You are on page 1of 601

www.jeebooks.

in
www.jeebooks.in

The One & Only Complete


Assessment & Practice Package

Test Drive for

NEET
2020
30 UNIT TESTS (PCB)

12 MOCK TESTS
www.jeebooks.in

The One & Only Complete


Assessment & Practice Package

Test Drive for

NEET
2020
30 UNIT TESTS (PCB)

12 MOCK TESTS
1000+ SUBJECTWISE
Most Difficult Questions of Last 15 Years' of NEET & AIPMT...

Er. Himesh Chakravarthy (Physics)


Romesh Thapar (Chemistry)
Dr. Nalini Banerjee (Biology)

ARIHANT PRAKASHAN (Series), MEERUT


www.jeebooks.in
Test Drive for

NEET
2020

Arihant Prakashan (Series), Meerut


All Rights Reserved

© PUBLISHERS
No part of this publication may be re-produced, stored in a retrieval system or distributed
in any form or by any means, electronic, mechanical, photocopying, recording, scanning,
web or otherwise without the written permission of the publisher. Arihant has obtained all
the information in this book from the sources believed to be reliable and true. However,
Arihant or its editors or authors or illustrators don’t take any responsibility for the absolute
accuracy of any information published and the damages or loss suffered there upon.
All disputes subject to Meerut (UP) jurisdiction only.

ADMINISTRATIVE & PRODUCTION OFFICES


Regd. Office
‘Ramchhaya’ 4577/15, Agarwal Road, Darya Ganj, New Delhi -110002
Tele: 011- 47630600, 43518550; Fax: 011- 23280316
Head Office
Kalindi, TP Nagar, Meerut (UP) - 250002
Tele: 0121-2401479, 2512970, 4004199; Fax: 0121-2401648

SALES & SUPPORT OFFICES


Agra, Ahmedabad, Bengaluru, Bareilly, Chennai, Delhi, Guwahati,
Hyderabad, Jaipur, Jhansi, Kolkata, Lucknow, Meerut, Nagpur & Pune

ISBN : 978-93-13199-33-5

Published by Arihant Publications (India) Ltd.


For further information about the books published by Arihant
log on to www.arihantbooks.com or email to info@arihantbooks.com

/arihantpub /@arihantpub Arihant Publications /arihantpub


www.jeebooks.in
Test Drive for

NEET
2020

PREFACE
Every year lakhs of students dreaming to get that venerable prefix Dr. before their names go
through intense rigors and grinds of pre-medical tests. In order to be eligible for admission to
MBBS/BDS courses for a particular academic year, it shall be necessary for a candidate to obtain
minimum marks in NEET held for the same academic year.
To help you go through NEET, we have come up with Test Drive for NEET 2020 which will give
you complete Assessment & Practice right before the real exam.
This book will help you to get higher score in the NEET, it consists of 12 Mock Tests, 30 Unit Tests
and more than 1000 questions asked in NEET & AIPMT. Each Mock Test containing questions
with their authentic solution having the conceptual approach.

SALIENT FEATURES
There are some special features associated with this book, which make it stand apart from others.
Some of them are
! 30 Unit Tests for all the three Sections: Physics, Chemistry and Biology.
! 12 Mock Tests strictly based on the latest examination pattern.
! Most Difficult Questions asked in last 15 Years' of NEET & AIPMT
! At the end of book, there are NEET Solved Papers 2019 (National & Odisha).

We are highly thankful to ARIHANT PRAKASHAN, MEERUT for giving us such an excellent
opportunity to write this book. The role of Arihant DTP Unit and Proofreading team is
praiseworthy in the making of this book.
Huge efforts have been made from our side to keep this book error free, but inspite of that if
any error or whatsoever is skipped in the book then that is purely incidental, apology for the
same, please write to us about that so that it can be corrected in the further edition of the book.
Suggestions for further improvement of the book will also be welcomed.
In the end, we would like to wish BEST OF LUCK to our readers!

Authors
www.jeebooks.in

Test Drive NEET 2020


Cutting Edge Features

The whole New Test Drive for NEET aims to


provides systematics practice. For achieving its
aim book is divided into 3 Modules, Module I
(Prep Analysis), Module II (Prep catalysis ) and
Module III (The NEET Edge).
Given here are the some glimpses of the book to
give you the idea of its organisation.

PREP ANALYSIS
This stage contains Unit Tests for all the three
parts of NEET; Physics, Chemistry and Biology.
The objective for the tests given in this stage
should be to qualify them with not
less than 75% marks, the score in different tests
will help to analyse the preparation of individual
unit.

PREP CATALYSIS
This stage contains 12 Mock Tests exactly based
on Pattern and Level of NEET to help you to be
acquainted with pattern and level of NEET and to
complete the same in a prescribed time limit of
3 hrs. You are advised to complete each Mock
Test in a single shot.
www.jeebooks.in
Test Drive for

NEET
2020

The NEET Edge


This stage contains subjectwise most difficult
questions asked in last 15 years NEET & AIPMT.
Practices there question will boost your confidence.

OTHER SPECIAL FEATURES


The Score Chart given with each
practice sets in this book will help you
to know the real level of your
performance.

Answer Key has been provided with


each practice sets, to help you to
score yourself. you can cross check the
answers selected by you with the
answer given here and mark your
answer as correct or incorrect.

with the detailed explanations given


for the Mock Tests (1-10) you can
check the point of mistake or the
complete solutions in case you do not
get the correct answer.
But we advise you to see explanations
only after you put your best efforts in
solving the questions.
www.jeebooks.in
Test Drive for

NEET
2020

CONTENTS
MODULE 1 : Prep Analysis
Unit Test of Physics 3-32
Unit Test of Chemistry 33-60
Unit Test of Biology 61-90

MODULE 2 : PREP CATALYSIS


Mock Test 1 93-114
Mock Test 2 115-136
Mock Test 3 137-159
Mock Test 4 160-182
Mock Test 5 183-203
Mock Test 6 204-225
Mock Test 7 226-248
Mock Test 8 249-271
Mock Test 9 272-295
Mock Test 10 296-317
Mock Test 11 318-331
Mock Test 12 332-344

MODULE 3 : THE NEET EDGE


Physics 347-414
Chemistry 415-474
Biology 475-532
NEET Solved Papers 2019 (National & Odisha) 1-64
www.jeebooks.in

www.jeebooks.in

MODULE 1
Prep Analysis
This section contains Unit Tests for all the three parts of
NEET; Physics, Chemistry and Biology. The objective
for the tests given in this stage should be to qualify them with
not less than 75% marks. The score in different tests will help
you to analyse the preparation of individual unit.
www.jeebooks.in
Prep Analysis for NEET ~ Physics 3

PREP ANALYSIS
for NEET
Full Length Tests for each Unit of NEET Physics to Tighten up the Loose Screws

PHYSICS
Instruction In each unit, question nos. 1 to 25 are of moderate difficulty level and question nos. 26 to 45 are of higher difficulty level.

1. Physical World, Measurement 6. The numbers 2.745 and 2.735 on rounding off to 3
significant figures will give
and Kinematics (a) 2.75 and 2.74 (b) 2.74 and 2.73
L (c) 2.75 and 2.73 (d) 2.74 and 2.74
1. A physical quantity have the same dimension as
C
that of 7. If random error in the arithmetic mean of 100
(a) (current) -1
(b) (time)2 observations is x, then the random error in the
(c) (voltage) (d) (resistance) 2 arithmetic mean of 500 observations would be
x x
(a) 5x (b) (c) 25x (d)
2. The force which can be attractive and repulsive 5 25
depending on the distance between the interacting 8. Name the quantity represented by the dimensional
particles is formula [ML-3 T 0 ].
(a) gravitational force (b) weak force (a) Specific gravity (b) Linear mass density
(c) electromagnetic force (d) strong force (c) Impulse (d) Density
3. A ball thrown vertically upwards with an initial 9. The pitch of screw gauge is 0.5 mm. The circular
1
velocity of 8 ms -1 returns in 1 s. The total scale of it has 200 divisions. The least count of
2 screw gauge is
displacement of the ball is
. ´ 10-2 cm
(a) 025 . ´ 10-3 cm
(b) 025
(a) 12 m (b) 5.34 cm (c) 0.05 ´ 10-2 cm (d) 0.05 ´ 10-3 cm
(c) zero (d) 24 m
10. The physical quantities not having same
4. The position of a particle moving along X-axis dimensions are
depends on time in accordance with following (a) momentum and Planck’s constant
equation x = at 2 + bt3 . Then, dimensions of a and b (b) speed and (m 0 e0 )-1/ 2
are respectively
p
(c) speed and
(a) [LT-2 ], [LT-3 ] (b) [LT-3 ], [LT-2 ] r
(c) [LT-2 ], [LT-2 ] (d) None of these (d) surface tension and spring constant
5. Which one of the following quantities has not been 11. A car is moving with same velocity and minimum
expressed in proper units ? distance over which it can be stopped is y. If the
Stress speed of the car is halved, then what will be
(a) = Nm-2 (b) Surface tension = Nm -1
minimum distance covered over which the car can
Strain
be stopped?
(c) Energy = kg-ms -1 (d) Pressure = Nm -2
(a) y / 4 (b) 4y (c) 2 y (d) 6y

MODULE 1
www.jeebooks.in
4 NEET Test Drive

12. A particle moves for 20 s with velocity 3 ms -1 and 21. If a plane flying horizontally at 100 m/s at a
then velocity 4 ms -1 for another 20 s and finally height of 1000 m releases a bomb from it, then the
moves with velocity 5 ms -1 for next 20 s. Find the velocity with which the bomb hits the ground is
value of average velocity. (a) 72.1 m/s
(a) 3 ms -1 (b) 4 ms -1 (b) 173.2 m/s
(c) 5 ms -1 (d) Zero (c) 197.2 m/s
(d) None of these
13. A particle starts its motion from rest under the
action of a constant force. If the distance in first 22. Which of the following is the altitude-time graph
10 s is s1 and distance in first 20 s is s2, then (figure) for a projectile thrown horizontally from
(a) s2 = 2 s1 (b) s2 = 3s1 the top of the tower?
(c) s2 = 4s1 (d) s2 = s1
Y Y
14. The velocity of an object is v = Pt + Qt 2 where
P and Q are constants, then the distance travelled h h
by it between 1 sec and 2 sec is (a) (b)
3 3 7 P Q
(a) P + 4Q (b) 3P + 7Q (c) P + Q (d) +
2 2 3 2 3
X X
15. Two bodies of mass 1 kg and 3 kg respectively t t
dropped from heights of 16m and 25m respectively. Y Y
The ratio of the time taken by them to reach the
ground is h
12 5 h
(a) (b) (c) (d)
5 12
4 5
(c) (d)
5 4
X X
16. A particle moves along a circular path of radius t t
0.5 m. Its speed increases at a uniform rate from
2 ms -1 to 4 ms -1 in 4 s. The angular acceleration of 23. The distance travelled by a particle starting from
4
the particle is rest and moving with an acceleration ms -2, in the
-2 -2 3
(a) 1 rad s (b) 2 rad s third second is
(c) 3 rad s -2 (d) 4 rad s -2
10 19
(a) m (b) m
17. A particle is moving with velocity 3 3
v = K ( yi$ + x$j) where K is a constant. The general (c) 6 m (d) 4 m
equation for its path is
24. The velocity of a bullet is reduced from 200 ms -1 to
(a) y = x2 + constant (b) y2 = x + constant 100 ms -1, while travelling through a wooden block
(c) xy = constant (d) y2 = x2 + constant of thickness 10 cm. The retardations assuming it to
be uniform, will be
18. For a body moving in a circular path, a condition
(a) 10 ´ 104 ms -2 (b) 12 ´ 104 ms -2
for no skidding, if m is coefficient of friction, is
mv 2 mv 2 (c) 13.5 ´ 104 ms -2 (d) 15 ´ 104 ms -2
(a) £ mmg (b) ³ mmg
r r 25. A cricketer can throw a ball to a maximum
2
v mv horizontal distance of 100 m. The speed with which
(c) = mg (d) = mmg
r r he throws the ball is (to nearest integer)
19. The value of acceleration due to gravity is 9.8 ms -2. (a) 10 ms -1 (b) 10 ms -1
Its value in km-min -2
will be (c) 10 10 ms -1 (d) 9.8 ms -1
(a) 35.3 (b) 58.8 26. A body of mass m is thrown with a velocity v
(c) 5.9 (d) 17.6 making an angle 45° with horizontal. The
20. If a particle has initial velocity ( 2$i + 3$j) and magnitude of change in momentum between
starting point at and maximum height is (neglect
acceleration ( 0.3$i + 0.2 $j). The magnitude of velocity
air resistance)
after 10 s will be
(a) 2 mv (b) 2 mv
(a) 5 units (b) 9 units
2 mv
(c) 5 2 units (d) 9 2 units (c) (d) None of these
2

MODULE 1
www.jeebooks.in
Prep Analysis for NEET ~ Physics 5
27. If velocity v, acceleration a and force F are taken as 35. If the dimensions of length are expressed as
fundamental quantities, then the dimension of G x c y h z , where G , c and h are the universal
Young’s modulus Y would be gravitational constant, speed of light and Planck’s
(a) Fa2 v -2 (b) Fa2 v -3 (c) Fa2 v -4 (d) Fa2 v -5 constant respectively, then
1 -3 1 -3
28. If in a Vernier calliper, one main scale division is (a) x = , y= z= (b) x = y = , z=
2 2 2 2
equal to M units and nth division of main scale 1 -3 -3 1
coincides with ( n + 1)th division of Vernier scale, (c) x = z = , y = (d) x = z = ,y=
2 2 2 2
then least count of Vernier is
(a)
nM
(b)
nM 36. A ball falling from rest was observed to fall
n+1 m+ 1 through 78.4 m in 2s. The time for which it had
M been falling before this observation is
(c) (d) None of these
n+1 (a) 5s (b) 4s (c) 2s (d) 3s

29. In the formula x = 3 yz 2, x and z have dimensions of 37. Two cars are moving with speeds 9 ms -1 and
15 ms -1. The relative velocity of first car w.r.t
capacitance and magnetic induction, respectively.
second car, if the first car is moving along North
The dimension of y should be
and second car is moving along West is
(a) [M3L2 T-4 A -4 ] (b) [M-2L-2 T3 A 2 ]
(a) 3 34 ms -1 at tan-1 æç ö÷ North to West
3
(c) [M-3L-2 T8 A 4 ] (d) [M-3L-2 T2 A 0 ] è 5ø
(b) 34 ms -1 at tan-1 æç ö÷ North to East
3
30. The position vectors of a particle moving in
è 5ø
xy-plane as a function of time are x = 5 + 0.20 t 2
(c) 8 ms -1 at tan-1 æç ö÷ North to East
5
and y = 1.5 t + 0.05 t 2. Find the unit vector along
è 3ø
particle velocity at t = 5 s.
(d) 2 2 ms -1 at tan-1 æç ö÷ North to West
5
1 $ $ è 3ø
(a) (2 $i + 2 $j ) (b) (i + j )
2
1 38. A particle is projected with velocity u along
(c) ($i + $j ) (d) ($i + $j )
2 X-axis. The acceleration of the particle is
-1 proportional to the square of distance from origin,
31. Consider an expression F = Ax sin ( Bt ), where F then the distance at which the particle stop is
represents force, x represents distance and t æ 2u 2 ö
1/ 3
æ 3u 2 ö
1/ 3 1/ 3
(d) æç ö÷
3u 2u
represents time. Dimensionally the quantity AB (a) (b) çç ÷÷ (c) çç ÷÷
2k è k ø è 2k ø è 3k ø
represents
(a) energy (b) surface tension 1/ 3
æ c ö
(c) intensity of light (d) pressure 39. A physical quantity Q = ab2 ç ÷ is determined
è d2 ø
32. A stone is dropped into a pond from a cliff of
by measuring a , b, c and d separately with the
height H meters, then the time after which the
percentage error of 1.5%, 2%, 3% and 3%
splash of sound is heard (where, v is speed of
respectively. Minimum percentage error in Q is
sound in air)
contributed by
2H H 2H
(a) + (b) (a) a (b) b (c) c (d) d
g v g
40. In an experiment to measure the height of a bridge
2H 2H 2H
(c) (d) + a stone is dropped. If the error in measurement of
v g v time is 0.5 s at the end of 2s, then the error in
33. A tennis ball is dropped on the floor from a height estimation of height of bridge is
of 5 m. It rebounds to a height of 2.5 m. If the ball (a) 1 m (b) 9.8 m (c) 19.6 m (d) 4.9 m
is in contact with the floor for 3 ´ 10-3 s, then its 41. A boy wants to throw a ball from a point P to Q as
average acceleration during the contact is (take, shown in the figure. The minimum velocity with
g = 10 ms -2) which the boy should throw the ball is
(a) 1260 ms -2 (b) 0 ms -2 (take, g = 10 ms -2)
(c) 5690 ms -2 (d) 4500 ms -2 P
1 Q
34. A bullet loses of its velocity in passing through 28 m
20 8m
15 m
a plank. What is the least number of planks
required to stop the bullet?
(a) 8 (b) 7 (c) 11 (d) 14 (a) 15 ms -1 (b) 30 ms -1 (c) 20 ms -1 (d) 7.5 ms -1

MODULE 1
www.jeebooks.in
6 NEET Test Drive

42. A police van moving on a highway with a speed of 5. A body is moving with a constant speed in a
30 kmh -1 fires a bullet at a thief’s car speeding straight line path. A force is not required to
away in the same direction with a speed of (a) increase its speed
192kmh -1. If the muzzle speed of the bullet is (b) decrease the momentum
150 ms -1, then with which speed does the bullet (c) keep it moving with uniform velocity
hit the thief’s car? (d) change the direction
(a) 105 ms -1 (b) 210 ms -1
(c) 315 ms -1 (d) 205 ms -1
6. A block is projected along a rough horizontal road
with the speed 10 ms -1. How far will it travel
43. A car moves a distance of 200 m. It covers the first before it comes to rest (u K = 010
. )?
half of the distance at a speed of 40 kmh -1 and (a) 20 m (b) 30 m (c) 40 m (d) 50 m
second half of the distance at a speed v. The
average speed is 48 kmh -1, then find the value of v. 7. A particle will leave a vertical circle of radius r,
when its velocity at the lowest point of the circle
(a) 56 km/h (b) 60 km/h
(c) 50 km/h (d) 58 km/h
( v2 ) is
(a) 2gr (b) 5gr (c) 3gr (d) 6gr
44. The acceleration a of a particle moving in a
straight line varies with displacement s. According 8. A block has been placed on an inclined plane and
to relation a = 2s, then velocity of the particle vary the slope angle q of the plane is such that the
with displacement as (given that velocity is zero at blocks slides down with a constant speed. The
zero displacement) coefficient of kinetic friction is equal to
(a) s (b) 2 s (a) sinq (b) tan q
(c) s 2 (d) None of these (c) cos q (d) cot q

45. A small block of mass m slides down from the top 9. Conservation of momentum in a collision between
of a hemisphere of radius R (figure). The surface of particles can be understood from
block and hemisphere are frictionless. The height (a) conservation of energy
at which the body loose contact with the surface of (b) Newton’s first law only
sphere is (c) Newton’s second law only
(d) Both Newton’s second and third law
10. A weightlifter holding a 100 kg mass steadily on
R his shoulder for 30 s does no work on the load
h during this time, because
(a) F = 0 (b) s = 0
3 2 1 2 v2
(a) R (b) R (c) gt (d) (c) q = 0º (d) None of these
2 3 2 2g
11. A shell in its flight explodes into four unequal
parts. Which of the following is conserved?
2. Laws of Motion and Work, (a) momentum (b) kinetic energy
Power & Energy (c) potential energy (d) Both (b) and (c)
1. A body whose momentum is constant must have 12. A ball with initial momentum p collides normally
constant with a rigid wall. If p¢ is its linear momentum after
(a) force (b) acceleration the perfectly elastic collision, then
(c) velocity (d) displacement (a) p¢ = - 2 p (b) p¢ = 2 p
2. The mud guards over the wheels of a car works on (c) p¢ = p (d) p¢ = - p
the basis of 13. When a spring is stretched or compressed a
(a) inertia of rest (b) inertia of motion restoring force is developed in it which is
(c) inertia of direction (d) None of these proportional to the elongation or compression in it.
3. The mass of a body weighing 100 dyne is (take, This is given by
g = 10 ms -2) (a) Hooke’s law
-1 -2 (b) Newton’s law
(a) 10 g (b) 10 g (c) 10 g (d) 100 g
(c) law of conservation of linear momentum
4. The force acting on a body, whose linear (d) principle of conservation of energy
momentum changes by 20 kgms -1in 10 s is
14. The coefficient of restitution/resilience of a
(a) 100 N (b) 200 N perfectly elastic collision in one dimension is
1 (a) 1 (b) 2
(c) 2 N (d) N
2 (c) 0.5 (d) 3

MODULE 1
www.jeebooks.in
Prep Analysis for NEET ~ Physics 7

15. A car drives along a straight level frictionless road 26. A particle of mass 100 g is thrown vertically
by an engine delivering constant powers. Then, upwards with a speed of 5 ms -1. The work done by
velocity is directly proportional to the force of gravity during the time the particle
(a) t (b) 1/ t goes up is
(c) t (d) None of these (a) -125
. J (b) 1.25 J (c) 0.5 J (d) –0.5 J
16. The work done by centripetal force in moving on 27. A body of mass 10 kg at rest is acted upon
body through half-cycle on a circular path of radius simultaneously by two forces 4 N and 3 N at right
30 m is angles to each other. The kinetic energy of the
(a) 30 J (b) 15 J (c) zero (d) 3 J body at the end of 10 s is
(a) 100 J (b) 300 J
17. If the momentum of a body is increased by 50%,
(c) 50 J (d) 125 J
then its kinetic energy becomes
(a) 100% (b) 150% (c) 125% (d) 225% 28. A ball dropped from a height of 2 m rebounds to a
height of 1.5 m after hitting the ground. Then, the
18. A body is falling freely under the action of gravity percentage of energy lost is
alone in vacuum. Which of the following quantities
(a) 25% (b) 30%
remain constant during the fall?
(c) 50% (d) 100%
(a) Kinetic energy
(b) Potential energy 29. A motorcycle moving with a velocity of 72 kmh -1
(c) Total mechanical energy on a flat road takes a turn on the road at a point
where the radius of curvature of the road is 20 m.
(d) Total linear momentum
The acceleration due to gravity is 10 ms -2. In order
^ ^ ^
19. A force F = ( 5 i + 2 j- k ) N produces acceleration of to avoid skidding, he must not bend with respect to
-2
1 ms in a body. The mass of the body is vertical plane by an angle greater than
(a) 15 kg (b) 30 kg (a) q = tan-1(2 ) (b) q = tan-1(6)
(c) 13 kg (d) 3 kg (c) q = tan-1(4) (d) q = tan-1(25.92 )
20. A cyclist bends while taking turn to 30. A stone is projected with kinetic energy K at an
(a) reduce friction angle of 45º with horizontal. The kinetic energy at
(b) generate required centripetal force highest point is
(c) reduce apparent weight K 2 K
(d) reduce speed (a) (b) (c) (d) K
2 K 2
21. If bullet of mass m and velocity v is fired into a 31. If a force, F = 6t 2$i + 4t$j is acting on a particle of
large block of mass M. The final velocity of the
system mass 3 kg, then velocity of particle at t = 3 s is (at
Mv mv m M
t = 0, particle is at rest)
(a) (b) (c) v (d) v (a) 2 $i + 3$j (b) 4$i + 6$j
M+m M+m M m
(c) 18i$ + 6$j (d) None of these
22. If the tension on a wire is removed at once, then 32. A uniform chain of length L and mass M is lying
(a) it will break on a smooth table and 1/6 of its length is hanging
(b) its temperature will reduce
down over the edge of table. If g is acceleration due
(c) its temperature will increase to gravity, then the work done to pull the hanging
(d) there will be no change in its temperature part on the table is
23. A body is said to be in equilibrium, if all the forces MgL MgL MgL
(a) (b) (c) (d) MgL
acting on it 72 18 6
(a) are in same direction (b) are equal in magnitude 33. A machine gun fires a bullet of mass 40 g with a
(c) have zero resultant (d) can be arranged in pair velocity 1200 ms -1. The man holding it can exert a
24. If the force on a rocket moving with a velocity of maximum force of 144 N on the gun. How many
300 ms -1 is 210 N, then the rate of combustion of bullets can he fire per second at the most?
the fuel is (a) 1 (b) 4 (c) 2 (d) 3
-1 -1
(a) 0.07 kg s (b) 1.4 kg s 34. A body moves along a circular path of radius 2m
(c) 0.7 kg s -1 (d) 10.7 kg s -1 and the coefficient of friction is 0.7. What should be
25. A body is moving in a circular path with its angular speed, so that it does not slip from the
acceleration a. If its velocity is doubled, then the surface? (Take, g = 9.8 ms -2 )
ratio of acceleration after and before the change is (a) 7 rad s -1 (b) 0.7 rad s -1
(a) 1 : 4 (b) 1 : 2 (c) 2 : 1 (d) 4 : 1 (c) 0.7 7 rad s -1 (d) 7 rad s -1

MODULE 1
www.jeebooks.in
8 NEET Test Drive

35. A monkey climbs up and another monkey climbs 41. The rate of mass of the gas emitted from rear of a
down a rope hanging from a tree with same rocket is initially 0.1 kg s -1. If the speed of the gas
uniform acceleration separately. If the respective releative to the rocket is 50 ms -1 and mass of the
masses of monkey’s are in the ratio 2 : 3, then the rocket is 2 kg, then the acceleration of the rocket
common acceleration must be (in ms -2) is
g g (a) 5 (b) 5.2 (c) 2.5 (d) 25
(a) (b) 6g (c) (d) 2g
5 2
42. A bullet losses 19% of its kinetic energy when
36. The resultant of two forces, one double the other in passes through an obstacle. The percentage change
magnitude, is perpendicular to the smaller of the in its speed is
two forces. The angle between the two forces is (a) increase by 10% (b) decrease by 10%
(a) 60º (b) 120º (c) 150º (d) 90º (c) increase by 20% (d) decrease by 20%
37. As shown in figure, the tension in the horizontal 43. A block of mass m is stationary with respect to a
cord is 30 N. The weight w and tension in the rough wedge as shown in figure. Starting from rest
string OA (in newton) are in time t, (m = 1 kg, q = 30° , a = 2 m/s 2 , t = 4s) work
A done on block

30°
30 N
O
a
w
m
(a) 30 3, 30 (b) 30 3, 60
(c) 60 3, 30 (d) None of these
38. For the given system as shown in figure, the pulley q
are light and frictionless. The tension in the string is
Column I Column II
A. By gravity p. 144 J
B. By normal reaction q. 32 J
M
C. By friction r. 56 J
D. By all the forces s. 48 J
M θ t. None
1 2 3
(a) 2Mg sinq (b) Mg sinq (c) Mg sinq (d) Mg sinq A B C D A B C D
2 3 2
(a) t p s q (b) p q r s
39. Block A of weight 100 kg rests on a block B and is (c) r p q t (d) t r p q
tied with a horizontal string to the wall at C. Block
B weighs 200 kg, and the coefficients of friction 44. A block of mass m placed on a rough inclined
between A and B is 0.25 and between B and surface plane of inclination q = 30º can just be prevented
is 1/3. The horizontal force F necessary to move the from sliding down by applying a force F1 up the
block B should be ( take, g = 10 ms -2 ) plane and it can be just made to slide up the plane
by applying force F2 up the plane. If coefficient of
friction between block and the inclined plane is
C A 1
, then
B F 2 3
(a) F2 = F1 (b) F2 = 2 F1
(a) 1150 N (b) 1250 N (c) 1300 N (d) 1420 N (c) F2 = 3F1 (d) F2 = 4F1

40. A body is projected at an angle q with the 45. A force acts on a 3 g particle in such a way that
horizontal. When it is at highest point, then the position of particle as a function of time is given by
ratio of the potential and kinetic energies of the x = 3t - 4t 2 + t3 , where x is in metre and t is in
body is second. The work done during first 4 s is
2u u (a) 570 mJ (b) 450 mJ
(a) tanq (b) tan2 q (c) (d)
g sinq g sinq (c) 490 mJ (d) 528 mJ

MODULE 1
www.jeebooks.in
Prep Analysis for NEET ~ Physics 9

3. Motion of System of Particles 12. For a hollow cylinder and a solid cylinder of the
same mass and radius rolling without slipping on
and Gravitation an inclined plane, which of these reaches the
1. Centre of mass of a body coincide with the ground earlier?
geometrical centre of the body, when body has a (a) hollow cylinder (b) Solid cylinder
(a) uniform motion (b) uniform mass density (c) Both simultaneously (d) Cannot say anything
(c) uniform momentum (d) uniform acceleration 13. If the radius of earth’s orbit is made 1/4th, then
durations of year will become
2. The moment of linear momentum is called
(a) 8 times (b) 4 times (c) 1/8 times (d) 1/4 times
(a) torque (b) couple
- 11
(c) angular momentum (d) None of these 14. The value of G is 6.67 ´ 10 N - m kg - 2 . On
2

3. A flywheel gains a speed of 540 rpm in 6 s. Its moon, the value of g is nearly one-sixth than that
angular acceleration is on earth, then value of G on moon is
G
(a) 3p rad/s 2 (b) 54 p rad/s 2 (a) (b) G (c) 6 G (d) 3G
6
(c) 9 p rad/s 2 (d) 18 p rad/s 2
15. If the earth suddenly shrinks to half of its present
4. The rotational analogue of force is called
radius, then the acceleration due to gravity will be
(a) angular momentum (b) moment of inertia
(a) g/2 (b) 4g (c) 2g (d) g/4
(c) torque (d) centre of mass
16. A man weighing 32 N goes from the surface of
5. Moment of inertia of a body depends upon
earth to a height equal to the radius of earth, then
(a) mass of body
his weight becomes
(b) shape and size of body
(a) 8 N (b) 32 N (c) 64 N (d) 16 N
(c) position and orientation of axis of rotation
(d) All of the above 17. If there is an infinite separation between two
interacting masses, then the gravitational
6. The radius of gyration of an uniform circular disc potential energy of the body will be
of radius R, about any diameter of the disc is
R (a) infinite (b) zero
(a) R (b) (c) dependent on masses (d) None of these
2
(c) 2R (d) None of these 18. What is the sense of rotation of stationary satellite
around the earth?
7. A couple is acting on a two particle system. The
(a) North to South (b) South to North
resultant motion will be
(c) East to West (d) West to East
(a) purely rotational motion (b) purely linear motion
(c) Both (a) and (b) (d) Neither (a) nor (b) 19. Mass of A satellite is 16 times that of B satellite,
then the ratio of the period of revolution of B to
8. If a person standing on a rotating disc stretches that of A is
out his hands, then the angular speed will (a) 1 : 1 (b) 1 : 4 (c) 4 : 1 (d) 16 : 1
(a) increases (b) decreases
(c) remains same (d) None of these 20. The value of orbital speed of a satellite orbiting
close to the surface of earth is
9. The orbital speed of jupiter is g
(a) v 0 = R (b) v 0 = gR
(a) less than the orbital speed of earth R+ h
(b) equal to the orbital speed of earth GM
(c) greater than the orbital speed of earth (c) v 0 = 2 hg (d) v 0 =
r
(d) zero
21. The escape speed of a satellite is 10 kms- 1, then its
10. If a solid sphere and solid cylinder of same mass orbital speed is
and radius rotate about their own axis, the 5
(a) 10 2 kms - 1 (b) kms - 1
moment of inertia will be greater for 2
(a) solid sphere (b) solid cylinder (c) 5 2 kms - 1 (d) 10kms - 1
(c) Both (a) and (b) (d) equal for both
22. A satellite is in an orbit around the earth. If its
11. Under a constant torque, the angular momentum kinetic energy is doubled, then
of a body changes from A to 4A in 4 s. The torque (a) It will rotate with greater speed
on the body will be (b) It will fall on earth
3 1 4 (c) It will maintain its path
(a) A (b) A (c) A (d) 4A
4 4 3 (d) It will escape out of earth’s gravitational field

MODULE 1
www.jeebooks.in
10 NEET Test Drive

23. Gravitational force on a particle lying inside a 30. Equal torques are applied on a cylinder and a
hollow body of any arbitrary shape is hollow sphere of equal mass and radius. The
(a) equal to that on its surface (b) zero cylinder rotates about its axis, while sphere rotates
(c) half of its value on surface (d) twice of its value on surface about one of its diameter. Which of them will
acquire greater speed?
24. The value of g at equator is smaller than that at (a) Cylinder (b) Hollow sphere
the poles due to (c) Have same speed (d) None of these
(a) rotation of earth (b) equatorial bulge
(c) Both (a) and (b) (d) None of these 31. A meery-go-round, made of a ring like plate form of
radius R and mass M, is revolving with angular
25. The intensity of gravitational field, when a force of speed w. A person of mass M standing on it jumps
150 N acts on a body of mass 30 kg in the off the round, radially away from the centre of
gravitational field is round. The speed of the round becomes
5 w
(a) 45 N-kg -1 (b) N-kg -1 (a) w (b) 2w (c) (d) 0
3 2
-1
(c) 10 N- kg (d) 5 N-kg -1
32. The radius of gyration of a body about a parallel
26. A particle having angular momentum L performs axis through its centre of mass is 8 cm. Then, the
uniform circular motion. When its frequency of radius of gyration about an axis at a distance of
motion is doubled and kinetic energy is halved, 6 cm from its centre of mass is
then the angular momentum becomes (a) 8 cm (b) 10 cm
L 1 (c) 6 cm (d) 5 cm
(a) L (b) 2 L (c) (d) L
2 4
33. Two wheels having radii in the ratio 1 : 2 are
27. A ball of mass 0.5 kg is suspended at a height of connected by a common belt. If the smaller wheel
1.5 m above the ground by a weightless string that is accelerating from rest at a rate of 1.25 rad s- 2
passes over a frictionless pulley as shown in the for 4 s, then find the velocity of bigger wheel.
figure. Then, the velocity with which the ball
(a) 2.5 rad s - 1 (b) 5 rad s -1
strikes the ground is (c) 2 rad s - 1 (d) 1.5 rad s - 1
(Take, g = 10 ms- 2)
M=4 kg 34. A man weighing 60 kg is standing on a trolley
weighing 360 kg. The trolley is resting on
frictionless horizontal rails. If the man starts
walking on the trolley along the rail at speed
2 ms - 1, then after 14 s his displacement relative to
m=0.5 kg the ground will be
1.5 m (a) 5 m (b) 10 m (c) 7 m (d) 2 m

Ground 35. A stone tied to a string takes 6 s to complete


-1 -1 -1 -1
revolution along a horizontal circle. If by pulling
(a) 6 ms (b) 6 ms (c) 3 ms (d) 3 ms the cord, the radius of the circle is reduced to
28. A body P of mass M while falling under gravity one-third to the previous value, then the time, the
1 ball will take in one revolution is
breaks into two parts, Q part of mass M and R
4 (a) 0.50 s (b) 0.67 s (c) 0.60 s (d) 0.99 s
3 36. The moment of inertia of a body about a given axis
part of mass M. If centre of mass of bodies Q and
4 is 1.5 kg-m 2. Initially, the body is at rest. To
R taken together, then there is a shift from that of produce a rotational KE of 300 J, the time duration
P towards for which an acceleration of 20 rad s- 2 must be
(a) body Q applied about the axis is
(b) body R (a) 1s (b) 2s (c) 0.5s (d) 1.5s
(c) no shift
37. The ratio of the acceleration due to gravity at the
(d) depend on height of breaking
bottom of a deep mine and that on the surface of
29. A grind stone has moment of inertia of 7 kg-m 2. A earth is
978
. If the density of earth is uniform
constant torque is applied to the grind stone and it 980
has a speed of 240 rpm in 8 s from rest position. throughout and the radius is 6300 km, then depth
The value of torque is of the mine is
(a) 2p N-m (b) 5p N-m (a) 90.9 km (b) 13.0 km
(c) 7p N-m (d) 6p N-m (c) 12.86 km (d) 12.38 km

MODULE 1
www.jeebooks.in
Prep Analysis for NEET ~ Physics 11

38. A satellite moves in a elliptical orbit about a 4. Properties of Bulk Matter and
planet. The maximum and minimum velocities of
satellites are 3 ´ 104 ms - 1 and 1 ´ 103 ms - 1 Thermodynamics
respectively. What is the maximum distance of 1. A rod elongates by l when a body of mass M is
satellite from planet, if minimum distance is suspended from it. The work done is
4 ´ 104 km? (a)
1
Mgl (b) Mgl (c) 2 Mgl (d) zero
4 4 2
(a) 1.2 ´ 10 km (b) ´ 103 km
3 2. A wire of length L and cross-section area A is
3
(c) 12 ´ 105 km (d) ´ 103 km made of material of Young’s modulus Y . The work
4
done in stretching the wire by an amount x is
39. Energy required to move a body of mass m from an YAx2 YAx2 2 YAx2 4YAx2
(a) (b) (c) (d)
orbit of radius 2R to 3R is L 2L L L
GMm GMm GMm GMm
(a) (b) (c) (d) 3. The value of Young’s modulus for perfectly rigid
12 R 2 3R 2 8R 6R
body is
40. The escape velocity of a body on the surface of the (a) 1 (b) less than 1 (c) zero (d) infinite
earth is 11.2 km/s. If earth’s mass is increases to
twice of its present value and radius becomes half, 4. A wire can sustain a weight of 15 kg. If it cut into
then the escape velocity will be four equal parts, then each part can sustain a
(a) 11.2 km/s (b) 22.4 km/s
weight.
(c) 44.8 km/s (d) None of these (a) 5 kg (b) 30 kg (c) 15 kg (d) 45 kg

41. A satellite is a heavily body which revolve around 5. What is the pressure on a swimmer 20 m below the
the earth at some height. Suppose a satellite is surface of water?
orbiting around the earth in a circular orbit of (a) 1 atm (b) 2 atm (c) 3 atm (d) 4 atm
radius R from centre of earth, then its period of 6. A beaker containing a liquid of density r moves up
revolution varies as with an acceleration a. The pressure due to the
3
(a) R 2 (b) R 2 (c) R (d) R liquid at a depth h below free surface of the liquid
(a) hrg (b) hr(g - a)
42. The radius of a planet is twice the radius of earth. (c) hr(g + a) (d) hr g (g - a)
Both have almost equal average mass-densities. If
v p and ve are escape velocities of the planet and the 7. The force of cohension is maximum in
earth respectively, then (a) solids (b) gases
(a) v p = 15
. ve (b) v p = 2 ve (c) liquids (d) All of these
(c) ve = 3v p (d) ve = 15. vp 8. If we dip capillary tubes of different radii in water
43. A person brings a mass of 0.5 kg from infinity to a and water rises to different heights in them, then
h h
point P. Initially, the mass was at rest but it (a) = constant (b) = constant
moves at a speed of 6 ms- 1 as it reaches at P. The r2 r
work done by the person on the mass is - 4 J. The (c) hr = constant (d) hr 2 = constant
potential at P is 9. After terminal velocity is reached, the acceleration
(a) - 13 J/kg (b) -26 J/kg (c) - 10 J/kg (d) - 15 J/kg of a body falling through a fluid is
44. A satellite in a force-free space sweeps stationary (a) equal to g (b) less than g
dM (c) more than g (d) zero
interplanetary dust at a rate = bv, where v is
dt 10. ‘‘A good absorber is a good emitter’’ is explained by
the speed of escaping dust w.r.t. satellite and M is (a) Stefan’s law (b) Wien’s law
the mass of satellite at that instant. The (c) Newton’s law of cooling (d) Kirchhoff’s law
acceleration of the satellite is
bv 2 bv 2 mb 11. Solar constant S depends upon the temperature of
(a) - bv 2 (b) - (c) - (d) - 2 the sun T as
2M M v
(a) S µ T 4 (b) S µ T 3 (c) S µ T 2 (d) S µ T
45. A satellite is orbiting just about the surface of a
planet of average density r with period T . If G is 12. Two spheres of same material and radius r and 2r
the universal gravitational constant, then the are heated to same temperature and are kept in
quantity T 2r is equal to identical surroundings, ratio of their rate of loss of
heat is
4p 2 3p 1
(a) 4p 2G (b) (c) (d) (a) 1 : 2 (b) 1 : 4 (c) 1 : 6 (d) 1 : 8
G G G

MODULE 1
www.jeebooks.in
12 NEET Test Drive

13. Which of the following factors affect the thermal 25. Carnot engine can not give 100% efficiency,
conductivity of a rod? because we can not
(a) Area of cross-section (b) Length of rod (a) eliminate friction
(c) Material of rod (d) All to these (b) find ideal sources
(c) prevent radiation
14. A solid ball of copper has a co-centric spherical
(d) reach absolute zero temperature
cavity within it. If the ball is heated, the volume of
the cavity will 26. Four wires of length L and radius r are given in
(a) increase (b) decrease options which are made of same material. Which of
(c) no change (d) None of these these will have the largest extension, when the
same tension is applied?
15. The intensive property among the following is
(a) L = 100 cm, r = 0.2 mm (b) L = 200 cm, r = 0.4 mm
(a) energy (b) volume
(c) entropy (d) temperature (c) L = 300 cm, r = 0.6 mm (d) L = 400 cm, r = 0.8 mm

16. The state of a thermodynamic system is 27. When a solid sphere of metal is taken to bottom of
represented by sea 2 km deep, it contracts by 0.04%. The bulk
(a) pressure only
modulus of elasticity of the material of sphere is
(given, density of water = 1 g cm -3 )
(b) volume only
(c) pressure, volume and temperature (a) 0.5 ´ 1010 Nm -2 (b) 5 ´ 1010 Nm -2
(d) numbers of moles (c) 0.6 ´ 1010 Nm -2 (d) 6 ´ 1010 Nm -2

17. Which of the following is unique function of initial 28. The energy stored per unit volume in copper wire,
and final states? which produces longitudinal strain of 0.2% is
(a) dQ (b) dW (c) dU (d) dQ and dW . ´ 1011 Nm -2)
(given, Y = 11
(a) 11 ´ 103 Jm -3 (b) 5.5 ´ 103 Jm -3
18. If the heat of 110 J is added to a gaseous system,
whose internal energy is 40J, then the amount of (c) 2.2 ´ 105 Jm -3 (d) 11 ´ 104 Jm -3
th
external work done is æ 7ö
29. A block of wood floats in water with ç ÷ of its
(a) 180J (b) 70J (c) 110J (d) 30J è 8ø
19. For an adiabatic process, the relation between V volume submerged. If the same block just floats in
and T is given by a liquid, the density of the liquid is (in kg-m -3 )
(a) TV g = constant (b) T g V = constant (a) 800 (b) 750 (c) 1250 (d) 875
(c) TV1- g = constant (d) TV g -1 = constant 30. An incompressible fluid flows steadily through a
cylindrical pipe, which has radius 3r at point
20. In which of the processes, does the internal energy
A and radius r at point B further along the flow
of the system remain constant?
direction. If the velocity at point A is v, its velocity
(a) Adiabatic (b) Isochoric
at point B is
(c) Isobaric (d) Isothermal
(a) v (b) 5v (c) 9v (d) 12v
21. In which process the temperature of the system
decreases
31. Water rises in a vertical capillary tube upto a
length of 5 cm. If the tube is inclined at 30°, the
(a) isothermal compression (b) free expansion
length of water column risen in the tube will be
(c) adiabatic expansion (d) isothermal expansion
10 20
(a) 10cm (b) cm (c) 20 cm (d) cm
22. A child is blowing air with open mouth. Now, it is 3 3
an example of
(a) isobaric process (b) isochoric process
32. A second’s pendulum clock has a steel wire. The
(c) isothermal process (d) adiabatic process
clock is calibrated at 30°C. How much time does
the clock loss or gain in 2 week when the
23. The temperature of the sink of a carnot engine is temperature is increased to 50°C?
27°C and its efficiency is 25%. The temperature of (a steel = 1.2 ´ 10-5 °C -1)
the source is (a) 140.3 s (b) 180.8 s (c) 145.1 s (d) 190.4 s
(a) 227°C (b) 27°C (c) 327°C (d) 127°C
33. If 120 cal of heat is required to raise the
24. An electric fan is switched ON in a closed room. temperature of 3 mol of an ideal gas at constant
The air in the room is pressure from 40°C to 45°C, then the amount of
(a) cooled heat required to raise the temperature of same
(b) heated gas through same range at constant volume is
(c) maintains its temperature (take, R = 2 cal/mol°C)
(d) heated or cooled depending on the atmospheric pressure (a) 50 cal (b) 70 cal (c) 90 cal (d) 120 cal

MODULE 1
www.jeebooks.in
Prep Analysis for NEET ~ Physics 13

34. Three rods of same dimensions have thermal 40. In an adiabatic change, the pressure and
conductivities 3K , 2K and K , they are arranged as temperature of diatomic gas are related with
shown in the figure. relation p µ T - x , where x is equal to
60° 7 7 7 5
(a) (b) - (c) - (d) -
T 2K 5 5 2 3
120°
3K 41. 3 moles of an ideal gas in initial state A undergoes
K a cyclic process QRSQ, as shown in the figure. Its

pressure at Q is p0. Choose the correct option(s)
What will be the temperature T of the junction? from the following.
(a) 120°C (b) 90°C (c) 80°C (d) 60°C V
2
35. A black body of mass 60 g and surface area 24 cm 3V0 R
is at an initial temperature of 400 K. It is allowed
to cool inside an evacuated enclosure kept at V0 Q
S
constant temperature 200K. The rate of cooling is
0.04 °C/s. The specific heat of body is (Stefan’s T0 T
constant, s = 5.73 ´ 10-8 Jm -2 K -4)
(a) Internal energies at Q and R are not the same.
(a) 2800 Jkg -1K -1 (b) 2100 Jkg -1K -1 (b) Work done by the gas in process QR is p0 V0 ln 3
(c) 1002 Jkg -1K -1 (d) 1500 Jkg -1K -1 p
(c) Pressure at point S is 0
36. A woman, on finding his gas stove out of order, 4
T
decides to boil the water for her husband’s coffee (d) Temperature at point S is 0
5
by shaking it in a thermos flask. Suppose that, she
42. An ideal gas is allowed to expand freely against
uses tap water at 20°C and that the water falls
vacuum in a rigid insulated container. The gas
15 cm each shake. The woman is making 40 shakes undergoes
each minute. Neglecting any loss of thermal energy (a) no change in temperature and internal energy
by the flask. How long must she shake the thermos (b) an increase in temperature
until the water reaches 100°C? (c) decrease in its internal energy
(a) 7.6 ´ 103 min (b) 8.4 ´ 103 min (d) increase in work done by gas
. ´ 103 min
(c) 57 (d) 4.5 ´ 103 min 43. Initially two gas samples X and Y are at the same
thermodynamic condition. The volume of the two
37. Two temperature scales A and B are related by a are doubled, one isothermally and the other
A - 50 B - 25
mathematical relation is = adiabatically. What is the relation between the
220 440 final pressure p1 and p2?
At which temperature the two scales have the (a) p1 = p2
same reading? (b) p1 > p2
(a) 12 (b) -78 (c) 75 (d) -12 (c) p2 > p1
(d) Can not be determined
38. The ratio of specific heats of a gas is g. The change
in internal energy of one mole of the gas, when the 44. Three experimental design are proposed for truck
volume changes from V to 3V at constant pressure engine which are operating between 700K and
p is 350K. For 1 kcal of heat input X1 claims to produce
pV pV g -1 1800 J of mechanical work, design X 2 claims to
(a) (b) pV (c) (d)
g g -1 pV produce 4000 J of work and design X3 claims to
produce 2100 J of work. Now, find which engine
39. 2 cm3 of water at its boiling point absorbs 1080 cal has shown wrong experimental result.
of heat to become steam with a volume of (a) X1, X 2 (b) X 2 only
1620 cm3 . If the atmospheric pressure is 1 atm and (c) X1 only (d) All of these
the mechanical equivalent of heat is 4.2 J cal -1, the 45. A carnot engine has efficiency 3/5. Efficiency
energy spent in this process in overcoming becomes 4/5, when temperature of sink is decrease
intermolecular forces is by 75 K. What is the temperature of sink?
(a) 501.48 J (b) 935.76 J (a) 300 K (b) 350 K
(c) 1041.48 J (d) 1305.25 J (c) 230 K (d) 150 K

MODULE 1
www.jeebooks.in
14 NEET Test Drive

5. Kinetic Theory of Gases 12. Two simple pendulums of unequal length meet
each other at mean position while oscillating. Their
Oscillations & Waves phase difference could be
1. Calculate the number of atoms in 39.4 g gold. (a) zero (b) 180º
-1 (c) 90º (d) Both (a) and (b)
Molar mass of gold is 197 g mol .
. ´ 1023
(a) 13 . ´ 1023
(b) 12 . ´ 1024
(c) 11 . ´ 1025
(d) 11 13. A loaded vertical spring executes SHM with time
period of 6s. The kinetic and potential energy of
2. The value of gas constant R in SI units is this system varies with a period of
(a) 8.314 J mol - 1 K - 1 (b) 8.325 J mol - 1 K - 1 (a) 2 s (b) 3 s
-1 -1
(c) 7.231 J mol K (d) 2.732 J mol - 1 K - 1 (c) 4 s (d) None of these
3. The ratio of the specific heats of a gas is related to 14. Which of the following phenomenon is not common
degrees of freedom by to sound and light wave?
Cp Cp 1 Cp 2 Cp (a) Interference (b) Diffraction
(a) = 1 + 2 f (b) = (c) = 1+ (d) =f (c) Polarisation (d) Coherence
CV CV 2f CV f CV
15. If the tension of a stretched wire is increased to 4
4. A mass M is suspended from a light spring. An times, the wave speed changed to
additional mass m added displaces the spring (a) 2 times (b) 1/2 times (c) 4 times (d) 16 times
further by a distance X. Now, the combined mass
will oscillate on the spring with period 16. A sound absorber attenuates the sound level by
mg (M + m)X 20 dB. The intensity decreases by a factor of
(a) T = 2p (b) T = 2p (a) 100 (b) 1000 (c) 10000 (d) 10
X (M + m) mg
mg (M + m) 17. The important property of a material medium
(c) T = p /2 (d) T = 2p responsible for propagation of wave through it is
X(M + m) mg
(a) elasticity (b) inertia
5. In linear SHM, at the extreme position, the (c) Both (a) and (b) (d) None of these
velocity and acceleration of the particle are 18. In case of interference of two waves each of
(a) minimum, maximum (b) maximum, minimum intensity I 0, the intensity at a point of constructive
(c) same as original (d) zero, infinity interference
6. The average value of total energy of a particle in will be
SHM in one complete oscillation is (a) 3 I0 for coherent source (b) 2 I0 for coherent source
1 1 1 1 (c) 4 I0 for incoherent source (d) 2 I0 for incoherent source
(a) mw2 a (b) mw2 a2 (c) mw2 a2 (d) mw2 a
2 4 2 4 19. If a person places his ear to one end of a long iron
7. The time period of a simple pendulum whose pipe line, he can hear distinctly how many sounds,
length is equal to radius of earth is when a workman hammers the other end?
(a) 60 s (b) 84.6 min (a) One (b) Two (c) Three (d) No sound
(c) 30 s (d) 30 min 20. When an explosion takes place at the bottom of a
8. If the length of pendulum is doubled in a simple lake, the shock waves produced are
pendulum, the time period will become (a) longitudinal wave (b) transverse wave
(a) 2 times (b) same (c) stationary wave (d) matter wave
(c) half (d) 2 times 21. Sound can be heard over a longer distances on a
9. The force which keeps the simple pendulum in rainy day because
simple harmonic motion is (a) temperature of air decreases (b) density of air decreases
(a) mg sinq (b) mg cos q (c) density of air increases (d) None of these
(c) Both a and b (d) None of these
22. A tunning fork is in resonance with a closed pipe.
10. The ratio of the potential energy and the total But the same fork cannot be in resonance with an
energy of a particle executing SHM, when its open pipe, because of frequency difference of
displacement is half of its amplitude v 4v
(a) v / 2 L (b) 2v / L (c) (d)
(a) 1 : 2 (b) 1 : 4 4L L
(c) 2 : 1 (d) 4 : 1 23. If oil of density higher than the density of water is
11. A pendulum watch cannot be used in an artificial used in a resonance tube, the frequency change is
satellite as 1
(a) Df µ density (b) Df µ
(a) its time period is zero (b) its time period is infinity density
(c) gravity is infinite (d) there is no atmosphere (c) Df µ density (d) no change

MODULE 1
www.jeebooks.in
Prep Analysis for NEET ~ Physics 15

24. A siren emitting a sound of frequency 1000 Hz 32. A stationary wave is represented by
moves away from you towards a cliff with a speed y = A sin(100t ) cos( 0.01x ), where y and A are in
of 10 ms-1. What is the frequency of the sound you millimeter, t is in second and x is in metre. The
hear coming directly from the siren? velocity of the wave is
(Take, speed of sound in air = 280 m/s) (a) 102 m/s (b) 104 m/s
(a) 33/34 ×1000 Hz (b) 34/33 × 1000 Hz (c) 1 m/s (d) not desirable
(c) 35/34 × 1000 Hz (d) 34/35 × 1000 Hz
33. If l1, l 2 and l3 are wavelength of waves giving
25. For an echo to be heard, the minimum distance
between the source of sound and the reflector is resonance with fundamental, first and second
overtones respectively of a closed organ pipe, then
(Take, velocity of sound = 340 ms- 1)
the ratio of wavelengths l1 : l 2 : l3 is
(a) 10 m (b) 17 m (c) 3.4 m (d) 34 m (a) 1 : 3 : 5 (b) 5 : 3 : 1
26. When an ideal diatomic gas is heated at constant (c) 15 : 5 : 3 (d) None of these
pressure, the fraction of the heat energy supplied, 34. If two SHM’s with same time period and same
which increases the internal energy of the gas is amplitudes are acting together in perpendicular
(a) 2/5 (b) 3/5 (c) 3/7 (d) 5/7
directions with a phase difference of p / 2, then the
27. 1 mole of H 2 gas is contained in a box of volume motion will be
V = 1 m 3 at T = 300 K. The gas is heated to a (a) straight motion (b) circular motion
temperature T = 3000 K and the gas gets converted (c) elliptical motion (d) None of these
to a gas of hydrogen atoms. The final pressure
would be (assume gas to be ideal) 35. Two springs of spring constant k1 and k2 have equal
(a) same as initial pressure maximum velocities, while executing simple
(b) 2 times that of initial pressure harmonic motion. The ratio of their amplitudes will
(c) 10 times that of initial pressure be (masses are equal in both cases)
(d) 20 times that of initial pressure 1/ 2 1/ 2
æk ö æk ö k1 k2
28. The resultant of two rectangular simple harmonic (a) ç 1 ÷ (b) ç 2 ÷ (c) (d)
motions of the same frequency and unequal è k2 ø è k1 ø k2 k1
amplitudes but differing in phase by p /2 is 36. A wave motion is described by
(a) simple harmonic (b) circular y( x , t ) = a sin( kx - wt ). Then, the ratio of the
(c) elliptical (d) parabolic maximum particle velocity to the wave velocity is
29. The period of a simple pendulum measured inside w 1
(a) :1 (b) :1 (c) ka : 1 (d) wa : 1
a stationary lift is found to be T . If the lift starts k ka
accelerating upwards with acceleration of g / 3, 37. A body sends 100 mm long waves through medium
then the time period of the pendulum is P and 0.25 m long in medium Q. If the velocity of
(a) 2T / 3 (b) T / 3 (c)
3
T (d) 3T wave in medium P is 80 cms- 1. Then, the velocity
2 of wave in medium Q is
30. The displacement of a particle from its mean (a) 100 cms - 1 (b) 200 cms - 1
position (in meter) is given by (c) 80 cms -1
(d) 320 cms - 1
y = 0.2 sin(10pt + 1.5p ) cos(10pt + 1.5p ). This motion
of the particle is 38. The density of air at NTP is 1.29 kgm–3. Assume
air to be diatomic with g = 1.4. The velocity of
(a) periodic but not SHM sound at 127ºC is
(b) non-periodic (a) 382.8 ms–1 (b) 350 ms–1
(c) simple harmonic motion with period 0.1 s (c) 350.6 ms–1 (d) 348.6 ms–1
(d) simple harmonic motion with period 0.2 s
39. The phase difference between two points separated
31. A mass M, attached to a horizontal spring executes by 1 m in a wave of frequency 120 Hz is 90º. The
SHM with amplitude A1. When the mass M passes wave velocity will be
through its mean position, then a smaller mass m (a) 720 ms - 1 (b) 480 ms - 1 (c) 240 ms - 1 (d) 180 ms - 1
is placed over it and both of them move together
with amplitude A2. The ratio of ( A1 / A2 ) is 40. Two waves are represented by
(a)
M
(b)
M+ m y1 = a sin( wt - kx ) and y2 = a cos( wt - kx )
M+ m M The resultant wave will have an amplitude
1/ 2 1/ 2 (a) a (b) 2a
æ M ö M + mö
(c) ç ÷ (d) æç ÷ (c) 2a (d) zero
è M + mø è M ø

MODULE 1
www.jeebooks.in
16 NEET Test Drive

41. A longitudinal wave is represented by 4. Path of a charged particle in electric field enters at
æ xö angle 30° from the direction of electric field is
x = x0 sin 2p ç nt - ÷
è lø (a) parabolic (b) circular
(c) elliptical (d) straight line
The maximum particle velocity will be four times
the wave velocity, if 5. If an electric dipole is enclosed by Gaussian
px px surface, then net electric flux passes through the
(a) l = 0 (b) l = 2 px0 (c) l = 0 (d) l = 4px0
4 2 surface is
(a) positive (b) negative
42. A racing car moving towards a cliff sounds its (c) zero (d) None of these
horn. The driver observes that the sound reflected
from the cliff has a pitch one octave higher than 6. If the length of a conducting wire of resistivity r is
the actual sound of the horn. If v is the velocity of increased to double by stretching it, then its new
sound, the velocity of car is resistivity will be
(a) v / 2 (b) v / 2 (c) v / 3 (d) v / 3 r
(a) 2r (b) 4r (c) (d) r
43. An organ pipe open at one end is vibrating in first 2
overtone and is in resonance with another pipe 7. A point charge q produces an electric field of
open at both ends and vibrating in third harmonic. magnitude 2 NC -1 at a point distance 0.25 m from
The ratio of length of two pipes is it. What is the value of charge?
(a) 1 : 2 (b) 4 : 1 (c) 8 : 3 (d) 3 : 8 (a) 1.39 ´ 10-11C (b) 1.4 ´ 1011C
44. Consider the three waves Z1, Z 2 and Z3 as (c) 4.16 ´ 10-11C (d) 9 ´ 1011C
Z1 = A sin( kx - wt ), Z 2 = A sin( kx + wt ) and 8. In the following figure, equivalent capacitance
Z3 = A sin ( ky - wt ). Then, which of the following between A and B is
represents a standing wave?
6 µF
(a) Z1 + Z 2 (b) Z 2 + Z 3 (c) Z 3 + Z1 (d) Z1 + Z 2 + Z 3
A B
45. At a pressure of 20 ´ 104 Nm - 2 , the volume strain of 6 µF 6 µF
water is 5 ´ 10- 5 . Then, the speed of sound in
(a) 2 mF (b) 18 mF
water is (density of water is 103 kg-m - 3 ) (c) 4 mF (d) 9 mF
(a) 2 ´ 103 ms - 1 (b) 2 ´ 103 ms - 1
9. A set of coloured rings band are printed on the
(c) 4 ´ 103 ms - 1 (d) 4 ´ 109 ms - 1
carbon resistor having colours of first, second, third
and fourth band namely green, orange, red and
6. Electrostatics and Current gold, respectively, then the value of resistance is
Electricity (a) 53 ´ 102 ± 10% (b) 64 ´ 102 ± 5%
(c) 53 ´ 102 ± 5% (d) 64 ´ 102 ± 10%
1. The electrostatic capacitance depends on
(a) nature of the conductor 10. When two charges are placed in a medium of
(b) size of the conductor dielectric constant 20, then they attract to each
(c) charge given to the conductor other by a force of 5N, then force of attraction
(d) None of these between them, when they are placed in air at same
distance is
2. A charge Q is enclosed by a Gaussian surface of
(a) 25 N (b) 40 N (c) 45 N (d) 100 N
radius r. If the radius of the surface is doubled,
then outward electric flux will be 11. A metal foil of negligible thickness is introduced
(a) reduced to half (b) doubled between two plates of a capacitor at the centre.
(c) remains same (d) increase four times The capacitance of capacitor will be
3. In the arrangement shown in the figure, the charge (a) same (b) half
supplied by battery will be (c) double (d) zero
C C 12. A source of emf 20 V and internal resistance 4 W
transfers maximum power to a load, then the value
of load resistance will be
(a) 2W (b) 4W (c) 8W (d) 5W
V 13. When temperature of a conductor increases, then
relaxation time of its free electrons
1 2 (a) remains same (b) increases
(a) CV (b) 2 CV (c) CV (d) CV 2 (c) decreases (d) None of these
2

MODULE 1
www.jeebooks.in
Prep Analysis for NEET ~ Physics 17

14. The potential difference between A and B in the 20. In the given distribution, what is the value of i ?
following figure is
12V 4V
2A 6 Ω 9Ω 5Ω i 1Ω
A B 1Ω
4A
1Ω
(a) 12 V (b) 24 V 5A 2A
(c) 48 V (d) None of these 3A
1Ω 1Ω
15. The distance between two point charges is increased
by 10%. The force of interaction between them
(a) increased by 10% (a) 8 A (b) 0 (c) 2 A (d) 5 A
(b) decreased by 10%
(c) decreased by 17% 21. P , Q and R are three points in a uniform electric
(d) decreased by 21% field. The electric potential is
16. In a given Wheatstone’s network Q
P = 2 W, Q = 2 W, R = 2 W and S = 3 W, calculate the P
E
value of resistance with which S is to be shunted, R
so that bridge get balanced. (a) maximum at R (b) maximum at Q
(a) 2 (b) 3 (c) 6 (d) 9 (c) maximum at P (d) same at all three points
17. A voltmeter is connected to resistance R in parallel 22. A 500 pF capacitor is charged by a 200 V supply. It
as shown in the figure. The value of R for which is then disconnected from the supply and is
reading in voltmeter is 2V is connected to another uncharged 500 pF capacitor.
V The loss of electrostatic energy in this process is
(a) 2 ´ 10-6 J (b) 5 ´ 10-6 J (c) 3 ´ 10-6 J (d) 6 ´ 10-6 J
R 2Ω 23. A parallel plate capacitor is charged. If the plates
are pulled apart, then
(a) the capacitance increases
(b) the total charge increases
4V (c) the charge and potential difference remain the same
(a) 8W (b) 4W (c) 2W (d) zero (d) the potential difference increases

18. The following diagrams shows arrangement for 24. An electric field required to keep a water drop of
meter bridge experiment. The balance length XY mass m just to remain suspended, when charged
corresponding to null deflection of the with one electron is
em mg
galvanometer is x. What would be the balance (a) (b) (c) emg (d) mg
g e
length, if radius of wire of XZ is doubled?
25. What is the angle between the electric dipole
moment and the electric field strength due to it on
R1 R2 the equatorial line?
(a) 0° (b) 90° (c) 45° (d) 180°
G
26. A gang capacitor is formed by interlocking a 7
X Z plates as shown in the figure. The distance
Y
x between the consecutive plates is 0.886 cm and the
2
(a) x (b) 2 x2 (c)
x
(d) 4x overlapping area of plate is 15 cm 2. The
2 capacitance of gang capacitor will be
19. The energy required to charge a parallel plate
capacitor having plate separation d and plate area
of cross-section A and the uniform electric field E
between plates is given by
1
(a) e0 E 2 A × d (b) e0 E 2 A × d
2 (a) 1.5 pF (b) 19 pF
1 (c) 9 pF (d) 38 pF
(c) e0 E 2 / A × d (d) e0 E 2 / A × d
2

MODULE 1
www.jeebooks.in
18 NEET Test Drive

27. Charge Q of mass m revolves around a stationary 33. In an experiment, charges on four tiny oil drops
charge q in a circular orbit of radius r. Then, the suspended in an electrostatic field is measured by
periodic time of charge Q would be 4.2 ´ 10-19 C, 2.8 ´ 10-19 C, 7.0 ´ 10-19 C and
1/ 2 1/ 2 5.6 ´ 10-19 C, respectively. Then, the charge of an
é 16 p 2 e0 mr ù é 16 p 2 e0 mr 2 ù
(a) ê ú (b) ê ú electron is deduced to be
êë Qq úû êë Qq úû (a) 2.1 ´ 10-19 C (b) 2.8 ´ 10-19 C
(c) 1. 6 ´ 10-19 C . ´ 10-19 C
1/ 2 1/ 2
é 16 p 3 e0 mr 3 ù é Qq ù (d) 14
(c) ê ú (d) ê 3 3ú
êë Qq úû êë 16 p e0 mr úû 34. A charge Q is placed at the centre of cube. Then,
total electric flux through upward and downward
28. If E denotes electric field in a uniform conductor, I
faces is
is corresponding current through it and vd is drift Q Q Q Q
velocity of e - , then the correct graph is (a) (b) (c) (d)
e0 2 e0 6e0 3e0
y y
35. For a uniform positive distribution of charge over
spherical shell of radius R, the potential will be
vd vd maximum
(a) (b)
(a) at the centre
3R
(b) at a distance from the surface
x x 2
E E (c) on the surface
y y (d) Both (a) and (c)
36. Two charges of 1 mC and 2mC are placed 0.3 m apart
as shown in the figure. A third charge of 3 mC is
(c) vd (d) vd moved along the arc of a circle of radius 0.4 m from
C to D, then change in potential energy of the
system is
x x 3µC
E E
C
29. The carbon resistor of ( 54 ± 10-2 ) kW to be marked
with a set of coloured co-axial rings or bands, then 0.4m
the colour code sequence will be
(a) Orange-Red-Green-Gold
(b) Yellow-Orange-Red-Silver 0.3m B
(c) Green-Yellow-Orange-Gold 1µC
A 2µC D
(d) Green-Yellow-Orange-Silver
(a) 0.432 J (b) 0.863 J (c) 0.608 J (d) 0.176 J
30. In the following figure three 60 W, 120 V light
bulbs are connected across a 120 V power line. The 37. 20 cells each of emf 2V and internal resistance
total power dissipated in the three bulbs is 0.5 W are connected in series and 5 such rows are
connected in parallel across the external resistance
of R W, then current through R W resistor is
maximum, if
(a) 8W (b) 4W (c) 2W (d) 1W
A B C
38. Match the following Column I with Column II and
choose the correct option from the codes given below.
(a) 40 W (b) 60 W (c) 90 W (d) 180 W
Column I Column II
31. If the resistance of a wire is 50 W. It is heated till it I. Electrical capacity A. [M L T 4 A2 ]
-1 -3
melts and stretches to four times its original
length, then change in resistance will be II. Permittivity of free space B. [M1L2 T -3 A -1 ]
(a) 800 W (b) 750 W (c) 200 W (d) 600 W III. Electrical potential C. [M1L2 T -2 ]
32. A charged body of mass 9 g and carrying charge of IV. Electrical energy D. [M-1L-2 T 4 A2 ]
0.5 mC is approaching a fixed charge of 0.8 mC with Codes
a velocity of 8 m/s. The distance of closest approach I II III IV I II III IV
of the charge is (a) A D B C (b) D A C B
(a) 1.25 cm (b) 1.25 m (c) 2.50 cm (d) 2.50 m (c) D A B C (d) A D C B

MODULE 1
www.jeebooks.in
Prep Analysis for NEET ~ Physics 19

39. A charge particle of 8 mC moves from A to D along 43. 81 cells each of emf 4 V and internal resistance 1 W
the path ACD in a uniform electric field of 200 N/C are to be arranged, so as to produce maximum
along positive direction of X-axis. The work done current in a 9 W resistance. Each row is to contain
by the electric field during this process is equal number of cells. The number of cells in each
row will be
5 cm C (a) 27 (b) 24 (c) 36 (d) 18
D
4 cm 44. If a parallel plate capacitor has capacity C. On
cm
3
inserting a dielectric slab of relative permittivity K
45°
and thickness equal to one-fourth of the plate
X separation is placed between the plates, then its
A
capacity becomes C¢. The value of C ¢/ C will be
(a) - 6.4 ´ 10-3 J (b) - 4.53 ´ 10-3 J (a)
K
(b)
2K
(c)
5K
(d)
4K
(c) 4.53 ´ 10-3 J (d) 6.4 ´ 10-3 J 2(K + 1) K+1 4K + 1 3K + 1

40. Three point charges A, B and C are at a distance of 45. The electric potential V at any point ( x , y , z ) in
2m, 4m and 2m from an infinitely long charged space is given according to relation V = 2x 2V. The
wire of linear density l coulomb/meter. A charge q electric field intensity at the point ( 2m , 0, 2m ) will
is taken from A to B, B to C and finally from C to A. be
Then, the correct statement about the work done in (a) - 4 Vm -1 (b) + 4 Vm -1 (c) - 8 Vm -1 (d) 8 Vm -1
above process is
(a) WAB = - WBC (b) WAB = 2WBC
(c) WAB = 0 (d) WCA ¹ 0 7. Magnetic Effect of Current,
41. Two batteries of emf E1 and E2 (E2 > E1 ) and Magnetism, EMI and AC
internal resistance r1 and r2 , respectively are 1. An electron and a proton enter a magnetic field
connected in parallel as shown in the figure. perpendicularly. Both have same kinetic energy.
Which of the following is true?
E1 r1 (a) Trajectory of electron is less curved
(b) Trajectory of proton is less curved
(c) Both trajectories are equally curved
A B (d) Both move on straight line path
E2 r2 2. The net magnetic flux passing through a closed
surface is always
(a) zero (b) positive
(a) The equivalent emf E is smaller than E1 (c) negative (d) None of these
(b) The equivalent emf E = E1 + E2 3. The secondary to primary turns ratio of
(c) The equivalent emf E is equal to zero transformer is given as 2 : 3. If the current passing
(d) The equivalent emf Eeq of two cells is between A and B through the primary coil is 3A. Find the current
always through the load resistance.
(a) 4.5 A (b) 1.5A (c) 2A (d) 1A
42. Two cells having emf’s E1 and E2 are connected to a
potentiometer wire PQ as shown in the figure. If 4. Torque on a current carrying rectangular loop
the balancing length for the two combinations of placed in uniform magnetic field is maximum,
the cells are 150 cm and 200 cm, then ratio of E1 when its axis makes an angle with the direction of
and E2 is magnetic field is
V (a) 0° (b) 180° (c) 90° (d) 120°
5. For L-C-R circuit, the power transferred from the
200 cm driving source to the driven oscillator is
P
150 m
Q P = I 2Z cos f.
E1 E2 (a) Here, the power factor cos f ³ 0, P ³ 0
+ – (b) The driving force can give no energy to the oscillator
G
– + (P = 0) in some cases
+ –+ –
G (c) The driving force cannot syphon out (P < 0) the
E1 E2 energy out of oscillator
(a) 3 : 4 (b) 1 : 7 (d) The driving force can take away energy out of the
(c) 4 : 3 (d) 7 : 1 oscillator

MODULE 1
www.jeebooks.in
20 NEET Test Drive

6. A conductor of length l is moving with velocity v in 16. In an AC circuit, the emf E and current I at any
uniform magnetic field B perpendicular to it, then æ pö
instant is given by E = 40 sin ç 314t - ÷ and
induced emf across the conductor is è 2ø
Bv 2 æ p ö
(a) (b) Bvl (c) Bvl 2 (d) 0 I = 4 sin ç 314t - ÷ , then the average power in the
l è 6ø
7. Two parallel wires having current 20 A each are circuit over one cycle of AC is
separated by a distance 20 cm. If direction of (a) 80 W (b) 40 W (c) 20 W (d) 60 W
currents is both wire is same, then the force
exerted by one wire on the other per unit length is 17. The magnetic flux through a coil perpendicular to
-4 -4 the plane is varying, according to the relation
(a) 4 ´ 10 , repulsive (b) 4 ´ 10 , attractive
f = ( 2t 2 + 3t - 1) Wb, then magnitude of induced
(c) 2 ´ 10- 4 , attractive (d) 2 ´ 10- 4 , repulsive
emf at t = 3 s, is
8. The magnetic potential energy stored in a inductor (a) 15 V (b) 26 V (c) 8 V (d) 0 V
coil is 40 mJ, when the current through the inductor 18. An electron and a proton are moving on straight
is 80 mA. Then, the inductance of the coil is parallel path with same velocity. They enter into a
(a) 12.5 H (b) 1.25 H (c) 25 H (d) 15 H uniform magnetic field in perpendicular direction.
9. If number of turns per unit length of a coil of If time period of electron and proton are Te and T p,
solenoid is doubled, then the self-inductance of the then
solenoid will be (a) Te = Tp (b) Te > Tp (c) Te < Tp (d) Te = 2Tp
(a) doubled (b) increases four times 19. A moving coil galvanometer given full scale
(c) remains unchanged (d) halved deflection when a current of 0.005 A is passed
10. The value of relative permeability (m r ) and through its coil. It is converted into a voltmeter
susceptibility ( c m ) for diamagnetic substance is reading upto 5V by using an external resistance of
(a) m r = 0.5, c m = - 0.5 (b) m r = 0.5, c m = 0.5
975 W. What is the resistance of the galvanometer
(c) m r = 1.5, c m = - 0.5 (d) m r = 1.5, c m = - 0.5
coil?
(a) 30 W (b) 25 W (c) 50 W (d) 40 W
11. When a magnetic dipole of dipole moment m is
placed in a uniform magnetic field B, then its 20. If a current is passed through a spring, then the
potential energy is maximum, when spring will be
(a) m is anti-parallel to B (b) m is like parallel to B (a) expand (b) remains same
(c) m is perpendicular to B (d) None of these (c) bend (d) compress

12. Lenz’s law is based on conservation of 21. A proton, a deutron and an a-particle enter a
(a) charge (b) momentum (c) current (d) energy magnetic field perpendicular to field with same
velocity. What is the ratio of the radii of circular
13. The current flowing through a wire having length paths?
L is I. If the wire is bent in a circular form, then (a) 1: 2 : 2 (b) 2 : 1 : 1 (c) 1 : 1 : 2 (d) 1 : 2 : 1
its magnetic dipole moment will be
IL I2 L I 2 L2 IL2 22. The reduce the resonant frequency in an L-C-R
(a) (b) (c) (d) series circuit
4p 4p 4p 4p
(a) resistance should be increased
14. The value of magnetic declination at a place on the (b) another capacitor should be added in parallel to the first
earth (c) The iron core of the inductor should be removed
(a) changes two times during each rotation of earth about its
(d) dielectric in the capacitor should be removed.
own axis
(b) changes four times during each rotation of earth about its 23. A transformer is used to light a 100W-110V lamp
own axis from 220 V mains. If main current is 0.5 A,
(c) changes only when earth reaches close to sun during each efficiency of transformer is
revolution (a) 90% (b) 95%
(d) does not depend on situation (a), (b) and (c) (c) 96% (d) 99%
15. Which of the following combination should be 24. The torque required to hold a small circular coil of
selected for better tuning of an L-C-R circuit used 10 turns, 2 ´ 10-4 m 2 area and carrying 0.5 A
for communication? current in the middle of a long solenoid of 103
(a) R = 20 W, L = 1.5 H, C = 35 mF turns m -1carrying 3 A current, with its axis
(b) R = 25 W, L = 2.5 H, C = 45 mF perpendicular to the axis of the solenoid, is
(c) R = 15 W, L = 3.5 H, C = 30 m F (a) 12 p ´ 10-7 Nm (b) 6 p ´ 10-7 Nm
(d) R = 25 W, L = 1.5 H, C = 45 mF (c) 4 p ´ 10-7 Nm (d) 2 p ´ 10-7 Nm

MODULE 1
www.jeebooks.in
Prep Analysis for NEET ~ Physics 21

25. A galvanometer needs 30 mV for a full scale 31. Magnetic field at the centre O produced by a
deflection of 60 divisions, then its voltage circulating point charge q as shown in the figure is
sensitivity is given by
(a) 2 ´ 10- 3 div/V (b) 0.5 ´10- 3 div/V v
q ω
(c) 2 ´ 103 div/V 3
(d) 0.5 ´10 div/V
r
^ ^ r
26. A charged particle with velocity v = x i + y j moves
^ ^ O
in a magnetic field B = y i + x j, then no force will
act on particle, if
(a) x > y (b) x < y
(c) x2 + y2 = 0 (d) x = y m 0 qv m 0 qv m 0 qw m 0 qw
(a) × (b) × (c) × (d) ×
2p r 4p r 4p r 2p r
27. In R-L-C series circuit, inductor L is removed from
the circuit. The phase difference between the 32. Three straight current carrying wire of current 2A
voltage and current in the circuit is p/4. If each are placed perpendicular to plane of paper
according to figure. Direction of currents in each
capacitor C is removed from the circuit, then phase
conductor is same, then the value of force per unit
difference between voltage and current is again
length on the middle of wire Y is
p/4. The power factor of the circuit is
3 1 Y √2 m Z
(a) (b) (c) 1 (d) 0 × ×
2 2
√2 m
28. Two semicircles of radii r1 and r2 having current I
are connected according to figure. The magnetic X × .
field at center O will be (a) 16 ´ 10- 7 N (b) 2 ´ 10- 7 N
(c) 8 ´ 10- 7 N (d) 4 ´ 10- 7 N
r1 33. A transformer having efficiency 80% is working on
O
200 V and 4 kW power supply. If the current in the
I secondary coil is 4 A, then the voltage across the
r2
secondary coil and the current in the primary coil
m 0 I é r1 - r2 ù m 0 I é r2 - r1 ù
respectively are
(a) ê ú (b) ê ú (a) 500 V, 20 A (b) 600 V, 20 A
4 ë r1r2 û 4 ë r1r2 û
(c) 800 V, 20 A (d) 800 V, 15 A
m 0 I é r1 + r2 ù m 0 I é r1 + r2 ù 34. A galvanometer has a resistance of 100 W. A
(c) ê ú (d) ê ú
2 ë r1r2 û 4 ë r1r2 û potential difference of 100 mV between its
terminals gives a full scale deflection. The shunt
29. A conducting ring of mass 3 kg and radius 0.75 m is resistance required to convert it into an ammeter
placed on a smooth horizontal plane. The ring carries
reading upto 5 A is
a current of 6 A. A horizontal magnetic field of 12 T is
then applied to the ring as shown in the figure. Then, (a) 0.01 W (b) 0.02 W
the initial angular acceleration of the ring will be (c) 0.03 W (d) 0.04 W
35. In the network shown below
15 V
B
A 1Ω i B
5 mH
If current is 5 A and is decreasing at the rate of
(a) 150.7 rad/s 2 (b) 113 rad/s 2
103 A/s, then V B - V A will be
(c) 226 rad/s 2 (d) 135 rad/s 2
(a) 20 V (b) 10 V
30. If the flux linked with a circuit is given by (c) 15 V (d) 5 V
f = t3 + 3t - t. The graph between time (X-axis) and
36. In a tangent galvanometer, when a current of
induced emf (Y -axis) will be a
10 mA is passed, the deflection is 31°. By what
(a) straight line through the origin percentage the current has to be increased, so as to
(b) straight line with positive intercept produce a deflection of 42°? (Use, tan 31° = 0.6,
(c) parabola through origin tan 42° = 0.9)
(d) parabola not through origin (a) 10% (b) 20% (c) 50% (d) 80%

MODULE 1
www.jeebooks.in
22 NEET Test Drive

37. An iron rod is placed inside a solenoid in which 45. An electron of mass 9.1 ´ 10- 31 kg is projected in a
number of turns is 500. The relative permeability perpendicular uniform magnetic field of 4 ´ 10- 2 T,
of the rod is 100. If a current of 1 A is flowing in moves on circular path of radius 5 mm, then
the solenoid, then the magnetisation of the rod kinetic energy of electron is
will be (a) 5.6 ´ 10- 16 J (b) 11.2 ´ 10- 10 J
(a) 5 ´ 104 A/m (b) 4.9 ´ 104 A/m (c) 2.8 ´ 10- 16 J (d) 1.6 ´ 10- 10 J
(c) 5 ´ 105 A/m (d) 6 ´ 104 A/m

38. A coil of resistance 40 W and inductive reactance 8. Electromagnetic Waves and Optics
30 W is connected to a battery of 15 V, then the 1. A convex lens is dipped in a liquid, whose
total current in the coil is refractive index is greater than the refractive index
(a) 0.3 A (b) 0.375 A (c) 0.5 A (d) 0.214 A of lens material, then convex lens will behave as
39. The magnetic susceptibility is negative for (a) convex lens (b) concave lens
(a) paramagnetic material only (b) plane glass (d) None of these
(b) diamagnetic material only 2. An object is placed at the focus of convex mirror.
(c) ferromagnetic material only The image will be at
(d) None of the above (a) centre of curvature (b) focus
-2
(c) infinity (d) no image will be formed
40. A magnetic field of 2 ´ 10 T acts at right angles
2 3. If an unpolarised light is polarised by reflection,
to a coil of area 100 cm with 50 turns. The
then the angle between refracted and reflected
average emf induced in the coil is 0.1 V, when it is
light is
removed from the coil in t s. The value of t is
(a) 120º (b) 60º (c) 180º (d) 90º
(a) 0.1 s (b) 0.01 s (c) 1 s (d) 10 s
4. In Young’s double slit experiment, the central
41. In an AC circuit, an alternating voltage
fringe is
V = 400 2 sin 100 t volt is connected to a capacitor
(a) always bright (b) always dark
of capacity 1 mF. The rms value of the alternating (c) Neither bright nor dark (d) None of these
current in the circuit is
(a) 2 ´ 10- 2 A (b) 4 ´ 10- 3 A 5. A Young’s double slit experiment uses a
monochromatic source. The shape of the
(c) 8 ´ 10- 3 A (d) 4 ´ 10- 2 A
interference fringe formed on a screen is
42. In an inductor, the current I varies with time t as (a) hyperbola (b) circle
I = 4 + 4 t. If the induced emf in the inductor is 8 (c) straight line (d) parabola
mV, then what is the self-inductance? (Here, 6. The maximum number of possible interference
current is an ampere and time is in second) maxima for slit-separation equal to twice the
(a) 2 ´ 10- 3 H (b) 6 ´ 10- 6 H wavelength in Young’s double slit experiment is
(c) 6 ´ 10- 3 H (d) 4 ´ 10- 3 H (a) infinite (b) five
(c) three (d) zero
43. A coil of self-inductance L is connected in series
with a bulb and an AC source. Brightness of the 7. A light ray passes through air to medium of
bulb decreases, when refractive index m, such that angle of refraction is
(a) number of turns in the coil increased half of angle of incidence, then the angle of
(b) an iron rod is inserted in the coil incidence will be
1 m m m
(c) frequency of AC source decreased (a) cos - 1 (b) cos - 1 (c) 2 cos - 1 m (d) 2 cos - 1
(d) None of these 2 2 2 2

44. In the given figure, the magnetic field induction at 8. In Young’s double slit experiment, the intensity of
point O will be maxima is I. If the intensity of light emitted by
i each slit is doubled, then the intensity of the
maxima will be
O
I
2r (a) (b) 2I (c) 4I (d) I
2
9. The refractive index of the material of an
m0 i m i
(a) (b) æç 0 ö÷ (p + 2 ) equilateral prism for which the angle of minimum
2 pr è 4pr ø
deviation is 30° is
m i m i
(c) æç 0 ö÷ (p + 1) (d) æç 0 ö÷ (p - 2 ) (a) 2 (b)
1
(c) 2 (d) 3
è 4pr ø è 4pr ø 2

MODULE 1
www.jeebooks.in
Prep Analysis for NEET ~ Physics 23

10. A ray is incident at 20º on one refracting surface of 21. The angle of incidence at which reflected light is
a prism of angle 55º suffers a deviation of 50º, then totally polarised for reflection from air to glass
what is the angle of emergence? (refractive index n) is
1
(a) 25º (b) 85º (a) sin- 1 n (b) cos - 1 n (c) tan- 1 n (d) tan- 1
(c) 75º (d) 125º n
11. When a monochromatic red light is used instead of 22. The nature of the image formed by an eye lens of a
blue light in a convex lens, its focal length will be compound microscope is
(a) not depends on the colour of light (a) real and diminished (b) real and enlarged
(b) remains same (c) virtual and diminished (d) virtual and enlarged
(c) increase 23. A person suffering from myopia can see only
(d) decreases (a) near objects clearly
12. A person of six feet in height can see his full size (b) far objects clearly
erect image in a mirror of 2 feet in height. The (c) the objects at least distance of distinct vision
mirror has to be (d) None of the above
(a) plane or convex (b) plane or concave
(c) necessarily convex (d) necessarily concave 24. If bG and b R are the fringe width for green and red
colour of light in Young’s double slit experiment,
13. The critical angle of prism is 30°. The velocity of
then which of the following is true?
light in the medium is
(a) bG = b R (b) b R > bG
(a) 1.5 ´ 108 m/s (b) 3 ´ 108 m/s
(c) b R < bG (d) None of these
(c) 4.5 ´ 108 m/s (d) 9 ´ 108 m/s
25. In Young’s double slit experiment, the width of the
14. If two lenses of power 4.5 D and - 2.5 D are placed fringe obtained on screen is 6 mm. If the entire
in contact, then power of combination of lenses apparatus is immersed in a liquid of refractive
will be index 1.5, then width of fringe will be
(a) 3 D (b) - 3.5 D (c) 2 D (d) - 2 D (a) 4 mm (b) 9 mm (c) 0.15 mm (d) 6 mm
15. The width of fringes produced in diffraction 26. Resolving power of a reflecting telescope depends
pattern are
upon
(a) equal (b) unequal
(a) directly proportional to diameter of objective lens
(c) equal only bright fringes (d) equal only dark fringes
(b) the focal length of objective lens
16. Electromagnetic wave are transverse in nature is (c) inversely proportional to the frequency of light
evident by (d) All of the above
(a) polarisation (b) interference
(c) reflection (d) diffraction 27. A plano-convex lens is kept in contact with a
plano-concave lens. Their plane surfaces are
17. The pressure exerted by an electromagnetic wave parallel to each other. If lenses are made of
2
of intensity I(W/ m ) on a non-reflecting surface is different materials of refractive indices m1 and m 2
(where, c is the velocity of light) and R is the radius of curvature of the curved
(a) I / c (b) Ic 2 (c) Ic (d) I / c 2 surface of the lenses, then the focal length of the
18. The magnetic and electric fields in an combination is
R R R 2R
electromagnetic waves are (a) (b) (c) (d)
(a) in phase and parallel to each other 2(m1 + m 2 ) 2(m1 - m 2 ) (m1 - m 2 ) (m 2 - m1 )
(b) in phase and perpendicular to each other 28. A convex lens of focal length 12.5 cm is used as a
(c) in opposite phase and parallel to each other magnifying glass. If the near point of the observer
(d) in opposite phase and perpendicular to each other is 25 cm from the eye and the lens is held close to
19. The displacement current is produced across the the eye, then angular magnification of magnifying
gap between the two plates of a capacitor, when glass is
the charge on the capacitor (a) 3 (b) 4 (c) 5 (d) 2
(a) increases (b) decreases 29. When a prism is immersed in a liquid of refractive
(c) Both (a) and (b) (d) does not change 4
index , then the angle of minimum deviation
20. An unpolarised light of intensity I 0 incident on a 3
produced by prism is reduced to [given, refractive
polariser, then the intensity of light which does not 3
get transmitted is index of prism in air = ]
2
(a) I0 (b) I0 / 2 (a) one third (b) one fourth
(c) I0 / 4 (d) zero (c) one half (d) None of these

MODULE 1
www.jeebooks.in
24 NEET Test Drive

2 38. A light beam travelling in the x-direction is


30. A biconvex lens has a focal length times the
3 æ xö
described by the electric field Ex = 200 sin w ç t - ÷.
radius of curvature of either surface, then the è cø
refractive index of lens material is An electron is constrained to move along the
(a) 1.5 (b) 2.75 (c) 4.3 (d) 1.75 y-direction with a speed of 106 m /s, then maximum
31. Two beams of light of intensity I1 and I 2 interfere magnetic force on electron is
(a) 1. 07 ´ 1017 N (b) 1. 07 ´ 10- 19 N
to give an interference pattern. If the ratio of
(c) 2 . 5 ´ 1015 N (d) 9.2 ´ 1017 N
maximum intensity to minimum intensity is 64 : 9,
then I1 / I 2 is 3
39. A lens of refractive index is immersed into the
(a) 8 : 3 (b) 64 : 9 (c) 121 : 25 (d) 25 : 121 2
4
32. The observation of diffraction produced by sound water of refractive index , then the ratio of focal
wave is more easier than the diffraction produced 3
by light waves because lengths of lens in air and in water is
(a) 1 : 4 (b) 4 : 1 (c) 2 : 1 (d) 1 : 2
(a) speed of sound is too much less than speed of light
(b) sound waves require medium for propagation 40. The maximum intensity in case of interference of
(c) sound waves are longitudinal 10 identical waves each of intensity I 0, if
(d) wavelength of sound waves is too much greater than interference is coherent, is
wavelength of light waves 1 1
(a) I0 (b) 10 I0 (c) I0 (d) 100 I0
10 100
33. In the biprism experiment, fringes of width 0.08
mm are observed on a screen at a distance of 2m 41. Match Column I with Column II and select the
from the source. When a convex lens is placed correct option from the codes given below.
between the source and the screen, then images of
Column I Column II
slits in two positions are 0.05 mm and 0.02 mm (EM wave) (its association/application)
respectively. The wavelength of light used is
A Infrared waves (i) To treat muscular strain
(a) 1.2 nm (b) 0.2 nm
(c) 0.126 nm (d) 0.8 nm B Radio waves (ii) For broadcasting
C X-rays (iii) To detect fracture of bones
34. A person can see clearly objects only when it lies
between 60 cm and 400 cm from his eye. In orderto D Ultraviolet rays (iv) Absorbed by the ozone layer
increase, the maximum distance of distinct vision of the atmosphere
to infinity, the type and power of the correcting Codes
lens, then the person has to use will be A B C D A B C D
(a) concave, - 0.25 D (b) convex, + 0.25 D (a) iii ii i iv (b) i ii iii iv
(c) concave, - 0. 3 D (d) convex, + 0. 4 D (c) iv iii ii i (d) i ii iv iii

35. If the focal length of objective lens is increased, 42. Light of wavelength 400 nm falls normally on a slit
then magnifying power of of width 1 m m producing Fraunhoffer diffraction
(a) microscope will increase but that of telescope decrease pattern on a screen. The angular width of central
(b) microscope and telescope both will increase maxima is
(c) microscope and telescope both will decrease (a) sin- 1(0.2 ) (b) 2 sin- 1(0.2 ) (c) sin- 1(0. 4) (d) 2 sin- 1(0. 4)
(d) microscope will decrease but that of telescope will 43. Light of wavelength 5 ´ 10- 6 m is diffracted by an
increase
aperture of width 8 ´ 10- 4 m. The distance travelled
36. The electric and the magnetic field associated with by the diffracted beam does the spreading due to
an electromagnetic waves, propagating along the diffraction become greater than the width of the
+ Z-axis, can be represented by aperture is
(a) [E = E0k$ , B = B0 $i ] (b) [E = E0 $j, E = B0 $j ] (a) 0.6 m (b) 0.5 m (c) 0.128 m (d) 0.120 m
(c) [E = E $j, B = B k$ ]
0 0 (d) [E = E0 $i , B = B0 $j ] 44. White light is used to illuminate the two slits in a
37. The electric field associated with an Young’s double slit experiment. The separation
electromagnetic wave in vacuum is given by between slits is b and the screen is at a distance
E = $i 40 cos( kz - 6 ´ 108 t ), where E, z and t are in d( > > b) from the slits. At a point on the screen
volt/m, metre and second, respectively. The value directly in front of one of the slits, certain
of wave factor is wavelengths are missing. One of the missing
wavelengths is
(a) 2 m- 1 (b) 0.5 m- 1
(a) l = b 2 / d (b) l = 2 b 2 / d
(c) 6 m- 1 (d) 3 m- 1 (c) l = 3b 2 / d (d) l = 2 b 2 / 3d

MODULE 1
www.jeebooks.in
Prep Analysis for NEET ~ Physics 25

45. Two periodic waves of intensities I1 and I 2 passes 7. The radiations emitted from a radioactive material
through a region at the same time in the same separated into two groups A and B, when a
direction. The sum of maximum and minimum magnetic field is directed into the plane of the
intensities is paper. According to figure, names of radiations A
(a) ( I1 + I2 )2 (b) ( I2 - I2 )2 and B are respectively.
(c) 2(I1 + I2 ) (d) I1 + I2 A B
× × × ×

9. Dual Nature of Matter and × × × ×


Atom & Nuclei
× × × ×
1. When intensity of incident photon increases in
photoelectric experiment, then
× × × ×
(a) kinetic energy of emitted electron increases
(b) value of stopping potential increase
(c) photoelectric current increases Radioactive Material
(d) All of the above
(a) g, a (b) a, g
2. An electron and proton are accelerated through (c) b, g (d) a, b
same potential, then the ratio of their de-Broglie 8. The half-life of a radioactive substance is 15 days.
waves ( l p / l e ) will be (where, m e and m p are 3
mass of electron and proton, respectively) The time taken for of its original mass to
4
mp mp disintegrate is
(a) (b)
me me (a) 15 days (b) 45 days
(c) 30 days (d) 60 days
me me
(c) (d) 1
mp mp 9. The activity of a radioactive element reduces to
32
3. The rest mass of photon moving with velocity c of its original value in 20 years. The half-life of
having wavelength l is element is
h lc (a) 2 yr (b) 4 yr
(a) (b)
lc h (c) 5 yr (d) 6 yr
(c) clh (d) zero 10. Which of the following represent the correct graph
4. Photoelectric emission takes place between the intensity of incident photons (I) and
(a) When frequency of incident radiation is greater than its
the photoelectric current i?
threshold frequency
(b) When wavelength of incident photon is less than its
threshold wavelength i i
(a) (b)
(c) When frequency of incident radiation is equal to its
threshold frequency
(d) All of the above
x x
5. The de-Broglie wavelength for electron by I I
accelerated 100 V potential is
(a) 1.23 nm i i
(b) 1.23Å (c) (d)
(c) 1.2 nm
(d) 1.25Å
6. Mass defect in nuclear fission reaction is 0.04%, x x
I I
then the energy liberated in 2 kg fission reaction
will be 11. The nuclei 14 13
7 N and 6 C can be described as
(a) 7.2 ´ 1013 J (b) 7 .2 ´ 1012 J
(a) isobars (b) isotones
(c) 3. 6 ´ 1013 J (d) 3. 6 ´ 1012 J (c) isotopes of carbon (d) isotopes of nitrogen

MODULE 1
www.jeebooks.in
26 NEET Test Drive

12. According to Bohr’s model, angular momentum of 22. The ratio of de-Broglie wavelength of molecules of
electron moving in circular orbit is integral hydrogen and helium which are at temperatures
multiple of 127°C and 27°C respectively is
2p h (a) 3 : 2 2 (b) 2 2 : 3
(a) (b)
h 2p (c) 3 : 1 (d) 2 : 3
h p
(c) (d) 23. Effective mass of a photon having wavelength l is
p h
cl hl
(a) (b)
13. If atoms are arranged in increasing order of mass h c
number, then their binding energy per nucleon h ch
(c) (d)
(a) increases (b) decreases cl l
(c) increase then decrease (d) remains same
240
24. The energy of a photon is 3 keV, then its linear
14. A radioactive element decays into 86Y 232,
92 X momentum is
then number of a and b - particles emitted during . ´ 10-24 kg-m/s
(a) 16 (b) 2.4 ´ 10-24 kg-m/s
this decays respectively are . ´ 10-21 kg-m/s
(c) 16 (d) 0
(a) 2, 1 (b) 1, 2
(c) 2, 2 (d) 1, 3 25. For a radioactive sample ratio of mean life and half
life is about
15. In ground state radius of Bohr’s hydrogen is 0.53Å. (a) 1.2 (b) 1.3
Radius of atom is 2 nd excited state is (c) 1.4 (d) 1.5
(a) 0.059 Å (b) 1.59 Å
(c) 4.77 Å (d) 2.12 Å 26. A source S1 is producing, 1015 photons/s of
wavelength 5000 Å. Another source S 2 is producing
16. The energy (E) of the electron in Bohr’s orbit is 1.02 ´ 1015 photons per second of wavelength
related to principal quantum number n as
1 1
5100Å. Then, the ratio of source power of S 2 to
(a) E µ 2
(b) E µ source power of S1 is
n n
(a) 1 : 2 (b) 2 : 1
(c) E µ n2 (d) E µ n
(c) 1 : 1 (d) 3 : 1
17. Series limit ( l min ) for Balmer series is (R-Rydberg 27. If the kinetic energy of the particle is increased to
constant) 9 times its previous value, then percentage change
1 9 4 5 in the de-Broglie wavelength of the particle is
(a) (b) (c) (d)
R R R R (a) 75%
18. If the ionisation energy of a H-atom is E, then its (b) 25%
ionisation energy in the second excited state is (c) 66.6%
E E (d) 33.3%
(a) (b)
9 4
E 28. In a discharge tube, ionisation of enclosed gas is
(c) (d) E produced due to collisions between
2
(a) positive ions and neutral atoms / molecules
19. When frequency of incident photon increases in (b) negative electrons and neutral atoms / molecules
photoelectric emission, then kinetic energy of (c) photons and neutral atoms/ molecules
liberated electrons (d) neutral gas atoms / molecules
(a) increases (b) decreases
(c) does not change (d) None of these 29. A photosensitive metallic surface has work
function, hv0. If photons of energy 2hv0 fall on this
20. Maximum kinetic energy of a liberated electron surface, the electrons come out with a maximum
from metal surface in photoelectric effect is velocity of 4 ´ 106 m/s. When the photon energy is
1.6 ´ 10-20 J, then magnitude of stopping potential increased to 5hv0, then the maximum velocity of
will be photoelectrons will be
(a) 0.001 V (b) 0.1 V (a) 2 ´ 106 m/s (b) 2 ´ 107 m/s
(c) 0.01 V (d) 1 V (c) 8 ´ 105 m/s (d) 8 ´ 106 m/s
21. Slope of the graph of cut-off voltage versus 30. The momentum of a photon of an electromagnetic
frequency of incident light in an experiment on radiation is 3.3 ´ 10-29 kg-m/s, then the frequency
photoelectric effect is of associated with it is [given, h = 6.6 ´ 10-34 J-s
(a) constant
and c = 3 ´ 108 m/s]
(b) depends on frequency of incident light
(a) 3 ´ 1013 Hz . ´ 1013 Hz
(b) 15
(c) depends on work function of metal
. ´ 1015 Hz
(c) 15 (d) 4.5 ´ 1013 Hz
(d) Both (a) and (c)

MODULE 1
www.jeebooks.in
Prep Analysis for NEET ~ Physics 27

31. Thermions are 42. Hydrogen atom in ground state is excited by a


(a) Electrons (b) Protons (c) Neutrons (d) a-particles monochromatic radiation of l = 975Å. Number of
32. A particle of mass 1 mg has the same wavelength spectral lines in the resulting spectrum emitted
as an electron moving with a velocity of will be
3 ´ 106 ms -1. The velocity of the particle is (mass of (a) 3 (b) 2 (c) 6 (d) 10
electron = 91 . ´ 10-31 kg) 43. Match the following Column I with Column II and
(a) 2.7 ´ 10-18 ms -1 (b) 9 ´ 10-2 ms -1 choose the correct option from the codes given
(c) 3 ´10-31ms -1 (d) 2.7 ´ 10-21ms -1 below.
33. The number of photoelectrons emitted for light of a Column I Column II
frequency n (higher than the threshold frequency
v0) is proportional to I. Thomson atomic A. Fixed for an atom
(a) n - n0 (b) threshold frequency (n0 ) model
(c) intensity of light (d) frequency of light (n) II. Rutherford atom B. Stationary orbits model
34. In a photoelectric emission, work function for III. Bohr atom model C. Charge and mass are
metal surface is 1.8 eV. The kinetic energy of most distributed uniformly in a
energetic electrons is 0.5 eV, then the stopping sphere
potential will be
IV. Ionisation potential D. Nucleus
(a) 1.2V (b) 1 V (c) 0.8 V (d) 0.5 V
Codes
35. Monochromatic light of frequency 6.0 ´ 1014 Hz
is produced by a laser. The power emitted is I II III IV I II III IV
2 ´ 10-3 W. The number of photons emitted on the (a) A B C D (b) D B A C
average, by the source per second is (c) C D B A (d) A C D B
(a) 5 ´ 1015 (b) 5 ´ 1016
44. Energy E of a hydrogen atom with principal
(c) 5 ´ 1017 (d) 5 ´ 1014 -13.6
quantum number n is given by E = eV. The
36. q
A nucleus p A emits one a and one b particle, then n2
resulting nucleus is energy of a photon ejected, when the electron
(a) p -2 Aq -4
(b) p -1A
q -4
(c) p + 1A
q -2
(d) p Aq -4 jumps from n = 3 state to n = 2 state of hydrogen is
approximately
37. In the fusion reaction, 12 H + 12 H ¾® 12 He + 10 n (a) 1.5 eV (b) 0.85 eV
The masses of deuteron, helium and neutron (c) 3.4 eV (d) 1.9 eV
expressed in amu are 2.015, 3.017 and 1.009,
45. The total energy of electron in the ground state of
respectively. If 1 kg of deuterium undergoes
hydrogen atom is -13.6 eV. The kinetic energy of an
complete fusion, then find the amount of total
electron in the first excited state is
energy released ( 1 amu = 931.5 MeV).
(a) 3.4 eV (b) 6.8 eV
(a) 9.0 ´ 1013 J (b) 20 ´ 105 J (c) 5 ´ 1016 J (d) 8 ´ 105 J
(c) 13.6 eV (d) 1.7 eV
38. Remaining quantity (in %) of radioactive element
after 5 half lives is
(a) 4.125% (b) 3.125% (c) 31.1% (d) 42.125%
10. Electronic Devices
1. In a half-wave rectifier, the minimum number of
39. For a radioactive sample, decay constant is 0.05 / diode(s) used
year, then the time for which sample will decay by
75% is (a) 1 (b) 2 (c) 3 (d) 4
(a) 20 loge 2 (b) 10 loge 2 (c) 30 loge 2 (d) 40 loge 2 2. Minimum resistance of a diode occurs in
40. The mass number of He is 4 and that for sulphur is (a) reverse bias (b) forward bias
32. The radius of helium nuclei is smaller than (c) cut-off region (d) None of these
sulphur by
1 1 1 1 3. Find the value of current in the figure, if junction
(a) (b) (c) (d) diode is ideal.
4 2 8 2
4V p-n 300Ω +1V
41. The radius of a hydrogen like atom are r1 and r2. The
wavelength of photon, when it jumps from r2 to r1 is
2chr1r2 2ch(r2 - r1 ) ke 2 (r2 - r1 ) ke 2ch (a) zero (b) 10-2 A
(a) (b) (c) (d)
ke 2 (r2 - r1 ) ke 2 (r1 r2 ) hr1r2 r1r2 (c) 10-1 A (d) 10-3 A

MODULE 1
www.jeebooks.in
28 NEET Test Drive

4. The relation between number of free electrons ( n ) 13. In an intrinsic semiconductor, the fermi level is
is a semiconductor and temperature (T ) is given by (a) nearer to valence band than conduction band
(a) n µ T (b) equidistance from conduction band and valence band
(b) n µ T 2 (c) nearer to conduction band than valence band
(c) n µ T (d) overlaps with the conduction band
(d) n µ T 3 / 2
14. A n-type material is electrically
5. A piece of copper and other of germanium are (a) positive
cooled from the room temperature to 80 K, then (b) negative
(a) resistance of each will increase (c) neutral
(b) resistance of each will decrease (d) depends on the concentration of p impurities
(c) the resistance of copper will increase, while that of
germanium will decrease 15. In a common-emitter configuration of a transistor,
(d) the resistance of copper will decrease, while that of maximum current flows in which of the following?
germanium will increase (a) Base region (b) Collector region
(c) Emitter region (d) Equal in all the region
6. Highest resistivity is found in
(a) pure semiconductors 16. A pure Si crystal has 5 ´ 1028 atoms m -3 . It is
doped by 1 ppm concentration of pentavalent As.
(b) impure semiconductors
The number of holes is (approx).
(c) insulators
(Given, n = 1.5 ´ 1016m -3 )
(d) conductors
(a) 2.2 ´ 106 m-3 (b) 4.5 ´ 109 m-3
7. The p-n junction, which generates an emf when . ´ 106 m-3
(c) 62 . ´ 109 m-3
(d) 81
solar radiation falls on it with no external bias
17. A representation that shows all possible input
applied is a
combinations and the corresponding output
(a) zener diode (b) photodiode
combinations for a logic gate is known by
(c) solar cell (d) light emitting diode
(a) graph (b) truth table
8. The part of a transistor which is heavily doped to (c) boolean expression (d) None of these
produce a large number of majority carriers is
18. In Boolean expression, de-Morgan’s theorem is
(a) collector
represented by
(b) base
(a) A + B = B + A (b) A + 1 = 1
(c) emitter
(c) A + A = 0 (d) A + B = A × B
(d) None of the above
19. Associative law is correctly given as
9. In a CB configuration of a transistor, emitter-base
junction and collector-base junction are (a) A + B = A + B (b) A × (B × C ) = ( A × B) × C
(c) A + B = B + A (d) None of these
(a) forward, forward (b) forward, reverse
(c) reverse, reverse (d) reverse, forward 20. Which of the following is an universal gate?
(a) AND (b) OR
10. The correct relation between a and b in a
(c) NOT (d) NOR
transistor is
b b 21. The figure shows the symbol of a
(a) a = (b) a =
1+ b 1- b A
B C
(c) a = b (d) a = 1 - b
(a) AND (b) OR (c) NOT (d) NOR
11. When aluminium is added as an impurity to
silicon, then the resulting material is 22. The given truth table is of
(a) n-type semiconductor
A B X
(b) n-type conductor
(c) p-type conductor 0 0 1
(d) p-type semiconductor 0 1 0
1 0 0
12. A forward bias diode amongst the following is
1 1 0
0V 5V 7V 3.5V
(a) (b) (a) AND gate (b) OR gate
5V 3.2V (c) NAND gate (d) NOR gate
(c) (d) None of these

MODULE 1
www.jeebooks.in
Prep Analysis for NEET ~ Physics 29

23. Boolean expression of NAND gate is 30. Amongst the following circuit which will not show
(a) Y = A× B current in ammeter?
(b) Y = A×B
(c) Y = A+ B
(d) Y = A× B
(a) (b)
24. Symbolic representation of OR gate is
A A
(a) (b) + – + –

(c) (d)
(c) (d)
25. The following figure represents
A A
A + – + –

Y 31. Main cause of zener breakdown is


B (a) the base semiconductor being germanium
(b) production of electron-hole pairs due to breakage of covalent
(a) OR gate (b) NOT gate bonds
(c) NOR gate (d) AND gate (c) low doping
26. In a common-emitter amplifier, using output (d) Both (b) and (c)
resistance of 5000 W and input resistance of 32. Energy gap between valence band and conduction
2000 W, if the peak value of input signal voltage band of a semiconductor is of the order of
is 10 mV and b = 50, then peak value of output (a) zero (b) 10 eV (c) 1 eV (d) infinite
voltage is
(a) 5 ´ 10-6 V (b) 12.50 ´ 10-6 V
33. The figure of a half-wave rectifier is
(c) 125
. V (d) 125.0 V + +

27. Determine the number of donor atoms which


have to be added to an intrinsic germanium
semiconductor to produce an n-type – –
semiconductor of conductivity 5W-1cm -1, given (a) (b)
that the mobility of electrons in n-type Ge is + +
3900 cm 2V -1s -1. Neglect the contribution of
holes to conductivity. Take, charge on electron,
e = 1.6 ´ 10-19 C.
(a) 8. 013 ´ 1021 m-3 – –
(c) (d)
20 -3
(b) 6.2 ´ 10 m
(c) 5.3 ´ 1019 m-3 34. Match the following Column I with Column II and
(d) 4.8 ´ 1018 m-3 choose the correct option from the codes given below.

28. In a zener regulated power supply of a zener Column I Column II


diode with V z = 6V is used for regulation.
The load current is to be 4 mA and the I. n-p-n transistor A.
unregulated input is 10 V. The value of series
resistor RS is
(a) less than 5 W II. p-n-p transistor B.
(b) Infinite
(c) greater than 100 W
E C
(d) zero Light emmiting
III. C.
29. Reverse bias applied to a p-n junction diode diode
B
(a) lowers the potential barrier
(b) decreases the majority charge carriers E C
(c) raises the potential barrier IV. Zener diode D
(d) change the mass of p-n junction diode
B

MODULE 1
www.jeebooks.in
30 NEET Test Drive

Code 40. Amongst the following which is 0 in Boolean


I II III IV I II III IV algebra?
(a) A B C D (b) D A B C (a) A + 1 (b) A + A
(c) C D B A (d) B A C D (c) A × A (d) A × A + 1
35. In a common base configuration, I E = 0.96 mA,
41. To get an output 1 from the circuit shown in the
IC = 0.93 mA, The value of base current is figure, the input must be
(a) 1.96 mA (b) 189
. mA
A
(c) 0.03 mA (d) None of these
B
36. In a transistor in common-emitter configuration, 1
the ratio of power gain to voltage gain is C
(a) a (b) b
(c) b 2 (d) b a (a) A = 0, B = 1, C = 1 (b) A = 0, B = 0,C = 0
(c) A = 1, B = 0,C = 1 (d) A = 1, B = 1,C = 1
37. The minimum number of NAND gates used to
construct OR gate is 42. The truth table is given below
(a) six (b) four
(c) three (d) two A Y

38. An p-n- p transistor can be considered to be 0 1


equivalent to a system of two diodes connected. 1 0
Then, which of the following figures is the correct
representation of that? The gate is
(a) OR (b) AND
p n p p n p
(a) (b) (c) NOT (d) None of these
E C E C
43. Distributive law is expressed by (Boolean
expression)
B B
(a) A + B = B + A
p n p p n p (b) A × B = B × A
(c) (d)
E C E C (c) ( A × B) × C = A × (B × C )
(d) A × (B + C ) = A × B + A × C
B B 44. In Boolean algebra, A × ( A × B) is equal to
39. In a CE transistor amplifier, the output (a) A × B (b) AB
resistance in 500 kW and current gain b = 49. If the (c) A + B (d) zero
power gain of the amplifier is 5 ´ 106, the input 45. A NAND gate is represented by the Boolean
resistance is expression
(a) 240 W (b) 165 W (a) A × B (b) A × B
(c) 180 W (d) 290 W (c) A + B (d) A + B

MODULE 1
www.jeebooks.in

Answer Sheet
1. Physical World, Measurement and Kinematics
1. (d) 2. (d) 3. (c) 4. (a) 5. (c) 6. (d) 7. (b) 8. (d) 9. (b) 10. (a)
11. (a) 12. (b) 13. (c) 14. (c) 15. (c) 16. (a) 17. (d) 18. (a) 19. (a) 20. (c)
21. (b) 22. (d) 23. (a) 24. (d) 25. (c) 26. (c) 27. (c) 28. (c) 29. (c) 30. (b)
31. (c) 32. (a) 33. (c) 34. (c) 35. (c) 36. (d) 37. (a) 38. (c) 39. (c) 40. (b)
41. (d) 42. (a) 43. (b) 44. (b) 45. (b)

2. Laws of Motion and Work, Power & Energy


1. (c) 2 (c) 3. (a) 4. (c) 5. (c) 6. (d) 7. (c) 8. (b) 9. (d) 10. (b)
11. (a) 12. (d) 13. (a) 14. (a) 15. (c) 16. (c) 17. (c) 18. (c) 19. (b) 20. (b)
21. (b) 22. (c) 23. (c) 24. (c) 25. (d) 26. (a) 27. (d) 28. (a) 29. (a) 30. (c)
31. (c) 32. (a) 33. (d) 34. (c) 35. (a) 36. (b) 37. (b) 38. (b) 39. (b) 40. (b)
41. (c) 42. (b) 43. (a) 44. (c) 45. (d)

3. Motion of System of Particles and Gravitation


1. (b) 2. (c) 3. (a) 4. (c) 5. (d) 6. (b) 7. (a) 8. (b) 9. (a) 10. (b)
11. (a) 12. (b) 13. (c) 14. (b) 15. (b) 16. (a) 17. (b) 18. (d) 19. (a) 20. (b)
21. (c) 22. (d) 23. (b) 24. (c) 25. (d) 26. (d) 27. (b) 28. (c) 29. (c) 30. (a)
31. (b) 32. (b) 33. (a) 34. (b) 35. (b) 36. (a) 37. (c) 38. (c) 39. (d) 40. (b)
41. (a) 42. (b) 43. (b) 44. (c) 45. (c)

4. Properties of Bulk Matter and Thermodynamics


1. (a) 2. (b) 3. (d) 4. (c) 5. (c) 6. (c) 7. (a) 8. (c) 9. (d) 10. (d)
11. (a) 12. (b) 13. (c) 14. (a) 15. (d) 16. (c) 17. (c) 18. (b) 19. (d) 20. (d)
21. (c) 22. (a) 23. (d) 24. (c) 25. (d) 26. (a) 27. (b) 28. (c) 29. (d) 30. (c)
31. (b) 32. (c) 33. (c) 34. (c) 35. (c) 36. (c) 37. (c) 38. (c) 39. (c) 40. (c)
41. (b) 42. (a) 43. (b) 44. (b) 45. (d)

5. Kinetic Theory of Gases and Oscillations & Waves


1. (b) 2. (a) 3. (c) 4. (b) 5. (a) 6. (c) 7. (b) 8. (d) 9. (a) 10. (b)
11. (b) 12. (d) 13. (b) 14. (c) 15. (a) 16. (a) 17. (a) 18. (d) 19. (b) 20. (a)
21. (b) 22. (c) 23. (d) 24. (a) 25. (b) 26. (b) 27. (d) 28. (c) 29. (c) 30. (c)
31. (d) 32. (b) 33. (c) 34. (b) 35. (b) 36. (c) 37. (b) 38. (a) 39. (b) 40. (b)
41. (c) 42. (c) 43. (a) 44. (a) 45. (b)

6. Electrostatics and Current Electricity


1. (b) 2. (c) 3. (b) 4. (a) 5. (c) 6. (d) 7. (a) 8. (b) 9. (c) 10. (d)
11. (a) 12. (b) 13. (c) 14. (c) 15. (c) 16. (c) 17. (c) 18. (a) 19. (a) 20. (c)
21. (c) 22. (b) 23. (d) 24. (b) 25. (d) 26. (c) 27. (c) 28. (a) 29. (d) 30. (a)
31. (b) 32. (a) 33. (d) 34. (d) 35. (c) 36. (a) 37. (c) 38. (c) 39. (b) 40. (a)
41. (d) 42. (d) 43. (a) 44. (d) 45. (c)

7. Magnetic Effect of Current, Magnetism, EMI and AC


1. (a) 2. (a) 3. (a) 4. (c) 5. (a) 6. (b) 7. (b) 8. (a) 9. (b) 10. (a)
11. (a) 12. (d) 13. (d) 14. (d) 15. (c) 16. (b) 17. (a) 18. (c) 19. (b) 20. (d)
21. (a) 22. (b) 23. (a) 24. (a) 25. (c) 26. (d) 27. (c) 28. (d) 29. (a) 30. (d)
31. (c) 32. (c) 33. (c) 34. (b) 35. (c) 36. (c) 37. (b) 38. (b) 39. (b) 40. (a)
41. (d) 42. (a) 43. (b) 44. (b) 45. (a)

MODULE 1
www.jeebooks.in
32 NEET Test Drive

8. Electromagnetic Waves and Optics


1. (b) 2. (d) 3. (d) 4. (a) 5. (a) 6. (b) 7. (d) 8. (b) 9. (a) 10. (b)
11. (c) 12. (c) 13. (a) 14. (c) 15. (b) 16. (a) 17. (a) 18. (b) 19. (c) 20. (b)
21. (c) 22. (d) 23. (a) 24. (b) 25. (a) 26. (a) 27. (c) 28. (a) 29. (b) 30. (d)
31. (c) 32. (d) 33. (c) 34. (a) 35. (d) 36. (d) 37. (a) 38. (b) 39. (a) 40. (d)
41. (b) 42. (d) 43. (c) 44. (a) 45. (c)

9. Dual Nature of Matter and Atom & Nuclei


1. (c) 2. (c) 3. (d) 4. (d) 5. (b) 6. (a) 7. (d) 8. (c) 9. (b) 10. (a)
11. (b) 12. (b) 13. (c) 14. (c) 15. (c) 16. (a) 17. (c) 18. (a) 19. (a) 20. (b)
21. (a) 22. (c) 23. (c) 24. (a) 25. (c) 26. (c) 27. (c) 28. (b) 29. (d) 30. (b)
31. (a) 32. (a) 33. (c) 34. (d) 35. (a) 36. (b) 37. (a) 38. (b) 39. (d) 40. (b)
41. (a) 42. (c) 43. (c) 44. (d) 45. (a)

10. Electronic Devices


1. (a) 2. (b) 3. (b) 4. (d) 5. (d) 6. (c) 7 (b) 8. (c) 9. (b) 10. (a)
11. (d) 12. (c) 13. (b) 14. (c) 15. (c) 16. (b) 17. (b) 18. (d) 19. (b) 20. (d)
21. (d) 22. (d) 23. (d) 24. (a) 25. (d) 26. (c) 27. (a) 28. (c) 29. (c) 30. (c)
31. (b) 32. (c) 33. (b) 34. (c) 35. (c) 36. (b) 37. (c) 38. (b) 39. (a) 40. (c)
41. (b) 42. (c) 43. (d) 44. (d) 45. (c)

For Explanation to all the Unit Tests of Physics, open the link
https://goo.gl/Q5C3Kq

MODULE 1
www.jeebooks.in

PREP ANALYSIS
for NEET
Full Length Tests for each Unit of NEET Chemistry to Tighten up the Loose Screws

CHEMISTRY
Instruction In each unit, question nos. 1 to 25 are of moderate difficulty level and question nos. 26 to 45 are of higher difficulty level.

1. Some Basic Concepts of Chemistry 7. If travelling at same speeds, which of the following
matter waves have the shortest wavelength?
and Atomic Structure (a) Electron (b) a-particle [He 2 + ]
1. 6 .02 ´ 1020 molecules of urea are present in 100 mL (c) Neutron (d) Proton
of its solution. The concentration of urea solution is
(a) 0.001 M (b) 0.01 M 8. What is the value of de-Broglie wavelength for an
(c) 0.02 M (d) 0.1 M a-particle having mass 6.6 ´ 10- 27 kg moving with
a speed of 105 cm s- 1?
2. An aqueous solution of glucose is 20% in strength.
(a) 2 ´ 10- 12 m (b) 3 ´ 10- 10 m
The volume in which 1 g mole of it is dissolved will - 10
be (c) 1 ´ 10 m (d) 2 ´ 10- 10 m
(a) 9 L (b) 1.8 L (c) 8 L (d) 0.9 L 9. Line spectrum of element is unique and there is
3. The uncertainty in momentum of an electron is regularity in the line spectrum of each element.
1 ´ 10- 5 kg m s- 1. The uncertainty in its position Which of the following line is observed in line
will be ( h = 6.62 ´ 10- 34 kg m 2s- 1 ) spectrum of hydrogen atom?
(a) Balmer series (b) Lyman series
(a) 5.27 ´ 10- 30 m (b) 1. 05 ´ 10- 26 m
(c) Brackett series (d) Pfund series
(c) 1. 05 ´ 10- 29 m (d) 5.25 ´ 10- 28 m
10. Amount of oxygen required for combustion of 1 kg
4. 0.5 mole of H 2SO4 is mixed with 0.2 mole of of a mixture of butane and isobutane is
Ca(OH)2. The maximum number of mole of CaSO4 (a) 1.8 kg (b) 2.7 kg
formed is (c) 4.5 kg (d) 3.58 kg
(a) 0.2 (b) 0.5 11. The volume in litres of CO 2 liberated at STP when
(c) 0.4 (d) 1.5
10 g of 90% pure lime is heated completely is
5. 2.76 g of silver carbonate on being strongly heated (a) 20.16 (b) 2.016 (c) 2.24 (d) 22.4
yields a residue weighing.
12. 0.5 g of a metal on oxidation give 0.79 g of its
(a) 3.5 g (b) 3.0 g
oxide. The equivalent weight of the metal is
(c) 1.36 g (d) 2.16 g
(a) 10 (b) 14 (c) 20 (d) 40
6. If the concentration of glucose (C6H12O6 ) in blood is 13. Quantum numbers describe values of conserved
0.9 g L- 1. What will be the molarity of glucose in quantities in the dynamics of quantum system.
blood? Orientation of an atomic orbital is governed by
(a) 5 M (b) 50 M (a) magnetic quantum number (b) principal quantum number
(c) 0.005 M (d) 0.5 M (c) azimuthal quantum number (d) spin quantum number

MODULE 1
www.jeebooks.in
34 NEET Test Drive

14. Hund’s rule of maximum multiplicity is the rule for 22. Amount of oxygen required for complete
filling of electrons in orbitals. For the practical combustion of 27 g Al is
evaluating of this rule, the term “maximum (a) 24 g (b) 12 g
multiplicity” is added in the rule. According to (c) 20 g (d) 6 g
Hund’s rule, which of the following is incorrectly
filled? 23. The number of radial nodes of 3s and 2 p orbitals
¼ respectively are
(a) ¼¼¼¼¼ (b) ¼ ¼¼¼¼ (a) 2, 0 (b) 1, 2
(c)
¼¼
(d)
¼¼ (c) 0, 2 (d) 2, 11
¼¼ ¼¼¼ ¼¼ ¼¼
24. M 3 + (mass number = 56) contains 30.4% more
15. The orbital with two radial and two angular nodes neutrons than electrons. Thus, atomic number of
is the ion is
(a) 5d (b) 3 p (c) 8d (d) 5f (a) 24 (b) 25
(c) 26 (d) 23
16. An insecticide contains 47.5% C, 2.54% H and
50% Cl by mass. Its empirical formula is 25. The orbital angular momentum of an electron in
(a) C13H8Cl 5 (b) C14H9Cl 5 (c) C12H10Cl 5 (d) C15H12Cl 6 2s-orbital is
1 h
17. The frequency of radiation emitted when the (a) + (b) zero
2 2p
electron falls from n = 4 to n = 1 in a hydrogen h h
atom will be [If ionisation energy of H (c) (d) 2
2p 2p
= 2.18 ´ 10- 18 J atom -1]
. ´ 1015 s - 1
(a) 134 (b) 3.08 ´ 1015 s - 1 26. To neutralise completely 20 mL of 0.1 M aqueous
solution of phosphorus acid (H 3PO3 ), the volume of
. ´ 1015 s - 1
(c) 198 (d) 3.00 ´ 1015 s - 1
0.1 M aqueous KOH solution required is
18. Planck’s constant is the proportionality constant (a) 60 mL (b) 20 mL
- 34 (c) 40 mL (d) 10 mL
whose value is 6.626 ´ 10 J-s. It has the same
dimension as that of 27. For which one of the following sets of four
quantum numbers, an electron will have the
(a) radiant energy (b) work
highest energy?
(c) angular momentum (d) power
n l m s
19. One mole of CO 2 contains
(a) 5 2 1 ½
(a) 6.022 ´ 1023 atoms of oxygen
(b) 4 1 1 -½
. ´ 1023 molecules of CO 2
(b) 181
(c) 4 2 -1 ½
(c) 3 grams of CO 2
(d) 2 1 -1 -½
(d) 6.02 ´ 1023 molecules of CO 2
28. Match the items given in Column I with the items
20. An alkane has a C/H ratio (by mass) of 5.1428 : 1. given in Column II and choose the correct code.
Its molecular formula is
(a) C 6H14 (b) C 5H12 (c) C 7H16 (d) C 8H18 Column I Column II
(H-spectral series) (Characteristic)
21. The plot of the radial wave function R as a function
of distance r of the electron from the nucleus for 1 é1 1ù
A. Balmer 1. = Rê 2 - 2 ú
2s-orbital is l êë 4 n ú

R2 R2 B. Paschen 2. Visible

(a) (b) C. Brackett 3. UV-visible

= R éê 2 - 2 ùú
1 1 1
D. Lyman 4.
r r l ë3 2 û

R2 R2 Codes
A B C D A B C D
(c) (d)
(a) 2 3 1 4 (b) 3 1 2 4
(c) 1 2 3 4 (d) 2 1 4 3
r r

MODULE 1
www.jeebooks.in
Prep Analysis for NEET ~ Chemistry 35

29. The number of moles of oxygen in 1 L of air 39. Liquid benzene (C6H 6 ) burns in oxygen to produce
containing 21% oxygen by volume at STP is carbon dioxide gas and steam. How many litres of
(a) 0.186 mol (b) 0.0093 mol O 2 at STP are needed to complete the combustion
(c) 0.16 mol (d) 2.10 mol of 39 g of liquid benzene?
30. The energy of first excited state of hydrogen atom (a) 60 L (b) 84.0 L (c) 34 L (d) 11.2 L
is -3.4 eV. Then, KE of same orbit of hydrogen 40. Which sets of quantum number is wrong?
atom is 1 1
(a) n = 2, l = 1, m = 0, s = + (b) n = 2, l = 2, m = 0, s = +
(a) + 3.4 eV (b) + 4.8 eV (c) + 13.6 eV (d) - 13.6 eV 2 2
1 1
31. A metal oxide has the formula, A2O3 . It can be (c) n = 3, l = 2, m = 0, s = + (d) n = 3, l = 1, m = 0, s = +
2 2
reduced by hydrogen to give free metal A and
water. 0.1596 g of the metal oxide requires 6 mg of 41. The amount of oxalic acid (hydrated) required to
hydrogen for complete reduction. The atomic prepare 500 mL of its 0.1 N solution is
weight of the metal is (a) 0.315 g (b) 6.3 g (c) 3.15 g (d) 63.0
(a) 77.9 (b) 159.6 (c) 55.8 (d) 79.8
42. What is the maximum number of electrons that
32. The angular momentum of an electron in can be associated with the following set of
2 p-orbital is quantum numbers?
h h n = 4, l = 2 and m = - 1
(a) (b)
2p 2p (a) 10 (b) 4 (c) 2 (d) 3
2h
(c) (d) None of these
p 43. Elements A and B combine to form compound AB2
and A 3 B 2. When 0.2 mole of AB 2 weighs 20 g and
33. The total number of valence electrons in 4.2 g of
0.10 mole of A 3 B 2 weighs 18 g, the atomic weights
azide ion is (N A is Avogadro’s number)
of A and B respectively are
(a) 2.8 NA (b) 1.2 NA (c) 1. 6 NA (d) 3. 8 NA
(a) 40, 30 (b) 60, 80 (c) 80, 30 (d) 30, 40
34. The frequency of first line of Balmer series in 44. In an atom, an electron is moving with a speed of
hydrogen atom is v0. This frequency of 600 m/s with an accuracy of 0.005%. Certainty
corresponding line emitted by singly ionised with which the position of the electron can be
helium atom is located is (Given, me = 9.1 ´ 10- 31 kg,
(a) 2 n0 (b) 4n0 (c) n0 / 2 (d) n0 / 4
h = 6.6 ´ 10- 34 kg m 2 s - 1)
35. Calculate the amount of Ca(OH)2 required to (a) 3.41 ´ 10- 3 m . ´ 10- 4 m
(b) 182
remove the hardness of water from 60,000 L (c) 4.91 ´ 10 -3
m . ´ 10- 3 m
(d) 192
containing 16.2 g of Ca(HCO3 )2 per 100 L.
(a) 1.11 kg (b) 2.22 kg 45. If concentrated aqueous sulphuric acid is 98%
(c) 3.33 kg (d) 4.44 kg H 2SO4 by mass and has a density of 1.80 g mL- 1.
36. If the uncertainty in momentum and position are Volume of acid required to make 1 L of 0.1 M
equal, then uncertainty in velocity is H 2SO4 is
(a) 18.25 mL (b) 5.55 mL (c) 20.85 mL (d) 11.10 mL
h 1 h 1 h h
(a) (b) (c) (d)
p m p 2m p 2p
2. Classification of Elements, Hydrogen
37. 25 mL of 0.50 M H 2O 2 solution is added to 50 mL of
and Chemical Bonding & Molecular
0.20 M KMnO4 in acidic solution. Which of the
following statements is true? Structures
(a) 0.010 mole of oxygen is liberated 1. Which of the following molecule doesn’t obey
(b) 0.005 mole of KMnO 4 does not react with H2O 2 electron octet rule?
(c) 0.0025 mole of H2O 2 does not react with KMnO 4 (a) CO 2 (b) H2O (c) O 2 (d) NO+
(d) In the final solution there are only water molecules and 2. Following transition elements (IE)1 drops abruptly
Mn2 + ions
(Ga, In and Tl). This is due to
38. If the energy of an electron in the first Bohr’s orbit (a) decrease in effective nuclear charge
of H- atom is - 313.6 kcal/mol, then the energy of (b) removal of an electron from the singly occupied np-orbitals of
the electron in the second orbit will be Ga than the ns-orbitals of Zn, Cd and Hg
(a) - 34.84 kcal/mol (b) - 12.5 kcal/mol (c) increase in atomic radius
(c) - 78.4 kcal/mol (d) - 313.6 kcal/mol (d) None of the above

MODULE 1
www.jeebooks.in
36 NEET Test Drive

3. The elements that can form interstitial hydrides 14. The compound which is used in softening of hard
are water is
(a) alkali and alkaline earth metals (a) Na 2HPO 4 (b) Na 6P6O18
(b) non-metals like halogens (c) Ca 3 (PO 4 )2 (d) Na 3PO 4
(c) metalloids
15. Which one of the following has a coordinate bond?
(d) transition elements
(a) Cl 2 (b) NaCl
4. Considering the elements, B, C, N, F and Si, the (c) NH4Cl (d) AlCl 3
correct order of their non-metallic character is 16. The species having pyramidal shape is
(a) F > N > C > B > Si (b) Si > C > B > N > F (a) OSF2 (b) SO 3
(c) B > C > Si > N > F (d) F > N > C > Si > B (c) SiO 3 2- (d) BrF3
5. The correct order of the lattice energies of the
following ionic compound is 17. Which one of the following statements is an
incorrect?
(a) Al 2O 3 > CaO > MgBr2 > NaCl
(a) Ionisation potential of nitrogen is greater than that of
(b) NaCl > MgBr2 > CaO > Al 2O 3
oxygen
(c) NaCl > CaO > MgBr2 > Al 2O 3
(b) The ionisation potential of beryllium is greater than that of
(d) Al 2O 3 > MgBr2 > CaO > NaCl
boron
6. Which of the following gas is evolved, when silicon (c) The electron affinity of fluorine is greater than that of
is boiled with caustic soda solution? chlorine
(a) SiH4 (b) O 2 (d) The electronegativity of fluorine is greater than that of
(c) H2 (d) None of these chlorine

7. The bond order and magnetic character in 18. Which one among the following does not have the
superoxide ion (O -2 ) respectively are hydrogen bond?
(a) Liquid HCl (b) Liquid NH 3
(a) 0, paramagnetic (b) 2.5, diamagnetic
(c) Phenol (d) Water
(c) 1.5, paramagnetic (d) 1, diamagnetic
8. The decreasing order of the ionisation potential in 19. Which one of the following compound has the
maximum dipole moment?
the following elements is
(a) NI3 (b) NF3
(a) Ne > Cl > P > S > Al > Mg
(c) NCl 3 (d) NH3
(b) Ne > Cl > S > P > Mg > Al
(c) Ne > Cl > P > S > Mg > Al 20. Which of the following species has lowest
(d) Ne > Cl > S > P > Al > Mg ionisation potential?
(a) O (b) O 2 (c) O +2 (d) O -2
9. Temporary hardness of water is caused due to the
presence of 21. Which of the following is correct order of ionic size?
(a) Ca(HCO 3 )2 (b) CaCO 3 (a) Ca 2+ > K+1 > Cl - > S2 -
(c) CaSO 4 (d) CaCl 2 (b) Ca 2+ > Cl -1 > K +1 > S2 -
10. The metal which evolves hydrogen on reacting with (c) S2 - > Cl - > K +1 > Ca 2 +
cold dil. HNO3 is (d) Cl - > K + > Ca 2 + > S2 -
(a) Mg (b) Cu (c) Al (d) Fe
22. The correct order of covalent, van der Waals’ and
11. Which of the following is the correct order of crystal radii is
increasing basicity? (a) rcovalent < rcrystal < rvan der Waals'
(a) MgO < BeO < CaO < BaO (b) rcovalent < rvan der Waals' < rcrystal
(b) BeO < MgO < CaO < BaO (c) rcrystal < rcovalent < r van der Waals'
(c) BaO < CaO < MgO < BeO (d) rcrystal < r van der Waals' < rcovalent
(d) CaO < BaO < BeO < MgO
23. Among the following species, the one which is
12. Which of the following has the highest percentage paramagnetic in nature is
of ionic character in its bonding? (a) CO (b) CN - (c) O 2-
2 (d) NO -
(a) LiI (b) MgCl 2 (c) CsF (d) CsI
24. Which of the following possess smallest bond angle?
13. Which of the following is used as a moderator in (a) N2O (b) H2S (c) BeCl 2 (d) H2O
nuclear reactor?
(a) R—O—R (b) H2O 2 25. Out of F2 , Cl 2, Br2 and I2 most oxidising agent is
(c) TEL (d) D2O (a) Cl 2 (b) F2 (c) Br2 (d) I2

MODULE 1
www.jeebooks.in
Prep Analysis for NEET ~ Chemistry 37

26. Which of the following statements is incorrect? 35. Stability of the species Li2 , Li2- and Li +2 increases
(a) Ordinary water is electrolysed more rapidly than D2O in the order of
(b) Reaction between H2 and Cl 2 is much faster than D2 and Cl 2 (a) Li 2 < Li 2 + < Li -2 (b) Li 2 < Li 2 + = Li -2
(c) D2O freezes at lower temperature than H2O
(c) Li 2 < Li 2 - < Li +2 (d) Li -2 < Li 2 < Li +2
(d) Bond dissociation energy for D2 is greater than H2
27. Which one has maximum ionic character? 36. Which one of the following arrangement is the
(a) PbBr 2 (b) PbCl 2 (c) PbF 2 (d) PbI 2 incorrect representation of the property indicated
with it?
28. Three elements X , Y and Z have atomic numbers (a) Br < Cl < F : Electronegativity
19, 37 and 55 respectively. Then the correct (b) F < Br < Cl : Electron affinity
statement is (c) Br 2 < Cl 2 < F 2 : Bond energy
(a) their ionisation potential would increase with increasing (d) Br 2 < Cl 2 < F 2 : Oxidising strength
atomic numbers
(b) Y would have an ionisation potential between those of X and Z 37. Which of the following given pair resembles CO2?
(c) Y would have the highest ionisation potential (a) HgCl 2 , C 2H2 (b) HgCl 2 , SnCl 4
(d) Z would have the highest ionisation potential (c) C 2H2 , NO 2 (d) N2O, NO 2
29. Which of the following statements is/are incorrect 38. Which one of the following is an incorrect
for hydrogen? statement for periodic classification of elements?
(a) Its molecule is diatomic (a) The properties of elements are a periodic function of their
(b) It exists both as H+ and H- in different chemical atomic numbers
compounds (b) Non-metallic elements are less in number than metallic
(c) It is the only species which has no neutrons in the nucleus elements
(d) Heavy water is unstable because hydrogen is substituted (c) The first ionisation energies of elements along a period do
by its isotopic deuterium not vary in a regular manner with increase in atomic number
(d) For transition elements, the d-subshells are filled with
30. Which of the following pairs of compounds is electrons monotonically with increase in atomic number
isoelectronic?
(a) Cl 2O, ICl 2 - (b) ICl -2 , ClO 2 (c) IF2+ , I-3 (d) ClO -2 , ClF2 + 39. Bond angle is the angle between the bonded
orbital. Generally, it decreases as the number of
31. The first ionisation energy of Na, Mg, Al and Si are lone pair of electrons increases or as the
in the order : electronegativity of the central atom decreases.
(a) Na < Mg > Al < Si (b) Na > Mg > Al < Si Which of the following set of species has same bond
(c) Na > Mg > Al > Si (d) Na < Mg < Al < Si angle around central atom?
(a) BCl 3 , BF3 (b) NF3 , NH3
32. In which of the following arrangements, the order
(c) AlCl 3 , NH3 (d) BeCl 2 , SnCl 2
is not according to the property indicated against
it? 40. On the basis of VSEPR model the angular
(a) Al 3+ < Mg 2+ < Na + < F - (Ionic size) arrangement of the bonding pairs corresponds to
(b) B < C < N < O (First ionisation enthalpy) the angular arrangement of the bonded atoms.
(c) I < Br < F < Cl (Electron gain enthalpy) Then, which of the following is correct regarding
(d) Li < Na < K < Rb (Metallic radius) bond angle?
(a) CO 2 > H2O > CH4 (b) CO 2 > CH4 > H2O
33. The bond order of NO is 2.5 while that of NO + is 3. (c) CH4 > CO 2 > H2O (d) H2O > CH4 > CO 2
Which of the following statements is true for these
two species? 41. Choose the correct statement.
(a) Bond length of NO + is equal to the bond length of NO (a) Silicon exhibits 4 coordination number in its compound
(b) Bond length of NO is greater than bond length of NO + (b) Bond energy of F2 is less than Cl 2
(c) Bond length of NO + is greater than bond length of NO (c) Mn(III) oxidation state is more stable than Mn(II) in
(d) Bond length is unpredictable aqueous state
(d) Elements of 15th group shows only +3 and +5 oxidation state
34. The hydride ion H - is a stronger base than its
hydroxide ion OH - . Which of the following 42. From VSEPR theory, we know central atoms are
reactions will occur if sodium hydride (NaH) is surrounded by both bonded and lone pair of
dissolved in water? electrons, give rise to an irregular geometry in
(a) H - (aq) + H2O(l ) ¾® H3O - (aq) molecule on account of different repulsive
(b) H- (aq ) + H2O(l ) ¾® OH- (aq ) + H2 (g ) interactions between electron pairs. Then, which of
(c) H- (aq ) + H2O)(l ) ¾® No reaction
the following is isostructural with C 2 H 2 ?
(a) H2O 2 (b) NO 2 (c) SnCl 2 (d) CO 2
(d) None of the above

MODULE 1
www.jeebooks.in
38 NEET Test Drive

43. Which of the following is incorrect order regarding 6. The enthalpies of elements in their standard states
the property indicated? are arbitrarily assumed to be
(a) Sc 3 + > Cr 3 + > Fe 3 + > Mn3 + (Ionic radius) (a) zero at 298 K and 1 atm (b) unity at 298 K and 1 atm
(b) Sc < Ti < Cr < Mn (Density) (c) zero at all temperatures (d) zero at 273 K and 1 atm
(c) Mn2 + > Ni 2 + < Co 2 + < Fe 2 + (Ionic radius)
7. Consider the following equilibrium,
(d) FeO < CaO < MnO > CuO (Basic nature) +
Ag+ + 2NH3- Ag(NH ) ; K = 1.8 ´ 10
3 2 1
7
44. The volume strength of 1.5 N H 2O2 solution is Ag+ + Cl-
- AgCl ; K = 5.6 ´ 10 9
2
(a) 8.0 (b) 8.4
Hence, for the equilibrium,
(c) 3.0 (d) 4.8 + -
AgCl + 2NH - Ag(NH ) + Cl3 3 2
45. Match the compounds given in Column I with the
the equilibrium constant is
shape given in Column II and assign the correct
code. (a) 0.32 ´ 10-2 (b) 0.31 ´ 10-21
(c) 1.01 ´ 10 17
(d) 1.01 ´ 10-17
Column I Column II
8. In the metallurgy of iron, when CaCO 3 is added to
A. BF 3 1. See-saw
blast furnace, calcium ion appears as
B. SF 6 2. Square planar
(a) slag (b) gangue (c) metallic (d) CaO
C. SF 4 3. Octahedral
9. For the reaction, H 2( g) + I2( g) 2HI( g) , the
-
D. XeF 4 4. Bent standard free energy is DG° > 0 . The equilibrium
5. Trigonal planar constant ( K ) would be
(a) K = 0 (b) K > 1
Codes
(c) K = 1 (d) K < 1
A B C D A B C D
(a) 1 2 3 4 (b) 5 3 1 2 10. On decomposition of NH 4HS the following
(c) 4 3 1 4 (d) 3 2 2 1 equilibrium is established,
NH 4HS ( s) e NH (g) + H S (g)
3 2
3. Equilibrium, Thermodynamics If the total pressure is p atm, the equilibrium
and Extraction of Metals constant K p is equal to
(a) p atm (b) p2 atm 2
1. The solubility product of CuS, CdS and HgS are (c) p2 / 4 atm2 (d) 2 p atm
10-31 , 10-44 , 10-54, respectively. The solubility of
these sulphides are in the order 11. At 27°C, latent heat of fusion of a compound is
(a) CdS >HgS >CuS (b) HgS >CdS >CuS
2930 J/mol. Entropy change during fusion is
(c) CdS >CuS >HgS (d) CuS >CdS >HgS (a) 9.77 J/mol K (b) 10.977 J/mol K
(c) 19.07 J/mol K (d) None of these
2. During the process of electrolytic refining of
copper, some metals present as impurity settle as 12. For a reaction at 25° C , enthalpy change (DH ) and
“anode mud”. These are entropy change (DS ) are - 11. 7 ´ 103 J mol-1 and
(a) Se and Ag (b) Pb and Zn - 105 J mol-1K -1 respectively. The reaction is
(c) Ag and Au (d) Fe and Ni (a) spontaneous (b) non-spontaneous
3. The process which is related with the removal of (c) instantaneous (d) None of these
sulphur present in the ore by heating it in the 13. What is the equilibrium expression for the
air is reaction?
(a) roasting (b) annealing P4( s ) + 5O2 ( g) eP O4 10( s )
(c) calcination (d) smelting [P4O10 ] [P4O10 ]
(a) KC = 5
(b) KC =
4. Temperature at which the equilibrium water [P4 ] [O 2 ] 5[P4 ][O 2 ]
vapour pressure is 1.00 bar, is 1
(c) KC = [O 2 ] 5 (d) KC =
(a) 107°C (b) 38°C (c) 215°C (d) 240°C [O 2 ] 5
5. Temperature and heat are
14. Which of the following decreases the H+ ion
(a) extensive properties
concentration of an acetic acid solution?
(b) intensive properties -+
(c) intensive and extensive properties respectively (a) AgNO 3 (b) CH3COONa
(d) extensive and intensive properties respectively (c) Al 2 (SO 4 )3 (d) NH4Cl

MODULE 1
www.jeebooks.in
Prep Analysis for NEET ~ Chemistry 39

15. Entropy decreases during 25. Which of the following options will be correct for
(a) crystallisation of sucrose solution the stage of half completion of the reaction
(b) rusting of iron A n B?
(c) melting of ice (a) DG° = 0 (b) DG° > 0
(d) vaporisation of camphor (c) DG° < 0 (d) DG° = - RT ln K

16. In which case, a spontaneous reaction is impossible 26. The pH of 0.05 M aqueous solution of diethylamine
at any temperature? is 12.0. The value of K b is
(a) D H > 0, DS > 0 (b) D H > 0, DS < 0 (a) 2 .5 ´ 10-3 (b) 2 .5 ´ 103
(c) D H < 0, DS < 0 (d) In all cases (c) 5.0 ´ 10 -6
(d) 5.0 ´ 106
17. The equilibrium constant for the reaction,
27. Among the following methods, the one which is
N 2 + 3H 2 - 2NH 3 is K C , used to refine Zr and Ti is
then equilibrium constant for the reaction (a) liquation (b) cyanide process
1 3 (c) Mond’s process (d) van-Arkel method
NH 3 - N 2 + H 2 will be
2 2 28. A person heats the surrounding at a rate of about
1 1 1 100 W/s. The entropy change contributed by the
(a) (b) (c) KC (d)
KC KC2 KC person on a normal day at a temperature 20°C is
(a) 25 (b) 30 (c) 15 (d) 10
18. The element which is present as the impurity to
the maximum extent in the pig iron is 29. Which of the following reactions will not be
(a) silicon (b) carbon affected by increasing the pressure?
(c) phosphorus (d) manganese (a) PCl 5 (g ) - PCl 3 (g ) + Cl 2 (g )
(b) N2 (g ) + O 2 (g )
- 2NO(g )
19. The ore which is not concentrated by (c) CaCO 3 (s )- CaO(s ) + CO 2 (g )
froth-floatation method is (d) N2 (g ) + 3H2 (g )
- 2NH3 (g )
(a) copper pyrites (b) zinc blende
(c) pyrolusite (d) galena 30. In the metallurgical process, the flux used for
removing basic impurities is
20. Which of the following is incorrect relation for (a) lime stone (b) sodium chloride
equilibrium constant of (c) silica (d) sodium carbonate
aA + bB - cC + dD2.303 RT log K 31. 50 mL each of 1M CH 3COOH and 0.50 M NaOH are
(a) DG° = - 2.303 T log K (b) E°cell =
nF mixed (K a for CH 3COOH = 1. 8 ´ 10-5 ) . The pH of
S (product) DH
(c) dK = (d) log K = log A - the mixture is
S (reactant) 2 .303 RT (a) 6.27 (b) 2.67 (c) 1.67 (d) 4.74

21. Which of the following gives the maximum number 32. The molar entropy change when a sample of
of ions per mole when dissolved in water? hydrogen gas expands isothermally to twice of its
(a) K 2MgI4 (b) CuSO 4 initial volume is
(c) FeCl 3 (d) KI3 (a) 2.83 (b) 6.72 (c) 5.76 (d) 3.96

22. Which of the following is not a state function? 33. The per cent dissociation of H 2S if 1 mole of H 2S is
(a) Internal energy (b) Entropy introduced into a 1.10 L vessel at 1000 K is
-6
(c) Work (d) Enthalpy (Given, 2H 2 S ( g) - 2H (g) + S ( g) is 1 ´ 10
2 2 )
23. An ideal gas undergoing expansion in vacuum (a) 0.8% (b) 1.3%
shows (c) 1.8% (d) 2.0%
(a) DU = 0 (b) w = 0 34. Which of the following method is used for the
(c) q = 0 (d) All of these concentration of the bauxite ore?
24. The relationship between K C and (a) Hydraulic separation (b) Leaching
Dn (c) Electromagnetic separation (d) Froth floatation
K p is K p = K C ( RT ) . What would be the value of
Dn for the reaction? 35. Which of the following reaction will not be affected
by change in pressure?
NH 4Cl( s) - NH ( g) + HI( g)
3
(a) H2 + I2- 2HI (b) N2 +3H2 - 2NH 3
(a) 1 (b) 0.5 (c) PCl 5 - PCl + Cl (d) CaO + CO 2 - CaCO
3 2 3
(c) 1.5 (d) 2

MODULE 1
www.jeebooks.in
40 NEET Test Drive

36. A bulb of 100 Watt is switched on in a room of 43. Match the following parameters with description for
dimensions ( 5 ´ 4 ´ 3) m3 . What will be the increase spontaneity and choose the correct codes given below.
in temperature of room after 15 min, if specific
Parameters (D r H° D r S° D r G°) Description
heat of air at room temperature and 1 atm is
0.71 Jg -1 K -1 and heat capacity of four walls and A. + – + 1. Non-spontaneous at
high temperature.
the roof is 50 ´ 103 JK -1?
(Density of air = 1 .22 ´ 10-6 kg mL-1) B. – – + at high T 2. Spontaneous at all
temperatures.
(a) 0.68 K (b) 0.98 K
(c) 0.59 K (d) 0.88 K C. – + – 3. Non-spontaneous at
all temperatures.
37. The equilibrium constant for a reaction,
N 2( g) + O2( g) ® 2NO ( g) is 4 ´ 10-4 at 200 K. In the Codes
A B C A B C
presence of catalyst, the equilibrium constant is
attained 10 times faster. The equilibrium constant (a) 1 3 2 (b) 3 2 1
in the presence of catalyst, at 2000 K is (c) 1 2 3 (d) 3 1 2
(a) 40 ´ 10-4
(b) 4 ´ 10-4 44. The solubility of solid silver chromate,
Ag 2 CrO 4 is determined in three solvents K sp or
(c) 4 ´ 10-2
Ag2CrO4 = 9 ´ 10-12.
(d) data insufficient
I. Pure water II. 0.1 M AgNO 3
38. The major role of fluorspar (CaF2 ) which is added in
III. 0.1 M Na 2 CrO 4
small quantity in the electrolytic reduction of
The relative solubility of Ag2CrO4 in the three
alumina dissolved in fused cryolite (Na 3 AlF6 ) is/are
solvents is
(a) to decrease the rate of oxidation of carbon at anode
(a) I = II = III (b) I < II < III
(b) to lower the temperature of melting
(c) II = III < I (d) II < III < I
(c) to make the fused mixture for reduction
(d) acts as a catalyst 45. Which one of the following statements is incorrect?
(a) The degree of ionisation of a weak electrolyte increases
39. If a mixture of 3 moles of H 2 and 1 mole of N 2 is with dilution
completely converted into NH 3 , then the ratio of the (b) Strong electrolytes are ionised completely at all
initial and final volume at same temperature and concentrations
pressure is (c) Addition of NH4Cl to NH4OH increases the ionisation of the
(a) 2 : 1 (b) 3 : 1 latter
(c) 4 : 1 (d) 1 : 1 (d) Increase of temperature increases the ionisation
40. The pressure of CO 2 gas at 700 K in the following
heterogenous equilibrium reaction is 4. s- and p-block Elements
CaCO3 ( s) CaO ( s) + CO2( g); DG° = 130.2 kJ mol-1
- 1. The element which forms oxide in all the oxidation
(a) 10-8 atm (b) 10-23 atm states from +1 to +5 is
-10 -6 (a) P (b) Sb (c) N (d) As
(c) 10 atm (d) 10 atm
2. Which of the following does not give oxygen on
41. 32 g oxygen gas expands at STP to occupy double heating?
of its original volume. The work done during the (a) HgO (b) KMnO 4
process is (c) KClO 3 (d) (NH4 )2 Cr2O 7
(a) 260 kcal (b) 180 kcal
(c) 130 kcal (d) 545.67 kcal 3. A colourless salt gives violet colour to bunsen flame
and also turns moistured litmus paper blue. It is
42. Three sparingly soluble salts M 2X , MX and MX3 (a) Na 2CO 3 (b) KNO 3
have same solubility product. Their solubilities will (c) K 2CO 3 (d) Cu(OH)2
be in the order
(a) MX 3 > MX > M 2 X 4. The metal ‘X’ is prepared by the electrolysis of
(b) MX 3 > M 2 X > MX
fused chloride. It reacts with hydrogen to form a
colourless solid from which hydrogen is released on
(c) MX > MX 3 > M 2 X
treatment with water. The metal is
(d) MX > M 2 X > MX 3
(a) Al (b) Ca (c) Cu (d) Zn

MODULE 1
www.jeebooks.in
Prep Analysis for NEET ~ Chemistry 41

5. On the addition of mineral acid to an aqueous 18. When carbon monoxide is passed over solid caustic
solution of borax, following compound is formed soda heated to 200° C under 5-10 atm pressure it
(a) orthoboric acid (b) metaboric acid forms
(c) boron hydride (d) boric anhydride (a) NaCO 3 (b) NaHCO 3
(c) HCOO -Na + (d) None of these
6. Concentrated sulphuric acid does not act as
(a) efflorescent (b) hygroscopic 19. Increasing order of strength of oxy-acids of chlorine is
(c) oxidising agent (d) sulphonating agent (a) HClO < HClO 3 < HClO 2 < HClO 4
(b) HClO 4 < HClO 2 < HClO < HClO 3
7. Which of the following halogen oxides is ionic? (c) HClO < HClO 2 < HClO 3 < HClO 4
(a) I4O 9 (b) I2O 5 (c) BrO 2 (d) ClO 3 (d) None of the above
8. Which of the following alkali metal bicarbonates 20. Geometry of XeOF4 molecule is
does not exist as solid ? (a) square planar (b) square pyramidal
(a) LiHCO 3 (b) KHCO 3 (c) CsHCO 3 (d) NaHCO 3 (c) triangular bipyramidal (d) None of these
9. The oxide of which metal is most stable to heat? 21. In the contact process of H 2SO4 , SO3 dissolves in
(a) K (b) Ag sulphuric acid to give
(c) Hg (d) All of these
(a) permonosulphuric acid (b) thiosulphuric acid
10. The ease of liquefaction of noble gases decreases in (c) pyrosulphuric acid (d) perdisulphuric acid
the order 22. Which of the following oxides is amphoteric in
(a) He > Ne > Ar > Kr > Xe nature?
(b) Xe > Kr > Ar > Ne > He (a) CaO (b) SiO 2 (c) CO 2 (d) SnO 2
(c) Kr > Xe > He > Ar > Ne
(d) Ar > Kr > Xe > He > Ne 23. The product obtained on strong heating of
aluminium oxide and carbon in dry chlorine gas is
11. pp- pp bonding is not present in (a) anhy. AlCl 3 (b) hydrated AlCl 3
(a) NO -3 (b) NO -2 (c) N-3 (d) PO 3-
4
(c) AlCl 3 (d) None of these

12. When H2 S is passed through nitric acid solution, 24. Carbogen is


(a) mixture of CO + CO 2
the product formed is (b) mixture of O 2 + CO 2
(a) milk of sulphur (b) colloidal sulphur (c) pure form of carbon
(c) g-sulphur (d) b-sulphur (d) unsaturated organic compound
13. Which of the following alloy is used in making the 25. Diborane is instantly hydrolysed by water to give
parts of aircraft? (a) H3BO 3 + H2 (b) H3BO 3 + B 2O 3
(a) Alumina (b) Bell metal (c) H3BO 3 + O 2 (d) H2O 3 + B 2O 3
(c) Duralumin (d) g-alloy
26. Boron is unable to form BF63 - ion. The reason for
14. The atomic radii of gallium is lesser than this is
aluminium. This is due to (a) absence of d-orbital (b) presence of d-orbital
(a) decrease in nuclear charge (c) low oxidation state (d) None of these
(b) poor shielding effect of d-electrons
27. Consider the following molecule :
(c) presence of higher orbitals
(d) higher atomic number
I. XeO 3 II. XeOF4 III. XeF6
The molecules having same number of lone pairs on
15. The substance which is solid at room temperature Xe are
forms ionic compounds and react with hydrogen (a) Both I and III (b) Both I and II
forming a hydride, the aqueous solution of which is (c) Both II and III (d) I, II and III
acidic, could be
(a) Al (b) Na (c) Br2 (d) I2
28. Bleaching action of SO 2 is due to
(a) reduction (b) hydrolysis
16. Which ions are possible? (c) oxidation (d) acidic nature
(a) He +2 , He 2 , Ne 2 , Ar 2+
(b) He +2 , +
HeH , HeH 2+

+ 2+
29. Which of the following statement is incorrect for
(c) He 2 ,HeH , HeH (d) All of these both B and Al?
17. When chlorine is passed through hot concentrated (a) Both are resistant to the action of conc HNO 3
solution of KOH, the compound formed is (b) Both BeCl 2 and AlCl 3 have chlorine bridged dimers in the
vapour phase
(a) KClO (b) KClO 2
(c) KClO 3 (d) KClO 4 (c) Both BeCl 2 and AlCl 3 act as strong Lewis bases
(d) Both have a tendency to form covalent compounds
MODULE 1
www.jeebooks.in
42 NEET Test Drive

30. The tendency of catenation in group 14 elements 39. The number of P—O—P bonds in cyclic
follows the order. metaphosphoric acid is
(a) Ge > Sn > Si > C (b) Si > C > Sn > Ge (a) zero (b) three (c) two (d) four
(c) C > Si > Ge > Sn (d) C >> Si > Ge » Sn
40. Sodium bicarbonate (NaHCO 3 ) on gentle heating
31. Gradual addition of electronic shells in the noble produces.
gases causes a decrease in their (a) Na 2CO 3 + CO 2 (b) Na 2CO 3 + CO
(a) ionisation energy (b) density (c) Na 2CO 3 + CO 2 + H2O (d) Na 2CO 3 + CO + H2O
(c) boiling point (d) atomic radius
41. The correct order of stabilities of adducts formed
32. Consider the following reaction, with boron-halides is
H 2 SO4 (a) BF3 < BBr3 < BCl 3 (b) BCl 3 < BBr3 < BF3
HCOOH ¾ ¾ ¾
¾® CO +H 2O
(c) BBr3 < BCl 3 < BF3 (d) BF3 < BCl 3 < BBr3
Here, H 2SO4 acts as
(a) reducing agent (b) oxidising agent 42. Which of the following cannot be obtained by direct
(c) dehydrating agent (d) All of these reaction of constituent elements ?
(a) XeO 3 (b) XeF2 (c) XeF6 (d) XeF4
33. Which of the following is incorrect for the
hydroxides of alkali metals ? 43. Among the halogens, the one which is oxidised by
(a) They are white crystalline solids nitric acid is
(b) These caustic alkalies are the strongest bases known in (a) iodine (b) bromine
aqueous solution (c) fluorine (d) chlorine
(c) The solubility in water decreases as we move down the 44. Match the species given in Column I with the
group properties mentioned in Column II.
(d) The basic strength of hydroxides increases on moving
down the group Column I Column II
34. Which of the following form of interhalogen A. BF -4 1. Oxidation state of central atom is +4
compounds does not exist ? B. AlCl 3 2. Strong oxidising agent
(a) IF7 (b) ClF3
(c) ICl (d) BrCl 7 C. SnO 3. Lewis acid

35. Consider the following reaction, D. PbO 2 4. Can be further oxidised

SO2 + H 2S ¾® Product 5. Tetrahedral shape

The final product is Codes


(a) H2SO 3 (b) H2SO 4 A B C D A B C D
(c) H2S2O 3 (d) H2O +S (a) 5 3 4 1, 2 (b) 3 4 2 1
(c) 5 3 4 2, 1 (d) 5,1 3 2 4
36. An element of group 2 forms covalent oxide which
is amphoteric in nature and dissolves in water to 45. In solid state, Al 2 Cl 6 exists as dimer. It also occur
give amphoteric hydroxide. The element ‘X’ is as same in the solution of non-polar solvents such
(a) Be (b) Mg as benzene. When dissolved in water, it gives
(c) Ca (d) Sr (a) Al 2O 3 + 6HCl (b) Al 3+ + 3Cl -

37. Which of the following statements is/are correct (c) [Al 2 (CH2O)6 ]3+ + 3Cl - (d) [Al(OH)6 ]3- + 3HCl
regarding group 13 elements?
(a) H3BO 3 is a weak acid but HBF4 is a very strong acid 5. General Organic Chemistry and
(b) BF3 is converted into adducts BF3 × H2O and BF3 × 2H2O with
small amounts of water at low temperature Hydrocarbons & Compounds
(c) BF3 fumes strongly in most air and is partially hydrolysed
by excess of water
Containing Halogen
(d) All of the above are correct statements 1. Total number of all isomeric ethers having
molecular formula C5H12O is
38. The chief source of iodine in which it is present as (a) 6 (b) 5 (c) 7 (d) 8
sodium iodate is +
(a) carnallite 2. The most stable species in the given set of ion CH3 ,
+ + +
(b) sea weeds CH 2Br, CH Br2, CBr3 is
(c) caliche + + + +
(d) iodine never exists as sodium iodate (a) CH2 Br (b) CHBr2 (c) CBr3 (d) CH3

MODULE 1
www.jeebooks.in
Prep Analysis for NEET ~ Chemistry 43

3. The product formed on reaction of 1-chlorobutane 14. Rearrangement reactions are shown by
with alcoholic potash is (a) carbanion (b) free radical
(a) butan-2-ol (b) butan-1-ol (c) carbene (d) carbocation
(c) but-2-ene (d) but-1-ene
15. Which of the following is a strongest nucleophile?
4. The IUPAC name of the compound -
(a) EtO - (b) OH
CH3 ¾ CH ¾ C ¾ CH ¾ CH3 is -
(c) CN (d) I-
½ ½½ ½
Br O CH3 16. The decreasing basic character of the following is
(a) 2-bromo-4-methylpentan-3-one I. PhO- II. CH3S-
(b) 4-bromo-2-methylpentan-3-one -
III. OH IV. MeO-
(c) 2, 4-bromo methylpentan-3-one
(a) I > II > III > IV (b) III > IV > II > I
(d) 4-bromo-2-methylpentan-2-one
(c) IV > III > II > I (d) I > II > IV > III
5. Chloroform, when kept open, is oxidised to
(a) COCl (b) COCl 2 (c) CO 2 , Cl 2 (d) CO 2
17. The decreasing order of stabilities of the following
carbanions is
6. The rate of SN 2 reaction is maximum when the - -
I. Ph3C- II. Ph2CH III. PhCH 2
solvent is
(a) benzene (b) DMSO (c) CH3 OH (d) H2O (a) I > II > III (b) II > I > III
(c) III > II > I (d) III > I > II
7. The compound which gives sooty flame on
combustion is 18. Identify the compound X in the following reaction
(a) C 6 H6 (b) CH4 (c) C 2 H6 (d) C 2 H4 + –
NH2 N2 Cl
8. Which of the following compound will react with NaNO2+HCl HBF4
X
NaBH 4 ? 273–278 K

(a) Acetic acid (b) Cyclohexanone


F
(c) Benzoic acid (d) Benzamide
9. The total number of s and p-bonds present in (a) (b)
H 2C == C == CH 2 is
(a) 6 s- bonds, 2p-bonds (b) 4 s- bonds, 2p-bonds F F
(c) 5 s- bonds, 2p-bonds (d) 6 s-bonds, 3p-bonds
F
10. In the organic compound (c) (d)
CH 2 == CH ¾ CH 2 ¾ CH 2 ¾ C ºº CH, the pair of F
hybridised orbitals involved in the formation of
C2 ¾ C3 bond is
19. Which of the following carbocations is least stable?
Å Å
3
(a) sp - sp 3 2
(b) sp and sp 3 (a) PhCH2 (b) CH == CH2
Å Å
(c) sp and sp2 (d) sp and sp3 (c) Me 2 CH (d) CH2 == CH ¾CH2

11. The best method for the preparation of 20. The decreasing order of acidic characters of the
p-nitroiodobenzene from p -nitroaniline is the following is
reaction with I. p-nitrophenol II. o-nitrophenol
(a) NaBH4 followed by I2
III. m-nitrophenol IV. phenol
(b) LiAlH4 followed by I2
(a) I > II > III > IV (b) II > I > III > IV
(c) NaNO 2 / HCl followed by CuCN
(c) I > II > IV > III (d) II > I > IV> III
(d) NaNO 2 / HCl followed by KI
12. Thermal decomposition of alkanes in the absence 21. Which one of the conformations of cyclohexane is
of air is called chiral?
(a) Twist boat (b) Rigid
(a) hydrogenation (b) combustion
(c) Chair (d) Boat
(c) cracking (d) oxidation
13. Propyne on passing through red hot copper tube 22. Which of the following compounds exhibit
forms stereoisomerism?
(a) 2-methylbutene (b) 3-methylbutyne
(a) toluene (b) benzene
(c) 3-methylbutanoic acid (d) 2-methylbutanoic acid
(c) dimethylbenzene (d) mesitylene

MODULE 1
www.jeebooks.in
44 NEET Test Drive

23. In sulphur estimation, 0.157 g of an organic 31. Carbocations are stabilised by three main
compound gave 0.4813 g of barium sulphate. The structural factors. Such factors are neighbouring
percentage of sulphur in the compound is carbon atoms, neighbouring carbon-carbon
(a) 23.62 (b) 72.23 (c) 42.10 (d) 15.02 multiple bonds and neighbouring atoms with lone
pairs. Most stable among the following
24. Among the following compounds, which one has carbocations is
the minimum boiling point?
r
(a) iso-butene (b) But-1-ene (c) n-butane (d) But-1-yne C +
(a) (b) C6H5—CH2
25. Which one of the following carbides forms ethyne
on reaction with water?
(a) Be 2C (b) CaC 2 (c) Al 4C 3 (d) Mg 2C 3 +
(c) (d) CH3—CH—CH3
26. Which of the following compound is least reactive
in Wurtz coupling reaction? r

32. Consider the following species :


(a) C6H5—CH2—Cl (b)
Cl CH3 CH3
Cl r r
Cl
(c) (d)
I. H3C O CH3 II. H3C CH3

NO2 CH3 CH3


r r
27. Match the following columns. III. H3C O CH3 IV. H3C CH3
Column I Column II The correct stability order is
(Reaction) (Directing influence) (a) II > I > III > IV (b) IV > III > II > I
Å
A. —N 1. o, p-directing due to resonance (c) I > III > II > IV (d) I > II > III > IV
H3
B. —CCl 3 2. m-directing due to inductive effect 33. An incorrect statement with respect to S N 1 and S N 2
mechanism for alkyl halide is
C. —CH3 3. o, p-directing due to hyperconjugation
(a) a strong nucleophile in an aprotic solvent increases the
D —NH2 4. m-directing due to reverse rate or favours SN2 reaction
hyperconjuugation (b) competing reaction for SN2 reaction is rearrangement
Codes (c) S N 1 reaction can be catalysed by some Lewis acids
(d) a weak nucleophile and a protic solvent increases the rate
A B C D A B C D
of S N1reaction
(a) 4 2 3 1 (b) 1 2 3 4
(c) 3 4 2 1 (d) 2 1 3 4 34. The substance obtained in between the reactant
and the product. Sometimes, the intermediates
28. Identify ‘X’ and ‘Y ’ in the following reaction, may be quite stable enough. Intermediate involved
Conc.
2 (i) Br in formation of pinacolone is
X ¾¾® Y ¾¾¾¾® CH3CH ºº CH
H 2SO4 (ii) Alc. KOH (a) carbocation (b) carbanion
(c) free radical (d) None of these
(a) CH3CH2Br (b) (CH3 )2 CHBr
(c) CH3CH2CH3 (d) CH3CH2CH2OH 35. Ammoniacal silver nitrate form a white precipitate
with
29. When ethylbromide and n-propyl bromide are
(a) CH2 == CH2 (b) CH3CH == CH2
made to react in presence of sodium in ether, then
the product formed is a (c) CH3C ºº CH (d) CH3C ºº C ¾CH3
(a) mixture of three alkanes (b) mixture of two alkanes 36. Phosphorus in any organic compound can be
(c) mixture of four alkanes (d) single alkane estimated by the formula
62 ´ mass of Mg2P2O7
30. Consider the following reactive species. = ´ 100
222 ´100 of organic compound
(CH3 )3 C, Cl3 C, (CH3 )2CH, C6H5 C H 2
This formula is related to
I II III IV
(a) Carius method
The correct order for the stability of carbanions is (b) Lassaigne’s method
(a) III > II > IV > I (b) II > IV > III > I (c) Kjeldahl’s method
(c) I > III > IV > I (d) IV > II > I > III (d) Duma’s method

MODULE 1
www.jeebooks.in
Prep Analysis for NEET ~ Chemistry 45

37. Essential oils are steam volatile organic Product formed is


compounds, responsible for the fragrance of * * NH2
flowers. At room temperature, these are generally (a) (b)
insoluble in water but are miscible with NH2
water-vapour in vapour phase. The best method for
NH2
the extraction of these oils from the flower is
*
(a) crystallisation (c) (d) Both (b) and (c)
(b) distillation
(c) steam distillation
(d) distillation under reduced pressure 44. When sodium nitroprusside is added to sodium
extract of organic compound it turns pink
38. Which of the following correctly represents the colouration. This test confirm presence of
decreasing order of reactivity of hydrogen halides (a) sulphur (b) chlorine
with propene?
(c) nitrogen (d) None of these
(a) HCl > HI > HBr (b) HI > HBr > HCl
(c) HBr > HI > HCl (d) HCl > HBr > HI 45. On mixing a certain alkane with chlorine and
irradiating it with UV light, it forms
39. The decreasing order of - I-effect of the following is monochlorinated alkane. The alkane could be
I. R 4 N Å II. NO2 (a) iso-pentane (b) pentane
III. CN IV. SO3H (c) neo-pentane (d) propane
V. COOH
(a) I > II > III > IV > V (b) II > I > III > IV > V 6. States of Matter and Solutions
(c) I > II > III > V > IV (d) II > I > V > IV > III
1. The ratio of velocities of diffusion of gases A and B
40. The product formed on reaction of CH3 MgX with is 1 : 5. If the ratio of their masses present in the
CH3C ºº C ¾ H is mixture is 3 : 2, the ratio of their mole fraction is
H H (a) 3 : 50 (b) 1 : 25
½ ½ (c) 50 : 3 (d) 25 :1
(a) CH4 (b) CH3 ¾C == C ¾CH3
2. A solution that is 20% ethanol by volume is found
(c) CH3CH == CH2 (d) CH3C ºº C ¾CH3
to have a density of 0.977 g/mL. Density of ethanol
41. Which of the following carbocations is most stable? is 0.789 g/mL. Thus, mass per cent of ethanol
Me solution is
r (a) 18.72% (b) 16.15% (c) 7.89% (d) 9.77%
r
(a) Me Me (b) Me CH2 3. 4.8 g of a gas at 25° C occupied the same volume as
0.152 g of hydrogen at 17° C and at the same
Me pressure. The molecular mass of the gas is
Me (a) 39.14 (b) 85.83 (c) 64.90 (d) 59.14
(c) Me (d) r
r Me
Me Me
4. The formula of an oxide of titanium that
crystallises as a closely packed array of oxygen
42. Which of the following is not a rearrangement atoms with titanium atoms in two third of the
reaction? octahedral voids is
(a) Ti 3O 2 (b) TiO 2 (c) TiO (d) Ti 2O 3
Me OH Me
Hr Mg/Hg
(a) N (b) O 5. Two bulbs A and B (containing an ideal gas)
Me H2O
Me having volume 5 L and 10 L and pressure of 9 atm
and 6 atm respectively are connected. If the
O O
temperature remains constant, then the final
∆ sOH
(c) R C N3 (d) R C NHOH pressure in the two bulbs will be
(a) 28 atm (b) 14 atm (c) 10 atm (d) 7 atm
43. In the following reaction (where, * represents 14C) 6. The molal freezing point constant for water is
Cl 1.86° C per mol, therefore, the freezing point of
* 0.1 M NaCl solution in water is expected to be
s NH3 (liquid)
+ NH2 (a) - 1.86° C (b) - 0.186° C
(c) + 0.372° C (d) - 0.372° C

MODULE 1
www.jeebooks.in
46 NEET Test Drive

7. The degree of dissociation of Ca(NO3 )2 in a dilute 15. An azeotropic solution of two liquids has a boiling
solution containing 14 g of the salt per 200 g of point greater than either of the boiling points of
water at 100° C is 70%. If the vapour pressure of the two liquids when it
water is 760 mm, then the vapour pressure of (a) is saturated (b) shows negative deviation
solution will be (c) shows no deviation (d) shows positive deviation
(a) 723.48 mm (b) 802.24 mm 16. Rate of diffusion of LPG (mixture of n-butane and
(c) 628.59 mm (d) 745.42 mm propane) is 1.25 times faster than that of SO3 .
8. Pressure of 1 g of an ideal gas A at 300 K is found Hence, mole fraction of n-butane in LPG is
to be 2 bar. When 2 g of another ideal gas B is (a) 0.75 (b) 0.25
introduced in the same flask at the same (c) 0.51 (d) 0.87
temperature, pressure becomes 3 bar. Thus,
17. A solution of 2.5 g of A in 100 g of water lowers the
(a) mA = mB (b) mA = 3mB freezing point by 0.3° C. If substance A
(c) mA = 4mB (d) mB = 4mA tetramerises to the extent of 80% in water, then
9. KCl has the same structure as NaCl. If the volume the molar mass of A is
of the unit cell is 247.7 cubic Å, calculate the (a) 62 (b) 122
distance between adjacent K + and Cl - centres. (c) 244 (d) 31
(a) 6.28 Å (b) 3.48 Å (c) 3.14 Å (d) 2.46 Å 18. The critical temperature of water is higher than
10. AB is a sparingly soluble salt in aqueous solution that of O2 because the H 2O molecules have
with K sp = 1 ´ 10- 8. Calculate the elevation in (a) dipole moment (b) fewer electrons than O 2
(c) two covalent bonds (d) V-shaped structure
boiling point of saturated solution, if
K b(H 2 O) = 0.520 kg mol- 1. 19. The solubility of any gas in the liquid at 1 bar
(a) 5.2 ´ 10 -4
(b) 5.2 ´ 10 -8
(c) 5.2 ´ 10 -5
(d) 5.2 ´ 10 -9 pressure is 0.05 mol/L. On keeping the
temperature constant, its solubility at 3 bar
11. Which of the following statements is/are correct pressure is
about non-stoichiometric compounds? (a) 0.15 mol/L (b) 0.05 mol/L
(a) They do not obey the law of constant composition (c) 0.18 mol/L (d) 0.017 mol/L
(b) They are also called as berthollide compounds
(c) Electrical neutrality is maintained either by having extra
20. If the ionic radii of Rb + and I - are 1.48 Å and
electrons in the structure or by changing the charge on 2.39 Å respectively, then the most probable type of
some of the metal ions structure shown by it will be
(d) All of the above (a) CaF2 type (b) ZnS type
(c) NaCl type (d) CsCl type
12. In two vessels of 1 L each at the same
temperature, 1 g of H 2 and 1 g of CH 4 are taken. 21. The concentration of NO -3 ions in the mixture of
For these, 0.1 M AgNO 3 and 0.2 M NaCl having equal volumes
(a) Vrms values will be same will be
(b) pressure will be same (a) 0.25 M (b) 0.18 M (c) 0.05 M (d) 0.3 M
(c) kinetic energy per mole will be same 22. The pressure of 3 moles of ammonia at 27°C when its
(d) total kinetic energy will be same volume is 8 L according to van der Waals’ equation is
13. An element X crystallises in fcc lattice having edge (Given : a = 4.27 and b = 0.0412)
length 400 pm. Maximum radius of the atom that (a) 8.77 atm (b) 9.33 atm
can be placed in the interstitial site without (c) 6.82 atm (d) 10.33 atm
distorting the structure is
23. A binary solid X +Y - has a fcc structure in which Y -
(a) 83 pm (b) 58.6 pm ions constitutes the lattice and X + ions occupies
(c) 166 pm (d) 178 pm 1
th of the tetrahedral voids. The formula of the
14. The interaction energy of London forces is 4
inversely proportional to sixth power of the solid is
distance between two interacting particles but (a) XY4 (b) XY2 (c) X 2 Y (d) XY
their magnitude depends upon
24. Calculate the total pressure in a 15.0 L cylinder
(a) mass which contains 0.3 g helium, 2.8 g oxygen and 1.2 g
(b) charge nitrogen at 27° C.
(c) polarisability (a) 0.337 atm (b) 33.7 atm
(d) strength of permanent dipoles (c) 3.37 atm (d) 0.0337 atm

MODULE 1
www.jeebooks.in
Prep Analysis for NEET ~ Chemistry 47

25. Which one of the following statements regarding 34. Number of N 2 molecules present in 1 L vessel at
defects in the crystalline solid is/are correct? NTP when compressibility factor is 1.2 is
(a) Frenkel defect is found in halides of alkaline earth metals (a) 2.23 ´ 1024 (b) 2.23 ´ 1022
(b) Frenkel defect is a dislocation defect (c) 2.7 ´ 1022 (d) 2.7 ´ 1024
(c) Frenkel defects decreases the density of crystalline solids
(d) Schottky defect have no effect on the density of crystalline 35. Which of the following gas shows real behaviour?
solids (a) 8 g O 2 at STP occupies 5.6 L
(b) 1 g H2 in 0.5 L flask exerts a pressure of 24.63 atm at 300 K
26. The SI unit of viscosity coefficient is
(c) 1 mol NH3 at 300 K and 1 atm occupies volume 22.4 L
(a) N s m–2 (b) Pa s
(d) 5.6 L of CO 2 at STP is equal to 11 g
(c) kg m -1 s -1 (d) All of these
36. Match the laws given in Column I with expressions
27. Which of the following has the maximum value of given in Column II.
mean free path?
(a) CO 2 (b) H2 Column I Column II
(c) O 2 (d) N2
A. Rault’s law 1. DTf = K f m
28. 25 mL of an aqueous solution of KCl was found to B. Henry’s law 2. p =CRT
require 20 mL of 1 M AgNO 3 solution when titrated
using a K 2 CrO 4 as indicator. The depression in C. Elevation of boiling point 3. p = X1 p10 + X 2 p20
freezing point of KCl solution with 100% ionisation
D. Depression in freezing point 4. DTb = K b m
will be
[K f = 2.0° mol - 1 kg and molarity = molality] E. Osmotic pressure 5. p = KH X
(a) 5.0° (b) 3.2° Choose the correct option.
(c) 1.6° (d) 0.8°
A B C D E
29. At moderate pressures, the van der Waals’ (a) 3 5 4 1 2
equation is written as (b) 5 3 4 1 2
é a ù (c) 1 2 3 5 4
êë p + V 2 úû V = RT (d) 4 1 3 5 2

The compressibility factor is then equal to 37. The unit cells present in a cubic-shaped ideal
crystal of NaCl of mass 1.0 g is
(a) æç1 -
a ö
(b) æç1 -
RTV ö
÷ ÷ (a) 1.28 ´ 1021 (b) 1.71 ´ 1021
è RTV ø è a ø
(c) 2.57 ´ 1021 (d) 5.14 ´ 1021
(c) æç1 +
a ö
(d) æç1 +
RTV ö
÷ ÷
è RTV ø è a ø 38. Which of the following statements is incorrect
about osmotic pressure ( p ), volume (V ) and
30. The edge length of unit cell of a metal ( M w = 24)
temperature (T ) ?
having cubic structure is 4.53 Å. If the density 1
(a) p µ , if T is constant
of metal is 1.74 g cm - 3 , the radius of metal is V
(N A = 6 ´ 1023 ). (b) p µ T, if V is constant
(a) 180 pm (b) 160 pm (c) 140 pm (d) 190 pm (c) p µ V, if T is constant
31. The edge length of a face-centred cubic unit cell is (d) pV is constant, if T is constant
508 pm. If the radius of the cation is 110 pm, the 39. The ratio of the volume of a tetragonal lattice unit
radius of the anion is cell to that of a hexagonal lattice unit cell is (both
(a) 144 pm (b) 288 pm having same respective lengths)
(c) 618 pm (d) 398 pm 3 2 2a2c
(a) abc (b) (c) (d) 1
32. The number of atoms in 100 g of an fcc crystal with 2 3 3b
-3
density = 10.0 g cm and cell edge equal to 200 pm 40. Schottky defect in crystals is observed when
is equal to (a) unequal number of cations and anions are missing from
(a) 5 ´ 1024 (b) 5 ´ 1025 (c) 6 ´ 1023 (d) 2 ´ 1025 the lattice
(b) equal number of cations and anions are missing from the
33. At what temperature will hydrogen molecules have lattice
the same KE as nitrogen molecules at 280 K?
(c) an ion leaves its normal site and occupies interstitial site
(a) 280 K (b) 40 K
(d) density of the crystal is increased
(c) 400 K (d) 50 K

MODULE 1
www.jeebooks.in
48 NEET Test Drive

41. A gas in an open container is heated from 27°C to 5. The gas which is adsorbed greatly by activated
127°C. The fraction of the original amount of the charcoal is
gas remaining in the container will be (a) water vapours (b) CO 2
3 1 1 1 (c) NO 2 (d) SO 2
(a) (b) (c) (d)
4 2 4 8
6. The rate constant for a reaction,
42. Which of the following solutions has minimum 2N 2 O 5 ¾® 4NO 2 + O 2 is 8.0 ´ 10-5 s -1. If the rate is
freezing point? . ´ 10-5 , then the concentration of N 2 O 5
412
(a) 0.01 M NaCl (b) 0.005 M C 2H5OH
(mmol/L) is
(c) 0.005 M MgI2 (d) 0.005 M MgSO 4
(a) 0.52 (b) 0.49 (c) 1.4 (d) 0.80
43. Which of the following contains greatest number of
7. The current in a given wire is 1.8 A. The number of
N-atoms?
coulombs that flow in 1.36 minutes will be
(a) 22.4 L nitrogen gas at STP
(a) 100 C (b) 147 C (c) 247 C (d) 347 C
(b) 500 mL of 2.00 M NH3
(c) 1.00 mole of NH4Cl 8. How many Faraday’s are required to generate one
(d) 6.02 ´ 1023 molecules of NO 2 gram atom of magnesium from MgCl2?
(a) 1 (b) 2 (c) 3 (d) 4
44. A quantity of hydrogen gas occupies a volume of
30.0 mL at a certain temperature and pressure. 9. The atomic weight of Al is 27. When a current of
What volume would half this mass of hydrogen 5 Faraday’s is passed through a solution of Al 3 +
occupy at triple the absolute temperature if the ions, the weight of Al deposited is
pressure were one ninth that of the original gas? (a) 27 g (b) 36 g (c) 45 g (d) 39 g
(a) 270 mL (b) 90 mL 10. For a chemical reaction 2A + B ¾® C, the rate of
(c) 405 mL (d) 137 mL appearance of C is 0.08 mol L -1 min -1. The rate of
45. If a thin slice of sugar beet is placed in disappearance of A will be
concentrated solution of NaCl, then (a) 0.04 mol/L (b) 0.16 mol/L (c) 0.25 mol/L (d) 0.29 mol/L
(a) sugar beet will lose water from its cells 11. Amount of electricity that can deposit 108 g of
(b) sugar beet will absorb water from solution silver from AgNO 3 solution is
(c) sugar beet will neither absorb nor lose water (a) 1A (b) 1C (c) 1F (d) 2 F
(d) sugar beet will dissolve in solution
12. In electrolysis of a fused salt, the weight of the
7. Chemical Kinetics, Redox Reactions, deposit on an electrode will not depend on
(a) temperature of the path
Surface Chemistry and Electrochemistry (b) current intensity
1. Which of the following statements is/are correct (c) electrochemical equivalent of ions
about the zero order reaction? (d) time for electrolysis
(a) Unit of rate constant is concentration -1 time -1 13. For a reaction X + Y ¾® Product, the rate law is
(b) Unit of rate constant is concentration -1
given by, r = k[X ]3/ 2 [Y ]1/ 2. The order of the
(c) Rate of reaction depends on decay constant
(d) Rate of reaction is independent of concentration reaction is
5 1
(a) 2 (b) (c) 1 (d)
2. The rate constant of a first order reaction is 2 2
69.3 ´ 10-3 s -1, then the time it will take to reduce
1 14. In Arrhenius equation, k = Ae- E a / RT ; A may be
the initial concentration to its th value will be called the rate constant at
16
(a) the boiling temperature of reaction mixture
(a) 80 s (b) 40 s (c) 400 s (d) 200 s
(b) very low temperature
æxö (c) zero activation energy
3. When log ç ÷ is plotted against log P, we get a
èmø (d) All of the above
straight line with slope 15. The molar conductance of CH3COOH at 0.1 M
1
(a) m (b) n (c) x (d)
n
concentration is 64.1 S cm 2mol -1.
Given, l° (H + ) = 349.6 S cm 2mol -1and
4. When gelatin is added to an ice-cream, it plays a
role as a/an l° (CH - = 63.5 S cm 2mol -1 the degree of
3 COO )
(a) coagulating agent (b) emulsifying agent
(c) peptising agent (d) flocculating agent
dissociation for CH3COOH will be
(a) 0.1005 (b) 0.155 (c) 1.50 (d) 1.05

MODULE 1
www.jeebooks.in
Prep Analysis for NEET ~ Chemistry 49

16. The half-life of a first order reaction having a rate 26. The experimental rate law for the reaction
constant k = 2.42 ´ 10-10 s -1 is S2O28- ( aq ) + 2I- ( aq ) ¾® 2SO24- (aq ) + I2( aq ) is
-
(a) 2.86 ´ 109 s (b) 1.68 ´ 10-10 s k[S2O2- 2-
8 ] [I ]. If the concentration of S2O8 is halved
-
(c) 4.30 ´ 109 s (d) 2.92 ´ 1010 s and that of concentration of I is increased by
4 times, then the new rate of the reaction is
17. Which of the following is a set of reducing agents ? 1 1
(a) Cr2O 27 - , CrO 24 - , Na (b) I- , Na + , Fe 2 + (a) r2 = 2 r1 (b) r2 = r1 (c) r2 = r1 (d) r2 = 8r1
2 4
(c) F - , Cl - , MnO 4- (d) HNO 3 , Fe 2 + , I2
27. If the rate constant of a first order reaction at 27° C
18. The correct order of oxidation state in the following is 10-4 min -1, then the rate constant (in min -1) at
oxides is 17° C for this reaction will be
(a) KO 2 < OF2 < O 3 < BaO 2 (b) BaO 2 < KO 2 < O 3 < OF2 [Given : Temperature coefficient of this reaction = 4]
(c) BaO 2 < O 3 < OF2 < KO 2 (d) OF2 < KO 2 < BaO 2 < O 3 (a) 4 ´ 10-4 (b) 5 ´ 10-5 (c) 2 .5 ´ 10-5 (d) 2 ´ 10-5
19. For a gaseous reaction X ( g) ¾® Y ( g), the rate of 28. Which of the following statements is/are correct?
reaction can be given as (a) Increase in surface area of catalyst reduces the surface
dX dY 1 dn phase reactions
(a) - = (b) - = k[ X ]n
dt dt V dt (b) In heterogenous catalytic reaction, no surface reaction
d[ X ] occurs
(c) - = k[ X ]n (d) Both (a) and (b)
dt (c) The rate of decomposition of the substance adsorbed on
a surface depends on the surface coverage
20. Which of the following statements is/are incorrect
(d) In chemisorption, there is no disruption of bonding in an
regarding catalyst ?
adsorbed molecule
(a) They are very specific in respect of reaction
(b) They does not alter the equilibrium in a reversible reaction 29. Consider the following reaction :
(c) They can initiate a reaction D
I. NH 4NO3 ¾¾® N 2O + 2H 2O
(d) They remain unchanged in composition and quantity at the
D
end of a reaction II. NH 4NO2 ¾¾® N 2 + 2H 2O
21. Consider the following reaction : D
III. PCl5 ¾¾® PCl3 + Cl2
P4 + 3NaOH + 3H 2O ¾® 3NaH 2PO2 + PH3 Among the given reaction, disproportionation
The given reaction is an example of reaction is not shown by
(a) disproportionation reaction (b) displacement reaction (a) Both I and II (b) Both II and III
(c) combination reaction (d) decomposition reaction (c) Both I and III (d) I, II and III
22. Oxidation numbers of iodine in IO -3 , IO -4 , KI and I 2 30. The number of coulombs required to reduce 12.3 g
respectively are of nitrobenzene to aniline, is
(a) +5, + 7, - 1, 0 (b) -1, - 5, - 1, 0 (a) 96500 C (b) 5790 C (c) 95700 C (d) 57900 C
(c) -2, - 5, - 1, 0 (d) +3, + 5, + 7, 0 31. Given, E°Cr3 + /Cr = -0.72 V
23. In the conversion of Br2 to BrO -3 , the oxidation state E°Fe 2+ / Fe = -0.42 V
of Br changes from
(a) 0 to +5
The potential for the cell
(b) 0 to -3
(c) +1to +5 (d) +2 to +5 Cr / Cr3 + (0.1 M) ||Fe2+ (0.01 M)|Fe is
(a) -026
. V (b) 026
. V (c) 0.339 V (d) -0.339 V
24. The rate of a chemical reaction doubles for every
10° C rise in temperature. If the temperature is 32. The activation energy of a reaction at a given
raised by 80° C, the rate of the reaction increases temperature is found to be 2.303RT J/mol. The
by about ratio of rate constant to the Arrhenius factor is
(a) 64 times (b) 1024 times (a) 0.001 (b) 0.1 (c) 0.002 (d) 0.02
(c) 256 times (d) 512 times 33. For the second order reaction, concentration (A) of
25. An endothermic reaction A ¾® B has an the reactant at time ‘t’ starting with initial
activation energy of 20 kcal/mol and the energy of concentration [ A0 ] is
reaction is 6 kcal/mol. The activation energy for the k + [ A0 ]2 1+ kt [ A0 ]2
(a) (b)
reaction B ® A is 1 + kt kt [ A0 ]2
(a) zero (b) 14 kcal/mol [ A0 ]
(c) (d) None of these
(c) 26 kcal/mol (d) 20 kcal/mol kt [ A0 ] + 1

MODULE 1
www.jeebooks.in
50 NEET Test Drive

34. Which of the following statements is incorrect 40. What amount of current will be required to evolve
about the collision theory of chemical reaction? 20 g of chlorine in 6 h from HCl solution?
(a) Molecules should collide with sufficient threshold energy (a) 10 A (b) 15 A
and proper orientation for the collision to be effective (c) 5 A (d) 2.45 A
(b) Collision of atoms or molecules possessing sufficient
threshold energy results into the product formation
41. Among the following ionic compounds, the one
which will be most effective in precipitating the
(c) Number of effective collisions determines the rate of
sulphur sol is
reaction
(a) Na 3PO 4 (b) AlCl 3
(d) It considers reacting molecules or atoms to be hard
spheres and ignores their structural features (c) BaCl 2 (d) KCl

35. In presence of a catalyst, the activation energy of 42. The value of DH and DS for exothermic
a reaction is lowered by 20 kJ/mol to 10 kJ/mol. chemisorption and physisorption respectively are
What will be the temperature at which uncatalysed (a) (+)ve and (-)ve (b) (-)ve and (+)ve
reaction will have the same rate as that of (c) Both are (-)ve (d) Both are (+)ve
catalysed at 37° C ? 43. In a second order reaction, when the concentration
(a) 327° C (b) 380° C of both the reactants are equal, the reaction is
(c) 347° C (d) 410° C completed in 400 seconds. How long will it take for
36. For the zero order reaction, variation t1 with log a a reaction to go to 80% completion?
2
(a) 6400 s (b) 3200 s
(where, t1/ 2 is half-life period and a is the initial (c) 1600 s (d) 2800 s
concentration) is given by 44. When copper metal is dipped in a solution of HNO 3 ,
log t 1 it produces copper nitrate and some gaseous
log t 1 2 molecule, the change in oxidation number of
2 45°
(a) 45° (b) nitrogen is
(a) +2 (b) +1 (c) +4 (d) 0
O log a O log a 45. Match the half reaction (in Column I) with change
in oxidation number (in Column II) and choose the
log t 1 log t 1 correct code.
2 2
Column I Column II
(c) (d) -
45° A. Cl ¾® ClO 4- 1. 3
O log a O log a B. Br2 ¾® HOBr 2. -1

æ dx ö C. CuSO 4 ¾® Cu2I2 3. +1
+ n
37. If ç ÷ = k[H ] and rate becomes 1000 times + 3
è dt ø D. Cr ¾® CrO 5 4. 8
when pH changes from 3 to 2. Codes
Hence, the order is A B C D
(a) 1 (b) 0 (a) 1 2 4 3
(c) 2 (d) 3
(b) 2 3 1 4
38. Which of the following reaction is not a redox (c) 4 3 2 1
reaction ? (d) 4 2 3 1
(a) CaCO 3 ¾® CaO + CO 2
1
(b) Na + H2O ¾® NaOH + H2
2
8. Coordination Compounds and
1
(c) MnCl 3 ¾® MnCl 2 + Cl 2 d and f-block Elements
2
(d) O 2 + 2H2 ¾® 2H2O
1. In the complex ion [Co(NH 3 )6 ]3 + , the NH 3 molecules
are linked to the central metal ion by
39. How much time is required to do the electroplating (a) ionic bonds (b) covalent bonds
of Ag layer on a coffee tray (30 cm ´ 15 cm) to a (c) coordinate bonds (d) hydrogen bonds
thickness of 1mm using a constant current of
1.0 A. Given that density of Ag is 10.5 g/cm3 . 2. Which one of the following is ambidentate ligand?
(a) 7720 s (b) 120 min (a) SO 2-
3 (b) CN-
(c) 772 s (d) 77.2 s (c) NH3 (d) H2O

MODULE 1
www.jeebooks.in
Prep Analysis for NEET ~ Chemistry 51

3. The coordination number of Cu in complex 13. HCl cannot be used to make the medium acidic in
[Cu(CN)4 ]3 - is oxidation reactions of KMnO 4 in acidic medium.
(a) 4 (b) 3 (c) 2 (d) 1 This is because
(a) KMnO 4 oxidises HCl into Cl 2 which is also an oxidising agent
4. Among the following species, the one which (b) KMnO 4 acts as a reducing agent in the presence of HCl
imparts colour to an aqueous solution is (c) KMnO 4 is a weaker oxidising agent than HCl
(a) Sc 3+ (b) Cu+ (c) Zn 2+ (d) Cr 3+ (d) Both HCl and KMnO 4 acts as oxidising agent
5. Which of the following statements is/are correct 14. Carnallite is solution in H 2 O shows the properties of
about the ionisation energy of Cu and Zn?
(a) K + , Mg 2 + , Cl - (b) K + , Cl - , SO 24 - , Br -
(a) IE 2 of Zn is more than that of Cu
+ 2+
(b) IE 2 of Cu is more than that of Zn (c) K , Mg , CO 23 - (d) K + , Mg 2 + , Cl - , Br -
(c) IE1 of Cu is less than that of Zn
15. Effective atomic number of the central metal ion,
(d) Both (b) and (c)
Pt, in the complex [Pt(NH3 )6 ]4+ is
6. Which one of the following is expected to be a (a) 74 (b) 90 (c) 86 (d) 84
paramagnetic complex?
(a) [Ni(H2O)6 ]2+ (b) [Ni(CO)4 ] 16. The IUPAC name for [Be4O(CH3COO)6 ] is
(c) [Zn(NH3 )4 ]2+ (d) [Co(NH3 )6 ]3+ (a) basic beryllium acetate (II)
(b) hexa - m - hexabis ( acetato) beryllium (II)
7. Which of the following is diamagnetic in nature? (c) hexa - m - acetato (o, o¢)- m 4 -oxo-tetraberyllium (II)
(a) Co 3+ octahedral complex with weak field ligands (d) hexaacetato - m - oxo - beryllate (I)
(b) Co 3+ octahedral complex with strong field ligands 17. The compound which is paramagnetic as well as
(c) Co 2+ in tetrahedral complex coloured is
(d) Co 2+ in square planar complex (a) K 3 [Cu(CN)4 ] (b) VOSO 4 (c) (NH4 )2 [TiCl 6 ] (d) K 2Cr2O 7
8. Which of the following complex does not show 18. The magnetic moment of a transition metal ion is
optical isomerism? 24 BM. Therefore, the number of unpaired
(a) [Cr(C 2O 4 )3 ]3- (b) cis [Pt(Br)4 (en)2 ]2+ electrons present in it, is
(c) [CrCl 2 (NH3 )2 en]+ (d) [Co(NH3 )(NO 2 )3 ] (a) 2 (b) 1 (c) 3 (d) 4

9. A blue solution of copper sulphate becomes darker 19. Among the following lanthanoids, the one for
when treated with excess of ammonia. This is which +2 and +3 oxidation states are common is
because (a) Pr (b) Nd (c) Eu (d) Ce
(a) ammonia molecules replace water molecules in the 20. The transition metal ion which is colourless in
solution aqueous solution is
(b) ammonia is a stronger ligand than water (a) Fe 3+ (b) Ni 2+ (c) Mn2+ (d) Ti 4+
(c) ammonia forms a stable complex ion [Cu(NH3 )4 ]2+ with
Cu2+ ions
21. The enthalpies of hydration of Ca 2+, Mn 2+ and
(d) All of the above are correct Zn 2+follows the order
(a) Mn2 + > Ca 2 + > Zn2 + (b) Zn2 + > Ca 2 + > Mn2 +
10. The oxidation state and effective atomic number (c) Mn2 + > Zn2 + > Ca 2 + (d) Ca 2 + > Mn2 + > Zn2 +
(EAN) of cobalt in [CoF6 ]2 - respectively are
22. Which one of the following is wrongly matched?
(a) 3 and 36 (b) 4 and 35 (c) 4 and 37 (d) 2 and 35
(a) [Cu(NH3 )4 ]2+ - square planar
11. Which among the following will be named as (b) [Ni(CO)4 ] - neutral ligand
dibromidobis (ethylenediamine) chromium (III) (c) [Fe(CN)6 ]3- - sp3d 2
bromide?
(d) [Co(en)3 ]3+ - follows EAN rule
(a) [Cr(en)3 ]Br3 (b) [Cr(en)2 Br2 ]Br
(c) [Cr(en)Br4 ]- (d) [Cr(en)Br2 ]Br 23. Which of these statements about [Co(CN)6 ]3 - is true?
12. Among the given complexes, which of the following (a) [Co(CN)6 ]3- has four unpaired electrons and will be in a
has a magnetic moment of 5.9 BM low-spin configuration
Ni(CO)4 , [Fe(H 2O)6 ]2+ , [Co(NH3 )6 ]3 + , [MnBr4 ]2- , (b) [Co(CN)6 ]3- has four unpaired electrons and will be in a
I II III IV high - spin configuration
2+ (c) [Co(CN)6 ]3- has no unpaired electrons and will be in a
[Zn(NH3 )4 ]
V
high-spin configuration
(a) Both I and II (b) Both II and III (d) [Co(CN)6 ]3- has no unpaired electrons and will be in a
(c) Only IV (d) Only V low-spin configuration

MODULE 1
www.jeebooks.in
52 NEET Test Drive

24. Which of the following species has an atom in 33. Which of the following statements is/are incorrect?
+6 oxidation state? (a) K 2Cr2O 7 solution becomes yellow on increasing the pH
(a) CrO 2Cl 2 (b) Cr(CN)6 3- beyond 7
(c) MnO 4 - (d) NiF62- (b) K 2Cr2O 7 solution is orange in acidic medium
(c) Na 2Cr2O 7 is preferred over K 2Cr2O 7 in volumetric analysis
25. A reduction in atomic size with increase in atomic (d) On passing H2S through acidified K 2Cr2O 7 solution, a milky
number is a characterstic of elements of colour is observed
(a) radioactive series (b) d-block
34. A compound Co(en) 2× (NO2 )2Cl exists in different
(c) f-block (d) high atomic masses
isomeric forms (i.e. A, B and C) with the
26. According to Werner’s theory, coordination characteristic properties as given below:
compounds show two types of valency called
primary and secondary valencies. Secondary Co(en)2 × (NO2 )2 Cl
valency of [Co(H 2 O)4 ]2 + is Property A B C
(a) 4 (b) 3 (i) Optical activity û û ü
(c) 2 (d) 1 (ii) Reaction with AgNO 3 û ü ü

27. Arrange the given complex ions in ascending order (iii) Reaction with en û û ü
of increasing crystal field splitting energy ( D o ).
The structure of B is
I. [Cr(Cl)6 ]3 - II. [Cr(CN)6 ]3 -
III. [Cr(NH3 )6 ]3 + NO2 NO2
(a) I < II < III (b) I < III < II
(c) II < III < I (d) III < II < I (a) en Co en NO2 (b) en Co en Cl

28. Which one of the following characterstics of the Cl NO2


transition metals is associated with their catalytic
activity? en
NO2
(a) Colour of hydrated ions
(b) Variable oxidation states (c) Co Cl (d) None of these
(c) High enthalpy of atomisation NO2
(d) Paramagnetic behaviour en

29. Which of the following statements is/are incorrect


35. The basic character of the transition metal
regarding lanthanoids?
monoxide follows the order (Atomic number,
(a) Lanthanoids have high ionisation energy
Ti = 22, V = 23, Cr = 24, Fe = 26 )
(b) Lanthanoids have high density values
(a) TiO > FeO > VO > CrO (b) CrO > VO > FeO > TiO
(c) Lanthanoids have high melting and boiling points
(c) VO > CrO > TiO > FeO (d) TiO > VO > CrO > FeO
(d) Lanthanoid metal ions are mostly coloured
36. The compound [Ni(CN)4 ]2 - is a complex and
30. Square planar complexes show cis-trans isomerism.
Any complex of the type MA2B2 can exist in diamagnetic in nature. Then, the structure of
cis-trans form. cis form of [Ni(CN)4 ]2 - is
diamminedichloroplatinate (II) is used as an (a) tetrahedral (b) square planar
(a) antiseptic drug (b) anaesthetic drug (c) see-saw (d) trigonal pyramidal
(c) anticancer drug (d) None of these
37. Which one of the following oxide of Mn is acidic in
31. Effective atomic number abbreviated as, EAN, nature?
which is defined as the resultant number of
(a) MnO 2 (b) Mn2O 3 (c) Mn2O 7 (d) MnO
electrons with the metal atom or ion after gaining
electrons from the donor atoms of the ligand. EAN 38. [Co(NH3 )4(NO2 )2 ]Cl exhibits
of Ni in [Ni(CN)4 ]2- is (a) ionisation isomerism, geometrical isomerism and optical
(a) 32 (b) 33 (c) 34 (d) 36 isomerism
32. Electronic configuration of a transition element X (b) linkage isomerism, geometrical isomerism and optical
in +3 oxidation state is [Ar] 3d5 and Y in +2 state isomerism
is [Ar] 3d 8. What are the atomic number of the (c) linkage isomerism, ionisation isomerism and optical
elements? isomerism
(a) 24 and 36 (b) 21 and 25 (d) linkage isomerism, ionisation isomerism and geometrical
(c) 23 and 26 (d) 26 and 28 isomerism

MODULE 1
www.jeebooks.in
Prep Analysis for NEET ~ Chemistry 53

39. Among the given configuration of transition 3. Consider the following reaction:
element, the one which shows highest magnetic (i) CH MgI
3
moment is (CH3 )2CHOH ¾¾¾¾¾® (Y )
(ii) Hydrolysis
(a) 3d 2 (b) 3d 5
(c) 3d 7
(d) 3d 8 In the above sequence of reaction, (Y ) is
(a) isobutyl alcohol (b) n-butyl alcohol
40. Werner’s theory was the first successful theory, (c) tertiary butyl alcohol (d) isobutylene
which explained the properties of complexes.
According to Werner’s theory the primary and 4. The conversion of acetophenone to acetanilide is
secondary valence of [CoCl3 (NH3 )3 ] Cl3 is best accomplished by using
(a) 3 and 3 respectively (b) 6 and 6 respectively (a) Beckmann rearrangement (b) Curtius rearrangement
(c) 3 and 6 respectively (d) 6 and 3 respectively (c) Lossen rearrangement (d) Hofmann rearrangement

41. KMnO4 acts as an oxidising agent in acidic 5. Consider the following reactions:
X
medium. The number of moles of KMnO4 that will CH3CH 2C ºº N ¾ ¾ ® CH3CH 2CHO
be needed to react with one mole of sulphide ion in
acidic solution is The compound X is
1 4 3 2 (a) SnCl 2 / HCl / H2O, boil (b) H2 / Pd-BaSO 4
(a) (b) (c) (d)
5 5 5 5 (c) LiAlH4 /ether (d) NaBH4 /ether/ H3O +

42. In which of the following pairs both the complexes 6. Among the following compounds, C3H7NH 2, NH3 ,
show optical isomerism? CH 3 NH 2 , C2H5NH 2 and C6H5NH 2, the least basic
(a) cis- [Cr(C 2O 4 )2 Cl 2 ]3- , cis-[Co(NH3 )4 Cl] compound is
(b) [PtCl (dien)] Cl, [NiCl 2Br2 ]2- (a) C 3H7NH2 (b) NH3 (c) CH3NH2 (d) C 6H5NH2
(c) [Co(NO 3 )3 (NH3 )3 ] , cis- [Pt(en)2 ]Cl 2 ] 7. Maximum dehydration to form a and
(d) [Co(en)3 ]Cl 3 , cis-[Co(en)2 Cl 2 ]Cl b-unsaturated acid takes place in
(a) lactic acid (b) glycolic acid
43. IUPAC name of [Cr(H 2O)4Cl2 ]Cl is
(c) 3-hydroxypropanoic acid (d) alanine
(a) tetraaquadichlorochromate
(b) tetraaquadichlorochromium (III) chloride 8. Ease of esterification of following acids with
(c) tetraaquadichlorochromium (II) chloride CH 3 OH is
(d) dichlorotetraaquachromium (III) chloride HCOOH, CH3COOH, CH3CH 2COOH
(I) (II) (III)
2-
44. Number of unpaired electrons of Ni in [Ni(CN)4 ] is (a) III < II < I (b) I < II < III
(a) 0 (b) 1 (c) 3 (d) 4 (c) II < I < III (d) Equal in all three
45. When ( n - 1)d orbitals are used in bond formation, 9. Which of the following is a correct IUPAC name for
the complex formed is called inner orbital complex. the compound shown?
Sometimes, in place of ( n - 1)d orbitals, outer nd COOH
orbitals are used for hybridisation. The complex ½
thus formed is called outer orbital complexes. (CH3CH 2 )2 ¾ C ¾ CH 2CH(CH 2CH3 )2
Hybridisation involved in [Fe(CN)5 (H 2O)]3 + is (a) 1, 1, 3-triethylhexanoic acid
(a) sp3d 2 (b) d 2 sp3 (b) 2, 2, 4-triethylhexanoic acid
(c) sp3d 3 (d) None of these (c) 3, 5-diethyl-3-heptyl carboxylic acid
(d) 3, 5, 5-triethyl-6-hexanoic acid
9. Organic Compound Containing 10. An ether reacts with cold H 2SO4 to give
Oxygen and Nitrogen (a) a Zwitter ion (b) an oxonium ion
1. Ethylidene chloride is hydrolysed with aq. NaOH. (c) an oxyanion (d) an alkyl free radical
The product formed is 11. When methanol reacts with formaldehyde in the
(a) ethanol (b) ethanal presence of excess of HCl, the product is
(c) ethanone (d) ethane (a) CH3OCH2Cl (b) CH2Cl 2
2. The vapours of ethyl alcohol are passed over red (c) CH3OCH2OH (d) CH2 (OCH3 )2
hot copper at 573 K to form 12. Aniline is prepared in presence of Fe/HCl from
(a) methane (b) formaldehyde (a) benzene (b) nitrobenzene
(c) acetone (d) acetaldehyde (c) dinitrobenzene (d) None of these

MODULE 1
www.jeebooks.in
54 NEET Test Drive

13. Which of the following reagent used to convert 22. Which one of the following on reaction with lithium
primary amides into primary amines containing aluminium hydride yields a secondary amine?
same number of carbon atoms? (a) Nitroethane (b) Methyl isocyanide
(a) LiAlH4 (b) Br2 + NaOH (c) Acetamide (d) Methyl cyanide
(c) Sn + HCl (d) Na + C 2H5OH
23. Which of the following will not undergo aldol
14. Which of the following is converted into an alcohol condensation reaction?
on treatment with HNO 2 ? (a) Acetaldehyde (b) Propanaldehyde
(a) Ethylamine (b) Aniline (c) Benzaldehyde (d) Tridentereo acetaldehyde
(c) Dimethylamine (d) Triethylamine 24. The pair of compounds in which both the
15. IUPAC name of m-cresol is compounds give positive test with Tollen’s reagent is
(a) 3-methylphenol (b) 3-chlorophenol (a) glucose and sucrose (b) fructose and sucrose
(c) 3-methoxyphenol (d) benzene-1, 3-diol (c) acetophenone and hexanal (d) glucose and fructose

16. OH 25. Which of the following will give iodoform test?


– (a) Isopropyl alcohol (b) Ethanol
O C2H5
‘A’. Here, ‘A’ is (c) Ethane (d) Benzyl alcohol
+ C2H5I
Anhy. (C2H5OH)
26. A substance C 5 H10 O does not reduce Fehling’s
(a) C 6H5I (b) C 6H5OC 6H5 solution, forms a phenylhydrazone, shows the
(c) C 6H5OC 2H5 (d) C 2H5OC 2H5 haloform reaction and can be converted into
17. Name the following reaction n-pentane by strong reduction. The structural
formula of the original substance is
(i) Pyridine
PhCHO + CH 2(COOEt)2 ¾¾¾¾¾¾® (a) CH3COC 3H7 (b) C 2H5COC 2H5
(ii) H2O (c) CH3COCH(CH3 )2 (d) C 4H9CHO
(iii) D, - CO 2
27. Consider the following structures :
PhCH == CHCOOH r
(a) Perkin reaction (b) Aldol condensation NH3 NH3
(c) Claisen reaction (d) Knoevenagel condensation r
18. Gabriel’s phthalimide synthesis is used for the
preparation of (I) (II)
(a) primary aromatic amine (b) secondary amine Which of the following statement is correct?
(c) primary aliphatic amine (d) tertiary amine (a) (II) is not an acceptable canonical structure because
19. The reaction of phenyl magnesium bromide and carbonium ions are less stable than ammonium ions
propanal followed by hydrolysis yields (b) (II) is not an acceptable canonical structure because it is
(a) 2-phenyl-1-propanol non-aromatic
(b) 2-phenyl-2-propanol (c) (II) is not an acceptable canonical structure because
(c) 3-phenyl-1-propanol nitrogen has 10 valence electrons
(d) (II) is an acceptable canonical structure
(d) 1-phenyl-1-propanol
28. To prepare an ether by Williamson’s synthesis, the
20. The major product of the following reaction is
reactants needed are
CrO3 , H 2SO4 (a) ethyl alcohol and tert butyl alcohol
PhCH 2OH ¾¾¾¾¾®
(b) sodium ethoxide and tert butyl bromide
+
O OH (c) sodium tert butoxide and ethyl bromide
(d) sodium ethoxide and sodium tert butoxide
(a) PhCH3 (b) C
Ph H 29. The major organic product of the following reaction is
O O H3 O + , catalyst
H3CC == CHCH3 ¾¾¾¾¾¾¾®
(c) C (d) C Heat
Ph OH
½
Ph H
OH
21. The product formed by the reaction of acetaldehyde
with excess of ethanol in the presence of sulphuric (a) (b)
acid is
(a) C 2H5OCH2CH2OC 2H5 (b) ketal
(c) CH3CHCO(OC 2H5 )2 (d) CH3CH(OH)2 (c) (d)

MODULE 1
www.jeebooks.in
Prep Analysis for NEET ~ Chemistry 55

30. The reagent used for the following conversion 35. The organic compound with formula C 3 H 5 N on
O hydrolysis forms an acid which reduces Fehling’s
solution. The compound can be
(i) – ?
C CH CH3 (a) ethane nitrile (b) isocyanoethane
(ii) – ?
(c) ethoxyethane (d) propane nitrile
In above reaction, reagent (i) and (ii) is 36. Which of the following correctly represents the
(a) (i) O3 /Red (ii) AlCl 3 (iii) MeCOOH decreasing order of reactivity with HBr?
(b) (i) H2 SO4 + HgSO4 (ii) H2O, heat
I. II. H3CO
(c) (ii) O3 /Zn AcOH (ii) H2 SO4 + HgSO4 (iii) H2O, heat OH OH
(d) (i) CH3 COOH (ii) H2O2 + O-H /
OCH3
31. Which of the following statement is incorrect?
III. OH IV. H3C OH
(a) Phenol and aniline give coupling reaction with diazonium
salt
(a) III > IV > II > I (b) II > IV > III > I
(b) Phenol couples with diazonium salt in mild basic
(c) IV > II > III > I (d) III > II > I > IV
conditions ( pH = 8 - 10 )
(c) Aniline couples with diazonium salt in mild acidic condition 37. A compound (X) has the molecular formula C3H7NO.
(pH = 4 - 6) With Br2 and KOH, (X) gives (Y). (Y) responds to
(d) Both phenol and aniline couple with diazonium salt in mustard oil reaction. (Y ) upon treatment with
neutral condition (pH = 7) HNO2 evolves N 2 and gives an alcohol (Z ) which
gives iodoform test. (X ) is likely to be
32. Which of the following method can be used to (a) C 2H5CONH2 (b) CH3CONH2 (c) CH3COONH4 (d) (CH3 )3 N
prepare the compound PhCH 2 CH 2 CH 2 OH ?
O 40% H2SO 4 Isomerisation
NaBH4 PBr4 CuI CH3— OH H3O
+ 38. CH3 ¾ C ºº CH ¾¾¾¾¾¾® A ¾¾¾¾¾¾®
(a) 1% HgSO 4
Ph H CH3 ¾ C ¾ CH3
O O ½½
NaBH4 PBr3 CuI H3O+ O
(b) Ph Structure of ‘A’ and type of isomerism exhibited in
H
O O
the above reaction are respectively
+
NaBH4 PBr4 Li H3O (a) prop-1-en-2-ol, tautomerism
(c) (b) prop-2-en-2-ol, geometrical isomerism
Ph H
O O (c) prop-1-en-1-ol, tautomerism
NaBH4 PBr3 Li H3O+
(d) prop-1-en-2-ol, metamerism
(d) Ph
H
39. Consider the following sequence of reaction:
O
33. The product of refluxing anisole with conc. HBr is C CH BH3 × THF I2 / NaOH D
(a) benzyl bromide and methyl bromide ¾¾¾¾¾® ¾¾¾¾¾® ¾¾ ® ‘X ’
H2O 2 , OH- H+
(b) bromobenzene and methanol
(c) bromobenzene and methyl bromide Here, ‘X ’ obtained is
(d) phenol and methyl bromide O O
34. Which of the following methods is not suitable for COOH COCH3
the preparation of methylene cyclohexane? (a) (b)
Cl
O O
– + CH2CHO
(a) + Me3C OK
(c) (d)
Cl
40. An optically active compound ‘X’ has molecular
(b) + Alc.KOH formula C 4 H 8 O 3. It evolves CO2 with aq. NaHCO 3 .
‘X’ reacts with LiAlH 4 to give an achiral compound.
r s
s
OH
Here, ‘X’ is
(c) O + [Ph3PCH2] I (a) CH3CH2 CHCOOH (b) CH3 CHCOOH
½ ½
OH OMe
OH
Conc.H2SO4 (c) CH3 CHCOOH (d) CH3 CHCH2COOH
(d) ½ ½
CH2OH OH

MODULE 1
www.jeebooks.in
56 NEET Test Drive

41. Which of the following statements is correct? 2. The gaseous envelope around the earth is known
(a) The boiling points of alkylhalides are higher than those of as atmosphere. The lowest layer of this which is
the corresponding alkanes extended upto 10 km from sea level is known as
(b) In water, the solubility of CH3OH > C 2H5OH > C 6H5OH (a) hydrosphere (b) mesosphere
(c) C 6H5NH2 is a weaker base than NH3 (c) stratosphere (d) troposphere
(d) All statements are correct
3. Which of the following vitamins can be stored in
42. The final product obtained in the following reaction is our body?
+ –
14 NaOC2H5 (a) B1 (b) B 2
ClCH2 CH CH2 (c) B 6 (d) B12
O 4. Two isomeric sugars that are cyclic and only differ
14 14
(a) CH2 CH CH2 O C2H5 (b) CH2 CH CH2 O C2H5 in the position of the —OH group at the hemiacetal
group are called
O O (a) enantiomers (b) mutarotations
14 14 (c) anomers (d) epimers
(c) Cl CH2CHCH2 OC2H5 (d) ClCH2 CH CH2 ONa
5. Which one of the following is not amongst the
OH OC2H5
components of photochemical smog?
43. Consider the following reaction, (a) Unsaturated hydrocarbon (b) O 3
OH (c) SO 2 (d) NO 2
Br2
A 6. In acid rain, the acid present in highest
CCl4
O
concentration is
Here, A is (a) carbonic acid (b) sulphuric acid
OH OH (c) nitric acid (d) hydrochloric acid
(a) (b) 7. Chemical compounds used for the treatment of
Br O Br O
depression and hypertension are called
Br (a) analgesics (b) tranquilizers
Br
(c) antibiotics (d) pheromones
OH
(c) (d) O
8. Which of the following enhances leathering
O O
property of soap?
(a) Sodium carbonate (b) Sodium rosinate
Br
(c) Sodium stearate (d) Trisodium phosphate
44. A nitrogeneous substance (X ) is treated with HNO2 9. Serum albumin is an example of a/an
and the product so formed is further treated with (a) enzyme (b) structural protein
NaOH solution, which produces blue colouration. (c) storage protein (d) transport protein
Which of the following can (X) be
(a) CH3CH2NH2 (b) CH3CH2NO 2
10. Which of the following base is normally found in
(c) CH3CH2ONO (d) (CH3 )2 CHNO 2
DNA but not in RNA?
(a) Thymine (b) Adenine
45. How will you distinguish the products obtained by (c) Guanine (d) Uracil
acidic hydrolysis of
Me
11. Which of the following method is used to determine
number-average molecular mass of a polymer?
(I) (II) OCOMe (a) Elevation in boiling point method
OCOMe Me (b) Osmotic pressure method
(a) Fehling’s solution (b) NaHSO 3 (c) Depression in freezing point method
(c) Bredig’s reagent (d) Lucas test (d) Light scattering and ultracentrifuge method
12. Among the following type of polymers, the one
10. Environmental Chemistry, Biomolecules, which has weakest interparticle forces is
Polymers and Chemistry in Everyday Life (a) fibres (b) thermoplastic
(c) thermosettling (d) elastomers
1. The smog is essentially caused by the presence of
(a) O 3 and N2 13. The enzyme which hydrolyses triglycerides to fatty
(b) O 2 and N2 acids and glycerol is called
(c) O 2 and O 3 (a) maltase (b) lipase
(d) oxides of sulphur and nitrogen (c) zymase (d) pepsin

MODULE 1
www.jeebooks.in
Prep Analysis for NEET ~ Chemistry 57

14. Identify the correct statement regarding enzymes. 24. The reactant in an enzyme catalysed reaction is
(a) Enzymes are specific biological catalysts that can normally called
function at very high temperatures (T ~1000 K) (a) hormone (b) substrate
(b) Enzymes are normally heterogeneous catalysts that are (c) co-factor (d) inhibitor
very specific in their action
25. The hormone which controls the processes like
(c) Enzymes are specific biological catalysts that cannot be
burning of fats, proteins and carbohydrates to
poisoned
liberate energy in the body is
(d) Enzymes are specific biological catalysts that possess well
defined active sites (a) cortisone
(b) thyroxine
15. If M w is the weight average molecular mass and (c) adrenalin
M n is the number average molecular mass of a (d) insulin
polymer, then the polydispersity index (PDI) of the
polymer is given by 26. Which of the following monomers is used in the
1 M M preparation of the polymer NBR?
(a) M w ´ M n (b) (c) w (d) n
Mw ´ Mn Mn Mw (a) Buta-1, 3-diene and acrylonitrile
(b) Ethylene glycol and terepthalic acid
16. In which of the following polymers, empirical (c) Formaldehyde and urea
formula resembles with monomer? (d) Adipic acid and hexamethylene diamine
(a) Bakelite (b) Teflon
(c) Nylon-6,6 (d) Dacron
27. Insulin production and its action in human body
are responsible for the level of diabetes. This
17. Which of the following statement is incorrect for compound belongs to which of the following
glucose? categories?
(a) It is an aldohexose (a) A co-enzyme (b) A hormone
(b) On heating with HI, it forms n-hexane (c) An enzyme (d) An antibiotic
(c) It is present in furanose form
28. Biochemical Oxygen Demand (BOD) is a measure
(d) It does not give 2,4-DNP test
of organic material present in water. If BOD value
18. Ozone is formed in the upper atmosphere from is less than 5 ppm, then it indicates that the water
oxygen by the action of sample is
(a) thermal radiation from sunlight (a) not suitable for aquatic life
(b) ultraviolet rays (b) poor in dissolved oxygen
(c) cosmic rays (c) highly polluted
(d) infrared rays (d) rich in dissolved oxygen
19. Which of the following is an antibiotic? 29. Hydrolysis of sucrose is called
(a) Insulin (b) Ascorbic acid (a) inversion (b) esterification
(c) Penicillin (d) Albumin (c) hydration (d) saponification
20. Which of the following is chemically known as 30. The number average molecular mass Mn of a
aspirin? polymer sample in which 20% molecules have a
(a) Methylbenzoate (b) Ethyl salicylate molecular mass of 30,000, 50% have 40,000 and
(c) Acetyl salicylic acid (d) o-hydroxy benzoic acid the rest 30% have 60,000 is
(a) 35,000 (b) 20,600
21. Glucose molecule reacts with ‘X’ number of
(c) 28,100 (d) 44,000
molecules of phenyl hydrazine to yield osazone.
The value of ‘X’ is 31. Today artificial sweeteners and other sugar
(a) four (b) one (c) two (d) three substitutes are found in a variety of food and
beverages marketed as “sugar-free”. Which of the
22. Which one of the following gives positive Fehling’s following is most powerful artificial sweetener?
test?
(a) Saccharin
(a) Sucrose (b) Glucose
(b) Aspartame
(c) Fats (d) Proteins
(c) Alitame
23. Terylene (a condensation polymer) is made up from (d) Sucralose
monomers of
(a) ethylene glycol and terepthalic acid 32. Which of the following reagent is used to convert
(b) ethylene glycol and benzoic acid
glucose to gluconic acid?
(c) ethylene glycol and salicylic acid (a) H2SO 4 (b) Br2 / H2O
(c) HNO 3 (d) Dil. HCl
(d) ethylene glycol and pthalic acid

MODULE 1
www.jeebooks.in
58 NEET Test Drive

33. The types of forces present in the nylon-6,6 39. Which of the following statements is incorrect
polymer is about aspirin?
(a) metallic bonding (a) Aspirin is analgesic
(b) three dimensional network of bonds (b) Aspirin is antipyretic
(c) hydrogen bonding (c) Aspirin is tranquilizer
(d) van der Waals’ forces of attraction (d) Aspirin doesn’t belong to narcotic analgesic
34. Which of the following is incorrect regarding 40. Which of the amino acid contain aromatic side
detergent? chain?
(a) Detergent is more better cleansing agent than soap (a) Histidine (b) Leucine
(b) Detergent can be used in hard water also (c) Glycine (d) Valine
(c) Detergent is soluble in water due to present of long
hydrocarbon chain
41. Vitamin B-12 also called cobalamin is a water
(d) Detergent is insoluble in water due to presence of long
soluble vitamin with a key role in the normal
hydrocarbon chain functioning of the brain and nervous system and
for the formation of blood. Deficiency of vitamin
35. Aspartame is one of the good artificial sweeteners B-12 causes
whose use is limited to cold foods and soft drinks (a) scurvy (b) pale skin
because
(c) night blindness (d) None of these
(a) it has very low boiling point
(b) it gets dissociated at cooking temperature 42. Carbon monoxide, emitted by automobiles,
(c) it is sweetener at low temperature only prevents transport of oxygen in the body. This is
(d) it is insoluble at high temperature because
(a) it combines with oxygen to form carbon dioxide
36. Starch is susceptible to acid hydrolysis which is (b) it prevents reaction between oxygen and haemoglobin
due to the nature of glucose-glucose linkage, which
(c) it forms stable compound with haemoglobin
of the following statements is correct for the nature
(d) it causes destruction of haemoglobin
of starch?
(a) Starch is hemiacetal 43. Which of the following statements is correct
(b) Starch is acetal regarding phenol- formaldehyde resin?
(c) Starch is polymer (a) It involves the formation of methylene bridges in ortho or
(d) Starch contains only few molecules of glucose para-position
(b) Novalac is a cross-linked polymer and is used in paints
37. The pollutants which come directly in the air from
sources are called primary pollutants. Primary (c) Bakelite is a linear-chain polymer and is used in making
pollutants are sometimes converted into secondary combs and handles of utensils
pollutants. Which of the following belongs to (d) None of the above
secondary pollutants? 44. In DNA, the complementary bases are
(a) Peroxyacetyl nitrate (b) NO (a) A ® T, C ® G
(c) CO (d) Hydrocarbon (b) A ® C, T ® G
38. Haemoglobin is the iron containing oxygen (c) Both (a) and (b) are possible
transport metalloprotein in the red blood cells of (d) None of the above
all vertebrates as well as the tissue of some 45. Which of the following reactions of glucose can be
invertebrate. The correct statement in respect of explained only by its cyclic structure?
protein haemoglobin is that it
(a) Glucose forms pentaacetate
(a) acts as an oxygen carrier in the blood
(b) Glucose reacts with hydroxylamine to form an oxime
(b) forms antibodies and offers resistance to diseased
(c) Pentaacetate of glucose does not react with hydroxylamine
(c) functions as a catalyst for biological reactions
(d) maintains blood sugar level (d) Glucose is oxidised by nitric acid to glycolic acid

MODULE 1
www.jeebooks.in

Answer Sheet
1. Some Basic Concepts of Chemistry and Atomic Structure
1. (b) 2. (d) 3. (a) 4. (a) 5. (d) 6. (c) 7. (b) 8. (c) 9. (b) 10. (d)
11. (b) 12. (b) 13. (a) 14. (d) 15. (a) 16. (b) 17. (b) 18. (c) 19. (d) 20. (a)
21. (b) 22. (a) 23. (a) 24. (c) 25. (b) 26. (c) 27. (a) 28. (a) 29. (b) 30. (a)
31. (c) 32. (b) 33. (c) 34. (b) 35. (d) 36. (c) 37. (b) 38. (c) 39. (b) 40. (b)
41. (c) 42. (c) 43. (c) 44. (d) 45. (b)

2. Classification of Elements, Hydrogen and Chemical Bonding & Molecular Structures


1. (d) 2. (b) 3. (d) 4. (a) 5. (a) 6. (c) 7. (c) 8. (c) 9. (a) 10. (a)
11. (b) 12. (c) 13. (d) 14. (b) 15. (c) 16. (a) 17. (c) 18. (a) 19. (d) 20. (d)
21. (c) 22. (a) 23. (d) 24. (b) 25. (b) 26. (c) 27. (c) 28. (b) 29. (d) 30. (d)
31. (a) 32. (b) 33. (b) 34. (b) 35. (b) 36. (c) 37. (a) 38. (d) 39. (a) 40. (b)
41. (b) 42. (d) 43. (a) 44. (b) 45. (b)

3. Equilibrium, Thermodynamics and Extraction of Metals


1. (d) 2. (c) 3. (a) 4. (a) 5. (c) 6. (a) 7. (a) 8. (a) 9. (d) 10. (c)
11. (a) 12. (a) 13. (d) 14. (b) 15. (a) 16. (b) 17. (d) 18. (b) 19. (c) 20. (c)
21. (c) 22. (c) 23. (d) 24. (d) 25. (a) 26. (a) 27. (d) 28. (b) 29. (b) 30. (c)
31. (d) 32. (c) 33. (b) 34. (b) 35. (a) 36. (d) 37. (b) 38. (c) 39. (a) 40. (b)
41. (d) 42. (b) 43. (d) 44. (d) 45. (c)

4. s- and p-block Elements


1. (c) 2. (d) 3. (c) 4. (b) 5. (a) 6. (a) 7. (a) 8. (a) 9. (a) 10. (b)
11. (d) 12. (b) 13. (c) 14. (b) 15. (d) 16. (b) 17. (a) 18. (c) 19. (c) 20. (b)
21. (c) 22. (d) 23. (a) 24. (b) 25. (a) 26. (a) 27. (d) 28. (a) 29. (c) 30. (c)
31. (a) 32. (c) 33. (c) 34. (d) 35. (d) 36. (a) 37. (d) 38. (c) 39. (b) 40. (c)
41. (d) 42. (a) 43. (a) 44. (a) 45. (c)

5. General Organic Chemistry and Hydrocarbons & Compounds Containing Halogen


1. (a) 2. (d) 3. (c) 4. (a) 5. (b) 6. (b) 7. (a) 8. (b) 9. (a) 10. (b)
11. (d) 12. (c) 13. (d) 14. (d) 15. (a) 16. (b) 17. (a) 18. (d) 19. (b) 20. (a)
21. (c) 22. (d) 23. (c) 24. (a) 25. (b) 26. (b) 27. (a) 28. (d) 29. (a) 30. (b)
31. (a) 32. (c) 33. (b) 34. (a) 35. (c) 36. (a) 37. (c) 38. (b) 39. (a) 40. (a)
41. (a) 42. (b) 43. (d) 44. (a) 45. (c)

6. States of Matter and Solutions


1. (a) 2. (b) 3. (c) 4. (d) 5. (d) 6. (d) 7. (d) 8. (d) 9. (c) 10. (c)
11. (d) 12. (c) 13. (b) 14. (c) 15. (b) 16. (c) 17. (a) 18. (a) 19. (a) 20. (c)
21. (c) 22. (a) 23. (b) 24. (a) 25. (a) 26. (d) 27. (b) 28. (b) 29. (a) 30. (b)
31. (a) 32. (a) 33. (a) 34. (b) 35. (c) 36. (a) 37. (c) 38. (c) 39. (b) 40. (b)
41. (a) 42. (a) 43. (a) 44. (c) 45. (a)

7. Chemical Kinetics, Redox Reactions, Surface Chemistry and Electrochemistry


1. (d) 2. (b) 3. (d) 4. (b) 5. (d) 6. (a) 7. (b) 8. (b) 9. (c) 10. (b)
11. (c) 12. (a) 13. (a) 14. (c) 15. (b) 16. (a) 17. (b) 18. (b) 19. (d) 20. (c)
21. (a) 22. (a) 23. (a) 24. (c) 25. (b) 26. (a) 27. (c) 28. (c) 29. (d) 30. (d)
31. (b) 32. (b) 33. (c) 34. (b) 35. (c) 36. (b) 37. (d) 38. (a) 39. (a) 40. (d)
41. (b) 42. (c) 43. (a) 44. (b) 45. (c)

MODULE 1
www.jeebooks.in
60 NEET Test Drive

8. Coordination Compounds and d and f-block Elements


1. (c) 2. (b) 3. (a) 4. (d) 5. (d) 6. (a) 7. (b) 8. (c) 9. (d) 10. (c)
11. (b) 12. (c) 13. (a) 14. (a) 15. (c) 16. (c) 17. (b) 18. (d) 19. (c) 20. (d)
21. (d) 22. (c) 23. (a) 24. (a) 25. (c) 26. (a) 27. (b) 28. (b) 29. (a) 30. (c)
31. (c) 32. (d) 33. (c) 34. (b) 35. (d) 36. (b) 37. (c) 38. (d) 39. (b) 40. (c)
41. (d) 42. (d) 43. (b) 44. (a) 45. (b)

9. Organic Compound Containing Oxygen and Nitrogen


1. (b) 2. (d) 3. (c) 4. (a) 5. (a) 6. (d) 7. (c) 8. (a) 9. (b) 10. (b)
11. (a) 12. (b) 13. (a) 14. (a) 15. (a) 16. (d) 17. (d) 18. (c) 19. (d) 20. (d)
21. (c) 22. (b) 23. (c) 24. (d) 25. (a) 26. (a) 27. (c) 28. (c) 29. (b) 30. (b)
31. (d) 32. (d) 33. (d) 34. (d) 35. (b) 36. (b) 37. (a) 38. (a) 39. (d) 40. (c)
41. (d) 42. (b) 43. (d) 44. (d) 45. (a)

10. Environmental Chemistry, Biomolecules, Polymers and Chemistry in Everyday Life


1. (d) 2. (d) 3. (d) 4. (c) 5. (c) 6. (b) 7. (b) 8. (b) 9. (d) 10. (a)
11. (b) 12. (d) 13. (b) 14. (d) 15. (c) 16. (b) 17. (c) 18. (b) 19. (c) 20. (c)
21. (d) 22. (b) 23. (a) 24. (b) 25. (d) 26. (a) 27. (b) 28. (d) 29. (a) 30. (d)
31. (c) 32. (b) 33. (c) 34. (c) 35. (b) 36. (b) 37. (a) 38. (a) 39. (c) 40. (a)
41. (b) 42. (c) 43. (a) 44. (a) 45. (c)

For Explanation to all the Unit Tests of Chemistry, open the link
https://goo.gl/J4Vx4S

MODULE 1
www.jeebooks.in

PREP ANALYSIS
for NEET
Full Length Tests for each Unit of NEET Biology to Tighten up the Loose Screws

BIOLOGY
Instruction In each unit, question nos. 1 to 25 are of moderate difficulty level and question nos. 26 to 45 are of higher difficulty level.

1. Diversity in Living World 8. The pioneers involved in initiating a plant


succession on rocks are
1. A protein rich green alga is
(a) lichens (b) fungi (c) diatoms (d) mushrooms
(a) Nostoc (b) Ulothrix (c) Chara (d) Chlorella
9. Which of the following birds cannot fly?
2. Pericardial gland of Unio which helps in excretion
(a) Pavo cristatus (b) Passer (c) Corvus (d) Aptenodytes
is also called
(a) green gland (b) Keber’s organ 10. Which one of the following is an exclusive
(c) organs of Bojanus (d) Renette cells character of living beings?
(a) Isolated metabolic reactions occurring in vitro
3. Without exception, which one of the following is (b) Increase in the body mass from inside only
true for all birds?
(c) Perception of happening of events in the environment and
(a) Omnivorous their memory
(b) Have feathers and fly (d) Increase in mass by accumulation of material both on
(c) Have calcareous shelled egg surface as well as internally
(d) Form nest and care them
11. The economically important product sago is
4. Among the following animals, which one will not obtained from which part of the plant Cycas
be considered a true amphibian? circinalis?
(a) Salamander (b) Frog (c) Toad (d) Tortoise (a) Pollen grains (b) Adventitious buds
(c) Seeds (d) Pith
5. Angiosperms have dominated the land flora
primarily because they can 12. In frog, the pharynx communicates with tympanic
(a) adapt in diverse habitats cavity ventrally through
(b) produce large number of seeds (a) semi-circular canal (b) Bidder’s canal
(c) self-pollinate (c) Eustachian tube (d) horizontal canal
(d) be domesticated
13. Which of the following is a consumer decomposer
6. Which division of plants is also called as ‘snakes of protist?
plant kingdom’? (a) Anabaena (b) Clostridium
(a) Pteridophyta (b) Bryophyta (c) Physarum (d) Trypanosoma
(c) Thallophyta (d) Gymnospermae
14. These plants are mostly found in moist and shady
7. Protein coat of a virus enclosing nucleic acid is places because they require the presence of water
known as for fertilisation. Name these plants.
(a) genome (b) vector (a) Pine (b) Teak
(c) plasmid (d) capsid (c) Cycas (d) Moss

MODULE 1
www.jeebooks.in
62 NEET Test Drive

15. Secondary radial symmetry can be found in 27. Which of the following classes belongs to the
(a) Echinodermata (b) Mollusca division of first vascular land plants?
(c) Hemichordata (d) Cephalochordata (a) Adiantum (b) Cycas
(c) Funaria (d) Sphagnum
16. The National Botanical Research Institute is
located at 28. During its life cycle, Fasciola hepatica infects its
(a) Hyderabad (b) Lucknow intermediate host and primary host at which of the
(c) Bangalore (d) New Delhi following larval stages?
(a) Redia–human, miracidium– sheep
17. If the generic and the specific names are similar in (b) Cercaria–sheep, redia–snail
binomial nomenclature, then they are called
(c) Miracidium–snail, metacercaria–sheep
(a) clones (b) intermediate categories
(d) Metacercaria–sheep, cercaria–humans
(c) tautonyms (d) monographs
29. Which one of the following is incorrectly matched?
18. The presence of a muscular diaphragm is an
exclusive character of which class? (a) Reptilia — Possess four-chambered heart
(a) Mammalia (b) Reptilia (c) Amphibia (d) Aves with an incompletely divided
ventricle
19. Which of the following organisms reproduce by (b) Mammalia — Give birth to young ones
longitudinal binary fission? (c) Chondrichthyes — Possess cartilaginous skeleton
(a) Plasmodium and Hydra (b) Paramecium and Planaria (d) Aves — Uropygial gland
(c) Euglena (d) All of these 30. Singing birds make excellent pets if you enjoy the
20. Brown algae such as Laminaria and Fucus are the songs of these flying creatures. The sound
main sources of which mineral? producing organ in these singing birds is
(a) Magnesium (b) Iodine (a) air sacs (b) larynx
(c) Calcium (d) Phosphorus (c) syrinx (d) lungs

21. Dawsonia is also called as 31. Which of the following is formed by the
(a) peat moss (b) bog moss
schizogenous cavity in gymnosperm cells?
(c) largest moss (d) All of these (a) Resin duct (b) Oil glands
(c) Ring porous wood (d) Bitegmic ovules
22. Canal system is present in the phylum
(a) Porifera (b) Arthropoda 32. The haemoglobinuric fever in cattle is caused by a
(c) Annelida (d) Chordata
parasite belonging to which protozoan group?
(a) Flagellated protozoans
23. Mosaic disease is caused by virus in (b) Ciliated protozoans
(a) banana (b) sugarcane (c) grape (d) tobacco (c) Sporozoans
24. Which amongst the following is not a characteristic (d) Amoeboid protozoans
feature of angiosperms? 33. Sycon belongs to a group of animals characterised
(a) These are seed bearing vascular plants by
(b) Jurassic and Cretaceous periods are known as the age of (a) acellular
angiosperms (b) multicellular but without tissue organisation
(c) Pollen grains have non-laminate intine and exine
(c) multicellular with tissue organisation
(d) Sieve tube and companion calls are present
(d) multicellular with gastrovascular system
25. Cyanobacteria share a common trait with 34. In Ulothrix and Spirogyra, reduction division
eubacteria which is the presence of occurs at the stage of
(a) peptidoglycan containing cell wall
(a) gamete formation (b) zoospore formation
(b) oxygenic photosynthesis
(c) zygospore germination (d) vegetative propagation
(c) photosynthetic lamellae
(d) gas vacuoles 35. Which of the following statements is not true for
pteridophytes?
26. Identify the plant by the characteristics given (a) Dominant phase in pteridophytes is sporophyte which is
below. differentiated into root, stem and leaf
I. Antheridial branch is called male flower. (b) Spores of ferns germinate to produce independent
II. Stomata are present on the apophysis. gametophyte
III. Spore on germination gives rise to protonema (c) Diploid zygote develops into sporophytic fern
Choose the correct option. (d) Some cells of sporangium undergo equational division to
(a) Selaginella (b) Cycas (c) Zea mays (d) Funaria form spores

MODULE 1
www.jeebooks.in
Prep Analysis for NEET ~ Biology 63

36. The organisms belonging to kingdom—Protista 43. Study the statements given below and find out the
show which of the following features? incorrect ones.
I. Unicellular and eukaryote (a) Pseudomonas causes denitrification as a result of which
II. Multicellular and eukaryote soil fertility is reduced
III. Unicellular and prokaryote (b) Spirillum rectiphysetaris is responsible for the spoilage of
marine food stuff
IV. Autotroph or heterotroph
(c) Acetobacter aerogenes is capable of partially
Select the correct option. decomposing the hazardous pesticide DDT
(a) I, III and IV (b) I and IV (d) Bacillus papillae is involved in the formation of biogas
(c) Only IV (d) II and IV
44. The male gametophyte in Selaginella is
37. Match the following columns and choose the
(a) 13 cells = 12 cells of antheridium + 1prothallial cell
correct option from the codes given below.
(b) 15 cells = 13 cells of antheridium + 2 prothallial cells
Column I Column II (c) 11 cells = 9 cells of antheridium + 2 prothallial cells
A. Tornaria 1. Starfish (d) 13 cells = 11cells of antheridium + 2 prothallial cells
B. Tadpole 2. Jellyfish 45. Which of the following is not correctly matched?
C. Trochophore 3. Balanoglossus (a) Solanum melongena – Brinjal
D. Brachiolaria 4. Herdmania (b) Cucurbita maxima – Tori
5. Nereis
(c) Brassica campestris – Mustard
Codes (d) Raphanus sativus – Radish
A B C D A B C D
(a) 5 1 4 2 (b) 3 4 5 1 2. Structural Organisation in
(c) 1 5 4 2 (d) 4 2 1 3
Plants and Animals
38. The correct sequence of water flow into body of
Sycon and out of it is 1. Vascular bundles are obtusely wedge-shaped and
(a) Ostia ® Apopyles ® Prosopyles ® Osculum definite in number in
(b) Ostia ® Prosopyles ® Apopyles ® Osculum (a) dicot stem (b) monocot stem
(c) dicot root (d) monocot root
(c) Ostia ® Osculum ® Prosopyles ® Spicules
(d) Osculum ® Apopyles ® Prosopyles ® Ostia 2. Compound epithelium is present in
(a) moist surface of buccal cavity
39. Ambulacral system is also called …A…, which is
(b) inner lining of ducts of salivary glands
the characteristic feature of phylum …B… .
(c) Both (a) and (b)
A B
(d) lining of the stomach
(a) blood vascular system Porifera
(b) blood vascular system Arthropoda 3. Which of the following statements is/are true for
(c) water vascular system Porifera erythropoiesis?
(d) water vascular system Echinodermata (a) RBC formation in 72 hours
40. Development of animals may be direct or indirect. (b) Erythropoietic organ after birth is bone marrow
Indirect development includes larval stage, e.g. (c) Erythropoietic organ in foetus is liver, lymph nodes and
Rhabditiform larva, which is the first stage larva of spleen
(a) Wuchereria (b) Ascaris (c) Ringworm (d) Plasmodium (d) All of the above
41. The conidia and conidiophores are aseptate, while 4. The functions of glial cells include
mycelium and setae are septate. This statement is (a) forming blood-brain barrier
true with respect to (b) digestion of parts of dead neurons
(a) Saccharomyces cerevisiae (c) manufacturing of myelin for neurons
(b) Colletotrichum falcatum (d) All of the above
(c) Streptococcus lactis
(d) Lactobacillus vulgaricus 5. Which among the following statements is/are
correct regarding sapwood?
42. Besides external morphology, it also includes (a) It represents the outward wood of the plant
internal characters. Such a multi-disciplinary (b) It consists of living cells
approach of taxonomy is called
(c) In this, tracheids and vessels are not plugged by tyloses
(a) alpha-taxonomy (b) chemotaxonomy
(d) Both (a) and (c)
(c) beta-taxonomy (d) karyotaxonomy

MODULE 1
www.jeebooks.in
64 NEET Test Drive

6. Mostly tight junctions are (junctions which are 16. While studying morphology and anatomy of
closely associated areas of the two cells) present following plants, roots are not seen in
between (a) Podostemon and Lemna (b) Wolffia and Utricularia
(a) myocardial tissue (b) epithelium cells (c) Pistia and Wolffia (d) Lemna and Utricularia
(c) blood-brain barrier (d) columnar cells 17. In a woody dicotyledonous tree, which of the
7. The schizocarpic fruits include following parts will mainly consist of primary
(a) lomentum, cremocarp, hesperidium, pome
tissues?
(a) Flowers, fruits and leaves (b) Stems, roots and leaves
(b) lomentum, cremocarp, regma, carcerulus
(c) Shoot tips and root tips (d) Whole tree body
(c) hesperidium, lomentum, cremocarp, amphisarca
(d) hesperidium, cremocarp, achenes, follicles 18. Straphiole is
(a) part of funiculus fused with seed wall
8. Phylloclade is photosynthetic succulent specialised
(b) an outgrowth along the hilum
plant part found in
(c) protective covering of the seed
(a) Asparagus (b) Opuntia
(d) place of origin of seed coats
(c) Lilium (d) Euphorbia
9. Which among the following is not the primary 19. Which among the following forms articular
function of stem? cartilage?
(a) Hyaline cartilage (b) Fibrous cartilage
(a) It bears leaves, fruits, flowers and seeds in position
(c) Calcified cartilage (d) Yellow elastic cartilage
(b) It holds flower in suitable position, so that pollination and
fertilisation can take place 20. Xylem vessels are blocked by the development of
(c) It conducts water and minerals to roots, leaves, flowers, (a) tyloses (b) callose plugs
etc (c) Both (a) and (b) (d) None of these
(d) Many stems store food as reserve food materials
21. Which among the following is not a characteristic
10. Choose the incorrect option. of monocot leaf?
(a) Amorphophallus is the largest inflorescence (a) Amphistomatic
(b) Lodicules are two scale-like structures that lie at the base (b) Bulliform cells are present
of the ovary of a grass flower
(c) Mesophyll differentiated into palisade and spongy
(c) The spadix contains a large green or coloured bracts parenchyma
called spathe
(d) Sclerenchymatous cells are present
(d) Fruit in members of Solanaceae is pepo
22. Which one of the following statements is incorrect
11. In cockroach, corpora cardiaca is closely associated about cardiac muscle fibres?
with (a) They are immune to fatigue
(a) heart (b) cloaca (b) They are not supplied with autonomic nervous system
(c) oesophagus (d) kidney
(c) They have a rich blood supply
12. In dorsiventral (dicot) leaf, (d) Property of contraction even when they are isolated from
(a) stomata are almost equally distributed in upper and lower body temporarily
epidermis 23. Which one among the following statements is not
(b) bundle sheath extensions are parenchymatous correct regarding monocot root?
(c) bulliform cells (motor cells) present
(a) Pericycle gives rise to lateral roots only
(d) upper and lower epidermal layers are not clearly
(b) Passage cells generally occur in endodermis opposite the
distinguished
protoxylem point
13. The seat of origin of lateral roots or the root (c) Endodermis is less thickened and contains prominent
branches in dicot root is Casparian strips
(a) epiblema (b) endodermis (d) Cortex is very wide
(c) pericycle (d) pith
24. The salivary gland is an example of
14. In cockroach, mushroom gland is located in (a) multicellular glandular epithelium
(a) 2-5 segments (b) 7-8 segments (b) unicellular glandular epithelium
(c) 3-5 segments (d) 6-7 segments (c) stratified columnar epithelium
(d) stratified compound epithelium
15. Which among the following statements is/are true?
(a) Androecium is diadelphous in Fabaceae 25. Mesangial cells are specialised smooth muscle cells
(b) Asparagus belongs to family–Liliaceae that function to regulate blood flow in the blood
(c) Both (a) and (b) vessels. They are found in which organ of the body?
(d) None of the above (a) Testis (b) Brain (c) Kidney (d) Ovary

MODULE 1
www.jeebooks.in
Prep Analysis for NEET ~ Biology 65

26. The secondary growth in dicot stem comprises of Codes


I. The vascular cambial ring which is completed A B C D A B C D
by both the primary (intrafascicular) strip and (a) 2 4 1 3 (b) 2 3 4 1
secondary (interfascicular) strip. (c) 4 1 2 3 (d) 3 2 4 1
II. The vascular rays are narrow since beginning. 33. Consider the following statements.
III. Annual rings or growth rings are absent. I. Pneumatophores are seen in Rhizophora.
IV. Outward layer of the cortex gives birth to cork II. Adventitious aerial roots are seen in banyan
cambium. tree.
Choose the correct option. III. In Tridax, the stem is decumbent.
(a) I and II (b) II and III IV. If the stem is jointed with solid nodes and hollow
(c) I, II and IV (d) I, II and III internodes, it is called caudex.
27. Which of the following is/are incorrectly matched? V. Maize and sugarcane have prop roots.
(a) Apical meristem — Tips of roots and shoots Choose the correct option with true statements.
(b) Intercalary meristem— Occurs between mature tissues (a) I, II and III (b) I, III and IV
(c) Secondary — Appears later than primary (c) II, IV and V (d) II, IV and III
meristem meristem 34. Which of the following are not characteristic
(d) None of the above features of family– Solanaceae?
28. Identify the correct statement. (a) Berry or capsule type of fruit
(b) Flowers are bisexual
(a) Parenchyma cells can perform photosynthesis
(c) Ovary is superior
(b) Collenchyma cells are responsible for providing
(d) None of the above
mechanical support
(c) Sclerenchyma cells are dead and without protoplasts 35. Which of the following statements is/are correct?
(d) All of the above (a) In non-endospermic seeds, food is stored in seed coat
29. The heart of cockroach is not characterised by (b) Caryopsis is one seeded, dry indehiscent fruit developed
(a) present below the tergal plates from a monocarpellary, unilocular and superior ovary
(b) pulsatile (c) Hesperidium is a modified berry
(d) Both (b) and (c)
(c) neurogenic
(d) 15-chambered 36. Cyathium inflorescence is characterised by
(a) bracts or involucre become fused to form a cup-shaped
30. Which of the following statements is incorrect?
structure
(a) Lymph differs from blood in possessing more WBCs
(b) tricarpellary and syncarpous female flower
(b) Lamina propria is found associated with liver
(c) centrifugally arranged and scorpioid male flowers
(c) Haversian canals are connected with one another via
(d) All of the above
Volkman’s canal
(d) Protein present in cartilage is chondrin 37. Match the following columns and choose the
correct option from the codes given below.
31. Which of the following statements is incorrect?
(a) Perianth is the collective name of the non-essential Column I Column II
floral organs, if there is no distinction between sepals A. Septal nephridia 1. Phallomere
and petals
B. Genitalia 2. Mosaic vision
(b) The term tepals is used to describe the perianth lobes,
C. Open circulatory system 3. Earthworm
which appear like petals
D. Compound eye 4. Cockroach
(c) The corolla of Hibiscus is polypetalous and twisted
(d) Ovary is superior in perigynous flower Codes
A B C D A B C D
32. Match the item in Column I with Column II and
(a) 2 4 1 3 (b) 3 1 4 2
choose the correct alternatives.
(c) 3 4 2 1 (d) 1 2 3 4
Column I Column II
38. Which one of the following options is true?
A. Pneumatophores 1. Ficus (a) Cockroach—10 pairs of spiracles
B. Haustoria 2. Heritiera (b) Earthworm—Setae are present on first clitellum
C. Prop roots 3. Ipomoea (c) Rat—Right kidney is slightly lower in position than left one
D. Tubercular storage roots Viscum (d) Frog—Body divisible into neck and trunk
4.

MODULE 1
www.jeebooks.in
66 NEET Test Drive

39. Identify the correct option. 2. The disc-shaped adhesive junctions between the
(a) Stratified squamous epithelium — Air sacs of lungs cells, containing cadherins are
(b) Stratified cuboidal epithelium — Tubular parts of nephrons (a) ER nucleus complex (b) Golgi bodies
(c) Stratified columnar epithelium— Lining of stomach (c) desmosomes (d) lysosomes
(d) Stratified ciliated epithelium — Move particles in a specific
3. Sedimentation coefficient of ribosome is measured
direction
in
40. Hypodermis in the stem of Cucurbita is made up of (a) Angstrom unit (b) Svedberg unit
angular collenchyma. The vascular bundles are (c) millimicron (d) nanograms
conjoint, open and bicollateral that means
(a) phloem is present towards outer side of xylem 4. Newly made proteins are modified by N-linked
(b) phloem is present towards inner side of xylem glycosylation in
(c) phloem is present on both sides of xylem (a) nucleus (b) Golgi complex
(d) phloem is present in small amount (c) mitochondria (d) endoplasmic reticulum

41. Which of the following correctly represents the 5. Phragmoplast is precursor of which of the
characteristics of smooth muscle fibres? following?
(a) Cylindrical, unbranched, striated, multinucleate and (a) Spindle apparatus (b) Chloroplast
voluntary (c) Plastids (d) Cell plate
(b) Spindle-shaped, unbranched, non-striated, 6. In which one of the following stages do the
uninucleate and involuntary chromosomes show minimum coiling ?
(c) Cylindrical, unbranched, unstriped, multinucleate and (a) Interphase (b) Prophase
involuntary (c) Leptotene (d) Diakinesis
(d) Spindle-shaped, branched, unstriped, multinucleate and
voluntary 7. Which vitamin acts as a coenzyme during the
initial steps of fatty acid synthesis ?
42. Racemose inflorescence is characterised by (a) B1 (b) B 6
(a) Flowers borne in acropetal manner (c) Biotin (d) E
(b) Indefinite growth of flower bearing shoot
(c) Both (a) and (b) 8. Glutathione peroxidase is an enzyme that catalyses
(d) The apex terminates into flower destruction of H 2 O 2 . Which of the following
elements does it contain?
43. In cockroach, central nervous system comprises of (a) Co (b) Fe
I. A bilobed brain. (c) Zn (d) Se
II. One suboesophageal or subpharyngeal ganglion. 9. For proper coordination of the body, cell to cell
III. Proventricular ganglia. communication is must and it is facilitated by one
IV. Ventral nerve cord. of the following structures.
Choose the correct option. (a) Gap junction (b) Tight junction
(a) I and II (b) II and III (c) I, II and IV (d) III and IV (c) Desmosomes (d) Adhering junction
44. In monocot stem, 10. Match the following columns and choose the
(a) the primary vascular tissue remains functional throughout correct option from the codes given below.
the life of the plant
(b) the older vascular tissues cease functioning after some Column I Column II
time and are replaced by newer ones (Cell type) (Cell division)
(c) secondary growth takes place A. Syncytium 1. Free nuclear division
(d) hypodermis is usually collenchymatous
B. Spermatocytes 2. Meiosis
45. Which of the following statements is incorrect (primary)
regarding the dicot root?
(a) Cortex is simple homogeneous C. Amoeba 3. Amitosis
(b) Secondary growth does not occur Codes
(c) Pith is very small A B C A B C
(d) Conjunctive tissue is parenchymatous (a) 1 2 3 (b) 3 1 2
(c) 2 1 3 (d) 1 3 2
3. Cell Structure and Functions 11. Amino acids are mostly synthesised from
1. The pH of the cytoplasm is (a) fatty acids (b) a-ketoglutaric acids
(a) slightly acidic (b) alkaline (c) neutral (d) highly acidic (c) mineral salts (d) biotin

MODULE 1
www.jeebooks.in
Prep Analysis for NEET ~ Biology 67

12. When normal cell has lost the usual control over 23. Which of the following is not consistent with
its division, differentiation and apoptosis, its double helical structure of DNA?
development and progress can usually be linked to (a) A = T, C = G
(a) decreased metastasis (b) hybridomas (b) A + T/ C + G > 1
(c) cell cycle regulators (d) increased contact inhibition (c) DNA density decreases with heat
(d) Both (a) and (c)
13. In a living cell, various biomolecules are found in
different compositions, the average composition of 24. The acidic pH of lysosome is brought about by the
proteins is presence of
(a) 70-90% (b) 10-15% (a) waste materials (b) hydrolytic enzymes
(c) 20-25% (d) 50-60% (c) anabolic enzymes (d) liposomal fractions

14. Which of the following does not take place during 25. In which region of the human chromosome does
prophase-I of meiosis? NOR (Nucleous Organising Region) occur?
(a) Terminalisation (b) Chiasma formation (a) Telomere (b) Centromere
(c) Disjunction (d) Synapsis (c) Secondary constriction (d) Primary constriction

15. In which sub-stage of the meiosis bivalents are 26. Ceramide is classified under which one of the
formed? following lipid categories?
(a) Leptotene (b) Zygotene (a) Glycerophospholipid (b) Sphingolipid
(c) Pachytene (d) Diplotene (c) Triacylglycerol (d) Sulpholipid (galactolipid)

16. During meiosis, in which stage enzyme 27. Which one of the following amino acids does not
recombinase is involved? have an aromatic group in its structure?
(a) Prophase-I (b) Anaphase-I (a) Lysine (b) Tyrosine
(c) Metaphase-II (d) Telophase (c) Phenylalanine (d) Tryptophan

17. Some of the areas on the nuclear membrane are 28. A cell having an extensive Golgi apparatus will
without ribosomes and appear to participate in (a) synthesise large amounts of toxins
(a) phosphorylation (b) photorespiration (b) possess large quantities of cholesterol
(c) produce excess amounts of lipids
(c) blebbing (d) cartilage breakdown
(d) secrete large amounts of proteins
18. Which one of the following organelles is known as
the endoskeleton of the cell? 29. Alkaline hydrolysis of a triglyceride is also known
as
(a) Golgi apparatus (b) Endoplasmic reticulum
(a) esterification (b) hydrogenation
(c) Cytoplasm (d) Cell membrane
(c) saponification (d) dehydration
19. In which one of the following categories are
30. In which of the following, there is no involvement
lecithin and cephalins classified?
of the actin filaments?
(a) Amino acids (b) Complex carbohydrates
(a) Cytoplasmic streaming
(c) Phospholipids (d) Sphingolipids
(b) Amoeboid movement
20. Which one of the following polysaccharides is not a (c) Flagellar movement in bacteria
polymer of glucose? (d) Contraction of smooth muscles
(a) Amylose (b) Insulin
31. The below figure shows the conversion of a
(c) Glycogen (d) Cellulose substatre to product by an enzyme. Identify the
21. Humans are unable to synthesise essential amino components labelled as A, B, C and D from the
acid but they can synthesise non-essential amino components I, II, III, IV given below.
acid on their own. Due to this reason, essential
amino acid has to be taken through food. From the
A
following options, identify the non-essential amino
Energy coordinates B

acid.
(a) Trypsin (b) Isoleucine
C
(c) Lysine (d) Valine
D
22. During which of the following phases does cyclin-B
breakdown? Substrate (S)
(a) During DNA replication (b) During cell wall formation Product (P)
(c) During anaphase (d) During G2/M transition Progress of reaction

MODULE 1
www.jeebooks.in
68 NEET Test Drive

I. Potential energy 38. In biological membranes, the interaction between


II. Transition state the lipids and integral proteins is due to
III. Activation energy with enzyme (a) H-bond (b) van der Waals force
IV. Activation energy without enzyme (c) hydrophobic interactions (d) covalent bond
Starting from A, the components are 39. Which of the following features is present only in
A B C D A B C D eukaryotes and not in prokaryotes?
(a) I ® II ® IV ® III (b) II ® I ® IV ® III (a) Genetic information is enclosed in DNA using identical
(c) I ® II ® III ® IV (d) III ® II ® IV ® I genetic code
(b) Glycolysis and TCA cycle are present
32. While observing the seeds at different stages, it (c) Energy is conserved as ATP
was observed that germinating seeds show higher
(d) Cell is divided into nucleus and cytoplasm, separated by
number of mitochondria than the dormant seeds. nuclear membrane
This is because
(a) dormant seeds become dry and loose many organelles 40. Which of the following is the role of carbohydrates
(b) dormant seeds become impotent therefore number of present on the plasma membrane?
mitochondria and nucleus decrease (a) They act as a carrier of various substances to and fro
(c) due to higher rate of respiration in germinating seeds and (b) They form the channel in membrane
higher energy required (c) They help in molecular recognition
(d) None of the above (d) They play structural role
33. In which of the following, G1-phase of the cell cycle 41. Which one of the following statements is correct
usually absent? with respect to the ER?
(a) Intestinal epithelial cells (b) Senescent fibroblast cells (a) Rough ER is the site for lipid synthesis
(c) HeLa cells in culture (d) Early cleavage stage cells (b) Smooth ER is the site for synthesis and modification of
proteins
34. Given below are statements about competitive
(c) Rough ER is the site for modification and sorting of
inhibition of an enzyme catalysed reaction. Identify
the correct statement. proteins
(a) Competitive inhibitor and substrate can simultaneously (d) RER is related to protein synthesis, whereas SER is
bind to an enzyme involved in sorting of proteins
(b) Vmax remains constant in the presence of competitive 42. Match the Column I (stages of meiosis) with
inhibitor
Column II (characteristic features) and select the
(c) Km for a reaction remains unchanged in the presence of
competitive inhibitor
correct option.
(d) Vmax and Km remain unchanged in the presence of Column I Column II
competitive inhibitor
A. Pachytene 1. Homologous condense and detach
35. Which of the following glycerophospholipids is from the nuclear envelope
the major constituent of the mitochondrial B. Metaphase-I 2. Pairing of homologous
membranes? chromosomes
(a) Lecithin (b) Cardiolipin
C. Diakinesis 3. Terminalisation of chiasmata
(c) Phosphatidyl inositol (d) Phosphatidyl choline
D. Zygotene 4. Crossing over
36. The daughter cells resemble the parent cell in 5. Alignment of chromosomes at
mitosis. In meiosis, they differ from the parent equatorial plate
cells in having half the chromosome number and
also among themselves in genetic condition Codes
because of A B C D A B C D
(a) disjunction and crossing over (a) 2 3 5 1 (b) 3 5 4 2
(b) independent assortment and segregation (c) 1 2 3 5 (d) 4 1 2 5
(c) crossing over, independent assortment and segregation 43. Which of the following stands true with respect to
(d) independent assortment only synapsis?
37. Which one of the following is incorrect, when a cell (a) In meiosis-I, pairing of non-homologous chromosomes
enters M-phase? takes place
(b) In meiosis-I, pairing of homologous chromosomes takes
(a) Condensation of chromatin fibres takes place
place
(b) Nuclear envelope, ER and Golgi apparatus disappear
(c) In meiosis-II, pairing of homologous chromosomes takes
(c) Spindle formation takes place
place
(d) H1 histone proteins are dephosphorylated
(d) Pairing of any two chromosomes takes place
MODULE 1
www.jeebooks.in
Prep Analysis for NEET ~ Biology 69

44. Enzymes increase the rate of reactions by 8. The transpiration pull overcomes
(a) decreasing the energy of activation (a) resistance of narrow xylem channels and their end walls
(b) changing the equilibrium constant of the reaction (b) resistance offered by water coming out of narrow capillary
(c) increasing the free energy change of the reaction pores to the soil
(d) increasing the free energy of activation (c) root pressure
(d) Both (a) and (b)
45. While separating proteins based on mass through
electrophoresis, which of the following detergents 9. The cyclic photophosphorylation
is employed? (a) takes place in the grana of chloroplasts
(a) Dansyl chloride (b) involves cyclic movement of electrons
(b) 1-Fluoro-2, 4-Dinitrobenzene (FDNB) (c) is a photochemical reaction
(c) Sodium Dodecyl Sulphate (SDS) (d) All of the above
(d) Phenylthiocarbamoyl (PTC) 10. Which combination of light is most effective for
photosynthesis?
4. Plant Physiology (a) Blue and green (b) Yellow and green
(c) Green and red (d) Blue and red
1. Major portion of the dry weight of a plant consists
of 11. A proton gradient develops in the lumen of the
(a) C, H, O (b) N, P, K thylakoid of chloroplast because
(c) Ca, Mg, Fe (d) Zn, Mn, Co (a) photolysis of water takes place in lumen of thylakoid
(b) hydrogen ions are transported from stroma of chloroplast
2. Deficiency of molybdenum leads to
to thylakoid lumen
(a) bending of the leaf tips
(c) Both (a) and (b)
(b) brown heart of turnip
(d) None of the above
(c) poor development of vasculature
(d) molting and marginal necrosis of leaves 12. The main features of phasic development theory of
vernalisation include
3. Which one of the following hormones can (a) the growth and development are two distinct
substitute long photoperiods essential for phenomenons
flowering? (b) new phase will come only when the preceding phase has
(a) Cytokinin (b) Auxin (c) Ethylene (d) Gibberellin been completed
4. The actual pressure that develops in a solution, (c) vernalisation accelerates that phase of development which
when it is separated from pure water by means of is dependent upon temperature
semipermeable membrane is (d) All of the above
(a) root pressure (b) turgor pressure 13. Which of the following is the correct sequence in
(c) osmotic pressure (d) wall pressure Krebs’ cycle?
5. The objections to starch-sugar interconversion (a) Oxalosuccinate – Isocitrate – a-ketoglutarate
theory of stomatal movement is/are (b) a- ketoglutarate – Isocitrate – Oxalosuccinate
(c) Isocitrate – Oxalosuccinate – a- ketoglutarate
(a) glucose is not found in guard cells at the time of stomatal
(d) Isocitrate – a-ketoglutarate – Oxalosuccinate
opening
(b) change in pH cannot be explained on the basis of CO 2 14. RQ (Respiratory Quotient) is the ratio of which
concentration gases during the respiration?
(c) starch-sugar changes are chemically slow, while opening Volume of CO 2 absorbed
and closing of stomata are rapid (a) RQ =
Volume of O 2 absorbed
(d) All of the above Volume of CO 2 evolved
(b) RQ =
6. Identify the correctly matched pair. Volume of O 2 evolved
(a) Apoplast pathway— Water moves exclusively through the Volume of CO 2 absorbed
(c) RQ =
cell wall Volume of O 2 evolved
(b) Transmembrane pathway— Water crosses at least one Volume of CO 2 evolved
(d) RQ =
membrane Volume of O 2 absorbed
(c) Symplast pathway— Water moves from one cell to the next
via plasmodesmata 15. Which among the following statements is/are
(d) All of the above correct regarding nitrogenase enzyme?
(a) The enzyme is exclusive to prokaryotes
7. The shield and accessory pigments are
(b) It is present within the bacteroids
(a) chlorophylls (b) carotenoids (c) It gets inactivated when exposed to molecular oxygen
(c) phycobilins (d) allophycocyanin (d) All of the above
MODULE 1
www.jeebooks.in
70 NEET Test Drive

16. Which element helps in the activation of enzymes 25. Krebs’ cycle
of both photosynthesis and respiration? (a) occurs in aerobic respiration only
(a) Calcium (b) Magnesium (b) does not consume ATP
(c) Phosphorus (d) Potassium (c) produces CO 2
(d) All of the above
17. Pasteur efffect is
I. Rate of fermentation is higher under 26. Which among the following statements is incorrect
anaerobic conditions. regarding osmotic pressure?
(a) Osmotic pressure of pure solvent (water) is zero
II. Rate of fermentation is lower under anaerobic
(b) Its value increases due to the addition of solute particles
conditions.
(c) Amount of pressure which must be applied in order to
III. It involves anaerobic breakdown of sugar into prevent the passage of solvent is due to osmosis
CO 2 and ethyl alcohol. (d) Its unit is ‘bars’ with negative sign
IV. It was confirmed by Meyerhoff and Warburg.
27. Consider the following statements.
Choose the correct option containing true
statements. I. ATP was discovered by Lohmann and ATP cycle
by Lipmann, who won Nobel Prize in 1953.
(a) I and II (b) I and III
(c) I, III and IV (d) I and IV II. Acetyl Co-A is an initial compound of Krebs’
cycle.
18. Photosynthesis consists of essentially two biological III. Krebs’ cycle takes place in the matrix and ATP
reactions, one followed by the other, the second of
formation takes place in cytoplasm.
these reactions involves which of the following
events given in the options? Which of the statements given above are correct?
(a) Fixation of CO 2 (b) Trapping of light energy (a) I and II (b) I and III
(c) Synthesis of starch (d) None of these (c) II and III (d) All of these

19. The first stable intermediate product formed 28. Isotopes popularly known to have been used in the
during dark reaction of photosynthesis is study of photosynthesis are
(a) ribulose-5-phosphate (a) P15 and C 32 (b) C11 and P15
(b) 3-phosphoglyceric acid (c) C14 and O18 (d) C15 and N15
(c) fructose-6-phosphate 29. Consider the following statements.
(d) glucose-1-phosphate
I. Calvin cycle is known as C3 - cycle.
20. Antitranspirants are the chemicals which reduce II. C3 -plants have Kranz type of leaf anatomy.
the transpiration in plants. Which hormone from III. CO2-fixation in C3 -plants is done through 3PGA.
the given below is known as the antitranspirant?
(a) Ethylene (b) Abscisic acid
IV. C4-plants show more photorespiration as
compared to C3 -plants, resulting in higher
(c) Gibberellin (d) Auxin
production of dry matter.
21. Dedifferentiation is shown by Which of the statements given above are correct?
(a) interfascicular cambium (b) cell wall
(a) I, II and III (b) I, III and IV (c) I and III (d) II and IV
(c) secondary cortex (d) cork
30. Imbibition plays major roles in
22. Which of the following is the common deficiency (a) swelling of seeds
disease caused due to the lack of potassium in the (b) the initial stage of water absorption by roots
soil? (c) seedling to come out of soil
(a) Die back (b) Bully habit (d) All of the above
(c) Both (a) and (b) (d) Sickle leaf disease
31. Which one of the statements given below is not
23. In indeterminate plants, correct?
(a) floral initiation is independent of photoperiodism (a) Abscisic acid is a naturally occurring growth inhibitor. It acts
(b) require light period of 14-16 hours for flowering as a stress hormone
(c) require continuous dark period for subsequent flowering (b) The most common auxin is indole acetic acid which is the
(d) require short period of light principal naturally occurring auxin in all higher plants
24. The Krebs’ cycle starts with the condensation of (c) Gibberellin was first discovered from bryophyte Gibberella
fujikuroi
(a) a- ketoglutaric acid with citric acid
(d) Barley seeds are rich in carbohydrates. The starch is
(b) acetyl group with oxaloacetic acid
hydrolysed by a-amylase to monosaccharides at the time of
(c) citric acid with succinic acid
germination of seeds
(d) malic acid to oxaloacetic acid

MODULE 1
www.jeebooks.in
Prep Analysis for NEET ~ Biology 71

32. Match the following columns and choose the 37. Which statement is incorrect regarding
correct option from the codes given below. non-oxygenic photosynthesis?
(a) Rhodospirillum carry out photosynthesis by use of H2S
Column I Column II
(b) H2S is an electron donor
A. Sorghum 1. C 3 -plants (c) O 2 is not evolved
B. PEP carboxylase 2. Law of limiting factors (d) Water is the electron donor and O 2 is evolved
C. Blackman 3. Mesophyll cells 38. Identify the correctly matched pair.
D. Photorespiration 4. Kranz anatomy I. Primary nutrients – N, P, K
Codes II. Secondary nutrients – Ca, Mg, S
A B C D A B C D III. Critical elements – N, P, K
(a) 1 2 3 4 (b) 4 3 2 1
IV. Microelements – Zn, Mn, B
(c) 3 2 4 1 (d) 2 1 3 4
Choose the correct option.
33. Consider the following statements.
(a) I and II (b) I, II and III
I. ATP synthesis in mitochondria occurs in (c) II and III (d) All of these
F1-particles or oxysomes present on cristae.
II. If volume of CO 2 liberated during respiration is 39. In amphibolic pathway,
(a) all carbohydrates are first converted into glucose before
more than volume of O 2 used, RQ will be more
they are used for respiration
than one and hence respiratory substrate is
(b) fats have to be converted into fatty acids and glycerol
organic acid.
(c) proteins are degraded by proteases into individual amino
III. Photorespiration is present in C 4-plants. acids
Which of the statements given above are correct? (d) All of the above
(a) I and II (b) I and III
40. In light reaction of photosynthesis, which of the
(c) II and III (d) All of these
following statements is/are true for cyclic
34. Manganese becomes toxic when absorbed by plants photophosphorylation?
in higher amounts because of I. In this process, only PS-I is functional.
I. reduction in uptake of Fe3 + and Mg 2+ . II. Electrons move in a closed circle.
II. Inhibition of binding of Mg 2+ to specific III. Oxygen is not evolved.
enzymes. Choose the correct option.
III. Inhibition of Ca 2+ translocation into shoot apex. (a) I and II (b) I and III
IV. Increase in uptake of Fe3 + and Mg 2+ . (c) II and III (d) All are true
Choose the correct option containing true 41. Which among the following is the natural method
statement(s). of breaking seed dormancy?
(a) I and II (b) II and III (c) I, II and III (d) Only IV I. Completion of over-ripening period.
35. Which of the following is not an advantage of II. Attainment of maturity during dormant
hydroponics over geoponics? period by immature embryo.
(a) Accumulation of toxic organic decomposition products is III. Action of digestive enzymes makes the seed
prevented coat soft and permeable.
(b) No tillering is required and there is no weed growth
IV. Weakening of seed coat by hot water, fat
(c) Soil colloids are present to immobilise nutrients through
adsorption
solvents, strong mineral acids, etc.
(d) Growth of bacteria and fungi is minimised Choose the correct option.
(a) I, II and III (b) III and II
36. In order to show that an element is truly essential,
(c) I and IV (d) II, III and IV
which of the following statements must stand true?
(a) Deficiency of the element should make it impossible for the 42. Which of the following come under roles and
plant to complete its vegetative and reproductive cycle applications of auxins?
(b) Role or function of element should be irreplaceble by I. Xylem differentiation II. Parthenocarpy
another element
III. Weed control IV. Cell elongation
(c) The effect of the element should not be result of interaction
with other elements and should have some role in Choose the correct option.
metabolism (a) I and II (b) I and III
(d) All of the above (c) I, II and III (d) All of these

MODULE 1
www.jeebooks.in
72 NEET Test Drive

43. Guttation occurs during night or early morning 5. Which of the following is not a catecholamine?
when there is (a) Dopamine (b) Epinephrine
(a) high atmospheric humidity and transpiration is less (c) Norepinephrine (d) Serotonin
(b) low atmospheric humidity and transpiration is more
6. The cystic vein which is a part of the hepatic portal
(c) low atmospheric humidity and transpiration is less
system
(d) high atmospheric humidity and transpiration is more
(a) collects blood from spleen and pancreas
44. In electron transport chain, (b) receives blood from the stomach
I. For every pair of electrons that enter by the way (c) collects blood from the wall of colon and caecum
of NADH, 3 ATPs result. (d) receives blood from gall bladder
II. For every pair of electrons that enter by the way 7. The blood vessel that drains the blood from the
of FADH 2, 2 ATPs result. glomerulus is
III. Oxygen is the final acceptor of the electrons. (a) renal artery (b) renal vein
IV. Cytochrome-a3 donates electrons to free (c) efferent arteriole (d) afferent arteriole
molecular oxygen.
8. Juxtaglomerular Apparatus (JGA) is a complex
Choose the correct option.
structure involved in
(a) I and II (b) I, III and IV
(c) I and III (d) All of these (a) mechanism for the functioning of smooth muscles
(b) regulation of glomerular filtration rate
45. Which of the following statements is/are correct in (c) reabsorption of urine filtrate
relation to respiration in plants? (d) concentrating the urine
(a) In respiration, potential energy is converted into kinetic
energy 9. Neurosecretory hormones are secreted in pars
(b) It is an exothermic process nervosa and stored in
(c) It leads to the formation of other necessary (a) Herring bodies (b) Starling bodies
compounds that are important as cell constituents (c) Carotid bodies (d) Rathke’s pouch
(d) All of the above 10. Which of the following cells secrete histamine,
serotonin, etc., and are involved in inflammatory
5. Human Physiology reactions?
(a) Neutrophils (b) Monocytes
1. In diphyodont dentition, how many teeth are (c) Basophils (d) Lymphocytes
present in milk dentition?
(a) 12 (b) 20 11. Which of the following valves does not allow the
(c) 28 (d) 32 passage of blood in the right auricle?
(a) Eustachian valve (b) Haversian valve
2. Castle’s Intrinsic Factor or CIF secreted by the (c) Thebesian valve (d) Semilunar valve
parietal or oxyntic cells of the gastric glands helps
in 12. In the process of haemodialysis, the blood of the
(a) absorption of monosaccharides patient that is pumped into the haemodialyser is
(b) releasing mucus for lubrication taken from
(c) absorption of vitamin-B12 (a) arteries (b) veins
(d) secreting proenzyme pepsinogen (c) either arteries or veins (d) interstitial fluid

3. Sarcomere which is the functional unit of 13. Nutrients absorbed by the intestinal villi go into
contraction is present the liver through
(a) in the centre of I-band (a) aorta (b) hepatic artery
(b) in isotropic band (c) posterior vena cava (d) hepatic portal vein
(c) in isotropic band 14. Volume of the air inspired or expired during a
(d) between two successive Z-lines normal respiration is called
4. A chronic disorder in which alveolar walls are (a) residual volume (b) inspiratory capacity
damaged thereby, decreasing the respiratory (c) tidal volume (d) vital capacity
surface is 15. Under normal physiological conditions, how much
(a) asthma oxygen can be delivered to the tissues by 100 mL of
(b) emphysema blood?
(c) tuberculosis (a) 10 mL (b) 5 mL
(d) occupational respiratory disorder (c) 2 mL (d) 0.9 mL

MODULE 1
www.jeebooks.in
Prep Analysis for NEET ~ Biology 73

16. Corpus callosum connects 25. Due to certain medical reasons, the thymus gland
(a) two ventricles of brain (b) two cerebral hemispheres was removed in an infant. What implications
(c) two cerebellar hemispheres (d) two optic thalamus would this have?
(a) There would be lack of lymphocytes and antibodies
17. Internal stimuli like hunger, thirst, etc., are
(b) There would be lack of lymph nodes in his body
received by
(a) interoceptors (b) statoreceptors (c) White blood cells will become more in number
(c) thermoreceptors (d) proprioceptors (d) Mast cells will decline

18. The function of the iris is 26. The reflex arc which has only two neurons viz.
(a) to control the colour of the eyeball sensory and motor neurons which form one
(b) to control the quantity of light entering the eye synapse is called as
(c) to help in vision as it contains visual proteins (a) asynaptic reflex arc (b) disynaptic reflex arc
(d) to nourish the lens and the cornea (c) polysynaptic reflex arc (d) monosynaptic reflex arc

19. A person travelling in a Boeing aeroplane suddenly 27. Choose the correct statement in reference to
primary myofilament.
hears an announcement that there is a technical
(a) They are more than secondary myofilament
fault in the plane and there would be an
(b) Primary myofilaments have smooth surface
emergency landing. Which of the following is likely
(c) Primary myofilament consists of two proteins
to happen to his neurohormonal control system? (d) They slide during muscle contraction
(a) Neurotransmitters become active and start
28. Spironolactone acts at which part of the nephron?
transmitting nerve impulses rapidly
(a) Its site of action depends upon its concentration
(b) The hypothalamus activates the pituitary by a
feedback mechanism (b) It is still the matter of research
(c) Sympathetic nervous system is activated and adrenal (c) It acts on collecting tubules
medulla secretes the hormones epinephrine and (d) It acts on PCT
norepinephrine
29. The enzyme that assists in the hydration of carbon
(d) Sympathetic nervous system is activated and adrenal
dioxide in the erythrocytes is
cortex secretes the hormones epinephrine and
(a) carboxy peptidase (b) carbonic anhydrase
norepinephrine
(c) succinic dehydrogenase (d) thrombokinase
20. Choline acetylase is the enzyme that helps in
(a) conduction of nerve impulse across synapse
30. If I. represents tyrosine,
(b) release of more adrenaline II. represents hydroxylation,
III. represents decarboxylation and
(c) repolarisation
IV. represents phenylalanine.
(d) excitation of nerve axon
Then, in the following process of the synthesis of
21. Tympanum or eardrum is present in which part of
norepinephrine, the markings A, B, C and D stand
the ear?
(a) External ear (b) Middle ear for. D
i. A ¾¾® Dopa
(c) Inner ear (d) Labyrinth
B
ii. Dopa ¾¾® Dopamine
22. Simmond’s disease is caused due to atrophy of
(a) anterior lobe of pituitary iii. Transport of dopamine into the vesicles.
C
(b) intermediate lobe of pituitary iv. Dopamine ¾¾ ® Norepinephrine.
(c) posterior lobe of pituitary Choose the correct option.
(d) Both (b) and (c) A B C D A B C D
23. In the mammalian skull, the suspensorium is (a) IV II III III (b) I II III III
formed by squamosal. Therefore, it is called (c) I III II II (d) IV III II II
(a) autostylic skull (b) craniostylic skull 31. What will be the impact on heart, if the sinoatrial
(c) chondro cranium (d) Both (a) and (c) node stops functioning?
24. Osteomyelitis is caused due to the (a) The opening and closure of tricuspid and bicuspid valves
(a) inflammation of bone due to pus producing organism is affected
(b) inflammation of bursae present in the joints (b) Both atria and ventricles contract simultaneously
(c) reduction of bone tissue mass causing weakening of (c) The rhythmic and coordinated contraction will cease in the
bones cardiac muscles
(d) inflammation of synovial membranes of many joints (d) The pulmonary arteries and veins will become
simultaneously non-functional

MODULE 1
www.jeebooks.in
74 NEET Test Drive

32. Vermis is found in which of the following brain 38. Raman is a 20-year-old and is studying in junior
structures? college. He is majoring in biology and hopes some
(a) Cerebrum (b) Medulla oblongata day to be a pediatrician. Beginning about a month
(c) Cerebellum (d) Brain stem ago, Raman noticed that he was waking up once,
sometimes twice at night, by the need to go to the
33. The granulocytes that are involved in bathroom. More recently, he has noticed that the
inflammatory reactions contain
need to go to the bathroom during the day is much
(a) B and T-lymphocytes more frequently than before, sometimes as often as
(b) serotonin and vasopressin once in every hour. His blood sugar is found to be
(c) histamine and heparin normal. The most probable cause is
(d) serotonin and relaxin (a) excess of insulin in blood
34. Which of the following statements is not correct? (b) deficiency of glucagon in blood
(a) Pineal gland secretes serotonin and melatonin (c) hyposecretion of posterior pituitary hormone
(b) Adrenaline is an amine (d) hypersecretion of pituitary hormones
(c) Kidneys do not secrete steroid hormones
(d) Oxytocin is secreted by posterior lobe of the pituitary
39. Which of the following statements is/are true for
the structure of ear?
35. Match the following columns and choose the I. Malleus is attached to tympanic membrane.
correct option from the codes given below. II. Anvil is attached to oval window.
Column I Column II III. Malleus is attached to oval window.
A. Dysuria 1. Output 0-100 mL/day IV. Stapes is attached to oval window.
B. Anuria 2. WBC or pus in urine Choose the option with correct statements.
(a) II and IV (b) I and III
C. Pyuria 3. Painful urination
(c) I and IV (d) None of these
D. Ketonuria 4. Presence of acetone bodies in urine
40. Which part of the digestive system normally
5. Urine output less than 17-21 mL/hr in harbours symbiotic bacteria?
adults (a) Oral lining and tongue surface
Codes (b) Colon
A B C D A B C D (c) Rectum and appendix
(a) 1 2 3 4 (b) 3 1 2 4 (d) Mucous lining of stomach
(c) 5 2 1 3 (d) 4 3 5 2
41. An integral protein ATPase is involved in the Na +-
36. Study the following pairs carefully and identify the K + pump found in the membranes of nerve cells.
pair which is not correctly matched. One ATP molecule is used to
(a) Tendon–Connective tissue (b) Smooth muscle–Involuntary (a) three ions of Na + are taken in and two K + are pumped out
(c) Myosin–Contractile protein (d) Troponin–Fibrous protein (b) three ions of Na + are pumped out and two K + are taken in
(c) two ions of K + are absorbed and two ions of Na + are
37. Paracetamol is the most common medicine used
pumped out
during the febrile condition. It is excreted by
following processes. (d) one ion of Na + is thrown out and two K + are absorbed

A. Sulphotransferase 42. Which of the following is a heterocrine gland?


I. Pancreas II. Gonads
Paracetamol B. Oxidation by cytochrome P450 III. Gastric gland IV. Parathyroid
Choose the correct option.
C. Glucuronide conjugation
(a) I, II, III and IV (b) I and II (c) I, II and III (d) I and IV
43. Aphasia, a condition in which there is word deafness
Excreted D Increase paracetamol and inability to speak is due to the injury of
(a) amygdaloid body (b) Broca’s area
Identify the location of reactions A, B, C and D of
(c) pineal gland (d) hindbrain
above process.
A B C D 44. After inspiration of a normal tidal volume, a
(a) Liver Liver Liver Kidney person exhales as much air as possible. This
(b) Stomach Kidney Liver Kidney volume of air expired is called
(c) Kidney Lung Kidney Liver (a) vital capacity (b) expiratory capacity
(d) Kidney Muscle Liver Kidney (c) expiratory reserve volume (d) total lung capacity

MODULE 1
www.jeebooks.in
Prep Analysis for NEET ~ Biology 75

45. Which of the following is not involved in the 10. In angiosperms, functional megaspore develops as
synthesis of testosterone? a result of free nuclear division into
(a) Cholesterol (b) ICSH (a) embryo sac (b) ovule
(c) Oxytocin (d) Oestrogen (c) endosperm (d) zygote
11. Identify the incorrect statement regarding
6. Reproduction in Plants oogenesis.
and Animals (a) It has long resting periods
(b) It forms one functional and three non-functional cells
1. The technique of vegetative propagation, in which (c) Its growth phase is very short
roots are artificially induced on the stem branches (d) The division of primary oocyte is not equal
is
(a) cutting (b) grafting 12. These processes are compulsory for the complete
(c) micropropagation (d) layering development of the male gametophyte from pollen
mother cell
2. A kind of plant apomixis in which the embryos and (a) Two meiotic cell divisions
seeds are formed without meiosis and fusion of (b) One mitotic cell division and two meiotic cell divisions
gametes is (c) One meiotic cell division and two mitotic cell divisions
(a) diplospory (b) agamospermy (d) Two meiotic cell divisions and one mitotic cell division
(c) adventive embryony (d) apospory
13. Barrier method of birth control includes
3. Which part of male reproductive system is the site (a) lactational amenorrhoea
of sperm maturation and storage? (b) diaphragms
(a) Vas deferens (b) Testes (c) IUDs
(c) Epididymis (d) Seminal vesicle
(d) vasectomy
4. Which of the following is true for the arrangement
of the nuclei in a polygonum type of embryo sac in 14. The most widely accepted method of contraception
the angiospermic plants? in India, in which a small piece of plastic or metal
device is placed in uterus is
(a) 2 + 4 + 2 (b) 2 + 3 + 3
(a) tubectomy (b) IUDs
(c) 3 + 2 + 3 (d) 3 + 3 + 2
(c) cervical caps (d) rhythmic method
5. The part of the gynoecium that determines the
compatibility with the right pollen grain is 15. Study the following statements about reproduction
(a) stigma (b) style in honeybees and select the correct one.
(c) ovary (d) nucellus (a) Fertilised eggs of honeybee develop into drone
(b) The unfertilised eggs develop into female honeybee
6. Which one of the following is a correctly matched
pair? (c) The female honeybee developed parthenogenetically
(a) Binary fission–Plasmodium (d) The eggs that develop without fertilisation produce male
drone bees
(b) Transverse fission–Paramecium
(c) Multiple fission– Yeast 16. Tubuli recti in male testis are formed by end to
(d) Sporulation–Marchantia end fusion of
(a) ejaculatory duct (b) vasa efferentia
7. Vegetative propagation in mint and (c) seminiferous tubules (d) vas deferens
Chrysanthemum occurs by
(a) rhizome (b) sucker (c) runner (d) stolon 17. What is meant by double fertilisation which is a
characteristic feature of all angiosperms?
8. After ovulation, Graafian follicle changes to (a) Fusion of male gamete with antipodal cell
(a) corpus callosum (b) corpus artesia
(b) Fusion of male gamete with synergid cell
(c) corpus luteum (d) corpus albicans
(c) Fusion between egg and male gamete
9. If the egg remains unfertilised, then (d) Fusion of one male gamete with egg cell and another with
(a) there is increase in the level of progesterone leading to secondary nucleus
ovulation
(b) there is decline in the level of progesterone leading to
18. In an angiospermic embryo sac; n , 2n , 3n
menstruation conditions are found respectively in
(c) there is degeneration of Graafian follicle leading to (a) egg, synergids, antipodal
ovulation (b) endosperm, egg, zygote
(d) levels of progesterone and oestrogen rise sharply leading (c) egg, zygote, endosperm
to menstruation (d) endosperm, synergids, nucleus

MODULE 1
www.jeebooks.in
76 NEET Test Drive

19. Gonorrhoea and syphilis are 26. Totipotency will be displayed in which amongst the
(a) MTPs (b) IUDs (c) STDs (d) IVF following plant cells?
(a) Xylem tracheids (b) Xylem vessels
20. ‘Saheli’, an oral contraceptive for females was
(c) Sieve tube cells (d) Meristematic cells
developed by
(a) CDRI, Lucknow (b) NBRI, Lucknow 27. Acrosome content that prevents polyspermy in
(c) NII, Delhi (d) IISc, Bangalore humans is
(a) hyaluronidase (b) neuraminidase
21. In humans, desiccation of the embryo inside the
(c) zona lysin (d) acrosin
uterus is prevented by
(a) allantois (b) placenta 28. Assisted reproductive technology, ZIFT is helpful
(c) chorion (d) amnion for the infertile couples to have children. This
technique involves
22. Given below is the diagrammatic sketch of a certain
(a) transfer of embryo of up to 8 blastomeres in the Fallopian
type of angiospermic ovule. Identify the type of
tube of the female
ovule.
Hilum (b) transfer of ovum from the donor in the Fallopian tube of
female
Funicle (c) semen collected from the healthy donor is artificially
Micropyle
introduced in the female reproductive tract
Micropylar pole
Outer integument (d) Both (b) and (c)

Inner integument 29. In obstetric and gynecology, the term ‘nulliparous’


is frequently used. It refers to woman having
Nucellus
Embryo sac (a) uterus absent
(b) no children
(c) irregular or absent menstrual cycle
Chalazal pole (d) ovary absent
30. Egg activation during fertilisation is related to
(a) Orthotropous (b) Anatropous (a) secreting hyaluronidase enzymes
(c) Amphitropous (d) Circinotropous (b) preventing polyspermy
23. Foetal sex-determination test based on the number (c) dissolving egg envelope
of chromosomes is (d) pH change of egg cytoplasm
(a) artificial insemination (b) sterilisation 31. Following is the diagram of TS of an anther.
(c) contraception (d) amniocentesis Identify the parts labelled as A, B, C and D.
24. IUD stands for Intrauterine Devices. They may be
of following types
A
I. Medicated
B
II. Fe (iron) releasing
III. Hormone releasing C
D
Choose the correct option.
(a) I and II (b) I and III A B C D
(c) II and III (d) None of these (a) Tapetum Epidermis Endothecium Connective
25. In the given figure identify (b) Connective Epidermis Endothecium Sporogenous tissue
(c) Tapetum Epidermis Endodermis Middle layers
(d) Connective Epidermis Endodermis Sporogenous tissue
A
32. Your one of the relatives has just married. She
called you and requested to suggest the most
appropriate method of contraception. Which of the
D B following contraceptions would be your first
C suggested choice?
I. Site of fertilisation II. Site of implantation (a) Oral contraceptive pills (b) IUDs (Intra Uterine Devices)
(a) I–A, II–B (b) I–D, II–C (c) I–A, II–C (d) I–C, II–D (c) Tubectomy (d) I-pill

MODULE 1
www.jeebooks.in
Prep Analysis for NEET ~ Biology 77

33. In a type of apomixis known as adventive (c) Anatropous ovules are types of ovules in which micropyle,
embryony, embryos develop directly from the funicle and chalaza lie in a straight line
(a) endosperm (b) nucellus or integuments (d) Circinotropous ovules are completely inverted
(c) synergids (d) antipodals 41. The following are the diagrams of three modes of
34. Transfer of pollen grains from male flower to the entry of pollen tube into ovule. Identify the modes
stigma of another female flower growing on the labelled as A, B and C.
same plant is called Pollen tube
(a) chasmogamy (b) geitonogamy
(c) karyogamy (d) autogamy
35. In flowering plants, after fertilisation, the ovules
mature into seeds. Inside the mature seed is the
progenitor of the next generation. Which of the
following statements is true about the progenitor
mentioned above?
(a) It is the first cell of the sporophytic generation
Pollen tube
(b) It is formed after some mitotic divisions and is a product
formed after syngamy (A) (B) (C)
(c) It occurs only in the cross-pollinated plants A B C
(d) It is highly evolved in monocot plants (a) Chalazogamy Mesogamy Porogamy
36. Which one of the following statements is true (b) Porogamy Chalazogamy Mesogamy
regarding the stages of embryological development? (c) Mesogamy Porogamy Chalazogamy
(a) Penetration of zona pellucida with the acrosomal lysins (d) Porogamy Mesogamy Chalazogamy
results in fertilisation
42. Identify the type of birth control devices based on
(b) Cleavage involves series of meiotic divisions
the features given below.
(c) Outer layer of blastocyst differentiates into embryo
I. Requires the erection of penis.
(d) Gastrulation occurs on fifth day to the end of second week
II. Can cause excess menstrual bleeding and
37. At puberty in males, the immature male germ cells pain.
produce sperms by the process of spermatogenesis. III. Relatively unreliable and sometimes messy.
Which of the following cells are in the haploid state I II III
during this process? (a) Diaphragm Cervical cap Implant
(a) Spermatogonial cells (b) Germinal epithelial cells
(b) Levonorgestrel Hormonal pill Norplant
(c) Secondary spermatocytes (d) Primary spermatocytes
(c) Condom IUD Vaginal suppositories
38. The formation of triploid primary endosperm (d) Cervical cap Oral contraceptive Condom
nucleus is as a result of which of the following
43. In the given diagram, part labelled as A, B, C and
events?
D are respectively identified as
(a) Double fertilisation (b) Triple fusion
(c) Parthenogenesis (d) Both (a) and (b) A
39. Which structure in the given diagram gives rise to
embryo?
A

B B
C
D D

C
(a) A (b) B (c) C (d) D
40. Study the following statements and select the
A B C D
correct one.
(a) Orthotropous ovules turn over completely forming a (a) Polar nuclei Egg Antipodals Synergids
complete circle around the ovule (b) Synergids Filiform apparatus Egg Central cell
(b) Amphitropous ovules are curved with (c) Antipodals Polar nuclei Central cell Egg
horse-shoe-shaped embryo sac (d) Polar nuclei Central cell Egg Antipodal

MODULE 1
www.jeebooks.in
78 NEET Test Drive

44. The pattern of cleavage in an egg will be affected 6. Which among the following statements is correct
by for reciprocal cross?
(a) oxytocin secreted by maternal pituitary I. It can be used to study inheritance of one,
(b) amount of yolk and its distribution in egg two or more traits.
(c) interstitial cell stimulating hormone II. Distinguishes between nuclear and
(d) period of conception in a woman cytoplasmic inheritance.
45. Match the following columns and choose the III. Studies inheritance of a single trait.
correct option from the codes given below. IV. Distinguishes between sex-linked and
autosomal inheritance.
Column I Column II
Choose the correct option.
A. Miscarriage 1. Premature degeneration of (a) I and II (b) I and III (c) I, II and IV (d) I, III and IV
corpus luteum
7. The sequence annotation involves
B. Pregnancy test 2. Animal pole (a) identifying all the genes that are expressed as RNA
C. Luteal phase 3. Progesterone secretion (b) sequencing the whole set of genome and then assigning
functions of different regions
D. Polar bodies 4. hCG (c) address ethical, legal and social issues
Codes (d) transfer related technologies to other sectors
A B C D 8. When packaging of DNA takes place in a
(a) 1 3 2 4 nucleosome, the DNA is wrapped around
(b) 1 4 3 2
(c) 3 2 4 1 (a) RNA (b) ribosomes (c) nucleus (d) histones
(d) 2 1 4 3 9. Which one of the following type of selection
operates in most of the natural populations?
7. Genetics and Evolution (a) Stabilising selection (b) Disruptive selection
(c) Kin selection (d) Directional selection
1. The first eukaryotic cells emerged during the
(a) Cenozoic (b) Proterozoic
10. Appearance of ancestral characters in newborns
like tail, etc., are
(c) Mesozoic (d) Palaeozoic
(a) analogous (b) vestigial (c) homologous (d) atavistic
2. During which of the following periods did life
originate on earth?
11. Which of the following is a biogeographical
evidence in favour of organic evolution?
(a) Proterozoic (b) Mesozoic
(a) Modern horse (b) Archaeopteryx
(c) Cambrian period (d) Archean eon
(c) Darwin’s finches (d) Reptiles
3. Herbaceous lycopods and arborescent lycopods 12. A bacterial gene that is transcribed continually is
evolved from known as
(a) Psilophyton (a) monocistronic (b) constitutive
(b) Zosterophyllum of Palaeozoic era (c) repressible (d) inducible
(c) vascular plants in Silurian period
13. When gene migration happens multiple times, it is
(d) bryophytes called
4. Phenylketonuria, a disorder caused by mutation is (a) gene flow (b) genetic drift
an example of (c) mutation (d) genetic recombination
(a) multiple alleles (b) pleiotropy 14. The significance of law of dominance includes
(c) polygenic inheritance (d) codominance (a) the explanation for occurrence of 3 : 1 ratio in F 2
individuals
5. Which among the following correctly represents the
(b) the explanation for F1-generation expressing trait of only
central dogma?
one parent
Transcription Translation (c) Both (a) and (b)
(a) DNA ¾¾¾¾® RNA ¾¾¾¾® Protein
Transcription (d) None of the above
Translation
(b) RNA ¾¾¾¾® DNA ¾¾¾¾® Protein 15. In homozygous condition, which of the following
Transcription Translation chromosomal changes will be the most harmful?
(c) Protein ¾¾¾¾® RNA ¾¾¾¾® DNA
Transcription (a) Translocation (b) Inversion
Translation
(d) DNA ¾¾¾¾® Protein ¾¾¾¾® RNA (c) Deletion (d) Duplication

MODULE 1
www.jeebooks.in
Prep Analysis for NEET ~ Biology 79

16. By careful analysis of human genetics, how many 28. If colourblind woman marries a normal visioned
linkage groups will be revealed? man, all their sons would be
(a) 2 (b) 4 (c) 23 (d) 46 (a) normal or colourblind
(b) colourblind
17. Lucy comes under which of the following species?
(c) one eye having colour blindness
(a) Australopithecus afarensis (b) Homo habilis
(d) normal vision
(c) Homo erectus (d) Australopithecus robustus
18. Random breeding within a deme is known as 29. With reference to Griffith experiment complete the
following.
(a) hybridisation (b) panmixia
I
(c) introgression (d) fecundity ...... Virulent S-cells + Live mice ® Mice died ®
19. When due to different needs, some structures Virulent S-cells
II
develop differently, the condition is called Non-virulent + Live mice ® Mice ...... No bacteria
(a) convergent evolution (b) adaptive radiation Heat-killed S-cells + Live mice ® Mice lived ¾®
(c) divergent evolution (d) chemical resistance No bacteria
III
20. With which of the following does an anticodon Heat-killed S-cells +Live mice ® Mice ...... Virulent
pair? + S-cells
(a) Amino acid (b) tRNA Live R-cells
(c) mRNA codons (d) DNA strand (a) I–Non, II–died, III–died (b) I–‘A’, II–lived, III–died
21. The probe used in DNA fingerprinting is (c) I–Live, II–lived, III–died (d) I–Non, II–lived, III–died
(a) VNTRs (b) SNPs (c) dystrophin (d) mRNA 30. Excessive hair development on the external ears is
22. DNA-dependent RNA polymerase catalyses the also known as hypertrichosis. It is
polymerisation in (a) autosomal trait
(a) 5¢ ® 3¢ direction (b) 3¢ ® 5¢ direction (b) sex-chromosome-Y related trait
(c) Both (a) and (b) (d) None of these (c) sex-chromosome-X related trait
(d) Both (b) and (c)
23. Which of the following is the reason for Mendelian
recombinations? 31. The enzyme which helps in reducing the tension of
(a) Evolution DNA helix during its unwinding in front of the
(b) Linkage replication fork is
(c) Independent assortment of characters (a) ligase (b) topoisomerase
(d) Mutation (c) helicase (d) endonuclease

24. The structure of the chromosomes may change 32. Which among the following statements depicts the
due to difference between DNA replication and
(a) duplication (b) deletion transcription?
(c) inversion (d) All of these (a) The total DNA of an organism gets duplicated in replication
(b) In transcription, only a segment of DNA or only one of the
25. Mutations may be caused due to exposure to two strands is copied into RNA
(a) blue light (b) infra-red light (c) Both (a) and (b)
(c) radio frequency (d) X-rays (d) None of the above
26. Which among the following statements is/are 33. The Untranslated Regions (UTR) are
incorrect for lac operon? I. additional sequences in an mRNA that are not
(a) Lactose is known to be the inducer and the substrate for translated.
enzyme b-galactosidase
II. present at 5¢ end (before start codon) and 3¢ end
(b) Consists of one regulatory gene
(after stop codon).
(c) a gene codes for b-galactosidase
(d) z gene codes for b-galactosidase III. improve efficiency of translation process.
IV. decrease efficiency of translation process.
27. Identify the correctly matched pair.
(a) Har Gobind Khorana – Synthesis of RNA molecule Choose the correct option.
(b) Marshall Nirenberg – Cell-free system for protein (a) I and II (b) II and III (c) I, II and III (d) I, II and IV
synthesis 34. Humans and monkeys can be categorised under
(c) Severo Ochoa – Polynucleotide phosphorylase which of the following taxonomic groups?
helped in polymerising RNA (a) Hominoids (b) Prosimians
(d) All of the above (c) Hominids (d) Anthropoids

MODULE 1
www.jeebooks.in
80 NEET Test Drive

35. The first true hominids have been placed under the 43. Examples of biogenetic law includes
genus (a) presence of fish-like characters in tadpole larva of frog
(a) Homo (b) Dryopithecus (b) presence of filamentous green algae-like structure,
(c) Australopithecus (d) Aegyptopithecus protonema during development of Funaria
(c) pteridophytes, primitive gymnosperms have
36. Read the following description.
flagellated sperm and water dependency
Fossils found in East Africa, first hominid, i.e. (d) All of the above
human-being-like, brain capacity 650-800 cc, did
not eat meat. 44. Which among the following statements is incorrect
Which among the following correctly possesses the for sickle-cell anaemia?
above characteristics? (a) Only homozygous individuals for Hbs show diseased
(a) Homo habilis (b) Homo erectus phenotype
(c) Homo sapiens (d) Dryopithecus (b) Heterozygous individuals are carriers
(c) Homozygous individuals are carriers
37. Identify the incorrectly matched pair. (d) It is an autosomal-linked recessive trait
(a) Incomplete dominance – Antirrhinum majus
45. Identify the correct sequence in the evolution of
(b) Codominance – ABO blood group
modern horse
(c) Polygenic inheritance – human skin colour
I. Miohippus II. Eohippus
(d) Dominant epistasis – coat colour gene in mice III. Equus IV. Mesohippus
38. Morgan selected fruitfly, Drosophila melanogaster V. Merychippus
for his experiments because Choose the correct option.
(a) they could be grown on simple artificial medium in the (a) I ® II ® V ® IV ® III (b) III ® IV ® II ® I ® V
laboratory (c) II ® IV ® I ® V ® III (d) V ® I ® II ® IV ® III
(b) a single mating could produce a large number of flies
(c) there is clear differentiation of the sexes
(d) All of the above
8. Biology in Human Welfare
1. Which of the following characteristics is/are shown
39. DNA helicase is a highly conserved group of by acquired immunity?
enzymes, defects in it may lead to various
(a) Specificity
disorders. The major function of DNA helicase is
(b) Memory
(a) opening supercoils of DNA
(c) Discrimination between self and non-self
(b) making DNA helix
(d) All of the above
(c) unwinding of major grooves specifically
(d) opening two chains of DNA helix 2. Which of the following is a vector for both dengue
and yellow fever?
40. Essential conditions of Hardy-Weinberg principle are
(a) Escherichia coli (b) Nitrosomonas
I. No mutation (c) Aedes aegypti (d) Mycobacterium
II. Genetic drift
III. Gene flow 3. The flocs in secondary treatment of sewage are
IV. No genetic recombination (a) masses of bacteria associated with fungal filament
Choose the correct option. (b) soil and small pebbles
(c) floating debris
(a) I and II (b) II and III (c) I and IV (d) None of these
(d) inoculum
41. Identify the correct statement from the following 4. In …… , superior males and females of the same
options. The frequency of Down’s syndrome breed are identified and then mated.
increases (a) inbreeding (b) outbreeding
(a) during 1st pregnancy (c) out-crossing (d) interspecific hybridisation
(b) in mothers of at least 3 children
(c) when age of mothers is below 35 years 5. An example of biofortification is
(d) when age of mother is above 35 years (a) atlas-66 (b) flat bean
(c) okra (d) rapeseed mustard
42. Which of the following is not a correct match? 6. In diabetes mellitus, the blood sugar level increases.
(a) Sex-determination — A chromosomal phenomenon Which of the following is not a symptom of diabetes
(b) Red-green colour — A sex-linked character mellitus?
blindness in human (a) Polydipsia
(c) Y-chromosome — Autosomal (b) Blood cholesterol level rises
(d) An abnormal — A case of polyploidy (number
(c) Excessive urination
chromosome in each cell)
(d) Ketoacidosis common

MODULE 1
www.jeebooks.in
Prep Analysis for NEET ~ Biology 81

7. Among the following which is/are the symptoms of 19. Kojic acid is
alcoholic withdrawal ? (a) result of fermentation
(a) Anxiety (b) Insomnia (b) obtained by Aspergillus oryzae
(c) Shaky hands (d) All of these (c) is used for skin whitening
8. IgA is present in (d) All of the above
I. Tears II. Saliva III. Colostrum 20. Chrysanthemum is used to prepare which
Choose the correct option. insecticide?
(a) I and II (b) II and III (c) I and III (d) I, II and III (a) Pyrethrum (b) Carbamide
(c) Allethrin (d) Thiocarbamate
9. Which one is not used for the detection of cancer or
internal organs? 21. Which one of the following is caused by Lyssa virus?
(a) Radiography (a) Mumps (b) Measles
(b) Computed Tomography scan (CT scan) (c) Trachoma (d) Rabies
(c) Magnetic Resonance Imaging (MRI)
22. Streptococcus lactis is useful in
(d) Enzyme Linked Immuno Sorbent Assay (ELISA)
(a) ripening and souring of cheese
10. Identify the incorrectly matched pair. (b) curdling of milk
(a) P. vivax® Quartan malaria (c) sewage treatment
(b) P. falciparum® Pernicious malaria (d) biogas production
(c) P. ovale® Mild tertian malaria
23. Kallu, a traditional drink of some parts of
(d) None of the above
Southern India is made by fermenting
11. Parbhani Kranti is a (a) palm (b) bamboo
(a) new variety of mung bean (c) grape (d) rice
(b) new variety of Abelmoschus esculentus
24. Which breed serves the dual purpose of good milk
(c) new variety of Triticum aestivum yield and also ploughing of fields?
(d) new variety of Brassica
(a) Sahiwal (b) Aseel
12. In somatic hybridisation, the fusion of parent (c) Deoni (d) Jersey
protoplasts is induced by 25. An example of biocontrol agent is
(a) polyethylene glycol (b) phytohormones
(a) Baculoviruses (b) Bacillus thuringiensis
(c) colchicine (d) None of these
(c) Trichoderma (d) All of these
13. Which of the following is an opiate narcotic? 26. Which among the following is/are characteristic(s)
(a) LSD (b) Barbiturates of benign tumours?
(c) Amphetamines (d) Morphine
I. The cells of this tumour resemble the original
14. Which among the following is not a protozoan mature cells.
disease? II. They do not show metastasis.
(a) Kala-azar (b) Malaria (c) Amoebiasis (d) Q-fever
III. Fibroids are benign tumours of connective
15. Sour taste of vinegar is due to which one of the tissue.
following acids? IV. They show metastasis and contact inhibition.
(a) Lactic acid (b) Acetic acid
(c) Butyric acid (d) Fumaric acid Choose the correct option.
(a) I and II (b) I, II and III (c) Only I (d) Only II
16. If a person is feeding only on meat, egg and milk
then he will suffer from 27. Choose the incorrect statements about biogas from
(a) night blindness (b) scurvy
the codes given below.
(c) rickets (d) beri-beri I. It is primarily composed of methane (50-60%)
and hydrogen sulphide (30-40%).
17. Which one of the following is the most important
II. Cattle dung is the most important substrate for
for transfection in animals?
biogas as it contains cellulose.
(a) Papilloma virus (b) Adenovirus
(c) Agrobacterium (d) Retrovirus III. Acidogenesis is the first step of biogas
production.
18. A bioactive molecule which is used as a blood
cholesterol lowering agent is Choose the correct option.
(a) I and II (b) I and III
(a) cyclosporin-A (b) statins
(c) II and III (d) All of these
(c) lipases (d) streptokinase

MODULE 1
www.jeebooks.in
82 NEET Test Drive

28. Triticale is an example of 35. Which among the following is/are the
(a) cross made between the plants belonging to different characteristics of definitive host?
genera of the same family I. In which sexual reproduction occurs.
(b) cross made between the two species of the same genus II. Which harbors the mature form.
(c) cross between the plants of different varieties
III. In which asexual reproduction occurs.
(d) cross between the plants of the same variety
IV. Which harbors the immature form.
29. Which among the following statements is/are Choose the correct option.
correct regarding symbiotic nitrogen-fixation? (a) I and II (b) I and IV
I. Rhizobium, group of bacteria, form nodules on (c) II and III (d) III and IV
the roots of leguminous plants.
II. Nodules contain an oxygen binding haem
36. Worker bees have been provided with some specific
features such as
protein called leghaemoglobin.
I. They have a powerful sting for defence.
III. Leghaemoglobin is present in the cytoplasm of
infected nodule cells at high concentration. II. They have long proboscis for sucking the
nectar.
IV. Rhizobium possesses an enzyme complex
nitrogenase. III. They have pollen baskets for the collection of
pollens.
Choose the correct option.
IV. They have four pairs of pocket-like wax
(a) I and II (b) II and III (c) Only III (d) All of these
secreting glands on ventral surface.
30. Third generation vaccines are
Choose the correct option.
(a) DNA vaccines which stimulate cell-mediated immunity
(a) I and II (b) I and III
(b) preparations that include the whole microorganism
(c) III and IV (d) All of these
(c) produced by subunits of microorganisms
(d) not of uniform quality 37. Choose the incorrect statement about antibodies.
(a) They are synthesised by an animal to combat foreign
31. Which among the following are the qualities of material
culturable fishes?
(b) They directly join an antigen to destroy the latter
(a) Able to feed on natural as well as artificial food diet
(c) They may occur on the surface of a microbe or occur as a
(b) Able to tolerate a sudden change in the climatic conditions
free molecule
(c) Their meat should be pleasing to the taste and much
(d) It is a protein molecule
nutritive
(d) All of the above 38. The microorganisms involved in the production of
biogas do not include
32. Monoamine Oxidase (MAO) inhibitors form the
(a) hydrolytic and fermentative bacteria
major class of antidepressants. It
(b) syntrophic hydrogen producing bacteria
I. is a mitochondrial enzyme.
II. is involved in oxidative deamination of (c) acetogenic bacteria
biogenic amines. (d) anaerobic nitrogen-fixing bacteria
III. occurs as two isoenzymes, i.e. MAO-A and 39. Teratogens affect the foetus at
MAO-B.
(a) fertilisation and implantation
IV. Liver contains both MAO-A and MAO-B.
(b) organogenesis which is the most vulnerable period
Choose the correct option. (c) growth and development where functional
(a) I and II (b) I and III (c) I, II and III (d) All of these abnormalities can occur
33. An insect hormone strategy used in controlloing (d) All of the above
pesticides includes 40. Identify the incorrectly matched pair.
I. ecdysone II. confusion technique (a) Biofortification – Protein content and quality
III. juvenile hormone IV. pheromone traps (b) Single cell protein – Spirulina
Choose the correct option. (c) Sterilisation strategy – Tomato mosaic virus
(a) I and II (b) I and III (c) III and IV (d) All of these
(d) Autopolyploids – Colchicine treatment
34. Smoother crops are
41. Indentify the incorrectly matched pair.
(a) genetically engineered plants
(b) the crops, which do not allow the weeds to grow nearby (a) Tuberculoid leprosy – Few lesions on the skin
(b) Lepromatous leprosy – No lesions on the skin
(c) weeds which may be made useful by using them as
(c) Syphilis – Primary chancre
vegetable or fodder
(d) Plague – Black death
(d) crops that fix atmospheric nitrogen

MODULE 1
www.jeebooks.in
Prep Analysis for NEET ~ Biology 83

42. Read the following statements about a synthetic 2. A ‘superbug’ strain of Pseudomonas, used in
hallucinogen, Phencyclidine Piperidine (PCP) and controlling water pollution due to oil spills, has
choose the incorrect statement about it. A different strains. Here A is
plasmids of ......
(a) It has analgesic properties
(a) Two (b) Four (c) Five (d) Six
(b) It results in chromosomal and foetal abnormalities
(c) It is widely used in veterinary medicine to immobilise large 3. Which of the following is a transgenic plant?
animals (a) Hirudin (b) Triticale (c) Flavr Savr (d) All of these
(d) It is obtained from Psilocybe mexicana
4. Most widely used bioweapon is
43. Read the following statements about complement (a) Bacillus subtilis (b) Pseudomonas putida
system. (c) Bacillus anthracis (d) None of these
I. It consists of over 30 proteins found in blood
5. Transgenic plants are
II. Complement proteins do not create pores in the (a) produced by somatic embryo in an artificial medium
cell wall of microbes. (b) produced by introducing foreign DNA into the cell and
III. Some components of the complement system developing a plant from that cell
form a coat around the invading microbes. (c) produced after the protoplast fusion in an artificial medium
IV. It causes agglutination of microbes, (d) grown in an artificial medium after hybridisation in the field
neutralisation of viruses, etc. 6. Which of the following techniques is based on the
Choose the correct option. principle of antigen-antibody interaction?
(a) I and II (b) II and III (c) III and IV (d) I, III and IV (a) PCR
(b) ELISA
44. Read the following statements about heterosis.
(c) Recombinant DNA technology
I. Heterosis is the hybrid over either of its
(d) Gene therapy
parents in one or more traits.
II. It results in increased yield and growth. 7. A single strand of nucleic acid tagged with a
III. It is achieved by inbreeding. radioactive molecule is called
IV. It is observed in only self-pollinated species. (a) plasmid (b) vector
Choose the correct option. (c) probe (d) selectable marker
(a) I and III (b) II and IV 8. C-peptide in human insulin is
(c) I and II (d) III and IV (a) a part of mature insulin molecule
45. You have hired a driver who is 55 years old. (b) responsible for the formation of disulphide bridges
During the interaction before joining, he had (c) removed during the maturation of pro-insulin to insulin
informed you very honestly that he is a diabetic (d) responsible for the biological activity of insulin
patient. Which of the following suggestions you 9. Which amongst the following is involved in the
will give him before going on a drive? process of genetic engineering?
I. Take a double dose of insulin, so that he does (a) Codon (b) Anticodon
not develop any problem during driving. (c) Explant (d) Plasmid
II. Always carry an identification card with full
details.
10. Restriction enzymes are used to cut
(a) single-stranded RNA (b) double-stranded DNA
III. Take regular snacks or meals during long
(c) single-stranded DNA (d) double-stranded RNA
journey.
IV. Drink only 500 mL of water in 24 hours to avoid 11. The first discovered restriction enzyme was
any hypoglycemic attack. (a) Eco RI (b) Hind II (c) Bam HI (d) Sma I
Choose the correct option. 12. cDNA in genetic engineering is
(a) I and III (b) III and IV (a) formed by reverse transcriptase
(c) I and II (d) II and III (b) cloned DNA
(c) circular DNA
9. Biotechnology : Principles, (d) recombinant DNA

Processes and Its Applications 13. The genetically engineered protein which is used to
treat the lung disease emphysema is
1. Genetically engineered human insulin, humulin (a) a-1 antitrypsin
was launched by American drug company on (b) a-glucosidase
(a) 5th July, 1998 (b) 5th July, 1993 (c) hGH (human Growth Hormone)
(c) 5th July, 1973 (d) 5th July, 1983 (d) factor VIII

MODULE 1
www.jeebooks.in
84 NEET Test Drive

14. More advancement in genetic engineering is due to 24. The enzymes that are generally used in the
the discovery of continuous process for bioreactors are
(a) restriction endonucleases (b) reverse transcriptases (a) mobilised (b) immobilised
(c) proteases (d) zymase (c) digestive (d) activation

15. An enzyme catalysing the removal of nucleotides 25. In the process of separation of DNA on gel
from the ends of DNA is electrophoresis, where would you find the smallest
(a) endonuclease (b) exonuclease segment of DNA?
(c) DNA ligase (d) Hind II (a) Near the positive electrode, farthest away from the wells
(b) Near the negative electrode, close to the wells
16. Biolistic gun (gene gun) is suitable for (c) Near the top, near the negative pole
(a) disarming of pathogen vectors (d) Near the middle, they tend to slow down, after the first few
(b) transformation of plant cell minutes
(c) construction of vectors
(d) DNA fingerprinting
26. RFLP (Restriction Fragment Length
Polymorphism) is
17. Patent granted to the medical centre, University of (a) the difference in the restriction maps between the two
Missisippi (USA) for the use of haldi powder as a alleles in a diploid cell
wound healing agent was challenged by which of (b) the difference in the restriction maps between two
the following? individuals of two species
(a) CCMB, Hyderabad (b) IARI, Pusa, New Delhi (c) the difference in the restriction maps between two
(c) CSIR, New Delhi (d) NGOs in India individuals of same species
(d) the technique used to fingerprint patterns of
18. PCR and restriction fragment length inheritance
polymorphism are methods for the
(a) study of enzymes (b) genetic transformation
27. Out of the following, which is the genetically
engineered drug used for treating infertility?
(c) DNA sequencing (d) early diagnosis of diseases
(a) Interleukin (b) Polymyxin-B
19. Recently RNA interference method has been used (c) Chorionic gonadotropin (d) Calcitonin
in the development of new varieties of tobacco
plants which are resistant to 28. Type I restriction enzymes cut
(a) viruses (b) bollworms (a) both strands of DNA (b) only one strand of DNA
(c) nematodes (d) bacterial blights (c) no strand of DNA (d) None of these

20. Stirred tank bioreactors (most common type of 29. Flavr Savr variety of tomato is the result of
aerobic bioreactor) have been designed for the (a) RNAi technique (b) somatic hybridisation
(a) availability of oxygen throughout the process (c) inbreeding (d) None of these
(b) ensuring anaerobic conditions in the culture vessel 30. Which of the following cannot be determined by
(c) purification of the product using the Northern blotting technique?
(d) addition of preservatives to the product (a) A specific mRNA in a sample
21. Construction of recombinant DNA involves (b) Recombinants by detecting the mRNA produced by the
(a) cleaving and rejoining DNA segments with endonuclease transgene
alone (c) A particular disease
(b) cleaving DNA segments with endonuclease and rejoining (d) A specific protein
them with ligase 31. In 1990, the procedure of humulin production was
(c) cleaving DNA segments with ligase and rejoining them A instead of E. coli.
with endonuclease improved by using ......
(d) cleaving and rejoining DNA segments with ligase alone Choose the correct option for A.
22. Human serum albumin can be produced by (a) Bacillus subtilis (b) Saccharomyces cerevisiae
(c) Pseudomonas sp. (d) Candida albicans
(a) transgenic tomato (b) gene gun
(c) Bacillus thuringiensis (d) transgenic potato 32. Which of the following vectors will be used to
23. Golden rice was created by transforming rice with transfer and express a gene from sunflower to
beans?
(a) only two beta-carotene biosynthesis genes
(a) pBR322 plasmid
(b) only three beta-carotene biosynthesis genes
(b) maize streak virus
(c) only one beta-carotene biosynthesis gene
(c) Ti plasmid
(d) all the beta-carotene biosynthesis genes
(d) Lambda phage

MODULE 1
www.jeebooks.in
Prep Analysis for NEET ~ Biology 85

33. The enzyme obtained from bacterium Thermus 41. Genetic markers are used to identify host cells that
aquaticus is employed for have successfully taken up a cloning vector. A
(a) DNA amplification in PCR common marker used in RDT is
(b) production of antibiotics (a) Ti-plasmid (b) R-plasmid
(c) isolation of enzymes (c) F-plasmid (d) Col-plasmid
(d) modification of biosynthetic pathway 42. What does ‘transfection’ refer to?
34. Which cells are required to obtain hybridoma cells (a) Introduction of foreign DNA into a eukaryotic cell
by which the monoclonal antibodies are produced? (b) Synthesis of protein based on a mRNA sequence
(a) Lymphoma cells and bone marrow cells (c) The process by which a normal cell becomes malignant
(b) T-lymphocytes and myeloma cells (d) Synthesis of mRNA from a DNA template
(c) B-lymphocytes and myeloma cells
43. Bacterial defence system involves fragmenting the
(d) B-lymphocytes and carcinoma cells viral DNA upon entry with the help of
35. Shotgun approach is used for the construction of (a) exonucleases (b) endonucleases
(a) genomic library (c) methylase (d) ligase
(b) cDNA library 44. Wine and beer are produced directly by
(c) gene bank fermentation. Brandy and whisky require both
(d) All of the above fermentation and distillation because
36. GMFs have proven their importance as (a) fermentation is inhibited at an alcohol level of 10-18%
(a) they cause less damage to environment (b) distillation prolongs storage
(b) they allow the increase of crop quantity (c) distillation improves quality
(c) they improve profitability (d) distillation purifies the beverages
(d) they improve convenience for customers 45. Plasmids are used in the process of making
37. Recombinant DNA technology can be used to multiple identical copies of any template DNA
produce medically useful protein. Which of the because they are
following can be produced by using this (a) small circular DNA molecules with their own replication
technology? origin site
(a) Somatostatin (b) Insulin (b) carrying antibiotic resistant sites
(c) Interferon (d) All of these (c) circular DNA with the capability to integrate with the host
DNA
38. Genetically Modified Organisms (GMOs) are (d) transcribed from a gene
developed by rDNA technology and genetic
engineering. They are useful in
(a) enhancing the nutritional value of food
10. Ecology
(b) reducing the post-harvest losses 1. Animals and plants with ambient water osmotic
(c) increasing the tolerance power of crops against the abiotic concentration are known as
stresses (a) endotherms (b) ectotherms
(d) All of the above (c) osmoconformers (d) diapause

39. Which of the following is not correct for the enzyme 2. The phytophagous insects are an example of
restriction endonucleases? (a) predation (b) competition
(a) It is present in both eukaryotes and prokaryotes (c) parasitism (d) mutualism
(b) Each restriction enzyme only recognises the same 3. An example of species diversity is
palindromic sequence, no matter what the source of DNA
(a) 1000 verieties of mango
is
(b) Western Ghats have a greater amphibian diversity than
(c) Restriction enzymes work in the presence of Mg + 2
Eastern Ghats
(d) Type II restriction endonucleases do not require ATP for (c) India with its deserts, rainforests, etc has a greater diversity
restriction activities than a Scandinavian country
40. In agarose gel electrophoresis, (d) total richness of species in all the habitats
(a) the larger molecules of DNA migrate faster than the smaller 4. Which of the following is not a control measure of
molecules air pollution?
(b) supercoiled plasmids migrate slower than their nicked (a) Combustion technique
counterparts (b) Scrubbers
(c) ethidium bromide can be used to visualise the DNA
(c) Effluent treatment plant
(d) DNA migrates towards the negative electrode
(d) Gravity setting chamber

MODULE 1
www.jeebooks.in
86 NEET Test Drive

5. According to Allen’s Rule, mammals from colder represented by a pyramid. Who occupies the tip of
climates have an ecological pyramid?
(a) Shorter ears (b) Shorter limbs (a) Herbivores (b) Carnivores
(c) longer ears (d) Both (a) and (b) (c) Producers (d) None of these
6. What is the mass of living matter at a trophic level 15. Which among the following is not the characteristic
in an area at specific time? of hotspots?
(a) Humus (b) Standing crop (a) Extremely rich in species diversity
(c) Standing state (d) Detritus (b) Have high endemism
(c) Under constant threat
7. There are many reasons as to why we should
(d) Cause pollution
conserve biodiversity. One is the broadly utilitarion
aspect. An example of it is 16. Which one of the following is the most efficient
(a) industrial products, products of medicinal device to eliminate particulate matter from the
importance industrial emissions?
(b) forest through photosynthesis produce 20% of the total (a) Cyclonic separators (b) Trajectory separators
oxygen in Earth's atmosphere
(c) Electrostatic precipitator (d) Scrubber
(c) a moral duty to core about every existing form of life
(d) pass on our biological legacy 17. In triangular age pyramid,
(a) number of pre-reproductive individuals is very large
8. Ozone depletion of stratosphere will result in lot of
(b) number of reproductive individuals is moderate
side effects to atmosphere. One of them is that it
(c) post-reproductive individuals are fewer
(a) causes forest fires
(d) All of the above
(b) increases incidence of skin cancer
(c) causes blood cancer 18. Which among the following statements is/are
(d) None of the above correct regarding climax community?
(a) It is a final biotic community that develops in an area
9. Which one of the following terms is used for local (b) Diversity is more
population, adapted genetically to its particular
(c) It is stable
environment?
(d) All of the above
(a) Biotic community (b) Demes
(c) Biome (d) Ecotype 19. Alluring type mimicry is shown by
(a) praying mantis (b) grasshopper
10. The factors affecting rate of decomposition are (c) spiders (d) None of these
(a) chemical composition of detritus
(b) availability of oxygen for processing 20. Which one of the following statements is incorrect
(c) warm and moist environment with reference to altruism?
(d) All of the above (a) It is a sacrificial behaviour (b) It is seen in spotted deer
(c) Both (a) and (b) (d) None of these
11. Identify the incorrect combination from the given
options. 21. Internal nutrient cycling refers to
(a) Periyar — Elephant (a) nutrients uptake by plants by the process of
(b) Sunderban — Tiger absorption
(c) Dachigam National Park — Snow leopard (b) nutrients are released from the detritus through the
(d) Rann of Kutch — Wild ass activity of decomposers
(c) Both (a) and (b)
12. Which of the following is incorrect about (d) None of the above
eutrophication?
(a) Found in all ecosystems 22. The release of inorganic substances from organic
(b) Leads to organic loading matter during the process of decomposition is
(c) Bloom formation in eutrophic water known as
(d) Enrichment with plant nutrients (a) humification (b) mineralisation
13. Which of the following has no effect on the second (c) leaching (d) detritus
organism? 23. Which of the following national parks is first to be
(a) Parasitism (b) Amensalism established in India for conserving wildlife?
(c) Predation (d) Commensalism (a) Gir National Park, Gujarat
14. Ecosystem has different trophic levels which are (b) Periyar National Park, Kerala
interrelated to each other. This relation can be (c) Jim Corbett National Park, Uttarakhand
(d) Kaziranga National Park, Asom

MODULE 1
www.jeebooks.in
Prep Analysis for NEET ~ Biology 87

24. The main aim of Earth Summit of Rio de Janeiro 30. Consider the following statements and choose the
(1992) was correct option given below.
(a) compilation of Red List I. Nepenthes khasiana is a green plant but
(b) establishment of biosphere reserves shows heterotrophic nature for nitrogen
(c) conservation of biodiversity supply, therefore, is called as carnivorous.
(d) conservation of tiger and stags II. Frankia is symbiont in root nodules of several
25. Which among the following is not an example of legume plants like Pisum sativum.
mutualism? (a) Statement I is correct, II is incorrect
(a) Lichen (b) Statement I is incorrect, II is correct
(b) Ruminant and bacteria (c) Both the statements are correct
(c) Mycorrhiza (d) Both the statements are incorrect
(d) Barnacles and whale
31. Which among the following statements is/are
26. Which among the following statements is/are correct regarding food web?
correct regarding ecosystem services? (a) Members of higher trophic level can feed on a number of
(a) These are the products of ecosystem processes alternative organisms of the lower trophic level
(b) Ecosystem services maintain biodiversity (b) Members of higher trophic level feed upon a single type of
(c) Ecosystem services are the transformation of a set organisms of lower trophic level
physical environment (c) Members of higher trophic level become unable to get
(d) All of the above food from the organisms of lower trophic level.
(d) None of the above
27. Arrange the correct order of biotic succession in a
pond/lake. 32. Which of the following is correct?
I. Woodland stage II. Plankton stage I. Critically endangered—Sus salvinus
III. Climax forest IV. Floating stage II. Endangered — Long tailed macaque
V. Marsh meadow stage III. Rare species —Great Indian bustard

Choose the correct option. Choose the correct option.


(a) I ® II ® III ® IV ® V (b) II ® IV ® V ® I ® III (a) I and II (b) III and II
(c) I and III (d) All of these
(c) I ® V ® IV ® II ® III (d) III ® I ® II ® V ® IV
28. Identify the incorrect statement. 33. Which one among the following statements is
incorrect regarding greenhouse gases?
(a) Species richness refers to the number of species per unit
(a) Carbon dioxide is increasing due to fossil fuel burning
area
(b) Major source of N2O is nylon industries and nitrogen rich
(b) Species evenness refers to relative abundance of species
fertilisers
in an area
(c) Major source of methane is propellants in aerosol
(c) Species diversity decreases if the species richness is
higher (d) CFCs present atmospheric concentration is 282 ppt.
(d) Number of individuals and variety determine the level of 34. Label the given graph.
diversity of an ecosystem
Size/weight of the organ

29. Consider the following statements. C


I. Green muffler scheme involves the growing
of green plants along roadside to reduce air B
pollution.
II. Delhi became the first city of the world to use
A
CNG for its public transport system and
autorickshaws by the end of 2002.
III. In India, the Air (Prevention and Control of
Time
Pollution) Act came into force in 1981, but
A B C
was amended in 1987 to include noise as an
air pollutant. (a) Lag phase Exponential phase Stationary phase
(b) Exponential phase Lag phase Stationary phase
Which of the statements given above are correct?
(a) I and II (b) I and III (c) Stationary phase Lag phase Exponential phase
(c) II and III (d) I, II and III (d) Lag phase Stationary phase Exponential phase

MODULE 1
www.jeebooks.in
88 NEET Test Drive

35. Identify the markings of the ecological pyramid Codes


given below, from following options, if A B C D A B C D
I. Herbivores II. Secondary predators (a) 1 2 3 4 (b) 4 1 2 3
(c) 4 1 3 2 (d) 1 2 4 3
III. Decay detritivores IV. Primary predators
V. Autotrophs 39. Study the following chain.
Sun
ß
C
Gives 1000 units energy to phytoplanktons
E ß
Phytoplanktons are consumed by herbivorous
D zooplanktons and in turn received A unit of energy
ß
B Carnivorous zooplanktons eat the herbivorous
zooplanktons and get B unit of energy
ß
Carnivorous fishes take carnivorous zooplanktons and
A receive C unit of energy
ß
A B C D E A B C D E In last, Tunas find their turn and engulf carnivore fishes
(a) II I III IV V (b) IV I III V II to get D unit of energy
(c) III V II I IV (d) V III II I IV Identify A, B, C and D. If
36. Which among the following statements is/are I. 0.1 unit II. 10 unit
correct regarding seral community? III. 100 unit IV. 1 unit
I. Takes lesser time for completion A B C D A B C D
II. Unstable (a) II III IV I (b) IV I III I
III. Stable (c) III II IV I (d) None of these
IV. Takes longer time for completion 40. The adaptations in animal parasites include the
(a) development of adhesive organs
Choose the correct option.
(b) rise in rate of multiplication
(a) I and II (b) III and IV (c) Only I (d) Only III
(c) presence of intermediate hosts
37. Realised mortality differs from specific mortality in (d) All of the above
I. actual rate of death occurring in a unit
population due to different environmental
41. The functions of predation include
(a) maintenance of species diversity
changes.
(b) keeping prey population in control
II. minimum death rate that occurs under ideal
(c) transfer of energy at trophic levels
conditions.
(d) All of the above
III. for being constant for a population.
IV. maximum mortality occurs in egg, larva, 42. Which among the following statements is/are
seedling and in old age. correct regarding J-shaped growth curve?
(a) Population seldom reaches an equilibrium phase with the
Choose the correct option. carrying capacity of the habitat
(a) I and IV (b) I and II (c) II, and IV (d) III and IV (b) An inflection in phase of diminishing growth is not reached
38. Match the following columns and choose the (c) Minor fluctuations are not observed
correct option from the codes given below. (d) All of the above

Column I Column II 43. Which among the following statements is/are


correct regarding the net primary productivity?
A. Ecads 1. individuals are genetically different
(a) It is the amount of organic matter stored by producers per
B. Ecotypes 2. one or more ecotype in a single unit time and area
coenospecies
(b) It is equal to organic matter synthesised by
C. Ecospecies 3. transition zone between two different
photosynthesis minus the rate of respiration and other loss
communities
(c) Both (a) and (b)
D. Ecotone 4. individuals differ in vegetative (d) None of the above
characters

MODULE 1
www.jeebooks.in
Prep Analysis for NEET ~ Biology 89

44. Match the following columns and choose the correct 45. Match the following columns and choose the correct
option from the codes given below. option from the codes given below.
Column I Column II Column I Column II
A. Rann of Kutch 1. Tiger A. Environment (Protection) Act 1. 1968
B. Corbett park 2. Elephant B. Insecticides Act 2. 1974
C. Kaziranga 3. Rhinoceros C. Water Act 3. 1986
D. Periyar 4. Wild ass D. Air Act 4. 1987

Codes Codes
A B C D A B C D A B C D A B C D
(a) 4 1 3 2 (b) 2 1 3 4 (a) 2 3 5 1 (b) 1 4 2 3
(c) 1 4 2 3 (d) 3 1 4 2 (c) 4 3 2 1 (d) 3 1 2 4

Answer Sheet
1. Diversity in Living World
1. (d) 2. (b) 3. (c) 4. (d) 5. (a) 6. (a) 7. (d) 8. (a) 9. (d) 10. (c)
11. (d) 12. (c) 13. (c) 14. (d) 15. (a) 16. (b) 17. (c) 18. (a) 19. (c) 20. (b)
21. (c) 22. (a) 23. (d) 24. (b) 25. (a) 26. (d) 27. (a) 28. (c) 29. (a) 30. (c)
31. (a) 32. (c) 33. (b) 34. (c) 35. (d) 36. (b) 37. (b) 38. (b) 39. (d) 40. (b)
41. (b) 42. (c) 43. (d) 44. (a) 45. (b)

2. Structural Organisation in Plants and Animals


1. (a) 2. (c) 3. (d) 4. (d) 5. (d) 6. (c) 7. (b) 8. (b) 9. (d) 10. (d)
11. (c) 12. (b) 13. (c) 14. (d) 15. (c) 16. (b) 17. (c) 18. (b) 19. (a) 20. (a)
21. (c) 22. (b) 23. (c) 24. (a) 25. (c) 26. (c) 27. (d) 28. (d) 29. (d) 30. (b)
31. (d) 32. (a) 33. (a) 34. (d) 35. (d) 36. (d) 37. (b) 38. (a) 39. (d) 40. (c)
41. (b) 42. (d) 43. (c) 44. (a) 45. (b)

3. Cell Structure and Functions


1. (a) 2. (c) 3. (b) 4. (d) 5. (d) 6. (a) 7. (c) 8. (d) 9. (a) 10. (a)
11. (b) 12. (c) 13. (b) 14. (c) 15. (b) 16. (a) 17. (c) 18. (b) 19. (c) 20. (b)
21. (a) 22. (c) 23. (d) 24. (b) 25. (c) 26. (b) 27. (a) 28. (d) 29. (c) 30. (c)
31. (b) 32. (c) 33. (d) 34. (b) 35. (b) 36. (c) 37. (d) 38. (c) 39. (d) 40. (c)
41. (c) 42. (b) 43. (b) 44. (a) 45. (c)

4. Plant Physiology
1. (a) 2. (d) 3. (d) 4. (c) 5. (d) 6. (d) 7. (b) 8. (d) 9. (d) 10. (d)
11. (c) 12. (d) 13. (c) 14. (d) 15. (d) 16. (b) 17. (c) 18. (a) 19. (b) 20. (b)
21. (a) 22. (c) 23. (a) 24. (b) 25. (d) 26. (d) 27. (a) 28. (c) 29. (c) 30. (d)
31. (c) 32. (b) 33. (a) 34. (c) 35. (c) 36. (d) 37. (d) 38. (d) 39. (d) 40. (d)
41. (a) 42. (d) 43. (a) 44. (d) 45. (d)

5. Human Physiology
1. (b) 2. (c) 3. (d) 4. (b) 5. (d) 6. (d) 7. (c) 8. (b) 9. (a) 10. (c)
11. (d) 12. (a) 13. (d) 14. (c) 15. (b) 16. (b) 17. (a) 18. (b) 19. (c) 20. (a)
21. (a) 22. (a) 23. (b) 24. (a) 25. (a) 26. (d) 27. (c) 28. (c) 29. (b) 30. (c)
31. (c) 32. (c) 33. (c) 34. (c) 35. (b) 36. (d) 37. (a) 38. (c) 39. (c) 40. (b)
41. (b) 42. (b) 43. (b) 44. (b) 45. (d)

MODULE 1
www.jeebooks.in
90 NEET Test Drive

6. Reproduction in Plants and Animals


1. (d) 2. (b) 3. (c) 4. (c) 5. (a) 6. (b) 7. (b) 8. (c) 9. (b) 10. (a)
11. (c) 12. (c) 13. (b) 14. (b) 15. (d) 16. (c) 17. (d) 18. (c) 19. (c) 20. (a)
21. (d) 22. (b) 23. (d) 24. (b) 25. (c) 26. (d) 27. (b) 28. (a) 29. (b) 30. (b)
31. (b) 32. (a) 33. (b) 34. (b) 35. (b) 36. (a) 37. (c) 38. (b) 39. (b) 40. (b)
41. (b) 42. (c) 43. (c) 44. (b) 45. (b)

7. Genetics and Evolution


1. (b) 2. (d) 3. (b) 4. (b) 5. (a) 6. (c) 7. (b) 8. (d) 9. (a) 10. (d)
11. (c) 12. (b) 13. (a) 14. (c) 15. (c) 16. (c) 17. (a) 18. (b) 19. (b) 20. (c)
21. (a) 22. (a) 23. (c) 24. (d) 25. (d) 26. (c) 27. (d) 28. (b) 29. (c) 30. (b)
31. (b) 32. (c) 33. (c) 34. (d) 35. (c) 36. (a) 37. (d) 38. (d) 39. (d) 40. (c)
41. (d) 42. (c) 43. (d) 44. (c) 45. (c)

8. Biology in Human Welfare


1. (d) 2. (c) 3. (a) 4. (a) 5. (a) 6. (b) 7. (d) 8. (d) 9. (d) 10. (a)
11. (b) 12. (a) 13. (d) 14. (d) 15. (b) 16. (b) 17. (d) 18. (b) 19. (d) 20. (a)
21. (d) 22. (b) 23. (a) 24. (c) 25. (d) 26. (b) 27. (b) 28. (a) 29. (d) 30. (a)
31. (d) 32. (d) 33. (d) 34. (b) 35. (a) 36. (d) 37. (c) 38. (d) 39. (d) 40. (c)
41. (b) 42. (b) 43. (d) 44. (c) 45. (d)

9. Biotechnology : Principles, Processes and Its Applications


1. (d) 2. (b) 3. (c) 4. (c) 5. (b) 6. (b) 7. (c) 8. (c) 9. (d) 10. (b)
11. (b) 12. (a) 13. (a) 14. (a) 15. (b) 16. (b) 17. (c) 18. (d) 19. (c) 20. (a)
21. (b) 22. (d) 23. (a) 24. (b) 25. (a) 26. (c) 27. (c) 28. (b) 29. (a) 30. (d)
31. (b) 32. (c) 33. (a) 34. (c) 35. (a) 36. (c) 37. (d) 38. (d) 39. (a) 40. (c)
41. (b) 42. (a) 43. (b) 44. (a) 45. (a)

10. Ecology
1. (c) 2. (a) 3. (b) 4. (c) 5. (d) 6. (b) 7. (b) 8. (b) 9. (d) 10. (d)
11. (c) 12. (a) 13. (d) 14. (b) 15. (d) 16. (c) 17. (d) 18. (d) 19. (c) 20. (d)
21. (c) 22. (b) 23. (c) 24. (c) 25. (d) 26. (d) 27. (b) 28. (c) 29. (c) 30. (a)
31. (a) 32. (d) 33. (c) 34. (a) 35. (c) 36. (a) 37. (a) 38. (b) 39. (c) 40. (d)
41. (d) 42. (d) 43. (c) 44. (a) 45. (d)

For Explanation to all the Unit Tests of Biology, open the link
https://goo.gl/hEf2PL

MODULE 1
www.jeebooks.in

www.jeebooks.in

MODULE2
Prep Catalysis
This section contains 12 Mock Tests exactly based on
Pattern and Level of NEET to help you to be acquainted
with pattern and level of NEET and to complete the same in
a prescribed time limit of 3 hrs. You are advised to complete each
mock test in a single shot with the estimation given with each test,
you can also predict your Rank.
www.jeebooks.in

PREP CATALYSIS
for NEET
Full Length Mock Tests for NEET to Make You Ready to Face the Challenge

MOCK TEST 1 (With Solutions)

720

Instructions
The test is of 3 hours duration and Test Booklet contains 180 questions. Each question carries 4 marks. For
each correct response, the candidate will get 4 marks. For each incorrect response, one mark will be deducted
from the total scores. The maximum marks are 720.
The question paper contains three parts of Physics, Chemistry and Biology respectively.
Part A contains 45 Questions from Physics section. Part B contains 45 Questions from Chemistry section.
Part C contains 90 Questions from Biology.

PART A PHYSICS
1. A capacitor of capacitance 5 µF is connected as shown 4. Output B is given by
in the figure. The internal resistance of the cell is X
0.5 Ω. The amount of charge on the capacitor plates
Y A
is B
1Ω 1Ω
Z
5 µF (a) X + Y + Z (b) XY + Z
2Ω
(c) X + YZ (d) X + XYZ

2.5 V 5. A particle moving with uniform acceleration has


average velocities v1 , v2 and v3 over the successive
intervals of 5s, 10 s and 15 s respectively. Then, the
(a) 80 µC (b) 40 µC (c) 20 µC (d) 10 µC v − v2
value of 1 will be
2. For a material the Young’s modulus is 3.2 times v2 − v3
that of rigidity modulus. Its Poisson’s ratio is (a)
5
(b) 1 (c)
3
(d)
1
(a) 0.6 (b) 0.3 (c) 0.2 (d) 0.5 3 5 2
3. In an L - C - R circuit inductance is changed from L 6. The density of a solid ball is to be determined in an
to L / 2. To keep the same resonance frequency, C experiment. The diameter of the ball is measured
should be changed to with a screw gauge, whose pitch is 0.5 mm and
C C there are 50 divisions on the circular scale.
(a) 2C (b) (c) 4C (d)
2 4

MODULE 2
www.jeebooks.in
94 NEET Test Drive

The reading on the main scale is 2.5 mm and that 12. A heat engine placed on neptune (planet) operating
on the circular scale is 20 divisions. If the between temperatures T1 and T2, where T1 > T2 has
measured mass of the ball has a relative error of 1
efficiency . When T2 is lowered by 60K its
2%, the relative percentage error in the density is 5
(a) 0.9% (b) 2.4% (c) 3.1% (d) 4.2% 1
efficiency increase to . Then, T1 and T2 are
7. An ideal solenoid having 5000 turns/m has an 2
aluminium core and carries a current of 5 A. If respectively
χ Al = 2.3 × 10−5 , then the magnetic field developed (a) 200 K and 180 K (b) 200 K and 160 K
(c) 252 K and 215 K (d) None of these
at centre will be
(a) 0.031 T (b) 0.048 T (c) 0.027 T (d) 0.050 T 13. In the given circuit, the current through the
resistor 2k Ω is
8. Find current i in circuit shown in figure.
1 kΩ
B

5Ω 10Ω +
20 V 12 V 2 kΩ

A D
10Ω

i
10Ω 20Ω (a) 2 mA (b) 4 mA (c) 6 mA (d) 1 mA

C 14. The longitudinal waves starting from a ship return


from the bottom of the sea to the ship after 2.60 s.
If the bulk modulus of water be 220 kg mm −2 and
5V . × 103 kgm −3 , the depth of sea is
the density is 11
(a) 0.5 A (b) 0.2 A (c) 1 A (d) 2 A (take, g = 9.8 N kg −1)
9. A mass M is broken into two parts of masses m1 (a) 1820 m (b) 1850 m
(c) 1400 m (d) 1420 m
and m2. For the force of attraction or gravitation to
be maximum between them, the m1 and m2 have 15. The maximum vertical distance through which a
value full dressed astronaut can jump on the earth is
M 2M 2M M 0.5 m. The maximum vertical distance through
(a) m1 = , m2 = (b) m1 = , m2 =
3 3 3 3 which he can jump on the moon, which has a mean
M M 3M density (2/3) rd that of earth and radius one
(c) m1 = m2 = (d) m1 = , m2 =
2 4 4 quarter that of earth is
(a) 1.5 m (b) 3 m (c) 6 m (d) 7.5 m
10. Kepler space telescope generated a plot of intensity
of radiation versus wavelength of three stars in 16. Two slits are separated by a distance of 0.5 mm
Andromeda galaxy. What is the relation between
their temperature? and illuminated with light of λ = 6000 A° . If the
TC
screen is placed 2.5m from the slits, the distance of
I
the third bright fringe from the centre will be
TB
TA (a) 1.5 mm (b) 3 mm
(c) 6 mm (d) 9 mm

17. If λ 1 and λ 2 are the wavelengths of the first


members of the Lyman and Paschen series
respectively, then λ 1 : λ 2 is
(a) 1 : 3 (b) 1 : 30 (c) 7 : 50 (d) 7 : 108
λ
18. The wavelength λ of a photon and the de-Broglie
(a) TA > TB > TC (b) TC > TB > TA
wavelength of an electron have the same value.
(c) TA > TC > TB (d) TB > TC > TA Find the ratio of energy of photon to the kinetic
11. The amplitude of a damped oscillator decreases to energy of electron in terms of mass m, speed of
0.9 times its original magnitude in 5 s. In another light c and Planck constant h.
10 s it will decrease to α times its original λmc hmc
(a) (b)
magnitude, then α is equal to h λ
2hmc 2λmc
(a) 0.7 (b) 0.81 (c) 0.729 (d) 0.6 (c) (d)
λ h

MODULE 2
www.jeebooks.in
Prep Catalysis for NEET ~ Mock Test 1 95

19. Potential energy of a particle in a force field is 24. The mass of jupiter is 1.9 × 1027 kg and that of sun
B A is 1.99 × 1030 kg. The mean distance of the jupiter
given by U = 2 − where, A and B are positive
x x from the sun is 7.8 × 1011 m. Assume that the
constant and x is distance of particle from centre of
jupiter moves in a circular orbit around the sun.
the field. For stable equilibrium, the distance of
Velocity of jupiter is
particle is
A B 2A B . × 104 ms −1
(a) 13 . × 104 ms −1
(b) 15
(a) (b) (c) (d)
B A B 2A (c) 1.2 × 104 ms −1 . × 104 ms −1
(d) 16

20. What will be the time constant for the given 25. In a match a sixer is hitted by a batsman and the
circuit? ball touches the ground outside the boundary.
R R Then, which of the following graph describes the
variation of the cricket ball’s vertical velocity v
C with time between the time t as it hits the bat and
time t′, when it touches the ground?
v v

(a) t′ (b) t′
2R R t t
A B t t
(a) RC (b) 2.5 RC
(c) 1.2 RC (d) 6 RC v v

21. The output Y of given logic circuit is (c) t′


t (d) t
t t t′
A
B
Y
C 26. The excess pressure inside a spherical drop of
D water is five times that of another drop, then their
respective mass ratio is
(a) AB + CD (b) ABCD (a) 5 : 3 (b) 25 : 137
(c) ABC + D (d) A ⋅ B + CD (c) 125 : 137 (d) 1 : 125
22. Match the corresponding entries of Column I with 27. A thin symmetrical double convex lens of
Column II and choose the correct option from the refractive index µ 2 = 1.5 is placed between a
codes given below medium of refractive index µ 1 = 1.4 to the left and
Column I Column II another medium of refractive index µ 3 = 1 . 6 to the
right. Then, the system behaves as
A. Astigmatism 1. Convex lens (a) a convex lens (b) a concave lens
B. Hypermetropia 2. Concave lens (c) a glass plate (d) a convex or concave lens
C. Myopia 3. Cylindrical lens
28. The half-life period of a radioactive element X is
Codes same as the mean life of another radioactive
element Y . Initially, both of them have the same
A B C A B C
numbers of atoms, then
(a) 3 2 1 (b) 3 1 2 (a) X and Y have the same decay rate initially
(c) 1 2 3 (d) 2 1 3 (b) X and Y decay at the same rate always
23. The effective resistance of n number of resistor (c) Y will decay at a faster rate than X
when connected in parallel is P Ω. If one of the (d) X will decay at a faster rate than Y
resistor is removed, the effective resistance 29. At room temperature ( 27° C ) a vehicle tyre has
becomes Q Ω. Then, the resistance of resistor
pressure of 3 atm. The temperature at which the
which is removed is 7
PQ PQ tyre suddenly, burst will be (given, γ air = )
(a) (b) 5
P+Q Q−P
(a) 246K (b) 274K
P 2Q 2 PQ (c) 217K (d) 204K
(c) (d)
(Q − P)2 P−Q

MODULE 2
www.jeebooks.in
96 NEET Test Drive

30. Which one of the following graph represents the 36. Two thin long parallel wires separated by a
variation of maximum kinetic energy EK of the distance a are carrying a current I A each. The
emitted electrons with frequency ν in photoelectric magnitude of the force per unit length will be
effect correctly ? µ 0I µ 0I µ 0 I2 µ 0I
(a) (b) (c) (d)
EK EK 4 πa 4 πa 2
2 πa 2 πa
37. A billiard ball of mass m and radius r hit by a cue
(a) (b) stick at a height h above the centre and it acquire
a linear velocity v. The angular velocity ω acquired
n n by the ball is
5vr 2 5vh 2 vr 2 2 vh
(a) (b) (c) (d)
EK EK 2h 2r 2
5h 5r 2
38. A current I flows in a conducting wire of length L.
(c) (d) If we bent it in a circular form, its magnetic dipole
moment would be
n n lL2 lL l2L l 2 L2
(a) (b) (c) (d)
4π 4π 4π 4π
31. In Meldi’s experiment a string vibrates in 3 loops, 39. Minimum excitation potential of Bohr’s first orbit
when 8 g were placed in the pan. To make the in hydrogen atom is
string vibrates in 5 loops, the mass should be (a) 3.6 V (b) 10.2 V (c) 13.6 V (d) 3.4 V
placed is of value
(a) 2 g (b) 2.88 g
40. When a positive charge particle is moving on a
(c) 2.5 g (d) 4 g
equipotential surface, then work done in this
process will be
32. A uniform solid right circular cone and a (a) positive (b) negative (c) zero (d) None of these
hemisphere of same base radius 2m and same
density are placed on each other, so as to have 41. Two plane mirror of length 24 m each are kept at a
common face. The centre of mass of the compositive distance of 8 cm parallel to each other. A ray of
solid lies on the common face. The height of the light incident on one end of one mirror at angle of
cone is 53°. Then, total number of reflections before it
4
reaches the other end is (given, tan 53° = )
h 3
r (a) 225 (b) 175 (c) 250 (d) 200
42. A particle executes SHM of amplitude 25 cm and
time period 3s. The minimum time required for the
(a) 2m (b) 6m (c) 4m (d) 8 m
particle to move between two points 12.5 cm on
either side of the mean position is
33. The resistance of the wire in platinum resistance (a) 0.25 s (b) 0.5 s (c) 1 s (d) 1.25 s
thermometer at ice point is 3 Ω and at steam point
is 9 Ω. When the thermometer is inserted in an 43. A voltmeter has a resistance of G Ohm and range
unknown hot bath its resistance is found to be of V volt. The value of resistance used in series to
12 Ω. The temperature of the hot bath is convert it into a voltmeter of range nV volt is
G G
(a) 100° C (b) 50° C (c) 75° C (d) 150° C (a) nG (b) (c) (n − 1)G (d)
(n − 1) n
34. One mole of monoatomic gas and three moles of
diatomic gas are put together in a container. The
44. A source emits electromagnetic waves of
wavelength 3m. One beam reaches the observer
molar specific heat (in JK −1 mol −1) at constant
directly and other after reflection from a water
volume is (R = 8.3 JK −1 mol −1)
surface, travelling 1.5 m extra distance with
(a) 18.7 (b) 18.9 (c) 19.2 (d) 20.7
intensity reduced to 1/4 as compared to intensity
35. A body weighs 72 kgf on the surface of earth. Then, due to the direct beam alone. The resultant
1 intensity will be
its weight on surface of mars, whose mass is and
9 (a) (1/4) fold (b) (3/4) fold (c) (5/4) fold (d) (9/4) fold
1
radius is of that of earth is 45. If 8 dipoles of charges of magnitude ± e are placed
2 inside the cube. The electric flux coming out of
(here, kgf = kilogram-force) cube will be
(a) 72 kgf (b) 54 kgf (c) 62 kgf (d) 32 kgf (a) 16 e / E0 (b) 8 e / E0 (c) e / E0 (d) zero

MODULE 2
www.jeebooks.in

PART B CHEMISTRY
46. The sulphide ore is converted to oxide before 53. Which of the following reagent is used to convert?
reduction because O CH2
(a) oxides are easier to reduce
(b) sulphide are easily decompose to
(c) oxide get easily decompose
(d) sulphide get oxidised to sulphur (a) Zn-Hg in HCl (b) PH3P == CH2
(c) Both (a) and (b) (d) None of these
47. The separation of primary, secondary and tertiary
amines can be done through fractional distillation. 54. Product obtained on reaction of 2-methylcyclohexene
Which of the following method will not used to with m-CPBA followed by hydrolysis is
distinguish primary, secondary and tertiary
OH OH
amine?
(a) Hinsberg method (a) OH (b) CH3
(b) Hofmann method CH3 OH
(c) Liebermann’s nitroso method
OH
(d) Victor Meyer’s method
(c) (d) CH3
48. The amino acid which is optically inactive is
CH3 OH
(a) alanine (b) lactic acid
(c) glycine (d) serine
55. One mole of the complex CoCl3 × 6H 2O on reaction
49. The substance which is added to soap to impart with excess of AgNO3 gives two moles of white
antiseptic properties is precipitate. Thus, complex is
(a) iodoform (b) terpineol (a) [Co(H2O)6 ]Cl 3 (b) [Co(H2O)5 Cl] Cl 2 × H2O
(c) chloroxylenol (d) bithional (c) [Co(H2O)4 Cl 2 ]Cl ×2H2O (d) [Co(H2O)3 Cl 3 ]× 3H2O
50. Strength of acid increases with the attachment of 56. The ligand N(CH 2CH 2NH 2 )3 is
group showing -I effect and decreases with the (a) tridentate (b) didentate
attachment of group showing + I effect. Which of (c) tetradentate (d) pentadentate
the following is correct sequence of basic strength
in aqueous solution? 57. The transition elements vary widely in their
(a) CH3NH2 < (CH3 )2 NH < (CH3 )3 N chemical behaviour. Many of them are sufficiently
(b) (CH3 )2 NH < CH3NH2 < (CH3 )3 N electropositive to dissolve in mineral acids, although
(c) (CH3 )3 N < (CH3 )2 NH < CH3NH2
a few are noble. Among the first series transition
metals, except copper, all the metals are relatively
(d) (CH3 )3 N < CH3NH2 < (CH3 )2 NH
reactive. Transition elements show variable valency
O due to
½½ (a) small size
51. Compound Ph ¾ O ¾ C ¾ Ph can be prepared by (b) large charge
the reaction of (c) presence of incompletely filled d-subshell
(a) phenol and benzaldehyde in the presence of palladium (d) All of the above
(b) phenol and benzoic acid in the presence of NaOH
(c) phenol and benzoyl chloride in the presence of pyridine 58. The oxidation state of Ni in tetracarbonyl nickel is
(d) phenol and benzoyl chloride in presence of ZnCl 2 (a) + 1 (b) + 2 (c) 0 (d) + 4

52. Arrange the following in correct sequence of basic 59. Chromatography is based on the principle of
strength. (a) vaporisation (b) concentration
(c) absorption (d) adsorption
O
60. If the electron is visualised as a classical charged
N N N N particle laterally rotating about an axis with
angular momentum L and magnetic dipole moment
H H H
M then find the magnetic moment of K3 [FeF6 ] ?
I II III IV
(a) 3.87 BM (b) 4.89 BM
(a) IV > I > III > II (b) III > I > II > IV (c) 5.91 BM (d) 6.92 BM
(c) II > I > III > IV (d) I > III > II > IV

MODULE 2
www.jeebooks.in
98 NEET Test Drive

61. Which of the following inert gas element, shows 70. Match the block of elements given in Column I
the highest chemical reactivity? with their electronic configuration given in
(a) Xe (b) He Column II and choose the correct code.
(c) Ne (d) Ar
Column I Column II
62. The reduction of C == O to CH 2 is done by A. s-block 1. [Xe] 4 f7 5d 16 s2
Wolf-Kishner reduction. The reagent used in
B. p-block 2. [Kr] 4 d 10 5 s1
Wolf-Kishner reduction is
(a) NH2 ¾NH2 in presence of base C. d-block 3. [Xe] 5 s1
(b) Zn ¾Hg in dil. HCl
D. f-block 4. [Xe] 4f14 5 d 10 6 s2 6 p2
(c) PH3P == CH2
(d) None of the above Codes
63. The mixture of concentrated HCl and HNO3 made A B C D A B C D
(a) 1 2 3 4 (b) 3 4 2 1
in 3 : 1 ratio contains (c) 2 3 1 4 (d) 3 4 1 2
(a) N2O 4 (b) NOCl
(c) NCl 3 (d) ClO 2 71. The shortest wavelength transition in the Paschen
series in hydrogen atom occurs at 821 nm. At what
64. Which of the following is/are use(s) of KMnO 4 ? wavelength does it occur in Li 2+ (isoelectronic to
(a) In analytical chemistry hydrogen)?
(b) As a favourite oxidant in preparative organic chemistry (a) 91.2 nm (b) 273.6 nm
(c) In bleaching of wool, cotton and silk (c) 821.0 nm (d) 7389.0 nm
(d) All of the above
72. Which of the following is the property of an
65. Which of the following statement is correct about isothermal expansion process?
salt bridge? (a) DU = 0 (b) W = - Q
V
(a) Forms insoluble precipitate with the electrolytes in the two (c) W = - nRT ln 2 (d) All of these
half-cells V1
(b) Enhances the flow of electrons by overcoming liquid 73. Which of following statement is correct?
junction potential (a) Glucose has furanose structure and fructose has pyranose
(c) Maintains the electrical neutrality of the two half-cells structure
(d) Provides the extra ions (b) Glucose has pyranose structure and fructose has furanose
structure
66. Which of the following is not correct for the
(c) Both have furanose structure
reaction?
2N 2O5 ¾® 4NO2 + O2 (d) Both have pyranose structure

(a) Rate of reaction is -d[N2O 5 ] 74. A container has hydrogen and oxygen mixture in
(b) Rate of reaction is proportional to [N2O 5 ]2 ratio of 1 : 1 by weight, then
(c) It is first order reaction (a) internal energy of the mixture decreases
(d) It is redox reaction (b) internal energy of the mixture increases
(c) entropy of the mixture increases
67. The products formed on the photochemical (d) entropy of the mixture decreases
decomposition of HNO 3 is
(a) N2O 5 and NO 2
(c) NO 2 and O 2
(b) N2O 4 and NO 2
(d) NO and O 2
75. In the reaction, N 2 O 4 c
2NO 2 , a is that part of
N 2 O 4 which dissociates, then the number of moles
at equilibrium will be
68. Which of the following expression is correct for
(a) (1 - a)2 (b) 3a
average kinetic energy of one molecule of an ideal
gas at 27°C and 1 atm pressure? (c) a (d) 1 + a
(a) 900 cal K -1 molecule -1 76. The compound A on heating gives a colourless gas
(b) 6.21 ´ 10-21 JK -1 molecule -1 and a residue that is dissolved in water to obtain
(c) 336.7 JK -1 molecule -1 B. Excess of CO 2 is bubbled through aqueous
solution of B, C is formed which recovered in the
(d) 3741.3 JK -1 molecule -1
solid form. Solid C on gentle heating gives back A.
69. Which of the following pair is isoelectronic? The compound is
(a) CO 2 and NO 2 (b) BF3 and NH3 (a) CaSO 4 × 2H2O (b) CaCO 3
(c) NO -3 and CCl 4 (d) NO + and SbH5 (c) Na 2CO 3 (d) K 2CO 3

MODULE 2
www.jeebooks.in
Prep Catalysis NEET ~ Mock Test 1 99

77. In Carius method, 0.099 g of organic compound 82. According to kinetic theory, the reactant molecules
gave 0.287 g of AgCl. The percentage of chlorine in are assumed to be hard spheres and reaction is
the compound will be postulated to occur when molecules collide with
(a) 71.71% (b) 29.39% each other. Pre-exponential factor in the
(c) 53.06% (d) 76.23% expression obtained during collision theory is
known as
78. Choose the correct option which will fill the A, B (a) collision diameter (b) collision frequency factor
and C correctly. (c) collision energy (d) None of these
4B + 3CO 2 ¾® A + C
83. The number of nearest neighbours and next
4B + 3SiO 2 ¾® B + Si nearest neighbours of Na + ion in a crystal of NaCl
4B + 6NO ¾® C + 3N 2 are, respectively,
Å -
(a) B2O 3 , BO 2 and BO (b) B2O 3 , B2O 2 and BO (a) 6Na , 12Cl - (b) 6Cl , 12Na Å
(c) B2O 3 , B2O 3 and B2O 3 (d) BO, BO 2 and B2O 3 (c) 12 Cl - , 12Na Å (d) 6Cl - , 6Na Å

79. Which of the following would react most readily 84. Each pair forms ideal solution except
with nucleophiles? (a) C 2H5Br and C 2H5I (b) C 6H5Cl and C 6H5Br
Cl Cl (c) C 6H6 and C 6H5CH3 (d) C 2H6I and C 2H5OH

85. A certain current liberates 0.500 g of H 2 in 2.00 hr.


(a) (b) How many gram of oxygen can be liberated by the
OCH3
same current in the same time?
(a) 0.500 g (b) 8.00 g
Cl (c) 4.00 g (d) 16.00 g
Cl Cl
86. Addition of 0.643 g of a compound to 50 mL of
CH3 benzene (density = 0.879 g mL -1), lowers the
(c) (d) freezing point from 50.51°C to 50.03°C. If K f for
benzene is 5.12, the molecular mass (in g mol -1) of
the compound is
NO2 (a) 156.05 (b) 312.00
(c) 78.00 (d) 468.00
80. Match the following name of elements to stable end
product. 87. The oxygen atom in phenol
(a) exhibits only inductive effect
Name of elements Stable end products (b) exhibits only resonance effect
A. Thorium 1. 208
Pb (c) has more dominating resonance effect than inductive
82
effect
209
B. Neptunium 2. 83 Bi (d) has more dominating inductive effect than the resonance
206 effect
C. Uranium 3. 82 Pb
207 88. At 500 K, the half-life period of a gaseous reaction
D. Actinium 4. Pb
82 at an initial pressure of 100 kPa is 364 s. When the
Codes pressure is 50 kPa, the half-life period is 182 s.
A B C D A B C D
The order of the reaction is
(a) 2 3 4 1 (b) 3 4 1 2 (a) zero (b) one (c) half (d) two
(c) 1 2 3 4 (d) 4 1 3 2 89. The effective atomic number of cobalt in
81. Sewage containing organic waste should not be [Co(NH 3 )5 H 2 O]3 + is
disposed in water bodies because it causes major (a) 36 (b) 33 (c) 24 (d) 30
water pollution. Fishes in such a polluted water die
90. Which of the following represents the correct order
because of
of solubility for the noble gases?
(a) large number of mosquitoes
(a) Ne > Ar > Kr > He > Xe
(b) increase in amount of dissolved oxygen
(b) He > Ne > Ar > Kr > Xe
(c) decrease in amount of dissolved oxygen in water
(c) Ar > Ne > He > Kr > Xe
(d) clogging of gills by mud
(d) Xe > Kr > Ar > Ne > He

MODULE 2
www.jeebooks.in

PART C BIOLOGY
91. The virus of hepatitis-A contains 96. Which of the following statements is/are incorrect?
(a) dsDNA (b) ssRNA I. The fast block polyspermy develops in response
(c) ssDNA (d) dsRNA to the opening of sodium gates in the plasma
92. Which of the following cranial nerves is incorrectly membrane.
matched with its nature? II. The slow block polyspermy develops in
(a) Olfactory – Sensory response to the release of bindin.
(b) Trochlear – Motor III. Capacitation is the process of entry of more
(c) Glossopharyngeal – Mixed than one sperm into the oocyte.
(d) Hypoglossal – Sensory Choose the correct option.
(a) Only III (b) I and II
93. Match the Column I with Column II and choose
the correct option from the codes given below. (c) Only I (d) I and III

Column I Column II 97. Consider the following statements about the


structure of ovule and select the one which is
A. Phycophages 1. Fungal viruses incorrect.
B. Mycophages 2. Carry enzyme reverse transcriptase (a) The point of attachment of funicle with the body is called
hilum
C. Retroviruses 3. Benign and malignant
(b) A collar-shaped outgrowth from the base of the ovule
D. Tumour viruses 4. Viruses which are parasitic on algae which forms the third integument is called aril
(c) In a polygonum type of embryo sac, the egg apparatus is
Codes
situated towards the chalazal end
A B C D A B C D
(d) Generally, both the polar nuclei fuse before fertilisation to
(a) 1 2 3 4 (b) 4 3 2 1
form a single diploid nucleus
(c) 4 1 2 3 (d) 1 3 2 4
98. Which of the following statements is true?
94. K-T boundary refers to
(a) Auxin translocation is non-polar
(a) mass extinction of 60 million years ago when dinosaurs
(b) Abscisic acid is transported through phloem only
disappeared
(c) Phototropism is induced by blue light
(b) deposits of iridium which are rare on earth
(d) Auxin is not affected by TIBA
(c) Both (a) and (b)
(d) None of these 99. Which among the following is/are adaptations of
hydrophytes?
95. Refer to the diagram given below and choose the
(a) No cuticle over the epidermis
option that indicates incorrect labelling.
(b) Aerenchyma is well-developed
(c) Secondary growth is absent
(d) All of the above
A
100. The enzyme that catalyses the conversion of
plasminogen to plasmin and is used in acute
B coronary arterial thrombosis is
(a) streptokinase (b) phosphotransferase
C (c) β-lactamase (d) penicillinase

101. The production of fertile soil on bare rocks is


(a) secondary succession (b) primary succession
(c) heterotrophic succession (d) allogenic succession
D
102. The Tumour inducing (Ti) plasmid of which plant
(a) A–Represents the storage tissue rich in starch pathogen has been modified into a cloning vector to
(b) B–Represents scutellum deliver genes of our interest?
(c) C–Represents coleorhiza (a) Providencia stuartii (b) Agrobacterium tumefaciens
(d) D–Represents the protective covering of radicle (c) Haemophilus haemolyticus (d) Bacillus globigii

MODULE 2
www.jeebooks.in
Prep Catalysis for NEET ~ Mock Test 1 101

103. Which of the following disorders occurs due to the 114. Which among the following statements is/are
deficiency in immune system? incorrect regarding significance of biopatents?
(a) Parkinson’s disease (b) Digeorges’ syndrome (a) Permit private, monopoly rights over cells, genes, animals
(c) Huntingtons’s chorea (d) Alzheimer’s disease and plants
(b) People would not research in such areas, which are
104. In which of the following the percentage of dominated by patents
lipoprotein is maximum? (c) Lead to research programmes dominated by potentiability
(a) Chylomicron (b) VLDL and profitability
(c) DHL (d) LDL (d) Philosophy and social commentary that deal with the
105. Which among the following statements is/are biological sciences and their potential impact on society
incorrect regarding Spirogyra? 115. Bordered pits are found on the radial walls of
(a) Fertilisation takes place in the female gametangium (a) xylem tracheids (b) sieve tubes
(b) Conjugation tube is not formed (c) xylem fibres (d) sieve plates
(c) Both (a) and (b)
(d) None of the above 116. In bacteria, glucose is utilised first, even if other
sources of sugar are available. This happens
106. Which of the following is the intermediate through a mechanism known as
compound linking glycolysis to Krebs cycle? (a) catabolite repression (b) enzyme repression
(a) Oxaloacetate (b) Acetyl Co-A (c) operon repression (d) positive feedback mechanisms
(c) Pyruvate (d) Citric acid
117. The life history traits of organisms have evolved in
107. Catabolite Activator Protein (CAP) is /an relation to
(a) inducer (b) corepressor (a) Darwinian fitness
(c) aporepressor (d) apoinducer (b) organisms evolve under selection pressure developed by
environmental factors
108. What will happen if corpora allata is removed from
(c) organisms achieve most efficient reproductive strategy
the body of cockroach?
(d) All of the above
(a) Digestion will not occur (b) It will become sterile
(c) It will become adult (d) It will become blind 118. In mitotic prophase, the sister chromatids are held
together by a multi-subunit protein complex called
109. Identify the incorrectly matched pair.
(a) actin (b) cytokinin (c) cohesin (d) tubulin
(a) Physical xerophytes – Soils which are physically dry
(b) Physiological xerophytes – Soils having sufficient water 119. Viruses are known as wandering genes because
(c) Both (a) and (b) (a) nucleic acid is the only active part of a virus
(d) None of the above (b) capsid is the outer protective coat
(c) nucleic acid is the central core
110. trp operon is regulated by the availability of amino (d) capsid is made up of specific protein
acid tryptophan, this mechanism is known as
(a) termination (b) attenuation 120. A person performing ‘Yoga’ breathes in as much air
(c) suppression (d) repression as possible. The volume of air inspired is called
(a) tidal volume (b) inspiratory capacity
111. In ‘protoplast fusion’, tissues of two different plants (c) vital capacity (d) inspiratory reserve volume
are cultured. Their cells are separated and
subjected to the treatment with 121. In which of the following tissues the benign
(a) IBA and NAA tumour is enclosed?
(b) ethylene and glycol (a) Connective tissue (b) Epithelial tissue
(c) coconut milk and auxin (c) Muscular tissue (d) Nervous tissue
(d) pectinase and cellulase
122. From which amino acid, creatine is metabolically
112. Which are the first organs to develop in a human formed?
embryo? (a) Histidine (b) Phenylalanine
(a) Heart and blood vessels (b) Eyes and ears (c) Tryptophan (d) Arginine
(c) Brain and spinal cord (d) Mouth and tongue
123. cAMP regulates the lac operon by
113. What would be the number of chromosomes in (a) binding to the lac repressor
aleurone cells of a plant when number of (b) binding to the operator
chromosomes in leaf cell is 24? (c) combining with CAP to form a complex which starts
(a) 12 (b) 24 transcription by binding near the promoter
(c) 36 (d) 48 (d) All of the above

MODULE 2
www.jeebooks.in
102 NEET Test Drive

124. Match the Column I with Column II and choose 130. Castle’s intrinsic factor is a glycoprotein secreted
the correct option from the codes given below. by parietal cells is required for the absorption of
(a) cobalamine (b) biotin (c) pyridoxine (d) niacin
Column I Column II
A. CO2 1. Freshwater wetlands 131. Match the Column I with Column II and choose
the correct option from the codes given below.
B. CH4 2. Non-toxic gas
C. CFC 3. Burning of fossil fuel Column I Column II
(Type of bryophytes) (Features)
D. N2O 4. Industrial processes
A. Hepaticopsida 1. Capsule without any
Codes columella
A B C D A B C D B. Anthocerotopsida 2. Sterile columella present
(a) 1 2 3 4 (b) 3 1 2 4
C. Bryopsida 3. Sporophyte
(c) 4 2 3 1 (d) 1 3 2 1
well-differentiated into
125. Match the Column I with Column II and choose foot, seta and capsule
the correct option from the codes given below. D. Marchantiales 4. Riccia
Column I Column II
Codes
A. Intrafascicular 1. Cambium between xylem and A B C D A B C D
cambium phloem (a) 1 2 4 3 (b) 1 2 3 4
B. Interfascicular 2. Parenchymatous cells of (c) 2 1 3 4 (d) 4 1 3 2
cambium medullary rays
132. In golden rice crtl (carotene desaturase) gene has
C. Tyloses 3. Ballon-like structures been inserted from the
D. Heartwood 4. Physiologically inactive (a) Narcissus pseudonarcissus (b) Erwinia uredovora
(c) Pseudomonas putida (d) E. coli
Codes
A B C D A B C D 133. Match the following columns and choose the
(a) 1 3 2 4 (b) 3 1 2 4 correct option from the codes given below.
(c) 1 2 3 4 (d) 4 1 2 3
Column I Column II
126. Which of the following is not a secondary sex organ (Hormone) (Function)
in human males? A. Human chorionic 1. Promotes growth of
(a) Seminal vesicle (b) Penis gonadotropin myometrium
(c) Epididymis (d) Testis
B. Progesterone 2. Maintains corpus luteum
127. With respect to eukaryotic gene regulation, which C. Oestrogen 3. Suppresses uterine
of the following statements is not true for contractions
enhancers? D. Relaxin 4. Softens cervix to prepare
(a) They function in tissue specific manner for dilation
(b) They function in any orientation
(c) They function as promoters Codes
(d) They function even when are at a distance from the gene A B C D A B C D
(a) 2 3 1 4 (b) 3 2 4 1
128. Thyroxine is produced from which of the following (c) 4 1 3 2 (d) 1 4 2 3
amino acids? 134. The individuals produced as a consequence of the
(a) Tryptophan (b) Alanine (c) Glycine (d) Tyrosine undersecretion of pituitary gland are called
129. Identify the incorrect statement(s). (a) midgets (b) fidgets (c) gadgets (d) maggots
I. Ability of an organism to maintain a state of 135. In which of the following, males have half the
dynamic constancy is called thermoregulation. number of chromosomes that females possess?
II. The maintenance of constant internal (a) Moth (b) Chicken (c) Honeybee (d) Pigeon
temperature by an organism is called
136. Which of the following statements is/are correct?
homeostasis.
(a) Bilirubin is transported in blood by being attached to
III. The oldest living plant on earth is the great plasma albumin
Indian banyan tree. (b) Bilirubin is insoluble in water
Choose the correct option. (c) Both (a) and (b) are correct
(a) Only I (b) I and II (c) Only III (d) All of these (d) None of the above

MODULE 2
www.jeebooks.in
Prep Catalysis for NEET ~ Mock Test 1 103

137. In which of the following is XX-XO sex chromosome (a) It is an action of time passing with calf
complement found? (b) Lysozymes present in the saliva of the mother protects the
(a) Chimpanzee (b) Birds (c) Honeybee (d) Cockroach calf from bacterial infection
(c) To remove insects sitting on the calf
138. The effect(s) of ozone depletion include(s) (d) Both (a) and (b)
(a) inhibition of photosynthesis in phytoplanktons
(b) damage in nucleic acids of living organisms 147. Craidukov’s phenomenon is
(c) increase in UV-radiations reaching the earth surface (a) the capacity of changing colour
(d) All of the above (b) the change in colour according to the wavelength of
incident light
139. In which of the following structures insulin does (c) also known as chromatic adaptation
not increase the glucose uptake? (d) All of the above
(a) Cardiac muscles and skeletal muscles
(b) Renal tubules and intestinal mucosa 148. Which of the following statements are not
(c) Adipose tissues and cardiac muscles disadvantages of hydroponics?
(d) Smooth muscles and skeletal muscles I. The cost of these experiments is high.
II. It requires skilled people.
140. Which of the following is/are required for full
III. It facilitates production of seasonal vegetables.
expression of the lac operon?
(a) Lactose (b) Allolactose
IV. It avoids soil borne pathogens.
(c) Lactose and cAMP (d) Allolactose and cAMP Choose the correct option.
(a) II and III (b) III and IV (c) I and IV (d) I, II and III
141. The human biochemical disorder Tay-Sachs
disease is an example of 149. Which of the following classes of Echinodermata
(a) incomplete dominance (b) codominance does not include free-moving echinoderms?
(c) epistasis (d) multiple alleles (a) Asteroidea (b) Holothuroidea
(c) Echinoidea (d) Crinoidea
142. Which among the following statements is/are
correct regarding water potential? 150. Identify the incorrect statement(s).
(a) It refers to the chemical potential of water in a system I. Amniotic fluid test is used for diagnosing
(b) Water potential of pure water is zero chromosomal abnormalities.
(c) Water potential increases when pure water is heated II. In ultrasound imaging, visible images are
(d) All of the above produced by the pattern of echos made by
143. In the figure given below, a diagrammatic view of organs.
human skull is given. Two of the bones are III. Chronic villi sampling is an invasive
wrongly labelled. They are technique used in prenatal diagnosis.
Frontal bone IV. Maternal blood sampling is an invasive
Sphenoid bone technique where mother’s blood is tested.
Parietal bone
Choose the correct option.
Occipital bone (a) Only I (b) Only IV (c) Only II (d) III and IV
151. Pick out the mismatched pair(s).
Temporal bone
I. Wind — Anemophily — Maize
II. Water — Hydrophily — Vallisneria
Hyoid bone
III. Insects — Entomophily — Poppy
(a) sphenoid and hyoid bone (b) frontal and parietal bone
IV. Snails — Myrmecophily — Lemna
(c) occipital and temporal bone (d) parietal and occipital bone
V. Ants — Malacophily — Chrysanthemum
144. Which of the following is a ‘double-headed snake’?
Choose the correct option.
(a) Python (b) Viper (c) Sand boa (d) Kraits
(a) Only IV (b) Only V
145. Which of the following is a pair of zoogeographical (c) IV and V (d) None of these
regions separated by high mountains?
(a) Oriental and Australian (b) Nearctic and Neotropical 152. Which among the following statements is incorrect
(c) Ethiopian and Nearctic (d) Palaearctic and Oriental
regarding viruses?
(a) Viruses possess no membranes of their own
146. A little boy was inquisitive about why a cow was (b) Viruses do not possess ribosomes to make proteins
licking its calf. What is the most probable (c) Viruses do not have any source of energy (ATP)
explanation given by his grandfather? (d) Viruses can reproduce by themselves
MODULE 2
www.jeebooks.in
104 NEET Test Drive

153. The primary treatment of effluents involves 163. MAB (Man And Biosphere) programme is
(a) removal of large particles (b) removal of small particles associated with
(c) removal of dissolved particles (d) Both (a) and (b) (a) studying impact of human interference on environment
(b) pollution in biotic and abiotic environments
154. On the basis of functions, the meristematic tissue (c) conservation strategies for present and future
is divided into
(d) All of the above
(a) protoderm, procambium, ground meristem
(b) apical, lateral, intercalary 164. It is important to make a newborn baby burp
(c) mass, plate, rib meristem immediately after feeding or else they may
(d) primary, secondary regurgitate. This is because of lesser developed
(a) cardiac sphincter (b) gastric cells
155. Identify the incorrectly matched pair. (c) pyloric sphincter (d) None of these
(a) Chlorophyceae–Chlorophyll-a and b, carotenoids
(b) Phaeophyceae–Chlorophyll-a and c, fucoxanthin 165. Identify the incorrectly matched pair.
(c) Rhodophyceae–Chlorophyll-a, phycocyanin, phycoerythrin, (a) Sclerophyllous–Stiff leaves
xanthophyll (b) Trichophyllous–Leaves covered with hairs
(d) None of the above (c) Malacophyllous–Soft and fleshy leaves
(d) None of the above
156. The MN blood group in humans is an example of
(a) partial dominance (b) codominance 166. Clones can be obtained by which of the following
(c) incomplete dominance (d) epistasis techniques?
(a) Vegetative propagation (b) Cultivation
157. Which of the following category of genes found in (c) Hybridisation (d) Sexual reproduction
normal cells have an antiproliferative function?
(a) Oncogenes (b) Proto-oncogenes 167. A health conscious lady takes a lot of germinated
(c) Retro genes (d) Tumour-suppressor genes lentils in breakfast. Which of the following
enzymatic reaction will be the first to occur after
158. In the glomerulus of nephron, the afferent
its intake in mouth?
arteriole is
(a) longer than efferent arteriole Substrate Enzyme Products formed
(b) of same diameter as efferent arteriole (a) Protein Pepsin Polypeptide
(c) wider than efferent arteriole
(b) Fats Lipase Diglycerides
(d) narrower than efferent arteriole
(c) Maltose Maltase Glucose + Glucose
159. Which among the following is/are method(s) of safe (d) Starch Amylase Glucose + Fructose
disposal of radioactive wastes?
(a) Radioactive waste is buried deep under ground surface 168. Which of the following organs of the cockroach
(b) Radioactive waste is reprocessed absorbs nitrogenous waste from haemolymph?
(c) Radioactive waste is buried in specially constructed (a) Mycetocytes (b) Urate cells
buildings (c) Utriculi majores (d) Anal cerci
(d) All of the above
169. Which among the following statements is/are true
160. The oldest known fossil cells were about the size of for carbon cycle?
modern prokaryotes. Which of the following (a) Atmospheric inputs of carbon through rainfall are higher
resembles them? (b) There is respiratory release of carbon into the atmosphere
(a) Red algae (b) Green algae (c) Amoeba (d) Bacteria (c) The gaseous exchange of carbon between organisms and
161. Which one among the following is not the environment is negligible
characteristic of virusoids? (d) Both (a) and (b)
(a) They are small circular RNA 170. Real time PCR utilises …… for detection and
(b) They are always associated with larger viral RNA estimation of DNA products.
(c) They may form part of the viral genome (a) methyl orange (b) SYBR green
(d) They are proteinaceous infectious particles (c) phenolphthalein (d) ethidium bromide
162. ‘Saheli’ is a new oral contraceptive pill for females 171. ‘Brown sugar’ is composed of
with (a) lysergic acid diethylamide
(a) low side effects and low contraceptive value (b) tetra-hydro cannibinols
(b) high side effects and high contraceptive value (c) diacetyl morphine hydrochloride
(c) low side effects and high contraceptive value (d) acetaldehyde
(d) high side effects and low contraceptive value

MODULE 2
www.jeebooks.in
Prep Catalysis for NEET ~ Mock Test 1 105

172. Match Column I with Column II and choose the 175. Refer to the following flowchart and select the
correct option from the codes given below. option which correctly indicates ‘a’ and ‘b’.
Column I Column II Anterior pituitary
A. Tropical rainforests 1. Sal a

B. Tropical deciduous forests 2. Acacia sp. Adrenal cortex


b
C. Desert 3. Dipterocarpus
Liver
D. Coastal biome 4. Mangroves
Codes Glucose Glycogen
A B C D A B C D (a) CRF and ACTH (b) ACTH and cortisol
(a) 3 1 2 4 (b) 1 2 3 4 (c) CRF and cAMP (d) STH and cortisol
(c) 2 1 3 4 (d) 4 1 3 2
176. In Drosophila, gynandromorphs occur due to the
173. Match the contents of Column I to the contents
(a) loss of autosomal chromosome
of Column II and choose the correct option from
(b) loss of X-chromosome
the codes given below.
(c) gain of Y-chromosome
Column I Column II (d) chemical induction
A. Barley 1. Atomita-2
177. Along with scales, scutes and bony plates are found
B. Peppermint 2. Aruna in which of the following?
C. Castor 3. Todd’s Mitcham (a) Torpedo (b) Salmon
D. Rice 4. Erectiferum (c) Gambusia (d) Hippocampus
5. Pusa Lerma 178. According to the ‘Blackman’s law of limiting factor’, at
Codes a particular time, photosynthesis can be limited by
A B C D A B C D (a) CO 2 concentration (b) light
(a) 1 2 3 4 (b) 4 3 2 1 (c) Both (a) and (b) (d) Either (a) or (b)
(c) 3 4 1 2 (d) 2 1 3 4
179. Identify the incorrectly matched pair.
174. Why the bronchioles and lungs in human beings do (a) Ebola → Filoviruses (b) Mumps → Paramyxovirus
not collapse even though they lack the (c) Rabies → Rhabdovirus (d) Yellow fever → Variola virus
cartilaginous rings?
(a) Surfactants secreted by the clara cells prevent the collapse 180. The arrangement of outer and central microtubules
(b) The intercostal muscles keep the pleura expanded in a cilium is called
(c) The elastic cartilage present on their surface prevents them (a) 9 + 2 pattern (b) 9 + 0 pattern
from collapsing (c) 8 + 2 pattern (d) 9 + 1 pattern
(d) None of the above

Answer Sheet
1. (d) 2. (a) 3. (a) 4. (a) 5. (c) 6. (c) 7. (a) 8. (a) 9. (c) 10. (c)
11. (c) 12. (b) 13. (c) 14. (a) 15. (b) 16. (d) 17. (d) 18. (d) 19. (c) 20. (c)
21. (b) 22. (b) 23. (b) 24. (a) 25. (a) 26. (d) 27. (c) 28. (c) 29. (c) 30. (d)
31. (b) 32. (c) 33. (d) 34. (a) 35. (d) 36. (c) 37. (b) 38. (a) 39. (b) 40. (c)
41. (a) 42. (b) 43. (c) 44. (d) 45. (d) 46. (a) 47. (d) 48. (c) 49. (d) 50. (d)
51. (c) 52. (d) 53. (c) 54. (b) 55. (b) 56. (c) 57. (c) 58. (c) 59. (d) 60. (c)
61. (a) 62. (a) 63. (b) 64. (d) 65. (c) 66. (c) 67. (c) 68. (b) 69. (d) 70. (b)
71. (a) 72. (d) 73. (b) 74. (c) 75. (d) 76. (b) 77. (a) 78. (c) 79. (c) 80. (c)
81. (c) 82. (b) 83. (b) 84. (d) 85. (c) 86. (a) 87. (c) 88. (a) 89. (a) 90. (d)
91. (b) 92. (d) 93. (c) 94. (c) 95. (c) 96. (d) 97. (c) 98. (c) 99. (d) 100. (a)

101. (b) 102. (b) 103. (b) 104. (a) 105. (b) 106. (b) 107. (d) 108. (c) 109. (d) 110. (b)
111. (d) 112. (c) 113. (c) 114. (d) 115. (a) 116. (a) 117. (d) 118. (c) 119. (a) 120. (b)
121. (a) 122. (d) 123. (c) 124. (b) 125. (c) 126. (d) 127. (c) 128. (d) 129. (c) 130. (a)
131. (b) 132. (b) 133. (a) 134. (a) 135. (c) 136. (c) 137. (d) 138. (d) 139. (b) 140. (d)
141. (a) 142. (d) 143. (c) 144. (c) 145. (d) 146. (b) 147. (d) 148. (b) 149. (d) 150. (b)
151. (c) 152. (d) 153. (d) 154. (a) 155. (d) 156. (b) 157. (d) 158. (c) 159. (d) 160. (d)
161. (d) 162. (c) 163. (d) 164. (a) 165. (d) 166. (a) 167. (c) 168. (b) 169. (d) 170. (b)
171. (c) 172. (a) 173. (b) 174. (a) 175. (b) 176. (b) 177. (d) 178. (d) 179. (d) 180. (a)

MODULE 2
www.jeebooks.in

EXPLANATIONS

PHYSICS
1. In steady state, there will be no current Mass M 9. Let m1 = m and m 2 = M - m, then
Density, r = =
in the capacitor branch. Net resistance Volume 4 æ D ö 3 Gm (M - m) G
of the first parallel branch and cell, pç ÷ F = = (Mm - m 2 )
3 è2ø r2 r2
R = 1 + 1 + 05
. = 2.5 W dF
percentage error in density For maximum force, =0
Current drawn from the cell, Dr dm
V 2.5 = ´ 100 G
i = = = 1A r Þ (M - 2m) = 0
R 2.5
DM 3 DD r2
Potential drop across two parallel = ´ 100 + ´ 100 M
M D Þ M = 2m or m =
branches, V = E - ir = 2.5 - 1 ´ 05. 0.01 2
= 2.5 - 05
. = 2.0 V = 2% + 3 ´ ´ 100 M
2.7 \ m1 = m 2 =
So, charge on the capacitor plates = 2% + 111
. % = 3.1 % 2
q = CV = 5 ´ 2 = 10mC 1
7. Given, n = 5000 turns/m, i = 5A 10. Wien’s law, l m µ and from the figure
T
2. As,Y = 2 h (1 + s) c Al = 2.3 ´ 10-5 , B = ? (l m ) A < (l m )C < (l m )B . Therefore,
3.2 h = 2 h (1 + s) Þ 1. 6 = 1+ s As, B = m 0 (H + I ) T A > TC > TB .
s = 0.6 B m ni 11. The amplitude of damped oscillator is
where, H = 0 = 0 = ni
1 m0 m0 b
3. n = A = A0e - kt , where k =
LC = 5000 ´ 5 2m
When L is changed to L / 2, C must be = 2.5 ´ 104 A/m At t = 5 s, 0.9 A0 = A0e -5 k
changed to 2C to keep n same. and I = cH = 2.3 ´ 10-5 ´ 2.5 ´ 104 Þ e -5 k = 0.9
4. As, A = X ×Y = X + Y = 0575
. A/m At t = 15 s, aA0 = A0e -15 k
B = m 0 (H + I ) Þ e -15 k = a
(QA × B = A + B )
= 4 p ´ 10-7 (2.5 ´ 104 + 0.575)T Þ (e -5 k ) 3 = a
B = X + Y × Z (NAND gate) = 0.031 T Þ a = (0.9) 3 = 0729
.
= X +Y + Z = X + Y + Z 8. The circuit given in the question, is a T2 1 T
balanced Wheatstone bridge, so no 12. We known, h1 = 1 - Þ = 1- 2
5. Let u be initial velocity and a be T1 5 T1
uniform acceleration current is flow in the branch BC and it
T2 4
O A B C behaves like an open circuit. Þ = …(i)
T1 5
So, circuit becomes as shown below
B T 2 - 60
u u+5 a u+15 a u+30 a h2 = 1 -
T1
Then average velocities are,
5Ω 10 Ω 1 T - 60
u + u + 5a 5 Þ = 1- 2
v1 = =u+ a
2 2 2 T1
A D
u + 5a + u + 15a 1 T 2 - 60
v2 = = u + 10a = …(ii)
2 2 T1
u + 15a + u + 30a 45 10 Ω 20 Ω
v3= =u+ a i From Eqs. (i) and (ii), we get
2 2 T1 = 200 K ,T 2 = 160 K
5 C
a - 10a 13. Voltage across 2 kW resistor = 12 V
v1 - v 2 -15a
\ = 2 =
v 2 - v 3 10a - 45 a -25a 5V Therefore current passing through 2 kW
2 12
Now, in the above circuit, resistor = = 6 ´ 10-3 A = 6 mA
15 3 2 ´ 103
= = R ABD = 5 + 10 = 15 W
25 5
14. Given, K = 220 kg mm -2
Pitch R ACD = 10 + 20 = 30W
6. Least count = = 220 ´ 106 kgm -2 .
Number of division Resistance. R ABD and R ACD in parallel,
so = 220 ´ 9.8 ´ 106 Nm -2
0.5 mm
= = 0.01mm 15 ´ 30 450 and r = 1.1 ´ 103 kgm -3 .
50 R net = = = 10W
Diameter of ball, 30 + 15 45 K 220 ´ 9.8 ´ 106
Velocity, v = =
D = MSR + CSR ´ LC E 5 r . ´ 103
11
i = = = 05
. A
= 2.5 + 20 ´ 0.01 = 2.7 mm R net 10
= 1400 ms -1

MODULE 2
www.jeebooks.in
Prep Catalysis NEET ~ Mock Test 1 107

vt d æA
- ö÷ = - 3 + 2 = 0
Depth of sea = B 2A B goes on decreasing with height unitll it
Þ ç
2 dx è x 2 xø x x become zero. After that the situation is
1400 ´ 2.60 2A reversed and the velocity goes on
= = 1820 m Þ x =
2 increasing untill the ball hits the ground.
B
This explanation can be interpreted by
15. The acceleration due to gravity on d 2U
4 For stable equilibrium, >0 graph (a), as the motion is in 2-D.
earth is ge = pGRr dx 2 2T
3 26. Pressure in spherical drop, p = .
d 2U 6A 2B R
That on moon, Þ = 4 - 3
dx 2 x x 2T 2T
g m = pG æç ö÷ æç r ö÷ = ge
4 R 2 1 =5´ (QGiven, p1 = 5 p 2 )
3 è 4 øè 3 ø 6 æ d 2U ö B4 R1 R2
ç 2÷ = >0
è dx ø x = 2 A 8A3 Þ R 2 = 5R1
As the potential energy remain same, B
i.e., mgehe = mg mhm m1 4 pR13d1 1
20. Time constant in R - C circuit is given and = =
g
hm = e he =
ge
he m2 4 pR 23d 2 125
by t = R eqC
gm 1 / 6ge
Equivalent resistance, t 27. For first refracting surface,
= 6 ´ 05
. m= 3 m 2R ´ 3R 6R For parallel beam of light,
= =
16. Distance of nth bright fringe from the 2R + 3R 5 m m m -1
- 1+ 2 = 2
centre, u v R
6R 6RC
nDl \ t = ´C = = 12
. RC 14
. 15
. . - 14
15 .
yn = 5 5 =- + =
d -¥ v R
3 ´ 6000 ´ 10-10 ´ 2.5 21. Y = AB + CD = AB × CD
y3 = v = 15R
-3 (By de-Morgan’s theorem)
. ´ 10
05 For second refracting surface,
= 9 ´ 10-3 m = 9 mm = ABCD m m m - m2
- + 3 = 3
17. For first line of Lyman series, n1 = 1 22. (A) Astigmatism-It can be corrected . R
15 v¢ R
with the use of cylindrical lens. 15
. 16
. . - 15
16 .
and n 2 = 2 Þ - + = Þ v¢ = ¥
(B) Hypermetropia-To overcome 15R v¢ R
= R æç 2 - 2 ö÷ = R æç1 - ö÷ =
1 1 1 1 3R
\ from this problem a convex lens is Hence, the combination behaves as a
l1 è1 2 ø è 4ø 4 used. glass plate.
For first line of Paschen series, (C) Myopia-It is also know by short 28. Half life of X = mean life of Y
n1 = 3 and n2 = 4 sightedness and can be
corrected with the use of Let l X and lY be decay constants of X
= R æç 2 - 2 ö÷
1 1 1
\ concave lens. andY , then condition (1) implies
l2 è3 4 ø
0.693 1 lx
23. When n resistors are connected in = Þ lY = Þ lY > l X
= R æç -
1 1 ö 7R lX lY
÷= parallel, then 0.693
è 9 16 ø 144
1 1 1 1 1 As rate of decay is proportional to decay
l1 7R 4 7 = + + + .......+ …(i)
= ´ = P R1 R 2 R3 Rn constant, hence decay of Y will be faster
l 2 144 3R 108 than that of X .
When last resistor is removed, then
18. The de-Broglie wavelength, 1 1 1 1 1 29. It is a adiabatic process so,
= + + + ....+ …(ii)
h h Q R1 R 2 R 3 Rn - 1 p (1 - g )T g = constant
l= Þv = …(i)
g -1 g
mv ml æ p2 ö æT ö
Subtracting Eq. (ii) from Eq. (i), then ç ÷ = ç 2÷
hc
Energy of photon, E p = 1 1 1 è p1 ø è T1 ø
l - =
P Q Rn g -1 1. 4
(since l is same) æ 1ö = æç 2 ö÷
T éQ 7 = 14
. ù
ç ÷
so, required ratio is, Q -P
=
1
Þ Rn =
PQ è3ø è 300 ø ëê 5 ûú
Ep hc / l PQ Rn Q -P
= =
2hc 0.4 [log 1 - log 3] = 14
. [logT 2 - log 300]
Ee 1
mv 2 l mv 2 24. As the gravitational force of attraction T 2 = 217 K
2 between sun and jupiter provide the
30. According to photoelectric equation,
Substituting value of v from Eq. (i), we necessary centripetal force to jupiter
to move in a circular orbit. E K = hn - hn 0 …(i)
get
Ep 2hc 2 lmc mv 2 GMm GM If the energy of photon hn is less than the
= = So, = Þ v = work function hn 0 of metallic surface, then
Ee æ h ö
2
h r r2 r
lm ç ÷ electrons will never be ejected from surface
è ml ø 6.67 ´ 10-11 ´ 199
. ´ 1030 regardless of intensity of incident light.
=
A B 7.8 ´ 1011 Also, from Eq. (i), when n = n 0 ,E K = 0
19. As, U = 2
-
x x 4 -1 \ Graph (d) represents variation ofE K with n.
= 13
. ´ 10 ms
dU 3 T1
For equilibrium, =0 25. At time t, when the bat hit the ball, it 31. In first case, n =
dx acquire maximum velocity, which 2L m

MODULE 2
www.jeebooks.in
108 NEET Test Drive

5 T2 m 0 I1I 2 d
In second case, n = 36. As, F = . tan53° =
2L m 2p r 8 ´ 10-2
m I × I m 0I 2
\
3 T1
=
5 T2 = 0. = d = 8 ´ 10-2 tan53°
2L m 2L m 2p a 2 pa 4
= 8 ´ ´ 10-2
T2 3 T 9 37. Impulse imparted to ball = Change in 3
Þ = or 2 = momentum = mv 0.32
T1 5 T1 25 =
As, angular momentum 3
9
Þ T2 =T1 Total number of reflections before it
25
9 h reaches the other end
= ´ 8 = 2.88 g v
24 24
25 ω = =
d 0.32 / 3
2
32. Mass of hemisphere = pr 3r 24 ´ 3
3 = = 225
= moment of momentum 0.32
1 2
Mass of cone = pr hr 2
3 Iw = mvh Þ mr 2 w = mvh 42. The displacement of particle from mean
5 position is y = A sin wt
The mass moments about the centre of 5vh
Þ w= 2p 2p
mass is zero. So, As, w= = rad s -1
2r 2 T 3
mH æç ö÷ = mc æç ö÷
r h
è2ø è4ø 38. Let a wire length L is bend in a circular 2p
Þ 12.5 = 25 sin t
2
form of radius r 3
2 3 r p r hr h
Þ pr r ´ = ´ 2pr = L 2p 12.5 1
3 2 3 4 Þ sin t = =
L 3 25 2
Þ h 2 = 4r 2 or h = 2r = 4 m Þ r = …(i)
2p 2p p
Þ sin t = sin
33. For resistance thermometers, 3 6
Rt - R 0 ° 1
t = ´ 100° C Þ t = = 0.25 s
R100 - R 0 ° 4
r L
9 Total time between two points of 12.5 m
t = ´ 100 = 150° C
6 is given by
R I 2t = 2 ´ 0.25 = 05
. s
34. As, CV =
r -1 43. I g =
V
R 3 The magnetic dipole moment of a
For monoatomic gas, CV = = R G
5 circular ring,
-1 2 nV
3 M = IA and R = -G
2 Ig
R 5 M = Ip r …(ii)
For diatomic gas, C ¢V = = R nV
7 On putting the the value of r from Eq. = - G = (n - 1) G
-1 2
5 (i), we get V /G
nCV + n ¢ C ¢V 2
L2 IL2 44. We know that a phase change of p
M = Ip æç
For mixture, C V¢ ¢ = L ö
n + n¢
Þ ÷ = Ip =
è 2p ø 4p 2
4p occurs, when the reflection takes place
3 5 at the boundary of denser medium. This
1´ R + 3 ´ R 39. From the relation,
2 2 is equivalent to a path difference of l / 2.
= 13.6eV
1+ 3 DE = 13.6 eV - Total phase difference = p - p = 0
9 9 n2
= R = ´ 8.3 Thus, the two waves superimpose in
4 4 13.6 eV
= 13.6 eV - = 10.2 eV phase.
= 18.7 JK -1 mol -1 (2) 2
Resultant amplitude = I + æI ö
ç ÷
35. The weight of body on earth is Therefore, excitation potential è4ø
GMm 10.2 3
mg = = 72 kgf = eV = 10.2 V = I
R2 e 2
Then on mars surface, the value of g 2
40. Work done on equipotential surface is
Resultant intensity = æç
3 ö
becomes zero because potential difference is I÷
è2 ø
GM ¢ GM / 9 4GM zero.
g¢ = = = 9 9
R ¢2 (R / 2) 2 9R 2 41. Let d be the distance travelled by ray in = I = fold
4 4
4 the direction of mirror, then
Þ g¢ = g 45. From Gauss theorem,
9 24 m
qin
The weight of body on mars is d fE =
4 4 E0
mg ¢ = m ´ g = mg 8 cm
9 9 +8e - 8e
53° 53° = =0
4 E0
= ´ 72 = 32 kgf
9

MODULE 2
www.jeebooks.in

CHEMISTRY
46. The sulphide ore is converted to oxide morpholine makes it less basic as This conversion is termed on
before reduction because oxides are compared to piperidine. Now, out of Wolf-Kishner reduction.
easier to reduce. In this way, ore pyridine and pyrolle, pyridine is much 63. The mixture of conc. HCl and HNO 3
conversion is suitable for reduction. stronger base as the lone pair of made in 3 : 1 ratio is called aqua-regia.
nitrogen atom in pyridine is not The reaction is given below :
47. Victor Meyer’s method is used to
involved in resonance. So, the order of
distinguish between primary, HNO 3 + 3HCl ¾® NOCl + Cl 2 + 2H2O
basic strength is : I > III > II > IV.
secondary and tertiary alcohols. By Conc. Conc.
identifying the colour produced, the O Clemmensen reduction CH2 64. KMnO 4 is used in analytical chemistry as
alcohols are identified. Rest all the Zn-Hg in dil. HCl
53. it is used in quantitative analysis for
other methods given are used to
detecting halides and oxalates. It is used
distinguish between primary, PPH3 CH2
Wittig reaction as an oxidising agent in the laboratory as
secondary and tertiary amines.
well as in industry. It is also used in
48. Glycine is only optically inactive amino 54. m-CPBA H2 O bleaching of wool, cotton and silk.
acid due to absence of chiral carbon O
65. Functions of salt-bridge Complete the
atom.
H 2-methylcyclohexene cell circuit, maintains electrical neutrality
½ OH
r
OH2
in the solution of two half-cells, prevent
H2N ¾ C ¾ COOH diffusion of the solutions from one
s
½ OH
H+
O half-cell to the other.
H CH3
Glycine 66. 2 N2O 5 ¾® 4NO 2 (g ) + O 2 (g )
m-CPBA causes formation of epoxide
49. Bithional, an antiseptic, is mixed to ring from less hindered side and ring Rate law for the above reaction is given
medicated soaps to impart antiseptic opening will also occur from less as :
properties. hindered side. Rate = k[N2O 5 ]
Cl OH OH Cl So, order of the reaction is 1.
55. CoCl 3× 6H2O + AgNO 3 ¾® 2 mol AgCl
1 mol
S 67. Photochemical decomposition of HNO 3
As two Cl atoms are outside the produces NO 2 and O 2 gases.
Cl Cl coordinate sphere. So, the coordination
hn 1
Bithionol number of Co = 6. Thus, 2HNO 3 ¾® H2O + 2NO 2 + O 2
[Co(H2O) 5 Cl] Cl 2 × H2O. 2
50. The basic character of the given
compounds follows the following trend 56. The ligand N(CH2 CH2NH2 ) 3 is 68. Average kinetic energy (per molecule)
in aqueous solution. tetradentate. The given ligand have 3
= KT
four donor atoms.They donate four pair 2
(CH3 ) 3 N < CH3NH2 < (CH3 )2 NH
of electrons to metal ion.
This is due to combined effects of steric, 3 8.314 JK -1
57. Due to presence of incomplete filled = ´ ´ 298
hindrance, solvation and +I-effect. 2 6.023 ´ 1023 mol -1
d-sub shell it shows variable valency.
O Hence, it can extend its valency. = 6.2 ´ 10-21 JK -1 molecule -1
OH C Cl 58. CO is a neutral ligand. 69. Species having same total number of
\ Oxidation state of Ni in [Ni(CO) 4 ] is 0. valence electrons are isoelectronic with
51. +
59. Chromatography is based on the each other. Total valence electrons in
Phenol Benzoylchloride principle of differential adsorption of NO + = 5 + 6 - 1 = 10
different components over an Total valence electrons in
–HCl adsorbent. In this method, weakly SbH5 = 5 + 5 = 10
N adsorbed component is eluted first and
the strongly adsorbed component is \ NO + and SbH5 are isoelectronic.
O
elucted afterwards. 70. (i) For s-block element, electronic
O C 60. Fe3 + = [ 18 Ar] 3d 5 4s 0 , n = 5 configuration is [Xe] 5s 1.
(ii) For p-block element, electronic
m = n (n + 2)
configuration is
m = 5 (5 + 2) = 35 = 5.91 BM
Phenyl benzoate [Xe] 4f 14 5d 10 6s 2 6p 2 .
61. Xe shows highest chemical reactivity (iii) For d-block element, electronic
This reaction is an example of because of its low ionisation energy. Xe
Schotten-Baumann reaction. forms most of the compounds with F configuration is [Kr] 4d 10 5s 1.
and O like XeF2 , XeF4 , XeF6 , XeO 3 etc. (iv) For f-block element, electronic
52. Piperidine and morpholine both are
62. CH3 configuration is [Xe] 4f 7 5d 1 6s 2 .
non-aromatic and N-atom is sp 3
O + NH2 — NH2 1
hybridised in them, so these are more CH3 – 71. l µ 2
–N2 OH Z
basic as compared to other two (which
are aromatic and have sp 2 hybridised CH3 l (Li 2 + ) 1 821
CH2 = Þ l (Li 2 + ) = = 912
. nm
N-atom). -I-effect of O-atom in CH3 l (H ) 9 9

MODULE 2
www.jeebooks.in
110 NEET Test Drive

72. An isothermal process is a change of a


system, in which the temperature remains 80. Series Name of Stable end 86. Weight of benzene = V ´ d
constant. For isothermal process series products = 50 ´ 0.879 g
208
DT = 0 4n Thorium 82 Pb = 43.95 g
Hence, DU = 0 = Q ± W 4n + 1 Neptunium 209 Weight of compound = 0.643 g
83 Bi
V2 DTf = 5051
. - 50.03 = 0.48
Q = -W = - nRT ln 4n + 2 Uranium 206
V1 82 Pb w
207 \ DTf = Kf ´ compound
73. Cyclic sugars that contain a 5-membered 4n + 3 Actinium 82 Pb M
ring are called furanose. e.g. Fructose. 1000
81. Dissolved oxygen is essential for =
Cyclic sugars that contain a 6-membered w benzene
ring are called pyranose. e.g. Glucose. aquatic life. Organic waste is
oxidised by microorganisms using 0.643 1000
0.48 = 5.12 ´ ´
74. Mixing of gases increases the entropy. dissolved oxygen. Hence, oxygen M 43.95
75. N2O 4 c 2NO 2 from water decreases which is \ M = 156.05 g mol -1
harmful for aquatic life.
Initial moles 1 0 87. The oxygen atom in phenol has more
Moles at equilibrium 1 - a 2a 82. According to collision theory, dominating resonance effect than
Total number of moles at equilibrium k = Z ABe -E a /RT inductive effect.
= 1 - a + 2a = 1 + a where, Z AB = collision frequency Increase in charge separation decreases
factor the stability of a resonating structure.
D
76. CaCO 3 (s ) ¾® CaO + CO 2 ­ 83. Na Å lies in octahedral voids formed +
OH
OH OH
A
CO2 ( excess) by Cl È . Here, Na + present at body
CaO + H2O ¾® Ca(OH) 2 ¾¾¾¾®
B
centre touches six Cl - ions. Each s
Ca(HCO 3 ) 2 Cl - ion in the lattice is present at the
C
D face centre of each cube.
Ca(HCO 3 ) 2 ¾® CaCO 3 + CO 2 + H2O I IV
a
C A
[Distance between Na + and Cl - = ]
77. Percentage of chlorine 2 + +
Distance between two nearest (one O—H O—H
35.5 mass of AgCl
= ´ ´ 100 at body centre and other at edge –
143.5 mass of the compound centres)
35.5 0.287
= ´ ´ 100 = 7171
. % Na Å = a / 2 –
143.5 0.099 III
\ The number of nearest neighbours II
78. As B is a powerful reducing agent of Na + ion
reduces CO 2 , NO and SiO 2 , B replaces Stability of resonating structures in
= 6 Cl - ions decreasing order will be : I > IV = II > III.
C, Si and N from CO 2 , SiO 2 and NO
The number of next nearest 88. p1 = 100 kPa, (t1/ 2 )1 = 364 s
respectively as
neighbours of Na + ions
4B + 3 CO 2 ¾® 2B 2O 3 + 3C p 2 = 50 kPa, (t1/ 2 ) 2 = 182 s
= 12 Na + ions
4B + SiO 2 ¾® 2B 2O 3 + 3Si
100 364
4B + 6 NO ¾® 2B 2O 3 + 3N2 84. C2H6I and C2H5OH pair will not form = =2
50 182
ideal solution. Here, C2H5OH will
79. Nucleophile always attack on electron p1 (t ) a
show H-bonding as well as polarity Q = 1/ 2 1 = 1
deficient site. Presence of electron P2 (t1/ 2 ) 2 a2
both.
withdrawing groups such as NO 2 , CHO
etc., decreases the electron density on 85. According to Faraday’s law, \ t1/ 2 µ a where, a = initial concentration
benzene nucleus, hence such groups w = z × it or presence of reactants and this is true
activate the ring towards nucleophilic for zero order reaction.
Given, the value of it is same for H2
attack. While presence of electron and O 2 . 89. EAN = (atomic number - oxidation state
releasing groups such as —R or —OR w H2 z + 2 ´ CN)
increases the electron density, thus Then, = H2
w O2 z O2 = 27 - 3 + 2 ´ 6
deactivates the benzene nucleus toward
nucleophilic attack. NO 2 group activates w H2 EH2 = 24 + 12 = 36
the ring more than —Cl towards =
w O2 E O2 90. Noble gases are slightly soluble in water
nucleophilic attack. and their solubility increases with
0500
. 2
Cl = increase in atomic number on moving
w O2 16 down the group. Thus, the correct order
16 ´ 05. of solubility is
Hence, reacts rapidly with w O2 =
2 Xe > Kr > Ar > Ne > He.
nucleophilic.
w O2 = 4.00 g
NO2

MODULE 2
www.jeebooks.in

BIOLOGY
91. The virus of hepatitis-A contains a single 102. Agrobacterium tumefaciens, a 109. Options (a) and (b) both are correctly
strand of RNA and has no envelope. pathogen of several dicot plants is able matched. The physical xerophytes are
92. Among the given options, option (d) is to deliver a piece of DNA, known as plants that grow in soils which are
incorrectly matched. Hypoglossal is a T-DNA, to transform normal plant cells physically dry due to the shortage of
motor nerve which controls the into a tumour cell and direct these water.
movements of the tongue. It originates tumour cells to produce the chemicals Physiological xerophytes are plants that
from the ventral side of the medulla required by the pathogen. The Ti grow in soils having sufficient water but it
oblongata. plasmid of Agrobacterium tumefaciens is not available to the plants due to high
has now been modified into a cloning salt concentration.
93. Phycophages are the viruses which are vector which is no more pathogenic to
parasitic on algae. 110. Attenuation is a mechanism of control of
the plants. It is able to deliver the
Mycophages are fungal viruses. some bacterial operons which results in
genes of our interest into a variety of
Retroviruses carry enzyme reverse premature termination of transcription
plants.
transcriptase. They have RNA genome. and is based on the fact that, in bacteria,
Tumour viruses cause benign and 103. Digeorges’ Syndrome (DGS) also transcription and translation proceed
malignant tumours in animals and called as thymic aplasia occurs simultaneously.
humans. because of the deficiency of T-cells. 111. In protoplast fusion, tissues of two
DGS is caused by abnormal formation different plants are cultured. Their cells
94. K-T boundary refers to the mass
of certain tissues during the foetal are separated and treated with
extinction of 60 million years ago when
development. pectinase and cellulase to produce
dinosaurs disappeared. It is connected
with deposits of iridium which are rare 104. The percentage of lipoprotein is naked protoplast which are then made to
on the earth. maximum in chylomicrons. fuse by electrofusion.
95. C–represents coleoptile which is a Chylomicrons transport lipids absorbed 112. In humans, in week 3, the neural tube
protective covering for plumule in the from the intestine to various tissues, develops and the brain and spinal cord
embryo. where their triglyceride components begin to develop.
96. Statements I and III are correct, while are hydrolysed by the activity of the 113. The number of chromosomes in leaf cells
statement II is incorrect. This can be lipoprotein lipase, allowing the is 24 (2n). Aleurone cells are the part of
corrected as : The slow block released free fatty acids to be endosperm which is triploid in nature.
polyspermy develops in response to the absorbed by the tissues. Therefore, the number of chromosomes
formation of fertilisation membrane and in it would be 36 (3n).
105. The statement (b) is incorrect
polyspermy is the process of entry of
regarding Spirogyra. In it, conjugation 114. The statement (d) is incorrect regarding
more than one sperm into the oocyte.
tube is formed. The male gamete significance of biopatents. It can be
97. In a polygonum type of embryo sac, passes through this tube to unite with corrected as
the egg apparatus is situated towards female gamete.
the micropylar end. Bioethics is the branch of ethics,
106. Acetyl Co-A is the intermediate philosophy and social commentary that
98. Phototropism is the curvature compound that links glycolysis to deals with the biological sciences and
movement that takes place when plant their potential impact on society.
Krebs cycle. The end product of
is provided with artificial or natural light
glycolysis is pyruvate. It is converted 115. Bordered pits are found on the radial
only from one direction. It can be
into acetyl Co-A before it enters the walls of xylem tracheids and vessels.
positive (towards light) or negative
(away from light). Krebs cycle. Bars of Sanio develop below these
107. Apoinducer is a protein that binds with pits.
99. The plants growing in aquatic habitat
are known as hydrophytes. They have the DNA to activate transcription. 116. Catabolic repression is a system of
no cuticle over the epidermis. The Catabolite Activator Protein (CAP) gene control in some bacterial operons
aerenchyma is well-developed and activates transcription through in which glucose is used preferentially
secondary growth is absent. interactions with RNA-polymerase. This and the metabolism of other sugars
100. Streptokinase is an enzyme produced interaction causes binding of DNA near is repressed in the presence of
by streptococci that catalyse the the transcription start site, thus glucose.
conversion of plasminogen to plasmin. effectively catalysing the transcription Catabolite repression allows
It is administered intravenously as a initiation process. microorganisms to adapt quickly to
thrombolytic agent in the treatment of 108. In cockroach, corpora allatum is an preferred carbon/energy source (like
acute coronary arterial thrombosis. endocrine gland which generates glucose) first. This is achieved through
101. The production of fertile soil on bare juvenile hormone. It plays a crucial role inhibition of synthesis of enzymes
rocks is primary succession. It begins in metamorphosis. Removal of the involved in catabolism of carbon
in an area where no living organisms corpora allata can cause immature sources.
ever existed. It takes several hundred
larva to pupate at its next molt, 117. Population of organisms evolves to
to thousand years under favourable
climatic conditions to produce fertile resulting in miniature adult. maximise their reproductive fitness, i.e.
soil. Darwinian fitness in the habitat in which

MODULE 2
www.jeebooks.in
112 NEET Test Drive

they live, under a particular set of 124. CO 2 is produced by the burning of 133. Hormone Function
selection pressures. Organisms evolve fossil fuel.
towards most efficient reproductive Human chorionic Maintains corpus
CH4 is produced by freshwater luteum
strategy. Life history traits of organisms gonadotropin
wetlands.
have evolved in relation to the CFC is a non-toxic gas. Progesterone Suppresses uterine
constraints imposed by components of contractions
N2O is produced by industrial
habitat in which they live. Oestrogen Promotes growth of
processes.
118. Cohesin is a multi-subunit protein myometrium
125. Intrafascicular cambium is cambium
complex that holds the sister Relaxin Softens cervix to
between xylem and phloem.
chromatids together after DNA prepare for dilation
replication, i.e. in prophase and Interfascicular cambium is
metaphase, during anaphase removal parenchymatous cells of medullary 134. Undersecretion of growth hormone
of cohesin leads to the separation of rays. causes dwarfism. Very small people are
sister chromatids. Tyloses are balloon-like structures. produced due to the undersecretion of
Heartwood is physiologically inactive. growth hormone. Such people are called
119. Viruses are known as wandering genes ‘midgets’.
because nucleic acid is the only active 126. Testis is a primary sex organ as it
part of it. The infectivity of virus is due produces male gametes and secretes 135. In honeybees, an offspring formed from
to nucleic acid, while host specificity is sex hormone, testosterone. the union of a sperm and an egg
determined by the protein coat. develops as a female and an unfertilised
127. Enhancers are DNA sequences that egg develops as a male (drone) by
120. Total volume of air a person can inspire facilitate the expression of a given means of parthenogenesis. Thus, males
after a normal expiration is the gene. They may be located few have half the number of chromosomes
inspiratory capacity. It includes Tidal hundred base pairs away from the than that of a female.
Volume (TV) + Inspiratory Reserve gene.
Volume (IRV). 136. Bilirubin is produced in the
Whereas, promoter is a DNA sequence
macrophage-monocyte system by the
121. Benign tumour does not invade other at which RNA polymerase may bind,
breakdown of haemoglobin to biliverdin
tissues or spread to other sites. It is leading to the initiation of transcription.
and then bilirubin. It is carried to the liver
usually well-encapsulated in 128. Thyroxine is formed by the amino acid by attaching to the albumin as it is
connective tissue. tyrosine, which is made by the body insoluble in water. In liver, bilirubin is
122. Creatine is an endogenous organic from another amino acid phenylalanine. conjugated with glucuronic acid by the
compound, which is synthesised from 129. Maintenance of favourable dynamic enzyme glucuronyltransferase, making it
simpler amino acids, namely glycine, constancy is called thermoregulation, soluble in water.
arginine and sulphur containing amino while maintenance of constant internal 137. In cockroach, XX-XO sex chromosome
acid, methionine. Its synthesis takes temperature by an organism is called complement is found. In this pattern, the
place in the kidney and liver which is homeostasis. The oldest living plant female has two X-chromosomes (called
then transported to muscles through on earth is bristle cone pine of XX) while male has only one
the blood. California. X-chromosome (called XO). The
Creatine is found in vertebrates where X-chromosome is completely absent
130. Vitamin-B12 or cyanocobalamin is a
it facilitates recycling of ATP here. Thus, the presence of unpaired
cobalt containing vitamin found in X-chromosome determines the
molecules. This is achieved by meat, liver and fish. It promotes DNA
converting ADP back to ATP through masculine sex. The female produces
synthesis and maturation of RBCs. The only one type of eggs and male
donation of phosphate group. absorption of cobalamin in the intestine produces two types of sperms, i.e. 50%
123. CAP-cAMP binds to a specific site on requires the action of a glycoprotein, with one X-chromosome and 50%
the DNA, stimulating transcription of Castle’s intrinsic factor. without any sex chromosome.
the lac genes and production of 131. Hepaticopsida has capsule without any
lactose-metabolising enzymes. 138. The depletion of ozone layer leads to an
columella. increase in ground level ultraviolet
The lac operon is an operon required In Anthocerotopsida, sterile columella radiation, because ozone is an effective
for the transport and metabolism of is present. absorber of ultraviolet radiation. The
lactose in E.coli and many other effects of ozone depletion include
In Bryopsida, sporophyte is
bacteria. inhibition of photosynthesis in
well-differentiated into foot, seta and
In the absence of lactose, the lac capsule. phytoplanktons which affects the food
repressor, halts the production of Marchantiales includes Riccia. chains. It damages nucleic acids in
enzymes encoded by lac operon. living organisms (mutation). The ozone
cAMP binds to Catabolite Activator
132. Golden rice is a b-carotene enriched depletion causes an increase in UV
Protein (CAP), which assists RNA variety of Oryza sativa. It was created radiations reaching the earth’ surface
polymerase in binding to the DNA. by transforming rice with two which can cause skin cancer.
This significantly increases the b-carotene biosynthesis genes, psy
139. Insulin increases glucose uptake in liver,
production of b-galactosidase, and crtl. The crtl gene was isolated
leucocytes and smooth, cardiac and
enabling the cell to hydrolyse lactose from the soil bacterium, Erwinia
skeletal muscles. It does not do so in
and release galactose and glucose. uredovora.
brain, renal tubules, intestinal mucosa
and RBCs.

MODULE 2
www.jeebooks.in
Prep Catalysis NEET ~ Mock Test 1 113

140. The lac operon of E.coli contains genes employed as a technique for the The photosynthetic pigments in
involved in lactose metabolism. commercial production of vegetables Rhodophyceae are chlorophyll-a,
Two regulators turn the operon ‘on’ and such as tomato, seedless cucumber, phycocyanin, phycoerythrin and
‘off’ in response to lactose and glucose etc. Thus, this technique facilitates the xanthophyll.
levels, the lac repressor and Catabolite production of seasonal vegetables. 156. The MN blood group in humans is an
Activator Protein (CAP). The lac It also avoids soil borne pathogens. example of codominance.
repressor senses lactose through However, the cost of these experiments In codominance, there is joint expression
allolactose and stops acting as is high and also requires skilled people of both alleles in a heterozygote. In this
repressor. CAP activates transcription to perform it. situation, two alleles of a single gene are
of operon sensing low glucose levels 149. Crinoidea is a class of stalked and responsible for producing two distinct
through cAMP. sedentary echinoderms. and detectable gene products.
141. Tay-Sachs disease is an example of 150. Maternal blood sampling is a 157. Tumour-suppressor genes (e.g. p 53
incomplete dominance. In this disease, non-invasive technique to detect foetal gene) are very important to maintain the
mutations in the gene coding for an disorders. genomic integrity. They help in stopping
enzyme, hexosaminidase that causes division of cells having damaged DNA.
neurological dysfunction. This enzyme The test measures the level of three or
four substances, e.g. alfa fetoprotein, 158. In glomerulus, the afferent arteriole is
breaks down lipid byproducts
hCG, etc., in a sample of mother’s shorter and wider than efferent arteriole
gangliosides in the cell’s lysosomes. In
blood during pregnancy. These tests which is longer and narrower.
Tay-Sachs disease, homozygous
recessive individuals are severely are done to detect birth defects in the 159. The radioactive waste should be
affected with a fatal lipid-storage foetus such as Down’s syndrome, brain disposed after sufficient pre-treatment, in
disorder, whereas, heterozygous or spinal defects, etc. suitably shielded containers buried
individuals have only 50% of enzyme 151. Pollination by snails is called within rocks, about 500m deep below the
activity found in normal individuals. malacophily and by ants is called Earth’s surface and in specially
myrmecophily, e.g. Anemone constructed buildings.
142. Water potential refers to the chemical
potential of water in a system. The nemarosa. The radioactive waste may be
water potential of pure water is zero. 152. The statement (d) is incorrect reprocessed, but this method is
Water potential increases when pure regarding viruses as they can abandoned as it increases the volume of
water is heated as there is increase in reproduce only inside a host cell. wastes almost 150 times.
free energy. Hence, all the statements 153. Sewage is treated in Sewage 160. Heterotrophic bacteria resemble oldest
are correct regarding water potential. Treatment Plants (STP) before disposal fossil.
143. The labellings of occipital and temporal to make it less polluting. This treatment 161. The options (a), (b) and (c) are the
bones have been interchanged. The is carried out in two stages, viz. characteristics of virusoids. Prions are
bone labelled occipital bone here is primary treatment and secondary the proteinaceous infectious particles.
actually temporal bone and the bone treatment. The primary treatment steps They have a distinct extracellular form
labelled as temporal bone is actually basically involve the physical removal which is entirely a protein.
occipital bone. of large and small particles from the
sewage through filtration and 162. Oral contraceptive pill is a popular
144. Eryx johnii (Sand boa) is a method of contraception used by
sedimentation.
double-headed snake. It has a small, females. They work by inhibiting
non-prehensile thick tail bearing a 154. On the basis of functions, the
ovulation and implantation.
resemblance to head. It is a species of meristematic tissue is divided into
protoderm, procambium and ground ‘Saheli’ is a new oral contraceptive pill
non-venomous snake in the subfamily
meristem. for females that contain a non-steroidal
Erycinae of the family Boidae. It is
preparation. It is once a week pill
adapted to burrowing so, the head is Protoderm is the outermost
with very few side effects and high
wedge-shaped. meristematic layer of apical meristem
contraceptive value.
145. Himalayan ranges separate Palaearctic which develops into epidermis.
and Oriental regions. Procambium is located inside 163. MAB is associated with studying impact
protoderm and gives rise to primary of human interference on environment.
146. Lysozymes present in the saliva act as vascular tissue. It is an international biological
an antibacterial agent that prevent programme of UNESCO. It is also
infections. Ground meristem is the precursor of
associated with pollution in biotic and
ground tissue which forms pith, cortex,
147. Craidukov’s phenomenon is the abiotic environments. It is involved in
hypodermis and pericycle.
capacity of changing colour according conservation strategies for present
to the wavelength of incident light. It is 155. All the pairs are correctly matched. The and future.
exhibited by species that develop photosynthetic pigments in
164. Cardiac sphincter is a valve that guards
green shades in red light, brownish- Chlorophyceae are chlorophyll-a and b
the oesophagus and stomach.
yellow in blue light, etc. It is also known carotenoids.
It prevents the food from the stomach to
as chromatic adaptation. The photosynthetic pigments in come back to mouth. In newborns, it is
148. Hydroponics is a technique of growing Phaeophyceae are chlorophyll-a and c, not fully matured, allowing the stomach
plants in a nutrient solution. fucoxanthin. contents to flow backwards.
Hydroponics have been successfully

MODULE 2
www.jeebooks.in
114 NEET Test Drive

165. Sclerophyllous are leaves which are 169. An appreciable amount of gaseous Its main function is to reduce surface
stiff and hard, e.g. Banksia. exchange of carbon between tension, thereby preventing the
Trichophyllous are leaves covered with organisms and environment occurs. bronchioles and the lungs from
hairs, e.g. Nerium. Malacophyllous are Thus, the statements (a) and (b) are collapsing.
leaves which are soft and fleshy, e.g. true for carbon cycle. 175. In the given flow chart, a and b
Begonia. 170. Real time PCR utilises SYBR green for respectively represent ACTH and
166. Vegetative propagation is a form of detection and estimation of DNA cortisol. Control of glucose to glycogen
asexual reproduction of plants where products. SYBR green is a cyanine dye is a result of the release of
new plants grow from a single parent used to stain DNA. It binds with DNA adrenocorticotropic releasing factor.
plant. The plants produced through molecules by intercalating between the
176. In Drosophila, gynandromorphs occur
vegetative propagation are clones and DNA bases.
due to loss of X-chromosome.
are morphologically as well as 171. Brown sugar is also called smack. It is A gynandromorph is an organism that
genetically similar to each other and an opioid derivative, derived from contains both male and female
their parent. morphine. Its chemical name is characteristics. It is mainly used in
diacetyl morphine hydrochloride and is entomology. These organisms can be
167. Maltose is a disaccharide containing
more powerful analgesic than butterflies, moths, etc.
two molecules of glucose. It is formed
morphine. 177. Hippocampus (seahorse) is a marine
during germination of starchy seeds
and is digested with the help of 172. Tropical rainforests–Dipterocarpus bony fish having bony plates and scutes
enzyme maltase produced in small Tropical deciduous forests–Sal besides scales. It has a brood pouch on
the belly of males for incubating eggs.
intestine. Desert–Acacia sp.
Coastal biome–Mangroves 178. According to Blackman’s law, when a
168. The excretory system of cockroach
process depends on a number of
helps in eliminating the nitrogenous 173. Crop Hybrid variety factors, its rate is limited by the pace of
wastes from the body in the form of uric the slowest factor. Therefore, at a
acid. The structures associated with Barley Erectiferum particular time, photosynthesis can be
excretory functions are Malpighian Peppermint Todd’s Mitcham limited by CO 2 concentration or light.
tubules, fat bodies, uricose glands,
Castor Aruna 179. The option (d) is incorrectly matched
nephrocytes and cuticle. Urate cells
pair. It can be corrected as follows
present in the fat bodies absorb and Rice Atomita-2 Yellow fever is caused by Flavivirus.
store uric acid throughout life. This is
Wheat Pusa Lerma Smallpox is caused by variola virus.
called storage excretion as they remain
stored in the cells of the corpora 180. Both flagella and cilia have a 9 + 2
174. Pulmonary surfactant is a mixture of arrangement of microtubules. Dynein
adipose. lipids and proteins which is secreted ‘arms’ attached to the microtubules
by the epithelial cells, viz. clara cells. serve as the molecular motors.

SCORE CHART
No. of Correct Answers : A .................. No. of Incorrect Answers : B ..................
Total Marks : x = (A × 4) – (B × 1)
Scores and Expected Rank : If the score lies above 665, then rank will be in between 1-50. For other scores, rank
estimations are given below
Score Rank
664 – 642 51 – 150
641 – 636 151 – 250
635 – 630 251 – 400
629 – 625 401 – 500

MODULE 2
www.jeebooks.in
Prep Catalysis for NEET ~ Mock Test 2 115

PREP CATALYSIS
for NEET
Full Length Mock Tests for NEET to Make You Ready to Face the Challenge

MOCK TEST 2 (With Solutions)

Duration : 3 Hours Max. Mark : 720

Instructions
The test is of 3 hours duration and Test Booklet contains 180 questions. Each question carries 4 marks. For
each correct response, the candidate will get 4 marks. For each incorrect response, one mark will be deducted
from the total scores. The maximum marks are 720.
The question paper contains three parts of Physics, Chemistry and Biology respectively.
Part A contains 45 Questions from Physics section. Part B contains 45 Questions from Chemistry section.
Part C contains 90 Questions from Biology.

PART A PHYSICS
1. Force required to move a train of 1500 quintals up 4. The output of the given logic circuit is
an incline of 1 in 75 (a slope that has a rise of A O2
1 unit for every 75 unit of displacement) with an
acceleration of 4 ms - 2 is
O1
(where, force of friction = 0.2 newton per quintal) Y
(a) 650000 N (b) 690000 N
(c) 619900 N (d) 625000 N
B O3
2. In a sonometer experiment, density of the material (a) AB + BA (b) ( A + B)A
of wire used is 7 . 5 ´ 103 kg m - 3 . If the stress in the (c) A × B + B × A (d) None of these
wire is 3 ´ 108 Nm - 2 , the speed of transverse wave
on wire, is 5. Relation between wavelength of photon and
(a) 225 ms - 1 (b) 250 ms - 1 electron of same energy is
(c) 200 ms -1
(d) 100 ms -1 (a) lph > le (b) lph < le
(c) lph = le (d) le / lph = constant
3. A radioactive sample with a half life of 1 yr has the
label ‘activity = 5m curie on 1-9-2018’. Then, what 6. A steam boat goes across a lake and comes back
would be its activity two years earlier? (A) on a quite day, when the water is still and (B)
(a) 0.5 m curie (b) 10 m curie
on a rough day, when there is uniform air current,
so as to help the journey onward and to impede the
(c) 20 m curie (d) 50 m curie
journey back.

MODULE 2
www.jeebooks.in
116 NEET Test Drive

If the speed of the launch on both days was same, 12. What is the current through an ideal p - n junction
in which case it will complete the journey in lesser diode shown below?
time?
100 Ω
(a) Case (A) (b) Case (B)
(c) Same in both (d) Nothing can be predicted +

– 1V – 3V
7. Two coherent light beams of intensities I and 4I +
are superposed. The maximum and minimum
possible intensities in the resulting beam are
(a) Zero (b) 10 mA (c) 20 mA (d) 40 mA
(a) 5I and I (b) 5I and 3I
(c) 9I and I (d) 9I and 3I 13. A star subtends an angle f at the pole of a concave
8. Indian space agency (ISRO) thinking about making mirror of focal length f. Then, the radius of the
space elevator on earth by using steel cables. If an image of the star formed by the mirror is
2 ff
untapered space elevator cable would need a (a) (b)
material capable of sustaining a length of 5000 km ff 2
of its own weight at sea level to reach a (c) 2ff (d) ¥
geostationary altitude of 35800 km without 14. The frequency f of a stretched string depends upon
yielding. So, for this idea, the length of the steel
cable which will break under its own weight when the tension force F, length l of the string and the
suspended vertically is (take, breaking strength of mass per unit length m of the string. The formula
steel = 1 . 2 ´ 109 Nm - 2, density of steel cable for frequency is (k = a constant)
k m kF
= 8 ´ 103 kg / m3 and gravitational acceleration, (a) f = (b) f =
l F m
g = 10 m / s2)
l
m F
(a) 20,000 m (b) 50 km (c) 15000 km (d) 15 km (c) f = kl (d) f = kl
F m
9. Two condensers, one of capacity C and the other of
C 15. The internal energy of an ideal diatomic gas
capacity , are connected to V volt battery, as decreases by the same amount as the work done by
2
shown in the figure. the system
(i) the process must be adiabatic
(ii) the temperature must decrease
V C C (iii) the process must be isobaric
2
(iv) the process must be isothermal
Which option satisfies the statements?
The work done in charging fully both the (a) (i), (iv) (b) (ii), (iii)
condensers is (c) (i), (ii) (d) All statements are correct
1 3 1
(a) CV 2 (b) CV 2 (c) CV 2 (d) CV 2 16. A particle of mass 10g is describing SHM along a
4 4 2
straight line with a period of 2s and amplitude of
10. A carnot engine is a device 10 cm. The difference in kinetic energy, when it is
(a) which convert mechanical energy into heat energy 2 cm and 5 cm from its equilibrium position is
(b) which convert heat energy into mechanical energy (a) 100p 2 erg (b) 105 p 2 erg
(c) absorbs heat from a sink at a lower temperature and (c) 480 p 2 erg (d) 375p 2 erg
rejects to the source at high temperature
(d) None of the above 17. A particle of mass 10 g is kept on the surface of an
uniform sphere of mass 100 kg and radius 10 cm.
11. A black body of mass 50 g and surface area Then, the work to be done to take the particle far
11 . 46 cm 2 is at an initial temperature of 800 K. It is away from the sphere is
allowed to cool inside an evacuated enclosure kept (take, G = 6 . 67 ´ 10- 11 N - m 2 kg - 2 )
at constant room temperature 300 K. The rate of (a) 3. 33 ´ 10- 10 J
cooling is 0.05ºC/s. The specific heat of body is (take,
(b) 6. 67 ´ 10- 10 J
Stefan’s constant, s = 5 . 73 ´ 10- 8 Jm - 2 K - 4 )
(c) 13. 34 ´ 10- 10 J
(a) 11. 5 kJ kg - 1 K - 1 (b) 15. 5 kJ kg - 1 K - 1
(c) 9. 5 kJ kg - 1 K - 1 (d) 10. 5 kJ kg - 1 K - 1 (d) 6. 67 ´ 10- 9 J

MODULE 2
www.jeebooks.in
Prep Catalysis for NEET ~ Mock Test 2 117

18. Pressure versus temperature graph of an ideal gas 25. Match the quantities of Column I with their values
is shown in the figure. Density of the gas at point in Column II and choose the correct option from
A is r 0. Density at point B will be the codes given below
p Column I Column II
h
3p0 B A. Planck constant 1.
p
B. Stopping potential 2. E0 - KEmax
p0 A KEmax
C. Work function 3. e
T0 2T0 T E
3 3 4 D. de-Broglie wavelength 4. n
(a) r 0 (b) r 0 (c) r 0 (d) 2r 0
4 2 3
Codes
19. Two soap bubbles of diameter dA and dB combine A B C D A B C D
under isothermal conditions in space outside (a) 1 3 2 4 (b) 3 2 1 4
international space station at low earth orbit. Now, (c) 4 3 1 2 (d) 4 3 2 1
find the new radius of bubble, if low earth orbit at
a distance of 37000 km from earth. 26. The threshold wavelength of photon for a metal
d + dB d Ad B surface is 36 A° . If a light of wavelength 2240 A
° is
(a) A (b)
2 dA + dB incident on metal surface, then velocity of ejected
electron will be
(c) d A d B (d) d A2 + d B2
(given, h = 6.63 ´ 10- 34, c = 3 ´ 108 m /s,
20. Two bodies of masses 10 kg and 2 kg are moving me = 91. ´ 10- 31 kg)
(a) 18. 4 ´ 106 m / s (b) 8. 6 ´ 105 m / s
with velocity ( - 2i$ + 7 $j + 3k
$ ) and (10$i - 35 $j + 3k$ ),
6
(c) 15 ´ 10 m / s (d) 12 . 5 ´ 105 m / s
respectively. The velocity of the centre of mass
will be 27. The reading of a spring balance corresponds to
(a) 2 k$ ms - 1 (b) 2 $j ms - 1 (c) 3k$ ms - 1 (d) 3$j ms - 1 100 N, while situated at the north pole and a body
is kept on it. The weight recorded on the same
21. A ractangular coil having 20 turns carries a
scale, if it is shifted to the equator is
current of 15 A. The coil is oriented with respect to
uniform magnetic field of 0.2 T, such that its area (take, acceleration due to gravity at poles
vector is - 0.05 $i m 2 . The potential energy of coil in g p = 10 ms - 2 and radius of earth = 6 . 4 ´ 103 m)
the given orientation is (a) 99.66 N (b) 110 N (c) 97.66 N (d) 106 N
(a) 0 (b) 7.5 J (c) 4 J (d) 3 J
28. Two similar coils of radius r and number of turns
22. In an adiabatic compression of a monoatomic gas N are lying concentrically with their planes at
and its volume reduces to one-third of initial right angles to each other. The current flowing in
volume, then its final pressure would be, if gas has them are 6 I and 3 I respectively, then magnetic
initial pressure p field induction at centre of the coils is
(a) 31/ 2 p (b) 33 / 2 p 3 m 0 NI m 0 NI 3 m 0 NI m 0 NI
(a) (b) (c) (d)
(c) 6.24 p (d) 33 / 5 p 4r 4r 2r 2r

23. An air bubble inside a cubical block of glass of side 29. At normal temperature and pressure, the speed of
7 cm appears to be at 3.5 cm from one face and sound in air is 330 ms- 1. The speed of sound in
1.5 cm from the other face opposite to the first hydrogen at 273ºC and 3 atm pressure is (take, air
when viewed normally. Then, the real depth of the as 16 times heavier than hydrogen)
bubble from the opposite face is [take, refractive
(a) 1850 ms - 1 (b) 1860 ms - 1
index of glass, m = 1 . 5] -1
(a) 3 cm (b) 2.5 cm (c) 2.1 cm (d) 2 cm (c) 1866.48 ms (d) 1806.48 ms - 1

24. A point mass oscillates along the X-axis according 30. A voltmeter has a resistance of 90 W is connected
to the relation x = x0 cos(wt - p / 4). If acceleration across a cell of emf 10 V and internal resistance
of the particle is written as a = a0 cos( wt + d ), then 10 W . The voltmeter reads the voltage of
(a) a0 = x0 w2 , d = - p / 4 (b) a0 = x0 w2 , d = p / 4 (a) 12 V (b) 6 V
2
(c) a0 = x0 w , d = – 3p / 4 (d) a0 = x0 w2 , d = 3p / 4. (c) 3 V (d) 9 V

MODULE 2
www.jeebooks.in
118 NEET Test Drive

31. If Young’s double slit experiment is performed in 39. A lens is placed between a source of light and a
water instead of air, then wall. It forms images of area A1 and A2 on the wall,
(a) no fringes would be seen for its two different positions. The area of the
(b) fringe width would decrease source of light is
(c) fringe width would increase 1 1 A1 - A2 A1 + A2
(a) + (b) (c) A1 A2 (d)
(d) fringe width would remain unchanged A1 A2 2 2

32. A current loop of radius r has a magnetic field 40. The mass of jupiter is 315 times that of earth and
induction B1, at a point on its axis at a distance r the diameter of jupiter is 11.5 times that of earth.
from its centre and B2 at its centre, then B1 : B2 is If value of g on earth is 9 . 8 ms - 2 , then its value on
(a) 1 : 2 2 (b) 2 2 : 3 jupiter is
(c) 1 : 2 (d) 3 : 2 2 (a) 23. 52 ms - 2 (b) 23. 90 ms - 2
-2
33. The voltage and current through a circuit element (c) 23. 60 ms (d) 23. 34 ms - 2
are V = 100 sin( 314t + 45º ) V,
41. The output Y , when all the three inputs are first
i = 10 sin( 314t + 315º )A. Then, circuit element is
high and then low, will respectively be
(a) capacitor (b) inductor
X
(c) resistor (d) None of these
Y
34. An aluminium sphere of radius 10 cm is heated
from 0º to 100ºC. Its volume changes by (a) 1, 1 (b) 1, 0 (c) 0, 1 (d) 0, 0
(where, coefficient of linear expansion for
aluminium = 23 ´ 106/º C ) 42. Which of the following statement(s) is (are) correct
regarding electric lines of forces?
(a) 28.9 cc (b) 2.89 cc
(i) The electric lines of forces are parallel for
(c) 9.28 cc (d) 49.8 cc
non-uniform electric field.
35. A projectile P is projected at an angle of 30º with the (ii) Closer are the electric lines of forces, stonger is
horizontal. If a second projectile Q is projected such the field, and further apart are lines of force,
that their speeds are in ratio 3 : 1 for same maximum weaker is electric field.
height, then the angle of projection of Q is
(iii) Electric lines of forces never cross each other
(a) sin- 1 æç ö÷ (b) sin- 1 æç ö÷
3 1
è 1ø è 3ø otherwise, there will be two dircetions of
electric field at a point of intersection which is
(c) sin- 1 æç ö÷ (d) sin- 1 æç ö÷
3 2
è2 ø è 3ø not possible.
(a) (i) only (b) (i) and (ii) only
36. In a region, electric field and magnetic field both (c) (ii) and (iii) only (d) all are correct
are acting in the same direction. If a proton is 43. In gamma ray, emission from a nucleus
projected along the direction of the fields with a (a) Both the neutron number and the proton number change
certain velocity, then (b) there is no change in the proton number and the neutron
(a) it will turn towards left (b) it will turn towards right number
(c) its velocity will increase (d) its velocity will decrease (c) only the neutron number changes
37. The earth has a mass of 6 ´ 1024 kg and a radius of (d) only the proton number changes
6 . 4 ´ 106 m. The amount of work done in joules, if 44. If potential difference across a capacitor is changed
its rotation were to be increased, so that the from 10 V to 20 V, then work done is W . What will
duration of the day becomes 20 h instead of 24 h. be the work done, if potential difference is changed
2 from 30 V to 40 V?
(Moment of inertia of earth = MR 2) W
5 (a) 10 W (b) W (c) (d) 2W
(a) - 9. 360 ´ 10 25
J (b) 9. 360 ´ 10 30
J 2
(c) 9. 358 ´ 1028 J (d) - 9. 358 ´ 1030 J 45. A research satellite of mass 200 kg circles the
3R
2
earth in an orbit of average radius , where R is
38. A current loop of area 0 . 05 m and carrying a 2
current of 4 A is held perpendicular to a magnetic the radius of earth. Assuming the gravitational
field induction of 0.5 T, then torque acting on the pull on a mass of 1 kg on the earth’s surface to be
loop is 10 N, the pull on the satellite will be
(a) 0. 01 N - m (b) 0.1 N - m (a) 880 N (b) 889 N
c) 1 N-m (d) 0 N-m (c) 885 N (d) 892 N

MODULE 2
www.jeebooks.in

PART B CHEMISTRY
46. If travelling at same speeds, which of the following 54. Solubility of AgCl in water, 0.01 M CaCl 2 , 0.01 M
matter waves have the shortest wavelength? NaCl and 0.05 M AgNO 3 are S1 , S 2 , S3 and S 4
(a) Electron (b) a-particle respectively, then
(c) Neutron (d) Proton (a) S1 < S 2 < S 3 < S 4 (b) S1 > S 3 > S 2 > S 4
(c) S1 > S 2 = S 3 > S 4 (d) S1 > S 3 > S 4 > S 2
47. The value of n in
MnO-4 + 8H + + ne - ¾® Mn2+ + 4H 2 O is 55. Among following reactions, which one will produce
dihydrogen?
(a) 5 (b) 4 (c) 2 (d) 3
(a) H2S4O 8 + H2O (b) BaO +HCl
48. In a flask at a certain temperature there are 2 g H 2 (c) Mg +H2O (d) Na 2O 2 + 2HCI
and 8 g O 2 . The mole fraction of O 2 in the given 56. Which of the following alkaline earth metals do not
mixture is impart any colour to the flame?
(a) 8/5 (b) 0.2 (c) 0.25 (d) 1.0 (a) Ca, Sr (b) Mg, Ca
49. Which of the following statements is in accordance (c) Be, Mg (d) Sr, Ba
with the Arrhenius equation? 57. The solution which maintains its pH constant even
(a) Rate of a reaction has no effect with increase in upon addition of small amounts of acid or base, is
temperature called buffer solution. Which can act as a buffer?
(b) Rate of a reaction increases with decrease in activation (a) NH4Cl +NH4OH
energy
(b) CH3COOH+CH3COONa
(c) Rate constant decreases exponentially with increase in
(c) 40 mL of 0.1 M NaCN + 20 mL of 0.1 M HCN
temperature
(d) Rate of reaction decreases with decrease in activation (d) All of the above
energy 58. The metallic luster exhibited by sodium is
50. Helium atom is two times heavier than hydrogen explained by the
molecule. At 298 K, the average kinetic energy of (a) existence of body centred cubic lattice
helium atom is (b) excitation of free electrons
(a) two times that of hydrogen molecule (c) diffusion of sodium ions
(b) same as that of hydrogen molecule (d) oscillation of loose electrons
(c) four times that of hydrogen molecule 59. Which one has maximum oxidation member of the
(d) half that of hydrogen molecule underlined atoms in the following compounds?
51. The work to be done on 2 moles of perfect gas at (a) MnO 2-
4 (b) CrO 5
27°C, if it is compressed reversibly and (c) Cr O 2Cl 2 (d) Equal in all the options
isothermally from a pressure of 1.01 ´ 105 Nm -2 to
5.05 ´ 106Nm 2 is 60. Perkin’s condensation is an example of chemical
4
(a) 2 .03 ´ 10 J 2
(b) 1.95 ´ 10 J
properties of aromatic aldehydes only. In this
4 8 reaction aromatic aldehydes condense with an acid
(c) 1.95 ´ 10 J (d) 1.95 ´ 10 J
anhydride to yield a , b-unsaturated acids.
52. pH of 0.5 M aqueous solution of HF ( K a = 2 ´ 10-4 ) Intermediate involved in Perkin reaction is
is (a) carbocation (b) carbanion
(a) 2 (b) 4 (c) 6 (d) 10 (c) free radical (d) carbene
53. An electrochemical cell consists of two half-cell
reactions 61. The per cent loss in mass after heating a pure
sample of 245 g potassium chlorate (mol. mass
AgCl( s ) + e - ¾® Ag( s ) + Cl - ( aq )
=122.5) will be
Cu( s ) ¾® Cu 2+( aq ) + 2e - (a) 12.25 (b) 24.50
The mass of copper (in grams) dissolved on passing (c) 39.18 (d) 49.0
0.5 A current for 1 hour is (Given : Atomic mass of 62. Anhy. AlCl 3 produces fumes in the air. This is due to
Cu is 63.6; F = 96500 C mol -1)
(a) hydrolysis (b) reduction
(a) 0.88 g (b) 1.18 g (c) oxidation (d) dimerisation
(c) 0.29 g (d) 0.56 g

MODULE 2
www.jeebooks.in
120 NEET Test Drive

63. Quartz is extensively used as a pizeoelectric 71. A graph of log [A] vs t for first order reaction is
material as it contains given below.
(a) Sn (b) Ti
(c) Si (d) Pb
- -

log [A]
64. Basic- strength of CH ºº C(I); CH 2 == CH (II) and Slope = X
CH 3 CH 2 (III) will be in order
(a) I <II<III (b) II <III> I
(c) III <II<I (d) III <I<II
t
65. The temperature of the slag zone in the metallurgy X is equal to
of iron using blast furnace is 0.693 k k
(a) 1200-1500°C (b) 1500-1600°C (a) (b) (c) - (d) log [ A] 0
k 2.303 2 .303
(c) 400-700°C (d) 800-1000°C
- d[ A] 2d[ B]
66. The one of the product obtained on reaction of 72. For the reaction A ® B, = then the
dt dt
ethyl magnesium bromide with methanol is
rate law is
(a) ethane (b) methane
d [B] d [B]
(c) propane (d) methoxyethane (a) - = k[B]2 (b) = k[ A]
dt dt
67. For ZnS, the coordination number of cation and d [ A] d[ A]
(c) = k[ A]2 (d) - = k[ A]2
anion are respectively dt dt
(a) 4,4 (b) 4,6 73. Which of the following is the most stable free
(c) 6,6 (d) 4,8
radical?
68. Which of the following statements is/are incorrect CH3
regarding corrosion?
(a) Impurities present in iron enhance rusting by settling a
number of miniature cells (a) (b)
(b) It is an electrochemical phenomenon
(c) The technique of protecting a metal from corrosion by
connecting it to a second metal is known as anodic (c) (d)
protection
(d) It is an oxidation deterioration of a metal
69. Which of the following represents the correct
increasing order of acidic behaviour of the 74. CoSO 4 Cl . 5NH 3 exists in two isomeric forms ‘A’ and
molecules given below ethyne, benzene and hexane? ‘B’. Isomer ‘A’ reacts with AgNO 3 to give white
(a) Benzene < Hexane < Ethyne precipitate, but does not react with BaCl 2. Isomer
(b) Hexane < Benzene < Ethyne ‘B’ gives white precipitate with BaCl 2 but does not
(c) Hexane < Benzene > Ethyne react with AgNO 3 . The IUPAC name of ‘A’ and ‘B’
(d) Ethyne < Hexane < Benzene respectively are
70. Which of the following curve show Freundlich (a) [A] is pentaamminesulphatocobalt (III) chloride
adsorption isotherm? [B] is pentaamminechloridocobalt (III) sulphate
(b) [A] is tetraamminedisulphatocobalt (III) chloride
[B] is pentaamminechloridecobalt (III) sulphate
x x (c) [A] is triamminedisulphatocobalt (III) chloride
(a) log (b) log
m m [B] is pentaamminechloridocobalt (III) sulphate
(d) [A] is pentaamminesulphatocobalt (III) chloride
log p log p [B] is tetraamminedichloridocobalt (III) sulphate

75. Solids are attracted by magnetic field due to the


presence of atoms, ions or molecules with unpaired
x x
(c) log (d) log electron, called paramagnetic. Which among the
m m
following is paramagnetic?
(a) Cl 2O 7 (b) Cl 2O
log p log p
(c) ClO 2 (d) Cl 2O 5

MODULE 2
www.jeebooks.in
Prep Catalysis for NEET ~ Mock Test 2 121

76. Which of the following statement(s) regarding Codes


actinides is/are correct? A B C D A B C D
(a) Pu4+ disproportionates to Pu3+ and PuO 2+
2 in strongly acidic
(a) 2 3 4 1 (b) 3 4 2 1
medium (c) 1 2 3 4 (d) 4 3 2 1
(b) Maximum oxidation state of Np is +7. 84. The major product of the following reaction is
(c) UO 2 2+ is stable
O
(d) All of the above statements are correct
H CH3MgBr (excess)
77. Count the total number of S ¾ O bonds, which are H O
H3O+
having equal length in bisulphate ion?
(a) 3 (b) 4 (c) 5 (d) 6 (a) Propanol (b) Propanal
(c) Acetaldehyde (d) None of these
78. The synthesis of alkyl fluorides is best
accomplished by 85. Which one of the following pairs of pyrimidine
(a) Swart’s reaction (b) free radical fluorination bases are present in DNA?
(c) Finkelstein reaction (d) Sandmeyer reaction (a) Cytosine and uracil
(b) Cytosine and guanine
79. In dichromate ion (Cr2 O 7 2 - ), each Cr is linked to (c) Cytosine and adenine
(a) two O- atoms (b) three O- atoms (d) Cytosine and thymine
(c) four O- atoms (d) five O- atoms
86. Phenol is heated with pthalic anhydride in
80. The coordination entity formed when excess of presence of conc. H 2 SO 4 . The product gives pink
KCN is added to an aqueous solution of CuSO 4 is colour with alkali. The product is
(a) [Cu(CN)2 ]+ (b) [Cu(CN)6 ]4- (a) phenolpthalein (b) bakelite
(c) [Cu(CN)4 ] 2-
(d) [Cu(CN)4 ]3+ (c) salicylic acid (d) fluorescein

81. Which of the following solvent is used in the 87. For the heterogenous reaction,
A(s) + 2B(g) 3D(s) + 2Y(g), the relationship
e
synthesis of chlorofluorocarbon?
between equilibrium constants, K p and K C is
(a) Methylene chloride
(a) K p = KC [RT]2 (b) K p = KC [RT]0
(b) Carbon tetrachloride
(c) Chloroform (c) K p = KC [RT] -2 (d) K p = KC [RT] -1
(d) Iodoform 88. Among the aldehydes and ketones, ‘‘the size of
82. A solution containing 2.665 g of CrCl3 × 6H 2O is alkyl group is inversely proportional to their
reactivity’’. Cannizzaro reaction is an example of
passed through a cation exchanger. The chloride
disproportionation reaction, i.e. self
ions obtained in solution were treated with excess
oxidation-reduction. Which of the following will
of AgNO 3 to give 2.87 g of AgCl. The structure of
undergo Cannizzaro reaction?
compound is
(a) Benzaldehyde (b) Acetaldehyde
(a) [CrCl × (H2O)5 ]Cl 2 × H2O
(c) Acetone (d) Pentanone
(b) [CrCl 2 × (H2O)4 ] Cl × H2O
(c) [CrCl × (H2O)5 ]Cl × H2O 89. Those polymers which get decomposed by the
(d) [CrCl 2 × (H2O)5 ]Cl 2 × H2O process of biodegradation are known as
biodegradable polymer. Among the following, the
83. Match the Column I with Column II and choose biodegradable polymer is
the correct option using the codes below. (a) cellulose (b) polyethene
Column I Column II (c) PVC (d) nylon-6
A. O2- 1. Paramagnetic 90. CH3CONH 2 and CH3CH 2NH 2 are distinguished by
B CN- 2. Diamagnetic reacting with
C. CO 3. Diamagnetic (a) aq. NaOH and heat (b) aq. HCl and heat
+ (c) Br2 water (d) acidic KMnO 4
D. NO 4. Diamagnetic

MODULE 2
www.jeebooks.in

PART C BIOLOGY
91. Leaves and branches falling from trees often 98. Which among the following statements is incorrect
damage smaller plants beneath them. It is an regarding dicotyledons?
example of (a) Possess tap root system
(a) amensalism (b) commensalism (b) Leaves have reticulate venation
(c) mutualism (d) competition (c) Stem is branched
(d) Secondary growth is absent
92. In bryophytes, the food conducting cells are known
as 99. Thermocycler is used in
(a) hydroids (b) leptoids (c) rhizoids (d) bulbils (a) micropropagation (b) hybridisation
(c) PCR (d) fermentation
93. Which among the following statements is/are
correct regarding eutrophication? 100. Identify the incorrectly matched pair?
(a) It is natural ageing of water body (a) J-shaped curve – Logistic growth
(b) It occurs due to nutrient enrichment of water body (b) S-shaped curve – Represented under limited
particularly with nitrogen and phosphorus resource conditions
(c) Both (a) and (b) (c) Both (a) and (b)
(d) None of the above (d) None of the above
94. Which of the following plants can grow in N 2 101. During eukaryotic transcription, what is the role or
deficient soil? function of an enhancer?
(a) Nymphaea (b) Pisum sativum (a) Increases mRNA stability
(c) Stellaria (d) Nepenthes (b) Elongation of transcript
(c) Transcription initiation
95. The bacterium commonly called ‘Weizman
(d) Stimulates the rate of transcription
organism’ helps in the fermentation of sugar to a
mixture of organic solvents for the production of 102. Which of the following is a freshwater sponge that
biofuel blend stock is harbours green algae in it?
(a) Clostridium acetobutylicum (b) Pseudomonas nitrificans (a) Euspongia
(c) Acetobacter aceti (d) Thiobacillus denitrificans (b) Euplectella
96. The biopatents are awarded for achievements in (c) Cliona
(d) Spongilla
I. strains of microorganisms.
II. DNA sequences. 103. Which factor(s) among the following is/are
III. proteins encoded by DNA sequences. responsible for decrease in absorption of water by
the roots at low temperature?
IV. cell lines.
(a) increased viscosity of water
Choose the correct option. (b) decreased rate of root elongation
(a) I and II (b) I and III
(c) decreased metabolic activity of root cells
(c) III and IV (d) I, II, III and IV
(d) All of the above
97. Match the era given in Column I with the
104. Choose the option that correctly explains the
organisms that appeared in these periods in
meaning of the terms related to ecological
Column II and choose the correct option from the
succession.
codes given below.
(a) Sere – Sequence of communities that
Column I Column II successively change in a given
A. Devonian 1. Fishes area
B. Ordovician 2. Man (b) Seral stage – Changes during ecological
succession that leads to a
C Pennsylvanian 3. Reptiles
community
D. Holocene 4. Amphibians (c) Climax community – Individual transitional
Codes communities
A B C D A B C D (d) Pioneer community – Gradual and predactable
(a) 4 1 3 2 (b) 1 3 4 2 change in a community
(c) 2 4 1 3 (d) 3 2 4 1

MODULE 2
www.jeebooks.in
Prep Catalysis for NEET ~ Mock Test 2 123

105. With respect to the mechanism by which the plant 112. The left phallomere, a part of the external
tissues sense gravity (gravisensing), which of the genitalia of male Periplaneta consists of which of
following statements is correct? the following sets of structures?
(a) It leads to redistribution of auxin in stems and roots, (a) Titillator, phallic gland
leading to differential growth (b) Phallic gland, mushroom gland
(b) It may be involving pressure-sensitive channels at the top (c) Vasa deferentia, an ejaculatory duct
and bottom of cells (d) Titillator, pseudopenis
(c) Starch grains inside of cells which fall to the lower side of
the cell (statoliths) may be involved 113. In protein structure, the strongest bonds are
(d) All are correct (a) ionic bonds (b) covalent bonds
(c) hydrogen bonds (d) van der Waals interactions
106. Which class of cyclin-CDK complex is present at
the G 2 checkpoint and is required for the cell cycle 114. The maximum transportation of oxygen in the
to enter the mitotic phase from G 2-phase? blood occurs
(a) CdK1/Cyclin B (b) CdK 4 /Cyclin D (a) in dissolved form (b) as bicarbonate
(c) CdK 6/Cyclin D (d) CdK 4 /Cyclin B (c) as oxyhaemoglobin (d) Both (a) and (b)

107. Match the following columns and choose the 115. Which one among the following statements
correct option from the codes given below. correctly describes a tracheid?
(a) Lignin is deposited in the cellulose wall to form annular or
Column I Column II spiral thickenings
A. PCT 1. Urine concentration (b) Dead, single cells with hard and lignified walls
(c) Both (a) and (b)
B. Vasa recta 2. Reabsorption
(d) None of the above
C. Glomerulus 3. Ultrafiltration
116. The tuberculated rhizoids are characterised by
D. DCT 4. Selective secretion
(a) narrow rhizoids
Codes (b) lack of cytoplasm at maturity
A B C D A B C D (c) peg-like ingrowth
(a) 2 4 3 1 (b) 4 1 3 2 (d) All of the above
(c) 2 1 3 4 (d) 1 2 3 4
117. Which of the following fungi is used for the
108. The two kingdom system of classification was production of antibiotic penicillin?
discarded because of the (a) S. griesus
(a) presence of characters of both plants and animals in (b) P. notatum
Euglena (c) P. chrysogenum
(b) difference of bacteria and cyanobacteria from other (d) Both (b) and (c)
organisms
(c) non-photosynthetic nature of fungi 118. CAM (Crassulacean Acid Metabolism) plants have
(d) All of the above so adapted, that allows them to
(a) function better in humid conditions
109. The host cells are made competent to take up DNA (b) fix CO 2 during the day
by treating them with
(c) save enzyme RuBisCO from denaturation
(a) potassium (b) strontium
(d) survive in arid conditions
(c) magnesium (d) calcium
119. Match the following columns and choose the
110. Which of the following techniques is/are highly
correct option from the codes given below.
effective for birth control?
I. Vasectomy II. Tubectomy Column I Column II
III. Periodic abstinence IV. Coitus interruptus A. Enterobius vermicularis 1. Whipworm
Choose the correct option. B. Wuchereria bancrofti 2. Eyeworm
(a) I and II (b) III and IV C. Trichuris trichiura 3. Filarial worm
(c) II and III (d) Only IV
D. Loa loa 4. Pinworm
111. Which of the following exhibits myxotrophic
Codes
nutrition?
A B C D A B C D
(a) Euglena (b) Cyanobacteria
(a) 4 3 1 2 (b) 3 1 2 4
(c) Amoeba (d) Sponges
(c) 2 3 4 1 (d) 4 2 1 3

MODULE 2
www.jeebooks.in
124 NEET Test Drive

120. Deficiency of which of the following is the cause for 130. The figure given below is showing the structure of
albinism? a microsporangium. Two of its labellings are
(a) Acetyl cholinesterase (b) Carbonic anhydrase wrong. Choose the option which correctly indicates
(c) Tyrosinase (d) Glutamine them.
Tapetum
121. Hybrid vigour (or heterosis) is a result of Connective
(a) homozygosity of the gametes Epidermis
(b) blending of male and female characteristics Endothecium
(c) increased heterozygosity in the offspring Middle layers
(d) retention of ancestral genes
Vascular strand
122. In collenchyma, the corners of cells are deposited
with
Sporogenous
(a) cellulose (b) pectic substances
tissue
(c) Both (a) and (b) (d) None of these

123. Which of the following is a major inorganic


Structure of microsporangium
chemical component of Murashige and Skoog’s
nutrient medium? (a) Tapetum and vascular strand
(a) Potassium nitrate (b) Sodium EDTA (b) Endothecium and connective
(c) Calcium sulphate (d) Cobalt chloride (c) Middle layers and sporogenous cells
(d) Tapetum and endothecium
124. The book written by Carolus Linnaeus, which lists
every species of plant known at that time is 131. Identify the correct statement(s) regarding
(a) Systema Naturae (b) The origin of life bulliform cells.
(c) Green Data Book (d) Species Plantarum (a) Bulliform cells are found in monocot leaves
(b) They help in rolling of leaves in dry weather
125. Which among the following is not an example of
(c) They are also known as motor cells
biopiracy?
(d) All of the above
(a) Basmati rice (Oryza sativa)
(b) Black pepper (Piper nigrum) 132. Ephemerals are short-lived xerophytes, which lack
(c) Indian mustard (Brassica campestris) the capacity to tolerate drought. They survive in
(d) Bacillus anthracis the critical dry period in the form of
(a) barks and leaves (b) leaves and thorns
126. A human gene was amplified, cloned and finally (c) fruits and seeds (d) thorns and seeds
transformed into E.coli. But, gene expression
turned out to be poor because of 133. The psychological disorder characterised by
(a) codon bias unreasonable thoughts and fear, repetitive
(b) presence of termination signals behaviours and panic attack is called
(c) presence of introns (a) clinical depression
(d) All of the above (b) Obsessive Compulsive Disorder (OCD)
(c) bipolar disorder
127. Genes whose expression (product) is constantly (d) attention deficit disorder
required for cellular activity are known as
(a) housekeeping genes 134. Match the following ‘man’ given in Column I with
(b) smart genes their cranial capacity given in Column II and
(c) regulatory genes choose the correct option from the codes given
(d) structural genes below.
Column I Column II
128. A series of genes, transcription of which is
regulated by DNA binding proteins A. Handy man 1. 900 cc
(a) promoter (b) operator B. Peking man 2. 1360 cc
(c) operon (d) terminator C. Java ape man 3. 700 cc
129. Diuretic drugs that promote secretion of excess D. Cro-Magnon man 4. 1075 cc
urine from the body, are commonly used in the
Codes
treatment of
A B C D A B C D
(a) high blood pressure (b) renal failure
(a) 1 2 3 4 (b) 2 3 4 1
(c) renal calculi (d) glomerulonephritis
(c) 3 4 1 2 (d) 4 1 2 3

MODULE 2
www.jeebooks.in
Prep Catalysis for NEET ~ Mock Test 2 125

135. Which among the following statement(s) is/are 145. Heavy Mero-Myosin (HMM) contains
correct regarding dioxin? (a) globular head
(a) It is a colourless organic compound (b) tail
(b) It is the most toxic herbicide (c) globular head with a short arm
(c) It was used in Vietnam War as a defoliating agent (d) short arm and a tail
(d) All of the above
146. Mycoplasma
136. An annelid which shows bioluminescence is (a) is a prokaryote and lacks cell wall
(a) Polynoe (b) Lumbricus (b) exhibits pleomorphism
(c) Chaetopterus (d) Nereis (c) does not show crystallisation
(d) All of the above
137. Two species of rose grow in the same garden but do
not interbreed because one blooms in winters and 147. ‘Gomphoses’ is an example of
one blooms in summers. This is best explained by (a) non-movable joints (b) imperfect movable joints
(a) temporal isolation (b) mechanical isolation (c) perfect movable joints (d) sutures
(c) geographical isolation (d) gametic isolation
148. Mycorrhiza is the symbiotic relationship between
138. The vital capacity includes ……… and roots of plants.
(a) inspiratory reserve volume + tidal volume (a) algae (b) cyanobacteria
(b) tidal volume + inspiratory reserve volume + expiratory (c) fungi (d) bacteria
reserve volume
(c) expiratory reserve volume + tidal volume 149. Which one of the following statements is false with
(d) expiratory reserve volume + inspiratory reserve volume respect to the condition phenylketonuria?
(a) It is the result of pleiotropy
139. In which type of placenta, the villi are present in (b) It is an autosomal recessive trait
the form of transverse bands? (c) It is a metabolic error
(a) Discoidal (b) Diffused (d) It is a case of aneuploidy
(c) Cotyledonary (d) Zonary
150. When a gene has two or more alleles that are
140. Which of the following statements is/are correct? common in a population, the phenomenon is
(a) HBOT can be used for wound healing known as
(b) Oxygen is a critical factor in wound healing (a) polyploidy (b) polygenic inheritance
(c) Both (a) and (b) (c) polymorphism (d) complementary genes
(d) None of the above
151. The sympathetic nervous system regulates all of
141. The oxides of sulphur are an important pollutant these except
because they cause (a) gall bladder (b) liver
(a) drying up of margin of leaf and portion of lamina in (c) adrenal glands (d) urinary bladder
between vascular bundles
(b) harm to eyes, throat and lungs 152. Match the Column I with Column II and choose
(c) Both (a) and (b) the correct option from the codes given below.
(d) None of the above Column I Column II
142. In certain body parts like finger tips, nose, pinnae, A. Dicot stem 1. Hypodermis is sclerenchymatous
etc., their is direct connection between the arteriole B. Monocot stem 2. Pericycle gives rise to lateral roots
and venule, which is known as, only
(a) arterio-venous anastomosis
C. Monocot root 3. Pericycle gives rise to lateral roots
(b) interelastic membrane
and secondary meristem
(c) diapedesis
(d) pleuripotential haemopoietic stem cells D. Dicot root 4. Hypodermis is collenchymatous

143. The leader region of the trp operon codes for RNA Codes
that may function as a A B C D A B C D
(a) inducer (b) repressor (a) 1 2 3 4 (b) 1 4 3 2
(c) 1 4 2 3 (d) 4 1 2 3
(c) attenuator (d) apoprotein
153. Name the class of antiviral proteins produced by
144. The basic structure of liver is composed of which T-lymphocytes after viral stimulation.
tissue? (a) Alpha interferon (b) Gamma interferon
(a) Mucous tissue (b) Reticular tissue (c) Beta interferon (d) None of these
(c) Areolar tissue (d) Lymphoid tissue
MODULE 2
www.jeebooks.in
126 NEET Test Drive

154. The injury to vagus nerve is unlikely to affect 161. Match Column I with Column II and choose the
which of the following? correct option from the codes given below.
I. Movement of eyelids. Column I Column II
II. Tongue movement. A. Endangered species 1. Facing a very high risk of
III. Cardiac movement. extinction in near future
IV. Peristaltic movement. B. Threatened species 2. Liable to become extinct
Choose the correct option. C. Extinct species 3. Last individual has died
(a) I, III and IV (b) I and II D. Vulnerable species 4. Not critically endangered
(c) III and IV (d) Only II
Codes
155. Which among the following statement(s) is/are A B C D A B C D
correct regarding cymose inflorescence? (a) 1 3 2 4 (b) 4 3 2 1
(a) Development of flowers is basipetal (c) 3 4 2 1 (d) 1 2 3 4
(b) The growth is definite
(c) Opening of flowers is centrifugal 162. Which hormone is responsible for maintaining the
(d) All of the above
circadian rhythms in non-mammalian vertebrates
as well as acts as a neurosecretory transducer?
156. A boy whose metabolic rate is abnormally high and (a) Pituitary gland (b) Thymus gland
has protruding eyeball is suffering from which of (c) Thyroid gland (d) Pineal gland
the following diseases?
(a) Goitre (b) Grave’s disease 163. In which stage of childbirth uterine contractions
(c) Hashimoto’s disease (d) Addison’s disease begin marking the beginning of labour?
(a) Expulsion stage (b) Placental stage
157. For the development of root and stem, a plant
requires (c) Dilation stage (d) Gestation stage
(a) calcium (b) copper 164. A farmer wished to produce bananas that were
(c) magnesium (d) sodium longer and sweeter. He opted for grafting method
158. Match the pancreatic hormones given in Column I to achieve this. What are the chances of his success
with the cells that are responsible for their to achieve this target?
secretion given in Column II. (a) 50% (b) 100% (c) 25% (d) Negligible

Column I Column II 165. Ecosystem services include


(a) healthy forest ecosystem purifies air and water
A. Insulin 1. δ -cells
(b) generate fertile soil
B. Glucagon 2. α-cells (c) maintain biodiversity
C. Somatostatin 3. γ-cells (d) All of the above
D. Pancreatic polypeptide 4. β-cells 166. Which one of the following pairs exhibits
5. F-cells mutualism between them?
I. Fig and Blastophaga wasp.
Choose the correct option from the codes given
II. Ophrys and male Colpa wasp.
below.
III. Honeybees and Salvia.
Codes
A B C D A B C D
IV. Coral tree and squirrels.
(a) 4 2 1 5 (b) 1 5 4 2 Choose the correct option.
(c) 4 2 5 1 (d) 1 3 2 4 (a) Only I (b) I and II (c) I, II and III (d) All are correct
159. Biomagnification occurs because 167. Smoking leads to which of the following diseases?
(a) pollutants like DDT, mercury are non-biodegradable (a) Emphysema (b) Tuberculosis
(b) pollutants like DDT cannot be metabolised by organisms (c) Bronchitis (d) Both (a) and (c)
(c) pollutants like DDT cannot be excreted out by organisms
consuming them 168. The condition where the anthers do not dehisce
(d) All of the above
and the germinated pollen tube pierces anther wall
and enters the stigma of same flower, is known as
160. A person is suffering from deficiency of a certain (a) geitonogamy (b) cleistogamy (c) herkogamy (d) dicliny
hormone due to which his I.Q. level is equivalent to
a 5 years old boy. Which hormone is deficient in 169. Cucurbita shows
this man? (a) reticulate multicostate divergent type venation
(a) Aldosterone (b) GH (b) reticulate multicostate convergent type venation
(c) ACTH (d) Thyroxine (c) parallel unicostate venation
(d) parallel multicostate venation

MODULE 2
www.jeebooks.in
Prep Catalysis for NEET ~ Mock Test 2 127

170. In which type of ovule the embryo sac becomes Choose the correct option.
curved-like horse shoe? (a) Only I (b) Only II (c) Only III (d) All of these
(a) Orthotropous (b) Anatropous
175. Genetically Modified Organisms (GMOs) are
(c) Campylotropous (d) Amphitropous
developed by rDNA technology and genetic
171. Tryptophan operon in E. coli, is an example of engineering. They are useful in
(a) positively regulated operon (a) enhancing food’s nutritional value
(b) inducible operon (b) reducing in the post-harvest losses
(c) repressible operon (c) increasing the tolerance power of crops against the abiotic
(d) None of the above stresses
(d) All of the above
172. Identify the correctly matched pair.
(a) Virus – Take over cellular machinery of the host cell 176. Which of the following statement(s) is/are correct
(b) Viroids – Modified hydrophobic glycoproteins about tocopherols, also called ‘beauty vitamins’?
(c) Both (a) and (b) (a) It is an antioxidant
(d) None of the above (b) It protects unsaturated fatty acids and membranes
(c) Its deficiency causes sterility and muscular atrophy
173. Varun gorges upon the potato finger chips which (d) All of the above
he is very fond of. Which is the correct order of
enzymes that are likely to act on it from mouth 177. The five kingdom system of classification is based
onwards? on
(a) Salivary amylase – Pancreatic amylase – Disaccharidases (a) cell structure (b) body structure
(b) Pancreatic amylase – Salivary amylase – Lipases (c) nutrition and lifestyle (d) All of these
(c) Salivary maltase – Lipases – Nucleases 178. The non-collapsible wall of trachea in human
(d) Disaccharidases – Lipases – Nucleases beings is because of the incomplete rings made of
174. Identify the incorrect statement(s). (a) elastic cartilage (b) hyaline cartilage
(c) Both (a) and (b) (d) None of these
I. Bioballistic is a method in which vectors of
pathogens are allowed to infect the cell to 179. Plasmids found in prokaryotic cells, are also known
transfer the recombinant DNA. as
II. Microinjection is a method when cells are (a) peroxisomes (b) extranuclear RNA
bombarded with high velocity particles of gold (c) microbodies (d) extranuclear DNA
coated with DNA.
180. Cellular totipotency was demonstrated by
III. Heat shock is a method when the recombinant
(a) AV Leeuwenhoek (b) Robert Hooke
DNA is incubated with the host cells at
(c) FC Steward (d) Schleiden and Schwann
80-100°C.

Answer Sheet
1. (c) 2. (c) 3. (c) 4. (c) 5. (a) 6. (a) 7. (c) 8. (d) 9. (c) 10. (b)
11. (d) 12. (d) 13. (b) 14. (b) 15. (c) 16. (b) 17. (d) 18. (b) 19. (d) 20. (c)
21. (d) 22. (c) 23. (c) 24. (d) 25. (d) 26. (b) 27. (a) 28. (c) 29. (c) 30. (d)
31. (b) 32. (a) 33. (b) 34. (a) 35. (c) 36. (c) 37. (c) 38. (d) 39. (c) 40. (d)
41. (a) 42. (c) 43. (b) 44. (b) 45. (b) 46. (b) 47. (a) 48. (b) 49. (b) 50. (b)
51. (c) 52. (a) 53. (d) 54. (b) 55. (c) 56. (c) 57. (d) 58. (d) 59. (d) 60. (b)
61. (c) 62. (a) 63. (c) 64. (a) 65. (d) 66. (a) 67. (a) 68. (c) 69. (b) 70. (c)
71. (c) 72. (d) 73. (c) 74. (a) 75. (c) 76. (d) 77. (a) 78. (a) 79. (c) 80. (c)
81. (b) 82. (a) 83. (c) 84. (a) 85. (d) 86. (a) 87. (b) 88. (a) 89. (a) 90. (a)
91. (a) 92. (b) 93. (c) 94. (d) 95. (a) 96. (d) 97. (a) 98. (d) 99. (c) 100. (a)

101. (d) 102. (d) 103. (d) 104. (a) 105. (d) 106. (a) 107. (c) 108. (d) 109. (d) 110. (a)
111. (a) 112. (d) 113. (b) 114. (c) 115. (b) 116. (d) 117. (d) 118. (d) 119. (a) 120. (c)
121. (c) 122. (c) 123. (a) 124. (d) 125. (d) 126. (d) 127. (a) 128. (c) 129. (a) 130. (a)
131. (d) 132. (c) 133. (b) 134. (c) 135. (d) 136. (a) 137. (a) 138. (b) 139. (d) 140. (a)
141. (c) 142. (a) 143. (c) 144. (b) 145. (c) 146. (d) 147. (a) 148. (c) 149. (d) 150. (c)
151. (c) 152. (d) 153. (b) 154. (b) 155. (d) 156. (b) 157. (a) 158. (a) 159. (d) 160. (d)
161. (d) 162. (d) 163. (c) 164. (d) 165. (d) 166. (c) 167. (d) 168. (b) 169. (a) 170. (d)
171. (c) 172. (a) 173. (a) 174. (d) 175. (d) 176. (d) 177. (d) 178. (b) 179. (d) 180. (c)

MODULE 2
www.jeebooks.in

EXPLANATIONS

PHYSICS
1. Given, m = 1500 quintals h h 1
5. For electron, le = = Hence, W =U = C ¢V 2
4
= 15 ´ 10 kg, p 2mE 2
= æç
1 hc 1 3C ö 2 3 2
sin q = , a = 4 ms - 2 \ l= ÷V = CV
75 2mE 2è 2 ø 4
Total force of friction hc
For photon, lph = 10. An engine uses heat energy to produce
= 0. 2 ´ 1500 = 300 N E mechanical work.
Force required against gravity in \ Wavelength of photon is greater than
11. Newton’s law of cooling is
moving the train up the inclined plane that of electron, i.e. l ph > le .
dT
= mg sin q mc = s(T 4 - T 04 )A
6. Case A Time in quite day, dt
1 l l 2l
= 15 ´ 104 ´ 9. 8 ´ tA = + = s(T 4 - T 04 )A
75 vB vB vB Þ c=
dT
= 19600 N Case B When v A be the velocity of air, m
dt
Force required to produce an then on rough day the time will be
acceleration of 4 ms - 2 = ma 5 .73 ´ 10- 8 ´ [8004 - 300] 4 ]
l l 11. 46 ´ 10- 4
tB = +
= 15 ´ 104 ´ 4 = 600000 N vB + v A vB - v A =
Total force required (50 ´ 10- 3 ) ´ 0. 05
2l
= 300 + 19600 + 600000 = . kJ kg -1 K -1
= 105
é æv ö ù
2
= 619900 N vB ê1 - ç A ÷ ú 12. It is clearly from below diagram that
êë è vB ø ú given diode will be forward biased,
2. Tension in wire,T = Stress ´ Area û
= 3 ´ 108 ´ A tB 1 100Ω
Þ =
Mass per unit length, tA é æv ö ù
2
ê1 - ç A ÷ ú +
m = A ´ l ´ r = A ´ 1 ´ 7 .5 ´ 103

êë è vB ø ú 1V –+
3V
û

Speed of transverse wave, 2
vA
T 3 ´ 108 ´ A As, 1- 2 < 1
v = = vB
m A ´ 7.5 ´ 103 Hence,
tB 1 - (- 3) 4
= 200 ms -1 Þ > 1or tB > t A I = = = 40 mA
tA 100 100
3. Given, activity A1 = - lN 0 = 5 m curie 13. For a star, u = ¥
7. Given, I1 = I , I 2 = 4I
2 yr earlier, the number of half life
2 I max = ( I1 + I2 ) 2 \ By mirror formula,
= =2
2
1 = ( I + 4I )
Activity, A2 = - lN ¢ 0 = - l 2 2 N 0 = (3 I ) 2 = 9I A′ φ
= - l4N 0 I min = ( I1 - I 2 ) 2
A2 - l 4N 0 Horizon
So, = = ( I - 4I ) 2 C f
A1 - lN 0 B′
= (- I ) 2 = I
A2
Þ =4 Alrg
5 8. We know that, stress = = rlg
A
Þ A2 = 20m curie Breaking stress 1 1 1
So, l = + =
4. Here, O1 = A × B rg v u f
. ´ 109 1 1 1
O 2 = A × (A × B ) l =
12 + =
(- v ) ¥ f
and O 3 = B × (A × B ) 8 ´ 103 ´ 10
= 15000 m = 15 km Þ v = -f
So, Y = A × (A × B ) × B × (A × B ) So, it is not possible to make space The image of star will be formed at focus
= A × (A × B ) + B . (A × B ) elevator by using steel cable. which is real, inverted and diminished.
Height of image, h = A ¢ B ¢
[QM = M ] 9. The two condensers in the circuit are in Arc h
C 3C \ Angle, f = Þ f=
= A. (A + B ) + B . (A + B ) parallel order, hence C ¢ = C + = Radius f
2 2
= A. A + A.B + A .B + B .B h = ff
The work done in charging the h ff
[QMM = 0] \ Radius of image of star = =
equivalent capacitor is stored in the
= A .B + B . A form of potential energy
2 2

MODULE 2
www.jeebooks.in
Prep Catalysis for NEET ~ Mock Test 2 129

14. Speed of wave over string, 8T From Eqs. (i) and (ii), we get
19. For first bubble, p1 =
F dA 7-x x
v = =
m 8T 3 .5 1.5
and p2 =
F 1 F dB 1.5 (7 - x ) = 3 .5 x
f×l = Þf =
m l m where,T = surface tension 10.5 - 1.5 x = 3 .5 x
2l Under isothermal condition, 10.5 = 3 .5 x + 1.5 x
As for a string, l =
n temperature remain constant. Þ 10.5 = 5 x
where, n = number of harmonic So, equating number of moles,
Þ x =
10.5
= 2 .1 cm
= 1,2,3, ..... pV = p1V1 + p 2V2 5
n F k F é 4 æD ö 3 ù x = x 0 cos(wt - p / 4)
8T é 4 æ d A ö ù 24.
3
So, f = = 8T
2l m l m ê pç ÷ ú = ê pç ÷ ú dx
D êë 3 è 2 ø úû d A ëê 3 è 2 ø úû v = = - x 0 w sin(wt - p / 4)
15. Adiabatic process dW = - dU dt
8T é 4 é dB ù ù
3
dv
dU = nCV dT + ê pê ú ú a= = - x 0 w 2 cos(wt - p / 4)
dB êë 3 ë 2 û úû dt
dU decrease, thenT decrease.
= + x 0 w 2 cos[p + (wt - p / 4)]
16. Velocity at displacement y is D = d A + dB2
2 2
= x 0 w 2 cos(wt + 3 p / 4)
v = w A2 - y 2 Þ D = d A2 + dB2 Comparing it with given equation,
Angular frequency, m1v1 + m 2v 2 a = a0 cos(wt + d)
20. As we know, v CM = 3p
2p 2p m1 + m 2 Þ a0 = x 0 w 2 and d =
w= = = p rad s - 1 4
T 2 10 (- 2 $i + 7 $j + 3 k$ )
when, y = 2 cm + 2 (10$i - 35 $j + 3 k$ ) 25. (a) Planck constant
=
v = w 102 - 2 2 = 96 p cms - 1 10 + 2 E
= [QE = hn]
1 1 30k$ + 6k$ n
KE = mv 2 = ´ 10 ´ 96 ´ p 2 =
2 2 12 (b) Stopping potential
= 480 p 2 erg 36k$ KE max
= = [QeV0 = KE max]
when, y = 5 cm 12 e
v = w 102 - 5 2 = 75 p 2 cms - 1 = 3 k$ ms - 1 (c) Work function (f 0 ) = E 0 - KE max
h
1 1
KE = mv 2 = ´ 10 ´ 75 ´ p 2 21. Potential energy of coil, (d) de-Broglie wavelength =
P
2 2 U = - m ×B
= 375 p 2 erg = - NI × A × B 26. By Einstein’s photoelectric equation,
Difference = 480p 2 - 375 p 2 = - 20 ´ 15 [- 0. 05 i$ × 0. 2 i$] 1 2 æ1 1ö
mev max = hc ç - ÷
= 105 p 2 erg =3J 2 è l l0 ø
17. Initial PE of the body, 5
22. For monoatomic gas, g = 2hc æ 1 1ö
GMm 3 Vmax = ç - ÷
Ui = - 5/3 me è l l 0 ø
R 5/3 æVö
So, pV = p¢ç ÷
6. 67 ´ 10- 11 ´ 100 ´ 10- 2 è3ø 2 ´ 6. 63 ´ 10- 34 ´ 3 ´ 108
=
0. 01 Þ p ¢ = p (3) 5 / 3 = 6. 24 p . = 9.1 ´ 10- 31
-9
= - 6. 67 ´ 10 J é 1010 1010 ù
23. Refractive index, ê - ú
When the particle is far away from the ë 2240 3640 û
sphere, the PE of the system will be 7 cm
= 8 . 6 ´ 105 m / s
zero, i.e.
(7–x) cm 27. The change in gravity from pole to
Uf = 0
x cm equator is
\ W = Uf - Ui = + 6. 67 ´ 10- 9 J
g p - ge = Rw 2 = 3 . 37 ´ 10- 2
m
18. From pV = RT , we get Þ ge = g p - 3 . 37 ´ 10- 2
M
Real depth = (10 - 3 . 37 ´ 10- 2 )
m pM m=
= = 9. 9663 ms - 2
V RT Apparent depth
m pM 7-x Weight of equator,
and r= = m= … (i) [For first face] w e = mge
V RT 3 .5 100
rB rBT A 3 p 0 .T 0 3 x = ´ 9. 9663
\ = = = and m = … (ii) [For opposite face] 10
r A TB p A 2T 0 . p 0 2 1.5 = 99. 663 N

MODULE 2
www.jeebooks.in
130 NEET Test Drive

28. As, we know B = B12 + B 22 From the above equations, it is 39. By displacement method, size of object
observed that voltage leads current by (O ) = l1l 2
[Qq = 90° so, cos q = 0]
90º. Hence, given circuit element is \ Area of source light = A1A2
m NI
= 0 3+ 6 inductor. M
2r 40. Given, j = 315
3m 0NI 34. Cubical expansion, Me
=
2r DV = gVDT = 3 a VDT Diameter of jupiter
and = 11.5
[Qg = 3 a] Diameter of earth
29. Speed of sound in hydrogen as
= 3 ´ 23 ´ 10- 6 Rj
ra \ = 11.5
vH = ´ æç p ´ 103 ö÷ ´ 100
va 4
rH Re
è3 ø
GM
16rH = 28 . 9 cc As, g = 2
= ´ 330 R
rH 2
35. As heights are same, gj M j æ Re ö
= 4 ´ 330 i.e. HP = HQ \ = ç ÷
ge Me è R j ø
= 1320 ms - 1 U A2 sin2 30º UB2 sin2 q
Þ = 2
The sound is unaffected by pressure. 2g 2g æ 1 ö
So, v 0 and v 273 be the speeds of sound = 315 ´ ç ÷
sin2 q
æU ö 9
2 è 11.5 ø
of hydrogen at 0ºC and 273ºC, Þ = ç A÷ = 315 ´ 9. 8
= 23 . 34 ms - 2
2
respectively. Then, sin 30º è UB ø 1 Þ gj =
v 273 273 + 273 9 11.5 ´ 11.5
= or sin2 q = ´ sin2 30º
v0 273 + 0º 1 41. When, all input are high
9 NOR (1, 1) = 0 = X
= 2 = 1. 414 =
v 273 = 1. 414 ´ 1320 4 and NAND (Y = 0, 1) = 1
q = sin- 1 æç ö÷
3
= 1866. 48 ms - 1 Þ When, all input are low
è2ø
æ e ö æ 10 ö NOR (0, 0) = 1 = X
30. V = ç ÷R = ç ÷ 90 36. Since, proton is moving in the direction and NAND (Y = 1, 0) = 1
èR + r ø è 90 + 10 ø
of magnetic field, hence magnetic field
10 42. Statement (ii) and (iii) are correct.
= ´ 90 = 9 V do not apply any force on proton, only
100 electric field will apply force on proton The electric lines of forces are parallel
Dl in the direction of motion, hence its for an uniform electric field.
31. Fringe width, b =
d velocity will increase. 43. In g-ray emission, the energy is released
or bµl 37. Given, M = 6 ´ 1024 kg, from nucleus, so that nucleus get
If the experiment is performed in water, R = 6. 4 ´ 106 m stabilised.
wavelength l decreases, so fringe
T1 = 24 h andT 2 = 20 h 44. Ratio of work done,
width also decreases.
Work done = Increase in rotational KE 2
32. As we know that, magnetic field at the W2 æ DV2 ö
1
= K 2 - K1 = I (w 22 - w12 ) =ç ÷
axis, 2 W1 è DV1 ø
m0 2 pIr 2 1 2 éæ 2p ö2 æ 2p ö2 ù (40 - 30) 2
B1 = × 2 = ´ MR 2 ê ç ÷ -ç ÷ ú = =1
4 p (r + r 2 ) 3 / 2 2 5 êë è T 2 ø è T1 ø ú (20 - 10) 2
û
m0 I \ W2 = W1 = W .
= × 1 2 2æ 1 1ö
4 2r = MR ´ 4 p ç 2 - 2 ÷
5 èT 2 T1 ø 45. Gravitational acceleration,
m 0I 10
= 6 ´ 1024 ´ (6. 4 ´ 106 ) 2 g = = 10 ms - 2
4 2r 1 ´ 4 ´ 9. 87 1
= ´ 2
Magnetic field at the centre, 5 (3600) 2
g ¢ = g æç
R ö
mI ÷
B2 = 0 æ 1 - 1 ö è 3R / 2 ø
2r ç ÷
è 400 576 ø 4
So, B1 : B 2 = 1 : 2 2 = 10 ´
= 9. 358 ´ 1028 J 9
33. Given,V = 100 sin(314t + 45 º) =
40
ms - 2
38. Here, q = 0
i = 10 sin (314t + 315 º) 9
\ t = NIAB sin 0° [Qsin 0° = 0]
= 10 sin [314t + (360º - 45 º)] Gravitational pull on the satellite
t = 0 N-m 40 ~
= 10 sin(314t - 45 º) = mg ¢ = 200 ´ - 889 N
9

MODULE 2
www.jeebooks.in

CHEMISTRY
46. From de-Broglie equation, Be and Mg are too strongly bound to 63. Quartz is the amorphous form of
h get excited by flame. In other words, silicon (Si).
wavelength, (l) = . For same speed the energy required to excite electrons
mv 64. CH ºº CH, CH2 == CH2 , CH3 ¾ CH3
of different particles, i.e. electron, in Be and Mg atoms does not lie in the ­ ­ ­
visible range. sp sp 2 sp 3
l
proton, neutron and a-particle l µ . Electronegativity of carbon atom :
m 57. Buffer is a solution of weak acid and
sp>sp 2 > sp 3 .
As h is constant, greater the mass of conjugate of weak acid or a weak base
matter waves, lesser, is wavelength and a conjugate of weak base, \ Acidic character follows the order
and vice-versa. In these matter waves, e.g. NH4 Cl +NH4OH, CH ºº CH > CH2 == CH2 > CH3 — CH3
a-particle has higher mass. Therefore, CH3 COOH + CH3 COONa,NaCN +HCN. Hence, order of basic strength:
- - -
has shortest wavelength.
58. The oscillation of loose electrons in CH ºº C < CH2 == CH < CH3 — CH2
(I) (II) (III)
47. MnO -4 + 8H+ + 5e - ¾® Mn2 + + 4H2O metallic sodium is responsible for
metallic luster. 65. Slag zone is present at the middle part of
48. Mole fraction of O 2 the furnace (where, temperature is about
8 / 32 59. MnO 24 - : Oxidation number of Mn = 6
= = 0.2 800-1000°C). In this region, limestone
2 / 2 + 8 / 32 CrO 5 : Oxidation number of Cr = 6 decomposes to from CuO and CO 2 .
49. According to Arrhenius equation, rate CrO 2 Cl 2 : Oxidation number of Cr = 6 d- d+ d - d+
of a reaction increases with decrease 60. Base catalysed condensation of acid 66. CH3 CH2Mg Br + H — O CH3 ® C2H6
Methanol Ethane
in activation energy. anhydride and aromatic aldehyde is Ethyl magnesium
bromide
50. The average kinetic energy of atom is known as Perkin reaction. +CH3OMgBr
3 CHO 67. ZnS has zinc blend structure
given as KT .
2 Zn2+occupy half of tetrahedral void and
It does not depends upon mass of any
+ Ac2 O S - - occupy fcc lattice points and
atom. Benzaldehyde coordination number of Zn2+ and S - - are
OH –
4 and 4 respectively.
51. For reversible process under
isothermal conditions. 68. The technique of protecting a metal from
CH CHCOOH corrosion by connecting it to a second
W = - 2.303nRT log (p1 / p 2 )
= -2 .303 ´ 2 ´ 8.314 ´ 300 ´ metal that is most easily oxidised is
. ´ 105
101 called cathodic protection.
log Cinnamic acid
5.05 ´ 106 69. Acidic character increases with increase
4
Reaction starts with the formation of in s-character of the orbital. The
= 1.95 ´ 10 J carbanion. increasing order of s-character in
52. [H+] = CK a O O
following molecule follows the order :
Hexane < Benzene < Ethyne
= 0.5 ´ 2 ´ 10-4 = 1.0 ´ 10-2 C C sp 3 sp 2 sp
CH3 O CH3
[H ] = 10-2
+
(25%) (33.3%) (50%)
pH = - log [H+] = -log (10-2 ) = +2 OH –
Thus, the acidic behaviour follows the
53. Faraday’s law of electrolysis, O O same order.
W= Z´ i´ t 70. According to Freundlich adsorption
where, Z = electrochemical equivalent – C C x
CH2 O CH3 isotherm, µ p 1/ n
(g/C) or gm/ionic charge m
i = current (Amps) x 1/ n
Carbanion = kp
t = time (sec) m
61. 2KClO 3 —® 2KCl + 3O 2 ­
63.6 Taking log on both sides,
W= ´ 05
. ´ 60 ´ 60 245 g KClO 3 on heating shows a mass
2 ´ 96500 x 1
loss=96 g log = log k + log p
W = 056
. g m n
\ 1 g KClO 3 on heating shows a mass
On compairing with equation of straight
54. Also the common-ion effect is 96 ´ 100
loss = line,
maximum in AgNO 3 . 245 x
So, order of solubilities: y = mx + c, the graph between log
= 3918
. % m
S1 > S 3 > S 2 > S 4
62. Anhy. AlCl 3 produces fumes in the air. and log p comes out to be a straight line
55. Only (c) will produce H2 as follows : with interrupt.
It is partially hydrolysed with
Mg +H2O —® Mg (OH) 2 + H2 ­ atmospheric moisture and HCl gas is 71. From first order rate equation,
56. In comparison to other group 2 liberated. This moist HCl gas appears kt
log [A]=log [A0 ]-
elements, Be and Mg atoms are white in colour. 2.303
smaller in size. Thus, the electrons in

MODULE 2
www.jeebooks.in
132 NEET Test Drive

On compairing this equation with 84. O O


equation of straight line. Ob δ– δ+
H CH3– MgBr
k
y=mx+c and m = - H O (Excess)
CH3
H
2.303 S c
a O δ– δ+
k O
\ Slope (X ) = - O—H CH3– MgBr
2.303
Resonance hybrid
Intercept (c) = log[A] 0 OH OMgBr
All three bond lengths are equal.
72. Rate law for the reaction, H3O + H
A ®B
78. Alkyl fluorides which cannot be H3C H
CH3 H3C CH3
d[A] 2d [B ] -1 d[ A] d [B ] prepared by Finkelstein reaction is best
- = or = accomplished by Swarts reaction. In Propanol
dt dt 2 dt dt
this reaction, the corresponding
Thus, actual stoichiometry of the 85. The two most common pyrimidines of
fluoride is obtained by the action of
-d [ A] DNA are cytosine (C) and thymine (T)
reactants, = k [A] 2 mercurous fluoride or antimony
dt and the two most common purines of
trifluoride on corresponding chlorides.
73. Most stable free radical is DNA are adenine (A) and guanine (G).
The reaction is as follows:
2CH3 Cl +Hg 2F2 ¾® 2CH3F +Hg 2 Cl 2 86. Phenol is heated with pthalic anhydride
in presence of conc. H2 SO 4 to give
79. In dichromate ion, two CrO 3 units have
phenolphthalein which gives pink colour
been joined by O, i.e. each Cr is linked with alkali.
to four O-atoms. The structure of O
It is due to conjugation. dichromate ion is as follows : O
O C
74. ‘A’ gives precipitate with AgNO 3 . So, in O O C
– O
it one Cl is present outside the Cr Cr O C
O
– Pthalic
coordination sphere. ‘B’ gives O O anhydride Conc. H2SO4
C
2- O
precipitate with BaCl 2 . So, in it SO 4 is O H H Heat
present outside the coordination (Dichromate ion)
sphere.
A is [Co (NH3 ) 5 SO 4 ]Cl. 80. CuSO 4 + 4KCN ¾® K 2 [Cu(CN) 4 ] OH OH
Excess
Phenolpthalein
B is [Co (NH3 ) 5 Cl]SO 4 . +K 2 SO 4
The IUPAC name of [A] is [Cu(CN) 4 ] 2 - is stable complex having OH
Phenol
OH
pentaamminesulphato cobalt (III) k = 2 ´ 1027 . It does not provide
chloride and [B] is pentaammine sufficient Cu2+ion to give precipitate of 87. K p = KC (RT ) Dn where, Dn = [number of
chloridocobalt (III) sulphate. CuS. gaseous products - number of gaseous
reactants].
75. Mole- Total number Magnetic 81. Carbon tetrachloride is used as a
Here, Dn= zero. Hence, K p = KC (RT ) 0 .
cules of electrons behaviour feedstock in the synthesis of
chlorofluorocarbon and other 88. Aldehyde having no a-hydrogen
Cl2O7 7´2+7´6 Diamagnetic pharmaceutical manufacturing. undergoes Cannizzaro reaction in
= 14 + 42 = 56 presence of base as follows:
82. Number of moles of Cl - ions ionised
Cl2O 7 ´ 2 + 6 = 20 Diamagnetic
from one mole of CrCl 3 × 6H2O. CHO

OH
ClO 2 7 ´ 1 + 6 ´ 2 = 19 Paramagnetic 2 .665 é QMolecular mass of ù
= = 0.01 ê ú
ë CrCl 3 × 6H2O = 266.5 û
Benzaldehyde
Cl2O5 7 ´ 2 + 6 ´ 5 = 44 Diamagnetic 2665.
COOH CH2OH
76. All of the above given statements are \ Moles of AgCl obtained
+
correct. = Moles of Cl - ionised
77. Structure of bisulphate ion is 2 .87
= Here, self-oxidation as well as reduction
143.5
O O takes place.
= 0.02
89. Cellulose in a biodegradable polymer
\ 0.01 mole of complex CrCl 3 × 6H2O
S S y - which occurs in nature and degragate
y O O gives 0.02 mole of Cl on ionisation.
O O easily.
OH OH
I III Thus, the fomula of the complex is 90. CH3 CONH2 (acetamide)andCH3 CH2NH2
O
y [CrCl (H2O) 5 ] Cl 2 × H2O. (ethylamine) are distinguished by
83. (i) O -2 = 8 + 8 + 1= 17 Paramagnetic reacting with NaOH followed by heating.
The reactions is as follows :
S (ii) CN- = 8 + 7 + 1= 16 Diamagnetic
O O
O (iii) CO = 8 + 6 = 14 Diamagnetic ||
II OH + NaOH / D
(iv) NO = 7 + 8 - 1= 14 Diamagnetic CH3 CNH2 ¾¾¾¾® NH3 ­
y
Resonance structures of HSO4 NaOH / D
CH3 CH2 NH2 ¾¾¾¾® No reaction

MODULE 2
www.jeebooks.in

BIOLOGY
91. Leaves and branches falling from trees 100. J-shaped curve is called exponential protein kinases (CdK 5 ) and cyclins. The
often damage smaller plants beneath growth curve. It occurs when the cyclin-CdK complex is required for the
them. It is an example of amensalism. It resources are unlimited. This type of exit from a particular phase of the cell
is an interaction, in which one curve represents the population cycle. CdK1/cyclin B complex is required
participant is harmed but the other growing exponentially or geometrically for the cell cycle to enter the mitotic
remains unaffected. until there is a resource limitation or phase from G 2 -phase.
92. In few bryophytes, the food conducting limitation of population growth by other 107. Renal tubule begins with the Bowman’s
cells are present which are known as factors. Growth of the population capsule that encloses the glomerulus,
leptoids. Many other mosses have declines rapidly until a favourable which is a tuft of capillaries formed by
water conducting cells known as period is restored. afferent arteriole. Blood from glomerulus
hydroids. Both these cells do not 101. An enhancer is a short sequence or is carried away by efferent arteriole. The
perform their functions efficiently. So, segment of DNA that can be bound to renal tubule continues further to form
most of the water is absorbed directly activators to stimulate the transcription PCT, Henle’s loop which has a
from the surface. of a particular gene. They are usually descending and ascending limb and
referred to as transcription factors. DCT (Distal Convoluted Tubule).
93. Both the statements (a) and (b) are
Urine formation starts in glomerulus and
correct regarding eutrophication. It is 102. Spongilla is a freshwater sponge
is called ultrafiltration.
the process of natural ageing of water belonging to family Spongillidae. It
Nearly all the essential nutrients are
body. It occurs due to nutrient appears green due to the presence of
absorbed by the PCT.
enrichment of water body particularly symbiotic green algae in it.
Conditional reabsorption of Na + and
with the nitrogen and phosphorus.
103. The factors responsible for decrease in water and selective secretion occur in
94. Nepenthes is an insectivorous plant absorption of water at low temperature DCT.
thus, it does not require nitrogen from include the increased viscosity of water Urine concentration occurs in Henle’s
soil to survive. and decreased metabolic activity of loop and vasa recta by counter-current
95. Clostridium acetobutylicum is a root cells. Rate of absorption of water is mechanism.
commercially valuable bacterium that also affected when rate of root
108. The two kingdom system of classification
ferments sugar to acetone, butanol and elongation is decreased.
was discarded because of the presence
ethanol. The methodology is used for 104. Sere The entire sequence of of characters of both plants and animals
the production of biofuel blend stock. communities that successively change in Euglena. The difference of bacteria
96. Biopatents are awarded for the in a given area are called seres. and cyanobacteria from other organisms
achievements in the following areas of Seral stages The individual transitional and the non-photosynthetic nature of
biotechnology communities are termed as seral fungi also were the reasons to discard
stages or seral communities. the two kingdom system of classification.
Strains of microorganisms, cell lines,
genetically modified strains of plants Pioneer species The species that 109. The host cells are treated with a specific
and animals, DNA sequences, proteins invade a bare area are called pioneer concentration of divalent cations of
encoded by DNA sequences, etc. species. calcium to increase the efficiency with
Climax communities The sequential which DNA enters the bacterium.
97. Devonian period marks the evolution of
amphibians. Fishes arose for the first changes that finally lead to a 110. The techniques vasectomy and
time in the Ordovician period. In community that is in near equilibrium tubectomy are highly effective for birth
Pennysylvanian period, there was with the environment is called climax control. In vasectomy, a small part of the
appearance of first reptile and the most community. vas deferens is removed or tied up in
recent Holocene era is the age of man. 105. Different mechanisms are present in a males.
plant to sense gravity. The principle of In tubectomy, a small part of the
98. In dicotyledons, the cambium is
establishing and maintaining a Fallopian tube is removed or tied up in
present. Therefore, secondary growth
gravitropic set point angle depends in female.
occurs.
gravisensing and a subsequent 111. In the given options, Euglena shows
Hence, statement (d) is incorrect
cascade of events. Gravisensing can myxotrophic nutrition being both
regarding dicotyledons.
be accomplished by various means holozoic or animal-like and autotrophic,
99. The thermocycler is a laboratory including redistribution of auxin in i.e. plant-like nutrition.
apparatus most commonly used to stems and roots, pressure sensitive
amplify segments of DNA via the channels and sedimentation of
112. The three phallomeres that constitute the
Polymerase Chain Reaction (PCR). external genitalia of Periplaneta are right,
statoliths (dense plastids).
They may also be used in laboratories left and ventral phallomeres. The left
106. The cell cycle is controlled by certain phallomere consists of a curved hook
to facilitate other temperature sensitive
proteins at certain points in the cell called titillator, pseudopenis, aspirate
reactions, including restriction enzyme
cycle called the check points. These lobe and inner acculobolus.
digestion.
proteins are called cyclin-dependent

MODULE 2
www.jeebooks.in
134 NEET Test Drive

113. In protein structure, covalent bonds are Loa loa is commonly called African 126. E. coli is one of the organisms of choice
the strongest. They are of two types, eyeworm, causes swelling and itching for the production of recombinant
peptide bonds and –S–S– (disulphide) in eyes. protein. However, if the gene expression
bonds. 120. Albinism results from inheritance of is poor, it may be due to differences in
114. About 97% of oxygen is carried in recessive gene alleles. It is due to the frequency of occurrence of synonymous
combination with haemoglobin of absence or defect of tyrosinase, which codons in coding DNA (codon bias), the
erythrocytes. Thus, the maximum is a copper containing enzyme presence of termination signals or due to
transportation of O 2 in blood occurs as involved in the production of melanin. the presence of introns.
oxyhaemoglobin. Melanin is a vital pigment producing 127. The expression of housekeeping genes
compound responsible for determining is constantly required for the cellular
115. The statement (b) correctly describes a
colour of skin and hair. Its deficiency activity. These are constitutive genes
tracheid. Tracheid is a type of cell in causes several disorders, e.g. that are required for the maintenance of
the xylem. Their main function is albinism. basic cellular function and are
conduction of sap. These are
121. Heterosis or hybrid vigour or expressed in all cells of an organism
elongated or tube-like cells with thick
outbreeding enhancement, is the under normal conditions.
and lignified walls and tapering ends.
These are dead and the inner layer of improvement or increased function of 128. An operon is a unit of genetic expression
the cell walls have thickenings. any biological quality in a hybrid consisting of one or more related genes
offspring. including the operator and promoter
116. The tuberculated rhizoids are found in
An offspring exhibits heterosis if its sequences that have regulatory function.
bryophytes, e.g. Riccia and help in
anchorage and absorption. They traits are enhanced as a result of 129. Diuretic drugs like acetozolamide,
possess internal cell wall projections. mixing the genetic contributions of the mannitol, etc., are frequently used in the
They originate from the lower parents. Such offsprings with
treatment of high blood pressure as they
superficial cells of the apical meristem. increased heterozygosity are superior
increase the removal of excess fluid from
They are narrower in comparison to over their parents for various
the body.
smooth walled rhizoids. They are dead characteristics.
122. In collenchyma, the corners of cells are 130. The labellings of tapetum and vascular
at maturity and lack cytoplasm. They
deposited with cellulose and pectic strand have been interchanged. The
possess peg-like ingrowths.
substances. It is mechanical tissue structure labelled tapetum is vascular
117. In 1928, Sir Alexander Fleming which provides support, flexibility and strand and the structure labelled
extracted penicillin from the fungus vascular strand is tapetum.
elasticity to the organ.
P. notatum. The fungus, P.
chrysogenum has been used 123. Murashige and Skoog’s medium is a 131. Bulliform cells are found in monocot
plant growth medium used in leaves. They are also known as motor
industrially to produce penicillin and
laboratory for cultivation of plant cell cells and help in rolling of leaves in dry
xanthocillin X to produce the enzymes
culture. Major inorganic chemical weather. Hence, all the statements are
polyamine oxidase, phosphogluconate
components present in it are correct regarding bulliform cells.
dehydrogenase and glucose oxidase.
ammonium nitrate, calcium chloride, 132. Ephemerals or drought escapers are
118. In CAM plants, the stomata remain magnesium sulphate and potassium short-lived xerophytes, which do not
close during the day in order to reduce nitrate, etc. About 1900 mg/L of KNO 3 have the capacity to tolerate drought.
evapotranspiration. is present in the Murashige and During critical dry periods, they survive
CAM or Crassulacean Acid Skoog’s medium with sucrose being in the form of fruits and seeds that have
Metabolism, also known as CAM the major organic component of this hard seed coat and pericarp
photosynthesis, is a carbon-fixation medium. respectively. At the return of favourable
pathway that is evolved in some plants 124. Carolus Linnaeus was a Swedish conditions, these seeds germinate into
as an adaptation to arid conditions. In naturalist. He is considered as the new plants.
such plants, the stomata remain shut father of taxonomy. Species Plantarum
during the day to reduce 133. OCD is a common, chronic and long
is a famous publication of Linnaeus
evapotranspiration and open at night lasting disorder in which person has
(1753), which lists every species of
during which malic acid breaks up into uncontrollable thoughts and behaviour
plant known at that time, classified into
CO 2 and pyruvic acid and CO 2 is which he/she feels the urge to repeat
genera. It is also the first work to again and again.
released. This CO 2 is utilised in consistently apply binomial names.
C3 cycle. 134.
125. Some organisations exploit biological Evolutionary Cranial
119. Enterobius vermicularis also known as resources of other nations without stages of man capacity
pinworm causes disease enterobiasis Handy man – 700 cc
proper authorisation from the countries
in human beings.
concerned. This is known as biopiracy. Peking man – 1075 cc
Wuchereria bancrofti or filarial worm
The plants such as basmati rice, black Java ape man – 900 cc
causes elephantiasis which is
characterised by lymphatic tumours pepper, Indian mustard, etc., are Cro-Magnon man – 1360 cc
and swelling in limb. related with biopiracy. However,
135. Dioxin is a colourless organic
Bacillus anthracis is the only organism
Trichuris trichiura or whipworm resides compound. It comprises of oxygen,
in large intestine of children and which is not linked with biopiracy.
hydrogen, carbon and chlorine
causes gastrointestinal problems. elements. It is the most toxic

MODULE 2
www.jeebooks.in
Prep Catalysis for NEET ~ Mock Test 2 135

herbicide. It was used in Vietnam War 147. Gomphoses is an example of 157. Calcium is required by meristematic and
as a defoliating agent. non-movable joints. It is an attachment differentiating tissues for the development
Hence, all the statements are correct of a tooth to its socket by a fibrous of root and stem apex.
regarding dioxin. periodontal ligament. It consists of
158. Insulin – b-cells
collagen.
136. Polynoe (scaleworm) shows Glucagon – a-cells
bioluminescence because of its 148. Mycorrhiza is the symbiotic relationship Somatostatin – d-cells
brightly illuminated elytra. between fungi and roots of higher Pancreatic polypeptide – F-cells
plants. The fungal symbiont absorbs
137. Given situation can be explained as phosphorus from soil and passes it to
159. Biomagnification is the phenomenon of
temporal isolation which means build up of pesticides in the food chain.
the plant.
‘isolated in time’. This is a mechanism These pesticides get magnified in the
that prevents species from mating 149. Statement (d) is incorrect about higher trophic levels.
because they breed at different times. phenylketonuria. It is an inborn error of
Biomagnification occurs because
metabolism that results in decreased
138. The maximum volume of air a person pollutants like DDT, mercury are
metabolism of the amino acid,
can breathe in after a forced expiration non-biodegradable. They cannot be
phenylalanine.
or the maximum volume of air a person metabolised or excreted out by the
can breathe out after a forced 150. In addition to having more than one organisms consuming them.
inspiration is called vital capacity. This allele at a specific locus, each allele
160. The man has deficiency of thyroxine
includes tidal volume, inspiratory must also occur in the population at a
hormone. This hormone causes
reserve volume and expiratory reserve rate of at least 1% to generally be
development and differentiation of brain.
volume (TV + IRV + ERV). considered polymorphic.
Hypothyroidism during pregnancy
139. In zonary placenta, the villi are in the 151. Autonomic nervous system is made up causes defective development of the
form of transverse zones and penetrate of two opposing divisions, sympathetic growing baby leading to stunted growth,
the uterus wall. and parasympathetic. Sympathetic mental retardation and low intelligence
nervous system relaxes the gall quotient, etc.
140. Hyperbaric Oxygen Therapy (HOBT) is
bladder and urinary bladder and
used to treat the wound, as O2 helps in 161. Endangered species is when it is not
reduces the bile production. However,
bacterial killing, angiogenesis, critically endangered but facing a very
it has no action on adrenal glands.
fibroblast growth and epithelisation. high risk of extinction in the wild in near
152. Dicot stem – Hypodermis is future.
141. The oxides of sulphur are an important
pollutant because they cause drying collenchymatous Threatened species is one which is liable
up of margin of leaf and portion of Monocot stem – Hypodermis is to become extinct if not allowed to
lamina in between vascular bundles. It sclerenchymatous realise its full biotic potential.
also causes harm to eyes, throat and Monocot root – Pericycle gives rise Extinct species when there is no
lungs at high concentration. to lateral roots only reasonable doubt that its last individual
142. Arterio-venous anastomosis is a direct Dicot root – Pericycle gives rise has died.
vascular connection between an to lateral roots and Vulnerable species is when it is not
arteriole and venule, by passing secondary meristem critically endangered, but it is facing a
capillary supply. It is meant for 153. g-IFN (Interferon) produced by high risk of extinction in the wild in the
controlling blood supply and medium term future.
T-lymphocytes stimulates the
temperature in exposed parts.
production of antiviral proteins which 162. Pineal gland secretes three hormones,
143. The leader region of the trp operon block viral protein synthesis. i.e. melatonin, serotonin and
codes for RNA that may function as an adrenoglomerulotropin. Melatonin acts
154. Vagus nerve innervates the larynx,
attenuator. This attenuator is involved on the cells of skin, i.e. melanophores
trachea, oesophagus, stomach,
in regulating the expression of certain and its production is lowest during the
intestine, lungs and heart. Movement of
genes. It functions as a transcription daylight hours and highest at night. In
eyelids is controlled by oculomotor
terminator. non-mammalian vertebrates, it is
nerve and tongue movement is
144. Reticular connective tissue forms the controlled by hypoglossal nerve. responsible for maintaining circadian
framework of stroma of liver. rhythms. Serotonin acts as a
155. All the statements are correct vasoconstrictor and
145. Myosin filaments are constituted by regarding cymose inflorescence. In it, adrenoglomerulotropin stimulates the
monomeric proteins ‘meromyosin’ the development of flowers is basipetal zona glomerulosa of adrenal cortex to
which has two important parts– and the growth is definite, i.e. the main secrete aldosterone.
a globular head with short arm (HMM) axis is terminated by a flower. The
and a tail (LMM). opening of flowers is centrifugal. 163. During dilation stage, uterine
contractions push the foetal head lower
146. Mycoplasma is characterised by being 156. The boy is suffering from exophthalmic in the uterus and cause the cervix to
a prokaryote and lacking cell wall. It goitre or Grave’s disease. The signs of dilate.
exhibits pleomorphism as it can this diseae include hyperthyroidism,
change its shape due to the absence exophthalmos, i.e. protrusion of eyeball 164. Grafting in monocots is rarely successful
of cell wall. They do not show due to the deposition of fat behind the because they have closed vascular
crystallisation and are thus different eyes. bundles and are without cambium.
from viruses. Banana is a monocot plant.

MODULE 2
www.jeebooks.in
136 NEET Test Drive

165. Ecosystem services are the products of 170. In amphitropous ovule, the curvature of 175. GMOs developed by rDNA technology
ecosystem processes which include the ovule is more and the embryo sac and genetic engineering are useful in
healthy forest ecosystem that purifies becomes curved-like horse shoe, e.g. enhancing the nutritional value of food.
air and water. It generates fertile soil poppy. They reduce the post-harvest losses.
and maintains biodiversity. 171. Tryptophan operon is an example of They are also useful in increasing the
166. Options I, II and III show mutualism. repressible operon. In negative tolerance power of crops against the
These flowers and their pollinating repressible operons, transcription of abiotic stresses.
agents show coevolution and display the operon normally takes place. 176. Vitamin-E or tocopherol is commonly
mutualism. Blastophaga wasp is Repressor proteins are produced by a called ‘antisterility vitamin’ or ‘beauty
completely dependent upon Ficus regulator gene but they are unable to vitamin’. It removes scars and wrinkles of
carcia (fig) for survival. The fig is bind to the operator in their normal skin. Its deficiency causes infertility and
dependent upon this wasp for conformation. muscular atrophy.
pollination.The flowers of Ophrys very 177. The five kingdom system of classification
172. The option (a) is a correctly matched
closely resemble the female wasp. The is based on cell structure (prokaryotic or
pair. Virus is a nucleoprotein entity. It is
male wasp mistaking them as females eukaryotic), body structure (unicellular or
capable of multiplication by taking over
and pollinating the female flowers. multicellular), nutrition (autotrophic or
cellular machinery of the host cell.
Honeybees get nectar while pollinating heterotrophic) and lifestyle (producers,
Prions are modified hydrophobic
Salvia. consumers or decomposers).
glycoproteins. Viroids are RNA entities
167. Smoking leads to emphysema and consisting of 250-370 nucleotides. 178. Trachea is supported by 16-20
bronchitis. Smoke particles stimulate C-shaped incomplete rings made of
173. Saliva and pancreatic juice contain
the secretion of mucus in excess and hyaline cartilage.
starch splitting enzyme amylase that
also indirectly destroy the walls of lung
hydrolyses starch present in the potato 179. Plasmids are also known as extranuclear
alveoli.
and change them into maltose. The DNA. It is a small DNA molecule that can
168. The given condition is known as disaccharidases present in the replicate independently. Often, the
cleistogamy. It is an adaptation for intestinal juice acts upon maltose to genes carried in plasmids provide
self-pollination in which the flowers break it in two molecules of glucose. bacteria with genetic advantage, like
never open. The pollen grains from antibiotic resistance.
174. Bioballistic is bombarding cell with
anther lobe fall on the stigma of same
high velocity DNA coated particles, 180. Cellular totipotency was demonstrated
flower, e.g. Commelina bengalensis.
while microinjection is injecting by FC Steward. It is the ability of a single
169. Cucurbita shows reticulate multicostate recombinant DNA directly into nucleus cell to produce all cell types and to
divergent type venation. In this type of of host cell. Heat shock is placing cell organise them into an entire organism
venation, the main veins diverge incubated with DNA alternatively in when cultured in a suitable medium.
towards periphery. cold and hot environment.

SCORE CHART
No. of Correct Answers : A .................. No. of Incorrect Answers : B ..................
Total Marks : x = (A × 4) – (B × 1)
Scores and Expected Rank : If the score lies above 665, then rank will be in between 1-50. For other scores, rank
estimations are given below
Score Rank
664 – 642 51 – 150
641 – 636 151 – 250
635 – 630 251 – 400
629 – 625 401 – 500

MODULE 2
www.jeebooks.in
Prep Catalysis for NEET ~ Mock Test 3 137

PREP CATALYSIS
for NEET
Full Length Mock Tests for NEET to Make You Ready to Face the Challenge

MOCK TEST 3 (With Solutions)

Duration : 3 Hours Max. Mark : 720

General Instructions
The test is of 3 hours duration and Test Booklet contains 180 questions. Each question carries 4 marks. For
each correct response, the candidate will get 4 marks. For each incorrect response, one mark will be deducted
from the total scores. The maximum marks are 720.
The question paper contains three parts of Physics, Chemistry and Biology respectively.
Part A contains 45 Questions from Physics section. Part B contains 45 Questions from Chemistry section.
Part C contains 90 Questions from Biology.

PART A PHYSICS
1. In the following figure, four curves A, B, C and D 3. Three particles each of mass m are placed at the
are shown below. The curves are corners of an equilateral triangle of side d. If the
side of the triangle is changed from d to 2d, then
p p work done on this system is
A 3Gm2 3Gm2 Gm2 3 Gm2
(a) - (b) (c) (d) -
2d 2d 2d d
B C 4. Two circular loops P and Q are made of same wire
D and their radii are in ratio 1 : n. Their moment of
V V inertia about the axis passing through the centre
and perpendicular to their plane are in ratio 1 : m.
(a) A and C for isothermal, while B and D for adiabatic Then, m and n are related as
(b) A and C for adiabatic, while B and D for isothermal (a) m = n (b) m2 = n
(c) A and B for isothermal, while C and D for adiabatic (c) m = n2 (d) m = n3
(d) A and B for adiabatic, while C and D for isothermal
5. A step-up transformer of turn ratio of 10 has 500
2. A particle is executing SHM of amplitude 8 cm. At turns in primary winding. The primary is
what displacement from the mean position, the connected to an AC supply 20 V, 50 Hz. The
energy is half kinetic and half potential? secondary will have an output of
(a) 4 cm (b) 2 2 cm (a) 200 V, 50 Hz (b) 400 V, 100 Hz
(c) 4 2 cm (d) 8 2 cm (c) 100 V, 50 Hz (d) 150 V, 100 Hz

MODULE 2
www.jeebooks.in
138 NEET Test Drive

6. The figure below shows, the transfer 12. Light from two coherent sources of the same
characteristics of a base biased CE transistor. amplitude A and wavelength l illuminate the
Which of the following statment is true? screen. The intensity of the central maximum is I 0.
V0 If the sources are incoherent, the intensity at the
same point will be
I0
(a) 4I0 (b) 2 I0 (c) I0 (d)
2
13. A body A is thrown up vertically from the ground
with a velocity v0 and another body B is
0 Vi
0.6V 2V simultaneously dropped from a height H . They
(a) At Vi = 0.4 V, transistor is in active state H
meet at a height , if v0 is equal to
(b) At Vi = 1V, it can be used as an amplifier 2
(c) At Vi = 0.5 V, it can be used as a switch truned ON 1 2g
(a) 2gH (b) gH (c) gH (d)
(d) At Vi = 2.5 V, it can be used as a switch truned OFF 2 H
7. A wooden block floats in water, where 50% of its 14. The forbidden energy gap in Ge is 0.75 eV, given
volume is outside water. When the same block hc = 11250 eVÅ. The maximum wavelength of
floats in alcohol 70% of its volume remains outside radiation that will generate electron hole pair is
the alcohol. The relative density of alcohol is (a) 1500Å (b) 15000Å (c) 150.52Å (d) 150Å
(a) 1.78 (b) 1.50 (c) 1.66 (d) 2.1
15. Activity of a radioactive sample decreases to (1/3)rd
8. A disc of mass 40g is kept floating horizontally by of its original value in 3 days. Then, in 9 days, its
throwing 20 marbles per second against it from activity will become
below. If the mass of each marble is 4.9 g, then the (a) (1/27) of the original value (b) (1/9) of the original value
velocity with which the marbles are striking the (c) (1/18) of the original value (d) (1/3) of the original value
disc is
(assume that, the marbles strike normally and 16. A charged particle q is shot with speed v towards
rebound downward with same speed) another fixed charged particle Q. It approaches Q
(a) 9.8 ms -1 (b) 4.9 ms -1 (c) 5 ms -1 (d) 2 ms -1 upto a closest distance r and then returns. If q
9. In photo electric emission from a metallic surface, were given a speed 2v. The closest distance of
wavelength of incident light is 2 ´ 10-7 m and approach would be
stopping potential is 2.5V. The threshold frequency q V Q
r
of the metal in Hz approximately
(charge on electron, e = 1.6 ´ 10-19 C. (a) r (b) 2r (c) r / 2 (d) r / 4
Planck’s constant, h = 6.6 ´ 10- 34 J-s) 17. A test tube weighing 10 g and external diameter
(a) 12 ´ 1016 (b) 9 ´ 1015 (c) 9 ´ 1014 (d) 9 ´ 1016 2 cm is floated vertically in water by placing 10 g
of mercury at its bottom. The tube is depressed in
10. In the diagram shown below, equivalent resistance water a little and then released. Find the time of
between A and B is oscillation. (Take, g = 10 ms -2)
B (a) 0.5 s (b) 1 s (c) 5 s (d) 3 s

2Ω 2Ω 2Ω
18. Two rods of copper and aluminium of length L1 and
2Ω 2Ω
L2, respectively. The coefficient of linear expansion
A of both metal rod are a 1 and a 2, respectively. If
they are joined together and ( L1 + L2 ) is maintained
(a) 7.5 W (b) 5 W same at all temperatures, which one of the
(c) 2.5 W (d) 10 W
following relations holds good?
11. A body when projected from Earth surface in (a) a1L22 = a2 L12 (b) a12 L2 = a22 L1
vertical direction, attains a height of 18 m. If it is (c) - a1L1 = a2 L2 (d) a1L2 = a2 L1
projected with the same velocity upwards from a
19. A body of mass 4 kg is acted upon by a force which
planet having density 3 times and radius half of
causes a displacement in it given by x = t 2 m,
Earth, then will rise to where t is time in second. The work done by force
(a) 12 m (b) 10 m in 4s is
(c) 15 m (d) 20 m (a) 64 J (b) 128 J (c) 200 J (d) 240 J

MODULE 2
www.jeebooks.in
Prep Catalysis for NEET ~ Mock Test 3 139

20. Three objects coloured, black, red and white can 28. Object distance u = ( 501
. ± 0.5) cm and image
withstand hostile condition at 3200º C. These distance v = ( 201
. ± 0.2) cm, then focal length is
objects are thrown into furnace, where each of
é1 1 1 ù
ê f = u + v , where u and v are + veú
them attains a temperature of 2335º C. Which
object will glow brightest? ë û
(a) Black (b) Red (a) (12.4 ± 0.4) cm (b) (12.4 ± 01
. ) cm
(c) White (d) All equally glow (c) (14.3 ± 0.4) cm (d) (14.3 ± 01
. ) cm
21. If a radioactive nucleus decays by two different 29. If a particle of mass m and charge q is thrown at a
processes, the half-life for first process is t1 and for speed u against an uniform electric field E. The
second process it is t2. Then, the effective half life t distance that it will travel before coming to
of the nucleus is given by momentary rest is
(a) t = t 1 + t 2 (b) t = t 1 + t 2 mu qE
(a) (b)
(c) t -1 = t 1-1 + t 2-1 (d) None of these qE m
mu 2
22. A light emitting diode(LED) has a voltage drop of (c) (d) None of these
2qE
2V across it and passes a current of 10 mA. When
it operates with a 6V battery through a limiting 30. A projectile is projected at an angle of 45º with the
resistor R, the value of R is horizontal at a speed of 12 ms -1. A second object is
(a) 40 KW (b) 4 K W (c) 200W (d) 400W simultaneously released from a point on the
23. A ray falls on a prism ABC( AB = BC ) and travels vertical line along maximum height of first
as shown in the figure. The minimum refractive projectile, where they collide. The inital height of
index of the prism material should be second object is ( g = 10 ms 2 )
(a) 3.6 m (b) 7.2 m (c) 2.5 m (d) 5 m
A
31. Magnetic field of the earth is H = 3 g. A magnet is
90° vibrating 5 oscillations per minute, then the
reduction required in the magnetic field of the
earth to increase time period upto 10 oscillations
per minute is
90° (a) 2.25 g (b) 0.6 g (c) 0.9 g (d) 0.12 g
B C
32. A circular disc rolls down an inclined plane
(a) 4/3 (b) 2 (c) 1.5 (d) 3
without slipping. What fraction of its total energy
24. Work done by 3 mole of gas at 47º C to triple its is translational?
volume at constant pressure is 1 1 2 1
(a) (b) (c) (d)
(R = 2 cal mol -1 ºC -1) 2 2 3 3
(a) 3402 cal (b) 3428 cal
33. Match the symbol gates or gate circuits given in
(c) 3832 cal (d) 3840 cal
Column I to the gate given in Column II and choose
25. Escape velocity of a body from Earth is 11 km s -1. the correct option from the codes given below.
Assuming the mass and radius of Earth to be Column I Column II
about 81 times and 4 times the mass and radius of
the Moon, the escape velocity (in km/s) from the A. A 1. NOT
Y
surface of the Moon will be B
(a) 0.54 (b) 2.44 (c) 11 (d) 22
B. A 2. OR
26. A current of I ampere flows along infinitely long Y
B
conductor, then magnitude of magnetic field at any
point on the conductor is C. A 3. NOR
Y
(a) infinity (b) zero B
m I m I
(c) 0 (d) 0
4pr 2 pr D. A 4. NAND
27. If an X-rays tube operates at 18 kV, then the Y
maximum speed of electron striking the anode is
B
(a) 2 ´ 107 m/s (b) 3 ´ 108 m/s
(c) 8 ´ 107 m/s (d) 8 ´ 1010 m/s
5. AND

MODULE 2
www.jeebooks.in
140 NEET Test Drive

Codes 40. In a radiation laboratory, a experiment is


A B C D A B C D conducted with two spherical bodies X (radius 12
(a) 3 2 4 5 (b) 3 4 1 2 cm) and Y (radius 36 cm), they have temperature
(c) 5 4 3 2 (d) 5 4 1 2 T1 and T2, respectively.
34. Electron in hydrogen atom first jumps from third The maximum intensity in the emission spectrum
excited state to second excited state and then from of X is at 500 nm and in that Y is at 1500 nm.
second excited state to first excited state, then the Considering them to be black bodies, what will be
ratio of wavelengths l1 : l 2 emitted in two cases is the ratio of the rate of total energy radiated by X
(a) 27 : 5 (b) 7 : 5 (c) 20 : 7 (d) 27 : 20 to that of Y ?
-1
(a) 9.5 (b) 9
35. A bat moving at 10 ms towards a wall sends a (c) 8.5 (d) 8
sound signal of 8000 Hz towards it. On reflection,
it hears a sound of frequency f. The value of f in 41. In Young’s double slit experiment, the 5th bright
Hz is close to (speed of sound = 320 ms -1) fringe from central maxima due to wavelength l1
(a) 9500 Hz (b) 8516 Hz (c) 8000 Hz (d) 8425 Hz coincides with 8th bright fringe from central
maxima due to wavelength l 2, then possible
36. In an ammeter, 0.4% of main current passes wavelengths of visible lights are
through galvanometer. If resistance of
(a) 500 nm and 800 nm
galvanometer is R g , then shunt resistance of
ammeter will be (b) 480 nm and 400 nm
Rg Rg Rg Rg (c) 640 nm and 400 nm
(a) (b) (c) (d)
299 120 200 249 (d) 640 nm and 560 nm

37. A person P is 600 m away from the station. When, 42. A copper wire of length 2m and radius 0.1 cm is
train is approaching station with 72 km h -1, it joined in series with an iron wire of length 4 m and
blows a whistle of frequency 800 Hz, when 800 m radius 0.4 cm and a current is passed through the
away from the station. If speed of sound is wires. The ratio of the current density in iron and
340 ms -1, then the frequency heard by the person is copper wires is
800 m (a) 1 : 2 (b) 4 : 1
S (c) 1 : 16 (d) 16 : 1
v
600 m 43. Three lenses of focal length +15 cm, + 150 cm and
+250 cm are available, for making an astronomical
P telescope. To produced the largest magnification,
the focal length of the eye-piece should be
(a) 800 Hz (b) 839.5 Hz (c) 829.5 Hz (d) 843.5 Hz
(a) +15 cm
38. A capacitor is charged in such a manner that it (c) +150 cm
stores energy of 2 J. Now, potential energy is (d) +250 cm
increased to 8 J, it is done by (d) None of these
(a) double the potential difference across the capacitor
without changing the charge 44. The mass and density of the Moon, if acceleration
(b) double both the potential difference and charge due to gravity on its surface is 1.62 ms -2 and its
(c) double the charge without changing the potential difference radius is 1.74 ´ 106 m, are respectively (Take,
(d) quadruple the potential difference across the capacitor G = 6.67 ´ 10-11N - m 2 kg -2)
without changing the charge (a) 7.35 ´ 1022 kg, 3.3 ´ 103 kg m -3
39. In the given circuit (as shown in the figure) (b) 3.3 ´ 103 kg, 7.35 ´ 1022 kg m - 3
neglecting source resistance, then voltmeter and (c) 7 ´ 1022 kg, 4 ´ 103 kg m -3
ammeter reading will be
(d) 5 ´ 1022 kg, 3 ´ 103 kg m -3
V
45. Electron revolving in circular path of radius r with
R=30 Ω
speed v produced magnetic filed B at centre.
XL=25 Ω Then, correct relation among v , B and r is
A XC=25 Ω
1 1
(a) B µ µ r2 (b) B µ v 2 µ
v2 r2
1 1 1
240 V (c) B µ v µ (d) B µ µ
r2 v r2
(a) 120 V, 3A (b) 120 V, 6A (c) 0V, 8A (d) 0V, 3A

MODULE 2
www.jeebooks.in

PART B CHEMISTRY
46. Which one of the following complex is an outer 54. Which one of the following octahedral complexes
orbital complex? will not show geometrical isomerism?
(a) [Fe(CN)6 ]4- (b) [Mn(CN)6 ]4- (c) [Co(NH3 )6 ]3+ (d) [CoF6 ]3- (a) [MA4 B2 ] (b) [MA5 B]
(c) [MA2 B4 ] (d) [MA3 B3 ]
47. The product P in following reaction is
Cl
55. On heating 3, 3-dimethyl-butan-2-ol with conc.
KCN H3O+
A P H 2 SO 4 , the major product obtained is
(a) 2, 3-dimethylbutene (b) 3, 3-dimethylbutene
(a) 2-phenylpropanoic acid (b) benzoic acid (c) 2, 3-dimethylbut-2-ene (d) 3, 3-dimethylbut-2-ene
(c) 2-phenylethanoic acid (d) phenylmethanoic acid
56. Among the given lanthanoid ion, the one which is
48. Formalin is a colourless solution in water that is not paramagnetic in nature is
mostly used as a preservative. It is also used as a (a) Yb 2+ (b) Sm2+
disinfectant and an antibacterial. It is (c) Eu2+ (d) Ce 2+
(a) a solution of 40% HCHO and 60% CH3CHO
57. Which one of the following represents correct order
(b) 40% aqueous solution of HCHO
of basic strength?
(c) solution of fructose
(a) NH2OH < NH3 < N2H4 (b) NH3 < N2H4 < NH4OH
(d) None of the above
(c) NH2OH < N2H4 < NH3 (d) N2H4 < NH2OH < NH3
49. Among the given compounds, the compound which
58. Which of the following statements is not correct?
is oxidised to prepare methyl ethyl ketone is
(a) Brass is an alloy of Cu and Sn
(a) tert-butyl alcohol (b) butan-2-ol
(b) Transition elements form alloy due to their similar radii
(c) propan-2-ol (d) butanol
(c) Steel is also an alloy
50. In [Ni(CO)4 ] , [Ni(CN)4 ]2- , [NiCl4 ]2- species, the (d) German silver is an alloy of Cu, Zn and Ni
hybridisation state of Ni atom in all the complexes 59. Which one of the following oxides is most acidic in
are respectively nature?
3 2 2 3 2 3
(a) sp , dsp , dsp (b) sp , dsp , sp
(a) Bi 2O 3 (b) Sb 2O 3
(c) sp3 , sp3 dsp2 (d) dsp2 , sp3 , sp3 (c) As 2O 3 (d) P2O 5

51. Methyl alcohol when reacted with carbon monoxide 60. The radial function depends upon quantum
using cobalt or rhodium as catalyst, compound A is number n and l whereas angular functions depend
formed. On heating A with HI in the presence of upon quantum numbers l and m, i.e. these are
red phosphorus as catalyst, B is formed. independent of n. The total wave function may
The compound B is therefore, be written as
(a) CH3COOH (b) CH3CHO (c) CH3CH2I (d) CH3 × CH3 y(r , q , f ) = rnl + q l , m fn
2+ 3- (Radial part) (Angular part)
52. The complex [Ru(H 2O)6 ] and [RuCl6 ] have
nearly the same D o values because The number of radial node present in 4d-orbital is
(a) 3 (b) 2 (c) 1 (d) 0
(a) both complexes contain the same metal atom
(b) both the complexes have a symmetrical octahedral 61. Among the following substituted silicone, the one
structure which will give cross-linked silicone polymer on
(c) the presence of weaker field ligand in one is compensated hydrolysis?
by the presence of the metal in a higher oxidation state (a) R 3SiCl (b) R 4Si
(d) H2O and Cl - occupy adjacent positions in the (c) RSiCl 3 (d) R 2SiCl 2
spectrochemical series
62. The ions O2- , F - , Na + , Mg2+ and Al3 + are
53. Although +3 is the characterstic oxidation state for isoelectronic. Their ionic radii show
lanthanoid yet cerium also shows +4 oxidation (a) an increase from Na + to Al 3+ and then decreases from
state because O 2- to F -
(a) it has tendency to attain noble gas configuration (b) a decrease from Na + to Al 3+ and then increases from O 2-
(b) it has variable ionisation enthalpy to F -
(c) it resembles Pb 4+ (c) a significant increase from Al 3+ to O 2-
(d) it has tendency to attain f 7 configuration (d) a significant decrease from Al 3+ to O 2-

MODULE 2
www.jeebooks.in
142 NEET Test Drive

63. From the given sets of quantum number which is 68. Which of the following is correct regarding
not consist with the theory? following chemical reaction?
n l m s n l m s 3H 2 + N 2 d
2NH3
1 1 (a) K p > KC (b) K p < KC
(a) 1 1 1 (b) 3 0 0
2 2 (c) K p = KC (d) None of these
1 1
(c) 1 0 0 (d) 2 0 0 - 69. Which of the following is an optically active
2 2 molecule?
64. O O H
½½ ½

A+B (a) C 6H5 ¾C ¾ OH (b) CH3 ¾C (OH)C 2H5
H
In the above reaction, products A and B are ½
O (c) C 6H5 ¾CH2OH (d) C 6H5 ¾C ¾ CH3
½
(a) O O + O CH3
70. 4 moles of X and 4 moles of Y are mixed together.
O For the reaction X + Y dP + Q, 2 moles of P
O O O and Q are formed at equilibrium. The equilibrium
constant for the reaction will be
(b) + 1 1
(a) (b) (c) 1 (d) 4
4 2
O
O 71. Which of the following statement is incorrect
regarding boric acid?
(c) O + (a) Boric acid has layered structure
(b) It is prepared by BCl 3 with H2O
O O (c) It is used as mild antiseptic in washing eye
(d) It has square planar structure
O
72. From the following reactions,
(d) O + O O - +
R ON a + H 2O d ROH + NaOH
- +
65. A, B and C are ideal gases. Their molecular weight ROH + NaNH 2 d R ONa + NH3
are 2, 4 and 28 respectively. The rate of diffusion of
these gases follow the order Which of the following order is correct regarding
(a) C > A > B (b) C > B > A basic strength?
(a) OH- < NH2- < RO - (b) NH-2 < OH- < RO -
(c) A = B = C (d) A > B > C
(c) OH- < RO - < NH2- (d) RO - < NH2- < OH-
66. Hess’s law states ‘‘the enthalpy change
accompanying a chemical reaction is independent 73. The entropy values ( in JK -1 mol-1 ) of H 2 = 130.6,
of the route by which the chemical change occurs’’. Cl2( g) = 223.0 and HCl( g) = 186.7 J mol -1 K -1 at
Then, we can conclude that the Hess’s law is used 298 K and 1 atm pressure then entropy change for
for determination of the reaction,
(a) heat of reaction
H 2( g) + Cl2( g) ¾® 2HCl( g) is
(b) heat of formation
(a) +540.3 (b) -166.9
(c) heat of transformation
(c) +727.3 (d) +19.8
(d) All of the above
74. Which of the following statements is correct about
67. On dissolving moderate amount of sodium metal in resonance?
liquid NH 3 at low temperature, which of the
(a) Resonating structures are more stable than hybrid
following does not occur?
+ resonance
(a) Na ions are formed in the solution
(b) Equivalent resonating structures do not contribute equally
(b) Blue coloured solution is obtained
(c) Resonance affects bond lengths but not bond angles
(c) Liquid NH3 becomes good conductor of electricity
(d) Resonating structures indicate that the molecule has no
(d) Liquid NH3 remains diamagnetic
real existence

MODULE 2
www.jeebooks.in
Prep Catalysis for NEET ~ Mock Test 3 143

75. CH ºº CH ¾ O¾
3 NaOH
¾¾ ¾® X ¾ Zn
¾CH COOH
¾3¾ ¾¾® Y , Y is 84. The process of gangue removal or increase in
(a) CH2OH (b) CH3CH2OH concentration of ore depends upon the nature of
½ ore and the impurities present in it. Froth
CH2OH floatation process is an important method for the
concentration of the ore. Which of the following
(c) CH3COOH (d) CH3OH
ores involves the froth floatation method during
76. It was found that a unique set of conditions was extraction of metal?
responsible for ozone hole. In summer season, NO2 (a) Chromite (b) Zincite (c) Bauxite (d) Iron pyrite
and CH 4 react with chlorine monoxide and chlorine 85. Most of the organic compounds are generally used
atoms forming chlorine sinks, prevents much ozone in the formation of drugs. 1.1% solution of phenol
depletion. Which of the following gas is harmful to is used as
ozone layer?
(a) antiseptic (b) antipyretic (c) disinfectant (d) tranquilizer
(a) CFC (b) MIC (c) Phosgene (d) CO 2
86. The rate of reaction A ® B quadruples on
77. The C¾ C bond length of the following molecules
is in the order increasing the concentration of A by 8 times, thus
the order of the reaction is
(a) C 2H6 > C 2H4 > C 6H6 > C 2H2
(a) 4 (b) 1 2 (c) 2 (d) 1
(b) C 2H2 < C 2H4 < C 6H6 < C 2H6
(c) C 6H6 > C 2H2 > C 2H6 > C 2H4 87. Proteins when heated with conc. HNO3 gives a
(d) C 2H4 > C 2H6 > C 2H2 > C 6H6 yellow colour. This test used in identification of
78. Diamond has a fcc structure with two atoms per protein is known as
lattice point and a unit cell of edge length 3.569 Å. (a) xanthoprotic test (b) biuret’s test
What will be the density of diamond? (c) acid base test (d) hoppe’s test
(a) 3.506 g cm-3 (b) 3 .0 g cm-3 88. NH2 N2+Cl–
. g cm-3
(c) 175 (d) 1 .5 g cm-3
A CuCN
B
79. Normality and molarity changes with temperature ∆
because they involve volumes. The normality of
0.3 M phosphoric acid, H3PO4 is A and B respectively are
(a) 0.9 N (b) 0.1 N (a) phenol and methyl nitrile
(c) 0.3 N (d) 0.6 N (b) HNO 3 and benzonitrile
(c) NaNO 2 HCl and benzonitrile
80. The equivalent conductance of NaCl at (d) HNO 2 H+ and methyl cyanide
concentration C and at infinite dilution are lC and
l ¥ respectively. The correct relationship between 89. Which of the following method can be used to
lC and l ¥ is given as synthesise fluorobenzene in the laboratory?
(a) lC = l ¥ - B C (b) lC = l ¥ - (B)C (a) Heating phenol with HF and KF
(c) lC = l ¥ + (B)C (d) lC = l ¥ + B C (b) Reacting bromobenzene with NaF solution
(c) Diazotisation of aniline followed by heating the diazonium
81. For a reaction 4 A ¾® 5B, if the rate of formation salt with HBF 4
of B is y mol/L, then the rate of consumption of A (d) Direct flourination of benzene with F2 gas
will be
(a)
4
y (b) 5y (c) y (d)
5
y
90. Aniline when diazotised in cold and then treated
5 4 with dimethylaniline gives a coloured compound,
the product is
82. Molality of aqueous solution of 8.0 M ethanol
having density 1.025 g/mL is (a) CH3—NH N N NHCH3
(a) 12.17m (b) 24.34m (c) 10.17m (d) 14.35m
83. Consider the following reactions,
(b) CH3 N N NH2
A ® X , kA = 1018 e- 8000 T
B ® X , kB = 1017 e- 4000 T
(c) (CH3)2N N N
Temperature (T K) at which kA = kB is
(a) 4000 K (b) 8000 K
4000 8000 (d) (CH3)2N NH
(c) K (d) K
2.303 2 .303

MODULE 2
www.jeebooks.in

PART C BIOLOGY
91. The chloroplast thylakoids are in the form of 99. Which among the following statements are correct
(a) interconnected tubules (b) interconnected sacs regarding ETC?
(c) independent discs (d) stacked discs I. Cytochrome-a3 donates the electrons to free
molecular oxygen.
92. The water column in xylem vessels or tracheids
does not undergo rupture or fractionation during II. Oxygen acts as the last electron acceptor at
ascent of sap. This is possible due to the end of respiratory chain.
(a) lignified thick walls of vessels III. Oxygen also receives two protons directly
(b) transpirational pull from the aqueous medium to form one
(c) cohesion and adhesion forces molecule of water.
(d) imbibition pressure IV. NADH and FADH bring electrons to the
93. Sclerenchymatous fibres are electron transport system.
(a) lignified (b) dead Choose the correct option.
(c) found in pericycle of stem (d) All of these (a) I and II (b) II and III (c) III and IV (d) All of these

94. A pendulus spike bearing unisexual flowers is 100. Which of the following is not an organ where
(a) umbel (b) catkin lymphocytes originate, mature or multiply?
(c) spadix (d) corymb (a) Spleen (b) Bone marrow
(c) Thymus (d) Pancreas
95. Physiological barriers include 101. Apospory is
I. Tears from the eyes II. Saliva in the mouth (a) development of sporophyte without fusion of male and
III. Acid in the stomach IV. Interferons female gametes
Choose the correct option. (b) development of gametophyte from any cell of sporophyte
(a) I and II (b) I, II and III other than spores
(c) II, III and IV (d) I and III (c) Both (a) and (b)
(d) None of the above
96. Pseudometamerism refers to
I. segmentation that simultaneously divides 102. Ommatidium is found in
body both externally and internally. (a) Porifera (b) Coelenterata (c) Annelida (d) Arthropoda

II. the body is divisible into parts or segments 103. In humans, cartilage bone forms by the
called proglottid. deposition of
III. proglottids develop from the neck but are not (a) bony matter by osteoclasts
embryonic in origin. (b) resorption of bone matter by osteoblasts
(c) deposition of bony matter by osteoblasts and resorption by
IV. repetition which appears due to the repeated
chondroclasts
budding.
(d) deposition of bony matter by chondrocytes
Choose the correct option.
(a) I and II (b) II, III and IV 104. The common ancestor of man and ape is
(c) III and IV (d) I, III and IV (a) Dryopithecus
(b) Australopithecus
97. The source of complementary RNA in RNA (c) Hylobates hoolock (gibbon)
interference is (d) Pongo pigmaeous (orangutan)
(a) infection by viruses having RNA genomes
(b) transposons that replicate via an RNA intermediate 105. What is a DNA molecule that is able to replicate
(c) Both (a) and (b) autonomously called?
(d) None of the above (a) Genome (b) Chromosome
(c) Replicon (d) Plasmid
98. The action of an inhibitor which diminishes the
catalysis via reverse binding at the active site of 106. A person with asthma has difficulty in breathing
enzyme is best described as because of the constriction of
(a) competitive (b) allosteric (a) bronchi (b) alveoli
(c) facultative (d) non-competitive (c) pharynx (d) terminal bronchioles

MODULE 2
www.jeebooks.in
Prep Catalysis for NEET ~ Mock Test 3 145

107. Filamentous thalli, hyphae generally non-septate Identify the phylum in which above characteristic
and multinucleate are vegetative features of features are included
(a) Zygomycetes – Rhizopus (a) Chordata (b) Non-Chordata
(b) Basidiomycetes – Chlamydomonas (c) Porifera (d) Hemichordata
(c) Ascomycetes – Neurospora
115. Which statement is incorrect regarding
(d) Deuteromycetes – Mycorrhiza
fermentation?
108. The flowers show trimerous symmetry in I. It is anaerobic respiration which does not
(a) Dicotyledonae (b) Monocotyledonae require oxygen.
(c) Cryptogamae (d) Phanerogamae II. It generally utilises NADH produced during
109. ‘Doom’ is a mixture of glycolysis.
I. Bacillus popilliae III. It does not produce ATP.
II. Bacillus lentimorbus IV. It produces pyruvic acid.
III. Bacillus thuringiensis Choose the correct option.
IV. Hirrutella thompsoni (a) I and IV (b) II, III
Choose the correct option. (c) Only IV (d) Only II
(a) I and III (b) I and II (c) III and IV (d) I and IV
116. Identify the incorrectly matched pair.
110. The fallen stars is the name given to (a) Codominance — ABO blood grouping in humans
(a) Oscillatoria (b) Nostoc (b) Pleiotropy — Phenylketonuria
(c) Zygnema (d) Cladophora (c) Polygenic inheritance — Human skin colour
(d) Incomplete dominance — Coat colour in mice
111. Families in plant kingdoms are characterised
based on which of the following features? 117. Adiantum caudatum is also called walking fern
(a) Seasonal similarities and variations because
(b) Anatomy of various parts (a) it’s leaves are in the shape of limbs
(c) External morphology (b) new plantlets grow wherever leaves touch the ground
(d) Vegetative and reproductive parts (c) the petiole is shaped like a leg
112. GIFT is a technique to overcome the problem of (d) the plant shifts from one place to another sometimes
infertility in females who 118. Given below are statements regarding viruses.
(a) cannot produce an ovum Identify the correct statement.
(b) cannot provide a suitable environment for fertilisation (a) All viruses contain both RNA and DNA
(c) cannot maintain a viable foetus (b) They are obligate parasites
(d) have two narrow cervical canal for sperm passage (c) Viruses possess their own metabolic system
113. Read the following statements. (d) Viral nucleic acid is called as capsid
I. Some species acquire heat from sun when the 119. Manganese becomes toxic when absorbed by plants
body temperature drops below the comfort in higher amounts. It is due to
zone. (a) reduction in uptake of Fe 3 + and Mg 2 +
II. Some species move into shade when the (b) inhibition of binding of Mg 2 + to specific enzymes
ambient temperature starts increasing. (c) inhibition of Ca 2 + translocation in shoot apex
III. Some species burrow into the soil and escape (d) All of the above
from the above ground heat. 120. One of the following statements is incorrect with
The above statements depict the adaptations in reference to mycoplasma. Identify it.
(a) kangaroo rat (b) desert lizards (a) Absence of cell wall
(c) mammals (d) humans (b) Covered with three layered plasma membrane
(c) 70S ribosomes are present
114. Given below are some characteristic features. (d) They are Gram positive
I. A ventral heart and closed circulatory system
is present. 121. Which of the following is the correct sequence of
II. Haemoglobin is present in red blood electron carriers between PS-II to PS-I in
corpuscles. photosynthesis?
(a) PQ, cyt-b 6 , cyt-f, PC (b) cyt-b 6 , cyt-f, PC, PQ
III. Digestive system is well-developed
(c) PC, cyt-b 6 , cyt-f, PQ (d) cyt-f, cyt-b 6 , PQ, PC
IV. Hepatic portal system is present.

MODULE 2
www.jeebooks.in
146 NEET Test Drive

122. Which option shows the correctly matched The correct option with matched pairs is
hormone with its source and function? (a) I and II (b) II and III
(c) I, II and III (d) I and III
Hormone Source Function
(a) MSH Pars intermedia Darkening of skin 130. Match Column I with Column II and choose the
correct option from the codes given below.
(b) STH Pars nervosa Micturition
Column I Column II
(c) ACTH Pars tuberalis Control’s blood
(Gene transfer) (Characteristics)
glucose level
(d) FSH Pars distalis Cushing’s disease A. Plant tumours 1. Delivering large genes into
bacterial cells
123. Which among the following is an example of root B. Bacteriophages 2. Delivering genes to living
climbers? patients in gene therapy
(a) Pothos (b) Cuscuta
C. Adenoviruses 3. Foreign genes are replicated
(c) Linospora (d) Bauhinia
into every daughter cell
124. Find the odd one out in the following list. D. Retroviruses 4. Usage of Agrobacterium
I. Pila II. Aplypsia tumefaciens
III. Unio IV. Octopus
Codes
V. Doris VI. Limax
A B C D A B C D
Choose the correct option. (a) 1 2 3 4 (b) 4 3 2 1
(a) Only I (b) I and VI (c) III and IV (d) Only IV (c) 4 1 2 3 (d) 1 2 4 3
125. In non-cyclic photophosphorylation, which of the 131. Which of the following statements are not true for
following electron carriers is a copper containing the synergids?
compound? I. They are generally two in number and occur
(a) Ferredoxin (b) Cytochrome-a3 laterally to egg cell.
(c) Plastocyanin (d) Plastoquinone II. These are characterised by the presence of
126. Identify the incorrectly matched pair regarding finger-like filiform apparatus towards
the development of plasmid vectors in following chalazal pole.
three phases. III. They secrete chemotactic substances to keep
(a) First phase → Poor replication ability the egg cell safe.
(b) Second phase → With unsuitable selectable markers IV. They have polarised protoplasm where nuclei
(c) Third phase → Incorporation of high expression promoters occur towards the micropylar pole and
for expression of large amounts of foreign proteins vacuoles towards the central cell.
(d) None of the above V. Synergids degenerate before the entry of the
pollen tube into the embryo sac.
127. Which among the following statements is/are
correct regarding active absorption? Choose the correct option.
(a) ATP is required (a) II and III (b) II, IV and V
(b) Rate of respiration increases during absorption (c) I, IV and V (d) All of these
(c) Both (a) and (b) 132. Tubular reabsorption involves
(d) None of the above
I. absorption of water and useful solutes from
128. Which among the following statements is/are the glomerular filtrate into blood.
correct regarding dicot leaf ? II. It occurs mostly in PCT.
(a) Leaves are dorsiventral III. It takes place only by active transport.
(b) Bulliform cells are absent
IV. It occurs mostly in DCT.
(c) Vascular bundle is surrounded by parenchymatous bundle
sheath The correct statements are
(d) All of the above (a) I and II (b) II and III (c) III and IV (d) I and IV

129. Identify the correctly matched pairs. 133. The top floor of the building caught fire due to
short circuit. The scared residents of that floor
I. RNA polymerase I → Transcribes rRNAs immediately started running downstairs. Which
II. RNA polymerase II → Transcribes precursor hormone instigated this action?
of mRNA (a) Thyroxine (b) Calcitonin
III. RNA polymerase III → Transcribes tRNA (c) Somatotropin (d) Adrenaline
MODULE 2
www.jeebooks.in
Prep Catalysis for NEET ~ Mock Test 3 147

134. Match Column I with Column II and choose the II. It provides the site for binding of RNA
correct option from the codes given below. polymerase.
Column I Column II III. The template and coding strands of DNA are
(Terms used in ecology) (Description) defined by the promoter site.
A. Community 1. All organisms that live in IV. The site is A and T rich region called TATA
a place together with their BOX (in prokaryotes) and Pribnow Box
physical environment (in eukaryotes).
B. Ecosystem 2. group of ecosystems that Choose the correct option.
share similar climates (a) I and II (b) III and IV
(c) I, III and IV (d) All of these
C. Biome 3. entire planet with all its
organisms and physical 138. Identify the incorrectly matched pair.
environments (a) Pink pages — Critically endangered species
(b) Green pages — Species which are no longer threatened
D. Biosphere 4. assemblage of different
(c) Both (a) and (b)
populations
(d) None of the above
Codes
A B C D A B C D 139. Arrange the following in sequence of steps in plant
(a) 1 2 3 4 (b) 1 4 2 3
breeding.
(c) 1 4 3 2 (d) 4 1 2 3 I. Collection of variability.
II. Cross hybridisation among selected parents.
135. Given below is the diagrammatic presentation of a
male reproductive tract. III. Evaluation and selection of parents.
IV. Selection and testing of superior
recombinants.
V. Testing, release and commercialisation of new
cultivars.
X
The correct sequence of steps in plant breeding is
as follows.
Z (a) I, II, III, IV, V (b) I, III, II, IV, V
(c) I, II, IV, V, III (d) I, II, III, V, IV

Y 140. E. histolytica, E. coli and E. gingivalis are a


group of
(a) symbiotic bacteria present in the intestine of most
primates
Which of the following correctly identifies part X, Y
(b) protozoan parasites found in gastrointestinal tract
and Z along with their functions?
(c) bacterial parasites that can be dealt with immunisation at
(a) X – Prostate gland, Function – Secrete testosterone
proper time
hormone
(d) None of the above
(b) X – Urethra, Function – Outlet for urine and semen
(c) Y – Testis, Function – Primary male reproductive organs 141. Which among the following statements is incorrect
(d) Z – Seminal vesicle, Function – Secretes sperms regarding a food web?
(a) It consists of number of interconnected food chains
136. Identify the incorrectly matched pair in reference
(b) It is never straight
to sewage treatment.
(c) A number of alternate sources of food are present
(a) Primary treatment → Physical removal of particles
(d) None of the above
(b) Secondary treatment → BOD is reduced
(c) Flocs → Masses of bacteria associated with fungal 142. Identify the correct plant growth regulator(s) for
filaments the formation of the corresponding plant tissue?
(d) Greater BOD → Less is the polluting potential
Plant Growth Regulator Plant Tissue
137. Which among the following statements is correct in (a) Auxin Shoots
reference to promoter of transcription unit?
(b) Auxin and cytokinin Callus
I. It is located towards 5′ end (upstream) of the
(c) Abscisic acid Stem elongation
structural gene.
(d) Cytokinin Somatic embryo

MODULE 2
www.jeebooks.in
148 NEET Test Drive

143. Which among the following is/are correctly (c) smooth muscles and cardiac muscles are more active than
matched regarding age pyramid? skeletal muscles
(a) Young population pyramid — Pyramid with broad base (d) Both (b) and (c) explain it correctly
(b) Stable population — Bell-shaped pyramid 151. Eukaryotic mature mRNAs have a 5′ cap that is
(c) Declining population — Urn-shaped pyramid residue of
(d) All of the above (a) adenylic acid (b) adenine
144. Identify the incorrectly matched pair. (c) 3-methyl guanosine (d) 7- methyl guanosine
(a) Biopiracy — Exploit biological resources of other nations 152. Which one of the following statements is correct for
(b) Biopatent — Right granted by the government to an Okazaki fragments ?
inventor to prevent others to make commercial use of (a) They are DNA - RNA hybrids
one’s invention (b) They are double-stranded
(c) Both (a) and (b) (c) They are formed due to nicking of the sugar-phosphate
(d) None of the above back bone of parental strand
145. The functions of endosperm include (d) They contain covalently linked RNA and DNA
(a) Forms sufficient food for the developing embryo 153. The properties of genetic material include
(b) Stores carbohydrates, fats, proteins, etc (a) it should be able to replicate itself
(c) Its contents are necessary for the growth of embryo (b) it should be stable both chemically and structurally
(d) All of the above (c) it should be able to express itself in the form of ‘Mendelian
146. The secondary immune response is not characters’
characterised by (d) All of the above
(a) more rapid 154. Lethal genes are
(b) lasts for longer periods (a) responsible for reappearance of ancestral traits
(c) occurs at the second and subsequent exposure of the (b) always recessive
same host to same antigen
(c) present on separate chromosomes and influence the same
(d) takes longer time to establish immunity
trait
147. Which among the following statements is/are (d) lethal and cause death of the organism in homozygous
correct regarding phagemids? state
(a) They give high yield of foreign DNA 155. Gauri visited her village with her parents for the
(b) They can provide single-stranded DNA first time. She was surprised to see many infants
(c) Due to their small size single-stranded DNA upto 10 kbp in her village who were very weak with stunted
can be easily obtained from them growth, slender legs and protruding belly. Which
(d) All of the above one of the following diseases are they likely to be
suffering from?
148. During expiration, (a) Kwashiorkor (b) Marasmus
I. external intercostal muscles relax. (c) Pellagra (d) Xerophthalmia
II. diaphragm bends upwards.
III. diaphragm straightens.
156. Identify the incorrectly matched pair.
(a) Dicot root — Pith absent or poorly developed
IV. external intercostal muscles contract.
(b) Monocot root — Cambium absent
The correct statements are (c) Both (a) and (b)
(a) I and III (b) I and II (d) None of the above
(c) III and IV (d) I and IV
157. Read the following statements.
149. Resource partitioning is
I. These muscle fibres are supplied with both
(a) avoidance of competition
central and autonomic nervous system.
(b) realised niches divide resources among several species
(c) Both (a) and (b)
II. They are not under the will of the animal.
(d) None of the above III. These fibres never get fatigue.

150. The reason why the skeletal muscles are syncytial In reference to above statements, which of the
while smooth and cardiac muscles are not, is following muscles possess the above
(a) skeletal muscle is made up of a number of muscle bundles characteristics?
held together (a) Smooth muscles (b) Cardiac muscles
(b) quantity of myoglobin is high (c) Striated muscles (d) None of these

MODULE 2
www.jeebooks.in
Prep Catalysis for NEET ~ Mock Test 3 149

158. Which part acts as a highway for the relay of many 166. Invertebrates have a bluish blood because of
signals to and from the cerebrum and cerebellum? respiratory pigment
(a) Pons Varolii (b) Medulla oblongata (a) haemoglobin (b) haemocyanin
(c) Cerebellum (d) Corpora quadrigemina (c) cytochrome (d) chlorocruorin
159. The four kingdom system of classification consists 167. In dorsiventral leaf
of (a) guard cells are kidney-shaped
(a) Monera, Protista, Plantae, Animalia (b) mesophyll is differentiated into palisade and spongy
(b) Monera, Protista, Fungi, Plantae parenchyma
(c) Monera, Protista, Fungi, Animalia (c) bundle sheath cells are colourless
(d) Monera, Protista, Fungi, Virus (d) bulliform cells (motor cells) absent
160. Arrange the following in reference to pollen 168. It two genes are located very close to each other on
development in flowering plants. the same chromosome
I. Microspore mother cell develops within pollen (a) no linkage is there
sac. (b) crossing over increases
II. Each microspore undergoes mitosis to form (c) occurrence of crossing over is very rare
immature pollen grain. (d) they merge to form a larger gene
III. Microspore mother cell undergoes meiosis to 169. The bulk of the semen is contributed by
form four haploid microspores. (a) prostate gland secretion (b) seminal vesicle fluid
IV. Generative cell undergoes mitosis to form two (c) bulbourethral gland’s fluid (d) Cowper’s gland secretion
sperm cells.
170. Which among the following statements is/are
Choose the correct option. correct regarding Protista?
(a) I → II → III → IV (b) I → III → II → IV
(a) Unicellular eukaryotes
(c) III → I → II → IV (d) IV → I → II → III
(b) Possess double nuclear membrane
161. Identify the incorrect statement. (c) They are the smallest free-living microorganisms
(a) In Arthropoda, jointed appendages help in locomotion (d) Both (a) and (b)
(b) Organs of Bojanus are present in Mollusca 171. Intermediate host is
(c) Ambulacral system is present in Echinodermata I. in which asexual reproduction occurs.
(d) Roundworm is an example of Platyhelminthes II. in which the immature form harbors.
162. Identify the incorrectly matched pair. III. in which sexual reproduction occurs.
(a) Siliqua → Capsella IV. which harbors the mature form.
(b) Silicula → Iberis The option with correct statements is
(c) Capsule → Cotton (a) I and II (b) III and IV (c) III and II (d) I and IV
(d) Follicle → Calotropis
172. Which among the following statements is/are
163. Which statement is incorrect regarding lac operon? correct regarding bryophytes?
(a) Lactose is known to be the inducer (a) They are non-vascular cryptogams
(b) z-gene codes for β - galactosidase (b) Sporophyte is attached to gametophyte
(c) y-gene codes for permease (c) Gametophyte is the dominant phase in life cycle
(d) a-gene codes for the repressor (d) All of the above
164. What may be caused as a result of a marriage 173. Match the items given in Column I with these in
between close relatives? Column II and select the correct option.
(a) Erythroblastosis foetalis
Column I Column II
(b) Mutations (Plant hormones) (Action)
(c) More recessive alleles will come together
(d) Incomplete dominance A. IAA 1. Stomatal closure
B. Gibberellins 2. Stem elongation
165. Which of the following is correctly matched pair?
C. Cytokinins 3. Cell wall elongation
(a) Leptotene — Homologous chromosomes get associated
as bivalents D. Abscisic acid 4. Cell division
(b) Zygotene — Chromatin begins to get compacted showing
Codes
fine threads
A B C D A B C D
(c) Pachytene — Homologous chromosomes come in pairs
(a) 3 2 4 1 (b) 1 2 3 4
(d) Diplotene — Chiasma formation and separation of
(c) 4 3 2 1 (d) 2 4 1 3
chromosomes

MODULE 2
www.jeebooks.in
150 NEET Test Drive

174. Identify the incorrect pair. Choose the correct option.


(a) G 0 –Non-dividing (a) I and II (b) II and IV
(b) G1 –Growth and differentiation (c) I and III (d) None of these
(c) G 2 –Chromosomes are replicated
178. Match Column I with Column II and choose the
(d) S–Synthesis of DNA
correct option from the codes given below.
175. Which of the following male sex organs are also Column I Column II
known as bulbouretrhal glands?
(Modifications of stem) (Characteristic features)
(a) Prostate glands (b) Pineal glands
(c) Bartholin glands (d) Cowper’s glands A. Prickles 1. Bear several nodes and
internodes
176. Match Column I with Column II and choose the B. Phylloclade 2. One internode long
correct option from the codes given below.
C. Cladodes 3. Help in vegetative propagation
Column I Column II
D. Bulbils 4. Act as climbing organs
(Type of pollutant) (Description)
A. Primary pollutant 1. Present in environment as Codes
it is produced A B C D A B C D
B. Secondary pollutant 2. Formed from primary (a) 4 1 2 3 (b) 1 2 3 4
(c) 4 1 3 2 (d) 1 4 3 2
pollutant
C. Qualitative pollutant 3. Produced by 179. Which statement(s) is/are correctly related to
anthropogenic activities
in situ conservation?
(a) It involves protection of species threatened with extinction
D. Quantitative pollutant 4. Concentration reaches
(b) Flora and fauna exist, without human interference
more than threshold value (c) It involves protection of species outside their natural homes
Codes (d) Both statements (a) and (b)
A B C D A B C D 180. Which assumptions should be met in
(a) 1 2 3 4 (b) 1 2 4 3 Hardy-Weinberg equilibrium?
(c) 2 1 3 4 (d) 2 1 4 3 (a) The population size is very large
(b) Individuals pair by chance, not according to their
177. Deuterostomes are the ones in which
genotypes or phenotypes
I. anus develops first. II. mouth develops later. (c) no gene flow
III. mouth develops first. IV. anus develops later. (d) All of the above

Answer Sheet
1. (a) 2. (c) 3. (b) 4. (d) 5. (a) 6. (b) 7. (c) 8. (d) 9. (c) 10. (c)
11. (a) 12. (d) 13. (b) 14. (b) 15. (a) 16. (d) 17. (a) 18. (c) 19. (b) 20. (a)
21. (c) 22. (d) 23. (b) 24. (d) 25. (b) 26. (b) 27. (c) 28. (c) 29. (c) 30. (b)
31. (a) 32. (c) 33. (b) 34. (a) 35. (b) 36. (d) 37. (b) 38. (b) 39. (c) 40. (b)
41. (c) 42. (c) 43. (a) 44. (a) 45. (c) 46. (d) 47. (c) 48. (b) 49. (b) 50. (b)
51. (d) 52. (d) 53. (a) 54. (b) 55. (c) 56. (a) 57. (c) 58. (a) 59. (d) 60. (c)
61. (c) 62. (c) 63. (a) 64. (b) 65. (d) 66. (d) 67. (d) 68. (b) 69. (b) 70. (c)
71. (d) 72. (c) 73. (d) 74. (c) 75. (a) 76. (a) 77. (b) 78. (a) 79. (d) 80. (a)
81. (a) 82. (a) 83. (c) 84. (d) 85. (c) 86. (b) 87. (a) 88. (c) 89. (c) 90. (c)
91. (d) 92. (c) 93. (d) 94. (b) 95. (b) 96. (b) 97. (c) 98. (a) 99. (d) 100. (d)

101. (b) 102. (d) 103. (c) 104. (a) 105. (c) 106. (d) 107. (a) 108. (b) 109. (b) 110. (b)
111. (d) 112. (a) 113. (b) 114. (a) 115. (c) 116. (d) 117. (b) 118. (b) 119. (d) 120. (d)
121. (a) 122. (a) 123. (a) 124. (c) 125. (c) 126. (b) 127. (c) 128. (d) 129. (c) 130. (c)
131. (a) 132. (a) 133. (d) 134. (d) 135. (c) 136. (d) 137. (d) 138. (d) 139. (b) 140. (b)
141. (d) 142. (b) 143. (d) 144. (d) 145. (d) 146. (d) 147. (d) 148. (b) 149. (c) 150. (a)
151. (d) 152. (d) 153. (d) 154. (d) 155. (a) 156. (d) 157. (b) 158. (a) 159. (a) 160. (b)
161. (d) 162. (a) 163. (d) 164. (c) 165. (d) 166. (b) 167. (a) 168. (c) 169. (b) 170. (d)
171. (a) 172. (d) 173. (a) 174. (c) 175. (d) 176. (a) 177. (a) 178. (a) 179. (d) 180. (d)

MODULE 2
www.jeebooks.in

EXPLANATIONS

PHYSICS
1. Amount of work done in isothermal 6. The transfer characteristics are as 11. As kinetic energy is same in both cases,
cycle is higher than in the adiabatic when transistor is used in cut-off or so potential energy will also be same, i.e.
cycle. Hence, the curves are isothermal saturation state, it acts as switch. But if g
mg php = mgehe Þ hp = e he
for A and C, while adiabatic for B and D. it is used in its active region it acts as gp
2. According to question, KE = PE an amplifier.
GM G 4
Cut-off Active Saturation Also, g = = 2 ´ pR 3 r
1 1 V0 R2 3
mv 2 = mw 2 y 2 region region region R
2 2 4
1 1 = p GR r
Þ mw 2 (A 2 - y 2 ) = mw 2 y 2 3
2 2 ge Re re 2
\ = ´ =
Þ A 2 - y 2 = y 2 or 2 y 2 = A 2 gp Rp r p 3
A 8 Vi
Þ y = = = 4 2 cm 2
2 2 \ hp = ´ 18 = 12 m
7. We know that, v s g = (05
. v )sw g 3
3. For two particle system potential 12. Let intensity of each source is I, then
[for water]
energy is
and v s g = (0.3v )s ag [for alcohol] intensity at central maxima =
Gm1m 2
U =- 5 s 4I = I 0 (given)
r 1= × w
3 sa I
C So, I = 0
m 4
sa 5
So, = = 166
.
60° sw 3 When sources are incoherant, intensity
d at any point on screen
8. Let velocity of each marble = v
= I + I = 2I = 2 æç 0 ö÷ = 0 .
I I
Change in momentum per second è4ø 2
= 20[mv - (-mv )] = 40 mv 1 2
Am mB 13. For body A, s = ut + at
\ Force exerted by marble on the disc 2
So, U a = U AB + UBC + UCA = 40 mv H 1 2
Þ = vot - gt
Gmm Gm 2 To keep the disc, floating, the force 2 2
= -3 = -3
d d must balances the weight of disc, i.e. H 1 2
For body B, = 0 + gt
3Gm 2 40 mv = Mg 2 2
When d = 2d Þ Ub = - 1 1
2d 40 ´ 4.9 ´ 10-3 ´ v = 40 ´ 10-3 ´ 9.8 Þ vot - gt 2 = gt 2
3Gm 2 9.8 2 2
Work done = Ub - U a = Þ v = = 2 ms -1 v0
2d 4.9 Þ t =
g
4. The masses of two loops are 9. By Einstein’s photoelectric equation,
H v 1 v2
MP = 2pmR æ1 1ö \ = v 0 ´ 0 - g 02
hc ç - ÷ = eV0 2 g 2 g
and MQ = 2pnmR è l l0 ø
H v 02 v 02 v2
where, m = mass per unit length of wire. Þ = - = 0
h æç - n 0 ö÷ = eV0
c
2 g 2g 2g
Their moments of inertia are èl ø
IP = MP × RP2 = 2 pmR ´ R 2 = 2 pmR 3 1 é hc Þ v 0 = gH
Þ n0 = - eV0 ù
and IQ = MQ × RQ2 = 2pnmR ´ n 2R 2 h êë l úû 14. Energy gap, E g =
hc 11250
= = 15000 Å
l 075
.
= 2 pn 3mR 3 1 é 6.6 ´ 10-34 ´ 3 ´ 108
IQ IQ = -34 ê 15. R = R 0e - lt
\ 3
= n but = m (given) 6.6 ´ 10 ë 2 ´ 10-7
IP IP
Þ æ 1 ö = e - l ´ 3 = e -3 l
ù ç ÷ …(i)
Þ m = n3 . ´ 10-19 ´ 2.5 ú
-16 è3ø
û

5. For a transformer,
Ns E
= s = 9 ´ 1014 Hz Again = e - l ´ 9 = e -9 l = (e -3 l ) 3
Np Ep R0
10. Given, resistance is in a parallel 3
= æç ö÷
é Q Ns = 10 ù combination 1
5000 E s [From Eq. (i)]
Þ = ê N ú 1 1 1 1 1 1 5 è3ø
500 20 ê p
ú So, = + + + + =
ë Þ N = 10 N pû R AB 2 2 2 2 2 2 1
s =
Þ E s = 200 V \ R AB = 2.5 W 27

Frequency will be same as 50 Hz.

MODULE 2
www.jeebooks.in
152 NEET Test Drive

R0 20. Black body has maximum emissivity. 1 2eV


\ R¢= 27. As, mv 2 = eV Þ v =
27 Hence, the black object at a 2 m
Hence, in 9 days activity will become temperature of 2335ºC will glow Here, V = 18 kV = 18 ´ 103 V,
æ 1 ö of the original value. brightest.
ç ÷ . ´ 10-19 C
e = 16
è 27 ø
21. Decay constant for first process, and . ´ 10-31 kg
m = 91
16. At closest distance r its whole KE is 0.693
l1 = . ´ 10-19 ´ 18 ´ 103
2 ´ 16
converted into PE. t1 v =
1 1 Q. q 9 ´ 10-31
\ mv 2 = and decay constant for second 7
2 4 pe 0 r = 8 ´ 10 m/s
process,
1 Q. q 1 1 1 uv
Þ r = 0.693 28. As, = + or f =
l2 = f u v u+v
4 pe0 mv 2 t2
1 Qq (501. ) (201.)
In next case, r ¢ = The probability of an active nucleus Þ f = = 14.3 cm
4 pe0 m (2v ) 2 501. + 201 .
decays by first process in small time is
Df Du Dv Du + Dv ù
1æ 1 Qq ö l1 dt, similarly for second process it is =± é + +
Þ r ¢= ç . ÷ Þ r ¢= r / 4 f êë u v u + v úû
l 2 dt.
4 è 4 pe0 mv 2 ø
. + 0.2 ù
= ±é
Total probability it decays by first 05
. 0.2 05
+ +
17. Mass = 10 + 10 = 20g = 0.02 kg, process or by second process is ëê 501. 201
. 501 . ûú
. + 201
r = 1 cm = 10-2 m (l1dt + l 2dt ) = ±0.0299
Area of cross-section, ldt = l1dt + l 2dt Þ Df = ±0.0299 ´ 14.3 = ± 0.4 cm
2
22 æ 1 ö Þ l = l1 + l 2
A = pr 2 = ´ç ÷ \ f = (14.3 ± 0.4) cm
7 è 100 ø
0.693 0.693 0.693 1 1 1 29. Force on charge particle against electric
22 = + Þ = +
= ´ 10-4 m 2 field,
7 t t1 t2 t t1 t 2
-1 F = qE
Density of water, r = 10 kg m 3 -3 Þ t = t1-1 + t 2-1
ma = qE
Force constant, 22. Voltage drop of LED = 2V qE
F a=
k = \ Voltage across resistor, m
y (V) = 6 - 2 = 4 V From equation of motion,
Ayrg V 4
= = Arg So, value of R = = = 400W v 2 = u 2 - 2as
y I 10 ´ 10-3 qE
22 0 = u2 - 2 s
= ´ 10-4 ´ 103 ´ 10 23. Angle i at both points will be 45º. For m
7 total Internal reflection to take place mu 2
22 i > qC or sini > sin qC Þ s =
= Nm -1 2qE
7 1 1
\ > or m > 2
m 2 m 30. Maximum height of 1st projectile,
Time period, T = 2p
k u 2 sin2 q (12) 2 sin2 45 º
24.
V1 T1
= Þ
V
=
320 h0 = = = 3.6 m
22 0.02 ´ 7 V2 T 2 3V T2 2g 2 ´ 10
= 2´ = 05
. s
7 22 Time of attaining maximum height,
[QT1 = 273 + 47 = 320]
18. Given that, length of copper and brass u sin q 12 ´ sin 45 º 3 2
Þ T 2 = 960 Þ DT = 640 t = = = s
are L1 and L2 . g 10 5
W = pDV = nR DT
The length change with temperature, 3 2
= 3 ´ 2 ´ 640 = 3840 cal Distance of vertical fall in s
l ¢1 + l ¢ 2 = L1 (1 + a1Dq) + L2 (1 + a2 Dq) 5
2GM
Q Given, L1 + L2 = l1¢ + l 2¢ 25. Escape velocity, ve = æ3 2 ö
2
R 1 2 1
s = gt = ´ 10 ´ ç ÷ = 3.6 m
Þ L1a1Dq + L2 a2 Dq = 0 2 2 è 5 ø
vm 2GM m / R m
So, L1a1Dq = - L2 a2 Dq Þ =
vE 2GME / RE \ Initial height of second object
Þ -L1a1 = L2 a2
= 3.6 + 3.6 = 7.2 m
æ M m ö æ RE ö
19. Velocity of body at time t is given by = ç ÷´ç ÷ 1
dx d 2 è Me ø è R m ø 31. We know that,T µ
v = = (t ) = 2t H
dt dt 1 2
Now, at t = 0 s, initial velocity vi = 0 = ´4 = T1 H2
81 9 \ =
T2 H1
at t = 4 s, final velocity vf = 8 2 2
Þ Vm = ´ VE = ´ 11 = 2.44 km/s 5 H2
Work done = increase in KE 9 9 =
1 1 1 10 3
= mvf2 - mvi2 = m (vf2 - vi2 ) 26. Every current element on the conductor
2 2 2 makes either 0º or 180º angle with its 1 H2
Þ =
1 position vector w.r.t. observation point. 2 3
= ´ 4 ´ (8 2 - 02 ) = 128 J
2 Hence, magnetic field is zero.
MODULE 2
www.jeebooks.in
Prep Catalysis for NEET ~ Mock Test 3 153

1 H2 v + v0 v + v0 v 41. Position of nth bright fringe from central


= f = ´ f2 = ´ ´ f1
4 3 v v v - vs maxima,
\ H 2 = 075
. v + v0 320 + 10 nD l
= ´ f1 = ´ 8000 xn =
Reduction in field = 3 - 075
. = 2.25 g v - vs 320 - 10 d
32. Rotational kinetic energy, 33 i.e. xn = xn 2
= ´ 8000 ~
- 8516 Hz 1
31 n1Dl1 n2Dl 2
(KE )R =
1 2 1 1
Iw = ´ (mR 2 )w 2 =
2 2 2 d d
36. Here, I g = 0.4% of I = 0.004 I
l1 n 2 8
1
= mR 2 w 2 = = =
Shunt current, IS = I - I g = 0.996 I l2 n1 5
4
I ×R
Translational kinetic energy, \ Shunt resistance, RS = g g Hence, only option (c) satisfies.
Is
1 1 1 42. In series combination, electric current is
(KE )T = mv 2 = m (Rw) 2 = mR 2 w 2 0.004R g Rg
2 2 2 = = same through both wires.
0.996 I 249 1
Total energy, Hence, currect density J µ
3 n A
KE = (KE )R + (KE )T = mR 2 w 2 37. n¢ = ´n
4 n - vs cos q J copper 2
riron
=
(KE )T 1 / 2mR 2 w 2 2 Jiron 2
r copper
\ = = v Q 800 m
KE 3 / 4 mR 2 w 2 3 S
θ (0.4) 2
= = 16
33. (A) This is NOR gate symbol (01. )2
(B) (A or B) NOT Þ A × B » NAND gate 600 m Jiron 1
1000 m \ =
(C) A NOR B = A + B J copper 16
Þ NOT of P fo æ f ö
A + B = A + B = A + B = OR gate 43. As, magnification | m| = ç1 + e ÷
vs = 72 km h -1 = 20 ms -1 fe è Dø
[QA = A] 800 1
cos q = = 0.8 So, mµ
1000 fe
(D) (Not A) NAND (NOTB)
vs cos q = 20 ´ 0.8 = 16 ms -1 \ In order to produce largest
= A × B = A + B = A + B = OR gate 340
Þ n¢ = ´ 800 = 8395
. Hz magnification the focal length of
1 æ 1 1ö 340 - 16 eye-piece should be small, i.e. + 15 cm
34. We know that, = Rç 2 - 2 ÷ lens is used.
l è n1 n2 ø 38. Uintial = 2 J, U final = 8 J, d= 4 ´ Uinitial
GM
1 1 q2 44. g = 2
= R æç 2 - 2 ö÷
1 1 1 but U = CV 2 =
For first case, R
l1 è2 3 ø 2 2 C
gR 2 162 . ´ 106 ) 2
. ´ (174
1 5R and q = CV ÞM = =
= K (i) G 6.67 ´ 10-11
l1 36 Since, C is constant, hence charge and
potential both should be doubled. = 7.35 ´ 1022 kg
1
=R æ1 - 1 ö M
For second case, ç ÷ And r =
l2 è1 2 2 ø 39. Since, X L = X C = 25W, so,VL andVC
V
3 are equal and opposite in phase, i.e.
= R gR 2 3g
4 volmeter reading = 0 V. = =
4
G ´ pR 3 4 p GR
1 3R \ Ammeter reading,
= ...(ii) 3
l2 4 V
I = 3 ´ 162
.
R 2 + (X L - X C ) 2 =
From Eqs. (i) and (ii), we get 4 ´ 3.14 ´ 6.67 ´ 10-11 ´ 174
. ´ 106
1 5R 240 = 3.3 ´ 103 kg m -3
= = 8A
l1 5 30 m I
\ = 36 = 45. Magnetic field at the centre, B = 0
1 3R 27 1 l T 500 1 2r
40. l m µ Þ\ X = Y = =
l2 4 T lY T X 1500 3 e
where, I = (t ® time period)
l1 27 E µT 4 × A t
=
l2 5 (where, A = surface area = 4 pR 2 ) e ev
= =
\ E µ T 4R 2 2pr / v 2pr
35. As bat approaches the stationary wall,
apparent frequency 4 2 m ev m ev
EX éT ù é R ù \ B = 0× = 0 2
v = ê Xú ê Xú 2r 2 pr 4 pr
f2 = ´ f1 EY ëTY û ë RY û
v - vs 1
2 \ B µ v and B µ 2
32
= [3] 4 é ù =
12 r
Apparent frequency of reflected wave, =9
ëê 36 ûú 1

MODULE 2
www.jeebooks.in

CHEMISTRY
46. [CoF6 ] 3 - complex is an outer orbital complex. In this complex, 55. H2 SO 4 s HSO 4- + H+
oxidation state of Co is +3 due to presence of 6F - ligands.
CH3 CH3
3d 4s 4p 4d
H+
Co3+= CH3 — C —CH—CH3 CH3 —C —CH—CH3
- 3+
Since, F is a weak field ligand, it cannot pair electrons of Co . CH3 OH CH3 OH2
3– +
[CoF6] = 3-3-dimethyl-butan-2-ol
–H2O Slow
sp3d 2 hybridisation
(outer orbital complex) CH3 CH3 2, 3-CH3
– CH3
Shift
47. Reaction sequence is as follows : CH3—C—CH—CH3 CH3—C—CH—CH3
+
H H (3º-carbocation) CH3
Cl KCN CN H3O+ COOH (2º-carbocation)
–H+
–H+
Benzyl cyanide 2-phenylethanoic
acid
CH3 CH3 CH3
48. Formalin is a 40% aqueous solution of formaldehyde (HCHO).
CH3—C== C—CH3 CH3—C—CH== CH2
49. Oxidation of butan-2-ol to ethyl methyl ketone can be made 2,3-dimethyl-but-2-ene
effective by using oxidising agent (pyridinium chlorochromate in (Major product) CH3
dichloromethane). 3,3-dimethyl-but-1-ene
O (Minor product)
PCC
½½
CH3 ¾ CH2 ¾ CH ¾ OH ¾¾® CH3 ¾ CH2 ¾ C ¾ CH3 56. Lanthanoid ion with no unpaired electron is diamagnetic in
DCM
½ Methyl ethyl ketone
CH3 nature. The electronic configuration of the given lanthanoid ions
Butan- 2 - ol are as follows :
Ce2+ = [Xe]4f 2 (Two unpaired electrons)
50. In Ni(CO) 4 , Ni is in zero oxidation state Ni(0) = 3d 8 4s 2
Sm2+ = [Xe]4f 6 , 5d 0 (Six unpaired electrons)
3d
Eu2+ = [Xe]4f 7, 5d 0 (Seven unpaired electrons)
In [Ni(CO)4]=
Yb 2+ = [Xe]4f 14 , 5d 0 (No unpaired electrons)
Thus, Yb 2+ is diamagnetic in nature.
CO CO CO CO
57. NH2OH and NH2 — NH2 may be considered as NH3 derivatives
It is sp 3 -hybridised.
in which H is replaced by—OH and — NH2 respectively. Due to
In [Ni(CN) 4 ] 2 - , Ni is in +2 oxidation state.
their electron withdrawing nature, these groups decreases
[Ni(CN)4]2–= electron density over nitrogen making them less basic. The
effect of —OH group is stronger than —NH2 .
CN CN CN CN 58. Brass in an alloy of Cu-Zn. It is composed of 60-80% of Cu
It is dsp 2 -hybridised. and 20-40% Zn. It is used in domestic utensils, condensers,
In [NiCl 4 ] 2 - , Ni is in +2 oxidation state. tubes etc.

[Ni(Cl)4]2–= 59. Acidic strength of oxides decreases on moving down the group.
Thus, the order of acidic strength is :
Cl Cl Cl Cl P2O 5 > As 2O 3 > Sb 2O 3 > Bi 2O 3 .
3
It is sp -hybridised. 60. Radial node = n - l - 1
Rh-catalyst + For d-orbital, n = 4, l = 2, i.e. 4 - 2 - 1= 1
51. CH3OH + CO ¾¾¾® CH3 COOH ¾H¾® CH3 — CH3
Methyl alcohol A (Ethanoic acid) Red P B (Ethane) 61. R SiCl 3 on hydrolysis produces cross-linked polymer.
52. The complex [Ru(H2O) 6 ] 2+ and [RuCl 6 ] 3 - have nearly the same –3HCl
R—SiCl3 + 3H2O —Si—O—Si—O—Si—O—Si—O
D o values because H2O and Cl - occupy adjacent positions in the
spectrochemical series. R O O O O
Polymerisation
53. Ce shows +4 oxidation state because it attains noble gas –H2O
—Si—O—Si—O—Si—O—Si—O
R—Si—OH
configuration and moreover it has tendency to attain
f 0 configuration. O O O O
OH
54. [MA5B] is the octahedral complex that will not show geometrical —Si—O—Si—O—Si—O—Si—O
isomerism. These complexes does not have two same ligands
that can either occupy adjacent positions or opposite positions.

MODULE 2
www.jeebooks.in
Prep Catalysis for NEET ~ Mock Test 3 155

62. Within a series of isoelectronic ions, as 71. Structure of boric acid consist of By comparing the equation with
the nuclear charge increases, the force number of layer hence known as y = mx + c, the following plot is obtained
of attraction by the nucleus on the layered structure. Each layers are λ∞
electrons also increases. As a result, linked together by van der Waals’ force
ionic radii decreases. Thus, their ionic of attraction. Each layer has various λC m=– B
radii increases in the order : trigonal planar B(OH) 3 units.
Al 3 + < Mg 2+ < Na+ < F - < O 2 - 72. The acidic character of conjugate
63. l value never equal to n value [l = (n - 1)]. acids is H2O > ROH > NH3 . A stronger
acid has weak conjugate base and O √C
64. O O O O vice-versa. Hence, the correct order of
- - 81. For reaction, 4 A ¾® 5B
basic strength is : OH < R O < NH2 .
hν -1 d [A] 1 d [B ]
+ =
A
73. DS° = 2S°HCl - (S°H 2 + S° Cl 2 ) 4 dt 5 dt
B
O d[B ]
= 2 ´ 1867
. - (130.6 + 223.0) Given , = y mol L
dt
65. According to Graham’s law of diffusion, = 19.8JK -1 mol -1
d[A] -4 d[B ] -4
1 \ = = y
rate of diffusion µ 74. Statement (c) is correct for resonance dt 5 dt 5
molecular weight because resonance affects bond
Hence, rate of consumption of A
lengths but not bond angles while (a),
As the molecular weight of the gas -d [A] 4
(b) and (d) are incorrect statements. = = y
increases, rate of diffusion decreases. dt 5
75. C H COOH
66. Heat of any type of reaction is O3 Zn/CH3COOH CH2OH 82. Mass of 1 L solution of ethanol = 1025 g
calculated by using Hess’s law NaOH
C COOH CH2OH Mass of solute in 1.0 L = 8 ´ 46 = 368 g
according to which heat change in any H
reaction is sum of heat change in 1st step is ozonolysis and 2nd step is \ Mass of solvent = 1025 - 368 g = 657 g
various small steps. reduction. Whole reaction is known as 8g
Molality (m) = ´ 1000 = 12 .17m
It is used for the determination of reductive ozonolysis. 657g
(a) enthalpies of formation of compound. 76. CFC is known as chlorofluorocarbon. It 83. Qk A = kB
(b) enthalpies of extremely slow causes ozone depletion. Hence, CFC
is harmful to ozone layer CFC gas \ 1018 e -8000 T = 1017 e -4000 T
reactions.
makes the hole in the ozone layer and 10 = e 4000 T
(c) enthalpies of transformation of one
hence the harmful ultraviolet light rays 4000
allotropic form into another. ln10 =
penetrate and reach the earth’s T
(d) bond energies etc. surface. 4000 4000
\ T = = K
67. On dissolving moderate amount of 77. C¾C, C == C and C ºº C bond In10 2.303
sodium metal in liquid NH3 at low
temperature, that liquid NH3 becomes lengths are 1.54 Å, 1.34 Å and 1.20 Å 84. Iron pyrite is a sulphide ore of iron
paramagnetic due to presence of free respectively. In benzene, C == C is having molecular formula FeS 2 . Sulphide
electrons. ores are extracted using froth-floatation
1.40 Å. method.
68. For given reaction, Dn = 2 - 4
Hence, correct option is (b). 85. 0.2% of phenol solution is used as
=-2 ZM antiseptic, while 1.1% phenol solution is
78. We know that, r =
We know that, K p = KC (RT ) Dn Na 3 used as disinfectant.
K p < KC Given, Z = 8, M = 12 86. For the reaction A ® B , on increasing
69. Compound (b) is an optically active N = 6.023 ´ 10 23 the concentration of reactant by 8, rate
molecule due to the presence of -10 becomes 4 times.
a = 3.569 ´ 10 m
asymmetric carbon. 1
8 ´ 12 \ Order of reaction =
H r= 2
6.0.23 ´ 1023 ´ (3.569 ´ 10-10 ) 3
½ Q Rate = k [A] n
CH3 ¾ * C ¾ C2H5 = 3.506 g cm-3
87. Certain proteins give yellow colour with
½
OH
79. Since, H3PO 3 is a diprotic acid. conc. HNO 3 . This yellow colour is same
\ N = M ´ n ÞN = M ´ 2 (Qn = 2) which is formed on skin when skin
70. X + Y P + Q d
No. of moles comes in contact with conc. HNO 3 . This
initially 4 4 0 0 N = 2 ´ 0.3 = 0.6 N test is known as xanthoprotic test.
No. of moles (4 - 2) (4 - 2) 2 2 80. By Debye-Huckel Onsager equation, 88. NaNO 2 HCl (or HNO 2 ) causes
at equilibrium
lC = l ¥ - B C diazotisation of aniline and results in the
Equilibrium constant is, where, lC = equivalent conductivity at formation of diazonium salt which when
[P ][Q ] concentration C. reacts with CuCN gives benzonitrile.
KC = =1
[X ][Y ] l ¥ = equivalent conductivity at infinite
dilution.

MODULE 2
www.jeebooks.in
156 NEET Test Drive

The reaction is as follow : 90. NO2 N2Cl


+ –
NH2 N2 Cl CN
0–5ºC
+ NaNO2 + HCl
NaNO2/HCl CuCN
0–5ºC ∆ Aniline
Aniline Benzene diazonium Benzonitrile N(CH3)2
chloride 'B'
CH3 Dimethylaniline
89. NH2 N2+ Cl– F N N N
CH3
NaNO2/HCl HBF4
∆ Azo dye
Diazotisation (Coloured compound)
Aniline Benzene Fluorobenzene
diazonium chloride

BIOLOGY
91. The chloroplast thylakoids are in the The source of this complementary RNA 104. The common ancestor of man and ape is
form of stacked discs which are could be either an infection (by viruses Dryopithecus. It lived about 25 million
referred to as grana. Thylakoids are the having RNA genomes) or transposons years ago during Miocene. Dryopithecus
sites of the light dependent reactions of (mobile genetic elements) that africans shows close similarity to
photosynthesis. replicate via an RNA intermediate. chimpanzee.
92. The cohesion and adhesion forces in 98. In competitive inhibition of enzymatic 105. A DNA molecule that is able to replicate
the water column are responsible for catalysis, binding of an inhibitor to autonomously is called replicon. It is a
conducting water to great heights. active site prevents binding of the DNA molecule or RNA molecule or a
These forces allow water molecules to specific substrate. This inhibition can region of DNA or RNA, that replicates
form an unbroken continuous water be reversed by increasing the from a single origin of replication.
column in the xylem. substrate concentration in the reaction. 106. A person with asthma has difficulty in
93. Sclerenchymatous fibres are lignified 99. All the statements I, II, III and IV are breathing because of the constriction of
and dead. They are long and tapering correct regarding ETC (Eelectron terminal bronchioles. It is a disorder that
at ends. These are found in pericycle Transport Chain). causes the muscles surrounding the
of stem forming a solid tissue 100. Pancreas is not an organ where airways of the lungs to become tight and
protecting the vascular bundles of lymphocytes originate, mature or the lining of the air passage swells. Thus,
dicots. multiply. It is involved in the process of reducing the amount of air that can pass
digestion. Bone marrow and thymus are by.
94. A pendulus spike bearing unisexual
flowers is catkin. It is observed in primary lymphoid organs. The spleen is 107. The features given are vegetative
Acalypha and oak. the secondary lymphoid organ. characteristics of fungi belonging to
101. Apospory is development of class Zygomycetes, e.g. Rhizopus
95. Physiological barriers include tears
gametophyte from any cell of (bread mould). Their hyphae are
from the eyes, saliva in the mouth, acid
sporophyte other than spores whereas generally non-septate and multinucleate,
in the stomach, etc. They prevent the
apogamy is development of i.e. coenocytic. The characteristic
growth of microbes.
sporophyte without fusion of male and component of hyphal wall is chitin but
Interferons are cytokine barriers, i.e. cellulose may also be present along
cells which are virus infected release female gametes.
with it.
protein called interferons that protect 102. The eyes of arthropod or insect are
non-infected cells from further viral called compound eyes as they are 108. The flowers show trimerous symmetry in
infection. made of ommatidia. These ommatidia Monocotyledonae.
function as a separate visual receptor. It is the presence of parts of a flower,
96. Pseudometamerism refers to the body
A single ommatidium consists of a lens, such as petals, sepals and stamens in
being divisible into parts or segments
a transparent crystalline core, light sets of three parts.
called proglottids. They develop from
the neck but are not embryonic in sensitive visual cells arranged in radial In Dicotyledonae, flowers exhibit
origin. Such a repetition which appears patterns and pigment cells. tetramerous or pentamerous symmetry.
due to the repeated budding is known 103. Bone tissue is unusual in that it is 109. Doom is a mixture of Bacillus popilliae
as false segmentation or continually being reconstructed. The and Bacillus lentimorbus.
pseudometamerism. osteoblasts secrete bone matrix and It is a bioinsecticide in which living
97. All eukaryotic organisms use RNAi as a osteoclasts destroy the same. These organisms or their products are used for
method of cellular defence. This twin processes of resorption and insect control.
method involves silencing of a specific reconstruction enable a particular bone
to remodel its structure and meet any It has been commercially used for
mRNA due to a complementary dsRNA
changes in the mechanical controlling the larvae of Japanese
molecule that binds to and prevents
requirements of an animal during its beetles.
translation of the mRNA (silencing).
development.
MODULE 2
www.jeebooks.in
Prep Catalysis for NEET ~ Mock Test 3 157

110. The fallen stars is the name given to ● Viruses enclose a single type of The ATP is obtained from cellular
Nostoc. It is the genus of nucleic acid, i.e either RNA or DNA. respiration and so rate of respiration
cyanobacteria that grow on the soil. ● Viruses do not possess their own increases during absorption.
The colonies of Nostoc are ordinarily metabolic machinery instead 128. Dicot leaves are dorsiventral. The
unobservable but after rainfall as they utilising their living host’s cellular bulliform cells are absent. The vascular
often swell up and appear as stars, machinery for multiplication. bundle is surrounded by
hence the name fallen stars.
119. Manganese becomes toxic when parenchymatous bundle sheath.
111. Family is a subdivision of an order absorbed by plants in higher amounts. Mesophyll is differentiated into palisade
consisting of a group of related genera The toxicity is expressed in the form of and spongy parenchyma. Hence, all the
and species. The families in plant brown spots surrounded by chloretic statements are correct regarding dicot
kingdom are classified based on the vein. It is due to the reduction in uptake leaf.
vegetative and reproductive parts of of Fe 3+ and Mg 2 + and inhibition of 129. All the options are correctly matched. In
the plant species. binding of Mg 2 + to specific Fe 3+ nucleus, there are atleast three RNA
112. GIFT is Gamete Intra Fallopian enzymes. It also causes inhibition of
polymerases in addition to RNA
Transfer. It can successfully provide Ca 2 + translocation in shoot apex. Thus,
polymerase found in organelles to allow
conception in females who cannot excess of Mg 2 + causes deficiency of
clear cut division of labour.
produce a viable ovum but can provide iron, magnesium and calcium.
suitable environment to aid fertilisation 130. Plant tumours involve infection of
120. The statement (d) is incorrect. It is
and further development of foetus. Agrobacterium tumefaciens.
corrected as
Bacteriophages deliver large genes into
113. The above statements depict the Mycoplasmas are Gram negative. They
adaptations in desert lizards. They bacterial cells.
are covered with three-layered plasma
absorb heat from sun when the body membrane and 70S ribosomes are Adenoviruses deliver genes to living
temperature drops below the comfort present. Cell wall is also absent. patients in gene therapy.
zone. They move into shade when the Retroviruses are involved in foreign
121. The correct sequence of electron
ambient temperature starts increasing. genes replication into every daughter
carriers between PS-II to PS-I in
They burrow into the soil. These are cell.
photosynthesis is PQ, cyt-b 6 , cyt-f, PC.
behavioural responses.
In non-cyclic photophosphorylation, 131. Synergids are specialised cells lying
114. The phylum in which above electron passes from a carrier at lower adjacent to the egg cell. They secrete
characteristic features are included is redox potential to one having higher chemotactic substance for attraction of
Chordata. They possess a ventral heart one. At each step energy is released. pollen tube during fertilisation. Synergids
and closed circulatory system. form a highly thickened structure called
Haemoglobin is present in red blood 122. MSH is the only hormone secreted by
the intermediate lobe of pituitary, i.e. filiform apparatus towards the micropylar
corpuscles. Digestive system is pole.
well-developed and hepatic portal pars intermedia. It affects spreading of
system is present. melanin granules so that skin colour 132. The tubular reabsorption involves the
darkens. absorption of water and useful solutes
115. The statement (IV) is incorrect from the glomerular filtrate into the
regarding fermentation. It is corrected 123. Pothos (money plant) is an example of
root climbers. The adventitious roots at blood. It occurs by reverse diffusion and
as follows
nodes of stem attach with the support active transport. It occurs mostly in PCT.
The common products released during by an adhesive disc to climb a support. It does not occur in animals that lack
fermentation process are ethanol (and glomerulus.
124. Unio belongs to the class Pelecypoda
CO 2 ) and lactic acid.
and Octopus belongs to the class 133. Adrenaline and noradrenaline are the
116. The option (d) is incorrectly matched Cephalopoda. All others belong to two hormones secreted from the adrenal
pair. It is corrected as follows Gastropoda. medulla. Adrenaline hormone is rapidly
Incomplete dominance — Snapdragon. 125. In non-cyclic photophosphorylation, secreted in response to stress and
plastocyanin is a copper containing during emergency and is known as the
117. Adiantum caudatum is also known as emergency hormone. Its secretion is
walking ferns because new plantlets compound. It is a small protein found
involved in electron transfer in all directly controlled by autonomous
grow wherever the arching leaves of nervous system.
higher plants.
this plant touch the ground, thus,
giving a walking effect. 126. The option (b) is incorrectly matched 134. Community is assemblage of different
pair. The second phase includes the populations.
118. Only statement (b) is correct. Viruses
vectors with greatly reduced size. The Ecosystem is all organisms that live in a
only become living when they come in
use of antibiotic resistance was seen for place together with their physical
contact with a living host. Hence, they
the selection of recombinant vectors. environment.
are called as obligate parasites.
127. Both the statements (a) and (b) are Biome is a group of ecosystems that
Incorrect statements can be corrected
correct regarding active absorption. share similar climates.
as
Active absorption requires ATP Biosphere is entire planet with all its
● Viruses contain a protein coat called because the movement of molecules
capsid. organims and physical environment.
occurs against the concentration
gradient.

MODULE 2
www.jeebooks.in
158 NEET Test Drive

135. X – Prostate gland (male accessory are in low percentage. Hence, an purine nucleoside. It is a methylated
organ) urn-shaped age pyramid is observed. version of guanosine.
Y – Testis (Primary reproductive organ, 144. Both the options are correctly matched. 152. The statement (d) is correct for Okazaki
produce gametes) Some organisations and multinational fragments . They contain covalently
Z – Urethra (carries urine from bladder companies exploit and/or patent linked RNA and DNA.
to outside) biological resources of other nations Okazaki fragments are short, newly
without proper authorisation from the synthesised DNA fragments that are
136. The option (d) is incorrectly matched
countries concerned and this is called formed on the lagging template strand
pair. The greater the BOD (Biochemical
biopiracy. during DNA replication.
Oxygen Demand), more is its polluting
potential. BOD is the measure of the Biopatent is the right granted by the 153. The properties of genetic meterial
organic matter present in the water. government to an inventor to prevent include, it should be able to replicate
others to make commercial use of itself, should be stable both chemically
137. All the options are correct in reference
one’s invention. and structurally and it should be able to
to promoter of transcription unit. It is
145. The functions of endosperm are as express itself in the form of ‘Mendelian
located towards 5¢ end (upstream) of
follows characters’. It should also provide the
the structural gene. It provides the site
It forms sufficient food for the scope for show changes (mutation)
for binding of RNA polymerase. The
developing embryo. which are required for evolution.
site is A and T rich region called TATA
Box (in prokaryotes) and Pribnow Box Endosperm stores carbohydrates, fats, 154. Lethal genes cause death of the
(in eukaryotes) to which specific proteins, etc., and provides them to the organism when present in homozygous
proteins combine, during transcription. seed during germination for their state.
138. The pink pages in IUCN Red List refers growth. Lethal alleles are usually a result of
to the critically endangered species. Many experiments have proved that mutations in genes that are essential to
the contents of endosperm are very growth or development.
Green pages include species which
are no longer threatened. These necessary for the growth of embryo. 155. The infants are suffering from the
species were formerly endangered but 146. The secondary immune response disease kwashiorkor. It is a protein
with time have recovered. occurs during the second and deficiency disease. This disease
subsequent exposure of the same host commonly affects the infants and
139. Correct sequence of plant breeding are
children between 1-3 years of age. The
given by option (b), i.e. I, III, II, IV, V. to same antigen. It is more rapid and
lasts for longer periods. common symptoms are underweight,
Collection of variability ® Evaluation stunted growth, poor brain development,
and selection of parents ® Cross The primary immune response takes
longer time to establish immunity. It loss of appetite, anaemia, slender legs,
hybridisation among selected parents protruding belly and bulging eyes.
® Selection and testing of superior declines rapidly and occurs as a result
recombinants ® Testing release and of the first contact of the animal with an 156. Both the options are correctly matched.
antigen. In dicot root, the pith is absent or poorly
commercialisation of new cultivars.
147. All the statements regarding developed.
140. Entamoeba histolytica present in the
gut of animals causes amoebic phagemids are correct. They are the In monocot root, the cambium is absent.
dysentery. plasmids with a fragment of 157. The cardiac muscles possess the given
filamentous phage DNA. Thus, characteristics. These muscle fibres are
Entamoeba gingivalis is found in mouth
combining the desirable features of supplied with both central and
of humans and causes oral infection.
both. autonomic nervous systems. They are
Entamoeba coli however are generally
148. During expiration, there is relaxation of contractile tissues present only in the
harmless but in some cases can cause
external intercostal muscles and heart and in the wall of large veins which
diarrhoea.
diaphragm bends upwards which enter the heart. They show the
141. Food web consists of number of reduces the vertical length of thoracic characters of both striped and unstriped
interconnected food chains and is cavity. muscle fibres.
never straight. In a food web,there are
During inspiration, diaphragm 158. Pons Varolii acts as a highway for the
a number of alternate sources of food.
straightens and external intercostal relay of many signals to and from the
Hence, all the options are correct muscles contract. cerebrum and cerebellum. It is the part
regarding the food web.
149. Resource partitioning is the way in of midbrain which is present at the axis
142. Auxin and cytokinin are plant growth which different species can use the of brain. It is considered as a link
promoters. When added in plant tissue same resource, such as food, without between the upper portions of the brain
culture media, they promote the occupying the same physical location and spinal cord through the medulla
formation of callus. Auxin promotes at the same point in time. oblongata. It helps in the regulation of
rooting and cytokinin accelerates breathing movements.
There is avoidance of competition and
shooting.
realised niches divide resources 159. The four kingdom system of
143. In young population pyramid, the birth among several species. classification consists of Monera,
rate is high and population growth may Protista, Plantae, Animalia. It was given
150. Skeletal muscles are made up of a
be exponential. Hence, age pyramid by Copeland.
number of muscle fibres fused together,
with broad base is observed. 160. Pollen development in flowering plants
therefore, they have multiple nuclei on
The stable population has moderate syncytium. comprises of following sequence
proportion of young to old. Hence, a
151. In eukaryotic cells, mature mRNAs I ® III ® II ® IV.
bell-shaped age pyramid is observed. In have a 5¢ cap that is residue of Microspore mother cell develops within
declining population, young individuals 7- methylguanosine. It is modified pollen sac and undergoes meiosis to
form four haploid microspores.
MODULE 2
www.jeebooks.in
Prep Catalysis for NEET ~ Mock Test 3 159

Each microspore undergoes mitosis to 168. If two genes are located very close to particular phase is spent in synthesising
form immature pollen grain. The each other on the same chromosome, molecules other than DNA.
generative cell undergoes mitosis to the occurrence of crossing over is very 175. Cowper’s glands are also known as
form two sperm cells. rare. bulbourethral glands. They are situated
161. The option (d) is incorrect statement. It Prevalence of recombination is beneath the bladder and behind urethra
is corrected as follows dependent on the distance between and add fluids to semen during
Roundworm is an example of linked genes. Frequency of crossing ejaculation.
Aschelminthes. over between two genes decreases if 176. Primary pollutants are pollutants that are
162. The option (a) is incorrectly matched they are closely placed. present in environment as it is produced.
pair. The correct example of siliqua is 169. The bulk of the semen is contributed by Secondary pollutants are formed from
Brassica (mustard). seminal vesicle fluid. The semen is primary pollutant.
This fruit develops from bicarpellary, composed of the fluid and sperms from Qualitative pollutants are produced by
syncarpous, bilocular and superior the vas deferens (about 10% of the human activities.
ovary with parietal placentation and total), fluid from the seminal vesicles Quantitative pollutants are pollutants
false septum called replum. Each (almost 60%) and prostate gland whose concentration reaches more than
locule has many seeds. (about 30%) and small amounts of threshold value.
163. The statement (d) is incorrect mucus from bulbourethral gland. 177. In deuterostomes, anus develops first
regarding lac operon. It is corrected as 170. Both the statements (a) and (b) are from blastopore region and mouth
follows correct regarding Protista. They are develops later. In protostomes, mouth
A-gene codes for a transacetylase. unicellular eukaryotes and possess develops from blastopore region and
One regulatory gene (the i gene), double nuclear membrane. anus develops later.
codes for the repressor of the lac 171. Intermediate host is in which asexual 178. Prickles act as climbing organs.
operon. reproduction occurs and the immature Phylloclades are green, flattened
164. Marriage between close family form harbors. structures bearing several nodes and
members can lead to genetic Definitive host is in which sexual internodes.
anomalies in future offsprings. This is reproduction occurs and which harbors Cladodes are green photosynthetic
because certain dormant recessive the mature form. stems generally one internode long.
genes may double up, increasing the 172. All the statements regarding Bulbils are modified vegetative or floral
chances of appearance of genetic bryophytes are correct. They are buds arising in the axil of scale or foliage
abnormalities in their children. non-vascular cryptogams and plant leaves. They help in vegetative
165. The option (d) is correctly matched body is thalloid. propagation.
pair. The points of attachment between The sporophyte is attached to 179. The statements (a) and (b) are correctly
the homologous chromosomes after gametophyte. The gametophyte is the related to in situ conservation. It involves
dissolution of the synaptonemal dominant phase in their life cycle. protection of species threatened with
complex are called chiasmata. This 173. The correct matches are extinction in their natural habitats. Also,
process takes place during the the protected flora and fauna remain
diplotene stage of prophase-I of (a) IAA (auxin) – Helps in cell wall
undisturbed, i.e. without human
meiosis-I. elongation
interference.
166. Invertebrates have a bluish blood (b) Gibberellins – Stem elongation 180. All the assumptions stated here should
because of respiratory pigment (c) Cytokinins – Cell division be met in Hardy-Weinberg equilibrium.
haemocyanin. It makes the blood look The population size is very large and
bluish because it contains copper. (d) Abscisic acid – Stomatal closure
there should be random mating.
167. In dorsiventral leaf, guard cells are 174. The option (c) is incorrect pair. It is There should be no gene flow, i.e.,
kidney-shaped. The stomata usually corrected as migration of individuals and therefore
absent or less abundant in upper G 2 – Growth and differentiation. alleles, into or out of the population does
epidermis, while numerous in lower not occur.
In this phase, the preparations are
epidermis.
made for genomic separations. This

SCORE CHART
No. of Correct Answers : A .................. No. of Incorrect Answers : B ..................
Total Marks : x = (A × 4) – (B × 1)
Scores and Expected Rank : If the score lies above 665, then rank will be in between 1-50. For other scores, rank
estimations are given below
Score Rank
664 – 642 51 – 150
641 – 636 151 – 250
635 – 630 251 – 400
629 – 625 401 – 500

MODULE 2
www.jeebooks.in

PREP CATALYSIS
for NEET
Full Length Mock Tests for NEET to Make You Ready to Face the Challenge

MOCK TEST 4 (With Solutions)

Duration : 3 Hours Max. Mark : 720

Instructions
The test is of 3 hours duration and Test Booklet contains 180 questions. Each question carries 4 marks. For
each correct response, the candidate will get 4 marks. For each incorrect response, one mark will be deducted
from the total scores. The maximum marks are 720.
The question paper contains three parts of Physics, Chemistry and Biology respectively.
Part A contains 45 Questions from Physics section. Part B contains 45 Questions from Chemistry section.
Part C contains 90 Questions from Biology.

PART A PHYSICS
1. A satellite is launched into a circular orbit of 4. In a biprism experiment, fringes are obtained on
radius R around the earth. A second satellite is the screen at a distance of 2m from the source,
launched into an orbit of radius 1.01 R. The period when a monochromatic light of wavelength 400 nm
of the second satellite is larger than that of the used. When, a convex lens is placed between the
first one by source and screen, images of the slits in two
(a) 0.5% (b) 1% position 0.03 mm and 0.12 mm respectively, then
(c) 1.5% (d) 3% fringewidth on the screen will be
(a) 0.0013 mm (b) 13.3 mm (c) 0.013 m (d) 1.33 mm
2. A point initially at rest moves along X-axis. Its
-2
acceleration varies with time as a = ( 3t - 2) ms . 5. A black body at a temperature of 127°C radiates
If it starts from origin, then the distance covered in heat at the rate of 9 cal cm -2 s -1. At a temperature
4s is of 527°C the rate of heat radiated per unit area in
(a) 12 m (b) 20 m cal cm -2 s -1 is
(c) 15 m (d) 28 m (a) 80 (b) 120 (c) 144 (d) 180

3. If a wire is stretched to make it 0.2% longer, its 6. The respective speeds of five molecules are 1, 2, 3,
resistance will be 4 and 5 km/s. The ratio of their rms velocity and
the average velocity will be
(a) increase by 0.4% (b) decrease by 0.4%
(a) 11 : 3 (b) 3 : 11 (c) 1 : 2 (d) 3 : 4
(c) increase by 01
. % (d) decrease by 01
. %

MODULE 2
www.jeebooks.in
Prep Catalysis for NEET ~ Mock Test 4 161

7. A coil in the shape of an equilateral triangle of side Mv 02 Mv 02 Mv 02


(a) zero (b) (c) (d)
0.02 m is suspended from its vertex, such that it is 2R 3R 5R
hanging in a vertical plane between the pole pieces
of permanent magnet producing a uniform field of 14. Work function of a metal is 5.2 ´ 10-18J, then its
5 ´ 10-2 T. If a current of 0.1 A is passed through threshold wavelength will be
the coil, what is the couple acting? (a) 736.7 Å (b) 760.7 Å (c) 301 Å (d) 344.4 Å
-7 -10
(a) 5 3 ´ 10 N-m (b) 5 3 ´ 10 N-m 15. A lift of mass 1000 kg is supported by thick steel
3 ropes. If maximum upward acceleration of the lift
(c) ´ 10-7 N-m (d) None of these
5 be 1.2 ms -2 and the breaking stress for the ropes
be 1.4 ´ 108 Nm -2, what should be the minimum
8. The radius of earth is 6400 km and g = 9.8 m/s 2. If diameter of rope?
the body placed at the equator has to become (a) 1 cm (b) 2 cm (c) 3 cm (d) 4 cm
weightless the earth should make one complete
rotation in 16. Maximum efficiency of half-wave rectifier is
(a) 12 h (b) 1.4 h (a) 100% (b) 80% (c) 50% (d) 40%
(c) 6 h (d) 24 h
17. The Rydberg constant for an electron revolving
9. A zener diode having breakdown voltage equal to around hydrogen atom is R. The Rydberg constant
15 V, is used in a voltage regulator. Circuit as for electron revolving about 10 times ionised
shown in the figure, the current through the sodium atom will be
diode is (a) R (b) less than R
(c) greater than R (d) arbitrary
(a) 5 mA (b) 10 mA 18. N small metal drops (identical) are charged to 10 V
(c) 15 mA (d) 20 mA each. If they coalesce to form a single big drop,
then its potential energy will be
10. A pipe of length 85 cm is closed from one end.
10 N2 / 3
The number of possible natural oscillations of (a) 10 N1/ 3 (b) 2/ 3
(c) 10 N2 / 3 (d)
air column in the pipe whose frequencies lie N 10
below 1250 Hz are 19. Two glass capillary tubes of length L and 3L and
(take, velocity of sound in air = 340 ms -1) 3R
radii R and are connected in series. The net
(a) 5 (b) 4 2
(c) 6 (d) 10 rate of flow of liquid through them will be (given,
rate of the flow through single glass capillary,
11. A uniform rod of length L and mass 2.0 kg is made ppR 4
to rest on two measuring scales at its two ends. A Q= )
uniform block of mass 2.8 kg is placed on the rod 8hL
L (a) 0.62 Q (b) 1.73 Q
at a distance of from its left end. The force
4 (c) 1.6 Q (d) 1.38 Q
experienced by the measuring scale on the right
end is
20. In an adiabatic change, the pressure and
temperature of a monoatomic gas are related as
(a) 10 N (b) 15 N
p µ T k, where k equals
(c) 17 N (d) 19 N
5 2 5 3
(a) (b) (c) (d)
12. Two pendulums of lengths 100 cm and 110.25 cm 2 5 3 5
start oscillating in phase. After how many
oscillations will they again be in same phase? 21. A source of emf 12 V and having negligible internal
resistance connected to a variable resistor, so that
(a) 20 oscillations of smaller pendulum
current in the circuit changes with time as
(b) 21 oscillations of larger pendulum
i = t - 1. Then, total charge transfer by the source
(c) 21 oscillations of smaller pendulum
(d) None of these in first second will be
(a) 0.46 C (b) 0.33 C
13. An insulated box containing a diatomic gas of (c) 2.1 C (d) 1.2 C
molar mass M moving with a speed v0 is suddenly
stopped. 22. A black hole is a body from whose surface nothing
can escape. For a body of mass 9 times that of
The increase in gas temperature as a result of earth the radius for being a black hole is
stopping the box is (take, mass of earth = 6 ´ 1024 kg and
G = 6.67 ´ 10-11 Nm 2 kg -2)

MODULE 2
www.jeebooks.in
162 NEET Test Drive

(a) 6 cm (b) 8 cm (c) 5 cm (d) 10 cm (a) 968 Nm -1 (b) 535.4 Nm -1 (c) 322.6 Nm -1 (d) 350 Nm -1
238 222
23. A radioactive element decays into
90 X 83 Y . 29. A motor of power P0 is used to deliver water at a
The number of b -particles emitted are certain rate through a given horizontal pipe. To
(a) 1 (b) 2 (c) 4 (d) 6 increase the rate of flow of water through the same
pipe n times, the power of the motor is increased to
24. The logic circuit shown below has the input P1. The ratio of P1 to P0 is
waveforms A and B as shown in the figure. Pick
(a) n : 1 (b) n2 : 1 (c) n3 : 1 (d) n4 : 1
out the correct output waveform.
A 30. A transparent cuboid contains a small air bubble,
which apparent distance is 1.5 cm when seen
Y through one face and 3.5 cm when seen through
B other face. If refractive index of transparent cuboid
is 1.5, then real length of the edge of cube is
(a) 6 cm (b) 5 cm (c) 9 cm (d) 7.5 cm

Input A 0 31. Consider the circular loop having current i and


with central point O. The magnetic field at the
central point O is
Input B 0 I
R
(a) 0 2R
O
II

(b) 0
i

(c) 0

2m 0 i 5m 0 i
(d) 0 (a) acting downward (b) acting downward
3p R 12 R
6m i 3m i
25. In which of the following case of spherical lenses (c) 0 acting downward (d) 0 acting upward
11 R 7R
(as shown in the figure), the emergent ray is
parallel to incident ray? 32. Two bodies of masses 2 kg and 5 kg are located at
(1, -2) and (1, 3) respectively. Then, the centre of
(a) R1 R2 (b) ∞ R1 mass have coordinates at
(a) æç1, ö÷ (b) æç , 1ö÷ (c) æç , ö÷ (d) æç , ö÷
11 11 1 11 11 1
R1 > R2 è 7ø è7 ø è7 7 ø è 7 7ø

33. If E , M , L and G denote energy, mass, angular


(c) R1 R2 (d) R1 R2 momentum and gravitational constant respectively,
æ EL2 ö
R1 < R2 R1 = R2 then the quantity ç 5 2 ÷ has the dimensions of
èM G ø
26. In a series L-C-R circuit with L = 2H, C = 8 mF and (a) time (b) length (c) mass (d) angle
R = 5 W, the Q-factor will be
34. Two waves each of amplitude a and frequency f
(a) 50 (b) 100 (c) 150 (d) 200
p
have a phase difference . The amplitude and
27. When a weight of 25 kg is suspended from a steel 2
wire of length 5m and radius 0.2 mm. Its length frequency of resultant wave due to their super
increases by 2.5 cm. If the diameter of the wire is position will be
doubled, then the extension in its length will be a f a f
(a) , (b) ,f (c) 2 a, (d) 2a, f
(a) 0.52 m (b) 0.62 cm (c) 0.75 cm (d) 0.83 cm 2 2 2 2
28. In a HCl molecule, the Cl can be treated on infinite 35. The temperature of a radiating body increases by
mass and H oscillating alone. If the oscillation of 50%. Then, the increase in amount of radiation
HCl molecule shows frequency of 7 ´ 1013 s -1, the emitted will be approximately
value of force constant is (a) 185° (b) 385° (c) 406° (d) 607°
(take, Avogadro number = 6 ´ 1026 per kg-mole)
MODULE 2
www.jeebooks.in
Prep Catalysis for NEET ~ Mock Test 4 163

36. Circular loop of a wire and a long straight wire required to be given to it, so that it escape from the
carry currents I c and I e , respectively as shown in earth is
the figure. Assuming that these are placed in the E
(a) 2E (b)
same plane. The magnetic field will be zero at the 2
centre of the loop when the separation H is (c) E (d) 3E

42. A block is dragged on a smooth plane with the help


R of a rope which moves with velocity v as shown in
the figure. The horizontal velocity of the block is
v
Ic Pulley
θ

m
H

(a) v (b) v sinq


Ie v v
(c) (d)
Straight wire sinq cos q
Ie R Ic R
(a) (b) 43. If a biconvex lens is cut into identical parts by a
Ic p Ie p horizontal plane xy as shown in the figure, then
pIc I p which of the following statement is correct?
(c) (d) e
Ie R Ic R

37. In the given figure parallel rays of light incident on


x y ⇒
spherical medium of radius of curvature R meet at
I. Then, refractive index of spherical medium is
Air µ Air (a) Focal length of each part will be equal and forms image
with same intensity.
I
(b) Focal length of each part will be same and forms image
Medium with reduced intensity.
(c) Focal length of each part will be different and forms image
(a) 2.5 (b) 1.5 with reduced intensity.
(c) 1.35 (d) 2 (d) None of the above
38. If one of the slits in Young’s double slit experiment 44. A mass m hangs with the help of a string wrapped
is covered with a black opaque paper, then around a pulley on a frictionless bearing. The
(a) the fringe width decreases pulley has mass m and radius R. Assuming pulley
(b) the bright fringes become fainter to be perfect uniform circular disc, the acceleration
(c) there will be uniform illumination all over the screen of the mass m, if the string does not slip on the
(d) diffraction pattern will be observed pulley is
3 2
39. In a non-uniform electric field, the electric dipole (a) g (b) g
2 3
experiences g
(a) torque only (b) force only (c) (d) g
3
(c) torque as well as net force (d) None of these
45. Total energy stored in the system given in figure is
40. The primary and secondary coils of a transformer
have 50 and 1500 turns respectively. If the 2µF
A B
magnetic flux f linked with the primary coil is
given by f = f 0 + 4t, where f is in weber, t is time 2µF 2µF 2µF 2µF
in second and f 0 is a constant, the output voltage
across the secondary coil is
(a) 90 V (b) 120 V (c) 220 V (d) 30 V 10 V
-4
41. A satellite is revolving around the earth, close to (a) 3 ´ 10 J (b) 2.4 ´ 10-4 J
its surface with a kinetic energy E, then the energy . ´ 10
(c) 12 -6
J (d) 2.4 ´ 10-6 J

MODULE 2
www.jeebooks.in

PART B CHEMISTRY
46. The volume of 1.5 M HCl required to completely 54. The equilibrium constant for the reaction,
react with 2.4 g Mg is PCl5 ( g) ¾® PCl3 ( g) + Cl 2 ( g) at 400 K if
Mg + 2HCl ¾® MgCl2 + H 2 ­ DH ° = 77.2 kJ mol -1 and DS° = 122 JK -1 mol -1 is
(a) 103 mL (b) 133 mL
(a) 1.958 ´ 10-4 (b) 2.931 ´ 10-6
(c) 24.6 mL (d) 139 mL -4
(c) 6.252 ´ 10 (d) 3.536 ´ 10-6
47. If proton, electron and a-particle have equal
kinetic energy, thier de-Broglie wavelengths would 55. For an octahedral complex, which of the following
be in the order: d- electron configuration will give maximum
(a) le = l p < l a (b) l a < l p < le CFSE?
(c) l a > l p < le (d) l p > le < l a (a) High spin, d 6 (b) Low spin d 4
(c) Low spin, d 5 (d) High spin, d 7
48. Among the following species, identify the
isostructural pairs. [NF3 , NO3- , BF3 , H3O, HN3 ]
+ 56. If at 350K the concentration of [H3O+ ] is 10-6 M,
then the value of K w at this temperature will be
(a) [NF3 , NO -3 ] and [BF3 , H3O + ]
(a) 10-13 M 2 (b) 10-6 M 2
(b) [NF3 , HN3 ] and [NO -3 , BF3 ] (c) 10-12 M 2 (d) 10-14 M 2
(c) [NF3 , H3O + ] and [NO -3 , BF3 ]
57. Which will not undergo S N 1 reaction?
(d) [NF3 , H3O + ] and [HN3 , BF3 ]
(a) Cl (b)
49. The strength of a solution for 6.82 g solute in Cl
250 mL solution is
(a) 32.42 g/L (b) 25.62 g/L Cl
(c) 27.28 g/L (d) 74.34 g/L (c) (d) Cl
50. The density of a gas is 1.964 g dm -3 at 273 K and
76 mm Hg. The gas is
(a) CH4 (b) CO 2 58. The major product obtained on reaction of phenol
(c) Xe (d) C 2H6 with sodium hydroxide and CO2 followed by
acidification is
51. In an octahedral crystal field, the t2 g orbitals are (a) pthalic acid (b) salicylic acid
(a) raised in energy by 0.4 Do (b) lowered in energy by 0.4 Do (c) benzoic acid (d) salicylaldehyde
(c) raised in energy by 0.5 Do (d) lowered in energy by 0.6 Do 59. Which of the following species will acts as a Lewis
52. Which of the following is not the assumption of acid but not as a Bronsted acid ?
kinetic theory of gases? (a) OH- (b) O 2- (c) BF3 (d) NH-2
(a) The actual volume of the gaseous molecules is negligible
60. Williamson’s synthesis of preparing dimethyl
as compared to the total volume of the gas
ether is an/a example of
(b) Molecules are perfectly elastic
(a) free radical substitution (b) SN2 mechanism
(c) The critical temperature is the measure of the kinetic
(c) SN 1 mechanism (d) electrophilic substitution
energy of the molecule
(d) The effect of gravity on motion of molecules is negligible 61. Which one of the following polymer is obtained by
condensation polymerisation?
53. The most reactive substrate towards S N 2 reaction
(a) Buna-N (b) Teflon
among the following is (c) Phenol-formaldehyde (d) Polyethene
Cl
(a) Cl (b) 62. Which of the following concentration term(s)
represents solubility of BaF2 in a solution of
Ba(NO3 )2?
Cl 1
(a) 2[NO -3 ] (b) [F - ]
(c) (d) Cl 2
(c) [Ba 2+ ] (d) [F - ]

MODULE 2
www.jeebooks.in
Prep Catalysis for NEET ~ Mock Test 4 165

63. Value of coefficient p, q, r, s and t in the following


redox reaction is (a) (b)
p Br2 + q OH - ¾® rBr - + sBrO3- + t H 2O
p q r s t p q r s t
(c) (d)
(a) 3 6 1 5 3 (b) 3 6 5 3 1
(c) 3 6 5 1 3 (d) 3 5 1 6 3
71. Which of the following species is the leaving group
64. Which of the following statement(s) is/are O
incorrect? ||
(a) Argentite and cuprite are oxides in the alkaline hydrolysis of CH3 CO C2H5 ?
(b) Calamine and siderite are carbonates
(c) Malachite and chalcopyrite are the copper oxides (a) C 2H5O - (b) C 2H+5
(d) Zinc blende and pyrites are sulphides O
||
65. Consider the following sequence of reaction : (c) CH3 C O - (d) CH-3
O3/H2O Ca(OH)2 ∆ NH2—NH2
OH–/glycol 72. The similarity between Li of group 1 and Mg of
group 2 is called a diagonal relationship . It arises
The end product of the given reaction is because
(a) Li and Mg atoms have similar sizes
(b) Li and Mg ions have similar sizes
(a) (b)
(c) Li and Mg have similar electronegativities
(d) All of the above

73. Four successive members of the first series of the


(c) (d) transition metals are listed below. For which one of
°
them, the standard potential E M 2
+/ M
value has a
66. 1 mole of FeC2O4 is oxidised by x mole of Cr2O72- in positive sign?
acidic medium. Here, x is (a) Co(Z = 27) (b) Ni(Z = 28)
(a) 3 (b) 1.5 (c) Cu(Z = 29) (d) Fe(Z = 26)
(c) 0.5 (d) 1.0 74. Which of the following reagent can be used to
67. The compound which cannot be stored in glass distinguish between propyne and propene ?
vessels is (a) AgNO 3 in NH3 (b) Alk. KMnO 4
(a) XeF2 (b) XeO 3 (c) Br2 in CCl 4 (d) Conc. H2SO 4
(c) XeF4 (d) XeF6 75. Which of the following statements is incorrect
68. Nucleophilic reagents are the species that are about sodium hydroxide ?
negatively charged or have an electron rich atom. (a) It is a white, translucent crystalline solid
They are called Lewis base. Which of the following (b) It is highly soluble in water to give a strong alkaline solution
has highest nucleophilicity ? which is bitter in taste, corrosive and soapy to touch
(a) F - (b) OH- (c) Crystals of NaOH are not hygroscopic
(c) CH-3 (d) NH-2 (d) If reacts with CO 2 in the atmosphere to form Na 2CO 3

69. Which of the following does not react with 76. Consider the following statements and choose the
dil. H 2SO4 to give H 2O2? incorrect one.
(a) SrO 2 (b) Na 2O 2 (a) Acetamide is reduced to ethylamine by LiAlH4
(c) BaO 2 (d) PbO 2 (b) Methyl isocyanide is hydrolysed to methylamine
(c) Methyl cyanide is reduced to methylamine by LiAlH4
70. Conjugated dienes are more stable than
(d) Homologated 1° amine is obtained in the process
non-conjugated dienes (both isolated and
comulated) due to factors such as delocalisation of 77. Which of the following compound can be most
charge through resonance and hybridisation easily sulphonated ?
energy. Which of the following compound is most (a) Chlorobenzene (b) Toluene
stable ? (c) Nitrobenzene (d) Benzene

MODULE 2
www.jeebooks.in
166 NEET Test Drive

78. The correct statement regarding cycloaddition (c) Micelle formation occurs above CMC
reactions is (d) Micelle formation occurs above Kraft temperature
(a) [2 + 2] is thermally allowed and [4 + 2] is
86. Radioactive material A has decay constant ‘8l’ and
photopchemically allowed
material B has decay constant ‘l’. Initially they
(b) [2 + 2] is photochemically allowed and [4 + 2] is thermally
have same number of nuclei. After what time the
allowed
ratio of number of nuclei of material B to that A
(c) Both [2 + 2] and [4 + 2] are thermally allowed
will be 1/e?
(d) Both [2 + 2] and [4 + 2] are photochemically allowed 1 1
(a) (b)
79. The general electronic configuration for second row 8l 7l
1 1
transition series is (c) (d)
(a) [Xe] 5d 1 -10 , 5s1- 2 (b) [Ar] 5d 1 -10 , 4s1- 2 9l l
1 -10 1- 2
(c) [Kr] 4d , 5s (d) [Ne] 3 d 1 -10 , 4s 2 87. Match the given compound to number of unpaired
electrons
80. Boric acid is used in carom boards for smooth
gliding of pawns because Compound Number of
(a) H3BO 3 molecules are loosely chemically bonded and Unpaired Electrons
hence, it is soft A. [Cr (H2O)6 ]3 + 1. 3
(b) Its low density maked it fluffy
(c) It can be powdered to a very small grain size. B. [ Fe(H2O)6 ]2+ 2. 4
(d) H-bonding in H3BO 3 give it a layered structure. 2+
C. [ Zn(H2O)6 ] 3. 0
81. Packing fraction of simple cubic crystal lattice is
D. [ Fe(CN)6 ]4- 4. 0
(a) 38% (b) 74%
(c) 68% (d) 52.4% Codes
82. 10Cl ( aq ) + 2MnO 4- ( aq ) + 16H + ( aq )
-
= A B C D
(a) 1 2 3 4
5Cl 2 ( g) + 2Mn 2 + ( aq ) + 8H 2 O( l ) . The
value of E° for
(b) 3 4 1 2
this reaction is 0.15 V. What is the value of the
equilibrium constant ( K ) for this reaction ? (c) 2 1 4 3
(a) 2 .6 ´ 1025 (b) 4.9 ´ 1012 (d) 4 3 1 2
(c) 1.2 ´ 105 (d) 3.4 ´ 102 88. The products obtained when copper metal is
reacted with HNO 3 is
83. The form of carbon which is used as black pigment (a) HNO 2 or N2 (b) NO or NO 2
in black ink and filler in automobile tyres is (c) NO 2 or N2O 5 (d) NO or N2O 5
(a) graphite (b) charcoal
(c) coke (d) carbon black 89. Glucose on reaction with ammoniacal silver nitrate
gives silver mirror because it has
84. If the rate constant of a reaction with virus is (a) alcoholic silver nitrate (b) ester group
4.8 ´ 10-4 s -1, then the time required for the virus (c) aldehyde group (d) ketone group
to become 80% inactivated is
(a) 48 mins (b) 62 mins 90. Dehydration of alcohols to alkenes on heating in
(c) 39 mins (d) 56 mins presence of conc. H 2 SO 4 involves
(a) protonation of alcohol molecule
85. Which of the following statement is incorrect (b) formation of an ester
regarding micelle? (c) formation of carbocation
(a) Aggregation of particles is known as micelle (d) elimination of water
(b) Micelle formation occurs below CMC

MODULE 2
www.jeebooks.in

PART C BIOLOGY
91. Which type of evolution is shown by wings of 100. In which of the following conversions does
butterfly and birds? oxidative decarboxylation take place?
(a) Convergent (b) Divergent (a) Succinic acid → Fumaric acid
(c) Adaptive radiation (d) Chemical evolution (b) Pyruvate → Acetyl Co-A
(c) Fumarate → Malate
92. Which among the following is not a RNA virus?
(d) Isocitrate → Citrate
(a) Avian leukemia virus (b) Bacteriophage MS-2
(c) Adenoviruses (d) Influenza virus 101. The substances like fructose and some amino acids
are absorbed with the help of the carrier molecules.
93. Which of the following carbohydrates is an This mechanism is known as the
oligosaccharide?
(a) simple diffusion (b) facilitated transport
(a) Glyceraldehyde (b) Raffinose
(c) active transport (d) osmosis
(c) Dextrin (d) Inulin
102. Monascus purpureus is used to produce
94. Which among the following is not an effect of
(a) cyclosporin-A (b) statins
deforestation?
(c) riboflavin (d) L-ascorbic acid
(a) Loss of biodiversity
(b) Disturbs the hydrologic cycle 103. In glycolysis, which of the following reactions is
(c) Soil erosion catalysed by enzyme aldolase?
(d) Genetic deformities (a) Fructose-6-phosphate → Fructose 1, 6- biphosphate
(b) Dihydroxyacetone phosphate → Glyceraldehyde
95. Which among the following statements is/are
3-phosphate
correct regarding velamen?
(c) Fructose 1, 6-biphosphate → Glyceraldehyde
(a) Its cells comprise of fibrous thickenings
3-phosphate+ Dihydroxyacetone phosphate
(b) It is highly hygroscopic
(d) 2-phosphoglycerate → Phosphoenol pyruvate
(c) Outermost tissue of the root
(d) All of the above 104. During photolysis of water, the oxygen evolving
complex contains the following elemental ions.
96. Which among the following is an incorrect (a) Mg 2+ ,Cl − and Ca 2+ (b) Mg 2+ ,K+ and Cl −
statement? (c) Mn2+ ,Cl − and Ca 2+ (d) Mn2+ ,K+ and Cl −
(a) Runner has long internodes
(b) Stolon grows obliquely upwards and then bends down 105. What are pyrenoids associated with?
(c) Offset develops from an axillary bud of an underground (a) Starch storage (b) Photosynthesis
part of stem (c) Fatty acid oxidation (d) Enzyme secretion
(d) None of the above
106. Apical cell theory states that
97. An immunoglobin molecule contains two each of I. a single apical cell is the structural and
the heavy and light chains. Among these, the functional unit of apical meristem.
antigen binding site is present in II. it governs the whole process of primary
(a) constant region of both the light and heavy chains growth.
(b) variable region of light chain III. found in lower plants like algae and bryophytes.
(c) variable region of heavy chain IV. histogens form vascular strand.
(d) variable region of both the light and heavy chains
Choose the correct option.
98. The schizont stage of Plasmodium parasite in (a) I and II (b) III and IV
human occurs in (c) I, II and III (d) II, III and IV
(a) liver cells
(b) spleen 107. Immunity is a state of protection from foreign
(c) liver and erythrocytes
molecules. It is of two types, i.e. innate and
adaptive immunity. Specificity, diversity , memory
(d) liver, spleen and bone marrow
are characteristics of
99. Which of the following is a function of boron? (a) innate immunity
(a) Sugar metabolism (b) non-specific immunity
(b) Nitrogen metabolism (c) acquired immunity
(c) Activation of enzymes (d) external defence
(d) Acting as enzyme cofactor

MODULE 2
www.jeebooks.in
168 NEET Test Drive

108. Match the items given in Column I with those in 114. Hydatid cyst is the larvae of
Column II and select the correct option. (a) Rickettsia (b) Taenia solium
Column I Column II (c) Phlebotomus sp. (d) Echinococcus

A. Root pressure 1. Cohesion and adhesion of water 115. Early diagnosis of diseases cannot be done by
molecules (a) recombinant DNA technology
(b) polymerase chain reaction
B. Capillary action 2. Evaporation of water from leaf
surface (c) enzyme linked immuno sorbent assay
(d) urine analysis
C. Transpiration 3. Entry of water into xylem by
pull osmosis 116. The pollen tube contains vegetative nucleus and a
generative nucleus which are identical because
Codes
(a) they are products of amitosis
A B C A B C
(b) they are products of mitosis
(a) 1 3 2 (b) 1 2 3
(c) they are products of meiosis
(c) 3 2 1 (d) 3 1 2
(d) they are products of meiosis-I
109. The advantage of a DNA molecule having two
complementary strands is that 117. Fertilizin is a compatibility receptor protein found
(a) easy pairing between chromatids
in
(a) inner plasma membrane (b) zona pellucida
(b) transcription and translation processes can occur
simultaneously (c) corona radiata (d) ooplasm
(c) semiconservative replication 118. Match Column I with Column II and choose the
(d) None of the above correct option from the codes given below.
110. Which of the following is an enzyme which Column I Column II
recognises specific base sequences in DNA and cut (Type of stem) (Characteristic)
the DNA at that site? A. Monocot stem 1. Vascular bundle closed
(a) Restriction enzymes (b) DNA ligase
B. Dicot stem 2. Vascular bundle open
(c) Helicase (d) Primase
C. Pith in monocot stem 3. Pith absent
111. Which among the following statements is/are
D. Pith in dicot stem 4. pith made up of
correct regarding gemmules?
parenchymatous cells
I. Gemmules arise as modified branches.
II. In case of Marchantia, the gemmae are Codes
multicellular. A B C D A B C D
(a) 1 2 3 4 (b) 2 1 3 4
III. In case of algae, the gemmae occur in
(c) 4 1 3 2 (d) 4 3 1 2
Sphacelaria.
IV. In bryophytes, the gemmae are approximately 119. Heterostyly is an outbreeding device in which,
not present in Bryopsida. (a) pollen grains are unable to germinate on the stigma of
same flower
Choose the correct option. (b) pollen grains mature earlier than the receptive time of
(a) I and II (b) III and IV (c) I, II and III (d) All of these stigma`
112. Which among the following is/are charcteristic of (c) style and stamens in a flower are at different heights
imbibition? (d) structure of pollen sac and stigma functions as hurdles
(a) Development of pressure (b) Production of heat 120. Select the incorrect statements.
(c) Volume change (d) All of these
I. Bulbils are fleshy buds produced in the axil of
113. A mango merchant desires to obtain all the foliage leaves in place of axillary buds.
mangoes in his orchard to have the taste and II. Segments of Opuntia and other cacti produce
quality of alphonso mango. He tries to achieve this new plants after falling on the ground.
by using the grafting technique. Which variety III. In Bryophyllum, marginal buds form plantlets
would be used for the scion and the stock, while they remain attached to the plants.
respectively, to get the desired result?
IV. Fleshy adventitious roots take part in
(a) Scion of desi mango on stock of alphonso mango
vegetative propagation in onion.
(b) Scion of alphonso mango on stock of desi mango
(c) It can be either way out depending on the age of the plants Choose the correct option.
(a) I and II (b) III and IV
(d) Scion of desi mango on stock of both desi and alphonso
are combined (c) Only II (d) Only IV

MODULE 2
www.jeebooks.in
Prep Catalysis for NEET ~ Mock Test 4 169

121. Repeated selfing will lead to 128. Starch is made up of two polymers.
(a) increased heterozygosity (a) amylose, amlodipine (b) amylose, amylopectin
(b) increased homozygosity (c) glucose, fructose (d) glucose, sucrose
(c) production of hybrids
129. Match Column I with Column II and choose the
(d) Homozygosity in some and heterozygosity in other traits
correct option from the codes given below.
122. In prophase-I of meiosis, during which substage
Column I Column II
can terminalisation be observed?
(Reproductive organ) (Characteristic)
(a) Pachytene (b) Diplotene
(c) Diakinesis (d) Karyokinesis A. Protonema 1. Vegetative propagation
B. Antheridium 2. Male sex organ
123. Given below are different types of microspore
tetrads with their examples. Select the option, C. Archegonium 3. Female sex organ
which is matched incorrectly. D. Sporophyll 4. Bears sporangia
(a) Decussate – Magnolia
Codes
(b) T-shaped – Crocus
(c) Isobilateral – Aristolochia elegans A B C D A B C D
(d) Linear – Halophia (a) 1 2 3 4 (b) 2 1 3 4
(c) 1 2 4 3 (d) 2 1 4 3
124. Choose the incorrect statement(s).
130. Creatine excretion is probably associated with
I. Dislocation involves a damage to the
ligaments surrounding the joints. I. histolysis.
II. histogenesis.
II. Sprain is a twisting of joint without
III. seen in foetus and pregnant women.
dislocating it .
IV. presence of benzoic acid in diet.
III. Muscular dystrophy is an autoimmune
The correct option is/are
disorder.
(a) I, II and III (b) III and IV (c) I and II (d) I and III
IV. Gout is an inherited disorder of purine
metabolism. 131. Identify the incorrectly matched pair.
(a) I and II (b) Only III (a) Dairy — Bacterium lactici acidi
(c) III and IV (d) II and III (b) Vinegar industry — Acetobacter aceti
(c) Alcohol industry — Clostridium acetobutylicum
125. Which among the following statements is incorrect (d) Tobacco industry — Mycococcus codisans
regarding gymnosperms?
(a) Seeds are naked 132. Which among the following are the chracteristics of
(b) Double fertilisation is absent resemblance between gymnosperms and
angiosperms?
(c) Xylem consists of vessels and tracheids
(d) Phloem consists of sieve cells I. Secondary growth is not present in roots.
II. Secondary wood may be monoxylic or polyxylic.
126. Which of the following statements is/are correct for III. Presence of ovule.
Abscisic Acid (ABA)?
IV. Presence of endosporic and reduced
(a) ABA is translocated by both xylem and phloem
gametophytes.
(b) It is synthesised in mature old leaves
Choose the correct option.
(c) It is synthesised from mevalonic acid
(a) I and II (b) II and III (c) III and IV (d) II, III and IV
(d) All of the above
133. Which of the following is not a hormone releasing
127. Identify the correctly matched pair. Intra Uterine Device (IUD)?
I. Acoelom – There is no body cavity (a) Progestasert (b) LNG-20
II. Pseudocoelom – Coelom is not lined by mesoderm (c) Cervical cap (d) Mirena

III. Eucoelom – Body cavity develops entirely 134. Which among the following statements are correct
within the mesoderm regarding cyanobacteria?
IV. Enterocoel – Deuterostomes are (a) Possess prokaryotic type of cells
enterocoeloms (b) Incipient nucleus
(c) Asexual reproduction takes place by fission and
(a) I and II (b) III and IV
fragmentation
(c) II, III and IV (d) All of these
(d) All of the above

MODULE 2
www.jeebooks.in
170 NEET Test Drive

135. Which among the following is the criteria for an 143. Which of the following is in the proper order for
element to be considered as essential? the vectors given below in terms of increasing
I. The element must be essential for normal cloning capacity?
growth and reproduction. (a) BAC, cosmid, phage, plasmid, YAC
II. The requirement of element must be specific. (b) YAC, BAC, cosmid, phage, plasmid
III. It cannot be replaced by any another element. (c) Plasmid, cosmid, phage, BAC, YAC
(d) Plasmid, phage, cosmid, BAC, YAC
IV. The requirement must be direct that is, not
the result of any indirect effect. 144. The presence of HCl in the stomach helps the
Choose the correct option. digestion process in which of the following ways?
(a) I and II (b) III and IV (a) Kills microorganims of food
(c) II and III (d) I, II, III and IV (b) Maintains the pH of the stomach
(c) Preserves the food when in stomach
136. The post-transcriptional modification involves (d) All of the above
I. Methyl guanosine triphosphate, an unusual
nucleotide is added to the 5' end of hnRNA. 145. Match the items in Column I (National parks /
Biosphere reserves) with Column II (location) and
II. Adenylate residues (200-300) are added at
select the correct option.
3'-end in a template independent manner.
III. Processed hnRNA is now called mRNA. Column I Column II
IV. mRNA is transported out of the nucleus. A. Keoladeo National Park 1. Odisha

Choose the correct option. B. Dachigam National Park 2. Bharatpur


(a) I and II (b) IV and III (c) II, III and IV (d) I, II III and IV C. Similipal 3. Kashmir
137. Human dentition is correctly described by which Codes
set of terminologies? A B C A B C
(a) Thecodont, diphyodont, homodont (a) 2 3 1 (b) 1 2 3
(b) Pleurodont, acrodont, homodont (c) 3 1 2 (d) 3 2 1
(c) Thecodont, diphyodont, heterodont
146. Choose the correct statements regarding sucrose.
(d) Pleurodont, acrodont, heterodont
I. It is a monosaccharide.
138. Which among the following is/are an evidence for II. It is a reducing sugar.
natural selection? III. It is a disaccharide made up of glucose and
(a) Rate of reproduction and competition in nature fructose.
(b) Production of new varieties of plants and animals
IV. Glucose and fructose are joined together by
(c) Mimicry and protective colouration
α-1,2 glycosidic linkage.
(d) All of the above
(a) Only II (b) Both II and III
139. Contagium vivum fluidum for viruses was given by (c) Both III and IV (d) None of these
(a) Mayer (b) Ivanowski (c) Stanley (d) Beijerinck
147. Okazaki fragments are
140. The five types of histone proteins are (a) lagging strand
(a) H1 ,H2 A,H2B,H3 and H4 (b) H1 ,H2 ,H3 ,H4 and H5 (b) each Okazaki fragment has its own RNA primer
(c) H1 A,H1B, H2 , H3 and H4 (d) H1 , H2 ,H3 A,H3B and H4 (c) replicated in a discontinuous manner
141. In the apoplast pathway, (d) All of the above
(a) water moves exclusively through the cell wall without 148. Arrange the following processes of biotic succession
crossing any membrane in a sequence.
(b) water travels from one cell to the next via the I. Nudation II. Migration
plasmodesmata
(c) water crosses at least two membranes
III. Ecesis IV. Aggregation
(d) transport across the tonoplast is also involved V. Competition VI. Reaction
VII. Stabilisation
142. During implantation, which of the following does
not occur? Choose the correct option.
(a) blastocyst attaches to the uterine wall (a) I → II→ III → IV → V → VI → VII
(b) the trophoblast cells produce enzymes that allow the (b) II → I → III → IV → V → VI →VII
blastocyst to ‘eat its way’ into the thick endometrium (c) II → I → IV → III → V→ VI → VII
(c) by 12th to 20th day, the blastocyst is completely implanted (d) I → II → IV → III → V → VI → VII
(d) None of the above

MODULE 2
www.jeebooks.in
Prep Catalysis for NEET ~ Mock Test 4 171

149. Nicotine is used for its stimulatory action. This is (c) Deamination is the removal of amine group from amino
because it mimics the effect of acids
(a) testosterone (b) dopamine (d) Nephritis is an inflammation of the urinary bladder
(c) adrenaline (d) acetylcholine 157. The factors responsible for Coronary Artery
150. The cross of superior males of one breed with Disease (CAD) include
superior females of another breed is referred to as (a) hypercholesterolemia (b) hypertension
(a) outcrossing (b) cross-breeding (c) diet low in antioxidants (d) All of these
(c) inbreeding (d) interspecific hybridisation
158. Which among the following statements is
151. Match the following columns and choose the incorrect?
correct option from the codes given below. (a) Luciferase enzyme responsible for light generation in
fireflies is present in glyoxysomes
Column I Column II (b) Peroxisomes are only seen in C 3 -plants where a wasteful
(Term) (Definition) phenomenon of photorespiration is seen
A. Maximum 1. Maximum production of (c) Glyoxysomes are more prominent in plant seedlings
natality individuals (d) Lomasomes are found to be associated with cell wall
formation in plant cells
B. Ecological 2. Increase in population under an
natality actual existing specific condition 159. The CO2 fertilisation effect relates to
C. Minimum 3. Minimum loss under non-limiting (a) increased atmospheric concentration of CO 2
mortality conditions (b) increased productivity of C 3 -plants
(c) decrease in the rate of transpiration
D. Ecological 4. Actual loss of individuals under
(d) All of the above
mortality given environmental condition
160. A person whose blood group is A, can have the
Codes genotype as
A B C D A B C D (a) IA IB ,IA IB (b) IA IB ,IO IO
(a) 1 2 3 4 (b) 1 2 4 3 (c) IA IA ,IA IO (d) IA IO , IB IO
(c) 1 3 2 4 (d) 3 1 2 4
161. A graphic representation of energy flow from one
152. Intake and release of O2 from blood depends upon
trophic level to next is shaped like a pyramid
(a) difference in the partial pressures of O 2 between the because
circulating fluid and static media (a) organisms at each level store most of the energy
(b) CO 2 tension (b) there are more producers than primary consumers
(c) H+ ion concentration (c) eventually organisms die as they age
(d) All of the above (d) most energy at each level is lost, leaving little for the next
153. Blind sac body plan is not characterised by level
(a) cells being organised into tissues 162. Identify the incorrectly matched pair.
(b) division of labour
(a) Flexor — Muscles, which bend one part of the
(c) single opening of the body for ingestion and egestion
body over the other
(d) presence of two openings for digestive system
(b) Abductor — Muscles, which pull a limb away from
154. Ecological succession is characterised by the median line
(a) formation and increase in thickness of soil (c) Depressor — Muscles, which lower some part
(b) increase in biomass (d) Supinator — Muscle that turns the palm downward
or backward
(c) higher species diversity
(d) All of the above 163. Different alleles of a gene are found on
(a) different chromosomes
155. The function of DNA gyrase includes
(b) same chromosome
(a) synthesis of RNA primers
(c) homologous chromosomes
(b) erases primer and fills gaps
(d) same DNA strand
(c) relieves torque or strain
(d) stabilises single-stranded regions 164. The RNA interference (RNAi) had been adopted to
prevent the infestation of
156. Identify the incorrect statement. (a) Escherichia coli
(a) Atrial natriuretic factor works opposite to RAAS (b) Meloidogyne incognita
(b) The colour of urine is dependent upon the pigment (c) Bacillus thuringiensis
urochrome
(d) Lepidopterans

MODULE 2
www.jeebooks.in
172 NEET Test Drive

165. Identify the incorrectly matched pair. (c) Biotechnology may pose unforeseen risks to the
environment
(a) Palaeocene — Modernisation of angiosperms
(d) All of the above
(b) Cretaceous — Birth of modern reptiles
(c) Miocene — Mammals at height of evolution 172. All-or-none law states that
(d) Ordovician — Land plants probably first appeared I. the response of a muscle or nerve to a stimulus
166. Read the following statements. is not proportionate to intensity of stimulus.
I. Grave’s disease is due to hypersecretion of II. the response of a muscle or nerve to a stimulus
thyroxine. is proportionate to intensity of stimulus.
II. Cretinism is due to hyposecretion of thyroxine. III. the degree of contraction shows independence
III. Acromegaly is due to hyposecretion of with the intensity of stimulus.
parathyroid. IV. a single muscle fibre does not show any
IV. Tetany is due to hyposecretion of somatostatin. gradation in contraction in relation to degree of
stimulus.
Choose the correct option.
The correct statement(s) is/are
(a) I and II are correct (b) III and IV are correct
(a) I, III and IV (b) II and III (c) III and IV (d) I and III
(c) I and II are incorrect (d) II and III are incorrect

167. Choose the incorrect statement regarding 173. Which among the following statements is/are
correct regarding population density?
earthworm.
(a) Some newly introduced species reach high population
(a) Chloragogen cells are related to storage and synthesis of
densities
glycogen and fat
(b) Species that use abundant resources often reach higher
(b) Mucocytes are elongated cells with fan-like processes
population densities
(c) Phagocytes are amoeboid in nature
(c) Species with small individuals generally reach higher
(d) Circular cells are enucleated
population densities
168. Which of the following is a case of recessive (d) All of the above
epistasis?
174. Bioherbicides are
(a) Fruit colour of summer squash
(b) Coat colour of mice
I.
organisms which destroy weeds.
(c) Flower colour in sweet pea II.
they do not harm the useful plants.
(d) Shape of comb in poultry III.
sporeine is bioherbicide.
IV.cochineal insect was used for the control of
169. Cilia are present in cacti.
I. epithelium of respiratory tract The correct statement(s) is/are
II. oviduct (a) I and II (b) I, II and IV
III. epithelial cells of epididymis (c) II and III (d) II, III and IV
IV. vas deferens
Choose the correct option. 175. The advantages of inbreeding include
(a) I and II (b) III and IV I. it increases homozygosity.
(c) II and III (d) I and IV II. it exposes harmful recessive genes that are
eliminated by selection.
170. A couple have a haemophilic son, a normal son and III. superior genes can be accumulated.
a haemophilic daughter. What are the most likely
genotypes of the parents in this case? The correct statement(s) is/are
Mother’s genotype Father’s genotype (a) I and II (b) II and III (c) I and III (d) I, II and III
(a) XX h Xh Y 176. Scotopic vision is the function of
(b) XX XY
(a) cones (b) rods (c) fovea (d) lens
(c) X h X h XhY h
(d) X h Y XY h 177. Arrange the following in order of basic steps in
genetic engineering.
171. Which among the following is a major bioethical I. Identification of DNA with desirable genes.
concern pertaining to genetic modification of
organisms? II. Maintenance of introduced DNA in host.
(a) Use of animals in biotechnology causes immense III. Introduction of identified DNA.
sufferings and pain to them Choose the correct option.
(b) Transfer of human genes into animals and plants weakens (a) I → II → III (b) I → III → II
the concept of humanity (c) II → I → III (d) II → III → I
MODULE 2
www.jeebooks.in
Prep Catalysis for NEET ~ Mock Test 4 173

178. Which component of neuron is associated with 180. Which among the following statements are correct
enzyme formation required for synthesis of regarding archaebacteria?
neurotransmitters? I. They are obligate anaerobes.
(a) Neurofibrils II. Their genetic material floats freely in the
(b) Schwann’s cells cytoplasm as they do not contain nuclei.
(c) Axon hillock
III. They do not contain membrane bound
(d) Nissl’s granules
organelles.
179. In spermiogenesis, IV. They are found in harsh habitats such as
(a) no change in number of cells as only one spermatid extreme salty areas, hot springs and marshy
develops into a spermatozoa areas.
(b) process of formation of sperms from immature germ cells Choose the correct option.
(c) number of cells increases (a) I and II (b) III and IV
(d) sperms released from the seminiferous tubules (c) I, II and III (d) I, II, III and IV

Answer Sheet
1. (c) 2 (d) 3. (a) 4 (b) 5. (c) 6. (a) 7. (a) 8. (b) 9. (a) 10. (c)
11. (c) 12. (c) 13. (d) 14. (d) 15. (a) 16. (d) 17. (c) 18. (c) 19. (c) 20. (a)
21. (b) 22. (b) 23. (a) 24. (b) 25. (a, d) 26. (b) 27. (b) 28. (c) 29. (a) 30. (d)
31. (b) 32. (a) 33. (d) 34. (d) 35. (c) 36. (a) 37. (d) 38. (d) 39. (c) 40. (b)
41. (c) 42. (c) 43. (b) 44. (b) 45. (a) 46. (b) 47. (b) 48. (c) 49. (c) 50. (a)
51. (a) 52. (c) 53. (a) 54. (a) 55. (c) 56. (c) 57. (b) 58. (b) 59. (c) 60. (b)
61. (c) 62. (b) 63. (c) 64. (a) 65. (c) 66. (c) 67. (d) 68. (c) 69. (d) 70. (a)
66. (c) 67. (d) 68. (c) 69. (d) 70. (a) 71. (a) 72. (d) 73. (c) 74. (a) 75. (c)
71. (a) 72. (d) 73. (c) 74. (a) 75. (c) 76. (c) 77. (b) 78. (b) 79. (c) 80. (d)
81. (d) 82. (a) 83. (d) 84. (d) 85. (b) 86. (b) 87. (a) 88. (b) 89. (c) 90. (a)
91. (a) 92. (c) 93. (b) 94. (d) 95. (d) 96. (c) 97. (d) 98. (c) 99. (a) 100. (b)

101. (b) 102. (b) 103. (c) 104. (c) 105. (a) 106. (c) 107. (c) 108. (d) 109. (c) 110. (a)
111. (d) 112. (d) 113. (b) 114. (d) 115. (d) 116. (b) 117. (b) 118. (a) 119. (c) 120. (d)
121. (b) 122. (c) 123. (b) 124. (b) 125. (c) 126. (d) 127. (d) 128. (b) 129. (a) 130. (a)
131. (d) 132. (d) 133. (c) 134. (d) 135. (d) 136. (d) 137. (c) 138. (d) 139. (d) 140. (a)
141. (a) 142. (c) 143. (d) 144. (d) 145. (a) 146. (c) 147. (d) 148. (a) 149. (d) 150. (b)
151. (a) 152. (d) 153. (d) 154. (d) 155. (c) 156. (d) 157. (d) 158. (a) 159. (d) 160. (c)
161. (d) 162. (d) 163. (c) 164. (b) 165. (d) 166. (a) 167. (d) 168. (b) 169. (a) 170. (a)
171. (d) 172. (a) 173. (d) 174. (b) 175. (d) 176. (b) 177. (b) 178. (d) 179. (a) 180. (d)

MODULE 2
www.jeebooks.in

EXPLANATIONS

PHYSICS
1. The percentage increase in the radius v12 + v 22 + v 32 + ¼ + v n2 10. For closed organ pipe,
. -1
101
6. vrms = (2n + 1)v
= ´ 100 = 1% n f = , n = 0, 1, 2, ¼
1 2 2 2 2 2 4l
1 + 2 + 3 + 4 +5
Time period of satellite, T µ r 3 / 2 = (2n + 1)v
5 Þ < 1250
\ Percentage change in time period 4l
= 11 km/s 1250 ´ 4 ´ 0.85
3 Þ (2n + 1) <
= (% change in radius) v + v2 + v3 + ¼ + vn
2 v avg = 1 340
n
3 Þ (2n + 1) < 12 .5
= ´ 1% = 15. % 1+ 2 + 3 + 4 + 5
2 = = 3 km/s Þ n < 5.75
5
2. As acceleration, a = 3t - 2 \ n = 0, 1, 2, 3, 4, 5
vrms 11
dv \ = = 11 : 3 So, number of oscillations = 6
Þ a= = 3t - 2 v avg 3
dt 11. As the rod is at rest, so torque about the
Þ dv = (3t - 2)dt 7. Torque, t = iAB sin q, i = 01
. A, q = 90° left end should be zero, i.e.
3t 2 1 L L
Þ v = - 2t + c A= ´ Base ´ Height t = 28 ´ + 20 ´ - F ´ L = 0
2 2 4 2
1 a 3 Left end L
(c is integration constant) or A= a´
2 2
At t = 0, v = 0 Þ c = 0
3 a2 3 ´ (0.02) 2 Right end
ds 3t 2 = = 28 N 20 N
Þ v = = - 2t 4 4
dt 2 L/4 L/4 L/2
æ 3t 2 ö = 3 ´ 10-4 m 2 , q = 90°
Þ ds = ç - 2t ÷ dt t = 01
. ´ 3 ´ 10 -4
´ 5 ´ 10 -2
sin 90° 28L 20L
è 2 ø Þ FL = +
= 5 3 ´ 10-7 N-m 4 2
s 4
æ 3t 2 ö Þ F = 7 + 10 = 17 N
Þ ò ds = ò çè 2
- 2t ÷
ø
8. The body will be weightless if weight is
12. For larger pendulum,
0 0 balanced by centrifugal force
4
é 3t 3 2t 2 ù i.e. mRw 2 = mg n=
1 g
=
1 g
Þ s =ê - ú 2p l 2 p 110.25
6 2 û0 g
ë Þ w=
R For smaller pendulum,
= 32 - 4 = 28 m
R 22 6400 1 g
T = 2p =2´ ´ n + 1=
3. For given volume of wire, g 7 9.8 2 p 100
DR Dl
=2 = 2 ´ 0.2% = 0.4% ~
- 5080 s ~
- 14
. h n+1 110.25 100 + 10.25
R l \ = =
n 100 100
Resistance of a wire increase with 9. The voltage drop across 1 kW 1/ 2
increase in length. 1 æ 10.25 ö
= V2 = 15 V Þ 1+
= ç1 + ÷
I 250 Ω n è 100 ø
4. Here, d = d1d 2 + 1 10.25
I2 I′1 = 1+ ´ = 1 + 0.05
= 0.03 ´ 012
. = 0.06 mm 2 100
= 6 ´ 10-5 m 20 V
Þ n=
1
= 20
Dl 1 kΩ 0.05
\ Fringe width b =
d 13. Decrease in kinetic energy = Increase in

2 ´ 400 ´ 10-9 internal energy of the gas
= The current through 1 kW is
6 ´ 10-5 1 m æ5 ö
I¢ =
15 V
= 15 ´ 10-3 A = 15 mA mv 02 = nCV DT = ç R ÷ DT
. ´ 10-2 m
= 133 1 ´ 10-3 W
2 M è2 ø
= 13.3 mm Mv 02
The voltage drop across 250 W DT =
5. According to Stefan’s law, 5R
= 20 V - 15 V = 5 V
E µT 4 The current through 250 W is 14. Work function,W =
hc
E T 4 (127 + 273) 4 (400) 4 l
So, 1 = 14 = = 5V
I = = 0.02 A = 20 mA hc 6. 6 ´ 10-34 ´ 3 ´ 108
E 2 T2 (527 + 273) 4 (800) 4 250 W l= =
W 5.26 ´ 10-18
E 2 = 16 ´ E 1 The current through Zener diode is
= 16 ´ 9 = 144 cal cm -2 s -1 I 2 = I - I ¢ = (20 - 15) mA = 5 mA = 344.4 Å

MODULE 2
www.jeebooks.in
Prep Catalysis for NEET ~ Mock Test 4 175

. ms -2 ,
15. Here, m = 1000 kg, a = 12 20. For adiabatic change, 26. Resonant frequency, wr =
1

Tension in the rope is p 1- gT g = constant LC


-g g 1
T = m (g + a) =
Þ p µT 1 - g Þ p µT g -1
2 ´ 8 ´ 10-6
= 1000(9.8 + 12
. ) = 11000 N
g
Force \ Value of constant, k = wr = 250 rad/s
Breaking stress = g -1
Area w L 250 ´ 2
\ Q = r = = 100
T 4T 5 R 5
= = g =for monoatomic gas
p D 2 / 4 pD 2 3 FL FL
5/3 5 27. Extension of wire = =
4 ´ 11000 ´ 7 \ k = = AY pr 2Y
. ´ 108 =
14 5/3 - 1 2
22 ´ D 2 1
So, l µ 2 [QF , L, Y are constant]
2 ´ 1000 ´ 7 21. Given, i = t - 1Þ i = t 2 - 2t + 1 r
Þ D2 = 2
. ´ 108
14 \ Charge transfer in first second, l 2 æ r1 ö æ 1 ö
=ç ÷=ç ÷
1 l1 è r 2 ø è 2 ø
= 10-4 m 2 q = ò i dt = 2

Þ D = 10 -2
m = 1cm
ò (t - 2t + 1) dt
l2 =
l1
=
2.5
= 0.62 cm
0
1
4 4
16. Efficiency of half wave rectifier, ét 3 ù 1
40.6 = ê - t 2 + t ú = = 0.33 C 28. Frequency, n = 7 ´ 1013 s -1
h= ë 3 û0 3
r M 1
1+ 1 Mass of H-atom, m = = kg
r2 22. The escape velocity for a black body is N 6 ´ 1026
where, r1 = forward bias resistance and 2 GM 2 GM 1 k
ve = £ c ÞR = As, n=
r 2 = load resistance R c2 2p m
As r 2 (max) = ¥ ; 2 ´ (6.67 ´ 10-11) ´ (9 ´ 6 ´ 1024 ) 1 k
= Þ n2 = or k = 4 p 2mn 2
So, hmax = 40.6% (3 ´ 108 ) 2 4p2 m
2
17. Rydberg constant, R =
R10 = 80.04 ´ 10-1 ~
- 8 cm = 4 ´ æç ö÷ ´
22 1
(7 ´ 1013 ) 2
m 238 - 222 è7 ø 6 ´ 1026
1+ 23. a-particles are emitted = =4
M 4 = 322.6 Nm -1
For sodium (Na) atoms, M is higher The atomic number is decreased
than that for H-atom, so R is greater for 29. Let, rate of flow of water = x
Na. 90 - 4 ´ 2 = 82 Work Fy éQ x = y ù
P0 = = = mgx
As atomic number of 83Y
222
. Time t êë t úû
18. Volume remains constant.
4 4 So, atomic number is increased by 1, If, x ® nx , then increased power
pR 3 = N × pr 3 therefore one b -particle is emitted.
3 3 Fy ¢
P1 = = Fnx = nFx
Þ R = N 1/ 3r 24. Y1 t
A
P1 nFx n
Potential of small drop, Y \ = =
1 q P0 Fx 1
V = × B
4 pe0 r Y2 or P1 : P0 = n : 1
Potential of the large drop, Real depth
Truth table of given circuit is 30. m=
1 Nq 1 Nq Apparent depth
V¢ = × = × A B Y1 Y2 Y
4 pe0 R 4 pe0 N 1/ 3r 1.5 =
Real depth
= N 2 / 3V = 10 N 2 / 3 0 0 1 1 0 1.5 + 3 .5
8 hL 1 0 0 1 1 Þ Real depth = 7 .5 cm
19. Fluid resistance is given by f =
pR 4
0 1 1 0 1 \ Real length of the edge of cube = 7 .5 cm
If two capillary are joined together, 1 1 0 0 1 31. From figure, magnetic field at O due to
8 hL 8 h3L straight wire AB and CD = 0
feq = f1 + f2 = + So, output is (b).
pR 4 4
p æç R ö÷
3
25. In case (a), the radii of two curvature of i
è2 ø R
two faces (convex and concave) of a
8 hL é 3 ´ 24 ù lens are equal. i.e., R1 = R 2 2R
= 1+ O B
4 ê ú A
pR ë 34 û
. ~
= f ´ 159 - 16
. f R1 R2
II
p p p C
Q Q¢= = =
feq 16 . f 16 . (8 hL / pR 4 ) R1 = R2 I
4
pR p 1 Q So, the incident and emergent rays will
= ´ = D
8 hL 16
. 16. be parallel.

MODULE 2
www.jeebooks.in
176 NEET Test Drive

Hence, total magnetic field at O due to 37. By formula of refraction through convex
circular arc having radii R and 3R spherical surface, dx dq
0= cos q - x sin q
m i 270° ö u0i æ 90° ö m 2 m1 m 2 - m1
= 0 æç ÷+ ç ÷ - = dt dt
2R è 360° ø 2 ´ 3R è 360° ø v u R dq v cos q
Þ =
3m 0i m i 5m 0i Here, u = ¥, v = 2R dt x sin q
= + 0 =
8R 24R 12 R m2 1 m - m1 Also, CB = x sin q
\ - = 2
m x + m2 x 2 2R -¥ R d (CB )
32. Centre of mass, x CM = 1 1 Then, velocity of block (u) =
m1 + m 2 m2 m2 dt
-1
m1 m dx dq
2 ´ 1+ 5 ´ 1 = 1 = sin q + x cos q
= =1 dt dt
2+5 2R R
v sin q
m1y1 + m 2 y 2 m m -1 ém 2 ù = v sin q + x cos q ´
and y CM = = ê m = mú x sin q
m1 + m 2 2R R ë 1 û
v (sin2 q + cos 2 q) v
2 ´ - 2 + 5 ´ 3 11 Þ m=2 = =
= = sin q sin q
2+5 7 38. If one of the slits in Young’s double slit
43. Since, refractive index m, radius of
experiment is covered with a black
\ Centre of mass is at æç1, ö÷.
11
opaque paper, then we will obtain a curvature R will remains constant, hence
è 7ø focal length will be same.
single slit diffraction pattern on the
é EL2 ù screen. Since, intensity µ (aperture) 2
33. Given, ê 5 2ú
ëM G û 39. In a non-uniform electric field, an Hence, intensity will be reduced.
2 -2 2 -1 2 electric dipole experiences net force as 44. From Newton’s IInd law,
[ML T ][ML T ]
= well as torque.
[M] 5 [M-1L3 T -2 ] 2 Net force = mass ´ acceleration,
40. The magnetic flux linked with the α
[ML2 T -2 ][M2L4 T -2 ]
= = [M0L0 T 0 ] primary coil is given by f = f 0 + 4t.
[M5 ][M-2L6 T -4 ]
So, voltage across primary,
The quantity having no dimension is df d m
angle. VP = = (f 0 + 4t ) R
dt dt
34. y1 = a sin 2 p f t = 4V (as, f 0 = constant)
T
p
y 2 = a sin æç 2 p f t + ö÷ Also, we have
è 2ø N p = 50 and Ns = 1500
\ y = y1 + y 2 V N m a
From relation, S = S
p p
= 2a sin æç 2 p f t + ö÷ cos VP NP
è 4ø 4 NS mg
pö or VS = VP ×
=
2a æ
sin ç 2 p f t + ÷ NP we have, mg - T = ma
2 è 4ø 1
= 4 æç
1500 ö and t = T ×R = mR 2 a
p ÷ = 120 V
= 2 a sin æç 2 p f t + ö÷ è 50 ø 2
è 4ø and a = Ra
Thus, amplitude = 2a, frequency = f 41. The escape velocity and orbital velocity 1 1
are related as ve = 2vo Þ T = mRa = ma
35. Given that,Ti andTf = 15
. Ti 2 2
Energy in a orbital velocity, 1
So, E µT 4 1 Þ mg - ma = ma
4 4 4 KE o = mvo2 = E 2
Þ E f µ Ti ´ 15 . = 5.06Ti 2 2
Ti 4 (5.06) - Ti 4 Energy for escape velocity, Þ a= g
So, DE % = ´ 100 = 406° 3
Ti 4 1 1
KE e = mve2 = m ( 2vo ) 2 = 2E 45. Equivalent circuit diagram is
2 2
36. Magnetic field at the centre O of the
2µF 2µF
loop of radius is Thus, energy required 2µF
m 0Ic = KE e - KE o = 2E - E = E
B1 = A 2µF B
2R 42. Let AB = x , AC = x cos q 2µF
Magnetic field due to the straight wire
at the centre is C B v 10 V
m I
B2 = 0 e
2 pH Equivalent capacitance between A and
x
For net field to be zero, θ B is
m I m I C ¢ = (2 + 2 + 2) = 6mF
B1 = B 2 Þ 0 c = 0 e A
2R 2 pH So, energy stored, E =
1
C ¢V 2
H = e
IR d (AC) dx dq 2
\ = cos q - x sin q
p Ic dt dt dt = 3 ´ 10-4 J

MODULE 2
www.jeebooks.in
CHEMISTRY
46. Mg + 2HCl ¾® MgCl 2 + H2 ­ 54. DG ° = DH ° - TDS° (at standard 62. Ba(NO 3 ) 2 gives NO -3 and Ba 2 + ions,
1 mol 2 mol
conditions) hence Ba 2 + ion increases. To keep
i.e. 24 g = 73 g K sp constant, [F - ] decreases. Thus, it is
\ DH ° = 77.2 ´ 103 J mol -1
\ HCl required for 2.4 g Mg 1
DS° = 122 JK -1 mol -1,T = 400 K represented as [F - ].
73 2
= ´ 2 .4 = 7.3 g \ DG° = 77200 - 400 ´ 122 = 28400 J
24 63. Complete balanced equation is as follows :
7.3 Also, DG° = - 2.303 RT log KC
= = 0.2 mol 3Br2 + 6OH- ¾® 5Br - + BrO -3 + 3H2O
36.5 28400 = - 2 .303 ´ 8.314 JK -1
´ 400K ´ logKC So, p = 3, q = 6, r = 5, s = 1, t = 3
But M1 ´ V1 = M 2 ´V2 -4 64. Argentite is Ag 2 S (a sulphide ore) and
KC = 1.958 ´ 10
\ . ´ V2 = 0.2 ´ 1000
15 cuprite is Cu2O (an oxide ore).
55. Using formula,
V2 = 133.3 mL 65. OH
D o = - 0.4 ´ nt 2g + 0.6 ´ ne g , O3/H2O Ca(OH)2
1 1
47. l µ or l µ The calculation reveals that the ∆
(KE)m m OH
maximum CFSE is for low spin
48. NF3 and H3O + have sp 3 -hybridisation. d 5 configuration.
NH2–NH2
NO -3 and BF3 have sp 2 -hybridisation. 56. The ionic product of water is given by O
OH–/glycol
w 6.82 [H3O + ] [OH- ] = Kw (Wolf-Kishner reduction)
49. S = ´ 1000 = ´ 1000
VmL 250 Q [H3O + ] = [OH- ] = 10-6 M
Due to ozonolysis, H2O 2 is also formed
= 27.28g/L \ Kw = 10-6 ´ 10-6 = 10-12 M 2 which oxidise — CHO to — COOH.
50. We can identify the gas through its 57. Due to rigid structure, it does not 66. FeCr2O 4 = Fe2 + + C2O 42 -
molar mass (M ). According to the ideal undergo formation of planar
carbocation. Hence, (c) will not Fe is oxidised to Fe3 + and C2O 42 - to
2+
gas law, CO 2 by Cr2O 72 - .
undergo SN 1 reaction.
pV = nRT (i) Oxidation of Fe2+ ¾® Fe3 + by Cr2O 72 -
pM = dRT Cl Fe2 + ¾® Fe3 + + e - ] ´ 6
dRT
or M = Cr2O 72 - + 14H+ + 6e - ® 2 Cr 3 + + 7H2O
p
Thus, 6Fe2 + ºº 1 Cr2O 72 -
(1 .964 gdm-3 )
Unstable 1
(0.0821 dm3 atm K -1 mol -1) (273 K) 1 Fe2 + ºº Cr2O 72 -
= 6
æ 76 cm ö 58. When phenol reacts with sodium
ç ÷ (ii) Oxidation of C2O 42 - ¾® Cr 3 +
è 76 cm atm-1 ø hydroxide, reactive sodium phenoxide
by Cr2O 72 -
= 44 g mol -1 is formed. So, electrophilic aromatic
2-
Thus, the gas is CO 2 whose molecular substitution takes place with CO 2 [C2O 4 ¾® 2 CO 2 ´ 2e - ] ´ 3
mass is 44 amu. under a pressure of 4-7 atm followed Cr2O 72 - + 14H+ + 6e - ® 2Cr 3 + + 7H2O
by acidification and ortho
51. In an octahedral crystal field, t 2 g 3C2O 42 - ºº Cr2O 72 -
hydrobenzoic acid (salicylic acid) is
1
orbitals are lower in energy by 0.4 D o . formed. 1 C2O 42 - ºº Cr2O 72 -
OH ONa
3
52. Assumptions of kinetic theory of gases OH
Thus, total Cr2O 72 - required
are : COOH
NaOH (i) CO2 1 1 1
● The gas consists of very small + = + = = 05 . mol
(ii) H 6 3 2
particles known as molecules.
Phenol Sodium Salicylic acid
● These particles have the same mass. phenoxide
67. XeF6 cannot be stored in glass vessels
because it reacts with SiO 2 of the glass
● The number of molecules is so large 59. BF3 can accept a pair of electrons but to give highly explosive XeO 3 .
that statistical treatment can be it cannot give H+ ions in the aqueous
applied. 2XeF6 + 3SiO 2 ¾® 2XeO 3 + 3SiF4
solution, hence BF3 acts as Lewis acid
● These molecules are in constant, but not as a Bronsted acid. 68. Stronger the acid, weaker is the
random and rapid motion. - conjugate base or weaker is
CH ONa +
● Except during collisions, the 60. CH3Br ¾ ¾ 3¾ ¾
¾® CH3 ¾ O ¾ CH3 nucleophilicity.
-NaBr
interactions among molecules are Order of acidic nature :
This reaction involves nucleophilic
negligible. substitution of the halide ion from the HF > H2O > NH3 > CH4
● The average kinetic energy of the alkyl halide by the alkoxide ion via SN 2 and order of nucleophilicity
-
gas particles depends only on the mechanism. CH-3 > NH-2 >OH > F - .
absolute temperature of the system.
61. Phenol formaldehyde polymer is
\ It is not the critical temperature but 69. PbO 2 is a dioxide, not a true peroxide
obtained by the condensation
temperature. polymerisation reaction of due to the absence of peroxide linkage.
formaldehyde and phenol in presence Hence, it does not give H2O 2 with
53. Least sterically hindered compound is
of either an acid or base as a catalyst. dil. H2 SO 4 .
most reactive towards SN 2 reaction.

MODULE 2
www.jeebooks.in
178 NEET Test Drive

log æç
70. 75. Crystals of NaOH are highly 2.303 100 ö
4.8 ´ 10-4 = ÷
deliquescent, i.e. have the ability to t è 20 ø
absorb moisture from the air to such an 2 .303
\ It is most stable compound is due to extent that they dissolve in it. Thus, NaOH 4.8 ´ 10-4 = log 5
conjugated diene. t
becomes liquid on exposure to air. 3353.7
O t = 3353.7 s = mins
|| 76. Methyl cyanide is reduced to 60
71. Hydrolysis of CH3 COC2H5 takes place ethylamine by LiAlH4 as shown below. t = 55.90 » 56 mins
as : CH3 CN ¾LiAlH
¾¾ 4 ® CH CH NH
3 2 2 85. The temperature above which micelle
Methyl cyanide Ethylamine
O formation takes place is known as CMC.

OH Rest all the other statements are correct. It is the minimum concentration required
CH3 C O C2H5 E for the micelle formation. CMC increases

77. Reactivity towards electrophilic
O substitution increases as the electron with increase in concentration of

– C2H5O density in the benzene ring increases. substances.
CH3 C OC2H5 E Since, CH3 is a strong electron 86. N A = Noe - 8lt Þ NB = Noe - lt
H donating group, thus toluene NB Noe - lt
(C6H5 CH3 ) can be most easily Given, = = e7 lt
O N A Noe - 8 lt
– sulphonated.
CH3 OH OC2H5 1
C 78. In cycloaddition reactions [2 + 2] is e 7 lt = e1 or 7 lt = 1 Þ t =
O 7l
photochemically allowed and [4 + 2] is
thermally allowed. 87. Electronic Configuration Unpaired
CH3 C – + C2H5OH of Metal Ion Electrons
O 79. Transition elements have three series
3d, 4d and 5d. 4d series has elements A. [Cr(H2O) 6 ] 3 + 3d 3 4s 0 1. 3
72. On descending a group, the atoms and 39(Y ) to 48(d) atomic number. Thus, 2+ 6 0
B. [Fe(H2O) 6 ] 3d 4s 2. 4
ions increase in size. On moving from [Kr]4d 1-10 , 5s 1- 2 is the general
left to right in the periodic table, the configuration of IInd row series. C. [Zn(H2O) 6 ] 2 + 3d 10 4s 0 3. 0
size decreases thus, on moving 4- 6 0
diagonally, the size remains the same. 80. Boric acid is used in carom boards for D. [Fe(CN) 6 ] 3d 4s 4. 0
smooth gliding of pawns because
r (Li) = 152 pm, r (Mg) = 160 pm H-bonding in H3BO 3 gives it a layered 88. The products of the reaction of copper
r (Li + ) = 76 pm, r (Mg 2 + ) = 72 pm structure. with HNO 3 depends upon the
Similarly, on moving diagonally, concentration of HNO 3 used.
81. The packing fraction is the fraction of
electronegativity remains nearly the volume in a crystal structure that is 3Cu + 8HNO 3 (dil.) ¾® 3Cu(NO 3 ) 2
same. occupied by atoms. + 2NO + 4H 2O
Li Be For simple cubic crystal lattice, Cu + 4HNO 3 (conc.) ¾® Cu(NO 3 )2
1.0 1.5
4 / 3 pr 3 p + 2NO 2 + 2H2O
Na Mg f= ´ Z = = 0524
. [QZ = 1]
0.9 1.2 a3 6 89. Glucose gives silver mirror with
Packing fraction = 0524
. ´ 100 = 52 .4% ammoniacal silver nitrate because of the
73. In electrochemical series metals with presence —CHO group (aldehyde
positive standard potential are placed 82. For the given reaction, group) in the structure of glucose.
below hydrogen. Out of the given 10Cl - (aq) + 2MnO -4 (aq) + 16H+ (aq) Conc. H SO
90. C2H5OH ¾ ¾ ¾ ¾
2
¾
¾4
® CH2 == CH2
transition metals, only Cu is lower than = 5 Cl 2 (g) + 2Mn2 + (aq) + 8H2O (l ) 44 K
hydrogen in electrochemical series.
The number of electrons exchanged + H2O
Therefore, it must have positive sign for
(n) = 10 This reaction is known as dehydration of
standard potential.
0.0591 alcohols. Mechanism proceeds via
From Nernst equation, E ° = log K
74. Propyne reacts with AgNO 3 in NH3 to n formation of protonated alcohol. The
give white ppt. of silver acetylide and nE ° ù
\ K = antilog é
mechanism is as follows :
ëê 0.059 ûú
propene does not react with it. Thus, it
is used to distinguish between propene H H
10 ´ 0.15 ù
and propyne. The reaction is as follows : = antilog é = 2 .6 ´ 1025 Fast
ëê 0.059 ûú H—C—C—O—H + H
CH3 ¾ C ºº CH + AgNO 3
+ NH3 ¾® CH3 ¾ C ºº CAg 83. Carbon black is the most amorphous H H
H H H
Silver acetylide (white ppt.) form of carbon. It is used for making
black pigment in black ink, plastics +
CH3 ¾ CH == CH2 + AgNO 3 H—C—C—O—H
and is a reinforcing pigment in rubber.
+ NH3 ¾® No reaction H H
–H2O Slow
On the other hand, both of them 84. Given : k = 4.8 ´ 10-4 s -1 Protonated alcohol
decolourise alk. KMnO 4 and Br2 in Since, it indicates first order reaction, H H
2.303 a H H +
CCl 4 and both of them give addition \ k = log H++ C == C H—C—C
reaction with conc. H2 SO 4 . t a-x H H
Ethene H H

MODULE 2
www.jeebooks.in

BIOLOGY
91. The convergent evolution is shown by and utilisation of Ca 2+ and membrane Diversity means it can recognise a large
wings of butterfly and birds. It is the functioning. variety of foreign molecules.
evolution where different structures 100. In option (b), the oxidative Memory is when a second encounter
evolve for the same function and hence decarboxylation takes place. In with the same microbe, it evokes a
share similarity. mitochondria, pyruvic acid (formed by heightened immune response.
92. Adenoviruses are DNA viruses. Avian the glycolytic catabolism of 108. The correct matches are
leukemia virus, bacteriophage MS-2 carbohydrates in cytosol) undergoes Root pressure is caused by upward
and influenza virus are RNA viruses. oxidative decarboxylation (i.e. removal movement of water in xylem from roots.
of CO 2 in aerobic conditions) forming a Capillary action Movement of water in
93. Raffinose is a trisaccharide composed
key compound, i.e. acetyl Co-A by the xylem due to attractive forces between
of galactose, glucose and fructose.
action of pyruvic acid dehydrogenase water molecules (cohesion), water
Raffinose is an oligosaccharide, which
(in mitochondrial matrix) through a molecules and tube walls (adhesion).
on hydrolysis produces two or more
series of reactions. Transpiration pull Suction force caused
molecules of some or different
monosaccharide unit held together by 101. The substances like fructose and some by evaporation of water from leaf
a glycosidic bond. amino acids are absorbed with the surfaces.
help of the carrier molecules. This
94. Deforestation encompasses the 109. A DNA molecule is constituted by two
mechanism is known as the facilitated
depletion of forest cover. Genetic complementary strands. This is
transport.
deformities is not an effect of advantageous because it promotes
deforestation. There is loss of 102. Monascus purpureus is used to semiconservative mode of replication,
biodiversity, disturbance of the produce statins. They are inhibitor of where each parental strand gets paired
hydrologic cycle and soil erosion due enzyme HMG Co-A reductase of liver with a newly formed daughter strand.
to deforestation. and lower LDL cholesterol level.
110. Restriction enzymes are the enzymes
95. Velamen cells comprise of fibrous 103. In glycolysis, the following reaction is which recognise specific base
thickenings. It is highly hygroscopic catalysed by enzyme aldolase. sequences in DNA and cut the DNA at
and outermost tissue of the root. It is Fructose 1, 6-biphosphate → that site. They are often referred to as
found in orchid root. Glyceraldeyde-3-phosphate molecular scissors.
Hence, all the statements are correct +Dihydroxyacetone phosphate. The 6 111. All the statements are correct regarding
regarding velamen. carbon molecule is splitted into two 3 gemmule. These arise as modified
carbon molecules-one G3P, another branches and are rich in food and
96. The statement (c) is incorrect and is
DHAP that is converted into G3P in germinate into new plant on detachment
corrected as sucker develops from an
another reaction. from the plant. In case of Marchantia, the
axillary bud of an underground part of
stem. Offset has short and thick 104. During photolysis of water, the Oxygen gemmae are multicellular and in algae,
internode. Evolving Complex (OEC) has a the gemmae occur in Sphacelaria.
metalloenzyme containing two different In bryophytes, the gemmae are
97. An antibody molecule is formed of metals, i.e. manganese and calcium in approximately not present in Bryopsida.
4- polypeptides, i.e. two heavy and two addition to chlorine.
light chains. 112. The phenomenon of imbibition includes
105. Pyrenoids are associated with starch volume change, production of heat and
These join to form a Y-shaped molecule.
storage. They are found in algal development of pressure.
The variable region is the amino acid
chloroplasts. It has a protein core
(110-130) sequence present at the tips Volume change occurs by increasing the
around which starch is deposited in
of this Y - molecule. It is responsible for volume of the system. Imbibition results
layers.
imparting the specificity to an antibody in the production of heat as molecules
for antigen binding. 106. Apical cell theory was postulated by are arranged on the surface of imbibant.
Hofmeister and supported by Nageli. It Pressure development results as the
The variable region includes the ends
states that a single apical cell is the swelling occurs if an imbibing system is
of both the light and heavy chains.
structural and functional unit of apical confined.
98. The schizont stage of Plasmodium is meristem and it governs the whole
observed in human erythrocytes and process of primary growth. It is found 113. The quality of food produced by the
liver cells. The sporozoite stage in some lower plants like algae, grafted plant is determined by the
circulates for about half an hour in bryophytes and pteridophytes. genotype of ‘scion’. So, the scion of
blood and later enters the liver cells. alphonso mango and stock of desi
107. Adaptive immune system is also known
Some of them escape into hepatocytes mango are used.
as acquired immune system.
and multiply further via schizogony. 114. Hydatid cyst is the larvae of
Specificity, diversity, memory are
99. Boron in green plants assists in sugar characteristics of acquired immunity. Echinococcus granulossus (hydatid
metabolism and translocation. It is also Specificity is able to distinguish worm). It is also called dog tapeworm.
involved in pollen germination and cell among various kinds of foreign The life cycle of individual involves a
differentiation and elongation, uptake molecules/particles. single intermediate host, which may be

MODULE 2
www.jeebooks.in
180 NEET Test Drive

man or cattle. The larva looks like a Muscular dystrophy is a genetic 132. The characteristics of resemblance
watery bladder or hydatid vesicle or disease that weakens the between gymnosperms and
hydatid cyst with a two-layered cyst musculoskeletal system and hamper angiosperms are as follows
wall. locomotion. The disease is The secondary growth is present in
115. Early diagnosis of diseases cannot be characterised by progressive skeletal roots. Secondary wood may be
done by urine and serum analysis. muscle weakness, defects in muscle monoxylic or polyxylic. There is the
They are the conventional methods of proteins and the death of muscle cells presence of ovule, endosporic and
and tissues. reduced gametophytes.
diagnosis.
125. The statement (c) is incorrect 133. Cervical cap is not an IUD. It is a barrier
116. The pollen tube contains vegetative
regarding gymnosperms. It is method of birth control. Cervical caps
nucleus and a generative nucleus
corrected as follows are made up of rubber and are inserted
which are identical because they are into female reproductive tract to cover
products of mitosis. Xylem consists of tracheids only in
the cervix during intercourse. Cervical
The entire generative cell resides within gymnosperms.
caps prevent conception by blocking
the cytoplasm of the tube cell or 126. All the statements are correct entry of sperms through cervix.
vegetative cell. These two cells form regarding Abscisic Acid (ABA). ABA is Progestasert, LNG-20, mirena are some
juvenile pollen grain (the male a naturally occurring growth inhibitor, It of well-known hormone releasing IUDs.
gametophyte). is also known as stress hormone. All
134. Cyanobacteria possess prokaryotic type
117. Fertilizin is a compatibility receptor the plant parts are known to synthesis of cells with incipient nucleus. In it,
protein found in zona pellucida. this hormone. It is synthesised from asexual reproduction takes place by
The ovum possesses three coverings, mevalonic acid. fission and fragmentation. Hence, all the
i.e. inner plasma membrane, middle 127. All the options are correctly matched. statements are correct regarding
glycoprotein, zona pellucida and outer In acoelom, there is no body cavity. In cyanobacteria.
cellular corona radiata. The cytoplasm pseudocoelom, there is the presence 135. All the statements are the criteria for an
of ovum is called ooplasm. of false coelom or perivisceral cavity element to be considered as essential,
118. Monocot stem has vascular bundle and coelom is not lined by mesoderm. i.e. it must be essential for normal growth
closed. Dicot stem has vascular bundle Eucoelom is true coelom in which true and reproduction, and plant cannot
body cavity develops entirely within the proceed its life without this element. The
open. Pith in monocot stem is absent.
mesoderm resulting in tube. requirement of element must be specific
Pith in dicot stem is made up of
Deuterostomes are enterocoeloms and cannot be replaced by any another
parenchymatous cells.
which are formed from the pouches of element. The requirement must be
119. In heterostyly, there is difference in the the archenteron or primitive gut. direct, that is, not the result of any
lengths of style and stamens in a indirect effect, e.g. for relieving-toxicity
flower, so that self-pollination is not 128. Starch is made up of two polymers caused by some other substance.
possible. e.g. Primula and some which are amylose and amylopectin.
Amylose is a linear molecule comprising 136. The post-transcriptional modification
members of Papilionaceae.
of 1,4 linked alpha-D-glucopyranosyl involves capping in which methyl
120. The statement IV is incorrect. It is units. Amylopectin is a highly branched guanosine triphosphate, an unusual
corrected as follows molecule comprising of both 1,4 linked nucleotide is added to the 5' end of hnRNA.
Fleshy adventitious roots also take part and 1,6 linked alpha-D-glucopyranosyl In tailing, adenylate residues (200-300)
in vegetative propagation as in sweet units. are added at 3' end in a template
potato, dahlia and Asparagus. independent manner. The fully
129. Protonema is involved in vegetative processed hnRNA is called mRNA which
121. Repeated selfing will lead to increased propagation. is transported out of the nucleus for
homozygosity. Selfing or self-fertilisation Antheridium is male sex organ. translation.
is the union of male and female gametes Archegonium is female sex organ.
from same organism. Self-fertilisation 137. In humans, the teeth are embedded in a
Sporophyll bears sporangia. socket of jaw bone which is called
results in the loss of genetic variation
130. Creatine excretion is probably thecodont. Diphyodont is a condition in
within the offspring.
associated with the process of histolysis which there are two sets of teeth in their
122. In prophase-I of meiosis, during the (breaking down of organic tissue during lifetime and there are four types of teeth
substage diakinesis, terminalisation’ metamorphosis) and histogenesis which is called heterodont.
can be observed. During (formation of different tissues from 138. All the options provide an evidence for
terminalisation, the chiasmata shifts undifferentiated cells). Creatine natural selection.
towards the ends of the chromosomes. excretion can be seen in foetus and Rate of reproduction and competition
By the end of diakinesis, the homologs pregnant and lactating women. can be seen in nature normally.
are held together only at their ends. Production of new varieties of plants and
131. The option (d) is incorrectly matched
123. In Crocus, the microspores are pair. It is corrected as follows animals through artificial selection shows
arranged in decussate manner, i.e. Tobacco industry—Bacillus evidence that the nature having vast
three microspores are in one lane and megatherium. resources can easily produce new
one at back. species through the natural selection.
It is used for developing flavour in
Mimicry and protective colouration also
124. The statement III is incorrect. It is tobacco leaves.
support natural selection.
corrected as follows

MODULE 2
www.jeebooks.in
Prep Catalysis for NEET ~ Mock Test 4 181

139. Contagium vivum fluidum for viruses own RNA primer. The Okazaki 156. The statement (d) is incorrect statement.
was given by Beijerinck. He called fragments are joined by polynucleotide It is corrected as follows
them as living infectious fluid. ligase, a joining enzyme to form Nephritis is the inflammation of renal
continuous strand. pelvis, calyces and interstitial tissue due
140. The five types of histone proteins are
H1,H2 A,H2B,H3 and H4 . Four of these 148. The processes of biotic succession in a to local bacterial infection.
types (H1,H2 A,H2B,H3 and H4 ) occur in sequence is as follows 157. The factors responsible for Coronary
pairs to produce histone octamer, Nudation → Migration → Ecesis → Artery Disease (CAD) include
called nu body or core of nucleosome. Aggregation → Competition → hypercholesterolemia, hypertension, diet
DNA connecting the two adjacent Reaction → Stabilisation. low in antioxidants, lack of exercise,
nucleosomes is called linker DNA. It 149. Nicotine is a stimulatory component hyperglycemia, etc.
bears H1. present in tobacco. It has an effect This disease is the end result of the
141. In the apoplast pathway, water moves similar to acetylcholine on human accumulation of plaques within the walls
exclusively through the cell wall without body, e.g. stimulation of nerve of the coronary arteries that supply the
crossing any membrane. The apoplast impulses, muscle relaxation, release of myocardium (the muscle) of the heart
is the continuous system of cell walls adrenaline, increased blood pressure, with oxygen and nutrients.
and intercellular air spaces in plant etc. 158. The statements (b), (c) and (d) are
tissues. 150. The cross of superior males of one correct. The statement (a) is incorrect
142. Implantation refers to the attachment of breed with superior females of another and is corrected as follows
blastocyst to the uterine wall. The breed is referred to as cross-breeding. It Luciferase enzyme responsible for light
trophoblast cells produce enzymes that helps to accumulate the desirable generation in fireflies is present in
allow blastocyst to get implanted into genes of the two breeds into a progeny. peroxisomes.
thick endometrium. This begins the 151. Maximum natality is maximum 159. The CO 2 fertilisation effect is related to
process of implantation. By 7th to 10th production of individuals. increase in atmospheric concentration of
days, the blastocyst is completely
Ecological natality is increase in CO 2 above a certain level.
implanted. Therefore, only statement
population under an actual existing It results in increased productivity of
(c) is incorrect during implantation.
specific condition. C 3 -plants. There is decrease in the rate
143. BAC (Bacterial Artificial Minimum mortality is minimum loss of transpiration because of the partial
Chromosomes), cosmids, phages, under non-limiting conditions. stomatal closure.
plasmids and YAC (Yeast Artificial Ecological mortality is actual loss of
Chromosomes) are all commonly used 160. Genotype of blood group A can be
individuals under given environmental written as IAIA ,IAIO because alleles I A and
vectors. These vectors differ in their condition.
cloning capacities, within a range from I A show codominance or both alleles are
approx 100 bp to 3000 kb. The correct 152. Intake and release of O 2 from blood dominant allele and IO is a recessive
order with respect to increasing depends upon O 2 tension, CO 2 tension, allele.
cloning capacity is given by option (d). i.e. difference in the partial pressure of 161. At each trophic level, most of the
CO 2 within the blood (circulating fluid energy, gets lost as heat in metabolic
144. All the statements depict the function
and static media). It also depends on reactions. As a result, there occurs a
of HCI in the stomach. It stores the
H + ion concentration, electrolyte progressive decrease in energy at each
food for 4-5 hours. The presence of
content and temperature. trophic level.
HCl preserves the food from spoilage.
It also acts as antiseptic and kills the 153. The statement (d) is not the 162. The option (d) is incorrectly matched
microorganisms, if any, present in the characteristic feature of blind sac body pair. It is corrected as follows
food. It makes the medium acidic for plan. Its examples include Supinator is antagonistic to pronator, i.e.
the functioning of pepsin. coelenterates, flatworms and turns the palm upward or forward.
ctenophore. The presence of two
145. National parks and biosphere reserves 163. Different alleles of a gene are found on
openings one on either sides of the
are part of in situ conservation strategy. homologous chromosomes. These
digestive system is the characteristic of
The correct matches are chromosomes are a set of one maternal
tube within a tube body plan.
Keoladeo National Park – Bharatpur and one paternal chromosome that pair
(Rajasthan) 154. Ecological succession is characterised up with each other inside a cell during
Dachigam National Park – Kashmir by the formation of soil, increase in meiosis. Homologues have the same
thickness of soil and biomass. It is also genes in the same loci.
Similipal Biosphere Reserve – Odisha
characterised by higher species
146. The statements III and IV are correct diversity. 164. The RNA interference (RNAi) had been
regarding the sucrose. Sucrose is a adopted to prevent the infestation of
155. The function of DNA gyrase includes Meloidogyne incognita. It infects the
disaccharide and is obtained naturally
the relieving of torque or strain. The roots of tobacco plants and causes great
from sugarcane, sugar beet and
unwinding of DNA imposes tension or reduction in yield.
Sorghum.
strain on the distal end of DNA
The glucose and fructose are joined 165. The option (d) is incorrectly matched
molecule. This strain is released by
together by α-1,2 glycosidic linkage. pair. It is corrected as follows
super helix relaxing protein (also called
147. Okazaki fragments are the lagging DNA gyrase or topoisomerase or In Ordovician, algae, fungi and bacteria,
strand. It replicates in a discontinuous swivelling protein). This protein has and first fossils of plant life were
manner. Each Okazaki fragment has its nick and joining activity. observed.

MODULE 2
www.jeebooks.in
182 NEET Test Drive

166. The statements I and II are correct. The 171. Bioethical concerns in relation to 175. The mating of closely related animals
statements III and IV are incorrect and genetic modification encompasses within the same breed for 4-6
are corrected as follows the moral implications of genetic generations is called inbreeding.
Acromegaly is due to hypersecretion of manipulation on animals, the concern The advantages of inbreeding include
somatostatin. Tetany is due to for environment, effect of modified increase of homozygosity, exposure of
hyposecretion of parathyroid. entity on people, etc. All the given harmful recessive genes and superior
statements are concerned with the genes can be accumulated.
167. The statements (a), (b) and (c) are
major bioethical concerns pertaining
correct regarding earthworm. The 176. The ‘scotopic’ vision refers to the vision
to genetic modification of organisms.
statement (d) is incorrect and is of eye in low light. It is the function of
corrected as follows 172. All-or-none law is a principle which rods. There are two types of
The circular cells are nucleated and states that the response of a muscle photoreceptor cells, viz, rods and cones.
make up about 10% of total corpuscles. or nerve to a stimulus is not The daylight or photopic vision is the
These may also be called leucocytes. proportionate to the intensity of function of cones and twilight or scotopic
168. The coat colour of mice is a case of stimulus, but is either present in full vision is the function of the rods.
recessive epistasis. It is when the strength or completely absent. A
177. The three basic steps in genetic
recessive allele of one gene in a single muscle fibre (striated,
engineering include
homozygous state masks the phenotypic unstriated or cardiac) does not show
(i) Identification of DNA with desirable
expression of the dominant allele of any gradation in contraction in
genes.
another gene. relation to the degree of stimulus.
(ii) Introduction of the identified DNA
169. Cilia are present in epithelium of 173. Population density is the number of into a suitable host to form
respiratory tract and oviduct. They are individuals of a species per unit of recombinant DNA (rDNA).
motile, hair-like, thin projections arising area. (iii) Maintenance of introduced DNA in
from the basal granule. They beat back Species that use abundant resources the host and transfer of the DNA
and forth to propel fluid or particles in often reach higher population (gene transfer ) to its progeny.
one direction. densities than species that use
scarce resources. 178. Nissl’s granules are associated with
170. Haemophilia is caused by a sex-linked
enzyme formation required for synthesis
recessive gene h located on the Species with small individuals
of neurotransmitters. Nissl’s granules are
X-chromosome. A marriage between a generally reach higher population
carrier woman and a haemophilic man riboprotein entities present in the
densities than species with large
produces 50% normal and 50% neuroplasm of neuron.
individuals.
haemophilic sons. Among the Some newly introduced species 179. In spermiogenesis, there is no change in
daughters, 50% will be carriers and reach high population densities. number of cells as only one spermatid
remaining 50% do not live. develops into a spermatozoa. It is a
Hence, all the statements are correct
XX
h
× XY
h
regarding population density. process of transformation of a non-motile
spermatid to a motile spermatozoa.
Gametes X X
h
X
h
Y 174. Bioherbicides are organisms which
destroy weeds without harming the 180. All the statements are correct regarding
h h h useful plants. In India and Australia, archaebacteria. They are obligate
XX XX anaerobes, since their energy liberating
Carrier Haemophilic cochineal insect (Cactoblastis
daughter daughter (dies) cactorum) was used for the control of metabolic process does not consume
cacti (Opuntia). oxygen. Genetic material floats freely.
Offsprings h They do not contain membrane bound
XY XY Sporeine was the first commercial
Normal son Haemophilic bioinsecticide. organelles and are found in harsh
son habitats.

SCORE CHART
No. of Correct Answers : A .................. No. of Incorrect Answers : B ..................
Total Marks : x = (A × 4) – (B × 1)
Scores and Expected Rank : If the score lies above 665, then rank will be in between 1-50. For other scores, rank
estimations are given below
Score Rank
664 – 642 51 – 150
641 – 636 151 – 250
635 – 630 251 – 400
629 – 625 401 – 500

MODULE 2
www.jeebooks.in
Prep Catalysis for NEET ~ Mock Test 5 183

PREP CATALYSIS
for NEET
Full Length Mock Tests for NEET to Make You Ready to Face the Challenge

MOCK TEST 5 (With Solutions)

Duration : 3 Hours Max. Mark : 720

Instructions
The test is of 3 hours duration and Test Booklet contains 180 questions. Each question carries 4 marks. For
each correct response, the candidate will get 4 marks. For each incorrect response, one mark will be deducted
from the total scores. The maximum marks are 720.
The question paper contains three parts of Physics, Chemistry and Biology respectively.
Part A contains 45 Questions from Physics section. Part B contains 45 Questions from Chemistry section.
Part C contains 90 Questions from Biology.

PART A PHYSICS
1. The forbidden energy gap between valence and Then, (rP , vP ) are related to the corresponding
conduction bands of Ge and Si are respectively, quantities at the aphelion (rA , vA ) as
(a) 5 eV, 1.1 eV (b) 0.78 eV, 1.1 eV
B
(c) 11
. eV, 0.38 eV (d) 078
. eV, 0.78 eV Planet

2. A rectangular frame of copper wire enclosing P A


a surface area A is covered with a soap film. If
the area of the frame of copper wire is reduced C
25%, the energy of the soap film will be changed
by (a) rP > rA , v P > v A (b) rP < rA , v P > v A
(a) 25% (b) 50% (c) rA > rP, v A > v P (d) rA < v P, v A = v P
(c) 75% (d) 35%
5. The path difference for constructive interference in
3. A ray of light falls on a transparent glass slab of terms of wavelength l is
refractive index 1.73. If the reflected ray and the (a) nl (b) (2 n + 1) l
refracted ray are mutually perpendicular, the l
(c) (2 n - 1) l (d) (n + 1)
angle of incidence is 2
(a) 30º (b) 48º
(c) 60º (d) 75.2º 6. A force F acting on a body depends on its
displacement s as F µ s1/ 3 . The power delivered by
4. Suppose the speed of planet at the perihelion P as F will depend on displacement as
in the figure be vP and the sun-planet distance SP (a) s 2 / 3 (b) s1/ 2 (c) s (d) s 0
be rP .
MODULE 11
www.jeebooks.in
184 NEET Test Drive

7. The equivalent resistance between A and B as 14. A cyclic process abca performed on 3 moles of an
shown in the figure, ideal gas. A total of 1500 cal of heat is withdrawn
A from the gas sample in the process. The work done
B by the gas during the path bc is
1Ω
c
1Ω 1Ω T
1Ω
700 K b
(a) 0.2 W (b) 0.4W
(c) 0.6W (d) None of these

8. Three balls of masses 4kg, 3 kg and 5 kg are 400 K a


arranged at the corners of an equilateral triangle
DABC of side 2 m. The moment of inertia of the
system about an axis through the centroid and O v
perpendicular to the plane of the triangle is (a) –3000 cal (b) + 3000 cal
(a) 16 kg-m 2 (b) 4 3 kg-m 2 (c) 4 kg-m 2 (d) 16 3 kg-m 2 (c) – 1500 cal (d) – 3300 cal
9. A block of mass 10 kg is moving horizontally with 15. The value of refractive index of a medium of
a speed of 1.5 ms -1 on a smooth plane. If a polarising angle of 76º is [given, tan 76º = 4]
constant vertical force of 10 N acts on it, then the (a) 2 (b) 4
displacement of the block from the point of
(c) 1.5 (d) None of these
application of the force at the end of 4s is
(a) 5 m (b) 20 m (c) 12 m (d) 10 m 16. A body hanging from a spring balance stretches it
by 1 cm at the earth’s surface. The same body
10. A copper wire has a mass 0.3 ± 0.003 g, radius stretches the spring at a place 3200 km below the
0.5 ± 0.005 mm and length 6 ± 0.06 cm. The surface of earth by a value of
maximum percentage error in the measurement of (take, radius of earth = 6400 km)
its density is (a) 2 cm (b) 0.5 cm
(a) 2% (b) 3% (c) 6% (d) 4% (c) 1 cm (d) 6 cm
11. A square loop of side 20 cm having resistance 50 W 17. The time period of the electron orbiting in second
is placed in an uniform magnetic field of 2.5 T to orbit of hydrogen atom is 4 ´ 10-16s. Hence, the
the plane of loop. If we attempt to pull it out of the time period in the third orbit will be
field with a constant velocity 25 m/s, then the (a) 2.7 ´ 10-16 s . ´ 10-16 s
(b) 135
power requirement is (c) 2.7 ´ 10-15 s . ´ 10-15 s
(d) 135
(a) 2.75 W (b) 3.75 W (c) 3.125 W (d) 5.125 W
18. Treating the bulb as a point source of
12. A brass rod 1 m long is firmly clamped in the electromagnetic radiation, i.e. about 5% efficient in
middle and one end is stroked by a resined cloth. converting electrical energy to visible light. The
What is the pitch of the note will be heard? maximum magnitudes of the electric field of the
(Young’s modulus for brass = 102 dyne cm -2 and light that is incident on a page, because of the
density = 9 g cm -3 ) visible light coming from a desk lamp is
(a) 1652.5 Hz (b) 1600 Hz (c) 1666.67 Hz (d) 1554.48 Hz (a) 40 V/m (b) 30 V/m
13. In the circuit shown below, A is joined to B for a (c) 45 V/m (d) 25 V/m
long and then, A is joined to C. The total heat 19. A metal rod of length 2.5 m and negligible mass
produced in R is has two masses tied to its ends. If they have
R masses 0.5 kg and 1.5 kg, respectively, then find
the location of the point on this rod, where the
2L C rotational energy is minimum, when rod is rotated
A
about that point.
B (a) 1.875 m from 1.5 kg
+ – 2R
(b) 1.525 m from 1.5 kg
e (c) 1.525 m from 0.5 kg
Le 2 Le 2 Le 2 Le 2 (d) 1.875 m from 0.5 kg
(a) (b) (c) (d)
8R 2 2R2 4R2 R2

MODULE 11
www.jeebooks.in
Prep Catalysis for NEET ~ Mock Test 5 185

20. Monochromatic light of wavelength 660 nm is 26. Infinite number of masses, each of 1 kg are placed
produced by a helium neon-laser. The power along the X-axis at x = ±1m,±2m , ± 4m , ± 8m ,
emitted is 9 mW. The number of photons emitted ±16m, ….. The magnitude of the resultant
per second is gravitational potential in terms of gravitational
(a) 4.5 ´ 1016 / s (b) 3 ´ 1016 / s constant G at the origin ( x = 0) is
(c) 3.6 ´ 1016 / s (d) 2 ´ 1016 / s (a) G (b) 2G (c) 3 G (d) 4G

21. A car accelerates from rest at a constant rate a for 27. A raw egg and a hard boiled egg are made to spin
sometime after which it deaccelerates at constant on a table with the same angular speed about the
rate b and finally comes to rest. If total time same axis. The ratio of time taken by the two eggs
elapsed is T1, then maximum velocity attained by to stop is
car is (a) equal to 1 (b) less than 1
a+ b ab (c) greater than 1 (d) zero
(a) T (b) T
ab a+ b
28. When temperature of radioactive substance
a b
(c) T (d) T increases, then its half-life
a+ b a+ b
(a) increases (b) decreases
22. The Young’s modulus of the material of a wire is (c) remains same (d) None of these
5 ´ 1010 N/m 2. If the elongation strain is 2%, then 29. A ray of light strikes a materials slab at an angle
the energy stored in the wire per unit volume of incidence 30º. If the reflected and refracted rays
(in Jm -3 ) is are perpendicular to each other, the refractive
(a) 107 (b) 2 ´ 107 index of the materials is
(c) 0.5 ´ 10 7
(d) 108 1 1
(a) 3 (b) 2 (c) (d)
2 3
23. Which of the following is not a path function?
(a) DQ (b) DQ - DW 30. Charge q2 of mass m revolves around a stationary
æV ö charge q1 in a circular orbit of radius r. The orbital
(c) DW (d) nRT loge ç 2 ÷
è V1 ø periodic time of q2 would be
1 1 1 1
-
é 4p 2 mr 2 ù 2 é kq1 q 2 ù 2 é 4p 3 mr 2 ù 2 é 4p 2 mr 4 ù 2
24. The power dissipated in the circuit shown below is (a) ê ú (b) ê 2 2 ú (c) ê ú (d) ê ú
50 W, then the value of R is ë q1 q 2 û ë 4p mr û ë kq1 q 2 û ë q1q 2 û
10 Ω
31. The width of one slit is double the another slit in
Young’s double slit experiment, then the ratio of
R maximum intensities to minimum intensities in
the interference pattern is
(a) 9 : 1 (b) 1 : 9 (c) 3 : 1 (d) 1 : 3

20 V
32. A policeman blows a whistle with a frequency of
500 Hz. A car approaches him with a velocity of
(a) 20W (b) 30W 15ms -1. The change in frequency as heard by the
(c) 40W (d) 50W driver of the car as he passes the policeman is
(speed of sound in air = 300 ms -1)
25. The output of the given circuit in the figure (a) 25 Hz (b) 50 Hz (c) 75 Hz (d) 45 Hz
R
33. A pure Si wafer is doped with Al. The number
density of acceptor atoms is approximately
1022 m -3 , if density of the hole in an intrinsic
Vm sin ωt D Output semiconductor is approximately 1020 m -3 . The
number density of electron in the wafer is
(a) 104 m-3 (b) 1012 m-3 (c) 1018 m-3 (d) 1024 m-3
(a) would be zero at all times 34. 20 g of water at 50ºC and 8 g of ice at -20º C are
(b) would be like that of a full-wave rectifier mixed together in a calorimetry. What is the final
(c) would be like a half- wave rectifier with positive cycle in temperature of the mixture? (Given, specific heat
output of ice = 0.5 cal g -1 ºC -1 and latent heat of fusion of
(d) would be like a half- wave rectifier with negative cycle in ice = 80 cal g -1)
output (a) -5 ºC (b) 5ºC (c) 20ºC (d) 28ºC

MODULE 11
www.jeebooks.in
186 NEET Test Drive

35. For the given an uniform square lamina ABCD, 40. Three blocks of mass 5 kg, 4 kg and 1 kg,
whose centre is O respectively are in contact on a frictionless table as
E shown in the figure. If a force of 25 N is applied on
A B
the 5 kg block, the contact force between 5 kg and
4 kg block will be
G H
25 N
O 5 4 1

D F C (a) 12.5 N (b) 12 N


(a) 2IAc = IEF (b) IAD = 3IEF (c) IAC = 2 IEF (d) IAC = IEF (c) 2.5 N (d) 10 N

36. An uncharged capacitor with a solid dielectric is 41. A bar magnet is oscillating in the earth’s
connected to a similar air capacitor charged to a magnetic field with a time period T . What happens
potential of 50 V. If the common potential after to its time period and motion, if its mass is
sharing the charges becomes 2V, then the dielectric quadrupled?
constant of dielectric must be (a) Motion remains SHM and time period remains nearly
(a) 16 (b) 24 (c) 36 (d) 20 constant
37. In the following circuit, current through 2 W (b) Motion remains SHM with time period = 2T
(c) Motion remains SHM with time period = 4T
resistor just after switch closed is
(d) Motion remains SHM and time period = T / 2

42. The length of a simple pendulum is about 100 cm


6V known to an accuracy of 1 mm. Its period of
2H 1mF 6Ω 0.5Ω
oscillation is 2s, determined by measuring the
time for 100 oscillations using a clock of 0.1 s
1H resolution. What is the accuracy in the determined
2mF
2Ω value of g ?
(a) 0.5% (b) 0.25%
(Switch) (c) 0.2% (d) 1.0%
(a) 3 A (b) 2.25 A (c) 0.71 A (d) 0A 43. The work function of metal is in the range of 2eV
38. Match the corresponding entries of Column I with to 5eV. Then which of the following wavelength of
Column II and choose the correct option from the light cannot be used for photoelectric effect. (take,
codes given below h = 4 ´ 10-15 eV, c = 3 ´ 108 m/s)
Column I Column II (a) 510 nm
(b) 650 nm
A. First law of P. Path dependent
(c) 400 nm
thermodynamics
(d) 570 nm
B. DW Q. DU = 0
C. Isothermal process R. pDV 44. A solid ball is rolling on a frictionless horizontal
surface with a translational velocity v ms -1 as
D. DQ S. Conservation of energy
shown in the figure. If it has to climb the inclined
Codes surface, then its velocity should be
A B C D A B C D
(a) S Q R P (b) S P R Q
(c) S R Q P (d) S Q P R h
v
39. The name of the following gate that the circuit
shown in the figure (a) ³ 2gh (b) 2gh

+10V (c) £ 10gh / 7 (d) ³ 10gh / 7
+

D1 R0
45. The velocity of a particle describing SHM is
16 cms -1 at a distance of 8 cm from mean position
V0 and 8 cms -1 at a distance of 12 cm from mean
position. The amplitude of motion
D2 (a) 15.5 cm (b) 13 cm
(c) 14.3 cm (d) 13.06 cm
(a) AND gate (b) OR gate (c) NOT gate (d) NOR gate

MODULE 11
www.jeebooks.in

PART B CHEMISTRY
46. The general molecular formula of the products 53. Which of the following coordination compound
obtained on heating lanthanoids (Ln) with sulphur would exhibit optical isomerism?
is (a) Pentaammine nitro cobalt (III) iodide
(a) Ln2S3 (b) Ln3S2 (b) Diammine dichloroplatinum (II)
(c) LnS (d) LnS3 (c) trans dicyano bis (ethylene diamine) chromium (III) chloride
(d) tris-(ethylene diamine) cobalt (III) bromide
47. The half-life time of a first order reaction is
1.26 ´ 1014 s, then rate constant of this reaction 54. Consider the reaction, 2N 2O5 ( g) ¾® 4NO2( g) + O2( g).
will be If the concentration of NO2 increases by 8.5 ´ 10- 3 M
(a) 5.5 ´ 10- 10 s -1 (b) 5.5 ´ 10- 15 s -1 in 100 s, then the rate of reaction will be
(c) 6.6 ´ 10- 14 s -1 (d) None of these (a) 1.3 ´ 10- 5 Ms - 1 (b) 2.8 ´ 10- 5 Ms - 1
(c) 7.6 ´ 10- 4 Ms - 1 (d) 2.13 ´ 10- 5 Ms - 1
48. Equivalent conductance of saturated MgSO4 is
380 W- 1 cm 2 equiv - 1 and specific conductance is 55. CFCs, HCFCs and halons destroy the earth’s
5 ´ 10- 4 W- 1 cm - 1. Thus, K sp of MgSO4 is protective ozone layer, which shields the earth
from harmful ultraviolet rays generated from the
- 10 2 -8 2
(a) 8.38 ´ 10 M (b) 43.56 ´ 10 M sun. Why CFC are widely used in air conditioners?
(c) 59.10 ´ 10- 4 M 2 (d) 23.25 ´ 10- 8 M 2 (a) Because CFC are highly reactive
(b) It is highly inflammable
49. Which of the following element does not belong to (c) It is non-reactive
4d series of transition elements?
(d) All of the above are true
(a) Molybednum (b) Titanium
(c) Technetium (d) Zirconium 56. Which of the following statements is incorrect for
cathode rays?
50. Which of the following pair of solution is expected (a) The cathode rays start from cathode and move towards the
to show isotonic at same temperature? anode
(a) 0.2 M urea and 0.2 M KCl (b) In the absence of electrical or magnetic field, these rays
(b) 0.1 M urea and 0.2 M CaCl 2 travel in straight lines
(c) 0.1 M KCl and 0.1 M Na 2SO 4 (c) The characteristics of cathode rays depend on the material
(d) 0.1 M Ca(NO 3 )2 and 0.1 M Na 2SO 4 of electrodes and the nature of the gas present in the
cathode rays tube
51. The product of the following reaction,
(d) Cathode rays consist of negatively charged particles
OH
57. The bond between carbon atom (1) and carbon
1 2
Br2/Fe
?, is atom (2) in compound N ºº C ¾ CH == CH 2 involves
CHO the hybrid orbitals
(a) sp2 and sp2 (b) sp3 and sp (c) sp and sp2 (d) sp and sp
OH OH
58. IP1 and IP2 of Mg are 178 and 348 kcal mol - 1. The
(a) (b) energy required for the reaction,
Br CHO CHO Mg ¾® Mg2 + + 2e - is
Br (a) + 170 kcal (b) + 526 kcal (c) - 170 kcal (d) - 526 kcal
OH OH 59. Which of the following has strongest hydrogen
Br Br bonding?
(c) (d) (a) O ¾H ...N (b) F ¾H ...F (c) O ¾H ...O (d) O ¾H ...F
CHO CHO
60. The product formed when acetone is treated with
phenyl magnesium bromide followed by acidic
52. Which of the following is used as an antifreeze in hydrolysis is
cars for cooling the engine? OH
(a) 10% ethanol solution in water OCH3 OH
OH
(b) 35% solution of ethylene glycol (a) (b) (c) (d)
(c) 45% solution of ethylene glycol
(d) 20% ethanol solution in water

MODULE 11
www.jeebooks.in
188 NEET Test Drive

61. Consider the following statement about chelating 67. The pK a of a weak acid HA is 4.92. The pK b of a
ligands: weak base BOH is 4.81. The pH of an aqueous
I. They have more than one donor atom. solution of the corresponding salt, BA, will be
II. They are non-linear and they must form (a) 7.05 (b) 8.29
bonds with reasonable angles at the metal (c) 9.58 (d) 4.79
atom.
68. Consider the following compounds :
III. Ethylenediamine is a bidentate chelating
ligand. I. cis-[Co(NH3 )4Cl2 ]+ II. trans-[Co(en)2Cl2 ]+
The correct statement is/are III. cis-[Co(en)2Cl2 ]+ IV. [Co(en)3 ]3 +
(a) Only I (b) Both I and II
Which of the given compound show optical
(c) Both II and III (d) I, II and III isomerism?
62. The reagent and condition required for the (a) Both I and II (b) Both II and III
following reaction is (c) Both III and IV (d) I, III and IV
CHO 69. Which of the following order is correct for the
½ acidic nature of the compounds?
( CHOH )4 ¾® CH3 ¾ (CH 2 )4 ¾ CH3 (a) CH3OH < CH3SH < CH3NH2
½ n hexane -

(b) CH3NH2 < CH3OH < CH3SH


CH 2OH
(c) CH3SH < CH3OH < CH3NH2
(a) HI, D (b) HBr, D
(d) CH3SH < CH3NH2 < CH3OH
(c) HF, D (d) HCl, D
70. Within each transition series, the oxidation state
63. In order to prepare a 1° amine from an alkyl halide
(a) first decreases till the middle of period and then increases
with simultaneous addition of one CH 2 group in (b) first increases till the middle of the period and then decreases
the carbon chain, the reagent used as a source of (c) decreases regularly on moving from left to right
nitrogen is (d) None of the above
(a) potassium phthalimide, C 6H4 (CO)2 N-K +
(b) potassium cyanide, KCN 71. Which of the following statement(s) is incorrect for
(c) sodium azide, NaN3 oxides of chlorine?
(d) sodium amide, NaNH2 (a) ClO 3 forms dimer due to unpaired electron
(b) Cl 2O 6 is the anhydride of HClO 3
64. If 8 g of NaOH is dissolved in water to make 1 L of (c) ClO 3 is paramagnetic and exists in liquid state
solution, then the pH of solution is (d) Cl 2O 6 is diamagnetic in solid state
(a) 13.70 (b) 10.28
(c) 8.42 (d) 12.89 72. In the estimation of nitrogen by Kjeldahl’s method,
2.8 g of an organic compound required 20 millimole
65. The number of chain isomers of C7H16 are of H 2SO4 for the complete neutralisation of NH 3 gas
(a) 6 (b) 4 evolved. The percentage of nitrogen in the sample
(c) 9 (d) 7 is
(a) 20% (b) 10%
66. Product of the following reaction is (c) 40% (d) 30%
NH2
73. Which of the following reagents will be used to
CH3COCI complete below conversion?
Pyridine / HCI NH2 NO2

NH2 NH2 ?

COCH3
(a) (b)
(a) KMnO 4 (b) H2SO 4
(c) P2O 5 (d) CF3COOOH
COCH3
74. When conc. H 2SO4 is heated with P2O5 , the acid is
NH2 NHCOCH3 converted to
(a) a mixture of sulphur dioxide and sulphur trioxide
(c) (d) (b) sulphur
COCH3 (c) sulphur dioxide
(d) sulphur trioxide

MODULE 11
www.jeebooks.in
Prep Catalysis for NEET ~ Mock Test 5 189

75. A flux is chemical leaning agent, flowing agent or 85. Match Column I with Column II, identify correct
purifying agent. They may have more than one answer by using the codes given below.
function at a time. They are used in both extractive
metallurgy and metal joining. Which of the Column I Column II (Entropy)
following is an example of acidic flux? æ DEcell ö
A. Endothermic process 1. DS = nF çè ÷
(a) SiO 2 (b) CaO DT ø
(c) MgO (d) None of these
B. Residual entropy 2. dS = 0
76. The electronic configurations of Cr2 + , Mn2 + , Fe2 + C. Entropy of cell reaction 3. DS > 0
and Ni2 + are 3d 4, 3d5 , 3d 6 and 3d 8 respectively.
Which one of the following aqua complexes will D. Cyclic process 4. S R = k InW = R InW
exhibit the minimum paramagnetic behaviour?
A B C D
(a) [Mn(H2O)6 ]2 + (b) [Fe(H2O)6 ]2 +
(a) 1 2 3 4
(c) [Ni(H2O)6 ]2 + (d) [Cr(H2O)6 ]2 +
(b) 4 3 2 1
77. Phosphorus trichloride PCl 3 , undergoes hydrolysis (c) 3 4 1 2
to form an oxoacid. The formula of this oxoacid is (d) 4 1 3 2
(a) H3PO 2 (b) H3PO 4 (c) HPO 3 (d) H3PO 3
86. Which of the following arrangement is possible?
78. Arrhenius proposed a quantitative relationship n l m s n l m s
between rate constant and temperature as 1 1
- Ea (a) 5 2 2 + (b) 2 2 0 -
2 2
k= Ae RT or ln k = ln A - Ea / RT . 1 1
(c) 3 -2 1 + (d) 0 0 1 +
What will be the slope of graph when ln k versus 2 2
1/T is plotted?
Ea Ea 1 87. Which of the following statement(s) is/are correct
(a) (b) - (c) (d) ln A regarding carbonyl group?
R R T
(a) The three groups attached to the carbonyl carbon lie in
79. Low level of noradrenaline is the cause of different planes
depression. The drug used to cure this problem is (b) The three groups attached to the carbonyl carbon lies in
(a) antidepressants (b) hypnotics the same plane
(c) sedatives (d) psychedic (c) The carbonyl carbon is sp-hybridised
80. Anomalous property of lithium is due to (Which of (d) The carbonyl carbon is sp3 -hybridised
the below is most correct choice?)
88. Which of the following statement is correct
(a) smaller size of metal ion
regarding elements of group 18?
(b) smaller charge density
(a) Helium can diffuse through rubber, PVC and even glass
(c) smaller size and higher charge density
(b) Ar, Kr and Xe form clathrate
(d) None of the above
(c) He has the lowest melting point and boiling point
81. The vapour density of a gas is 11.2. The volume (d) All of the above are correct statements
occupied by 11.2 g of this gas at N.T.P is
89. Doping can be done with an impurity which is
(a) 1 L (b) 11.2 L (c) 22.4 L (d) 20 L
electron rich or electron deficient as compared to
82. DE° of combustion of iso-butylene is - x kJ mol - 1. the intrinsic semiconductor, silicon or germanium.
The value of DH ° is Doping of silicon with boron leads to
(a) = DE° (b) > DE° (c) = zero (d) < DE° (a) n-type semiconductor
(b)p-type semiconductor
83. The repeating unit present in asbestos is (c) metal
(a) [Si 4O11 ]6 - (b) [SiO 4 ]4 - (c) [Si 2O 7 ]2 - (d) [Si 3O 9 ]3 -
(d) insulator
84. Consider the following reaction, H 2O( s) - H 2O( l ) 90. Which of the following reactants are used to obtain
The water and ice is at equilibrium at 273 K and methylphenyl ether?
enthalpy change of process is 6 kJ mol -1. The entropy (a) Phenolate ions and methyl iodide
change of conversion of ice into liquid water is (b) Methoxide ions and bromobenzene
(a) 21.98 JK -1 mol -1 (b) - 21.98 JK -1 mol -1 (c) Methanol and phenol
(c) - 21.98 JK -1 . kJ K -1
(d) + 2198 (d) Bromobenzene and methylbromide

MODULE 11
www.jeebooks.in

PART C BIOLOGY
91. The drug whose plant leaves contain atropine and 101. Which of the following ideas was not a part of
is used by doctors to dilate pupil of eye is Charles Darwin’s theory of ‘origin of species by
(a) reserpine (b) aspirin natural selection’?
(c) codein (d) belladonna (a) More offsprings are produced by organism than the
number an environment can support
92. Which of the following enzymes is the replicative (b) Gene mutation is responsible for variation among
polymerase in E. coli ?
individuals
(a) DNA polymerase IV (b) DNA polymerase II
(c) Individuals best adapted to environment can successfully
(c) DNA polymerase III (d) DNA polymerase I
survive and reproduce
93. Transposons are sequences of (d) Individuals of an area tend to compete for scarce
(a) polypeptide chain (b) RNA resources
(c) DNA (d) chromosomes
102. Which of the following genetic disorders runs in
94. Hypothyroidism is characterised by all of the the British royal family?
following except (a) Haemophilia (b) Phenylketonuria
(a) coarse dry skin (c) Thalassemia (d) Colour blindness
(b) Gull’s disease
103. Which method is used to analyse inheritance
(c) increased demand for energy pattern of traits in humans?
(d) heat intolerance
(a) Karyotyping (b) DNA fingerprinting
95. Penicillium undergoes sexual reproduction by the (c) Pedigree analysis (d) All of these
(a) formation of cleistothecia
104. The occurrence of haemophilia is more likely in
(b) formation of conidiophores human males compared to females because
(c) formation of buds (a) females are heterogametic
(d) fragmentation
(b) females can only be carriers
96. Which one of the following diseases is an arthropod (c) the haemophilic allele is present only on X-chromosome
borne disease ? (d) the haemophilic allele is present only on Y-chromosome
(a) Syphilis (b) Leprosy
(c) Typhus fever (d) Cholera 105. Rh factor is named after
(a) mouse (b) human
97. In human eyes, retina has cones which are (c) monkey (d) rabbit
responsible for the coloured vision and are
sensitive to entire visible spectrum. The three 106. Certain glands produce hormones that stimulate
principal cones are other endocrine glands. Identify the hormone
(a) red, blue and green (b) violet, indigo and blue which specifically acts to trigger hormone secretion
(c) red, orange and yellow (d) red, blue and yellow by another gland.
(a) Growth hormone
98. Which among the following is not an advantage of (b) Thyroxine
GM crops ? (c) Adrenocorticotropin hormone
(a) GM plants are able to tolerate cold climate (d) Antidiuretic hormone
(b) GM plants can withstand long periods of drought
(c) GM plants help to eliminate the use of chemical pesticides 107. Which of the following organisms has been
(d) GM plants cause unintended harm to other organisms commercially utilised as blood cholesterol lowering
agent?
99. Which of the following statements is incorrect ? (a) Torulopsis utilis (b) Saccharomyces cerevisiae
(a) JBS Haldane — Natural selection theory (c) Monascus purpureus (d) Pseudomonas putida
(b) de Vries — Mutation theory
(c) Lamarck — Inheritance of acquired characters
108. Who discovered X–body, but was not able to
explain its role ?
(d) Louis Pasteur — Germ theory
(a) Morgan (b) Calvin
100. Non-endocrine hormones are secreted by the cells (c) Henking (d) Beadle
and widely spread in the body. Which of the
following does not represent such hormone? 109. Hyposecretion of anterior pituitary leads to
(a) Insulin (b) Motilin (a) acromegaly (b) Simmond’s disease
(c) Ghrelin (d) Calcitonin (c) Hashimoto’s disease (d) Basedow’s disease

MODULE 11
www.jeebooks.in
Prep Catalysis for NEET ~ Mock Test 5 191

110. Identify the incorrectly matched pair. 119. Match the regions of the vertebral column correctly
(a) Transformation — Free DNA is incorporated into a with their numbers present in humans.
recipient cell
Column I Column II
(b) Transduction — Donor DNA transfer is
mediated by a virus A. Cervical 1. 7
(c) Conjugation — Transfer of DNA involves cell to cell B. Thoracic 2. 1
contact C. Lumbar 3. 12
(d) Sporulation — Formation of cysts
D. Sacral 4. 5
111. The pH of blood in the arteries is 7.4 which is
Codes
(a) higher than the pH in veins (b) lower than the pH in veins
A B C D A B C D
(c) equal to the pH in veins (d) varies in different positions
(a) 1 4 2 3 (b) 4 2 1 3
112. Match the following columns (I and II) and select (c) 1 3 4 2 (d) 4 1 3 2
the correct option.
120. High copy number in a vector is
Column I Column II (a) to provide more replicates of inserted DNA sequences in
the cell
A. Operator gene 1. Synthesises a molecule to block a
gene adjacent to structural genes (b) easy transformation to the host cells
(c) easy isolation and purification
(d) small size or low molecular weight for enhanced stability
B. Structural gene 2. Provides binding site for
repressor protein 121. Identify the correctly matched pair.
(a) Wet deposition — Acidic water received through rain
C. Promoter gene 3. DNA sequences that initiate
transcription (protein produced a (b) Dry deposition — Deposition of wind blown acidic gases
particular gene) (c) Air pollution — Biomagnification
(d) Both (a) and (b)
D. Regulator gene 4. Provides binding site for activator
proteins and RNA polymerase 122. Which of the following is not exclusively composed
of glucose molecules ?
Codes
(a) Maltose (b) Glycogen
A B C D A B C D (c) Lactose (d) Cellulose
(a) 2 3 4 1 (b) 2 1 3 4
(c) 3 1 4 2 (d) 3 4 1 2
123. Quaternary structure of protein is associated with
(a) three-dimensional structure of a peptide chain
113. Charging of the tRNA is also known as (b) orientation of multi-subunits of a protein
(a) transcription (b) polymerisation (c) amino acid sequence
(c) aminoacylation (d) adenylation (d) hydrogen bonding
114. Myxoxanthin is found only in 124. Arrange the following steps in the process of
(a) Euglenophyta (b) Cyanophyta decomposition.
(c) Phaeophyta (d) Pyrrophyta I. Fragmentation II. Catabolism
115. Presence of human chorionic gonadotropin in the III. Leaching IV. Humification
urine suggests V. Mineralisation
(a) Grave’s disease (b) goitre Choose the correct option.
(c) pregnancy (d) diabetes (a) I II III IV V
(b) I III II IV V
116. The pattern of cleavage in the human zygote is
(a) determinate (b) holoblastic (c) indeterminate (d) meroblastic (c) III I II IV V
(d) III I IV II V
117. Rigor mortis appears first on
(a) neck (b) lower jaw 125. The term (K - N/K) is
(c) legs (d) lower abdomen (a) carrying capacity for any given population
(b) greatest when K is large
118. Endarch is (c) zero when population size gets equal to carrying capacity
(a) protoxylem lying to the inner side of the metaxylem (d) increased when N approaches K
(b) protoxylem lying in the middle of metaxylem
(c) protoxylem lying outside the metaxylem 126. Largest herbarium of the world, National Museum
(d) protoxylem lying in the centre of metaxylem of Natural History is situated in
(a) New York (b) Paris (c) Kew (d) Geneva
MODULE 11
www.jeebooks.in
192 NEET Test Drive

127. Which of the following is an example of buffer 132. Zoochlorella is an example of


system in blood? (a) epizoic algae (b) endozoic algae
(a) Oxygen and carbon dioxide (c) epiphytic algae (d) holophytic algae
(b) Oxyhaemoglobin and carboxyhaemoglobin
(c) Bicarbonate and carbonic acid
133. Match the Column I with Column II and choose
the correct option from the codes given below.
(d) Sodium sulphate and oxalic acid
Column I Column II
128. Match the tools in Column I with their
explanations in Column II and choose the correct A. Obligate 1. Kill the host cells or tissues.
option from the codes given below. parasites

Column I Column II B. Destructive 2. Depend upon the host for the


parasites supply of the needs of parasite.
A. Shotgun 1. To make a gene
C. Balanced 3. Normally live as parasites, but
B. Antisense genes 2. To separate fragments of
parasites become saprophytes under
DNA
emergent conditions.
C. Gene synthesis 3. To stop the expression of a
D. Facultative 4. Grow only upon suitable living
gene in a cell
saprophytes host tissues.
D. Electrophoresis 4. To find a particular gene in a
whole genome Codes
A B C D A B C D
Codes
(a) 1 2 3 4 (b) 4 3 2 1
A B C D A B C D
(c) 1 2 4 3 (d) 4 1 2 3
(a) 1 2 3 4 (b) 4 2 1 3
(c) 4 3 1 2 (d) 3 2 1 4 134. Identify the incorrect statements.
I. Rabbit eats its own faeces.
129. Given below is a part of a polysaccharide chain. II. Mouse never drinks water.
CH2OH CH2OH
III. Hippopotamus is called the ‘Horse of the
H O H H O H river’.
X H X H X IV. Gibbon is the African dog-like monkey.
OH H H HO
(a) I and III (b) II and IV
(c) I and IV (d) II and III
H OH OH O
135. Which of the following is not a larval stage of blood
Part labelled X, represents which type of bond? fluke ?
(a) ¾ O ¾, hydroxyl bond (b) ¾ C ¾, glycosidic bond (a) Sporocyst (b) Redia (c) Cercaria (d) Miracidium
½½
O 136. The lymph vessels within the villi help in
(c) — N — , peptide bond
(a) absorption of glycerol, fatty acids and monoglycerides

H (b) absorption of glucose, galactose and fructose


O (c) absorption of fatty acids, glycerols and galactose
(d) O ==P , phosphodiester bond (d) absorption of amino acids and monosaccharides
O–
O– 137. Match the Column I with Column II and choose
the correct option from the codes given below.
130. Which of the following is incorrect with respect to
plant hormones? Column I Column II
(a) They are synthesised in specific glands and tissues A. Negative soil 1. Reduction in soil productivity due to
(b) They are active even in very small concentrations pollution addition of undesirable substances
(c) They are pleiotropic in their effects
B. Positive soil 2. Congregating fertile land into
(d) Responses to plant hormones are different depending on
pollution barren area
the tissue or time
C. Landscape 3. Reduction in soil productivity due
131. Which of the following events does not take place pollution to erosion and overuse
during the mitotic prophase?
Codes
(a) Disappearance of nuclear envelope
(b) Nucleolar disaggregation A B C A B C
(a) 1 2 3 (b) 1 3 2
(c) DNA replication
(c) 3 1 2 (d) 2 1 3
(d) Coiling of chromosome

MODULE 11
www.jeebooks.in
Prep Catalysis for NEET ~ Mock Test 5 193

138. Which variety of silk is produced in Asom ? 146. The reduction of nitrate to nitrite during nitrogen
(a) Tussar silk (b) Shantung silk assimilation is carried out by
(c) Muga silk (d) Mulberry silk (a) nitrite reductase and Mg
(b) nitrate reductase and FAD
139. Which of the following is true for F1-particle of the
(c) nitrate reductase and ferredoxin
oxysome ? (d) nitrite reductase and aspartate
(a) It utilises proton energy
(b) It releases proton energy 147. Which method is used when there are only a very
(c) It has no role related to energy consumption few cells available, such as fertilised egg cells ?
(d) It is present in the membrane of chloroplast (a) Heat shock (b) Electroporation
(c) Gene gun (d) Microinjection
140. Which statement is true about the climacteric
fruits ? 148. Which of the following cofactors is/are required for
(a) These fruits show increased respiratory activity at the time the formation of acetyl Co-A?
of ripening (a) Mg 2+ and Co-A
(b) These fruits are developed by artificial method of (b) TPP (Thiamine Pyrophosphate)
fertilisation at a certain period of year (c) Lipoic acid
(c) These fruits are produced throughout the year in all climates (d) All of the above
(d) None of the above
149. A sugar is called reducing because of the presence of
141. The relationship between species X and species Y (a) a free aldehyde or ketone group
is described as commensalism. This indicates that (b) a free oxygen
(a) both species derive mutual benefits like insect pollinators (c) a free hydroxyl group
(b) both species suffer as between Plasmodium and its vector (d) None of the above
(c) X derives benefit while Y is harmed like lice living on
human body 150. Glissons’ capsule and Kupffer cells are present in
(d) X derives benefit whereas Y remains unaffected as which of the following parts?
between Remoras and sharks (a) Stomach (b) Gall bladder
(c) Liver (d) Duodenum
142. Find the odd one out in the given list.
I. Diaphragms II. Cervical caps 151. The transpiration pull overcomes
III. Spremicidal jellies IV. Condoms (a) gravitation pull
(b) resistance of narrow xylem channels and their end walls
V. Nirodh VI. Saheli
(c) resistance offered by water coming out of narrow capillary
VII. Vaults
pores to the soil
Choose the correct option. (d) All of the above
(a) Only I (b) Only III (c) Only V (d) Only VI
152. In case of Gloriosa superba, autogamy is
143. The protein present in raw egg white responsible prevented due to the presence of a barrier in
for biotin deficiency is between stamens and style. This is technically
(a) aleurone (b) riboflavin (c) avidin (d) isoniazid known as
(a) dichogamy (b) herkogamy
144. The disadvantage of breeding activities over the
(c) protogyny (d) heterostyly
technique of GM crops production is
(a) breeding activities can use only those genes, which are 153. Which among the following is/are functions of
present in such species that can be hybridised with them cortex ?
(b) any gene can be used for transfer (a) It serves as a protective zone
(c) change in genotype can be precisely controlled (b) It helps in absorption and translocation of water
(d) produces a desired phenotype (c) Both (a) and (b)
(d) None of the above
145. A distinctive feature of an insect pollinated plant is
that 154. A patient had developed characteristic points on
(a) their pollen grains tend to have rougher surface teeth called ‘Hutchinson’s teeth’. Waksman test
(b) they produce sugary fruits was used to detect this disease as
(c) they have nectar, which guides on their flowers (a) chicken pox (b) smallpox
(d) pollen grains are produced in abundance to make up any (c) French pox (d) None of these
loss occurring due to wind

MODULE 11
www.jeebooks.in
194 NEET Test Drive

155. In plants of family Solanaceae, the ovary is 162. Mark the incorrect statement.
(a) bicarpellary, syncarpous, superior (a) F Redi proposed the theory of special creation
(b) monocarpellary, syncarpous, superior (b) Spallanzani stated that air carried microorganisms
(c) tricarpellary, synocarpous, superior (c) Arrhenius is the father of modern palaeontology
(d) multicarpellary, apocarpous, superior (d) Oparin proposed that life originated on earth by abiogenic
molecular evolution
156. Match the methods of asexual reproduction given
in Column I with their explanation given in 163. Which of the following enzymes promotes the
Column II and choose the correct option from the spermatozoan to penetrate the egg during
codes given below. fertilisation ?
(a) Acid phosphatase (b) Hyaluronidase
Column I Column II
(c) Amylase (d) Phosphatase
A. Fragmentation 1. Simple cell division
164. Which of the following amino acids has only a
B. Fission 2. Outgrowth proliferates from single codon ?
thallus (a) Isoleucine (b) Tryptophan
C. Budding 3. Arise as modified branches (c) Asparagine (d) Glutamic acid
D. Gemmae 4. Thallus breaks down into many 165. In which of the following can formylmethionine
fragments tRNA be found ?
Codes (a) Insects (b) Mammals
(c) Bacteria (d) Yeasts
A B C D A B C D
(a) 4 1 2 3 (b) 1 2 3 4 166. Glans of the penis are equivalent to ..........in
(c) 2 1 4 3 (d) 3 2 1 4 female reproductive system.
(a) vestibule (b) hymen
157. Rhodospirillum is an example of (c) vagina (d) clitoris
(a) cyanobacteria (b) prochlorobacteria
(c) purple or green bacteria (d) mycoplasma 167. The parasites harm the host by
(a) reducing the survival, growth and reproductive ability of
158. Keratinised dead layer of skin is made up of the host
(a) stratified columnar epithelium (b) reducing its population density
(b) simple columnar epithelium (c) might render the host more vulnerable to predation by
(c) stratified squamous epithelium making it physically weak
(d) simple cuboidal epithelium (d) All of the above
159. Phycomycetes are characterised by 168. Name the enzyme involved in ornithine cycle.
I. Aseptate, coenocytic hyphae. (a) Oxidase (b) Dehydrogenase
II. Asexual spores, produced endogenously. (c) Amylase (d) Arginase
III. Show hetero or isogamous sexual
reproduction. 169. After consumption of a meal containing greasy
French fries, which hormones and enzymes would
IV. Divided into oomycetes and zygomycetes.
be most active during digestion process?
Choose the correct option containing true
I. Bile salts, lipase and enterogastrone.
statements.
(a) I and II (b) I and III II. Salivary and pancreatic amylase along with
(c) I and IV (d) All are correct disaccharidases.
III. Pepsin, trypsin, chymotrypsin,
160. Identify the incorrectly matched pair. aminocarboxypeptidases, dipeptidases and
(a) Stem rust of wheat — Puccinia graminis tritici CCK.
(b) Brown rust of wheat — Puccinia recondita IV. Salivary amylase, lipase and protein
(c) Loose smut of wheat — Puccinia striiformis degrading enzymes.
(d) Covered smut of barley— Ustilago hordei Choose the correct option.
(a) I and II (b) I and III
161. Which of the following characteristics is never
(c) III and IV (d) II and IV
present at any stage of development in primary
chordates ? 170. Which among the following causes damages to
(a) Post anal tail (b) Pharyngeal slits chloroplast in plants ?
(c) Dorsal tubular nerve cord (d) None of these (a) Ozone (b) PAN (c) Phenol (d) Aldehyde

MODULE 11
www.jeebooks.in
Prep Catalysis for NEET ~ Mock Test 5 195

171. Which of the following statements is true for 176. It is not advised to sleep in a closed room with fire
arrhenotoky and thelytoky? burning inside as it could be fatal because
(a) They are forms of sexual reproduction (a) excessive heat
(b) Arrhenotoky is found in Peiplaneta species (b) non-availability of oxygen
(c) In thelytoky, the diploid egg parthenogenetically develops (c) Hb has higher affinity to CO
into males (d) combined effect of (b) and (c)
(d) In arrhenotoky, the male develops from unfertilised egg
177. Which among the following does not come under
and the female from fertilised egg
the factors affecting primary productivity?
172. The causes of biodiversity loss include (a) Environmental factor
(a) habitat fragmentation (b) Nutrients availability
(b) overexploitation (c) Photosynthetic efficiency of plants
(c) intensive agriculture (d) Leaching
(d) All of the above
178. Which one of the following diseases is due to
173. Waksman and Woodruff were responsible for the hypersensitivity?
isolation of (a) Hay fever (b) Breakbone fever
(a) actinomycin and streptothricin (c) Scarlet fever (d) Enteric fever
(b) penicillin and streptomycin
179. Select the option which is not the true reason for
(c) actinomycin and vancomycin
polyembryony in angiosperms.
(d) erythromycin and vancomycin
(a) Cleavage of proembryo
174. A portion of the endoplasmic reticulum is present (b) Formation of many embryos due to the presence of more
in plasmodesmata, it is known as than one embryo sac in the same ovule
(a) myeloid body (b) cisternae (c) Development of many embryos from egg
(c) vesicles (d) desmotubule (d) Formation of many embryos from structures like nucellus
and integuments
175. Which of the following pigments is not present in
the chloroplast? 180. Which one of the following pigments is water soluble?
(a) Anthocyanin (b) Chlorophyll-b (a) Chlorophyll-a (b) Chlorophyll-b
(c) Carotenoid (d) Xanthophyll (c) Phycobilin (d) Xanthophyll

Answer Sheet
1. (b) 2. (a) 3. (c) 4. (b) 5. (a) 6. (d) 7. (c) 8. (a) 9. (d) 10. (d)
11. (c) 12. (c) 13. (c) 14. (d) 15. (b) 16. (b) 17. (d) 18. (c) 19. (d) 20. (b)
21. (b) 22. (a) 23. (b) 24. (c) 25. (c) 26. (d) 27. (c) 28. (c) 29. (d) 30. (c)
31. (a) 32. (b) 33. (c) 34. (d) 35. (d) 36. (b) 37. (a) 38. (c) 39. (a) 40. (a)
41. (b) 42. (c) 43. (b) 44. (d) 45. (d) 46. (a) 47. (b) 48. (b) 49. (b) 50. (d)
51. (b) 52. (b) 53. (d) 54. (d) 55. (c) 56. (c) 57. (c) 58. (b) 59. (b) 60. (b)
61. (d) 62. (a) 63. (b) 64. (a) 65. (c) 66. (d) 67. (a) 68. (c) 69. (b) 70. (b)
71. (b) 72. (a) 73. (d) 74. (d) 75. (a) 76. (c) 77. (d) 78. (b) 79. (a) 80. (c)
81. (b) 82. (d) 83. (a) 84. (a) 85. (c) 86. (a) 87. (b) 88. (d) 89. (b) 90. (a)
91. (d) 92. (c) 93. (c) 94. (d) 95. (a) 96. (c) 97. (a) 98. (d) 99. (a) 100. (d)

101. (b) 102. (a) 103. (c) 104. (c) 105. (c) 106. (c) 107. (c) 108. (c) 109. (b) 110. (d)
111. (a) 112. (a) 113. (c) 114. (b) 115. (c) 116. (b) 117. (b) 118. (a) 119. (c) 120. (a)
121. (d) 122. (c) 123. (a) 124. (b) 125. (c) 126. (b) 127. (c) 128. (c) 129. (b) 130. (a)
131. (c) 132. (b) 133. (d) 134. (b) 135. (b) 136. (a) 137. (c) 138. (c) 139. (a) 140. (a)
141. (d) 142. (d) 143. (c) 144. (a) 145. (a) 146. (b) 147. (d) 148. (d) 149. (a) 150. (c)
151. (d) 152. (b) 153. (c) 154. (c) 155. (a) 156. (a) 157. (c) 158. (c) 159. (d) 160. (c)
161. (d) 162. (c) 163. (b) 164. (b) 165. (c) 166. (d) 167. (d) 168. (d) 169. (a) 170. (b)
171. (d) 172. (d) 173. (a) 174. (d) 175. (a) 176. (d) 177. (d) 178. (a) 179. (c) 180. (c)

MODULE 11
www.jeebooks.in

EXPLANATIONS
PHYSICS
0.003 2 ´ 0.005 0.06 ù
1. The forbidden energy gap of Ge and Si 3/2 Ω =é + + ´ 100
are 078
. eV and 11
. eV, respectively. A B êë 0.3 05
. 6 úû
2. Surface energy, E = T ´ 2 A = 4%
New surface energy, df BA
11. As we know, e = =
E 1 = T ´ 2 æç A ö÷ = TA
3 3 dt t
1Ω
è4 ø 2 2.5 ´ 20 ´ 20 ´ 10-4 ´ 25
= = 12.5 V
E - E1 20 ´ 10-2
% decrease in E = ´ 100 = 25% 3/5 Ω
E Displacement 20 ´ 10-2
A B QDt = =
3. As, m = tan q p Velocity 25
\ R eq = 0.6W
\ Here, e2 (12.5) 2
8. Let G be the centroid of DABC. So, power = = = 3.125 W
q p = qi = tan-1 173
. = tan-1 3 = 60º R 50
4. The radius vector and velocity vector Then, CD = AC 2 - AD 2 = 2 2 - 12
12. Speed of sound in brass rod,
of the planet are mutually = 4 -1 = 3
Y 1012 106
perpendicular at any point. Then, the C (5 kg) v = = = cm s -1
angular momentum at point P is r 9 3
LP = mP rP vP 2m Length of rod, L = 1m = 100 cm
Similarly at point A is LA = mP r Av A v v
2m Frequency of note, n = =
By conservation of angular momentum, l 2L
LP = LA Þ rP vP = r Av A G 106
= = 1666.67 Hz
or
vP r
= A 3 ´ 2 ´ 100
(4 kg) D (3 kg)
vA rP 13. When A is joined to B, then peak value
A B
As, r A > rP 1m 1m current,
So, vP > v A e
2 2 2 I0 = …(i)
AG = BG = CG = CD = 3 = 2R
5. Path difference for constructive 3 3 3
interference is nl. Now, when A is connected to C, then
Moment of inertia about an axis current flowing through R be I 0 .
(where, n = 1, 2, 3, 4, 5) through centroid G and perpendicular
to plane of DABC is Hence, total heat produced across R is
6. As, F µ s -1/ 3 given by
2 2 2
I = 4 ´ AG + 3 ´ BG + 5 ´ CG
Þ a µ s -1/ 3 [QF = ma] 2
Le 2
(2L)I 02 = (2L) æç
4 1 1 e ö
-1/ 3 2 2/3 = (4 + 3 + 5) CG 2 = 12 ´ = 16 kg-m 2 H = ÷ =
Þ v × dv µ s ds or v µs 2 2 è 2R ø 4R 2
3
Þ v µ s 1/ 3 F 10
9. ay = = = 1ms -2 14. Line ab, passing through origin, so
As P = Fv Þ P µ s -1/ 3 s 1/ 3 = s 0 m 10 isobaric process
7. Equivalent circuit is shown in the figure. Distance in vertical direction in 4 s, DWab = pDV = nRT = 3 ´ 2 ´ (700 - 400)
1Ω –1 = 1800 cal
10 kg 1.5 ms
1Ω DWca = 0
1Ω In cyclic process, DQ = DW
DQ = DWab + DWbc + DWca
F = 10 N
-1500 = 1800 + DWbc + 0
1 1
1Ω sy = ayt 2 = ´ 1 ´ 16 = 8 m DWbc = -3300 cal
2 2
15. Refractive index, m = tani p = tan76º = 4
Distance in horizontal direction in 4s,
On further solving we will have s x = 15
. ´ 4 = 6m 16. Weight of suspended body = Stretching
following equivalent figures force
\ Displacement
1/2 Ω 1Ω Þ mg = kx and mg ¢ = kx ¢
A B = s x2 + s y2 = 2
8 + 6 = 10 m2
g¢ x¢
Þ =
⇒ 10. As we know, density r =
m g x
pr 2L g¢
1Ω Dr Dm 2 Dr DL ù or x ¢ = ×x
\ ´ 100 = é + + ´ 100 g
r êë m r L úû

MODULE 2
www.jeebooks.in
Prep Catalysis for NEET ~ Mock Test 5 197

g¢ R - d 6400 - 3200 1 22. Energy stored per unit volume 29. By Brewster’s law,
Also, = = =
g R 6400 2 1 Dl
2
Y (strain) 2 = Y æç ö÷
1
1 =
Þ x ¢ = ´ 1 = 05
. cm 2 2 èL ø 30°
2
Given, elongation strain = 2%
17. Time period,T µ n 3 2 90°
1 æ 2L ö
T2 (2) 3 Þ Y = ´ 5 ´ 1010 ´ ç ÷
\ = 2 è 100 L ø
T3 (3) 3
= 107 Jm -3 1
T3 =
27T 2 m = tani p = tan 30º =
8 23. Internal energy is not a path function. 3
27 So, DV = DQ - DW 30. As we know, force
= ´ 4 ´ 10-16
8 24. Voltage across R is 20 V. 1 q1 q 2
F = …(i)
. ´ 10-15 s
= 135 V2 4 pe0 r 2
P =
Pav E 2 max R 4 p 2mr 2
18. As, wave intensity, I = = Also, F = mrw 2 = …(ii)
4 pr 2
2m 0c (20) 2 T2
P1 =
mccPav R By Eqs (i) and (ii), we get
Þ E max =
2 pr 2 (20) 2 (4 pe0 ) r 2 (4 p 2mr )
P2 = T2 =
10 q1 q 2
(4 p ´ 10-7 ) (3 ´ 108 ) (3)
= (Power dissipated by 10W) 1
2 p (0.30) 2 é 4 p 3mr 2 ù 2
So, P1 + P2 = 50 So, T =ê
= 45 V / m ú
(20) 2 (20) 2 ë kq1 q 2 û
+ = 50
19. Rotational energy is minimum when R 10 31. Given, a1 = 2a Þ a2 = a
moment of inertia is minimum at Þ R = 40W
distance of x from mass 0.5 kg, then amax = a1 + a2 = 3a
25. The diode become reverse bias in a min = a1 - a2 = a
. x 2 + 15
I = 05 . (2.5 - x ) 2
positive half cycle, so only positive half 2
dI I max a max (3a) 2 9
Now, =0 cycles appear across the output. \ = 2 = =
dx I min a min (a) 2 1
26. Total gravitational potential at point O
Þ x + 3 (2.5 - x )(-1) = 0 I max : I min = 9 : 1
due to each of mass 1 kg is
Þ x = 1875
. m
32. When car approaches policeman,
20. Number of photons emitted per –8 –4 –2 –1 0 1 2 4 8 v - vo 300 - (-15)
n¢ = ´n= ´ 500 = 525
second, v - vs 300 - 0
G ´1 G ´1 G ´1 G ´1 ù
P Pl V = 2é- - - - ...
N = = êë 1 2 4 8 úû When car passes the policeman,
æ hc ö hc
ç ÷ v - vo 300 - 15
= -2G é1 + + 2 + 3 + ....ù
èl ø 1 1 1 n¢¢ = ´n= ´ 500 = 475 Hz
êë 2 2 2 úû v - vs 300 - 0
9 ´ 10-3 ´ 6.6 ´ 10-9 ~ 3 ´ 1016 / s
N = - é ù Change in frequency
6.6 ´ 10-34 ´ 3 ´ 108 ê 1 ú = n¢- n¢¢ = 525 - 475 = 50 Hz
21. LetT1 be the time of acceleration and =- ê
2G ú = -4G
ê æç1 - 1 ö÷ ú 1020
T 2 for deacceleration, then êë è 2 ø úû 33. For intrinsic semiconductor, ni =
m3
T1 + T 2 = T
Magnitude ofV = 4G 10 22
Density of holes, n h =
27. The raw egg has some fluid in it and m3
thus on spinning the egg fluid is thrown As, .
nh ne = ni2
vmax outward while the hard boiled egg is
(1020 ) 2
solid from inside. So, Þ ne = = 1018 m-3
1022
I r > Ib
As, Iw = constant
34. Let final temperature of mixture be t º C.
T Heat lost by water in calories,
O \ wr = wb
T1 T2 H1 = 20 ´ 1 ´ (50 - t ) = 1000 - 20t
wr
or <1
v max v wb Heat from ice, H 2 = msDt + mL + msw t
From figure, a = Þ T1 = max
T1 a tr éQ w = 2 p ù = 8 ´ 05
. ´ 20 + 8 ´ 80 + 8 ´ 1 ´ t
or >1
v max v tb êë T úû H 2 = 720 + 8t = 1000 - 20t [QH1 = H 2 ]
and b = Þ T 2 = max
T2 b t = 28º C
28. Half-life depends only on nature of
v max v max abT radioactive substance and it does not 35. Let I be the moment of inertia of square
\ + = T Þ v max =
a b a+ b depend on the temperature. lamina about an axis passing through O
and perpendicular to its plane.

MODULE 2
www.jeebooks.in
198 NEET Test Drive

By perpendicular axes theorem, 38. A. First law of thermodynamics is 42. For a pendulum, the time period,
consequence of conservation of l
I AC + IBD = I energy. T = 2p
I g
or I AC =
2
[QI AC = IBD] B. DW = pDV for constant pressure p.
4 p 2lDg Dl DT
C. For isothermal process DT = 0 and g = Þ = +2
Again, IEF + IGH = I DU µ DT T2 g l T
I 01
. 2 (01
.)
Þ IEF = Þ DU = 0 = + = 0.002
2 100 2 (100)
D. DQ is path dependent.
\ I AC = IEF Therefore, the accuracy in the value of
39. (i) If D1 = D2 = 0, thenV0 = 0. Dg
36. Common potential, g = ´ 100 = 0.2%
Because there is no voltage drop g
C1V1 + C2V2
V = Þ 2 across an ideal diode. 1242 eV ´ nm
C1 + C2 43. l min = = 248.4 nm
(ii) If D1 or D2 any one is high and other 5eV
C0K ´ 0 + C0 ´ 50 is grounded, then alsoV0 = 0.
= 1242 eV ´ nm
C0K + C0 (iii) If both D1 and D2 are +10 V, then l max = = 621nm
2eV
50 output is 10 V which mean high.
2= = 2K + 2 = 50 Thus, 650 nm cannot be used.
K+1 So, it performs like an AND gate.
44. For conversion of energy,
K = 24 25
40. a = = 2.5 ms -2 Initial KE ³ Final PE
37. Just after switch is closed, inductor 5+ 4+1
KE translational + KE rotational ³ mgh
acts like an open switch and capacitor 25 N 5 kg R 1 1
acts like closed switch. Hence, mv 2 + Iw 2 ³ mgh
equivalent circuit will be 2 2
a 1 1 2 v2
mv 2 + ´ mr 2 2 ³ mgh [Qv = rw]
I The equation of motion is 2 2 5 r
6V
6Ω F - R = ma 7
mv 2 ³ mgh Þ v ³
10
gh
25 - R = 5 ´ 2.5 10 7
Þ R = 12.5 N 45. v = w(A 2 - y 2 )
2Ω
41. Time period of oscillation of magnet For 1st case, 16 = w A 2 - 8 2 …(i)
having mass m,
2´6 For 2nd case, 8 = w A 2 - 12 2 …(ii)
\ R eq = = 15
. W I mK 2
T = 2p = 2p On dividing Eq. (i) by Eq. (ii), we get
2+ 6 MB MB
6 (I ® moment of inertia) 16 A2 - 82 A 2 - 64
\ Current, I = = 4A Þ = Þ 22 =
15
.
T µ m
8 A 2 - 12 2 A 2 - 144
\ Current through 2W resistor T1 m1 m 1 Þ 4 A 2 - 576 = A 2 - 64
6 or = = =
= ´I T2 m2 4m 2 Þ A 2 = 170.6 or A = 13.06 cm
2+ 6
6 \ T 2 = 2T1
= ´4=3A
8

CHEMISTRY
46. When lanthanoids (Ln) are heated with t1/ 2 =
0.693 Molarity = 6.6 ´10-4 M
14
sulphur, then Ln2 S 3 are formed. This 1.26 ´ 10 \ K sp = S 2
sulphides are highly stable because Ln = 5.5 ´ 10- 15 s - 1 = (6.6 ´ 10- 4 M) 2
is in + 3 oxidation state. 48. MgSO 4 - Mg 2+
+ SO 24 - = 43.56 ´ 10- 8 M2
S S
47. Given, half-life time of a first order 49. Titanium (Ti) having atomic number 22
reaction = 1.26 ´ 1014 s 1000 ´ Specific conductance
LMgSO4 = belongs to the 3d series of transition
Half-life for a first order reaction is Normality
elements.
0.693 1000 ´ 5 ´ 10-4
given by, t1/ 2 = Normality = 50. Solutions having same number of ions
k 380
-2
will exert equal osmotic pressure and
where, k = rate constant, = 0.132 ´ 10 N expected to show isotonic.
t1/ 2 = half-life time 0.132 ´ 10-2
\ Molarity = Ca(NO 3 ) 2 - Ca 2 + + 2NO -3 , i = 3
2
Na 2 SO 4 - 2Na + + SO 42 - , i = 3

MODULE 2
www.jeebooks.in
Prep Catalysis for NEET ~ Mock Test 5 199

51. Activation OH 60. 65. C7H16 has 9 chain isomers which are as
group follows:
CH3 CH3 + —MgBr
1. CH3 ¾ CH2 ¾ CH2 ¾ CH2 ¾ CH2
Acetone n - heptane
CHO Phenyl magnesium – + ¾ CH2 ¾ CH3
bromide O MgBr CH3
Br2/Fe
Deactivating CH3 ½
group CH3 2. CH3 ¾ CH ¾ CH2 ¾ CH2
OH n - methyl hexane

¾ CH2 ¾ CH3
H3O+ 3. CH3 ¾ CH2 ¾ CH ¾ CH2
CHO ½ ¾ CH2 ¾ CH3
OH CH3
Br 3 - methyl hexane
\ Strongly activating group generally CH3 CH3
H 3C
win over the deactivating group and ½
¾ OH is a ortho/para directing group. 4. CH3 ¾ C ¾ CH2 ¾ CH2 ¾ CH3
p-products predominants. ½
2-phenyl CH3
52. 35% solution of ethylene glycol is used propan-2-ol 2, 2 - dimethyl pentane
as an antifreeze in cars for cooling the CH3
61. All statements are correct. Chelating
engine because at this concentration it ½
ligands have more than one donor
lowers the freezing point of water to
atom. They are non-linear and they 5. CH3 ¾ CH2 ¾ C ¾ CH2 ¾ CH3
255.4 K. This is why this solution is used must form bonds with reasonable ½
as an antifreeze agent. angles at the metal atom. Ethylene CH3
3, 3 - dimethyl pentane
53. Coordination complex diamine is a bidentate chelating ligand.
CH3 CH3
tris-(ethylenediamine) cobalt(III) 62. On prolonged heating with HI, glucose
bromide would exhibit optical ½ ½
forms hexane, suggesting that all 6 6. CH3 ¾ CH ¾ CH ¾ CH2 ¾ CH3
isomerism. carbon atoms are linked in a straight 2, 3 - dimethyl pentane
3+ 3+ chain. CH3 CH3
en en CHO
½ ½
½ 7. CH3 ¾ C ¾ C ¾ CH3
en Co Co en ( CHOH ) 4 ¾HI
¾® ½
½ D
CH3
en en CH2OH 2, 2, 3 - trimethyl butane

CH3 CH2 CH2 CH2 CH2 CH3 CH3 CH3


54. For, the reaction, n hexane
-
½ ½
63. In order to prepare 1° amine from an 8. CH3 ¾ CH ¾ CH2 ¾ CH ¾ CH3
2N2O 5 (g) - 4NO 2 (g) + O 2 (g)
alkyl halide with simultaneous addition
2, 4 - dimethyl pentane
Rate of reaction of one CH2 group in the carbon 9. CH3 CH2 ¾ H C ¾ CH2 CH3
1 d [NO 2 ] 8.5 ´ 10- 3 ½
= = chain, the reagent used as a source of
4 dt 4 ´ 100 nitrogen is KCN. Chemical CH2 CH3
3 -ethyl pentane
= 2.13 ´ 10- 5 Ms - 1 transformation can be shown as:
KCN 66. NH2 NHCOCH3
55. CFC (chlorofluorocarbon) is X C N
–KX
non-reactive and non-toxic hence used Alkyl halide
CH3COCI
in air conditioner. Pyridine/HCI
Na/C2H5OH
CH2NH2
56. The characteristics of a cathode rays
Amine
not depend on the material of Formation of N-acylated product is due
electrodes and the nature of gas 64. Weight of NaOH dissolved = 8 g to presence of lone pair of electrons on
present in the cathode ray tube. \ Equivalent of NaOH dissolved nitrogen atom.
8
57. C1 has 2s and 2p-bonds = = 0.2 67. For a salt of weak acid and weak base
(sp-hybridised) while C2 has 3s and 40
1
1p-bond (sp 2 -hybridised). \ NNaOH = 0.2 N pH = - [log K a + log Kw - log Kb ]
2
58. Removal of two electrons from an atom \ [OH- ] = 2 ´ 10- 1
1 1 1
requires Now, pOH = - log [OH- ] = pK a + pKw - pKb
2 2 2
= IP1 + IP2 = 178 + 348 = 526 kcal = - log [2 ´ 10- 1] 1 1
= ´ 4.92 + 7 - ´ 4.81
59. F is most electronegative and smaller = 0.3010 2 2
in size hence, it has strongest Q pH + pOH = 14 = 7.05
H-bonding in F ¾H .....F. \ pH = 14 - 0.3010
= 13.699 » 13.70

MODULE 2
www.jeebooks.in
200 NEET Test Drive

68. Optical isomerism is shown by complex: Electronic configuration of Cr 2 + is 3d 4 . 82. We know that, DH ° = DE ° + Dng RT
cis-[Co(en) 2 Cl 2 ] + (III) 3d 4
4s 0
4p 0
For the reaction,
and [Co(en) 3 ] 3 + (IV) 2+ CH3
Cr = C == CH2 (g) + 6O 2 (g) ¾®
+ 3+
en en Four unpaired electrons CH3
Cl 4CO 2 (g) + 4H2O (l )
Co and Co en Electronic configuration of Mn2 + is 3d 5 . Dng = 4 - 7 = - 3
Cl 3d5 4s0 4p0 \ DH ° = DE ° - Dn g RT
en en 2+
Mn = [QDng = negative]
(III) (IV)
Five unpaired electrons i.e. DH ° < DE °
69. Greater the electronegativity difference
83. Molecular formula of asbestos is
of M¾H bond, more is acidic nature or Electronic configuration of Fe2 + is 3d 6 . Ca × Mg 3O(Si 4O11) in which silicate is
a strong acid has weak conjugate base
3d6 4s0 4p0 present as (Si 4O11) 6 - . Two simple chains
and vice-versa.
are joined together through the third
70. In transition series, oxidation state first Fe2+=
oxygen atom of SiO 4 tetrahedral. This is
increases from Sc to Mn due to Four unpaired electrons an example of double chain silicates.
increase in number of unpaired
Number of unpaired electrons is DH fus
electrons and then decreases because 84. We know that, DS fus =
pairing takes place. minimum in [Ni(H2O) 6 ] 2+, thus it show Tm
minimum paramagnetic behaviour.
71. Cl 2O 6 (dimer of ClO 3 ) is a mixed 6000 J mol -1
77. PCl 3 + 3H2O ¾® H3PO 3 =
anhydride of HClO 3 and HClO 4 . Phosphorus Orthophosphorus
273 K
Cl 2O 6 + H2O ¾® HClO 3 + HClO 4 trichloride acid
= 21.98 JK -1 mol -1
+ 3HCl
72. Percentage of 85. For endothermic process, (heat is
1.4 ´ Milliequivalent of H2 SO 4 78. According to Arrhenius equation, we absorbed), DS > 0.
- Ea
used to neutralise NH3 know that k = Ae RT For a cyclic process, entropy change is
N=
Weight of compound zero.
Taking log on both sides,
Millimoles of H2 SO 4 = 20 E 86. l = 0 to n - 1
In k = ln A - a
Milliequivalent of H2 SO 4 = 20 ´ 2 = 40 RT 1 1
m = - l to + l; s = + ,-
1.4 ´ 40 On compairing with straight line 2 2
Percentage of N = = 20%
2 .8 equation, y = mx + c, 87. In the carbonyl group, carbon atom is in
73. CF3 COOOH (peroxytrifluoroacetic E a state of sp 2 -hybridisation one
Slope = - a
acid) is used to convert aniline to R sp 2 -hybrid orbital overlap with a
nitrobenzene. 79. Antidepressants such as phenelzine, unhybridised p-orbital of oxygen to form
NH2 NO2 equanil, iproniazid etc., acts on central C—O s-bond. The remaining two
nervous system and produce a feeling sp 2 -orbitals of carbon form s-bonds with
CF3COOOH of well being and confidence in the two alkyl groups. Here, all the groups lie
person of depressed mood. on the same plane.

Aniline Nitrobenzene Thus, antidepressants drug are used to 88. All of the given statements are correct.
cure depression.
89. The p-type semiconductor have a larger
74. When conc. H2 SO 4 is heated with 80. Anomalous property of lithium is due hole concentration than electron
P2O 5 , the acid is converted to sulphur to smaller size and high charge concentration. p-type semiconductors
trioxide. The reaction is as follows: density. are created by doping an intrinsic
2H2 SO 4 + 2P2O 5 ¾® 2SO 3 + 4HPO 3 81. Molecular weight = Vapour density ´ 2 semiconductor with acceptor impurities.
Sulphur
trioxide The most common doping element for
= 11.2 ´ 2 Þ 22.4 p-type semiconductor are silicon and
75. Acidic flux is used to remove basic
Volume of 22.4 g of substance at NTP boron.
impurity. Here, SiO 2 is an acidic flux.
= 22.4 L 90. Methyl phenyl ether is obtained by the
76. The electronic configurations of Ni 2 + is 22.4
8 1 g of substance at NTP = L reaction of phenolate ions and methyl
3d .
22.4 iodide.
3d8 4s0 4p0
2+
11.2 g of substance at NTP = 11.2 L C6H5O - + CH3I ¾® C6H5OCH3 + I-
Ni = Methyl phenyl ether

Two unpaired electrons

MODULE 2
www.jeebooks.in

BIOLOGY
91. Belladonna is a narcotic, diuretic and 101. Darwin’s theory of natural selection did 110. Some bacteria produce non-motile
antispasmodic drug extracted from not encompass the role of genetic spores, such as conidia,
Atropa belladonna plant. Belladonna variation between individuals produced sporangiospores, etc. Under favourable
drops act as a muscarinic antagonist due to genomic mutation. environmental conditions, these spores
that block receptors in the muscles of 102. Haemophilia is an inherited disorder germinate and the vegetative cell comes
the eyes that constrict pupil size. Thus, where the blood fails to clot. It is due to out and grows.
they are used to dilate pupils. reduced levels of clotting factor. The 111. pH of venous blood is 7.35, which is lower
92. Being the primary holoenzyme involved family pedigree of queen Victoria than the pH of arterial blood. The
in replication activity, the DNA shows a number of haemophilic presence of free hydrogen ions increases
polymerase III holoenzyme also has descendants as she was a carrier of acidity, lowers pH of venous blood.
proofreading capabilities that correct the disease. 112. Operator — Switches on the cistron
replication mistakes by means of 103. Pedigree analysis is a tabular gene activity. It provides binding
exonuclease activity working 3 ¢ ® 5 ¢. representation of a family history by site for repressor protein
93. A transposable element or transposon taking a particular disease or character Promoter — Provides a site for binding
is a DNA sequence that can change its into consideration. gene activator proteins and RNA
position within a genome, sometimes polymerase
104. Haemophilia is a sex-linked recessive
altering the cells genetic identity. Regulator — Synthesise a molecule that
disease whose (recessive) allele is
94. Hypothyroidism leads to cretinism, which gene blocks a gene adjacent to
present on X-chromosomes. Females
causes coarse dry skin, Gull’s disease structural genes
are homogametic, i.e. have
and demand for increased energy Structural — Makes enzymes that control
XX-chromosomes, whereas males are
specially by muscles during exercise. gene metabolism, i.e. DNA
heterogametic i.e XY.
95. Penicillium undergoes sexual sequence, which initiates
Males only require one recessive allele transcription process to
reproduction by the formation of for expressing haemophilic trait. produce a particular protein
cleistothecia. It is a spherical-shaped However, females need two copies of
fruiting structure, which is formed as a recessive alleles for the diseased 113. Charging of the tRNA is also known as
result of sexual reproduction, which condition, which is very rare. aminoacylation.
also helps fungus for perennation in 114. The colour of algal thallus is due to the
105. Rhesus (Rh) factor is an inherited
adverse climatic conditions. presence of definite pigments.
protein found on the surface of RBCs of
96. Epidemic typhus (typhus fever) is Rhesus monkey. It is called Rh antigen Myxoxanthin is found only in
caused by Rickettsia prowazekii and is or Rh factor. The formation of which is Cyanophyta. Antheraxanthin is found in
transmitted to humans by head and controlled by dominant gene ‘R’. Euglenophyta. Peridinin is found only in
body lice of genus Pediculus. People with genotype RR/Rr are Rh Pyrrophyta. Fucoxanthin is a
97. Retina has cones for coloured vision. + ve (positive) person and those characteristic pigment of Phaeophyta.
The three principal cones are red, blue whose blood is devoid of it are Rh - ve 115. Chorionic gonadotropin is secreted by
and green. The green and red cones (negative) people. placenta and is responsible for the
are concentrated in fovea centralis and maintenance of corpus luteum. Its
106. Adrenocorticotropin Hormone (ACTH)
blue cones are mostly found outside presence in urine indicates pregnancy.
is secreted by the pituitary gland. This
the fovea.
hormone stimulates the adrenal cortex 116. Human beings have isolecithal eggs, in
98. The statement (d) does not come (in response to biological stress) to which there is sparse amount of yolk,
under the advantage of GM crops. The produce secrete increased amount of which is fairly uniformly distributed. Such
criticism against GM crops includes cortisol. eggs display holoblastic cleavage or
that they cause unintended harm to complete cleavage.
other organisms. 107. Monascus purpureus is a species of
mold belonging to division 117. Rigidity that develops in muscles after
99. JBS Haldane proposed chemical theory
Ascomycetes. It is commercially death is known as rigor mortis. It first
of evolution. He stated that the early
important as it produces number of develops in the lower jaw.
atmosphere of the Earth was reducing
statins, which are used to lower blood 118. Endarch is protoxylem lying to the inner
UV rays, lightening, reacted with
cholesterol. side of the metaxylem.
gaseous atmosphere to form organic
molecules, which later gave rise to amino 108. A Barr body is the inactive Mesarch is protoxylem lying in the
acids, proteins, carbohydrates, etc. X-chromosome in a female somatic middle of metaxylem.
100. Calcitonin hormone is secreted by the cell, it is inactivated in a process Exarch is protoxylem lying outside the
C-cells in thyroid gland. It decreases known as lyonisation. Further metaxylem.
blood calcium level and is investigations led to the conclusion that
the ‘X-body’ of Henking was a 119. Cervical vertebrae are seven in number
proteinaceous in nature. and present in weak first cervical
Insulin is secreted by the exocrine chromosome and was named
‘X-chromosome. vertebra is called atlas.
portion of the pancreas. Motilin is
Thoracic are present in the chest and
secreted from mucosa of the small 109. Atrophy or destruction of anterior lobe
are 12 in numbers.
intestine and ghrelin is secreted from of pituitary gland causes atrophy of
stomach and stimulates the secretion gonads, thyroid and adrenal cortex Lumbar are five in number and present
of human Growth Hormone (hGH) from called Simmond’s disease. in the abdomen.
pituitary gland.

MODULE 2
www.jeebooks.in
202 NEET Test Drive

Sacral have five and are fused in the monosaccharide units to form a organisms. Climacteric fruits show
adult forming one structure called polysaccharide. These bonds are increased respiratory activity at the time
sacrum. formed with elimination of a water of ripening.
120. High copy number is to provide more molecule (dehydration reactions). 141. In commensalism, none of the
replicates of inserted DNA sequences 130. Plants do not have any specific glands associating species are harmed. One
in the cell. It refers to the average or or tissues where hormone is species can derive benefits from other
reported number of copies per host synthesised exclusively. A plant species, which in turn remains
cell. Plasmids have either low, medium hormone is synthesised in one part of a unaffected.
or high copy number. plant and translocated to another part e.g., Remoras obtain locomotion and
121. Wet deposition is the acidic water in very low concentrations it causes food from sharks.
received through rain, fog, snow, dew, physiological response.
142. All the others are barriers methods of
mist, etc. Dry deposition is deposition 131. DNA replication does not take place contraception except for ‘Saheli’, which
of wind blown acidic gases and during the M-phase. It takes place is an oral contraceptive pill.
particles on the ground. during the S-phase of the cell cycle.
143. Avidin is a protein present in raw egg
122. Lactose consists of one galactose and 132. Zoochlorella is an example of endozoic white, which prevents the absorption of
one glucose molecules attached by 1-4 algae. It is found inside the body of biotin. Thus, biotin deficiency may be
glycosidic linkage in beta (b) aquatic animals. caused by eating the raw egg in large
orientation. Maltose is a disaccharide Zoochlorella is found inside Hydra quantities.
with 2 glucose molecules. Glycogen viridis. 144. The breeding activities can use only
and cellulose are polymers of glucose. 133. Obligate parasites – Grow only as those genes, which are present in such
123. Quaternary structure is the parasites upon suitable living host species that can be hybridised with
three-dimensional structure consisting tissues. them. It changes all those characters for
of the aggregation of two or more Destructive parasites – Kill the host which the parents used in hybridisation
individual polypeptide chains cells or tissues. differ from each other.
(subunits) that operate as a single Balanced parasites – Depend upon the 145. Rough surface of pollen grains helps
functional unit (multimer). host for the supply of the needs of them to cling to the body of insect
124. Different steps involved in the process parasite. pollinators. Option (b), (c) and (d) are
of decomposition are Facultative saprophytes – Normally live incorrect because
Fragmentation ® Leaching ® as parasites, but become saprophytes Not all insect pollinated plants produce
Catabolism ® Humification ® under emergent conditions. sugary fruit.
Mineralisation 134. Kangaroo rat is a desert rodent, which Nectar guides are also present in only
125. K is the carrying capacity for a given never drinks water and Mandrillus is some of insect pollinated flowers.
population. the African dog-like monkey. Pollen grains in abundance are
(K - N/K) depicts the fraction of 135. Blood fluke (Schistosoma) does not produced by wind pollinated plant/
carrying capacity available for further have redia and metacercaria stages of flowers.
growth of a population. When N = K , larva. 146. Nitrate reductase is a flavoprotein and
population growth stops because then requires reduced co-enzyme NADH for
K -N 136. Absorption is the process, by which the
K - N = 0 and = 0, end products of digestion pass through its activity. This co-enzyme contacts with
K the intestinal mucosa to the blood or nitrate by FAD.
Hence, option (c) is correct. lymph. Glycerol, fatty acids and 147. Microinjection method is used when
126. National Museum of Natural History, monoglycerides are absorbed via the there are only a very few cells available,
Paris contains approximately 68 million lymph. such as fertilised animal egg cells. In this
specimens of plants. It is considered as 137. Negative soil pollution – Reduction in method, a cell is held on a glass pipette,
one of the largest herbarium in the world. soil productivity due to erosion and under a microscope and the foreign
overuse. DNA is injected directly into the nucleus
127. Human blood has its own buffer system
using an incredibly fine micropipette.
of bicarbonate (HCO -3 ) and carbonic Positive soil pollution – Reduction in
acid (H2 CO 3 ). This system maintains soil productivity due to addition of 148. The oxidative decarboxylation of
the blood pH in the range of 7.35 and undesirable substances, e.g. industrial pyruvate into acetyl Co-A involves the
7.45. waste. presence of at least five essential
Landscape pollution – Congregating cofactors and a complex enzyme.
128. Shotgun is used to find a particular
gene in a whole genome. fertile land into barren area by 149. A reducing sugar is any sugar that is
dumping wastes. capable of acting as a reducing agent
Antisense genes are used to turnoff the
138. In Asom, muga silk is obtained from the because it has a free aldehyde group or
expression of a gene in a cell.
semi-domesticated species of silk a free ketone group. All
Gene synthesis is to synthesise gene.
worm, A. assamensis. monosaccharides are reducing sugars,
Electrophoresis method is used to
e.g. glucose, maltose, fructose, etc.
separate fragments of DNA on agarose 139. F1-particle of the oxysome is involved in
gel. 150. Kupffer cells are phagocytic in nature,
ATP synthesis. This synthesis is
O while Glisson’s capsules are membrane
possible due to proton motive force.
having dense connective tissue. Both
129. Glycosidic bonds C are present 140. Climacteric is a term referred to critical are present in liver.
in carbohydrates. It links adjacent phase or period of change in

MODULE 2
www.jeebooks.in
Prep Catalysis for NEET ~ Mock Test 5 203

151. The transpiration pull overcomes 160. Loose smut of wheat is caused by 171. Arrhenotoky and thelytoky are forms of
gravitation pull. It also overcomes Ustilago tritici. natural parthenogenesis. In Arrhenotoky,
resistance of narrow xylem channels Yellow rust of wheat is caused by as in honeybee, the male develops from
and resistance offered by water Puccinia striiformis. Rest pairs are unfertilised egg and the female from
coming out of narrow capillary pores. correctly matched. fertilised egg. In thelytoky, the diploid
152. Due to the presence of a barrier, unfertilised egg parthenogenetically
161. A unique combination of four
autogamy is not possible in flowers like develops into female.
characteristics, i.e. notochord, post
Gloriosa. This is techanically known as anal tail, pharyngeal slits and dorsal 172. The causes of biodiversity losses include
herkogamy. It is also reported in tubular nerve cord remain present at habitat fragmentation, overexploitation
Caryophyllaceae, Orchidaceae, etc. It some stages during development of and intensive agriculture.
is one of the contrivances for chordates. Habitat fragmentation leads to decrease
cross-pollination in plants. 162. Georges Cuvier is considered to be the in core area. Overexploitation leads to
153. The cortex zone lies between epidermis father of modern palaeontology. extinction of species.
and pericycle. It is predominantly made Arrhenius was one of the founders of Intensive agriculture also leads to loss of
of parenchymatous tissues. physical chemistry. biodiversity.
Cortex serves as a protective zone and 163. Hyaluronidase is an important enzyme 173. Waksman and Woodruff well responsible
also helps in absorption and produced by human sperm involved in for isolation of actinomycin in 1941 and
translocation of water in plants. the catabolism of hyaluronic acid. This streptothricin in 1942.
154. Syphilis is a sexually transmitted enzyme helps to disperse the cells of 174. The narrow cylindrical structure running
disease, commonly called as French corona radiata of the oocyte. through the centre of the
pox, caused by Treponema pallidum. 164. Amino acid tryptophan has only one plasmodesmata, which are in continuity
The disease is confirmed by Waksman codon UGG. Rest of the amino acids with the Smooth Endoplasmic Reticulum
test. Hutchinson’s teeth is a sign of given in the choices are coded by (SER) membranes of the connected cell
congenital syphilis. more than one codons. are called as desmotubule.
155. Gynoecium in Solanaceae family are 165. N-formylmethionine (fMet) is 175. Anthocyanin has a protective role in
characterised as bicarpellary, specifically important for initiation of plants against many abiotic stressors.
syncarpous, ovary superior, bilocular, translation in bacteria and other They belong to parent class of molecules,
axile placentation, swollen placentae, prokaryotes. called flavonoids and are synthesised via
multiple ovules in each locule. 166. In females, clitoris is a highly sensitive the phenylpropanoid pathway. They occur
156. Fragmentation – Thallus breaks down organ and like the penis has in all tissues of higher plants, including
specialised nerve endings. leaves, stems, roots, flowers and fruits.
into many fragments.
Fission – Simple cell division. It is homologous to glans penis in 176. Incomplete combustion of charcoal/coal
Budding – Outgrowth proliferates from males, but has no reproductive produces carbon monoxide that has
thallus. significance. greater affinity for haemoglobin than
Gemmae – Arise as modified branches. oxygen. Thus, the person dies of
167. The parasites harm the host by
asphyxiation.
157. Rhodospirillum is an example of purple reducing the survival, growth and
or green bacteria. They are Gram reproductive ability of the host. It 177. Leaching does not come under the
reduces the host population density. It factors affecting primary productivity. It
negative and generally occur in
might also render the host more is the process, by which water soluble
anaerobic sediments of lakes and
vulnerable to predation by making it inorganic nutrients go down into the soil
ponds.
physically weak. horizon and get precipitated as
158. Stratified squamous epithelium unavailable salts.
protects skin from chemical abrasions 168. Arginine decomposes into ornithine
and urea in the presence of enzyme 178. Hypersensitivity is also known as allergy
and stress. Keratinised dead layer of
arginase in the ornithine cycle. and hay fever is an allergic reaction to
skin is made up of stratified squamous
airborne allergens.
epithelium. 169. French fries contain starch, fats and oil.
Enzymes and hormones involved in 179. One of the reasons for polyembryony in
159. Phycomycetes belong to kingdom
carbohydrate and lipids digestion will angiosperms is due to development of
Fungi and characterised by aseptate,
be most active during digestion. many embryos from other cells of
coenocytic hyphae and production of
Hence, option (a) is correct. embryo sac except egg.
asexual spores endogenously in
sporangia as zoospores or 170. PAN, i.e. Peroxyacetyl Nitrate is an 180. Phycobilins are unique among the
aplanospores. These reproduce atmospheric oxidant. photosynthetic pigment in that they are
sexually and show hetero or isogamous bonded with certain water soluble
It damages chloroplast so, reduces
sexual reproduction. It is divided into proteins such as phycobiliproteins.
photosynthetic efficiency and growth of
Oomycetes and Zygomycetes. These are found in cyanobacteria, in
plants.
chloroplast of red algae, etc.

SCORE CHART
No. of Correct Answers : A .................. No. of Incorrect Answers : B ..................
Total Marks : x = (A × 4) – (B × 1)
Scores and Expected Rank : If the score lies above 665, then rank will be in between 1-50. For other scores, rank
estimations are given below Score Rank Score Rank
664 – 642 51 – 150 635 – 630 251 – 400
641 – 636 151 – 250 629 – 625 401 – 500
MODULE 2
www.jeebooks.in

PREP CATALYSIS
for NEET
Full Length Mock Tests for NEET to Make You Ready to Face the Challenge

MOCK TEST 6 (With Solutions)

Duration : 3 Hours Max. Mark : 720

General Instructions
The test is of 3 hours duration and Test Booklet contains 180 questions. Each question carries 4 marks. For
each correct response, the candidate will get 4 marks. For each incorrect response, one mark will be deducted
from the total scores. The maximum marks are 720.
The question paper contains three parts of Physics, Chemistry and Biology respectively.
Part A contains 45 Questions from Physics section. Part B contains 45 Questions from Chemistry section.
Part C contains 90 Questions from Biology.

PART A PHYSICS
1. Loop A of radius r << R moves towards loop B with a 2. The magnitude of acceleration of centre of mass of
constant velocity v in such a way that their planes two blocks as shown in the figure is
are always parallel. The distance between the two (neglect friction and g = 10 ms −2 )
loops x, when the induced emf in loop A is maximum
B

m
r I R
m

v 65° 25°

A x
(a) 5 ms −2 (b) 5 2 ms −2
R R
(a) (b) (c) 2R (d) 2R (c) zero (d) 1.25 ms −2
2 2

MODULE 2
www.jeebooks.in
Prep Catalysis for NEET ~ Mock Test 6 205

3. In the cyclic process shown in the p-V digram, then 8. An alternating current, I = I 0 sinω t is flowing in
magnitude of the work done is AC circuit. The ratio of rms current in the interval
V of 0 to T and average current in the circuit for the
T T
2V0 time interval to ( where, T is time period) is
8 4
π π π
(a) (b) 2 π (c) (d)
V0 2 4 2

9. A vibration magnetometer placed in a magnetic


p meridian has a small bar magnet. The magnet
p0 3 p0
executes oscillations with time period 3 s in earth’s
πp0 V0 horizontal magnetic field of 32 µT. When, a
(a) πp0 V0 (b) −
2 horizontal field of 16µT is produced opposite to the
πp0 V0
(c) π 0 V02 (d) earth’s field, then the new time period of magnet
2 will be
3
4. A ball of mass 2 kg moving with a speed of 100ms−1 (a) s (b) 3 2 s
strikes a wall at an angle of 30°. The ball rebounds 2
at the same speed and same angle and remains in (c) 6 s (d) 1.5 s
contact with the wall for 0.4 s, then the force 10. If the pitch of the sound of a source appears to
exerted by the ball on the wall is drop by 10% to a moving person, then the velocity
A of motion of the person is
(take, velocity of sound = 330 ms−1)
30°
(a) 30 ms −1 (b) 11 ms −1 (c) 35 ms −1 (d) 33 ms −1
O
30°
11. From the top of a tower a ball is dropped. If the
total distance covered by it in last second of its
B motion is equal to the distance covered by it in its
first 5s, then the time for which it remains in air is
(a) 1500 N (b) 500 N (c) 1500 2 N (d) 700 N
(a) 5 s (b) 10 s (c) 13 s (d) 9 s
5. In the following arrangement as shown in figure, 12. An ideal gas contained in a cylinder is given 210 J
the magnetic field at O is given by of heat. The gas does 80 J of work in the expansion
Y resulting from heating, then the increase in
θ2 internal energy in the process is
i (a) 210 J (b) 140 J (c) 130 J (d) 30 J

φ2 13. The ratio of radii of gyration of a circular disc and


P
a
O a circular ring of the same radii and same mass
φ1 about a tangential axis to the plane is
(a) 1 : 2 (b) 5 : 6 (c) 2 : 3 (d) 2 : 1
θ1
14. The diameter of a cylinder is measured using a
X Vernier callipers with no zero error. It is found that,
µ0 i µ0 i the zero of the Vernier scale lies between 6.10 cm
(a) . (cos θ1 + cos θ2 ) (b) . (sin φ1 + sin φ2 ) and 5.95 cm of the main scale and there is no other
4π a 4π a
main scale division in between there two. The
(c) Both (a) and (b) (d) None of these
Vernier scale has 50 division equivalent to 1.75 cm.
6. Two lenses of power +10 D and −2D are combined The 25th division of the Vernier scale exactly
together, then focal length of combination is coincides with one of the main scale division, the
(a) −20 cm (b) −12.5 cm (c) 12.5 cm (d) 15 cm diameter of the cylinder is
(a) 8.825 cm (b) 17.106 cm
7. A wheel is subjected to uniform angular (c) 11.95 cm (d) 13.70 cm
acceleration about its axis. Initially, its angular
velocity is zero. In the first 3 s, it rotates through 15. The distances of two planets from the sun are 1013 m
an angle θ1 and in next 3 s it rotates through θ 2, and 1012m, respectively. The ratio of speeds of two
θ
then 2 is planets around sun is
θ1 1
(a) 10 (b) 10 (c) (d) 1025
(a) 2 (b) 4 (c) 3 (d) 5 10
MODULE 2
www.jeebooks.in
206 NEET Test Drive

16. The incident intensity on a horizontal surface at of disc. A small point mass m detaches from the
1 kW rim of disc and continue to move with same
sea level from sun is about . If 50% of this
m2 angular velocity. The angular velocity of remaining
intensity is reflected and 50% is absorbed, then disc just after detaching will be
find the ratio of radiation pressure to atmospheric  M − 2 m  M + 2 m
(a)  ω (b)  ω
pressure p0(1 × 105 Pa ) at sea level  M + m  M+ m
(a) 5 × 10−11 (b) 5 × 10−6  M − 2 m  M + 2 m
(c)  ω (d)  ω
(c) 1 × 10−11 (d) 7 × 10−7  M−m  M−m
17. There are 20 cells connected in series, each having
24. What will be the height of the image when an
emf E and internal resistance r. Some of cells are
object of 4 mm is placed at a distance 30 cm in
connected wrongly, A battery of emf 3E and internal
front of the axis of a convex mirror of radius of
resistance 5r is connected with above arrangement,
curvature 60 cm?
then an ammeter reads 5A, when both add each
(a) 5 mm (b) 4 mm
other and 3A, when both oppose each other. Then,
(c) 3 mm (d) 2 mm
number of cells wrongly connected is
(a) 2 (b) 3 25. Two soap bubbles combine to form a single bubble,
(c) 5 (d) 4 then what is the ratio of surface energy of bigger
drop to smaller one? If both have equal radius.
18. The colour seen in the reflected white light from a 1 2
thin oil film are due to (a) 2 2 : 1 (b) 2 3 : 1
(a) diffraction (b) interference 3
(c) dispersion (d) polarisation (c) 1 : 1 (d) 2 2 : 1

19. A motor car running at the rate of 7ms−1 can be 26. In a Fresnel biprism experiment, the two positions
stopped by applying breaks in 10m, If w is the of lens give separation between the slits as 32 cm
weight of the car, then the total resistance to the and 2 cm respectively. Then, the actual distance of
motion, when breaks are on is separation is
w w
(a) (b) w (c) (d) 2w (a) 4 cm (b) 16 cm
2 4 (c) 24 cm (d) 8 cm
20. Two radioactive materials X and Y have 3000 and 27. A battery of emf E and internal resistance r is first
6000 number of nuclei respectively of initially. If connected to an external resistance r1 and then
decay constant of material X and Y are 12 min −1 connected to another external resistance r2 for the
and 10 min −1, then the time in which both same time. The value of r for which heats
elements have same number of nuclei is dissipated in r1 and r2 will be same
(a) loge  
1 1 r1 r2
(b) loge 2
 2  2 (a) (b) r1r2 (c) r1r2 (d)
r2 r1
(d) loge  
1 1
(c) loge 2
2  2  28. A metal wire 1 cm in diameter breaks, if the
21. Assuming earth to be a sphere of radius R, a tension in it exceeds 500N. The maximum tension
plateform is arranged at a height of R from earth that may be given to similar rope of diameter 4 cm
surface. Escape velocity is kv from this plateform, is
where v is escape velocity from earth’s surface, then (a) 500 N (b) 800 N
k have value of (c) 1000 N (d) 8000 N
1 1
(a) (b) 29. If the energy of a photon corresponding to a
2 2
1
wavelength of 6000Å is 3.32 × 10−19 J, then photon
(c) 2 (d) energy corresponding to wavelength 4000Å will be
3
(a) 5 × 10−19 J (b) 3.5 × 10−18 J
22. A photodetector used to detect the wavelength of (c) 5.5 × 10−20 J (d) 4.5 × 10−17 J
1700 nm, has energy gap of about
(a) 0.82 eV (b) 0.73 eV
30. The total charge induced in a conducting loop,
(c) 1.13 eV (d) 0.92 eV
when it is moved in magnetic field depends on
(a) the rate of change of magnetic flux
23. A circular disc of mass M and radius R is rotating (b) initial magnetic flux only
with an angular velocity ω about an axis passing (c) the total change in magnetic flux
through its centre and perpendicular to the plane (d) final magnetic flux only

MODULE 2
www.jeebooks.in
Prep Catalysis for NEET ~ Mock Test 6 207

31. In the following combinations of logic gates, the 38. Three electric dipoles are enclosed with in
output of A, B and C are respectively Gaussian spherical surface of radius 2 cm. If
1 electric dipole moment of each dipole is 20 µC, then
(i) total electric flux passing through the surface is
1 A
0 10 × 10−8 5 × 10−8 20 × 10−8
(a) (b) 0 (c) (d)
0 ε0 ε0 ε0
(ii) B
1 39. A satellite orbits the earth at a height of 400 km
above the surface. How much energy must be
(iii) 0 C expended to rocket the satellite out of the
0 gravitational influence of earth?
Mass of satellite = 200 kg, mass of earth
Codes = 6 × 1024 kg, radius of earth = 6.4 × 106 m and
A B C A B C G = 6.67 × 10−11Nm 2 kg−2.
(a) 0 0 0 (b) 0 1 1 (a) 6 × 109 J (b) 4.5 × 109 J (c) 7.2 × 109 J (d) 2 × 1010 J
(c) 1 0 0 (d) 1 1 1 40. The diameter of an iron wire is 1.20 mm. If the
32. The ratio of intensities of two waves are given by speed of transverse wave in the wire be 50 ms−1 ,
16 : 4, then the ratio of amplitudes of the two then the tension in the wire be (density of iron
waves is = 7.7 × 103 kg m −3 )
(a) 2 : 1 (b) 1 : 2 (c) 16 : 1 (d) 1 : 4 (a) 20 N (b) 25.5 N (c) 18.23 N (d) 21.78 N
33. In a hydrogen atom, a transition takes place from 41. A uniform cylinder has a radius R and length L. If
third orbit to second orbit, then wavelength of the moment of inertia of this cylinder, about an
emitted photon lies in [ given, R = 1.097 × 107 m −1] axis passing through its centre and normal to its
(a) infrared region (b) ultravoilet region circular face is equal to the moment of inertia of
(c) visible region (d) X rays the same cylinder about an axis passing through
its centre and perpendicular to its length, then
34. The molecules in a perfect gas has mean kinetic R 3
energy (at 27°C) of 7 × 10−4 erg and acceleration (a) L = R (b) L = 3R (c) L = (d) L = R
3 2
due to gravity 980 cm s−2. Then, the number of
molecules in 2 cc of this gas at 27° C at a pressure 42. The value of electric field inside the capacitor is 50
of 10 cm of Hg is V/m. If a dielectric material of dielectric constant
. × 10
(a) 428 7
(b) 3.52 × 10 8 6.5 is filled between the plates of capacitor, then
. × 108
(c) 475 . × 108
(d) 571 the magnitude of polarisation is
(a) 300 (b) 375 (c) 275 (d) 325
35. A body of mass 4m is lying in x-y plane at rest. It
43. The equivalent resistance between A and B in the
suddenly explodes into three pieces. Two pieces
given circuit is
each of mass m moves perpendicular to each other
with equal speeds v,the total kinetic energy 2Ω
generated due to explosion is
3 2Ω 2Ω
(a) 2 mv 2 (b) 4 mv 2 (c) mv 2 (d) mv 2
2 A B
2Ω
36. Study the following statements and find which
is/are true.
I. The motion of pendulum with the effect of (a) 0.8 Ω (b) 1 Ω
dissipative force is known as damped (c) 5 Ω (d) None of these
oscillation.
44. The value of current in the following diagram will
II. Swinging of a child in a swing (without
be
external push) is the example of free
oscillation. 5V 200 Ω 3V
Choose correct option (a) 10 mA (b) 1 mA (c) 0.1 mA (d) zero
(a) I only (b) II only
(c) I and II (d) All are wrong 45. When a proton is accelerated through 2V, then its
kinetic energy will be
37. A constant volume gas thermometer works on
(a) 4.8 × 10−19 J (b) 3.2 × 10−19 J
(a) Boyle’s law (b) Charles’ law −19
(c) Pascal’s law (d) Archimedes’ principle (c) 6.4 × 10 J . × 10−19 J
(d) 16

MODULE 2
www.jeebooks.in

PART B CHEMISTRY
46. Which of the following is the strongest nucleophile? 53. Among the following compounds which will not be
(a) CH3 ¾CH-2 (b) HC ºº C - hydrolysed under normal condition?
-
(c) NH2 (d) H2C == CH- (a) GeCl 4
(b) CCl 4
47. The formation of the oxide ion O 2 - ( g ) requires first
(c) SiCl 4
an exothermic and then endothermic step as shown (d) SnCl 4
below.
O( g) + e- ¾® O - ( g); DH = - 142 kJ mol - 1 54. Which of the following is incorrect statement
regarding group 16 elements?
O -(g) + e - ¾® O 2 - ( g); DH = + 844 kJ mol - 1 (a) Oxygen is most abundant element in the earth’s crust
This is because followed by sulphur in group 16
(a) oxygen is more electronegative (b) The transition temperature of rhombic and monoclinic
(b) oxygen has high electron affinity sulphur is 369ºC
(c) O - will tend to resist the addition of another electron (c) Sulphur shows tendency to catenation while oxygen shows
(d) O- ion has comparatively large size than oxygen atom this tendency to a limited extent
(d) In OF2 , oxidation state of oxygen is + 2
48. In the electrometallurgy of aluminium, graphite
rod acts as 55. The bond length of HCl is 3.40 ´ 10- 10 m.
(a) anode (b) cathode The percentage ionic character of HCl, if measured
(c) oxidising agent (d) reducing agent dipole moment is 6.28 ´ 10- 30 C-m, is
49. Physisorption is reversible in nature. The extent of (a) 12% (b) 8%
adsorption increases with the increase of surface (c) 25% (d) 50%
area of the adsorbent. In physisorption, the
56. The potential energy of an electron in the first
molecules of gases adsorbed are attached to
Bohr orbit in the He+ ion is
adsorbent by
(a) - 13.6 eV (b) - 27.2 eV
(a) chemical force (b) electrostatic force
(c) - 54.4 eV (d) - 108.8 eV
(c) van der Waals’ force (d) None of these
57. When acidified K 2 Cr2 O 7 solution is added to Sn 2 +
50. Among the group 14 elements the one having the
lowest first ionisation energy is salt then Sn 2 + changes to
(a) Pb (b) Sn (c) Si (d) Ge (a) Sn+ (b) Sn3 +
(c) Sn (d) Sn4 +
51. IUPAC name of
CH3 H3C 58. Which of the following dissolves in water to
and
produce hydroxides, hydrogen peroxide and
oxygen?
CH3 H3C
(a) Na (b) Na 2O
respectively are (c) Na 2O 2 (d) NaO 2
(a) 1, 2-dimethyl cyclobutane and 2, 3-dimethyl butene
59. What is the temperature at which the kinetic
(b) 1, 2-dimethyl cyclobut 1-ene and 3, 4-dimethyl
energy of 0.3 mole of helium is equal to the kinetic
cyclo-but-1-ene energy of 0.4 mole of argon at 400 K?
(c) 2, 3-dimethyl cyclobutene and 2, 3-dimethyl cyclobutane (a) 400 K
(d) 2, 3-dimethyl butene and 1, 2-dimethyl cyclobut-1-ene (b) 873 K
52. Cl (c) 533 K
COOH (d) 300 K

(A) 60. Which of the following statements is incorrect for


IUPAC name of A is n-butyl lithium?
(a) 3-methyl 2-chloro butanoic acid (a) It is hexameric in the solid
(b) 2-methyl 3-chloro butanoic acid (b) Used as polymerisation catalyst
(c) 3-chloro 2-methyl butanoic acid (c) Used for alkylation
(d) 2-chloro 3-carboxy, 2, 3-dimethyl propane (d) Used for synthesis of vinyl and allyl lithium

MODULE 2
www.jeebooks.in
Prep Catalysis for NEET ~ Mock Test 6 209

61. Match Column I with Column II and identify 67. Hydrocarbon on reaction with steam produces
correct answer by using the codes given below: hydrogen gas. Chemical reaction occuring in this
process is given by
Column I Column II
H 2 O( g) + CH 4 ( g) CO( g) + 3H 2 ( g)
-
(Compounds) (Oxidation state)
-4
A. [Fe(CN)6 ]-3 1. +5
If K C = 3.0 ´ 10 at 1000 K, then the K p would be
(a) 202 (b) 2.02
B. Na3 [O5 Cl5 N]] 2. +1 (c) 20.2 (d) 0.6740
C. [HMn(CO)5 ] 3. +3
68. Gammexane is chemically known as
D. Fe 4 [Fe(CN)6 ] 4. +2 (a) benzene hexabromide
(b) benzene hexachloride
Codes
(c) hexabromo benzene
A B C D A B C D
(d) hexachloro benzene
(a) 4 3 1 2 (b) 2 4 1 3
(c) 3 1 2 4 (d) 1 2 3 4 69. What volume of 0.8 M solution contains 0.1 m mole
of solute?
62. A gas can be liquefied most suitably at a (a) 100 mL (b) 125 mL
temperature T K and pressure p. It provided that (c) 500 mL (d) 0.125 mL
(a) T = Tc and p < pc (b) T < Tc and p = pc
(c) T < Tc and p > pc (d) T > Tc and p > pc 70. Secondary structure of proteins are classified into
two different types a-helix and b-pleated sheet.
63. Consider the values of Eº for the following Out of these two, the a-helix structure is stabilised
reactions : by
º
EFe 3+
/ Fe 2 +
= + 0.77; EI° - = + 0.54 V (a) dipole-dipole interactions (b) peptide bonds
2/I
º ° (c) van der Waals’ forces (d) hydrogen bonds
ECu 2+
/ Cu
= + 0.34; EAg +
/Ag
= 0.80 V
71. The maximum work done in expanding 16 g of
The pair between which redox reaction is not oxygen at 300 K and occupying a volume of 5 dm 3
feasible is isothermally until the volume becomes 25 dm 3 is
(a) Fe 3 + and I- (b) Ag + and Cu (a) 2.01 ´ 103 J (b) 4.03 ´ 104 J
(c) Fe 3 + and Cu (d) Ag and Fe 3 + (c) - 2.01 ´ 103 J (d) 2.67 ´ 104 J
64. Cl 2 ( g) + 3F2 ( g) - 2ClF3 ( g); DH r = - 329 kJ
72. Glucose on reaction with CH 3 OH in presence of dry
The above reaction represents an exothermic HCl gas forms a- and b-methyl glucosides because
process. Which of the following will increase it contains
quantity of ClF3 in an equilibrium mixture?
(a) five ¾OH groups (b) ¾CH2OH group
(a) Removing Cl 2
(c) a ring structure (d) an aldehydic group
(b) Adding F2 or Cl 2
(c) Increasing temperature
(d) Increasing the volume of container
73. For the chemical reaction, N 2O 4 ( g) - 2NO 2( g)
The observed molecular weight of N 2 O 4 is
65. The oxidation state of S-atoms in S 4 O 26 - from left to 60 g mol- 1 at 300 K. The percentage dissociation of
right respectively are N 2O4( g) at 300 K is
O O (a) 10% (b) 15%
½½ ½½ (c) 20% (d) 53%
È
O ¾ S ¾ S ¾ S ¾ S ¾ OÈ
74. The electrolytic reduction of nitrobenzene in
½½ ½½
strongly acidic medium produces
O O
(a) aniline (b) azobenzene
(a) + 3, + 1, + 1, + 3 (b) + 4, + 1, + 1, + 4
(c) + 5, 0, 0, + 5 (d) + 6, 0, 0, + 6 (c) azoxybenzene (d) p-aminophenol

66. Which of the following alkali-metal hydroxides is 75. Tertiary alkyl halides are partially inert to
the least soluble in water? substitution by S N 2 mechanism because of
(a) CsOH (b) KOH (a) insolubility (b) instability
(c) NaOH (d) LiOH (c) inductive effect (d) steric hindrance

MODULE 2
www.jeebooks.in
210 NEET Test Drive

76. Cycloketone on reacting with NH 2 OH gives B which 83. Which of the following trivalent ion has the largest
further reacting with H 2 SO 4 gives C. Find the atomic radii in the lanthanide series?
product C and choose the correct option. (a) Lu (b) Ce
O (c) Gd (d) La

NH2OH H2SO4 84. When conc. HCl is added to an aqueous solution of


B C CoCl 2, its colour changes from reddish pink to deep
blue. Which complex ion gives blue colour in this
(a) Caprolactam (b) Cyclohexanone oxime
reaction?
(c) Aniline (d) Anilide
(a) [CoCl 4 ]2 - (b) [CoCl 6 ]3 -
77. The reaction of propene with HOCl proceeds via (c) [CoCl 6 ]4 - (d) [Co(H2O)6 ]2 +
the addition of
(a) OH- in the first step (b) H+ in the first step 85. The amount of bromine required to convert 3 g of
(c) Cl + in the first step (d) Cl + and OH- in a single step phenol into 2, 4, 6-tribromophenol will be
(a) 15.32 g (b) 20.82 g
78. A plot of 1/T versus k for a reaction gives the slope (c) 10.22 g (d) 4.89 g
2 ´ 104. The energy of activation for the reaction is
(Given, R = 8. 314 JK - 1 mol- 1) 86. Which one of the following compound has the
smallest heat of hydrogenation per mole?
(a) 166 kJ mol - 1 (b) 400 kJ mol - 1
(a) Buta-1, 3-diene
(c) 67 kJ mol - 1 (d) 280 kJ mol - 1 (b) trans-but-2-ene
79. What kind of isomerism exists between (c) 1-butene
[Cr(H 2 O)6 ]Cl 3 (violet) and [Cr(H 2 O)5 Cl]Cl 2 × H 2 O (d) cis-but-2-ene
(greyish-green)? 87. Benzoin condensation is the chemical properties of
(a) Linkage isomerism (b) Solvate isomerism aromatic aldehydes only. The resultant product of
(c) Ionisation isomerism (d) Coordination isomerism this reaction is a-hydroxy ketones. Reagent used in
80. In AgBr crystal, the ion size lies in the order benzoin condensation is
Ag + < < Br - . The AgBr crystal should have the (a) KOH (b) KCN
(c) H2SO 4 (d) C 2H5ONa
following characteristics
(a) Perfect crystal 88. What will be the value of equilibrium constant for
(b) Schottky defect the reaction at 300 K?
(c) Frenkel defect AgCl( s) ¾® Ag+ ( aq ) + Cl- ( aq )
(d) Both (b) and (c) º
If EAgCl( s )/ Ag( s ), Cl -
= 0.22 V
81. For which type of reaction order and molecularity º
have the same value? EAg +
( aq)/ Ag( s )
= 0.80 V
(a) Elementary reaction (b) Termolecular reaction (a) 44.88 (b) 3.07 ´ 109
(c) Biomolecular reaction (d) First order reaction (c) 3.07 ´ 1019 (d) 3.42 ´ 1018
(CH3 )3 CCl Cl 2/ FeCl3 HBr
¾® ¾ ¾ ¾ ¾
82. Anisole ¾ ¾¾¾¾ ¾® ¾He
¾¾ ® ‘A’.
Anhy. AlCl3 at 89. Which of the following contains greatest number of
ammonia atoms?
The product ‘A’ in the above series of reaction is (a) 0.17 g of ammonia
OH Br (b) 34 mL of ammonia
Cl Br (c) 11.2 L of ammonia vapours at 1 atm and 273 K
(a) (b) (d) All have equal number of atoms
POCl3
90. + H—C—N(CH3)2 A; Product A will be
C(CH3)3 C(CH3)3
S
OCH3 Br O
Br Cl (a) (b)
(c) (d) S CH2Cl S CHO

(c) (d)
S Cl S C
C(CH3)3 C(CH3)3 CH3
O

MODULE 2
www.jeebooks.in

PART C BIOLOGY
91. Bundle sheath extensions are sclerenchymatous in 101. Match Column I with Column II and choose the
(a) isobilateral leaf (b) dorsiventral leaf correct option from the codes given below.
(c) dicot stem (d) monocot stem Column I Column II
92. An adult kidney weighs approximately A. Dithecous 1. One-lobed anther
(a) 350 g (b) 250 g (c) 50 g (d) 150 g
B. Monothecous 2. Two-lobed anther
93. Principal embryonic membranes are C. Inserted 3. Anther face is towards the
(a) umbilicus and chorion (b) amnion and chorion
periphery
(c) placenta and amnion (d) placenta and umbilicus
D. Exerted 4. Anther face is towards the
94. When both stipules of a leaf combine together and centre
form a tube-like structure, then it is known as
(a) interpetiolar (b) ochreate Codes
(c) tendrillar (d) scaly A B C D A B C D
95. Which among the following conditions is/are (a) 1 2 3 4 (b) 2 1 4 3
necessary for seed formation? (c) 1 2 4 3 (d) 4 3 1 2
(a) Production of two types of spores 102. Dextran is a complex polysaccharide used in blood
(b) Retention of megaspore within the megasporangium transfusions. Name the microbe responsible for its
(c) Development of receptive areas like stigma for catching production.
microspores (a) Cunninghamella blakesleeana
(d) All of the above (b) Leuconostoc mesenteroides
96. Archaebacteria differ from prokaryotes in (c) Propionibacterium shermanii
(a) the absence of membrane-bound organelles (d) Aspergillus flavus
(b) the genetic material that floats freely in the cytoplasm 103. Identify the incorrectly matched pair.
(c) having proteins, glycoprotein or non-cellulosic (a) Gymnosperms – Ovule is naked
polysaccharides in their cell wall
(b) Angiosperms – Endosperm is haploid
(d) occurrence and live at only moderate temperature
(c) Both (a) and (b)
97. Prophase-I of meiosis-I is the longest phase, (d) None of the above
requiring 90% of the total time of entire meiosis
process. Arrange phases of prophase-I in the 104. Which amino acids are found abundant in the
correct sequence. histone proteins?
(a) Lysine and methionine (b) Lysine and cysteine
I. Leptotene II. Pachytene
(c) Arginine and citrulline (d) Arginine and lysine
III. Zygotene IV. Diplotene
V. Diakinesis 105. With reference to photophosphorylation, which one
of the following statements is correct?
The correct sequence is
(a) Non-cyclic photophosphorylation involves only PS-I
(a) I, II, III, IV, V (b) I, III, IV, II, V
(b) Non-cyclic photophosphorylation involves only PS-II
(c) I, III, II, IV, V (d) I, IV, II, III, V
(c) Cyclic photophosphorylation involves only PS-II
98. Activity of nitrogenase enzyme depends on (d) Cyclic photophosphorylation involves only PS-I
(a) non-availability of nitric acid
106. The characteristic feature of atactostele is
(b) availability of nitric acid
(a) vascular strands scattered in vascular tissue
(c) non-availability of ATP
(b) a number of parallel vascular bundles
(d) availability of ATP
(c) hollow stele with the overlapping leaf gaps
99. Xylem vessels are arranged in Y-shaped manner in (d) scattered leaf gaps
(a) monocot stem (b) dicot stem
107. Which among the following does not come under
(c) monocot root (d) dicot root
the category of inclusions present in bacteria?
100. Which among the following is correct regarding the (a) Metachromatic granules
root? (b) Carboxysomes
(a) Negatively geotropic (b) Bear nodes and internodes (c) Gas vacuoles
(c) Positively phototropic (d) None of these (d) Plasmids

MODULE 2
www.jeebooks.in
212 NEET Test Drive

108. Which of the following amino acids have a 114. Which among the following is not an application of
non-polar, aliphatic R group? DNA fingerprinting?
(a) Glycine and proline (a) It is a highly reliable method for identification of individuals
(b) Alanine and valine involved in crime
(c) Isoleucine and methionine (b) It is useful in guiding breeding programmes for
(d) All of the above endangered animals
(c) It provides information about human lineage and
109. Refer to the given figure showing anterior view of relationship with great apes
larynx. Choose the correct option for the parts (d) Production of organisms which are genetically identical to
labelled as, A, B, C and D in the figure. their parents
A
115. Which of the following is not correctly paired?
(a) G1/S checkpoint– Cell monitors size and DNA integrity
B
(b) G 2 /M checkpoint–Cell monitors DNA synthesis and damage
Thyrohyoid
(c) M checkpoint– Cell monitors spindle formation and
membrane
Thyrohyoid attachment to kinetochores
ligament (d) All are correctly matched
C
116. The enzyme that catalyses the dissociation of
carbonic acid into bicarbonate and hydrogen ion in
Cricothyroid
ligament
blood is
(a) carbonic anhydrase (b) carbonic ligase
D (c) carbonic lyase (d) carboxypeptidase
Thyroid gland 117. Identify correct statements about thylakoids.
Trachea Cartilaginous rings
I. Thylakoids are absent in blue-green algae and
A B C D some bacteria which are photoautotrophic.
(a) Epiglottis Hyoid bone Thyroid cartilage Cricoid cartilage II. Thylakoids are present in blue-green algae
(b) Hyoid bone Epiglottis Cricoid cartilage Thyroid cartilage and some photoautotrophic bacteria.
(c) Cricoid cartilage Epiglottis Hyoid bone Thyroid cartilage III. In blue-green algae and some
(d) Thyroid cartilage Hyoid bone Epiglottis Cricoid cartilage photoautotrophic bacteria, thylakoids lie
110. Identify the incorrectly matched pair. freely in the cytoplasm.
(a) Dwarf yeast cells – Haploid IV. In blue-green algae and some
(b) Large yeast cell – Diploid photoautotrohpic bacteria, thylakoids are
(c) Fission yeast – Diploid phase is reduced organised into chloroplasts.
(d) Helobial yeast – Haploid phase is more prominent Choose the correct option.
111. The soil salination is caused due to (a) I and III (b) II and IV
(a) poor drainage (b) addition of fertilisers (c) I and IV (d) II and III
(c) closeness to sea areas (d) All of these 118. During cell divison, migration of the chromosomes
112. In which structure are the histones organised in a is made possible by binding of spindle fibres to
nucleosome? the………, which is an assembly of multilayered
plates of proteins associated with the centromere.
(a) Hexamer (b) Octamer
(a) tubulin (b) actin
(c) Dimer (d) Tetramer
(c) seperase (d) kinetochore
113. Phenotypic adaptations include
119. The minute scar in the seed called hilum is the
I. favourable changes in morphology and
remnant of
physiology.
(a) integument (b) funiculus
II. non-genetic changes
(c) raphe (d) antipodals
III. genetic variations
IV. occurs by natural selection 120. …… are the pioneer species in the primary
Choose the correct option. succession on rocks.
(a) I and II (b) I and III (a) Lichens (b) Phytoplanktons
(c) Free-floating angiosperms (d) Bryophytes
(c) I and IV (d) Only I

MODULE 2
www.jeebooks.in
Prep Catalysis for NEET ~ Mock Test 6 213

121. The hepatic portal vein carries the digested food to 128. Which of the following does not hold true for the
the liver to remove the excess of which of the function of bile?
following? (a) Emulsification of fat
(a) Glucose and galactose (b) Cholesterol and vitamins (b) Increases alkalinity
(c) Glucose and amino acids (d) Glucose and cholesterol (c) Elimination of some heavy metals
(d) Formation of fat soluble vitamins
122. Which of the following statements are correct with
respect to mitochonodrial matrix? 129. Net primary productivity can be obtained by
I. It possesses single circular DNA molecule. (a) GPP – R (b) R – GPP (c) GPP + R (d) None of these
II. It possesses few RNA molecules. 130. Parthenocarpy can artificially be induced by
III. It possesses ribosomes (70S). spraying hormones auxins and gibberellins. The
proper time for applying auxin and gibberellins,
IV. It possesses 80S ribosomes. respectively are
Choose the correct option. (a) before anthesis, after anthesis
(a) I and III are correct (b) II and IV are correct (b) at anthesis, after anthesis
(c) III and IV are correct (d) I, II and III are correct (c) after anthesis, at anthesis
(d) at anthesis for both
123. The demand for O2 is directly related to increasing
input of organic waste and is expressed as 131. What is the larval name of acorn or tongueworm?
(a) biochemical oxygen demand (a) Tadpole (b) Tornaria (c) Brachidaria (d) Wriggler
(b) chemical oxygen demand 132. Identify incorrect statement with reference to
(c) Dobson unit ornithine cycle/urea cycle.
(d) Both (a) and (b) (a) It was studied and described by Krebs and Hanseleit
124. Which of the following post-fertilisation changes is (b) The first step of urea formation in ornithine cycle involves
incorrectly matched? the formation of carbamoyl phosphate from the two
molecules of ammonia and CO 2
(a) Ovary — Forms seed
(c) All the enzymes involved in ornithine cycle are located
(b) Egg cell — Forms embryo
inside the mitochondria
(c) Nucellus — Forms perisperm
(d) It takes place in the kidney
(d) Outer integument — Forms testa
133. Which of the following nucleotides is a common
125. The process of transmitting growth signals from second messenger (regulatory molecule)?
the external environment to the cell nucleus is
(a) UMP (b) CMP (c) cAMP (d) ADP
known as
(a) signal transduction (b) tumorigenesis 134. The aquatic animals that change the osmotic
(c) metastasis (d) proliferation concentration of their body fluids according to the
environment are known as
126. Match the Column I with Column II and choose (a) osmoconformers (b) endotherms
the correct option from the codes given below. (c) eurythermal (d) stenothermal
Column I Column II 135. There are four layers in the wall of alimentary
A. α-diversity 1. Diversity between two communities canal. Which of these layers forms crypts of
Lieberkuhn in the intestine?
B. β-diversity 2. Diversity within a community (a) Serosa (b) Muscularis (c) Sub-mucosa (d) Mucosa
C. γ-diversity 3. Regional diversity 136. Which of the following is an aromatic amino acids?
(a) Arginine (b) Tryptophan (c) Lysine (d) Histidine
Codes
A B C A B C 137. Match the Column I with Column II and choose
the correct option from the codes given below.
(a) 2 1 3 (b) 1 2 3
(c) 3 1 2 (d) 1 3 2 Column I Column II
(Class) (Feature)
127. In eukaryotic genome, telomeres are present at the
A. Cyclostomata 1. All living members are
ends of a chromosome to
ectoparasites
(a) code for genes associated with apoptosis and PCD
(b) silence genes at chromosomal ends B. Chondrichthyes 2. Cartilaginous endoskeleton
(c) protect them from breakdown C. Osteichthyes 3. Bony endoskeleton
(d) help in crossing over during meiosis D. Amphibia 4. Most have two pairs of limbs

MODULE 2
www.jeebooks.in
214 NEET Test Drive

Codes 144. What connects scrotal sacs with abdominal cavity?


A B C D A B C D (a) Urogenital tract (b) Inguinal canal
(a) 1 2 3 4 (b) 2 1 3 4 (c) Spermatic canal (d) Haversian canal
(c) 1 2 4 3 (d) 3 2 4 1 145. Which of the following hormones is incorrectly
138. Which of the following hormones is associated with matched with its features?
Richmond-Lang effect? (a) Parathyroid — Presence of chief and oxyphil cells
(a) Ethylene (b) GA (c) ABA (d) Cytokinin (b) Adrenal cortex — Secretion of glucocorticoids and
mineralocorticoids
139. What does the term ‘explant’ mean?
(c) Adrenal medulla — Maintenance of electrolyte balance in
(a) A plant which is extinct and exists no more
body, also called salt retaining hormone
(b) A plant from which small pieces are cut and used for
(d) Thymus — Degenerates in aged people
propagation
(c) A plant part used in tissue culture 146. Endodermis generally lacks passage cells in
(d) A dioecious plant in which male and female sex organs lie (a) monocot root (b) dicot root
in such a direction which is the shape of letter ‘X’ (c) monocot stem (d) dicot stem
140. Match the Column I with Column II and choose 147. Trypsin activates the other enzymes in the
the correct option from the codes given below. pancreatic juice. Which enzyme activates
Column I Column II trypsinogen to active trypsin?
(a) Succus entericus (b) Enterokinase
A. Saxicolous 1. Lichen growing on bark
(c) Gamma globulins (d) Amylases
B. Corticolous 2. Lichen growing on wood
148. In what aspect, the GM (Genetically Modified) food
C. Lignocolous 3. Lichen growing on rock differs from the food prepared from the produce of
D. Terricolous 4. Lichen growing on soil conventionally developed varieties?
(a) The GM food contains the proteins, produced by the
Codes transgene (Cry protein in case of insect resistant varieties)
A B C D A B C D (b) GM food contains the enzymes produced by the antibiotic
(a) 3 1 2 4 (b) 1 2 3 4 resistance gene that was used during gene transfer by
(c) 4 3 2 1 (d) 1 3 2 4 genetic engineering
(c) It contains the antibiotic resistance gene itself
141. What type of gametes will be formed by genotype
AaBb? (d) All of the above
(a) AB, Ab, aB, ab (b) AB, Ab, ab, ab 149. Which of the following is a special feature of the
(c) Ab, Ab, Bb, ab (d) Aa, AA, Bb, BB earthworm (Pheretima)?
(a) It has a long dorsal tubular heart
142. Which of the following varieties of peppered moth
demonstrates industrial melanism? (b) It has open circulatory system
(c) It has setae for defence against predators
(a) Limenitis archippus
(b Danais plexippus (d) It has typhlosole that greatly increases the absorption area
of digested food in intestine
(c) Archaeospheroides barlsertonensis
(d) Biston betularia 150. Which method of direct gene transfer is involved in
delivering genes to cells in vivo?
143. Match the following autoimmune diseases given in
(a) Heat shock (b) Electroporation
Column I with the location of self-antigens given in
(c) Liposomes (d) Microinjection
Column II and choose the correct option from the
codes given below. 151. In human genome project, the largest known
Column I Column II human gene is of
(a) dystrophin (b) insulin
A. Grave’s disease 1. β-cells (pancreas)
(c) growth hormone (d) lactotropin
B. Rheumatoid arthritis 2. lgG
152. How does CO2 concentration affect stomatal
C. Type I diabetes mellitus 3. Thyroid cells
movement?
D. Addison’s disease 4. Adrenal cortex
I. Reduced CO2 concentration favours opening of
Codes stomata.
A B C D A B C D II. Increase in CO2 concentration promotes
(a) 1 2 3 4 (b) 3 2 1 4 closing of stomata.
(c) 4 3 2 1 (d) 2 1 4 3

MODULE 2
www.jeebooks.in
Prep Catalysis for NEET ~ Mock Test 6 215

III. Reduced CO2 concentration favours closing of 160. Consider the diagram given below and choose the
stomata. correct option in which the parts labelled as (A to E).
IV. Increase in CO2 concentration favours opening A
of stomata.
The correct statement(s) is/are
(a) I and II (b) Only I (c) III and IV (d) II and III B
153. If a dsDNA has 30% thymine, what is the
percentage of cytosine present?
C
(a) 70% (b) 30% (c) 40% (d) 20%
154. Which theory states that the ‘life came from outer D
space’? E
(a) Special Creation (b) Cosmozoic Origin Theory
(c) Abiogenesis (d) Naturalistic A B C D E
155. In respect to effects of noise pollution, consider the (a) Scapula, Clavicle, Radius, Carpals, Phalanges
following statements. (b) Clavicle, Scapula, Ulna, Carpals, Phalanges
I. Hearing disability (c) Clavicle, Scapula, Radius, Carpals, Metacarpals
II. Sleeplessness (d) Scapula, Clavicle, Ulna, Carpals, Metacarpals
III. Altered breathing pattern 161. The gymnosperms and angiosperms resemble in the
IV. Psychological disorder (a) presence of ovule (b) presence of pollen tube
Identify the effects which are not correct about (c) production of seeds (d) All of these
noise pollution. 162. In plasma membrane, receptor-mediated
(a) I and II (b) II and III endocytosis requires which of the following coat
(c) III and IV (d) None of these proteins?
156. Which of the following are involved in (a) Glycophorin (b) SNARE (c) Arrestin (d) Clathrin
erythropoiesis? 163. In alcoholics, which body part gets damaged due to
(a) Lymph nodes (b) Bone marrow accumulation of excessive fats?
(c) Yolk sac (d) All of these (a) Pancreas (b) Liver (c) Kidney (d) Stomach
157. Which of the following statements is incorrect 164. Which of the following cannot be found in
regarding use of herbarium? cytoplasm of an ovum of human female?
(a) Herbarium provides information about the local flora as (a) Endoplasmic reticulum (b) Centrosomes
well as flora of distant areas (c) Golgi bodies (d) Chromosomes
(b) Herbaria are used for correct and authentic identification of
unknown plants 165. Vitamin-K is essential for blood clotting because it
(c) Herbaria depict the morphological variations found in is necessary for the synthesis of
species (a) fibrinogen in kidney (b) prothrombin in liver
(d) The information contained in herbaria cannot be useful in (c) glass factor in liver (d) Hageman factor in kidney
locating wild varieties and relatives of economically 166. Match the Column I with Column II and choose
important plants the correct option from the codes given below.
158. Which of the following statements is correct? Column I Column II
(a) When an amino acid is dissolved in water, it exists in (Type of leaf) (Shape of leaf)
solution as a dipolar ion or zwitter ion
(b) A zwitter ion can act as either an acid (proton donor) or a A. Reniform 1. Lyre-shaped leaves
base (proton acceptor) B. Lyrate 2. Arrow head-shaped leaves
(c) Substances having such dual nature are amphoteric also C. Saggitate 3. Wedge-shaped leaves
known as ampholytes
D. Cuneate 4. Kidney-shaped leaves
(d) All of the above
Codes
159. Severe Acute Respiratory Syndrome (SARS) is
caused by A B C D A B C D
(a) Paramyxovirus (b) influenza virus (a) 1 2 3 4 (b) 1 2 4 3
(c) variola virus (d) Flavivirus (c) 4 1 2 3 (d) 4 1 3 2

MODULE 2
www.jeebooks.in
216 NEET Test Drive

167. The eyes of gazelle and camels are protected from 174. Which of the following molecules is associated with
sand and excessive glare by a modification which is cell adhesion?
(a) umbraculum (b) otoconia (a) Keratin (b) Myosin (c) Integrin (d) Cystein
(c) osphradium (d) ulna
175. Which of the following is not a hormone of
168. Which protein is responsible for sliding of outer gastrointestinal tract?
microtubule doublets against each other to cause (a) Gastrin (b) Secretin
ciliary bending? (c) Erythropoietin (d) Cholecystokinin
(a) Dynein (b) Tubulin (c) Nexin (d) All of these
176. GEAC is involved in
169. C2-cycle is shown by which of the following plants? (a) validity of GM research
(a) CAM plants (b) C 4 -plants (c) C 3 -plants (d) Xerophytes (b) safety of introducing GM- organisms
(c) Both (a) and (b)
170. Wood is light in colour in (d) None of the above
(a) springwood (b) autumnwood
(c) softwood (d) hardwood 177. In glycolysis, which of the following reactions is
catalysed by enzyme enolase?
171. Which of the following is a viral poultry disease? (a) Phosphoenol pyruvate → Pyruvate
(a) Fowl cholera (b) Fowl coryza (b) 3-Phosphoglycerate → 2-Phosphoglycerate
(c) Fowl coccidiosis (d) Fowl pox
(c) Glyceraldehyde-3-phosphate → 1, 3– Biphosphoglycerate
172. A certain gas which is considered to be a pollutant (d) 2-Phosphoglycerate → Phosphoenolpyruvate
and can cause health hazards, if present in high
178. Which of the following is not an IUD?
concentrations does not obstruct the oxygen
(a) Lippes loop (b) Progestasert (c) LNG-20 (d) Norplant
transport. Identify the gas.
(a) SO 2 (b) CO (c) NO (d) SO 3 179. The division Gnathostomata can be divided into
which superclasses?
173. The transcellular streaming theory states that
(a) Pisces and Amphibia (b) Tetrapoda and Reptilia
(a) sieve tubes contain tubular strands which demonstrate a
(c) Amphibia and Reptilia (d) Pisces and Tetrapoda
kind of peristaltic movement
(b) nitrates move up with the transpiration current 180. The culture media used in hydroponics contains
(c) food materials are transported through the living cells of which chemical compound in highest
the phloem concentration?
(d) source-sink relationship may be reversed depending on (a) Ammonium phosphate (b) Copper sulphate
the season (c) Boric acid (d) Potassium nitrate

Answer Sheet
.

1. (b) 2. (d) 3. (b) 4. (b) 5. (c) 6. (c) 7. (c) 8. (c) 9. (b) 10. (d)
11. (c) 12. (c) 13. (b) 14. (a) 15. (c) 16. (a) 17. (d) 18. (b) 19. (c) 20. (d)
21. (b) 22. (b) 23. (c) 24. (d) 25. (b) 26. (d) 27. (b) 28. (d) 29. (a) 30. (c)
31. (b) 32. (a ) 33. (c) 34. (d) 35. (d) 36. (a) 37. (b) 38. (b) 39. (a) 40. (d)
41. (b) 42. (c) 43. (a) 44. (a) 45. (b) 46. (a) 47. (c) 48. (a) 49. (c) 50. (b)
51. (b) 52. (c) 53. (b) 54. (b) 55. (a) 56. (d) 57. (d) 58. (d) 59. (c) 60. (a)
61. (c) 62. (c) 63. (d) 64. (b) 65. (c) 66. (d) 67. (b) 68. (b) 69. (d) 70. (d)
71. (c) 72. (c) 73. (d) 74. (d) 75. (d) 76. (a) 77. (c) 78. (a) 79. (b) 80. (c)
81. (a) 82. (a) 83. (d) 84. (a) 85. (a) 86. (a) 87. (b) 88. (c) 89. (d) 90. (b)
91. (a) 92. (d) 93. (b) 94. (b) 95. (d) 96. (c) 97. (c) 98. (d) 99. (a) 100. (d)

101. (b) 102. (b) 103. (b) 104. (d) 105. (d) 106. (a) 107. (d) 108. (d) 109. (a) 110. (d)
111. (d) 112. (b) 113. (a) 114. (d) 115. (d) 116. (a) 117. (d) 118. (d) 119. (b) 120. (a)
121. (c) 122. (d) 123. (a) 124. (a) 125. (a) 126. (a) 127. (c) 128. (d) 129. (a) 130. (c)
131. (b) 132. (d) 133. (c) 134. (a) 135. (d) 136. (b) 137. (a) 138. (d) 139. (c) 140. (a)
141. (a) 142. (d) 143. (b) 144. (b) 145. (c) 146. (b) 147. (b) 148. (d) 149. (d) 150. (c)
151. (a) 152. (a) 153. (d) 154. (b) 155. (d) 156. (d) 157. (d) 158. (d) 159. (a) 160. (c)
161. (d) 162. (d) 163. (b) 164. (b) 165. (b) 166. (c) 167. (a) 168. (a) 169. (c) 170. (a)
171. (d) 172. (a) 173. (a) 174. (c) 175. (c) 176. (c) 177. (d) 178. (d) 179. (d) 180. (d)

MODULE 2
www.jeebooks.in

EXPLANATIONS

PHYSICS
1. Magnetic field at A due to current or ∆P = 2mv sin 30°  1 
−I 0 0 −
carrying loop B, 1  2  = 4I 0
= 2 × 2 × 100 × = 200 kg-ms -1 =
µ 0IR 2 2 2π T 2π
BA = ×
3 ∆P 200 T 8
2(R 2 + x 2 ) 2 ∴ F = = = 500 N I0
∆t 0.4

Irms
= 2 = π
Magnetic flux passing through loop A, 5. Magnetic field B,
I avg 4I 0 4
φ = B . πr 2 ...(ii) µ i
A A
= 0 . (sin φ1 + sin φ 2 ) 2π
Induced emf will be maximum, if 4π a
I
d d  −dφA  But φ1 = 90° − θ1 9. Time period, T = 2 π
eA =   =0 M ⋅ BH
dx dx  dt  and φ2 = 90° − θ 2
∴ 1
[where, e A = Induced emf in loop A] or T ∝
µ i BH
dφ B = 0 [sin(90° − θ1) + sin(90° − θ 2 )]
and eA = − A 4 πa
dt T1 BH 2 3 32 − 16
µ i ∴ = or =
R = 0 . [cos θ1 + cos θ 2 ] T2 BH 1 T2 32
On solving, we get x = 4π a
2 3 1
6. Power of combination is ⇒ = ⇒ T2 = 3 2 s
2. T2 2
P = P1 + P2
v − v0 
a a P = 10 − 2 = 8 D 10. Apparent frequency, ν′ =  ν
1 100  v 
m ∴Focal length, F = = = 12.5 cm
m P 8 ν′ v − v 0 90 330 − v 0
⇒ = ⇒ =
1 ν v 100 330
65° 25° 7. For first 3 s, θ1 = ω 0t + α t 2
2 [As pitch of the sound of a source is
Acceleration of system is, As ω 0 = 0, dropped by 10%]
mg sin 65 ° − mg sin 25 ° ∴
1 9
θ1 = × α (3) 2 = α ⇒ 330 − v 0 = 9 × 33
a=
2m 2 2 ⇒ v 0 = 33 ms −1
mg (sin 65 ° − sin 25 ° ) In first 6s,
= 11. As s nth = s 5
2m 1
(θ1 + θ 2 ) = ω 0t +
αt 2 1 1
10 × 05
. 2 ⇒u + g (2t − 1) = ut + gt 2
= = 2.5 ms −2 2 2
2  9 α + θ  = 1 α (6) 2 = 36 α = 18α
 2 1 1
m1a 1 + m 2a 2 2  2 2 ⇒ 0 + g (2t − 1) = 0 + g × 25 [Qu = 0]
aCM = 2 2
m1 + m 2 27
⇒ θ2 = α 25 + 1
2 ⇒t = = 13 s
a 2.5
= = ms −2 = 1. 25 ms −2 θ2 27α 2
2 2 ∴ = =3
θ1 9α 12. From first law of thermodynamics,
3 p − p 0   2V0 − V0 
3. ∆WCyclic = − π  0   T /4 ∆Q = ∆U + ∆W
 2  2 
8. I rms =
I0
and I avg =
∫T / 8 I 0 sinωt ⇒ ∆U = ∆Q − ∆W = 210 − 80 = 130 J
πp 0V0 2 T − T 
=−   13. By theorem of perpendicular axes,
2  4 8
 − cos ωt 
T /4
I disc = ID + MR 2
4. Change in momentum,
 ω T / 8 1 5
∆P = Pf − Pi = I0 = MR 2 + MR 2 = MR 2
T − T  4 4
= mvsin 30° − ( − mv sin 30° )  
 4 8 Iring = ID + MR 2
A mv
 cos 2 π × T − cos 2 π × T  1 3
= MR 2 + MR 2 = MR 2
− I 0  T 4 T 8  2 2
30° =
– mv sin 30° O ω T 5
I 5
mv sin 30° 8 ∴ disc = 4 =
30°  as, ω = 2 π  Iring 3 6
 T  2
B mv

MODULE 2
www.jeebooks.in
218 NEET Test Drive

2
Mk disc 5 Current in the above two cases must 6.6 × 10−34 × 3 × 108
⇒ = =
2
Mkring 6 be proportional to the emf in the two 1700 × 10−9
cases. i.e.,
k disc 5 = 0.012 × 10−17 J
⇒ = or 5 : 6 (20 − 2n)E + 3E 5
kring 6 = 0.012 × 10−17
(20 − 2n)E − 3E 3 = eV = 073
. eV
1. 6 × 10−19
14. Given, 50 VSD = 175
. cm 20 − 2n + 3 5
⇒ =
175
. 20 − 2n − 3 3 23. By conservation of angular momentum,
⇒ 1 VSD = = 0.035 cm 1 1
50 ⇒ n =4 MR 2ω = (M − m)R 2ω′ + mR 2ω
2 2
1 MSD = 610
. − 5.95 = 015
. cm 18. The colour seen in the reflected white 1 1
∴Least count of Vernier light from a thin oil film are due to ⇒ (M − 2m)R 2ω = (M − m)R 2ω′
2 2
= 1 MSD − 1 VSD interference.
(M − 2m)
19. Given, u = 7ms −1, v = 0, s = 10 m and ⇒ ω′ = ω
= 015
. − 0.035 (M − m)
= 0115
. cm a=?
24. O = 4 mm, u = − 30 cm
The diameter of cylinder From equation of motion,
R 60
= MSR + (VSR × LC) v 2 − u 2 = 2as f =
= = 30 cm
2 2
= 5.95 + (25 × 0115
. ) or 0 − 7 2 = 2a × 10
1 1 1
⇒ a = − 2.45 ms −2 From mirror formula, = +
= 8.825 cm f v u
−9.8 −g
15. The time period of satellite is or a= ms −1 = 1 1
= −
1
4 4
T 2 = kr 3 30 v 30
3 ∴Total resistance ⇒ v = 15 cm
⇒ T = kr 2 = − ma = +
mg
=+
w I v I 15
3 4 4 Q = ⇒ =
o u 4 30
T1  r1  2 −λ Xt −12t
or =  N x Noxe 3000 × e ⇒ I = 2 mm
T 2  r2  20. = =
Ny Noye −λYt 6000 × e −10t
25. Let radius of larger drop is R and each of
If v1 and v 2 be the orbital speeds of the N x 1 −2t
or = e small drop is r .
planets,
Ny 2 As per given condition,
2 πr1
v1 = 1 4 4
T1 1 = e −2t πR 3 = 2 ⋅ πr 3
2 πr 2 2 3 3
and v2 = 1
T2 (when, N x = N y )
So, R = 2 3r
v1 r1 T 2 1
∴ = × e 2t =
2 Surface energy,
v 2 r 2 T1 2
1
3 1 loge e = loge
2t
W1 = T ⋅ A = 4 πR 2T = 4 π 2 3 r 2T
r1  r 2  2 r  2 2
=   =  2 2
r 2  r1   r1  2t = loge  
1 W1 23
 2 W2 = 4 πr T ⇒ 2
=
1 W2 1
 1012  2 1
loge  
=  13  = 1 1
t = ⇒ W1 : W2 = 2 2 / 3 : 1
 10  10 2  2 
26. Actual distance of separation, d = d1d 2
1 S  21. Using law of conservation of energy,
16. Pressure by absorbed light =   = 32 × 2 = 8 cm
2 c 1 GMm
m (kv ) 2 − =0 27. Heat dissipated in r1 in time t = Heat
1  2S  2 (R + R )
Pressure by reflected light =   dissipated in r 2 in time t
2 c  GM
⇒ kv = I12r1t = I 22r 2t
Total radiation pressure on surface is R 2
3  E  E2
S gR 2   .r1t = .r 2t
. × 103
15 = = gR  r1 + r  (r 2 + r ) 2
p rad = 2 = = 5 × 10−6 Pa R
c 3 × 108
The escape velocity from earth’s ⇒ r = r1r 2
p rad 5 × 10−6
∴ = = 5 × 10−11 surface, 28. Breaking stress of both wires would be
p0 1 × 105
v = 2gR same, so
17. Let n cells are connected with wrong 1 T (max)1 T (max) 2 T T
So, k = = ⇒ (max)21 = (max)22
polarity 2 πr12 πr 22  1  4
∴Effective emf of the battery    
22. Given,  2  2
= (20 − n)E − nE λ = 1700 nm = 1700 × 10−9 m ⇒ T (max) 2 = 16T (max)1 = 16 × 500 N
= (20 − 2n)E ∴Energy gap =
hc
in joule = 8000 N
λ

MODULE 2
www.jeebooks.in
Prep Catalysis for NEET ~ Mock Test 6 219

29. Energies are ∴Number of molecules in the given


hc hc volume . × 10−3 ) 2 × 7.7 × 103 × (50) 2
22 (12
E1 = and E 2 = = ×
λ1 λ2 KE of given gas 7 4
=
E1 λ 2
KE of a molecule of gas = 2178
. N
∴ = 3
E 2 λ1 × 10 × 13.6 × 980 × 2 41. (i) Moment of inertia of a cylinder about
λ1 . = 2 = 5.71 × 108 its centre and parallel to its length
E2 = E1 7 × 10−4
λ2 MR 2
=
35. Initial momentum, pi = 0 2
6 × 10−7 × 3.32 × 10−19
= Final momentum, R
4 × 10−7
pf = mv $i + mv $j + p 3
= 4.98 × 10−19 J ~− 5 × 10−19 J
From conservation of linear momentum,

30. Induced emf is given by e = − pf = pi
dt L
e 1 dφ or p 3 = mv 2
As, i = =−
R R dt p 32 1 1
∴Total KE = + mv 2 + mv 2
∴Total charge induced = ∫ idt 2 × 2m 2 2
(i) (ii)
1 dφ 1 φ2 2m 2v 2 3mv 2
= −∫ dt = − ∫ d φ = + mv 2 = (ii) Moment of inertia about its centre and
R dt R φ1 4m 2
1 1 36. In damped oscillations the motion of
= − [φ2 − φ1] = (φ1 − φ2 ) perpendicular to its length
R R simple pendulum die out gradually due
 L2 R2 
Thus, the induced charge in a to air drag and friction of the support. = M + 
The oscillation of a child swinging in a  12 4 
conducting loop, moving in a magnetic
field depends on the total change in swing cannot be sustained due to According to question,
magnetic flux. resistive forces present in nature.
ML2 MR 2 MR 2
37. Charles’ law for pressure, + = ⇒L = 3R
31. In case of AND gate 12 4 3
output (Y) = (Input A) ⋅ (Input B) p ∝T when,V = constant 42. Given, E = 50 V/m
⇒ Y = A ⋅B 38. Since, electric dipole has equal
∴ εr = 1 + χ
In case of NAND gate amount of positive and negative
charge, therefore total outward flux is [where, χ is electrical susceptibility and
Y = A ⋅B equal to total inward flux. Hence, net εr is dielectric constant]
In case of NOR gate electric flux will be zero. . = 1 + χ ⇒ χ = 5.5
65
Y = A+B 39. Total energy of orbiting satellite at a ∴Polarisation, P = χ E = 5.5 × 50 = 275
In case of NOT gate height, 43. Redraw the circuit,
Y =A h=−
GMm 1
+ mv 2 2Ω
(a) Here A = 0⋅ 0 = 0 (R + h) 2
(b) B = 0 ⋅ 1 = 1 GMm 1  GM  GMm
=− + m  =− A B
(c) C = 1 ⋅ 0 = 1 (R + h) 2 R + h 2 (R + h) 2Ω
32. Ratio of intensities,
I1 a12 a 2 16 a 4 Energy expended to rocket the satellite
= 2 ⇒ 12 = , 1 = 2Ω 2Ω
out of earth’s gravitational field
I 2 a2 a2 4 a2 2
= − Total energy of the orbiting satellite
∴ a 1 : a2 = 2 : 1 GMm
=
33. By Balmer formula, we have 2(R + h) Let R AB is equivalent resistance between
1  1 1 (6.67 × 10−11) × (6 × 1024 ) × 200 A and B
= R 2 − 2  =
λ  n1 n2  2 (6.4 × 106 + 4 × 105 ) ∴
1 1 1 1 5
= + + =
R AB 2 2 4 4
× 107  2 − 2 
1 1
= 1097
. = 6 × 109 J
2 3  4
or R AB = Ω = 0.8 Ω
40. Mass per unit length of iron wire is 5
⇒ λ = 6558Å (visible region)
πd 2 × 1 × ρ
m = A ×l ×ρ = 44. Here, p-n junction diode is in forward
34. KE of the given gas at 27° C 4 bias with voltage,
1 3
= MC 2 = pV As velocity, v =
T
⇒v2 =
T V = 5 − 3 = 2V
2 2 m m 2 1
3
= × (10 × 13.6 × 980) × 2 ergs ∴Current, I = = A = 10 mA
πd 2ρ 200 100
2 ⇒ T = mv 2 = ×v2
4 . × 10−19 × 2
45. Kinetic energy, K = eV = 16
= 3.2 × 10−19 J

MODULE 2
www.jeebooks.in

CHEMISTRY
46. In case of same nucleophilic site, 52. Cl 58. All the alkali metals, their oxides,
nucleophilicity parallels the basicity COOH peroxides and superoxides readily
3 1
while in case of different sites, 4 2 dissolves in water to produce the
nucleophilicity varies inversely with corresponding hydroxides which are
electronegativity. Substituent ® 2-methyl strong alkalies.
Order of electronegativity, N > C. 3-chloro 2Na + 2H2O ¾® 2NaOH + H2
Thus, the order of nucleophilicity C > N. Parent chain ® butan-1-oic acid Na 2O + H2O ¾® 2NaOH
- - IUPAC name = 3 chloro-2-methyl Na 2O 2 + 2H2O ¾® 2NaOH + H2O 2
Among, CH3 CH2 , HC ºº C and butanoic acid. 2NaO 2 + 2H2O ¾®
H2 C == CH– , CH3 CH2– is the strongest
53. Carbon tetrachloride (CCl 4 ) is not 2NaOH + H2O 2 + O 2
nucleophile, as it is less electronegative
(due to least s-character). hydrolysed under normal conditions. The last three reactions correspond to
This is because of absence of the hydrolysis of oxides, peroxides and
47. When an electron is added to O-atom d-orbitals in case of carbon as a result superoxides, respectively.
to form O - ion, energy is released. But it cannot form five-coordinated
when another electron is added to O - 59. Number of moles of He = 0.3
intermediate.
to form O 2 - ion, energy is absorbed to Number of moles of Ar = 0.4
overcome the strong electrostatic 54. The transition temperature of rhombic
and monoclinic sulphur is 369 K or We know that, KE = nRT
repulsion between the two negatively
KE of He = 0.3 ´ R ´ T
charged O - ion. 96ºC. The stable form at room
temperature is rhombic sulphur which KE of Ar = 0.4 ´ R ´ 400
48. Graphite rod acts as anode during the transforms to monoclinic sulphur when According to question,
electrolytic reduction of alumina. At heated about 369 K. KE of Ar = KE of Ar
anode, O 2 gas is produced which
55. N cal = e ´ d 0.3 ´ R ´ T = 0.4 ´ R ´ 400
reacts with the carbon of anode (rods) l

0.4 ´ R ´ 400
to produce CO 2 gas. = 1.6 ´ 10 - 19
´ 3.40 ´ 10 C -m - 10 T = » 533 K
0.3 ´ R
49. In physisorption, gaseous molecules = 5.44 ´ 10- 29 C - m
are attached to the surface of N 60. n-butyl lithium is an organolithium
% ionic character = exp ´ 100
adsorbent by weak van der Waals’ N cal reagent. It is widely used as a
force of attraction. - 30 polymerisation initiator in the formation of
6.28 ´ 10
= ´ 100 elastomers, such as polybutadiene. Also,
50. The first ionisation energy decreases 5.44 ´ 10- 29 it is broadly employed as a strong base in
down the group from C to Sn as the synthesis of organic compounds such
= 12%
expected, but there is slightly increase as in the pharmaceutical industry.
as we move to Pb. 56. Potential energy = - 2 ´ total energy
61. A. [Fe(CN) 6 ] -3
The increase in ionisation energy from æ 13.6 Z 2 ö
=-2´ç 2
÷ x + (-1) ´ 6 = -3
Sn to Pb is due to the poor screening è n ø x + (-6) = -3
effect of the 4f and 5d-electrons. The
decreasing order is For helium, Z = 2 and n = 1in first Bohr x-6= -3
orbit x = -3 + 6
C > Ge > Si > Pb > Sn.
Thus, potential energy x = +3
CH3 13.6 ´ (2) 2 B. Na 3 [O 5 Cl 5N]
=-2´
51. and (1) 2 x + (-1) ´ 5 + 1 ´ (-3) = -3
= - 108.8 eV x + (-5) + (-3) = -3
CH3
x + (-8) = -3
IUPAC name = 57. When acidified K 2 Cr2O 7 solution is
1,2-dimethyl cyclobut-1-ene x = -3 + 8
added to Sn2 + salts then Sn2 + x = +5
H3C changes to Sn4 + . The reaction is given
below. C. [HMn(CO) 5 ]
(-1) + x + (0) ´ 5 = 0
H3C Oxidation
-1 + x + 0 = 0
IUPAC name = +6 2–
3,4-dimethyl cyclobut-1-ene Cr2O7 + 14H+ + 3Sn2+ 2Cr3+ + 3Sn4+ x = +1
+ 7H2O D. Fe4 [Fe(CN) 6 ]
Reduction
x + (-1) ´ 6 = -4

MODULE 2
www.jeebooks.in
Prep Catalysis for NEET ~ Mock Test 6 221

x + (-6) = -4 69. Millimole = M ´ V R R R


x - 6 = -4 Þ x = -4 + 6 01.
V = = 0125
. mL R—C—Br + Nu– Nu—C—Br
x = +2 0.8
R R
62. For liquefaction of gases, 70. In a-helix structure, hydrogen bonds
The transition state is involved which is
temperature should be lower than are present between ¾ NH group of
one amino acid residue to the more stable in case of unhindered
critical temperature and
carbons.
pressure should be higher than critical C O group of another amino acid
pressure. residue. 76. H
O NHOH
63. The reaction with positive E º are 71. Maximum work is always obtained in a
N
feasible reactions whereas with reversible process. NH2OH H2SO4
negative E º are non-spontaneous \ W = - 2.303 nRT log(V2 / V1)
reactions. Q nO2 = (w / M ) = (16 / 32); Cycloketone (B) Caprolactam
2Fe3 + + 2I- ¾® 2Fe2 + + I2 ; (C)
º
R = 8.314 JK -1mol -1,
E cell = + 0.23 V
T = 300 K,V1 = 5 dm3 , 77. The reaction of propene with HOCl
Cu + 2Ag + ¾® 2Cu2 + + 2Ag; proceeds via the addition of Cl + in the first
º V2 = 25 dm3
E cell = + 0.46V step. HOCl has Cl + and OH- ions.
2Fe3 + + Cu ¾® 2Fe2 + + Cu2 + ; \ W = - 2.303 ´ (16 / 32) The reaction is takes place as follows:
8.314 ´ 300 ´ log æç ö÷
º
25
E cell = + 0.43 V è5 ø Electrophilic
CH3—CH == CH2 + Cl+
Ag + Fe3 + ¾® Ag + + Fe2 + ; = - 2.01 ´ 103 J addition
º Propene
E cell = - 0. 03 V 72. Mixture of a- and b- form indicates that +
º
Last reaction is not feasible since E cell CH3—CH—CH2—Cl
one ¾OH group is different from other
is negative. OH–
which may combine with ¾ CHO group
64. QDng = - ve to form intramolecular hemiacetal. So, CH3—CH—CH2—Cl
this reaction is an evidence of the
\ DH = - ve OH
cyclic structure of glucose in solutions.
Hence, reaction will shift towards
forward direction if extra amount of F2
73. Given, M = 92, m = 60, n = 2 78. Arrhenius equation is given by
or Cl 2 is added. M -m k - ln A = -
Ea
Degree of dissociation (x ) = ln
(n - 1)m RT
65. For corner S-atom, + 2 each from
92 - 60 32 On comparing with equation of straight
(== O), + 1from (O - ) and zero from - S. = = = 0.53 line, y = mx + c
Therefore, its oxidation number is + 5. (2 - 1) ´ 60 60
E
Middle S-atoms has zero oxidation % dissociation = 0 × 53 ´ 100 = 53% Slope = - a
R
number.
74. Under weakly acidic conditions 4 Ea
66. Except LiOH, all the alkali metal Þ - 2 ´ 10 = -
nitrobenzene on electrolytic reduction 8.314
hydroxides are highly soluble in water. gives aniline but under strongly acidic
LiOH is much less soluble on account E a = 166.28
conditions it gives p-aminophenol.
of high lattice enthalpy.
NO2 NHOH NH2 E a » 166 kJ / mol.
67. Dng = (3 + 1) - 2 = 2 79. Solvate isomerism is shown when two
Dn g 4H+ 2H+ compounds having same molecular
K p = KC (RT )
Weakly formula differ by water or solvent
= 3 ´ 10- 4 (0. 0821 ´ 1000) 2 Nitrobenzene Aniline molecule directly bonded to metal ion or
H+
= 3 ´ 6740.41 ´ 10- 4 strongly present as free solvent molecules in a
= 3 ´ 0.6740 = 2.02 crystal lattice.
NH2
Cordination compounds, [Cr(H2O) 6 ]Cl 3
68. Gammexane is chemically known as
and [Cr(H2O) 5 Cl]H2O × Cl 2 are solvate
benzene hexachloride (BHC).
isomers because water is exchanged by
Cl chloride ion.
Cl Cl 80. The AgBr crystal should have Frenkel
Sunlight OH
+ 3Cl2 p-aminophenol defect. The Frenkel defect causes by
Cl Cl displacement of an atom from its lattice
75. Tertiary alkyl halides are partially position to an interstitial site, creating a
Cl inert to substitution by SN 2 mechanism vacancy at the original site and an
Benzene hexachloride
(BHC) because of steric hindrance. In SN 2 interstitial defect at the new location.
reaction, nucleophile and alkyl halide
81. Order and molecularity are same for an
reacts in one step.
elementary reaction.

MODULE 2
www.jeebooks.in
222 NEET Test Drive

82. OCH3 OCH3 OCH3


O OH
Cl C
(CH3)3 CCl Cl2
H CN H CN
Anhy.AlCl3 FeCl3 CN
Anisole
C(CH)3 C(CH3)3
∆ HBr H

OH
Cl O OH

CN H
OH O–
C(CH3)3 Benzoin
(A)
88. Half-cell reactions are :
83. In 4f-series, as the atomic number increases from La to Lu, the
atomic and ionic radii decreases gradually due to lanthanide (i) AgCl + e - ¾® Ag (aq) + Cl - (aq); E º = 0.22 V
contraction. Thus, lanthanum (La) has the largest atomic radii in (ii) Ag+ + e - ¾® Ag; E º = 0.80 V
the lanthanide series. Subtracting Eqs. (i) from (ii)
84. When concentrated HCl is added in an aqueous solution of AgCl ¾® Ag + + Cl -
CoCl 2 then it forms [CoCl 4 ] 2 - complex. º
E cell = 0.80 - 0. 22 = + 058
. V
85. OH OH We know that,
Br Br DG = nFE º
+ 3 Br2 - RT lnk = - nFE º
480 g nFE º 2 ´ 96500 ´ 058
.
In k = =
94 g RT 8.314 ´ 300
Br
In k = 44.88
Q94 g of phenol requires = 480 g Br2
k = Antilog æç
44.88 ö 19
480 ÷ = 3.07 ´ 10
\ 3 g of phenol will require = ´ 3 = 15.32 g è 2.303 ø
94
89. Number of atoms = moles ´ N A
86. Heat of hydrogenation is inversely proportional to the stability of
the compound. Since, among the given compounds, POCl3
buta-1-3-diene is resonance stabilised, i.e. more stable. Thus, it + H—C—N(CH3)2
90.
has the lowest heat of hydrogenation per mole. S S CHO
O
87. Reagent used in benzoin condensation is KCN and reaction This reaction is known as Vilsmeier Hack formylation.
occurs as follows :

BIOLOGY
91. Bundle sheath extensions are sclerenchymatous in isobilateral The retention of megaspore within the megasporangium and
leaf. Bundle sheath extensions are parenchymatous in development of receptive areas like stigma for catching
dorsiventral leaf. microspores are also necessary.
92. An adult kidney weighs around 150 g that is about 0.1% of body 96. Archaebacteria differ from prokaryotes in having proteins,
weight. glycoproteins or non-cellulosic polysaccharides in their cell wall
instead of peptidoglycan or mucopeptide substances.
93. The principal embryonic membranes are amnion and chorion.
Amnion is developed by the amniogenic cells forming the wall of 97. Prophase-I of meiosis-I is divided into following stages (in their
the amniotic cavity. Chorion is formed by the parietal correct order).
extraembryonic mesoderm. Leptotene ® Zygotene ® Pachytene ® Diplotene ® Diakinesis
98. Nitrogenase catalyses conversion of atmospheric nitrogen to
94. When both stipules of a leaf combine together and form a ammonia. The reaction is as follows
tube-like structure, then it is known as ochreate. It is found in Nitrogenase
Polygonum. N 2 + 8 e - + 8H + + 16ATP ¾¾¾¾®2NH 3 + H 2 + 16ADP + 16Pi

95. The conditions necessary for seed formation include production From the above reaction, it is clear that nitrogenase requires
of two types of spores, which is known as heterospory. availability of ATP (8 ATP for each NH3 produced) for its activity.

MODULE 2
www.jeebooks.in
Prep Catalysis for NEET ~ Mock Test 6 223

99. Xylem vessels are arranged in 109. In the given figure, 120. Lichens are the pioneer species in the
Y-shaped manner in monocot stem. A – Epiglottis primary succession on rock. They
In dicot stem, xylem vessels are usually secrete some acids that dissolve rocks
B – Hyoid bone
arranged in rows. and thus, help in their weathering and
C – Thyroid cartilage
soil formation.
100. None of the option is correct regarding D – Cricoid cartilage
the root. It is positively geotropic and 110. The option (d) is incorrectly matched 121. Excess of glucose and amino acids from
negatively phototropic. It does not bear the digested food reach to the liver.
pair. It can be corrected as follows
nodes and internodes. Excess of glucose is converted into
In helobial yeast, life cycle pattern is
glycogen and stored in liver for later use.
101. Dithecous is two-lobed anther. diplobiontic as haploid phase is greatly
reduced. It is represented by only 122. Statements I, II and III are correct.
Monothecous is one-lobed anther.
ascospores, whereas diploid phase is Mitochondrial matrix have few RNA
In inserted type, anther face is towards more prominent. molecules, single circular DNA molecule,
the centre. ribosomes of 70S type. Mitochondrial
111. The increase in the concentration of
In exerted type, anther face is towards salts in the soil is known as soil single circular DNA molecule is passed
the periphery. salination. It is due to poor drainage from mother to the offspring through egg
and addition of fertilisers to soil. It may cell. RNA molecules and ribosomes are
102. Dextran is a microbially produced
also occur due to the closeness of soil required for the synthesis of proteins.
biopolymer used as blood expander
and absorbent. The microbe to sea area. 123. The demand for O 2 is directly related to
responsible for its production is a increasing input of organic waste and is
112. In a nucleosome, histones are
bacteria named Leuconostoc expressed as biochemical oxygen
organised as an octamer. A histone
mesenteroides. demand. The lower O 2 content kills many
octamer is a eight protein complex, it
sensitive aquatic organisms like
103. In angiosperms, endosperm is triploid consists of two copies of each H2A,
planktons, molluscs, fish, etc.
(3n) and formed after fertilisation. It is H2 B, H3 and H4 .
2 3 4

triploid because it is formed when one 113. Phenotypic adaptations include 124. The option (a) is incorrectly matched.
of the two sperms (haploid) enters A seed is a fertilised mature ovule that
non-genetic changes in individuals
inside the embryo sac and fuses with possesses an embryonic plant, stored
such as physiological modifications,
two polar nuclei (each polar nucleus is food and protective coverings. Ovary
e.g. acclimatisation. These are the
haploid). forms the fruit.
favourable changes in the morphology
104. Arginine and lysine are found abundant and physiology, which develop in 125. Cells in G 0 can be stimulated to re-enter
in the histone proteins. Histones are response to changes in environmental the cell cycle by external growth signals.
highly alkaline proteins found in conditions. These signals can be growth factors or
eukaryotic cell that package and 114. Cloning is the production of living hormones. Signal transduction (also
arrange the DNA into necleosomes. structures or organisms, which are called cell signaling) is the transmission
105. Statement (d) is correct as, cyclic genetically identical to their parents. it of molecular signals from the exterior of
photophosphorylation involves only is not an application of DNA a cell to its interior.
PS-I. On the other hand, non-cyclic fingerprinting. 126. a-diversity is the diversity within a
photophosphorylation involves both 115. All the options are correctly matched. community. It is also called as local
PS-I and PS-II. Cell cycle has three major checkpoints diversity.
106. In atactostele, vascular strands remain to monitor any error during cell divison. b-diversity is the diversity between two
scattered in the vascular tissue, e.g. If any parameter is not met, cell communities.
monocots. division stops. g-diversity is regional diversity. It refers
107. Inclusions are the reserve deposits. 116. Carbonic anhydrase found in the blood to the total richness of species in all
They include metachromatic granules catalyses the reaction that leads to habitats in a region.
(usually glycogen or starch), dissociations of carbonic acid to 127. A telomere is a region of repetitive
carboxysomes and gas vacuoles. bicarbonate and hydrogen ion. nucleotide sequences at each end of a
A plasmid is a small, self-replicating, 117. The statements II and III are correct. chromosome, which protects the
circular, extrachromosomal DNA. Blue-green alage and some bacteria chromosomal ends from deterioration.
108. Structurally amino acids are made up are photoautotrophic. Their thylakoids 128. Bile helps in the absorption of fat soluble
of hydrogen, carboxyl group, amino lie freely in the cytoplasm. They are not vitamins (A, D, E and K) in the small
group and a variable group. The group organised into chloroplasts. intestine, not in their formation.
which is variable is designated as R
118. The kinetochore structure forms on 129. Net primary productivity is the available
group. Based on nature of R group,
opposite sides of each paired biomass for the consumption by
amino acids are of many types.
centromere, in intimate association with heterotrophs (herbivores and
R-group of amino acids like glycine,
the two sister chromatids. It is decomposers).
proline, alanine, valine, isoleucine and
kinetochore where the spindle fibres
methionine are non-polar and aliphatic. Net primary productivity can be obtained
attach during cell division to pull sister
These amino acids are hydrophobic in by GPP (Gross Primary Productivity) – R
chromatids apart.
nature and their side chains cluster (Respiration losses).
together within the proteins to stabilise 119. Funiculus forms the stalk of the seed,
protein structure by hydrophobic which ultimately withers and leaves a 130. For parthenocarpy induction by auxins,
interactions. minute scar called hilum. these should be applied after anthesis

MODULE 2
www.jeebooks.in
224 NEET Test Drive

(first opening of flower) and by 139. Tissue culture is a technique of is called genetically modified food or GM
gibberellins, these should be applied growing of cells, tissues, etc., in food. The GM foods differ from the
earlier, i.e. at anthesis. artificial medium. The plant part used in produce of conventionally developed
tissue culture is called explant. varieties in aspects like the presence of
131. Acorn or tongueworm is the common protein produced by transgene, the
name of Balanoglossus. Tornaria is 140. Saxicolous is lichen growing on rock, enzymes produced by antibiotic
larva of some species of Hemichordata e.g. Dermatocarpon. resistance gene and antibiotic resistance
such as acornworms. Corticolous is lichen growing on bark, gene.
Balanoglossus is genus of great e.g. Usnea. 149. In the typhlosole region, the mid-dorsal
zoological interest because, being a Lignocolous is lichen growing on wood, wall of the intestine is thrown into a
hemichordate, it is an evolutionary link e.g. Cyphelium. longitudinal fold known as typhlosole.
between invertebrates and vertebrates. Terricolous is lichen growing on soil, It increases the absorptive surface of the
e.g. Cladonia. intestine.
132. Statement (d) is incorrect. It can be
corrected as follows 150. Liposome is the method of direct gene
141. Based on the law of independent
transfer which is involved in delivering
Ornithine cycle/urea cycle takes place assortment, four types of gametes will genes to cells in vivo.
in the liver. be produced by the individual with
genotype AaBb, i.e. AB, Ab, aB, ab. Vectors can be encased in liposomes,
This cycle was studied and described
which are small membrane vesicles. The
by Krebs and Hanseleit. So, it is also 142. Biston betularia was the light coloured liposomes fuse with the cell membrane
called as Krebs-Hanseleit cycle. variety of peppered moth, which got delivering the DNA into the cell.
Carbamoyl phosphate is formed from naturally eliminated because of
two molecules of ammonia and (CO 2 ), 151. HGP goal is to determine the sequence
industrial revolution.
in the presence of enzyme carbamoyl of nucleotide base pairs that make up
phosphate synthetase and ATP. All 143. In autoimmune diseases, our immune human DNA and identify and map all of
enzymes involved in the cycle are system fails to recognise ‘self’ from them.
located inside the mitochondria. ‘non-self’ and starts destroying body In human genome project, the largest
proteins. known human gene is of dystrophin.
133. cAMP (adenosine 3¢, 5¢- cyclic
monophosphate) is the most common Grave’s disease is an autoimmune 152. Reduced CO 2 concentration favours
second messenger. It is formed from disease that causes hyperthyroidism, opening of stomata, while increase in
ATP in a reaction which is catalysed by or overactive thyroid. In this disease, CO 2 concentration promotes closing of
enzyme adenyl cyclase. immune system attacks the thyroid, stomata.
causing it to produce more thyroid
134. The aquatic animals that change the hormone than body needs. 153. Thymine (T) pairs with Adenine (A) and
osmotic concentration of their body Cytosine (C) pairs with Guanine (G). If
Rheumatoid Arthritis (RA) is an thymine is 30%, adenine will also be
fluids according to the environment are autoimmune disease caused by
known as osmoconformers. equal, i.e. 30%. Now, as (T + A) = 60%,
antibody against IgG. This antibody is
rest of 40% = (C + G), and as the
135. Mucosa is the innermost layer lining the produced by the immune system of self
quantity of C = G, C can be calculated
lumen of the alimentary canal. It forms (patient) and is directed against 40
small finger-like folds called villi in the individual’s own proteins. as = 20%.
2
small intestine. It also forms crypts in In type I diabetes mellitus, pancreatic
154. Cosmozoic origin theory was proposed
between the bases of villi in the beta cells are destroyed by T-cells of
by Richter in 1865, who believed that the
intestine. These crypts are called patient’s own immune system.
protoplasm reached on earth from the
crypts of Lieberkuhn. Addison’s disease is an autoimmune other parts of universe in the form of
136. Aromatic amino acids contain aromatic disease that affects the function of the
spores, seeds, sperms (cosmozoa), etc.,
rings in their side chain, e.g. adrenal glands.
and various type of living beings formed
phenylalanine, tyrosine, tryptophan. 144. Inguinal canals are the two passages in later.
All other amino acids are basic amino the anterior abdominal wall connecting
scrotum to abdominal cavity.
155. The effects of noise pollution include
acids. These contain two amino groups hearing disability, sleeplessness, altered
and one carboxyl group per molecule, 145. The option (c) is incorrectly matched. breathing pattern and psychological
e.g. arginine, lysine, histidine. Adrenal medulla secretes hormones disorders. Sound level higher than 150
adrenaline and noradrenaline. dB or more may damage eardrums,
137. All living members of the class
It stimulates fight and flight reactions. resulting into hearing disability.
Cyclostomata are ectoparasites on
some fishes. Condrichthyes and 146. Endodermis generally lacks passage 156. In a few weeks, old embryo, primitive
Osteichthyes have cartilaginous and cells in dicot root. nucleated erythrocytes develop in yolk
bony endoskeletons, respectively. Most Passage cells generally occur in sac. In middle trimester, a good number
of amphibians have two pairs of limbs. endodermis opposite the xylem bundle of erythrocytes are produced in lymph
region in monocot root. nodes. In last month of gestation and
138. Richmond-Lang effect is also referred
thereafter, erythropoiesis occurs only in
to as ‘delay of senescence’. Cytokinin 147. Trypsinogen is activated by
enterokinase secreted by intestinal bone marrow.
is able to postpone, for a number of
days the disappearance of chlorophyll mucosa. 157. The information in herbaria is useful in
and degradation of proteins, which 148. The food prepared from the product of locating wild varieties and relatives of
occur with ageing process. genetically modified (transgenic) crops economically important plants. Therfore,
option (d) is incorrect.

MODULE 2
www.jeebooks.in
Prep Catalysis for NEET ~ Mock Test 6 225

158. All the statements are correct. 166. Reniform leaves are shaped like a 173. The transcellular streaming theory states
H H kidney (oval with an inward curve on that the sieve tubes contain tubular
½ ½ one side). Leaves shaped like lyre are strands, which are continuous from one
R ¾ C ¾ COO - - R ¾ C ¾ COO – + H+ called lyrate. These are pinnately tube cell to the other through the sieve
½ ½ lobed with a bigger terminal lobe and pores. These strands demonstrate a kind
+NH NH2 a smaller lateral lobe. Saggitate are of peristaltic movement, which helps in
3
Zwitter ion (Dipolar ion) arrow head-shaped with the lower the passage of organic substances.
[Proton donor] lobes folded, or curled downward. 174. Integrins are proteins that function
H H Cuneate leaves are triangular mechanically by attaching the cell
½ ½ wedge-shaped. cytoskeleton to the Extracellular Matrix
R ¾ C ¾ COO- + H+ - R ¾ C ¾ COOH
½ 167. Umbraculum is the modification of iris (ECM), and biochemically, by sensing
½
+NH +NH in gazelle and camel. whether adhesion has occurred.
3 3
Zwitter ion (Dipolar ion) It is a covering on the pupil to protect 175. Erythropoietin is produced by the juxta
[Proton acceptor] them from sand and excessive glare. glomerular cells of kidney. This hormone
168. Dynein is a family of cytoskeletal stimulates formation of RBCs, i.e
159. Severe Acute Respiratory Syndrome erythropoiesis. All other hormones are
(SARS) is a viral disease caused by motor proteins that move along
microtubules in cells. They drive the secreted by endocrine cells that are
Paramyxovirus of Coronavirus family.
beat of eukaryotic cilia and flagella. present in different parts of the
160. In the given figure, gastrointestinal tract.
169. C2 cycle is shown by C3 -plants.
A – Clavicle, B – Scapula, C – Radius 176. Genetic Engineering Approval Committee
Photorespiration (C2 cycle) involves a
D – Carpals, E – Metacarpals complex network of enzyme reactions (GEAC) is involved in validity of GM
that exchange metabolites between research and the safety of introducing
161. The gymnosperms and angiosperms
chloroplasts, peroxisomes and GM-organisms for public usage. It is an
resemble in the presence of ovule,
mitochondria. Photosynthetic Indian Government organisation.
pollen tube and production of seeds.
efficiency is reduced, about 25% of 177. In glycolysis, reaction given in option (d)
162. In plasma membrane, carbon fixed by photosynthesis is is catalysed by enzyme enolase. It is
receptor-mediated endocytosis released as CO 2 . The the second last step of glycolysis. In
requires clathrin coat proteins. photorespiration is also called
this step, two water molecules are
Coat-proteins like clathrin are used to C2 cycle since, the first stable
released.
product of the photorespiration
build small vesicles in order to
reaction is a 2-carbon compound 178. Norplant are capsules surgically
transport molecules within the cells.
called glycolate. implanted under the skin to produce
163. In alcoholics, liver gets damaged due 170. Wood is light in colour in springwood. hormones that inhibit ovulation.
to the accumulation of excessive fats. It is also called as early wood. It forms 179. The division Gnathostomata can be
The accumulation of fats in liver results plenty of xylem vessles with wider divided into two superclasses–Pisces and
in ‘fatty liver syndrome’ leading to cavities. Tetrapoda.
cirrhosis.
171. Fowl pox is a viral disease, which Superclass Tetrapoda is divided further
164. Centrosomes are not demonstrable in causes formation of greyish blisters into four classes
ovum since, the maternal centrosome
and scales in fowls.
is inactive. Amphibia, Reptilia, Aves and Mammalia.
172. Oxygen transport is reduced in acidic
165. Vitamin-K is essential for blood clotting 180. Culture media used in hydroponics
because it is necessary for the conditions. SO 2 makes the blood less
contains potassium nitrate (KNO 3 ) in the
synthesis of prothrombin in the liver. acidic compared to SO 3 , whereas CO
highest concentration of 1.02 g/L.
If vitamin-K is not sufficient in the body, and NO combine with haemoglobin
then blood clotting becomes thus reducing oxygen carrying
insufficient. capacity.

SCORE CHART
No. of Correct Answers : A .................. No. of Incorrect Answers : B ..................
Total Marks : x = (A × 4) – (B × 1)
Scores and Expected Rank : If the score lies above 665, then rank will be in between 1-50. For other scores, rank
estimations are given below Score Rank Score Rank
664 – 642 51 – 150 635 – 630 251 – 400
641 – 636 151 – 250 629 – 625 401 – 500

MODULE 2
www.jeebooks.in

PREP CATALYSIS
for NEET
Full Length Mock Tests for NEET to Make You Ready to Face the Challenge

MOCK TEST 7 (With Solutions)

Duration : 3 Hours Max. Mark : 720

Instructions
The test is of 3 hours duration and Test Booklet contains 180 questions. Each question carries 4 marks. For
each correct response, the candidate will get 4 marks. For each incorrect response, one mark will be deducted
from the total scores. The maximum marks are 720.
The question paper contains three parts of Physics, Chemistry and Biology respectively.
Part A contains 45 Questions from Physics section. Part B contains 45 Questions from Chemistry section.
Part C contains 90 Questions from Biology.

PART A PHYSICS
1. Two balls of masses M1 and M 2 are placed on a 3. Match the terms given in Column I with Column II and
horizontal frictionless table connected by a spring choose the correct option from the codes given below
as shown in figure, now mass M 2 is pulled to the Column I Column II
right with a force F. If the acceleration of mass M1
is a, then acceleration of mass M 2 will be A. Positive acceleration P.

M1 M2 F t
O
F F − M1a F B. Zero acceleration Q.
(a) (b) (c) (d) None of these
M1 M2 M2
2. Which one of the graphs represents the temperature
dependency of volume of water correctly? O t

C. Variation of acceleration R.
Volume (V)

Volume (V)

with time for free fall O


(a) 50°C (b) 50°C
Temperature (T )
Temperature (T ) t
D. Variation of velocity S.
Volume (V)

Volume (V)

with time for free fall


O t
(c) 50°C (d)
50°C
Temperature (T ) Temperature (T )

MODULE 2
www.jeebooks.in
Prep Catalysis for NEET ~ Mock Test 7 227

Codes 10. In the potentiometer experiment, electron in


A B C D A B C D the potentiometer wire experiences a force
(a) P Q R S (b) Q P S R 4.8 × 10−19 N. The length of potentiometer wire is
(c) Q P R S (d) P R S Q 4m. The emf of the battery connected across the
4. The moment of inertia of a uniform rod about a potentiometer wire is
perpendicular axis passing through one end is I1. [given, e = 1.6 × 10−19 C]
When, it is bend into a ring, the moment of inertia (a) 12 V (b) 9 V (c) 16 V (d) 6 V
I
about a diameter is I 2. Then, ratio of 1 is 11. Following diagram performs logic function of
I2
4π2 2 π2 Y
(a) (b)
3 3
π2 8π2 (a) AND gate (b) NAND gate
(c) (d)
3 3 (c) XOR gate (d) OR gate

5. The total energy of a particle executing SHM of 12. A piece of solid weighs 150 g in air, 120 g in water
period 2π seconds is 10.24 × 10−3 J. and 90 g in liquid. The relative density of the solid
π and that of liquid are respectively.
The displacement of the particle at s is 0.08 2 m. (a) 5, 2 (b) 3, 2
4
The mass of particle is (c) 4, 5 (d) 2, 5
(a) 0.08 kg (b) 0.8 kg 13. If a ball rolls ( u = 0) along a plane of 0.5 m of
(c) 0.16 kg (d) 1.6 kg
length in 4 second. The upper end of which is
6. A car moves at a constant speed on a straight but raised 0.02 m above the lower end, then the
hilly road. One section has a crest and dip of the acceleration due to gravity has value of
same 250 m radius. As the car passes over the (a) 9.871 ms −2 (b) 4.520 ms −2
crest, the normal force on the car is one-half the (c) 4.375 ms −2 (d) 9.970 ms −2
16 kN weight of the car. The normal force on the
car as it passes through bottom of the dip is 14. An electromagnetic wave is propagating in medium
with relative magnetic permeability 50 and
(given, g = 10 ms −2) relative dielectric constant 2. The wave impedance
(a) 24 × 103 N (b) 12 × 103 N of such medium is
(c) 32 × 10 N 3
(d) 16 × 103 N (a) 2000 Ω (b) 1883 Ω
(c) 1550 Ω (d) 1222 Ω
7. A charge of q µC is placed at the centre of the line
15. Two resistor of resistance ( 5 ± 0.25) Ω and
joining two exactly equal positive charge of 60 µC.
The system of the three charges will be in ( 7 ± 0.35) Ω are connected in parallel. The
equilibrium, if the value of charge q is percentage error in the equivalent resistance is
(a) 45 µC (b) 60 µC (a) 12% (b) 0.16%
(c) 15 µC (d) −15µC (c) 15% (d) 0.15%

8. The masses of neutron and proton are 1.0087 and 16. The speed of sound in a gas, in which two sound
1.0073 amu respectively. If neutrons and protons waves of wavelengths 1.00 m and 1.01 m produce
combine to form helium nucleus of mass 24 beats in 6 s is
4.0015 amu, the binding energy of the helium (a) 404 ms −1 (b) 402 ms −1
nucleus will be (c) 420 ms −1 (d) 400 ms −1
(a) 28.4 MeV (b) 20.8 MeV (c) 27.3 MeV (d) 14.2 MeV
17. At what point on the line joining two masses of
9. A simple harmonic wave train of amplitude 1 cm 500 kg and 2000 kg, will the resultant
and frequency 100 vibrations is travelling in gravitational field intensity is zero, if their
positive x-direction with velocity 15 ms −1. The separation is 2 m ?
displacement y, at x = 180 cm from the origin at (Take, G = 6.67 × 10−11 N-m 2 kg −2)
t = 5 s, is 2
(a) 1 m from 500 kg (b) m from 2000 kg
(a) zero (b) 2400 cm 3
(c) 1200 cm (d) 900 cm 2
(c) 1 m from 2000 kg (d) m from 500 kg
3

MODULE 2
www.jeebooks.in
228 NEET Test Drive

18. An α-particle is accelerated by V volt, experiences 26. When a conductor is connected to a battery of
a force F, when it enters in a uniform magnetic supply voltage of 220 V, then drift velocity of
field. When α-particle is accelerated by 3V volt, in electron across the conductor is 3.6 m/s. If the
same magnetic field, then force experienced by length of conductor is stretched to double and same
α-particle is potential difference is applied, then drift velocity of
(a) 3 F (b) 2 F electrons will be
F F (a) 2.4 m/s (b) 1.8 m/s
(c) (d)
3 2 (c) 4 m/s (d) 7.2 m/s

19. A block shown in figure slides on a semi-circular 27. Three bricks each of length L and mass M are
frictionless track. If it starts from rest at position arranged as shown in figure. The distance of the
A, what is its speed at the point B ? centre of mass of the system from the wall is
Wall
A
1m
45°
B

L L/2 L/4
(a) 3 ms −1 (b) 3.5 ms −1 L L
(a) (b)
(c) 3.76 ms −1 (d) 2.75 ms −1 2 4
11 5
20. If a capacitor having capacitance 2F and plate (c) L (d) L
12 6
separation of 0.5 cm will have area
(a) 1130 cm 2 (b) 1130 m 2 1
28. A heat engine has efficiency . Efficiency becomes
(c) 1130 km 2 (d) None of these 6
1
21. The average depth of Indian ocean is about 4 km. when temperature of sink is decreased by 100 K.
3
∆V
The fractional compression, of water at the What is the temperature of sink?
V (a) 68 K (b) 72.2 K
bottom of ocean is (given that the bulk modulus of
−2 −2 (c) 100.7 K (d) 55.55 K
the water = 2.2 × 10 Nm 9
and g = 10 ms )
(a) 1.36% (b) 1.81% 29. Which of the following transitions gives photon of
(c) 2.43% (d) 1.17% maximum energy?
(a) n = 2 to n = 1 (b) n = 1to n = 2
22. In a Carnot engine, sink is fitted at temperature (c) n = 2 to n = 6 (d) n = 6 to n = 2
37°C and heat of 1000 kcal is taken from source at
temperature 777°C, then work done in joule is 30. The power of a biconvex lens is 10 dioptre and the
(a) 5 × 106 (b) 8 × 106 (c) 3.8 × 106 (d) 2.9 × 106
radius of curvature of each surface is 10 cm. Then,
the refractive index of the material of the lens is
3 4
23. The electric potential at point ( x , y , z ) is given by (a) (b)
2 3
V = − xy 2 − x 2z + 5, then the value of electric field 9 5
(c) (d)
at point (1, 1, 1) is 8 3
(a) $i + 2 $j + k$ (b) 3i$ + 2 $j + k$
(c) 2 i$ + 3$j + k$ (d) 3$i − 2 $j + k$ 31. In a common emitter configuration of a transistor,
the voltage drop across a 700 Ω resistor in the
24. Assuming earth to be a perfect sphere of radius collector circuit is 0.7 V, when the collector supply
6400 km. The value of acceleration due to gravity voltage is 7V. If the current gain in the common
changes by what value from poles to equator for a base mode is 0.98, the base current is
period of 1 day? 1 1
(a) mA (b) mA
(a) 3.5 × 10−2 ms −2 (b) 3.38 × 10−2 ms −2 49 32
1 1
(c) 3 × 10−2 ms −2 (d) 2.25 × 10−2 ms −2 (c) µA (d) µA
49 39
25. A 220V input is supplied to a transformer. The
32. A magnetic needle lying parallel to a magnetic
output circuit draws a current of 2.0 A at 440 V. If
field requires W units of work to turn it through
efficiency of transformer is 80%, the current drawn
60°. What is the torque needed to maintain the
by the primary winding of the transformer is
needle in this position?
(a) 3.6 A (b) 2.8 A (c) 2.5 A (d) 5.0 A
(a) W (b) 2 W (c) 3 W (d) 2 W

MODULE 2
www.jeebooks.in
Prep Catalysis for NEET ~ Mock Test 7 229

33. If the effective value of acceleration due to gravity 40. In the given figure what will be the coefficient of
becomes zero of the equator, the length of a day mutual inductance?
becomes a
(radius of earth = 6400 km, g = 10 ms −2)
i
(a) 1.4 h (b) 1.5 h
(c) 1 h (d) 2 h

34. In Fraunhoffer’s diffraction pattern, slit is


illuminated by a light source of wavelength b
4200 Å . If slit is illuminated by another µ a µ 0a 
(a) 0 ln  1 +
a b
monochromatic light source, then angular width is  (b) ln  1 + 
2π  2b  π  2a
increased by 40%. Wavelength of new source is µ 0a  a µ 0a  b
(a) 5880 Å (b) 4200 Å (c) ln  1 +  (d) ln  1 + 
2π  b 2π  a
(c) 6000 Å (d) 5580 Å
41. A football having mass 100 g and radius 7.5 cm
35. A heat engine absorbs an amount of heat Q from a
and is initially at rest. A player hit it at a height h
reservoir at an absolute temperature T and rejects above the centre and it begins to roll without
T
heat to a sink at a temperature of . The amount slipping, then find h.
2 (a) 5 cm (b) 2.5 cm
of heat rejected is (c) 3 cm (d) 1.5 cm
Q Q 3Q
(a) Q (b) (c) (d)
2 4 2 42. An electron of mass m and charge q is travelling
36. The work functions of three metal A, B and C are with a speed v along a circular path of radius r at
1.92 eV, 2 eV and 5 eV respectively. Which metals right angle to a uniform magnetic field B. If speed
will emit photoelectrons for a radiation of of the electron is double and the magnetic field
wavelength 4100 Å? is halved, then resulting path would have a
(a) A only (b) B only
radius of
r r
(c) Both A and B (d) C only (a) (b)
4 2
37. A set of n identical resistors, each of resistance (c) 2r (d) 4r
R Ω, when connected in series have an effective
resistance of A Ω and when connected in parallel, 43. In a pure silicon ( n i = 1016 / m3 ) crystals at 300 K,
the effective resistance is B Ω, then the value of 1021 atoms of phosphorous are added per cubic
R is metre. The new hole concentration will be
A 2 − B2 A 2 + B2 (a) 105 per m 3 (b) 1011 per m 3
(a) AB (b) AB (c) (d) (c) 1019 per m 3 (d) 1021 per m 3
A2 B2
44. Two polaroids P1 and P2 are placed with their axis
38. A flywheel rotating at 840 rpm slows down at a perpendicular to each other. Unpolarised light of
−2
constant rate of 11 rad s . The time required to
intensity I 0 is incident on P1. A third polaroid P3 is
stop the flywheel is
kept between P1 and P2 , such that its axis makes
(a) 8 s (b) 5 s an angle 45° with that of P1. The ratio of intensity
(c) 2 s (d) 10 s of transmitted light to unpolarised light is
1 1
39. In the figure, a proton moves a distance d in a (a) (b)
uniform electric field E as shown in the figure. 4 2
1 1
Does the electric field do a positive or negative (c) (d)
work on the proton ? Does the electric potential 8 16
energy of the proton increase or decrease ? 45. The radioactivity of a sample is I1 at a time t1 and
E
I 2 at a time t2. If the half-life of the sample is τ1/ 2,
then the number of nuclei that have disintegrated
P in the time t2 − t1 is proportional to
d
(a) I1 t 2 − l2 t 1 (b) l1 − l2
(a) Negative, increase (b) Positive, decrease l −l
(c) 1 2 (d) (l1 − l2 )τ1/ 2
(c) Negative, decrease (d) Positive, increase τ1/ 2

MODULE 2
www.jeebooks.in

PART B CHEMISTRY
46. The radius of hydrogen atom in the ground state is 52. The solubility product of a salt having general
0.53 A° .The radius of Li2+ ion (atomic number = 3) formula MX 2 in water is 32 ´ 10-15 .
in a similar state is
The concentration of M 2+ ions in the aqueous
° °
(a) 017
. A (b) 0.265 A solution of the salt is
(c) 0.53 A° (d) 1.06 A° (a) 2 ´ 10-5 M (b) 16 ´ 10-15 M
-15
(c) 8 ´ 10 M (d) 0.8 ´ 10-5 M
47. The correct order of decreasing second ionisation
enthalpy of Ti, V, Cr and Mn is 53. Consider the following statements :
(a) Cr > Mn > V > Ti (b) Ti > V > Cr > Mn I. Alkali metal hydrides are less reactive than
(c) V > Mn > Cr > Ti (d) Mn > Cr > Ti > V alkaline earth metal hydrides.
48. Select correct statements and choose the correct II. Alkali metals have more reducing power than
code out of the following. alkaline earth metal.
I. CH3 ¾ C ¾ CH3 angle in (CH3 )C ==CH 2 is III. Alkali metals are more basic than alkaline
smaller and the CH3 ¾ C== CH 2 angle is earth metals.
larger than trigonal (120°). IV. Alkali metals have lower ionisation energy
II. O ¾ I ¾ F angle is less than 90° ( 89° ) in IOF4- . than alkaline earth metal.
III. In SeOCl2 , Cl ¾ Se ¾ Cl angle is less than the Choose the correct statement(s) and select the
Cl ¾ Se ¾ O angle. correct option.
(a) I and II (b) Only II (c) I, II III (d) II, III and IV
IV. POCl3 is tetrahedral with a double bond
between P and O, there is no lone pair on 54. For the reaction,
central atom. 1
SO2( g) + O 2( g ) =SO ( g), the K /K
3 p C is equal to
(a) I and II are correct. (b) II and III are correct 2
(c) I, II and III are correct (d) All are correct 1 1
(a) (b) RT (c) (d) 1
RT RT
49. If Z is a compressibility factor, van der Waals’
equation at low pressure can be written as 55. During electrophilic substitution reaction of
RT a fullerene, the hybridisation of carbon atom
(a) Z = 1 + (b) Z = 1 -
pb VRTn (a) remains sp2 in reactant as well as product
pb pb (b) changes from sp2 to sp3 from reactant to product
(c) Z = 1 - (d) Z = 1 +
RT RT (c) changes from sp3 to sp2 from reactant to product
50. The free energy change for the following reactions (d) cannot be determined
are given 56. The IUPAC name of the compound
5
C2H 2( g) + O2( g) ¾® 2CO2( g) + H 2O ( l ) ; CH3 ¾ CH 2 ¾ C ¾ CH 2CH3 is
2 ||
DG° = -1234kJ N ¾ OH
C ( s) + O2( g) ¾® CO2( g) ; DG° = - 394 kJ (a) N-hydroxy - 3- aminopentane (b) N-hydroxy aminopentane
1 (c) N-hydroxy -3- imino pentane (d) None of these
H 2( g) + O2( g) ¾® H 2O( l ) ; DG° = - 237 kJ
2 57. Which of the following statement is incorrect for
Calculate the standard free energy change for the chemisorption?
reaction? (a) It is irreversible
H 2( g) + 2 C ( s) ¾® C2H 2 ( g) (b) Enthalpy of adsorption is high
(c) Low temperature favours adsorption
(a) -2259 kJ (b) +209 kJ
(d) It is highly specific in nature
(c) +2259 kJ (d) -209 kJ
58. Which of the following order is correct regarding
51. If 10 moles of an ideal gas expands reversibly and the acidity of carboxylic group?
isothermally from 10 L to 100 L at 300 K, then
(a) CH3CH2CH(Cl) COOH > CH3CH(Cl)CH2COOH
entropy change will be
-1 -1 -1 > ClCH2CH2CH2COOH
(a) -19147
. JK mol (b) 191.24 JK
(b) CH3CH2CH(Cl) COOH < CH3CH(Cl)CH2COOH
(c) 83.03 JK -1 (d) 83.03 JK -1 mol -1
< ClCH2CH2CH2COOH

MODULE 2
www.jeebooks.in
Prep Catalysis for NEET ~ Mock Test 7 231

(c) CH3CH2CH(Cl) COOH > CH3CH(Cl)CH2COOH 68. Which of the following contain N¾ N bond?
< ClCH2CH2CH2COOH N 2O4 , N 2O5 , N 2O3
(d) CH3CH2CH(Cl) COOH < CH3CH(Cl)CH2COOH
(a) N2O 4 , N2O 3 and N2O 5 (b) N2O 5 and N2O 4
< ClCH2CH2CH2COOH
(c) N2O 3 and N2O 5 (d) N2O 3 and N2O 4
59. A greenhouse gas is a gas in an atmosphere that 69. For which of the following order of reaction, the
absorbs and emits radiation within the thermal
unit of rate constant and rate of reaction is same?
infrared range. This process is the fundamental
(a) Zero (b) First (c) Second (d) Third
cause of greenhouse effect. Among the following
which one is not a greenhouse gas? 70. Which of the following drug is used for the
(a) CO 2 (b) CH4 prevention of heart attack?
(c) O 2 (d) Vapour of water (a) Chloramphenicol (b) Valium
60. Duma’s method involves the determination of nitrogen (c) Aspirin (d) Cephalosporin
content in the organic compound in the form of 71. Which kind of isomerism is exhibited by octahedral
(a) NH3 (b) N2 (c) NaCN (d) (NH4 )2 SO 4 [Co(NH3 )4 Br2 ]Cl?
61. In the fluorite structure if the radius ratio is (a) Geometrical and ionisation (b) Geometrical and optical
æ 3 ö (c) Optical and ionisation (d) Only geometrical isomerism
ç - 1÷, then how many ions does each cation
è 2 ø 72. Propyl nitrite may be converted into butyl amine
touch? by using reduction.Which of the following reagent
(a) 4 anions (b) 12 anions (c) 8 anions (d) No cations can be used for this reduction?
(a) Sn / HCl (b) LiAlH4
62. 0.5 g of hydrocarbon gave 0.9 g water on (c) Conc. HCl (d) All of these
combustion. The percentage of carbon in the
hydrocarbon is 73. Suggest the suitable oxidising agent for the
(a) 60.6 (b) 28.8 (c) 80.0 (d) 68.6 following conversions :
I. CH3 CH2CH == CHCH2OH ¾® CH3 CH2CH == CHCHO
63. An aqueous solution of glucose is 20% in strength. The
volume in which 1 mole of it is dissolved will be
(a) 9 L (b) 1.8 L (c) 8 L (d) 0.9 L II.
+ OH O
64. If the limiting molar conductance of l 0(H )
2 -1 - 2
= 350 S cm equiv. and l 0(OH ) = 200 S cm
(a) MnO 2 in I and CrO 3 (in glacial acetic acid) in II
equiv. -1, then the equivalent conductance of water
(b) CrO 3 in I and MnO 2 in II
is
(c) CrO 3 in both
[Given, degree of dissociation of very pure water
(d) MnO 2 in both
= 1 .9 ´ 10-9]
(a) 1045
. ´ 10-10 S m 2 equiv -1 (b) 3.098 ´ 10-10 S m 2 equiv -1 74. The degree of dissociation of Ca(NO3 )2 in dilute
(c) 2 .085 ´ 10-8 S m 2 equiv -1 (d) 7.048 ´ 10-6 S m 2 equiv -1 aqueous solution, containing 7g of the salt per
100 g of water at 100° C is 70%. If the vapour
65. For the first order reaction, the ratio of t1/ 2 to t 1/3 pressure of water at 100° C is 760 mm Hg, the
for the amount of substance left will be vapour pressure of the solution is
(a) 0.520 (b) 0.413 (c) 0.328 (d) 0.631 (a) 746.3 mm Hg (b) 1492.6 mm Hg
(c) 373.2 mm Hg (d) 74.63 mm Hg
66. Which of the following statements is/are correct about
the electrolysis of Al2O3 by Hall-Heroult process? 75. NHCOCH3
(a) Al is obtained at the cathode and probably CO 2 at anode Br2(aq) H3O+
(b) Cryolite Na 3 [AlF6 ] lowers the melting point of Al 2O 3 and A B
increases its electrical conductivity
(c) Both (a) and (b) are correct B may be
(d) None of the above is correct (a) o-bromoaniline (b) p-bromoaniline
(c) mixture of (a) and (b) (d) None of these
67. The reaction rate for a reactant or product in a
particular reaction is defined as how fast or slow a 76. Which of the following aldehydes can be
reaction takes place. Effect of temperature on distinguish by Fehling’s solution?
reaction rate is given by (a) CH3CHO and C 6H5CHO
(a) Claisen-Clapeyron equation (b) Arrhenius equation O
||
(c) Gibbs Helmholtz equation (d) Kirchhoff’s equation (b) CH3CHO and CH3 CCH2OH

MODULE 2
www.jeebooks.in
232 NEET Test Drive

OH O 84. Which among the following has maximum number


| ||
(c) CH2 CH ¾C CH3 and HCHO of P—H bond?
(a) H4P2O 7 (b) H3PO 2 (c) H3PO 3 (d) H3PO 4
(d) CH3CHO and HCHO
85. What will be correct stereochemistry of product
77. The transition elements are more metallic than
obtained on reaction of but-2-ene with OsO4?
the representative elements. The reason behind
this is OH OH
(a) unpaired electron in metallic bonds
(b) the electrons in d-orbitals (a) (b)
(c) availability of d-orbitals for bonding OH OH
(d) electron pairs in d-orbitals
OH
78. [Ni(CN)4 ]2- and [Ni(CO)4 ] have
(a) sp3 -hybridised Ni in both cases (c) (d) None of these
OH
(b) sp3 and sp2d -hybridised Ni OH
(c) dsp2 and sp3 -hybridised Ni
(d) dsp2 in both cases 86. Among the following compounds, the one which
will be coloured in solid state is
79. The product X in the reaction given below is (a) Cu2Cl 2 (b) ZnF2
(c) Ag 2SO 4 (d) CuF2
H+ H2O
—C—CH3 + CN– C9H9NO X, X is 87. Which of the following is not correctly matched?
H2SO3
CN
(a) XeO3 ® Tetrahedral
(b) XeF4 ® Square planar
(a) —CH2—C—OCH3 (b) —CH—CH3 (c) XeF2 ® Linear
(d) XeO2F2 ® Square planar
COOH
88. The pH range at which a particular amino acid
(c) —C==CH2 (d) —CH==CH—COOH does not migrate under the influence of an electric
field is called
80. The coordination number of Pt in the complex ion (a) diffusion (b) yielding point
[Pt (en)2 Cl 2 ]2 + is (c) isoelectric point (d) eutectic point
(a) 3 (b) 4 (c) 5 (d) 6 89. Match Column I with Column II and identify
81. Major product of the following reaction is correct answer by using the code given below.
(i) HCN Column I Column I
(Electronegativity)
(ii) H3O
A. 1s 2 , 2 s 2 , 2 p5 1. 3.04
(a) (b)
H CN H CH2OH B. 1s2 ,2 s2 ,2 p4 2. 2.58
2 2 3
C. 1s ,2 s ,2 p 3. 3.98
H
(c) COOH (d) OH COOH 2 2 6 2 4
D. 1s ,2 s ,2 p ,3 s ,3 p 4. 3.44
82. Which of the following reaction will not produce Codes
an amine?
CH2—CN A B C D A B C D
LiAlH4 (b) CH3C==CH NaNH2 (a) 1 2 3 4 (b) 1 4 3 2
(a)
(c) 3 4 1 2 (d) 2 4 1 3
NO2
NH3 (d)
Sn/HCl 90. The acids are called hard acids if they tend to have
(c) 2CH3CH2—Cl the following characteristics such as small atomic
and ionic radii, high oxidation state and low
83. In the complex [Pt(O2 )(en)2(Br)]2+ , coordination polarisability. Which of the following order is
number and oxidation number of platinum are correct regarding acidic strength?
(a) 4, 3 (b) 4, 5 (a) Cl 2O > Cl 2O 3 > Cl 2O 7 (b) Cl 2O > Cl 2O 7 > Cl 2O 3
(c) 4, 6 (d) 6, 4 (c) Cl 2O < Cl 2O 3 < Cl 2O 7 (d) Cl 2O < Cl 2O 7 < Cl 2O 3

MODULE 2
www.jeebooks.in

PART C BIOLOGY
91. Which of the following is a larval canal system 98. Of the six names given below identify the diploid cells.
found in sponges? I. Nucellus II. Egg cell
(a) Rhagon (b) Leucon III. Male gamete IV. Antipodal cell
(c) Ascon (d) Sycon
V. Endosperm VI. Polar nuclei
92. Identify the correct statements with reference to Choose the correct option.
loop of Henle. (a) I, III and VI (b) I, IV and VI
I. It is also called as pars recta. (c) I and VI (d) All of these
II. It is bell-shaped.
99. Refer to the given figure showing a simplified
III. It can be divided into three regions. diagram of a part of a liver lobule. Identify the part
IV. It is the main site for urine concentration. labelled as A, B, C and D.
(a) I, II and III (b) I, II, III and IV B ile
(c) I, III and IV (d) II and IV canaliculus
B ranch of hepatic
93. Consider the given statements regarding muscle portal vein Fat storage
cell
protein.
I. Thick filaments constitute mainly of myosin
protein.
II. Myosin is splitted by an enzyme trypsin into
LMM and HMM. Hepatic
B ranch
III. ATPase requires Na + for its functioning. of hepatic sinusoids Central vein
artery Radial plates (branch of
IV. Subfragment S2 contains ATPase binding of hepatocytes hepatic vein)
sites. A B C D
(a) Hepatocyte Endothelial cell Kupffer cell Branch of
Which of the above statements is/are correct?
interlobular bile
(a) Only I (b) I and II duct
(c) II and III (d) Only IV (b) Kupffer cell Hepatocyte Endothelial Branch of
cell interlobular bile
94. The dominance of gametophyte is observed in duct
(a) bryophytes (b) pteridophytes (c) Branch of interlobular Endothelial cell Kupffer cell Hepatocyte
bile duct
(c) angiosperms (d) gymnosperms
(d) Endothelial cell Kupffer cell Branch of Hepatocyte
interlobular
95. Myasthenia gravis is a term that describes bile duct
(a) a notch lying in the concavity of left lung
(b) an autoimmune disorder affecting neuromuscular junction 100. Gartner’s duct in females is homologous to which
leading to paralysis of skeletal muscles male reproductive organ?
(c) change in the ear pressure as we move away from the (a) Wirsung duct (b) Wolffian duct
gravity (c) Stensen duct (d) Perihilar duct
(d) joints containing fibrocartilage
101. Match the diseases in Column I with their
96. Which of the following statements is true for the diagnostic tests in Column II and choose the
antibiotic neomycin? correct option from the codes given below.
(a) It resembles streptomycin but is highly toxic to kidneys and Column I Column II
ears (Disease) (Diagnostic test)
(b) It is antifungal
A. Breakbone fever 1. Weil-Felix test
(c) Sometimes can kill anthrax bacterium
(d) Always used with tetracycline B. Rocky mountain spotted fever 2. Wayson stain test

97. Which among the following statements is/are C. Black death 3. Schick test
correct regarding meristematic cells? D. Diphtheria 4. Tourniquet test
(a) These cells have the power of division
Codes
(b) Their cell wall is thin
A B C D A B C D
(c) Their nuclei are prominent
(a) 4 1 2 3 (b) 1 2 3 4
(d) All of the above
(c) 4 3 2 1 (d) 3 1 2 4

MODULE 2
www.jeebooks.in
234 NEET Test Drive

102. Fasting blood sugar level in a normal adult will be 112. Identify the pair which is correctly matched with
(a) 80-100 mg/100 mL of blood reference to ornithine cycle.
(b) 100-150 g/100 mL of blood (a) Carbamoyl phosphate – Arginine
(c) 50-70 mg/100 mL of blood (b) Citrulline – Argininosuccinic acid
(d) 80-100 g/100 mL of blood (c) Ornithine – Arginine
(d) Arginine – Fumaric acid
103. The valve that is present at the opening of inferior
vena cava is 113. Read the following statements and choose the
(a) Thebasian valve (b) Eustachian valve correct ones about the mechanism of hormonal
(c) Mitral valve (d) SA node control.
I. Protein hormones like epinephrine require
104. From what did the life originated on Earth membrane bound receptors to bind to specific
according to Oparin and Haldane? receptor sites.
(a) Thick red hot liquid
II. Thyroid hormone does not require any
(b) Snowball earth receptor for binding with its target site.
(c) Primordial soup
III. The hormones behave as a second messenger
(d) Earthshake due to Tsunami in the cytoplasm.
105. In which of the following techniques, the activated IV. Steroid hormones such as testosterone enter
charcoal is used to control gaseous pollutants? the cytoplasm of target cell to produce effect.
(a) Adsorption technique (b) Absorption technique Choose the correct option.
(c) Combustion technique (d) Incineration (a) Only I (b) Only II (c) I and IV (d) All of these

106. What is an operon? 114. Identify the incorrect matching regarding pyramid
(a) A gene sequence that regulates other genes of biomass.
(b) A protein binding subunit that enhances gene expression (a) Grassland — Upright (b) Forest — Upright
(c) A cluster of structural genes with related functions (c) Pond — Upright (d) None of these
(d) A protein that inhibits translation
115. Match the respiratory capacities and volumes and
107. According to distribution of yolk in the cytoplasm their values in a normal human adult. Choose the
of the ova, the ovum of a human female can be correctly matching pairs.
classified as Respiratory Values in normal
(a) homolecithal (b) telolecithal capacities/Volumes human adult
(c) centrolecithal (d) macrolecithal
1. Vital capacity 4800 mL
108. Which of the following statements explains the
2. Inspiratory capacity 3100 mL
teminism theory correctly?
(a) Retroviruses perform central dogma reverse with the help 3. Tidal volume 500 mL
of an enzyme, reverse transcriptase 4. Expiratory Reserve Volume 1700 mL
(b) RNA is translated into polypeptide chain, which folds into
an enzyme 5. Inspiratory Reserve Volume 2600 mL
(c) One amino acid can be coded by many codons (a) 1,2 and 3 (b) 1, 2, 3, 4 and 5
(d) Ribosomes may form group of 5-20, called polyribosomes (c) 2 and 3 (d) Only 5
109. The largest sesmoid bone found in the human 116. How is an eukaryotic genome different from a
body is bacterial genome?
(a) flexor carpi ulnaris (b) lenticular process of incus (a) In having enzyme reverse transcriptase
(c) patella (d) flexor hollucis brevis (b) In having start and stop codons
110. Which one of the following pituitary hormones does (c) Presence of introns
not stimulate the other endocrine glands? (d) Having thymine instead of uracil
(a) Thyrotropin (b) Gonadotropin 117. Which of the following is an atavistic character ?
(c) Somatotropin (d) Adrenocorticotropin
(a) Presence of tail (b) Dense body hairs
111. A combination of genetic and demographic factors (c) Monstral face (d) All of these
that drive population size downwards to extinction
118. During DNA replication, Okazaki fragments are
is known as
joined by which enzyme?
(a) mass extinction (b) anthropogenic extinction
(a) RNA polymerase I (b) DNA polymerase II
(c) extinction vortex (d) speciation
(c) DNA ligase (d) DNA polymerase III

MODULE 2
www.jeebooks.in
Prep Catalysis for NEET ~ Mock Test 7 235

119. Which among the following is incorrectly matched? Choose the correct option.
(a) Minamata — Hg (b) Cancer — Th (a) I and II (b) I, II and III (c) III and IV (d) Only IV
(c) Itai - itai — Cd (d) Black foot disease — Pb
131. What is the role of water (H 2 O) during
120. Puncturing of the diaphragm of a person would photosynthesis?
result in (a) It supplies electrons in the reactions of the light dependent
(a) sudden unconsciousness which would be revived after phase
some time (b) It absorbs the excess light energy
(b) stoppage of breathing and would be fatal (c) It transports hydrogen ions in Hill reaction
(c) diffusion of the residual volume of air (d) It provides oxygen for Hill reaction
(d) the intercostal muscles would collapse
132. Which of the following statements is incorrect with
121. The terrestrial ecosystems include respect to the plants showing photorespiration?
(a) forest (b) grassland (c) desert (d) All of these (a) There is enhancement of CO 2 assimilation in light and low
122. Milk is converted to the casein by the action of oxygen
which hormone given below? (b) Rate of CO 2 evolution in light is high
(a) Rennin (b) Amylase (c) CO 2 assimilation rate in intense light is about 50-70 mg
CO 2 / dm2 /hr
(c) Intestinal bacteria (d) Invertase
(d) CO 2 compensation point in light is between 15-35 mg
123. The outer layer of nuclear membrane is a part of CO 2 /dm2 /hr
(a) mitochondrial membrane (b) plasma membrane
(c) endoplasmic reticulum (d) nucleoplasm 133. ……… meristem is involved in the development of
cortex and pith in dicots.
124. 9 + 2 fibrillar arrangement is present in (a) Rib (b) Mass (c) Ground (d) Plate
(a) bacterial flagella (b) eukaryotic flagella
(c) T4 bacteriophage (d) bacterial fimbriae
134. The stone cells that possess suberised thick
sccondary walls are
125. The chief component of the middle lamella in plant (a) sclereids (b) aerenchyma
cells is (c) stellate parenchyma (d) angular collenchyma
(a) zinc (b) potassium (c) calcium (d) iron
135. The rolling mechanism of seed dispersal is found in
126. Identify the correct pair. (a) Amaranthus albus (b) Antirrhinum
(a) N, S, Mo – Inhibition of cell division (c) Taraxacum (d) Papaver
(b) N,K – Delay in flowering
(c) Ca, Cu – Yellowing of leaves
136. The pollution caused due to overusage of DDT
(d) Mn, Zn – Chlorosis
affects the birds by disturbing
(a) calcium metabolism (b) wings and feather formation
127. DNA bands of which colour are visible in an (c) flight muscles formation (d) ultrafiltration unit
ethidium bromide gel, when exposed to UV-light?
(a) Bright violet (b) Bright blue 137. Archegonia neck is solid in
(c) Bright orange (d) Bright green (a) gymnosperms (b) bryophytes
(c) pteridophytes (d) algae
128. An aerobic prokaryote without cell wall is
(a) mycoplasma (b) spirochaetes 138. During the life cycle of Plasmodium vivax, asexual
(c) archaebacteria (d) rickettsias phase occurs in
(a) man (b) female Anopheles mosquito
129. The nucleic acid hybridisation method of selection
(c) Both (a) and (b) (d) None of these
and screening of transformed cell is based on
(a) the fact that recombinants can be differentiated on the 139. Mesosomes are also known as
basis of their ability to produce colour (a) mitochondria of bacterial cell (b) chloroplast of bacterial cell
(b) detection by molecular probes (c) Golgi body of bacterial cell (d) nucleus of bacterial cell
(c) Western blotting method
(d) Northern blotting method 140. The NADH formed in glycolysis must be recycled
to regenerate NAD + , under anaerobic conditions.
130. The instruments used to measure osmotic pressure Yeast regenerates NAD + by
include (a) transferring electrons from NADH to pyruvate
I. Plasmolytic method by de Vries (d) transferring electrons from NADH to O 2 in mitochondrial
II. Pfeffer’s osmometer respiration
III. Cryoscopic osmometer (c) reducing pyruvate to ethanol and CO 2
IV. Cobalt chloride method (d) None of the above

MODULE 2
www.jeebooks.in
236 NEET Test Drive

141. Which of the following German biochemists was 149. Polyembryony is very common in
the first to show (1928) that tumours have a higher (a) gymnosperms (b) angiosperms
rate of glucose metabolism than other tissues? (c) bryophytes (d) pteriodophytes
(a) Otto Warburg (b) Severo Ochoa
(c) Fritz Lipmann (d) Andre Lwoff 150. PCR uses which of the following?
(a) Primase (b) RNA polymerase
142. The secondary structure of protein which exists in (c) Taq polymerase (d) Ligase
hair is
(a) α - helix (b) β - pleated sheet 151. Which of the following statements is not true?
(c) sulphur group (d) phosphodiester bond (a) Fragile X- syndrome is also known as Martin-Bell
syndrome, is the most common form of inherited mental
143. A plant based spermicidal cream ‘consap’ has been retardation
developed from which of the following plants? (b) Duplication of chromosomal segments has the potential to
(a) Prunus armeniaca amplify the number of copies of individual genes
(b) Sapindus mukrossi (c) In certain cases, multiple copies of genes code for rRNA,
(c) Prunus serotina such a DNA is called rDNA and the phenomenon is
(d) Coriandrum sativum referred to as gene redundancy
(d) All are true
144. The natural pollution is caused by
(a) volcanic eruptions (b) ozone 152. Identify the correct statements.
(c) dust storm (d) All of these I. Tiger salamander shows neoteny.
145. Which among the following statements is/are II. Frog aestivates during winter and hibernates
correct regarding J-shaped growth curve ? during summers.
(a) Minor fluctuations are not observed III. Bufo marinus is a poisonous amphibian.
(b) Environmental resistance does not operate IV. Male frogs croak louder than female frogs.
(c) Equilibrium phase is rarely reached Choose the correct option.
(d) All of the above (a) I and III (b) II, III and IV (c) I, II and IV (d) I, III and IV
146. Match the transgenic animals given in Column I 153. Which of the following statements is true?
with the genes inserted given in Column II and (a) Genetic characters are controlled by unit factors existing in
choose the correct option from the codes given pairs in individual organism
below. (b) When two unlike unit factors responsible for single
Column I Column II character are present in a single individual, one unit factor
(Transgenic animals ) (Gene inserted) is dominant to the other, which is said to be recessive
(c) During the formation of gametes, the paired unit factors
A. Sheep 1. 2 DNA coding lactoferrin separate or segregate randomly, so that each gamete
B. Cow 2. Breast cancer causing receives one or the other with equal likelihood
human gene (d) All are true

C. Mouse 3. Human growth hormone 154. The increase in the girth of palms is accomplished
gene by
D. Fish 4. Antihaemophilic factor IX (a) primary thickening meristems
(b) lateral meristems
Codes (c) intercalary meristems
A B C D A B C D (d) Both (b) and (c)
(a) 1 2 3 4 (b) 4 2 1 3
(c) 3 1 4 2 (d) 4 1 2 3 155. Match Column I with Column II and select the
correct option from the codes given below.
147. Which of the following statements is not true?
(a) Auxin is synthesised from amino acid tryptophan Column I Column II
(b) Methionine is the precursor of ethylene A. Photosynthetic protists 1. Myxomycetes
(c) Kinetin is the most prevalent natural cytokinin in plants
(d) Root apical meristem is the main site for cytokinin B. Saprophytic protists 2. Zooflagellata
synthesis C. Protozoan protists 3. Diatoms
148. Which of the following genes is referred to as the Codes
guardian of the genome? A B C A B C
(a) RB1 (b) Ras (a) 1 2 3 (b) 3 1 2
(c) p53 (d) map (c) 2 1 3 (d) 1 3 2
MODULE 2
www.jeebooks.in
Prep Catalysis for NEET ~ Mock Test 7 237

156. Reverse transcriptase Codes


(a) replicates host DNA A B C D A B C D
(b) translates bacterial DNA (a) 2 4 1 3 (b) 1 3 2 4
(c) transcribes viral RNA to cDNA (c) 3 2 4 1 (d) 4 1 3 2
(d) translates DNA in proteins
163. Which of the following is/are the first most
157. Which of the following conventions used in human common sign of pregnancy?
pedigree is not paired correctly ? I. Amenorrhoea
II. Quickening
(a) Female
III. Ballottement of foetus
IV. Foetal movement perceived by examiner
(b) Male Choose the correct option.
(a) II and III (b) I and IV
(c) Only I (d) I, II, III and IV

(c) Identical twins of same sex 164. Body shapes and proportions of endotherms vary
by climatic temperatures. This statement is in
accordance with
(a) Allen’s rule (b) Grinnel’s rule
(d) Parents (unrelated) (c) Hardy-Weinberg principle (d) Gause’s principle

165. Nitrogen-fixation genes can be introduced into


158. …… help in maintaining natural balance of non-legumes by which of the following methods?
organisms in intestine and promote a healthy (a) Haploid production by pollen culture
digestive system. (b) Embryo culture
(a) Antibiotics (b) Vitamins (c) Vaccines (d) Probiotics (c) Protoplast culture
(d) Meristem tip culture
159. Microsporogenesis is a process of the
(a) formation of pollen grains from microspore mother cells 166. What is cystolith composed of ?
(b) formation of archesporial initials from young anther (a) Calcium chloride (b) Calcium carbonate
(c) formation of microsporocytes from mass of sporogenous (c) Magnesium chloride (d) Calcium oxide
cells
(d) formation of microsporocytes from primary sporogenous 167. Which of the following statements is incorrect
cells regarding community?
(a) It is a group of single species in an area
160. Which among the following is not an adaptation of (b) It comprises of individuals that are different in morphology
halophytes? (c) It comprises of food chain and food webs
(a) Stomata are more in number (d) It involves competition among different species
(b) Water storage tissue is well-developed
(c) Presence of thick cuticle 168. Which of the following is not an artificial method of
(d) Presence of calcium oxalate crystals vegetative propagation?
(a) Cutting (b) Layering
161. Fructose is absorbed from the small intestine by (c) Fragmentation (d) Grafting
the process of
(a) osmosis (b) facilitative diffusion 169. SCID is a rare genetic disease that affects
(c) capillary action (d) hormones (a) nervous system (b) immune system
(c) excretory system (d) integumentary system
162. Match the following columns and choose the
correct option from the codes given below. 170. Match the organisms given in Column I with their
scientific names in Column II and choose the
Column I Column II correct option from the codes given below.
A. Haemopoiesis 1. Formation of RBCs
Column I Column II
B. Ossification 2. Production of blood cells
A. Neem 1. Bos taurus
and platelets
B. Panda 2. Fragaria
C. Erythropoiesis 3. Accumulation of calcium
C. Potato 3. Azadirachta indica
salts in tissues
D. Cow 4. Ailuropoda
D. Calcification 4. Bone formation
E. Strawberry 5. Solanum tuberosum

MODULE 2
www.jeebooks.in
238 NEET Test Drive

Codes
A B C D E A B C D E
(a) 3 4 5 1 2 (b) 4 1 2 5 3 (c) Monozygotic twins
(c) 1 2 3 4 5 (d) 2 5 4 3 1
171. γ- rays, β- rays UV-rays, etc., are agents capable of
(d) Parents (unrelated)
causing mutations. These are included in which
category of mutagens?
(a) Chemical mutagens (b) Airborne mutagens
177. Which of the following statements is not true with
(c) Physical mutagens (d) Waterborne mutagens
reference to neurotransmitters?
(a) Norepinephrine is inactivated by an enzyme monoamine
172. An ideal vector can be best defined as the one oxidase
possessing (b) Cholinergic fibres release acetylcholine
(a) selectable marker (b) an origin of replication (c) Glycine is a neurotransmitter
(c) restriction sites (d) All of these (d) GABA promotes post-synaptic regeneration of action potential

173. In split genes, the coding sequences are known as 178. Paramecium is heterokaryotic because of the
(a) cistrons (b) introns presence of
(c) exons (d) operator region (a) macronucleus and micronucleus
(b) trichocysts
174. Balanoglossus is the connecting link between (c) cytopharynx
which of the following? (d) cytoproct
(a) Annelida and Mollusca (b) Reptiles and mammals
(c) Protozoa and Porifera (d) Invertebrates and vertebrates 179. Which of the following fishes is a living fossil?
(a) Labeo rohita
175. Which of the following vaccines are inactivated (b) Latimeria chalumnae
toxins?
(c) Chimaera
(a) Attenuated (b) Conjugated (c) Toxoids (d) Subunit
(d) Remora
176. Which of the following conventions used in human
pedigrees is not paired correctly? 180. A series of proteases responsible for initiating
apoptosis (programmed cell death) and for
(a) Sex unknown digesting intracellular components is
(a) caspases (b) cyclins
(b) Deceased individual (Female) (c) CDKs (d) map kinase

Answer Sheet
1. (b) 2. (c) 3. (c) 4. (d) 5. (b) 6. (a) 7. (d) 8. (a) 9. (a) 10. (a)
11. (a) 12. (a) 13. (c) 14. (b) 15. (c) 16. (a) 17. (d) 18. (a) 19. (c) 20. (c)
21. (b) 22. (d) 23. (b) 24. (b) 25. (d) 26. (b) 27. (c) 28. (d) 29. (a) 30. (a)
31. (a) 32. (c) 33. (a) 34. (a) 35. (b) 36. (c) 37. (a) 38. (a) 39. (a) 40. (c)
41. (a) 42. (d) 43. (b) 44. (c) 45. (d) 46. (a) 47. (a) 48. (d) 49. (b) 50. (b)
51. (a) 52 (a) 53. (d) 54 (a) 55 (b) 56. (c) 57. (c) 58. (a) 59. (c) 60. (b)
61. (c) 62. (c) 63. (d) 64. (a) 65. (d) 66. (c) 67. (b) 68. (d) 69. (a) 70. (c)
71. (a) 72. (b) 73. (a) 74. (a) 75. (c) 76. (a) 77. (c) 78. (c) 79. (c) 80. (d)
81. (d) 82. (b) 83. (d) 84. (b) 85. (b) 86. (d) 87. (d) 88. (c) 89. (c) 90. (c)
91. (a) 92. (c) 93. (b) 94. (a) 95. (b) 96. (a) 97. (d) 98. (c) 99. (a) 100. (b)

101. (a) 102. (a) 103. (b) 104. (c) 105. (a) 106. (c) 107. (a) 108. (a) 109. (c) 110. (c)
111. (c) 112. (b) 113. (c) 114. (c) 115. (b) 116. (c) 117. (d) 118. (c) 119. (d) 120. (b)
121. (d) 122. (a) 123. (c) 124. (b) 125. (c) 126. (d) 127. (c) 128. (a) 129. (b) 130. (b)
131. (a) 132. (c) 133. (c) 134. (a) 135. (a) 136. (a) 137. (a) 138. (a) 139. (a) 140. (c)
141. (a) 142. (a) 143. (b) 144. (d) 145. (d) 146. (d) 147. (c) 148. (c) 149. (a) 150. (c)
151. (d) 152. (d) 153. (d) 154. (a) 155. (b) 156. (c) 157. (d) 158. (d) 159. (a) 160. (a)
161. (b) 162. (a) 163. (c) 164. (a) 165. (c) 166. (b) 167. (a) 168. (c) 169. (b) 170. (a)
171. (c) 172. (d) 173. (c) 174. (d) 175. (c) 176. (c) 177. (d) 178. (a) 179. (b) 180. (a)

MODULE 2
www.jeebooks.in
EXPLANATIONS

PHYSICS
1. Force acting on mass M1 is f = M1a. L 2pn ( frequncy)
As, R = = A sin (vt - x )
Mass M1 will pull mass M 2 towards left 2p v
with a force f. ML2 Given,A = 1 cm, n = 100 Hz, v = 15 ms -1
\ I2 =
8p2 = 1500 cms -1, x = 180 cm, t = 5s, then
Net force acting on mass M 2 = F - f 2 p ´ 100
= F - M1a I (1 / 3) ML 8p2 2 y = 1 sin (1500 ´ 5 - 180)
Then, 1 = 2 2
= 1500
F - M1a I 2 ML / 8 p 3 2p
Acceleration of mass M 2 = = sin ´ 7320 = sin 2 p ´ 488 = 0
M2 2p 15
5. As, y = A sin wt = A sin t
1 T V
2. We know that, volume µ 2p p p 10. Force on electron, F = qE = q.
density 0.08 2 = A sin ´ = A sin l
2p 4 4 Fl 4.8 ´ 10-19 ´ 4
and water has minimum volume at 4° C V = = = 12 V
Þ A = 0.08 ´ 2 = 016
. m q . ´ 10-19
16
for a fixed mass. 2
2p ö 2
mw 2 A 2 = m æç
1 1
3. In positive time graph, upward motion Total energy = ÷ A 11. Truth table of given combination is,
is for positive acceleration and straight 2 2 èT ø
2
A B Y1 Y
line for zero acceleration as shown in 2p ö
´ m ´ æç
1
10.24 ´ 10-3 = . )2
÷ ´ (016 0 0 1 0
figure below. 2 è 2p ø 0 1 1 0
x 10.24 ´ 10-3 ´ 2 1 0 1 0
Þm = = 0.8 kg 1 1 0 1
+ve acceleration . )2
(016
A Y1
mv 2 Y
6. Here, mg - N = , B
r The diagram performs logic function of
t (16000 - 8000) 250
O For crest, v 2 = AND gate.
1600 12. Relative density of solid
x v = 35.35 ms -1 Weight in air
a=0 =
For dip, Weight in air - Weight in water
mv 2 mv 2 150 150
N ¢ - mg = ÞN ¢ = mg + = = =5
r r 150 - 120 30
1600 (35.35) 2 Relative density of liquid
t N ¢ = 16000 +
O 250 Weight in air - Weight in liquid
=
= 24000 N = 24 ´ 103 N Weight in air - Weight in water
As the body is moving under free fall,
thus acceleration is constant, i.e. 7. For equilibrium, net force on charge A 150 - 90 60
= = =2
a = - 9. Hence, it is a straight line must be zero, 150 - 120 30
parallel to time axis as shown. 60µC q 60µC 1 2
13. s = ut + at
a x x 2
A 2 2 B 1
. = 0´t +
05 ´ a ´ (4) 2
O t -6 2
1 60 ´ 10 .q
i.e. × . ´2
05 1
4 pe0 æx ö
2
Þa = = ms -2 .
ç ÷ 16 16
–g è2ø g sin q g sin q
But, a = =
1 (60 ´ 10-6 ) 2 I 2
Under free fall, v (t ) = - 9.8t . Thus, it is + . =0 1+ MR 2
straight line having negative slope as 4 pe0 x2 MR 2 1+ 5
shown below 4q = - 60 ´ 10-6 MR 2
5 1 5
v q = - 15 ´ 10-6 = g sin q Þ = ´ g ´ 0.02
7 16 7
q = - 15 mC 7
Þ g = = 4.375 ms -2
O t 8. Mass defect, Dm = 2(m n + m p) - mHe 5 ´ 16 ´ 0.02
(as helium nucleus has two-protons 14. Given, relative magnetic permeability = 50
and two-neutrons) Dielectric constant = 2
Dm = 2 (10087
. + 10073
. ) - 4.0015 The impedance is given as,
4. Moment of inertia of rod about one end
1 4.032 - 4.0015 = 0.0305 mr m 0 æm ö æm ö
is I1 = ML2 Z = = ç r÷ ç 0÷
3 Binding energy, E B = Dm ´ 931MeV er . e0 è er ø è e0 ø
Moment of inertia of this rod, when bent = 0.0305 ´ 931 = 28.4 MeV 50
into a ring 9. The displacement of wave is given by = ´ 376.6 W = 1883 W
2
MR 2 2p
I2 = y = A sin (vt - x )
2 l
MODULE 2
www.jeebooks.in
240 NEET Test Drive

15. Given, R1 = (5 ± 0.25) W, 20. QCapacitance of a parallel plate Efficiency of the transformer is given by
e A Output power VsIs
R 2 = (7 ± 0.35)W capacitor, C = 0 h= =
d Input power VpI p
The equivalent resistance in parallel
combination is, Cd . ´ 10-2
2 ´ 05 VsIs (440) (2)
A= = or Ip = = = 5A
Rp =
R1R 2
and e0 8.85 ´ 10-12 hVp æ 80 ö (220)
ç ÷
R1 + R 2 è 100 ø
= 1130
. ´ 109 m2 = 1130 km 2
R1 + R 2 = RDR = (12 ± 0.60) W 26. When the conductor is stretched to
P
DR p DR1 DR 2 DR 21. We know that, B = double, then its resistance becomes 4
= + + DV / V
Rp R1 R2 R times and area becomes half
DV p r ´g ´h
So, = = I = neAvd
0.25 0.35 0.60 V B B
= + + = 015
. V
5 7 12 DV 1000 ´ 10 ´ 4000 = neAvd
DR p ´ 100 = ´ 100 R
% error = ´ 100 V 2.2 ´ 109
Rp V 1
= 181
. % vd = Þ vd µ
neAR AR
= 015
. ´ 100 = 15% T2 310 74
22. h = 1 - = 1- = A1
. 4R1
16. If v be the speed of sound in gas, then T1 1050 105 vd 1 AR
v v v v = 2 2 = 2 =2
n1 = = and n 2 = = 74 3 vd 2 A1R1 A1R1
andW = hq1 = 1000 ´ 10 ´ ´ 4.2J
l1 100
. l 2 1. 01 105 vd 1
3.6
24 6
= 2.96 ´ 10 J vd 2 = = 18 =
. m/s
Beat frequency = n1 - n 2 = = 4 s -1. 2 2
6 23. V = - xy 2 - x 2 z + 5 L L L
27. x1 = , x 2 = + = L,
Þ v æç
1 1 ö
- ÷=4 ¶V 2 2 2
è 100
. . ø
101 Electric field, E = -
¶r
v ´ 0.01 ¶V $ ¶V $ ¶V $ Wall
Þ =4 =- i - j - k
101
. ¶x ¶y ¶z
Þ v = 404 ms -1 ¶ ¶
=- (- xy 2 - x 2 z + 5) $i - L/4
17. Let x be the distance from 500 kg ¶x ¶y L/2
mass, then L/4
(- xy 2 - x 2 z + 5) $j
G ´ 500 G ´ 2000 L/2
= ¶
x2 (2 - x ) 2 - (- xy 2 - x 2 z + 5) k$ L
¶z x1
Þ 2
(2 - x ) = 4 ´ x 2
= (y 2 + 2 xz ) $i + 2 xy $j + x 2 k$ x2
x3
Þ 2 - x = 2x At (1, 1, 1) = (1 + 2) $i + 2 $j + (1) 2 k$
2 L L L 5L
Þ x = m (from 500 kg) = 3 $i + 2 $j + k$ x3 = + + =
3 2 4 2 4
24. Given, R = 6400 km = 6.4 ´ 106 m
18. Work done = Kinetic energy Centre of mass of system,
T = 1day = 24 ´ 60 ´ 60 = 8.64 ´ 104 s
1 2qV m1x1 + m 2 x 2 + m3 x 3
qV = mv 2 Þ v = The value of g with latitude is X cm =
2 m m1 + m1 + m 3
g ¢ = g - Rw 2 cos 2 l
Force experienced by a-particle, At poles, l = 90° L 5L
2qV M + ML + M
F = Bqv = Bq. Þ cos l = 0, then 2 4
m =
g p = g - Rw 2 (0) = g …(i) M +M +M
F2 V2
F µ V Þ = = 3 At equator, l = 0° 11 / 4ML
F1 V1 =
Þ cos l = 1, then 3M
F2 = 3F ge = g - Rw 2 …(ii) =
11
L
The change in value of g is 12
19. Total energy at point A
2
= Total energy at point B 2p ö 1 T
g p - ge = Rw 2 = 6.4 ´ 106 ´ æç ÷ 28. = 1- L …(i)
1 æ 1 ö èT ø 6 TH
0 + mgR = mvB2 + mgR ç1 - ÷
2 è 2ø 2 1 T - 100
æ 2 ´ 3.14 ö = 1- L …(ii)
A = 6.4 ´ 106 ´ ç 4
÷ 3 TH
R è 8.64 ´ 10 ø 6TL
45° From Eq. (i),TH = …(iii)
= 3.38 ´ 10-2 ms -2 5
B 25. Given, (Input voltage)Vp = 220 V and from Eqs. (ii) and (iii), we get
(Output voltage)Vs = 440 V 2 5T - 500
- = L
1 3 6TL
R 1– (Input current) I p = ?
√2 1500
(Output current) Is = 2A So, TL = = 55.55 K
vB2 = 2gR = 3.76 ms -1(QR = 1m, given) Efficiency of the transformer, h = 80% 27

MODULE 2
www.jeebooks.in
Prep Catalysis for NEET ~ Mock Test 7 241

29. Energy level of H atom is given by g ¢ = g - Rw 2 cos 2 l = 0 As, proton is moving in electric field from
2
-13.6 Z -13.6 Þ g - Rw 2 = 0 low potential region to high potential
En = eV = eV region. Hence, its potential energy
n2 n2 or g = Rw 2
increases.
Photons are emitted only when electron g 10
jumps from higher energy level to lower And w = = 40. Magnetic field due to wire,
R 6400 ´ 103 m0
energy level. B = .i
Hence, option (b) and (c) are wrong. . ´ 10-3 rad s -1
= 125 2 px
Time period, dA = adx
DE 2 ®1 = 13.6 æç 2 - 2 ö÷ eV
1 1
è1 2 ø 2p 2 ´ 3.14 a
T = = = 5024 s = 14
. h
= 10.2 eV w . ´ 10-3
125 i
DE 6 ® 2 = 13.6 æç 2 - 2 ö÷
1 1 34. Angular width, w = 2 q
è2 6 ø l
w=2
= 3.02 eV d
Hence, DE 2 ®1 is maximum. wd
l= x
30. Power of lens, 2
l1 w w1
P (in dioptre) = 1= [w 2 = 14
. w1] b dx
100 l2 w 2 14
. w1
=
Focal length f (in cm) l1 1
= and df = B × dA
100 l 2 14
.
f = = 10 cm On integrating both sides, we get
10 l 2 = 14
. l1 = 14
. ´ 4200 = 5880 Å
According to lens maker’s formula,
DW Q T
ò df = ò BdAa + b
35. h = = 1 - rejectd = 1 - sink m i a × dx
1 æ1 1ö f= 0 ò
= (m - 1) ç - ÷ DQ Q taken T source 2p b x
f è R1 R 2 ø T /2 Q a+b
Qrejected = Q = m 0i
For biconvex lens, R1 = + R , R 2 = - R T 2 f= a ln x
2p b
= (m - 1) æç + ö÷ hc 6.62 ´ 10-34 ´ 3 ´ 108
1 1 1
36. W0 = = m 0i
f èR Rø = a [ln(a + b ) - lnb ]
l 4100 ´ 10-10 2p
= (m - 1) æç ö÷
1 2
= 4.8 ´ 10-19 J = 3 eV m i é a ù
= 0 a êlnæç1 + ö÷ ú = Mi
f èR ø
WA = 192
. eV 2p ë è b øû
= (m - 1) æç ö÷
1 2
WB = 2 eV where, M = coefficient of mutual
10 è 10 ø
WC = 5 eV inductance
1 3 m 0a æ
(m - 1) = or m = Since,WA < W0 M = lnç1 + ö÷
a
2 2 2p è bø
07
. WB < W0
31. Collector current, IC = = 1 mA Hence, A and B will emit photoelectrons. 41. Due to force F ,
700
IC 1 50 37. When n resistors are connected in Linear impulse, I = ò Fdt = Change in
Emitter current = = mA = mA linear momentum
Ia 0.98 49 series, then
A = nR …(i) ω
Therefore, base current F
50 1 When n resistors are connected in h
IB = IE - IC = - 1= mA parallel, then
49 49 O
R
32. The work done B = …(ii)
n
W = MB (cos q1 - cos q 2 ) From Eqs. (i) and (ii), we get I = mv - 0 = mrw …(i)
Here, q1 = 0° and q 2 = q = 60° AB = R 2 and angular impulse, J = Ih = change in
Þ W = MB (1 - cos q) Þ R = AB angular momentum
q 840 = Iw - 0 = Iw
= 2 MB sin2 …(i) 38. v 0 = 840 rpm = rps = 14 rps
2
Ih = æç mr 2 ö÷ w
60 2
22 or …(ii)
Torque, t = MB sin q = 2MB sin q w 0 = 2 pv 0 = 2 ´ ´ 14 = 88 rad s -1 è3 ø
q q 7 2
sin cos …(iii) From Eqs. (i) and (ii), we get [I = mr 2 for
2 2 w = 0, a = - 11rad s -2 3
From Eqs. (i) and (ii), we get w - w0 0 - 88 hollow sphere]
t = = = 8s
W q a -11 (2 / 3) mr 2 w 2 2
= tan = tan 30° h= = r = ´ 7.5 = 5 cm
t 2 39. Since, the proton is moving against the mrw 3 3
Þ t = 3W direction of electric field, so work is
42. In a perpendicular magnetic field,
done by the proton against electric
33. At equator latitude, l = 0°, then Magnetic force = Centripetal force
field and hence work done by the field
cos l = 1 mv 2
is negative. i.e., Bqv =
So, according to question, r

MODULE 2
www.jeebooks.in
242 NEET Test Drive

mv 44. Intensity of transmitted light from I0


Þ r = I2 =
Bq I 8
polaroid P1, I1 = 0
r1 v1 B 2 2 I2 1
Þ = ´ \ =
r2 v 2 B1 Intensity of transmitted light from I0 8
polaroid P3 , 0.693
r1 1 1 45. Half life, t1/ 2 =
= ´ I 3 = I1 cos 2 45°
r2 2 2 l
I 1 I l1 = lN1 and l 2 = lN 2
Þ r 2 = 4r [Qr1 = r ] = 0 ´ , I3 = 0 Activity
2 2 4
l ® disintegration constant
43. Using mass action law, ni2 = nenh Intensity of transmitted light from 0.693
ni2 polaroid P2 , l1 - l 2 = (N1 - N 2 )
nh = t 1/ 2
ne I 1
I 2 = I 3 cos 2 45° = 0 ×
4 2 Þ (N1 - N 2 ) µ (l1 - l 2 )t1/ 2
(1016 ) 2
= = 1011 per m 3
1021

CHEMISTRY
n 2a0
46. rn =
Z 49. At low pressure, van der Waals’,
r1 (H) =
a0
= a0 = 053
. A
° equation becomes 54. As we know that, K p = KC (RT ) Dn g
1 a
Z = 1- For the reaction,
a 053
. ° VRT n
r1 (Li 2 + ) = 0 = = 017
. A 1
SO 2 (g) + O 2 (g) =
SO 3 (g );
3 3 5 2
50. C2H2 (g) + O 2 (g) ¾® 2CO 2 (g )
47. Among Ti, V, Cr and Mn generally 2 3 1
+ H2O(l ) Dn g = 1 - = -
second ionisation energy increases 2 2
DG° = - 1234 kJ …(i)
with increase in atomic number but
C + O 2 ¾® CO 2 ; \ K p = KC (RT ) -1/ 2
second ionisation energy of Cr
(3d 5 , 4s 1) is greater than that of Mn due DG° = - 394 kJ …(ii) KC K 1
Kp = \ p =
to the presence of exactly half-filled 1 RT KC RT
H2 + O 2 ¾® H2O;
d-subshell in Cr. Thus, the order of 2 55. This problem includes concept of
second ionisation enthalpy is
DG° = - 237 kJ …(iii) allotropic form of carbon (fullerene) and
Cr > Mn > V > Ti
Eq. (ii) ´ 2 + Eq. (iii) - Eq. (i) electrophilic substitution reaction of
48. All the given statements are true. DG° = 2 (-394) + (-237) - (-1234) fullerene.
CH2 When fullerene undergoes electrophilic
DG° = 209 kJ
C , θ <120° substitution, hybridisation of carbon
51. Given,V2 = 100 L,
changes from sp 2 to sp 3 from reactant to
CH3 CH3 V1 = 10 L product.
æV ö | |
This is due to larger space occupied by DS = - 2.303nR logç 2 ÷
p electron cloud which pushes both è V1 ø 56. — C — N group is called amino while
C ¾ CH3 to reduce CH3 — C ¾ CH3 | |
= - 2.303 nR logæç
100 ö
bond angle. ÷ — C == N group is called imino.
è 10 ø
● O ¾I ¾F bond angle is also less -1 -1
| |
= - 2.303 ´ 10 ´ 8.314 JK mol
than 90° in IOF4- due to lone pair on The nomenclature of the compounds is
I-atom. . JK -1 mol -1
= - 19147
CH3 — CH2 — C ¾ CH2 ¾ CH3
● SeOCl 2 has Cl ¾ Se ¾ Cl bond 52. MX 2 = M 2+ + 2X - ||
angle less than Cl ¾ Se ¾O due to N ¾OH
K sp = [M 2 + ] [X - ] 2
p-electron. N-hydroxy-3-iminopentane
\ K sp = (S) (2S) 2 = 4S 3
● POCl 3 has tetrahedral structure due 57. Chemisorption is a kind of adsorption
to presence of p-bond. 4S 3 = 32 ´ 10-15
which involves a chemical reaction
O Thus, S = 3 8 ´ 10-15 = 2 ´ 10-5 M between the surface and the adsorbate.
P Þ [M 2 + ] = 2 ´ 10-5 M New chemical bonds are generated at
Cl the adsorbent surface.
Cl 53. (I) is incorrect because alkali metal
Cl hydrides have high reactivity than With increase in temperature
alkaline earth metal. chemisorption first increase as sufficient
energy is being provided for the

MODULE 2
www.jeebooks.in
Prep Catalysis for NEET ~ Mock Test 7 243

molecules to reach the activation 65. For first order reaction, NH3 +
Br +

energy. But after certain degree it


log æç
2 .303 1 ö 2 .303
decreases, as the further high
t1 = ÷= log 2 …(i) H3N Br H3N NH3
k è1 / 2 ø k Co Co
2
temperature helps in breaking of the
log æç
bond between the adsorbate and 2 .303 1 ö 2 .303 H3N Br H 3N NH3
t1 = ÷= log 3…(ii)
adsorbed molecules. k è1 / 3 ø k NH3 Br
3
cis trans
58. Presence of -I-showing group like Cl On dividing Eq. (i) by Eq. (ii), we get
LiAlH4
increases the acidic character of t1/ 2 log 2 0.3010 72. CH3 CH2 ¾ C ºº N ¾ ¾ ¾
¾®
= = = 0.631 Propyl nitrite
carboxylic acids and the acidity reduces t1/ 3 log 3 0.4771 CH3 CH2 CH2NH2
with increase in the distance between Butylamine
¾ COOH and -I-showing group. 66. In the electrolysis of Al 2O 3 by
Hall-Heroult process, following reaction 73. MnO 2 is used to convert alcohol into
59. O 2 is used in respiration. High
occurs : aldehyde whereas CrO 3 (in glacial
concentration of O 2 means good
Oxidation at anode acetic acid) in B used to convert cyclic
atmosphere. It does not causes
alcohol to ketone.
greenhouse effect hence, O 2 is not a [C + 2O 2 - ¾® CO 2 + 4e - ] ´ 3 MnO
greenhouse gas. CH3 CH2 CH == CHCH2OH ¾ ¾¾
2
®
Reduction at cathode
60. Duma’s method involves the CH3 CH2 CH == CHCHO
[Al 3 + + 3e - ¾® Al] ´ 4
determination of nitrogen content in the 3+ 2-
3C + 4Al + 6O ¾® 4Al + 3CO 2 CrO3
organic compound in the form of N2 .
Cryolite improves the electrical
N2O + Cu ¾® N2 + CuO (In glacial acetic
conductivity of the cell as Al 2O 3 is a OH acid) in B O
% of N poor conductor.
28 Volume of N2 at NTP
= ´ ´ 100 67. Effect of temperature on reaction rate is 74. Total moles of Ca (NO 3 ) 2
22400 Weight of compound
given by Arrhenius equation, 7
r+ n = (1 + 2 a)
61. =
3
- 1 = 0.225 k = Ae -E a /RT 164
-
r 2 where, A = pre-exponential factor 7
= (1 + 2 ´ 07
. )´ = 01
.
Hence, it is the limiting case where R = gas constant 164
cation in the void of fcc structure is not
T = absolute temperature [Molecular weight of Ca(NO 3 ) 2 = 164]
distorted. So, number of cations
E a = energy of activation 100
surrounding the particular cation = 12. Mole of solvent (H2O) ,N =
But at the same time 8 anions touch 68. N2O 4 and N2O 3 has one N—N bond 18
the particular cation. as shown below. From Raoult’s law,
62. % of H p° - ps n
=
2 Weight of H2O p° n+N
= ´
18 Weight of organic compound N—N N—N 760 - ps 01.
or =
´ 100 760 100
. +
01
2 0.9 18
= ´ ´ 100 = 20% 69. Unit of rate constant (k )
18 05 . ps = 746.3 mm Hg
\ The percentage of carbon = Ln - 1 mol1 - n s -1 75. NHCOCH3
= 100 - 20 = 80% where, n= order of a reaction
63. Let, 20 g glucose is dissolved in For zero order reaction,
100 mL solution. k = L0 -1 mol1 - 0 s -1
\ 180 g (1 mol) glucose is dissolved in \ k = L-1 mol s -1 = mol L -1 s -1 Br2(aq)
100 ´ 180 Rate of reaction = k [A] 0
= NHCOCH3 NHCOCH3
20 Thus, rate of reaction = k
= 900 mL = 0.9 L Br
\ Unit of rate constant and rate of
l reaction is same in zero order reaction. +
64. Degree of dissociation (a) = c .
lo 70. Aspirin (2-acetoxy benzoic acid) due to
\ lc = a ´ l 0 …(i) its anti-blood clotting action, is widely H 3O +
Br
+ -
Now, l 0 (H2O) = l 0 (H ) + l 0 (OH ) used to present heart attack. H3O+
= (350 + 200) S cm 2 equiv -1 71. Geometrical and ionisation isomerism NH2
= 550 S cm 2 equiv -1 is exhibited by octahedral Br NH2
\ lc = 1.9 ´ 10 -9
´ 550 [from eq. (i)] [Co(NH3 ) 4 Br2 ]Cl. Ionisation isomer of
[Co(NH3 ) 4 Br2 ]Cl is [Co(NH3 ) 4 BrCl]Br .
= 1.045 ´ 10-6 S cm 2 equiv -1
Geometrical isomers are cis and trans o-bromoaniline
= 1.045 ´ 10-10 S m 2 equiv -1 as shown below.
Br
p-bromoaniline

MODULE 2
www.jeebooks.in
244 NEET Test Drive

76. CH3 CHO gives positive Fehling’s test 82. CH2—CN Thus, H3PO 2 has maximum number of
whereas C6H5 CHO gives negative (a) P—H bond.
LiAlH4
Fehling’s test. Hence, these aldehydes 85. OsO 4 causes cis hydroxylation as shown
can be distinguish by Fehling’s CH2—CH2—NH2 below.
solution.
CH3 OH
77. Transition elements are more metallic OsO4
than the representative elements due
to the availability of d-orbitals for NaNH2 CH3 OH
bonding. (b) CH3C CH CH3C C :– Nar cis 2, 3 diol
78. NH3
Hybridi- Unpaired Effect of (c) 2 CH3CH2Cl (CH3CH2)2 NH 86. The transition metal ions having d 1 to d 9
Complex
sation electrons ligand configuration are coloured whereas the
NO2 NH2 transition metal ions having d 0 and d 10
[Ni(CN)4 ]2 - dsp2 zero CN- is a Sn/HCl
strong (d) configuration are colourless.
ligand
Among the given compounds, only
[Ni(CO)4 ] sp3 zero CO is a CuF2 , having Cu in +2 oxidation state
strong
ligand
Hence, (b) will not produce an amine. has 3d 9 configuration due to which it is
83. O -2 (superoxide) – Monodentate coloured.
79. The reaction proceeds as : 87. XeO 2F2 has see-saw structure as shown
– en – Bidentate
Br– Monodentate below.
—C—CH3 + CN– —C—CH3
Thus, coordination number of Pt is 6.
H+
CN
[ Pt (O 2 ) (en) 2 (Br)] 2 + F
Xe
OH OH x + (- 1) + (0) ´ 2 + (- 1) = + 2 F
H2O
—C—CH3 —C—CH3 x - 1- 1= + 2
COOH CN x - 2 = + 2 Þx = + 4 see-saw structure
H2SO4 –H2O
Thus, oxidation number of Pt = +4
88. The point (pH range) at which the amino
84. No. of acid molecule has equal positive and
—C==CH2
OH OH P—H negative charges is called the
COOH bond
H4P2O7 O==P—O—P ==O isoelectric point. At this point, the
0
80. The coordination number of Pt in the amino acids do not migrate in an electric
complex ion, [Pt(en) 2 Cl 2 ] 2 + is 6. OH OH field.
2+ O 89. On moving down the group, atomic
en size increase and electronegativity
Cl
H3PO2 P 2 decrease because of the atomic radius
Pt H H
increases.
OH
Cl 1
en O Electronegativity µ
Size
81. Reaction takes place as follows : H3PO3 P 1 F, O, N, S Þ F > O > N > S
H OH
HCN (order of electronegativity).
OH
HO CN 90. As oxidation state of Cl increases, acidic
H3O O
strength of oxy acid of chlorine
H3PO4 P increases.
COOH HO OH 0
OH
OH

BIOLOGY
91. Rhagon canal system is found in larval 93. Thick filaments mainly consist of 94. The dominance of gametophyte is
stage of sponges, which gets myosin protein, which is splitted into observed in bryophytes.
converted to leucon in the adult stage. two fragments LMM (Light This means that the more prominent
92. Henle’s loop is divided into three Meromyosin) and HMM (Heavy longer lived stage is the haploid
regions, i.e. descending limb, Meromyosin). HMM consists of two gametophyte. The diploid sporophyte
ascending limb and main loop. Another subfragments S1 and S2. Each S1 appears only occassionally and remains
name of Henle’s loop is pars recta. It is fragment consists of an ATPase site attached to gametophyte. It nutritionally
the main site for urine concentration. and a binding site for actin. ATPase depends on the gametophyte (in
requires Mg 2 + for its functioning. bryophytes).

MODULE 2
www.jeebooks.in
Prep Catalysis for NEET ~ Mock Test 7 245

95. Myasthenia gravis is an autoimmune and protein components were hard and soft tissues and does not
disease affecting neuromuscular synthesised and they accumulated till stimulate other endocrine glands.
junction, leading to fatigue, weakening the primitive oceans reached to Whereas, all the other three, i.e.
and paralysis of skeletal muscles. consistency of hot dilute soup, the thyrotropin stimulates the thyroid gland,
primordial soup from the living things gonadotropin stimulates the gonadal
96. Neomycin is extracted from
were created. activity and adrenocorticotropin
Streptomyces fradiae. It acts against
acid fast and Gram negative bacilli, but 105. In adsorption technique, the activated stimulates adrenal glands.
toxic to kidneys and ears. It is an charcoal is used to control gaseous 111. A combination of genetic and
antibiotic, should not be taken with pollutants. It consists of very fine solid demographic factors that drive
tetracycline and anthrax bacterium is particles. It removes toxic gases and population size downwards to extinction
not killed. inflammable compounds. In absorption is known as extinction vortex. A small
technique, gaseous pollutants are population will become extinct sooner
97. All the statements are correct
absorbed by scrubber having packing than a large population because of
regarding meristematic cells. They
materials. In combustion technique, the inbreeding depression and periods like
have the power of division. Their cell
emission are burnt at high temperature drought, harsh winter and summer.
wall is thin and nucleus is prominent.
to remove gaseous pollutants. The
98. Nucellus and polar nuclei are diploid solid waste is disposed off at very high
112. Ornithine cycle consists of three main
among the given options. Endosperm parts of conversion of ornithine to
temperature, i.e. 900-1300°C.
is triploid, while others are haploid. citrulline, the conversion of citrulline into
106. An operon is a cluster of genes whose arginine and splitting of arginine into
99. In the given figure, expression is controlled by a single urea and ornithine. The reaction of
A.–Hepatocyte operator, e.g. lac operon. This lac citrulline with aspartic acid to form
B.–Endothelial cell operon is required for the transport and argininosuccinic acid and
metabolism of lactose in E. coli and pyrophosphate is as follows
C.–Kupffer cell many other enteric bacteria.
D.–Branch of interlobular bile duct Citrulline + Aspartic acid + ATP
107. According to distribution of yolk in ¾¾¾¾¾® Argininosuccinic acid +
100. Wolffian duct in males and Gartner’s cytoplasm of the ova, the ovum of a AMP + PPi
duct in females are potential human female can be classified as
113. Statements I and IV are correct, while
embryological remnants of homolecithal as yolk is evenly
statements II and III are incorrect.
mesonephric ducts. Thus, they are distributed. In centrolecithal eggs, yolk
Hormones are first messengers and
homologous structures having similar is concentrated in the middle of egg
thyroid hormone needs receptors to bind
embryology. Wirsung duct is also surrounded by cytoplasm. The yolk is
target sites. Hormones that are too large
called as pancreatic duct. Stensen concentrated in one end of the egg in
like epinephrine to pass cell membranes
duct is also called as parotid duct. telolecithal eggs. Yolk is present in
require specific receptors to reach their
Perihlar duct is also called as large amount as compared by
target sites. Steroid hormones like
extrahepatic bile duct. cytoplasm in macrolecithal eggs.
testosterone are lipid soluble and can
101. Breakbone fever–Tourniquet test 108. Temin (1970) found that reverse of enter cytoplasm of target cells.
Rocky mountain spotted central dogma takes place in 114. The option (c) is incorrectly matched.
fever–Weil-Felix test retroviruses. Here, DNA is synthesised
In a pond ecosystem, the standing crop
from RNA using RNA dependent DNA
Black death–Wayson stain test of phytoplanktons, the major producers
polymerase (or reverse transcriptase).
Diphtheria–Schick test will be lower than the mass of
During translation, RNA is translated
heterotrophs such as fish and insects.
102. Normal blood sugar levels in a normal into polypeptide chain, which folds into
Thus, the pyramid of biomass will be
adult are an enzyme. One amino acid is
inverted for pond ecosystem. Rest pairs
Fasting 80-100 mg/100 mL encoded by many codons (degenerate
are correctly matched.
genetic code). Polyribosomes act to
After meal 100-120 mg / 100 mL. 115. Among the given options, option (b) is
translate mRNA instructions into
103. Eustachian valve is present at the polypeptides. correct.
opening of inferior vena cava. It allows Vital Capacity = ERV + TV + IRV
109. Sesmoid bone is a bone embedded
the passage of blood in right atrium.
within a tendon or a muscle. Often they = 1700 + 500 + 2600
Thebasian (coronary) valve is present = 4800 mL
are formed in response to strain. The
over the opening of coronary sinus. It
knee cap or patella is the largest Inspiratory Capacity = TV + IRV
also allows passage of blood in right
sesmoid bone in the body. Sesmoids
atrium. Mitral valve is present between = 500 + 2600 = 3100 mL
act like pulleys, providing a smooth
left atrium and left ventricle. SA node is 116. Eukaryotic genome has introns and
surface to tendons to slide over,
not a valve. exons. Introns are non-coding segments
increasing the tendons ability to
104. Alexander Oparin and JBS Haldane transmit muscular forces. Flexor carpi of a gene, whereas exons are coding
proposed that the life originated from ulnaris and flexor holloucis brevis are regions. During the process of RNA
carbon based molecules in the muscles and lenticular process of splicing, introns are removed and exons
primordial soup. According to them, incus is most inferior part of incus. are joined.
UV-rays from sun induced reactions on Bacteria contain a single DNA molecule
110. Somatotropin or growth hormone plays
a mixture of water, CO 2 and ammonia. that makes up the entire genome.
an important role in the growth of both
Organic substances such as sugars

MODULE 2
www.jeebooks.in
246 NEET Test Drive

117. Atavism is reappearance of some cylinder and the 2 unfused 134. Sclereids are often called stone cells or
anscetral characters. They are often microtubules in the centre. sclerotic cells. They are dead cells with
seen as evidences of evolution. The core of the flagella, called small lumen. These are found in hard
Evolutionarily traits that have axoneme, has nine pairs of doublets of part of the plants and possess strongly
disappeared phenotypically, remain in radially arranged peripheral lignified and sometimes even suberised
the genome for many generations and microtubules and a pair of centrally thick secondary walls.
these become expressed through a located microtubules. Such an 135. The rolling mechanism of seed dispersal
mutation, e.g. presence of tail, dense arrangement of axonemal microtubules is found in Amaranthus albus. In this
body hairs, monstral face, etc. is referred to as 9 + 2 array. type of seed dispersal, the plant breaks
118. Okazaki fragments are short 125. The middle lamella is a layer of calcium off and rolls over the ground. This results
sequences of DNA nucleotides which pectate, which cements the cell walls in the dispersal of seeds as they are
are synthesised discontinuously and of two adjoining plant cells together. shed all along the way.
later linked together by the enzyme
126. Deficiency of N, K, Mg, S, Fe, Mn, Zn 136. Calcium metabolism in birds gets
DNA ligase. affected by DDT pollution. It results in
and Mo leads to chlorosis which is loss
119. The option (d) is incorrectly matched. shell thinning.
of chlorophyll in leaves making them
Black foot disease is an endemic pale. The affected plant has little or no 137. Archegonia neck is solid in
peripheral vascular disease. It is found ability to manufacture carbohydrate gymnosperms. It lacks neck canal cell.
in areas with high concentration of through photosynthesis. In bryophytes and pteridophytes,
arsenic in water.
127. The most commonly used fluorescent archegonia neck is hollow. In algae,
Minamata disease is a neurological archegonia are not present.
DNA stain is ethidium bromide. This
syndrome caused by severe mercury
molecule binds with the DNA in the gel. 138. During the life cycle of Plasmodium
(Hg) poisoning. Thallium (Th) and its
compounds are highly toxic, contact When exposed to UV-light, the vivax, asexual phase occurs in man. The
with the skin is dangerous as it is ethidium bromide fluorescences and sexual phase occurs in female
readily absorbed by the skin. It is a produces a bright orange light. Anopheles mosquito. Man is the
human carcinogen. 128. An aerobic prokaryote without cell wall intermediate host of the malarial parasite
is mycoplasma. They are the simplest and female Anopheles mosquito is its
Itai-itai disease is caused by mass
definitive host as well as vector.
cadmium (Cd) poisoning. It causes and smallest unicellular organisms.
softening of bones and kidney failure. Spirochaetes, archaebacteria and 139. Mesosomes are also known as
rickettsias possess cell wall and are mitochondria of bacterial cell. They
120. The diaphragm is a muscular structure,
included in prokaryotic cells. contain respiratory enzymes. Thus, they
which separates the thoracic and
help in respiration.
abdominal cavities. Most important 129. The nucleic acid hybridisation method
function of diaphragm of mammals is to of selection and screening of 140. In anaerobic respiration (fermentation),
aid in respiration and any damage transformed cells is based on detection there is no net oxidation or reduction of
would stop breathing and prove fatal. by molecular probe. It is also called as the carbons of glucose.
121. The terrestrial ecosystems include The anaerobic regeneration of NAD is
colony hybridisation method. In this
forest, grassland and desert. It is a called fermentation. In yeast, the
method, the recombinant DNA is
type of natural ecosystem. These regeneration of NAD from NADH takes
detected by molecular probes, i.e.
place under anaerobic conditions. Here
ecosystems are capable of operating either radioactive DNA or RNA probes.
pyruvate is converted into ethanol and
and maintaining themselves without 130. The instruments used to measure CO 2 in a two steps reaction.
any major interference by man. osmotic pressure include plasmolytic 141. Warburg first reported that cancer cells
122. Rennin is the enzyme secreted by method by de Vries, Pfeffer’s exhibit high rate of glucose uptake and
stomach. It hydrolyses the soluble milk osmometer and cryoscopic lactic acid production even in the
protein casein into paracasein. osmometer. presence of oxygen.
Paracasein is spontaneously Cobalt chloride method is used for the Warburg purified and crystallised seven
precipitated in the presence of calcium measurement of transpiration. of the enzymes of glycolysis. His
as insoluble, calcium paracaseinate 131. In PS-II (during light reaction), experimental tools revolutionised
forming coagulated milk. electrons lost need to be replaced. biochemical studies of oxidative
123. The nuclear envelope consists of two Water is split into O 2 , H+ and e - . The metabolism.
lipid bilayer membranes, i.e. an inner electrons needed to replace those 142. The fibrous protein keratin which exists
nuclear membrane and an outer removed from photosystem-I are in hair is a-helix. The two most important
nuclear membrane. The outer nuclear provided by photosystem-II. secondary structures of proteins are
membrane is continuous with the 132. In plants showing photorespiration, the a-helix and the b-sheet.
endoplasmic reticulum membrane. The rate of CO 2 assimilation is very low. The a-helix is a common motif in the
space between the membranes is About 25% of carbon fixed is released secondary structure of proteins and is a
called the perinuclear space. as CO 2 . right-handed spiral conformation. Here,
124. The 9+2 fibrillar arrangement is present every backbone N-H group donates a
133. Ground meristem is involved in the hydrogen bond to the backbone C == O
in eukaryotic flagella. This arrangement development of cortex and pith in group of the amino acid located 3-4
refers to the 9 fused pairs of dicots. Plate, mass and rib meristem residues earlier along the protein
microtubules on the outside of a are classified on the basis of plane of sequence.
cell divisions.

MODULE 2
www.jeebooks.in
Prep Catalysis for NEET ~ Mock Test 7 247

143. Spermicidal cream ‘consap’ is a plant having two or more embryos body for proper functioning of biological
based product developed by Central developing from a single fertilised egg. systems. Vaccines are provided to boost
Drug Research Institute (Lucknow). It is 150. PCR uses Taq polymerase. This Taq immune system that helps in fighting
obtained from Reetha, i.e. Sapindus DNA polymerase is obtained from against microbes.
mukrossi. Thermus aquaticus that is naturally 159. The process of formation and
144. The natural pollution is the pollution found in hot springs. It does not differentiation of microspores (pollen
caused by natural sources. These denature at high temperatures, which grains) from Microspore Mother Cells
sources include volcanic eruptions, is necessary for amplifying DNA using (MMCs) by reductional division is called
natural gas emission, ozone, nitrogen PCR. microsporogenesis.
oxides, dust storm, etc. 151. All the given statements are true. In 160. Halophytes are plants of saline habitats,
145. All the given statements are correct fragile X syndrome, a specific area which have not only the ability to tolerate
regarding J-shaped growth curve. along the chromosome may fail to stain high concentrations of salts in their
This type of curve represents the giving the appearance of a gap. These rooting medium but are able to obtain
population growing exponentially or areas are known as fragile sites, as their water supply from the same.
geometrically until there is a resource they are susceptible to chromosome In halophytes, the stomata are less in
limitation or population growth is breakage. number. They may be sunken and
limited by other factors. The main source of gene redundancy present on inner surface of leaves.
In it, minor fluctuations are not is gene duplication process. 161. Facilitative diffusion is the major
observed. The equilibrium phase is 152. Statements I, III and IV are correct, transport route for fructose. A common
rarely reached and environmental while II is incorrect. Frog aestivates secondary sodium dependent active
resistances do not operate. during summer and hibernates during transporter absorbs glucose and
146. Human genes for blood clotting factor winters. galactose.
IX have been transferred in sheep and 153. All the statements are Mendel’s first Lipids are transported via simple
is expressed in mammary tissue. Cow three postulates. His fourth postulate diffusion from the small intestine.
is inserted with 2 DNA coding states that during gamete formation,
lactoferrin and has therapeutic human 162. Haemopoiesis–Production of blood cells
segregating pairs of unit factors assort and platelets
protein in its milk. independently of each other.
Transgenic mice are being developed Ossification – Bone formation
for the use in testing the safety vaccine 154. Secondary growth is absent in
Erythropoiesis – Formation of RBCs
before they are used in humans. These monocot stems as their vascular
bundles do not possess vascular Calcification–Accumulation of calcium
transgenic mice contain breast cancer
combium. In most of the palms, the salts in tissues
causing human gene. Transgenic fish
contains human growth hormone. They secondary growth is accomplished by 163. Generally amenorrhoea (cessation of
attains a size twice of that shown by primary thickening meristem. menses) is the first and most common
non-transgenic fish. 155. Diatoms are photosynthetic protists, sign of pregnancy.
147. The statement in option (c) is incorrect. which are the chief producers in the Quickening (perception of foetal
It can be corrected as follows– oceans. movements by mother) occurs at
Kinetin (6-furfurylaminopurine) does not Myxomycetes or slime moulds are the 16 weeks after conception.
occur naturally. It is a synthetic saprophytic protists. Protozoan protists 164. Allen’s rule was formulated by Joel Allen.
hormone. Rest all statements are are zooflagellates. They are It states that animals adapted to cold
correct. heterotrophs and live as predators or climates have shorter limb and body
148. Tumour protein P 53 is also known as parasites. appendages than animals adapted to
P 53 . It is crucial in the prevention of 156. In retroviruses, with the help of enzyme warm climates. It is to reduce the surface
cancer thus, functioning as a tumour reverse transcriptase, reverse central areas for homeothermic animals.
suppressor. dogma takes place where DNA is 165. Due to fragile nature of protoplast
It is called as guardian of genomes synthesied from RNA. This enzyme membrane, desirable characters can be
because of its role in conserving transcribes viral RNA to cDNA. incorporated in plants by protoplast
stability by preventing genome culture.
157. The option (d) is not paired correctly. It
mutation.
symbolises consanguineous parents 166. Cystolith is a botanical term for
Cells lacking P 53 are unable to arrest at (closely related). Rest options are outgrowths of the epidermal cell wall,
cell cycle checkpoints or enter correctly matched. usually of CaCO 3 . It is formed in
apoptosis in response to DNA damage. cellulose matrix in special cells called
158. Probiotics are good bacteria that
RB1 gene encodes for retinoblastoma lithocysts, generally in the leaf of plants.
reduce the growth of harmful bacteria
protein that is a tumour suppressor
in human digestive tract. Thus, they 167. The statement (a) is incorrect regarding
protein. Ras gene synthesises proteins
help in maintaining the natural balance a community. It can be corrected as
that are involved in cellular signal
of organisms in intestine and promote a follows
transduction.
healthy digestive system. Community refers to a group of
149. Polyembryony is very common in
Antibiotics are those chemical individuals of different species found in
gymnosperms. It is regarded as an
compounds that either retard the an area. Rest options are correct.
important characteristic of this group.
growth of bacteria or kill them. Vitamins 168. Fragmentation is a natural method of
Polyembryony is the phenomenon of
are needed in small amounts in our vegetative propagation. It is a form of

MODULE 2
www.jeebooks.in
248 NEET Test Drive

asexual reproduction, in which the Examples of chemical mutagens are but it can elicit an immune response
parent individuals breakdown into alkylating agents, base analogues, against the toxin. These are based on
small pieces of fragments, which then acridine dyes and others (nitrous acid, the toxin produced by certain bacteria,
develop into new individuals. Cutting, sodium azide). e.g. tetanus.
layering and grafting are artificial 172. An ideal vector possesses all, i.e. a 176. The option (c) is not paired correctly.
methods of vegetative propagation. selectable marker, an origin of It is the symbol of fraternal (dizygotic)
169. Severe Combined Immuno Deficiency replication (ori) and restriction sites. Ori twins, sex may be the same or different.
Disease (SCID) is a rare genetic is a specific sequence of nucleotide in
177. Gama Amino Butyric Acid (GABA) is
disease that affects immune system. DNA from where replication starts,
released by synaptic knobs of the fibres
It is caused by a mutation in the gene selectable marker permits the selection
of some interneurons in CNS. It is an
for the enzyme Adenosine Deaminase of host cells and restriction sites allow
inhibitory neurotransmitter as it inhibits
(ADA). cleavage of specific sequence.
the post-synaptic regeneration of action
170. A. Neem – Azadirachta indica 173. Eukaryotic cells possess split genes, potential.
here the coding bases are not
B. Panda – Ailuropoda 178. Paramecium is heterokaryotic because
continuous, but are interrupted by
C. Potato – Solanum tuberosum of the presence of macronucleus and
non-coding sequences. Exons are the
D. Cow – Bos taurus micronucleus, i.e. dimorphic nuclei.
coding regions, whereas introns are
E. Strawberry – Fragaria non-coding. 179. Latimeria chalumnae is an ancient fish,
171. Mutagen is a physical or chemical which is considered as a living fossil. It is
174. Balanoglossus is an ocean dwelling
agent that changes the genetic the sole remaining member of a taxon
acornworm. It is an evolutionary link
material, usually DNA, of an organism otherwise known only from fossils. It is
between vertebrates and and
and thus increases the frequency of also known as West Indian ocean
invertebrates.
mutations above the natural coelocanth.
175. Toxoids are vaccines that are made
background level. 180. The family of proteases involved in
from toxin produced by a pathogen.
Harmful radiations of sun, X-rays, programmed cell death and intracellular
g-rays, b-rays and UV-rays are physical Toxoids are made from the toxin digestion of components is caspases.
mutagens. (poison) that has been made harmless,

SCORE CHART
No. of Correct Answers : A .................. No. of Incorrect Answers : B ..................
Total Marks : x = (A × 4) – (B × 1)
Scores and Expected Rank : If the score lies above 665, then rank will be in between 1-50. For other scores, rank
estimations are given below
Score Rank
664 – 642 51 – 150
641 – 636 151 – 250
635 – 630 251 – 400
629 – 625 401 – 500

MODULE 2
www.jeebooks.in

PREP CATALYSIS
for NEET
Full Length Mock Tests for NEET to Make You Ready to Face the Challenge

MOCK TEST 8 (With Solutions)

Duration : 3 Hours Max. Mark : 720

Instructions
The test is of 3 hours duration and Test Booklet contains 180 questions. Each question carries 4 marks. For
each correct response, the candidate will get 4 marks. For each incorrect response, one mark will be deducted
from the total scores. The maximum marks are 720.
The question paper contains three parts of Physics, Chemistry and Biology respectively.
Part A contains 45 Questions from Physics section. Part B contains 45 Questions from Chemistry section. Part
C contains 90 Questions from Biology.

PART A PHYSICS
1. Stationary waves are set up by the superposition of 3. A body of mass m1 , moving with a velocity 3 ms −1
two waves given by collides with another body at rest of mass m2. After
y1 = 0.05 sin( 5πt − x ) collision, the velocities of the two bodies are 2 ms −1
and 5 ms −1 respectively, along the direction of motion
and y2 = 0.05 sin( 5πt + x ) m
of m1 . The ratio of 1 is
where, x and y are in metre and t is in second. The m2
displacement of a particle situated at a distance, 5 1 12
(a) (b) 5 (c) (d)
x = 1 m is 12 5 5
(a) 0.54 m (b) 0.054 m (c) 0.25 m (d) 0.025
4. 3 moles of an ideal gas is heated at constant
2. Five equal resistors, each of 5 Ω are connected in a pressure, so that its temperature increases from
network as shown in the following figure, then 27°C to 177° C, then work done is
equivalent resistance between P and Q is (a) 5732 J (b) 3735 J
5Ω (c) 3243 J (d) 1775 J

5Ω 5Ω 5Ω 5. A helium nucleus makes full rotation in a circle of


B D
P
C
Q radius 0.8 metre in two seconds. The value of the
A
magnetic field B at the centre of the circle will be
5Ω 10−19 2 × 10−10
(a) (b) 10−19 µ 0 (c) 2 × 10−10 µ 0 (d)
(a) 5Ω (b) 10Ω (c) 25Ω (d) 20Ω µ0 µ0

MODULE 2
www.jeebooks.in
250 NEET Test Drive

2 2
6. Which graph of the following closely depicts the  9   
 24 − g  
correct variation of the gravitational potential V (r ) g  2  g  (8 − 3g ) 
(a)   (b)  
due to a large planet of radius R and uniform mass 2  (8 − 3g )  2   24 − 9 g  
 
density?     2  
2 2
V (r) V (r) g  (8g − 3)  g  (24g − 9)
(c) (d)
2  (24g − 9) 2  (3g − 8) 
(a) (b)
$,
13. Electric field in the space is given by 3i$ − 2$j + 4k
then the value of electric flux passing through an
O r O r
area 36 unit lying in the xy-plane is
V (r) r V (r) r (a) 180 unit (b) 108 unit (c) 144 unit (d) 72 unit
O 14. A set of 24 tunning forks is arranged in a series of
O
(c) (d) increasing frequencies. If each fork gives 4 beats
per second with the preceding one and the last
sounds the octave of the first. Find the frequencies
of the first and the last forks.
7. A sphere has bulk modulus B. It is placed inside a (a) 184 Hz, 92 Hz (b) 92 Hz, 184 Hz
pressure chamber, if a constant pressure p is (c) 85 Hz, 162 Hz (d) 162 Hz, 85 Hz
maintained inside chamber. The fractional
decrement in diameter is 15. A copper rod of length 250 cm and radius 2 mm is
p 3p p p joined to a steel rod of same length and radius at
(a) (b) (c) (d) 25°C. If the coefficients of linear expansion of
2B B 3B B
copper and steel are 2.5 × 10−5 °C −1 and
8. A body is sliding down an inclined plane having 1.25 × 10−5 °C −1 , then change in length of the
coefficient of friction 0.5. If the normal reaction is combined rod at 200°C is
twice that of the resultant downward force along (a) 15 mm (b) 13.32 mm (c) 16.4 mm (d) 18.3 mm
the incline, then the angle between the inclined
plane and the horizontal is 16. Time period of a freely suspended thin magnet is
(a) 15° (b) 30° (c) 45° (d) 60° 8 s. If it is broken in length into two equal parts
and one part is suspended in the same way, then
9. A piece of wood is floating in water. When the its time period in seconds will be
temperature of water rises, then the apparent (a) 2 s (b) 4 s
weight of the wood will be (c) 8 s (d) None of these
(a) increase
17. A capacitor of capacitance 2 µF is charged by
(b) decrease
connecting a battery of 100 V across its plates.
(c) may increase or decrease
Now, battery is disconnected and another capacitor
(d) remain same
of capacitance 6 µF is connected in parallel to first
10. The angular speed of a motor wheel is increased capacitor. Then, the ratio of total energy stored by
from 1800 rpm to 3600 rpm in 12s, then the both capacitors to initial energy of first capacitor is
angular acceleration is 2 3 1 1
(a) (b) (c) (d)
(a) 4π rad s −2
(b) 5π rad s −2 3 4 4 2
(c) 2 π rad s −2 (d) 6π rad s −2 18. Particle each of mass m are falling into weighing
pan from height H at a rate of n particles per
11. The efficiency of heat engine is 0.8, when the second. Find reading of weighing pan after time T ,
temperature of the source is T1 and that of sink is
if collision of particle with weighing pan is perfectly
T2. The efficiency of another heat engine is also 0.8,
inelastic. (Neglect oscillation of weighing pan).
then the temperature of source and sink of the m
second engine are respectively
(a) KT1 , KT2 (b) 3T1 , 0.5 T2 H

(c) T1 + 10, T2 − 10 (d) T1 − 3, T2 + 3

12. A stone is dropped from the top of a fall cliff and


after 3 s another stone is thrown vertically
downwards with a speed of 8 ms−1 . Then, the
second stone will overtake the first, below the top (a) mgH + mgnT (b) mn(gT + 2 gH )
of cliff at a distance of (c) mng 2 gH (d) None of these

MODULE 2
www.jeebooks.in
Prep Catalysis for NEET ~ Mock Test 8 251

19. A 20 m long wire of resistance 25 Ω is connected in 26. A cylinder of length 1.4 m, radius 0.5 m and
series with a battery of emf 4V and a resistance density 4 × 102 kgm −2. The moment of inertia of
15Ω. If the internal resistance of battery is cylinder about the axis is
negligible, then the potential gradient along the (a) 0.55 kg-m 2 (b) 5.5 kg-m 2 (c) 55 kg-m 2 (d) 5.0 kg-m 2
wire is 27. A proton and an α-particle move on same circular
(a) 0.25 V/m (b) 0.125 V/m path in a perpendicular magnetic field, then their
(c) 0.45 V/m (d) 0.5 V/m kinetic energies are in the ratio
20. Resonance frequency of L- C- R series AC circuit is (a) 2 : 1 (b) 1 : 1 (c) 2: 3 (d) 1 : 4
f0. Now, the capacitance is made 4 times, then the 28. Magnification of a compound microscope is 30.
new resonance frequency will be Focal length of eye-piece is 5 cm and the image is
f f formed at a distance of distinct vision of 25 cm.
(a) 0 (b) 2 f0 (c) f0 (d) 0
4 2 The magnification of the objective lens is
21. The radius of gyration k of a solid sphere of mass (a) 6 (b) 5 (c) 7.5 (d) 10
M and radius R about a certain axis is equal to
29. The electric field of an electromagnetic wave in
2 R. Find the distance of that axis from the centre
free space is given by
of the sphere.
2 2 2 2 2 E = 20 cos(108 t + kx ) $j V/m
(a) R (b) R (c) R (d) R
5 5 5 5 where, t and x are in second and metre, then
4 1. wavelength λ is 18.9 m
22. If the refractive index of water is and that of
3 2. wave number k is 0.33 rad/m
5
given slab of glass immersed in it is . What is the 3. amplitude is 20 V/m
3
critical angle for a ray of light tending to go from 4. The wave is propagating along y-direction
glass to water? Correct statements are
(a) sin−1  
3 (a) 1, 2 and 3 (b) 1, 2 and 4 (c) 1, 3 and 4 (d) 2, 3 and 4
 4
30. If the length of wire is increased by 20% by
(b) sin  
−1 3 drawing it, then its resistance
 5 (a) is increased by 44% (b) is decreased by 44%
(c) sin  
−1 4 (c) is increased by 20% (d) is decreased by 20%
 5
31. The escape velocity of a projectile on the earth’s
(d) Data given are insufficient to make any calculation surface is 11.2 kms −1 . A body is projected with
23. When a weight of 1 kg is suspended from a thrice this speed from the earth’s surface, then the
weightless spring, its length increases by 2 cm. The speed of body far away from the earth is
22.4
weight is pulled down by 10 cm and released, then (a) kms −1 (b) 31.68 kms −1
2
the period of oscillation of spring and its kinetic
energy are (c) 22.4 kms −1 (d) 31.68 kms −1

(take, g = 10 ms −2) 32. A beam of light composed of red and green rays is
(a) 2.5 s, 0.28 J (b) 0.5 s, 5 J incident obliquely at a point on the face of a
(c) 0.28 s, 2.5 J (d) 5 s, 0.5 J rectangular glass slab. When coming out on the
opposite parallel face, then the red and green rays
24. If the temperature of the sink of a Carnot engine is emerge from
57°C and its efficiency is 10%, then the (a) two points propagating in two different non-parallel
temperature of the source is directions
(a) 227°C (b) 93.6°C (b) two points propagating in two different parallel directions
(c) 127.5°C (d) 85.7°C (c) one point propagating in two different directions
25. A star 2.5 times the mass of the sun and collapsed (d) one point propagating in the same direction
to a size of 12 km rotates with a speed of 1.5 rev/s. 33. A galvanometer of resistance 35 Ω is shunted by a
An object placed on its equator will (mass of the 3.5 Ω wire. The part of total current that flows
sun = 2 × 1030 kg) I 
(a) remain stuck to its surface through the galvanometer  g  is given as
I
(b) leave is surface
3 2 1 4
(c) move towards sun (a) (b) (c) (d)
11 11 11 11
(d) remain/leave depending on object’s mass

MODULE 2
www.jeebooks.in
252 NEET Test Drive

34. Two polaroids are crossed. If now one of them 40. In the following common emitter circuit, if β = 96,
is rotate through 30° and unpolarised light of V CE = 5V, VBC = negligible, RC = 3 kΩ, then IB = ?
intensity I 0 is incident on the first polaroid,
then the intensity of transmitted light will be –
15 V
+
I0 3l0
(a) (b) IB
4 4
3l l RB RC
(c) 0 (d) 0
8 8
Ic
35. A solid sphere of radius r is rotating about its c
diameter at an angular speed of ω. If it is B
1 E
cooled to reduce its radius to of its original
4
value, then the angular speed becomes
(a) 4 ω (b) 8 ω
(c) 16 ω (d) 2 ω (a) 0.012 mA (b) 0.044 mA (c) 0.034 mA (d) 0.082 mA
36. A change of 9.0 mA in the emitter current 41. In which case, there is no Coil
brings a change of 8.8 mA in collector current, current induced in the coil as
the values of α and β are shown in the figure?
(a) 0.99, 90 (b) 0.97, 42.4
(c) 0.97, 49.3 (d) 0.98, 65 (a) When both moves towards each
other N S
37. Two coherent sources of intensity ratio β (b) When magnet moves towards coil Bar
( I max − I min ) (c) When coil and magnet both move
magnet
interfere, then the value of is
( I max + I min ) in same direction with same velocity
(d) When both moves away from each other with same speed
1+ β 1 + β
(a) (b)   42. In Young’s double slit experiment with sodium vapour
β  β 
lamp of wavelength 589 nm and the slits 0.589 mm
1+ β 2 β
(c) (d) appart, then the half angular width of the central
2 β 1+ β maximum is
(a) sin−1 (0.01) (b) sin−1 (0.0001)
38. If a proton and anti-proton come close to each
(c) sin−1 (0.001) (d) sin−1 ( 01
.)
other and annihilate, then how much energy
will be released ? 43. The activity of a radioactive substance decreases to
. × 10−10 J
(a) 15 (b) 3 × 10−10 J one-third of the original activity number N 0 in a period
(c) 4.5 × 10−10 J (d) 2 × 10−10 J of 9 yr. After a further lapse of 9 yr, its activity will be
3 N0
39. Four balls each of mass 2 kg are placed at the (a) N0 (b) (c) N0 (d) None of these
4 9
corners of a square of side 10 cm. If mass m3 is
removed as shown in the figure, then the shift 44. Study the statement given below and find which is true?
in the centre of mass of the system is I. When a bus suddenly stopped the passengers tend
Y
to fall forward.
II. When stone is rotated in a circle of smaller radius,
greater force is required.
m4 m3 III. At optimum speed frictional force is not needed at
all to provide the necessary centripetal force in case
C of banked road.
(a) I only (b) I and II
X (c) I, II and III (d) All are false
m1 m2
45. The energy that should be added to an electron to
(b)  $i +
10 10 $ 
(a) (5$i + 5$j) j reduce, its de-Broglie wavelength from 1 nm to
 3 3  0.5 nm is
(c)  $i + $j
5 5
(d) (3$i + 3$j) (a) equal to initial energy (b) two times of initial energy
3 3  (c) three times of initial energy (d) four times of initial energy

MODULE 2
www.jeebooks.in

PART B CHEMISTRY
46. In reaction, C5H10Cl2 + NaOH ¾® C5H10O 51. Mn2+ ( aq ) ion is light pink coloured due to
(A) (B) (a) d-d transition is spin forbidden
B is an aldehyde with no a-hydrogen. Thus, A is (b) d-d transition is laporte forbidden (Dl = ± 1)
(a) (CH3 )3 CCHCl 2 (b) CH3CH2 CHC HCl 2 (c) d-d transition is allowed
½ (d) Both (a) and (b)
Cl CH3
½ 52. Density of 2.05 M solution of acetic acid in water is
(c) CH3 CHC H2CHCl 2 (d) CH3 CC H2CH2CH3 1.02 g/mL. The molality of the solution is
½ ½ (a) 0.44 mol kg -1 (b) 1.14 mol kg -1
CH3 Cl
(c) 3.28 mol kg - 1 (d) 2.29 mol kg - 1
47. Hydrogen reacts with different elements to form
variety of hydrides. Their stability depends upon 53. Which one of the following do not decolourise
lattice energy and ionic character. Covalent KMnO 4 in aqueous solution?
hydride is most common type of hydrides. Which of (a) SO 23 - (b) CO 23 - (c) C 2O 24 - (d) HSO -3
the following is an example of polymeric hydride?
(a) CaH2 (b) MgH2 (c) BaH2 (d) SrH2 54. Aldohexoses are six carbon sugars with four
chirality centres and an aldehyde carbonyl group.
48. The most basic property of a polymer is the There are 16 possible stereoisomeric aldohexoses.
identity of its constituent monomers. A second set D-glucose and D-mannose are X of each other.
of properties, known as microstructure essentially Here, ‘X’ refers to
describe the arrangement of these monomers (a) epimers (b) anomer
within the polymer at the scale of a single chain. (c) enantiomer (d) diasteromer
Orlon is a polymer of
(a) vinyl cyanide (b) chloroethene 55. Which of the following statements regarding
(c) methyl methacrylate (d) caprolactam transition elements is incorrect?
(a) Ti 2 + and Cr 2 + are reducing agents in aqueous medium
49. Correct order of basic strength is
(b) Mn3 + and Co 3 + are oxidising agents in aqueous solution
NH2 NH2 NH2 NH2
(c) In its higher oxidation state, manganese forms stable
NO2
compounds with oxygen and fluorine
(a) > > > (d) Copper liberates hydrogen from acids
56. C3H 6Cl2 on reaction with NaOH forms C3H 6O which
NO2 CH3
gives yellow precipitate on heating with NaOH and
NH2 NH2 NH2 NH2
I2. Thus, C3H 6Cl2 is
NO2
(a) 1, 1-dichloropropane
(b) > > > (b) 1, 2-dichloropropane
(c) 2, 2-dichloropropane
CH3 NO2 (d) 1, 3-dichloropropane
NH2 NH2 NH2 NH2
57. Which one of the following is correct order?
NO2
(a) ¾OH > ¾OR (order of - I effect)
(c) > > >
(b) ¾CH3 > ¾CO 3 > CCl 3 (order of + I power)
(c) ¾F > ¾OH > ¾NH2 (order of - I effect)
CH3 NO2 +
(d) ¾NO>SO 2 R > SO 3H>NO 2 (set of meta directors)
NH2 NH2 NH2 NH2
NO2 58. One gas bleaches the colour of flowers by reduction
(d) > > > and another gas by oxidation. The gases
respectively are
CH3 NO2 (a) H2S and Br2 (b) NO and Cl 2
(c) CO 2 and Cl 2 (d) SO 2 and Cl 2
50. Nyctalopia also called ‘Night Blindness’ is a
condition making it difficult or impossible to see in 59. The qualitative analysis of an organic compound
relative low light. It is symptom of several eye implies the detection of all major elements which
diseases. It may exist from birth, or be caused by can be present in it with the help of suitable
injury or malnutrition. It is caused in human due chemical tests. Lassaigne’s method cannot be used
to deficiency of vitamin for analysis of organic compound containing
(a) A (b) B (c) C (d) D (a) P (b) Cl (c) N (d) S

MODULE 2
www.jeebooks.in
254 NEET Test Drive

60. In perchloric acid (HClO4 ) , the number of s and 68. The values of standard electrode potentials of
p-bonds are Zn2 + / Zn and Ag+ /Ag are - 0.763 V and + 0.799 V
(a) 5s and 4p-bonds (b) 6s and 3p-bonds respectively. The standard potential of the cell is
(c) 5s and 3p-bonds (d) 5s and 2p-bonds (a) 1.56 V
61. An ether is more volatile than an alcohol having (b) - 1.56 V
the same molecular formula. This is due to (c) - 0.036 V
(a) dipolar character of ethers (d) + 0.036 V
(b) alcohols have resonance structures
(c) intermolecular hydrogen bonding in ethers 69. Elimination of bromine from a-bromobutane
(d) intermolecular hydrogen bonding in alcohols
results in the formation of
(a) predominantly but-2-yne
62. Which of the following statements is not correct for (b) predominantly but-2-ene
Joule-Thomson’s experiment? (c) predominantly but-1-ene
(a) Process is adiabatic
(d) equimolar mixture of but-1-ene and but-2-ene
(b) Process is iso enthalpic
(c) Lower temperature is observed on low pressure side 70. Consider a third order reaction which involves a
(d) None of the above single reactant. If the concentration of the reactant
is doubled, the rate of the reaction will
63. Which one of the following statements is incorrect (a) increases by a factor of two
about alkoxy mercuration-demercuration of alkenes?
(a) Identical regiochemistry is seen in product (b) increases by a factor of four
(b) It involves addition of water according to Markownikoff’s (c) increases by a factor of eight
rule (d) increases by a factor of nine
(c) It involves no rearrangment 71. Identify the Z in the following reaction,
(d) It involves carbonium ion formation
3Cl 2 / D Br2 / Fe Zn / HCl
64. Which of the following statement is incorrect? C7H 8 ¾¾¾® X ¾¾¾® Y ¾¾¾® Z
(a) Boron shows only + 3 oxidation state while all other show (a) 2, 4, 6-trichlorotoluene (b) o-bromotoluene
+ 1as well as + 3 oxidation state (c) p-bromotoluene (d) m-bromotoluene
(b) Stability of + 1oxidation state decreases from top to
bottom along group 13 72. Which of the following statement(s) is/are incorrect
(c) Compound of Ti(I) shows great resemblence with alkali metals regarding alkanes?
(d) Aluminium trichloride occurs as dimeric structure (a) The solubility of alkanes diminishes with increase in
molecular weight
65. Which of the following forms cationic micelles (b) The dipole moment of all alkanes, whether straight or
above certain concentration? branched chain, will be zero
(a) Urea (c) Melting point of alkanes decreases with increase in
(b) Sodium dodecyl sulphate molecular weights
(c) Sodium acetate (d) Alkanes are also called as paraffins
(d) Cetyltrimethylammonium bromide
73. The decomposition of NH 3 on Pt surface is zero
66. Arrange halides of group 13 elements in correct order reaction. Calculate the rate of production
order of Lewis acidic strength. [Hint X : halogen] of N 2 and H 2 if k = 2.5 ´ 10-4 mol-1L-1s -1?
(a) BX 3 > AlX 3 > GaX 3 > InX 3 (b) BX 3 < AlX 3 < GaX 3 < InX 3 (a) 1.25 ´ 10- 4 and 3.75 ´ 10- 4 respectively
(c) BX 3 < AlX 3 > GaX 3 < InX 3 (d) BX 3 < AlX 3 > GaX 3 > InX 3 . ´ 10- 4 and 1.25 ´ 10- 4 respectively
(b) 375
67. What will be the product B in the following (c) 1.25 ´ 10- 3 and 3.75 ´ 10- 3 respectively
reaction? (d) 3.75 ´ 10- 3 and 1.25 ´ 10- 3 respectively
HBr
A
aq. KOH
B 74. Cr3 + form four complexes with four different
H 2 O2
ligands which are [Cr(Cl)6 ]3 - , [Cr(H 2O)6 ]3 + ,
OH [Cr(NH3 )6 ]3 + and [Cr(CN)6 ]3 - . The order of CFSE
(a) (b) OH (D o ) in these complexes is
(a) [Cr(Cl)6 ]3 - = [Cr(H2O)6 ]3 + = [Cr(NH3 )6 ]3 + = [Cr(CN)6 ]3 -
OH
(b) [Cr(Cl)6 ]3 - < [Cr(H2O)6 ]3 + < [Cr(NH3 )6 ]3 + < [Cr(CN)6 ]3 -
OH
(c) [Cr(Cl)6 ]3 - > [Cr(H2O)6 ]3 + > [Cr(NH3 )6 ]3 + < [Cr(CN)6 ]3 -
(c) OH (d)
(d) [Cr(Cl)6 ]3 - < [Cr(H2O)6 ]3 + = [CrNH3 )6 ]3 + < [Cr(CN)6 ]3 -

OH

MODULE 2
www.jeebooks.in
Prep Catalysis for NEET ~ Mock Test 8 255

75. Potassium carbonate cannot be made by the Solvay Br Br Br H


process because ½ ½ ½ ½
(a) CH3 ¾C ¾ C ¾ CH3 (b) CH3 ¾C ¾ C ¾CH3
(a) potassium hydrogen carbonate is unstable ½ ½ ½ ½
(b) potassium hydrogen carbonate is rather too soluble in H H Br H
water to be precipitated Br Br Br
(c) potassium carbonate is insoluble in water ½ ½ ½
(d) potassium carbonate is soluble in water (c) CH3 ¾ C == C ¾ CH3 (d) CH3 ¾C == C ¾ CH3
½ ½ ½
76. 100 mL of a mixture of NaOH and Na 2SO4 is Br Br Br
neutralised by 10 mL of 0.5 M H 2SO4. The amount 82. The radius of first Bohr orbit of hydrogen atom is
of NaOH in 100 mL solution is 0.52 Å. The radius of the third orbit of H + will be
(a) 0.2 g (b) 0.4 g (a) 8.46 Å (b) 0.705 Å (c) 1.59 Å (d) 4.76 Å
(c) 0.6 g (d) None of these
83. Consider the following reactions:
77. The magnetic moment of [Cu(NH3 )4 ]2 + was found I. Zn + Dil. H 2SO4 ¾® ZnSO4 + H 2
to be 1.73 BM. The number of unpaired electrons II. Zn + Conc. H 2SO4 ¾® ZnSO4 + SO2 + H 2O
in the complex is
III. Zn + HNO3 ¾® Zn(NO3 )2 + NH 4NO3 + H 2O
(a) 0 (b) 1
(c) 2 (d) 3 Oxidation number of hydrogen changes in
(a) I, II and III (b) Both I and II
78. Which of the following statement is incorrect about (c) Both II and III (d) Only I
group 2 metals?
84. Changes in a system from an initial state to the
(a) They all dissolve in liquid NH3
final state were made by a different manner that
(b) Dilute solutions of group-2 metals in liq. NH3 are bright DH remains same but q changes because
blue in colour but the concentrated solutions are bronze (a) DH is a path function but q is a state function
coloured
(b) DH is a state function and q is a path function
(c) Solutions of group-2 metals in liquid NH3 decompose very (c) Both DH and q are state function
slowly, forming amides and evolving H2 (d) Both DH and q are path function
(d) The evaporation of NH3 from the solutions of group 2 metals
give the metals 85. If the equilibrium constant for the reaction,
NH3 ( aq ) + H + ( aq ) -NH +4 ( aq ) is 2.8 ´ 108.
79. Match Column I with Column II. Identify correct
Then, the equilibrium constant for
answer by using the codes given below :
NH3 ( aq ) + H + NH +4 ( aq ) + OH - ( aq ) is
-
Column I Column II (Units) (a) 2.8 ´ 10- 22 (b) 2.8 ´ 10- 6 (c) 3.2 ´ 10- 10 (d) 1.4 ´ 10- 6
-2 2 -1
A. First order reaction 1. k = mol L s 86. Which one of the following cannot exist in stable
-1 state according to the molecular orbital theory?
B. Zero order reaction 2. k = s
-1 -1 (a) He 2 (b) H-2 (c) H2 (d) H+2
C. Second order reaction 3. k = mol Ls
D. Third order reaction 4. k = mol L -1s -1
87. pH of solution prepared by mixing 800 mL of 0.05N
NaOH and 200 mL of 0.1 N HCl assuming the
Codes complete ionisation of acid and a base is
A B C D A B C D (a) 5 (b) 10.4 (c) 12.3 (d) 1.7
(a) 1 3 4 2 (b) 2 4 3 1
88. What will be the molar mass of A if hydrogen
(c) 2 3 1 4 (d) 4 1 3 2
diffuses six times faster than gas A?
80. The correct balanced equation when CuS is (a) 36 (b) 72 (c) 24 (d) 6
dissolved in dil. HNO 3 is
(a) CuS + 2H+ + 3 NO 3- ¾® Cu(NO 3 )2 + H2S + H2O + NO 2
89. For the equilibrium in a closed vessel
PCl 5 ( g) PCl 3 ( g) + Cl 2 ( g)
-
(b) 3CuS + 8H+ + 8NO 3- ¾® 3Cu(NO 3 )2 + 3S + 4H2O + 2NO
K p is found to be double of K C . This attained when
(c) CuS + 4NO 3 ¾® Cu(NO 3 )2 + H2S + H2O
(a) T = 2 K (b) T = 12.18 K (c) T = 24.36 K (d) T = 27.3 K
(d) None of the above is correct statement
90. For monoatomic gas, kinetic energy (E) relation
81. What will be the final product of following with rms velocity ( u rms ) is
reaction? 1 1 1 1

(a) u = æç ö÷ (b) u = æç
E ö2 æ 3E ö 2 (d) u = æç
Br2 /CCl 4 2E 2 E ö2
CH 3 ¾ C ºº C ¾ CH 3 ¾ ¾ ¾ ¾
¾® ÷ (c) u = ç ÷ ÷
è mø è2 mø è2 mø è 3m ø

MODULE 2
www.jeebooks.in

PART C BIOLOGY
91. Which of the following is a correctly matched pair 99. Identify the correct statement.
with reference to anti-cancer drug? (a) DNA fingerprinting is the production of living organisms,
(a) Vincristine — Cinchona officinalis which are genetically identical to their parents
(b) Vinblastine — Chrysanthemum cinerarifolium (b) The idea of cloning was first suggested by Hans Spemann
(c) Vinblastine — Cinnamomum camphora in 1938
(d) Vincristine — Catharanthus roseus (c) The idea of cloning was given by John Gordon in 1970
(d) The technique of cloning was developed by Sir Alec
92. Antibiosis refers to
Jeffreys
(a) the production of chemical poisons
(b) the production of a bactericide Chlorella that inhibits the 100. Which among the following statement(s) is/are
growth of Daphnia correct regarding carrying capacity?
(c) the production of a toxic substance by Microcystis which (a) Maximum number of individuals that can survive in an
causes the death of fishes ecosystem
(d) All of the above (b) Number of individuals per species in as ecosystem
93. A condition in humans where all the 46 (c) Both (a) and (b)
chromosomes are present, but a small portion of (d) None of the above
chromosome 5 is missing (deletion) 101. Refer to the given figure showing TS of human
(a) Edward’s syndrome (b) Patau’s syndrome spinal cord. Identify the parts labelled as A, B, C
(c) Cri du chat syndrome (d) Monosomy and D in the figure.
Poster ior median sulcus Poster ior
94. Which of the following is not a criteria for inter mediate septum
Poster ior
essentiality of a mineral nutrient for plants? median septum Poster ior root
(a) The element must be able to support the process of of spinal ner ve
normal growth and reproduction L ater al Poster ior
funiculus funiculus
(b) Requirement of the particular element in appropriate
Poster ior
amount must be specific and it should not be replaced by gr ey hor n
any other element
(c) The element must have the property of involving in the A nter ior gr ey hor n
functioning and metabolism of the plant directly
A nter ior root of
(d) Plant must absorb the mineral nutrient from the soil spinal ner ve
A nter ior median fissur e
95. With regard to eugenics, identify the correct
statement(s). A B C D
(a) Sir Francis Galton is known as the ‘Father of Eugenics’ (a) White matter Grey matter Central canal Anterior funiculus
(b) In negative eugenics, people with inferior and undesirable (b) Grey matter White matter Central canal Posterior funiculus
traits are prevented from reproducing
(c) Anterior Central Grey White
(c) In positive eugenics, people with better and desirable traits
funiculus canal matter matter
are encouraged to produce offspring
(d) Central Posterior White Grey
(d) All are correct
canal funiculus matter matter
96. Which amongst the following have non-nucleated 102. Which of the following is not a marine bird?
blood cells?
(a) Mallard (b) Goose
(a) Monocytes, Erythrocytes (b) Erythrocytes, Lymphocytes (c) Arctic tern (d) Albatross
(c) Erythrocytes, Thrombocytes (d) Erythrocytes, Eosinophils
103. Which of the following is incorrect in relation with
97. Identify the incorrectly matched pair. the Human Genome Project (HGP) ?
(a) Escherichia coli Ry13  Sticky ends (a) Its aim was to map the whole human genome at the level
(b) Haemophilus aegypticus  Sticky ends of nucleotide sequences
(c) Serratia marcescens  Blunt ends (b) It was started in 1988 and completed in 2002
(d) Anabaena viriabilis  Sticky ends (c) Its aim included storing the information gathered from the
98. Name a species of fungus which can be used as a project in databases and develop tools for data analysis
biocontrol agent. (d) Its aim was to solve any ethical, legal or social problems
(a) Ashbya gossypii (b) Trichoderma which may arise from the project
(c) Aspergillus giganteus (d) Penicillium chrysogenum

MODULE 2
www.jeebooks.in
Prep Catalysis for NEET ~ Mock Test 8 257

104. Which of the following statements is not correct 113. A type of aneuploidy where unmasking of a
with respect to Down syndrome? recessive lethal allele that is otherwise tolerated in
(a) It is caused due to non-disjunction of chromosome heterozygotes carrying the corresponding wild-type
number 21 during meiosis allele, may lead to failure of individual to survive
(b) It is a monosomic condition is
(c) Children with this syndrome have affectionate and loving (a) trisomy (b) monosomy
nature (c) disomy (d) amphidiploidy
(d) Individual with this syndrome have a characteristic 114. Which of the following genes is one of the most
prominent epicanthic fold in each eye mutated in the human tumours?
105. Consider the following statements. (a) BAX (b) ras gene family
I. The sperm identifies the egg by (c) MDM2 (d) pRB
fertilizin-antifertilizin reaction. 115. Eyes of Octopus are analogous to which of the
II. Fertilizin initiates acrosomal reactions. following?
III. Fertilisation between different species is (a) Eyes of poriferans (b) Eyes of annelids
promoted by fertilizin. (c) Eyes of mammals (d) Eyes of aves
IV. Fertilizin prevents polyspermy. 116. What are the bead-like thickened portions of
The incorrect statement is leptotene chromosomes known as?
(a) Only I (b) Only II (c) Only III (d) Only IV (a) Puffs (b) Centromeres
(c) Chromomeres (d) Telomere
106. An indicator of lead in water is
(a) Chladophora glomerata (b) Aspergillus niger 117. Which among the following statement(s) is/are
(c) Lichen (d) Both (a) and (b) correct regarding detritus food chain?
I. It begins from dead organic matter.
107. Endosperm culture would not be desirable for the
commercial production of which of the following? II. Energy comes from the primary producers.
(a) Grapes (b) Pomegranate (c) Orange (d) Papaya
III. It begins from the primary producers.
IV. Energy comes from sun.
108. Consider the following statements. Choose the correct option.
I. Boutonneuse fever’s causative agent is (a) I and II (b) Only I (c) Only II (d) III and IV
Rickettsia conorii.
118. Toxicity of manganese causes deficiency of which
II. Rickettsialpox is transmitted to man from
elements?
rodents.
(a) Fe 3 + , Mg 2 + , Ca 2 + (b) K + , NO 3− , PO 34 −
III. The causative agent of Q-fever is Coxiella (c) Mn2 + , Zn2 + , Cu2 + (d) SO 24 − , Cl − , MoO 22 +
burnetii.
119. In plant species like Tamarindus and Orchidus,
IV. Typhus is caused by Rickettsia rickettsii.
the endosperm is utilised by embryo and no
The incorrect statement is endosperm is left at maturity. Such seeds are
(a) Only I (b) Only II (c) Only III (d) Only IV called
109. Narrow utilitarian aspect of biodiversity (a) exalbuminous (b) albuminous
conservation includes (c) xenia (d) metaxenia
(a) pine resin (b) gums and tannin sources 120. Which of the following are responsible for an
(c) dyes (d) All of these inflammatory response?
110. The Bhakarwal breed of sheep is majorly found in (a) Histamines (b) Prostaglandins
(a) Gujarat (b) Himachal Pradesh (c) Kinins (d) All of these
(c) Uttarakhand (d) Jammu and Kashmir 121. The cattle or goats do not browse on the plant
111. Vitamin-D 3 is essential for the Ca 2 +
absorption. Calotropis because
Active form of vitamin-D 3 is also called as (a) it produces highly poisonous cardiac glycosides
(a) calciferol (b) cholecalciferol (b) it is an endoparasite
(c) calcitriol (d) calcidiol (c) it is ectoparasitic
(d) it shows cannibalism
112. Study the following pairs carefully and identify the
pair which is not correctly matched. 122. Proteins after complete hydrolysis yield amino
(a) Tendon — Connective tissue acids. The excess amino acids are stored in
(b) Smooth muscle — Involuntary (a) pancreas (b) kidney
(c) Myosin — Contractile protein (c) adipose tissues (d) None of these
(d) Troponin — Smooth muscle

MODULE 2
www.jeebooks.in
258 NEET Test Drive

123. Consider the following statements. 132. The lymph capillary present in the intestinal villi
I. The presence of heterozygosity is the basis of concerned with the absorption of fat is called
mass selection. (a) Peyer’s patches (b) lacteal
(c) crypts of Lieberkuhn (d) haustra
II. Pureline is the progeny of a single,
homozygous, self-pollinated plant. 133. In humans, the bacteria present in alimentary
III. Progeny selection is mainly employed in canal could become difficult to manage because
self-pollinated crops. (a) it could take up the antibiotic resistance gene present in
the GM food
IV. Clonal selection is the selection of desirable
(b) it may develop resistance to insects
clones from the mixed population.
(c) it can cause toxicity and allergies
The incorrect statement(s) is/are
(d) it can result in cold and drought tolerance
(a) Only I (b) Only II
(c) Only III (d) Only IV 134. Match column I with column II and choose the
correct option from the codes given below.
124. Which among the following statements is/are
incorrect regarding Gram-positive bacteria? Column I Column II
(a) Lipopolysaccharides are absent A. Epiphytes 1. Oedogonium
(b) Prominent mesosomes are present
B. Endophytes 2. Anabaena
(c) Basal body of flagellum contains four rings
(d) Highly sensitive to antibiotics C. Epizoic 3. Cladophora crispata

125. Consider the following matches. D. Endozoic 4. Zoochlorella


I. Apolar neurons—Neurons of Hydra Codes
II. Unipolar neurons—Vertebrate embryos A B C D A B C D
III. Bipolar neurons—Olfactory epithelium (a) 1 2 3 4 (b) 1 2 4 3
(c) 2 I 4 3 (d) 2 1 3 4
IV. Multipolar neurons—Retina of eyes
The incorrect match is 135. Which of the following statement(s) is/are incorrect
with respect to urine?
(a) Only I (b) Only II
(c) Only III (d) Only IV I. It is amber coloured watery fluid.
II. It is slightly basic with pH around 8.0.
126. Plants with large and odourless flowers possessing
nectaries and some edible parts are pollinated by III. It consists of 96% of urea.
(a) honeybee (b) ants IV. It is faintly aromatic in fresh form.
(c) birds (d) wasps Choose the correct option.
127. Where is the major amount of cytokinin produced (a) I and IV (b) II and III
in vascular plants? (c) II, III and IV (d) I, II and III
(a) Flowers (b) Leaves 136. Arrange the following steps in the sequence of
(c) Shoots (d) Roots construction of genomic library.
128. Graafian follicle contains large central follicular I. Entire genome is broken into fragments with
cavity called the help of restriction endonucleases.
(a) stroma (b) antrum II. Recombinant DNA molecules produced are
(c) infundibulum (d) ostium introduced into prokaryotic cells.
129. The tertiary structure of a protein molecule results III. Fragments are joined to cloning vectors.
from IV. rDNA molecules replicate to form the clones.
(a) electrostatic bonding (b) hydrophobic bonding
(c) hydrogen bonding (d) All of these
Choose the correct option.
(a) I → II → III → IV (b) I → III → II → IV
130. Leaf base which completely encircles the stem is (c) I → IV → II → III (d) IV → I → II → III
known as
(a) amplexicaul (b) sheathing leaf base 137. Which among the following does not come under
(c) pulvinus leaf base (d) modified leaf base the category of internal factors affecting
transpiration?
131. Name the enzyme used in dissolving blood clots in (a) Sunken stomata
heart patients. (b) Thick cuticle
(a) Cyclosporin-A (b) Statins
(c) Light
(c) Amylase (d) Streptokinase
(d) Water content of the leaves

MODULE 2
www.jeebooks.in
Prep Catalysis for NEET ~ Mock Test 8 259

138. Among the following which one is the most advanced 150. Cryopreservation is an example of
form of kidney and can be found in humans? (a) Ex situ conservation strategies
(a) Metanephric (b) Archinephric (b) In situ conservation strategies
(c) Mesonephric (d) Pronephric (c) Tissue culture
139. The function of companion cell is to (d) Both (a) and (b)
(a) help the sieve tube members in translocation of food material 151. Consider the following matches regarding pigments
(b) provide mechanical support and light absorbed.
(c) store food material I. Chlorophyll-a and b — 400-500 nm
(d) accumulate tannins II. Carotenoids — maximum at 500 nm
140. Staphylococcus and Streptococcus are III. Phycocyanin — minimum at 600 nm
(a) Gram-positive cocci (b) Myxophyceae The incorrect match is
(c) Actinomycetes (d) Chemoautotrophs (a) Only I (b) Only II (c) Only III (d) I and III

141. Which among the following is a total root parasite? 152. Refer to the given figure showing Waldeyer’s ring.
(a) Rafflesia (b) Orobanche Choose the option that correctly identifies the
(c) Both (a) and (b) (d) None of these parts labelled as A, B, C and D in the figure.
142. What is Necturus called commonly? A
(a) Congoeel (b) Cave salamander Tubal tonsil
(c) Hellbender (d) Mud puppy B

143. Cephaleuros virescens is an example of


(a) water blooms (b) algal parasitism C
(c) algal toxicity (d) All of these
144. Reproduction without multiplication is shown by
(a) conidia (b) zoospores
(c) endospores (d) aplanospores D
145. Identify the correct sequence.
(a) Species < Genus < Family < Order < Class < Division (a) A–Lingual tonsil B–Eustachian tube
(b) Species < Class < Family < Order < Genus < Kingdom C–Pharyngeal tonsil D–Palatine tonsil
(c) Genus < Order < Family < Class < Division < Kingdom (b) A–Pharyngeal tonsil B–Eustachian tube
(d) Species < Genus < Class < Order < Division < Kingdom C–Palatine tonsil D–Lingual tonsil
146. Consider the following matches. (c) A–Eustachian tube B–Pharyngeal tonsil
I. Lactose — β-1, 4-glycosidic linkage. C–Palatine tonsil D–Lingual tonsil
II. Maltose — α-1, 2-glycosidic linkage. (d) A–Eustachian tonsil B–Palatine tonsil
C–Lingual tonsil D–Pharyngeal tonsil
III. Sucrose — α-1, 2-glycosidic linkage.
The incorrect match is
153. Soil is made up of various layers called soil
horizons. Identify correct statements regarding soil
(a) Only I (b) Only II (c) I and II (d) II and III
horizons.
147. The variations in chromosome structure is due to I. A-horizon is topmost part of soil.
I. deletion II. duplication II. B-horizon is subsoil.
III. translocation IV. inversion III. C-horizon contains water table.
The correct option is IV. D-horizon is made up of unweathered parent
(a) I and II (b) III and IV (c) II, III and IV (d) I, II, III and IV
bed rock.
148. Consider the following matches. Choose the correct option.
I. Allopatric speciation—Darwin’s finches (a) I and III (b) I, II, III and IV (c) III and IV (d) II and IV
II. Sympatric speciation—Polyploids 154. The problems of GM foods are
III. Peripatric speciation—Darevskia rudis (a) the transgene products may cause toxicity and/or produce
The incorrect match is allergies
(a) Only I (b) Only II (c) Only III (d) I and II (b) The enzyme produced by the antibiotic resistance gene
could cause allergies because it is a foreign protein
149. Indicators of water pollution include
(c) Bringing GM food to market is an expensive process
(a) Escherichia coli (b) Daphnia
(c) Mycorrhiza (d) Both (a) and (b) (d) All of the above

MODULE 2
www.jeebooks.in
260 NEET Test Drive

155. Which among the following statements is/are (c) male gametes that are carried by pollen tube
correct regarding secondary growth in dicot stem? (d) Both (a) and (c)
(a) The vascular cambial ring develops as a circular ring since 163. Which of the following functions is associated with
beginning spherosomes?
(b) The vascular rays are narrow since beginning (a) Metabolism of alcohol
(c) Annual rings occur quite commonly (b) Synthesis of proteins
(d) All of the above (c) Synthesis and storage of fats
156. Which of following statements is incorrect about (d) Storage of excess glucose
climbers? 164. X-ray diffraction studies were critical evidence
(a) The stem is weak but is not sensitive to contact indicating that DNA has a double-helix structure.
(b) A climber is able to rise upward a support These studies were done by
(c) Stem tip does not perform circumnutation (a) Watson and Crick
(d) Clinging organs are absent (b) Chargaff
157. How many molecules of CO 2 are evolved in (c) Rosalind Franklin and Maurice Wilkins
glycolysis? (d) Meselson and Stahl
(a) 1 (b) 2 (c) 3 (d) None 165. Pituitary gland or hypophysis is a pea-shaped
158. Match Column I with Column II and choose the gland which lies in the cavity called sella turcica of
correct option from the codes given below. (a) ethmoid (b) sphenoid (c) nasal (d) vomer

Column I Column II 166. The causes of seed dormancy include


(a) the production of compounds that restrict germination
A. Bryophytes 1. Non-vascular, lacking seeds
(b) hard seed coat
B. Pteridophytes 2. Vascular, lacking seeds (c) seed coat impermeable to oxygen
C. Gymnosperms 3. Vascular, naked seeded plants (d) All of the above
D. Angiosperms 4. Seeds well-protected in fruits 167. Identify the incorrectly matched pair.
(a) Latex cells  Papaver (b) Latex vessels  Calotropis
Codes
(c) Both (a) and (b) (d) None of these
A B C D A B C D
(a) 1 2 4 3 (b) 2 1 4 3 168. Broca’s area present in the left frontal lobe of the
(c) 1 2 3 4 (d) 2 1 3 4 human brain controls
(a) learning and reasoning (b) movement of tongue
159. Endospore-forming rods and cocci include
(c) breathing and hiccups (d) All of these
(a) Bacillus and Clostridium (b) Lactobacillus
(c) Mycoplasma (d) Chlamydia 169. DNA is the transforming factor in bacterial cells,
was first proven by
160. Which one of the following four glands is
(a) Griffith (b) Avery (c) Meischer (d) Crick
incorrectly matched with the accompanying
description? 170. What is wobble effect or phenomenon?
(a) Transfer of tRNA from A site to P site
(a) Thyroid — Hyposecretion in children causes
cretinism (b) Movement of many ribosomes along a single mRNA
(c) The ability of a tRNA to pair with different codons that may
(b) Parathyroid — Secretes parathormone that regulates
differ in the third base
amount of calcium and phosphate
(d) Shifting of the reading frame due to mutation
between blood and other tissues
(c) Pineal — Secretes three groups of steroid 171. Potometer method of measurement of transpiration
hormones from cholesterol is based on the assumption that the
(d) Thymus — Starts undergoing atrophy after puberty (a) rate of water uptake is approximately equal to the rate of
transpiration
161. Pseudomonas putida is involved in the (b) rate of water uptake is dependent upon external factors
(a) production of human insulin like temperature
(b) biological control of fungal disease (c) rate of water uptake is not equal to the rate of transpiration
(c) digestion of hydrocarbon of crude oil (d) rate of water uptake does not depend on any internal
(d) control of greenhouse gases factor like water content of leaves
162. The siphonogamous plants refer to the plants 172. Mineralocorticoids are steroid hormones produced
having from
(a) non-motile male gametes (a) zona fasciculata (b) zona glomerulosa
(b) motile male gametes (c) zona reticularis (d) zona pollucida

MODULE 2
www.jeebooks.in
Prep Catalysis for NEET ~ Mock Test 8 261

173. Match Column I with Column II and choose the 177. The gene transfer to non-target species causes
correct option from the codes given below. (a) transfer of the herbicide resistant genes from the crops into
Column I Column II the weeds
(b) crop improvement
A. Restriction enzymes 1. Molecular binder
(c) genes for nitrogen-fixation transferred to cereal crops
B. DNA ligase 2. Dissolve the cell wall
(d) transfer of genes for cold tolerance
C. Lyases 3. Molecular scissors
178. Epiphysis cerebri is also called as … A … .
D. Synthetases 4. In vitro synthesis of DNA TCT (Thyrocalcitonin) … B … the calcium level is
Codes high in blood.
A B C D A B C D Choose the correct option for A and B from the
given options.
(a) 1 2 3 4 (b) 4 3 2 1
(c) 3 1 2 4 (d) 1 4 3 2 A B
174. The muscle contraction requires energy which (a) Pituitary gland Increases
comes from ATP hydrolysis. The enzyme ATPase (b) Adrenal gland Increases
for this is present in
(a) globular head of meromyosin (c) Pineal gland Decreases
(b) tropomyosin (d) Parathyroid gland Dereases
(c) light meromyosin
(d) cisternae of sarcoplasmic reticulum
179. Consider the following matches.
I. Body surface respiration — Hydra
175. What is RNA processing?
II. Branchial respiration — Prawns
(a) Processing of old, worn-out RNAs
(b) It is an another name for transcription III. Tracheal respiration — Leech
(c) It is related to tRNA function during protein synthesis IV. Cutaneous respiration — Earthworm
(d) It is an event that occurs after RNA has been transcribed The incorrect match is
176. What is a polysome? (a) Only I (b) Only II
(a) Initiation site for transcription (c) Only III (d) Only IV
(b) Initiation complex for translation 180. The stage between the two meiotic division is
(c) A number of ribosomes on the same transcript (a) interkinesis (b) cytokinesis
(d) Elongation factor in DNA synthesis or replication (c) diakinesis (d) karyokinesis

Answer Sheet
1. (b) 2. (a) 3. (b) 4. (b) 5. (b) 6. (c) 7. (c) 8. (c) 9. (c) 10. (b)
11. (a) 12. (a) 13. (c) 14. (b) 15. (c) 16. (b) 17. (c) 18. (b) 19. (b) 20. (d)
21. (b) 22. (c) 23. (c) 24. (b) 25. (a) 26. (c) 27. (b) 28. (b) 29. (a) 30. (a)
31. (b) 32. (b) 33. (c) 34. (d) 35. (c) 36. (b) 37. (d) 38. (b) 39. (c) 40. (c)
41. (c) 42. (c) 43. (b) 44. (b) 45. (c) 46. (a) 47. (b) 48. (a) 49. (d) 50. (a)
51. (d) 52. (d) 53. (b) 54. (a) 55. (d) 56. (c) 57. (c) 58. (d) 59. (a) 60. (c)
61. (d) 62. (d) 63. (d) 64. (b) 65. (d) 66. (a) 67. (b) 68. (a) 69. (b) 70. (c)
71. (d) 72. (c) 73. (a) 74. (b) 75. (b) 76. (a) 77. (b) 78. (d) 79. (b) 80. (b)
81. (c) 82. (d) 83. (d) 84. (b) 85. (b) 86. (a) 87. (c) 88. (b) 89. (c) 90. (a)
91. (d) 92. (d) 93. (c) 94. (d) 95. (d) 96. (c) 97. (b) 98. (b) 99. (b) 100. (a)

101. (a) 102. (c) 103. (b) 104. (b) 105. (c) 106. (a) 107. (b) 108. (d) 109. (d) 110. (d)
111. (c) 112. (d) 113. (b) 114. (b) 115. (c) 116. (c) 117. (a) 118. (a) 119. (a) 120. (d)
121. (a) 122. (d) 123. (c) 124. (c) 125. (d) 126. (c) 127. (d) 128. (b) 129. (d) 130. (a)
131. (d) 132. (b) 133. (a) 134. (a) 135. (b) 136. (b) 137. (c) 138. (a) 139. (a) 140. (a)
141. (c) 142. (d) 143. (b) 144. (c) 145. (a) 146. (b) 147. (d) 148. (c) 149. (d) 150. (a)
151. (c) 152. (b) 153. (b) 154. (d) 155. (d) 156. (d) 157. (d) 158. (c) 159. (a) 160. (c)
161. (c) 162. (d) 163. (c) 164. (c) 165. (b) 166. (d) 167. (c) 168. (b) 169. (b) 170. (c)
171. (a) 172. (b) 173. (c) 174. (a) 175. (d) 176. (c) 177. (a) 178. (c) 179. (c) 180. (a)

MODULE 2
www.jeebooks.in
262 NEET Test Drive

EXPLANATIONS

PHYSICS
1. The resultant displacement, 3 GM é r2 ù Angular acceleration,
For r < R , V = - ê1 - ú
y = y1 + y 2 2 R ë 3R 2 û w - w1 120p - 60p
a= 2 = = 5p rad s -2
= 0.05 [sin(5 pt - x ) + sin(5 pt + x )] t 12
v( r) r
= 0.05 ´ 2 sin5 pt cos x r=R T2
11. Efficiency of Carnot engine, h = 1 - .
éQ sin A + sinB ù O T1
ê = 2 sin A + B cos A - B ú r>R
êë 2 2 úû Efficiency remains the same, when both
r<R
temperaturesT1 andT 2 are increased by
Þ y = 01
. sin5 p t cos x same factor.
At x = 1m, 12. Let two stones meet at time t.
180° 7. We know that, bulk modulus,
y = 01
. cos(1) = 01
. cos The distance travelled by 1st stone is
p F 1
= 01
. ´ 05402
. = 0.054 m DV p s1 = gt 2 [Qu = 0]
B = A Þ = 2
DV V B
2. Given question satisfies the condition The distance travelled by 2nd stone is
V
of Wheatstone’s bridge principle, 1
hence Fractional decrement in volume, s 2 = 8 (t - 3) + g (t - 3) 2
2
RPQ = (5 + 5) || (5 + 5) DV DD é p
QV = pr 3 = D 3 ù
4
Þ =3 1 9
10 ´ 10 V D êë 3 6 úû = 8t - 24 + gt 2 + g - 3gt
= 2 2
10 + 10 Volume of the sphere decreases due to At time t, s1 = s 2
= 5W the decrease in its diameter. 1 2 1 9
DV DD p DD p Þ gt = 8t - 24 + gt 2 + g - 3gt
3. The final velocities of bodies are Hence, =3 = So, = 2 2 2
u1 V D B D 3B 9
u2=0 24 - g
8. From figure, Þ t = 2
m1 m2 8 - 3g

co

Thus, distance after which second stone


N
g
m

v1 v2 2
éæ 9 öù
µ

ç 24 - g ÷ ú
g êè 2 ø .
m1 m2 overtakes is s1 = ê ú
θ 2 ê (8 - 3g) ú
æ m - m2 ö mg sin θ mg cos θ êë úû
v1 = ç 1 ÷u1 [Qu 2 = 0] θ
è m1 + m 2 ø 13. Since, area lying in xy-plane, hence
mg
v2 =
2m1u1 A = 36k$
m1 + m 2 Resultant downward force along incline \ Electric flux, f = E × A
v1 m1 - m 2 = mg (sin q - m cos q) = (3 $i - 2 $j + 4 k$ ) × (36 k$ ) = 144 unit
\ =
v2 2m1 Normal reaction, N = mg cos q 14. Let, frequency of 1st fork = n
2 m1 - m 2 m1 According to question,
Þ = Þ =5 frequency of 2nd fork = n + 4
5 2m1 m2 mg cos q = 2mg (sin q - m cos q)
So, frequency of 24th fork = n + 23 ´ 4
4. DW = nRDT [Qp DV = nR DT ] Þ cos q = sin q [Qm = 05
. (given)]
But frequency of last is the octave of 1st,
= 3 ´ 8.3 ´ (177 - 27) = 3735 J or q = 45°
\ 2 n = n + (23 ´ 4) Þ n = 92 Hz
5. Current, 9. Water has abnormal property, it has Frequency of last fork = 2 n = 184 Hz
q . ´ 10-19
2 ´ 16 highest density at 4°C. So, apparent
i = = = 1. 6 ´ 10-19 A weight of the wood may increase or 15. Change in length of copper rod
t 2 Dlc = aclc (T 2 - T1)
decreases.
\ Magnetic field,
1800 = 2.5 ´ 10-5 ´ 2500 ´ (200 - 25)
. ´ 10-19
m i m ´ 16 10. v1 = 1800 rpm = rps = 30 rps
B = 0 = 0 = 10-19 m 0 60 = 10.93 mm
2r 2 ´ 0.8 3600
v 2 = 3600 rpm = rps = 60 rps Similarly, Dls = (a)s ls (T 2 - T1) = 5.46 mm
GM 60
6. For r > R , V = - So, change in combined rod = Dlc + Dls
r w1 = 2 pn1 = 60p rad s, -1
GM = 10.93 + 5.46 » 16.4 mm
For r = R , V = - and w 2 = 2 pn 2 = 120p rad s -1
R

MODULE 2
www.jeebooks.in
Prep Catalysis for NEET ~ Mock Test 8 263

16. Distinguish between I (inertia) & l dp d Time period,


= [m 2gH ´ nT ] = mn 2gH
(length) in rotations everywhere. dt dt m 1
T = 2p = 2 ´ 3.14 ´ = 0.28 s
æ l ö Force due to weighing k 500
As,T = 2p ç ÷
è MBH ø = mg ´ nT = mgnT For x = 10 cm = 01
. m
where, I = moment of inertia of magnet, \ Reading of weighing pan 1
Kinetic energy = kx 2
M = magnetic moment of magnet and = mgnT + mn 2gH 2
BH = the earth’s horizontal component. = mn (gT + 2gH ) 1
. ) 2 = 2.5 J
= ´ 500 ´ (01
l 2
When it is broken in length = 19. Total resistance, R = 25 + 15 = 40W
2 T2 10 330
\ Current through potentiometer wire, 24. h = 1 - Þ = 1-
m T1 100 T1
and mass will be (if m is total mass) V 4
2 I = = = 01
. A T1
R 40 = T1 - 330 Þ 9T1 = 3300
Now, after cutting, M ¢ = m æç ö÷ 10
l
è2ø Potential difference across wire,
So,T1 = 93.6° C
V = IR = 01. ´ 25 = 2.5 V
[m is pole strength] 25. M = 2.5 ´ Mass of the sun
V
M \ Potential gradient =
Þ New magnetic moment M ¢ = l = 2.5 ´ 2 ´ 1030 = 5 ´ 1030 kg
2
2.5 R = 12 km = 12 ´ 103 m
1 2 = = 0125
. V/m
and I = ml [Thin bar] 20 Acceleration due to gravity on star’s
12
20. We know that, surface
1 æm ö æ l ö I
2
After breaking, I ¢ = ç ÷ç ÷ = f0 = resonance frequency GM 6.67 ´ 10-11 ´ 5 ´ 1030
12 è 2 ø è 4 ø 8 1 1 g = 2
=
= ´ R (12 ´ 103 ) 2
æI ö 2p LC
ç ÷ = 2.316 ´ 10 12
ms -2
è8ø T 8 1
T ¢ = 2p = = = 4s Þ f0 µ As, v = 15
. rps,
æ öB
M 2 2 C
ç ÷ H \ w = 2 pn = 3 p rad/s
è2ø When capacitance of the circuit is
Centripetal acceleration (ac ) = Rw 2
17. Initial energy of first capacitor, made 4 times, let new resonance
1 frequency is f ¢ 0 . = 12 ´ 103 ´ (3 ´ 3.14) 2
E1 = C1V 2
2 f0¢ C 1 f = 1065
. ´ 106 ms -2
Þ = = Þ f0¢ = 0
1 f0 4C 2 2 As g > ac , so object will remain stuck to
= ´ 2 ´ 10-6 ´ 1002 = 10-2 J
2 star’s surface.
21. From parallel axes theorem
When both capacitors are connected in A X 26. Mass of cylinder,
parallel, then equivalence capacitance M = pR 2l ´ r
C = C1 + C2 22
= . ) 2 ´ 14
´ (05 . ´ 4 ´ 102
= 2 + 6 = 8mF 7
Now, potential across each capacitor x = 440 kg
O
q CV Moment of inertia of the cylinder about its
(V ¢ ) = 1 = 1 1
C C own axis, I = MR 2
2
2 ´ 10-6 ´ 100
= = 25 V 1
8 ´ 10-6 B Y = ´ 440 ´ (05 . ) 2 = 55 kg-m 2
2
Therefore, final total stored energy I XY = I AB + Mx 2 1
27. Kinetic energy, E K = mv 2
1 1 2 2
= C1V ¢ 2 + C2V ¢ 2 Þ Mk 2 = MR 2 + Mx 2
2 2 5 2
1 q 2B 2r 2
= m × æç
1 Bqr ö
1 k = ÷ =
E 2 = ´ 2 ´ 10-6 ´ 25 2 Given, 2R 2 è m ø 2 m
2 2 2 2
\ 2R 2
= R2 + x2 Þ x = R (E K ) p q p2 m a e2 4m p
= 0.25 ´ 10-2 J 5 5 \ = = × = 1: 1
-2
(E K ) a q a2 × mp (2e) 2 mp
E2 0.25 ´ 10 1
\ = = 22. Critical angle,
E1 10-2 4 v0 æ Dö æ Dö
æ4ö 28. m = ç1 - ÷ = m 0 ç1 + ÷
-1 æ mw ö
ç ÷ u0 è fe ø è fe ø
C = sin ç ÷ = sin ç ÷ = sin-1 æç ö÷
-1 3 4
18. Reading of weighing pan is due to two
m
è gø ç5 ÷ è5 ø æ 25 ö
reasons Þ 30 = m 0 ç1 + ÷ = m0 ´ 6 Þ m0 = 5
è3ø è 5 ø
= Force due to momentum transfer to it
+ Force due to weighing 23. The force constant of spring is 29. E = 20 cos(108 t + kx ) $j V/m
F mg 1 ´ 10 Comparing with,
Force exerted due to momentum k = = = = 500 Nm -1
transfer, x x 2 ´ 10-2 E = E 0 cos(wt + kx )$j V/m

MODULE 2
www.jeebooks.in
264 NEET Test Drive

Amplitude, E 0 = 20 V/m, w = 108 rad/s 34. Intensity of transmit light from one When m 3 is removed, then the new
l coordinates of centre of mass are
Wave number, polaroid, l1 = 0
2 2 ´ 0 + 2 ´ 10 + 2 ´ 0
w 108 x ¢ CM =
k = = = 0.33 rad/m Therefore, intensity of light transmitted 2+ 2+ 2
c 3 ´ 108 from second polaroid
20 10
2p 2p l 2 = l1 cos 2 q = = cm
Wavelength, l = = = 18.9 m 6 3
k 0.33 l
= 0 cos 2 (90° - 30° ) 2 ´ 0 + 2 ´ 0 + 2 ´ 10
Hence, 1, 2 and 3 are correct. 2 y ¢ CM =
2
2+ 2+ 2
= cos 2 60° = 0 ´ æç ö÷ = 0
30. Let initial length of wire is l1, therefore l0 l 1 l
10
final length 2 2 è2ø 8 = cm
3
6l1
\ l2 = l1 + 20% of l1 =
\ Shift = (5 i$ + 5 $j) - æç i$ +
35. Applying law of conservation of 10 10 $ ö
5 j÷
angular momentum, è3 3 ø
l2 6
\ = I1w1 = I 2 w 2
= æç i$ + $jö÷
5 5
l1 5 2
For solid sphere, I = mr 2 è3 3 ø
By drawing the wire, volume remains 5
constant. 2 2 40. Here,V = VCE + ICRC
\ mr12 w1 = mr 22 w 2 V - VCE 15 - 5
\ pr12 l1 = pr 22 l2 5 5 IC = = = 3.33 mA
r12 l2 6 Þ r12 w1 = r 22 w 2 RC 3 ´ 103
= = …(i) IC 3.33
r 22 l1 5 æ r1 ö
2
2 Q b= Þ Ib = = 0.034 mA
w2 = ç ÷ w1 = (4) w = 16w IB 96
l2 r12 è r2 ø
New, resistance R 2 = × ´ R1
l1 r 22 41. When coil and magnet both move in
D IC 8.8
36. As, a = Þ a= = 0.97 same direction with same velocity, so
6 6 36 D Ie 9.0
= × ´ R1 = ´ R1 there will be no change in flux and due
5 5 25 a 0.977
and b = = = 42 .4 to this, no current will induce.
R - R1 1 - a 1 - 0.977
Increases in resistance = 2 ´ 100 v Coil
R1
37. We know that, I µ a 2
36
R1 - R1 aµ l
= 25 ´ 100 = 44% v
R1 I max (a + a2 ) 2 (1 + b ) 2
= 1 = N S
I min (a1 - a2 ) 2 (1 - b ) 2
. kms -1
31. Escape velocity, ve = 112 Bar
I max + I min (1 + b ) 2 + (1 - b ) 2 magnet
Velocity of projection, v = 3ve Þ =
I max - I min (1 + b ) 2 - (1 - b ) 2
By the law of conservation of energy, 42. In Young’s double slit experiment, half
1 1 1 2 (1 + b) angular width is given by
mv 02 = mv 2 - mve2 =
4 b l 589 ´ 10-9
2 2 2 sin q = = = 10-3
I max - I min 2 b d 0589. ´ 10-3
or v 0 = v 2 - ve2 = (3ve ) 2 - ve2 Þ =
I max + I min 1+ b Þ q = sin-1 (0.001)
= 8 ve2 = . )2
8 ´ (112 N
38. Mass of proton = Mass of anti-proton 43. As, N = N 0e - lt Þ 0 = N 0e - 9 l
. kms -1
= 22.4 2 = 3168 . ´ 10-27 kg = 1amu
= 167 3
1
32. We know that, in any medium except Energy equivalent to 1 amu = 931MeV Þ = e -9 l
3
vacuum or air, then the velocities of So, energy equivalent to 2 amu Now, after further 9 yr,
different colours are different.
= 2 ´ 931MeV 2
N ¢ = N 0e -18l = N 0 (e -9 l ) 2 = N 0 æç ö÷ = 0
Therefore, both red and green colours 1 N
are refracted at different angles of . ´ 10-19
= 1862 ´ 106 ´ 16 è3ø 9
refraction, So, after emerging from = 2.97 ´ 10-10 J = 3 ´ 10-10 J 44. Except III all statements are correct.
glass slab through opposite parallel Friction is needed for turning on banked
39. The coordinates of centre of mass are
faces, they appear at two different track also.
points and move in two different parallel
2 ´ 0 + 2 ´ 10 + 2 ´ 10 + 2 ´ 0 45. de-Broglie wavelength,
direction. x CM =
2 + 2+ 2 + 2 h 1
l= i.e. l µ
33. Shunt resistance, 2mK K
40
I gG I gG = = 5 cm
S = Þ I - Ig = 8 l1 K2 1 ´ 10-9 K2
I - Ig S \ = Þ =
y CM =
2 ´ 0 + 2 ´ 0 + 2 ´ 10 + 2 ´ 10 l2 K1 05. ´ 10-9 K1
I G 35
- 1= = = 10 2+ 2+ 2+ 2 Þ K 2 = 4K1
Ig S 3.5 40
= = 5 cm \ Addition of energy = K 2 - K1
I I 1 8
= 11 Þ g = = 4K1 - K1 = 3K1
Ig I 11

MODULE 2
www.jeebooks.in

CHEMISTRY
46. (CH3 ) 3 CCHCl 2 + NaOH ¾® 51. In complexes, where Mn(II) is present, Cl
(CH3 ) 3 CCH(OH) 2 configuration of metal ion is d 5 . The ½
56. CH3 ¾ C ¾ CH3 + NaOH ¾®
arrangement of electrons in these ½
¯ - H 2O
complexes can be shown as : Cl
(CH3 ) 3 CCHO
eg O
­ ½½ D
a - C with no H d-d transition is spin
CH3 ¾ C ¾ CH3 ¾¾¾¾® CHI3 +
forbidden (∆s=0) and 3I 2 + NaOH (Yellow ppt. )
47. Polymeric hydrides are formed by laporte forbidden
elements having electronegativity in the (∆ l=±1 forbidden) CH3 COONa + 3H2O+3NaI
range of 1.4 to 2.0. Thus, among the t2g 57. Problem is based on concept of Inductive
following MgH2 is a polymeric hydride. effect and electronegativity of element. As
High spin complex
H H F is most electronegative element with
Mg Mg In high spin complexes, d-d transition smallest size rather than ¾ OH and
H H requires reversion of spin which is ¾ NH2 where O and N has larger size as
n
against spin selection rules and due to well as lower electronegative than F.
48. Orlon is a polymer of vinyl cyanide or this reason d-d transition are spin ¾ F > ¾ OH > ¾ NH2
acrylonitrile. The reaction is as follows : forbidden and colour is of only about Electronegativity decreases.
1/100 the intensity if transition is ® Inductive effect decreases.
H CN allowed. Hence, correct order is (c).
n 52. 1 L solution of acetic acid which weight While all other are conceptually very wrong.
58. SO2 bleaches the colour of flowers by
H H = 1.02 kg
reduction while Cl 2 bleaches by oxidation.
Vinyl cyanide It contains 2.05 M acetic acid with molar
mass of 60.05 g/mol or 123.1 g. 59. Lassaigne’s method is used to qualitative
Polymerisation analysis of C, halogen, N, S only.
then mass of solvent
60. The structure of perchloric acid (HClO 4 ) is
= 1.02 kg-0.1231 kg
CN σ
O—H
= 0.8969 kg σ
—CH2—CH— 2.05 mol
n Molality (m) = = 2.29 mol kg -1 σ Cl σ
0.8969 kg π
πσπ
Polyacrylonitrile (Orlon)
53. Potassium permaganate is a powerful
49. Electron releasing group increases oxidising agent in neutral, alkaline or 61. Alcohols have stronger intermolecular
basic strength while electron acidic solution because it liberates H-bonding and ethers have weaker
withdrawing group decreases basic nascent oxygen. The aqueous solution intermolecular H-bonding. Therefore,
strength. Moreover, electron releasing of KMnO 4 can be decolourised by ethers are more volatile than alcohols.
and withdrawing effect is more C2O 42 - , HSO -3 and SO 23 - while CO 23 -
prominent at ortho-position. Thus, the
62. Due to adiabatic expansion in Joule
cannot decolourise KMnO 4 in aqueous Thomson’s experiment, kinetic energy
correct sequence is as follows : solution.
NH2 NH2
(internal energy) of the gas molecule is
54. D-mannose and D-glucose differ at used to break down the intermolecular
forces (van der Waals’ forces) due to this
configuration at C-2 only. Hence, they
> > internal energy of gas decreases.
are epimers. The structure of
Hence, corresponding temperature of
D-mannose and D-glucose are as the molecule also decreased.
CH3 follows :
63. This reaction does not involve carbonium
NH2 NH2 CHO CHO ion formation, thus no rearrangments
NO2 are possible to affect the result of reaction.
H C OH HO C H
> H+ +
H3C C ==CH2 CH3—C—CH2
HO C H HO C H
NO2 H C OH H C OH CH3 CH3 H
OCH3
50. Vitamin A is a group of unsaturated H C OH H C OH –H+
nutritional organic compounds, that CH3—C—CH3
O—CH3
includes retinol, retinal, retinoic acid CH2OH CH2OH CH3
and several provitamin A carotenoids, D-glucose D-mannose H
among which beta carotene is the most 64. Stability of +1oxidation state increases
important. Vitamin A is the vitamin 55. Copper lies below hydrogen in the from top to bottom due to inert pair effect,
whose deficiency may cause night electrochemical series and hence does because of reductant in the participation
blindness. not liberate H2 from acids. The other of inner s-electrons to participate in bond
three options are correct. formation.
MODULE 2
www.jeebooks.in
266 NEET Test Drive

65. R (NH3 )Br ¾® R (NH3 ) +3 + Br - 72. Melting point of alkanes increases with 78. The evaporation of NH3 gives
Alkyl trimethyl ammonium ions increase in molecular weights but hexammoniates of the metals
aggregates to form cationic micelles. alkanes with even number of carbon [M (NH3 ) 6 ] 2 + .
atoms have higher melting point than the
66. Greater the electronegativity of central proceeding and succeeding members 79. The basic formula of unit of rate
metal ion, greater the electron density constant expressed by
due to their well packed structures.
accepting ability. Hence, greater is the Rate mol L-1 s -1
73. 2NH3 N2 + 3H2 k= =
Lewis acidic strength. Thus, the correct
order of Lewis acid strength is.
-
By dividing the equation by 2
[R ] n [mol L-1] n
1- n
k=é
mol ù
BX 3 > AlX 3 > GaX 3 > InX 3 1 3
N2 + H2 s -1, where n = 0,1, 2, 3
NH3 - êë L úû
67. In presence of H2O 2 , reaction 2 2
- d [NH3 ] 2d [N2 ] 80. The correct balanced equation when
undergoes through free radical Rate = =+
pathways. dt dt CuS is dissolved in dil. HNO 3 is
· · =+
2 d [H2 ] 3CuS + 8H+ + 8NO -3 ¾® 3Cu(NO 3 ) 2
H ¾Br + H2O 2 ¾® H + Br 3 dt + 3S + 4H2O + 2NO
H For zero order reaction,
H - d [NH3 ] 2d [N2 ] 2 d [H2 ] 81. Reaction sequence is as follows :
So, = =
dt dt 3 dt CH3 C C CH3
= 2.5 ´ 10 mol L- 1s - 1
-4
Br2/CCl4
d [N2 ]
\ Rate of production of N2 =
H Br -4 dt Br
H aq. KOH 2.5 ´ 10
Br = mol L- 1s - 1
OH 2 CH3 C C CH3
= 1.25 ´ 10- 4 mol L- 1s - 1
Br
68. Half-cell reactions are : d[H2 ]
\ Rate of production of H2 = Br2 / CCl4
2+ - dt
(i) Zn ¾® Zn + 2e ; E ° = + 0763
. V
3
(ii) Ag + + e ¾® Ag; E ° = 0.799 V = ´ (2.5 ´ 10- 4 mol L- 1s - 1) Br Br
2
From Eq. (i) and (ii), we get = 3.75 ´ 10- 4 mol L- 1s - 1 CH3 C C CH3
Zn + Ag + ¾® Zn2 + + Ag; 74. The order of ligand field strength is : Br Br
E ° = 0.763 + 0.799 = 1.56 V CN- > NH3 > H2O > Cl - .
82. According to Bohr model, radius of
69. CH3 CH2CHCH 3 75. Unlike NaHCO 3 , which is sparingly hydrogen atom
½ –Brs soluble in water, KHCO 3 is fairly soluble 0.529 ´ n 2
Br rn = Å
in water. Thus, when CO 2 is passed Z
r
CH3 CH2 CHCH 3 through ammoniated brine, NaHCO 3 where, n = number of orbit
¯ -H+ gets precipitated, while KHCO 3 does not
Z = atomic number
CH3 CH2 CH== CH 2 + CH3 CH== CHCH 3 get precipitated when CO 2 is passed
But -1-ene, less substituted But -2 -ene, more substituted through an ammoniacal solution of KCl. +1 0
83.
I. Zn + 2 H+ ¾® H2
I II
However, it can be made by passing
Stability of II > I, hence II (i.e. CO 2 into a solution of potassium (change in oxidation number)
hydroxide. Evaporation and subsequent +
II. Zn + H + SO 24 - ¾® Zn2 + + SO 2
but-2-ene) is the predominant product.
ignition gives the carbonate. ­
70. QFor third order reaction, + - + - +1 + H2O
KO H + CO 2 ¾® KHCO 3 ­
Rate1 = k [A] 3 +1
On increasing the concentration of A 2KHCO 3 ¾® K 2+ CO 32 - + H2O + CO 2 ­ (No change in oxidation number)
+
by 2 times, new rate will be 76. 2NaOH + H2 SO 4 ¾® Na 2 SO 4 + 2H2O III. Zn + H + NO 3- ¾® Zn2 + + NH+4 + H2O
­
Rate2 = k[2 A] 3 +1
­
MNaOH VNaOH = MH 2 SO4 VH 2 SO4 +1
Rate2 8
\ = 100 mL ´ 0.5M (No change in oxidation number)
Rate1 1 MNaOH = = 0.05 M Thus, only in (I) oxidation number of
100 mL
Thus, rate will increase by a factor of 8. hydrogen changes.
Amount of NaOH per litre
71. CH3 CCl3 84. As DH remains same by different paths,
= MNaOH ´ molecular mass of NaOH.
it is a state function, As q changes by
3Cl2, ∆ 77. The hybridisation is sp 2d with one different paths, it is a path function.
–3HCl unpaired electron. 85. As we know that, ionic product of water
Toluene (X) [(Cu (NH3)4]2+= is given by [H+ ] [OH- ] = 1 ´ 10- 14 = Kw
CH3 CCl3 NH3 (aq) + H+ (aq) -NH4+ (aq)
+
[NH4 ]
Br2/Fe K1 = = 2.8 ´ 108
Zn/HCl NH3 NH3 NH3 [NH3 ] [H+ ]
Br Br NH3 (aq) + H2O (l ) - NH4+ (aq)
(Z ) (Y ) + OH- (aq)
meta-bromotoluene NH3
MODULE 2
www.jeebooks.in
Prep Catalysis for NEET ~ Mock Test 8 267

[NH+4 ] [OH- ] Since, bond order of He2 molecule is rH 2 MA


K2 = (Q [H2O] = 1) \ =
[NH3 ] zero. Hence, it cannot exist in stable rA MH 2
state.
[NH+4 ] [OH- ] Since, rH 2 = 6r A
\ K1Kw =
[NH3 ] 87. Number of milliequivalents of
NaOH = 800 ´ 0.05 = 40 MA M
= 2.8 ´ 108 ´ 10- 14 = 6 or A = 36
Number of milliequivalents of HCl MH 2 MH 2
= k2
= 200 ´ 0.1= 20
\ K 2 = 2.8 ´ 10- 6 or M A = 36M H 2
Total volume = 200 + 800 = 1000 mL MH 2 = (36) (2) = 72 g mol - 1
86. According to molecular orbital theory, Number of milliequivalents of NaOH
the bond order of the molecules given present in 89. PCl 5 - PCl 3 + Cl 2 ; Dng = 1
are as follows :
40 - 20 = 20 K p = KC (RT )
Dn g
H2 = s1s 2
pOH = - log [OH - ]
2-0 Þ K p = KC RT ¢ (QK p = 2KC )
\ B.O = =1 = - log [2 ´ 10- 2 ]
2
= 1.7 Þ 2KC = KCRT ¢
H+2 = s1s 1 RT = 2
also pH + pOH = 14
1- 0 2 2
\ B.O = = 05. pH = 14 - pOH T = = = 24.36 K
2 R 0.0821
= 14 - 1.7 = 12.3
H-2 = s1s 2 s* 1s 1 90. As we know that,
2-1 88. According to Graham’s law, 1 2
\ B.O = = 05 . K.E. = murms
2 1 2
r µ (at constantT and p)
He2 = s1s 2 , s *1s 2 M 2E
or = u rms
2-2 m
\ B.O = =0
2

BIOLOGY
91. The option (d) is correctly matched 95. All the statements are correct. Other type of restriction enzymes
pair. A common weed Catharanthus Sir Francis Galton, a British Scientist sourced from bacteria like Escherichia
roseus is the source of two anti-cancer who coined the term eugenics is known coli and Anabaena viriabilis make cuts in
drugs, vincristine and vinblastine. as the ‘Father of Eugenics’. It is a two strands of DNA at two different sites,
branch of science that deals with the producing two strands with protruding
92. Antibiosis refers to the production of improvement of human race genetically ends. These strands are called sticky
chemical poisons which inhibit the either by preventing people with ends. These are called so because of
growth of other organisms. They are inferior and undesirable traits from their ability to form hydrogen bond with
mostly produced by bacteria, moulds reproduction (negative eugenics) or by their complementary cut part.
and actinomycetes. Its examples encouraging people with better and 98. Trichoderma species are free-living
include Chlorella and Microcystis. desirable traits to produce offspring fungi. These species of fungus can be
(positive eugenics). used as a biocontrol agent.
93. Individuals with Cri du chat syndrome
have a cry similar to the meowing of a 96. Erythrocytes and thrombocytes are It can suppress the growth of the
cat. All 46 chromosomes are present in non-nucleated blood cells. pathogen population in the rhizosphere
Cri du chat syndrome, except a small through competition and thus reduce
Monocytes, lymphocytes are
disease development.
portion of the short-arm of agranulocytes, i.e. without granules
chromosome no. 5. and monomorphic nucleus, while 99. Only statement (b) is correct. The idea of
eosinophils are granulocytes with cloning was first suggested by German
94. In order to determine whether the embryologist Hans Spemann in 1938. The
granules and bilobed nucleus.
particular element is essential or production of living structures or organisms,
non-essential, each and every element 97. The option (b) is incorrectly matched which are genetically identical to their
must follow the criteria for essentiality pair. It can be corrected as follows parents is called cloning. DNA fingerprinting
as given above (in options a, b and c). Haemophilus aegypticus – Blunt ends. is a technique to identify a person on the
Option (d) is not a criterian for the The restriction enzymes from different basis of person’s DNA specificity. DNA
essentiality of mineral nutrients for bacteria cut the DNA molecule in two fingerprinting was developed by Sir Alec
plants. It is not necessary that plant different ways. Jeffreys.
must absorb mineral nutrient from the Restriction enzymes sourced from 100. The statement (a) is correct regarding
soil only. Plants obtain carbon, bacteria like Haemophilus aegypticus carrying capacity. It is the maximum
hydrogen and oxygen from CO 2 (air) and Serratia marcescene cut across number of individuals that can survive in
and H2O while the other elements are both the strands of DNA at the same an ecosystem.
obtained from the soil itself. position. Thus, DNA segments with Species diversity is the number of
blunt ends are produced. individuals per species in an ecosystem.

MODULE 2
www.jeebooks.in
268 NEET Test Drive

101. In the given figure, 109. Narrow utilitarian aspect of biodiversity Detritus food chain is made up of
A – White matter conservation includes pine resin, microbes (fungi, bacteria) and then to
B – Grey matter gums, tannin sources and dyes. detritus-feeding organisms (detritivores)
C – Central canal Pine resin is obtained from Pinus. and their predators. It begins from dead
D – Anterior funiculus Gums are obtained from Butea organic matter.
102. Arctic tern is not a marine bird. It is a monosperma. 118. The symptoms of Mn toxicity are brown
champion long distance migrant, Tannin sources include Acacia. spots around chlorotic veins. In addition,
travels a distance of 11,000 miles to Dye sources include Acacia catechu. toxicity of manganese causes deficiency
reach Antarctica in winters. Its
110. The Bhakarwal breed of sheep is of Fe3 + , Mg 2 + and Ca 2 + .
scientific name is Sterna paradisaea.
majorly found in Jammu and Kashmir. 119. Non-endospermic or exalbuminous
103. The statement (b) is incorrect. It can be The Bhakarwal breed of sheep is seeds are those in which no endosperm
corrected as follows named after the nomadic tribe which is left at maturity. Cotyledons of such
An International Human Genome reared it. Its wool is used in making seeds are thick due to storage of food.
Project was started in 1990 with the shawls. For example, in plant species like
triple aim of mapping the whole human Tamarindus and Orchidus, the
111. calcitriol is the active form of vitamin endosperm is utilised by embryo and no
genome at the level of nucleotide
cholecalciferol (D3 ). It regulates endosperm is left at maturity.
sequences, to store the information
concentation of Ca 2 + and phosphorus
from project in databases and develop 120. Histamines, prostaglandins and kinins
and accelerates bone formation.
tools for data analysis, to solve ethical, are responsible for an inflammatory
legal or social problems which may 112. The option (d) is incorrectly matched. response. An infection or tissue injury
arise from the project. The project was Troponin is a globular protein of usually causes redness, swelling, pain
completed on June 26, 2000. muscles that binds tropomyosin which and production of heat which is known
is absent in smooth muscle cells. as inflammatory response.
104. The statement (b) is incorrect
regarding Down syndrome. Down 113. A recessive allele is not expressed if a All of the above are important chemical
syndrome results due to trisomy corresponding allele (which is mediators released by a variety of cells
(2n + 1) of chromosome 21. dominant) is present. In case, where, in response to tissue injury.
Non-disjunction of chromosome 21 only a single copy of a recessive allele is 121. The cattle or goats do not browse on the
results into trisomy of chromosome 21. present, its expression may lead to plant Calotropis because it produces
Hence, statement (a) is correct. Down failure of individual to survive. This is highly poisonous cardiac glycosides.
syndrome, also known as 21 trisomy, is known as monosomy. They are organic compounds.
not a monosomic condition. 114. The ras gene family encodes signal 122. Excess of amino acids are not stored in
So, statement (b) is incorrect. transduction molecules that are the body but are deaminated and used
Children with Down syndrome have associated with the cell membrane and to produce energy.
affectionate and loving nature and regulate cell growth and division.
Individuals have a characteristic Mutated (changed) forms of the ras
123. The statement (III) is incorrect. Progeny
prominent epicanthic fold in each eye. selection is mainly employed in
gene are found in the human tumours.
cross-pollinated crops and it overcomes
105. The statement III is incorrect. It can be 115. The eyes of Octopus and mammals some of the demerits of mass selection. It
corrected as follows appear similar but have different is based on the principle of testing the
Fertilisation between different species structure and origin. Because of progenies of selected plants, and the final
is prevented by fertilizin as the reaction difference in structure and origin selection is based on this test.
between fertilizin and antifertilizin is despite similar appearance, eyes of
species specific. Octopus and eyes of mammals are 124. The statement (c) is incorrect regarding
106. An indicator of lead in water is called analogous. Eyes of an Gram-positive bacteria. The basal body
Octopus and a mammal both differ in of flagellum contains two rings in
Chladophora glomerata. Lead causes
retinal position but their functions are Gram-positive bacteria.
retardation of mental growth in children.
It also causes damage of kidney, liver same. Gram positive bacteria retain Gram stain.
and gastro-intestinal tracts. The lipid content is very low due to the
116. The bead-like thickened portions of
absence of outer membrane.
107. Endosperm culture would not be leptotene chromosomes are known as
desirable for the commercial chromomeres. Chromomeres can be 125. The option IV is incorrectly matched.
production of pomegranate. visualised when chromosomes are Multipolar neurons are neurons with
Endosperm culture is used for triploid highly condensed. The tips of unidirectional flow of information, but
plant production. As triploids are chromosomes are called telomeres. with one axon and many dendrites. They
usually seed sterile, so this method is Centromere is where two chromatids occur in the nervous system of adult
undesirable for plants where seeds are are attached to each other. vertebrates.
commercial products. 117. The statements I and II are correct Bipolar neurons are found in the retina of
regarding detritus food chain. eyes, olfactory epithelium, cochlear
108. The statement IV is incorrect. It can be
ganglia, etc.
corrected as follows The series of organisms eating one and
Typhus is caused by Rickettsia 126. Such plants are ornithophilous plants.
being eaten by the other is called the
prowazekii. It is also called classical or They have flowers of larger size and
food chain.
European typhus. possess nectaries.

MODULE 2
www.jeebooks.in
Prep Catalysis for NEET ~ Mock Test 8 269

They produce large amount of honey Many algae grow on the shells of 145. Species is the smallest taxa and
and are generally odourless. The molluscs, turtles and fins of fishes and kingdom is the largest taxa.
pollination by the agency of birds is are called as epizoic algae, e.g. In biological classification, taxonomic
known as ornithophily. Cladophora. rank is the relative level of a group of
127. Root tip is an important site of cytokinin Endozoic algae are found inside the organisms (a taxon) in a taxonomic
synthesis. Evidences suggest that body of aquatic animals, e.g. hierarchy. Examples of taxonomic ranks
translocation of cytokinin takes place in Zoochlorella. in correct order from smallest taxon to
the upward direction. 135. The statements II and III are incorrect. largest taxon are species, genus, family,
Cytokinin is a basic hormone and They can be corrected as follows order, class, division.
purine (adenine) derivative. It primarily Urine is acidic with pH around 7.0 and 146. The incorrect match is option (b).
acts on cell division. it consists of 96% of water.
Maltose is a disaccharide made up of
128. Graafian follicle contains large central 136. Option (b) is correct. I ® III ® II ® IV
two glucose units, which are joined by
follicular cavity called antrum. It is filled In the formation of genomic library, first a-1, 4-glycosidic linkage. This sugar is
with a fluid called liquor folliculi. the entire genome is randomly broken found in germinating seeds and is also
129. The tertiary structure of a protein into fragments with the help of produced during the digestion of starch
molecule results from electrostatic restriction endonucleases. by an enzyme a-amylase.
bonding, hydrophobic bonding and These fragments are then joined to 147. The variation in chromosome structures
hydrogen bonding. suitable cloning vectors in vitro. The is due to deletion, duplication,
recombinant DNA molecules, thus translocation and inversion.
Tertiary structure refers to the
produced are then introduced into
three-dimensional structure of Deletion is the absence of a
prokaryotic cells inside which these
monomeric and multimeric protein chromosomal portion.
r DNA molecules replicate to form the
molecules. Duplication is the presence of one block
clones. Thus, producing the genomic
The a-helixes and b-pleated sheets are of genes more than one time.
library.
folded into a compact globular Translocation is a kind of rearrangement
structure. 137. Light is an external factor that affects of genes. In inversion, a section of a
transpiration. Light increases the rate
130. The leaf base which completely chromosome becomes changed after
of transpiration because in light,
encircles the stem is known as rotation through 180°.
stomata are open. In dark, due to the
amplexicaul. Its example includes
closure of stomata, stomatal 148. The option (c) is incorrectly matched.
Polygonum.
transpiration is almost stopped. The example of peripatric speciation is
131. Streptokinase is produced by the Australian bird, Pectroica multicolor. It is
138. Metanephric kidney is the most
bacterium Streptococcus. It is used as
advanced kidney. It is also called as a subform of allopatric speciation in
a clot buster. It is used in dissolving
posterior kidney and consists of loop of which new species are formed in
blood clots in heart patients.
Henle. It is found in mammals. isolation, smaller peripheral populations
Amylase is a starch digesting enzyme. that are prevented from exchanging
139. The function of companion cells is to
Statins are inhibitor of enzyme HMG genes with the main population.
help the sieve tube members in
Co-A reductase of liver. Cyclosporin Darevskia radios is an example of
translocation of food material. These
are immunosuppressive.
thin-walled elongated cells are living parapatric speciations.
132. The absorption of the digested food and possess cytoplasmic content with 149. Escherichia coli and Daphnia
mainly occurs in small intestine. a conspicuous nucleus. These cells are (Crustacean) both are the indicators of
The villi present in the intestine provide absent in pteridophytes and water pollution.
large surface area for absorption. gymnosperms. They are affected by water impurities
Lymph capillary present in a villus is 140. Staphylococcus and Streptococcus are and their number increases in polluted
called lacteal that is concerned with Gram-positive cocci. They are found in water.
the absorption of fat. soil, skin and mucous membranes of Mycorrhiza are an association of fungi
133. The bacteria present in the alimentary animals. with roots of higher plants.
canal of the humans could take up the 141. Rafflesia and Orobanche both are total 150. Cryopreservation is an example of ex situ
antibiotic resistance gene, which is root parasites. They depend on other conservation strategies. It is mostly used
present in the GM food. These bacteria plants for water and minerals.
for conserving vegetatively propagated
would then become resistant to the
142. Mud puppy is an aquatic salamander crops.
concerned antibiotic. Thus, these of genus Necturus with persistent gills.
bacteria could become difficult to Ex situ conservation literally means,
manage. 143. Cephaleuros virescens is an example offsite conservation. It is the technique of
of algal parasitism. It causes red rust of conservation of biodiversity outside their
134. Algal forms which grow on the other tea.
aquatic plants are called epiphyte, e.g. natural environment. In situ conservation,
Oedogonium. 144. Reproduction without multiplication is on the other hand is conservation of
Algae that grow within the plants are shown by endospores. It is a method to biodiversity inside their natural
called endophytes, e.g. Anabaena. overcome unfavourable conditions. environment.

MODULE 2
www.jeebooks.in
270 NEET Test Drive

151. In phycocyanin, light absorption is Gymnosperms are vascular, naked 167. Both the options (a) and (b) are
maximum at 600 nm. seeded plants. incorrectly matched.
The maximum absorption occurs In angisoperms, seeds are Latex cells are found in Calotropis while
between 450 and 700 nm and well-protected in fruits. latex vessels are found in Papaver. They
chlorophylls are responsible for it. 159. Endospore-forming rods and cocci are specialised to contain latex. They are
include Bacillus and Clostridium. They multinucleated.
152. In the given figure,
are Gram positive bacteria. They are Latex cells are non-articulated laticifers
A – Pharyngeal tonsil
found in soil and animal gut. while latex vessels are articulated
B – Eustachian tonsil laticifers.
C – Palatine tonsil 160. Option (c) is incorrectly matched. The
D – Lingual tonsil pineal gland is located on the dorsal 168. Broca’s area controls movement of
side of forebrain. The pineal gland tongue. It is involved in speech
153. All the statements regarding soil secretes melatonin, sarotonin and production and is situated in left cerebral
horizons are correct. A horizon is also adrenoglomerulotropin. hemisphere in right-handed people.
called topsoil because it is the topmost
161. Pseudomonas putida is involved in the 169. Avery performed transformation
part of soil. B is called subsoil.
digestion of hydrocarbon of crude oil. experiments. He used DNA extracts from
C-horizon shows the presence of water
table while D-horizon contains This soil bacterium uses a wide variety virulent and non-virulent type of bacteria.
unweathered parent bed rocks. of organic compounds as carbon and Using enzyme DNAase, he was able to
energy sources. It can be used in the prove that DNA was the transforming
154. All the statements are correct digestion of hydrocarbon of crude oil. factor.
regarding problems of GM foods. The
problems of GM foods include the 162. The siphonogamous plants refer to the 170. Many codons are synonyms of one
transgene products that may cause plants where male gametes are another and they usually differ in only the
toxicity and/or produce allergies. non-motile and are carried by pollen third base. Example GUU, GUC, GUA,
tube for fertilisation. The seed plants GUG are the codes for valine. This
The enzyme produced by the antibiotic
are siphonogamous. flexibility with respect to the third base of
resistance gene could cause allergies
because it is a foreign protein. 163. Spherosomes or oleosomes are small a codon is called wobble effect.
Bringing GM food to market is a lengthy cell organelles bounded by single 171. Potometer method of measurement of
and expensive process and of course membrane which take part in storage transpiration is based on the assumption
agri-biotech companies wish to ensure and synthesis of lipid. These are small that the rate of water uptake is
a profitable return on their investment. spherical vesicles of 0.2-0.8 m. They approximately equal to the rate of
155. All the statements are correct regarding are formed from smooth ER. They are transpiration. The apparatus used for
secondary growth in dicot stems. found in only plant cells. such purpose is called potometer and
the method is called potometer method
The vascular cambial ring develops as 164. X-ray crystallographic data collected of measurement of transpiration.
a circular ring since beginning. The by Franklin and Wilkins played a major
vascular cambial ring is completed by role in the double-helix model of DNA 172. Adrenal cortex is subdivided into three
both the primary (intrafascicular) strip proposed by Watson and Crick. zones viz., zona glomerulosa which
and secondary (interfascicular) strip. Erwin Chargaff gave base equivalence produces mineralocorticoids, zona
The vascular rays are narrow since rule or Chargaff’s rule. Meselson and fasciculata produces glucocorticoids
beginning. Outward layer of the cortex stahl proved that DNA replication was and zona reticularis which produces
gives birth to cork cambium semiconservative. gonadocorticoids.
(phellogen). 173. All the given enzymes are used in various
165. Pituitary gland, also called master
156. The statement (d) is incorrect. processes of genetic engineering.
gland, is situated below the
Climbers possess organs for clinging Restriction enzymes are often referred to
hypothalamus and connected to the
to the support. In climbers, the stem is as molecular scissors because they can
brain by a stalk. It lies in the cavity
weak and flexible but is unable to coil cut DNA. DNA ligases called as
called sella turcica of sphenoid bone molecular binder because they can join
around an upright support by itself. It
above the roof of nasal cavity. two nucleotides in a DNA strand. Lyases
requires the help of certain clinging
structures. 166. The causes of seed dormancy include are used to open the cells or dissolve the
the production of compounds that cell wall. Synthetases help in in vitro
Clinging organs are absent in twiners.
restrict germination. Other causes synthesis of DNA.
157. There is no release of CO 2 in
include hard seed coat and the 174. The globular head of meromyosin is an
glycolysis. CO 2 may be produced only
impermeability of seed coat to oxygen. active ATPase enzyme and has binding
after glycolysis, if pyruvate enters
Hard seed coat is the cause of sites for ATP and active sites for actin.
anaerobic respiration.
dormancy as it provides mechanical 175. After transcription is complete, the newly
158. Bryophytes are non-vascular plants
resistance to the embryo growth. synthesised mRNA strand undergoes
lacking seeds.
Impermeability of seed coat to oxygen processing at both 5¢ and 3¢ end to
Pteridophytes are vascular plants that produce a mature mRNA transcript.
also causes seed dormancy because
lack seeds.
sufficient oxygen is required for the 176. A polysome is a number of ribosomes on
germination of seeds. the same transcript.

MODULE 2
www.jeebooks.in
Prep Catalysis for NEET ~ Mock Test 8 271

A number of ribosomes may attach to 178. Pineal gland can be found deep in the through which gaseous exchange
an mRNA transcript. These ribosomes brain at the top of third ventricle, where occurs.
simultaneously undergo translation it has close communication with the Cutaneous respiration is seen in leech in
(protein synthesis) process, each cerebrospinal fluid. Due to its location, which moist skin acts as respiratory
being at different stages of the it is also called epiphysis cerebri. surface.
process. TCT (Thyrocalcitonin) is secreted when
180. The stage between the two meiotic
177. The crop plants engineered for calcium level is high in blood.
divisions is interkinesis. It is generally
herbicide tolerance and weeds would It decreases the calcium level by short-lived. During this process, no
cross-breed, resulting in transfer of the supressing the release of calcium ions replication of DNA occurs. It is
herbicide resistant genes from the from blood. necessary for bringing true haploid DNA
crops into the weeds. These 179. Tracheal respiration is found in insects. in daughter cells. It is in fact considered
‘superweeds’ would then be herbicide It is an intercommunicating tube as incipient interphase.
tolerant as well.

SCORE CHART
No. of Correct Answers : A .................. No. of Incorrect Answers : B ..................
Total Marks : x = (A × 4) – (B × 1)
Scores and Expected Rank : If the score lies above 665, then rank will be in between 1-50. For other scores, rank
estimations are given below
Score Rank
664 – 642 51 – 150
641 – 636 151 – 250
635 – 630 251 – 400
629 – 625 401 – 500

MODULE 2
www.jeebooks.in

PREP CATALYSIS
for NEET
Full Length Mock Tests for NEET to Make You Ready to Face the Challenge

MOCK TEST 9 (With Solutions)

Duration : 3 Hours Max. Mark : 720

Instructions
The test is of 3 hours duration and Test Booklet contains 180 questions. Each question carries 4 marks. For
each correct response, the candidate will get 4 marks. For each incorrect response, one mark will be deducted
from the total scores. The maximum marks are 720.
The question paper contains three parts of Physics, Chemistry and Biology respectively.
Part A contains 45 Questions from Physics section. Part B contains 45 Questions from Chemistry section. Part
C contains 90 Questions from Biology.

PART A PHYSICS
1. A solid cylinder of radius 5 cm and mass 300 g 4. Radius of an air bubble at certain depth of Indian
rolls down an inclined plane (1 in 20). The velocity ocean is r and it becomes 18r, when air bubble rises
of cylinder after 5s will be to the top surface of the ocean. If t cm of water be
(a) 1. 63 ms − 1 (b) 1. 56 ms − 1 the atmospheric pressure, then the depth of the
(c) 2 ms −1
(d) 3.26 ms − 1 ocean is
(a) 3835 t cm (b) 3400 t cm (c) 4852 t cm (d) 5831t cm
2. The work done by a gas is maximum, when it
5. A battery of emf 8V and internal resistance r is
expands
connected to a load resistance of 5Ω. Then, power
(a) isothermally (b) adiabatically
at load resistance will be maximum if the value of
(c) isentropically (d) isobarically internal resistance r is
3. When a charged particle moving with velocity v is (a) 5 Ω (b) 10 Ω (c) 15 Ω (d) 20 Ω
subjected to a magnetic field of induction B, the 6. A well with vertical side and water at the bottom
force on it is non-zero. This implies that resonates at 3 Hz and at no other lower frequency.
(a) angle between them is either zero or 180º The air in the well has density 1 . 10 kg m − 3 and
(b) angle between them can have any value other than zero
bulk modulus of 1 . 32 × 105 Nm − 2, then the depth of
or 180º
(c) angle between them is necessary 90º
well is
(d) angle between them can have any value other than 90º (a) 30 m (b) 29 m (c) 25 m (d) 32 m

MODULE 2
www.jeebooks.in
Prep Catalysis for NEET ~ Mock Test 9 273

7. A brass sphere of mass 5 kg is heated in a 14. Match the corresponding entries of Column I with
furnance to a temperature 500ºC and then placed Column II and choose the correct option from the
on a large ice block. The mass of ice that will melt codes given below.
in this process will be (specific heat of brass
Column I Column II
= 500 J kg − 1 ° C − 1 and heat of fusion of ice
(Rotation) (Moment of inertia)
= 336 kJ kg − 1)
A. Thin rod of length L about P. MR2
(a) 5.25 kg (b) 3.86 kg
an axis perpendicular to 2
(c) 2.56 kg (d) 3.72 kg
the rod and passing
8. A planet of mass m revolves around the sun of through mid point
mass M in an elliptical orbit. The maximum and B. Circular disc of radius R Q. ML2
minimum distances of the planet from the sun are about an axis passing 12
r1 and r2, respectively. The time period of the through the diameter
planet in terms of r1 and r2 is
(a) T ∝ (r1 + r2 )2 C. Hollow cylinder of radius R R. 2 MR2
about an axis passing 5
(b) T ∝ (r1 + r2 )3
through the axis of cylinder
(c) T ∝ (r1 + r2 )1/ 2
Codes
(d) T ∝ (r1 + r2 )3 / 2
A B C A B C
9. A 6 µF capacitor is charged to 360 V. If its plates (a) Q P R (b) P R Q
are joined through a resistance, then heat (c) Q R P (d) R Q P
produced in the resistor is
15. The vertical motion of a huge piston in a machine is
(a) 0.78 J (b) 0.68 J approximately simple harmonic with a frequency of
(c) 1.2 J (d) 0.39 J 0 . 50 s− 1. A block of 20 kg is placed on the piston.
10. In the case of free expansion, The maximum amplitude of the piston’s SHM, for
(i) ∆W = 0 (ii) ∆Q = 0 the block and the piston to remains together is
(a) 0.99 m (b) 1 m
(iii) ∆U = 0 (iv) ∆T = 0
(c) 1.99 m (d) 2.8 m
Correct statements are
(a) (iii) and (iv) (b) (i), (ii) and (iii) 16. The length of a rod is 40 cm and area of
(c) (i) and (iv) (d) (i), (ii), (iii) and (iv) cross-section 4 cm 2 . The Young’s modulus of the
material of wire is 4 × 1010 Nm − 2. If the rod is
11. A shell of mass 200 g is ejected from a gun of compressed by 10 kg-wt along its length, then
mass 4 kg by an explosion that generates 1.05 kJ
increase in the energy of the rod in joules will be
of energy. The initial velocity of the shell is
(a) 40 ms − 1 (a) 8. 57 × 10− 6 J (b) 2 . 36 × 10− 4 J
(b) 120 ms − 1 (c) 1.25 × 10− 3 J (d) 1.25 × 10− 4 J
(c) 100 ms − 1
17. Infinite number of charges of magnitude 6µ C each
(d) 10 ms − 1
are lying at y = 2, 4, 8, 16 … metre on Y -axis. The
12. In the figure shown, m1 = 10 kg, m2 = 6 kg, value of electric field intensity at point y = 0 due to
these charges will be
m3 = 4 kg. If T3 = 40 N, then T2 is
(a) 6. 5 × 103 N / C
T1 T2
m1 m2 m3 T3 (b) 4. 05 × 104 N / C
(c) 7 .2 × 103 N / C
(a) 13 N (b) 32 N
(d) 1. 8 × 104 N / C
(c) 25 N (d) 35 N

13. The magnetic flux through a circuit of resistance 18. The driver of a train A moving with a uniform speed
R changes by an amount ∆φ in a time ∆t. Then, of 30 ms− 1 sees another train B moving with
the total quantity of electric charge Q that passes uniform speed 10 ms− 1 on the same track in the
at any point in the circuit during the time ∆t is same direction. He immediately applies brakes and
represented by achieves a uniform retardation of 2ms −1 and and
∆φ 1 ∆φ finally stops. To avoid collision, the minimum
(a) (b)
R R ∆t distance between the trains must be
∆φ ∆φ (a) 140 m (b) 75 m
(c) R (d)
∆t ∆t (c) 80 m (d) 120 m
MODULE 2
www.jeebooks.in
274 NEET Test Drive

19. A star of mass 4 times the solar mass and radius the maximum acceleration with which monkey can
108 km rotates about its axis with an angular speed slide down without breaking the branch is
of 10− 4 rad s − 1. When the star collapses to a radius (a) g (b)
3g
(c)
g
(d)
g
at 105 km, then its angular speed is 4 4 2
(Take, solar mass = 1 . 99 × 1030 kg) 25. In Young’s double slit experiment, the fringe width
(a) 10 rad s − 1 (b) 0.1 rad s − 1 is found to 0.5 mm. If the whole apparatus is
5
(c) 102 rad s − 1 (d) 10− 2 rad s − 1 immersed in liquid of refractive index , without
3
20. Two batteries of emf 3V and 6V with internal disturbing the geometrical arrangement, then new
resistances 2Ω and 4Ω are connected in a circuit fringe width will be
with resistance of 10Ω as shown in the figure. The (a) 0.35 m (b) 0.6 mm
current and potential difference between the points (c) 0.45 mm (d) 0.3 mm
P and Q are
3V 6V 26. A rectangular bar magnet of area 20 cm 2 has
2Ω 4Ω
P Q ability to produce a magnetic field of 2.5 T at a
r1 r2 point 30 cm away from its mid point on axial line.
If it is assumed that dipole is surrounded by a
spherical region in space, then net magnetic flux
passes through the sphere is
(a) 0 (b) 0.005 Wb
10 Ω (c) 0.15 Wb (d) 0.05 Wb
3 8 16 15
(a) A and V (b) A and V 27. A charged particle is moving in a uniform magnetic
16 15 3 8
3 3 15 field, then
(c) A and 8 V (d) A and V (a) both momentum and total energy will change
16 16 8
(b) its momentum changes but total energy remains the same
21. Figure shows two processes X and Y on a ideal gas (c) its total energy changes but momentum remains the same
system. Let ∆QX and ∆QY be the heat given to the (d) both momentum and total energy remain the same
system in processes X and Y respectively, then
28. The gravitational field due to a mass distribution
is I = k / r3 in the x-direction (k is a constant). The
X
2 gravitational potential is taken to be zero at
infinity, then its value at a distance x is
p (a) k / x (b) k / 2 x
1
(c) k / x2 (d) k / 2 x2
Y
29. The power factor of the circuit shown in the
V figure is
20 Ω XL=100 Ω
(a) ∆QX = ∆QY (b) ∆QX > ∆QY
(c) ∆QX < ∆QY (d) ∆QX ≤ ∆QY
XC=20Ω
22. A positively charged particle enters a magnetic 40 Ω
field of value B $i with a velocity v $j , then the
instantaneous direction of particle will be along
(a) Z-axis (b) − Z-axis
(c) − X-axis (d) − Y-axis 220 V, 50 Hz

23. A resonance air column shows resonance with a (a) 0.2 (b) 0.8 (c) 0.4 (d) 0.6
tuning fork of frequency 256 Hz at column lengths
30. A uniform cylinder has a radius R and length L. If
32.5 cm and 112.9 cm. The end correction and
the moment of inertia of this cylinder about an axis
speed of sound in air are
passing through its centre and normal to its
(a) 4.1 cm, 77 .7 ms − 1 (b) 7 .7 cm, 411. 65 ms − 1
circular face is equal to the moment of inertia of
−1
(c) 5 cm, 224. 5 ms (d) 6.7 cm, 352 .7 ms − 1 the same cylinder about an axis passing through
its centre and perpendicular to its length, then
24. A monkey is descending from the branch of a tree (a) L = 3R (b) L = R
with constant acceleration. If the breaking (c) L = R / 3 (d) L = 2 R
strength is 25% of the weight of the monkey, then

MODULE 2
www.jeebooks.in
Prep Catalysis for NEET ~ Mock Test 9 275

31. The plane face of a plano-convex lens is silvered. If L1 L2 (L1L2 )2 L1L2


(a) (b) (c) (d)
µ be the refractive index and R be the radius of L2 L1 (L12 + L22 )2 (L1 + L2 )2
curvature of curved surface, then system will
behave like a concave mirror of radius of curvature 1
39. A radioactive substance decays to of its initial
(a) µR (b) R / µ
2 64
(c) R / (µ − 1) (d) [(µ + 1) / (µ − 1)]R quantity in 30 days. The time during which it will
1
32. The equivalent resistance between A and B is decay of its initial quantity is
128
1Ω 2Ω
A (a) 50 days (b) 38 days (c) 35 days (d) 46 days
6Ω 2Ω 40. In Young’s double slit experiment, the intensity at
centre of screen is I. If one of the slit is closed, the
B
1Ω 2Ω intensity at centre now will be
I I I
(a) 5 Ω (b) 8 Ω (c) 2 . 5 Ω (d) 6.8 Ω (a) I (b) (c) (d)
3 4 2
33. A ray of light from rarer medium strikes a denser
medium such that reflected and refracted rays
41. The angular momentum of an electron
orbiting in the third orbit of an atom is
make an angle of 90º with each other. The angle of
reflection and angle of refraction are 60º and 30º [given, h = 6.6 × 10− 34 J- s]
respectively, then refractive index of the medium is (a) 8.25 × 10− 34 J - s (b) 6. 55 × 10− 34 J - s
− 34
(a) 1.5 (b) 1.73 (c) 1.41 (d) 1.75 (c) 3.15 × 10 J-s (d) 4 × 10− 34 J - s
34. A square wire of side 2.0 cm is placed 20 cm in 42. The temperature T dependence of resistivity ρ of a
front of a concave mirror of focal length 10 cm with
its centre on the axis of the mirror and its plane semiconductor is represented by
normal to the axis. The area enclosed by the image
of wire is
(a) 7.5 cm2 (b) 6 cm2 (c) 2 cm2 (d) 4 cm2 (a) ρ (b) ρ

35. In photoelectric effect, electrons are liberated from


metal’s surface, if the incident light has a certain
T T
maximum
(a) wavelength (b) frequency (c) amplitude (d) intensity
36. An electromagnetic wave is propagating in a (c) ρ (d) ρ
medium with a velocity v = − a $j. Magnetic field
oscillates in the direction of + X -axis. Then, the
direction of oscillating electric field of T T
electromagnetic wave will be
(a) along − x-direction (b) along + y-direction 43. When a uniform solid sphere and a disc of same
(c) along − z-direction (d) along + z-direction mass and of same radius rolls down an inclined
smooth plane from rest to the same distance, then
37. The dominant mechanism for motion of charge the ratio of the time taken by them is
carriers in forward and reverse biased silicon p - n (a) 15 : 14 (b) 15 : 14
junction are
(c) 14 : 15 (d) 14 : 15
(a) diffusion in forward biased, drift in reverse bias
(b) drift in forward biased, drift in reverse biased 44. The centripetal acceleration of a satellite of mass
(c) diffusion in both forward and reverse bias 1000 kg orbiting at 1600 km above the earth’s
(d) drift in both forward and reverse bias surface is
(take, radius of earth = 6 . 4 × 106 m and
38. Two inductors of self inductance L1 and L2 are
g = 9 . 8 ms − 2)
connected in parallel and a time varying current
flows as shown in the figure. Then, the ratio of (a) 6.272 ms − 2 (b) 2 . 563 ms − 2
−2
i
currents 1 at any time t is (c) 4. 525 ms (d) 6. 562 ms − 2
i2
L1 45. Maximum efficiency of full-wave rectifier is
i1 (a) 100% (b) 81%
i i
(c) 53% (d) 42%
i2 L2

MODULE 2
www.jeebooks.in

PART B CHEMISTRY
46. t1/ 4 can be defined as the time taken for the Choose the correct option.
3 (a) I and II (b) I and III (c) I and IV (d) I, II and III
concentration of a reactant to drop to th of its
4
initial value. If the rate constant for a first order 53. An organic compound containing nitrogen atom
reaction is k, then t1/ 4 can be given as when treated with the mixture of reagent used
in Riemer-Tiemann reaction in 1 : 3 ratio, it
(a) 0.10/k (b) 0.69/k produces methyl isocyanide. The organic
(c) 0.29/k (d) 0.75/k compound is
47. van-Arkel method of purification of metals involves (a) CH3NH2 (b) CH3CH2NH2
converting the metals to a (c) CH3CONH2 (d) CH3CH2CONH2
(a) volatile stable compound 54. Consider the following groups :
(b) non-volatile stable compound
(c) volatile unstable compound —OAc ; ¾ OMe ; ¾ OSO2Me ; ¾ OSO2CF3
I II III IV
(d) None of the above

48. Choose the correct order of acidic strength of the The correct order of leaving group ability is
following. (a) I > II > III > IV (b) IV > III > I > II
OH OH COOH COOH (c) III > II > I > IV (d) II > III > IV > I

55. Ethanal reacts with HCN and the addition product


so obtained is hydrolysed to form a new compound.
This compound shows
NO2 NO2 CH3 (a) optical isomerism (b) geometrical isomerism
(I) (II) (III) (IV) (c) tautomerism (d) metamerism

(a) I < II < III < IV (b) I < II < IV < III 56. Number of open chain isomers possible for C4H 8 are
(c) I > II > IV < III (d) I < II > II < IV (a) 4 (b) 3 (c) 2 (d) 5

49. Calculated spin magnetic moment of Eu 3+ is 57. Consider the reaction equilibrium,
(a) 6.93 (b) 7.94 (c) 2.83 (d) 3.87 2SO 2( g)+ O 2( g) d 2SO 3( g); DH ° = - 198 kJ
50. Consider the following reaction: On the basis of Le-Chatelier’s principle, the
(i) CH 3MgBr condition favourable for the forward reaction is
CH 3 C CH 2 CH 2 CH 2 Cl ¾¾¾® ‘A’ (a) lowering of temperature as well as pressure
(ii) H 3O+
½½ (b) lowering the temperature and increasing the pressure
O
Here, A is (c) increasing the temperature and pressure
CH3 (d) any value of temperature and pressure
(a) CH3—C—CH2 CH2 CH2 Cl (b) CH3 C CH2CH2CH2CH3 58. What will be the temperature at which liquid and
vapour of water will be at equilibrium with each
OH
other? Given data are
H3C CH3 DH vap = 40.73 kJ mol -1
CH3
(c) (d)
O CH3 O DS vap = 0109
. kJ mol -1 K -1
H3C (a) 273 K (b) 373 K (c) 37° C (d) 110°C
51. Which of the following method is correct for the 59. For the following reaction, H 2( g) + I 2( g) d 2HI( g)
formation of benzene?
(a) Mixture of calcium benzoate and calcium acetate is heated If value of equilibrium constant is 50 at 25°C.
(b) Calcium benzoate is heated
Then, the value of DG will be
(a) - 6.964 kJ (b) - 4.964 kJ (c) -9.694 kJ (d) - 6.496 kJ
(c) Sodium benzoate is heated with sodalime
(d) All of the above method is correct 60. Minimum amount of work done required to
compress 5.00 moles of an ideal gas isothermally
52. Methyl amine reacts with HNO 2 in the presence of
from 200 L to 40 L is
HCl to form (a) + 201
. kJ (b) - 201
. kJ
I. CH 3 ONO II. CH 3 OCH 3 (c) - 201
. J (d) + 201
. J
III. CH 3 OH IV. CH 3 CHO
MODULE 2
www.jeebooks.in
Prep Catalysis for NEET ~ Mock Test 9 277

61. What is the lowest energy of the spectral line 69. Consider the reaction, M 3+ + 3 e- ¾® M ; If 0.415 g
emitted by the hydrogen atom in the Lyman of metal M is deposited at the cathode due to
series? passage of 2A of current for 965 sec. then the
5hcRH 4hcRH 3hcRH 7 hcRH atomic weight of the metal M is
(a) (b) (c) (d)
36 3 4 144 (a) 124.5 g (b) 62.25 g
(c) 18.75 g (d) 27.5 g
62. In IF7 molecule, central atom has all the bonded
electrons, i.e. no lone pair is present on the central 70. What will be the value of relative lowering of
atom. Structure of IF7 is vapour pressure when 3 g urea is dissolved in 45 g
(a) pentagonal bipyramidal of water?
(b) octahedral (a) 0.05 (b) 0.04
(c) square pyramidal (c) 0.02 (d) 0.01
(d) octahedral monopyramidal
71. Which of the following statements is correct for
63. Which of the following outer electronic ethyne molecule?
configuration may exhibit the largest number of (a) Ethyne molecule consist of two C¾C s-bonds
oxidation states? (b) It has one p-bond
(a) 3d 3 , 4s 2 (b) 3d 5 , 4s1 (c) It is a non-linear molecule
(c) 3d 5 , 4s 2 (d) 3d 2 , 4s 2 (d) Electron cloud between two C-atoms is cylindrically
symmetrical
64. A and B are ideal gases, the molecular weights of A
and B are in ratio of 1 : 4. The pressure of a gas 72. Consider the following compounds,
mixture containing equal weights of A and B is + + + +
p atm. What is the partial pressure (in atm) of B in CH2 CH2 CH2 CH2
the mixture?
p p p 3p
(a) (b) (c) (d)
5 2 2.5 4
65. Potassium superoxide is the inorganic compound NO2 OH CH3 NH2
with the formula, KO 2 . It is a yellow solid that (I) (II) (III) (IV)
decomposes in moist air. It is a rare example of a
stable salt of the superoxide ion. Which of the The correct decreasing order of stability of
following statement is incorrect regarding compounds is
potassium superoxide? (a) I > III > II > IV (b) IV > II > III > I
(a) Oxidation state of oxygen in KO 2 is - 1 2 (c) III > II > IV > I (d) IV > I > III > II
(b) Oxidation state of potassium is +1 73. Molarity of a solution obtained by mixing 800 mL
(c) It is used in preparation of breathing mask of 0.6 M HCl with 200 mL of 1 M HCl will be
(d) It is a reducing agent (a) 0.4 M (b) 1.6 M
66. Which of the following statement is not true (c) 0.68 M (d) 1.68 M
regarding silicones? 74. Which of the following is not considered as a part
(a) Silicones have high thermal stability of the tertiary structure of proteins?
(b) Silicones are chemically inert (a) Hydrogen bonds
(c) Disubstituted silicones on hydrolysis gives cyclic trimer of (b) Electrostatic interactions
silicones (c) Hydrophobic effect
(d) Trisubstituted silicones on hydrolysis gives cyclic trimer of (d) Disulphide bonds
silicones
75. When stearic acid reacts with polyethylene glycol,
67. Ozone hole is a term which refers to then the product so formed belongs to a class of
(a) hole in ozone layer
(a) cationic detergents (b) non-ionic detergents
(b) reduction in thickness of ozone layer in stratosphere
(c) anionic detergents (d) None of these
(c) reduction in thickness of ozone in troposphere
(d) increase concentration of ozone 76. An aromatic compound A(C 7 H 9 N) on reacting with
NaNO 2 HCl at 0°C forms benzyl alcohol and
68. What will be the percentage of iron (III) present in
nitrogen gas. The number of isomers possible for
Fe0.93 O ?
the compound A is
(a) 84.95% (b) 45.64%
(a) 5 (b) 3
(c) 15.05% (d) 44.36%
(c) 7 (d) 6

MODULE 2
www.jeebooks.in
278 NEET Test Drive

77. Consider the following complexes : [Ni(CN)4 ]2- (A) 84. For a reversible reaction,
and [Ni(CO)4 ] (B). Both of them possess similar N 2 + O2 e 2NO
magnetic behaviour but their geometries are
different. The geometries of (A) and (B) are Activation energy of the backward reaction is lower
(a) (A) has tetrahedral structure, (B ) has square planar than that of forward reaction. The slope of ln k
structure versus 1 T graph will be
H
(b) (A) has tetrahedral structure, (B) has trigonal planar (a) zero (b) -
structure 2.303 R
H DH
(c) (A) has square planar structure, (B) has tetahedral (c) (d) -
structure 2.303 R R
(d) (A) has tetrahedral structure, (B) has trigonal pyramidal
structure
85. OsO4 is highly toxic volatile substance. It is
518 times more soluble in CCl4 than in water.
78. Rate of hydration of the given compounds follows Product obtained on reaction of cyclopentene with
the order. osmium tetraoxide at 25°C followed by reaction
CH3 with Na 2 SO3 is
—C == CH3
(a) cis pentan-1, 2-diol (b) trans pentan-1, 3-diol
—CH== CH2 —CH == CHCH3
(c) trans pentan-1, 2-diol (d) cis pentan-1, 3-diol
I II III
86. Which of the following statements is not correct for
(a) I < II < III (b) I < III < II zero order reactions?
(c) II < I < III (d) III < II < I
(a) In [R ] vs t is straight line plot
+ (b) [R ] vs t is straight line plot
79. The hybridisation state of Co in cis-[Co(NH 3 )4 Cl 2 ]
is (c) kt = [R ]0 - [R ]
(a) d 2 sp3 (b) dsp2 (d) t 1 / 2 = [R ]0 /2k
(c) sp3 (d) sp3d 2 87. When MnO 2 is fused with KOH, a purple coloured
80. Concentrated nitric acid upon long standing turns compound is formed. The compound is
yellow brown. This is due to the formation of (a) Mn2O 4 (b) KMnO 4 (c) K 2MnO 4 (d) Mn2O 3
(a) N2O 4
88. Match Column I with Column II and identify
(b) N2O correct answer by using the code given below.
(c) NO
(d) NO 2 Column I Column II
[Reaction Name] [Product]
81. The type of isomerism exhibited by
[Co(NH 3 )5 (NO 2 )]2+ complex is A. Etard reaction 1. Cinnamic acid
(a) functional isomerism B. Reformatsky reaction 2. Benzaldehyde
(b) linkage isomerism
(c) hydrate isomerism C. Friedel-Craft’s acylation 3. Dibenzoketone
(d) coordination isomerism
D. Perkin reaction 4. a, b-unsaturated ester
82. The unit of rate constant for 1st and zero order Codes
reaction in terms of molarity are respectively
-1 -1 -1 A B C D A B C D
(a) s , Ms (b) s , M
(a) 2 4 3 1 (b) 3 4 1 2
(c) Ms -1 ,s -1 (d) M, s -1
(c) 1 2 3 4 (d) 4 3 2 1
83. Choose the correct option regarding the product of 89. If the last electron (i.e. the differentiating electron)
following reaction. occupies 5f-orbital, the element is said to belong
(i) Hg (OAc2)/H2 5f-series. In this series, ions having 5 f 0 , 5 f 7 and 5
Product f 14 are colourless. The colour of U 3+ is red which is
(ii) NaBH4
(A) (B) due to
(a) d - d transition (b) f - f transition
(a) B is a secondary alcohol
(c) crystal defect (d) L ® M charge transfer
(b) B is a primary alcohol
(c) B is a tertiary alcohol 90. Which of the following correctly represents the
(d) B is an achiral molecule molecular formula of dithionic acid?
(a) H2S2O 7 (b) H2S2O 5 c. H2S2O 4 d. H2S2O 6

MODULE 2
www.jeebooks.in

PART C BIOLOGY
91. Which of the following variations in chromosome 97. Identify correct statement with regard to different
number is not correctly paired? types of spores present in plants.
(a) Aneuploidy – 2 n ± 1 chromosomes (a) In Chlorophyceae and Phaeophyceae, motile and
(b) Monosomy – 2 n − 1 chromosomes flagellated zoospores are produced
(c) Trisomy – 2 n + 1 chromosomes (b) During unfavourable conditions, some unicellular,
(d) Tetrasomy – 2 n + 4 chromosomes uninucleate, non-motile and thin-walled spores called
aplanosporers are produced
92. Which of the following represents a random change (c) In red algae, non-motile spores known as tetraspores are
in allele frequencies over the generation? produced
(a) Bottleneck effect (b) Sewall Wright effect (d) All are correct
(c) Hardy-Weinberg effect (d) Ernst Mayr effect
98. Name the antibiotic which inhibits the formation of
93. Match the following columns and choose the cell wall in bacterial cells.
correct option from the codes given below. (a) Terramycin (b) Aureomycin
Column I Column II (c) Penicillin (d) Clavicin

A. Chlamydospores 1. Formed in lower group of 99. Consider the following statements.


fungi of Phycomycetes I. Back cross is the cross of the hybrids of
B. Zoospores 2. Non-motile structures F1-generation with either of its parents.
II. Test cross is the cross of the individuals with
C. Aplanospores 3. Exogenous spores
its heterozygous dominant parent.
D. Conidia 4. Resistant spores formed III. Reciprocal cross is the cross that could be made
under unfavourable conditions
either way or independent of the sex of parents.
Codes The incorrect statement(s) is/are
A B C D A B C D (a) Only I (b) Only II (c) Only III (d) I and II
(a) 1 2 3 4 (b) 4 3 2 1
(c) 1 2 4 3 (d) 4 1 2 3 100. The physical location of a gene in relation to other
genetic material may influence its expression. This
94. Which of the following statements regarding the is called as
transport of CO2 is not true? (a) penetrance (b) expressivity
(a) CO 2 is carried by haemoglobin as carbaminohaemoglobin (c) position effect (d) epigenetics
(b) pCO 2 is high and pO 2 is low in the tissues
(c) In the tissues, pCO 2 is high and it diffuses into blood and 101. Match the following columns and choose the
forms CO 2 and H2O correct option from the codes given below.
(d) Every 100 mL of deoxygenated blood delivers Column I Column II
approximately 4 mL of CO 2 to the alveoli
A. Plantigrade 1. Cat
95. Discolouration of toe nails, finger nails and lips to
blue colour is called as B. Unguligrade 2. Man
(a) lathyrism (b) paraplegia C. Digitigrade 3. Horse
(c) cyanosis (d) leukaemia
Codes
96. Consider the following statements. A B C A B C
I. Phenylketonuria is an autosomal recessive (a) 3 1 2 (b) 1 2 3
trait. (c) 2 1 3 (d) 2 3 1
II. Thalassemia is an autosomal recessive trait. 102. The type of endosperm in which the first division
III. Down’s syndrome is due to the trisomy of of the nucleus is followed by wall formation, but
chromosome number 21. further nuclear divisions are free divisions is
(a) nuclear type (b) helobial type
IV. Turner’s syndrome is caused due to the
(c) ruminate type (d) mosaic type
absence of one of the Y-chromosomes.
The incorrect statement is 103. Presence of four long and two short stamens can be
associated with family
(a) Only I (b) Only II
(a) Asteraceae (b) Liliaceae
(c) Only III (d) Only IV
(c) Brassicaceae (d) Fabaceae

MODULE 2
www.jeebooks.in
280 NEET Test Drive

104. Identify the incorrect statement(s). 110. Consider the following statements about lytic
I. Diseases or infections transmitted through phase of virus reproduction.
sexual intercourse are called venereal I. During lytic phase, viral genome does not
diseases. integrate with host DNA.
II. AIDS is completely curable if detected early II. The host DNA is not hydrolysed during lytic
and treated properly. phase.
III. There can be many reasons for infertility III. The host cell does not get lysed.
including psychological. IV. A number of virus particles is liberated
Choose the correct option. during lytic phase.
(a) I and III (b) II and III
Which of the above statements are incorrect?
(c) Only II (d) Only I
(a) II and III (b) I and III (c) II and IV (d) III and IV
105. Which of the following statements is true about 111. In the question given below, identify what the
members of family Cucurbitaceae? letters ‘x’ and ‘y’ stand for.
I. The pollen grain produces only one pollen HCl
tube, i.e. monosiphonous. ‘x’ Pepsin
II. The pollen grain produces many pollen tubes,
i.e. polysiphonous. Proteins Proteoses + ‘y’
III. The pollen tube enters the ovule through (a) Propepsin, Dipeptides (b) Pepsinogen, Peptones
integument. (c) Pepsinogen, Amino acids (d) Propepsin, Amino acids
IV. The pollen tube enters the ovule through the 112. Which among the following statements is incorrect
funicle. regarding α-1 Antitrypsin (AAT)?
Choose the correct option. (a) It is a human protein made in the liver and found in the
(a) Only II (b) Both II and III blood
(c) Both I and III (d) Both II and IV (b) It is an inhibitor of protease enzymes like trypsin and elastase
106. Which among the following statements are correct (c) Inhibition of protease enzymes leads to lung disease
regarding heartwood? emphysema
(a) It represents the central wood of the plant (d) AAT can be extracted from blood donations in large
amounts
(b) Living cells are absent
(c) Tracheids and vessels plugged by tyloses 113. If the location of food source is towards the sun, a
(d) All of the above honeybee will wag its tail in which pattern?
(a) Vertically downward (b) Right angle towards left
107. Which of the following statements are incorrect?
(c) Vertically upward (d) Right angle towards right
I. Hemichordates are exclusively marine
organisms. 114. Name the leucocytes which is the largest in all
II. Hemichordates are bilaterally symmetrical types of leucocytes and constitute 2-10% of blood
and diploblastic. leucocytes.
(a) Neutrophils (b) N-K cells
III. They reproduce asexually as sexes are not
(c) Monocytes (d) Both (b) and (c)
separate.
IV. Fertilisation is internal. 115. Consider the following matches.
Choose the correct option. I. Ectoparasites—Human body lice
(a) II and III (b) III and IV II. Endoparasites—Rat flea
(c) I, II and III (d) II, III and IV III. Facultative parasites—Oyster prawn and pea
108. Which of the following is not a human X-linked crab
trait? IV. Brood parasitism—Cuckoo
(a) Hunter syndrome The incorrect match is
(b) Duchenne muscular dystrophy
(a) Only I (b) Only II (c) Only III (d) Only IV
(c) G-6-PD deficiency
(d) Pattern baldness 116. Allochemicals are
(a) chemicals produced by a species which affects other
109. Which of the following amino acids has the species
greatest number of codons? (b) sex hormones
(a) aspartic acid (b) tryptophan (c) produced by epiphytes
(c) leucine (d) proline (d) a positive interaction

MODULE 2
www.jeebooks.in
Prep Catalysis for NEET ~ Mock Test 9 281

117. Identify the incorrect statements. 125. In plant, apical dominance is a condition where
I. Bowman’s capsule acts as filtering apparatus (a) foliar buds are inhibited by gibberellin
of blood. (b) accessory buds are inhibited by cytokinin
II. Kidneys help in excretion and osmoregulation. (c) axillary buds are inhibited by auxin
(d) extra axillary buds are inhibited by abscisic acid
III. Urethra stores about 0.5 to 1 L of urine.
IV. The main role of loop of Henle is 126. Where does the maximum amount of carbon
ultrafiltration. dioxide fixation take place through the process of
photosynthesis?
Choose the correct option.
(a) Tropical rainforests (b) Crop fields
(a) I, II and III (b) II, III and IV
(c) Oceans (d) Savanna
(c) I and II (d) III and IV

118. Which one of the following pairs of cells and their 127. Which of the following cannot be classified as a
mental disorder?
secretions is correctly matched?
(a) Cirrhosis (b) Neurosis
(a) Mucous neck cells—HCl
(c) Psychosis (d) Epilepsy
(b) Oxyntic cells—Mucus
(c) Crypts of Lieberkuhn —Succus entericus 128. Identify the incorrect statement.
(d) β-cells of islets of Langerhans—Secretion that increases (a) Some photosynthetic pigments absorb light of a particular
blood sugar level wavelength at a greater strength, while very less at other
wavelength
119. Which statement is false regarding the flagellum of (b) Chlorophyll-a absorbs blue-violet and red colour
bacteria? wavelength at a greater strength, while carotenoids absorb
(a) The flagellum is made up of flagellin protein light of blue and green region
(b) Each flagellum arises from a basal granule called (c) Action spectrum is a graph which shows the degree of
blepharoplast absorption of light by a pigment as a function of wavelength
(c) Flagella are organs of motility (d) Plant leaves appear green due to high amount of
(d) 9 + 2 pattern of flagellum structure is present chlorophyll in plant, which reflects green colour

120. The following hormones are released from the 129. The figure given below shows stalked particles on a
adenohypophysis except region of mitochondrial cristae membrane.
(a) adrenocorticotropic hormone 100 nm
(b) growth hormone
(c) somatotropin
(d) prolactin release inhibiting hormone 1

121. Find the odd one out with reference to their sexual 2 Pi
cycles. Cytosolic
(a) Bear, fox (b) Mouse, horse medium +
H
(c) Cat, pig (d) Sheep, cow

122. Which among the following statements is incorrect


regarding opening of stomata?
(a) Glucose collects in the guard cells causing an increase in
concentration of the cell sap H+
3
(b) Guard cells become flaccid
(c) The outer wall stretches more than thick inner wall due to Identify the process which occurs in the numbered
turgor pressure regions.
(d) Increase in the rate of gaseous exchange and transpiration 1 2 3

123. The dicotyledonous stem hypodermis comprises of (a) Glycolysis ADP synthesis Krebs cycle
(a) collenchyma (b) sclerenchyma (b) ATP synthesis Krebs cycle Electron
(c) parenchyma (d) aerenchyma transport chain

124. For proper coordination of the body, cell to cell (c) Krebs cycle Oxidative ATP synthesis
communication is must and it is facilitated by one phosphorylation
of the following structures. (d) Electron Glycolysis Oxidative
(a) Gap junction (b) Tight junction transport chain phosphorylation
(c) Desmosomes (d) Adhering junction

MODULE 2
www.jeebooks.in
282 NEET Test Drive

130. Modern synthetic theory of evolution involves 138. Which portion of endometrium is shed during
I. Gene mutation menstrual phase?
II. Changes in chromosome structure and number (a) Myometrium (b) Antrum
(c) Serosa (d) Stratum functionale
III. Genetic recombination
IV. Natural selection 139. Control of gene expression in eukaryotes includes
all of following given below except
The correct terms are (a) methylation of DNA
(a) I and II (b) III and IV (c) I, II and III (d) All of these (b) feedback inhibition of enzyme activity via allosteric
131. Identify the incorrectly matched pair. modification
(a) Eusporangiate—Selaginella (c) transcription factors activity
(b) Leptosporangiate—Dryopteris (d) alternative splicing of RNA transcripts
(c) Both (a) and (b) 140. Match the items given in column I with those in
(d) None of the above Column II and select the correct option from the
codes given below.
132. Which of the following is a pair of alarm
chemicals? Column I Column II
(a) Opsonin and histones (b) Opsonin and dopamine A. Trichoderma polysporum 1. Citric acid
(c) Interferons and dopamine (d) Histamine and kinins
B. Aspergillus niger 2. Lysine
133. The function of glycocalyx in bacteria is
C. Pseudomonas denitrificans 3. Cyclosporin-A
(a) protection from phagocytosis
(b) prevention of desiccation D. Enterobacter aerogens 4. Cyanocobalamin
(c) enable them to adhere to the surfaces Codes
(d) All of the above A B C D A B C D
134. Arrange the steps in development of new varieties (a) 3 1 4 2 (b) 1 2 3 4
in correct order. (c) 4 3 2 1 (d) 2 4 1 3
I. Selection of the desirable characteristics. 141. The two important types of secondary structures of
II. Creation of genetic variation. proteins are
III. Evaluation of selected strains or lines or (a) motifs and domains
populations. (b) α-helix, β-pleated sheet
IV. Multiplication of evaluated lines. (c) peptide bond, hydrogen bond
Choose the correct option. (d) R group, amino group
(a) I → II → III → IV (b) I → III → II → IV 142. The major disadvantage of cosmids is
(c) II → I → III → IV (d) III → IV → I → II (a) inability to accept more than 40-50 kbp of DNA
135. Electronic smog refers to (b) allow the packaging of DNA in phage in vitro
(a) invisible electromagnetic radiations (c) also perpetuate in bacteria
(b) it causes harm to eyes (d) can produce a complete genome library of 108
(c) it affects brain cells of man
143. HIV virus destroys which of the following T-cells?
(d) All of the above
(a) Helper T-cells (b) Cytotoxic T-cells
136. The sarcoplasmic reticulum of skeletal muscle is a (c) Killer T-cells (d) Suppressor T-cells
(a) form of smooth endoplasmic reticulum 144. Which among the following statements is/are
(b) site where calcium is released during muscle relaxation incorrect regarding secondary growth in dicot
(c) site of calcium binding protein storage stem?
(d) Both (a) and (c) (a) The vascular cambial ring develops as a circular ring since
137. Consider the following statements. beginning
(b) Annual rings or growth rings occur quite commonly
I. Habitat is the place or set of environmental
(c) Both (a) and (b)
conditions, in which a particular organism lives.
(d) None of the above
II. Habitat can have number of niches.
III. Niche have many species. 145. Which of these nitrogen-fixing bacteria does not
live in symbiotic association with host plant?
IV. Niche is profession of an organism. (a) Rhizobium (b) Azotobacter
The incorrect statement is (c) Frankia (d) Xanthomonas
(a) Only I (b) Only II (c) Only III (d) Only IV

MODULE 2
www.jeebooks.in
Prep Catalysis for NEET ~ Mock Test 9 283

146. Procumbent is 155. The species which is susceptible to extinction has


(a) the stem that totally creeps on the ground (a) large body size
(b) woody perennial climbers (b) low reproductive rate
(c) the stem that grows vertically (c) In food chain, occupies the highest trophic level
(d) the stem that has a long internode (d) All of the above
147. Identify correct statement about protocells. 156. Match the items given in Column I (types of RNA
(a) Protocells were first organic molecules formed from water polymerase) with those in Column II (function).
before the origin of life
(b) Protocells were first RNA molecules having enzymatic Column I Column II
properties A. RNA polymerase I 1. Precursors of rRNA are
(c) Protocells were first protein-lipid microspheres enclosing synthesised except 5S rRNA
RNA molecules
B. RNA polymerase II 2. Synthesises hnRNA coding
(d) Protocells were first cell-like organisms, formed during
genes
origin of life
C. RNA polymerase III 3. Synthesises precursors of
148. A plant hormone which allows the seeds to ignore
tRNA, 5 S rRNA and SnRNA
environmental conditions and germinate is
(a) abscisic acid (b) ethylene Codes
(c) cytokinin (d) gibberellins A B C A B C
(a) 1 2 3 (b) 3 2 1
149. Given below is a stage of meiosis in an animal cell.
A (c) 1 3 2 (d) 2 3 1

157. The initiation codon for translation to start is


AUG, it is the codon for
(a) methionine (b) valine (c) leucine (d) isoleucine
158. Ductus choledochus and main pancreatic duct join
together to form
Identify the stage being represented by this figure (a) hepatopancreatic duct (b) cystic duct
along with structure labelled A. (c) bile canaliculi (d) duct of Santorini
Structure A Stage
159. Which type of food chain is depicted in the
(a) Meiotic spindle Telophase-I following examples?
(b) Centrioles Anaphase-I
Dead leaves → Wood louse → Black bird
(c) Centromere Metaphase-I (a) Detritus food chain (b) Grazing food chain
(d) Microtubules Telophase-II (c) Both (a) and (b) (d) None of these
150. In which phase of the cell cycle does synthesis of 160. High myoglobin content and intermediate
histone proteins take place? glycolytic enzyme activity are the characteristics of
(a) Anaphase-I (b) S-phase (c) G1-phase (d) G2-phase which of the following?
151. Chlamydomonas yellowstonensis is a (a) White fibres type of muscles
(a) thermophyte (b) cryophyte (c) symbiotic (d) parasitic (b) Fast oxidative or aerobic muscles
(c) Cross-bridge structure
152. In date palm, maturity time as well as size of (d) None of the above
fruits can be changed by using different pollens.
This effect is technically known as 161. Phylogenetic system brings about groupings or
(a) xenia (b) metaxenia (c) ruminate (d) mosaic association according to
153. Identify the incorrectly matched pair. (a) reproductive similarities
(b) morphological characters
(a) I-131 — Causes thyroid cancer
(c) increasing complexity of body organisation
(b) Sr-90 — Causes bone cancer
(d) evolutionary trends
(c) Radon — Causes lung cancer
(d) None of the above 162. Oligotrophic lakes have
(a) low plankton density (b) low primary productivity
154. In Najas and Zostera, the type of pollination is (c) poor in nutrients (d) All of these
called hypohydrophily because in these plants,
(a) the pollination occurs by a whirlpool created in water 163. Upon exposure to light, the endosperm becomes
(b) the pollination occurs in submerged plants chlorophyllous in
(c) the pollination occurs in floating plant (a) Raphanus (b) Mathiola
(d) Both (a) and (c) (c) Both (a) and (b) (d) None of these

MODULE 2
www.jeebooks.in
284 NEET Test Drive

164. Identify the incorrect match of a water pollutant Codes


with its effect (disease ). A B C D
(a) Nitrate – Black foot disease (a) 1 2 3 4
(b) Mercury – Minamata disease (b) 4 3 2 1
(c) 3 1 4 2
(c) Cadmium – Itai - Itai disease
(d) 2 4 1 3
(d) Fluoride – Fluorosis
171. A person suffering from long-sightedness is
165. Stem cuttings of grapes, sugarcane, etc., are dipped wearing spectacles with convex lenses. If he
in which solution prior to being sowed? removes the glasses the image of the near objects
(a) IBA (b) GA3 (c) ABA (d) Ethylene will be formed
166. In pest-resistant legumes, (a) on the yellow spot
(a) a gene for an enzyme that synthesises a chemical toxic to (b) behind the retina
weevils has been transferred (c) in front of the retina
(b) Bt toxins are thousands of times more powerful (d) on the blind spot
(c) these crops increase the use of chemical pesticides 172. Which among the following is not an objection to
(d) more resistant to viral attack root pressure theory?
167. A patient with the deficiency of vitamin-B1, should (a) The rapidly transpiring plants do not show any root
pressure
avoid intake of alcohol because, (b) Root pressure has been found in all plants
(a) alcohol has bad effect on brain tissues (c) Water continues to rise even in the absence of roots
(b) alcohol makes the person loose control over his reflexes (d) It is seen only during the most favourable periods of
(c) alcohol interferes with the metabolism of vitamin-B1 in liver growth like spring or rainy season
thus causes brain related diseases
(d) alcohol increases the respiratory quotient which may 173. The aleurone layer secretes some hydrolytic
cause restlessness enzymes which cause breakdown of
starch and protein. Their secretion is
168. Which among the following is not the characteristic stimulated by
feature of DNA probes? (a) cytokinin ‘zeatin’, released in endosperm
(a) Probes are always single-stranded (b) triple fusion instigates the secretion of these enzymes
(b) They can be made of DNA or RNA (c) gibberellins, released by the embryo
(c) The universal DNA probe is made up of repeated GATA (d) embryo scutellum
sequence
(d) It is the DNA which is transferred from one organism into 174. Which among the following helps in attachment
another with other cells?
(a) Pili
169. Transcription or RNA synthesis is terminated by
(b) Flagella
(a) sigma factor (b) rho factor (c) alpha factor (d) beta factor
(c) Fimbriae
170. Match the items in Column I with Column II and (d) Cell wall
select the correct option from the codes given below.
175. Condensation (ATP dependent)
Column I Column II A+B A–B
ATP ADP+Pi
A. DNA ligase 1. Endonuclease that digests
ss or ds DNA The above reaction is successfully accomplished by
B. DNase I 2. Unpackaging DNA strands enzymes of class
(a) Oxidoreductase – Alcohol dehydrogenase
C. DNA topoisomerase 3. Catalyses the covalent
(b) Lyases – Aldolase
bonds of segments of an
(c) Ligases – Glutamine synthetase
interrupted sugar
(d) Transferases – Hexokinase
phosphate strand in
ds DNA 176. In plants, Golgi apparatus is not found in
D. DNA helicase 4. Produces single or ds (a) male gametes of pteridophytes
breaks in DNA during (b) male gametes of bryophytes
replication to release (c) cells of sieve tube
tension brought about by (d) All of the above
DNA uncoiling

MODULE 2
www.jeebooks.in
Prep Catalysis for NEET ~ Mock Test 9 285

177. Injury to adrenal cortex is not likely to affect the 179. A cell will not enter M-phase if
secretion of which one of the following? (a) cell is not sufficiently large
(a) Cortisol (b) there is lack of nutrients
(b) Adrenaline (c) mitotic cyclin is overexpressed
(c) Aldosterone (d) replication of DNA is not complete
(d) Both androstenedione and dehydroepiandrosterone
180. Which of the following organisms have the richest
178. The cell wall of bacteria comprises of blood in the animal kingdom?
(a) NAG (b) NAM (a) Reptiles (b) Amphibians
(c) peptidoglycan (d) All of these (c) Birds (d) Mammals

Answer Sheet
1. (a) 2. (d) 3. (b) 4. (d) 5. (a) 6. (b) 7. (d) 8. (d) 9. (d) 10. (d)
11. (c) 12. (b) 13. (a) 14. (c) 15. (a) 16. (d) 17. (d) 18. (b) 19. (d) 20. (d)
21. (b) 22. (b) 23. (b) 24. (b) 25. (d) 26. (a) 27. (b) 28. (d) 29. (d) 30. (a)
31. (c) 32. (a) 33. (b) 34. (d) 35. (a) 36. (c) 37. (a) 38. (b) 39. (c) 40. (c)
41. (c) 42. (c) 43. (c) 44. (a) 45. (b) 46. (c) 47. (a) 48. (b) 49. (a) 50. (c)
51. (c) 52. (d) 53. (a) 54. (b) 55. (a) 56. (a) 57. (b) 58. (b) 59. (c) 60. (a)
61. (c) 62. (a) 63. (c) 64. (a) 65. (d) 66. (c) 67. (c) 68. (c) 69. (b) 70. (c)
71. (d) 72. (b) 73. (c) 74. (c) 75. (b) 76. (a) 77. (a) 78. (a) 79. (a) 80. (d)
81. (b) 82. (a) 83. (a) 84. (d) 85. (a) 86. (a) 87. (c) 88. (a) 89. (b) 90. (d)

91. (d) 92. (b) 93. (d) 94. (c) 95. (c) 96. (d) 97. (d) 98. (c) 99. (b) 100. (c)
101. (d) 102. (b) 103. (c) 104. (c) 105. (b) 106. (d) 107. (d) 108. (d) 109. (c) 110. (a)
111. (b) 112. (d) 113. (c) 114. (c) 115. (b) 116. (a) 117. (d) 118. (c) 119. (d) 120. (d)
121. (a) 122. (b) 123. (a) 124. (a) 125. (c) 126. (c) 127. (a) 128. (c) 129. (b) 130. (d)
131. (d) 132. (d) 133. (d) 134. (c) 135. (d) 136. (d) 137. (c) 138. (d) 139. (b) 140. (a)
141. (b) 142. (a) 143. (a) 144. (d) 145. (b) 146. (a) 147. (d) 148. (d) 149. (b) 150. (b)
151. (b) 152. (b) 153. (d) 154. (b) 155. (d) 156. (a) 157. (a) 158. (a) 159. (a) 160. (b)
161. (d) 162. (d) 163. (c) 164. (a) 165. (a) 166. (a) 167. (c) 168. (d) 169. (b) 170. (c)
171. (b) 172. (b) 173. (c) 174. (a) 175. (c) 176. (d) 177. (b) 178. (d) 179. (d) 180. (c)

MODULE 2
www.jeebooks.in

EXPLANATIONS

PHYSICS
1. Acceleration with which the cylinder It is similar to a closed organ pipe, Þ E 1 = 1000 J
rolls down, hence fundamental frequency of well is 1
g sin q g sin q \ KE = m1v12 = 1000
a= = v 346 2
n= =
I 1 4L 4 ´ L
1+ MR 2 Þ v1 = 10000 = 100 ms - 1
MR 2 1+ 2 2 346 ~
MR Þ L= - 29 m 12. Let a be the acceleration of each block,
4´3 then
2
= g sin q T 3 = (m1 + m 2 + m3 )a … (i)
3 7. Fall in temperature of brass sphere,
2 1 DT = 500 - 0 = 500º C and T 2 = (m1 + m 2 )a … (ii)
= ´ 9. 8 ´
3 20 Heat loss by sphere, From Eqs. (i) and (ii), we get
= 0. 326 ms - 2 Q = msDT = 5 ´ 500 ´ 500 æ m1 + m 2 ö
T2 = ç ÷ ´ T3
Using first equation of motion, = 1250 kJ è m1 + m 2 + m 3 ø
v = u + at = 0 + 0. 326 ´ 5 Heat for melting m 2 kg of ice,
æ 10 + 6 ö
= 1. 63 ms - 1 Q 2 = m 2L = m 2 ´ 336 =ç ÷ ´ 40
è 10 + 6 + 4 ø
2. Work done is maximum, when process From principle of calorimetry,
involved is isobaric. = 32 N
= 1250 kJ = m 2 ´ 336
3. As, F = q (v ´ B) Þ F = qvB sin q 1250 13. Induced emf is given by
m2 = = 3 .72 kg Df
At q = 0º, F = 0 336 E ind =
Dt
At q = 90º, F = qvB 8. The semi-major axis of the elliptical Q
orbit around the sun is given by Current, i =
At q = 180º, F = 0 Dt
r + r2
So, for non-zero force angle between v r = 1 Df 1
2 = ´
and B can have any value other than Dt R
zero or 180º. According to Kepler’s third law,
Df
T2 µr3 Þ Q =
4. Let the depth of Indian ocean is x cm R
3
r + r2 ö
\ p1V1 = p 2V2 (tdg + xdg) æç pr 3 ö÷ T 2 µ æç 1
4 14. A ® Q; B ® R; C ® P
Þ ÷
è3 ø è 2 ø
15. The maximum acceleration for SHM is
= tdg é 4
p(18r ) 3ù 3/2
r + r2 ö
êë 3 úû or T µ æç 1 ÷
given by
è 2 ø a max = w 2 A = (2 p n) 2 A = 4 p 2 n 2 A
(t + x ) = t × 18 3
or T µ (r1 + r 2 ) 3 / 2 . The block will remains in contact with the
18 3 t - t = x piston, if a max £ g or 4 p 2 n 2 A < g
9. Heat produced in the resistor
So, x = t 5831 cm = 5831 t cm 1 \ Maximum amplitude of piston will be
= Energy of capacitor = CV 2
5. Output power will be maximum, if 2 g
A max =
internal resistance of battery is equal to 1 4 p 2 n2
external resistance or load resistance, = ´ 6 ´ 10- 6 ´ 360 ´ 360
2 9.8
hence r = 5W. = = 0. 99 m
= 0. 39 J 4 p 2 (0.5) 2
6. For air, r = 1.10 kgm- 3 ,
10. All thermodynamic changes are zero. 1 1 F 2L éQY = FL ù
K = 1. 32 ´ 10 Nm 5 -2 16. Energy = Fl =
2 2 AY êë Al úû
Speed of sound in air, 11. Total energy = E 1 + E 2 = 1050 J
2 -2
p2 1 (100) ´ 40 ´ 10
K As, E =
E m
Þ 1 = 2 =
4
= 20 = ´
v = 2 4 ´ 10- 4 ´ 4 ´ 1010
r 2m E2 m1 0. 2
where, m1 = mass of shell = 1. 25 ´ 10- 4 J
1. 32 ´ 105
= and m 2 = mass of gun. 17. The value of resultant electric field at
1.10
E y =0
= 3 . 46 ´ 102 Þ E2 = 1 q é 1
+ 2 + 2 + .. ¥ù
1 1
20 E =
= 346 ms - 1 E1 4 pe0 êë 2 2 4 8 úû
Þ E1 + = 1050
20

MODULE 2
www.jeebooks.in
Prep Catalysis for NEET ~ Mock Test 9 287

é 1 ù Q Potential difference across 29. R = 40 + 20 = 60 W


-6 ê ú 3
= 6 ´ 10 ´ 9 ´ 10 ê 4 ú 9
PQ = ´ 10 Power factor =
R
1 16
ê1 - ú R 2
+ (X L - X C ) 2
ë 4û 15
= V
1 8 60
3
= 54 ´ 10 ´ = = 0.6
3 21. DU X = DUY (For cyclic process) 602 + (100 - 20) 2
= 1. 8 ´ 104 N / C
So, DWX = DQ X and DWY = DQY 30. Moment of inertia of a cylinder about its
18. The distance of A is given by MR 2
As from figure, DWX > DWY centre and parallel to its length = .
30 ms–1 10 ms–1 2
So, DQ X > DQY
A smin B Moment of inertia about its centre and
10 ms–1 22. Force on the charged particle in perpendicular to its length
v=0 magnetic field is
æ L2 R2 ö
A B F = q (v $j ´ B $i) = qvB (- k$ ) = Mç + ÷.
which is -Z -axis. è 12 4 ø
sB
sA L2 - 3L1 Equating both values, we get
23. End correction =
2 MR 2 ML2 MR 2
1 = +
sA = ut + at 2 112 . 9 - 3 ´ 32 .5 2 12 4
2 =
2 1 2 1 2
1 15 . 4 R = L
= 30t + (- 2)t 2 = = 7 .7 cm 4 12
2 2
Þ 3R = L
= 30t - t 2 Speed of sound in air,
or L= 3R
and of B, sB = 10t v = 2 n (L2 - L1)
From figure, s A - sB = s min
31. When an object is placed in front of such
= 2 ´ 256 ´ (1.129 - 0. 325)
a lens, the rays first of all refracted from
Þ s min = 20t - t 2 …(i) = 411. 65 ms - 1 the convex surface, then reflect from the
Thus for A, v = u + at 24. From second law of motion, polished plane surface and again
Þ 0 = 30 - 2t Þ t = 15 s mg - T = ma refracts from convex surface. If fl and fm
25 1 be the focal lengths of lens (convex
Putting the value of t in Eq. ()
i , we get and T = mg = mg surface) and mirror (plane polished
\ s min = 20 ´ 15 - 225 = 75 m 100 4
surface) respectively, then effective focal
1 3
19. As total momentum remains Þ mg - mg = ma Þ a = g length F is given by
conserved, 4 4 1 1 1 1
= + +
I1w1 = I 2 w 2 25. New fringe width is F fl fm fl
2 2 éQI = 2 MR 2 ù b
b¢ = 2 æ
MR1 w1 = MR 22 w 2 = ¥ö÷
2 2 1 R
5 5 êë 5 úû m = + = çQfm =
fl fm fl è 2 ø
0 .5
Þ R12 w1 = R 22 w 2 = = 0. 3 mm
= (m - 1) æç ö÷
1 1
2 5/3 Now,
æR ö fl èR ø
Þ w 2 = ç 1 ÷ ´ w1 26. Net magnetic flux through a closed
èR 2 ø 1 2 (m - 1)
surface is always zero. \ =
2 F R
æ 108 ö 27. Charged particle moves on circular
= ç 5 ÷ ´ 10- 4 R
è 10 ø path in magnetic field with uniform or F =
2 (m - 1)
-2 -1 speed. Force on charged particle is
= 10 rad s
always perpendicular to speed, hence R
As, R = 2F =
20. Applying Kirchhoff’s voltage law in the work done is zero. (m - 1)
given loop. Due to change in velocity of particle on
circular path, momentum changes. 32. Equivalent circuit is shown below
2Ω 3V 6V 4Ω 1Ω C 2Ω
P Q - dV
28. As, I = A
r1 I r2 dr
I 6Ω 2Ω
Þ dV = - Idr
V x x
-3 B
So, ò dV = ò - Idr = ò - kr dr 1Ω D 2Ω
0 0 0
x
RCD = 2 + 2 + 2
10 Ω ær-3+1 ö = 6W
Þ V = - kç ÷
- 4I + 6 - 3 - 2I - 10I = 0 è - 3 + 1ø 0 6 ´ 6ö
So, R AB = 1 + 1 + æç ÷
- 16I = - 3 k è 6 + 6ø
=
3 2x 2 = 5W
I = A
16

MODULE 2
www.jeebooks.in
288 NEET Test Drive

33. According to given question, Voltage across L1 and L2 is same.


r = 60º = i i.e., e1 = e2 1 2h æ k2 ö
43. Time, t = ç1 + 2 ÷
Given, condition satisfies Brewster’s di1 di sin q g è R ø
law, L1 = L2 2
dt dt
\ m = tani p (i p = i ) æk2 ö
or L1i1 = L2i 2 1+ ç 2÷
= tan 60º = 3 i1 L2 t sphere è R ø sphere
= \ =
= 1.73 i2 L1 t disc æk2 ö
1+ ç 2÷
34. Given, square wire of side = 2 cm, N 1 è R ø disc
39. We know that, =
æ t ö
u = - 20 cm, f = - 10 cm, N0 çç ÷÷
è T1 / 2 ø 2
Area of second wire = ? 2 1+
1 1 = 5
1 1 1 Þ = 1
\ + = 64 æ 30 ö
çç ÷÷ 1+
v u f è T1 / 2 ø 2
2
1 1 1
Þ + = 1 1 =
7 /5
=
14
v (- 20) - 10 Þ =
2 6
2
30 /T1/ 2 3/2 15
1 1 1
Þ =- + 30 or 14 : 15
v 10 20 Þ 6=
T1/ 2 44. Given, h = 1600 km = 1. 6 ´ 106 m,
Þ v = - 20 cm
v Þ T1/ 2 = 5 days R = 6 × 4 ´ 106 m and
We know that, m = -
u \
1
= x
1 g = 9. 8 ms - 2
- (- 20) 128 The distance of satellite from earth’s
= = -1 25
- 20 centre is
1 1
Þ = x /5 r =R + h
Area of image 27 2
Q = m2
Area of object Þ x = 35 days = 6. 4 ´ 106 + 1. 6 ´ 106
Ai = 8 ´ 106 m
Þ = (- 1) 2 40. If I 0 is the intensity after closing one
Ao mv 2
slit, then it satisfies Centripetal force, F =
Ai I0 µ a2 r
=1
2´2 (where, a = amplitude of light wave) \ Centripetal acceleration,
2 F
Þ Ai = 4 cm When both slits open, then intensity a=
m
35. Electrons are emitted from metal \ I µ (2a) 2 = 4a 2
surface only when maximum v2 ( gR 2 / r ) 2
Þ I0 = I / 4 = =
wavelength is equal to or less than r r
41. Angular momentum,
threshold wavelength. gR 2 R2
nh 3 ´ 6. 6 ´ 10- 34 = =g
36. Given, v = - a $j L= = r2 (R + h) 2
2p 2 ´ 3 .14
\ E ´ B = v , which satisfies, (6. 4 ´ 106 ) 2
(- k$ E ´ B $i) = - a $j = 3 .15 ´ 10- 34 J - s = 9. 8 ´
(8 ´ 106 ) 2
i.e., direction of oscillating electric field 42. The temperature coefficient of
of electromagnetic wave will be along resistance of semiconductors is = 6. 272 ms - 2
negative z -direction. negative that is their electrical 81. 2
resistance decreases with rise in 45. Efficiency, h = %
37. In p - n junction, the diffusion of 1 + r1 / r 2
temperature.
majority carriers takes place when So, h = 81. 2% ~
max = 81%
junction is forward biased and drifting where, r1 = forward bias diode resistance
of minority carrier takes place across r 2 = load resistance.
ρ and
the junction when reverse biased.
For maximum efficiency, r 2 >> r1
38. From given figure,
As inductors are in paralled, so T

MODULE 2
www.jeebooks.in

CHEMISTRY
46. A ¾®P r s
O 52. CH3NH2 + HONO ¾HCl
¾¾® CH3 N2 Cl +2H2 O
Ice cold
Initially a 0 Unstable ½
C ¯
After time, t a - x x OH r
CH3 + N2 + Cl s
a a
After t1/ 4 a- ½ HONO.
4 4 ¯

For first order reaction, O O CH3OCH3 + CH3ONO + CH3OH + Hr

log æç
2.303 a ö
k= ÷ O O CHCl + 3KOH ®
t èa - x ø 53. CH 3NH 2 ¾ ¾ ¾3 ¾ ¾¾® R ¾N ==C
Reimer -Tiemann
2.303 a reagent
k = log More stability of conjugate base is due + 3KCl + 3H 2O
t1/ 4 3a This reaction is known as carbylamine
to resonance. Thus, the correct order
4 of acidic strength of the given reaction.
2.303 4 compounds is I < II < IV < III.
Þ t1/ 4 = log 54. Leaving group ability depends upon
k 3
49. Electronic configuration of Eu3+ is basicity of group. The weaker the base,
0.29 [Xe]4f 6 5d 0 6s 0 the greater is its leaving group ability.
\ t1/ 4 =
k Alternatively, the stronger is conjugate
47. Ultra pure metals, are prepared by the acid, the greater the leaving group ability
van-Arkel method. Arkel method is \ m cal = N (N + 2) of the base.
used to purify oxide of titanium metal. The strength of conjugate acid is
Q N = 6 Þ m cal = 48 = 6.93
Ti is heated with iodine to about 500 K HOSO2 CF3 > HOSO2Me > HOAc > HOMe.
to form volatile compound. The Til 4 is 50. CH3 C CH2 CH2 CH2Cl
55. H3 C ¾ CHO + HCN ¾® H3 C ¾ CH ¾OH
reheated at 1700 K, when it ½
C
decomposes to give pure Ti. CN
(i) CH3MgBr (ii) H3O +
500K 1700 K COOH
Ti +2l 2 ¾® Til 4 ¾® Ti + 2l 2
Volatile stable Pure metal H 2O ½
compound CH3 ¾® H3 C ¾ C* ¾ OH
½
48. Presence of electron withdrawing CH3—C—CH2 CH2 CH2 Cl H
group on ring or aromatic nucleus
OH Optically active
increases its acidic strength by
decreasing electron density.
Intramolecular Hence, (a) is the correct answer.
rearrangement
COOH COOH 56. Number of open chain isomers possible
CH3
for C4H8 are 4. These are as follows:
H3C—C—CH2 H3C CH3 H3C H
< ⇒
C ==C C ==C
O CH2
O H H H CH3
(I) CH3 CH2
H CH2 CH3 CH3 H
(II) 51. C ==C C ==C
– +
—COO Na H H CH3 H
On the other hand, electron donating
group present on aromatic nucleus Sodium benzoate Hence, correct answer is 4.
(here) decreases acidic strength due
to increase in acidic strength around ∆, sodalime 57. An exothermic reaction is a chemical
nucleus. reaction that releases energy in the form
of light or heat.
COOH COOH
The given reaction is an exothermic
Benzene
reaction, hence lowering the temperature
> and increasing pressure will favour
D
(C6H5 COO) 2 Ca ¾¾ ® C6H5OC6H5 forward reaction.
Calcium benzoate Diphenyl
NO2 CH3 ether 58. We know that at equilibrium, DG = 0 for
+ CaCO 3 given reaction
(III) (IV)
(C6H5 COO) 2 Ca + (CH3 COO)2 Ca H2O(l ) s H2O(v )
Also, we know that carboxylic acid is Calcium Calcium
more acidic than phenol due to benzoate acetate At equilibrium, DG = 0
D
formation of more stable conjugate ¾¾ ® 2C6H5OCH3 + CaCO 3 DG = DH - TDS = 0
base of benzoic acid. Methyl phenyl ether D H = TD S
Thus, the correct option is (c).

MODULE 2
www.jeebooks.in
290 NEET Test Drive

DH 65. KO 2 is a very good oxidising agent 71. Ethyne molecule is a linear molecule
T =
DS KO 2 . It is even more good oxidising which consists of one C¾C s-bonds,
. kJ mol -1
4073 agent than K 2O 2 . two C¾H s-bonds and two
= C¾ C p-bonds. Electron cloud between
0.109 kJ mol -1 K -1 66. two C-atoms is cylindrically symmetrical
= 373 K R Cl +2H2O R R about the internuclear axis.
Si Si
59. We know that, R Cl OH OH 72. + M effect of — NH2 > + M of ¾OH
DG = - 2.303RT log K group that disperse the charge of
R OH OH R
= - 2.303 ´ 8.314 ´ 298 log 50 Si Si carbocation. Hence, increases the
= - 9694 J = - 9.694 kJ R OH HO R stability —CH3 group shows + I effect,
60. Work done in isothermal compression so it will disperse the charge less than
can be calculated as follows: Polymerisation –3H2O ¾NH2 , ¾OH group whereas ¾NO 2
V group shows - M -I effect due to which
W = - nRT ln 2 the positive charge on the carbocation
V1 R R
increases. Hence, stability decreases.
40 Si
= - 5 ´ 8.314 ´ 300 ´ 2.303 log M1V1 + M 2V2
200 O O 73. M =
= - 1500 ´ 8.314 ´ 2.303 ´ (- log 5) R R V1 + V2
Si Si
= + 201
. kJ R O R 0.6 ´ 800 + 1 ´ 200
Cyclic trimer
=
é 1 1ù 1000
61. Using formula, DE = hCRH ê 2 _ 2 ú of silicon
480 + 200
n
ë 1 n 2 û
=
67. Ozone is found in stratosphere, which 1000
For lowest energy of the spectral line in 680
is depleted by CFC and hence the =
Lyman series n1 = 1, n2 = 2.
thickness of ozone is reduces. 1000
62. Structure of IF7 is determined by using 68. According to valency concept, 3Fe2+ = 0.68 M
hybridisation as follows: will be replaced by 2Fe3+. 74. Except hydrophobic effect, all the given
V + M -C + A Total loss of iron from one mole of FeO are involved in the tertiary structure of
H =
2 = 1 - 0.93 = 0.07 proteins.
7+7 Total Fe3+ present in one molecule of
= =7 75. When stearic acid reacts with
2 FeO is polyethylene glycol, then the product
Hybridisation = sp 3d 3 2 ´ 0.07 = 014
. formed belongs to a class of non-ionic
Structure = Pentagonal bipyramidal Total number of Fe (II) and Fe (III) detergent. The reaction is as follows:
F present in one molecule of FeO = 0.93 CH3 (CH2 )16 COOH
F F 014
. Stearic acid
Percentage of Fe (III) = ´ 100
0.93 +
I
= 15.05% HO(CH2 CH2O) n CH2 CH2OH
F F Polyethylene glycol
F 69. According to first law of electrolysis,
F ½- H O
Mass of metal deposited (w)
63. The sum of number of electrons ¯ 2
atomic weight
(unpaired) in d-orbital and number of = ´ i ´t CH3 (CH2 )16 COO(CH2 CH2O) n CH2 CH2OH
electrons exchanged ´ 96500
electrons in s-orbital gives the number
atomic weight ‘Non-ionic detergent’
of oxidation states (OS) exhibited by \ 0.415 = ´ 2 ´ 965
d-block elements. Therefore, 3 ´ 96500 76. Compound A (C7H9N) is benzylamine
(a) 3d 3 , 4s 2 Þ OS = 3 + 2 = 5 Thus, atomic weight (CH3 CH2NH2 ) as it reacts with
(b) 3d 5 , 4s 1 Þ OS = 5 + 1 = 6 0.415 ´ 3 ´ 96500 NaNO 2 HCl at 0°C to form benzyl
(c) 3d 5 , 4s 2 Þ OS = 5 + 2 = 7 = = 62.25 g alcohol and N2 gas.
2 ´ 965
(d) 3d 2 , 4s 2 Þ OS = 2 + 2 = 4 CH2NH2 CH2OH
Hence, elements with 3d 5 4s 2 3 45
70. nu rea = = 0.05 Þ n H 2 O = = 2.5
configuration exhibits largest number 60 18
NaNO2/HCl
of oxidation states. éQn = weight ù + N2
êë 0ºC
molecular mass úû
64. M A = a and MB = 4a (A)
WA =WB = m (say) We know that,
Dp Benzyl amine shows following five
cB =
m 4a = c urea
m 4a + m a p° isomers.
0.05
1 p = = 0.0196 = 0.02
= = 2.5 + 05.
5 5

MODULE 2
www.jeebooks.in
Prep Catalysis for NEET ~ Mock Test 9 291

NH3 2+ 88. CH3


CH2—NH2 CH3 CH3 Etard reaction + CrO2Cl2
H 3N NO2
NH2 CS2
Co
; ;
NH2 H 3N NH3 CH(OCrOHCl2)2
I II III NH3
CH2 NHCH3 [Co(NO2) (NH3)5]2+
+
H3O
82. R = k [A]1 (first order)
CHO
;
mol L-1 s -1 = k (mol L-1)
V Unit of k = s -1
NH2 Benzaldehyde
IV R = k [A] 0 (zero order reaction)
-1 -1 O
mol L s =k Reformatsky
77. (A) has tetrahedral structure, (B) has
square planar structure. Unit of k = mol L-1s -1 or Ms -1 C + CH2—COOEt

78. Greater the stability of the carbocation 83. This reaction is an example of R R
Br
intermediate formed, greater will be the oxymercuration-demercuration reaction
(i) Zn/Benzene
hydration. Substituted alkenes are which proceeds as follows: (ii) H2O/H+
more hydrated than less substituted.
(i) Hg (OAc2)/H2O OH
Thus, the order of rate of hydration is
I < II < III. (ii) NaBH4 R
C—CH2
OH R
79. In cis - [Co(NH3 ) 4 Cl 2 ]+, Co is d 2sp 3 Butan-2-ol
–H2O COOEt
hybridised because NH3 is a strong
field ligand thus, pair up the 3d 6 84. k = e -DH /RT [Q DH = E f - E b] R
electrons of Co. C—
— CH—COOEt
DH R
Co3+ = 3d 6 4s 0 In k = constant = - + 0
RT α, β-unsaturated ester
[Co(NH3)4Cl2]+= Compare with y = mx + c O
CH3
3d 4s 4p Given, E a (forward) > E a (backward) Friedel-Craft + Ar / R—C—Cl
In endothermic reaction,
Dry AlCl3
DH > 0
NH3 NH3 NH3 NH3 Cl Cl
DH O
Hence, slope for reaction will be - .
It is d 2sp 3 hybridised. R
80. Concentrated nitric acid upon long OH C
85. (i) OsO4, 25ºC Ar/R
standing turns yellow brown. This is
(ii) Na2SO3
due to the formation of brown coloured
gas (NO 2 ). OH Perkin reaction
cis pentan-1, 2-diol O O
4HNO 3 ¾® 2H2O + 4NO 2 + O 2 Osmium tetraoxide causes cis
81. The complex, [Co(NH3 ) 5 NO 2 ] 2+ hydroxylation via formation of cyclic CHO + CH3—C—O—C—CH3
exhibits linkage isomerism as NO 2 oximate ester. Base
group being an ambidentate ligand 86. For zero order reaction,
can bind to a metal atom either through CH CH—COOH
R = -kt + [R ] 0
N or O-atom and hence two different Cinnamic acid
isomers are formed. These are On comparing with y = mx + c, we get 3+
[Co(ONO)(NH3 ) 5 ] 2+ and a straight line as shown below. 89. Colour of U is due to transition of
[Co(NO 2 )(NH3 ) 5 ] 2+. electron from lower electronic energy
[R]0
level of f-orbital to higher electronic
NH3 2+
energy level of f-orbital. This type of
H3N ONO k=– slope transition is known as f - f transition.
[R]

Co 90. O O
H3N NH3
NH3 t S S
[Co(ONO) (NH3)5] 2+ O O
87. 2MnO 2 + 4KOH +O 2 ¾® 2K 2MnO 4 OH OH
Purple green
H2S2O6
+ 2H2O Dithionic acid

MODULE 2
www.jeebooks.in

BIOLOGY
91. The option (d) is not correctly paired. In 98. Penicillin was discovered by Alexander outer two stamens are short and inner
aneuploidy, an organism gains or loses Fleming in 1928. It is derived from four stamens are long (i.e. 2 + 4).
one or more chromosomes but not a Penicillium fungi. Penicillium have a 104. The statement II is incorrect. Venereal
complete set. In tetrasomy (2n + 2), b-lactam ring, which is responsible for diseases are the sexually transmitted
there is gain of 2 chromosomes. their antibacterial activity. The b-lactam infections. AIDS can be transmitted by
Aneuploid condition 2n - 1 is called ring interacts with proteins in the infected sexual fluids, infected blood
monosomic. The aneuploid 2n + 1 is bacterial cell responsible for the final transfusion and from mother to her baby.
called trisomic. step in the assembly of the cell wall It is not completely curable even if it gets
92. Genetic drift is also called as Sewall and inhibits the synthesis of detected early and treated properly.
Wright effect. Also known as allelic peptidoglycan.
105. In members of Cucurbitaceae, many
drift. Genetic drift is the change in the 99. The statement II is incorrect. It is pollen tubes are produced
frequency of an existing gene variant corrected as follows (polysiphonous) and the pollen tube
(allele) in a population. Test cross is the cross of the enters the ovule through its middle part,
The evolutionary biologist Ernst Mayr individuals with its homozygous i.e. through integument, which is also
proposed biological species concept. recessive parent. In other words, it is a called mesogamy.
Hardy-Weinberg law is the fundamental specialised back cross of F1 hybrid with 106. All the statements are correct regarding
law of population genetics. The its homozygous recessive parent.
heartwood. It represents the central wood
attractions and loss of genetic 100. If a gene is relocated near genetically of the plants. It is also called duramen.
variability has been termed as bottle inert regions like heterochromatin, its
Living cells are absent in heartwood.
neck effect. normal expression may be modified.
Tracheids and vessels are plugged by
This is called position effect.
93. Chlamydospores are resistant spores tyloses. Tyloses are outgrowths on
formed under unfavourable conditions. Gene penetrance is the ability of a parenchyma cells of xylem vessels of
Zoospores are formed in lower group gene to extpress itself in an individual heartwood.
of fungi of Phycomycetes. that carries it.
107. The statements II, III and IV are incorrect.
Aplanospores are non-motile structures. Expressivity of a gene is the ability to
Hemichordates are triploblastic, they
Conidia are exogenous spores. express itself uniformly in all the
reproduce sexually as their sexes are
individuals with same genotype.
94. The statement (c) is incorrect. CO 2 is separate and the fertilisation is external.
Epigenetics refers to the study of Sexes are usually separate, but sexual
trapped as bicarbonates at the tissue heritable phenotype changes. It does
site and is transported to the alveoli dimorphism not visible.
not involve DNA alternations.
where it is released out as CO 2. Carbon 108. Pattern baldness is a autosomal trait in
101.
dioxide is immediately converted into Feet of No. of humans. The trait may be displayed by
H2 CO 3 after entering into the blood. mammals toes both males and females, but the
This H2 CO 3 forms bicarbonates. expression of these genes is dependent
Plantigrade 5 digits (man)
95. Cyanosis is bluish colouration of skin on the hormonal constitution of the
and mucous membranes due to Unguligrade 1 digit (horse) individual.
excessive deoxygenated haemoglobin Digitigrade 4 digits (cat) Muscular dystrophy is a genetic disorder
in blood. characterised by progressive skeletal
Humans are plantigrade because they muscle weakness. It involves mutated
Leukemia is a cancer of blood.
walk on the entire soles of the foot. X -chromosome gene. G-6-PD
Lathyrism is characterised by tremors. Horse is called unguligrade because it deficiency syndrome and Hunter
Paraplegia is due to spinal cord injury walks on its hooves. Cats are digitigrade syndrome are also due to X -linked
which results in paralysis of lower limbs. because they walk on their digits. inheritance.
96. The statement IV is incorrect. It is 102. Helobial type of endosperms are found 109. Amino acid, leucine has 6 codons –
corrected as follows mostly in monocots. Here the first UUA, UUG, CUU, CUC, CUA, CUG.
Turner’s syndrome is caused due to division of primary endosperm is It has the greatest number of codons.
the absence of one of the followed by the formation of wall forming
110. The statements II and III are incorrect.
X-chromosomes, i.e. 45 with XO. two chambers. Further divisions in both
The host DNA is not hydrolysed during
Such persons are sterile females who chambers are free nuclear division.
lysogenic phase whereas the host DNA
have rudimentary ovaries and In nuclear endosperm, primary often gets hydrolysed along with the host
undeveloped breasts. endosperm nucleus cell divides by free cell in the lytic phase. In lysogenic phase,
nuclear divisions, which are not followed the host cell does not get lysed.
97. All the given statements are correct
by the cell wall formation. In mosaic
about spores found in plants. Motile, 111. Proenzyme pepsinogen on exposure to
endosperm, the tissue of endosperm is
flagellated zoospores are produced in HCl gets converted to pepsin. Pepsin
not homogeneous. Mature endosperm
Chlorophyceae and Phaeophyceae converts proteins into proteoses and
with any degree of irregularity and
from zoosporangia during favourable peptones.
unevenness in its surface is called
conditions. Aplanospores are 112. The statement (d) is incorrect regarding a-1
ruminate endosperm.
produced during unfavourable Antitrypsin (AAT). The condition of
conditions. Tetraspores, non-motile 103. In plants of family – Brassicaceae,
emphysema can be treated by inhaling an
spores are found in red algae. stamens are 6, arranged in two whorls,

MODULE 2
www.jeebooks.in
Prep Catalysis for NEET ~ Mock Test 9 293

aerosolspray containing AAT, so that it 121. Bear and fox are monoestrous whereas 128. The statement (c) is incorrect. The graph
reaches the alveoli and inhibits the all the others are polyestrous. showing degree of absorption of light by
elastase activity there. AAT for this Monoestrous species have only one a pigment as a function of wavelength is
treatment can be extracted from blood breeding season per year. called absorption spectrum.
donations, but only in very small 122. The statement (b) is incorrect Action spectrum on the other hand, shows
amounts. regarding opening of stomata. It is the degree to which different wavelengths
113. Tail wagging dance is a method of corrected as affect the rate of photosynthesis.
communication used by honeybees to Guard cells become turgid during the
129. At position,
convey the location of food. opening of stomata.
By performing this dance, honeybee 1 – The proton motive force is
The movement of stomata is shown in
shares information about the direction generated, powering ATP synthesis.
response to entry or exit of water from
and distance of food. If it moves the guard cells. 2 – Krebs cycle occurs oxidising two
vertically upward, the direction of food molecules of acetyl Co-A.
123. The dicotyledonous stem hypodermis
source is directly towards the sun. 3 – Electron transport chain occurs
comprises of collenchyma.
114. Monocytes are the largest of all types coupled to proton transport across
The monocotyledonous stem
of leucocytes and amoeboid in shape. hypodermis comprises of the inner membrane.
Monocytes constitute 2-10% of blood sclerenchyma. 130. Modern synthetic theory of evolution
leucocytes. They generally change into involves gene mutation, changes in
124. The cells of epithelial tissues are very
macrophages after entering tissue chromosome structure and number,
closely packed. Neighbouring cells are
spaces. Neutrophils constitute about genetic recombination and natural
held together by cell junctions.
65% of blood leucocytes. NK cells are selection.
natural killer cells. (i) Gap junction facilitates cells to
communicate by connecting the Gene mutations are changes in
115. Endoparasites are parasites, which live cytoplasm of adjoining cells. chromosome structure and number.
within the host’s body as in the
(ii) Tight junction helps to store Genetic recombination provides the
digestive tract, body cavities, various
substances from leaking across a genetic variability without which changes
organs, or blood or other tissues of the
tissue. cannot take place.
host, e.g. Plasmodium malariae.
(iii) Adhering junction performs Natural selection guides populations of
Rat flea is an ectoparasite. These are cementing of cells. organisms for adaptation.
the parasites, which live on the outside
Desmosomes are cell structure 131. Eusporangiate is the sporangium which
of host body.
specialised for cell to cell adhesion. is developed from a superficial group of
116. Allochemicals are chemicals produced These mechanically integrate cells cells, e.g. Selaginella.
by a species which affects other species, within tissues and function to resist
e.g. Chlorella, a common alga, produces mechanical stress. Leptosporangiate is the sporangium
a bacteriocide that not only kills bacteria which is developed from a single
125. Apical dominance is a condition where, superficial cell, e.g. Dryopteris.
but also retards the growth of Daphnia, axillary buds are inhibited by auxin
which feeds on Chlorella. produced by apical meristem. When 132. Histamine and kinin are alarm chemicals
117. The statement III and IV are incorrect. apical meristem is removed, the axillary which may cause inflammation at the
Urethra serves as a common passage bud becomes free from hormone spot of insect bite.
for semen and urine. Storage of urine is inhibition and like apical meristem, it Interferons are proteins secreted by
a function of urinary bladder. The loop develops into a stem or flower. virus-infected cells that inhibit
of Henle plays a major role in 126. The oceans cover around 70% of the neighbouring cells from making new
conserving water. earth’s surface. Phytoplanktons present viruses. Dopamine is a neurotransmitter.
118. The option (c) is correctly matched. in the ocean obtain energy through Opsonin are antibodies which make
Mucous neck cells and oxyntic cells photosynthesis. Hence, maximum microbes more susceptible to
present in the gastric glands secrete amount of carbon dioxide fixation phagocytosis.
mucus and HCl, respectively. b-cells of through the process of photosynthesis 133. The function of glycocalyx in bacteria is
the islets of Langerhans secrete insulin takes place in oceans. protection from phagocytosis and
which reduces the blood sugar level. prevention of desiccation. It also enables
127. Cirrhosis of liver is caused by the
119. The statement (d) is false regarding the them to adhere to the surfaces.
chronic intake of alcohol.
flagella of bacteria and is corrected as Glycocalyx is made up of gelatinous,
Neurosis is a class of mental disorders polysaccharide or polypeptide.
9 + 2 pattern of flagellum structure is involving chronic distress but neither
absent. delusions nor hallucinations. 134. The development of new variety involves
Flegellum is a single-stranded structure a series of inter-related and largely
Psychosis is an abnormal condition of
similar to microtubular fibre, but with interdependent activities as in following
the mind. The patient experiences
(9 + 0) arrangement. steps
disconnection from reality, e.g.
120. Prolactin release inhibiting hormone is First step is the creation of genetic
schizophrenia.
a polypeptide hormone produced in variation. Second step involves selection
Epilepsy is mental disorder which of desirable characteristics. Third and
the hypothalamus. occurs as a result of brain injury, fourth steps include evaluation of
Growth or somatotropin hormone and stroke, brain tumours. It is selected strains or lines or populations
adrenocorticotropic hormone is characterised by seizures. and multiplication of these evaluated
secreted by adenohypophysis. lines.

MODULE 2
www.jeebooks.in
294 NEET Test Drive

135. Electronic smog refers to invisible 144. Both the statements are correct each other. Thermophytes are plants
electromagnetic radiations. It causes regarding secondary growth in dicot which can tolerate high temperature,
harm to eyes and also affects brain stem. The vascular cambial ring 152. Term metaxenia is used for denoting the
cells of man. These electromagnetic develops as a circular ring since effect of pollens on structure outside
radiations are emitted from computers, beginning. endosperm.
televisions, etc. Annual rings or growth rings occur Xenia refers to the effect of pollen on
136. The sarcoplasmic reticulum is a quite commonly. seeds and fruit in a fertilised plant.
modified smooth ER that serves 145. Azotobacter, a free-living 153. I-131 causes thyroid cancer
alternatively a storage site for cellular nitrogen-fixing bacterium does not form
calcium, which are necessary for Sr-90 causes bone cancer
any association with the host plant.
muscle contraction. Radon causes lung cancer
Rhizobium, Frankia, Xanthomonas are
137. The statement III is incorrect. It is symbiotic nitrogen-fixing bacteria that All are the components of radioactive
corrected as follows live in a mutualistic association with the pollution. Hence, all the options are
host plant. These live in small knob-like correctly matched.
Niche have only one species. It is a
functional description of the role, a protuberances known as nodules. 154. Plants which occur below the water level
species plays in a community. 146. Procumbent is the stem that creeps on are called submerged plants. When
138. Stratum functionale is 2/3rd superficial the ground totally, e.g. Tribulus, pollination occurs in these plants, it is
of endometrium supplied with spiral Convolvulus microphyllus. called hypohydrophily. where pollination
arteries which is shed during 147. Generally, it is considered that before occurs by agency of water.
menstruation. The thick membranous the origin of life, some organic 155. The species which is susceptible to
myometrium is composed of smooth molecules were formed from water and extinction has large body size, low
muscle layers. some components from primordial reproductive rate and occupies the
Antrum is a fluid-filled space in the earth’s atmosphere. These molecules highest trophic level in food chain.
follicle. probably included nucleic acids, amino Generally, larger the body size of an
acids, proteins and lipids. animal, the longer it lives and the fewer
139. Except option (b), all other strategies offsprings it produces each year.
By chance some molecules of RNA
are used for controlling the expression Relatively large animals also tend to
of genes in eukaryotes. Feedback may have enzymatic properties,
catalysing the assembly of copies of have relatively low population densities.
inhibition via allosteric modification Large animals due to their low
controls the enzyme activity. themselves. Protein-lipid microspheres
population densities are at increased risk
enclosing these RNA molecules may
140. The correct matches are as follows of extinction.
have formed the first cell-like
Trichoderma polysporum – 156. Eukaryotic nuclei contain three distinct
Cyclosporin-A organisms called protocells.
types of RNA polymerases that differ in
Aspergillus niger – Citric acid 148. Gibberellins break the dormancy of RNA they synthesise.
Pseudomonas denitrificans - seeds due to environmental conditions
The correct match is given by option (a).
Cyanocobalamin and promote germination.
Seed dormancy sets in the seeds to 157. The genetic code is triplet, the order of
Enterobacter aerogens – Lysine
overcome the unfavourable conditions base pairs along DNA molecule controls
141. The two important types of secondary the kind and order of amino acids found
for seedling and germination.
structures of proteins are a-helix and in the proteins of an organism. The start
b-pleated sheet. b-sheets consist of 149. The structure A is centriole, which codon is the first codon of an mRNA
b-strands connected laterally by atleast helps in assemblage of meiotic spindle transcript translated by ribosome. The
two or three backbone H-bonds, fibres at both ends. The stage of cell most common start codon is AUG which
forming a generally twisted, pleated division is Anaphase-I. Here the codes for methionine.
sheet. A b-strand is a stretch of homologous chromosomes separate,
polypeptide chain, 3-10 amino acids 158. Ductus choledochus combines with
with sister chromatids remaining
long with backbone in an extended pancreatic duct or duct of Wirsung to
attached to each other.
conformation. form common hepatopancreatic duct.
150. Synthesis of the DNA and the histone This hepatopancreatic duct then opens
In a-helix, the chain is spirally coiled, proteins takes place during the into the duodenum.
generally in a right-handed manner. S-phase (synthetic phase) of
This structure is formed through 159. The given example is of detritus food chain
interphase in cell cycle.
H-bonding in single amino acid chain Dead leaves ¾® Wood louse
only. In this phase, the cell synthesises a
replica of its genome by the process of (Producer) (Primary consumer)
142. The major disadvantage of cosmids is DNA replication. Thus, DNA content ¾® Black bird
their inability to accept more than becomes doubled. (Secondary consumer)
40-50 kbp of DNA. Cosmids are the
151. Chlamydomonas yellowstonensis is a Detritus food chain begins from dead
vectors, which can accommodate DNA
cryophyte. These are the plants (algae) organic matter. It is made up of
segments upto 45 kbp.
that grow on snow. microbes (fungi, bacteria) and then to
143. HIV virus attaches to CD4 receptor site detritus-feeding organisms (detritivores)
Parasites are those organisms which
of helper T-cells with the help of GP 120 and their predators.
feed on another organism called host.
present on the protein coat of the virus. Grazing food chain starts from green
The decrease in number of TH cells Symbionts are those organisms which
plants and ends at carnivores by
results in decline of immune capacity. live in close physical association with
passing through the herbivores.

MODULE 2
www.jeebooks.in
Prep Catalysis for NEET ~ Mock Test 9 295

160. Myoglobin content and number of vitamin-B1 deficiency may cause brain simpler substances to be utilised during
mitochondria are high in some related diseases like Wernike’s germination. Gibberellins released by
muscles. They are called red muscle syndrome. embryo stimulate amylase and protease
fibres. The high myoglobin content in 168. The option (d) is not the characteristic secretion at aleurone layer.
them facilitates production of ATP by feature of DNA probes. 174. Pili help in the formation of conjugation
oxidative phosphorylation. They also Passenger DNA is the DNA which is tube during conjugation in bacteria. It
have a moderate ability to generate transferred from one organism into also helps in attachment with other cells
ATP through glycolytic pathway. another by combining it with the which is also known as agglutination or
vehicle DNA. clump formation. Flagellum is a single-
In white muscle fibres, there is stranded structure and performs rotation
accumulation of low or no myoglobin. DNA probes are used to identify and movement. Fimbriae are small bristle-like
label DNA fragments that contain a fibres sprouting from cell surface.
161. In phylogenetic system of
specific sequence.
classification, organisms are classified Cell wall of bacteria prevents the cell
according to the evolutionary trends 169. In some prokaryotes, rho factor helps from bursting.
and genetic affinities. This system is in dissociating RNA from polymerase
175. Ligases are enzymes catalysing the
also called cladistics. and template DNA, terminating
synthetic reactions where two molecules
162. Oligotrophic lakes have low plankton transcription. This type of are joined together utilising ATP, e.g.
(producers) density. As a result, they transcriptional termination is called glutamine synthetase.
have low primary productivity and are rho-dependent termination. 176. In plants, Golgi apparatus is not found in
poor in nutrients. 170. The correct matches are given by male gametes of pteridophytes, male
163. Cells of endosperm are generally option (c). gametes of bryophytes and cells of seive
non-green or non-chlorophyllous. But in DNA ligase joins the ends of DNA tube. The Golgi apparatus packages
segments. It catalyses the covalent proteins into membrane bound vesicles
Raphanus, Viscum and Mathiola, etc.,
bonds of segments of an interrupted inside the cell before the vesicles are
chlorophyllous endosperm is present sent to their destination.
(when exposed to light). sugar phosphate strands in ds DNA.
DNase I is an endonuclease that 177. Adrenaline hormone is released by
164. All the options are correct except (a). adrenal medulla. Adrenal cortex releases
The presence of nitrate in water body, digests ds or ss DNA.
hormones mineralocorticoids and
causes blue baby syndrome when DNA topoisomerse relieves torque or glucocorticoids. Aldosterone is
consumed. This is because the body strain. mineralocorticoid and cortisol is
turns bluish in colour. DNA helicase unwinds the double glucocorticoid. So, secretion of
Arsenic causes black foot disease. helix. adrenaline will not be affected in case
165. IBA or Indole-3 butyric acid, is a plant 171. The person suffering from there is injury to adrenal cortex.
hormone in auxin family. It is an hypermetropia or long-sightedness has 178. The cell wall of bacteria comprises of
ingredient in many commercial difficulty in seeing objects near to him. NAG, NAM and peptidoglycan. NAG,
horticulture plant rooting products In this condition, the eyeball becomes NAM (N-acetyl glucosamine and
because stem cuttings of some plants too short and the light rays fall on a N-acetyl muramic acid) are the two
do not produce roots readily and have point of focus behind the retina. important sugar derivatives.
to be treated with a root promoting The cell wall comprises of peptidoglycan
hormone such as IAA, IBA, etc.
172. Root pressure is a positive hydrostatic
pressure developed in xylem vessels in which NAG and NAM ore joined by
166. In pest-resistant legumes, a gene for due to the metabolic activities of roots. short peptide chains.
an enzyme that synthesises a chemical The root pressure has not been found 179. A cell will not enter M-phase if replication
toxic to weevils has been transferred in all plants. of DNA is not complete. DNA replication
from Bacillus bacteria to the Rhizobium takes place in the S-phase. Doubling of
bacteria that live in the root nodules of Rest all the statements are correct
regarding the objections to root chromosomes is necessary before the
these plants. cell division takes place.
pressure theory.
167. Alcohol interferes with the metabolism
173. Aleurone layer secretes amylase and 180. Birds have more RBCs per cubic mm of
of thiamine in liver. Thus, intake
protease enzyme which cause blood than in any other organisms. Thus,
of alcohol by people with
breakdown of starch and protein to they have the richest blood in the animal
kingdom.

SCORE CHART
No. of Correct Answers : A .................. No. of Incorrect Answers : B ..................
Total Marks : x = (A × 4) – (B × 1)
Scores and Expected Rank : If the score lies above 665, then rank will be in between 1-50. For other scores, rank
estimations are given below
Score Rank
664 – 642 51 – 150
641 – 636 151 – 250
635 – 630 251 – 400
629 – 625 401 – 500 MODULE 2
www.jeebooks.in

PREP CATALYSIS
for NEET
Full Length Mock Tests for NEET to Make You Ready to Face the Challenge

MOCK TEST 10 (With Solutions)

Duration : 3 Hours Max. Mark : 720

Instructions
The test is of 3 hours duration and Test Booklet contains 180 questions. Each question carries 4 marks. For
each correct response, the candidate will get 4 marks. For each incorrect response, one mark will be deducted
from the total scores. The maximum marks are 720.
The question paper contains three parts of Physics, Chemistry and Biology respectively.
Part A contains 45 Questions from Physics section. Part B contains 45 Questions from Chemistry section. Part
C contains 90 Questions from Biology.

PART A PHYSICS
1. The stopping potential, when a metal with work 4. A short bar magnet of magnetic moment 0.5 JT -1 is
function 0.6 eV is illuminated with the radiation of placed in a uniform magnetic filed of 0.4 T. The magnet
energy 2 eV will be is in stable equilibrium, when potential energy is
(a) 3 V (b) 2.4 V (c) 1.8 V (d) 1.4 V (a) 0.2 J (b) - 0.2 J (c) zero (d) None
2. A pole PQ is held vertically with its one end P on 5. A cyclic process is shown on the p - T diagram.
the floor and is then allowed to fall. The speed of C B
the other end Q, when it strikes the floor is p
(a) 2gL (b) 3gL
2g 3g
(c) (d) A
L L
T
3. Figure shows current in a part of electrical circuit, Which of the curve shows the same process on p - V
then value of current i is diagram?
3A B C B
2Ω p p
1Ω (a) (b)
1Ω 2Ω
A C A
1Ω i V V
2Ω C B
3A C B
p p
(c) (d)
(a) 6 A (b) 3 A
(c) 2 A (d) Insufficient information A A
V V

MODULE 2
www.jeebooks.in
Prep Catalysis for NEET ~ Mock Test 10 297

6. Phase difference between two coherent sources is 15. Study following statements and find which is true.
(a) 90° (b) constant I. Turbulence promotes mixing and increasing the
(c) variable (d) None of these rates of transfer of mass, momentum and energy.
7. Three uniform spheres, each having mass m and II. For an aerofoil, the orientation of the wing
radius R are kept in such a way that each touches relative to flow direction causes the streamline to
other two. Then, the magnitude of gravitational crowd above wing surface.
force on any sphere due to other two is Choose correct option.
3 Gm 2Gm Gm 3 Gm2 (a) I only (b) II only
(a) (b) (c) (d) (c) I and II (d) Both are wrong
2 R2 R 2
2R 2
4 R2

8. The fundamental frequency of a closed organ pipe 16. If the angle between the polariser and analyser is
30°. Then, what percentage of the incident light is
is equal to the first overtone of an open organ pipe.
transmitted?
If the length of the open pipe is 40 cm, the length
of closed pipe is (a) 75% (b) 25% (c) 50% (d) 100%
(a) 10 cm (b) 15 cm (c) 7 cm (d) 20 cm 17. A coin placed on a rotating table is just slips, if it
is placed at a distance 4r from the centre. If the
9. Two discs of moment of inertia I1 and I 2 and
angular velocity of the table is doubled, then the
angular speeds w1 and w2 are rotating along coin will just slip when at a distance from the
collinear axes passing through their centre of mass centre equal to
and perpendicular to their plane. If both discs are r
pressed face to face to rotate combinedly along (a) 4r (b) 2r (c) r (d)
4
same axis, then rotational KE of system is
I1w1 + I2 w2 (I1 + I2 )(w1 + w2 )2 18. Two wires of same materials and same diameters have
(a) (b) lengths in the ratio 2 : 9. They are stretched by same
2(I1 + I2 ) 2
force. The ratio of work done in stretching them is
(I1w1 + I2 w2 )2
(c) (d) None of these (a) 9 : 2 (b) 4 : 8 (c) 9 : 2 (d) 2 : 9
2(I1 + I2 )
19. In the given circuit, the reading of voltmeter V L
10. The optical path of a monochromatic light is same, and VC are 300 V each. The reading of the
when it goes through 4.0 cm of glass or 4.5 cm of voltmeter V R and ammeter A are respectively,
4 C
water. If the refractive index of water is , then L R=100Ω
3
refractive index of glass will be
VL VC VR
(a) 1.6 (b) 1.45 (c) 1.5 (d) 1.55 A
11. A mass m is moving with a constant velocity along
a line parallel to X-axis. Its angular momentum
with respect to origin or Z-axis is 220 V, 50 Hz
(a) increasing (b) decreasing (a) 100 V, 2.0 A (b) 220 V, 2.2 A
(c) remains same (d) zero (c) 150 V, 2.2 A (d) 220 V, 2.0 A
12. If number of turns per unit length of a coil of solenoid 20. A car is moving at a speed of 90 kmh -1. The
is doubled, the self-inductance of the solenoid will diameter of its wheel is 0.5 m. If wheels are
(a) be halved (b) remain unchange stopped in 25 rotations by applying brakes, then
(c) be doubled (d) become four times angular retardation produced by the brakes is
(a) -7 .25 rad s -2 (b) -7.50 rad s -2
13. A body of mass 50 kg is on the surface of earth. -2
The difference in its weight when moved 200 km (c) -7.15 rad s (d) -7.95 rad s -2
above the surface of earth and 200 km below the 21. If a resistance coil is made by joining in parallel
surface of earth is two resistances each of 10 W. An emf of 1.0 V is
(take, g = 10 ms -2 and R = 6400 km) applied between two ends of coil for 5 min. The
(a)
100
N (b)
125
N (c)
125
N (d)
100
N
heat produced in calories will be
8 6 8 6 (a) 10.3 cal (b) 14.3 cal (c) 16.3 cal (d) 18.3 cal
14. n equal resistors are used to produce a total 22. The net force on an electric dipole placed parallel
resistance in circuit. The ratio of maximum to the to the X-axis, where electric field is also parallel to
minimum resistance is X-axis, will be
(a) 1 : n2 (b) n2 : 1 (a) directed along, Y-axis (b) directed along X-axis
(c) 1 : 1 (d) None of these (c) directed along Z-axis (d) None of these
MODULE 2
www.jeebooks.in
298 NEET Test Drive

23. The potential energy of a normal hydrogen atom in 29. An electric dipole is placed in a uniform electric
ground state is 27.2 eV, then its kinetic energy in field, then its potential energy will be minimum,
the same state will be when the angle between its axis and direction of
(a) 9.07 eV (b) 13.6 eV (c) -13.6 eV (d) 0 electric field is
p p
24. A block of mass M moving on the frictionless (a) (b) p (c) (d) zero
2 4
horizontal surface collides with a spring of spring
constant k and compresses it by length L. The 30. a-particle and proton have same momentum, then
maximum momentum of the block is what is same for both of them?
ML2 kL2 (a) Frequency (b) Wavelength
(a) zero (b) (c) Mk × L (d)
k 2M (c) Time period (d) Torque

25. Water of volume 3 L in a container is heated with a 31. The diode used in the circuit shown in the figure
heating coil of 1.5 kW at 27°C. The lid of the has a constant voltage drop of 0.7 V at all currents
container is open and energy dissipates at rate of and a maximum power rating of 140 mW. What
180 Js -1. In how much time temperature will rise should be the value of the resistance R, connected
from 27°C to 87°C? in series with diode, for obtaining maximum
[Given, specific heat of water is 4.2 kJ/kg -1] current?
(a) 2 min (b) 9 min 53 s (c) 8 min 53 s (d) 12 min 23 s R 0.7 V

26. In the given p - V diagram, A is the initial state


and B is the final state.
p
B
S
1.5 V
P (a) 1.2 W . W
(b) 07 (c) 4W (d) 5W
R T
32. To convert a galvanometer into a ammeter, then
Q one needs to connect a
A (a) high resistance in series (b) high resistance in parallel
O (c) low resistance in series (d) low resistance in parallel
The gas goes from A to B by 33. A solid cylinder rolls without slipping down an
(i) AQPB (ii) ARB (iii) ATSB inclined plane at an angle 60° with the horizontal.
The acceleration of the cylinder is
The heat absorbed by gas is g g 2
(a) (b) (c) 3g (d) g
(a) the same in all three processes 3 3 3
(b) the same in (i) and (ii)
(c) the greater in (iii) than in (ii) 34. The work required in raising a body of mass 90 kg
(d) the greater in (i) than in (ii) to a height of 800 km from the surface of earth is

27. A train approaches a stationary observer, the (take, g = 10 ms -2, radius of earth = 6400 km)
1 (a) 2.5 ´ 106 J (b) 2.3 ´ 106 J (c) 32
. ´ 106 J (d) 15
. ´ 106 J
velocity of train being of the velocity of sound.
20
A sharp blast is blown with the whistle of the 35. Three photodiodes D1 , D2 and D3 are made of
engine at equal intervals of a second. Then, the semiconductors having band gaps of 2.6 eV, 1.9 eV
interval between the successive blasts as heard by and 3.2 eV, respectively. Which one will be able to
the observer is detect light of wavelength of 6000Å?
19 21 20 20 (a) D1 (b) D2 (c) D3 (d) D1 and D2
(a) s (b) s (c) s (d) s
20 20 19 21
36. A balloon with mass 16 kg is descending down with
28. A man stands on a weighing machine placed on a an acceleration 6.2 ms -2. How much mass should
horizontal plateform. The machine reads 50 kg. By be removed from it, so that it starts acceleration up
means of a suitable mechanism, the plateform is with 6.2 ms -2? (take, g = 9.8 ms -2)
made to execute harmonic vibrations of frequency 2 (a) 12.4 kg (b) 12 kg (c) 11.5 kg (d) 13 kg
vibration per second. The change in the reading of
the weighing machine observed due to oscillatory 37. The level of the doubly ionised lithium ( Li + + ) has
motion is (take, amplitude = 5 cm and g = 10 ms -2) the same energy as the ground state energy of the
(a) 79 kg-f (b) 89.5 kg-f
hydrogen atom
(c) 10.5 kg-f (d) 100 kg-f (a) 1 (b) 2 (c) 3 (d) 4

MODULE 2
www.jeebooks.in
Prep Catalysis for NEET ~ Mock Test 10 299

38. The energy density of an electromagnetic wave 43. In Young’s double slit experiment, the distance
moving with velocity c in space of permittivity e 0 between two slits is 0.8 mm and the fringes are
and permeability m 0 is given by [E0 and B0 are observed 1m away. If it produces the second dark
amplitudes electric field and magnetic field] fringe at a distance of 1.5 mm from the middle
B02 1 position of central bright fringe. The wavelength of
(a) (b) e0 E 2
2m 0 2 the monochromatic source of light used is
1 (a) 600 nm (b) 500 nm
(c) e0 B02c 2 (d) All of these (c) 450 nm (d) 480 nm
2

39. A double convex thin lens made of glass (refractive 44. Two radioactive materials x1 and x2 have decay
index m = 1.83) has both radii of curvature of constants 5l and l, respectively. If initially they
magnitude 30 cm. Incident light rays parallel to have the same number of nuclei, then the ratio of
the axis of the lens will converge at a distance L, 1
the number of nuclei of x1 to that of x2 will be
such that e
(a) 30 m (b) 25 cm (c) 18 cm (d) 13 cm after a time.
1 1 e
(a) l (b) l (c) (d)
40. From the top of a tower a stone is thrown up which 2 4l l
reaches the ground in a time t1. A second stone is 45. Calculate the work done by a gas as it is taken
thrown down with the same speed reaches the from 1 to 2, 2 to 3, and 3 to 1 as shown in the
ground in time t2. A third stone is released from figure. The work done in cyclic process
rest from the same point which reaches ground in 1 ® 2 ® 3 ® 1 is
time t3 , then
t1 p
(a) t 3 = t 1 + t 2 (b) t 32 = t 1t 2 (c) t 3 = (d) t 1 = t 2t 3
t2 3
2p0
41. Let the junction diode is ideal. The value of current
in the figure is
+5 V –3 V
p0 2
D1 800 Ω 1

(a) zero (b) 10 ´ 10-2 A (c) 10-1 A (d) 10 mA


V0 3V0 V
42. For a coil having L = 2 mH, current flows through it
(a) p0 V0 (b) 2 p0 V0
is i = t 2e - t , then the time at which emf becomes zero.
(c) -2 p0 V0 (d) - p0 V0
(a) 1 s (b) 3 s (c) 4 s (d) 2 s

PART B CHEMISTRY
46. The reaction which is induced in the preparation of 48. Which of the following product is formed when
ethene from chloroethane is cyclohexanone reacts with OH - ions followed by
(a) b-elimination heating?
(b) reduction
(c) a-elimination (a) (b)
(d) None of the above
47. What will be the product formed when B obtained
in following reaction undergo oxidation? (c) (d) O

CH3CI
(B)
AlCI3 49. Which one of the following equations does not
(A) correctly represents the first law of
thermodynamics for the given process?
(a) Benzyl alcohol (a) Isothermal process : q = - W
(b) Benzoic acid (b) Cyclic process : q = - W
(c) Toluene (c) Isochoric process : DE = q
(d) None of the above (d) Adiabatic process : DE = - W

MODULE 2
www.jeebooks.in
300 NEET Test Drive

50. Which of the following statement is not 59. Value of equilibrium constant for following
characteristic of free radical chain reaction? reaction, N 2( g) + 3H 2 ( g) 2NH 3 ( g) is 0.50 at
-
(a) It gives major product derived from most stable free radical 400°C. What is the value of K p at 400°C.
(b) It is usually sensitive to change in solvent polarity (a) 1.64 ´ 10-3 (b) 1.64 ´ 104
(c) It proceeds in three main steps like initiation, propagation -4
(c) 1.64 ´ 10 (d) 164 ´ 104
and termination
(d) It may be initiated by UV light UV
+ 3Cl2 500 K A,
51. Which of the following cannot be oxidised by H 2O2? 60.
(a) O 3 (b) PbS A is
(c) KI (d) Na 2SO 3 (a) cyclohexane (b) gammexane
Å D (c) cyclobenzene chloride (d) cyclohexane chloride
52. [(CH3 )3 N CH 2CH3 ]OH ¾¾® È

(CH3 )3 N + CH 2 == CH 2 + H 2O 61. Which one of the following statements is not true


about the effect of an increase in temperature on
Which of the following statement(s) is/are correct
the distribution of molecular speeds in a gas?
about the given reaction?
(a) The area under the distribution curve remains the same as
(a) Less substituted alkene is the predominant alkene
under the lower temperature
(b) OH- ion function as a base in E2 elimination
(b) The distribution becomes broader
(c) The above reaction is known as Hoffmann elimination
(c) The most probable speed increases
(d) All of the above statements are correct
(d) The fraction of the molecules with the most probable
53. The method which is used to convert hydrated speed increases
alumina into anhydrous alumina is known as
62. Which of the following process represents the
(a) calcination (b) dressing
change in the optical rotation of freshly prepared
(c) smelting (d) roasting
solution of glucose?
54. XeF6 on complete hydrolysis yields ‘X’. The (a) Mutarotation (b) Specific rotation
molecular formula of X and its geometry (c) Racemisation (d) Tautomerism
respectively are 63. NH 2COONH 4 ( s) 2NH3 ( g) + CO 2 ( g),
-
(a) XeO 4 and tetrahedral (b) XeO 3 and trigonal planar
equilibrium constant of given reaction is
(c) XeO 3 and pyramidal (d) XeO 2 and linear
2 .9 ´ 10-5 atm 3 . The total pressure of gases in
55. Solubility of CaCl 2 in term of solubility product equilibrium with NH 2 COONH 4 ( s) at 30°C is
( K sp ) can be given as: (a) 0.0194 atm (b) 0.194 atm
1/ 3 (c) 5.82 atm (d) 0.0582 atm
æK ö
(a) (Ksp )1/ 3 (b) ç sp ÷
è 2 ø 64. What are the spin-only magnetic moments for
1/ 3 1/ 2 Ni (II) ion in square-planar and octahedral
æK ö æK ö
(c) ç sp ÷ (d) ç sp ÷ geometry, respectively?
è 4 ø è 2 ø (a) 0 and 2.83 (b) 2.83 and 2.83
(c) 0 and 1.73 (d) 0 and 0
56. Which one of the following halogen is oxidised by
nitric acid? 65. The spatial arrangement of the two or more
(a) Chlorine (b) Fluorine polypeptide chains with respect to each other is
(c) Bromine (d) Iodine known as
(a) quaternary structure (b) tertiary structure
57. Among the following pair, which pair has same (c) secondary structure (d) primary structure
size?
(a) Zr 4+, Ti 4+ (b) Zr 4+, Hf 4+ 66. Product obtained on reaction of
(c) Zn2+, Hf 4+ (d) Fe 2+, Ni 2+ 3-chlorocyclopentene with AgNO 2 is

58. Bulbs A and B are connected by a tube having (a) (b)


stopcock. Bulb A has 100 mL volume and contain NO2 ONO
H 2 gas. After opening the gas from A to the
evacuated bulb B, the pressure falls down by 40%. NO2
(c) (d)
The volume of B must be
(a) 200 (b) 125 (c) 66 (d) 75 NO2 Cl

MODULE 2
www.jeebooks.in
Prep Catalysis for NEET ~ Mock Test 10 301

67. Which of the following statement is correct for SN 1 The progress of the reaction can be best
mechanism ? represented by
(a) The first step is dissociation into an intimate ion pair
(b) In second step the leaving group attacks from back side x
(c) There is inversion of configuration x
(d) An external nucleophile attacks the substrate (a) (b)
A+B A+B
68. A solid is made up of two elements P and Q. Atoms AB AB
P are is ccp arrangement and atoms Q occupy all
the octahedral voids and half of the tetrahedral
voids, then the simplest formula of the compound
is
(a) PQ2 (b) P2Q x
(c) PQ (d) P2Q2
(c) (d) All of these
69. Product of the following reaction will be A+B
AB
HCl
?

Cl 75. Which of the following are Newman projections of


meso-2, 3-butanediol are
Me Me
(a) Cl (b)
H OH H OH

Cl
Cl
H OH HO H
(c) (d) Me H
(A) (B)
70. Which of the following complex will give a pair of Me Me
enantiomorphs? Me H HO Me
(a) [Co(NH3 )4 Cl 2 ] NO 2 (b) [Cr(NH3 )6 ] [Co(CN)6 ]
(c) [Co(en) Cl ]+
2 2 (d) [Pt (NH3 )4 ] [PtCl 6 ]
H OH HO H
71. A solution of sodium metal in liquid ammonia is
OH H
strongly reducing due to the presence of
(a) solvated electrons (b) sodium amide (C) (D)
(c) sodium hydride (d) sodium atoms
Choose the correct option.
72. Select the correct statement for the following (a) A, D (b) A, B (c) C, D (d) B, C
reaction,
76. Which of the following statement is incorrect
NH +4 +NO-2 ¾® N 2 + 2H 2O regarding second order reaction?
(a) oxidation number of N has changed from - 2 to + 2 (a) Half-life period for second order reaction is given by
(b) oxidation number of N in NH+4 changed from - 3 to 0 and 1
t 1/ 2 = [where, a0 = initial concentration]
that in NO -2 changed from + 3 to 0 Kao
(c) oxidation number of N in NH+4 changed from + 1to 0 and (b) Time to complete 75% reaction is twice of half-life
that in NO -2 changed from -1to 0
(c) Its half-life is inversely proportional to its initial
(d) no change in oxidation number occurs
concentration
73. Which of the following method is used to purify (d) The units of K is mol -1 L time -1
blood?
(a) Filtration (b) Coagulation
77. Which of the following is a disproportionation
reaction?
(c) Dialysis (d) Electro-osmosis
(a) CaCO 3 + 2H+ ¾® Ca 2+ + H2O + CO 2
74. An exothermic chemical reaction occurs in two (b) CuS + O 2 ¾® Cu + SO 2
steps as follows : (c) 2 HCuCl 2 ¾® Cu + Cu2+ + 4Cl - + 2H+
I. A + B ® X (Fast)
(d) 3ClO - + 2Cr(OH)4- ¾® 2 CrO24 - + 3Cl -
II. X ® AB (Slow)

MODULE 2
www.jeebooks.in
302 NEET Test Drive

78. Which one of the following pairs of compound can 84. Which one of the following C ¾ H bonds is the
be used as starting material in the Grignard weakest for heterolytic fission?
synthesis of 2-phenyl-2-butanol? (a) CH3 ¾H (b) (C 6H5 ) CH2 ¾H
(a) C 6H5Br and (CH3 )3 CCH == O (c) C 6H5 ¾H (d) (C 6H5 )3 C ¾H
O
½½ 85. 0.7 g of Na 2 CO 3 × x H 2 O were dissolved in water and
(b) C 6H5 C CH2CH3 and CH3Br the volume was made upto 100 mL. 20 mL of this
Br
N
solution required 19.8 mL of HCl for complete
½ 10
(c) C 6H5 ¾ CHCH3 and CH3CHO neutralisation. The value of x is
O OH3 (a) 7 (b) 3
½½ ½ (c) 2 (d) 5
(d) C 6H5 C H and CH3 ¾C ¾ CH3
½ 86. Which of the following is an incorrect statement?
CH3 (a) Electron withdrawing inductive effect of the carbonyl group
in ¾COOH group weaken the O ¾ H bond and favours
79. Match Column I with Column II and identify ionisation of a carboxylic acid compared with an alcohol
correct answer by using the codes given below: (b) Inductive effect of the chlorine destabilises the acid and
Column I Column II stabilises the conjugate base
(Elements) (Electronic configuration) (c) Aniline is weaker base than ammonia
(d) Phenol is a weaker acid than water
A. Thorium 1. 4f 0 , 5d 1, 6s 2
B. Americium 2. 3d 5 , 4s1
87. Calculate the value of E° cell if standard reduction
potential of Fe2+ / Fe and Sn2+ / Sn electrodes are
C. Lanthanum 3. 5f1, 6d 1,7 s 2 - 0.44 V and - 014
. V respectively. The cell reaction
D. Chromium 4. 5f 7, 6d 0 ,7 s 2 is
Fe2+ + Sn ¾® Fe + Sn2+
Codes (a) 0.42 V (b) - 0.42 V
A B C D A B C D (c) - 0.30 V (d) - 110
. V
(a) 3 4 1 2 (b) 2 1 3 4
(c) 1 2 3 4 (d) 4 1 2 3 88. Which of the following code is incorrect regarding
allotropes of carbon?
80. Ether in contact with air for a long time form I. Carbon exist in more than one allotropic form.
peroxides. The presence of peroxide in ether can be
tested by adding Fe2+ ion and then adding. II. Diamond is semiconductor while graphite is
(a) KCN (b) SnCl 2 conductor.
(c) HgCl 2 (d) KCNS III. Diamond has layered structure while graphite
has tetrahedral structure.
81. If the bond forming entities are dissimilar with
IV. Diamond is a non-conductor of electricity.
different electronegativity, the bond formed has
(a) I and II (b) II and III
partial ionic character as the electron pair is
(c) I and IV (d) All are correct
attracted by more electronegativity. Such a bond is
called polar covalent bond. Which of the following 89. Alkali metals and alkaline earth metals are s-block
have lowest degree of covalency? elements, electropositive in nature and form
(a) NaCl (b) MgCl 2 electrovalent compounds. Some of such metals are
(c) AgCl (d) CsCl very essential for living organisms. Which of the
following is incorrect regarding biological
82. The rate constant for the reaction significance of Mg, Ca, Na and K ions?
2N 2O5 ¾® 4NO2 + O2 is 3.0 ´ 10-5 s-1. If the rate is
(a) Na + is present outside the cell while K + is present inside
2.40 ´ 10-5 mol L-1s-1, then the concentration of the cell
N 2O5 (in mol L-1) is (b) Na + is present inside the cell while K+ is present outside
(a) 0.04 (b) 1.2 the cell
(c) 1.4 (d) 0.8 (c) Ca 2+ is present as apatite in teeth
(d) Mg 2+ is an essential constituent found in leaf of plants
83. de-Broglie wavelength of electron accelerated
through V volt is nearly given by 90. The monomer of nylon-6 is
2 3
(c) æç
150 150 150 ö 150 (a) caprolactam (b) tetrafluoroethene
(a) Å (b) Å ÷ Å (d) 2 Å
V V è V ø V (c) ethylene glycol (d) isoprene

MODULE 2
www.jeebooks.in

PART C BIOLOGY
91. Industrial melanism is an example of Codes
(a) directional selection (b) normal selection A B C D A B C D
(c) disruptive selection (d) diverging selection (a) 1 2 3 4 (b) 4 2 3 1
(c) 1 2 4 3 (d) 4 1 2 3
92. Acrosome formation is attributed to
(a) Golgi complex (b) endoplasmic reticulum 101. What are Balbiani rings?
(c) ribosomes (d) nucleus (a) Sites of protein synthesis
(b) Sites of RNA synthesis
93. Which pattern of life cycle is found in pteridophytes?
(c) Sites of nucleotide synthesis
(a) Haplontic type (b) Diplontic type
(d) Both (a) and (b)
(c) Diplo-haplontic type (d) All of these
94. Which of the following statements is correct about 102. The guard-cells of stomata are dumb-bell-shaped in
potato crop? (a) doob grass (Cynodon)
(a) Potato tubers are modified underground stem (b) maize (Zea mays)
(b) Each eye of the potato tuber is a bud which grows into a (c) wheat (Triticum aestivum)
new plant (d) Both (a) and (b)
(c) Potato crop can be raised by both tubers and seeds
103. Greenstick is a term that describes
(d) Both (a) and (b)
(a) a kind of rod that is surgically inserted to correct the shape
95. Which among the following organisms is also of bone
known as nutritional supplement organisms? (b) a fracture in which one side of the bone is broken and the
(a) Clostridium sporogenes (b) Saccharomyces cerevisiae other only bent
(c) Streptococcus lactis (d) Clostridium indolis (c) a stick which is organically obtained and manufactured
specially for the use of old people
96. The secretory granules originating from Golgi
(d) a tree in which the trunk remains green throughout its life
complex fuse with
(a) cell wall (b) plasmalemma 104. Which among the following can be achieved when a
(c) vacuole (d) endoplasmic reticulum transgene is introduced into the genome of an
organism?
97. Identify the incorrectly matched pair.
I. Produces a protein of interest
(a) Stilt roots – Brace roots (b) Prop roots – Pillar roots
(c) Buttress roots – Ballast roots (d) None of these II. Produces a desired phenotype
III. Modifies an existing biosynthetic pathway
98. Which of the following is the outermost covering
linking kidney to abdominal wall? IV. Masks expression of a native gene
(a) Renal capsule (b) Renal fascia Choose the correct option.
(c) Renal papilla (d) Renal pelvis (a) I and II (b) I and III
(c) I and IV (d) I, II, III and IV
99. In Southern hybridisation technique, which of the
following procedures is not involved? 105. Consider the following statements.
(a) Autoradiography (b) PCR I. Seed formation with sexual union is called
(c) Electrophoresis (d) Blotting agamospermy.
100. Match the following columns and choose the correct II. Apospory a type of agamospermy involves
option from the codes given below. nucellar.
III. Diplospory is when an unreduced megaspore
Column I Column II
mother cell produces embryo sac.
A. Autogenic 1. Vegetation itself is responsible IV. In adventious apomixis, no embryo sac is
succession for replacing itself formed.
B. Allogenic 2. Other conditions are responsible The incorrect statement is
succession for replacing communities (a) Only I (b) Only II (c) Only III (d) Only IV
C. Retrogressive 3. Succession does not proceed 106. Link species include
succession through normal course (a) mycorrhizal fungi
D. Deflated 4. Human interference and (b) succulents
succession environmental conditions cause (c) perennial herbs
climax community to retrograde (d) lichens

MODULE 2
www.jeebooks.in
304 NEET Test Drive

107. Consider the given figure which alkaloid is 114. With reference to species diversity, identify the
obtained from it? incorrect statement.
(a) Species diversity means species richness in any habitat
(b) Greater the species richness, greater will be their diversity
(c) Species diversity is same as species abundance
(d) Example of species diversity includes Western Ghats that
have a greater amphibian species diversity than Eastern
Ghats

(a) Psilocybin (b) Morphine 115. Brunner’s glands are present only in the
submucosa of
(c) LSD (d) Atropine
(a) duodenum (b) ileum (c) jejunum (d) colon
108. Consider the following statements regarding
reptilies. 116. During a coitus, the human male ejaculates about
(a) 200-300 million sperms (b) 500 million sperms
I. They are poikilothermic animals.
(c) 400 million sperms (d) 100-150 million sperms
II. An endoskeleton of horny epidermal scales is
always present. 117. Which among the following statements is incorrect
III. Skull is monocondylic with single median regarding origin of replication?
occipital condyle. (a) It is the sequence from where the replication starts
(b) Any piece of DNA when linked to this sequence can be
IV. Heart is incompletely four-chambered with
made to replicate within the host cell
two auricles and partly divided ventricles.
(c) This sequence provides the ability to replicate autonomously
The incorrect statement is/are
(d) Generates single copy within the host itself
(a) I and II (b) Only II (c) Only III (d) Only IV
118. Refer to the given figures (A-D) showing hearts of
109. The ventricular systole involves the closure of
different vertebrates. Choose the option that
(a) semilunar valve and aortic valve
correctly identifies A-D.
(b) tricuspid and bicuspid valves
(c) pulmonary valve and vena cava Sinus Truncus
(d) semilunar valve and pulmonary valve venosus arteriosus
Aorta

110. Consider the following statements regarding DNA V RA LA

fingerprinting. A V
I. VNTRs are also known as minisatellites. Vein
Sinus
venosus
II. The technique identifies a person on the basis A B
of person’s DNA specificity.
Sinus venosus
III. VNTRs are used as genetic markers. Pulmonary arch Pulmonary
IV. VNTRs are dissimilar in monozygotic twins. Systemic trunk
Aorta
The incorrect statement(s) is/are arches
(a) I and III (b) Only II RA LA
RA LA
(c) Only III (d) Only IV V
R L RV LV
111. The branch of science dealing with the
Septum
improvement of human race genetically is
C D
(a) Eugenics (b) Euthenics
(c) Genetics (d) Genomics A B C D
(a) Pisces Amphibia Reptile Bird
112. The rate of decomposition is slower if (b) Amphibia Pisces Bird Reptile
(a) detritus is rich in lignin (b) detritus is rich in nitrogen (c) Reptile Bird Amphibia Pisces
(c) detritus is rich in sugars (d) detritus is rich in cellulose (d) Bird Reptile Pisces Amphibia
113. In Loranthus, the ovules are ategmic and they 119. The main criteria for classification used by
have a unique type of endosperm called Whittaker are
(a) chlorophyllous endosperm I. Cell structure II. Mode of nutrition
(b) composite endosperm III. Thallus organisation IV. Reproduction
(c) helobial endosperm The correct options are
(d) free nuclear endosperm (a) I and II (b) III and IV
(c) II and III (d) I, II, III and IV

MODULE 2
www.jeebooks.in
Prep Catalysis for NEET ~ Mock Test 10 305

120. Endoplasmic reticulum is absent in 126. Which among the following statements is/are
I. mature human RBCs incorrect regarding Flavr Savr tomato?
II. kidney cells (a) It has a shorter self-life
(b) Flavr Savr is the result of antisense RNA technology
III. skin cells
(c) The antisense RNA technology has been used to suppress
IV. Germinal cells ethylene biosynthesis
The correct option is/are (d) The fruits remain green and hard on the plants
(a) I and IV (b) Only II (c) Only III (d) Only IV
127. Consider the following statements.
121. Consider the following statements regarding
I. Mucous or Goblet cells secrete alkaline mucus.
functions of insulin.
II. Peptic or Chief or Zymogenic cells, secrete
I. Insulin stimulates glycogenesis in target cells.
active precursors of gastric enzymes.
II. Insulin increases gluconeogenesis.
III. Parietal or Oxyntic cells secrete HCl and CIF
III. Insulin increases glycogenolysis.
(Castle’s Intrinsic Factor).
IV. Insulin reduces the catabolism of proteins and
The incorrect statement is/are
fats. (a) Only I (b) Only II (c) Only III (d) I and III
The incorrect statement(s) are
(a) II and III (b) I and IV (c) I and III (d) III and IV 128. The first extraembryonic membrane to make
appearance in mammal is
122. Rumen, reticulum, omasum and abomasum are the (a) amnion (b) allantois (c) chorion (d) yolk sac
names of
(a) parts of cell organelle, endoplasmic reticulum 129. Clostridium is a
(b) chambers of stomach in cattle and sheep (a) cyanobacteria
(c) parts of small intestine (b) aerobic bacteria
(d) kinds of teeth present in mammals (c) anaerobic non-photosynthetic bacteria
(d) anaerobic photosynthetic bacteria
123. Soil texture is important agriculturally and is
determined by relative proportions of sand, silt and 130. The incorrect characteristic feature of Chordata is
clay. Which type of soils are generally more (a) Notochord is a solid, rod-like structure present on the
suitable for plant growth? dorsal side of the body
(a) Silt (b) Nerve cord is a hollow tube-like structure present on the
(b) Fine sand dorsal side of the body
(c) Clays, clay loams and silt loams (c) Pharyngeal gill slits are like paired openings formed on the
(d) Coarse sand lateral sides of the pharynx
(d) A ventral heart and open circulatory system is present
124. Consider the following statements regarding
cockroach. 131. Which among the following is not the function of
I. The main excretory organs of cockroach are DNA probes?
Malpighian tubules. (a) Identify the short DNA sequences
(b) Identify genes from one species
II. The circulatory system of cockroach is of open
type. (c) Identify genetic defects
(d) Cutting of DNA
III. The respiratory system of cockroach
comprises of a network of tracheae. 132. Factors affecting the rate of water absorption include
IV. Cockroaches are monoecious animals. (a) Low temperature inhibits water absorption
The incorrect statement is (b) Concentration of soil solution inhibits absorption of water
(a) Only I (b) Only II (c) Increased concentration of CO 2 retards absorption of water
(c) Only III (d) Only IV (d) All of the above
125. Which of the following statement(s) is/are correct? 133. Consider the following statements.
I. Both nuclear and cellular types of endosperm I. Pollen grains are very light, small, dusty, dry,
are present in coconut. non-sticky.
II. Angiospermic endosperm is triploid while II. Flowers are without nectar, small,
gymnospermic endosperm is haploid. inconspicuous and nectarless.
III. Development of endosperm occurs from the III. Stamens have long filaments and extorse anthers.
wall of embryo sac by syngamy. IV. Stigmas are sticky, hairy having large
(a) Only I (b) I and II receptive area.
(c) II and III (d) All of these

MODULE 2
www.jeebooks.in
306 NEET Test Drive

V. Often have a single ovule in each ovary and 143. A condition where the expression of one gene pair
many flowers packed into an inflorescence. masks or modifies the effect of another gene pair is
Which type of biotic pollination is being described by (a) codominance
the above characteristics? (b) epistasis
(a) Anemophily (b) Hydrophily (c) dominance
(c) Zoophily (d) Entomophily (d) incomplete dominance
134. The DNA made from mRNA by using enzyme 144. Which of the following statements is true about
reverse transcriptase is known as double-fertilisation and triple fusion?
(a) passenger DNA (b) synthetic DNA (a) Double fertilisation is so called as there is fusion of male
(c) complementary DNA (d) DNA probes and female gametes resulting in the formation of 2n zygote
(b) Triple fusion occurs only after double fertilisation
135. Consider the following statements.
(c) Syngamy is a step of triple fusion
I. Antigen is a protein or polysaccharide (d) Triple fusion is faster than double fertilisation
molecule.
II. Antibody is synthesised by an animal to 145. Consider the following statements regarding Porifera.
combat foreign material. I. The body wall contains outer dermal layer or
III. Antigen is usually a foreign material that pinacoderm and inner gastral layer or
stimulates antibody formation. choanoderm.
IV. Haptens are complete antigens. II. Digestion in animals of this phylum is
The incorrect statement is extracellular in nature.
(a) Only I (b) Only II (c) Only III (d) Only IV III. Respiration occurs through the body surface
by diffusion.
136. The haplontic life cycle is observed in
IV. They are triploblastic.
(a) Spirogyra (b) Fucus
(c) Ectocarpus (d) Polysiphonia
The incorrect statement(s) is/are
(a) II and IV (b) I and II
137. Select the incorrect statement. (c) Only III (d) Only IV
(a) Halophiles are Gram negative
146. Which among the following is affected by
(b) Methanogens are strict anaerobes
greenhouse gases?
(c) Halophiles live as symbionts
(a) Sea level changes (b) Species distribution
(d) Methanogens occur in marshy areas
(c) Food production (d) All of these
138. The capture and storage of atmospheric 147. Inspiratory capacity is
greenhouse gases is known as
(a) TV + ERV (b) TV + IRV
(a) carbon sequestration (b) biosequestration
(c) RV + ERV (d) TV + IRV + ERV
(c) biomagnification (d) Both (a) and (b)
148. Parthenium hysterophorus and Lantana camara
139. An association between Paramecium and alga are examples of
Zoochlorellae is
(a) overexploitation (b) alien species invasion
(a) amensalism (b) mutualism
(c) coextinction (d) habitat fragmentation
(c) parasitism (d) competition
149. The human placenta is referred to as
140. Which muscle forms about 40% of total body
(a) chorioallantoic placenta (b) haemochorial placenta
weight?
(c) Both (a) and (b) (d) None of these
(a) Striated or Skeletal muscles
(b) Non-striated or Smooth muscles 150. Dichogamy, dicliny, herkogamy and
(c) Cardiac muscles self-incompatibility are the examples of
(d) Visceral muscles (a) adaptations for cross-pollination
(b) adaptations for self-pollination
141. Choose the incorrect match.
(c) Both (a) and (b)
(a) Triticum aestivum × Secale cerele — Triticale
(d) None of the above
(b) Raphanus sativus × Brassica oleracea — Raphanobrassica
(c) Potato × Tomato — Pomato 151. Which among the following statements is/are
(d) Brinjal × Tomato — Rabbage correct regarding trophic level?
(a) It is based on the relationships among the organisms
142. Tasmanian wolf is an example of (b) It is based on the source of their nutrition or food
(a) natural extinction (b) mass extinction
(c) Both (a) and (b)
(c) anthropogenic extinction (d) coextinction
(d) None of the above

MODULE 2
www.jeebooks.in
Prep Catalysis for NEET ~ Mock Test 10 307

152. Phenylketonuria is a genetic disorder of 158. A ring of meristematic cells that divides and forms
(a) autosomal dominant gene cork on outside and secondary cortex on the inner
(b) autosomal recessive gene side is
(c) X-linked (a) phellogen (b) phelloderm
(d) monosomic condition (c) intrafascicular cambium (d) interfascicular cambium

153. Which of the following is an example of point 159. Bryophytes resemble algae in
mutation? (a) thalloid plant body
(a) Down syndrome (b) Sickle-cell anaemia (b) absence of roots
(c) Cri-du chat syndrome (d) Haemophilia (c) autotrophic mode of nutrition
(d) All of the above
154. Which of the following statements are correct?
I. Umbilical cord connects the placenta to the 160. In which of the following inflorescences, the older
abdomen of foetus. flowers have longer pedicle than the newer one, so
all flower reach the same level?
II. Placenta acts as an endocrine tissue and
(a) Catkin (b) Spadix
produces several hormones.
(c) Corymb (d) Umbel
III. Placenta helps in the exchange of nutrients
between mother and foetus. 161. Arrange the sequence of the development of root
Choose the correct option. nodules.
(a) I and II (b) II and III I. Chemical recognition.
(c) I and III (d) All of these II. Nodule formation.
155. Which among the following statements is/are III. Curling of root hair.
correct regarding kingdom-Protista? IV. Formation of infection thread.
I. Food reserve is starch, glycogen, fat, Choose the correct option.
paramylon. (a) I – II – III – IV (b) I – III – IV – II
II. Locomotion is through pseudopodia, cilia or (c) I – IV – III – II (d) IV – I – III – II
flagella. 162. Consider the following matches.
III. The organisms possess double and porous I. Housekeeping genes – constitutive genes
nuclear membranes. II. Non-constitutive genes – Luxury genes
IV. Mode of reproduction is both asexual and III. Transposons – Regulatory genes
sexual.
The correct match is/are
Choose the correct option. (a) Only I (b) I and II (c) Only III (d) I and III
(a) I and II (b) I, II and III
(c) I, II, III and IV (d) III and II 163. Respiratory quotient is greater than one in
(a) carbohydrates
156. Leuconoid canal system is found in (b) carbohydrates in anaerobic respiration
I. Spongilla II. Plakina (c) fat associated with carbohydrate synthesis
III. Leucosolenia IV. Sycon (d) carbohydrate with associated organic acid synthesis
The correct option is/are
(a) I and II (b) III and IV (c) only III (d) Only IV 164. Thermoacidophiles are
I.
Gram-negative
157. Consider the following statements regarding II.
mainly aerobic
Neo-Darwinism.
III.
occur in hot sulphur springs
I. Mutations become more effective than
continuous variations during new species IV.under anaerobic conditions, reduce sulphur to
formation. H 2S.
II. Variations accumulate in individuals. Choose the correct option.
(a) I and II (b) III and II
III. Isolation is a significant component of (c) I and IV (d) I, II, III and IV
evolution.
IV. Adaptations result from the multiple forces. 165. An example of autonyms is
The incorrect statement is (a) Rattus rattus (b) Homo sapiens sapiens
(a) Only I (b) Only II (c) Only III (d) Only IV (c) Ziziphus jujuba (d) Rosa indica

MODULE 2
www.jeebooks.in
308 NEET Test Drive

166. In which method, the plants take nutrients in the 172. Which among the following pigments are accessory
form of mist? pigments?
(a) Aeroponics I. Chlorophyll-a II. Chlorophyll-b
(b) Tank system of hydroponics III. Xanthophylls IV. Carotenoids
(c) Film system of hydroponics The correct pigment(s) is/are
(d) All of the above (a) I and II (b) III and IV
167. Which of the following is incorrect about criticism (c) I and IV (d) II and IV
against GM crops? 173. Which among the following is not a plant virus?
(a) Disease resistance (a) Tobacco mosaic virus
(b) Unintended harm to other organisms (b) Abutilon mosaic virus
(c) Gene transfer to non-target species (c) Filoviruses
(d) Reduced effectiveness of pesticides (d) Potato virus-X
168. The Variable Number Tandem Repeats (VNTRs) 174. The age group vulnerable to STDs is
are (a) 15-24 years (b) 10-15 years
(a) 10-100 nucleotides long (b) 20-200 nucleotides long (c) 30-40 years (d) 45-60 years
(c) 30-300 nucleotides long (d) 40-400 nucleotides long
175. Colchicine causes arrest of cell division at
169. Which among the following statements is/are (a) metaphase of mitosis (b) telophase of mitosis
correct regarding cortex? (c) metaphase of meiosis (d) telophase of meiosis
(a) The secondary function of cortex is the storage of food in
stems 176. The cause of gout is
(b) The primary function of cortex is the storage of food in (a) renal failure (b) hyperuricemia
roots (c) hypertension (d) fungal infection
(c) Both (a) and (b)
(d) None of the above
177. Lateral stem development is regulated by relative
levels of
170. Match the following columns and choose the (a) auxins and cytokinins
correct option from the codes given below. (b) auxins and gibberellins
Column I Column II (c) auxins and ethylene
(d) auxins and ABA
A. Vectors 1. To carry DNA into cells and
ensure replication 178. Medullary rays are
(a) parenchymatous pith cells
B. Plasmids 2. A common vector
(b) found in peripheral region between the vascular bundle
C. Genetic markers 3. To identify cells that have been (c) serve the function of radial transport of food and water
transformed (d) All of the above
D Replica plating 4. To make exact copies of bacterial
. colonies on an agar plate
179. Consider the following matches.
I. Aminotelic — Starfish
Codes
II. Ammonotelic — Some molluscs
A B C D
(a) 4 3 2 1 III. Ureotelic — Lizards
(b) 1 2 4 3 IV. Uricotelic — Snakes
(c) 3 2 1 4 The incorrectly matched pair is
(d) 1 2 3 4 (a) Only I (b) Only II
171. Which among the following statements is/are true (c) Only III (d) Only IV
for bryophytes? 180. The carbon sequestration refers to the
(a) In bryophytes, oogamous type of sexual reproduction is (a) process of removing carbon from the atmosphere
present (b) process of addition of carbon in atmosphere
(b) Sporophyte is dependent upon gametophyte
(c) eutrophication
(c) Both (a) and (b)
(d) Both (a) and (b)
(d) None of the above

MODULE 2
www.jeebooks.in

Answer Sheet
1. (d) 2. (b) 3. (d) 4. (b) 5. (b) 6. (b) 7. (d) 8. (a) 9. (c) 10. (c)
11. (c) 12. (d) 13. (c) 14. (b) 15. (c) 16. (a) 17. (c) 18. (d) 19. (b) 20. (d)
21. (b) 22. (d) 23. (b) 24. (c) 25. (b) 26. (c) 27. (a) 28. (a) 29. (d) 30. (b)
31. (c) 32. (d) 33. (a) 34. (b) 35. (b) 36. (a) 37. (c) 38. (d) 39. (c) 40. (b)
41. (d) 42. (d) 43. (d) 44. (c) 45. (d) 46. (a) 47. (b) 48. (c) 49. (d) 50. (b)
51. (a) 52 (d) 53 (a) 54 (c) 55 (c) 56. (d) 57. (b) 58. (c) 59. (c) 60. (b)
61. (c) 62. (a) 63. (d) 64. (a) 65. (a) 66. (a) 67. (a) 68. (a) 69. (b) 70. (c)
71. (a) 72. (b) 73. (c) 74. (c) 75. (a) 76. (b) 77. (c) 78. (b) 79. (a) 80. (d)
81. (d) 82. (d) 83. (a) 84. (d) 85. (c) 86. (d) 87. (c) 88. (b) 89. (b) 90. (a)
91. (a) 92. (a) 93. (c) 94. (d) 95. (b) 96. (b) 97. (d) 98. (b) 99. (b) 100. (c)

101. (d) 102. (d) 103. (b) 104. (d) 105. (a) 106. (a) 107. (b) 108. (b) 109. (b) 110. (d)
111. (a) 112. (a) 113. (b) 114. (c) 115. (a) 116. (a) 117. (d) 118. (a) 119. (d) 120. (a)
121. (a) 122. (b) 123. (c) 124. (d) 125. (b) 126. (a) 127. (b) 128. (d) 129. (c) 130. (d)
131. (d) 132. (d) 133. (a) 134. (c) 135. (d) 136. (a) 137. (c) 138. (b) 139. (b) 140. (a)
141. (d) 142. (c) 143. (b) 144. (d) 145. (a) 146. (d) 147. (b) 148. (b) 149. (c) 150. (a)
151. (c) 152. (b) 153. (b) 154. (d) 155. (c) 156. (a) 157. (b) 158. (a) 159. (d) 160. (c)
161. (b) 162. (b) 163. (b) 164. (d) 165. (b) 166. (a) 167. (a) 168. (a) 169. (c) 170. (d)
171. (c) 172. (b) 173. (c) 174. (a) 175. (a) 176. (b) 177. (a) 178. (d) 179. (c) 180. (a)

EXPLANATIONS

PHYSICS
1. As, we know, K max = hn - f 0 v v At point P current will be divided. This
Also, w = =
r L division can not be made as remaining
= 2 eV - 0.6eV = 14
. eV
part of circuit is not given.
\ Stopping potential, Now, Gain in rotational KE = Loss in PE
1 2 L . JT -1
4. Given, magnetic moment, M = 05
K 14
. eV Iw = Mg
V0 = max = = 1.4V 2 2
e e magnetic field, B = 0.4 T
1 ML2 v 2 MgL
2. Let M be the mass and L be the length ´ × = Now, potential energy
2 3 L2 2 U = - M × B = - MB cos q
of the pole. When, the upper end of
pole strikes the ground, the centre of Þ v 2 = 3gL
For stable equilibrium, q = 0°
L or v = 3gL
gravity falls through height . Then, \ U = - MB = - (05 . ) ´ (0.4)
2 3. By Kirchhoff’s first law, current in the = - 0.2 J
L circuit will be as shown in the figure.
loss in PE = Mg × pV
2 5. We know, = constant
T
Moment of inertia of the pole about the 3A AB - Isochoric process
lower end P , 1Ω 2Ω
BC - Isobaric process
I = moment of inertia of the pole about CA - Isothermal process
the parallel axis through centre of 6A 1Ω 6A 2A 2Ω
P 2A So, option (b) is correct.
gravity + Md 2
ML2 ML2 1Ω
6. Phase difference between two coherent
= I 0 + Md 2 = + 2Ω sources is constant.
12 4 3A
ML2 éQd = L ù 7. If A, B and C be the centres of three
=
3 êë 2 úû spheres, then force of attraction on B
due to sphere A is

MODULE 2
www.jeebooks.in
310 NEET Test Drive

A a 4 ´ 4.5 17. As we know,


Þ mg = = 15
.
3´4 mg
mrw 2 ³ mmg Þ r ³
w2
11. Angular momentum of mass m w.r.t.
origin or z -axis = Linear momentum ´ mg
Thus, coil will slip when, r =
60º Perpendicular distance of line of action w2
B C 2
of linear momentum from origin r 2 æ w1 ö
Þ =ç ÷
Y r1 è w 2 ø 2
2
w ö
= æç
Gm ´ m Gm 2 r2
F1 = = (along BA) m v Þ ÷
4r è 2w ø
(2R ) 2 4R 2
Þ r2 = r
Force of attraction on B due to sphere d
C is YA
18. Stretching force, F = l
X L
Gmm Gm 2 O
F2 = = along BC
(2R ) 2 4R 2 So, L µl
= mv ´ d = mvd = constant L1 l1 2
As F1 = F 2 , the resultant force on Þ = = [QF , Y and A constant]
12. Self-inductance of solenoid = m n 2 Al L2 l 2 9
sphere C is
n = number of turns per unit length 1 l2
F = F12 + F 22 + 2F1F 2 cos 60° \ Self-induction µ n 2 Work done,W = YA
2 L
= F12 + F12 + F12 So, inductance becomes 4 times when 2 2
W1 æ l1 ö L2 æ 2 ö 9 2
2 n is doubled. =ç ÷ × =ç ÷ =
3 Gm W2 è l 2 ø L1 è 9 ø 2 9
= 3F1 = 13. The value of g above earth’s surface,
4 R2 W1 : W2 = 2 : 9
æ 2 ´ 200 ö 75
g h = g æç1 -
2h ö
8. Fundamental frequency of a closed ÷ = 10 ç1 - ÷= 19. As,VL = VC = 300 V
è R ø è 6400 ø 8
organ pipe,
v and below earth’s surface, Given, L-C-R circuit in resonance
n¢1 =
4L \ VR = V = 220 V
gd = g æç1 - ö÷ = 10 æç1 -
d 200 ö 155
÷=
Fundamental frequency of an open è Rø è 6400 ø 16 And circuit will be purely resistive,
organ pipe, Z = R = 100 W
Difference in weight = mgd - mg h
v V 220
n¢1 = = m (gd - g h ) Current, i = = = 2.2 A
2L ¢ Z 100
= 50 æç
155 75 ö 125
- ÷ = N
For 1st overtone of open pipe, è 16 8ø 8 20. Given, v 0 = 90 km h -1
v 5
n 2¢ = 2 n1¢ = = 90 ´ = 25 ms -1
L¢ 14. Resistance will be maximum when
18
connected in series,
As given, n1 = n 2¢ v 25
\ R series = R + R + R ......... upto n w0 = 0 = = 50 rad s -1
v v r 05
.
Þ = R series = nR
q = 2 pN
4L L ¢ Resistance will be minimum when
L ¢ 40 (QN = number of rotation)
Þ L= = = 10 cm connected in parallel,
q = 2 p ´ 25 = 50p rad
4 4 1 1 1
\ = + + ……upto n As, w 2 - w 20 = 2 aq
9. According to law of conservation of Rparallel R R
Þ 02 - (50) 2 = 2 a ´ 50p
angular momentum, R
Þ Rparallel = -2500
I1w1 + I 2 w 2 = (I1 + I 2 )w n Þ a= = - 7.95 rad s -2
100p
R series nR n 2
(I1w1 + I 2 w 2 ) \ = = 21. As two resistances are in parallel
Þ w= Rparallel R 1
(I1 + I 2 ) combination.
n
1 So, the effective resistance
\ Rotational KE = (I1 + I 2 )w 2 15. In turbulent flow different fluid layers 10 ´ 10
2 mix with each other. In aerofoil distance R¢ = =5W
1 (I w + I 2 w 2 ) 2 10 + 10
= (I1 + I 2 ) 1 1 travelled by air on lower surface is more
2 (I1 + I 2 ) 2 compared to distance travelled on top. V2
Heat produced = ´t
(I1w1 + I 2 w 2 ) 2
Hence, both statements are correct. R¢
= 12
2 (I1 + I 2 ) 16. From the law of Malus, I = I 0 cos 2 30° = ´ 5 ´ 60 J
3 5
10. If x1 and x 2 are the distances travelled = I0 60
4 = cal = 14.3 cal
by light in glass and water respectively, 4.2
then \ Percentage of incident light
a transmitted 22. Resultant force on electric dipole can
m g ´ x1 = amw ´ x 2 ® ®
I 3
a 4 = ´ 100 = ´ 100 = 75% be equal to zero as E || p .
m g ´ 4 = ´ 4.5 I0 4
3

MODULE 2
www.jeebooks.in
Prep Catalysis NEET ~ Mock Test 311

23. We know that, PE = - 2KE 30. Wavelength is same because Let m 0 mass is removed, then
-27.2 = - 2KE l=
h
çæQ P ® momentum,÷ö Fb - (m - m 0 )g = (m - m 0 )a ...(ii)
KE = 13.6 eV P è h ® constant ø Fb
24. From law of conservation of energy, \ l ® remains same
a
1 1 k Voltage drop (V )
Mv 2 = kL2 Þ v = ×L 31. Resistance, R =
2 2 M Current (I )
Maximum momentum, P (m – m0) g
Current, I =
k V From Eqs. (i) and (ii), we get
p = Mv = M × L = Mk × L 140 ´ 10-3
M = = 0.2 A 2ma
m0 =
07. g +a
25. Energy gained by water in 1s . - 07
15 . 0.8
R = = = 4W 2 ´ 16 ´ 6.2
= Energy supplied - Energy lost 0.2 0.2 = = 12.4 kg
9.8 + 6.2
= 1500 - 180 = 1320 Js -1 32. To convert galvanometer into ammeter
The required time, Z2
one needs to connect a low resistance 37. En µ
msDq in parallel. n2
t = ++
Energy gaining rate 33. Acceleration of a body rolling without \ E n [Li ] = E 1[H]
3 ´ 4.2 ´ 103 ´ 60° sliding down is éZ 2 ù éZ 2 ù
=
g sin q ê 2ú =ê 2ú
1320 a= ë n û Li ë n û H
» 572 s = 9 min 53 s K2
1+ 32 12
26. Initial and final states are same in all the R2 Þ 2
= Þ n=3
1 n 12
process. For solid cylinder, K 2
= R2
2 38. Average electric energy density,
Hence, DU = 0 in each case
1
So, DQ = DW . and q = 60° UE = e0E 2
g sin 60° 2 3 g 2
\ Area of (iii) is greater than (ii). \ a= = ´g ´ = 2
1 3 2 3 1 æE0 ö 1 2
So, option (c) is correct. 1+ = e0 ç ÷ = e0E 0 …(i)
2 2 è 2ø 4
27. Let, velocity of sound = v
34. Work done = Change in potential energy Average magnetic energy density,
v 2
- æç -
Velocity of train = GMm GMm ö æ B0 ö
W =- ÷
20 (R + h) è R ø ç ÷
1B2 1è 2ø 1 B 02
Frequency of blasts, n = 1s -1 UB = = = …(ii)
2 m0 2 m0 4 m0
= GMm æç -
1 1 ö
Apparent frequency of blasts, ÷
èR R + h ø Now, E 0 = cB 0 …(iii)
v v 20 -1 GMmh
n¢ = ´n= ´ 1= s =
v - vs v 19 From Eqs. (i) and (ii), we get
v - R (R + h)
20
gR 2mh mgh UE = UB
1 19 or W = =
Time interval = = s R (R + h) æ hö \ Energy density of electromagnetic
n ¢ 20 ç1 + ÷ wave,
è Rø
U = UE + UB = 2UE = 2UB
28. The maximum acceleration is (QGM = gR 2 )
90 ´ 10 ´ 800 18 B2 1
2 2 2
a max = w A = 4 p n A Þ W = ´ \ U = 0 = e 0E 02
æ1 + 800 ö 2m 0 2
= 4 ´ 9.87 ´ 4 ´ 0.05 = 7.9 ms -2 ç ÷
5
è 6400 ø =
1
e0B 02c 2
Maximum force on the man
= 2.3 ´ 106 J 2
= m (g + a max)
= 50 (10 + 7.9) 35. Energy of incident light, 39. The parallel rays converge at a
-34 8 distance equal to focal length of lens
= 895.0 N or 89.5 kg-f 6.6 ´ 10 ´ 3 ´ 10
hn = -7 -19
so, f = L
Minimum force on the man 6 ´ 10 ´ 16
. ´ 10
1 æ1 1ö
= m (g - amax) = 2.06 eV But = (m - 1)ç - ÷
f è R1 R 2 ø
= 50(10 - 7.9)
2.06 is greater than the band gap of
. - 1) æç
= 105.0 N or 105 1 1 1ö
. kg-f photodiode D2 . = (183 + ÷
f è 30 30 ø
Change in reading = 895
. - 105
. 36. In 1st case, mg - Fb = ma …(i)
f = 18 cm
= 79 kg-f
Fb
40. Let h be the height of tower and u be
29. When axis of dipole is parallel to the
a the speed of projection of stone
direction of electric field, then potential
1 1
energy is minimum. \ h = - ut1 + gt12 = ut 2 + gt 22 …(i)
i.e., f = 0° 2 2
mg 1 1
\ U = - pE cos 0° = - pE Also, h = 0 + gt 32 = gt 32 …(ii)
2 2

MODULE 2
www.jeebooks.in
312 NEET Test Drive

From Eq. (i), we get Þ 2te - t - t 2e -t = 0 44. We know that, N = N 0e - lt


1
ut 2 + ut1 = g (t12 - t 22 ) Þ te -t (t - 2) = 0 N = number of radioactive nuclei
2 As, t ¹ ¥ and t ¹ 0 present at some instant
1
Þ u = g (t1 - t 2 ) \ t = 2s N0 = nuclei at t = 0
2
43. If b is fringe width, then spacing N1 e - l1t e -5 lt
From Eqs. (i) and (ii), we get Þ = - l t = - lt = e -4 lt
1 2 1 1 between second dark fringe and middle N2 e 2 e
gt 3 = - g (t1 - t 2 )t1 + gt12 position of central bright fringe N1 1
2 2 2
b 5b As, = (given)
Þ t 32 = t1t 2 +b+b= N2 e
2 2 1
41. As, If = =
V 5 - (-3) 5b Þ = e -4 lt Þ e 4lt = e
Given, . ´ 10-3
= 15 e
R 800 2 1
= 10-2 A = 10 mA 5 Dl Þ 4 lt = 1 Þ t =
´ . ´ 10-3
= 15 4l
di 2 d
42. As, i = t 2e - t and | e| = L 45. DW1 ® 2 ® 3 ® 1 = Area of 1 ® 2 ® 3
5 1´ l
dt Þ ´ = 15. ´ 10-3
di 2 0.8 ´ 10-3 1
So, emf will be zero when =0 =- (2 p 0 - p 0 )(3V0 - V0 ) = - p 0V0
dt Þ l = 480 nm 2
di
Now, = 2te -t - t 2e -t = 0
dt

CHEMISTRY
46. Alkyl halides on heating with alcoholic 49. According to first law of proceeds in the direction that gives the
potash eliminate one molecule of thermodynamics, less substituted alkene.
halogen acid to form alkenes. This DE = q + W 53. Calcination is a process in which ore is
reaction is known as heated below its melting point to expel
For isothermal process, DE = 0
dehydrohalogenation reaction. This is water from a hydrated oxide or carbon
an example of b -elimination reaction. Hence, q = - W.
dioxide from carbonate ore. The
The reaction is as follows : For cyclic process, DE = 0 reaction showing conversion of
b a
H3 C — C H2 X ¾¾¾¾®
Alc. KOH Hence, q = -W hydrated alumina into anhydrous
- HX alumina is as follows :
For isochoric process, DV = 0
°C
CH2 == CH2 (X = Cl, Br, I) Hence, DE = q 2Al(OH) 3 ¾1500
¾¾ ¾® Al 2O 3 + 3H2O
47. CH3 (W = pDV = 0) 54. XeF6 + 3H2O ¾® XeO 3 + 6HF
CH3CI
AlCl3 For adiabatic process, q = 0 XeO 3 is a colourless explosive solid
Hence, DE = W and has a pyramidal molecular
A Toluene
B structure.
Thus, only option (d) is incorrect.
55. For the reaction,
B on oxidation produces benzoic acid. 50. Free radical chain reaction is initiated
by UV light. It proceeds in three main CaCl 2 - Ca 2+ + 2Cl -
CH3 COOH S 2S
[O] steps like initiation, propagation and K sp = S ´ (2S ) 2 = 4S 3
termination. It gives major product 1/ 3
K sp æK ö
derived from most stable free radical. s = 3 = ç sp ÷
Benzoic acid 4 è 4 ø
51. O 3 cannot be oxidised by H2O 2 as it is
48. OH reduced to O 2 by H2O 2 . The reaction is 56. Iodine is oxidised by nitric acid. The
O O reaction is as follows :
shown below.
H2O 2 + O 3 ¾® H2O + 2O 2 I2 +10HNO 3 ¾® 2HIO 3
O In this reaction, infact both H2O 2 and +10NO 2 +4H2O
O
O 3 reduces each other. 4+ 4+
+ 57. Zr and Hf have similar ionic radii
52. All the given statements are correct. due to lanthanide contraction.
H Quaternary ammonium hydroxide on
heating strongly decomposes to yield
58. According to Boyle’s law, at constant n
H+
water, a tertiary amine and alkene. andT , we have
O O Å -
O D pinitial ´ Vinitial = p final ´ Vfinal
OH [(CH3 ) 3 N CH2 CH3 ]OH ¾¾®
p AVA = 0.6 p A (VA + VB )
(CH3 ) 3 N + CH2 == CH2 + H2O
This method proceeds via E 2 p A (100) = 0.6 p A (100 + VB )
H
elimination in which the hydroxide ion 100 = 0.6 (100 + VB )
This is an example of aldol functions as the base. Elimination in 166 = 100 + VB
condensation reaction. alkyl-trimethyl ammonium hydroxide or VB = 166 - 100 = 66

MODULE 2
www.jeebooks.in
Prep Catalysis NEET ~ Mock Test 313

59. We know that, K p = KC (RT ) Dn g 66. AgNO2 Reduction property is mainly due to
-2 -2 – AgCl solvated electrons.
= 050
. (RT ) = 050
. (0.082 ´ 673)
Cl NO2 72. NH + + NO –
. ´ 10-4
= 164 4 2 N2 + 2H2O
67. SN1 = Internal nucleophilic substitution
Cl
reaction –3 +3 0
Cl Cl Reduction
60. UV In SN 1mechanism, retention of
+ 3Cl2 500 K configuration will be observed. Oxidation
Cl Cl The first step is dissociation into an
intimate ion pair.
73. Purification of blood which is a colloidal
Cl sol is done by the process of dialysis.
Gammexane
(A) Mechanism The most important application of
61. dialysis is the artificial kidney machine
R1 O
Fraction of molecules

T1 used to purify blood in case of patients


R3
S whose kidneys have failed to work.
R2 O HCl Cl 74. The reaction occuring in two steps has
T2 T2>T1
First step –HCl two activation energy peaks. The first
R1 R1 step being fast needs less activation
O Cl
energy. The second step being slow,
R3 ~ R3
S S needs more activation energy.
R2 O Cl O O Therefore, second peak will be higher
R2
Molecular speed than the first.
75. Me
The peak of the distribution curve R1 R1 Cl Me Me
R3 R3 H OH
corresponds to the most probable Rotate,
speed. We can notice that with increase Cr s S 180º
HO
R2 Cl O O
in temperature, the fraction of the R2 HO H
Product obtain with H OH H
molecules with most probable speed
retention in configuration Me Eclipsed
decreases as the graph becomes (A)
broader. Me
68. Four atoms of P contributes to one unit
62. A spontaneous change in the specific cell from ccp arrangement and 4 atoms HO H
rotation of a solution of an optically of Q from the all octahedral voids and 4 HO H
active compound is called mutarotation. atoms Q from the half of the tetrahedral
The specific rotation gradually voids contributes one unit cell. So, Me
decreases from 111° to + 52.5° in case formula of solid is P4Q 8 so, the simplest Me
Me
of a-glucose and increases from + 19.2° formula of the solid is PQ 2 . HO Me
HO Me
to + 52.5° in case of b -glucose. Rotate,
69. 60º
63. NH2 COONH4 (s ) - 2NH3 (g) H+ +
1- x 2x
HO H HO H
+ CO 2 (g) 1, 2-alkyl
H H
x Cl shift Eclipsed
K p = (p NH 3 ) 2 ´ p CO2 +
(D)
2.9 ´ 10-5 = (2 x ) 2 ´ x Cl
Me
2.9 ´ 10-5 = 4 x 3 HO H
x = 0.0194 atm HO H
This reaction proceeds through
Total pressure Me
carbocation rearrangement. meso
= 2 x + x = 3 x = 3 ´ 0.0194
70. A pair of molecules which are not super Here, A and D represent meso
= 0.0582 atm imposable on its mirror image, are compounds.
64. In square planar geometry, number of known as enantiomorphs or 76. For second order reaction,
unpaired electron in Ni 2+ = 0 enantiomers. e.g. [Co(en) 2 Cl 2 ] + . 1
t 50 or t1/ 2 =
In octahedral geometry, number of + + ka0
en en
unpaired electron in Ni 2+ = 2 Cl Cl 3
t 75 % = = 3t1/ 2
Thus, m (magnetic moment) = n (n + 2) Co Co ka0
= 2 ´ 4 BM Thus, time to complete 75% reaction is
Cl Cl
= 8 BM = 2.83 BM en en thrice of half-life.
65. Two or more polypeptide chains may 71. Solution of sodium metal in liquid NH3 77. A reaction in which same species is
associate to give rise to the quaternary + oxidised as well as reduced is a
consists of solvated Na ions and disproportionation reaction.
structure. These are held together by
solvated electrons.
non-covalent forces such as hydrogen HCuCl2 Cu + Cu2+
bonds, electrostatic interactions and Na + (x + y ) NH3 ¾® [Na(NH3 ) x ] +
van der Waals’ interactions. + [e (NH3) y ] - +1 0 +2

MODULE 2
www.jeebooks.in
314 NEET Test Drive

6.626 ´ 10-34
Et O
78. CH3Br + Mg ¾ ¾2¾® CH3MgBr Thus, C6H5OH is stronger acid than water.
=
OH 5.396 ´ 10-25 [V ]1/ 2 87. Half-cell reactions are :
½
C 6H 5 COCH 2 CH 3 1227
. ´ 10-9 12.27 ´ 10-10 (i) Fe2+ + 2e - ¾® Fe ;
¾¾¾¾¾® C6H5 ¾ C ¾ CH2 CH3 = m = m
H+ ½ [V ] 1/ 2 1/ 2
[V ] E °Fe 2+ / Fe = - 0.44 V
CH3 1/ 2 (ii) Sn ¾® Sn2 + + 2e - ;
Å =é
12.27 150 ù
2 - phenyl - 2 - butanol = Å
[V ]1/ 2 êë V úû E ° Sn/ Sn2+ = + 014
. V
79. A. Thorium (90) = 5f 1 6d 17s 2 °
E cell = E °Fe 2+ / Fe + E ° Sn/ Sn2+
84. On heterolytic cleavage,
B. Americium (95) = 5f 7 6d 0 7s 2 Å -
O = - 0.44 + 014
.
C. Lanthanum (57) = 5d 6s 1 2 (C6H5 ) 3 C ¾ H ¾® (C6H5 ) 3 C + H = - 0.30 V
Å
D. Chromium (24) = 5d 5 4s 1 The species (C6H5 ) 3 C is resonance 88. Diamond is a non-conductor while
80. Peroxide will oxidise Fe2+ to Fe3+ which stabilised carbocation. Hence, most graphite is a conductor. Diamond has
gives a blood colour with KCNS. stable. tetrahedral structure while graphite has
O 22 - + 2Fe2+ + 4H+ ¾® 2Fe3+ + 2H 2O layered structure.
85. Na 2 CO 3 × x H2O + 2HCl ¾®
Fe3+ + 3KCNS ¾® Fe(CNS) 3 + 3K+ 89. Ca 2+ is present in teeth as apatite
2NaCl + CO 2 + (x + 1) H2O
(Blood red colour) Ca 3 (PO 4 ) 2 . Magnesium is present in
81. Higher the electronegativity difference, M1V1 = M 2V2 plant as chlorophyll. K+ is present inside
lower is its degree of covalency. Here, 19.8 1 1 the cell and Na+ is present outside
\ M Na 2 CO3 × x H 2 O = ´ ´
CsCl has highest electronegativity 2 10 20 the cell.
difference hence it has lowest degree = 0.0495 M 90. The monomer of nylon-6 is
of covalency.
Strength = molar mass ´ molarity caprolactam.
82. For first order reaction,
7 = (106 + 18 x ) ´ 0.0495 H O
rate = k[N2O 5 ] 106 + 18 x = 141. 4 N
C 533-543 K
2.40 ´ 10-5 = 3.0 ´ 10-5 [N 2O 5 ] Hence, x » 2.0 H 2O
[N2O 5 ]= 0.8 86. Statement (d) is incorrect. Caprolactam

83. de-Broglie wavelength of electron, OH O O H
l=
h
=
h æ\ eV = 1 mv 2 ö
ç ÷
mv 2eVm è 2 ø C  (CH2)5  N
+ H+
-34 n
6.626 ´ 10
= Nylon - 6
. ´ 10-19 ´ V ´ 91
2 ´ 16 . ´ 10-31 Resonance stabilised
phenolate ion

BIOLOGY
91. Industrial melanism is an example of It is an excellent source of protein and 100. In autogenic succession, vegetation is
directional selection. This occurs, vitamins, especially the B-complex responsible for replacing itself due to
whenever, the environment changes in vitamins required in metabolism. change in environmental conditions.
a particular way. Therefore, selective 96. The secretory granules originating from In allogenic succession, other
pressure for species to change in Golgi complex fuse with plasmalemma. conditions apart from vegetation are
response to the environmental change Thus, it contributes to the renewal of responsible for replacing communities.
works. membrane constituents. Retrogressive succession causes
92. Acrosome formation is attributed to 97. All the options are correctly matched. community to retrograde.
Golgi complex. Acrosome is a vesicular Stilt roots are also known as brace In deflated succession, succession does
structure that lies in front of the nucleus roots. Prop roots are also known as not proceed through normal course.
in the sperm. pillar roots. Buttress roots are also 101. Balbiani rings are sites of protein and
93. Diplo-haplontic type of life cycle is found known as ballast roots. RNA synthesis. Balbiani rings are very
in pteridophytes. In this type of life cycle, 98. Renal fascia is the outermost fibrous large puffs on the polytene
sporophytic phase is dominant. covering of kidney. It helps the kidneys chromosomes. These puffs are
Gametophyte is less developed but in their attachment to the body wall. It is particularly well-suited for studies on
nutritionally it is independent. a fibrous connective tissue layer. active genes and on RNA protein
94. The statements (a) and (b) are correct 99. In Southern hybridisation technique, particles.
about potato crop. The statement (c) is PCR procedure is not involved. The 102. In some monocots, such as doob grass
incorrect. Potato crops can be raised Southern hybridisation technique is used (Cynodon), maize (Zea mays), the
only by means of potato tubers. to detect specific DNA sequences in guard cells are dumb-bell-shaped. The
95. Saccharomyces cerevisiae is also DNA samples. It involves transfer of DNA guard cells are involved in controlling
known as nutritional supplement fragments separated by electrophoresis the opening and closing of stomata for
organisms. It is a deactivated yeast. to a filtre membrane. gaseous exchange.

MODULE 2
www.jeebooks.in
Prep Catalysis NEET ~ Mock Test 315

103. Greenstick is a partial fracture, typically it is quicker if it is composed of nitrogen 122. Stomach of ruminant animals (cow,
occurring in children, in which one side and water-soluble substances like buffalo, sheep, goat, camel, etc.)
of the bone is broken and the other side sugars. consists of four chambers, i.e. rumen,
is only bent. It is simple crack without 113. In Loranthus, the ovules are ategmic, reticulum, omasum and abomasum.
breaking the bone into two pieces. It is i.e. without integuments and Rumen is the largest of the four and
also called as hairline fracture. endosperm of different embryo sacs along with reticulum is the site of
104. A gene that is transferred into an unite together to form a unique type of cellulose digestion.
organism by genetic engineering is endosperm called composite 123. Medium and fine textured soils such as
called transgene. When a transgene is endosperm. clays, clay loams and silt loams are
introduced into the genome of an 114. The statement (c) is incorrect. Species generally more suitable for plant
organism, it can achieve one of the diversity refers to species richness in growth because they have most
following, i.e. produces a protein of any habitat. Greater the species satisfactory nutrient and water retention
interest, produces a desired phenotype, richness, greater will be their diversity. capacity.
modifies an existing biosynthetic However, it should not be confused with 124. The statement IV is incorrect. It can be
pathway and masks expression of a species abundance. India is among the corrected as follows
native gene. world’s 15 nations that are exceptionally Cockroaches are dioecious animals, i.e.
105. The statement I is incorrect. It can be rich in species diversity. Greater sexes are separate and both the sexes
corrected as follows amphibian diversity in Western ghats is have well-developed reproductive
Seed formation without sexual union is an example of species diversity. organs.
called agamospermy. It can be divided 115. Brunner’s glands are present only in the
125. The statements I and II are correct. The
into two categories, i.e. gametophytic submucosa of duodenum (not in ileum
statement III is incorrect.
apomixis and adventitious apomixis. and jejunum). The epithelium of
intestine bears a large number of Coconut water is free nuclear
106. Link species include mycorrhizal fungi. endosperm and white kernel is cellular
glands like crypts of Leiberkuhn and
These species help other species to endosperm.
Brunner’s glands.
complete their life cycle. Mycorrhiza is
116. During a coitus, the human male Triple fusion is rare in gymnosperms
an association of fungi with roots of
ejaculates about 200-300 million and the endospam is haploid.
higher plants.
sperms. For the normal fertility, at least A typical angiospermic endosperm
107. The given figure is showing a flowering 60% sperms must have normal shape develops from the primary endospermic
branch of Papaver somniferum. and size and at least 40% of them must nucleus by triple fusion.
Morphine is obtained from it. It is an show vigorous motility.
alkaloid with molecular formula 126. The statement (a) is incorrect regarding
C17H19O 3N. It is a strong sedative and 117. The statement (d) is incorrect regarding Flavr Savr tomato. It can be corrected
analgesic. It is useful for patients who origin of replication. This sequence as follows The expression of
have undergone surgery. provides the ability to replicate polygalacturonase has been reduced in
autonomously, so as to generate its Flavr Savr which leads to improvement
108. The statement II is incorrect regarding multiple copies within the host itself. Its in flavour, total soluble solids and
reptilies. It can be corrected as follows representation in many copies in the longer self-life (2 weeks).
An exoskeleton of horny epidermal cell provides more replicates of
scales or scutes (contains a waterproof inserted DNA sequences. 127. The statement II is incorrect.
protein keratin) is always present. 118. In the given figures, Peptic or Chief or Zymogenic cells,
109. The ventricular systole involves the secrete inactive precursors of gastric
A – Pisces
enzymes. These include pepsinogen,
closure of tricuspid and bicuspid B – Amphibia
prorennin (in young mammals), gastric
valves. C – Reptile amylase and gastric lipase.
Ventricular systole increases the D – Bird
128. The first extraembryonic membrane to
ventricular pressure causing the closure 119. The main criteria for classification used make appearance in mammals is yolk
of tricuspid and bicuspid valves to by Whittaker are cell structure, mode of sac. It is a membrane that surrounds
prevent the backflow of blood into atria. nutrition, thallus organisation, the yolk. In mammals, it is reabsorbed
110. The statement IV is incorrect. It can be reproduction and phylogenetic before the embryo is horned.
corrected as follows relationships.
129. Clostridium is an anaerobic
VNTRs (Variable Number Tandem 120. Endoplasmic reticulum is absent in non-photosynthetic bacteria. It is an
Repeats) are similar in monozygotic mature human RBCs and germinal anaerabic free-living nitrogen-fixing
twins only. cells. ER is known to be absent in
bacteria. They add about 10-25 kg of
prokaryotes but is present in all
111. The branch of science dealing with the nitrogen per hectare per annum.
eukaryotic cells except RBCs and
improvement of human race genetically 130. The incorrect characteristic feature of
germinal cells.
is Eugenics. It has two forms, i.e. Chordata is statement (d).
negative eugenics and positive 121. The statements II and III are incorrect
In chordates, a ventral heart and closed
eugenics. In negative eugenics, people regarding the functions of insulin. The
circulatory system is present. It consists
with inferior and undesirable traits are statements can be corrected as follows
prevented from reproducing. Under of a heart as pumping organ along with
Insulin decreases gluconeogenesis and blood vessels.
positive eugenics, people with better glycogenolysis. Glycogenolysis is
and desirable traits are encouraged to breakdown of glycogen. 131. The option (d) is incorrect regarding
produce offspring. Gluconeogenesis is synthesis of functions of DNA probes.
112. The rate of decomposition is slow, if glucose molecules from fat and protein. The cutting of DNA is done by
detritus is rich in lignin and chitin while, restriction endonuclease which result in

MODULE 2
www.jeebooks.in
316 NEET Test Drive

the fragments of DNA. These fragments 141. The incorrect match is option (d). It can for a specific purpose. Some of them
can be separated by using the be corrected as follows turn invasive and cause decline or
technique known as gel Brinjal ´ Tomato ® Bromato extinction of indigenous species.
electrophoresis. Radish ´ Cabbage ® Rabbage
149. The human placenta is referred to as
The crosses made between the plants
132. Factors affecting the rate of water chorioallantoic placenta because
belonging to different genera of the
absorption include low temperature chorion and allantois take part in the
same family are known as intergeneric
which inhibits water absorption. The hybridisation. formation of placenta. The human
concentration of soil solution inhibits placenta is also referred to as
The hybrids produced by this method
absorption of water. The increased haemochorial placenta because in it
are both scientifically as well as
concentration of CO 2 and deficiency of agronomically significant. uterine epithelium, uterine connective
O 2 in the soil also retards absorption of tissue and endothelium of maternal
water. 142. Tasmanian wolf is an example of blood vessels are present.
anthropogenic extinction. These are
133. Anemophily is the pollination that extinctions of organisms by human
150. Dichogamy, dicliny, herkogamy and
occurs by the agency of air. The activities like hunting, overexploitation self-incompatibility are the examples of
pollination by air is being described by and habitat destruction. adaptations for cross-pollination. It is
the above characteristics. the transfer of pollen grains from the
143. A condition where the expression of anther of a flower to the stigma of
134. Complementary DNA (cDNA) is a DNA one gene pair masks or modifies the
made from mRNA by using enzyme another flower, borne on a separate
effect of another gene pair is epistasis. plant of the same species. It results in
reverse transcriptase, which does the In epistasis, genes involved may affect
reverse of transcription. It synthesises combination or mixing up of characters
phenotypic characteristic in an thus, improving the quality or vigour of
DNA from RNA template. antagonistic manner, leading to the species.
135. The statement IV is incorrect. masking or, they may influence each
Haptens are incomplete or partial other in a complementary fashion. 151. The statements (a) and (b) both are
antigens. They are substances, which correct regarding trophic levels. They
144. The statement (d) is true about double
are incapable of inducing antibody are based on the relationships among
fertilisation and triple fusion.
formation by themselves. However, they the organisms and the source of their
Triple fusion occurs more quickly than
can be capable of inducing antibodies nutrition or food. Each organism occupies
double fertilisation.
formation on combining with larger a specific place in the food chain.
molecules (generally proteins) which The second male gamete fuses with
secondary nucleus to triploid primary 152. Phenylketonuria is a genetic disorder of
serve as carriers.
endosperm mother cell. This is known autosomal recessive gene.
136. The haplontic life cycle is observed in as triple fusion. The simultaneous Phenylketonuria is an inherited disorder
Spirogyra. The dominant photosynthetic occurrence of 2 fusion (one sperm in which the level of amino acid,
phase is a gametophyte produced by fuses with egg cell and other sperm phenylalanine increases in the body.
haploid spores. The gametophyte fuses with 2 polar nuclei) events in the 153. The sickle-cell anaemia is an example
produces gametes by mitosis. The embryo sac is called double of point mutation. A point mutation is a
gametes fuse and produce a diploid fertilisation. genetic mutation where a single
zygote, that represents sporophytic
145. The statements II and IV are incorrect. nucleotide base is changed, inserted or
generation. This kind of life cycle is
Digestion in animals of phylum–Porifera deleted from a sequence of DNA.
called haplontic life cycle.
is intracellular (inside collar cells) in 154. Placenta acts as a barrier between the
137. The statement (c) is incorrect. Some nature. The animals of this phylum are foetus and the mother. It is contributed by
methanogens live as symbionts in the diploblastic, i.e. having ectoderm and both foetal and maternal tissue.
rumen of the cattles and help in the endoderm. It synthesises large quantities of
fermentation of cellulose. They occur in 146. The effect of concentration of proteins and some hormones such as
marshy areas where they convert formic greenhouse gases in atmosphere hCG, chorionic thyrotropin, oestrogens,
acid and CO 2 into methane with the causes change in sea level, species progesterone, etc.
help of hydrogen. distribution and food production.
Sea level will rise due to thermal 155. All the statements regarding
138. Biosequestration is the capture and
expansion of oceans. kingdom-Protista are correct. In the
storage of atmospheric greenhouse
organisms of this kingdom, the food
gases by biological processes, i.e. by It shifts the temperature ranges and
affects the distribution pattern of reserve is starch, glycogen, fat and
increasing photosynthesis, aforestation,
carbon trapping in agriculture or by the organisms. paramylon.
use of algae. Locomotion is through pseudopodia,
Increase in temperature results in the
eruption of diseases which will cilia or flagella and the organisms
139. An association between Paramecium possess double and porous nuclear
and alga Zoochlorellae is mutualism, decrease the food production.
membranes. The mode of reproduction
i.e. both the species are benefitted. 147. Inspiratory capacity is TV + IRV. It is the is both asexual and sexual.
140. The striated or skeletal muscles form total volume of air a person can inspire
with maximum effort. It is about 156. Leuconoid canal system is found in
about 40% of the total body weight. Spongilla and Plakina. It is considered
These muscles bring about the 3000-3500 mL. It includes tidal volume
and inspiratory reserve volume. as the most complex canal system. This
movement of the various bones of the
system develops from syconoid type by
skeleton, so are called skeletal 148. Parthenium hysterophorus and Lantana
further folding of walls of radial canals.
muscles. The striated muscles give camara are examples of alien species
shape to the body and also release invasion. The alien species are 157. The statement II is incorrect regarding
heat during contraction. introduced unintentionally or deliberately Neo-Darwinism. It can be corrected as
follows
MODULE 2
www.jeebooks.in
Prep Catalysis NEET ~ Mock Test 317

Variations accumulate in gene pool 165. An example of autonyms is Homo They are also known as ‘antenna
according to Neo-Darwinism. sapiens sapiens. These are the names pigment’.
158. A ring of a meristematic cells that in which specific epithet and 173. Filoviruses are not a plant virus. It
divides and forms cork on outside and infraspecific epithet, i.e. subspecies are causes disease Ebola.
secondary cortex on the inner side is identical. Tobacco mosaic virus, abuliton mosaic
phellogen. It is a couple of layers thick. 166. The plants take nutrients in the form of virus and potato virus-X are plant
It is made of narrow, thin-walled and mist in aeroponics. It is a method of viruses.
nearly rectangular cells. Phellogen cuts growing plants in a soil-free media. It 174. The age group vulnerable to STDs is
off cells on both sides. utilises air or mist environment. 15-24 years. Though all persons are
159. Bryophytes resemble algae in having 167. One of the advantages of GM crops is vulnerable to these infections, their
thalloid plant body, absence of roots and disease resistance. Plant biologists are incidences are reported to be very high
autotrophic mode of nutrition. working to create plants, which are among the adolescent age group.
Bryophytes include the various mosses resistant to viruses, bacteria and fungi. 175. Colchicine causes arrest of cell division
and liverworts. These are non-vascular All others are criticisms against GM at metaphase of mitosis. It is an alkaloid
embryophytes, characterised by the crops. which acts as a poison for mitosis as it
presence of an independent 168. The Variable Number Tandem Repeats does not allow the formation of mitotic
gametophyte and parasitic sporophyte. (VNTRs) are 10-100 nucleotides long. spindle by preventing the assembly of
160. In corymb inflorescence, flowers are These are specific sequences which microtubules.
borne on short peduncle, the older are repeated many times and found at 176. The cause of gout is hyperuricemia. It is
flowers have longer pedicle than the many places within the genome. The the increased level of uric acid in blood.
newer one. So, all flowers reach the loci at which these sequences are Gout is a kind of arthritis which is
same level, e.g. candy tuft, Capradla. present are known as the VNTRs or characterised by repeated attacks of
161. The sequence of development of root minisatellites. burning pain and stiffness.
nodules is as follows 169. Both the statements are correct regarding The decrease in excretion of uric acid
Chemical recognition – curling of root cortex. The secondary function of cortex is through kidneys is the primary cause of
hair – formation of infection thread – the storage of food in stems. hyperuricemia.
nodule formation. The primary function of cortex in roots is 177. Lateral stem development is regulated
During N2 -fixation roots of the legumes the storage of food whereas, the by relative levels of auxins and
and Rhizobium bacteria interact with protective function is the secondary one. cytokinins.
each other. As a result of these nodules 170. Vectors carry DNA into cells and ensure Auxin causes apical bud dominance,
are formed. replication. whereas cytokinin counteracts apical
162. The options I and II are correctly matched Plasmids are type of vector. bud dominance. Ratio of both of these
while the statement III is incorrectly Genetic markers are used to identify hormones will affect development of
matched. cells that have been transformed. lateral stem.
Transposons are also known as Replica plating is used to make exact 178. Medullary rays are parenchymatous
jumping genes. copies of bacterial colonies on an agar pith cells, which are smaller, densely
Regulatory genes include promoters, plate. packed and found in peripheral region
terminators, operators and repressor. 171. Both the statements are true regarding between the vascular bundle. These
163. Respiratory quotient is greater than one bryophytes. In them, oogamous type of serve the function of radial transport of
in carbohydrates undergoing anaerobic sexual reproduction is present and food and water.
respiration. It is the ratio of volume of sporophyte is dependent upon 179. The option III is incorrectly matched pair.
carbon dioxide released to the volume gametophyte. Ureotelic organisms are those
of oxygen consumed in respiration. Its Fertilisation is internal and produces organisms in which excretory product is
value varies for different respiratory diploid zygote in bryophytes. urea, e.g. cartilaginous fishes, snail,
substrates. 172. Xanthophylls and carotenoids are prawn, mammals and aquatic reptiles.
164. Thermoacidophiles are Gram-negative accessory pigments. These support the Lizard is a uricotelic organism in which
and mainly aerobic. They occur in hot function of primary pigments. excretory product is uric acid.
sulphur springs. Under aerobic conditions, Xanthophylls are yellow in colour and 180. The carbon sequestration refers to the
they oxidise sulphur to sulphuric acid and are oxidised carotenoids. process of removing carbon from the
when the conditions are anaerobic, they The carotenoids are yellow to atmosphere and depositing it in a
reduce sulphur to H 2 S. yellow-orange in colour. reservoir.

SCORE CHART
No. of Correct Answers : A .................. No. of Incorrect Answers : B ..................
Total Marks : x = (A × 4) – (B × 1)
Scores and Expected Rank : If the score lies above 665, then rank will be in between 1-50. For other scores, rank
estimations are given below
Score Rank Score Rank
664 – 642 51 – 150 635 – 630 251 – 400
641 – 636 151 – 250 629 – 625 401 – 500 MODULE 2
www.jeebooks.in
318 NEET Test Drive

PREP CATALYSIS
for NEET
Full Length Mock Tests for NEET to Make You Ready to Face the Challenge

MOCK TEST 11 (Unsolved)

Duration : 3 Hours Max. Mark : 720

Instructions
The test is of 3 hours duration and Test Booklet contains 180 questions. Each question carries 4 marks. For
each correct response, the candidate will get 4 marks. For each incorrect response, one mark will be deducted
from the total scores. The maximum marks are 720.
The question paper contains three parts of Physics, Chemistry and Biology respectively.
Part A contains 45 Questions from Physics section. Part B contains 45 Questions from Chemistry section. Part
C contains 90 Questions from Biology.

PART A PHYSICS
1. The potential difference between X and Y in the 4. An open door refrigerator is kept inside a closed
following figure is room. The temperature of the room is
5V (a) decrease (b) increase
3A 2Ω + – – + 4Ω – 1V
+ 1Ω
X Y (c) remain same (d) depend on the atmosphere
2Ω
6V 5. A charge q is enclosed in a cube at its centre, then
(a) 24 V (b) 25 V
electric flux associated with two opposite faces of
(c) 18 V (d) 27 V
the cube is
2. A charged particle (charge q) is moving in a circle q 2q q
(a) (b) (c) (d) zero
of radius R with an uniform speed v. The 3e0 3e0 6e0
associated magnetic moment m is given by
qvR 6. An apple of mass 0.25 kg falls from a tree. The
(a) (b) qvR 2 ratio of acceleration of the apple towards the earth
2
and of the earth towards the apple is (mass of
qvR 2
(c) (d) qvR earth = 6 ´ 1024 kg, radius of earth = 6.4 ´ 106 m
2
and G = 6.67 ´ 10-11 N-m 2kg -2)
3. The refractive index of glass is 1.5. The speed of 1
(a) (b) 24 ´ 1024
light in glass is 24 ´ 10-24
(a) 3 ´ 108 m/s (b) 2 ´ 108 m/s 1
(c) 24 ´ 10-24 (d)
(c) 1 ´ 108 m/s (d) 4 ´ 108 m/s 24 ´ 1024

MODULE 2
www.jeebooks.in
Prep Catalysis for NEET ~ Mock Test 11 319

7. A body is projected with a velocity of 40 ms -1. 12. The current in the resistance of 2W in the following
After 2 s, it crosses a vertical pole of circuit is (diodes are ideal)
height 20.4 m. The angle of projection be
( g = 9.8 ms -2) D1 D2 2Ω
(a) 15° (b) 30° D3
(c) 45° (d) 60°
+ –
8. A monoatomic ideal gas is used in a Carnot 4V
engine as working substance. If during the (a) 2 A (b) 1 A (c) 4 A (d) 0 A
adiabatic expansion part of the cycle, volume of
the gas increases from V to 8V . The efficiency 13. A monochromatic light of wavelength 6000Å is
of the engine is incident on a thin glass plate of refractive index 2,
(a) 15% (b) 35% such that the angle of refraction in the plate is 45°,
(c) 55% (d) 75% then smallest thickness of the plate which will make it
dark by reflection
9. A solenoid of length 1m and 3 cm diameter has (a) 3000 Å (b) 1500 Å
five layer of windings of 850 turns each and (c) 4000 Å (d) 4500 Å
carries a current of 5A.
14. Assuming Bohr model of hydrogen atom. If an electron
Then, study the following statements regarding jumps from first excited state to the ground state of
the solenoid. hydrogen atom, then the percentage change in the
(I) The magnetic field at the centre of speed of electron will be
solenoid is directly depends on the current (a) 25% (b) 50% (c) 100% (d) 200%
and the numbers of turn per unit length of
15. A sound wave propagating in air has a frequency of
the solenoid.
8000 Hz. The percentage change in wavelength, when
(II) The magnetic flux and the magnetic field the wave front, initially in a region where temperature
at the centre are 1.89 ´ 10-5 Wb and 27°C enters a region where temperature decreases to
2.671 ´ 10-2T respectively. 9°C, is
(a) 3% (b) 7% (c) 5% (d) 10%
Choose the correct statements and mark the
correct option given below. 16. If the earth be at one and half of its present distance
(a) I only from the sun, the change in the number of days in one
(b) II only year will be
(c) I and II (a) 300 days (b) 250 days
(d) None of the above (c) 305.5 days (d) 275.2 days

10. In our galaxy (milky way), there are billions of 17. A ring of radius r is first rotated with an angular
stars. Scientists say there are 3 red dwarf stars velocity of w0 and then carefully placed on a rough
out of each 4. Which law they use for horizontal surface. The coefficient of friction between
determination of surface temperature of a star? the surface and the ring is m, then the time after which
2
(a) Planck’s law its angular speed becomes is
3
(b) Kirchhoff’s law w0 r w0 r 3w0 r w0 r
(c) Rayleigh-Jeans law (a) (b) (c) (d)
3m g 2mg 2mg mg
(d) Wien’s displacement law
A 18. Calculate the dispersive power for crown glass from
11. A nucleus X has mass represented by
Z the given data mV = 1.523 and m R = 1.5145.
m( A. Z ). If mP and mn denote the mass of (a) 0.01639 (b) 1.05639
proton and neutron, respectively and BE the (c) 0.05639 (d) 2.05639
binding energy (in MeV), then
(a) BE = [m( A, Z ) - Zmp - ( A - Z )mn ]c 2 19. An electromagnetic wave passing through vacuum is
described by E = E0 sin( kx - wt ), B = B0 sin( kx - wt ).
(b) BE = [Zmp + ( A - Z )mn - m( A, Z )] c 2 The correct equation is
(c) BE = [Zmp - Amn - m( A, Z )] c 2 (a) E0 B0 = wk (b) E0 k = B0 w
k E0
(d) BE = m( A, Z ) - Zmp - ( A - Z )mn (c) E0 w = B0 k (d) =
w B0

MODULE 2
www.jeebooks.in
320 NEET Test Drive

20. When a metal plate of negligible thickness is The frequency becomes 400 Hz, then the original
introduced between the two plates of a capacitor length of the wire is
at middle position. Now, capacitance of capacitor (a) 20 cm (b) 25 cm
will be (c) 50 cm (d) 80 cm
(a) half (b) double
(c) same (d) infinite 27. A body weighing 10g has a velocity of 6 cms -1 after
one second of its starting from mean position. Its
21. For an p-type semiconductor, which of the time period is 6s, then the total energy is
following statements is false? (a) 720 erg (b) 180 erg
(a) Electrons are majority carriers and pentavalent atoms are (c) 540 erg (d) 520 erg
the dopants.
28. An open U-tube contains mercury, when 10 cm of
(b) Holes are majority carriers and trivalent atoms are the
water is poured into one of the arms of the tube.
dopants.
How high does the mercury rise in the other arm
(c) Electrons are minority carriers and doping are of trivalent
from its initial level?
impurities.
(a) 0.45 cm (b) 0.68 cm
(d) Impurities of third group (B, Al, Ga, In) are added
(c) 1.27 cm (d) 1.39 cm
22. In a L-C circuit, angular frequency at resonance is
w . What will be the new angular frequency, when 29. A body is released from 1600 km above the surface
inductance is made two times and capacitance is of earth. The velocity with which it strikes the
made four times? surface of earth is
w w (Take, radius of earth = 6400 km, g = 10 ms -2)
(a) (b)
2 2 2 (a) 18.2 ´ 103 kmh -1 (b) 3. 5 ´ 102 kmh -1
2w 2
(c) 4 ´ 10 kmh -1
(d) 2 .25 ´ 102 kmh -1
(c) 2w (d)
2
23. A solid sphere rolls down from the top of an 30. A car starts from rest and move with uniform
inclined plane, its velocity on reaching the bottom acceleration a on a straight road from time t = 0 to
of the plane is v. When the same sphere slides t = 5s. After that, a constant deceleration brings it
down from the top of the plane, its velocity on to rest. In this process, the average speed of the
v¢ car is
reaching the bottom is v¢. The ratio of is 5
v (a) a (b) 5a
2
3 3 7 2 25
(a) 1 (b) (c) (d) (c) a (d) a
5 5 5 5 2
24. The work function of a given material is 6.1 eV.
The wavelength of the incident radiation for which 31. When a wire is subjected to a force along its
the stopping potential is 5V lies in the length, the length increases by 0.5% and the radius
(a) ultraviolet region decreases by 0.15%. Then, the Poisson’s ratio of the
(b) visible region material of the wire is
(c) X-ray region (a) 0.8 (b) 0.5
(d) infrared region (c) 0.3 (d) 0.1

25. A 5 kg shell kept at rest suddenly splits up into 32. The current gain of a common base transistor
three parts. If two parts of mass 2 kg each are circuit is 0.94. On changing the emitter current by
found flying due north and east with a velocity of 9mA, the change in the collector current will be
5 m/s each, then what is the velocity of the third (a) 7.42 mA (b) 9 mA
part after explosion? (c) 8.46 mA (d) 0
(a) 10 ms -1 due north-east
33. Consider a compound slab consisting of two
10
(b) ms -1 due south-east different materials having length in the ratio 1 : 3
2 and thermal conductivity, K and 2K but equal
(c) 10 2 ms -1 due south-west thickness. The equivalent thermal conductivity of
(d) 10 2 ms -1 due south-east the slab is
3 8
26. A stretched wire emits a fundamental frequency of (a) K (b) K
256 Hz. Keeping the stretching force constant and 5 5
reducing the length of wire by 18 cm. 8 5
(c) K (d) K
5 3

MODULE 2
www.jeebooks.in
Prep Catalysis for NEET ~ Mock Test 11 321

34. The graph between applied potential difference V 39. In a vessel, the gas is at a pressure p. If the mass
and electric current I for a metallic wire having of all the molecules is halved and their speed is
length l and area of cross-section A is given in doubled, then the resultant pressure will be
figure. The slope of the curve becomes (a) 4p (b) 2 p
I p
(c) p (d)
2

40. A block having 12g of an element is placed in a


room. This element is a radioactive element with
half-life of 15 yr. After how many years will there
be just 1.5 g of the element in the box?
(a) 40 yr (b) 45 yr
(c) 20 yr (d) 15 yr
V
(a) more if the length of the wire is increased 41. The dimensional formula for acceleration,
(b) more if the area of cross-section is decreased velocity and length are ab -2 , ab -1 and ag. What is
(c) less if the length of the wire is increased the dimensional formula for the coefficient of
(d) more if temperature of the wire is increased friction?
(a) a0b 0 g -1 (b) abg
35. A small block is shot into each of the four track as
shown below. Each of the fricitons, track rises to (c) a-1b 0 g 0 (d) a0b -1g 0
the same height, then the speed with which the
block enters the tracks is same in all cases. At 42. At what distance from earth a body should be
highest point of the track normal reaction is placed, so that the gravitational pull on it due to
maximum in earth and sun is zero? The distance between sun
and earth is 1.5 ´ 1010 km and mass of sun is
3 ´ 105 times the mass of earth.
(a) 2.63 ´ 107 km (b) 2.73 ´ 107 km
(c) 2.50 ´ 107 km (d) 2 ´ 107 km

43. In an L-R circuit, time constant is that time in


which current grows from zero to the value
36. A man requires spectacles to see a object clearly at (where, I 0 is steady state current).
a distance of 50 cm but he can see clearly the (a) 0.63 I 0
object, when it is placed at a distance of 25 cm. The (b) 0.50 I 0
focal length of the lens of his spectacles is
50 50 (c) 0.37 I 0
(a) - cm (b) cm (c) -25 cm (d) -50 cm (d) I 0
3 3
37. A particle executes SHM with a time period of 2s 44. In a single slit diffraction pattern, distance
and amplitude 5 cm. Starting from the mean between screen and slit is 2.5 m and slit width is
1 0.25 mm. The separation of second dark band from
position, its displacement and velocity after s are central maxima is 1.5 cm, then the wavelength of
3
light is
(a) 4.33 cm, 7.85 cms -1 (b) 4.5 cm, 6.75 cms -1
(a) 7000 Å (b) 6000 Å
(c) 7.85 cm, 4.33 cms -1 (d) 6.75 cm, 4.5 cms -1
(c) 6500 Å (d) 7500 Å
38. When a dielectric material is placed between the
45. A coin of mass 4 kg and radius half meter is rolling
plate of capacitor, then its capacitance increases,
on a surface without sliding with angular speed of
because
300 rotation/min. The total kinetic energy of the
(a) potential difference between the plates decreases
coin is
(b) electric field between the plates decreases
(a) 100 p 2 J (b) 50 p 2 J
(c) charges on plates decreases 2
(d) Both (a) and (b) (c) 75 p J (d) 25 p 2 J

MODULE 2
www.jeebooks.in

PART B CHEMISTRY
46. Glucose is a monosaccharide, aldohexose and (a) P is acidic in nature
reducing sugar. In the combined state, it occur in (b) P is picric acid
glucosides, dissaccharides and polysaccharides. (c) P is obtained through nucleophilic substitution reaction
Glucose on reaction with Br2 /H 2O produces (d) P is obtained by carbocationic mechanism
(a) gluconic acid (b) laevulic acid
(c) glucaric acid (d) None of these 54. Rate constant of any reaction which 20% completes
in 10 min is
47. The pair whose both species are used in antacid (a) 0.223 (b) 0.0223 (c) 0.0322 (d) 0.322
medicinal preparation is
(a) Ca(OH)2 and NaHCO 3 (b) Ca(HCO 3 )2 and Mg(OH)2
55. Atomic number of Cr and Fe are respectively 24 and
26. Which of the following complex is
(c) NaHCO 3 and Mg(OH)2 (d) Na 2CO 3 and Ca(HCO 3 )2
paramagnetic?
48. Which of the following will give positive (a) [Cr(CO)6 ] (b) [Fe(CO)5 ]
carbylamine test? (c) [Fe(CN)6 ]4 - (d) [Cr (NH3 )6 ]3+
(a) N, N-diethylaniline (b) N-methylpropylamine
56. Oxidation numbers of Mn and Cr in KMnO 4 and
(c) N, N-diethylbutylamine (d) 2,4-dimethylaniline
K 2 Cr2 O 7 are
49. On the basis of different possible values of a, b and (a) + 6 and + 7 respectively (b) + 7 and + 6 respectively
g and different axial distances or edge lengths a , b (c) + 7 and + 7 respectively (d) + 6 and + 6 respectively
and c unit cells are divided into following seven types
of three dimensional primitive cells or crystal 57. A gaseous carbon compound is soluble in dil. HCl.
systems. Which of the following represent triclinic The solution on treating with NaNO 2 gives off
crystal ? nitrogen leaving behind a solution which smells of
(a) a = b = c, a = b = g = 90° (b) a ¹ b = c, a ¹ b = g = 90°
wood spirit. The carbon compound is
(a) HCHO (b) CO
(c) a ¹ b ¹ c, a ¹ b ¹ g = 90° (d) a ¹ b ¹ c, a = b = g ¹ 90°
(c) C 2H5NH2 (d) CH3NH2
50. Which of the following is the product of Hofmann
+
58. In which of the following compounds transition
elimination reaction of CH2N (CH3)3OH ? metal has zero oxidation state?
(a) [Fe(CO)5 ] (b) [Ni(CN)4 ]2 -
(a) CH2N (CH3)2 (b) NH2 + (CH3)2N
(c) Fe 2O 3 (d) CrO 5
r s 59. Molarity ( M ) of aqueous solution of acetic acid
(c) CH2 + (CH3)3N (d) + (CH3 )4 N OH
having density 1 .12 g/mL and molality 5.0 m is
51. There are substantial solvent effects in the Claisen (a) 4.31 (b) 4.13
reaction, more polar solvents tend to accelerate to (c) 8.62 (d) 5.0
a greater extent. Which of the following reaction is 60. Hydrolysis of o-tolunitrite using 75% H 2 SO 4 at
an example of Claisen rearrangement?
150-160°C gives
(a) Photolysis of alkyl allyl ether
(a) o-toluene (b) o-toluic acid
(b) Photolysis of alkyl phenyl ether
(c) benzene (d) benzoic acid
(c) Photolysis of aryl allyl ether
(d) Photolysis of aryl aryl ether 61. Lanthanoid compound which is used as a most
powerful liquid lasers after dissolving it in selenium
52. The correct order of the stoichiometries of AgCl oxychloride is
formed when AgNO3 in excess is treated with (a) ceric sulphate (b) promethium sulphate
1 mole each of the complexes CoCl3 × 6NH3 , (c) cerium oxide (d) neodymium oxide
CoCl3 × 5NH3 , CoCl3 × 4NH3 respectively is
(a) 2AgCl, 3AgCl, 1AgCl (b) 1AgCl, 3AgCl, 2AgCl 62. The usual covalency of an atom (except hydrogen
(c) 3AgCl, 1AgCl, 2AgCl (d) 3AgCl, 2AgCl, 1AgCl covalencyl) is equal to 8-number of the group to
which element belongs. This is applicable for
53. Choose the incorrect statement regarding P. non-metals of p -block only. N 2 ,P,As, Sb all are
Cl
NO2
p -block elements. Among the following, which is
NO2
aq. KOH most acidic?
P (a) NH3 (b) PH3
(c) AsH3 (d) SbH3
NO2

MODULE 2
www.jeebooks.in
Prep Catalysis for NEET ~ Mock Test 11 323

63. In the extraction of copper from its sulphide ore, 72. Equilibrium constant of a chemical reaction is 1.5
the metal is formed by the reduction of Cu 2 O with and rate constant of forward reaction is 7.5 ´ 10-4.
(a) SO 2 (b) Cu2S (c) SO 2 (d) FeS Then, rate constant of backward reaction will be
64. XeF6 is colourless crystalline solid. It undergoes (a) 1125
. ´ 10-3 (b) 2 .225 ´ 10-3
-5
hydrolysis in water. The final product obtained is (c) 3.335 ´ 10 (d) 1125
. ´ 10-1
an explosive solid. This explosive solid will be Cl
73.
(a) XeO 3 (b) XeO
(c) XeO 2 (d) Xe (i) HNO3/H2SO4 (i) HNO3/H2SO4
x y
(ii) Na2CO3/H2O
65. Which of the following statement is correct
regarding catalyst? In the above reaction, the product ‘y’ is
(a) phenol (b) nitrobenzene
(a) Catalyst decreases rate of reaction
(c) 2, 4-dinitrophenol (d) picric acid
(b) Catalyst is consumed during course of reaction
(c) Catalyst decreases activation energy of reaction 74. The pH of the solution is defined as the negative
(d) Catalyst increases activation energy of reaction logarithm of the concentration of hydrogen ions
which it contains. pH of 0.365 g L-1 HCl solution
66. Electron gain enthalpy increases across the period
and decreases down the group. Which of the will be
following order of electron gain enthalpy is correct? (a) 1 (b) 2
(a) F < Cl < Br < l (b) F < Cl > Br > l (c) 3 (d) 0.1
(c) l > Br > F < Cl (d) l < Cl < Br > F 75. Spontaneity of a process is decided by the value of
67. An olefin on ozonolysis yields a mixture of acetone DG. A negative value of DG shows a spontaneous
and methyl ethyl ketone. The possible structure of process. Since, the value of DG depends upon DS
the olefin is and DH , these two also play an important role for
CH3 CH 3 C 2H5 deciding spontaneity of a process. The condition for
½ ½ ½ spontaneity of any chemical reaction, if
(a) CH3 C == C ¾ CH3 (b) C 2H5 C == CH2 (a) TDS = DH and both DH and DS are positive
CH3 CH3 (b) TDS > DH and both DH and DS are positive
½ ½ (c) TDS < DH and both DH and DS are positive
(c) CH3C == C CH2CH3 (d) CH3CH == CH CH3 (d) TDS > DH and DH positive and DS is negative

68. The rate of diffusion of a gas is 76. The ionic radii of isoelectronic species N 3 - , O 2 - and
(a) directly proportional to its density F - in Å are in the order
(b) directly proportional to its molecular weight (a) 1.36,1.40,1.71 (b) 1.36,1.71,1.40
(c) directly proportional to squre root of its molecular weight (c) 1.71,1.40,1.36 (d) 1.71,1.36,1.40
(d) inversely proportional to square root of its molecular weight
77. Using the given data, calculate heat of combustion
69. When propionic acid is treated with aqueous of ethene
sodium bicarbonate, CO 2 is liberated. The C from Bond Bond Energy (kJ mol -1 )
CO 2 comes from
(a) methyl group (b) carboxylic acid group C == C 619
(c) methylene group (d) bicarbonate C ¾H 414
70. Borax, disodium tetraborate is an important boron O == O 499
compound, a mineral and a salt of boric acid. The C == O 724
number of terminal B ¾ OH present in borax is
O ¾H 660
(a) 3 (b) 4
(c) 5 (d) 6 H H
C==C + 3O ==O 2O == C == O + 2H—O—H
H H
71. A compound A is obtained by reaction between first
(a) + 964 kJ mol -1 (b) - 964 kJ mol -1
member of 1st group element and lightest element -1
of periodic table, then correct statement about (c) - 3772 kJ mol (d) + 3772 kJ mol -1
compound A is
78. A metal is said to be diamagnetic, if it does not have
(a) A is acidic in nature
any unpaired electron. Among the following,which one
(b) A is basic in nature
is diamagnetic?
(c) A is amphoteric in nature
(a) Co 2- (b) Cu2+
(d) A turns blue litmus paper red
(c) Mn2+ (d) Sc 3+

MODULE 2
www.jeebooks.in
324 NEET Test Drive

79. Which is a possible set of quantum numbers for a 85. What will be the product obtained when A reacts
valence electron in ground state atom of with heavy water?
phosphorus ( Z = 15)? Br
Mg/ether D2O
A ?
n l me ms
(a) 2 1 0 +1 / 2 Choose the correct option regarding product.
(b) 3 0 0 0 D

(c) 3 1 -1 + 1/ 2 (a) (b)


OH OD
(d) 3 2 0 -1 / 2
(c) (d)
80. Match Column I with Column II and select the
correct answer using the codes given below : 86. Consider the compounds (I) and (II):
Column I Column II O OH OH OH
A. Solid I 2 1. Insulator at 0 K
Ph CH3 and Ph CH3
B. Ag 2. Good conductor
H H
C. LiF 3. Insulator up to melting point (I) (II)
D. Ge 4. Insulator Which of the following statement is true for the
Codes above two compounds?
A B C D A B C D (a) Above pairs are resonance contributors
(a) 4 2 3 1 (b) 3 2 4 1 (b) They are chain isomers
(c) 1 4 2 3 (d) 2 3 1 4 (c) They are group isomers
(d) They are tautomers
81. At what temperature, the sample of neon gas
would be heated two double of its pressure, if 87. Among the following statements about control of
the initial volume of gas is/are reduced to 15% at particulate pollution which is incorrect?
75°C (a) Cyclone collector removes fine particles in the diameter
(a) 319°C (b) 592°C range 5-20 microns
(c) 128°C (d) 60°C (b)Wet scrubbers are used to wash away all types of
particulates
82. In the following structure, which carbon atom is (c) In electrostatic precipitator, the particulates are made to
most electronegative? acquire positive charge which are then attracted by the
Å
negative electrode and removed
CH 3 ¾ CH 2 ¾ CH == CH
I II III IV (d)Gravity settling chamber removes larger particles from the
(a) I (b) II air
(c) III (d) IV 88. The properties of a dilute solution which depends
only upon the number of particles present in a
83. Consider the following dehydration reaction : solution are called colligative properties. Which of
P2 O 5
CH 3 CO NH 2 ¾ ¾¾® CH 3 C ºº N the following is not a colligative property?
The hybrdisation state of carbon changes from (a) Osmotic pressure
(a) sp3 to sp2 (b) sp to sp (b) Relative lowering of vapour pressure
(c) sp2 to sp (d) sp to sp3 (c) Elevation in boiling point
(d) Vapour pressure
H
89. If the temperature coefficient for the saponification
COOH H COOH
of ethylacetate by NaOH is 3.25, then its activation
energy will be
84. Consider the molecule H H , which of (a) 10.22 kcal mol -1 (b) 21.50 kcal mol -1
following statement is true for the above (c) 42 kcal mol -1 (d) 38.24 kcal mol -1
compound?
(a) cis form is more stable due to H-bonding 90. 200 mL of an aqueous solution contains 0.01 mole of
(b) cis form is less stable due to steric hinderance the solute AB. If its specific conductance is x S cm -1,
(c) trans form is more stable due to crowding its molar conductance in S m 2 mol-1 will be
(d) trans form is less stable in the absence of H-bonding (a) x (b) 2 x (c) 200 x (d) 40x

MODULE 2
www.jeebooks.in

PART C BIOLOGY
91. In which ratio, ova and sperms will be produced, if 99. Match Column I with Column II and choose the
a germ cell in female and male gonads undergoes correct option from the codes given below.
meiosis simultaneously? Column I Column II
(a) 4 : 1 (b) 1 : 4
A. Downward 1. Translocation in direction of root
(c) 2 : 1 (d) 1 : 2
translocation to leaves
92. In an archaeological excavation which of the B. Upward 2. Source and sink lie on the
following is/are most likely to be obtained in a translocation opposite sides
well-preserved form? C. Lateral 3. Carbohydrates move in both
(a) Petals of the flower (b) Leaf of the flower translocation upward and downward
(c) Pollen grains (d) Pistil directions
93. Which of the following is/are factor(s) which affects D. Bidirectional 4. Most common mode of
the electrophoretic mobility? translocation translocation
(a) Molecular weight of DNA molecule Codes
(b) Size of DNA molecule
A B C D A B C D
(c) Charge
(a) 1 2 3 4 (b) 4 1 2 3
(d) All of the above (c) 4 1 3 2 (d) 1 2 4 3
94. Which statement is/are true regarding 100. In gymnosperms,
prey-predator relationship? (a) xylem lacks vessels
(a) Prey try to escape from predators (b) phloem lacks companion cell
(b) Predators choose a prey that can be easily trapped (c) xylem vessels are present
(c) A selection pressure develops on prey (d) Both (a) and (b)
(d) All of the above
101. Which among the following are the reasons of
95. For the formation of a nucleotide, it is essential infertility in female?
that
I. Irregular menstrual cycle.
(a) a phosphate group is linked to 5' OH of a nucleoside
through a phosphoester linkage II. Polycystic ovarian disease.
(b) a phosphate group is linked to 3' OH of a nucleoside III. Blockage of Fallopian tube.
through a phosphoester linkage IV. Hypothalamic and pituitary dysfunction.
(c) a nitrogenous base is linked to a pentose sugar Choose the correct option.
(d) a nitrogeous base is linked to phosphate group (a) I and II (b) II, III and IV
(c) I and IV (d) All of these
96. In Ascaris, the excretory system comprises of
(a) flame cells 102. Choose an incorrect statement.
(b) general body surface (a) Simple squamous epithelium is also called pavement
(c) renette cells epithelium
(d) Malpighian tubules (b) The cells of cuboidal epithelium form microvilli on their free
surface
97. The effluent will have more polluting potential if (c) Non-keratinised stratified squamous epithelium contains
(a) COD and BOD values are low goblet cells
(b) COD is high but BOD is low (d) Simple cuboidal epithelium is also called germinative layer
(c) BOD is high but COD is low
(d) Both BOD and COD are high 103. Consider the following statements.
I. Bacteria do not have histones.
98. Select the correct option for where in nature do II. Bacteria have 70S ribosomes.
the restriction enzymes occur and what is their
III. Bacteria have introns.
role.
(a) In bacteria – defence against viral invasion
IV. Bacterial cell wall comprises of peptidoglycan.
(b) In yeast – defence against foreign DNA invasion The incorrect statement(s) is/are
(c) In virus – splicing host cell’s DNA (a) Only I (b) Only II
(d) In bacteriophage – defence against mutation (c) Only III (d) Only IV

MODULE 2
www.jeebooks.in
326 NEET Test Drive

104. In the epigeal germination of seeds, the cotyledons 112. In VAM fungi mostly belong to
are pushed above the soil due to the (a) Zygomycetes (b) Phycomycetes
(a) elongation of hypocotyl (c) Deuteromycetes (d) Ascomycetes
(b) elongation of epicotyl
(c) lot of water imbibed for germination
113. Match the items given in Column I with those in
Column II and select the correct option given below.
(d) faster respiration due to oxygen excess
Column I Column II
105. Which among the following statement(s) is/are
correct regarding symbiotic relationship of lichens? A. Endometrium 1. Implantation occurs

I. The algal partner performs photosynthesis. B. Placenta 2. Fertilisation occurs to form a zygote
II. The algal partner provides vitamins. C. Fallopian tube 3. Vascular connection between
III. The fungal partner is involved is outer covering foetus and mother
for protection. Codes
IV. The algal partner provides attachment to A B C A B C
substratum. (a) 1 2 3 (b) 1 3 2
Choose the option with correct statements. (c) 3 1 2 (d) 3 2 1
(a) I and II (b) II, III and IV 114. If a condition arises where a plant is devoid of
(c) I, II and III (d) I, II and IV carotenoid pigment, under normal sunlight
following can be observed
106. Match the kinds of fruits given in Column I with
(a) Increase in chlorophyll synthesis
their examples in Column II.
(b) Increase in the rate of photosynthesis
Column I Column II (c) Increase in the rate of cellular respiration
(d) Increase in chlorophyll oxidation and necrosis
A. Lomentum 1. Castor
B. Cremocarp 2. Chilbil
115. Which among the following statements are correct
regarding somaclonal variation?
C. Samara 3. Coriander I. Somaclonal variations do not affect
D. Regma 4. Radish cytoplasmic genome.
Codes II. Somaclonal variations affect nuclear genome.
A B C D A B C D III. It is involved in developing rust resistance in
(a) 4 3 2 1 (b) 1 2 3 4 wheat.
(c) 4 3 1 2 (d) 3 4 1 2 IV. The variations are useful if they are heritable.
107. Which of the following is a pleiotropic gene? Choose the correct option.
(a) Hb A (b) HbS (a) I and II (b) II, III and IV
(c) IA (d) IB (c) I, II and III (d) All of these

108. Psychotria has 116. The anatomical peculiarity of C 4 -plants which is


(a) moniliform root (b) annulated root not valid is
(c) pillar root (d) fasciculated roots (a) the leaf mesophyll consists of compactly arranged cells
(b) it is differentiated into palisade and spongy mesophyll
109. Read the following statements w.r.t. apomixis and (c) the vascular bundles in the leaves are surrounded by a
identify the incorrect statement. distinct bundle sheath
(a) It refers to asexual mode of reproduction (d) the chloroplasts in leaf cells are dimorphic
(b) Apomixis is helpful in maintaining genetic purity of mother
plant 117. Which of the following statements are incorrect?
(c) It results in genetic variability I. Nitrogen is required by whole plant for cell
(d) There is no segregation in apomixis division, growth and photosynthesis.
II. Zinc is involved in RNA, IAA and protein
110. An etaerio of drupes is found in
synthesis.
(a) raspberry (b) strawberry
(c) lotus (d) custard apple III. Chlorine is responsible for photolysis of water.
IV. Iron is required by 40 enzymes as cofactor.
111. In horse, the shape of the gall bladder is Choose the correct option.
(a) sac-shaped (b) U-shaped (a) I, II, III and IV are incorrect (b) II and IV are incorrect
(c) C-shaped (d) gall bladder is absent (c) Only IV is incorrect (d) None is incorrect

MODULE 2
www.jeebooks.in
Prep Catalysis for NEET ~ Mock Test 11 327

118. Which one of the following depicts the correct set of 128. In parthenogenesis,
hormones secreted from the posterior pituitary? I. Formation of embryo occurs from an
(a) Melatonin, thyroxine and calcitonin unfertilised egg.
(b) Oxytocin and vasopressin II. Useful characteristics of plants can be
(c) Somatostatin, insulin and glucagon preserved for many generations.
(d) Gonadotropin, somatotropin and prolactin
III. Parthenogenetic plants are heterozygous.
119. The least number of genes is present in IV. Induced parthenogenesis can be achieved
(a) chromosome-1 (b) chromosome-2 through chemical agents.
(c) chromosome-X (d) chromosome-Y
The incorrect option is
120. Accumulation of which of the following hormones (a) I and II (b) Only III
may cause seed dormancy? (c) III and IV (d) II and III
(a) Auxin (b) Gibberellic acid
(c) ABA (d) Zeatin 129. What were the reasons for selecting garden pea
plant as a sample by Mendel for his experiments?
121. Vivipary is an example of I. Pea plants are self-pollinated and can be
(a) interspecific competition cross-pollinated also.
(b) intraspecific competition
II. Pea plants are annual plants with short life
(c) parasitism
cycle.
(d) commensalism
III. Pea plants do not differ in distinct/contrasting
122. L-sorbose is produced by characteristics.
(a) Blakeslea trispora (b) Ashbya gossypii IV. Pea plants are readily available on a large
(c) Gluconobacter oxidans (d) Bacillus coagulans scale.
123. Which among the following statement(s) is/are Choose the correct option.
false for pineal gland? (a) I, II and IV (b) I, II and III
I. The pineal gland secretes melatonin, serotonin (c) II, III and IV (d) I and IV
and adrenoglomerulotropin.
130. Biological magnification is shown by
II. Melatonin is a derivative of amino acid (a) insecticides (b) pesticides
tryptophan. (c) biopesticides (d) Both (a) and (b)
III. Serotonin acts as a vasoconstrictor.
131. The microbe not used as biocontrol agent is
IV. Serotonin plays an important role in regulation
(a) Trichoderma species
of rhythm of our body.
(b) Bacillus thuringiensis
Choose the correct option. (c) baculoviruses of genus-Nucleopolyhedrovirus
(a) I and II (b) II and III (c) I and IV (d) Only IV (d) Azotobacter
124. The term non-disjunction refers to 132. The antibody responsible for Rh-factor in blood is
(a) failure of separation of homologous chromosomes during (a) IgG (b) IgA
anaphase-I of meiosis (c) IgM (d) IgE
(b) failure of arrangement of homologous chromosome at
equitorial plane of cell during metaphase 133. Select the option that shows the correct sequence
(c) failure of cytokinesis by cell furrow or cell plate formation of embryo development.
(d) failure of DNA strand separation during S-phase (a) Two-celled stage → Heart-shaped → Globular type →
Mature embryo
125. The longer is the loop of Henle in the nephron, the (b) Globular type → Two-celled stage → Heart-shaped →
greater is the ability of an animal to form Mature embryo
(a) hypertonic urine (b) hypotonic urine (c) Two-celled stage → Globular type → Heart - shaped →
(c) isotonic urine (d) None of these Mature embryo
126. The trophoectoderm or trophoblast forms (d) Heart - shaped → Two-celled stage → Gobular type →
Mature embryo
(a) amnion (b) allontois (c) chorion (d) yolk sac
127. Which among the following is an example of 134. The phenomenon in which a single gene product
external glands? may produce multiple or more than one phenotypic
effect is
(a) Nectar secreting glands (b) Oil secreting glands
(a) multiple alleles (b) pleiotropy
(c) Resin secreting glands (d) Mucilaginous glands
(c) polygenic inheritance (d) epistatic gene

MODULE 2
www.jeebooks.in
328 NEET Test Drive

135. Which among the following statement(s) is/are 142. Identify the incorrectly matched pair.
incorrect regarding pericycle? (a) Exonucleases —
Remove nucleotides from the ends of
(a) Outermost layer of stele the DNA
(b) It forms part of cambium ring in dicot roots (b) Endonucleases — Make cuts at specific position
(c) It gives rise to lateral roots in angiosperms within DNA
(d) None of the above (c) Endonucleases — Act on single strand of DNA
(d) Exonucleases — They do not cut RNA
136. Choose the incorrect statement.
(a) Bryophytes were the first plant to colonise land 143. Nearly 20-25% of CO2 is transported by RBCs,
(b) Vascular plants first originated in Devonian period whereas about 7% is carried in a dissolved state
(c) Herbaceous lycopods evolved from Zosterophyllum of through plasma. The remaining 70%
Palaeozoic era (a) is carried as bicarbonates in blood
(d) Psilophyton is the common ancestor for horse tails, ferns (b) diffuses in the lymph capillaries
and gymnosperms (c) remains in the tissues
(d) None of the above
137. The dietary fat is partially digested by the
pancreatic lipase in the intestine converting it into 144. An inverted omega-shaped arrangement of
glycerol and fatty acids. The remaining product of vascular bandles in rachis is observed in
fat digestion is called (a) Cycas (b) Pteridium
(a) chyle (b) chyme (c) Dryopteris (d) Pinus
(c) micelles (d) bolus
145. The total lung capacity includes
138. The simplest sporogonium is observed in (a) RV, ERV, TV and IRV (b) RV, ERV and TV
(a) Marchantia (b) Riccia (c) VC, ERV and TV (d) RV, ERV, TV and FRC
(c) Pellia (d) Porella
146. Many protists can resist harsh environmental
139. Which of the following is not present in the conditions surrounding them. This feature is
digestive tract? facilitated by
(a) Sphincter of oddi (b) Eustachian valve (a) formation of a cyst and slow metabolism
(c) Cardiac sphincter (d) Valves of Kerckring (b) possessing a single mode of nutrition
(c) spending a part of life cycle in non-human host
140. Which among the following is not valid for factors
(d) oxidising inorganic chemical sources
affecting respiration?
(a) CO 2 has a limited direct inhibitory effect on the respiration 147. Where are the enzymes required for steps of
rate glycolysis and Krebs’ cycle are respectively present
(b) Respiration decreases with the temperature between 0°C in?
and 30°C (a) Cytoplasm and mitochondria
(c) Dehydration slows down the rate of respiration (b) Mitochondria
(d) The respiration rate increases due to injury
(c) Both peroxisome and mitochondria
141. Match Column I with Column II and choose the (d) Endoplasmic reticulum and cytoplasm
correct option from the codes given below.
148. Which of the following forms the most stable
Column I Column II product with haemoglobin?
(Population (Examples) (a) O 2 (b) CO 2 (c) CO (d) SO 2
interactions)
149. In which among the following the secondary
A. Mutualism 1. Orchids pollutants are absent?
B. Commensalism 2. Sea anemone and hermit crab (a) Classical smog (b) Photochemical smog
C. Protocooperation 3. Human liver fluke (c) Global warming (d) Both (a) and (b)

D. Parasitism 4. Lichens 150. Identify the incorrectly matched pair.


Codes (a) Exoskeleton — Produced by mesoderm
A B C D A B C D (b) Endoskeleton — Consists of living tissues
(a) 4 1 2 3 (b) 1 2 3 4 (c) Endoskeleton — Occurs inside the body
(c) 2 1 3 4 (d) 4 1 3 2 (d) Endoskeleton — Does not restrict the growth of the body

MODULE 2
www.jeebooks.in
Prep Catalysis for NEET ~ Mock Test 11 329

151. In G0-phase, the cell does not undergo 158. Khecheopalri in Sikkim is a
(a) also known as quiescent stage (a) sacred lake (b) orchard
(b) the cell division (c) sanctuary (d) botanical garden
(c) cells functions as reserve cells
159. Monoclonal antibodies have not been used for
(d) muscle cells remain in G 0 -phase permanently
(a) identifying and measuring levels of gene product
152. Which among the following statement(s) is/are (b) pregnancy testing diagnosis of disease
correct regarding genetic diversity? (c) preventing rejection of transplants
(a) Diversity in the number and types of genes (d) production of vitamins
(b) Diversity used by humans for raising domesticated plants
and animals 160. The factors that affect decomposition include
(c) Both (a) and (b) I. chemical nature of detritus
(d) None of the above II. soil pH
III. moisture
153. Foramen of Magendie is the median aperture of
medulla oblongata which in turn is the component IV. aeration
of Choose the correct option.
(a) diencephalon (b) mesencephalon (a) I, II, III and IV (b) I and II
(c) rhombencephalon (d) prosencephalon (c) Only III (d) I and IV
154. In bacteria, initiation of transcription takes place 161. Which among the following statement(s) is/are
through binding of correct regarding quiescent centre concept?
(a) promoter (b) enhancer (a) It is a region of passive cells
(c) sigma factor (d) operator (b) Have low DNA and RNA contents
155. Which one of the following pairs is incorrectly (c) Have very few mitochondria
matched with respect to humans? (d) All of the above
(a) Family–Hominidae (b) Class–Primata 162. The technique which is beneficial in case of male
(c) Phylum–Chordata (d) Genus–Homo infertility is
156. Which of the following is a subclass of Amphibia (a) GIFT (b) ICSI
which is extinct now? (c) Al technique (d) ZIFT
(a) Lissamphibia (b) Gymnophiona
163. Which among the following is not valid for vestigial
(c) Salientia (d) Stegocephalia
organs?
157. Given below is the pyramid showing the total (a) They are non-functional organs
amount of energy content within each trophic level (b) They were functional in their ancestors
of a food chain. (c) Splint bones in horse is an example of vestigial organs
Snakehead 10 J (d) They are functional in related animals

164. Which among the following statements is incorrect


Clown loach 100 J regarding the structure of tRNA
(a) anticodon loop has bases complementary to the code
(b) T-loop helps in binding to ribosome
Ramshom snail 1000 J (c) D-loop helps in binding aminoacyl synthetase
(d) variable loop is specific for specific amino acid

Algae 10000 J 165. A river near an agricultural farm is being polluted


by chemical fertilisers run-off.
1000000 J of sunlight
Concentration

Concentration

Identify possible reasons explaining why the


amount of energy transferred between levels is at (a) (b)
most 10%.
I. Some energy is lost as heat during respiration.
II. Energy stored is consumed to drive Time Time
photosynthesis in algae. Run-off pollution Run-off pollution
III. Not all algae are consumed by the ramshorn Oxygen Oxygen
snails. Bacteria Bacteria
(a) Only I (b) Only III (c) I and II (d) I and III

MODULE 2
www.jeebooks.in
330 NEET Test Drive

172. The method in which recombinant DNA is directly

Concentration
Concentration

injected into the nucleus of an animal cell is


(a) biolistics (b) heat shock method
(c) (d)
(c) microinjection (d) disarmed pathogen vectors

173. HIV infection has a longer incubation period


Time Time before the appearance of full extent of symptoms.
Run-off pollution Run-off pollution This renders HIV infection as more dangerous
because
Oxygen Oxygen
(a) the infection can unknowingly transmit to others
Bacteria Bacteria
(b) longer incubation periods result in more serious
Identify the graph which correctly shows the consequences
resulting changes in levels of oxygen and bacteria (c) Both (a) and (b)
in this particular river. (d) virus can mutate to more lethal form
166. Allograft is grafting of tissue or organ between 174. Microglia are
(a) individuals of same species but with different genetic (a) cells which are specialised macrophages and the
background scavengers of the nervous system
(b) animals of different species (b) epithelial cells that line the cerebrospinal fluid-filled
(c) one's own tissue ventricles in the brain
(d) donor and recipient are genetically identical (c) produce secretions called neurohormones
(d) play a metabolic role in the formation of myelin sheath
167. An example of feedback inhibition occurring during
a metabolic pathway is 175. The presence of barrel-shaped air pores on thallus
(a) binding of non-competitive inhibitor irreversibly to substrate is observed in
(b) competition between two enzymes for a common (a) Marchantia (b) Riccia
substrate (c) Pellia (d) Porella
(c) accumulation of end product, showing the rate of an
enzyme controlled reaction 176. ........... are used for the preparation of Havan
(d) All of the above Samagri.
(a) Ramalina (b) Evernia
168. Which one of the following statements is not true (c) Both (a) and (b) (d) Spirullina
about exophthalmic goitre?
(a) It is caused by the hyposecretion of thyroxine 177. Which of the following is true for eukaryotic DNA
(b) Basal Metabolic Rate (BMR) increases replication?
(c) Eyes bulge forward (a) It occurs inside the nucleoid along with cytoplasm
(d) There is an increase in the heart rate and pulse (b) Multiple origin of replications (ori) are present
(c) DNA gap filling is done by DNA polymerase-I.
169. Identify the incorrectly matched pair.
(d) RNA primer is removed by DNA polymerase-I
(a) Commensalism with permanent contact — Epiphytic
plants 178. Two curly-winged (Cy) flies are crossed and they
(b) Commensalism without continuous contact — Epizoans produced 150 eggs. What will be the proportion of
(c) Both (a) and (b) straight-winged flies expected among the living
(d) None of the above offsprings?
(a) 25% (b) 33% (c) 66% (d) 75%
170. Identify the group of plants which have
underground stems. 179. In Periplaneta, the cleavage is
(a) Potato, ginger, turmeric, Euphorbia (a) holoblastic, spiral and determinate
(b) Watermelon, zamikand, Colocasia, water hyacinth (b) holoblastic, spiral and indeterminate
(c) Potato, ginger, turmeric, Colocasia (c) meroblastic, spiral and determinate
(d) Cucumber, radish, Allium cepa, Euphorbia (d) meroblastic, spiral and indeterminate

171. Which among the following statements is incorrect 180. In humoral immune response
regarding transcription? (a) B-lymphocytes produce an army of proteins called
(a) Formation of single-stranded RNAs takes place antibodies
(b) A primer is required (b) T-lymphocytes help B-lymphocytes to produce antibodies
(c) Synthesis of RNA from DNA takes place (c) Killer T-cells kill the specific target cells
(d) Both (a) and (b)
(d) Transcribed strand separates from its template

MODULE 2
www.jeebooks.in

Answer Sheet
1. (b) 2. (a) 3. (b) 4. (b) 5. (a) 6. (b) 7. (b) 8. (d) 9. (c) 10. (d)
11. (b) 12. (d) 13. (a) 14. (b) 15. (a) 16. (c) 17. (a) 18. (a) 19. (b) 20. (c)
21. (a) 22. (a) 23. (d) 24. (c) 25. (c) 26. (c) 27. (a) 28. (b) 29. (a) 30. (a)
31. (c) 32. (c) 33. (b) 34. (c) 35. (a) 36. (d) 37. (a) 38. (d) 39. (b) 40. (b)
41. (a) 42. (b) 43. (a) 44. (d) 45. (c) 46. (a) 47. (c) 48. (d) 49. (c) 50. (c)
51. (c) 52. (d) 53. (d) 54. (b) 55. (d) 56. (b) 57. (d) 58. (a) 59. (a) 60. (b)
61. (d) 62. (d) 63. (b) 64. (a) 65. (c) 66. (b) 67. (c) 68. (d) 69. (b) 70. (b)
71. (b) 72. (a) 73. (d) 74. (b) 75. (b) 76. (c) 77. (b) 78. (d) 79. (c) 80. (a)
81. (a) 82. (d) 83. (c) 84. (b) 85. (b) 86. (d) 87. (c) 88. (d) 89. (b) 90. (b)
91. (b) 92. (c) 93. (d) 94. (d) 95. (a) 96. (c) 97. (d) 98. (a) 99. (b) 100. (d)

101. (d) 102. (c) 103. (c) 104. (a) 105. (c) 106. (a) 107. (b) 108. (b) 109. (c) 110. (a)
111. (d) 112. (a) 113. (b) 114. (d) 115. (b) 116. (b) 117. (c) 118. (b) 119. (d) 120. (c)
121. (b) 122. (c) 123. (d) 124. (a) 125. (a) 126. (c) 127. (a) 128. (b) 129. (a) 130. (d)
131. (d) 132. (a) 133. (c) 134. (b) 135. (d) 136. (b) 137. (c) 138. (b) 139. (b) 140. (b)
141. (a) 142. (c) 143. (a) 144. (a) 145. (a) 146. (a) 147. (a) 148. (c) 149. (a) 150. (a)
151. (b) 152. (c) 153. (c) 154. (c) 155. (b) 156. (d) 157. (d) 158. (a) 159. (d) 160. (a)
161. (d) 162. (b) 163. (d) 164. (d) 165. (a) 166. (a) 167. (c) 168. (a) 169. (b) 170. (c)
171. (b) 172. (c) 173. (a) 174. (a) 175. (a) 176. (c) 177. (b) 178. (a) 179. (a) 180. (d)

For explanations, open the link


https://goo.gl/9cW8L1

MODULE 2
www.jeebooks.in

PREP CATALYSIS
for NEET
Full Length Mock Tests for NEET to Make You Ready to Face the Challenge

MOCK TEST 12 (Unsolved)

Duration : 3 Hours Max. Mark : 720

Instructions
The test is of 3 hours duration and Test Booklet contains 180 questions. Each question carries 4 marks. For
each correct response, the candidate will get 4 marks. For each incorrect response, one mark will be deducted
from the total scores. The maximum marks are 720.
The question paper contains three parts of Physics, Chemistry and Biology respectively.
Part A contains 45 Questions from Physics section. Part B contains 45 Questions from Chemistry section. Part
C contains 90 Questions from Biology.

PART A PHYSICS
1. An aluminium block of fixed volume V is drawn The angular acceleration produced is
into wire of length l. When this wire is hanged by a (take, g = 10ms−2)
mass of M, the extension produced in the wire is (a) 1.5 rad s −2 (b) 3 rad s −2
∆l, which of the following graph is a parabolic line? (c) 2 rad s −2 (d) 2.5 rad s −2
1
(a) ∆l versus (b) ∆l versus l 2
l 5. Two springs are joined and connected to a mass m
1 as shown in the figure. If the force constants of the
(c) ∆l versus (d) ∆l versus l
l2 two springs are k1 and k2, then the frequency of
oscillation of mass m is
2. A reverse bias in a junction diode is changed from
4V to 12V, then the value of the current changes k2 k1
from 20 µA to 60µA. The resistance of junction m
diode will be
(a) 2 × 105 Ω (b) 2.5 × 105 Ω (c) 3 × 105 Ω (d)4 × 105 Ω
1 k1k2 1 (k1 + k2 )
3. If the earth shrinks by 25% of its present radius (a) ν = (b) ν =
and mass remaining the same, the value of 2 π (k1 + k2 )m 2 π (k1k2 )m
acceleration due to gravity increases by 1 (k1k2 )m 1 (k1 + k2 )m
(c) ν = (d) ν =
(a) 50% (b) 75% (c) 25% (d) 60% 2 π k1 + k2 2π k1k2
4. A rope is wound around the circumference of a 6. The electrostatic potential V at a distance r from
bicycle wheel of diameter 1.5 m. A mass of 5 kg is
the centre of a charged metallic sphere of radius R
tied to end of rope and then it is allowed to fall
for r < R will vary according to relation
from rest. The weight falls 3 m in 2s. The wheel 1 1
has horizontal axle and rotates in vertical plane. (a) V ∝ (b) V ∝ r (c) V ∝ (d) V ∞ r 0
r r2

MODULE 2
www.jeebooks.in
Prep Catalysis for NEET ~ Mock Test 12 333

7. In the following figure, the capacitance is 13. If a solid cylinder of mass 3 kg is rolling on a
d/2 horizontal surface with velocity 4 m/s. It collides
with a horizontal spring of force constant 200 N/m.
The maximum compression produced in the spring
will be
K (a) 0.6 m (b) 0.5 m (c) 0.7 m (d) 0.2 m
14. In a capillary tube, water rises to 4 mm. The
height of water that will rise in another capillary
tube having one-fourth radius of the first is
d
(a) 16 mm (b) 14 mm (c) 12 mm (d) 9 mm
2 KAε0 2 KAε0
(a) (b) 15. Half mole of helium is stored in a container at
(K + 1)d d
STP. The amount of heat energy needed to double
(K + 1)Aε0 2 KAε0
(c) (d) the pressure of the gas, keeping the volume
2d (K 2 + 1)d constant is (heat capacity of gas is 3 Jg −1K −1)
8. If the earth is supposed to be a sphere of uniform (a) 1680 J (b) 1638 J (c) 1600 J (d) 1538 J
2
mass density, then the weight of a body rd way 16. How many NOR gates are required to form an
3 AND gate?
down the surface of earth, if its weight on surface (a) 1 (b) 2 (c) 3 (d) 4
of earth is 330 N is
(a) 100 N (b) 220 N 17. A flint glass prism (refractive index 3) is made
(c) 495 N (d) 110 N such that a beam of light incident on it retrace its
path after reflection from other refracting surface
9. If the density of oxygen is 8 times, the density of due to silvering. If angle of prism is 30°, then angle
hydrogen, then at what temperature the velocity of of incidence is
sound in hydrogen is same as in oxygen at 87° C ?
(a) 230°C (b) − 230° C
30°
(c) − 228° C (d) 228° C

10. A radioactive nucleus of mass number A, initially


at rest, emits an α-particle with a speed v. The µ = √3
recoil speed of the daughter nucleus will be
2v 2v
(a) (b) (a) 30° (b) 60° (c) 90° (d) 0°
A−4 A+ 4
4v 4v 18. A smooth hemispherical bowl 30 cm diameter
(c) (d)
A−4 A+ 4 rotates with a constant angular velocity ω, about
its vertical axis of symmetry. A particle at P of
11. In the following circuit diagram, current through weighing 5 kg is observed to remain at rest relative
5Ω resistor is to the bowl at a height 10 cm above the base as
shown in figure. The magnitude of speed of
rotation of the bowl is
15 cm
30 V 15 Ω 3Ω 9V
O

P
5Ω 10 cm

(a) 4A (b) 2.5A (a) 15 rad /s (b) 13 rad /s


(c) zero (d) 3A (c) 10 rad /s (d) 16 rad /s
12. A transformer is used to light a 100 W, 110 V 19. The magnetic moment of an electron orbiting in a
lamp. If the mains current and voltage are 0.5 A circular orbit of radius r with a speed v is equal to
and 220 V respectively, then the efficiency of the evr
(a) (b) evr
transformer is approximately 2
(a) 30% (b) 50% er
(c) (d) None of these
(c) 90% (d) 10% 2v

MODULE 2
www.jeebooks.in
334 NEET Test Drive

20. The dependence of binding energy per nuclear 25. The frequency of a light wave in a material is
(∆E bn ) on the mass number A is represented by 2 × 1014 Hz and wavelength is 5000Å. The
refractive index of material will be
(a) 1.40 (b) 1.50 (c) 3 (d) 1.33
∆ Ebn
∆ Ebn 26. A wire of resistance 10 Ω is bent to form a complete
circle. Find its resistance between two
A=56 diametrically opposite points.
A=124
A

∆ Ebn ∆ Ebn
D B

A=96 A=96
C
21. A tray of mass 12 kg is supported by two identical
springs as shown in the figure. When the tray is 10
(a) 5Ω (b) 2.5 Ω (c) 1.25 Ω (d) Ω
pressed down slightly and released, it executes 3
SHM with a time period of 2.4 s. The force
27. If binding energy of electron in a hydrogen atom is
constant of each spring is
13.6 eV, then energy required to remove the
m electron from the first excited state of Li + + is
(a) 13.6 eV (b) 3.4 eV (c) 30.6 eV (d) 122.4 eV
28. 72 × 10 J of heat is conducted through 2 m 2 wall of
5

12cm thick in one hour temperature difference


between the two sides of the wall is 30° C the
(a) 244 Nm−1 (b) 246.49 Nm−1 thermal conductivity of the material of the wall is
(c) 298.54 Nm−1 (d) 123.25 Nm−1 (in Wm −1K −1)
(a) 1.5 (b) 2.4 (c) 4 (d) 3
22. Parallel plate capacitor is constructed using three
different dielectric materials as shown in the figure. 29. For an adiabatic expansion of ideal gas, the value
The parallel plates, across which potential difference ∆p
of is equal to
is applied of area 200 cm 2 and separated by a distance p
10 cm, then the capacitance across P and Q is ∆V ∆V ∆V
(a) − (b) γ 2 (c) zero (d) − γ
P + V V V
30. Two parallel wires of length 10 m each are
5 cm K2=4 separated by a distance of 0.1 m. If they carry
K1=2 equal currents in the same direction and exerts a
5 cm K3=6 total force of 2 × 10−7 N on each other, then the
value of current must be
(a) 0.5 A (b) 0.4 A (c) 0.3 A (d) 0.1 A
Q –
(a) 6 pF (b) 6µF (c) 12pF (d) 12µF 31. A carpet of mass m, made of inextensible material
−2
is rolled along its length in the form of a cylinder of
23. Following data are given : g = 9.8ms , radius of earth, radius r and kept on a rough floor. If it unrolled to
RE = 6.4 × 106 m, distance of moon, R = 4 × 108 m r
a radius without sliding, the decrease in
and time period of revolution of the moon is 27.3 2
days. Then, the mass of earth using time period potential energy is
method is 3 7 1 1
(a) mgr (b) mgr (c) mgr (d) mgr
(a) 6.52 × 1024 kg (b) 6.6 × 1024 kg 4 8 2 8
(c) 6.82 × 1024 kg (d) 6 × 1024 kg 32. An observer moves towards a stationary source of
1
24. A star is moving towards the earth with a speed of sound with a velocity th of the velocity of sound.
6 × 10 m/s . If the wavelength of a particular
6 5
spectral line emitted by it is 4200Å, then apparent The percentage increase in the apparent frequency is
wavelength is (a) 20% (b) 50%
(a) 4005Å (b) 4116Å (c) 3600Å (d) 4525Å (c) 25% (d) 40%

MODULE 2
www.jeebooks.in
Prep Catalysis for NEET ~ Mock Test 12 335

33. Two concentric metallic spheres kept in air have 40. Efficiency of a Carnot engine working between
radii r1 and r2 (r1 > r2 ). They have similar charge and reservoirs 60° C and 30° C is
equal surface density σ, the electric potential at (a) 5% (b) 10% (c) 20% (d) 40%
their common centre is
σ σ 41. A man is 45 m behind the bus, when the bus starts
(a) (r1 + r2 ) (b) (r2 − r1 ) accelerating from rest with acceleration 2.5ms−2.
εο εo
σ σ With what minimum velocity should the man start
(c) (r1 − r2 ) (d) running to catch the bus?
εo εo (r2 − r1 )
(a) 15 ms −1 (b) 10 ms −1
34. The height at which the weight of body becomes (c) 7.5 ms −1 (d) 5 ms −1
5% less than that at the surface of earth is 42. The output Y of given logic circuit is
(radius of earth = 6400 km)
(a) 100 km (b) 150 km A
B
(c) 160 km (d) 75 km Y
A
35. An isotropic point source emits light with C
wavelength 500 nm. The radiation power of the
source is P = 10 W. Find the number of photons (a) AB + BC + AC (b) A + B + C
passing through unit area per second at a distance (c) ( A + B)(B + C ) (d) A + BC
of 3 m from the source. 43. A man sits on a chair which supported by a rope
(a) 5.92 × 1017 / m2s (b) 2.23 × 1017 / m2s passing over a frictionless fixed pulley. The man of
(c) 2.23 × 1018 / m2s (d) 5.92 × 1018 / m2s weight 1000 N exerts a force of 450 N on the chair
downwards, while pulling the rope. If the chair
36. A beam of ions enters normally into a uniform
weight 250 N and g is 10 ms−2, then the
magnetic field of 4 × 10−2 T with velocity 2 × 105 m/s.
If the specific charge of the ion is 5 × 107 C/kg, then acceleration of the chair is
9
the radius of the circular path is (a) 0.45 ms −2 (b) zero (c) ms −2 (d) 2 ms −2
(a) 0.10 m (b) 0.08 m 25
(c) 0.25 m (d) 0.35 m 44. Three rods made of same material and having
cross-section in ratio 1 : 2 : 3 are joined as shown
37. An AC current is given by I = I1 cos ωt + I 2 sin ωt.
in the figure. Each rod is of same length. The
The root mean square current is temperature at the junction of the three rods
I1 + I2 (I1 + I2 )2 in °C is
(a) (b)
2 2 45°C
I12 + I22 I12 − I22
(c) (d) B
2 2
A
0°C O
38. If minimum deviation = 30°, then speed of light in
shown prism will be
C

60°C
60°
(a) 45 (b) 60 (c) 80 (d) 35
45. The condition for producing sustained interference
are
I. phase difference between interfering waves
remains constant with time.
3 1
(a) × 108 m/s (b) × 108 m/s II. interfering waves have nearly same
2 2 amplitude levels
2 2
(c) × 108 m/s (d) × 108 m/s III. interfering waves are of same frequency
3 3
IV. interfering waves are moving in opposite
39. A plane transparent glass slab is placed over directions.
various coloured letters, the letter which appears
Choose the correct statements and mark the
to be raised the least is
correct option given below.
(a) green (b) yellow
(a) I, II and III (b) II and III
(c) violet (d) red
(c) III and IV (d) All are correct

MODULE 2
www.jeebooks.in

PART B CHEMISTRY
46. The coordination number and oxidation number of CH3
‘X’ in the compound [X (SO4 )(NH3 )5 ] Cl will be ½
(a) 10 and 3 (b) 2 and 6 (c) 6 and 3 (d) 6 and 4 (b) CH3 ¾ C ¾ Cl +Na + O -CH2CH3 ¾¾®
½
47. The sulphonation of aromatic hydrocarbon comes CH3
under eletrophilic substitution reaction. Generally, CH3
it is difficult to introduce more than one substituent ½
at the aromatic hydrocarbon. Which of the following CH3 CH2 ¾O ¾ C ¾ CH3
reagent is involved in sulphonation of aromatic ½
hydrocarbon? CH3
(a) HSO -4 (b) H2SO 4 (c) SO 2 (d) SO 3 (c) Both (a) and (b)
48. Which pair of electrolytes could not be distinguished (d) None of the above
by the product of electrolysis using inert electrodes?
54. The differential rate law for the reaction
(a) 1 M CuBr2 solution, 1 M CuSO 4 solution
H 2( g) + I2( g) = 2HI( g) is
(b) 1 M KCl solution, 1 M NaCl solution
- d [I2 ] - d [H2 ] d [HI]
(c) 1 M KCl solution, 1 M KI solution (a) = =
(d) 1 M AgNO 3 solution, 1 M Cu(NO 3 )2 solution dt dt dt
d [H2 ] d [I2 ] d [HI]
49. Why do 2º and 3º amines fail to undergo the (b) = =
carbylamine reaction? dt dt dt
1 d [H2 ] 1 d [I2 ] - d [HI]
(a) They react with alc. KOH (c) = =
(b) They combine with chloroform to give a stable compound 2 dt 2 dt dt
-2d [H2 ] -2d [I2 ] d [HI]
(c) The nitrogen atom of the amine group does not have the (d) = =
dt dt dt
required number of hydrogen atoms
(d) All the given reasons are correct 55. Which one of the following compounds does not
50. Predict product of the following reaction. give positive iodoform test?
(a) C 2H5OH (b) C 6H5OH
D O
CH3 ¾ C == CH 2 ¾¾¾®
2
(c) (CH3 )2 CHOH (d) All of these
½
OH 56. CHCl3 on reaction with conc. HNO 3 produces
(a) CH3 ¾ C == CH2 (b) CH3 ¾ C == CH2 (a) chloropicrin (b) nitromethane
½ ½½ (c) acetylene (d) picric acid
OD O
(c) CH2 == C — CH2D (d) CD2 == C ¾ CD3 57. All the following statements about the 3d-series
½ ½ elements are true except that
OH OD (a) scandium has lowest melting point
51. When soap is used for cleaning, it couples with dirt (b) enthalpy of atomisation of zinc is lowest
and remove from the clothes. Aggregates of (c) first ionisation energy of Sc is lowest and zinc is highest
particles is known as (d) scandium has lowest density
(a) micelles (b) sols (c) gels (d) emulsion
58. Which of the following has maximum EAN of the
52. In the reaction, underlined atoms?
CH3CHO + HCN ¾¾® CH3CH(OH)CN, a chiral
centre is produced. This product is (Cr = 24, Co = 27, Fe = 26, Ni = 28)
(a) meso compound (b) laevorotatory (a) [Cr(EDTA)]– (b) [Co(en)3 ]3+
3–
(c) dextrorotatory (d) racemic mixture (c) [Fe(C 2O 4 )3 ] (d) [Ni(CN)4 ]2-

53. Which of the following reactions is/are feasible? 59. Choose the correct product among four options
CH3
regarding following reaction.
½ Br
(a) CH3 CH2Br +Na + O- C ¾ CH3 ¾¾®
½ + –
KOBut
CH3 ?
CH3
½
CH3 CH2O ¾ C ¾ CH3
½ (a) (b) (c) (d)
CH3

MODULE 2
www.jeebooks.in
Prep Catalysis for NEET ~ Mock Test 12 337

60. Which of the following statements is not correct 68. Distillation is a method of separating mixture
regarding indicator according to Ostwald’s theory? based on differences in volatility of components in
(a) Indicator is a weak acid or weak base a boiling liquid mixture. Which of the following is
(b) Indicator changes its colour when the medium changes purified by distillation method?
from acid to alkaline or vice-versa (a) Zn (b) Al
(c) Indicator changes its colour when the solution becomes (c) Fe (d) None of these
exactly neutral
69. If K a1 , K a2 and K a3 are the respective ionisation
(d) colour of the solution depends on the relative
concentration of unionised and ionised form of indicator constants for the following reactions.
H 2S H + + HS-
-
61. Among the following oxyacids of sulphur, which + 2-
one has more than one oxidation state for sulphur? HS– -H + S + 2-
(a) H2S2O 6 (b) H2S2O 7 (c) H2SO 4 (d) H2S2O 3 H 2S - 2H + S
62. The compound used in enhrichment of uranium for Then the correct relationship between K a1 K a2 and
nuclear power plant is K a is
3
(a) UO 2 (NO 2 )2 (b) UCl 4 (c) UF6 (d) U3O 8 (a) K a 3 = K a1 / K a 2 (b) K a 3 = K a1 - K a 2
(c) K a 3 = K a1 + K a 2 (d) Ka 3 = K a1 ´ K a 2
63. The enthalpy of hydrogenation of cyclohexene is
-119.5 kJ mol -1. If resonance energy of benzene is 70. The space occupied in different packing is called
-150.4 kJ mol-1, its enthalpy of hydrogenation packing fraction.
would be Total volume of sphere
(a) -269.9 kJ mol -1 (b) -358.5 kJ mol -1 Packing fraction = .
Volume of the unit cell
(c) -508.9 kJ mol -1 . kJ mol -1
(d) -2081 Fraction of total volume occupied by atoms in a
64. 0.5 M ammonium benzoate is hydrolysed to simple cubic unit cell is
0.25 per cent. Hence, it hydrolysis constant is p 3p 2p p
(a) (b) (c) (d)
(a) 6.25 ´ 10-6 (b) 3125
. ´ 10-6 (c) 1 .25 ´ 10-5 (d) 2 .5 ´ 10-5 2 8 6 6

65. Dihydrogen on the industrial scale is prepared by 71. Among the given conformations of ethane, the one
the water gas shift reaction from petrochemical. It having least torsional strain is
is obtained as a by-product by the electrolysis of (a) boat (b) eclipsed (c) staggered (d) skew boat
brine. Which of the following is not true regarding 72. In cyanide method, silver metal is obtained as
hydrogen? 2K[Ag(CN)2 ] + Zn ¾¾® K 2 [Zn(CN)4 ] + 2Ag
(a) Hydrogen forms complexes in +1 as well as –1 oxidation
In this
state
(a) Ag + has been oxidised and Zn has been reduced
(b) Hydrogen has more than one isotope (b) Ag + has been reduced and Zn has been oxidised
(c) Hydrogen has mainly two allotropes (c) Both metals have been oxidised
(d) Sum of electron, proton and neutron of lightest isotope of (d) Both metals have been reduced
hydrogen is one
73. Choose the process involved in the separation of
66. Match Column I with Column II and select the ortho and para nitrophenol and the reason behind
correct answer. respectively.
Column I Column II (a) Steam distillation, metallic bonding
A. Cyanide process 1. Ultra-pure Ge (b) Sublimation, hydrogen bonding
(c) Steam distillation, van der Waals’ forces
B. Floatation process 2. Pine oil (d) Steam distillation, hydrogen bonding
C. Electrolytic reduction 3. Extraction of Al
74. What will be the partial pressure of He and O2
D. Zone refining 4. Extraction of Au
respectively, if 200 mL of He at 0.66 atm and
Codes 400 mL of O2 at 0.52 atm pressure are mixed in
A B C D A B C D 400 mL vessel at 20ºC.
(a) 4 2 3 1 (b) 3 1 2 4 (a) 0.33 and 0.56 (b) 0.33 and 0.52
(c) 2 4 1 3 (d) 1 3 4 2 (c) 0.38 and 0.52 (d) 0.25 and 0.45

67. The products obtained on disproportionation 75. For the first order reaction with, C0 as the initial
reaction of aqueous solution of hypochlorous acid is concentration and C at time t, (C0 - C ) is
(a) HCl and HClO 3 (b) HCl and Cl 2O (a) C 0 × e kt (b) C 0 × e - kt (c) C 0 [1 + e kt ] (d) C 0 [1 - e - kt ]
(c) HClO 2 and HClO 4 (d) HClO 3 and Cl 2O

MODULE 2
www.jeebooks.in
338 NEET Test Drive

76. On the extreme left of periodic table, groups I and 84. Which of the following is not correctly matched?
II elements. These include alkali and alkaline (a) Neoprene ¾® ¾ [ CH2 ¾ C == CH ¾ CH2]¾
n
earth metals or s-block elements. These elements |
Cl
have strong electropositive character. Which of the (b) PVC ¾® ¾ [ CH2 ¾ CH ¾
]n
following has largest size in aqueous ion? |
(a) Rb + (b) Na + (c) K + (d) Li + Cl
O
||
77. Densities of two gases are in the ratio 1 : 2 and (c) Nylon-6, 6 ¾® ¾ [ NH ¾(CH2 )6 ¾NH ¾C ¾
their temperatures are in the ratio 2 : 1, then the O
ratio of their respective pressures is ||
(a) 1 : 1 (b) 1 : 2 (c) 2 : 1 (d) 4 : 1 (CH2 )6 ¾C ¾O ]¾
n
CH2
78. Which of the following comparison is incorrect? |
(d) PMMA —CH
[ 2—C—]n
(a) —OH > CH3OH (acidic nature) |
COOCH3

(b) —NH2 > CH3NH2 (basic nature) 85. The rate constant, the activation energy and
Arrhenius parameter of a chemical reaction at 25ºC
are 3.0 ´ 10-4 s-1, 104.4 kJ mol -1 and 6.0 ´ 1014 s-1
(c) —CH2 > CH3CH2 (stability) respectively. The value of rate of constant at T ® ¥ is
(a) 2 .0 ´ 1018 s -1 (b) 6.0 ´ 1014 s -1
(c) ¥ (d) 3.6 ´ 1030 s -1
(d) —COH > CH3COH (acidic nature)
86. Which of the following statement(s) is/are correct?
79. Which of the following elements do not impart (a) Sucrose is a dextrorotatory sugar
colour to flame? (b) Sucrose has a pyranose and furanose ring linked together
by 1, 1¢- glycosidic linkage
Ca, Mg, Li, Na, K, Rb
(c) Maltose has one glycosidic linkage and a free hemiacetal end
(a) Ca and Mg (b) Li and Na
(c) K and Rb (d) Na and K (d) All of the above statement are correct

80. 25 mL of 3.0 M HNO 3 are mixed with 75 mL of 87. Which one is an incorrect statement?
(a) Principal quantum number gives the average distance of
4.0 M HNO3 . If the volumes are additive, the
an electron from the nucleus
molarity of the final mixture would be
(b) Azimuthal quantum number gives the information of
(a) 3.25 M (b) 4.0 M
angular momentum of an electron present in an subshell
(c) 3.75 M (d) 3.50 M
(c) The value of m for p z -orbital is +1
81. In which of the following oxides, central metal is (d) Five d-orbitals of a particular d-subshell are degenerate
showing variable valencies?
(a) Fe 2O 3 (b) BaO 2
88. The AsF5 molecule is trigonal bipyramidal. The
(c) PbO 2 (d) Pb 3O 4 hybrid orbitals used by the As atoms for bonding are
(a) d 2 2 ,d 2 , s, px , py (b) d xy , s, px , py , pz
x -y z
82. Identify the correct statement regarding enzymes. (c) s, px , py , pz , d 2 (d) d 2 2 , s, px , py
z x -y
(a) Enzymes are specific biological catalysts that can normally
function at very high temperature (T ~ 1000K ) 89. PVC is a thermoplastic which is used for making
(b) Enzymes are normally heterogeneous catalyst that are sheets, rain coats, table clothes, plastic dolls etc.
very specific in their action
Addition homopolymer is obtained by the repeated
addition of single monomer unit then, which of the
(c) Enzymes are specific biological catalysts that cannot be
following is a monomer unit of PVC?
poisoned
(a) Ethene (b) Propylene
(d) Enzymes are specific biological catalysts that possess well (c) Chloroethene (d) Tetrachloroethene
defined active sites
X
90. (CH3 )3 CCH == CH 2 ¾¾ ®(CH3 )3 CCH ¾ CH3
83. Which of the following property is correct ½
regarding the product which is obtained by OH
reaction of boron with nitrogen gas above 900º C? X can be
(a) It is very hard material (a) BH3 × THF / H2O 2 × OH-
(b) It has planar structure (b) H3O +
(c) It is less harder than boron carbide (c) Hg(OAc)2 / NaBH4 , NaOH
(d) It is isomorphous with diamond (d) None of the above
MODULE 2
www.jeebooks.in

PART C BIOLOGY
91. Breathing is a physiological process that involved 101. Arrange the evolutionary stages of man in correct
drawing air in and out from the atmosphere. sequence.
Which of the following is not involved in breathing Cro-Magnon-I Ramapithecus-II Java-ape man-III
movement? Peking man-IV Neanderthal man-V
(a) Diaphragm (b) Ribs Australopithecus-VI Handy man-VII
(c) Trachea (d) External intercostal muscles
Choose the correct option.
92. Identify the incorrectly matched pair. (a) VI → II → III → VII → IV → V → I
(a) Simple tissues—Parenchyma (b) II → V → III → IV → I → VII → VI
(b) Complex tissues—Xylem (c) VII → II → VI → III → IV → V → I
(c) Special tissues—Glandular tissue (d) II → VI → VII → III → IV → V → I
(d) None of the above
102. Which among the following statements is incorrect
93. Which of the following statements is correct? regarding continuous culture?
(a) Archenteron is the cavity which occurs inside the gastrula (a) Continuous cultivation needs sophisticated equipment to
(b) Yolk sac contains amniotic fluid maintain constant conditions
(c) The gestation period is 350 days (b) Volume of suspension is kept variable
(d) Blastocoel is found in morula stage (c) Fermenter does not have to be emptied, cleaned and
refilled very often
94. Identify the correct match. (d) Nutrients are added and cells are harvested at a constant
(a) UAA – Tryptophan (b) UGG – Phenylalanine rate
(c) CAA – Glutamine (d) AGC – Cystine
103. Which of the following statements is not true about
95. Given below is the representation of the region of pollen tube germination?
DNA encoding for β -haemoglobin. (a) The wall of the pollen tube is the exception of exine
Exon Intron Exon Intron Exon (b) The tube secretes exogenous pectinoses and other
hydrolytic enzymes
(c) It carries vegetative and generative nuclei
Which statement with respect to transcription and (d) It absorbs nourishment from the transmitting tissue of the
translation of exons and introns is correct? style
(a) Both exons and introns are transcribed, but only exons are
translated 104. Which of the following acts by competitively
(b) Both are transcribed, but only introns are translated inhibiting the enzyme responsible for synthesis of
(c) Exons are transcribed and translated cholesterol?
(d) Introns are transcribed and translated (a) Dextran (b) Streptokinase (c) Prednisone (d) Statin

96. Volant adaptation is characterised by 105. Powdery mildew of wheat is caused by


(a) flight or gliding type (b) webbing of feet (a) Erysiphe graminis (b) Helminthosporium oryzae
(c) pneumatic bones (d) All of these (c) Erysiphe polygoni (d) Ustilago tritici

97. If the diploid number of chromosomes is 36, i.e. 106. The term given to the type of teeth that have
2n = 36, then the numbers of chromosomes in crown with small, blunt and round cusps is
nucellus, endosperm and antipodals respectively (a) secodont (b) bunodont (c) lophodont (d) hypsodont
would be 107. Fascicles are
(a) 36, 18 and 54 (b) 36, 54 and 18 (a) bundles of skeletal muscle fibres
(c) 18, 36 and 54 (d) 18, 54 and 36 (b) bundles of cardiac muscles
98. Which of the following combinations of drugs has a (c) bundles of smooth muscles
dramatic rapid increase in sedative effect? (d) bundles of white muscle fibres
(a) Alcohol + Antihistamines 108. Which among the following statements depicts the
(b) Alcohol + Benzodiazepines function of insertional inactivation?
(c) Alcohol + Aspirin (a) The recombinants and non-recombinants cannot be
(d) Amphetamine + Insulin differentiated
99. Litmus, an acid-base indicator is derived from (b) The recombinants and non-recombinants can be
(a) Parmelia molliuscula (b) Roccella montaignei differentiated on the basis of colour production
(c) Cladonia rangiferina (d) Cladonia crispata (c) The recombinants can be differentiated on the basis of
antibiotic resistance
100. Chloride shift is a term associated with (d) The recombinants can be differentiated by the use of
(a) electrostatic neutrality of plasma physical methods
(b) transfer of macromolecules in cytoplasm
(c) conduction of nerve impulse 109. The pollen-pistil interaction exhibits an example of
(d) activation of muscles for contraction (a) phototropism (b) chemotropism
(c) thigmotropism (d) hydrotropism

MODULE 2
www.jeebooks.in
340 NEET Test Drive

110. The semipermeable membrane 118. Match Column I with Column II and choose the
(a) allows passage of solvent as well as some selective solute correct option from the codes given below.
particles Column I Column II
(b) allows passage of only solvent molecules
(Type of migration) (Characteristic)
(c) requires energy
(d) possesses specific sites for passage of particles A. Diurnal migrations 1. Physical environment
B. Metamorphic migrations 2. Animal’s life cycle
111. Stercobilin is a pigment which causes
(a) yellow colour of bile juice (b) pale yellow colour of urine C. Periodic migrations 3. Feeding capacity
(c) brown-red colour of human faeces D. Annual migrations 4. Latitude, altitude
(d) Both (a) and (c)
Codes
112. Which among the following is incorrectly matched A B C D A B C D
pair? (a) 1 3 2 4 (b) 1 2 3 4
(a) Silicified epidermal layer—Equisetum (c) 3 2 1 4 (d) 1 4 3 2
(b) Waxy deposition—Banana leaves 119. An atrioventricular valve prevents the backflow of
(c) Epidermal hairs of seed coat—Cotton fibres blood from the
(d) None of the above (a) right ventricle into right atrium
113. Examples of chasmo-cleistogamous plant are (b) left atrium into left ventricle
(a) Coccinia and Corica (c) aorta into left ventricle
(b) Commelina and Oxalis (d) pulmonary vein into right atrium
(c) Commelina and Trichosanthes 120. Various requirements for the production of
(d) Dracaena and Yucca microbes in fermenters include
114. ‘One gene-one enzyme’ hypothesis was given by (a) oxygen is needed for aerobic respiration of some
Beadle and Tatum. On which of the following were microorganisms
their studies based on? (b) a source of carbohydrate as an energy source
(a) E. coli (b) Neurospora crassa (c) a source of nitrogen for protein synthesis
(c) Streptococcus pneumoniae (d) Agaricus bisporus (d) All of the above

115. Which among the following codons has a dual 121. Which of the following is an isomer of
function? dihydroxyacetone phosphate?
(a) UAA (b) UGA (c) UAG (d) AUG (a) d PGA (b) PGAL (c) PGA (d) Pyruvate

116. Match Column I with Column II and choose the 122. With respect to photosynthesis in bacteria, which
correct option from the codes given below. of the following statements is incorrect?
(a) In bacterial photosynthesis, Emerson effect is not found
Column I Column II (b) Plastocyanin (PC) is present
(Type) (Component) (c) P 890 is the reaction centre and light is absorbed at
A. Producers 1. Heterotrophic component wavelengths 800-900 nm or infrared
(d) All are correct
B. Macroconsumers 2. Decomposers
C. Microconsumers 3. Herbivores 123. Plants having scattered vascular bundles cannot be
propagated by air layering because
D. Primary consumers 4. Autotrophic component
(a) vascular bundles may be harmed
Codes (b) cortex may be harmed
A B C D A B C D (c) pith is harmed
(a) 4 1 2 3 (b) 1 2 3 4 (d) sunlight is blocked
(c) 4 3 2 1 (d) 1 2 4 3
124. Which of the following statements is incorrect?
117. Match the following columns and choose the (a) mRNA in eukaryotic cells is monocistronic
correct option from the codes given below. (b) tRNA has four loops
Column I Column II (c) mRNA in eukaryotic cell is 5′-capped and 3′-
polyadenylated
A. Diabetes mellitus 1. Pancreatic disorder (d) Eukaryotic mRNAs have a very short lifespan
B. Diabetes insipidus 2. Osteoporosis
125. Which of the following accessory excretory organs
C. Adrenal virilism 3. Hypothalamic disorder excretes sodium chloride, lactic acid, etc?
D. Hyperparathyroidism 4. Sexocorticoids (a) Lungs (b) Liver
(c) Intestine (d) Skin
Codes
A B C D A B C D 126. Which of the following hormones is involved in
(a) 1 3 4 2 (b) 1 2 4 3 phototropism?
(c) 4 2 1 3 (d) 2 4 3 1 (a) Zeatin (b) 2, 4-D (c) IAA (d) GA

MODULE 2
www.jeebooks.in
Prep Catalysis for NEET ~ Mock Test 12 341

127. In a patient, whose voluntary movements are slow 136. In lysosome, a type of enzyme called cathepsins is
and limbs are trembling there is present, it helps in digesting
(a) reduced dopamine in brain (a) glycosides (b) glycogen (c) sphingolipids (d) proteins
(b) reduced or loss of acetylcholine
137. Identify the molecule shown below and select the
(c) multiple sclerosis
correct option for its source and use.
(d) All of the above OH
128. Find out the correctly matched pair.
(a) Cyanobacteria – Biopesticide
(b) Single cell protein – Rhizobia O
(c) Glomus – Solubilisation of phosphate H
(d) Bacillus thuringiensis – Bioherbicide
Molecule Source Use
129. Gametangial contact is observed in (a) Cocaine Erythroxylum coca
Accelerates dopamine
(a) Mucor (b) Puccinia transport
(c) Rhizopus (d) Phytophthora albugo (b) Heroin Atropa belladona Depressant
130. Heart muscles in normal conditions are (c) Cannabinoid Cannabis sativa Affects cardiovascular
controlled by system
(a) abducens nerve (b) trigeminal nerve (d) Morphine Papaver somniferum Sedative and pain reliever
(c) accessory nerve (d) vagus nerve
138. Organisation of stem apex into corpus and tunica
131. The hormones secreted by placenta include is mainly determined by the
I. Human placental lactogen (a) planes of cell division
II. Oestrogens (b) region of meristematic activity
(c) cellular growth rate
III. Progesterones
(d) presence of vascular bundles
IV. Human chorionic gonadotropin
Choose the correct option. 139. In cyton of nerves, Nissl’s granules are found
(a) I and II (b) II and III (c) I, III and IV (d) All of these which are rich in
(a) proteins (b) RER (c) cholesterol (d) SER
132. The haplobiontic life cycle is shown in
(a) Schizosaccharomyces octosporous 140. Consider the following statements regarding
(b) Saccharomyces ludwigii reductive amination and choose the correct option.
(c) Saccharomyces cerevisiae (a) Ammonia combines with a keto acid
(d) None of the above (b) Ammonia combines with amino acid glutamate
(c) Transfer of amino groups from an amino acid with carboxyl
133. Which hormone released from hypothalamus group of a keto acid
inhibits the release of growth hormone from the (d) Ammonia changes from gaseous to ionic form in the soil
pituitary?
(a) Somatostatin 141. Which of the following statements is/are true about
(b) Thyrotropin-releasing hormone muscle contraction?
(c) Prolactin I. During shortening, I bands get reduced
(d) Luteinising releasing hormone whereas ‘A’ bands retain their length.
134. In an accident, an individual’s spinal cord was II. The part between the two ‘H’ zones is
severed at the neck. As a result, the patient will sarcomere.
show III. Binding of Ca 2 + with a subunit of troponin
I. Inability to carry out reflex action below the leads to the formation of cross bridge.
neck region.
IV. A-bands contain actin and are light bands.
II. Inability of voluntary movement from below
Choose the correct option.
the neck region.
(a) I, II and III (b) II and III (c) I and II (d) Only III
III. Inability to perceive sensory stimuli from
below the neck region. 142. The sclerenchyma is stained by
(a) safranin (b) aniline blue (c) fast green (d) orange dye
Choose the correct option.
(a) I and II (b) II and III (c) I and III (d) All of these 143. Consider the following statements.
135. Colchicine inhibits the formation of spindle fibres. I. The early symptom of STDs include swelling
In a cell, exposed to colchicine in the genital region.
(a) chromosomes are observed to be randomly distributed II. STDs can be prevented by using condoms.
(b) chromosomes appear as loose chromatin threads III. STDs can be transmitted through transfusion
(c) sister chromatids are separated rather than being held at of infected blood.
centromere IV. Filariasis is a STD.
(d) no centrioles will be present

MODULE 2
www.jeebooks.in
342 NEET Test Drive

The incorrect statement(s) is/are 151. Consider the following statements.


(a) I and II (b) II and III (c) III and IV (d) Only IV I. Trochophore larvae are found in both annelids
144. A person is suffering from diabetes insipidus. This and molluscs.
results due to reduction in vasopressin secretion II. Birds embryo has tooth buds which are
from the posterior pituitary. He is likely to show present in adults.
which of the following symptoms? III. Retrogressive metamorphosis is seen in adult
(a) Passes much urine and drinks much water ascidians.
(b) Has high blood glucose level IV. Neoteny is a condition in which larvae fail to
(c) Blood glucose levels are below normal undergo metamorphosis.
(d) Both (a) and (d) The incorrect statement(s) is/are
145. In gel electrophoresis, (a) I and III (b) Only II (c) Only III (d) Only IV
I. Smaller the fragment size, the farther it will move. 152. Consider the following statements regarding
II. Larger the fragment size, the farther it will sanctuary and choose the correct option.
move. (a) Grazing is allowed
(b) Cultivation of land is permitted
III. Smaller the fragment size, the lesser distance (c) Protection of both flora and fauna
it will move. (d) Both (a) and (b)
IV. Larger the fragment size, the lesser distance 153. An example of blind sac body plan is
it will move. (a) flatworms (b) roundworms
The correct statements are (c) arthropods (d) echinoderms
(a) I and IV (b) III and IV (c) II and III (d) II and IV
154. Which among the following is the side effect of
146. In cockroach, true coelom occurs in drugs on females?
(a) embryonic stage (b) adult stage (a) Reduction in size of testicles
(c) Both (a) and (b) (d) None of these (b) Premature baldness
(c) Enlargement of prostate gland
147. Choose the correct option for A, B and C from the (d) Masculinisation
given options.
155. Manubrium, gladiolus and xiphoid process are
I. Decrease in number of RBCs is called
parts of
.........A...... . (a) ribs (b) coxal bone (c) sternum (d) patella
II. Rise in number of RBCs is called ...... B ..... .
156. Two plants can be conclusively said to belong to
III. Erythropoietin is released by ......... C..... . same species if they
A B C (a) can freely reproduce with each other and produce seeds
(b) share 90 per cent similar genes
(a) Polycythemia Erythrocytopenia Erythropoietin (c) appear similar and possess identical secondary
(b) Erythropoietin Erythrocytopenia Polycythemia metabolites
(d) have same number of chromosomes
(c) Erythropoietin Polycythemia Erythropoietin
157. Leaf spines are seen in
(d) Erythrocytopenia Polycythemia Kidneys (a) Nepenthes (b) Zingiber officinale
(c) Opuntia (d) Acacia
148. Mutation breeding is carried out by using
(a) chemicals like aniline 158. The complementary codon present on tRNA is
(b) radiations like gamma radiations (a) stop codon (b) anticodon
(c) conventional breeding method (c) start codon (d) nonsense codon
(d) Both (a) and (b) 159. Consider the following statements.
149. Organic pollutants include I. Outbreeding refers to the mating of unrelated
I. DDT II. HCl animals.
III. Pathogenic microorganisms IV. Oils II. Outcrossing refers to the mating between
unrelated members of the same breed.
The correct option is
(a) I and IV (b) II and III (c) III and IV (d) I and II III. Cross-breeding refers to the cross of superior
males of one breed with superior females of
150. Which is the correct set of a single endocrine gland another breed.
hormone? IV. Interspecific hybridisation refers to crossing
(a) Gonadotropins, Growth hormone, Prolactin between male and female animals of two same
(b) Thyroxine, Growth hormone, ACTH species.
(c) Oxytocin, Vasopressin, ICSH
The incorrect statement is
(d) Oxytocin, Vasopressin, Prolactin
(a) Only I (b) Only II (c) Only III (d) Only IV

MODULE 2
www.jeebooks.in
Prep Catalysis for NEET ~ Mock Test 12 343

160. Which of the following statements is incorrect? 167. Consider the following statements.
(a) The results of Mendel’s monohybrid crosses were not I. Australopithecus — Fossils found in
sex-dependent Tanzania.
(b) Reginald C. Punnett devised the approach of constructing II. Homo erectus— Fossils found in Java.
Punnett square
(c) Test cross method to discover the genotype of the III. Homo habilis — Fossils found in East and
organisms was devised by Mendel Central Asia.
(d) Down’s syndrome is a sex-chromosomal disorder IV. Homo sapiens neanderthalensis — Fossils
found in Ethiopia.
161. Choose the incorrect match.
(a) Haplostele — Rhynia Choose the incorrect matches.
(b) Actinostele — Lycopodium cernum (a) I and II (b) III and IV
(c) Plectostele — Lycopodium volubile (c) IV and I (d) I and III
(d) Solenostele — Adiantum pedatum 168. In sedimentary cycles, the main reservoir is
162. Consider the following statements. (a) lithosphere
(b) biosphere
I. Macromolecules are involved in building up of
(c) atmosphere
plant body. (d) hydrosphere
II. Microelement potassium is involved in turgor
movement. 169. Identify the incorrectly matched pair.
(a) Gloger’s rule — organisms in warm regions are dark in
III. Microelements become toxic if they are in colour
excess. (b) Bergman’s rule — tail, snout, ears of organisms in colder
IV. Macromolecules form the different parts are shorter than in warmer parts
constituents of protoplasm. (c) Both (a) and (b)
(d) None of the above
The incorrect statement is
(a) Only I (b) Only II (c) Only III (d) Only IV 170. Intracellular enzymes do not involve the
characteristic such as
163. Gonyaulax is a (a) difficult to isolate
(a) dinoflagellate (b) golden algae (b) often stable only in environment inside intact cell
(c) diatom (d) euglenoids (c) downstreaming processing is difficult
164. Retroperitoneal refers to (d) no need to break cells
(a) location of kidney outside the peritoneal cavity 171. The chemical method of making a cell competent
(b) external structure of kidney includes
(c) internal structure of kidney (a) micro-injection method
(d) function of kidney (b) gene gun method
165. Consider the following statements. (c) heat shock treatment
I. Mycoplasmas occur saprotrophically in soil (d) disarmed pathogen vectors
and sewage water. 172. Which of the following convention used in human
II. Mycoplasmas are unicellular. pedigrees is not paired correctly?
III. Mycoplasmas possess plasma membrane as
the outermost boundary. (a) Affected individuals
IV. There is the presence of histones in
mycoplasma DNA.
The incorrect statement(s) is/are
(a) I and II (b) III and IV (b) Offspring
(c) II and III (d) Only IV (in birth order)
166. Consider the following statements regarding
amphibians.
I. Brain is well-developed with 12 pairs of
(c) 4 4 Multiple individuals (unaffected)

cranial nerves.
II. Amphibians have smallest RBCs of animal (d) Heterozygous carriers
kingdom.
III. Amphibians are poikilothermic.
IV. Exoskeleton is absent. 173. The cellulosic cell wall is present in
(a) parenchyma (b) sclerenchyma
The incorrect statements are (c) glandular tissue (d) xylem and phloem
(a) I and II (b) II and III (c) III and IV (d) I and IV

MODULE 2
www.jeebooks.in
344 NEET Test Drive

174. St. Anthony’s fire is caused by 178. Consider the following statements.
(a) Claviceps purpurea (b) Aspergillus flavus I. Diakinesis is also known as terminalisation.
(c) Penicillium (d) Amanita muscaria
II. Chiasmata are X-shaped structures.
175. In genetic engineering process of insulin III. The process of association of homologous
production, the vector used for modification of host chromosomes is synapsis.
cell is a
(a) bacteria (b) phage (c) YAC (d) plasmid IV. In diplotene stage, the process of crossing over
takes place.
176. Consider the following statements.
The incorrect statement(s) is/are
I. Barr body is a deeply stained chromatin body. (a) I and II (b) Only II (c) Only III (d) Only IV
II. Barr body was observed in nerve cells of
179. Which statement is false regarding Hydra
female cat.
vulgaris?
III. Barr body is present in males. (a) Hydra is a freshwater solitary polyp
IV. The sex chromatin is observed in the (b) Nematocyst contains a poisonous fluid or hypnotoxin
metaphase. inside it
(c) The germ cells are seen during summer, which is the
The correct statements are
breeding season for Hydra
(a) I and II (b) III and IV (d) Hydra is a triploblastic animal
(c) II and III (d) I and IV
180. The two nucleotides are joined by
177. Choose the incorrectly matched pair. (a) 3′ − 5′ phosphodiester linkage
(a) Incomplete dominance — Mirabilis jalapa (b) 5′ − 3′ phosphodiester linkage
(b) Codominance — MN blood group (c) N-glycosidic linkage
(c) Dominant epistasis — Coat colour in dogs (d) hydrogen bond
(d) Pleiotropy — Lathyrus odoratus

Answer Sheet
1. (b) 2. (a) 3. (a) 4. (c) 5. (a) 6. (d) 7. (a) 8. (d) 9. (c) 10. (c)
11. (c) 12. (c) 13. (a) 14. (a) 15. (b) 16. (c) 17. (b) 18. (c) 19. (a) 20. (a)
21. (d) 22. (a) 23. (c) 24. (b) 25. (c) 26. (b) 27. (c) 28. (c) 29. (d) 30. (d)
31. (b) 32. (a) 33. (a) 34. (c) 35. (b) 36. (a) 37. (c) 38. (a) 39. (d) 40. (b)
41. (a) 42. (d) 43. (d) 44. (a) 45. (a) 46. (c) 47. (d) 48. (b) 49. (a) 50. (a)
51. (a) 52. (d) 53. (a) 54. (d) 55. (b) 56. (a) 57. (a) 58. (b) 59. (b) 60. (c)
61. (d) 62. (c) 63. (d) 64. (b) 65. (d) 66. (a) 67. (a) 68. (a) 69. (d) 70. (d)
71. (c) 72. (b) 73. (d) 74. (b) 75. (d) 76. (d) 77. (a) 78. (b) 79. (a) 80. (c)
81. (d) 82. (d) 83. (a) 84. (c) 85. (b) 86. (d) 87. (c) 88. (c) 89. (c) 90. (c)
91. (c) 92. (d) 93. (a) 94. (c) 95. (a) 96. (d) 97. (b) 98. (b) 99. (b) 100. (a)

101 (d) 102. (b) 103. (a) 104. (d) 105. (a) 106 (b) 107. (a) 108. (b) 109. (b) 110. (b)
111. (c) 112. (d) 113. (b) 114. (b) 115. (d) 116. (a) 117. (a) 118. (b) 119. (a) 120. (d)
121. (b) 122. (d) 123. (a) 124. (d) 125. (d) 126. (c) 127. (a) 128. (c) 129. (d) 130. (d)
131. (d) 132. (a) 133. (a) 134. (b) 135. (a) 136. (d) 137. (c) 138. (a) 139. (b) 140. (a)
141. (b) 142. (a) 143. (d) 144. (a) 145. (a) 146. (a) 147. (d) 148. (d) 149. (a) 150. (a)
151. (b) 152. (d) 153. (a) 154. (d) 155. (c) 156. (a) 157. (c) 158. (b) 159. (d) 160. (d)
161. (b) 162. (b) 163. (a) 164. (a) 165. (d) 166. (a) 167. (b) 168. (a) 169. (b) 170. (d)
171. (c) 172. (d) 173. (a) 174. (a) 175. (d) 176. (a) 177. (d) 178. (d) 179. (d) 180. (a)

For explanations, open the link


https://goo.gl/tmEYgH

MODULE 2
www.jeebooks.in

www.jeebooks.in

MODULE 3
The NEET Edge
This section contains Most difficult questions of Physics,
Chemistry & Biology asked in last 15 years (2004-18)
AIPMT & NEET. Solving these questions without referring
to solutions, will boost your confidence.
www.jeebooks.in
The NEET Edge ~ Physics 347

THE NEET EDGE


Chapterwise Collection of Most Difficult Physics Questions asked in last 15 Years’ NEET/AIPMT

PHYSICS
1. Units and Measurements 6. If dimensions of critical velocity vc of a liquid
1. A student measured the diameter of a small steel flowing through a tube are expressed as [hx r yr z ],
ball using a screw gauge of least count 0.001 cm. where h , r and r are the coefficient of viscosity of
The main scale reading is 5 mm and zero of liquid, density of liquid and radius of the tube
circular scale division coincides with 25 divisions respectively, then the values of x , y and z are
above the reference level. If screw gauge has a zero given by [CBSE-AIPMT 2015]
error of -0.004 cm, the correct diameter of the (a) 1, - 1, - 1 (b) - 1, - 1, 1
ball is [NEET 2018] (c) - 1, - 1, - 1 (d) 1, 1, 1
(a) 0.053 cm (b) 0.525 cm (c) 0.521 cm (d) 0.529 cm 7. If force F, velocity v and time t are taken as
2. In an experiment, four quantities a , b, c and d are fundamental units, then the dimensions of
measured with percentage error 1%, 2%, 3% and mass are [CBSE-AIPMT 2014]
4% respectively. Quantity P is calculated (a) [FvT -1] (b) [FvT -2 ]
a3 b2 -1 -1
(c) [Fv T ] (d) [Fv -1T]
P= %. Error in P is [NEET 2013]
cd 1
8. The dimensions of e 0E 2, where e 0 is permittivity
(a) 14% (b) 10% (c) 7% (d) 4% 2
3. If the error in the measurement of radius of a of free space and E is electric field, are
sphere is 2%, then the error in the determination [CBSE-AIPMT 2010]
of volume of the sphere will be [CBSE-AIPMT 2008] (a) [ML2 T -2 ] (b) [ML-1T -2 ]
(a) 4% (b) 6% (c) 8% (d) 2% (c) [ML2 T -1] (d) [MLT -1]
4. A physical quantity of the dimensions of length 9. If the dimensions of a physical quantity are
e2 given by [M a LbT c] , then the physical quantity
that can be formed out of c, G and is [c is
4pe 0 will be [CBSE-AIPMT 2009]
velocity of light, G is universal constant of (a) pressure if a = 1, b = - 1, c = - 2
gravitation and e is charge] [NEET 2017] (b) velocity if a = 1, b = 0, c = - 1
1/ 2 1/ 2 (c) acceleration if a = 1, b = 1, c = - 2
1 é e2 ù é e2 ù (d) force if a = 0, b = - 1, c = - 2
(a) 2 ê
G ú (b) c 2 êG ú
c ë 4pe0 û ë 4pe0 û
1/ 2
10. Which two of the following five physical parameters
1 é e2 ù 1 e2 have the same dimensions?
(c) 2 ê ú (d) G
c ë G 4pe0 û c 4pe0 (i) Energy density (ii) Refractive index
(iii) Dielectric constant (iv) Young’s modulus
5. If energy E, velocity v and time T are chosen as the (v) Magnetic field [CBSE-AIPMT 2008]
fundamental quantities, the dimensional formula
of surface tension will be [CBSE-AIPMT 2015] (a) (ii) and (iv) (b) (iii) and (v)
-2 -1 -1 -2 -2 -2 -2 -1 -3 (c) (i) and (iv) (d) (i) and (v)
(a) [E v T ] (b) [E v T ] (c) [E v T ] (d) [E v T ]

MODULE 3
www.jeebooks.in
348 NEET Test Drive

11. Dimensions of resistance in an electrical circuit, in the position of the particle. The acceleration of the
terms of dimension of mass M , of length L, of time particle as a function of x, is given by
T and of current I , would be [CBSE-AIPMT 2007] [CBSE-AIPMT 2015]
2 -3 -1
(a) [ML T I ] (b) [ML T ] 2 -2 2 -1 -1
(c) [ML T I ] (d) [ML T I ] 2 -3 -2 (a) -2 nb 2 x-2 n -1 (b) -2 nb 2 x-4 n -1
(c) -2b 2 x-2 n + 1 (d) -2 nb 2 e -4 n + 1
12. The velocity v of a particle at time t is given by
b 18. A particle moving along X-axis has acceleration f,
v = at + , where a , b and c are constants. The
t+c
at time t, given by f = f0 æç1 - t ö÷, where f0 and T
dimensions of a , b and c respectively are è Tø
[CBSE-AIPMT 2006] are constants. The particle at t = 0 has zero
(a) [LT –2 ], [L] and [ T ] (b) [L2 ], [T] and [LT2 ] velocity. In the time interval between t = 0
(c) [LT2 ], [LT] and [L] (d) [L], [LT] and [T2 ] and the instant when f = 0, the particle’s velocity
vx is [CBSE-AIPMT 2007]
1 2 1
(a) f0T (b) f0T (c) f0T 2 (d) f0T
2. Motion in a Straight Line 2 2

13. Preeti reached the metro station and found that 19. A stone falls freely under gravity. It covers
the escalator was not working. She walked up the distances h1 , h2 and h3 in the first 5s, the next 5s
stationary escalator in time t1. On other days, if and the next 5s respectively. The relation between
she remains stationary on the moving escalator, h1 , h2 and h3 is [NEET 2013]
then the escalator takes her up in time t2. The time h2 h3
(a) h1 = 2 h2 = 3 h3 (b) h1 = =
taken by her to walk up on the moving escalator 3 5
will be [NEET 2017] (c) h2 = 3 h1 and h3 = 3 h2 (d) h1 = h2 = h3
t + t2 t t t t
(a) 1 (b) 1 2 (c) 1 2 (d) t 1 - t 2 20. A ball is dropped from a high rise platform at t = 0
2 t 2 - t1 t 2 + t1 starting from rest. After 6 s, another ball is thrown
14. A particle shows distance-time curve as given in downwards from the same platform with a speed v.
this figure. The maximum instantaneous velocity The two balls meet at t = 18 s. What is the value of
of the particle is around the point [CBSE-AIPMT 2008] v? (Take, g = 10 ms -2 ) [CBSE-AIPMT 2010]
(a) 75 ms -1 (b) 55 ms -1
-1
D (c) 40 ms (d) 60 ms -1
Distance

C 21. A man throws balls with the same speed vertically


upwards one after the other at an interval of 2 s.
A B What should be the speed of the throw so that
Time
more than two balls are in the sky at any time?
(Take, g = 9.8 m / s2) [CBSE-AIPMT 2003]
(a) B (b) C (c) D (d) A (a) Any speed less than 19.6 m/s
15. If the velocity of a particle is v = At + Bt , where A 2 (b) Only with speed 19.6 m/s
(c) More than 19.6 m/s
and B are constants, then the distance travelled by
(d) At least 9.8 m/s
it between 1s and 2s is [NEET 2016]
3 7 22. The distance travelled by a particle starting from
(a) 3 A + 7 B (b) A+ B
4
2 3 rest and moving with an acceleration ms-2 , in the
A B 3 3
(c) + (d) A + 4B
2 3 2 3 s is [CBSE-AIPMT 2008]
10 19
16. Two cars P and Q start from a point at the same (a) 6 m (b) 4 m (c) m (d) m
time in a straight line and their positions are 3 3
represented by X P ( t ) = at + bt 2 and XQ ( t ) = ft - t 2.
At what time do the cars have the same velocity? 3. Motion in a Plane
[NEET 2016] 23. The x and y coordinates of the particle at any time
a-f a+ f a+ f f-a are x = 5 t - 2 t 2 and y = 10 t respectively, where x
(a) (b) (c) (d)
1+ b 2 (b - 1) 2 (1 + b) 2 (1 + b ) and y are in metres and t in seconds. The
17. A particle of unit mass undergoes one-dimensional acceleration of the particle at t = 2 s is [NEET 2017]
motion such that its velocity varies according to (a) 0 (b) 5 m/s 2
v ( x ) = b x -2n where, b and n are constants and x is (c) -4 m/s 2 (d) -8 m/s 2

MODULE 3
www.jeebooks.in
The NEET Edge ~ Physics 349

24. If the magnitude of sum of two vectors is equal to 31. In the given figure, a = 15 m/s 2 represents the total
the magnitude of difference of the two vectors, the acceleration of a particle moving in the clockwise
angle between these vectors is direction in a circle of radius R = 2.5 m at a given
[NEET 2016, CBSE-AIPMT 1991] instant of time. The speed of the particle is
(a) 90° (b) 45° (c) 180° (d) 0° [NEET 2016]
25. Two particles A and B, move with constant
velocities v1 and v 2. At the initial moment, their R 30° a
position vectors are r1 and r2 respectively. The
condition for particles A and B for their collision is O
[CBSE-AIPMT 2015]
r1 - r2 v - v1
(a) = 2 (b) r1 × v1 = r2 × v 2
| r1 - r2 | | v 2 - v1 | (a) 4.5 m/s (b) 5.0 m/s (c) 5.7 m/s (d) 6.2 m/s
(c) r1 ´ v1 = r2 ´ v 2 (d) r1 - r2 = v1 - v 2
32. Two stones of masses m and 2m are whirled in
26. If vectors A = cos wt $i + sin wt $j and r
wt $ wt $ horizontal circles, the heavier one in a radius and
B = cos i + sin j are functions of time, then the 2
2 2 the lighter one in radius r. The tangential speed of
value of t at which they are orthogonal to each other, lighter stone is n times that of the value of heavier
is [CBSE-AIPMT 2015] stone when they experience same centripetal forces.
p p p The value of n is
(a) t = (b) t = (c) t = (d) t = 0 [CBSE-AIPMT 2015]
4w 2w w (a) 2 (b) 3 (c) 4 (d) 1
27. The position vector of a particle R as a function of 33. A stone tied to the end of a string of 1 m long is
time is given by R = 4 sin ( 2pt ) $i + 4 cos( 2pt ) $j where, whirled in a horizontal circle with a constant
R is in meter, t is in seconds and $i and $j denote unit speed. If the stone makes 22 revolutions in 44 s,
vectors along x and y-directions, respectively. Which what is the magnitude and direction of acceleration
one of the following statements is wrong for the of the stone? [CBSE-AIPMT 2005]
motion of particle? p2
(a) ms -2 and direction along the radius towards the centre
[CBSE-AIPMT 2015]
(a) Acceleration is along-R 4
v2 (b) p 2 ms -2 and direction along the radius away from centre
(b) Magnitude of acceleration vector is where v is the
velocity of particle R (c) p 2 ms -2 and direction along the radius towards the centre
(c) Magnitude of the velocity of particle is 8 m/s (d) p 2 ms -2 and direction along the tangent to the circle
(d) Path of the particle is a circle of radius 4 m
28. A ship A is moving Westwards with a speed of 4. Newton's Laws of Motion
10 km h –1 and a ship B 100 km South of A, is
moving Northwards with a speed of 10 km h–1. The 34. Three forces acting on a body are shown in the
time after which the distance between them become figure. To have the resultant force only along the
shortest is [CBSE-AIPMT 2015]
y-direction, the magnitude of the minimum
additional force needed is [CBSE-AIPMT 2008]
(a) 0 h (b) 5 h (c) 5 2 h (d) 10 2 h
y
29. A projectile is fired from the surface of the earth 1N
with a velocity of 5 ms -1 at angle q with the 4N
horizontal. Another projectile fired from another 30°
planet with a velocity of 3ms -1 at the same angle 60°
follows a trajectory which is identical with the x
trajectory of the projectile fired from the earth. The
30°
value of the acceleration due to gravity on the
planet (in ms-2 ) is (given, g = 9. 8 ms -2) 2N
[CBSE-AIPMT 2014]
(a) 0.5 N (b) 1.5 N
(a) 3.5 (b) 5.9 (c) 16.3 (d) 110.8 3
(c) N (d) 3 N
30. For angles of projection of a projectile at angles 4
( 45° - q ) and ( 45° + q ), the horizontal ranges 35. A 0.5 kg ball moving with a speed of 12 m/s
described by the projectile are in the ratio of strikes a hard wall at an angle of 30° with the
[CBSE-AIPMT 2006] wall. It is reflected with the same speed and at
(a) 1 : 1 (b) 2 : 3 (c) 1 : 2 (d) 2 : 1 the same angle.

MODULE 3
www.jeebooks.in
350 NEET Test Drive

If the ball is in contact with the wall for 0.25 s, the 39. A block of mass m is in contact with the cart C as
average force acting on the wall is [CBSE-AIPMT 2006] shown in the figure

30°
C m

30°
The coefficient of static friction between the
block and the cart is m. The acceleration a of the
(a) 48 N (b) 24 N (c) 12 N (d) 96 N cart that will prevent the block from falling
36. A block A of mass m1 rests on a horizontal table. A satisfies [CBSE-AIPMT 2010]
mg g g g
light string connected to it passes over a (a) a > (b) a > (c) a ³ (d) a <
frictionless pulley at the edge of table and from its m mm m m
other end another block B of mass m2 is suspended. 40. The coefficient of static friction, m s , between block
The coefficient of kinetic friction between the
A of mass 2 kg and the table as shown in the figure
block and the table is m k. When the block A is
sliding on the table, the tension in the string is is 0.2. What would be the maximum mass value of
[CBSE-AIPMT 2015] block B, so that the two blocks do not move ? The
(m + m k m1 )g (m 2 - m k m1 ) g string and the pulley are assumed to be smooth
(a) 2 (b)
(m1 + m2 ) (m1 + m 2 ) and massless (Take, g = 10 m/s2 ) [CBSE-AIPMT 2004]
m1m 2 (1 + m k ) g m1m 2 (1 - m k ) g
(c) (d) 2 kg A
(m1 + m 2 ) (m1 + m 2 )

37. A system consists of three masses m1, m2 and m3 T


connected by a string passing over a pulley P. The
mass m1 hangs freely and m2 and m3 are on a rough B
horizontal table (the coefficient of friction = m). The
pulley is frictionless and of negligible mass. The (a) 2.0 kg (b) 4.0 kg (c) 0.2 kg (d) 0.4 kg
downward acceleration of mass m1 is 41. A monkey of mass 20 kg is holding a vertical rope.
(Assume, m1 = m2 = m3 = m ) [CBSE-AIPMT 2014] The rope will not break, when a mass of 25 kg is
m2 m3 suspended from it, but will break, if the mass
P
exceeds 25 kg. What is the maximum acceleration
with which the monkey can climb up along the
rope? (Take, g = 10 m/ s2 ) [CBSE-AIPMT 2003]
(a) 25 m/s 2 (b) 2 .5 m/s 2 (c) 5 m/s 2 (d) 10 m/s 2

m1 42. A block of mass m is placed on a smooth inclined


wedge ABC of inclination q as shown in the figure.
g(1 - gm ) 2 gm
The wedge is given an acceleration a towards the
(a) (b) right. The relation between a and q for the block to
9 3
g(1 - 2 m ) g (1 - 2 m ) remain stationary on the wedge is [NEET 2018]
(c) (d)
3 2 A
m
38. The upper half of an inclined plane of inclination q
is perfectly smooth while lower half is rough. A
a
block starting from rest at the top of the plane will
again come to rest at the bottom, if the coefficient θ
of friction between the block and lower half of the C B
plane is given by [NEET 2013]
(a) a = g cos q (b) a =
g
1 2 sin q
(a) m = (b) m =
tan q tan q g
(c) a = (d) a = g tan q
(c) m = 2 tan q (d) m = tan q cosec q

MODULE 3
www.jeebooks.in
The NEET Edge ~ Physics 351

43. Two blocks A and B of masses 3m acceleration, if the kinetic energy of the particle
and m respectively are connected by a becomes equal to 8 ´ 10-4 J by the end of the second
massless and inextensible string. The revolution after the beginning of the motion?
whole system is suspended by a [NEET 2016]
massless spring as shown in figure. (a) 0.15 m/s 2 . m/s 2
(b) 018 (c) 0.2 m/s 2 (d) 0.1 m/s 2
A 3m
The magnitudes of acceleration of A
and B immediately after the string is 49. A mass m moves in a circle on a smooth horizontal
B m plane with velocity v0 at a radius R0. The mass is
cut, are respectively [NEET 2017]
g g g g attached to a string which passes through a
(a) g, (b) , g (c) g , g (d) ,
3 3 3 3 smooth hole in the plane as shown in the figure.
v0
44. A particle moves, so that its position vector is given
by r = cos wtx$ + sin wt y$ , where w is a constant. m
R0
Which of the following is true? [NEET 2016]
(a) Velocity and acceleration both are parallel to r.
(b) Velocity is perpendicular to r and acceleration is directed
towards to origin.
(c) Velocity is perpendicular to r and acceleration is directed The tension in the string is increased gradually and
R
away form the origin. finally m moves in a circle of radius 0 . The final
(d) Velocity and acceleration both are perpendicular to r. 2
value of the kinetic energy is [CBSE-AIPMT 2015]
45. A car is negotiating a curved road of radius R. The 1 1
road is banked at angle q. The coefficient of friction (a) mv 20 (b) mv 20 (c) 2 mv 20 (d) mv 20
4 2
between the tyres of the car and the road is m s . The
maximum safe velocity on this road is [NEET 2016] 50. A particle of mass m1 is moving with a velocity v1
and another particle of mass m2 is moving with a
æ m + tan q ö g æ m s + tan q ö velocity v2. Both of them have the same
(a) gR ç s ÷ (b) ç ÷
è 1 - m s tan q ø R è 1 - m s tan q ø momentum, but their different kinetic energies are
E1 and E2 respectively. If m1 > m2, then
g æ m s + tan q ö æ m + tan q ö
(c) ç ÷ (d) gR 2 ç s ÷ [CBSE-AIPMT 2004]
R 2 è 1 - m s tan q ø è 1 - m s tan q ø E m
(a) E1 < E2 (b) 1 = 1 (c) E1 > E2 (d) E1 = E2
E2 m2

5. Work, Energy and Power 51. If kinetic energy of a body is increased by 300%,
then percentage change in momentum will be
46. Two similar springs P and Q have spring constants
K P and K Q , such that K P > K Q . They are
[CBSE-AIPMT 2002]
(a) 100% (b) 150% (c) 265% (d) 73.2%
stretched, first by the same amount (case a), then
by the same force (case b). The work done by the 52. Consider a drop of rain water having mass 1 g
springs W P and WQ are related as, in case ( a ) and falling from a height of 1 km. It hits the ground
case ( b), respectively [CBSE-AIPMT 2015] with a speed of 50 m/s. Take, g constant with a
(a) WP = WQ , WP > WQ (b) WP = WQ , WP = WQ value of 10 m/s 2. The work done by the
(c) WP > WQ , WQ > WP (d) WP < WQ , WQ < WP (i) gravitational force and (ii) the resistive force of
air is [NEET 2017]
47. A force F acting on an object varies with distance x (a) (i) -10 J, (ii) - 8.25 J (b) (i) 1.25 J, (ii) - 8.25 J
as shown in the figure. The force is in newton and (c) (i) 100 J, (ii) 8.75 J (d) (i) 10 J, (ii) - 875. J
x is in metre. The work done by the force in
moving the object from x = 0 to x = 6 m is 53. A block of mass 10 kg, moving in x-direction with a
[CBSE-AIPMT 2005] constant speed of10 ms-1, is subjected to a retarding
F(N) force F = 01. x J/m during its travel from x = 20 m to
3 30 m. Its final KE will be [CBSE-AIPMT 2015]
2 (a) 475 J (b) 450 J (c) 275 J (d) 250 J
1 54. A body of mass 1 kg begins to move under the
0 x (m) action of a time dependent force F = ( 2t $i + 3t 2$j) N,
1 2 3 4 5 6 7
where $i and $j are unit vectors along X and Y axes.
(a) 4.5 J (b) 13.5 J (c) 9.0 J (d) 18.0 J What power will be developed by the force at the
time t ? [NEET 2016]
48. A particle of mass 10 g moves along a circle of
(a) (2 t 2 + 4 t 4 ) W (b) (2 t 3 + 3 t 4 ) W
radius 6.4 cm with a constant tangential 3 5
acceleration. What is the magnitude of this (c) (2 t + 3 t ) W (d) (2 t + 3 t 3 ) W

MODULE 3
www.jeebooks.in
352 NEET Test Drive

55. A particle of mass m is driven by a machine that 62. A body of mass 4m is lying in xy-plane at rest. It
delivers a constant power k watts. If the particle suddenly explodes into three pieces. Two pieces
starts from rest, the force on the particle at time t each of mass m move perpendicular to each other
is [CBSE-AIPMT 2015] with equal speeds v. The total kinetic energy
mk -1 2 -1 generated due to explosion is [CBSE-AIPMT 2014]
(a) t (b) mk t 2
3
2 (a) mv2 (b) mv2 (c) 2 mv2 (d) 4 mv2
-1 1 -1 2
(c) 2 mk t 2 (d) mk t 2
2
56. The heart of a man pumps 5 L of blood through 6. System of Particles and
the arteries per minute at a pressure of 150 mm
of mercury. If the density of mercury be Rotational Motion
13.6 ´ 103 kg/ m3 and g = 10 m/ s2, then the power 63. A light rod of length l has two masses m1 and m2
of heart in watt is [CBSE-AIPMT 2015] attached to its two ends. The moment of inertia of
(a) 1.70 (b) 2.35 (c) 3.0 (d) 1.50 the system about an axis perpendicular to the rod
and passing through the centre of mass is
57. The potential energy of a particle in a force field is [NEET 2016]
A B m + m2 2
U = - , where A and B are positive constants (a)
m1m2 2
l (b) 1 l (c) (m1 + m2 ) l 2 (d) m1m2 l 2
r2 r 1
m1 + m2 m1m2
and r is the distance of particle from the centre of
the field. For stable equilibrium, the distance of 64. The moment of inertia of a thin uniform rod of
the particle is [CBSE-AIPMT 2012] mass M and length L about an axis passing
(a) B/2A (b) 2A/B (c) A/B (d) B/A through its mid-point and perpendicular to its
length is I 0. Its moment of inertia about an axis
58. What is the minimum velocity with which a body passing through one of its ends and perpendicular
of mass m must enter a vertical loop of radius R, so to its length is [CBSE-AIPMT 2011]
that it can complete the loop? [NEET 2016]
(a) I0 + ML2 /4 (b) I0 + 2 ML2 (c) I0 + ML2 (d) I0 + ML2 /2
(a) 2gR (b) 3gR
(c) 5gR (d) gR 65. Three identical spherical shells, each of mass m
and radius r are placed as shown in the figure.
59. A stone is tied to a string of length l and is whirled
Consider an axis XX ¢, which is touching to two
in a vertical circle with the other end of the string
shells and passing through diameter of third shell.
as the centre. At a certain instant of time, the
Moment of inertia of the system consisting of these
stone is at its lowest position and has a speed u.
three spherical shells about XX ¢ axis is
The magnitude of the change in velocity as it
reaches a position where the string is horizontal
[CBSE-AIPMT 2015]
X
(g being acceleration due to gravity) is
[CBSE-AIPMT 2004]
2 2
(a) 2 (u - gl ) (b) u - gl
2
(c) u - u - 2 gl (d) 2 gl

60. A moving block having mass m, collides with


another stationary block having mass 4m. The X′
11 2 2 16 2
lighter block comes to rest after collision. When the (a) mr (b) 3 mr (c) mr (d) 4 mr 2
initial velocity of the lighter block is v, then the 5 5
value of coefficient of restitution e will be [NEET 2018] 66. From a disc of radius R and mass M, a circular
(a) 0.8 (b) 0.25 hole of diameter R, whose rim passes through the
(c) 0.5 (d) 0.4 centre is cut. What is the moment of inertia of the
61. A ball is thrown vertically downwards from a remaining part of the disc about a perpendicular
height of 20m with an initial velocity v0. It collides axis passing through the centre ? [NEET 2016]

with the ground, loses 50 % of its energy in (a) 13 MR 2 /32 (b) 11 MR 2 /32 (c) 9 MR 2 /32 (d) 15 MR 2 /32
collision and rebounds to the same height. 67. The instantaneous angular position of a point on a
The initial velocity v0 is (take, g = 10 ms-2) rotating wheel is given by the equation
[CBSE-AIPMT 2015] Q( t ) = 2 t3 - 6 t 2. The torque on the wheel becomes
(a) 14 ms - 1 (b) 20 ms - 1 zero at [CBSE-AIPMT 2011]
-1
(c) 28 ms (d) 10 ms - 1 (a) t = 0.5 s (b) t =0.25 s (c) t = 2 s (d) t = 1s

MODULE 3
www.jeebooks.in
The NEET Edge ~ Physics 353

68. An automobile moves on a road with a speed of 1 1


(a) I(w1 + w2 )2 (b) I(w1 - w2 )2
- 1 2 4
54 kmh . The radius of its wheels is 0.45 m and
I
the moment of inertia of the wheel about its axis of (c) I(w1 - w2 )2 (d) (w1 - w2 )2
rotation is 3 kg m 2 . If the vehicle is brought to rest 8
in 15 s, the magnitude of average torque 73. A circular disc of moment of inertia I t is rotating in
transmitted by its brakes to the wheel is a horizontal plane, about its symmetry axis, with a
[CBSE-AIPMT 2015] constant angular speed wi . Another disc of
(a) 6.66 kgm2 s - 2 (b) 8.58 kgm2 s - 2 moment of inertia I b is dropped coaxially onto the
2 -2
(c) 10.86 kgm s (d) 2.86 kgm2 s - 2 rotating disc. Initially the second disk has zero
angular speed. Eventually both the discs rotate
69. A uniform rod of length l and mass m is free to
rotate in a vertical plane about A. The rod initially with a constant angular speed wf . The energy lost
in horizontal position is released. The initial by initially rotating disc due to friction is
angular acceleration of the rod is (moment of inertia [CBSE-AIPMT 2010]
ml 2 1 Ib2 1 It2
of rod about A is ) (a) w2i (b) w2i
2 (It + Ib ) 2 (It + Ib )
[CBSE-AIPMT 2007]
3
1 Ib - It 1 Ib It
l (c) w2i (d) w2i
2 (It + Ib ) 2 (It + Ib )
A B
74. A solid sphere of mass m and radius R is rotating
3g 2l 3g l about its diameter. A solid cylinder of the same
(a) (b) (c) (d) mg
2l 3g 2 l2 2 mass and same radius is also rotating about its
geometrical axis with an angular speed twice that
70. A solid cylinder of mass 50 kg and radius 0.5 m is of the sphere. The ratio of their kinetic energies of
free to rotate about the horizontal axis. A massless rotation ( Esphere / Ecylinder ) will be [NEET 2016]
string is wound round the cylinder with one end (a) 2 : 3 (b) 1 : 5
attached to it and other hanging freely. Tension in (c) 1 : 4 (d) 3 : 1
the string required to produce an angular 75. A small object of uniform density rolls up a curved
acceleration of 2 rev/s2 is [CBSE-AIPMT 2014] surface with an initial velocity v ¢. It reaches upto a
(a) 25 N (b) 50 N (c) 78.5 N (d) 157 N 3v 2
maximum height of with respect to the initial
71. Point masses m1 and m2 are placed at the opposite 4g
ends of a rigid rod of length L and negligible mass. position. The object is [NEET 2013]
The rod is to be set rotating about an axis (a) ring (b) solid sphere
perpendicular to it. The position of point P on this (c) hollow sphere (d) disc
rod through which the axis should pass, so that
the work required to set the rod rotating with 76. A wheel has angular acceleration of 3 rad/s 2 and
angular velocity w0 is minimum, is given by an initial angular speed of 2 rad/s. In a time of 2s,
[CBSE-AIPMT 2015] it has rotated through an angle (in radian) of
[CBSE-AIPMT 2007]
ω0 (a) 6 (b) 10
(c) 12 (d) 4
m1 P m2 77. The ratio of the accelerations for a solid sphere
x (L – x) (mass m and radius R) rolling down an incline of
angle q without slipping and slipping down the
incline without rolling is [CBSE-AIPMT 2014]
(a) 5 : 7 (b) 2 : 3
m1L m m m2 L (c) 2 : 5 (d) 7 : 5
(a) x = (b) x = 1 L (c) x = 2 L (d) x =
m1 + m2 m2 m1 m1 + m2
78. A disc and a sphere of same radius but different
72. Two discs of same moment of inertia rotating about masses roll off on two inclined planes of the same
their regular axis passing through centre and altitude and length. Which one of the two objects
perpendicular to the plane of disc with angular gets to the bottom of the plane first? [NEET 2016]
velocities w1 and w2. They are brought into contact (a) Sphere
face to face coinciding the axis of rotation. The (b) Both reach at the same time
expression for loss of energy during this process is (c) Depends on their masses
[NEET 2017] (d) Disc

MODULE 3
www.jeebooks.in
354 NEET Test Drive

79. Three objects, A (a solid sphere), B (a thin circular If t1 is the time for the planet to move from C to D and
disc) and C (a circular ring), each have the same t2 is the time to move from A to B, then
mass M and radius R. They all spin with the same [CBSE-AIPMT 2009]
angular speed w about their own symmetrical axes. m v
C
The amounts of work W required to bring them to B
rest, would satisfy the relation [NEET 2018]
A S
(a) WB > WA > WC (b) WA > WB > WC
D
(c) WC > WB > WA (d) WA > WC > WB
(a) t 1 > t 2 (b) t 1 = 4 t 2
(c) t 1 = 2 t 2 (d) t 1 = t 2
7. Gravitation 85. The density of newly discovered planet is twice
80. The kinetic energies of a planet in an elliptical that of the earth. The acceleration due to gravity at
orbit about the sun, at positions A, B and C are the surface of the planet is equal to that at the
K A , K B and K C , respectively. AC is the major axis surface of the earth. If the radius of the earth is R,
and SB is perpendicular to AC at the position of the radius of the planet would be [CBSE-AIPMT 2004]
the sun S as shown in the figure. Then, [NEET 2018] (a) 2 R (b) 4R
1 1
B (c) R (d) R
4 2

A C 86. The acceleration due to gravity at a height 1 km


S above the earth is the same as at a depth d below
the surface of earth. Then [NEET 2017]
(a) K B < K A < KC (b) K A > K B > KC (a) d =
1
km (b) d = 1km (c) d =
3
km (d) d = 2 km
(c) K A < K B < KC (d) K B > K A > KC 2 2
81. If the mass of the sun were ten times smaller and 87. Starting from the centre of the earth having radius
the universal gravitational constant were ten times R, the variation of g (acceleration due to gravity) is
larger in magnitude, which of the following is not shown by [NEET 2016]
correct? [NEET 2018]
g
(a) Time period of a simple pendulum on the earth would g
decrease (a) (b)
(b) Walking on the ground would become more difficult r
O R
(c) Raindrops will fall faster O R r
(d) g on the earth will not change
g g
82. Kepler’s third law states that square of period of
revolution (T ) of a planet around the sun is (c) (d)
proportional to third power of average distance r
O O R r
between the sun and planet i.e. T 2 = Kr 3 , here K is R r

constant. If the masses of the sun and planet are 88. Dependence of intensity of gravitational field ( E ) of
M and m respectively, then as per Newton’s law of the earth with distance (r ) from centre of the earth
gravitation force of attraction between them is is correctly represented by [CBSE-AIPMT 2014]
GMm
F= , here G is gravitational constant.
r2 E
R
E
The relation between G and K is described as (a) O (b) O
r R r
[CBSE-AIPMT 2015]
I
(a) GK = 4p 2 (b) GMK = 4p 2 (c) K = G (d) K =
G
E
83. A geostationary satellite is orbiting the earth at a R E
height of 5 R above the surface of the earth, R (c) O (d) O
r R r
being the radius of the earth. The time period of
another satellite (in hour) at a height of 2 R from
the surface of the earth is [CBSE-AIPMT 2012] 89. At what height from the surface of earth, the
(a) 5 (b) 10 (c) 6 2 (d) 6/ 2 gravitation potential and the value of g are
-5.4 ´ 107 J kg -2 and 6.0 ms -2 respectively?
84. The figure shows elliptical orbit of a planet m about
the sun S. The shaded area SCD is twice the shaded (Take, the radius of earth as 6400 km) [NEET 2016]
area SAB. (a) 1600 km (b)1400 km (c) 2000 km (d) 2600 km

MODULE 3
www.jeebooks.in
The NEET Edge ~ Physics 355

90. Infinite number of bodies, each of mass 2 kg are 97. Two wires are made of the same material and have
situated on X-axis at distances 1m, 2 m, 4 m and the same volume. The first wire has cross-sectional
8 m, respectively from the origin. The resulting area A and the second wire has cross-sectional area
gravitational potential due to this system at the 3A. If the length of the first wire is increased by Dl
origin will be [NEET 2013] on applying a force F, how much force is needed to
8 4 stretch the second wire by the same amount?
(a) - G (b) - G (c) - G (d) - 4G
3 3 [NEET 2018]
(a) 4F (b) 6F (c) 9F (d) F
91. A body projected vertically from the earth reaches
a height equal to earth’s radius before returning to 98. The bulk modulus of a spherical object is B. If it is
the earth. The power exerted by the gravitational subjected to uniform pressure p, the fractional
force is greatest [CBSE-AIPMT 2011] decrease in radius is [NEET 2017]
(a) at the instant just before the body hits the earth p B 3p p
(a) (b) (c) (d)
(b) it remains constant all through B 3p B 3B
(c) at the instant just after the body is projected
(d) at the highest position of the body
99. Coefficient of linear expansion of brass and steel
rods are a 1 and a 2. Lengths of brass and steel rods
92. The ratio of escape velocity at earth ( ve ) to the are l1 and l2 respectively. If ( l2 - l1 ) is maintained
escape velocity at a planet (v p) whose radius and same at all temperatures, which one of the
mean density are twice as that of earth is following relations holds good? [NEET 2016]
[NEET 2016] (a) a1l22 = a2 l12 (b) a12 l2 = a22 l1
(a) 1 : 2 2 (b) 1: 4 (c) 1 : 2 (d) 1: 2 (c) a1l1 = a2 l2 (d) a1l2 = a2 l1
93. A black hole is an object whose gravitational field
is so strong that even light cannot escape from it.
To what approximate radius would earth 9. Mechanical Properties of Fluids
(mass = 5.98 ´ 1024 kg) have to be compressed to be
100. A small sphere of radius r falls from rest in a
a black hole? [CBSE-AIPMT 2014]
viscous liquid. As a result, heat is produced due to
(a) 10-9 m (b) 10-6 m viscous force. The rate of production of heat when
(c) 10-2 m (d) 100 m the sphere attains its terminal velocity, is
94. The earth is assumed to be a sphere of radius R. A proportional to [NEET 2018]
platform is arranged at a height R from the surface (a) r 5 (b) r 2 (c) r 3 (d) r 4
of the earth. The escape velocity of a body from 101. The approximate depth of an ocean is 2700 m. The
this platform is fv e , where ve is its escape velocity compressibility of water is 45.4 ´ 10-11 Pa -1 and
from the surface of the earth. The value of f is
[CBSE-AIPMT 2006] density of water is 103 kg/ m3 . What fractional
1 1 1 compression of water will be obtained at the
(a) 2 (b) (c) (d)
2 3 2 bottom of the ocean? [CBSE-AIPMT 2015]
(a) 0.8 ´ 10-2 (b) 1.0 ´ 10-2 (c) 1.2 ´ 10-2 (d) 1.4 ´ 10-2

8. Mechanical Properties of Solid 102. A U-tube with both ends open to the atmosphere,
is partially filled with water. Oil, which is
95. Copper of fixed volume V is drawn into wire of immiscible with water, is poured into one side until
length l. When this wire is subjected to a constant it stands at a distance of 10 mm above the water
force F, the extension produced in the wire is Dl. level on the other side. Meanwhile the water rises
Which of the following graphs is a straight line? by 65 mm from its original level (see diagram). The
density of the oil is
[CBSE-AIPMT 2014]
1
[NEET 2017]
(a) D l versus (b) D l versus l 2 Pa Pa
l
1 F
A
(c) D l versus (d) D l versus l 10 mm
l2 E Final water level
65 mm
96. The following four wires are made of same material. Oil D Initial water level
Which of these will have the largest extension when 65 mm
the same tension is applied? [NEET 2013]
B C
(a) Length = 50 cm, diameter = 0.5 mm
Water
(b) Length = 100 cm, diameter = 1mm
(c) Length = 200 cm, diameter = 2 mm
(a) 650 kg m-3 (b) 425 kg m-3
(d) Length = 300 cm, diameter = 3 mm
(c) 800 k g m-3 (d) 928 kg m-3

MODULE 3
www.jeebooks.in
356 NEET Test Drive

103. Two non-mixing liquids of densities r and nr(n > 1)


are put in a container. The height of each liquid is 10. Thermal Properties of Matter
h. A solid cylinder of length L and density d is put 109. On a new scale of temperature (which is linear)
in this container. The cylinder floats with its axis and called the W scale, the freezing and boiling
vertical and length pL (p < 1) in the denser liquid. points of water are 39°W and 239°W respectively.
The density d is equal to [NEET 2016] What will be the temperature on the new scale,
(a) {2 + (n + 1)p}r (b) {2 + (n - 1)p}r corresponding to a temperature of 39°C on the
(c) {1 + (n - 1)p}r (d) {1 + (n + 1)p}r celsius scale ? [CBSE-AIPMT 2008]
104. The cylindrical tube of a spray pump has radius R, (a) 78°W (b) 117°W (c) 200°W (d) 139°W
one end of which has n fine holes, each of radius r. 110. The value of coefficient of volume expansion of
If the speed of the liquid in the tube is v, the speed glycerin is 5 ´ 10- 4 K - 1. The fractional change in
of the ejection of the liquid through the holes is
[CBSE-AIPMT 2015]
the density of glycerin for a rise of 40°C in its
temperature is
nR 2 nR 2 nR 2 n2 R
[CBSE-AIPMT 2015]
(a) 2 2
(b) 2
(c) 3 2
(d) (a) 0.015 (b) 0.020 (c) 0.025 (d) 0.010
n r nr n r nr
111. Two identical bodies are made of a material for
105. A wind with speed 40 m/s blows parallel to the roof which the heat capacity increases with
2
of a house. The area of the roof is 250 m . temperature. One of these is at 100°C, while the
Assuming that the pressure inside the house is other one is at 0°C. If the two bodies are brought
atmospheric pressure, the force exerted by the into contact, then assuming no heat loss, the final
wind on the roof and the direction of the force will common temperature is [NEET 2016]
be ( p air = 1.2 kg /m 3 ) [CBSE-AIPMT 2015] (a) 50°C
(a) 4.8 ´ 105 N, downwards (b) more than 50°C
(c) less than 50°C but greater than 0°C
(b) 4.8 ´ 105 N, upwards
(d) 0°C
(c) 2.4 ´ 105 N, upwards
112. A piece of ice falls from a height h, so that it melts
(d) 2.4 ´ 105 N, downwards completely. Only one-quarter of the heat produced
106. Three liquids of densities r1 , r 2 and r3 (with is absorbed by the ice and all energy of ice gets
r1 > r 2 > r3 ), having the same value of surface converted into heat during its fall. The value of h is
tension T , rise to the same height in three identical [latent heat of ice is 3.4 ´ 105 J/kg and g = 10 N/ kg]
capillaries. The angles of contact q 1 , q 2 and q 3 obey [NEET 2016]
[NEET 2016] (a) 544 km (b) 136 km (c) 68 km (d) 34 km
p p
(a) > q1 > q2 > q3 ³ 0 (b) 0 £ q1 < q2 < q3 < 113. Steam at 100°C is passed into 20 g of water at
2 2 10°C. When water acquires a temperature of 80°C,
p p
(c) < q1 < q2 < q3 < p (d) p > q1 > q2 > q3 > the mass of water present will be [take, specific
2 2
heat of water = 1 cal g-1° C -1 and latent heat of
107. A rectangular film of liquid is extended from steam = 540 cal g -1 ] [CBSE-AIPMT 2014]
(4 cm ´ 2 cm) to ( 5 cm ´ 4 cm ). If the work done is
(a) 24 g (b) 31.5 g (c) 42.5 g (d) 22.5 g
3 ´ 10-4 J, the value of the surface tension of the
liquid is [NEET 2016] 114. Liquid oxygen at 50 K is heated to 300 K at
(a) 0.250 Nm -1 (b) 0.125 Nm -1 constant pressure of 1 atm. The rate of heating is
(c) 0.2 Nm -1 (d) 8.0 Nm -1 constant. Which one of the following graphs
represents the variation of temperature with time?
108. A certain number of spherical drops of a liquid of [CBSE-AIPMT 2012]
radius r coalesce to form a single drop of radius R
Temperature

and volume V . If T is the surface tension of the


Temperature

liquid, then [CBSE-AIPMT 2014]


(a) (b)
(a) energy = 4VT æç - ö÷ is released
1 1
èr Rø
Time Time
(b) energy = 3VT æç + ö÷ is absorbed
1 1
èr Rø
Temperature
Temperature

(c) energy = 3VT æç - ö÷ is released


1 1
èr Rø (c) (d)
(d) energy is neither released nor absorbed
Time Time

MODULE 3
www.jeebooks.in
The NEET Edge ~ Physics 357

115. A body cools from a temperature 3T to 2T in 10 122. A black body is at a temperature of 5760 K. The
minutes. The room temperature is T . Assume that energy of radiation emitted by the body at
Newton’s law of cooling is applicable. The wavelength 250 nm is U 1, at wavelength 500 nm is
temperature of the body at the end of next U 2 and that at 1000 nm is U 3 . Wien’s constant,
10 minutes will be [NEET 2016] b = 2.88 ´ 106 nmK. Which of the following is
7 3 4 correct?
(a) T (b) T (c) T (d) T [NEET 2016]
4 2 3 (a) U 3 = 0 (b) U1 > U 2
116. Certain quantity of water cools from 70°C to 60°C (c) U 2 > U1 (d) U1 = 0
in the first 5 min and to 54°C in the next 5 min.
The temperature of the surroundings is 123. A black body at 227°C radiates heat at the rate
[CBSE-AIPMT 2014]
of 7 cal cm -2s -1. At a temperature of 727°C, the
rate of heat radiated in the same units will be
(a) 45°C (b) 20°C (c) 42°C (d) 10°C
(a) 60 cal-cm 2 s -1 [CBSE-AIPMT 2009]
117. A cylindrical metallic rod in thermal contact with (b) 50 cal-cm s 2 -1
two reservoirs of heat at its two ends conducts an
(c) 112 cal-cm 2 s -1
amount of heat Q in time t. The metallic rod is
melted and the material is formed into a rod of half (d) 80 cal-cm 2 s -1
the radius of the original rod. What is the amount 124. Assuming the sun to have a spherical outer surface
of heat conducted by the new rod when placed in of radius r, radiating like a black body at
thermal contact with the two reservoirs in time t? temperature t° C, the power received by a unit
[CBSE-AIPMT 2010] surface, (normal to the incident rays) at a distance
(a) Q/4 (b) Q/16 (c) 2Q (d) Q/2 R from the centre of the sun is (where, s is the
118. Which of the following circular rods, (given radius Stefan’s constant) [CBSE-AIPMT 2007]
r and length l) each made of the same material and 4 pr 2t 4 r 2 s(t + 273)4
(a) 2
(b)
whose ends are maintained at the same R 4p R 2
temperature will conduct most heat? 2 2
16p r st 4
r s(t + 273)4
2
(c) 2
(d)
R2
[CBSE-AIPMT 2005]
R
(a) r = 2 r0 , l = 2 l0 (b) r = 2 r0 , l = l0
(c) r = r0 , l = l0 (d) r = r0 , l = 2 l0
119. Consider a compound slab consisting of two 11. Thermodynamics
different materials having equal thickneses and 125. A sample of 0.1g of water at 100° C and normal
thermal conductivities K and 2 K , respectively. The pressure (1.013 ´ 105 Nm -2 ) requires 54 cal of heat
equivalent thermal conductivity of the slab is energy to convert to steam at 100° C. If the volume
[CBSE-AIPMT 2003] of the steam produced is 167.1 cc, the change in
4 2 internal energy of the sample, is
(a) 3K (b) K (c) K (d) 2 K [NEET 2018]
3 3 (a) 42.2 J
120. Consider two rods of same length and different (b) 208.7 J
specific heats ( s1 , s2 ), thermal conductivities (c) 104.3 J
( K 1 , K 2 ) and areas of cross-section ( A1 , A2 ) and both (d) 84.5 J
having temperatures (T1 , T2) at their ends. If their
rate of loss of heat due to conduction are equal, 126. During an isothermal expansion, a confined ideal
then [CBSE-AIPMT 2002]
gas does 150 J of work against its surroundings.
K1 A1 K 2 A2 This implies that [CBSE-AIPMT 2011]
(a) K1 A1 = K 2 A2 (b) =
s1 s2 (a) 300 J of heat has been added to the gas
K 2 A1 K1 A2 (b) no heat is transferred because the process is isothermal
(c) K 2 A1 = K1 A2 (d) = (c) 150 J of heat has been added to the gas
s2 s1
(d) 150 J of heat has been removed from the gas
121. The power radiated by a black body is P and it 127. If DU and DW represent the increase in internal
radiates maximum energy at wavelength l 0. If the energy and work done by the system respectively
temperature of the black body is now changed, so
3 in a thermodynamical process, which of the
that it radiates maximum energy at wavelength following is true? [CBSE-AIPMT 2010]
4
l 0, the power radiated by it becomes nP. The (a) DU = - DW , in an adiabatic process
value of n is [NEET 2018] (b) DU = DW , in an isothermal process
256 4 3 81 (c) DU = DW , in an adiabatic process
(a) (b) (c) (d)
81 3 4 256 (d) DU = - DW , in an isothermal process

MODULE 3
www.jeebooks.in
358 NEET Test Drive

128. The volume (V ) of a monoatomic gas varies with its 132. Figure below shows two paths that may be taken
temperature (T ), as shown in the graph. The ratio by a gas to go from a state A to a state C.
of work done by the gas, to the heat absorbed by it, [CBSE-AIPMT 2015]
when it undergoes a change from state A to state
p
B, is B C
6×104Pa
[NEET 2018]
V

A
B 2×104Pa
A
2×10–3m3 4×10–3m3
O T V
1 2 2 2
(a) (b) (c) (d) In process AB, 400 J of heat is added to the system
3 3 5 7
and in process BC, 100 J of heat is added to the
129. Thermodynamic processes are indicated in the system. The heat absorbed by the system in the
following diagram [NEET 2017]
process AC will be
p (a) 380 J (b) 500 J
(c) 460 J (d) 300 J
IV
f 133. One mole of an ideal diatomic gas undergoes a
I III transition from A to B along a path AB as shown in
II f the figure. [CBSE-AIPMT 2015]
f 700 K
f 500 K
300 K
V A
Match the following : 5

Column-I Column-II p (in kPa) 2 B


P. Process I a. Adiabatic
Q. Process II b. Isobaric 4 6
V(in m3)
R. Process III c. Isochoric
S. Process IV d. Isothermal The change in internal energy of the gas during the
transition is
(a) P ® a, Q ® c, R ® d, S ® b
(a) 20 kJ (b) - 20 kJ
(b) P ® c, Q ® a, R ® d, S ® b
(c) 20 J (d) - 12 kJ
(c) P ® c, Q ® d, R ® b, S ® a
(d) P ® d, Q ® b, R ® a, S ® c 134. A monoatomic gas at a pressure p, having a
volume V expands isothermally to a volume 2V and
130. A gas is compressed isothermally to half of its then adiabatically to a volume 16 V. The final
initial volume. The same gas is compressed æ 5ö
separately through an adiabatic process until its pressure of the gas is ç Take, g = ÷
è 3 ø [CBSE-AIPMT 2014]
volume is again reduced to half. Then [NEET 2016]
p
(a) compressing the gas through adiabatic process will (a) 64p (b) 32 p (c) (d) 16p
require more work to be done 64
(b) compressing the gas isothermally or adiabatically will 135. During an adiabatic process, the pressure of a gas
require the same amount of work is found to be proportional to the cube of its
(c) which of the case (whether compression through Cp
isothermal or through adiabatic process) requires more temperature. The ratio of for the gas is
work will depend upon the atomicity of the gas CV [NEET 2013]
(d) compressing the gas isothermally will require more work to 4 5 3
(a) (b) 2 (c) (d)
be done 3 3 2
131. One mole of an ideal monatomic gas undergoes a 136. One mole of an ideal gas goes from an initial state
process described by the equation pV 3 = constant. A to final state B via two processes. It first
The heat capacity of the gas during this process is undergoes isothermal expansion from volume V
[NEET 2016] to 3V and then its volume is reduced from 3V to
3 5 V at constant pressure. The correct p-V
(a) R (b) R (c) 2R (d) R
2 2 diagram representing the two processes is
[CBSE-AIPMT 2012]

MODULE 3
www.jeebooks.in
The NEET Edge ~ Physics 359

B A 143. Two vessels separately contain two ideal gases A


p p and B at the same temperature, the pressure of A
(a) A (b) B being twice that of B. Under such conditions, the
density of A is found to be 1.5 times the density of
V 3V V 3V B. The ratio of molecular weight of A and B is
V V [CBSE-AIPMT 2015]
2 3 1
A A (a) (b) (c) 2 (d)
3 4 2
p p
(c) B (d) B 144. At what temperature will the rms speed of oxygen
molecules become just sufficient for escaping from
V 3V V 3V the Earth’s atmosphere?
V V (Given, mass of oxygen molecule, m = 2.76 ´ 10-26 kg,
137. One mole of an ideal gas at an initial temperature Boltzmann’s constant kB = 1.38 ´ 10-23 JK -1)
of T K does 6 R joules of work adiabatically. If the (a) 5.016 ´ 104 K (b)8.326 ´ 104 K
ratio of specific heats of this gas at constant 4
(c) 2.508 ´ 10 K (d) 1.254 ´ 104 K
pressure and at constant volume is 5/3, the final
temperature of gas will be [CBSE-AIPMT 2004] 145. A gas mixture consists of 2 moles of O2 and
(a) ( T + 2 .4) K (b) ( T - 2 .4) K 4 moles of Ar at temperature T. Neglecting all
(c) ( T + 4) K (d) ( T - 4) K vibrational modes, the total internal energy of the
system is
138. The efficiency of an ideal heat engine working
[NEET 2017]

between the freezing point and boiling point of (a) 4RT (b) 15RT
(c) 9RT (d) 11RT
water, is [NEET 2018]
(a) 6.25% (b) 20% (c) 26.8% (d) 12.5% 146. The amount of heat energy required to raise the
temperature of 1 g of helium at NTP from T1K to
139. A Carnot engine having an efficiency of 1 as heat
T2K is [NEET 2013]
10 3 3
engine is used as a refrigerator. If the work done (a) Na K B (T2 - T1 ) (b) Na K B (T2 - T1 )
8 2
on the system is 10 J, the amount of energy æT ö
3 3
absorbed from the reservoir at lower temperature (c) Na K B (T2 - T1 ) (d) Na K B ç 2 ÷
is [NEET 2017]
4 4 è T1 ø
(a) 1 J (b) 90 J (c) 99 J (d) 100 J 147. 4.0 g of a gas occupies 22.4 L at NTP. The specific
140. A refrigerator works between 4°C and 30°C. It is heat capacity of the gas at constant volume is
required to remove 600 calories of heat every 5.0 JK - 1 mol- 1. If the speed of sound in this gas at
second in order to keep the temperature of the NTP is 952 ms- 1, then the heat capacity at
refrigerated space constant. The power required is constant pressure is (Take, gas constant,
(Take, 1 cal = 4.2 joules) [NEET 2016] R = 8.3 JK - 1 mol- 1) [CBSE-AIPMT 2015]
(a) 23.65 W (b) 236.5 W (a) 8.0 JK - 1 mol - 1 (b) 7.5 JK - 1 mol - 1
(c) 2365 W (d) 2.365 W (c) 7.0 JK - 1 mol - 1 (d) 8.5 JK - 1 mol - 1
141. A Carnot engine whose sink is at 300 K has an 148. At 10°C, the value of the density of a fixed mass of
efficiency of 40%. By how much should the an ideal gas divided by its pressure is x. At 110°C,
temperature of source be increased so as to this ratio is [CBSE-AIPMT 2008]
increase its efficiency by 50% of original efficiency? 383 10 283
(a) x (b) x (c) x (d) x
(a) 275 K (b) 325 K [CBSE-AIPMT 2006] 283 110 383
(c) 250 K (d) 380 K

13. Oscillations
12. Kinetic Theory of Gases 149. A particle executes linear simple harmonic motion
142. A given sample of an ideal gas occupies a volume V with an amplitude of 3 cm. When the particle is at
at a pressure p and absolute temperature T . The 2 cm from the mean position, the magnitude of its
mass of each molecule of the gas is m. Which of the velocity is equal to that of its acceleration. Then,
following gives the density of the gas? [NEET 2016] its time period (in seconds) is [NEET 2017]
(a) p / (kT ) (b) pm / (kT ) 5 5 4p 2p
(a) (b) (c) (d)
(c) p / (kTV ) (d) mkT p 2p 5 3

MODULE 3
www.jeebooks.in
360 NEET Test Drive

150. A particle is executing SHM along a straight line. 155. Which one of the following equations of motion
Its velocities at distances x1 and x2 from the mean represents simple harmonic motion ?
position are v1 and v2, respectively. Its time period (a) Acceleration = - k0 x + k1 x2 [CBSE-AIPMT 2009]
is
(b) Acceleration = - k( x + a)
[CBSE-AIPMT 2015]
x12 + x22 x22 - x12 (c) Acceleration = k( x + a)
(a) 2 p (b) 2 p
v12 + v 22 v12 - v 22 (d) Acceleration = kx
(where, k, k0 , k1 and a are all positive)
v12 + v 22 v12 - v 22
(c) 2 p (d) 2 p 156. A point performs simple harmonic oscillation of
x12 + x22 x12 - x22
period T and the equation of motion is given by
151. A particle is executing a simple harmonic motion. æ pö
x = a sin ç w t + ÷. After the elapse of what fraction
Its maximum acceleration is a and maximum è 6ø
velocity is b. Then, its time period of vibration of the time period, the velocity of the point will be
will be [CBSE-AIPMT 2015] equal to half of its maximum velocity ?
b2 a b2 2pb
(a) (b) (c) (d) [CBSE-AIPMT 2008]
a 2
b a a (a)
T
(b)
T
(c)
T
(d)
T
8 6 3 12
152. The oscillation of a body on a smooth horizontal
surface is represented by the equation, x = A cos wt 157. A body of mass m is attached to the lower end of a
where, x = displacement at time t and w = frequency spring whose upper end is fixed. The spring has
of oscillation negligible mass. When, the mass m is slightly
pulled down and released, it oscillates with a time
Which one of the following graphs shows correctly period of 3 s. When the mass m is increased by
the variation a with t ? [CBSE-AIPMT 2014] 1 kg, the time period of oscillations becomes 5 s.
The value of m (in kg) is [NEET 2016]
a a 3 4
(a) (b)
(a) (b) 4 3
O T t O T t 16 9
(c) (d)
9 16
158. A mass is suspended separately by two springs of
spring constants k1 and k2 in successive order. The
a a time periods of oscillations in the two cases are T1
(c) O T (d) and T2 respectively. If the same mass be suspended
t O T t
by connecting the two springs in parallel (as shown
in figure), then the time period of oscillations is T.
Here, a = acceleration at time t and T = time period. The correct relation is [CBSE-AIPMT 2002]

153. Out of the following functions representing motion


of a particle which represents SHM?
[CBSE-AIPMT 2011] k1 k2
3
I. y = sin wt - cos wt II. y = sin wt
æ 3p ö
III. y = 5 cos ç - 3 wt ÷ IV. y = 1 + wt + w2t 2
è 4 ø
(a) Only (IV) does not represent SHM (a) T 2 = T12 + T22 (b) T -2 = T1-2 + T2-2
(b) (I) and (III) -1
(c) T = T1-1 + T2-1 (d) T = T1 + T2
(c) (I) and (II)
(d) Only (I) 159. A rectangular block of mass m and area of
154. The displacement of a particle along the X-axis is cross-section A floats in a liquid of density r. If it is
given by x = a sin2 wt. The motion of the particle given a small vertical displacement from
corresponds to [CBSE-AIPMT 2010]
equilibrium, it undergoes oscillation with a time
period T. Then,
(a) simple harmonic motion of frequency w / p
[CBSE-AIPMT 2006]
1
(b) simple harmonic motion of frequency 3 w /2 p (a) T µ r (b) T µ
A
(c) non-simple harmonic motion 1 1
(d) simple harmonic motion of frequency w /2 p (c) T µ (d) T µ
r m

MODULE 3
www.jeebooks.in
The NEET Edge ~ Physics 361

160. When a damped harmonic oscillator completes 100 166. A point source emits sound equally in all directions
1 in a non-absorbing medium. Two points P and Q
oscillations, its amplitude is reduced to of its
3 are at distance of 2m and 3m respectively from the
initial value. What will be its amplitude, when it source. The ratio of the intensities of the waves at
completes 200 oscillations? [CBSE-AIPMT 2002] P and Q is [CBSE-AIPMT 2005]
1 2 1 1 (a) 9 : 4 (b) 2 : 3 (c) 3 : 2 (d) 4 : 9
(a) (b) (c) (d)
5 3 6 9 167. If n1, n 2 and n3 are the fundamental frequencies of
three segments into which a string is divided, then
14. Waves the original fundamental frequency n of the string
is given by [CBSE-AIPMT 2014]
161. A wave of amplitude 0 . 2 m, velocity 360 m / s and 1 1 1 1 1 1 1 1
(a) = + + (b) = + +
wavelength 60 m is travelling along positive n n1 n2 n3 n n1 n2 n3
X-axis, then the correct expression for the wave is
(c) n = n1 + n2 + n3 (d) n = n1 + n2 + n3
(a) y = 0.2 sin 2 p æç 6 t +

÷
è 60 ø [CBSE-AIPMT 2002] 168. The fundamental frequency in an open organ pipe
is equal to the third harmonic of a closed organ
(b) y = 0.2 sin p æç 6 t +

÷ pipe. If the length of the closed organ pipe is 20 cm,
è 60 ø
the length of the open organ pipe is
(c) y = 0.2 sin 2 p æç 6 t -
xö [NEET 2018]
÷ (a) 12.5 cm (b) 8 cm (c) 13.3 cm (d) 16 cm
è 60 ø
(d) y = 0.2 sin p æç 6 t -
xö 169. The two nearest harmonics of a tube closed at one
÷
è 60 ø end and open at other end are 220 Hz and 260 Hz.
What is the fundamental frequency of the system?
162. An uniform rope of length L and mass m1 hangs [NEET 2017]
vertically from a rigid support. A block of mass m2
(a) 10 Hz (b) 20 Hz (c) 30 Hz (d) 40 Hz
is attached to the free end of the rope. A transverse
pulse of wavelength l1 is produced at the lower end 170. The second overtone of an open organ pipe has the
of the rope. The wavelength of the pulse when it same frequency as the first overtone of a closed
reaches the top of the rope is l 2. The ratio l 2 / l1 is pipe L metre long. The length of the open pipe will
[NEET 2016] be [NEET 2016]
m1 + m2 m2 (a) L (b) 2L (c) L/2 (d) 4L
(a) (b)
m2 m1 171. Three sound waves of equal amplitudes have
m1 + m2 m1 frequencies ( n - 1), n and ( n + 1). They
(c) (d) superimpose to give beats. The number of beats
m1 m2
produced per second will be [NEET 2016]
163. An siren emitting a sound of frequency 800 Hz (a) 1 (b) 4 (c) 3 (d) 2
moves away from an observer towards a cliff at a 172. A source of unknown frequency gives 4 beat/s,
speed of 15ms -1. Then, the frequency of sound that when sounded with a source of known frequency
the observer hears in the echo reflected from the 250 Hz. The second harmonic of the source of
cliff is (take, velocity of sound in air = 330 ms -1) unknown frequency gives 5 beat/s, when sounded
[NEET 2016] with a source of frequency 513 Hz. The unknown
(a) 800 Hz (b) 838 Hz (c) 885 Hz (d) 765 Hz frequency is [NEET 2013]

164. Sound waves travel at 350 m/s through a warm air (a) 254 Hz (b) 246 Hz (c) 240 Hz (d) 260 Hz
and at 3500 m/s through brass. The wavelength of 173. A source of sound S emitting waves of frequency
a 700 Hz acoustic wave as it enters brass from 100 Hz and an observer O are located at some
warm air [CBSE-AIPMT 2011] distance from each other. The source is moving
(a) increases by a factor 20 with a speed of 19.4 ms - 1at an angle of 60° with the
(b) increases by a factor 10 source observer line as shown in the figure. The
(c) decreases by a factor 20 observer is at rest. The apparent frequency
(d) decreases by a factor 10 observed by the observer (velocity of sound in air is
330 ms - 1), is
165. The time of reverberation of a room A is 1s. What vs
[CBSE-AIPMT 2015]
will be the time (in second) of reverberation of a
room, having all the dimensions double of those of
room A? [CBSE-AIPMT 2006]
(a) 2 (b) 4 60°
1 S O
(c) (d) 1
2 (a) 100 Hz (b) 103 Hz (c) 106 Hz (d) 97 Hz

MODULE 3
www.jeebooks.in
362 NEET Test Drive

174. A speeding motorcyclist sees traffic jam ahead of 179. Three point charges + q , - 2q and + q are placed at
him. He slows down to 36 km/h. He finds that points ( x = 0, y = a , z = 0),( x = 0, y = 0, z = 0) and
traffic has eased and a car moving ahead of him at ( x = a , y = 0, z = 0), respectively. The magnitude
18 km/h is honking at a frequency of 1392 Hz. If and direction of the electric dipole moment vector
the speed of sound is 343 m/s, the frequency of the of this charge assembly are [CBSE-AIPMT 2007]
honk as heard by him will be [CBSE-AIPMT 2014] (a) 2 qa along + y direction
(a) 1332 Hz (b) 1372 Hz (b) 2 qa along the line joining points ( x = 0, y = 0, z = 0) and
(c) 1412 Hz (d) 1454 Hz ( x = a, y = a, z = 0)
(c) qa along the line joining points ( x = 0, y = 0, z = 0) and
175. The driver of a car travelling with speed 30 ms -1,
( x = a, y = a, z = 0)
towards a hill sounds a horn of frequency 600 Hz. (d) 2 qa along + x direction
If the velocity of sound in air is 330 ms -1,
180. A square surface of side L metre is in the plane of
the frequency of reflected sound as heard by
the paper. A uniform electric field E (V/m), also in
driver is [CBSE-AIPMT 2009]
the plane of the paper, is limited only to the lower
(a) 550 Hz (b) 555.5 Hz half of the square surface, (see figure). The electric
(c) 720 Hz (d) 500 Hz flux in SI units associated with the surface is
176. A whistle revolves in a circle with angular velocity [CBSE-AIPMT 2006]
w = 20 rad/s, using a string of length 50 cm. If the
actual frequency of sound from the whistle is
385Hz, then the minimum frequency heard by the
observer far away from the centre is (velocity of
E
sound, v = 340 m/s) [CBSE-AIPMT 2002]
(a) 385 Hz (b) 374 Hz
(c) 394 Hz (d) 333 Hz
EL2 EL2
(a) (b) (c) zero (d) EL2
15. Electric Charges and Fields (2 e0 ) 2

177. Suppose the charge of a proton and an electron 181. A hollow cylinder has a charge q coulomb within it.
differ slightly. One of them is -e and the other is If f is the electric flux in unit of voltmeter
( e + De). If the net of electrostatic force and associated with the curved surface B, the flux
gravitational force between two hydrogen atoms linked with the plane surface A in unit of
placed at a distance d (much greater than atomic voltmeter will be [CBSE-AIPMT 2007]
size) apart is zero, then De is of the order [given, B
mass of hydrogen, mh = 1.67 ´ 10-27 kg] [NEET 2017]
C A
(a) 10-20 C (b) 10-23 C
-37 -47
(c) 10 C (d) 10 C
1 æq ö q f q
178. Two pith balls carrying equal charges are (a) ç - f÷ (b) (c) (d) -f
suspended from a common point by strings of equal 2 è e0 ø 2 e0 3 e0
length, the equilibrium separation between them is 182. What is the flux through a cube of side a, if a point
r. Now, the strings are rigidly clamped at half the charge of q is at one of its corner? [CBSE-AIPMT 2012]
height. The equilibrium separation between the 2q q q q
balls now becomes (a) (b) (c) (d) 6a2
e0 8 e0 e0 2 e0
[NEET 2013]

16. Electrostatic Potential and


y
Capacitance
y/2
183. Three concentric spherical shells have radii a, b
r r
and c ( a < b < c) and have surface charge densities
2 s , - s and s respectively. If V A , V B and VC denote
(a) æç ö÷ (b) æç ö÷
1 r
the potentials of the three shells, then for c = a + b,
è 2ø è 2ø
we have [CBSE-AIPMT 2009]
(c) æç ö÷ (d) æç ö÷
2r 2r (a) VC = VA ¹ VB (b) VC = VB ¹ VA
è 3ø è3ø (c) VC ¹ VB ¹ VA (d) VC = VB = VA

MODULE 3
www.jeebooks.in
The NEET Edge ~ Physics 363

184. In a region, the potential is represented by 188. A parallel plate air capacitor is charged to a
V ( x , y , z ) = 6x - 8xy - 8 y + 6 yz, where V is in volts potential difference of V volts. After disconnecting
and x, y, z are in metres. The electric force the charging battery the distance between the
experienced by a charge of 2 C situated at point plates of the capacitor is increased using an
(1, 1, 1) is [CBSE-AIPMT 2014] insulating handle. As a result the potential
(a) 6 5 N (b) 30 N difference between the plates [CBSE-AIPMT 2006]
(c) 24 N (d) 4 35 N (a) decreases (b) does not change
(c) becomes zero (d) increases
185. The electric potential at a point ( x , y , z ) is given by
V = - x 2 y - xz3 + 4. The electric field E at that 189. A parallel plate capacitor of area A, plate
point is [CBSE-AIPMT 2009] separation d and capacitance C is filled with four
(a) E = (2 xy + z )i$ + x2 $j + 3 xz2 k$
3 dielectric materials having dielectric constants
k1 , k2 , k3 and k4 as shown in the figure. If a single
(b) E = 2 xy$i + ( x2 + y2 )$j + (3 xz - y2 ) k$ dielectric material is to be used to have the same
(c) E = z3 i$ + xyz$j + z2 k$ capacitance C in this capacitor, then its dielectric
(d) E = (2 xy - z3 )i$ + xy2 $j + 3 z2 xk$
constant k is given by [NEET 2016]
A/3 A/3 A/3
186. An electric dipole of moment p is placed in an
electric field of intensity E. The dipole acquires a k1 k2 k3 d/2
position such that the axis of the dipole makes an
angle q with the direction of the field. Assuming d
that the potential energy of the dipole to be zero k4
when q = 90° , the torque and the potential energy of
the dipole will respectively be [CBSE-AIPMT 2012] A
(a) pE sin q, - pE cos q (b) pE sin q, - 2 pEcos q 2
(a) k = k1 + k2 + k3 + 3k4 (b) k = (k1 + k2 + k3 ) + 2 k4
(c) pE sin q, 2 pE cos q (d) pE cos q, - pE sinq 3
2 3 1 1 1 1 1 3
187. Two thin dielectric slabs of dielectric constants K 1 (c) = + (d) = + + +
k k1 + k2 + k3 k4 k k1 k2 k3 2 k4
and K 2 ( K 1 < K 2 ) are inserted between plates of a
parallel plate capacitor, as shown in the figure. 190. Three capacitors each of capacitance C and of
P + – Q breakdown voltage V are joined in series. The
+ – capacitance and breakdown voltage of the

combination will be [CBSE-AIPMT 2009]
+
C V V
+ – (a) , (b) 3C,
3 3 3
+ – C
(c) , 3V (d) 3C, 3V
+ – 3
+ –
191. A capacitor is charged by a battery. The battery is
+ –
removed and another identical uncharged capacitor
K1 K2
is connected in parallel. The total electrostatic
The variation of electric field E between the plates energy of resulting system [NEET 2017]
with distance d as measured from plate P is correctly (a) increases by a factor of 4 (b) decreases by a factor of 2
shown by [CBSE-AIPMT 2014] (c) remains the same (d) increases by a factor of 2
192. A series combination of n1 capacitors, each of value
E E C1 is charged by a source of potential difference 4V .
When another parallel combination of n 2
(a) (b) capacitors, each of value C 2 is charged by a source
of potential difference V , if both the combination
0 0 have the same (total) energy stored in them, the
d d
value of C2, in terms of C1, is then
E E [CBSE-AIPMT 2010]
2 C1 n2
(a) (b) 16 C1
(c) (d) n1 n2 n1
n 16C1
0 0 (c) 2 2 C1 (d)
d d n1 n1 n2

MODULE 3
www.jeebooks.in
364 NEET Test Drive

193. A parallel plate air capacitor of capacitance C is 198. Two metal wires of identical dimensions are
connected to a cell of emf V and then disconnected connected in series. If s1 and s 2 are the
from it. A dielectric slab of dielectric constant K , conductivities of the metal wires respectively, the
which can just fill the air gap of the capacitor, is effective conductivity of the combination is
now inserted in it. Which of the following is [CBSE-AIPMT 2015]
incorrect? 2 s1 s2 s + s2
[CBSE-AIPMT 2015]
(a) (b) 1
(a) The potential difference between the plates decreases K s1 + s2 2 s1 s2
times s1 + s2 s s
(b) The energy stored in the capacitor decreases K times (c) (d) 1 2
s1 s2 s1 + s2
(c) The change in energy stored is CV 2 æç - 1ö÷
1 1
2 èK ø 199. A carbon resistor of (47 ± 4.7 ) kW is to be marked
(d) The charge on the capacitor is not conserved with rings of different colours for its identification.
The colour code sequence will be [NEET 2018]
194. The electrostatic force between the metal plates of
(a) yellow - green - violet - gold
an isolated parallel plate capacitor C having a
charge Q and area A is [NEET 2018]
(b) yellow - violet - orange - silver
(c) violet - yellow - orange - silver
(a) proportional to the square root of the distance between
(d) green - orange - violet - gold
the plates
(b) linearly proportional to the distance between the plates 200. A battery consists of a variable number n of
(c) independent of the distance between the plates identical cells (having internal resistance r each)
(d) inversely proportional to the distance between the plates which are connected in series. The terminals of the
195. Two condensers, one of capacity C and the other of battery are short-circuited and the current I is
C measured. Which of the graphs shows the correct
capacity , are connected to a V volt battery, as relationship between I and n ?
2 [NEET 2018]
shown in the figure.
I I

(a) (b)
C
V C O
2 O n n
I I

The work done in charging both the condensers is (c) (d)


[CBSE-AIPMT 2007]
O n O n
1 3 1
(a) 2CV 2 (b) CV 2 (c) CV 2 (d) CV 2
4 4 2 201. The potential difference (V A - V B ) between the
196. A capacitor of 2m F is charged as shown in the points A and B in the given figure is [NEET 2016]
figure. When the switch S is turned to position 2, 3V
the percentage of its stored energy dissipated is VA 2Ω + – 1Ω VB
[NEET 2016]
AI = 2 A B
1 2
S (a) –3 V (b) +3 V
(c) +6 V (d) +9 V
V 2µ F
8µ F 202. The resistances of the four arms P , Q , R and S in a
Wheatstone bridge are 10 W, 30 W, 30 W and 90 W
respectively. The emf and internal resistance of the
(a) 20% (b) 75% (c) 80% (d) 0% cell are 7 V and 5 W respectively. If the
galvanometer resistance is 50 W, the current drawn
from the cell will be [NEET 2013]
17. Current Electricity (a) 1.0 A (b) 0.2 A
(c) 0.1 A (d) 2.0 A
197. The resistance of a wire is R ohm. If it is melted and
stretched to n times its original length, its new 203. The resistances in the two arms of the meter
resistance will be [NEET 2017] bridge are 5 W and R W, respectively. When
R 2 R the resistance R is shunted with an equal
(a) nR (b) (c) n R (d)
n n2
MODULE 3
www.jeebooks.in
The NEET Edge ~ Physics 365

resistance, the new balance point is at 1.6 l1. The 209. The charge following through a resistance R
resistance R is [CBSE-AIPMT 2014] varies with time t as Q = at - bt 2, where a and b
() are positive constants. The total heat produced in
5Ω RΩ R is [NEET 2016]
a3 R a3 R
(a) (b)
3b 2b
a3 R a3 R
G (c) (d)
b 6b

A l1 100 –l 1 B
210. An electric kettle takes 4 A current at 220 V. How
much time will it take to boil 1 kg of water from
(a) 10 W (b) 15 W (c) 20 W (d) 25 W temperature 20°C? [CBSE-AIPMT 2008]
204. A potentiometer is an accurate and versatile device (a) 6.3 min (b) 8.4 min
to make electrical measurement of emf because the (c) 12.6 min (d) 4.2 min
method involves [NEET 2017] 211. An electric kettle has two heating coils. When one
(a) cells of the coils is connected to an AC source, the water
(b) potential gradients in the kettle boils in 10 min. When the other coil is
(c) a condition of no current flow through the galvanometer used the water boils in 40 min. If both the coils are
(d) a combination of cells, galvanometer and resistances connected in parallel, the time taken by the same
205. A potentiometer wire is 100 cm long and a constant quantity of water to boil will be [CBSE-AIPMT 2003]
potential difference is maintained across it. Two (a) 25 min (b) 15 min
cells are connected in series first to support one (c) 8 min (d) 4 min
another and then in opposite direction. The 212. A filament bulb (500 W, 100 V) is to be used in a
balance points are obtained at 50 cm and 10 cm 230 V main supply. When a resistance R is
from the positive end of the wire in the two cases. connected in series, it works perfectly and the bulb
The ratio of emf is [NEET 2016] consumes 500 W. The value of R is [NEET 2016]
(a) 5 : 4 (b) 3 : 4 (a) 230 W (b) 46 W
(c) 3 : 2 (d) 5 : 1 (c) 26 W (d) 13 W
206. A potentiometer wire has length 4 m and resistance 213. Two cities are 150 km apart. Electric power is sent
8W. The resistance that must be connected in series from one city to another city through copper wires.
with the wire and an accumulator of emf 2V, so as The fall of potential per km is 8 V and the average
to get a potential gradient 1 mV per cm on the resistance per km is 0. 5 W. The power loss in the
wire is [CBSE-AIPMT 2015] wire is [CBSE-AIPMT 2014]
(a) 32 W (b) 40 W (c) 44 W (d) 48 W (a) 19.2 W (b) 19.2 kW
207. A potentiometer wire of length L and a resistance r (c) 19.2 J (d) 12.2 kW
are connected in series with a battery of emf E0 214. If voltage across a bulb rated 220 V-100 W drops
and a resistance r1. An unknown emf is balanced at by 2.5% of its rated value, the percentage of the
a length l of the potentiometer wire. The emf E will rated value by which the power would decrease is
be given by [CBSE-AIPMT 2015] (a) 20% (b) 2.5% [CBSE-AIPMT 2012]
LE0 r E0 r l
(a) (b) . (c) 5% (d) 10%
l r1 (r + r1 ) L
E l LE0 r 215. If power dissipated in the 9 W resistor in the circuit
(c) 0 (d) shown is 36W, the potential difference across the
L (r + r1 ) l
2 W resistor is [CBSE-AIPMT 2011]
208. A potentiometer circuit has been set up for finding 9Ω
the internal resistance of a given cell. The main
battery, used across the potentiometer wire, has an
6Ω
emf of 2.0 V and a negligible internal resistance.
The potentiometer wire itself is 4 m long. When the
resistance R, connected across the given cell, has
values of (i) infinity and (ii) 9.5 W , the balancing
lengths, on the potentiometer wire are found to be + –
V 2Ω
3 m and 2.85 m, respectively. The value of internal
resistance of the cell is [CBSE-AIPMT 2014] (a) 8 V (b) 10 V
(a) 0.25 W (b) 0.95 W (c) 0.5 W (d) 0.75 W (c) 2 V (d) 4 V

MODULE 3
www.jeebooks.in
366 NEET Test Drive

216. A current of 3 A flows through the 2 W resistor


shown in the circuit. The power dissipated in the 18. Moving Charges and Magnetism
5 W resistor is [CBSE-AIPMT 2008] 221. Two identical long conducting wires AOB and COD are
2Ω placed at right angle to each other, such that one is
above the other and O is their common point. The
4Ω wires carry I1 and I 2 currents, respectively. Point P is
lying at distance d from O along a direction
perpendicular to the plane containing the wires. The
magnetic field at the point P will be [CBSE-AIPMT 2014]
1Ω 5Ω m0 æ I1 ö m0
(a) ç ÷ (b) (I1 + I2 )
2 pd è I2 ø 2 pd
m m
(c) 0 (I12 - I22 ) (d) 0 (I12 + I22 )1/ 2
(a) 4 W (b) 2 W 2 pd 2 pd
(c) 1 W (d) 5 W
222. A long straight wire of radius a carries a steady
217. The total power dissipated in watts in the circuit current I. The current is uniformly distributed over
shown below [CBSE-AIPMT 2007] its cross-section. The ratio of the magnetic fields B
6Ω
a
and B¢ at radial distances and 2a respectively,
2
from the axis of the wire is [NEET 2016]
1 1
(a) (b) 1 (c) 4 (d)
3Ω 2 4
223. A wire carrying current I has the shape as shown
+ – in adjoining figure. Linear parts of the wire are
4Ω 18 V very long and parallel to X-axis while semicircular
(a) 16 (b) 40 portion of radius R is lying in YZ-plane. Magnetic
(c) 54 (d) 4 field at point O is [CBSE-AIPMT 2015]
Z
218. Power dissipated across the 8 W resistor in the
circuit shown below is 2 W. The power dissipated I
(in watt) units across the 3 W resistor is
R Y
[CBSE-AIPMT 2006] O
1Ω 3Ω I
I

i X
m0 I $ m0 I $
(a) B = (p i + 2k$ ) (b) B = - (p i - 2k$ )
4p R 4p R
8Ω m I m I $
(c) B = - 0 (p $i + 2k$ ) (d) B = 0 (p i - 2k$ )
(a) 2.0 (b) 1.0 4p R 4p R
(c) 0.5 (d) 3.0
224. When a proton is released from rest in a room, it
219. In producing chlorine by electrolysis 100 kW power starts with an initial acceleration a0 towards west.
at 125 V is being consumed. How much chlorine When it is projected towards north with a speed v0,
per minute is liberated? (ECE of chlorine is it moves with an initial acceleration 3a0 towards
0.367 ´10-6 kg C-1 ) [CBSE-AIPMT 2010] west. The electric and magnetic fields in the room
. ´ 10-3 kg
(a) 176 are [NEET 2013]
(b) 9.67 ´ 10- 3 kg (a)
ma0
west,
2 ma0
up (b)
ma0
west,
2 ma0
down
(c) 17.61 ´ 10-3 kg e ev 0 e ev 0
(d) 3.67 ´ 10-3 kg ma0 3ma0 ma0 3ma0
(c) east, up (d) east, down
e ev 0 e ev 0
220. A steady current of 1.5 A flows through a copper
voltameter for 10 min. If the electrochemical 225. An electron is moving in a circular path under the
equivalent of copper is 30 ´ 10-5 g C-1, the mass of influence of a transverse magnetic field of
copper deposited on the electrode will be 3. 57 ´ 10-2 T. If the value of e/m is 1.76 ´ 1011 C/kg,
[CBSE-AIPMT 2007] the frequency of revolution of the electron is
(a) 0.40 g (b) 0.50 g (a) 1 GHz (b) 100 MHz [NEET 2016]
(c) 0.67 g (d) 0.27 g (c) 62.8 MHz (d) 6.28 MHz

MODULE 3
www.jeebooks.in
The NEET Edge ~ Physics 367

226. A metallic rod of mass per unit length 0.5 kg m -1 is 230. A closed loop PQRS carrying a current is placed in
lying horizontally on a smooth inclined plane a uniform magnetic field. If the magnetic forces on
which makes an angle of 30° with the horizontal. segments PS , SR and RQ are F1 , F2 and F3
The rod is not allowed to slide down by flowing a respectively, in the plane of the paper and along
current through it when a magnetic field of the directions shown in the figure, the force on the
induction 0.25 T is acting on it in the vertical segment QP is [CBSE-AIPMT 2008]
direction. The current flowing in the rod to keep it Q
stationary is [NEET 2018]
(a) 14.76 A (b) 5.98 A
(c) 7.14 A (d) 11.32 A P
F3
227. A square loop ABCD carrying a current i, is placed F1
near and coplanar with a long straight conductor
XY carrying a current I, the net force on the loop
S R
will be [NEET 2016]
F2
B C
Y
(a) F3 - F1 - F2 (b) (F3 - F1 )2 + F22
(c) (F3 - F1 )2 - F22 (d) F3 - F1 + F2
I i L
231. A rectangular coil of length 0.12 m and width
X
0.1 m having 50 turns of wire is suspended
A D vertically in a uniform magnetic field of strength
L/2 L 0.2 Wb/m 2. The coil carries a current of 2A. If the
m Ii 2m IiL m 0 IiL 2m 0 Ii plane of the coil is inclined at an angle of 30° with
(a) 0 (b) 0 (c) (d)
2p 3p 2p 3p the direction of the field, the torque required to
keep the coil in stable equilibrium will be
228. A current carrying closed loop in the form of a
right angled isosceles D ABC is placed in a uniform
[CBSE-AIPMT 2015]
(a) 0.15 Nm (b) 0.20 Nm
magnetic field acting along AB. If the magnetic
(c) 0.24 Nm (d) 0.12 Nm
force on the arm BC is F, the force on the arm AC
is [CBSE-AIPMT 2011] 232. Current sensitivity of a moving coil galvanometer
is 5 div/mA and its voltage sensitivity (angular
A
deflection per unit voltage applied) is 20 div/V. The
resistance of the galvanometer is [NEET 2018]
(a) 250 W (b) 25 W (c) 40 W (d) 500 W
233. In an ammeter 0.2% of main current passes
B through the galvanometer. If resistance of
C
galvanometer is G, the resistance of ammeter will
(a) -F (b) F (c) 2 F (d) - 2 F be [CBSE-AIPMT 2014]
229. An arrangement of three parallel straight wires 1 499
(a) G (b) G
placed perpendicular to plane of paper carrying 499 500
same current I along the same direction is shown 1 500
(c) G (d) G
in the figure. Magnitude of force per unit length on 500 499
the middle wire B is given by [NEET 2017] 234. A millivoltmeter of 25 mV range is to be converted
d C into an ammeter of 25 A range. The value (in ohm)
B
of necessary shunt will be [CBSE-AIPMT 2012]
90° (a) 0.001 (b) 0.01 (c) 1 (d) 0.05

d
19. Magnetism and Matter
A 235. Following figures show the arrangement of bar
m 0 I2 2 m 0 I2 magnets in different configurations. Each magnet
(a) (b)
2 pd pd has magnetic dipole moment M. Which
2m 0 I 2 m 0 I2 configuration has highest net magnetic dipole
c) (d)
pd 2 pd moment? [CBSE-AIPMT 2014]

MODULE 3
www.jeebooks.in
368 NEET Test Drive

240. A vibration magnetometer placed in magnetic


N meridian has a small bar magnet. The magnet
N S executes oscillations with a time period of 2 s in
(i) (ii)
the earth’s horizontal magnetic field of 24 mT.
S S N S N When a horizontal field of 18 mT is produced
opposite to the earth’s field by placing a current
N carrying wire, the new time period of magnet will
N be [CBSE-AIPMT 2010]
(a) 1s (b) 2 s (c) 3 s (d) 4 s
(iii) 30° (iv) 60°
241. If a diamagnetic substance is brought near the
S N S N north or the south pole of a bar magnet, it is
(a) repelled by both the poles [CBSE-AIPMT 2009]
(a) (i) (b) (ii)
(b) repelled by the north pole and attracted by the south pole
(c) (iii) (d) (iv)
(c) attracted by the north pole and repelled by the south pole
236. A bar magnet of length l and magnetic dipole (d) attracted by both the poles
moment M is bent in the form of an arc as shown
in figure. The new magnetic dipole moment will be 242. A thin diamagnetic rod is placed vertically between
[NEET 2013]
the poles of an electromagnet. When the current in
the electromagnet is switched on, then the
diamagnetic rod is pushed up, out of the horizontal
magnetic field. Hence, the rod gains gravitational
potential energy. The work required to do this
comes from [NEET 2018]
r r (a) the lattice structure of the material of the rod
60° (b) the magnetic field
(c) the current source
(d) the induced electric field due to the changing magnetic field
3 2 M
(a) M (b) M (c) M (d) 243. If the magnetic dipole moment of an atom of
p p 2
diamagnetic material, paramagnetic material and
237. A bar magnet having a magnetic moment of ferromagnetic material are denoted by m d , m p and
2 ´ 104 J T -1 is free to rotate in a horizontal plane. m f respectively, then [CBSE-AIPMT 2005]
A horizontal magnetic field B = 6 ´ 10-4 T exists in (a) md ¹ 0 and m f ¹ 0 (b) m p = 0 and m f ¹ 0
the space. The work done in taking the magnet (c) md = 0 and m p ¹ 0 (d) md ¹ 0 and m p = 0
slowly from a direction parallel to the field to a
direction 60° from the field is
20. Electromagnetic Induction
[CBSE-AIPMT 2009]
(a) 0.6 J (b) 12 J
(c) 6 J (d) 2 J 244. A uniform magnetic field is restricted within a
238. A bar magnet is hung by a thin cotton thread in a region of radius r. The magnetic field changes with
dB
uniform horizontal magnetic field and is in time at a rate . Loop 1 of radius R > r encloses
equilibrium state. The energy required to rotate it dt
by 60 is W . Now, the torque required to keep the the region r and loop 2 of radius R is outside the
magnet in this new position is [NEET 2016] region of magnetic field as shown in the figure.
W Then, the emf generated is [NEET 2016]
(a) (b) 3W
3
3W 2W
(c) (d) r
2 3 R R
239. If q 1 and q 2 be the apparent angles of dip observed 1 2
in two vertical planes at right angles to each other,
then the true angle of dip q is given by [NEET 2017] (a) zero in loop 1 and zero in loop 2
(a) cot 2 q = cot 2 q1 + cot 2 q2 (b) -
dB 2
pr in loop 1 and -
dB 2
pr in loop 2
(b) tan2 q = tan2 q1 + tan2 q2 dt dt
dB 2
(c) cot 2 q = cot 2 q1 - cot 2 q2 (c) - pR in loop 1 and zero in loop 2
dt
(d) tan2 q = tan2 q1 - tan2 q2 dB 2
(d) - pr in loop 1 and zero in loop 2
dt

MODULE 3
www.jeebooks.in
The NEET Edge ~ Physics 369

245. A long solenoid of diameter 0.1 m has 2 ´ 104 turns 250. A coil of self-inductance L is connected in series
per metre. At the centre of the solenoid, a coil of with a bulb B and an AC source. Brightness of the
100 turns and radius 0.01 m is placed with its axis bulb decreases when [NEET 2013]
coinciding with the solenoid axis. The current in the (a) frequency of the AC source is decreased
solenoid reduces at a constant rate to 0 A from 4 A in (b) number of turns in the coil is reduced
0.05 s. If the resistance of the coil is 10p 2 W, the total (c) a capacitance of reactance XC = X L is included in the
charge flowing through the coil during this time is same circuit
(d) an iron rod is inserted in the coil
(a) 32p mC (b) 16 mC [NEET 2017]
(c) 32 mC (d) 16p mC 251. A series R-C circuit is connected to an alternating
voltage source. Consider two situations.
246. The current i in a coil varies with time as shown in [CBSE-AIPMT 2015]
the figure. The variation of induced emf with time
would be [CBSE-AIPMT 2011] 1. When capacitor is air filled (voltage across air
i filled capacitor = V a and current = ia )
2. When capacitor is mica filled (voltage across
mica filled capacitor = V b and current = ib)
t
Current through resistor is i and voltage across
0
T/4 T/2 3T/4 T capacitor is V , then
(a) Va < Vb (b) Va > Vb
(c) i a > i b (d) Va = Vb

emf emf 252. An AC voltage is applied to a resistance R and an


inductor L in series. If R and the inductive
reactance are both equal to 3 W, the phase difference
T/4 T/2 3T/2 T between the applied voltage and the current in the
(a) 0 t (b) 0 t
T/4 T/2 3T/2 T circuit is [CBSE-AIPMT 2011]
(a) p/4 (b) p/2
emf
(c) zero (d) p/6
emf
253. An inductor 20 mH, a capacitor 100 mF and a
resistor 50 W are connected in series across a source
T/2 3T/4 T T/4 of emf, V = 10 sin 314 t. The power loss in the
(c) 0 t (d) 0 t
circuit is
T/4 T/2 3T/4 T [NEET 2018]
(a) 2.74 W (b) 0.43 W
(c) 0.79 W (d) 1.13 W
247. A long solenoid has 500 turns. When a current of 254. The potential differences across the resistance,
2 A is passed through it, the resulting magnetic capacitance and inductance are 80 V, 40 V and
flux linked with each turn of the solenoid is 100 V respectively in an L-C-R circuit. The power
4 ´ 10-3 Wb. The self-inductance of the solenoid is factor of this circuit is [NEET 2016]
[CBSE-AIPMT 2008] (a) 0.4 (b) 0.5
(a) 2.5 H (b) 2 H (c) 1 H (d) 4 H (c) 0.8 (d) 1.0
248. A long solenoid has 1000 turns. When a current of 255. In an electrical circuit, R , L , C and an AC voltage
4A flows through it, the magnetic flux linked with source are all connected in series. When L is
each turn of the solenoid is 4 ´ 10-3 Wb. The removed from the circuit, the phase difference
self-inductance of the solenoid is [NEET 2016] between the voltage and the current in the circuit
(a) 3 H (b) 2 H is p/3. If instead, C is removed from the circuit, the
(c) 1 H (d) 4 H phase difference is again p/3. The power factor of
the circuit is [CBSE-AIPMT 2012]
(a) 1/2 (b) 1/ 2 (c) 1 (d) 3/2
21. Alternating Current 256. What is the value of inductance L for which the
249. In an AC circuit, an alternating voltage current is maximum in a series L-C-R circuit with
e = 200 2 sin 100 t volt is connected to a capacitor of C = 10 mF and w = 1000 s-1 ? [CBSE-AIPMT 2007]
capacity 1 mF. The rms value of the current in the (a) 100 mH
circuit is [CBSE-AIPMT 2011] (b) 1 mH
(a) 100 mA (b) 200 mA (c) Cannot be calculated unless R is known
(c) 20 mA (d) 10 mA (d) 10 mH

MODULE 3
www.jeebooks.in
370 NEET Test Drive

257. In a circuit, L , C and R are connected in series with 263. Out of the following options which one can be used
an alternating voltage source of frequency f. The to produce a propagating EM wave? [NEET 2016]
current leads the voltage by 45°. The value of C is (a) A stationary charge
[CBSE-AIPMT 2005] (b) A chargeless particle
1 1 (c) An accelerating charge
(a) (b)
2 pf(2 pfL + R ) pf(2 pfL + R ) (d) A charge moving at constant velocity
1 1
(c) (d) 264. The energy of the EM waves is of the order of 15 keV.
2 pf(2 pfL - R ) pf(2 pfL - R )
To which part of the spectrum does it belong?
258. Figure shows a circuit that contains three identical [CBSE-AIPMT 2015]
resistors with resistance R = 9.0 W each, two (a) X-rays (b) Infrared rays
identical inductors with inductance L = 2 .0 mH (c) Ultraviolet rays (d) g-rays
each and an ideal battery with emf e = 18 V. The 265. If l v , l X and l m represent the wavelengths of
current i through the battery just after the switch visible light, X-rays and microwaves respectively,
closed is [NEET 2017] then [CBSE-AIPMT 2005]
(a) l m > l X > l v (b) l v > l m > l X
L R R (c) l m > l v > l X (d) l v > l X > l m
+
ε –
R L C 23. Ray Optics and Optical Instruments
266. A beam of light from a source L is incident
(a) 2 mA (b) 0.2 A normally on a plane mirror fixed at a certain
(c) 2 A (d) 0 A distance x from the source. The beam is reflected
259. Which of the following combinations should be back as a spot on a scale placed just above the
selected for better tuning of an L-C-R circuit used source L. When the mirror is rotated through a
for communication? [NEET 2016] small angle q, the spot of the light is found to move
(a) R = 20W, L = 15
. H, C = 35 mF through a distance y on the scale. The angle q is
(b) R = 25W, L = 2.5 H, C = 45 mF given by [NEET 2017]
(c) R = 15W, L = 3.5 H, C = 30 mF y y x x
(a) (b) (c) (d)
(d) R = 25W, L = 15
. H, C = 45 mF 2x x 2y y

260. A transformer having efficiency of 90% is working 267. A beam of light composed of red and green rays is
on 200 V and 3 kW power supply. If the current in incident obliquely at a point on the face of a
the secondary coil is 6 A, the voltage across the rectangular glass slab. When coming out on the
secondary coil and the current in the primary coil opposite parallel face, the red and green rays
respectively are [CBSE-AIPMT 2014] emerge from [CBSE-AIPMT 2004]
(a) 300 V, 15 A (b) 450 V, 15 A (a) two points propagating in two different non-parallel
(c) 450 V, 13.5 A (d) 600 V, 15 A directions
(b) two points propagating in two different parallel directions
(c) one point propagating in two different directions
22. Electromagnetic Waves (d) one point propagating in the same direction
261. An EM wave is propagating in a medium with a 268. Two identical glass ( m g = 3 / 2) equiconvex lenses of
velocity v = v $i . The instantaneous oscillating focal length f each are kept in contact. The space
electric field of this EM wave is along + Y -axis. between the two lenses is filled with water
Then, the direction of oscillating magnetic field of ( m w = 4 / 3). The focal length of the combination is
EM wave will be along [NEET 2018] [NEET 2016]
(a) - y-direction 4f 3f
(a) f / 3 (b) f (c) (d)
(b) + z-direction 3 4
(c) - z-direction 269. Two identical thin plano-convex glass lenses
(d) - x-direction (m g = 1.5) each having radius of curvature of 20 cm
262. In an EM wave in free space the root mean square are placed with their convex surfaces in contact at
value of the electric field is E rms = 6 V/m. The peak the centre. The intervening space is filled with oil
value of the magnetic field is [NEET 2017] ( m oil = 1.7) The focal length of the combination is
(a) 1.41 ´ 10-8 T (b) 2.83 ´ 10-8 T [CBSE-AIPMT 2015]
(c) 0.70 ´ 10-8 T (d) 4.23 ´ 10-8 T (a) - 20 cm (b) - 25 cm (c) - 50 cm (d) 50 cm

MODULE 3
www.jeebooks.in
The NEET Edge ~ Physics 371

270. A concave mirror of focal length f1 is placed at a 276. The refracting angle of a prism is A and refractive
distance of d from a convex lens of focal length f2. index of the material of the prism is cot A /2. The
A beam of light coming from infinity and falling on angle of minimum deviation is [CBSE-AIPMT 2015]
this convex lens and concave mirror combination (a) 180° - 3 A (b) 180° - 2 A
returns to infinity. The distance d must be equal (c) 90° - A (d) 180° + 2 A
[CBSE-AIPMT 2012]
277. A ray of light is incident at an angle of incidence i,
(a) f1 + f2 (b) - f1 + f2
on one face of a prism of angle A (assumed to be
(c) 2 f1 + f2 (d) -2 f1 + f2
small) and emerges normally from the opposite
271. A lens having focal length f and aperture of face. If the refractive index of the prism is m, the
diameter d forms an image of intensity I. Aperture angle of incidence i, is nearly equal to
d
of diameter in central region of lens is covered [CBSE-AIPMT 2012]
2 mA
(a) m A (b) (c) A /m (d) A/2m
by a black paper. Focal length of lens and intensity 2
of image now will be respectively [CBSE-AIPMT 2010] 278. The ratio of resolving powers of an optical
I 3f I
(a) f and (b) and microscope for two wavelengths l1 = 4000 Å and
4 4 2 l 2 = 6000 Å is
3I f I
[NEET 2017]
(c) f and (d) and (a) 8 : 27 (b) 9 : 4 (c) 3 : 2 (d) 16 : 81
4 2 2
272. A boy is trying to start a fire by focusing sunlight 279. A microscope is focussed on a mark on a piece of
paper and then a slab of glass of thickness 3 cm and
on a piece of paper using an equiconvex lens of
refractive index 1.5 is placed over the mark. How
focal length 10 cm. The diameter of the sun is should the microscope be moved to get the mark in
1.39 ´ 10 9 m and its mean distance from the earth focus again? [CBSE-AIPMT 2006]
is 1.5 ´ 10 11 m. What is the diameter of the sun’s (a) 1 cm upward (b) 4.5 cm downward
image on the paper ? [CBSE-AIPMT 2008] (c) 1 cm downward (d) 2 cm upward
(a) 9.2 ´ 10-4 m (b) 6.5 ´ 10-4 m
280. In an astronomical telescope in normal adjustment
(c) 6.5 ´ 10-5 m (d) 12.4 ´ 10-4 m
a straight black line of length L is drawn on inside
273. The refractive index of the material of a prism is part of objective lens. The eye-piece forms a real
2 and the angle of the prism is 30°. One of the image of this line. The length of this image is I.
two refracting surfaces of the prism is made a The magnification of the telescope is
mirror from inwards, by silver coating. A beam of [CBSE-AIPMT 2015]
monochromatic light entering the prism from the L L L+1 L
(a) + 1 (b) - 1 (c) (d)
other face will retrace its path (after reflection I I L-1 I
from the silvered surface) if its angle of incidence 281. A telescope has an objective lens of 10 cm diameter
on the prism is [NEET 2018] and is situated at a distance of one kilometre from
(a) 30° (b) 45° two objects. The minimum distance between these
(c) 60° (d) zero two objects, which can be resolved by the telescope,
274. A thin prism having refracting angle 10° is made of when the mean wavelength of light is 5000 Å, is of
glass (refractive index, m1 = 1.42). This prism is the order of [CBSE-AIPMT 2004]
combined with another thin prism of glass (a) 0.5 m (b) 5 m (c) 5 mm (d) 5 cm
(refractive index, m 2 = 1.7). This combination 282. The angular resolution of a 10 cm diameter telescope
produces dispersion without deviation. The at a wavelength of 5000 Å, is of the order of
refracting angle of second prism should be [CBSE-AIPMT 2005]
(a) 4° (b) 6° -2 -4
6
(a) 10 rad (b) 10 rad (c) 10 rad (d) 10- 6 rad
[NEET 2017]
(c) 8° (d) 10°
275. The angle of incidence for a ray of light at a
refracting surface of a prism is 45°. The angle of 24. Wave Optics
prism is 60°. If the ray suffers minimum deviation 283. The interference pattern is obtained with two
through the prism, the angle of minimum deviation coherent light sources of intensity ratio n. In the
and refractive index of the material of the prism I - I min
respectively, are interference pattern, the ratio max will be
[NEET 2016] I max + I min
(a) 30°, 2 (b) 45°, 2
[NEET 2016]

1 1 n 2 n n 2 n
(c) 30°, (d) 45°, (a) (b) (c) (d)
2 2 n+ 1 n+ 1 (n + 1)2 (n + 1)2

MODULE 3
www.jeebooks.in
372 NEET Test Drive

284. In Young’s double slit experiment, the separation d (c) Reflected light is polarised with its electric vector parallel to
between the slits is 2 mm, the wavelength l of the the plane of incidence
light used is 5896 Å and distance D between the æ 1ö
(d) i = tan-1 ç ÷
screen and slits is 100 cm. It is found that the èm ø
angular width of the fringes is 0.20°. To increase
the fringe angular width to 0.21° (with same l and 291. Two polaroids P1 and P2 are placed with their axis
D) the separation between the slits needs to be perpendicular to each other. Unpolarised light I 0 is
changed to [NEET 2018] incident on P1. A third polaroid P3 is kept in
(a) 2.1 mm (b) 1.9 mm (c) 1.8 mm (d) 1.7 mm between P1 and P2 such that its axis makes an
angle 45° with that of P1. The intensity of
285. Young’s double slit experiment is first performed in transmitted light through P2 is [NEET 2017]
air and then in a medium other than air. It is I0 I0 I0 I0
found that 8th bright fringe in the medium lies, (a) (b) (c) (d)
2 4 8 16
where 5th dark fringe lies, in air. The refractive
index of the medium is nearly [NEET 2017]
(a) 1.25 (b) 1.59 (c) 1.69 (d) 1.78 25. Dual Nature of Radiation and Matter
286. The intensity of the maxima in a Young’s double 292. The photoelectric threshold wavelength of silver is
slit experiment is I 0. Distance between two slits is 3250 ´ 10-10 m. The velocity of the electron ejected
d = 5l, where l is the wavelength of light used in from a silver surface by ultraviolet light of
the experiment. What will be the intensity in front wavelength 2536 ´ 10-10 m is (take, h = 4.14 ´ 10-15
of one of the slits on the screen placed at a distance
D = 10 d ? [NEET 2016]
eV-s and c = 3 ´ 108 ms-1 ) [NEET 2017]
I0 3 I0 (a) » 6 ´ 105 ms -1 (b) » 0.6 ´ 106 ms -1
(a) (b) I0 (c) (d) I0 -1
4 4 2 3
(c) » 61 ´ 10 ms (d) » 0.3 ´ 106 ms -1
287. Two slits in Youngs experiment have widths in the 293. When a metallic surface is illuminated with
ratio 1 : 25. The ratio of intensity at the maxima radiation of wavelength l, the stopping potential is
I
and minima in the interference pattern max is V. If the same surface is illuminated with radiation
I min V
of wavelength 2l, the stopping potential is . The
9 121 4
(a) (b) [CBSE-AIPMT 2015]
4 49 threshold wavelength for the metallic surface is
49 4
(c) (d) [NEET 2016]
121 9 5
(a) 5l (b) l (c) 3l (d) 4l
288. In a diffraction pattern due to a single slit of width 2
a, the first minimum is observed at an angle 30° 294. A certain metallic surface is illuminated with
when light of wavelength 5000 Å is incident on the monochromatic light of wavelength l. The stopping
slit. The first secondary maximum is observed at potential for photoelectric current for this light is
an angle of [NEET 2016] 3V 0. If the same surface is illuminated with light of
(a) sin-1 æç ö÷ (b) sin-1 æç ö÷ (c) sin-1 æç ö÷ (d) sin-1 æç ö÷ wavelength 2l, the stopping potential is V 0. The
2 1 3 1
è 3ø è2 ø è 4ø è 4ø threshold wavelength for this surface for
photoelectric effect is [CBSE-AIPMT 2015]
289. For a parallel beam of monochromatic light of l l
wavelength l diffraction is produced by a single (a) 6l (b) 4l (c) (d)
4 6
slit whose width is a. If D is the distance of the
screen from the slit, the width of the central 295. When the energy of the incident radiation is
maxima will be [CBSE-AIPMT 2015] increased by 20%, the kinetic energy of the
2Dl Dl Da 2Da photoelectrons emitted from a metal surface
(a) (b) (c) (d)
a a l l increased from 0.5 eV to 0.8 eV. The work function
of the metal is
290. Unpolarised light is incident from air on a plane
[CBSE-AIPMT 2014]

surface of a material of refractive index m. At a (a) 0.65 eV (b) 1.0 eV (c) 1.3 eV (d) 1.5 eV
particular angle of incidence i , it is found that the 296. When the light of frequency 2n 0 (where, n 0 is
reflected and refracted rays are perpendicular to threshold frequency) is incident on a metal plate,
each other. Which of the following options is the maximum velocity of electrons emitted is v1.
correct for this situation? [NEET 2018] When the frequency of the incident radiation is
æ 1ö
(a) i = sin-1 ç ÷
increased to 5n 0, the maximum velocity of electrons
èm ø emitted from the same plate is v2. The ratio of v1 to
(b) Reflected light is polarised with its electric vector v2 is [NEET 2018]
perpendicular to the plane of incidence (a) 4 : 1 (b) 1 : 4 (c) 1 : 2 (d) 2 : 1

MODULE 3
www.jeebooks.in
The NEET Edge ~ Physics 373

297. Light of two different frequencies whose photons 305. A radiation of energy ‘E’ falls normally on a
have energies 1 eV and 2.5 eV respectively perfectly reflecting surface. The momentum
illuminate a metallic surface whose work function transferred to the surface is
is 0.5 eV successively. Ratio of maximum speeds of (where, c = velocity of light) [CBSE-AIPMT 2015]
emitted electrons will be E 2E 2E E
[CBSE-AIPMT 2011]
(a) (b) (c) (d)
(a) 1 : 2 (b) 1 : 1 (c) 1 : 5 (d) 1 : 4 c c c2 c2
298. Photons with energy 5 eV are incident on a cathode 306. If the kinetic energy of the particle is increased to
C in a photoelectric cell. The maximum energy of 16 times its previous value, the percentage change
emitted photoelectrons is 2 eV. When photons of in the de-Broglie wavelength of the particle is
energy 6 eV are incident on C, no photoelectrons [CBSE-AIPMT 2014]
will reach the anode A, if the stopping potential of (a) 25 (b) 75 (c) 60 (d) 50
A relative to C is [NEET 2016]
307. The momentum of a photon of energy 1 MeV in
(a) + 3 V (b) + 4 V (c) – 1 V (d) – 3 V kg-m/s will be [CBSE-AIPMT 2006]
299. A 200W sodium street lamp emits yellow light of (a) 0.33 ´ 106 (b) 7 ´ 10–24 (c) 10–22 (d) 5 ´ 10–22
wavelength 0.6 mm. Assuming it to be 25% efficient
in converting electrical energy to light, the number 308. Light of wavelength 500 nm is incident on a metal with
of photons of yellow light it emits per second is work function 2.28 eV. The de-Broglie wavelength
[CBSE-AIPMT 2012]
of the emitted electron is [CBSE-AIPMT 2015]

(a) 1.5 ´ 1020 (b) 6 ´ 1018 (c) 62 ´ 1020 (d) 3 ´ 1019 (a) < 2.8 ´ 10- 10 m (b) < 2.8 ´ 10- 9 m
(c) ³ 2.8 ´ 10- 9 m (d) £ 2.8 ´ 10- 12 m
300. Monochromatic light of wavelength 667 n-m is
produced by a helium neon laser. The power
emitted is 9 mW. The number of photons arriving 26. Atoms
per second on the average at a target irradiated by 309. In a Rutherford scattering experiment, when a
this beam is [CBSE-AIPMT 2009] projectile of charge Z1 and mass M1 approaches a
(a) 9 ´ 1017 (b) 3 ´ 1016 (c) 9 ´ 1015 (d) 3 ´ 1019 target nucleus of charge Z 2 and mass M 2, the
distance of closest approach is r0. The energy of the
301. The de-Broglie wavelength of a neutron in thermal projectile is [CBSE-AIPMT 2009]
equilibrium with heavy water at a temperature T
(a) directly proportional to M1 ´ M 2
(Kelvin) and mass m, is [NEET 2017]
(b) directly proportional to Z1Z 2
h h 2h 2h
(a) (b) (c) (d) (c) inversely proportional to Z1
mkT 3mkT 3mkT mkT (d) directly proportional to mass M1
302. An electron of mass m and a photon have same 310. When an a-particle of mass m moving with velocity
energy E. The ratio of de-Broglie wavelengths v bombards on a heavy nucleus of charge Ze, its
associated with them is [NEET 2016] distance of closest approach from the nucleus
1 1 depends on m as
(a) æç
E ö2 [NEET 2016]
÷ (b) c(2 mE )2 1 1 1
è2 mø (a) (b) (c) m (d)
1 1 m m2 m
1 æ2 mö 2
(d) æç ö2
1 E
(c) ç ÷ ÷ 311. The ratio of kinetic energy to the total energy of an
cè E ø c è2 mø
electron in a Bohr orbit of the hydrogen atom, is
(where, c being velocity of light) [NEET 2018]
(a) 2 : - 1 (b) 1 : - 1 (c) 1 : 1 (d) 1 : - 2
303. Electrons of mass m with de-Broglie wavelength l 312. Consider 3rd orbit of He+ (Helium),using non-relativistic
fall on the target in an X-ray tube. The cut-off approach, the speed of electron in this orbit will be
wavelength (l 0) of the emitted X-ray is [NEET 2016] [given, K = 9 ´ 109 (constant) Z = 2 and h (Planck's
(a) l 0 =
2 mcl2
(b) l 0 =
2h constant) = 6.6 ´ 10-34 J-s] [CBSE-AIPMT 2015]
h mc (a) 2.92 ´ 106 m/s (b) 1.46 ´ 106 m/s
2 m2c 2 l3 (c) 0.73 ´ 106 m/s (d) 3.0 ´ 108 m/s
(c) l 0 = (d) l 0 = l
h2
313. Ionisation potential of hydrogen atom is 13.6 eV.
304. The wavelength l e of an electron and l p of a Hydrogen atoms in the ground state are excited by
photon of same energy E are related by [NEET 2013] monochromatic radiation of photon energy 12.1 eV.
(a) l p µ l2e (b) l p µ l e According to Bohr’s theory, the spectral lines emitted
1 by hydrogen will be
(c) l p µ l e (d) l p µ
[CBSE-AIPMT 2006]
le (a) 2 (b) 3 (c) 4 (d) 1

MODULE 3
www.jeebooks.in
374 NEET Test Drive

314. The total energy of an electron in the first excited 323. A radioactive nucleus of mass M emits a photon of
state of hydrogen is about – 3.4 eV. Its kinetic frequency n and the nucleus recoils. The recoil
energy in this state is [CBSE-AIPMT 2005] energy will be [CBSE-AIPMT 2011]
(a) – 3.4 eV (b) – 6.8 eV (c) 6.8 eV (d) 3.4 eV (a) h2 n2 /2 Mc 2 (b) zero (c) hn (d)Mc 2 - hn
315. The ratio of wavelengths of the last line of Balmer 324. The binding energy per nucleon of 73 Li and 42 He
series to the last line of Lyman series is
nuclei are 5.60 MeV and 7.06 MeV, respectively. In
[NEET 2017]
(a) 2 (b) 1 (c) 4 (d) 0.5
the nuclear reaction 73 Li + 11H ® 42He + 42He + Q, the
316. If an electron in a hydrogen atom jumps from the value of energy Q released is [CBSE-AIPMT 2014]
3rd orbit to the 2nd orbit, it emits a photon of (a) 19.6 MeV (b) - 2.4 MeV (c) 8.4 MeV (d) 17.3 MeV
wavelength l. When it jumps from the 4th orbit to
the 3rd orbit, the corresponding wavelength of the 325. For a radioactive material, half-life is 10 minutes.
photon will be [NEET 2016] If initially there are 600 number of nuclei, the time
16 9 20 20 taken (in minutes) for the disintegration of 450
(a) l (b) l (c) l (d) l nuclei is
25 16 7 13 [NEET 2018]
(a) 30 (b) 10 (c) 20 (d) 15
317. In the spectrum of hydrogen, the ratio of the
longest wavelength in the Lyman series to the 326. Radioactive material A has decay constant 8l and
longest wavelength in the Balmer series is material B has decay constant l. Initially, they
[CBSE-AIPMT 2015] have same number of nuclei. After what time, the
4 9 27 5 ratio of number of nuclei of material B to that A
(a) (b) (c) (d) 1
9 4 5 27 will be ? [NEET 2017]
318. Hydrogen atom in ground state is excited by a e
1 1 1 1
monochromatic radiation of l = 975 Å. Number of (a) (b) (c) (d)
spectral lines in the resulting spectrum emitted l 7l 8l 9l
will be [CBSE-AIPMT 2014] 327. The half-life of a radioactive substance is
(a) 3 (b) 2 (c) 6 (d) 10 30 minutes. The time (in minutes) taken between
319. An electron of a stationary hydrogen atom passes 40% decay and 85% decay of the same
from the fifth energy level to the ground level. The radioactive substance is [NEET 2016]
velocity that the atom acquired as a result of photon (a) 15 (b) 30 (c) 45 (d) 60
emission will be [CBSE-AIPMT 2012]
328. A radio isotope X with a half-life 1.4 ´ 109 yr decays
24 hR 25 hR 25 m 24 m
(a) (b) (c) (d) into of Y which is stable. A sample of the rock from
25 m 24 m 24 h R 25 h R a cave was found to contain X and Y in the ratio
(where, m is the mass of electron, R is Rydberg 1 : 7. The age of the rock is [CBSE-AIPMT 2014]
constant and h is Planck’s constant.) . ´ 109 yr
(a) 196 (b) 3.92 ´ 109 yr
320. The ionisation energy of the electron in the (c) 4 .20 ´ 109 yr (d) 8.40 ´ 109 yr
hydrogen atom in its ground state is 13.6 eV. The
atoms are excited to higher energy levels to emit
329. Two radioactive substances A and B have decay
constants 5 l and l respectively. At t = 0, they have
radiations of 6 wavelengths. Maximum wavelength of the same number of nuclei. The ratio of number of
emitted radiation corresponds to the transition 2
between æ1ö
[CBSE-AIPMT 2009] nuclei of A to those of B will be ç ÷ after a time
(a) n = 3 to n = 2 states (b) n = 3 to n = 1 states è eø
(c) n = 2 to n = 1 states (d) n = 4 to n = 3 states interval [CBSE-AIPMT 2007]
1 1
(a) (b) 4l (c) 2 l (d)
4l 2l
27. Nuclei
27
321. If radius of the 13 Al nucleus is taken to be RAl ,
28. Semiconductor Electronics
then the radius of 125
53 Te nucleus is nearly 330. Carbon, silicon and germanium atoms have four
1
[CBSE-AIPMT 2015]
1
valence electrons each. Their valence and
conduction bands are separated by energy band
(a) æç ö÷ RAl (d) æç ö÷ R Al
53 3 5 3 13 3
(b) R Al (c) R AI gaps represented by ( E g )C , ( E g )Si and ( E g )Ge
è 13 ø 3 5 è 53 ø
respectively. Which one of the following
322. The radius of germanium (Ge) nuclide is measured relationships is true in their case? [CBSE-AIPMT 2005]
to be twice the radius of 94Be. The number of (a) (Eg )C > (Eg ) Si (b) (Eg )C = (Eg ) Si
nucleons in Ge are
(c) (Eg )C < (Eg ) Ge (d) (Eg )C < (Eg ) Si
[CBSE-AIPMT 2006]
(a) 73 (b) 74 (c) 75 (d) 72

MODULE 3
www.jeebooks.in
The NEET Edge ~ Physics 375

331. C and Si both have same lattice structure, having 336. The given graph represents V-I characteristic for a
4 bonding electrons in each. However, C is semiconductor device. Which of the following
insulator whereas Si is intrinsic semiconductor. statement is correct? [CBSE-AIPMT 2014]
This is because [CBSE-AIPMT 2012]
(a) in case of C, the valence bond is not completely filled at I
absolute zero temperature A
(b) in case of C, the conduction band is partly filled even at V
absolute zero temperature B
(c) the four bonding electrons in the case of C lie in the
second orbit, whereas in the case of Si they lie in the (a) It is V-I characteristic for solar cell where point A represents
third open circuit voltage and point B short circuit current
(d) the four bonding electrons in the case of C lie in the third (b) It is for a solar cell and points A and B represent open
orbit, whereas for Si they lie in the fourth orbit circuit voltage and current, respectively
(c) It is for a photodiode and points A and B represent open
332. If a small amount of antimony is added to
circuit voltage and current, respectively
germanium crystal [CBSE-AIPMT 2011]
(d) It is for a LED and points A and B represent open circuit
(a) the antimony becomes an acceptor atom voltage and short circuit current, respectively
(b) there will be more free electrons than holes in the
semiconductor 337. A p-n photodiode is made of a material with a band
(c) its resistance is increased gap of 2.0 eV. The minimum frequency of the
(d) it becomes a p-type semiconductor radiation that can be absorbed by the material is
nearly [CBSE-AIPMT 2008]
333. In a p-n junction diode, change in temperature due (a) 10 ´ 1014 Hz (b) 5 ´ 1014 Hz(c) 1 ´ 1014 Hz (d) 20 ´ 1014 Hz
to heating [NEET 2018]
(a) does not affect resistance of p -n junction 338. Application of a forward bias to a p-n junction
(b) affects only forward resistance [CBSE-AIPMT 2005]
(c) affects only reverse resistance (a) increases the number of donors on the n-side
(d) affects the overall V-I characteristics of p - n junction (b) increases the electric field in the depletion zone
(c) increases the potential difference across the depletion zone
334. The given circuit has two ideal diodes connected as
(d) widens the depletion zone
shown in the figure. The current flowing through
the resistance R1 will be [NEET 2016] 339. In the circuit shown in the figure, the input voltage
2Ω V i is 20 V, V BE = 0 and VCE = 0. The values of I B , IC
and b are given by [NEET 2018]
R1 D2 20 V
D1
10 V
R2 RC 4 kΩ
3Ω R3 2Ω
C
RB
(a) 2.5 A (b) 10.0 A Vi
(c) 1.43 A (d) 3.13 A 500 kΩ B
E
335. If in a p-n junction, a square input signal of 10 V is
applied as shown in the figure [CBSE-AIPMT 2015]

+5 V

RL (a) IB = 20 mA, IC = 5 mA, b = 250


(b) IB = 25 mA, IC = 5 mA, b = 200
–5 V (c) IB = 40 mA, IC = 10 mA,b = 250
(d) IB = 40 mA, IC = 5 mA, b = 125
Then, the output across RL will be
340. In a common emitter transistor amplifier, the
10 V audio signal voltage across the collector is 3 V. The
(a) (b) resistance of collector is 3 kW. If current gain is 100
–10 V and the base resistance is 2 kW, the voltage and
power gain of the amplifier is [NEET 2017]
5V
(c) –5 V (d) (a) 200 and 1000 (b) 15 and 200
(c) 150 and 15000 (d) 20 and 2000

MODULE 3
www.jeebooks.in
376 NEET Test Drive

341. A n-p-n transistor is connected in common emitter 348. A transistor is operated in common-emitter
configuration in a given amplifier. A load configuration at constant collector voltage
resistance of 800 W is connected in the collector V c = 1.5 V, such that a change in the base current
circuit and the voltage drop across it is 0.8V. If the from 100 mA to 150 mA produces a change in the
current amplification factor is 0.96 and the input collector current from 5 mA to 10 mA. The current
resistance of the circuits is 192 W, the voltage gain gain b is [CBSE-AIPMT 2006]
and the power gain of the amplifier will (a) 67 (b) 75 (c) 100 (d) 50
respectively be [NEET 2016]
349. A transistor-oscillator using a resonant circuit with
(a) 3.69, 3.84 (b) 4, 4 (c) 4, 3.69 (d) 4, 3.84
an inductor L (of negligible resistance) and a
342. For CE transistor amplifier, the audio signal capacitor C in series produce oscillations of
voltage across the collector resistance of 2 kW is frequency f. If L is doubled and C is changed to
4 V. If the current amplification factor of the 4 C, the frequency will be [CBSE-AIPMT 2006]
transistor is 100 and the base resistance is l kW, f
(a) (b) 8 f
then the input signal voltage is [NEET 2016] 4
(a) 10 mV (b) 20 mV (c) 30 mV (d) 15 mV f f
(c) (d)
2 2 2
343. The input signal given to a CE amplifier having a
æ pö 350. The output X of the logic circuit shown in figure
voltage gain of 150 is V i = 2 cos ç15 t + ÷. The
è 3ø will be [NEET 2013]

corresponding output signal will be A


X
[CBSE-AIPMT 2015] B
p 2p ö
(a) 300 cos æç15 t + ö÷ (b) 75 cos æç15 t + ÷
è 3ø è 3ø (a) X = A × B (b) X = A × B
æ 5p ö 4p ö
(c) 2 cos ç15 t + ÷ (d) 300 cos æç15 t + ÷ (c) X = A × B (d) X = A + B
è 3ø è 3ø
351. To get an output Y = 1 from the circuit shown
344. In a common emitter CE amplifier having a below, the input must be [CBSE-AIPMT 2010]
voltage gain G, the transistor has
A
transconductance 0.03 mho and current gain 25. If
B Y
the above transistor is replaced with another one C
with transconductance 0.02 mho and current gain
20, the voltage gain will [NEET 2013] A B C A B C
2 1 5 (a) 0 1 0 (b) 0 0 1
(a) G (b) 1.5 G (c) G (d) G (c) 1 0 1 (d) 1 0 0
3 3 4
345. In a CE transistor amplifier, the audio signal 352. The following figure shows a logic gate circuit with
voltage across the collector resistance of 2 kW is 2V. two inputs A and B and the output C. The voltage
If the base resistance is 1 kW and the current waveforms of A, B and C are as shown below
amplification of the transistor is 100, the input A Logic gate
signal voltage is [CBSE-AIPMT 2012] C
B circuit
(a) 0.1 V (b) 1.0 V
(c) 1 mV (d) 10 mV 1
346. A common emitter amplifier has a voltage gain of t
A
50, an input impedance of 100 W and an output
impedance of 200 W. The power gain the 1
amplifier is [CBSE-AIPMT 2010, 07] B t
(a) 500 (b) 1000 (c) 1250 (d) 50 1
347. A transistor is operated in common-emitter
t
configuration at VC = 2 V, such that a change in C
the base current from 100 mA to 200 mA produces a
change in the collector current from 5 mA to The logic circuit gate is [CBSE-AIPMT 2006]
10 mA. The current gain b is [CBSE-AIPMT 2009] (a) AND gate (b) NAND gate
(a) 75 (b) 100 (c) 150 (d) 50 (c) NOR gate (d) OR gate

MODULE 3
www.jeebooks.in

Answer Sheet
1. (d) 2. (a) 3. (b) 4. (a) 5. (c) 6. (a) 7. (d) 8. (b) 9. (a) 10. (c)
11. (d) 12. (a) 13. (c) 14. (b) 15. (b) 16. (d) 17. (b) 18. (d) 19. (b) 20. (a)
21. (c) 22. (c) 23. (c) 24. (a) 25. (a) 26. (c) 27. (c) 28. (b) 29. (a) 30. (a)
31. (c) 32. (a) 33. (c) 34. (a) 35. (b) 36. (c) 37. (c) 38. (c) 39. (c) 40. (d)
41. (b) 42. (d) 43. (b) 44. (b) 45. (a) 46. (c) 47. (b) 48. (d) 49. (c) 50. (a)
51. (a) 52. (d) 53. (a) 54. (c) 55. (a) 56. (a) 57. (b) 58. (c) 59. (a) 60. (b)
61. (b) 62. (b) 63. (a) 64. (a) 65. (d) 66. (a) 67. (d) 68. (a) 69. (a) 70. (d)
71. (d) 72. (b) 73. (d) 74. (b) 75. (d) 76. (b) 77. (a) 78. (a) 79. (c) 80. (b)
81. (d) 82. (b) 83. (c) 84. (c) 85. (d) 86. (d) 87. (b) 88. (b) 89. (d) 90. (d)
91. (a) 92. (a) 93. (c) 94. (b) 95. (b) 96. (a) 97. (c) 98. (d) 99. (c) 100. (a)

101. (c) 102. (d) 103. (c) 104. (b) 105. (c) 106. (b) 107. (b) 108. (c) 109 (b) 110. (b)
111. (b) 112. (b) 113. (d) 114. (a) 115. (b) 116. (a) 117. (b) 118. (b) 119 (b) 120. (a)
121. (a) 122. (c) 123. (c) 124. (d) 125. (b) 126. (c) 127. (a) 128. (c) 129. (b) 130. (a)
131. (d) 132. (c) 133. (b) 134. (c) 135. (d) 136. (d) 137. (d) 138. (c) 139. (b) 140. (b)
141. (c) 142. (b) 143. (b) 144. (b) 145. (d) 146. (a) 147. (a) 148. (d) 149. (c) 150. (b)
151. (d) 152. (c) 153. (b) 154. (c) 155. (b) 156. (d) 157. (d) 158. (b) 159. (b) 160. (d)
161. (c) 162. (a) 163. (b) 164. (b) 165. (a) 166. (a) 167. (a) 168. (c) 169. (b) 170. (b)
171. (a) 172. (a) 173. (b) 174. (c) 175. (c) 176. (b) 177. (c) 178. (b) 179. (b) 180. (c)
181. (a) 182. (b) 183. (a) 184. (d) 185. (a) 186. (a) 187. (c) 188. (d) 189. (c) 190. (c)
191. (d) 192. (d) 193. (d) 194. (c) 195. (c) 196. (c) 197. (c) 198. (a) 199. (b) 200. (c)

201. (d) 202. (b) 203. (b) 204. (c) 205. (c) 206. (a) 207. (b) 208. (c) 209. (d) 210. (a)
211. (c) 212. (c) 213. (b) 214. (c) 215. (b) 216. (d) 217. (c) 218. (d) 219. (c) 220. (d)
221. (d) 222. (b) 223. (c) 224. (b) 225. (a) 226. (d) 227. (d) 228. (a) 229. (d) 230. (b)
231. (b) 232. (a) 233. (c) 234. (a) 235. (c) 236. (b) 237. (c) 238. (b) 239. (a) 240. (b)
241. (a) 242. (c) 243. (c) 244. (c) 245. (c) 246. (d) 247. (c) 248. (c) 249. (c) 250. (d)
251. (b) 252. (a) 253. (c) 254. (c) 255. (c) 256. (a) 257. (a) 258. (*) 259. (c) 260. (b)
261. (b) 262. (b) 263. (c) 264. (a) 265. (c) 266. (a) 267. (b) 268. (d) 269. (c) 270. (c)
271. (c) 272. (a) 273. (b) 274. (b) 275. (a) 276. (b) 277. (a) 278. (c) 279. (a) 280. (d)
281. (c) 282. (d) 283. (b) 284. (b) 285. (d) 286. (c) 287. (a) 288. (c) 289. (a) 290. (b)
291. (c) 292. (a,b) 293. (c) 294. (b) 295. (b) 296. (c) 297. (a) 298. (d) 299. (a) 300. (b)

301. (b) 302. (d) 303. (a) 304. (a) 305. (b) 306. (b) 307. (d) 308. (c) 309. (b) 310. (d)
311. (b) 312. (b) 313. (b) 314. (d) 315. (c) 316. (c) 317. (d) 318. (c) 319. (a) 320. (d)
321. (b) 322. (d) 323. (a) 324. (d) 325. (c) 326. (b) 327. (d) 328. (c) 329. (d) 330. (a)
331. (c) 332. (b) 333. (d) 334. (a) 335. (d) 336. (a) 337. (b) 338. (a) 339. (d) 340. (c)
341. (d) 342. (b) 343. (d) 344. (a) 345. (d) 346. (c) 347. (d) 348. (c) 349. (c) 350. (c)
351. (c) 352. (a)

MODULE 3
www.jeebooks.in
378 NEET Test Drive

EXPLANATIONS
mv
and é 2 ù = 2 -2 = [L-2 T 2 ]
1 1
1. Units and Measurements êë c úû [L T ]
...(iii) 7. We know that, F = ma Þ F =
t
1. Given, least count of screw gauge Ft
Now, checking optionwise, Þ m=
(LC) = 0.001 cm 1/ 2 v
Main scale reading 1 æ Ge 2 ö [F][T]
= ç ÷ [M ] = = [Fv -1T ]
(MSR) = 5 mm = 0.5 cm c 2 è 4 pe0 ø [v]
Number of coinciding divisions on the = [L-2 T 2 ] [L6 T -4 ]1/ 2 = [L]. 8. As we know that,
circular scale, i.e. Vernier scale reading 5. We know that, Dimensions of e0 = [M-1L-3 T 4A 2 ]
(VSR) = 25
Surface tension S =
Force, F Dimensions of E = [MLT -3 A -1]
Here, zero error = -0.004 cm Length, L 1
Final reading obtained from the screw Dimensions of e0E 2
[MLT -2 ] 2
gauge is given as So, [S] = = [ML0 T -2 ] = [M-1L-3 T 4 A 2 ] ´ [MLT -3 A -1] 2
[L]
= MSR + VSR ´ LC - zero error = [ML-1T -2 ].
Final reading from the screw gauge Energy E = Force ´ Displacement
9. (i) Dimensions of velocity = [M0L1T -1]
= 0.5 + 25 ´ 0.001 - (-0.004). Þ E = [ML2 T -2 ]
Displacement Here, a = 0, b = 1, c = - 1
= 0.5 + 0.025 + 0.004 Velocity v =
Time (ii) Dimensions of acceleratio n
= 0.5 + 0.029 = 0.529 cm = [M0L1T -2 ]
Þ [v ] = [LT -1]
Thus, the diameter of the ball is 0.529 cm. Here, a = 0, b = 1, c = - 2
As, S µ E a v bT c
a 3b 2 (iii) Dimensions of force = [M1L1T -2 ]
2. As given, P = where, a, b, c are constants.
cd Here, a = 1, b = 1, c = - 2
DP From the principle of homogeneity,
\ ´ 100 (iv) Dimensions of pressure = [M1L-1T -2 ]
[LHS] = [RHS]
P \ Here, a = 1, b = - 1, c = - 2
D 2 Db Dc Dd ö Þ [ML0 T -2 ] = [ML2 T -2 ] a [LT -1]b [T]c
= æç
3 a
+ + + ÷ ´ 100 \ The physical quantity is pressure.
è a b c d ø Þ [ML0 T -2 ] = [MaL2 a + b T -2 a - b + c ] Energy E
Da Db Dc 10. Energy density = Þ u=
=3 ´ 100 + 2 ´ 100 + ´ 100 Equating the power on both sides, Volume V
a b c
we get a = 1, 2a + b = 0, b = - 2 Dimensions of u =
Dimensions of E
Dd
+ ´ 100 Þ - 2a - b + c = - 2 Dimensions of V
d
Þ c = (2a + b) - 2 = 0 - 2 = - 2 [ML2 T -2 ]
= 3 ´ 1+ 2 ´ 2 + 3 + 4 = = [ML-1T -2 ]
So, [S] = [Ev - 2 T - 2 ] [L3 ]
= 3 + 4 + 3 + 4 = 14%
4 6. According to principle of homogeneity of Refractive index is a dimensionless
3. Volume of a sphere,V = pr 3 dimension a physical quantity equation quantity.
3
will be dimensionally correct, if the Dielectric constant is a dimensionless
DV 3 ´ Dr
\ ´ 100 = ´ 100 dimensions of all the terms occurring on quantity.
V r both sides of the equations are same.
Dr Young’s modulus
Here ´ 100 = 2% Given, critical velocity of liquid flowing
Longitudinal stress F /A
r through a tube are expressed as = = = [ML-1T -2 ]
Longitudinal strain Dl
DV vc µ hxr yr z
\ ´ 100 = 3 ´ 2% = 6%. l
V Coefficient of viscosity of liquid, Force F
e2 e2 Magnetic field = =
4. As force, F = Þ = r 2 ×F h = [ML -1T -1] Charge ´ Velocity qv
4pe0r 2 4pe0
Density of liquid, r = [ML -3 ] [MLT -2 ]
Putting dimensions of r and F, we get = = [MT -2 A -1].
Radius of a tube, r = [L] [AT] [LT -1]
é e2 ù 3 -2 Critical velocity of liquid, vc = [M0L1T - 1]
⇒ ê ú = [ML T ] ...(i) 11. According to Ohm’s law,V µ I
ë 4pe 0 û Þ [M 0 L1T -1] = [ML - 1T - 1] x [ML - 3 ] y [L] z
and V = IR
Gm 2 [M 0 L1T - 1] = [M x + y
L- x - 3y + z
T- x]
Also, Force, F = Potential difference
r2 Comparing exponents of M, L and T, Resistance, R =
Current
we get
[MLT -2 ] [L2 ] V W
⇒ [G ] = x + y = 0, - x - 3 y + z = 1, = =
[M2 ] I qI
- x = -1
⇒ [G ] = [M-1L3 T -2 ] ...(ii) æQPotential difference is equal ö
Þ x = 1, y = - 1, z = - 1 ç ÷
è to the work done per unit chargeø

MODULE 3
www.jeebooks.in
The NEET Edge ~ Physics 379

So, dimensions of R 16. Velocity of each car is given by 1 100


s2 = 0 + g (10) 2 = g
Dimensions of work dx p (t ) 2 2
= vP = = a + 2bt
Dimensions of charge dt \ Distance covered in second 5 s,
´ Dimensions of current 100 25 75
dx (t ) h2 = s 2 - h1 = g - g = g ...(ii)
and vQ = Q = f - 2t
[ML2 T -2 ] dt 2 2 2
= = [ML2 T -3I-2 ].
[IT][I] It is given that, vP = vQ Distance covered in first 15 s,
1 225
12. The given expression is Þ a + 2bt = f - 2t s 3 = 0 + g (15) 2 = g
b f -a 2 2
v = at + Þ t =
t +c 2 (b + 1) \ Distance covered in last 5 s,
h3 = s 3 - s 2
-2 n
From principle of homogeneity 17. Given, v = bx 225 100 125
= g - g = g ...(iii)
[a] [t ] = [v] dv dx dv 2 2 2
–1 a= = ×
[v] [LT ] dt dt dx
[a] = = = [LT –2 ] From Eqs. (i), (ii) and (iii), we get
[t ] [T] dv
Þ a=v = (bx -2 n )(- 2n bx -2 n - 1) h1 : h2 : h3 =
25
g:
75
g:
125
g = 1: 3 : 5
[b ] dx 2 2 2
Further, = [v]
[t + c] Þ a = - 2nb 2 x - 4 n - 1 h h
Þ h1 = 2 = 3
or [b] = [v] [t + c] 18. Acceleration, 3 5
or –1
[b] = [LT ] [T] = [L] f = f0 æç1 - ö÷
t 20. For first ball, u = 0
è Tø 1 2 1
[c] = [T] \ s1 = gt1 = ´ g (18) 2
= f0 æç1 - ö÷ éQf =
dv t dv ù 2 2
or f =
dt è T ø êë dt úû For second ball, initial velocity = v
2. Motion in a Straight Line
or dv = f0 æç1 - ö÷ dt
t 1
…(i) \ s 2 = vt 2 + gt 22
13. Speed of walking =
h
= v1 è Tø 2
t1 t 2 = 18 - 6 = 12 s
Integrating Eq. (i) on both sides, we get
h æ tö 1
Speed of escalator = = v2 Þ s 2 = v ´ 12 + g (12) 2
t2 ò dv = ò f0 çè1 - T ÷ø dt 2
Time taken when she walks over running f0 t 2 Here, s1 = s 2
escalator \ v = f0 t - × +c …(ii) 1 1
T 2 Þ g (18) 2 = 12 v + g (12) 2
h tt 2 2
Þ t = ⇒ t = 12 where, c is constant of integration.
v1 + v 2 t1 + t 2 Þ v = 75 ms -1
Now, when t = 0, v = 0
14. Maximum velocity point means, the point 21. From equation of motion time taken by
dx So, from Eq. (ii), we get c = 0
at which i.e. the slope of the graph ball to reach maximum height,
f0 t 2
dt \ v = f0 t - × …(iii) v = u - gt
is maximum. T 2
At maximum height, final speed is zero,
f = f0 æç1 - ö÷
At point C, slope is maximum. t i.e. v = 0
As,
è Tø u
15. Velocity of the particle is given as So, u = gt or t =
v = At + Bt 2 When,f = 0 g

0 = f0 æç1 - ö÷
where, A and B are constants. t In 2s , u = 2 ´ 9.8 = 19.6 m/s
dx éQ v = dx ù è Tø If man throws the ball with velocity of
Þ = At + Bt 2
dt êë dt úû 19.6 m/s then after 2 s it will reach the
As, f0 ¹ 0
maximum height. When, he throws 2nd
Þ dx = (At + Bt 2 ) dt t
So, 1 - = 0 ball, 1st is at top. When he throws 3rd
Integrating both sides, we get T
ball, 1st will come to ground and 2nd will
x2 2 \ t =T be at the top. Therefore, only 2 balls are
òx dx = ò1 (At + Bt 2 ) dt
1 Substituting, t = T in Eq. (iii), we get in air. If he wants to keep more than 2
Þ Dx = x 2 - x1 f0 T 2
balls in air he should throw the ball with a
2 2 2
vx = f0T - × speed greater than 19.6 m/s.
=A ò1 t dt +B ò1 t dt T 2
22. Distance travelled in n th second is
fT 1
2 2 = f0T - 0 = f0T given by
ét 2 ù ét 3 ù 2 2
=Aê ú +B ê ú 1
sn = u + a (2 n - 1)
ë 2 û1 ë 3 û1 19. For free fall from a height, u = 0 2
A 2 B 3 4
= (2 - 12 ) + (2 - 13 ) \ Distance covered by stone in first 5 s, Here, u = 0, a =
2 3 1 25 3
h1 = 0 + g (5) 2 = g ...(i) 1 4
\ Distance travelled between 1s and 2s is 2 2 \ s3 = 0 + ´ ´ (6 - 1)
2 3
3 A 7B \ Distance covered in first 10 s,
= + 10
2 3 = m
3

MODULE 3
www.jeebooks.in
380 NEET Test Drive

r1 - r2 (v - v1) t 28. It is clear from the diagram that the


3. Motion in a Plane So, = 2
| r1 - r2 | | v 2 - v1| t shortest distance between the ship A
2 r1 - r2 (v - v1) and B is PQ
23. Given, x = 5t - 2t = 2 N
| r1 - r2 | | v 2 - v1|
Velocity of the particle,
dx d 26. For perpendicular vector, we have
vx = = (5t - 2t 2 ) = 5 - 4t
dt dt A ×B = 0
d [cos wt $i + sin wt $j] vA =10 km/h
Acceleration, ax = v x = - 4 ms -2 W E
dt é cos wt $i + sin wt $j ù = 0 O
Also, y = 10t êë 2 2 úû 45°
P
dy wt wt 100 km
Velocity, vy =
= 10 Þ cos wt cos + sin wt sin =0
dt 2 2 vB =10 km/h
dv y [Qcos (A - B )
\ Acceleration, a y = =0 Q
dt = cos A cos B + sin A sinB] PQ
\ Net acceleration of the particle, Here, sin 45 º =
wt ö
Þ cos æç wt - ÷=0 OQ
a net = ax $i + ay $j = (-4 ms -2 ) $i è 2 ø 1
Þ PQ = 100 ´ = 50 2 m
or a = - 4 $i ms -2
net wt 2
Þ cos =0
24. Suppose two vectors are P and Q. 2 Also, v AB = v A2 + vB2
It is given that, wt p
Þ =
2 2 = 102 + 102
| P+ Q | = | P- Q |
p = 10 2 km/h
Let angle between P and Q is f Þ t =
2 2 w So, time taken by them to reach shortest
\ P +Q + 2PQ cos f
Thus, time taken by vectors which are path is
= P 2 + Q 2 - 2PQ cos f p
orthogonal to each other is . PQ 50 2
Þ 4PQ cos f = 0 w t = = =5h
v AB 10 2
Þ cos f = 0 [Q P , Q ¹ 0] 27. (i) The position vector of a particle R
p as a function of time is given by
29. The trajectory of a projectile projected at
Þ f = or 90° some angle q with the horizontal direction
2 R = 4 sin (2 pt ) $i + 4 cos (2 pt )$j from ground is given by
25. For two particles A and B move with X -axis component, gx 2
constant velocities v1 and v2 . Such y = x tan q -
x = 4 sin 2 pt …(i) 2u cos 2 q
2
that, two particles to collide, the
direction of the relative velocity of one Y -axis component, For equal trajectories for same angle of
with respect to other should be directed y = 4 cos 2 pt …(ii) projection,
towards the relative position of the other g
Squaring and adding both = constant
particle.
equations, we get x 2 + y 2 u2
r - r2 9. 8 g¢
i.e., 1 ¾® direction of relative = 4 2 [sin2 (2 pt ) + cos 2 (2 pt )] Þ = 2
| r1 - r2 | 52 3
i.e. x 2 + y 2 = 4 2 , i.e. equation of 9.8 ´ 9
position of 1 w.r.t. 2
circle and radius is 4 m. g¢ = = 3.5 ms -2
v - v1 25
Similarly, 2 ¾® direction of v2
| v 2 - v 1| (ii) Acceleration vector, a = (-R$ ), 30. Horizontal range of projectile,
R
velocity of 2 w.r.t. 1. u 2 sin 2 a
where, v is the velocity of particle. R =
So, for collision of A and B, g
(iii) Magnitude of acceleration vector,
r -r v - v1 v2 For a = (45° – q),
we get 1 2 = 2 a=
| r1 - r2 | | v 2 - v 1| R u 2 sin 2 (45° – q)
R1 =
Alternate Method As resultant (iv) As, we have v x = + 4 (cos 2 pt ) 2 p g
displacement of a particle, and v y = - 4 (sin 2 pt ) 2 p u 2 sin (90° – 2 q) u 2 cos 2 q
v2 t = =
Net resultant velocity, g g

v = v x2 + v y2 For a = (45° + q),


r1 R u 2 sin 2 (45° + q)
v1t R2 =
A g
= (8 p) 2 (cos 2 2 p t + sin2 2 p t )
u 2 sin (90° + q) u 2 cos 2 q
O B v = 8p [Q cos 2 2 pt + sin2 2 pt = 1] = =
r2 g g
So, option (c) is incorrect. R1 1
R = r1 - v1t = r2 - v 2 t Hence, = or R1 : R 2 = 1 : 1
R2 1
i.e. r1 - r2 = (v 2 - v1) t

MODULE 3
www.jeebooks.in
The NEET Edge ~ Physics 381

31. Centripetal acceleration of a particle moving Breaking all the forces in X-Y axis 2 mv sin 30°
or F =
v2 Total force along + X -axis t
on a circular path is given by ac =
R = 1cos 60° + 2 sin 30° Given, m = 05
. kg, v = 12 m / s, t = 0.25 s
In the given figure, ac = a cos 30° q = 30°
along -X -axis = 4 sin 30°
= 15 cos 30° m/s 2 2 ´ 05
. ´ 12 sin 30°
along +Y -axis = 4 cos 30° + 1sin 60° Hence, F = = 24 N
v2 0.25
Þ = 15 cos 30° along -Y -axis = 2 cos 30°
R 36. Free body diagram of block A,
Þ Net force along X -axis
2 3 a
Þ v = R ´ 15 ´ = - (1cos 60° + 2 sin 30° ) + 4 sin 30°
2
Þ - æç + 2 ´ ö÷ + 4 ´
1 1 1
3 T m1
= 2.5 ´ 15 ´ è2 2ø 2
2 fk
-3 1
\ v = 5.7 m/s Þ + 2=+
2 2 T - m1a = fk …(i)
32. Given, that two stones of masses m and Net force along Y-axis Free body diagram of block B,
2 m are whirled in horizontal circles, the
r = 4 cos 30° + 1sin 60° - 2 cos 30° T
heavier one in a radius and lighter one
2 3 3 3
Þ 4´ + -2´
in radius r as shown in figure 2 2 2
5 3 2 3 3 3 m2
v2 = - = a
2 2 2
r/2 2m To have, resultant only inY -axis we
1
must have N (0.5 N) force towards
2 m2g
r + X -axis, so that it can compensate
v1 the net force of -X -axis. m 2g - T = m 2a …(ii)
m
35. The vector OA represents the Adding Eqs. (i) and (ii), we get
As, speed of lighter stone is n times that momentum of the object before the m 2g - m1a = m 2a + fk
of the value of heavier stone when they
collision and the vector OB that after Þ m 2g - m1a = m 2a + m km1g
experience same centripetal forces, we
the collision. The vector AB represents
get (m 2 - m km1)g
the change in momentum of the object Þ a=
(Fc ) heavier = (Fc ) lighter m1 + m 2
Dp.
2m (v ) 2 m (nv ) 2 From Eq. (ii),
Þ = B
(r / 2 ) r é (m 2 - m km1) ù
OB cos 30° T = m 2 (g - a) = m 2 ê1 - g
ë m1 + m 2 úû
Þ n2 = 4 Þ n = 2
30°

m m (1 + m k )
33. Since, speed is constant throughout the T = 1 2 g
OB sin 30° m1 + m 2
motion, so it is a uniform circular motion. 60°
O
Therefore, its radial acceleration is given by 60° 37. First of all consider the forces on the
2 OA sin 30° blocks
ar = rw
30°

a
2 2 2
2 pn ö 4p n
= r æç ÷ =r ´ OA cos 30°
è t ø t2 T1 2 T2 T3 3
A
1 ´ 4 ´ p ´ (22) 2 2 m m
= = p 2 m/ s 2
(44) 2 As the magnitudes of OA and OB are µmg µmg
equal, the components of OA and OB
This acceleration is directed along a T1
along the wall are equal and in the same
radius of circle.
direction, while those perpendicular to m 1
the wall are equal and opposite. Thus,
4. Newton’s Laws of Motion the change in momentum is only due to mg
y 4 cos 30° + 1 sin 60° the change in direction of the
34. For the Ist block, [Qm1 = m 2 = m 3 ]
1N perpendicular components.
mg - T1 = m ´ a …(i)
4N Hence,
Dp = OB sin 30° – (– OA sin 30° ) Let us consider 2nd and 3rd block as a
30° system
= mv sin 30° – (– mv sin 30° )
60° So, T1 - 2 mmg = 2m ´ a …(ii)
x′ x = 2 mv sin 30°
Adding Eqs. (i) and (ii), we get
1 cos 60° + 2 sin 30° It is time rate of change of momentum
30° that will appear in the form of average mg (1 - 2 m) = 3m ´ a
2N force acting on the wall. g
Þ a = (1 - 2 m)
\ F ´ t = 2 mv sin 30° 3
y′

MODULE 3
www.jeebooks.in
382 NEET Test Drive

38. Net work done by the block in going Thus, from Eqs. (i) and (ii), we get 43. Initially system, is in equilibrium with a
from top to bottom of the inclined plane, Mg = ms mg or M = ms m total weight of 4mg over spring.
must be equal to the work done by
Given, ms = 0.2, m = 2 kg
frictional force.
\ M = 0.2 ´ 2 = 0.4 kg kx
2 41. Maximum bearable tension in the rope, (3m+m)
R L/
A 3m
T Cutting
2
L/ in θ 4 mg plane
s
θ
B m
mg Monkey
mg cos θ
θ \ kx = 4mg
mg
a
When, string is cut at the location as
The block may be stationary, when shown above.
L Free body diagram for m is
mg sin q × L = m mg cos q
2 mg
m
mg sin q × L
or m=
mg cos q × (L / 2) T = 25 ´ 10 = 250 N
sin q mg
=2 = 2 tan q From the figure,
cos q T - mg = ma So,force on mass m = mg
m = 2 tan q T - mg \ Acceleration of mass, (m) aB = g
or acceleration, a =
39. When a cart moves with some m For mass 3m, free body diagram is
acceleration towards right, then a Given, mass m = 20 kg,
kx=4mg
pseudo force ma acts on block towards g = 10 m/ s 2 ,
left. This force ma is action force by a
Taking, T = 250 N
block on cart a 3m
250 - 20 ´ 10 50
F r = µR Hence, a = =
20 20
= 2 .5 m/ s 2
3 mg
ma m R 42. According to the question, the free body
If aA = acceleration of block of mass 3m,
diagram of the given condition will be
then
A
R cosθ F net = 4mg - 3mg
mg R g
θ ⇒ 3m × a A = mg or aA =
Now, block will remain static w.r.t. cart, 3
if frictional force mR ³ mg ma R sinθ So, accelerations for blocks A and B are
(Pseudo
Þ m ma ³ mg [as, R = ma] θ g
force) aA =
g mg θ a 3
Þ a³
m B C and aB = g
40. Let the mass of the block B be M . Since, the wedge is accelerating 44. Position vector of the particle is given by
R
towards right with a, thus a pseudo force r = cos wt x$ + sin wt y$
acts in the left direction in order to keep where, w is a constant.
the block stationary. As, the system is in
T Velocity of the particle is
equilibrium.
fs = µmg 2 kg dr d
\ SF x = 0 v = = (cos wt x$ + sin wt y$
dt dt
mg T or SF y = 0 = (- sin wt ) wx$ + (cos wt ) wy$
M Þ R sin q = ma …(i) = - w (sin wt x$ - cos wt y$ )
Similarly, R cos q = mg …(ii) Acceleration of the particle,
Mg
dv
In equilibrium, Dividing Eq. (i) by Eq (ii), we get a =
dt
T - Mg = 0 Þ T = Mg …(i) R sin q ma a d
= Þ tan q = = [- w sin wt x$ + w cos wt y$ ]
If blocks do not move, thenT = fs R cos q mg g dt
where, fs = frictional force or a = g tan q = - w 2 cos wt x$ - w 2 sin wt y$
= msR = ms mg \ The relation between a and g for the = - w 2 (cos wt x$ + sin wt y$ )
\ T = ms mg …(ii) block to remain stationary on the wedge Þ a = - w 2r = w 2 (-r)
is a = g tan q.

MODULE 3
www.jeebooks.in
The NEET Edge ~ Physics 383

Assuming the particle as P , then its i.e. x1 = x 2 = x 1 1


Kf = mv ¢ 2 = m (2v 0 ) 2
position vector is directed as shown in 1 1 2 2
the diagram. So, WP = KP x 2 and WQ = KQ x 2 1
2 2 = 4 mv 02 = 2mv 02
Y P 2
WP K
\ = P >1 Þ WP > WQ 50. Kinetic energy is given by
r WQ KQ
1 1
E = mv 2 = (mv ) 2
In case (b), the spring force is same 2 2m
X i.e. F1 = F 2 = F But, mv = momentum of the particle = p
O
F F p2
So, x1 = , x2 = [\ F = k x ] \ E = or p = 2 mE
Therefore, acceleration is directed KP KQ 2m
towards -r that is towards ‘O’ (origin). 1 m1E 1
\ WP = KP x12 Therefore,
p1
=
Now, we have v × r = - w 2 p2 m2 E 2
-w(sin wt x$ - cos wt y$ )(cos wt x$ + sin wt y$ ) 1 F2 1F 2
= KP 2 = But it is given that, p1 = p 2
Þ - w[sin wt × cos wt + 0 + 0 2 KP 2 KP
\ m1E 1 = m 2E 2 [Q m1 > m 2 ]
- sin wt × cos wt ] 1 1 F2 1F 2
and WQ = KQ x 22 = KQ × 2 = Þ E1 < E 2
Þ - w[0] = 0 [Q x$ ^ y$ ] 2 2 KQ 2 KQ
51. Kinetic energy,
v^ r WP K 1
\ = Q < 1 Þ WQ < WP K = (p 2 ) or p = 2 mK
Thus, velocity is perpendicular to r. WQ KP 2m
45. According to question, a car is 47. Work done in moving the object from If kinetic energy of a body is increased
negotiating a curved road of radius R. x = 0 to x = 6 m, is given by area under by 300%, let its momentum becomes p ¢
the curve. 300
The road is banked at angle q and the K¢= K + K = 4K çæintial KE = K ÷ö
coefficient of friction between the tyres W = Area of square + area of triangle 100 è final KE = K ¢ ø
of car and the road is ms . So, this given 1 Therefore, momentum is given by
=3´3+ ´3´3
2
situation can be drawn as shown in p ¢ = 2m ´ 4K æçInitial momentum = p ö÷
figure below. = 9 + 4.5 = 13 .5 J èFinal momentum = p ¢ø
N cosθ 48. Given, mass of particle m = 0.01 kg = 2 2mK = 2 p
N
Radius of circle along which particle is Hence, percentage change (increase) in
θ moving, r = 6.4 cm momentum
N sinθ
Dp p¢- p
f1 cosθ Kinetic energy of particle, KE ´ 100 = ´ 100
p p
f1
θ = 8 ´ 10-4 J
1 æp¢ ö
f1 sinθ Þ mv 2 = 8 ´ 10-4 J = ç - 1÷ ´ 100
θ 2 èp ø
mg 16 ´ 10-4 æ 2p ö
Þ v2 = = 16 ´ 10-2 …(i) =ç - 1÷ ´ 100 = 100%
Considering the case of vertical 0.01 è p ø
equilibrium, As it is given that KE of particle is equal 52. By work-KE theorem, we have change in
N cos q = mg + f1 sin q to 8 ´ 10-4 J by the end of second KE = work done by all of the forces.
Þ mg = N cos q - f1 sin q …(i) revolution after the beginning of motion
Work done by gravitational force,
of particle. It means, its initial velocity u
Considering the case of horizontal Wg = mgh = 10-3 ´ 10 ´ 1 ´ 103 = 10 J
is 0 m/s at this moment.
equilibrium,
Q By Newton’s 3rd equation of motion, Now, from work-KE theorem, we have
mv 2
N sin q + f1 cos q = …(ii) v 2 = u 2 + 2a t s DK = Wgravity + Wair resistance
R
2 1
Divide Eqs. (i) and (ii), we get Þ v = 2a t s or v 2 = 2a t (4 pr ) ⇒ ´ mv 2 = mgh + Wair resistance
2
v2 sin q + ms cos q (Q particle covers 2 revolutions) 1
= [f1 µ ms ] ⇒Wair resistance = mv 2 - mgh
Rg cos q - ms sin q v2 16 ´ 10-2 2
Þ at = =
8pr 8 ´ 3.14 ´ 6.4 ´ 10-2 = 10-3 æç ´ 50 ´ 50 - 10 ´ 103 ö÷
1
æ sin q + ms cos q ö è2 ø
Þ v = Rg ç ÷ (Qfrom Eq. (i), v 2 = 16 ´ 10-2 )
è cos q - ms sin q ø
\ . m/s 2
a t = 01 = -8.75 J
æ tan q + ms ö 49. Conserving angular momentum, 53. From work-energy theorem,
Þ v = Rg ç ÷
è 1 - ms tan q ø Li = Lf Work done = Change in KE

Þ mv 0R 0 = mv ¢ æç 0 ö÷
R Þ W = Kf - Ki
5. Work, Energy and Power è 2 ø x2 1
mv 2
Þ Kf = W + Ki = òx Fx dx +
2
46. Given, KP > KQ Þ v ¢ = 2v 0 1
30 1
So, final kinetic energy of the particle is = ò20 - 01
. × x dx + ´ 10 ´ 102
In case (a), the elongation is same 2

MODULE 3
www.jeebooks.in
384 NEET Test Drive

30
éx2 ù 57. Given, the potential energy of a particle At the horizontal position, it has energy
= - 01
. ê ú + 500 A B 1
in a force field, U = - E = K + U d= mu ¢ 2 + mgl
ë 2 û 20 r2 r1 2
= - 0.05 [302 - 202 ] + 500 dU
= - 0.05 [900 - 400] + 500 For stable equilibrium, F = - =0 According to conservation of mechanical
dr energy, K = E
Þ Kf = - 25 + 500 = 475 J dU
= = - 2 Ar -3 + Br -2 u′
54. According to question, a body of mass dr
-dU
0 = - 3 + 2 æç As, = 0ö÷
1 kg begins to move under the action of 2A B
time dependent force, r r è dr ø
2A l
F = (2t $i + 3t 2 $j ) N or =B O
where $i and $j are unit vectors along X r l
andY -axes. The distance of particle from the centre
2A
F = ma Þ a =
F of the field r = 1 1
Q B \ mu 2 = mu ¢ 2 + mgl
m 2 2
(2t$i + 3t 2 $j ) 58. According to question, we have 1 1
Þ a = (Q m = 1 kg) or mu ¢ 2 = mu 2 - mgl
1 Let the tension at point A beT A . So, from 2 2
Newton’s second law u ¢ 2 = u 2 - 2gl
Þ a = (2t$i + 3t 2 $j ) m / s 2 or
mvc2
Q Acceleration, a =
dv T A - mg = or u ¢ = u 2 - 2gl ...(i)
dt R
1 So, the magnitude of change in velocity
Þ dv = a dt …(i) Energy at point A = mv 02 …(i)
2 | Du | = | u ¢ - u |
Integrating both sides, we get
Energy at point C = u ¢ 2 + u 2 + 2 u ¢ u cos 90°
ò dv = a dt = (2t$i + 3t 2 $j ) dt
ò ò2 1
= mvc2 + mg ´ 2R …(ii) u
v = t $i + t 3 $j 2
Q Power developed by the force at the C
time t will be given as
P = F . v = (2t $i + 3t 2 $j ) × (t 2 $i + t 3 $j ) R TC u′
= (2t × t 2 + 3t 2 × t 3 ) B
TA TB ∆u
P = (2t 3 + 3t 5 ) W
55. As the machine delivers a constant v0
power. A
So, F × v = constant = k (watts) mg | Du | = u ¢ 2 + u 2
dv
Þ m ×v = k Applying Newton’s 2nd law at point C,
dt = 2 (u 2 - gl) [from Eq. (i)]
k mvc2
Tc + mg =
Þ ò vdv = m ò dt R 60. Since, the collision mentioned is an
elastic head-on collision. Thus,
v2 k 2k To complete the loop Tc ³ 0 accroding to the law of conservation of
Þ = t Þv = t
2 m m mvc2 linear momentum, we get
So, mg = Þ vc = gR …(iii)
Now, force on the particle is given by R m1u1 + m 2u2 = m1v1 + m2v 2
1 From Eqs. (i) and (ii) by conservation of Where, m1 and m 2 are the masses of the
= m æç ö2
dv d 2kt energy, we get two blocks, respectively and u1 and u2
F =m ÷
dt dt è m ø 1 1 are their initial velocities and v1 and v 2
mv 02 = mvc2 + 2mgR
æ 1 -1 ö -1 2 2 are their final velocities, respectively.
mk
= 2km × ç t 2 ÷ = ×t 2
1 1 2 Here, m1 = m, m 2 = 4m
ç2 ÷ 2 Þ mv 02 = mgR + 2mgR ´
è ø 2 2 2 u1 = v , u2 = 0 and v1 = 0
56. Given, pressure = 150 mm of Hg (Q vc = gR ) mv + 4m ´ 0 = 0 + 4mv 2
Pumping rate of heart of a man Þ v 02 = gR + 4gR Þ v 0 = 5gR Þ mv = 4mv 2
-3
dV 5 ´ 10 59. When stone is at its lowest position, it v
= = m3 /s or v2 = …(i)
dt 60 has only kinetic energy, given by 4
dV 1
Power of heart = P × K = mu 2 As, the coefficient of restitution is given
dt 2 as,
dV relative velocity of
= rgh × [P = rgh] separation after collision
dt e=
O relative velocity of approach
. ´ 5 ´ 10-3 )
(13.6 ´ 103 kg / m3 ) (10 ´ 015
= | v - v1|
60
l = 2
| u 2 - u1|
= 170
. W u

MODULE 3
www.jeebooks.in
The NEET Edge ~ Physics 385
v \ Moment of inertia of the point masses 67. According to question, torque t = 0
| - 0|
= 4 [from Eq. (i)] about the given axis is
It means that, a = 0
|0 - v| I = å mi ri 2 Þ I = m1r12 + m 2 r 22
d 2Q
1 2 2 a=
= æ m2 l ö æ m1 l ö dt 2
4 = m1 ç ÷ + m2 ç ÷
è m1 + m 2 ø è m1 + m 2 ø Given, Q (t ) = 2 t 3 - 6t 2
\ e = 0.25 2
2
m1m 2 l m1m 2 l dQ
61. Suppose a ball rebounds with speed v, = (m 2 + m1) = So, = 6t 2 - 12 t
(m1 + m 2 ) 2 (m1 + m 2 ) dt
v = 2gh = 2 ´ 10 ´ 20
64. According to parallel axes theorem of d 2Q
= 20 m / s a= = 12 t - 12
moment of inertia, dt 2
Energy of a ball just after rebound,
1 I = I CM + Mh 2 12 t - 12 = 0
E = mv 2 = 200 m 2
Þ t = 1s
ML2
I = I 0 + M æç ö÷ Þ I = I 0 +
2 L
So,
As, 50% of energy loses in collision è2ø 4 68. As velocity of an automobile vehicle,
means just before collision energy is 5
65. The total moment of inertia of the v = 54 km/h = 54 ´ = 15 m/s
400 m. system is 18
According to law of conservation of I = I1 + I 2 + I 3 …(i) Angular velocity of a vehicle, v = w 0 R
energy, we have 2 v 15 100
Here, I1 = mr 2 Þ w0 = = = rad/s
1 R 0.45 3
mv 02 + mgh = 400 m 3
2 So, angular acceleration of an
2
1 I2 = I3 = mr 2 + mr 2 automobile,
Þ mv 02 + m ´ 10 ´ 20 = 400 m 3
2 100
[From parallel axis theorem] 0-
Þ v 0 = 20 m/s Dw wf - w 0 3
5 a= = =
62. According to question, the third part of = mr 2 t t 15
3 - 100
mass 2m will move as shown in the = rad/s 2
figure because the total momentum of From Eq. (i), we get 45
I = mr 2 + 2 ´ mr 2 = mr 2 æç +
the system after explosion must remain 2 5 2 10 ö Thus, average torque transmitted by its
÷
zero. Let the velocity of third part be v ¢. 3 3 è3 3ø brakes to wheel,
From the conservation of momentum I = 4mr 2 t = Ia
2 (mv ) = (2m) ´ v ¢ 100
66. Considering the information given in the Þ =3 ´ = 6.66 kgm 2 s -2
v question, let us draw the figure 45
Þ v¢=
2 I 69. The moment of inertia of the uniform rod
So. total kinetic energy generated by the about an axis through one end and
explosion perpendicular to length is
1 1 1 R R/2 ml 2
= mv 2 + mv 2 + (2m) v ¢2 I =
2 2 2 3
2
æ v ö where, m is mass of rod and l its length.
= mv 2 + m ´ ç ÷
è 2ø Torque (t = Ia) acting on centre of
If the above figure is considered, then
gravity of rod is given by
2mv 2 3 moment of inertia of disc will be given as
= mv + = mv 2 l
2 2 I = I remain + I (R / 2 ) Þ I remain = I - I (R / 2 ) t = mg
2
+Y mv 2 (mv) (Resultant Putting the values, we get As we know that, t = Ia
momentum of éM 2 ù l ml 2 l
æR ö so, Ia = mg or a = mg
two small masses) 2 ê ç ÷ 2ú 2 3 2
MR 4 è2ø M æR ö ú
= -ê + ç ÷ 3g
–X 45° 2 ê 2 4 è2ø ú \ a=
45° +X ê ú 2l
ë û
v' 70.
2m

MR 2 é MR 2 MR 2 ù Fixed horizontal
= -ê + ú axis
–Y 2 ë 32 16 û
T
MR 2 é MR 2 + 2MR 2 ù
6. System of Particles and = -ê ú Given, m = 50 kg,
2 ë 32 û
Rotational Motion r = 0.5 m,
MR 2 3MR 2 16MR 2 - 3MR 2 a = 2 rev / s 2
= - =
63. Centre of mass of m1 and m 2 masses 2 32 32
Þ Torque produced by the tension in the
lies at 13MR 2 string
Þ Iremain =
m1r1 + m 2 r 2 32
r = T
= T ´ r =T ´ 0.5 = N-m …(i)
m1 + m 2 2
MODULE 3
www.jeebooks.in
386 NEET Test Drive

We know that, t = Ia …(ii) 73. By conservation of momentum, dq


Eq. (i) becomes = w 0 + at
From Eqs. (i) and (ii), we get æ I ö dt
It wi = (It + Ib) wf Þ wf = ç t ÷ wi
T è I t + Ib ø i.e. dq = (w 0 + at ) dt
= Ia
2 Loss in energy, So, if in time t angular displacement is q.
æ MR 2 ö 1 1 q t

è 2 ø
÷ ´ (2 ´ 2 p) rad / s
2
DE =
2
It wi2 - (It + Ib ) wf2
2
ò 0 d q = ò 0 (w 0 + at ) dt

1æ II ö 1
at 2
MR 2 = ç t b ÷ wi2 or q = w0 t + …(ii)
[because I Solid cylinder = ] 2 è I t + Ib ø 2
2
1 2 Given a = 3 rad / s 2
T 50 ´ (0.5) 2 74. KE of a rotating rigid body, KE = Iw
= ´ 4p 2 w 0 = 2 rad / s, t = 2 s
2 2
1 2 1
1 \ KE of sphere, E S = Iw1 Here, q=2´2+ ´ 3 ´ (2) 2
T = 50 ´ ´ 4p = 50 p = 157 N 2 2
4 1 2
= mR 2 w12 or q = 4 + 6 = 10 rad
71. As two point masses m1 and m 2 are 2 5
placed at opposite ends of a rigid rod of 1 Alternative
= mR 2 w12
length L and negligible mass as shown in 5 As we know that, equation of circular
the figure. 1 1 motion,
KE of cylinder, E C = mR 2 w 22
ω0 2 2 1
q = wt + at 2 çæ where symbols have÷ö
1 2 è their usual meaning ø
= mR 2 w 22
x (L – x) 4 Putting the value of w, t, a from question,
mR 2 w12 1
q=2´2+ ´ 3 ´ 2 ´ 2 = 10 rad
m1 P m2 ES 5 4 w12 4 w12 2
\ = = =
EC 2 2
mR w 2 5 w 22 5 (2 w1) 2 77. A solid sphere rolling without slipping
down an inclined plane
4
1

ω
= (given, w 2 = 2 w1)
Total moment of inertia of the rod, 5
ES : EC = 1 : 5

v
I = m1x 2 + m 2 (L - x ) 2
I = m1x 2 + m 2L2 + m 2 x 2 - 2m 2Lx 2gh
75. As, v =
k2 θ
As, I is minimum i.e. 1+
dI r2
= 2m1 x + 0 + 2 xm2 - 2m2L = 0 g sin q g sin q
dx 3 v2 In this case, a1 = =
Given, h = k2 (2 / 5) R 2
Þ x (2m1 + 2m 2 ) = 2m2L
4g 1+ 2
1+
R R2
m 2L 2 2gh 2g 3 v 2
Þ x = So, v = = é\ for solid sphere,k 2 = 2 R 2 ù
m1 + m 2 k2 æ k2 ö êë úû
1+ 4g ç1 + 2 ÷ 5
When I is minimum, then work done on r2 è r ø g sin q
rotating a rod 1/ 2 Iw 2 with angular =
6 gv 2 3 7 /5
velocity w 0 will be minimum. = Þ1 =
æ k2 ö æ k2 ö 5
72. When no external torque acts on system 4 g ç1 + 2 ÷ 2 ç1 + 2 ÷ Þ a1 = g sin q
then, angular momentum of system è r ø è r ø 7
remains constant. For a sphere slipping down an inclined
k2 3 k2 3 1
or 1 + =
or = - 1= plane without rolling without rolling
Angular momentum before contact r2
2 r2 2 2
= I1w1 + I 2 w 2 1 Þ a 2 = g sin q
k2 = r 2 (Equation of disc)
Angular momentum after the discs 2 a1 5 / 7g sin q 5
brought into contact Þ = =
Hence, the object is disc. a2 g sin q 7
= I net w = (I1 + I 2 ) w dw
76. By definition, a = 78. Acceleration of an object rolling down an
So, final angular speed of system dt inclined plane is given by
I w + I 2 w2 i.e. dw = a dt g sin q
= w= 1 1 a=
I1 + I 2 So, if in time t the angular speed of a 1 + I / mr 2
Now, to calculate loss of energy, we body changes from w 0 to w
w t where, q = angle of inclination of the
subtract initial and final energies of
system. òw 0
dw = ò 0 a dt inclined plane
m = mass of the object
⇒ Loss of energy If a is constant
I = moment of inertia about the axis
1 1 1
= Iw12 + Iw 22 - (2I ) w 2 w - w 0 = at or
through centre of mass
2 2 2 w = w 0 + at …(i)
dq I 1/ 2 mr 2 1
1
w= For disc, = =
= I (w1 - w 2 ) 2 Now, as by definition mr 2
mr 2 2
4 dt
MODULE 3
www.jeebooks.in
The NEET Edge ~ Physics 387

I 2 / 5mr 2 2 Since, as per the result the Kepler’s The time period of planet will be
For solid sphere, = =
mr 2
mr 2 5 second law of area, that the planet will 2pr 4 p 2r 2 4 p 2r 3
move slowly (v min) only when it is farthest T = ÞT 2 = = …(i)
I 2 / 3mr 2 2 v GM GM
For hollow sphere, = = from the sun and more rapidly (v max)
mr 2 mr 2 3 r
when it is nearest to the Sun.
g sin q 2 Also, from Kepler’s third law
\ a disc = = g sin q Thus, v A = v max,vC = v min
1+
1 3 T 2 = Kr 3 …(ii)
Therefore, we can write
2 From Eqs. (i) and (ii), we get
v A > vB > vC …(i)
= 0 .66 g sin q 4 p 2r 3
g sin q 5 Kinetic energy of the planet at any point = Kr 3 Þ GMK = 4 p 2
a = = g sin q 1 GM
solid sphere
2 7 is given as, K = mv 2
1+ 2 83. From Kepler’s third law,
5
Thus, at A,
1
K A = mv A2 T2 µr3
= 0.71 g sin q 2
g sin q 3 é where,T = time period of satellite ù
a hollow sphere = = g sin q At B,
1
KB = mvB2 ê and r = radius of elliptical orbit ú
2 5 ë (semimajor axis) û
1+ 2
3 1
At C, KC = mvC2 Hence, T12 µ r13 and T 22 µ r 23
= 0.6 g sin q 2
T 22 r 23 (3R ) 3 T 22 1
Clearly, a solid sphere > adisc > ahollow sphere From Eq. (i), we can write So, = = or =
K A > KB > KC T12 r13 (6R ) 3
T12 8
Type of sphere is not mentioned in the
question. Therefore, we will assume the 81. Let the original mass of sun was Ms and 1 2 24
given sphere as solid sphere. T 22 = T1 ÞT 2 = =6 2h
gravitational constant G. 8 2 2
\ asolid sphere > adisc According to the question, 84. Apply Kepler’s second law.
79. Work done required to bring an object to M
1 2 new mass of sun, Ms¢ = s The line joining the sun to the planet
rest is given as,W = Iw 10 sweeps out equal areas in equal time
2 New gravitational constant, G ¢ = 10G interval, i.e. areal velocity is constant.
where, I is the moment of inertia and w is As, the acceleration due to gravity is dA A1 A2
the angular velocity. = constant or =
given as dt t1 t2
Since, here all the objects spin with the GMe
same w, this means, g = …(i) where, A1 = area under SCD and
R2
W µI A2 = area under ABS.
where, Me is the mass of earth and R is A
2 Þ t1 = 1 t 2
As, I A (for a solid sphere) = MR 2 the radius of the earth.
A2
5
Now, new acceleration due to gravity,
1
IB (for a thin circular disc) = MR 2 G ¢Me 10MeG Given, A1 = 2 A2
2 g¢ = = …(ii)
R2 R2 \ t1 = 2 t 2
IC (for a circular ring) = MR 2 85. The acceleration due to gravity on an
\ g ¢ = 10g [from Eqs. (i) and (ii)]
\ WA : WB : WC = I A :IB : IC F
This means the acceleration due to object of mass m, g =
2 1 m
= MR 2 : MR 2 : MR 2 gravity has been increased.
5 2 But from Newton’s law of gravitation
The force of gravity, F = mg¢ = 10mg
2 1 GMm
= : : 1 = 4 : 5 : 10 Thus, force on body increase and it F =
5 2 R2
becomes difficult to move or walk on
Þ WA < WB < WC ground. The critical velocity is directly where, M is the mass of the earth and R
propor tional to acceleration due to is the radius of the earth.
or WC >WB > WA
gravity i.e. vc µ g ¢ GMm / R 2 GM
\ g = = 2
So, the raindrops will fall faster. m R
7. Gravitation As, time period of the simple pendulum is 4 4
80. According to the question, [QM p = p R p Pp and Me = p Re3 Pe ]
3

l 3 3
T = 2p
vC g¢ Given, rplanet = 2 r earth
B 1 GM p GMe
S Or T µ Also, gplanet = g earth Þ =
A C g¢ R p2 Re2
Thus, time period of the pendulum 4 4
vA G ´ pR p3 r p G ´ pRe3 re
decreases with the increase in g. So, 3 = 3
82. The gravitational force of attraction R p2 Re2
The figure above shows an ellipse
between the planet and sun provide the or R p r p = Re re
traced by a planet around the sun, S.
The close point A is known as perihelion centripetal force or R p ´ 2 re = Re re
(perigee) and the farthest point C is GMm mv 2 GM or
R
Rp = e =
R
i.e. = Þv =
known as aphelion (apogee). r2 r r 2 2

MODULE 3
www.jeebooks.in
388 NEET Test Drive

86. g h = Acceleration due to gravity at 5.4 ´ 107 Þ c2 = 2


GM
=R + h
height h above earth’s surface 6.0 R¢
2
Þ 9 ´ 106 = R + h 2 GM
æ R ö æ1 - 2h ö R¢ =
=g ç ÷ =g ç ÷ c2
èR + h ø è R ø Þ h = (9 - 6.4) ´ 106
2 ´ 6.67 ´ 10-11 ´ 6 ´ 1024
= 2.6 ´ 106 m =
where, gd = Acceleration due to gravitay 9 ´ 1016
at depth d below earth’s surface Þ h = 2600 km
4 ´ 6.67
90. The resulting gravitational potential, = ´ 10-3
= g æç1 - ö÷
d
3
è Rø V = - 2G é + + ¼ù
1 1 1 1
+ +
êë 1 2 4 8 úû = 8.89 ´ 10-3 = 0.889 ´ 10-2
Given, when h = 1 km, gd = g h
- 10-2 m
~
Þ V = - 2 G é1 + + 2 + 3 + ¼ù
1 1 1
g æç1 - ö÷ = g æç1 -
d 2h ö
or ÷ êë 2 2 2 úû 94. At a platform at a height h,
è Rø è R ø
-1 escape energy = binding energy of
Þ V = - 2 G æç1 + ö÷
⇒ d = 2h or d = 2 km 1
sphere
è 2ø
87. Acceleration due to gravity at a depth d 1 GMm
2G 2G or m (fve ) 2 =
below the surface of the earth is Þ V =- =- = - 4G 2 R +h
given by æ1 - 1 ö æ 1ö
ç ÷ ç ÷ 2 GM 2 GM
è 2ø è2ø or fve = =
gdepth = gsurface æç1 - ö÷
d
R +h 2R
è Rø 91. We know that,
power, P = F × v = Fv cos q (Qh = R ) …(i)
Also, for a point at height h above
So, just before hitting, q is zero, then But at surface of the earth,
é R2 ù
surface, gheight = gsurface ê 2ú
2 GM
power will be maximum. ve = …(ii)
ë (R + h) û R
92. Since, the escape velocity at earth can be
Therefore, we can say that value of g given as Dividing Eq. (ii) by Eq. (i),
increases from centre to maximum at 8 GM
the surface and then decreases as ve = 2gR = R p Gr fve
3 Hence, = R
depicted in graph (b).
ve 2 GM
GM GM éQg = GM = 4 pGRP ù
88. E = 3 r (if r < R ) and E = 2 R
R r êë R2 3 úû
1
(if r ³ R ) Þ f =
2
[where, r = density of earth]
E∝r 1
E E∝ 8
r2 Þ ve = R p Gr …(i) 8. Mechanical Properties
3
O R r of Solid
As, it is given that the radius and mean
density of planet are twice as that of 95. Youngs' modulus is given by
earth. So, escape velocity at planet F ´l
89. Gravitational potential at some height h will be Y = …(i)
from the surface of the earth is given A ´ Dl
8 V = A ´ l = constant
by v p = 2R pG 2r …(ii) As, …(ii)
GM 3
V =- …(i) From Eqs. (i) and (ii), we get
R +h Dividing Eq. (i) by Eq. (ii), we get F ´ l2 F
8 Y = Þ Dl = ´ l2
and acceleration due to gravity at some R p Gr V ´ Dl V ´Y
ve 3 1
height h from the earth surface can be = = Þ Dl µ l 2
given as vp 8 2 2
2R pG 2r F ´L mg .L
GM 3 96. AsY = =
g¢ = …(ii) DL ´ A DL. A
(R + h) 2 93. Problem Solving Strategy For the black mgL L
or DL = Þ DL µ ,
From Eqs. (i) and (ii), we get hole, the escape speed is more than c Y.A A
(speed of light). We should compare the
|V | GM (R + h) 2 escape speed with the c (Note that the
which is maximum for option (a).
= ´
g¢ (R + h) GM escape speed should be at least just 97. According to the question,
|V | greater than c). For first wire,
Þ =R + h …(iii)
g¢ 2 GM Area of cross-section = A1
-2
ve =
Given, 7
V = - 5.4 ´ 10 J-kg R¢ Force applied = F1
and g ¢ = 6.0 ms -2 [R ¢ = New radius of the earth] Increase in length = Dl
Radius of earth, R = 6400 km. 2 GM From the relation of Young’s modulus of
c= [ve » c]
Substitute these values in Eq. (iii), we R¢ elasticity,
get

MODULE 3
www.jeebooks.in
The NEET Edge ~ Physics 389

Fl As, given difference in length (l 2¢ - l1¢) is 103. According to question, the situation can
Y =
ADl same for all temperature. be drawn as following.
Substituting the values for wire 1 in the So, l 2¢ - l1¢ = l 2 - l1 A
above relation, we get (1–p)L d ρ
F l Þ l 2 (1 + a2 Dt ) - l1 (1 + a1 Dt ) = l 2 - l1
Þ Y1 = 1 1 …(i) pL nρ
A1Dl Þ l 2 a2 = l1a1
For second wire, or l1 a1 = l 2 a2
Applying Archemedies’ principle,
Area of cross-section = A2 Weight of cylinder
Force applied = F 2 9. Mechanical Properties = (Upthrust)1 + (Upthrust) 2
Increase in length = Dl of Fluids i.e. ALdg = (1 - p) LArg + (pLA) nrg
Similarly, Þ d = (1 - p) r + pnr = r - pr + n pr
F 2 l2 100.The rate of heat generation is equal to = r + (n - 1) p r = r [1 + (n - 1) p]
Y2 = …(ii) the rate of work done by the viscous
A2 Dl 104. During the streamline flow of viscous and
force which in turn is equal to its power. incompressible fluid through a
Q Volume,V = Al dQ
Rate of heat produced, = F ´ nT pipe-varying cross-section, the product
V
or l= dt of area of cross-section and normal fluid
A velocity (An) remains constant
where, F is the viscous force and nT is
Substituting the value of l in Eqs. (i) and the terminal velocity. throughout the flow.
(ii), we get Consider a cylindrical tube of a spray
As, F = 6phr nT
FV FV pump has radius R, one end having n
Y1 = 21 and Y2 = 22 dQ
A1 Dl A2 Dl Þ = 6phr nT ´ nT fine holes, each of radius r and speed of
dt
liquid in the tube is v as shown in the
As, it is given that the wires are made up = 6phr nT2 …(i) figure.
of same material, i.e. Y1 = Y2
From the relation of terminal velocity,
F1V FV r
Þ = 2 2 r 2 (r - s) R
A12 Dl A22 Dl nT = g , we get r
9 h
F1 A2 A2 1
Þ = 12 = = nT µ r 2 …(ii)
F2 A2 9A 2 9 According to equation of continuity,
From Eq. (ii), we can rewrite Eq. (i) as
(Q A1 = A and A2 = 3 A) dQ An = constant
µ r × (r 2 ) 2
or F 2 = 9F1 = 9F (given, F1 = F ) dt where, A is area of cylindrical tube and n
dQ is velocity of liquid in a tube.
98. The object is spherical and the bulk or µr5
modulus is represented by B. It is the dt Volume in inlet flow rate = Volume in
ratio of normal stress to the volumetric 3 3 outlet flow rate
101. Given d = 2700 m, r = 10 kg/m ,
strain. R 2n
compressibility = 45.4 ´ 10-11 per pascal pR 2 n = n pr 2 n¢ Þ n' =
Hence, B =
F /A nr 2
The pressure at the bottom of ocean is Thus, speed of the ejection of the
DV /V
given by R 2n
DV p DV p liquid through the holes is 2 .
Þ = Þ = p = rgd = 103 ´ 10 ´ 2700 nr
V B V B
= 27 ´ 106 Pa 105. From Bernoulli’s theorem,
Here, p is applied pressure on the
DV 1 2 1
object and is volume strain. So, fractional compression = p1 + rv1 = p 2 + rv 22
V compressibility ´ pressure 2 2
Fractional decreases in volume, = 45.4 ´ 10-11 ´ 27 ´ 106 = 1.2 ´ 10-2 where, p1, p 2 are pressure inside and
outside the roof and v1, v 2 are velocities
DV DR 4 102. Pressure of two points lie in the same
Þ =3 [QV = pR 3 ] of wind inside and outside the roof.
V R 3 horizontal level should be same and Neglect the width of the roof.
Volume of the sphere decreases due to p = hpg
Both ends of the U-tube are open, so the Pressure difference is
the decrease in its radius a.
1
DV 3 DR p DR p pressure on both the free surfaces must p1 - p 2 = r (v 22 - v12 )
Hence, = = Þ = be equal. 2
V R B R 3B 1
i.e., p1 = p 2 = ´ 1.2 (402 - 0) = 960 N/m 2
99. According to question, 2
hoil × S oil g = hwater × S water × g Force acting on the roof is given by
Coefficient of linear expression of
S oil = specific density of oil F = (p1 - p 2 )A = 960 ´ 250
brass = a1
h ×S ×g = 24 ´ 104 N = 2.4 ´ 105 N
S oil = water water
Coefficient of linear expression of hoil × g
steel = a2 As the pressure inside the roof is more
(65 + 65) ´ 1000 than outside it. So the force will act in the
From figure S oil =
Length of brass and steel rods are l1 and (65 + 65 + 10) upward direction,
l 2 respectively.
= 928 kgm-3 i.e. F = 2.4 ´ 105 N, upwards.

MODULE 3
www.jeebooks.in
390 NEET Test Drive

106. According to ascent formula for capillary 111. Heat lost by Ist body = Heat gained by 10
= K (65° C - t ° C) …(i)
tube, IInd body. Body at 100°C temperature 5 min
2T cos q has greater heat capacity than body at (60 - 54)
h= For second case, = K (57 - t ) …(ii)
rgr 0°C. So, final temperature will be closer 5 min
to 100°C. So,Tc > 50° C.
cos q1 cos q 2 cos q 3 (57°C is average of 60°C and 54°C)
\ = = 112. According to question as conservation
r1 r2 r3 From Eqs. (i) and (ii), we get
of energy, energy gained by the ice
Thus, cos q µ r during its fall from height h is given by 10 (65 - t )
=
\ r1 > r 2 > r 3 E = mgh
6 (57 - t )
\ cos q1 > cos q 2 > cos q 3 As given, only one quarter of its energy So, t = 45° C
p is absorbed by the ice 117. In steady state the amount of heat
0 £ q1 < q 2 < q 3 <
2 mgh flowing from one face to the other face in
So, = mLf KA (q1 - q 2 ) t
107. Increase in surface energy (work done) 4 time t is given by Q = ,
mLf ´ 4 l
= increase in area ´ Surface tension Þ h= where, K is coefficient of thermal
mg
Increase in surface area, conductivity of material of rod.
DA = (5 ´ 4 - 4 ´ 2) ´ 2 Lf ´ 4 3.4 ´ 105 ´ 4 Q A r2
= = Þ µ µ …(i)
(Q film has two surfaces) g 10 t l l
= (20 - 8) ´ 2 cm 2 = 24 cm2 = 13.6 ´ 104 = 136000 m As the metallic rod is melted and the
= 24 ´ 10 -4
m 2 = 136 km material is formed into a rod of half the
radius
So, work done,W = T × DA 113. According to principle of calorimetry,
V1 = V2
3 ´ 10-4 = T ´ 24 ´ 10-4 Heat lost by steam = Heat gained by éQ r = r1 ù
p r12 l1 = pr 22 l2
ëê 2 ûú
1 water 2
\ T = = 0.125 Nm -1
8 Let m¢ be the amount of steam that l2
converts into water. Þ l1 = …(ii)
108. As energy released 4
= (Af - Ai )T m ¢ ´ L + m ¢ s DT = ms Dt Now, from Eqs. (i) and (ii), we get
3 R3 3V é where,s = Specific heat of water ù Q1 r12 l r2 4 l1
where, Ai = 4 pR = ´ 4 p 2
= = ´ 22 = 1 ´
3 R R ëê L = Latent heat of water ûú Q2 l1 r2 l1 (r1 / 2) 2
2 V 2 3V m ¢ ´ 540 + m ¢´1 ´ (100 - 80) Q1
and Af = 4 pr = 4 pr = Þ Q1 = 16 Q 2 Þ Q2 =
4 3 r = 20 ´ 1 ´ (80 - 10) 16
pr
3 20 ´ 70
m¢ = = 2.5 g 118. As from law of heat transfer through
560
Energy released = 3VT é - ù
1 1 DQ æT1 - T 2 ö
\ conduction, H = = KA ç ÷
êë r R úû Now, net mass of water è l ø
Dt
= 20 + 2.5 = 22.5 g
r2
114. Graph (a) shows the variation of Þ H µ ...(i)
10. Thermal Properties l
temperature with time. At first
of Matter temperature will increase upto boiling (a) When r = 2 r 0 , l = 2 l0
point, then there will be state change (2 r 0 ) 2 2 r 02
Þ H µ Þ H µ
109. The relation between true scale and new from liquid to gas and then again the 2 l0 l0
scale of temperature is given by temperature rises with time.
(b) When r = 2 r 0 , l = l0
æ t - LFP ö = æç
t - LFP ö 115. According to Newton’s law of cooling,
ç ÷ ÷ (2 r 0 ) 2 4r 2
è UFP - LFP ø true è UFP - LFP ø faulty Þ H µ Þ H µ 0
DT = DT 0 e -lt
39° C - 0° C t - 39° W l0 l0
= Þ 3T - 2T = (3T - T ) e - l ´ 10 …(i) r 02
100° C - 0° C 239° W - 39° W (c) When r = r 0 , l = l0 Þ H µ
Again for next 10 minutes, l0
Þ t = 117°W T ¢ - T = (2T ) ´ e - l ( 20 ) …(ii) r2
110. Given, the value of coefficient of volume (d) When r = r 0 , l = 2 l0 Þ H µ 0
From Eqs. (i) and (ii), we get 2 l0
expansion of glycerin is 5 ´ 10-4 K -1. 2
T ¢ - T = (2T ) (e -l ´ 10 ) 2 = (2T ) æç ö÷ =
1 T
As, orginal density of glycerin, It is obvious that, heat conduction will be
è2ø 2 maximum in case (b).
r = r 0 (1 + YDT ) T 3T
\ T ¢ =T + = 119. The quantity of heat flowing across a
Þ r - r 0 = r 0YDT 2 2
slab in time t,
Thus, fractional change in the density of 116. Apply Newton’s law of cooling. Let the
KADq
glycerine for a rise of 40°C in its temperature of the surrounding be t° C. Q =
For first case, l
temperature,
r - r0 (70 - 60) where, K = thermal conductivity,
= YDT = 5 ´ 10-4 ´ 40 = K (65° C - t ° C )
5 min Dq = change in temperature,
r0
(65° is average of 70°C and 60°C) A = area of slab
= 200 ´ 10-4 = 0.020

MODULE 3
www.jeebooks.in
The NEET Edge ~ Physics 391

and l = thickness T1 3 The intensity of this power at the earth’s


or = …(ii)
For same heat flow through each slab T2 4 surface (under the assumption R > > r 0 )
and (composite slab), we have is
As we know that, from Stefan’s law, the
K1A (Dq1) K 2 A (Dq 2 ) power radiated by a body at P s ´ 4 pr 2T 4
= I = =
l l temperatureT is given as 4 pR 2 4 pR 2
K ¢ A (Dq1 + Dq 2 ) P = sAeT 4 2
sr T 4
sr 2 (t + 273) 4
= = =
2l 4 2
i.e. P µT R R2
or K1Dq1 = K 2 Dq 2 (Qthe quantity sAe is constant for a body)
K¢ 4 11. Thermodynamics
= (Dq1 + Dq 2 ) = C (say) P1 T14 æ T1 ö
2 Þ = =ç ÷
P2 T 24 èT 2 ø 125. According to the question,
C C
So, Dq1 = , Dq 2 = From Eq. (i), we get heat spent during the conversion of
K1 K2 sample of water at 100° C to steam is
4
P1 æ 3 ö 81
and
2C
(Dq1 + Dq 2 ) = =ç ÷ = DQ = 54 cal = 54 ´ 4.18 J = 225.72 J
K¢ P2 è 4 ø 256
Normal pressure, p = 1.013 ´ 105 Nm-2
C C 2C Given, P1 = P and P2 = nP
or + = Net work done during the conversion
K1 K 2 K¢ P1 P 81 256
Þ = = or n = would be given as
æK + K 2 ö 2 C P2 nP 256 81
or C ç 1 ÷= DW = pDV = p [Vsteam - Vwater ]
è K1 K 2 ø K¢ 122. Given, temperature,T1 = 5760 K Here,Vsteam = 167.1 cc
\ K¢ =
2 K1 K 2 Since, it is given that energy of radiation = 167.1 ´ 10-6 m3
K1 + K 2 emitted by the body at wavelength 250 Vwater = 0.1 g = 0.1cc
nm in U1, at wavelength 500 nm is U 2
Given, K1 = K and K 2 = 2 K = 0.1 ´ 10-6 m3
and that at 1000 nm is U 3 .
2K ´ 2K 4 \ DW = 1.013 ´ 105
So, K¢ = = K Q According to Wien’s law, we get
K + 2K 3 [(167.1 - 0.1) ´ 10-6 ]
l mT = b
120. Rate of loss of heat by conduction is where, b = Wien’s constant = 1.013 ´ 167 ´ 10-1
DQ T - T2 ö
H = = K A æç 1 ÷ = 2.88 ´ 106 nmK Þ l m =
b = 16.917 J
Dt è l ø T Now, by the first law of thermodynamics,
All the symbols have their usual 2.88 ´ 106 nmK DQ = DU + DW
Þ lm = = 500 nm
meaning. 5760 K where, DU is the change in internal
æT - T 2 ö Q l m = wavelength corresponding to energy of the sample.
For first rod, H1 = K1A1 ç 1 ÷
è l1 ø maximum energy So, U 2 > U1 Þ DU = DQ - DW
æT - T 2 ö 123. According to Stefan’s law, Substituting the values in the above
For second rod, H 2 = K 2 A2 ç 1 ÷
è l2 ø E = sT 4 , equation, we get
where, s = Stefan’s constant and DU = 225.72 - 16.917
But, l 1 = l 2 i.e. of same length
T = temperature = 208.7 J
and H1 = H 2 i.e. same rate of loss of 4
heat through conduction E 1 é T1 ù 126. Apply first law of thermodynamics.
=
E 2 êëT 2 úû According to first law of
So, we have
4 4
thermodynamics, DQ = DU + DW
K1A1 (T1 - T 2 ) = K 2 A2 (T1 - T 2 ) é 273 + 727 ù é 1000 ù
Þ E2 = 7 ê ú =ê ´7 For isothermal process, DU = 0
K1A1 = K 2 A2 ú
or ë 273 + 227 û ë 500 û So, DQ = - DW
121. According to Wien’s law, = 112 cal-cm 2 s -1 Given, DW = - 150 J
1
l max µ i.e. l maxT = constant 124. From Stefan’s law, the rate at which So, DQ = + 150 J
T energy is radiated by sun at its surface is
When Q is positive, the heat is added to
where, l max is the maximum wavelength P = s ´ 4 pr 2T 4 the gas.
of the radiation emitted at temperatureT .
[Sun is a perfectly black body as it emits 127. From first law of thermodynamics
\ l max 1T1 = l max 2T 2 radiations of all wavelengths and so for
l max 2 DQ = DU + DW
or
T1
= …(i) it e = 1]
T2 l max 1 For adiabatic process, DQ = 0
TK
Here, l max 1 = l 0 and l max
3
l0 = R r0 So, DU = - DW
2
4 r Earth 128. According to the given graph,
Substituting the above values in Eq. (i),
we get volume (V ) µ temperature (T )
3 V
l0 or = constant
T1 3 Sun T
= 4 =
T2 l0 4 Thus, the process is isobaric.

MODULE 3
www.jeebooks.in
392 NEET Test Drive

V Q Process, pV 3 = constant Þ a = 3 = nR
5 æ pBVB p AVA ö
-
ç ÷
R fR R 2 è nR nR ø
\ C = CV + = +
B 1- a 2 1- 3 5
= (2 ´ 103 ´ 6 - 5 ´ 103 ´ 4)
VB 2
fR 3R
A where, CV = = 5 4 ´ 104
2 2 = ´ (- 8 ´ 103 ) = - = -20 kJ
VA 2 2
[QFor monoatomic gas, f = 3 ]
O 3R R 134. For isothermal expansion process,
TA TB T Þ C = - =R
2 2 pV = p ¢ ´ 2V [\V ¢ = 2V ]
\ Work done,
p
DW = pDV = nRDT = nR (TB - T A ) …(i)
132. Since, initial and final points are same. p¢ =
2
So, DU A ®B ®C = DU A ®C …(i)
Heat absorbed, DQ = nCp DT
For adiabatic expansion,
Also, A ® B is isochoric process.
= nCp (TB - T A ) …(ii)
So, dWA ®B = 0 pV g = constant
gR æ where, g = 1 + 2 ö
As, Cp = ç ÷ dQ = dU + dW Þ p ¢V g = p ²V ² g
g -1 è fø and
p
So, dQ A ®B = dU A ®B = 400 J Þ (2V ) 5 / 3 = p ² (16V ) 5 / 3
For a monoatomic gas, f = 3 2
æ 5R ö 5/3 5/3
ç ÷ 5 p é 2V ù p æ 1ö
Þ Cp = ç 3 ÷ = R p Þ p² = = ç ÷
5 B C 2 êë 16V úû 2 è8ø
ç - 1÷ 2 6×104Pa
è3 ø pæ 1ö
= ç ÷ = p / 64
Substituting the value of Cp in Eq. (ii), we 2 è 32 ø
get A
2×104Pa 135. According to question,
DQ = n æç R ö÷ (TB - T A )
5
p µT 3 …(i)
è2 ø 2×10–3m3 4×10–3m3 æç p = pressure ÷ö
DW nR (TB - T A ) 2 V èT = temperatureø
Hence, = =
DQ æ 5 ö 5 Next B ® C is isobaric process.
n ç R ÷ (TB - T A ) and we know that
è2 ø
So, dQB ®C = dUB ®C + dWB ®C pV = nRT and pV µ T …(ii)
129. In isochoric process, the curve is = dUB ®C + pDVB ®C
parallel to Y-axis because volume is So, putting Eq. (ii) in Eq. (i),
Þ 100 = dUB ®C + 6 ´ 104 (2 ´ 10-3 )
constant. Isobaric is parallel to X-axis p µ (pV ) 3
because pressure is constant. Along the Þ dUB ®C = 100 - 120 = -20 J
Þ p 2V 3 = constant
curve, it will be isothermal because From Eq. (i),
temperature is constant.
Q DU A ®B ®C = DU A ®C Þ pV 3 / 2 = constant …(iii)
So,P ® c Þ Q ® a Þ R ® d Þ S ® b Þ Comparing Eq. (iii) with pV g = constant.
Þ DU A ®B + DUB ®C = dQ A ®C - dWA ®C
130. The solution of this question can be Þ 400 + (-20) = dQ A ®C é Cp ù
understood by plotting a p-V graph for We have, g = 3 / 2. êQr = C ú
the compression of a gas isothermally - (pDVA + Area of DABC) ë V û

and adiabatically simultaneously to half Þ dQ A ®C = 380 136. According to the question, in first case
of its initial volume. i.e. æ 2 ´ 104 ´ 2 ´ 10-3 ö the volume changeV to 3V isothermally,
+ ç 1 -3 4
÷ which gives a hyperbolic curre and in
ç + ´ 2 ´ 10 ´ 4 ´ 10 ÷
è 2 ø second case the volume reduced from
Adiabatic curve 3V to V isochorically; which gives a
= 380 + (40 + 40)
Isothermal curve parallel line to volume axis.
p dQ A ®C = 460 J
137. In an adiabatic process,
5
133. For a diatomic gas, CV = R DQ = 0
2
So, from first law of thermodynamics,
V/2 V
Volume (V ) 5
A DQ = DU + DW
Since, the isothermal curve is less As, DQ = 0
p (in kPa) 2 B
steeper than the adiabatic curve. So, so, DW = - D U = - nCV D T
area under the p-V curve for adiabatic
4 6 æ R ö
process has more magnitude than V(in m3) = -n ç ÷ (Tf - Ti )
isothermal curve. Hence, work done in è g - 1ø
adiabatic process will be more than in The change in internal energy of gas in nR
isothermal process. the transition from A to B is = (Ti - Tf ) …(i)
g -1
131. As, we know that for polytropic process DU = nCV dT
of index a, specific heat capacity Given, work done,W = 6R J, n = 1mol,
= n æç
5R ö
R ÷ (TB - T A )
= CV + è 2 ø 5
R = 8.31 J/mol-K, g = ,Ti = T K
1- a 3

MODULE 3
www.jeebooks.in
The NEET Edge ~ Physics 393

Substituting given values in Eq. (i), we When this engine is reversed, it takes in h¢ = 40% + 50% of h
get work W and heat q 2 from cold reservoir 40 50
= + ´ 0.4
and ejects 100 J of heat to hot reservoir. 100 100
R
\ 6R = (T - Tf ) \ W + q 2 = q1 = 0.4 + 05
. ´ 0.4 = 0.6
(5/3 - 1)
⇒ 10 + q 2 = 100 or q 2 = 90 J 300
3R Hence, 0.6 = 1 –
Þ 6R = (T - Tf ) 500 + x
2 140. Given, temperature of source,
T = 30° C = 30 + 273 300
Þ T - Tf = 4 Þ = 0.4
Þ T1 = 303 K 500 + x
\ Tf = (T - 4) K
Temperature of sink,T 2 = 4° C = 4 + 273 300
NOTE Adiabatic expansions of mono, Þ 500 + x = = 750
0.4
dia and polyatomic gases are shown T 2 = 277 K
below. Q1 T1 \ x = 750 - 500 = 250 K
1 ® monoatomic 2 ® diatomic As, we know that =
Q2 T2 NOTE All reversible heat engines
3 ® polyatomic Q2 + W T
working between same temperatures are
Þ = 1 {Q W = Q1 - Q 2 } equally efficient and no heat engine can
p Q2 T2 be more efficient than Carnot engine (as
where,Q 2 is the amount of heat drawn from it is ideal).
the sink (atT 2 ), W is work done on working
3
γ = 1.33 substance, Q1 is amount of heat rejected to
12. Kinetic Theory of Gases
2 source (at room temperatureT1).
γ = 1.4
Þ WT 2 + T 2Q 2 = T1Q 2 142. As, we know that
1 γ = 1.67 Þ WT 2 = T1Q 2 - T 2Q 2 1 nm 2
pressure, p = × vrms
Þ WT 2 = Q 2 (T1 - T 2 ) 3 V
V where, nm = mass of the gas,
æT ö
Þ W = Q 2 ç 1 - 1÷
138. Efficiency of an ideal heat engine is çT ÷ V = volume of the gas and
è 2 ø
given as mn
\ = density of the gas = r
W = 600 ´ 4.2 ´ æç - 1ö÷
303
T2 Þ V
h= 1- è 277 ø
T1 1 2 1 3RT
Thus, p = rvrms = r
3 3 M0
W = 600 ´ 4 . 2 ´ æç
where,T1 is the temperature of the 26 ö
÷
source andT 2 is the temperature of the è 277 ø rRT æ 3RT ö
sink. = çQvrms = ÷
W = 2365
. J
M0 è M0 ø
Here, T1 = 100 + 273 = 373 K pM 0 pmN A
Work done W 2365. r= =
T 2 = 0 + 273 = 273 K Power = = =
Time t 1 RT k NA T
273 [Q R = N Ak and M 0 = mN A ]
Þ h= 1-
373 = 2365
. W pm
373 - 273 100 r=
= = = 0.268 141. The efficiency of Carnot engine is kT
373 373
defined as the ratio of work done to the 143. Vessel-I Vessel-II
\ h % = 0.268 ´ 100 = 26.8 %
heat supplied, i.e. Ideal Ideal
139. Consider schematic diagram for a gas A
Work done W Q – Q2 gas B
Carnot engine as shown below. h= = = 1
T2 Low temperature Heat supplied Q1 Q1 T T
reservoir Q T
q2 = 1– 2 = 1– 2 r A = 15
. rB
Q1 T1
p A = 2 pB
W E Here,T1 is the temperature of source,T 2 According to ideal gas equation, we
is the temperature of sink, Q1 is heat rRT
have pressure, p = , where M is
q1 absorbed and Q 2 is heat rejected. M
High temperature 40 molecular weight of ideal gas.
T1 As given, h = 40% = = 0.4
reservoir 100 p RT rRT
Such that, = Þ M =
In case of engine, and T 2 = 300 K r M p
work W 300 300
Engine efficiency = = So, 0.4 = 1 – Þ T1 = where, R andT are constants.
heat absorbed q1 T1 1 - 0.4 r MA rA p
So, M µ Þ = ´ B
W 1 10 J 1 300 p MB rB pA
\ = Þ = T1 = = 500 K
q1 10 q1 10 0.6 1
= 15
. ´
or q1 = 100 J Let temperature of the source be 2
3
increased by x K, then efficiency = 075
. =
becomes 4

MODULE 3
www.jeebooks.in
394 NEET Test Drive

144. The minimum velocity with which the 148. Use ideal gas equation to find the ratio 151. For a particle executing SHM, we have
body must be projected vertically between density of a fixed mass of an maximum acceleration,
upwards, so that is could escape from ideal gas and its pressure. a = Aw 2 …(i)
the Earth’s atmosphere, is its escape Ideal gas equation, where, A is maximum amplitude and w is
velocity (ve ).
pV = nRT angular velocity of a particle.
As, ve = 2gR
pV
=
1 æQn = m ö Maximum velocity,
Substituting the value of g (9.8 ms -2 ) and RT ç ÷
m M è Mø b = Aw …(ii)
radius of Earth (R = 6.4 ´ 106 m), we get
p RT æ V 1ö Dividing Eq. (i) by Eq. (ii), we get
ve = 2 ´ 9.8 ´ 6.4 ´ 10 6 = çQ = ÷
r M è m rø a Aw 2
-1 -1 =
- 11.2 km s = 11200 ms
~ r 1 b Aw
\ µ
Let the temperature of molecule be T p T a 2p 2pb
when it attains ve . Þ =w= i.e. T =
\ Molecular mass M and universal gas b T a
According to the question, vrms = ve constant R remains same for a gas. Thus, its time period of vibration,
where, vrms is the rms speed of the So, for two different situations, i.e. at two 2pb
oxygen moelcule. different temperatures and densities. T =
a
3kBT r1 / p1 T 2
Þ . ´ 103
= 112 \ = 152. As, x = A cos w t
m O2 r 2 / p 2 T1
dx
. ´ 103 ) 2 (m O2 ) x 383 K \ v = = - Aw sin w t …(i)
(112 Þ = dt
or T = (r 2 / p 2 ) 283 K
(3kB ) d 2x
r2 283 and a= = - Aw 2 cos w t …(ii)
Substituting the given values, i.e., Þ = x dt 2
p2 383
. ´ 10-23 JK -1 and
kB = 138 We can find the correct graph by putting
-26 different values of t in Eq. (ii)
m O2 = m = 2.76 ´ 10 kg 13. Oscillations
3 -26
At t = 0, a = - Aw 2
. ´ 10) (2.76 ´ 10
(112 ) 149. Magnitude of velocity of particle when it
we get, T = 2p T ö
At t = , a = - Aw 2 cos æç
T
. ´ 10-23 )
(3 ´ 138 is at displacement x from mean position ´ ÷=0
4 èT 4ø
= w A2 - x 2
= 8.326 ´ 104 K 2p T ö
, a = - Aw 2 cos æç
T
Also, magnitude of acceleration of At t = ´ ÷
145. Total internal energy of system = Internal 2 èT 2ø
particle in SHM = w 2 x
energy of oxygen molecules + Internal = - Aw cos p = + Aw 2
2

energy of argon molecules Given, when x = 2 cm 3T


At t = ,
f f | v | =|a | 4
= 1 n1RT + 2 n 2RT
2 2 ⇒ A - x 2 = w2 x
2 2 p 3T ö
5 3
w a = - Aw 2 cos æç ´ ÷=0
= ´ 2RT + ´ 4RT = 11 RT èT 4 ø
2 2 A2 - x 2 9- 4
Þ w= = At t = T ,
f x 2
146. We know that, Q = nR DT 2p
2 5 a = - Aw 2 cos æç ´ T ö÷ = - Aw 2
⇒ Angular velocity, w = èT ø
\ Amount of heat required, 2
This condition is represented by graph in
3 1 3 \ Time period of motion,
Q = ´ ´ KBN a DT = N aKB (T 2 - T1) option (c).
2 4 8 2p 4p
T = = s
w 5 153. For a simple harmonic motion,
147. Given, M = 4 gm,V = 22.4 L,
d 2y
CV = 5 JK -1 mol -1
150. Let A be the amplitude of oscillation, aµ µ-y
then dt 2
v sound = 952 m / s, Cp = ?
v12 = w 2 (A 2 - x12 ) ¼ (i) Hence, equations y = sin wt - cos wt
gpV
As, velocity of sound, v sound = 3p
v 22 and y = 5 cos æç - 3 w t ö÷ are
2 2
M = w (A - x 22 ) ¼ (ii)
è 4 ø
M 2 Cp Subtracting Eq. (ii) from Eq. (i), we get
Þ g = v sound = satisfying this condition and equation
pV CV v12 - v 22 = w 2 (x 22 - x12 )
y = 1 + wt + w 2t 2 is not periodic and
So, heat capacity at constant pressure, v12 - v 22 y = sin3 wt is periodic but not simple
Þ w=
éM ù 2 x 22 - x12 hormonic motion.
Cp = CV ê ú v sound
ë pV û 2p 154. For a particle executing SHM,
as w=
T Acceleration (a) µ - w 2 displacement (x)
é 4 ´ 10-3 ù
= 5ê 5 -3 ú
(952) 2 2p v12 - v 22 …(i)
ë 10 ´ 22.4 ´ 10 û \ =
T x 22 - x12 Given, x = a sin2 wt …(ii)
20
= ´ (952) 2 ´ 10-5 Differentiating the above equation w.r.t.
22.4 x 22 - x12
Þ T = 2p time, we get
= 809,200 ´ 10-5 = 8.09 J/mol K v12 - v 22

MODULE 3
www.jeebooks.in
The NEET Edge ~ Physics 395

dx New mass = m + 1 159. Let block be displaced through x m, then


= 2aw (sin wt ) (cos wt )
dt m+1 weight of displaced liquid or upthrust,
\ T2 = 2p ...(i) (upwards) is given by Archimedes’
Again differentiating, we get K
principle
d 2x From Eqs. (ii) and (i), we get
= a = 2aw 2 [cos 2 wt - sin2 wt ] Fb = – Axrg
dt 2 T2 m+1 5 m+1
2 = Þ = where, A is the area of cross-section of
= 2aw cos 2 wt T1 m 3 m the block and r is density at liquid. This
The given equation does not satisfy the 25 m + 1 1 must be equal to force (=ma) applied,
condition for SHM [Eq. (i)]. Therefore, Þ = = 1+
9 m m where m is the mass of the block and a
motion is not simple harmonic. is the acceleration.
1 16
155. As we know that, the condition for a Þ =
m 9 \ ma = – Axrg
body executing SHM is F = -kx Arg
9 a=– x = – w2x
F k \ m= kg or
So, a = =- x or a = - w 2 x 16 m
m m
158. Problem solving strategy Calculate the This is the equation of simple harmonic
Acceleration µ - Displacement motion.
effective force constant of parallel
(where, acceleration = A and spring, then by putting the values of time Time period of oscillation,
displacement = y) M
period,T = 2p , we get the new T =
2p
= 2p
m
ÞT µ
1
Aµ-y K
2 w Arg A
A=-w y time period of spring.
k We can write time period for a vertical 160. In case of damped oscillation, amplitude
A=- y
m spring-block system as at any instant t is given by
A = - ky l a = a0 e -bt
T = 2p
Here, y =x +a g where, a 0 = initial amplitude and
\ Acceleration = - k (x + a) Here, l is extension in the spring when the b = damping constant
156. According to the question, equation of mass m is suspended from the spring. a
Case I t = 100T and a = 0
motion of SHM is This can be seen as under 3
p a0
x = a sin æç w t + ö÷ kl = mg (in equilibrium position) \ = a0 e -b (100 T )
è 6ø 3
m l 1
Þ = Þ e -100 bT =
Velocity of body is given by k g 3
p
= aw cos æç w t + ö÷
dx Case II t = 200T
v = \ T = 2p
m
dt è 6ø k \ a = a 0 e -bt = a0 e -b ( 200 T )
aw p
= aw cos æç wt + ö÷
2
= a0 (e -100 bT ) 2 = a0 ´ æç ö÷ = 0
m 1 a
2 è 6ø \ T1 = 2 p
k1 è3ø 9
p
= cos æç w t + ö÷
1
m Thus, after 200 oscillations, amplitude
2 è 6ø Þ k1 = 4 p 2 ...(i)
T12 1
will become times.
p p éQ cos p = 1 ù 9
wt + = m
6 3 êë 3 2 úû T2 = 2p
p 2p p T
k2 14. Waves
Þ wt = Þ t = Þt = m
6 T 6 12 Þ k2 = 4 p 2
...(ii) 161. The general expression of travelling
157. As we know that, time period, T 22 wave can be written as
Since, springs are in parallel, effective y = a sin (wt ± kx ) ...(i)
m
T = 2p force constant For travelling wave along positive X-axis
k
k = k1 + k 2 we should use minus (–) sign only
m \ y = a sin (wt - kx )
\ T = 2p 2 pv 2p
k1 + k 2 But w= and k =
l l
2 m
k Þ k1 + k 2 = 4 p ...(iii) 2p
T2 So, y = a sin (vt - x ) ...(ii)
l
Substituting values of k1 and k 2 from
Given, a = 0.2 m, v = 360 m/s and
Eqs. (i) and (ii) in Eq. (iii), we get
m l = 60 m
m m m
4p2 2 + 4p2 2 = 4p2 2 Substituting in Eq. (ii), we have
T1 T2 T
T1 = 2 p
m 2p
Case I …(i) 1 1 1 y = 0.2 sin (360 t - x )
k Þ = + 60
T2 T12 T 22
y = 0.2 sin 2 p æç 6 t -
Case II When the mass m is increased x ö
or ÷
by 1 kg, then or T -2 = T1-2 + T 2-2 è 60 ø

MODULE 3
www.jeebooks.in
396 NEET Test Drive

162. According to question, we have 165. Sabine’s formula for reverberation 168. Fundamental frequency for an open
time is v
wavelength of transverse pulse, organ pipe is given as f1 =
v 016
. V 2L
l= …(i) T =
f Sas where, L is the length of the open organ
V pipe.
(where, v = velocity of the wave and f = T µ
frequency of the wave) s Third harmonic for a closed organ pipe
where, V is volume of hall in m3 . is given as
T
As we know, v = …(ii) 3v
m Sas = a1s1 + a2s 2 + ... f¢ =
4L ¢
(where,T = tension in the spring and m = = total absorption of the hall (room)
where, L ¢ is the length of the closed
mass per unit length of the rope) Here, s1, s 2 , s 3 ... are surface areas of organ pipe.
From Eqs. (i) and (ii), we get the absorbers and a1, a2 , a3 ... are their According to the question,
1 T respective absorption coefficients. f = f¢
l=
f m So, for two different cases of v 3v
=
Þ lµ T reverberation. 2L 4L ¢
So, for two different cases, we get T ¢ V ¢ s (2) 3 8 2
L = L¢
\ = ´ = = =2
l2 T2 T2 T s¢ V (2) 2 4 3
= Given, L ¢ = 20 cm
l1 T1 m1 Hence, T ¢ = 2T = 2 ´ 1 = 2 s
L 2
p Þ L = ´ 20 cm
m1 + m 2 166. Intensity of sound, I = 3
=
m2 T1 4 pr 2 40
m2 = cm
é where, p = pressure of sound ù 3
T1 = m 2g ê waves, and ú
ê r = distance between ú = 13.3 cm
T 2 = (m1 + m 2 )g ë source and the point û
169. Frequency of nth harmonic in a closed
2
163. According to question, situation can be 1 I1 æ r 2 ö end tube
drawn as follows or I µ or =ç ÷
r 2
I 2 è r1 ø (2n - 1) v
Þ f = n = 1, 2, 3, K
4l
Here, r1 = 2 m, r 2 = 3 m
Also, only odd harmonics exists in a
Substituting the values, we have
closed end tube.
Cliff I1 æ 3 ö 9
2
15 m/s =ç ÷ = Now, given two nearest harmonics are of
Source I2 è 2 ø 4 frequency
Observer 167. In this problem, the fundamental 220 Hz and 260 Hz.
frequencies of each part is given. The (2n - 1) v
\ = 220 Hz ...(i)
Frequency of sound that the observer fundamental frequency of the complete 4l
hear in the echo reflected from the cliff wire could be find. One should check Next harmonic occurs at
is given by each option for the given values. (2n + 1) v
æ v ö = 260 Hz ...(ii)
f¢ = ç ÷f l1 l2 l3 4l
è v - vs ø
On subtracting Eq. (i) from Eq. (ii), we
where, f = original frequency of source, get
v = velocity of sound and l {(2n + 1) - (2n - 1)} v
= 260 - 220
vs = velocity of source 4l
2 æç ö÷ = 40
æ 330 ö v
So, f¢ = ç ÷ 800 = 838 Hz v v è 4l ø
è 330 - 15 ø For Ist part, n1 = Þ l1 =
2 l1 2n1 v
164. Velocity of a wave is given by Þ = 20 Hz
v v 4l
v = nl For IInd part, n2 = Þ l2 =
2 l2 2n 2 \ Fundamental frequency of the system
é where, n = frequency of wave, ù v
êë and l = wavelength of waveúû v v = = 20 Hz
For IIIrd part, n 3 = Þ l3 =
2 l3 2n 3 4l
So, for two different cases, we get
v v 170. For an open organ pipe,
v1 = n1l1 For the complete wire, n = Þ l= n
2l 2n nn = v (where, n = 1, 2, 3, … )
v 2 = n2 l 2 2L
v 3500 We have, l = l1 + l2 + l3
l 2 = l1 2 = l1 ´ For second overtone n = 3,
v1 350 v v v v
= + + 3
2n 2n1 2n2 2n3 (vO ) 2 = v …(i)
= l1 ´ 10 [Qn1 = n2 ] 2L1
1 1 1 1
l 2 = 10 l1 = + + L1 = length of open organ pipe
n n1 n2 n3

MODULE 3
www.jeebooks.in
The NEET Edge ~ Physics 397

2n + 1ö æ v ö
For closed organ pipe, n n = æç ÷v
174.
36 km/h fmin = ç ÷f
è 4L ø 18 km/h
f=1392 Hz è v + vs ø
(where, n = 0, 1, 2, 3, ...) 340
fmin = ´ 385
Ist overtone for closed organ pipe, n = 1 As both observer and source are 340 + 10
3 moving, we can use the formula of
(vc )1 = v …(ii) 34
apparent frequency as = ´ 385
4L 35
From Eqs. (i) and (ii), we get æv + v 0 ö
f = f0 ç ÷ = 34 ´ 11 = 374 Hz
Q (vO )1 = (v c )1 è v + vs ø

Þ
3v
=
3
v
é 343 + 10 ù
= 1392 ê
15. Electric Charges and Fields
ú
2L1 4L ë 343 + 5 û 177. Net charge on one H-atom
Þ L1 = 2L [Qv 0 = 36 km / h = 10 m / s and q = - e + e + De = De
171. As we know that, vs = 18 km / h = 5 m / s] Net electrostatic repulsive force between
two H-atoms
= 1392 é
Beat frequency = f1 ~ f2 353 ù
= 1412 Hz Kq 2 K ( De )2
êë 348 úû Fr = =
= n - (n - 1) = 1
d2 d2
and similarly for n and n + 1 175. According to Doppler's effect, whenever
there is a relative motion between a Similarly, net gravitational attractive force
Beat frequency = n + 1 - n = 1 between two H-atoms
source of sound and the observer
172. Given, Gmh2
(listener), the frequency of sound heard FG =
254 508 5 bea by the observer is different from the d2
t
actual frequency of sound emitted by It is given that, Fr - FG = 0
250 513 source. K ( De )2 Gmh2
at ⇒ - =0
246 492 21 be hill d2 d2
Gmh2
Hence, unknown frequency is 254 Hz. Case I n n′ Þ ( De )2 =
30 ms–1 K
173. Given, as a source of sound S emitting s
O (6.67 ´ 10-11) (1.67 ´ 10-27 ) 2
waves of frequency 100 Hz and an ( De )2 =
9 ´ 109
observer O are located at some n′ -37
Case II n′′ ⇒ De = 1.437 ´ 10 C
distance. Such that, source is moving 30 ms–1
with a speed of 19.4 m/s at angle 60° O 178. Case I Fe cos q = mg sin q
with source-observer line as shown in v
the figure. [for case I] n¢ = n …(i) Þ
Fe
= tan q
v - 30 mg
vs sin 60°
vs é where, n = frequency emitted by car ù
Fe cosθ
ëê and v = velocity of sound ûú
v + 30
[for case II] n¢ ¢ = n ¢ …(ii) θ y
v
é where, n¢ ¢ = frequency heard by ù Fe r
ëê the driver after reflectionûú (90 – θ)
θ
60° Ist case

S Doppler From Eqs. (i) and (ii), we get


s
vs cos 60° line v + 30 360
n¢ ¢ = n= ´ 600 = 720 Hz mg
mg sinθ
v - 30 300
The apparent frequency heared by
Fe
observer 176. Velocity of source (whistle) is given by Þ mg = …(i)
tan q
é v ù vs = rw
fO = fs ê ú Case II Fe ¢ cos q1 = mg sin q1
ë v - vs cos 60° û = (0.5 m) ( 20 rad/s) = 10 m/s
Fe ¢
The frequency of sound observed by the Þ = tan q1
é ù mg
ê 330 ú observer will be minimum when he is at
= 100 ê
1ú point A. Thus, at this point minimum
ê 330 - 19.4 ´ ú frequency of source as observed by Fe cosθ1
ë 2û
observer is
é 330 ù
= 100ê θ1
ë 330 - 97 . úû Fe y/ 2
Observer Fe
= 100é
330 ù C O (90 – θ) r
êë 320.3 úû
θ1
A v
» 103.02 Hz 2nd case
Source X
vs
mg mg sinθ1

MODULE 3
www.jeebooks.in
398 NEET Test Derive

Fe ¢ qinside = -[(6 - 8 y )i$ + (-8 x - 8 + 6z )$j + 6y k$ ]


Þ mg = …(ii) i.e. f total =
tan q1 e0 The value of E at coordinate (1, 1, 1)
From Eqs. (i) and (ii), we get Let electric flux linked with surfaces E = - [-2 i$ - 10$j + 6k$ ]
Fe F ¢ F tan q A, B and C are fA , fB and fC So, E net = (-2) 2 + (-10) 2 + 62
= e Þ e =
tan q tan q1 Fe ¢ tan q1 respectively, i.e. f total = fA + fB + fC
= 2 35 N/C
As, tan q =
P (perpendicular)
=
r Since, fC = fA
and force on charge q due to E net is
b (base) 2y q
\ 2 fA + fB = f total = given by
r r e0
tan q1 = P / b = ´2= F = q E net = 2 ´ 2 35 = 4 35 N
2´y y 1 æq ö
or fA = ç - fB ÷ 185. Electric potential relates with electric
Fe r 2 è e0 ø field according to the following relation,
= ´y
Fe ¢ 2 y ´ r But fB = f (given) dV
E =-
Þ Fe ¢ = 2Fe 1 æq ö dr
Hence, fA = ç - f÷
kq 2 2kq 2 1 2 2 è e0 ø As V = - x 2 y - xz 3 + 4
So, = Þ = ¶V $ ¶V $ ¶V $
r¢ 2
r2 r¢ r So, E = - i- j- k
182. According to Gauss's law, the electric ¶x ¶y ¶z
r flux through a closed surface is equal to
\ r¢ = E = (2 xy + z 3 )i$ + x 2 $j + 3 xz 2 k$
2 1
times, the net charge enclosed by
179. Choose the three coordinate axes as e0 186. The given dipole is shown in the figure
X , Y and Z and plot the charges with the surface. given below
the given coordinates as shown in the As the charge enclosed = q / 8
figure. O is the origin at which - 2 q +q
q q
charge is placed. The system is So, electric flux = enclosed Þ f =
e0 8 e0 θ E
equivalent to two dipoles along X and
Y -directions respectively. The dipole
moments of two dipoles are shown in 16.Electrostatic Potential and
figure.
Capacitance –q
Y
1 s 4 pa 2 1 s 4 pb 2 Here, torque, t = pE sin q
183.VA = -
4 pe 0 a 4 pe 0 b Potential energy of the dipole,
(0, a, 0) 0
q p(a, a, 0)
+
1
×
s 4 pc 2 U = ò t dq = òp/ 2 pE sin qdq
4 pe 0 c
a p = - pE [cos q] qp/ 2
pnet s s
= (a - b + c) = (2 a) = - pE cos q
–2q e0 e0
X (Qc = a + b) 187. We know that, the electric field inside
(0, 0, 0) p q(a, 0, 0)
O 1 s 4 pa 2 1 s 4 pb 2 the dielectrics will be less than the
a VB = × - electric field outside the dielectrics.
4 pe 0 c 4 pe 0 b
Z Also, the electric field inside the
1 s 4 pc 2 dielectrics could not be zero.
+ ×
The resultant dipole moment will be 4 pe 0 c Here, as K 2 > K1, the drop in electric field
directed along OP , where coordinate of
s æa 2 ö s æa 2 ö for K 2 dielectric must be more than K1.
p net is given by (a, a, 0). The magnitude = ç - b + c÷ = ç + a÷
of resultant dipole moment is e0 è b ø e0 è b ø \ The correct graph representing the
2 2 variation of E with d is as shown below.
p net = 2
p + p 2 1 s 4 pa 1 s 4 pb
and VC = × -
4 pe 0 c 4 pe 0 c E
2 2
= (qa) + (qa) 2
1 s 4 pc
= 2 qa + ×
4 pe 0 c
180. As we know, the electric flux f through
s æa 2 b 2 ö s
any surface area is given by, = ç - + c÷ = (2 a)
e0 è c c ø e0
f = E × d S = E dS cos q (Qc = a + b) 0
As according to question, surface area Hence,VC = VA ¹ VB d
is in plane of paper and E is also in
184. As we know that, relation between 188. If the battery is removed after charging,
plane of paper. So, angle between area
potential difference and electric field E then the charge stored in the capacitor
vector and E is 90°. in a particular region is given by remains constant.
So, f = E dS cos 90° = 0 dV é ¶V $ ¶V $ ¶V $ ù q = constant
E=- =-ê i+ j+ k
181. Gauss’s law states that, the net electric dr ë ¶x ¶y ¶z úû Change in capacitance, C ¢ = 0
e A
flux through any closed surface is equal d¢
As V = 6x - 8 xy - 8 y + 6yz
to the net charge inside the surface
dV As d¢ > d
divided by e0 . So, E = -
dr Hence, C¢ < C
MODULE 3
www.jeebooks.in
The NEET Edge ~ Physics 399

As, potential difference between the \ V = 3V 1 q2


Also energy stored is U = CV 2 =
plates of capacitor is given by The equivalent capacitance Cs is given 2 2C
q by
V = As the battery is disconnected from the
C C1 C2 C3 capacitor the charge will not be
+Q –Q +Q –Q +Q –Q
So, for the new capacitor formed, destroyed, i.e. q ¢ = q with the
1 introduction of dielectric in the gap of
V¢ µ [q = constant] V1 V2 V3
C¢ capacitor but the new capacitance will be
q q
As capacitance decreases, so potential C ¢ = CK andV ¢ = =
difference increases. + – C ¢ CK
NOTE If in the above situation, the The new energy stored will be
V
battery remains connected, then the q2
charge stored increases. Thus, the 1 1 1 1 U¢ = Þ DU = U ¢ - U
= + + 2CK
potential difference V becomes constant. Cs C1 C2 C3
q2 æ 1 ö 1 2æ 1 ö
189. Given, capacitor is equivalent to [QC1 = C2 = C3 = C ] = ç - 1÷ = CV ç - 1÷
capacitors with dielectric constant k1, k 2 2C è K ø 2 èK ø
C
Cs =
and k 3 in parallel and k 4 in series with 3 Hence, option (d) is incorrect.
them.
191. Energy stored in a system of capacitors 194. As we know that, the total work done in
Thus, we can write, transferring a charge to a parallel plate
k e (A / 3) capacitor is given as
C1 = 1 0 C
d/2 Q2
W = …(i)
k e (A / 3) 2C
C2 = 2 0 C
d/2 where, C is the capacitance of the
+ –
k 3 e0 (A / 3)
C3 = V capacitor.
d/2 We can also write a relation for work
1
Since, C1, C2 and C3 are in parallel. So, =S CV 2 done as,
their equivalent capacitance is 2
W = F ×d …(ii)
Ceq = C1 + C2 + C3 Also, potential drop remains same in
where, F is the electrostatic force
2 e0 A parallel combination across both
= (k1 + k 2 + k 3 ) between the plates of capacitor and d is
3d capacitors.
the distance between the plates.
k e A 2k 4 e0 A Initially stored energy,
Also, C4 = 4 0 = From Eqs. (i) and (ii), we get
d/2 d 1
U1 = CV 2 Q2
As, Ceq and C4 are in series. 2 W = = Fd
C ´ Ceq 2C
\ Net capacitance, C = 4 Finally potential drop across each
Q2
Ceq + C4 capacitor will be still V. Þ F = …(iii)
2k 4 e 0 A 2 e0 A 2Cd
´ (k1 + k 2 + k 3 ) So, finally stored energy,
1 1 As, the capacitance of a parallel plate is
d 3d
= U 2 = CV 2 + CV 2 given as
2k 4 e 0 A 2 e0 A 2 2 e A
+ (k1 + k 2 + k 3 ) C = 0
= (2C) V 2 = 2 æç CV 2 ö÷ = 2U1
d 3d 1 1
d
k e A 2 e0 A æ k 4 (k1 + k 2 + k 3 ) ö 2 è2 ø
or 0 = ç ÷ Substituting the value of C in Eq. (iii), we
d d è 3k 4 + k1 + k 2 + k 3 ø 192. Case I When the capacitors are joined get
æ k (k + k 2 + k 3 ) ö in series, Q 2d Q2
Þ k = 2ç 4 1 ÷ F = =
è 3k 4 + k1 + k 2 + k 3 ø 1 C1 2 e0 Ad 2 e0 A
U series = (4V ) 2
2 3 1 2 n1 This means, electrostatic force is
or = +
k k1 + k 2 + k 3 k 4 Case II When the capacitors are joined independent of the distance between
in parallel, the plates.
190. In series arrangement charge on each
1 195. The two condensers in the circuit are in
plate of all the capacitors have same Uparallel = (n2 C2 ) V 2
magnitude and equal to the main 2 parallel order. Hence,
charge supplied by the battery. But Given, U series = Uparallel C 3C
C¢ = C + =
potential difference across each 1 C1 1 2 2
So, (4V ) 2 = (n 2 C2 ) V 2
capacitor is different. Such that 2 n1 2 The work done in charging the
V = V1 + V2 + V3 equivalent capacitor is stored in the
16C1 form of potential energy.
Also, in this combination potential Þ C2 =
difference distributes in the inverse ratio n2 n1 1
Hence, W = U = CV 2
1 2
of capacitance, i.e.,V µ 193. When a parallel plate air capacitor
C So, for the equivalent capacitor C ¢
connected to a cell of emf V , then
Here, C1 = C2 = C3 = C charge stored will be 1 æ 3C ö 2 æ 3C ö 3 2
Þ V1 = V2 = V3 = V q = ç ÷ V çC ¢ = ÷ = CV
q = CV ÞV = 2 è 2 ø è 2 ø 4
V =V + V + V C
MODULE 3
www.jeebooks.in
400 NEET Test Derive

196. When the switch S is connected to point 199. Given, R = (47 ± 4.7) kW 1 1 1
= + …(i)
1, then initial energy stored in the 3
= 47 ´ 10 ± 10% W R eff R1 R 2
capacitor can be given as Then, R1 = 10 + 30
1 As per the colour code for carbon
= (2mF) ´ V 2 . resistors, the colour assigned to numbers. R1 = 40
2
4 – Yellow Now R 2 = 90 + 30 = 120
When the switch S is connected to point 7 – Violet R 2 = 120
2, energy dissipated on connection
3 – Orange By Eq. (i), we get
across 8 mF will be
1 æ CC ö For ±10% accuracy, the colour is silver. 1 1 1
= +
= ç 1 2 ÷ ×V 2 Hence, the bands of colours on carbon Reff 40 120
2 è C1 + C2 ø
resistor in sequence are yellow, violet, 40 ´ 120 4800
1 2 mF ´ 8 mF orange and silver. Reff = = = 30 W
= ´ ´V2 120 + 40 160
2 10 mF Note To remember the colour code
1 sequence for carbon resistor, the In the balancing condition,
= ´ (16 . mF) ´ V 2 7
2 following sentence should be kept in \ Current, I =
Therefore, % loss of energy memory. B B Roy of great Britain has a (30 + 5)
16
. very good wife. 7 é E ù
= ´ 100 = 80%. = = 0.2 A êQI =
2 200. If n identical cells are connected in 35 ë R + r úû
series, then
Equivalent emf of the combination, 203. For first case, balanced condition of
17. Current Electricity meter bridge will be
E eq = nE
5 R
197. Volume of material remains same in Equivalent internal resistance, = ...(i)
stretching. l1 (100 - l1)
r eq = nr
As volume remains same, A1L1 = A2L2 E nE Now, by shunting resistance R by an
\ Current, I = eq = equal resistance R, new resistance in that
Now, given L2 = nL1 r eq nr
AL A R
\ New area A2 = 1 1 = 1 E arm become .
L2 n or I= = constant 2
r
So, new balanced condition will be
Resistance of wire after stretching, Thus, current I is independent of the 5 R /2
L nL1 number of cells n present in the circuit. = ...(ii)
R2 = r 2 = r × 1.6l1 (100 - 1.6 l1)
A2 A1 / n Therefore, the graph showing the
From Eqs. (i) and (ii), we get
æ l ö é æ L1 ö ù relationship between I and n would be
= çr 1 ÷ × n 2 = n 2 × R êQR = çr ÷ ú 1.6 (100 - 1.6l1)
è A1 ø è A1 ø û
as shown below. = ´2
ë 1 100 - l1
198. Net resistance of a metal wire having I Þ 160 - 1.6 l1 = 200 - 3.2 l1
resistivity r, we have 1.6 l1 = 40
40
L L l1 = = 25 m
A 1.6
ρ1 ρ2 Þ Substituating the value of l1 in Eq.(i),
we get,
L O n
R1 = r1 5 R
= Þ R = 15 W
A 201. V 2A 2 Ω 1Ω 25 75
+ –
L A VB
Similarly, R 2 = r 2 3V
A 204. When a cell is balanced against
Applying KVL, potential drop across a certain length of
Then, net effective resistance of two
VA + 3 = VB + 2 ´ 2 + 2 ´ 1 potentiometer wire, no current flows
metal wires,
VA - VB - 3 = 4 + 2, VA - VB = 9 V through the cell.
R eq = R1 + R 2
202. Effective resistance, \ emf of cell = potential drop across
2L L L
Þ r = r1 + r 2 balance length of potentiometer wire.
A A A P Q
10 Ω 30 Ω So, potentiometer is a more accurate
Þ 2r = r1 + r 2
device for measuring emf of a cell or no
1 current flows through the cell during
As, conductivity s = , we have
r G 50 Ω measurement of emf.
2 1 1
= +
s s1 s2 R S 205. According to question, emf of the cell is
2 s1 + s 2 30 Ω 90 Ω directly proportional to the balancing
Þ =
s s1 s 2 length,
Þ Net effective conductivity of i.e. E µl …(i)
combined wires, Now, in the first case, cells are
2 s1s 2 connected in series to support one
s=
s1 + s 2 7 V, 5 Ω another,
i.e. Net emf = E 1 + E 2
MODULE 3
www.jeebooks.in
The NEET Edge ~ Physics 401

From Eq. (i), andV = IR for the second case, R


Þ t = ´ t1 [t1 = 10 min]
E 1 + E 2 = 50 cm (given) …(ii) I (r + R ) l R1
= 1
IR 4
Again cells are connected in series in l2 \ t = ´ 10 = 8 min
opposite direction i.e.Net emf = E 1 - E 2 æl ö 5
So, r = R ç 1 - 1÷ = 9.5 æç - 1ö÷
3
From Eq. (i), E 1 - E 2 = 10 …(iii) è l2 ø è 2.85 ø 212. If a rated voltage and power are given,
2
From Eqs. (ii) and (iii), we get Vrated
= 9.5 (1.05 - 1) = 9.5 ´ 0.05 then Prated =
E 1 + E 2 50 R
= = 0.475 ~
- 0.5 W
E1 - E 2 10 P
209. Given, charge Q = at - bt 2 …(i) \ Current in the bulb, i = (Q P = Vi )
E1 5 + 1 6 3 V
Þ = = = dQ 500
E2 5-1 4 2 Q We know that current, I = i ==5A
dt 100
206. Given, l = 4 m,
R = potentiometer wire resistance = 8W So, Eq. (i) can be written as \ Resistance of bulb,
d
and Potential gradient =
dV
= 1 mV/cm I = (at - bt 2 ) Þ I = a - 2bt …(ii) V 2 100 ´ 100
dt Rb = = = 20 W
dr P 500
So, for 400 cm, For maximum value of t, till the current
Q Resistance R is connected in series.
exist is given by
DV = 400 ´ 1 ´ 10-3 = 0.4 V E 230
Þ a - 2bt = 0 \ Current, i = =
Let a resistor RS connected in series, so a Rnet R + Rb
as \ t = …(iii)
V 2 2b 230
DV = ´ R Þ 0.4 = ´8 Þ R + 20 = = 46
Q The total heat produced H can be 5
R + RS 8+R
given as \ R = 26 W
16 t a / 2b
Þ 8+R = = 40 Þ R = 32 W I 2R dt = (a - 2bt ) 2 R × dt
0.4
H = ò0 ò0 213. Potential drop between two cities is
ìQ t = a ü = 150 ´ 8 = 1200 V
207. Consider a potentiometer wire of length í ý
L and a resistance r are connected in a / 2b î 2b þ Average resistance of total wire
(a - 2bt ) 3
series with a battery of emf E 0 and a = R = 05
. ´ 150 = 75 W
3 (-2b )
resistance r1 as shown in the figure. 0 V2
So, power loss P =
E0 Solving above equation, we get R
Current in wire, AB =
r1 + r a 3R 1200 ´ 1200
Þ H = = = 19200 W
r1 6b 75
210. Vit = ms DT = 19.2kW
E0 ms D T 1 ´ 4200 ´ 80 V2
Þt = = = 38182
. s 214. Power, P =
l P Vi 220 ´ 4 R
A r
B 381.82 DP
L = = 6.3 min For small variation, ´ 100
60 P
DV
211. Let R1 and R 2 be the resistances of the =2´ ´ 100 = 2 ´ 2.5 = 5%
E V
Potential gradient, coils, V be the supply voltage, H be the
Therefore, power would decrease by 5%.
é E ùr heat required to boil the water.
lr
x = =ê 0 ú V 2t1 215. Electric power, P = i 2R
L ë r1 + r û L For first coil, H = ...(i)
R1 P
emf produced across E will be given by \ Current, i =
V 2t 2 R
For second coil, H = ...(ii)
é E r ùl
R2 For resistance of 9 W,
E = x×l = ê 0 ú
Equating Eqs. (i) and (ii), we get 36
ë r1 + r û L i1 = = 4 = 2A
t1 t 9
= 2
208. Given, e = 2 V, l = 4 m, l1 = 3 m and R1 R 2 i ´R 2´ 9
i2 = 1 = =3A
l2 = 2.85 m R2 t 2 40 ét1 = 10 min ù 6 6
i.e. = = =4
R1 t1 10 êët 2 = 40 minúû
Potential drop per unit length, I = i1 + i 2 = 2 + 3 = 5 A
e 2 Þ R 2 = 4R1 ...(iii) So, V (2W) = I ´ 2 = 5 ´ 2 = 10 V
f = = = 0.5 V / m
l 4 When the two heating coils are in parallel,
216. Voltage across 2 W = 3 ´ 2 = 6 V
Case I e ¢ = fl1 ...(i) equivalent resistance is given by
(e ¢ = emf of the given cell) R1R 2 R ´ 4R1 4R1 Voltage across 4 W and (5 W + 1 W)
R = = 1 =
Case II V = fl2 ...(ii) R1 + R 2 R1 + 4R1 5 resistor is same.
V 2t So, current across,
From Eqs. (i) and (ii), we get and H = ...(iv)
5W=
6
= 1A éQi = V ù
e¢ l R
= 1 1+ 5 êë R úû
V l2 Comparing Eqs. (i) and (iv), we get
V 2t1 V 2t Power across 5 W = P = i 2R
Q e ¢ = I (r + R ) =
R1 R = (1) 2 ´ 5 = 5 W

MODULE 3
www.jeebooks.in
402 NEET Test Derive

217. The resistance of 6 W and 3 W are in Given, i = 1.5 A,t = 10 min = 10 ´ 60 s, Similarly, for bigger amperian loop.
parallel in the given circuit, their Z = 30 ´ 10-5 g C-1 m I
B ¢ ´ 2 p (2a) = m 0I Þ B ¢ = 0
equivalent resistance is Hence, mass of copper deposited on 4 pa
1 1 1 1+ 2 1 the electrode at distance 2a from the axis of the wire.
= + = =
R1 6 3 6 2 m = 30 ´ 10-5 ´ 15
. ´ 10 ´ 60 B m I 4 pa
So, ratio of = 0 ´ =1
or R1 = 2 W = 27 ´ 10-2 = 0.27 g B ¢ 4 pa m 0I
Again, R1 is in series with 4 W
223. The magnetic field in the different
resistance, hence 18.Moving Charges and regions is given by
R = R1 + 4 = 2 + 4 = 6 W
Thus, the total power dissipated in the
Magnetism Z
V2 221. As from question,
circuit, P = I
R 2
A R
Y
Here, V = 18 V, R = 6 W O
I1 1I I
2 3
(18) O
Thus, P = = 54 W C D
6
P
218. Resistances 1 W and 3 W are connected I2 X
m0 I
in series, so effective resistance Z B1 =´ (- k$ )
B 4p R
R¢ = 1+ 3 = 4 W m I
The point P is lying at a distance d B 3 = 0 (- k$ )
Now, R ¢ and 8 W are in parallel. We 4 pR
know that, potential difference across along the Z-axis m I
m 0 I1 B 2 = 0 (- $i)
resistances in parallel is same. As, magnetic field B1 is given by = 4 pR
1Ω 3 Ω i1 2p d The net magnetic field at the centre O is
and magnetic field B 2 is given by B = B1 + B 2 + B 3
i m I
i = 0 2 m I m I
2p d = 0 (-2 k$ ) + 0 (- i$)
4 pR 4R
P m I
8Ω i2
B1 = - 0 (2 k$ + pi$ )
4 pR
So, from Kirchhoff’s law,V1 = V2 224. Initial acceleration is given by
R ¢ ´ i1 = 8 i 2 or 4 ´ i1 = 8 i 2 eE
a0 = (west) (F = eE )
8 m
or i1 = i 2 = 2 i 2 or i1 = 2 i 2 ...(i)
4 B2 a m
Þ E = 0 …(i)
Power dissipated across 8 W resistance is e
B1 and B 2 are ^ to each other. ev 0B + eE
i 22 (8) = 2 W [QP = i 2R ] \ = 3a 0
2 2 So, B net is given by m
or i 2 = = 0.25 W ...(ii) m0 1 2
8 B net = B12 + B 22 = (I1 + I 22 )1/ 2 or ev 0B + eE = 3a0m
Power dissipated across 3 W resistance is 2p d
E (West)
P = i12 (3) = (2 i 2 ) 2 (3) = 12 i 22 222. Consider two amperian loops of radius
but i 22 = 0.25 W a
and 2a as shown in the diagram.
\ P = 12 ´ 0.25 = 3 W 2
Applying Ampere’s circuital law for
219. Mass of the substance deposited at the
these loops, we get North
cathode is given by
m = Z it East
(Z = electrochemical equivalent)
a/2
= Z æç ö÷ t
W
èV ø
B (down)
100 ´ 103 2a
= 0.367 ´ 10-6 ´ ´ 60
125 \ ev 0B = 3ma0 - eE
-3 Þ = 3 ma0 - ma0 [from Eq. (i)]
= 17.61 ´ 10 kg
220. Mass deposited is directly proportional ò B . dl = m 0I enclosed \ B =
2ma0
,
to the charge flows. For the smaller loop ev 0
2 (in vertically downward direction)
i.e. m µ q or m = Zq a I æa ö
Þ B ´ 2p = m0 ´ ´ p ç ÷
where, Z is a constant of proportionality 2 pa 2 è2ø 225. As we know that, radius of a charged
and is called electrochemical 1 m 0I particle in a magnetic field B is given by
= m 0I ´ = mv
equivalent (ECE) of the substance. 4 4 r = K (i)
If an electric current i flows through the m 0I a qB
Þ BI = , at distance from the
electrolyte, then 4 pa 2 where, q = charge on the particle,
m = Zit [q = it ] axis of the wire. v = speed of the particle
MODULE 3
www.jeebooks.in
The NEET Edge ~ Physics 403

\ The time taken to complete the circle, 227. Consider the given figure, 230. As the net force on closed loop is equal
2pr T m to zero. So, force on QP will be equal
T = Þ = [from Eq. (i)] B C
v 2p qB Y and opposite to sum of forces on other 3
2 p qB sides.
\ w= = I
T m i L F1 F3
e
Q q = e and = 1.76 ´ 1011 C/kg
m X A D
B = 3.57 ´ 10-2 T L/2 L
F2
B æçQ = f ö÷
2p eB 1 e 1
Þ = Þf = From the above figure, it can be seen
T m 2p m è T ø So, from vector laws,
that the direction of currents in a long
1 11 -2 straight conductor XY and arm AB of a FQP = (F 3 - F1) 2 + F 22
= ´ 1.76 ´ 10 ´ 3.57 ´ 10
2p square loop ABCD are in the same
231. Given, N = 50, B = 0.2 Wb/m2 , I = 2 A,
= 1.0 ´ 109 Hz = 1 GHz direction. So, there exist a force of
attraction between the two, which will q = 60° , A = 012 . = 0.012 m2
. ´ 01
226. Firstly, make a free body diagram of the
system and indicate the magnitude and be experienced by FBA as
m 0IiL B
direction of all the forces acting on the FBA =
2 p æç ö÷
body. Then, choose any two mutually L
perpendicular axes say X andY in the è2ø 30°
plane of forces in case of coplanar M
In the case of XY and arm CD, the 60°
forces. direction of currents are in the opposite
According to the question, direction. So, there exist a force of
θ repulsion which will be experienced by
s m 0IiL
co CD as FCD =
B F
2 p æç ö÷
3L
è 2 ø
θ Therefore, net force on the loop ABCD Thus, torque required to keep the coil in
F will be stable equilibrium, i.e.
θ t = NIAB sin q
m IiL é 1 1 ù
θ F loop = FBA - FCD = 0 ê - ú = 50 ´ 2 ´ 0.012 ´ 0.2 ´ sin 60°
mg 2p ë (L / 2) (3L / 2) û
sin θ 3
m
g 2m 0iI = 50 ´ 2 ´ 0.012 ´ 0.2 ´ = 0.20 N-m
Floop = 2
3p
As, the system is in equilibrium, 232. Current sensitivity of a moving coil
SF x = 0 228. Force on AB is given by FAB = 0
galvanometer is the deflection q per unit
or mg sin q = F cos q …(i) According to the question, current I flowing through it, i.e.
A q NAB
where, F is the magnitude of force IS = = …(i)
experienced by the rod when placed in I k
a magnetic field and current I is flowing where, N = number of turns in the coil,
through it. A = area of each turn of coil,
But the force experienced by the given B C B = magnetic field and
rod in a uniform magnetic field is FAB + FBC + FCA = 0 k = restoring torque per unit twist
F = ILB FBC + FCA = 0 of the fibre strip.
\ Eq. (i) becomes, FCA = - FBC = - F Similarly, voltage sensitivity is the
mg sin q = ILB cos q 229. As force on wire B due to A and C are deflection per unit voltage, i.e.
mg sin q q
VS = = æç
NAB ö I NAB
Þ I = attractive, so we have following ÷ = …(ii)
LB cos q condition V è k øV kRG
mg F where, RG is the resistance of the
= tan q B
LB galvanometer.
m g tan q
I = æç ö÷ …(ii) From Eqs. (i) and (ii), we get
èL ø B I
RG = S …(iii)
m VS
Here, = 0.5 kg m-1, g = 9.8 ms -2 ,
L Here, IS = 5 div/mA = 5 ´ 103 div/A
q = 30° and B = 0.25 T and VS = 20 div/V
F Resultant
Substituting the given values in Eq. (ii), Substituting the given values in Eq. (iii),
we get m I2
F = 0 we get
0.5 ´ 9.8 2 pd
I = tan 30° 5 ´ 103
0.25 Resultant force on B = F12 + F 22 RG = = 250
0.5 ´ 9.8 1 20
= ´ = 11.32 A m 0I 2 m I2
0.25 3 = 2F = 2 ´ = 0 \ The resistance of the galvanometer is
2 pd 2 pd 250 W.

MODULE 3
www.jeebooks.in
404 NEET Test Derive

233. For ammeter, 238.Q Work done in rotating the magnet, 240. Find out the relation of time period with
a W = MB (cos q 0 - cos q) the earth’s horizontal magnetic field and
0.002 I G where, M = magnetic moment of the then compare it for the two given cases.
A magnet Time period in vibration magnetometer
0.9988 I and B = magnetic field. is given by
W = MB (cos 0° - cos 60° ) l
rs T = 2p
= MB æç1 - ö÷ =
1 MB M ´ BH
0.002I ´ G = 0.998I ´ rs è 2ø 2 1
Þ T µ
0.002 1 \ MB = 2W …(i) BH
rs = G Þ rs = ´G
0.998 499 Torque on a magnet in this position is
So, for two different cases
Equivalent resistance of ammeter, given by, t = M ´ B
T1 (BH ) 2 2 18
1 1 1 = MB × sin q = 2W × sin 60° [from Eq. (i)] Þ = Þ =
= + 3 T2 (BH )1 T2 24
R G rs = 2W =W 3
2 \ T = 2.3 s » 2 s
1 1 1 G
\ = + ÞR =
R G G / 499 500 239. Let BH and BV be the horizontal and 241. Diamagnetic substances are weakly
234. The full scale deflection current, vertical component of earth’s magnetic magnetised in a direction opposite to
-3 field B. that of applied magnetic field. These are
25 ´ 10
ig = A Plane 1 repelled in an external magnetic field i.e.
G Magnetic meridian
have a tendency to move from high to
where, G is the resistance of the meter. low field region, i.e. it is repelled by both
The value of shunt required for north and south poles of a bar magnet.
converting it into ammeter of range 25 A BH sin θ 242. As, the source of current is switched on,
i G G a magnetic field sets up in between the
is S = g Þ S = ig (as i >> i g )
θ′

poles of the electromagnet. As, we know


°–

i - ig i
90

θ′ that a diamagnetic substance when


So that, placed in a magnetic field acquires a
25 ´ 10-3 G 25 ´ 10-3 BH cos θ Plane 2
S » × = = 0.001W feeble magnetism opposite to the
G i 25 B direction of magnetic field.
tan q = V
BH Also, in the presences of the field
19. Magnetism and Matter ⇒
B
cot q = H ...(i)
(non-uniform), these substances are
BV attracted towards the weaker field, i.e.
235. Magnetic moment is from S to N they move from stronger to weaker
So, M net = M 2 + M 2 + 2M 2 cos q Let plane 1 and 2 are mutually magnetic field. Due to these reasons,
perpendicular planes making angle q
M net will be maximum if cos q is the rod is repelled by the field produced
and (90° - q) with magnetic meridian.
maximum. cos q will be The vertical component of earth of
to the current source. Hence, it is
maximum when q will be minimum. earth’s magnetic field remain same in
pushed up, out of horizontal field and
So, at q = 30°, M net will be maximum. gains gravitational potential energy.
two plane but effective horizontal
components in the two planes is given by 243. In diamagnetic substances, in each pair
236. The magnetic moment, M = ml of electrons, the spin of both the
B1 = BH cos q¢ ...(ii)
æç where, m = pole strength andö÷ and B 2 = BH sin q¢ ...(iii)
electrons are in opposite directions.
è l = length of magnet ø Hence, the electrons of each pair
B BV
p Then, tan q1 = V = completely cancel the magnetic moment
According to question, l = ´ r B1 BH cos q¢
3 of each other. Thus, the net magnetic
3l B cos q¢ moment of each atom of such
So, r = cot q1 = H ...(iv)
BV substances is zero, i.e. md = 0.
p
New magnetic moment, M ¢ = m ´ r Similarly, Ms
3l 3 3M BV BV
=m ´ = × ml = ⇒ tan q 2 = =
p p p B 2 BH sin q¢
B sin q¢
237. When magnetic dipole is rotated from ⇒ cot q 2 = H ...(v)
initial position q1 to final position q 2 , BV
then work done = MB (cos q1 - cos q 2 ) From Eq. (iv) and Eq. (v), we get
Given, q1 = 0°, q 2 = 60° ⇒ cot 2 q1 + cot 2 q 2 Ms
Magnetic moment, M = 2 ´ 104 J/T BH2 cos 2 q¢ BH2 sin2 q¢ The property of paramagnetism is found
= +
Magnetic field, B = 6 ´ 10-4 T BV2 BV2 in those substances whose atoms or
molecules have an excess of electrons
So, W = MB æç1 - ö÷
1 ⇒ cot 2 q1 + cot 2 q 2
è 2ø spinning in same direction. Hence,
B H2 atoms of paramagnetic substances have
2 ´ 104 ´ 6 ´ 10-4 = (cos 2 q¢ + sin2 q¢ )
= = 6J B V2 permanent magnetic moment, i.e.
2 m p ¹ 0.
⇒ cot 2 q1 + cot 2 q 2 = cot 2 q

MODULE 3
www.jeebooks.in
The NEET Edge ~ Physics 405

The property of ferromagnetism is found V


247. Net flux through solenoid is =
in substances which acquire very strong 2
R 2 + æç
f net = Nf 1 ö
magnetism when placed in an external ÷
\ f net = 500 ´ 4 ´ 10-3 = 2 Wb è 2pfC ø
magnetic field. Like the paramagnetic
where, f = flux through each turn and 2 pfC
substances each atom of ferromagnetic Þ I = ´V
substances also has a permanent N = total number of turns. 4 p 2f 2C 2R 2 + 1
magnetic moment, i.e. mf ¹ 0. Also, fnet = Li = 2 Wb Voltage drop across capacitor,
Ms Now, L ´ 2 = 2 2 pfC 1
VC = I ´ X C = ´
Ms Þ Self-inductance, L = 1 H 4 p 2f 2C 2R 2 + 1 2 pfC
S
248. Given, number of turns of solenoid, V
i.e. VC =
N = 1000 2 2 2 2
4p f C R + 1
N
Current, I = 4A where, K is the dielectric constant or the
Magnetic flux, fB = 4 ´ 10-3 Wb medium.
QSelf-inductance of solenoid is 1
Þ VC µ
20. Electromagnetic Induction given by KC + 1
f ×N
244. Induced emf in the region is given by L= B …(i) When mica is introduced, capacitance
I
df will increase, hence voltage across
e=- Substitute the given values in Eq. (i), we capacitor get decrease (Va > Vb )
dt
get
where, f = BA = pr 2B 252. As we know that,
4 ´ 10-3 ´ 1000
df dB L= = 1H XL wL 3
Þ = - pr 2 4 tan f = = Þ tan f =
dt dt R R 3
Rate of change of magnetic flux
21. Alternating Current \ tan f = 1 Þ f = 45°
associated with loop 1 p
So, phase difference = rad
df dB 249. Compare the given equation with the 4
e1 = - 1 = - pR 2 equation of an alternating voltage
dt dt 253. Here, inductance,
-3
i.e. e = em sin w t L = 20 mH = 20 ´ 10 H
Similarly, e 2 = emf associated with loop 2
df 2 where, e m = erms 2 Capacitance, C = 100mF = 100 ´ 10-6 F
=- =0 [Q f 2 = 0]
dt Given, emf, e = 200 2 sin 100 t Resistance, R = 50 W
245. Current induced in the coil is given by and C = 1 mF = 1 ´ 10-6 F emf, V = 10 sin 314 t …(i)
1 æ df ö ⇒ Dθ = 1 æ Df ö QThe general equation of emf is given as
i = ç ÷ ç ÷ As, erms = 200V and w = 100 V = V0 sin wt …(ii)
R è dt ø Dτ Ρ è Dτ ø
1 1 \ Comparing Eqs. (i) and (ii), we get
Given, resistance of the solenoid, \ XC = = = 104 W
w C 1 ´ 10-6 ´ 100 V0 = 10 V, w = 314 rad s -1
R = 10p 2 W e 200
Radius of second and coil, r = 10-2 irms = rms = = 2 ´ 10-2 A = 20mA The power loss associated with the
XC 104 given AC circuit is given as
Dt = 0.05 s, Di = 4 - 0 = 4 A
P = Vrms Irms cos f = Vrms æç rms ö÷ æç ö÷
V R
Charge flowing through the coil is 250. When a bulb of resistance R is
è Z ø èZ ø
given by connected in series with a coil of
2 2
æ V ö
= æç rms ö÷ R = ç 0 ÷ R
Df 1 self-inductance L, then current in the V
Dq = æç ö÷ (Dt ) …(iii)
è Dt ø R circuit is given by è Z ø è 2 × Zø
Di 1 E
= m 0N1N 2 pr 2 æç ö÷ Dt I = , where E is the voltage \ Impedance, Z = R 2 + (X L - X C ) 2
è Dt ø R w 2L2 + R 2
2
= 4 p ´ 10-7 ´ 2 ´ 104 ´ 100 ´ p of an AC source. = R 2 + æç wL -
1 ö
÷
è wC ø
´ ( 10-2 ) 2 ´ æç
4 ö 1 m m N 2A
÷ ´ ´ 0.05 As, L = 0 r Þ L µ mr
è 0.05 ø 10p 2 l \ Substituting the given values in the
-6
= 32 ´ 10 C = 32 mC When iron rod is inserted, L increases, above equation, we get
2
246. As we know that, therefore current I decreases. é 1 ù
di = (50) 2 + ê (314 ´ 20 ´ 10-3 ) - ú
emf induced, e = - L 251. Net reactive capacitance, ë 314 ´ 10-4 û
dt C
T di R = 2500 + [6280 ´ 10-3 - 0.00318 ´ 104 ]2
During 0 to , = constant
4 dt = 2500 + (25.56) 2 = 56.15 W ~
- 56 W
i Vc
So, e = negative
T T di Now, substituting this values in Eq. (iii),
For to , =0 we get
4 2 dt V=V0 sin ωt 2
So, e=0 æ 10 ö 100
XC =
1 P =ç ÷ ´ 50 = ´ 50
T 3T di è 2 ´ 56 ø 2 ´ 3136
For to , = constant 2pfC
4 4 dt V = 0.79 W
So, current in circuit, I = Thus, power loss in the circuit is 0.79 W.
So, e = positive Z
MODULE 3
www.jeebooks.in
406 NEET Test Derive

254. Power factor of the L-C-R circuit Given, f = - 45° (minus sign shows the
R IR 80 voltage lagging)
22. Electromagnetic Waves
= cos f = = =
Z IZ I (X L - X C ) 2 + R 2 1
wL – 261. Here, velocity of EM wave, v = v $i
=
80 \ tan (-45° ) = w C
Instantaneous oscillating electric field,
(IX L - IX C ) 2 + (IR ) 2 R
E = E $j
1
80 wL - As we already know that, during the
= w C Þ -R = wL - 1
Þ -1 = propagation of EM waves through a
(100 - 40) 2 + (80) 2 R wC
80 80 1 1 medium, oscillating electric and
= = = 0.8 Þ wC = ÞC = magnetic field vectors are mutually
100 (wL + R ) w (wL + R )
(60) 2 + (80) 2 perpendicular to each other and to the
1
= direction of each other and to the
255. As, we know that phase difference for L, 2 pf (2 pfL + R ) direction of propagation of the wave
C, R series circuit is given by (E ´ B).
X - XC 258. Just after switch is closed, inductor acts
tan f = L Þ (E $j) ´ B = v $i …(i)
R like an open switch (open path) and
p X capacitor acts like a closed switch As we know that from vector algebra,
When L is removed, tan = C $j ´ k$ = $i …(ii)
3 R (closed path) because in DC circuit
XC inductive resistance becomes zero. Comparing Eqs. (i) and (ii), we get
3 = Þ XC = R 3
R Just after switch is closed, given circuit B = B k$ ,
p X is equivalent to the circuit shown below. where, B (say) be the magnitude of
When C is removed, tan = 3 = L
3 R magnetic field.
Þ XL = R 3 R Thus, we can say that the direction of
+ oscillating magnetic field of the EM
Hence, in resonant circuit – R
3R - 3R R wave will be along + z-direction.
tan f = =0
R 262. Given, root mean square value of electric
So, f=0 field, E rms = 6 V / m
Power factor, cos f = 1 We know that, peak value of electric field,
It is the condition of resonance, ⇒ + E 0 = 2 E rms
R R
therefore phase difference between – ⇒ E0 = 2 ´ 6 V / m
voltage and current is zero and power E
Also, we know that, c = 0
factor, cos f = 1. B0
R 9
256. Current in L-C-R series circuit, So, equivalent resistor = = ohms where, c = speed of light in vacuum,
V 2 2
i = B 0 = peak value of magnetic field
R 2 + (X L - X C ) 2 Battery emf, V = 18 volts
E
V 18 ´ 2 ⇒ B0 = 0
\ Current in circuit = = =4A c
where,V is rms value of voltage, R is R 9
resistance, X L is inductive reactance 2 ´6
Þ B0 =
and X C is capacitive reactance. No option is matching. 3 ´ 108
For current to be maximum, 259. For better tuning, peak of current growth 8.48
⇒ B0 = ´ 10-8
denominator should be minimum which must be sharp. This is ensured by a high 3
can be done, i.e. during the resonance value of quality factor Q. ⇒ B 0 = 2.83 ´ 10-8 T
of series LCR circuit. Now, quality factor is given by,
1 1 L 263. It is known that, an electric charge at
X L = X C , i.e. w L = Q = rest has electric field in the region
wC R C
around it, but no magnetic field. A
1 From the given options, highest value of moving charge produces both the
or L = 2 …(i)
w C Q is associated with R = 15 W,L = 3.5 H electric and magnetic fields. If a charge
Given, w = 1000 s -1, and C = 30 mF. is moving with a constant velocity, the
C = 10 mF = 10 ´ 10-6 F 260. Initial power = 3000 W electric and magnetic fields will not
1 change with time, hence no EM wave
Hence, L = As efficiency is 90%, then final power
(1000) 2 ´ 10 ´ 10-6 will be produced. But if the charge is
90
= 3000 ´ = 2700 W moving with a non-zero acceleration,
= 0.1 H = 100 mH 100 both the electric and magnetic field will
Þ V1I1 = 3000 W change with space and time, it then
257. Phase difference between current and V2I 2 = 2700 W
voltage in LCR series circuit is given by produces EM wave. This shows that
2700 900 accelerated charge emits EM waves.
1 V2 = = [QI1 = 6 A]
wL – 6 2
wC 264. Given, energy of EM waves is the order
tan f = Þ V2 = 450 V
R of 15 keV
3000 c
f being the angle by which the current and I1 = [QV1 = 200 V] i.e. E = hn = h ´
200 l
leads the voltage.
Þ I1 = 15 A
MODULE 3
www.jeebooks.in
The NEET Edge ~ Physics 407

h´c 6.6 ´ 10-34 ´ 3 ´ 108 268. Consider the situation shown in 2 -14.
Þl = = = - 07
. ´
=
E . ´ 10-19
15 ´ 103 ´ 16 figure.Let radius of curvature of lens R R
surfaces is R. The combination is 1 1 1
. ´ 10-29
132 So, =2´ +
= = 0.825 ´ 10-10 m equivalent to three lenses in contact. feq flens fconcave
. ´ 10-19
16 1 1 1 1 2 1 -14
+ æç
\ = + + = + (Qf1 = f3 ) 05
. . ö 1 14 . 0.4
=2´ ÷= - =-
= 0.825 Å [Q 1Å = 10-10 m] feq f1 f2 f3 f1 f2 R è R ø R R R
l = 0.825 Å R 20
\ feq = - =- = - 50 cm
Thus, this spectrum is a part of X-rays. 0.4 0.4
µ=3/2
265. Wavelength order of given rays are 270. According to question, the ray diagram
listed below will be

Waves Wavelength (in Å)


f1 f3
Visible light 4000–7000
f2
X-rays 10 -3 - 102 f2 2f1
1 1
= = (m - 1) æ2ö = 1
Now, ç ÷
Microwaves 107 - 109 f1 f3 èR ø f So, distance d between convex lens
1 and concave mirror is given by
So, l X < l v < l m Þ R =
2 (m - 1) f d = 2f1 + f2
-2
= (mw - 1) æç - ö÷ = æç - 1ö÷ æç ö÷
1 2 4 271. As we know that,
23. Ray Optics and Optical f2 è R ø è3 øèR ø Intensity, I µ A (Area exposed)
Instruments -2 æ ö pd 2 pd 2 / 4
= æç ö÷ ç
1
÷ -
è 3 ø è 2 (m - 1)f ø I 2 é A2 ù 3
266. According to the question, Þ = = 4 4 =
I1 êë A1 úû pd 2 4
L = Light source æ ö
ç 1 ÷ æ 1ö
1 æ 1ö 2 1 4
When the light beams incident normally Þ = ç- ÷
ç ÷ ç ÷= - ´
f2 è 3 ø
3 è f ø 3 f 3
on the plane mirror, it is reflected back ç - 1÷ Þ I 2 = I1
è2 ø 4
to the point from which it was coming.
1 2 2 1 6- 2 4 and focal length remains unchanged.
When the plane mirror is rotated by an \ = - ´ = =
angle q, the reflected ray or beam of feq f 3 f 3f 3f 272. The diameter of the sun,
light must rotate by angle 2q. 3f O = 1.39 ´ 109 m and its image is I. As
Þ feq =
B 4 the distance of sun from lens
M u = 1.5 ´ 1011 m and from paper is
269. Given, m g = 15
. and moil = 17
.
M' v = 0.1 m.
y
I v
θ Then by using relation, =
θ O u
θ I 0.1
L x O Þ =
1.39 ´ 109 1.5 ´ 1011
I = 9.2 ´ 10-4 m
Glass Glass 273. According to the question, the figure of
mentioned prism is given as
moil = 1.7 A
BL y
From the figure, tan 2q = =
LO x Þ R = 20 cm
30°
If the angle is small, From Lens Maker’s formula for the plano
R
tan 2 q » 2 q convex lens, O
i r2=0
y y 1 é1 1ù r1
So, 2q = Þ q = = (m - 1)ê - ú µ=√2
x 2x f ë R1 R 2 û O′
P
267. In any medium other than air or Here, R1 = R B C
vacuum, the velocities of different and for plane surface, R 2 = ¥
colours are different. Therefore, both (since, there is no refraction at the face
. - 1)æç - 0ö÷ Þ
1 1 1 05
.
\ = (15 = AC)
red and green colours are refracted at f lens èR ø f lens R
different angles of refraction. Hence, Given, Refractive index of the material of
after emerging from glass slab through When the intervening medium is filled prism, m = 2
opposite parallel face, they appear at with oil, then focal length of the concave
Angle of prism, A = 30°
two different points and move in the two lens formed by the oil
If the ray OR has to retrace its path after
. - 1)æç - - ö÷
different parallel directions. 1 1 1
= (17 reflection (as per the given condition),
fconcave è R Rø

MODULE 3
www.jeebooks.in
408 NEET Test Drive

then the ray has to fall normally on the angle of emergence, e = 45° x 3
or x¢ = = = 2 cm
surface AC. Therefore, minimum deviation suffered m 15.
This means, ÐARO = ÐORC = 90° by the ray is As image appears to be raised by 1 cm,
In DAOR, d min = i + e - (r + r ¢) = 90° - 60° = 30° therefore microscope must be moved
ÐAOR + ÐARO + ÐOAR = 180° Also we know that, upward by 1 cm.
Þ ÐAOR + 90° + 30° = 180° A + dm ö
sin æç ÷
280.
Þ ÐAOR = 180° - 120° = 60° …(i) è 2 ø
m= L
As we know, ÐAOR + Ðr1 = 90° A
sin
Þ Ðr1 = 90° - 60° = 30° [from Eq. (i)] 2 I
Applying Snell’s law at the face AB, we where, m = refractive index of the fo+fe
sin i material of the prism.
get, m=
sin r1 A = angle of prism = 60° We know, magnification of telescope,
60° + 30° ö
Substituting the given values, we get sin æç ÷
we have
è 2 ø sin 45° f
sin i \ m= = M = o, …(i)
2 = 60° fe
sin 30° sin sin 30°
2 fe -I
Þ sin i = sin 30° ´ 2 Here =
1/ 2 2 fe + u L
1 1 æ 1ö = = = 2
= ´ 2= çQ sin 30° = ÷ 1/ 2 fe -I f I
2 2 è 2ø
2 Þ = Þe =
A + Dmin ö fe - (fo + fe ) L fo L
sinæç
or i = sin-1 æç
1 ö æ 1 ö ÷
÷ = 45° çQ sin 45° = ÷ è 2 ø i.e. M =
L
[From eq. (i)]
è 2ø è 2ø 276. As we know that, m = I
A
The angle of incidence of the ray on the sin
2 281. Resolving limit of telescope is given by
prism is 45°.
æ A + Dmin ö x l
sinç ÷ qµ =
274. For dispersion without deviation, net A è 2 ø D d
deviation produced by the combination Þ cot =
2 A where, l = wavelength,
of prisms must be zero. sin
2 d = diameter of objective lens of
Let, prism angle of the first and second æ A + Dmin ö telescope and
A sinç
prisms are A1 and A2 respectively. cos ÷
2 = è 2 ø D = distance between object
Similarly, their refractive indices are Þ
A A and telescope
m1 and m 2 . sin sin
2 2 lD
Condition for dispersion without Þ x =
p A æ A + D min ö
sinæç - ö÷ = sinç
d
deviation is d1 - d 2 = 0 ÷
è2 2ø è 2 ø Given, l = 5000 Å = 5000 ´ 10-10 m,
⇒ (m1 - 1) A1 - (m 2 - 1) A2 = 0
p A A D min D = 1 km = 1000 m and
æ m - 1ö Þ - = +
⇒ A2 = ç 1 ÷; 2 2 2 2 d = 10 cm = 01. m
è m 2 - 1ø Þ D min = p - 2 A = 180° - 2A 5000 ´ 10-10 ´ 1000
hence, x =
1.42 - 1ö
A1 = æç
0.1
÷ (10° ) ⇒ A2 = 6° 277. As refracted ray emerges normally from
è 1.7 - 1 ø = 5 ´ 10-3 m = 5 mm
opposite surface, r 2 = 0
275. Consider a ray of light PQ incident an As, A = r1 + r 2 Þ r1 = A 282. Angular resolution of telescope is given by
the surface AB and moves along RS, . ´ 5000 ´ 10-10
. l 122
122
Refractive index, = =
after passing through the prism ABC.
sini sini d 10 ´ 10-2
A m= and m =
sinr1 sin A . ´ 10-6 » 10-6 rad
= 61
60° In case of small angle, sin i » i
and sin A » A 24. Wave Optics
45° 30° 45° I2
i
P 30° 30° S m = Þ i = mA 283. It is given that, = n Þ I 2 = nI1
Q R A I1
278. As, resolving power of a microscope, \ Ratio of intensities is given by,
B C RP ∝
1 I max - I min ( I 2 + I1 ) 2 - ( I 2 - I1 ) 2
=
l I max + I min ( I1 + I 2 ) 2 + ( I 2 - I1 ) 2
It is given that the incident ray suffers RP1 l 6000 3
minimum deviation. Therefore, the ray \ = 2 = = æ I2 ö
2
æ I ö
2
RP2 l1 4000 2 ç + 1÷ - ç 2 - 1÷
inside the prism must be parallel to the
è 1
I ø è 1
I ø
base BC of the prism. 279. Apparent depth of mark as seen = 2 2
From the geometry of the prism and the through a glass slab of thickness x æ I2 ö æ I ö
ç + 1÷ + ç 2 - 1÷
ray diagram, it is clear that and refractive index m is è 1
I ø è 1
I ø
angle of incidence, i = 45° Real depth ( n + 1) 2 - ( n - 1) 2
Apparent depth = = =
2 n
r = r ¢ = 30° Refractive index n+1
angle of refraction, ( n + 1) 2 + ( n - 1) 2

MODULE 3
www.jeebooks.in
The NEET Edge ~ Physics 409

284. In a YDSE, angular width of a fringe is 2p l Hence, width of central maxima = 2Y


f= ´ = p / 2 = 90 °
l l 4 2lD
given as, q= = .
d As we know, a
where, l is the wavelength of the light f 90°
I = I 0 cos 2 = I 0 cos 2 290. The figure shown below represents the
source and d is the distance between 2 2 course of path an unpolarised light
the two slits. 2
æ 1 ö I follows when it is incident from air on
1 Þ I = I0 ´ ç ÷ ÞI = 0
Þ qµ è 2ø 2 plane surface of material of refractive
d index m.
q1 d 2 287. Given, YDSE experiment having two
or = …(i) Incident (Normal)
q2 d1 slits of width are in the ratio of 1 : 25. Plane polarised
unpolarised reflected light
Here, q1 = 0.20° , q 2 = 0.21°, d1 = 2 mm So, ratio of intensity, light
I1 W1 1 I 25
Substituting the given values in Eq. (i), = = Þ 2 =
I 2 W2 25 I1 1
we get
2
0.20° d2 é I2 ù
= + 1ú Medium
0.21° 2 mm ê
I max ( I 2 + I1 ) 2 I1 (µ)
0.20 0.40 \ = 2
=ê ú
Þ d2 = 2 ´ = I min ( I 2 - I1 ) ê I2 ú
0.21 0.21 ê - 1ú
I1 Partially polarised
ë û
\ = 1.9 mm 2
refracted light
2
é 5 + 1ù æ 6ö 36 9
285. According to question, Þ ê 5 - 1ú = çè 4 ÷ø = 16 = 4 When the beam of unpolarised light is
5th dark fringe in air = 8th bright fringe ë û reflected from a medium (refractive
in the medium I max 9 index = m) and if reflected and refracted
Thus, =
lD lD I min 4 light are perpendicular to each other.
(2 ´ 5 - 1) =8
2d md Then, the reflected light is completely
288. As, the first minimum is observed at an
lD lD 9 8 plane polarised at a certain angle of
Þ 9 =8 ⇒ = angle of 30° in a diffraction pattern due to
incidence. This means, the reflected
2d md 2 m a single slit of width a.
light has electric vector perpendicular to
8´2 i.e. n = 1, q = 30°
⇒ m= incidence plane.
9 Q According to Bragg’s law of 291. According to the question,
\ Refractive index of the medium, diffraction,
16 a sin q = nl Þ a sin 30° = (1) l Angle between P1 and P2 is 90°
m= = 1.7777 » 1.78
a = 2lìíQ sin 30° = üý …(i)
9 1
Þ
286. Screen î 2þ
P1 P3 P2
For first secondary maxima,
3l I1 I2 I3
S1 Þ a sin q1 =
5λ O 2 I0
3l
S2 Þ sin q1 = …(ii)
50λ 2a
Substitute value of a from Eq. (i) to 45°
Eq. (ii), we get
In the above figure, S 1 and S 2 are the 3l 3
two different slits. sin q1 = Þ sin q1 =
4l 4 From the above diagram, intensity
Given, distance between slits S 1 and S 2 ,
q1 = sin-1 æç ö÷
3 transmitted through P3
d =5l Þ
è4ø I
I 2 = 0 cos 2 45°
and distance between screen and slits, 2
289. For the condition of maxima,
D = 10d = 50l 2
I0 æ 1 ö I0
Here, l is the wavelength of light used ⇒ I2 = ´ç ÷ ⇒ I2 =
2 è 2ø 4
in the experiment. Y
According to question, the intensity at Similarly, intensity transmitted through P2 ,
a θ 2
maximum in this Young’s double slit I0 I æ 1 ö
I3 = cos 2 45° ⇒ I 3 = 0 ´ ç ÷
experiment is I 0 . 4 4 è 2ø
Þ I max = I 0 I 1 I
D ⇒ I3 = 0 ´ ⇒ I3 = 0
Q Path difference 4 2 8
d l
d ´ sin q =
=
dYn
= 2 = d = l {Q d = 5l } a 25. Dual Nature of Radiation
D 10d 20 4 Y
A path difference of l corresponds to
From the geometry, sin q » q =
D
and Matter
phase difference 2p. (for small angle) 292. Applying Einstein’s photoelectric
So, for path difference l / 4, phase Y l lD equation, kinetic energy of emitted
So, = ÞY =
difference D a a electron can be given by

MODULE 3
www.jeebooks.in
410 NEET Test Drive

1 hc hc The threshold wavelength is given by v12 1 éQKE = 1 mv 2 ù


K = mv 2 = hn - hn 0 = - or =
2 l l0
l th =
hc 4eV0 l
= = 4l v 22 4 ëê 2 ûú
Given, threshold wavelength, W eV0 v1 1 1
or = =
l 0 = 3250 ´ 10-10 m v2 4 2
295. For photoelectric equation,
Wavelength of ultraviolet light, 298. Use Einstein’s photoelectric equation,
Einstein’s equation can be written as
l = 2536 ´ 10-10 m
(KE) max = hn - f 0 we know that,
Let velocity of ejected electron be v. E = (KE) max + work function (f)
For the first condition,
Now, applying Einstein’s photoelectric 0.5 = E - f 0 …(i) where, f = hn 0 , E = hn
equation, we have, E = K + f 0 1
1 For the second condition, (KE) max = mv 02
⇒ hn = mev 2 + hn 0 0.8 = 1.2 E - f 0 ...(ii) 2
2
From Eqs. (i) and (ii), Þ (KE) max = hn - f
1 æ1 1ö
⇒ mev 2 = hn - hn 0 = hc ç - ÷ -0.3 = -0.2E Þ 2eV = 5 eV - f (given)
2 è l l0 ø
⇒ Velocity of electron, 0.3 Þ f = 3 eV
E = = 1.5 eV
0.2 Thus, Vcathode - Vanode = 3 V
2hc æ 1 1ö Þ Vanode - Vcathode = - 3 V
v = ç - ÷ From Eq. (i), 0.5 = 1.5 - f 0
me è l l 0 ø f 0 = 1.5 - 0.5 = 1 eV 299. Efficient power (P ) is given by
-15 -19 18
2 ´ 4.14 ´ 10 ´ 1.6 ´ 10 ´ 3 ´ 10 296. According to the Einstein’s P =
N hc æN = total number of ö
´ ç ÷
= 9.1 ´ 10-31 photoelectric equation, t l è photons ø
æ 3250 - 2536 ö
ç ÷ 1
K max = mv max2
= h n - f0 N P ´l 50 ´ 0.6 ´ 10-6
è 3250 ´ 2536 ø = =
2 t hc 6.6 ´ 10-34 ´ 3 ´ 108
= 0.6 ´ 106 ms -1 » 6 ´ 105 ms -1 = h n - hn 0 …(i)
[Qefficiency, 25% of 200W = 50W]
where, K max is the maximum kinetic
293. In first case, when a metallic surface is . ´ 1020
= 15
illuminated with radiation of wavelength energy of photoelectrons having
l, the stopping potential isV . maximum velocity v max. 300. Here, l = 667 ´ 10-9 m,
So, photoelectric equation can be When incident frequency of light, P = 9 ´ 10-3 W
n = 2 n0 Energy (E ) nhc Nhc
written as Power = = =
hc hc Substituting the value of n in Eq. (i), we Time (t ) lt l
eV = - …(i) é ù
l l0 get
1 ê where, n = total number of photons andú
In second case, when the same surface m v12 = h (2 n0 ) - h n0 ê N = number of photons emitted ú
is illuminated with radiation of 2 ê n ú
per second =
wavelength 2l, the stopping potential is = 2h n0 - h n0 = h n0 …(ii) êë t úû
V
. So, photoelectric equation can be If incident frequency of radiation, P ´ l 9 ´ 10-3 ´ 667 ´ 10-9
So, N = =
4 n = 5 n0 hc 6.6 ´ 10-34 ´ 3 ´ 108
eV hc hc
written as, = - Substituting the value of n in Eq. (i), we = 3 ´ 1016 m /s
4 2l l 0 get
4hc 4hc 1 301. de-Broglie wavelength associated with a
Þ eV = - …(ii) mv 22 = h (5 n0 ) - h n0 moving particle can be given as
2l l0 2
h h
= 5h n0 - h n0 = 4h n0 …(iii) l= =
From Eqs. (i) and (ii), we get p 2m ((KE)
hc hc 4hc 4hc On dividing Eq. (ii) by Eq (iii), we get
Þ - = - 1 At thermal equilibrium, temperature of
l l0 2l l0 mv12
2 h n0 v2 1 neutron and heavy water will be same.
1 1 2 4 = Þ 12 =
Þ - = - Þ l0 = 3l 1 2 4 h n v2 4 This common temperature is given as, T.
l l0 l l0 mv 2 0
2 Also, we know that, kinetic energy of a
294. From photoelectric equation, v1 1 particle
or = p2
hn = W + eV0 v2 2 KE =
2m
(where,W = work function) \ v1 : v 2 = 1 : 2
hc where, p = momentum of the particle
So, = W + 3eV0 …(i) 297. Kinetic energy in photoelectric effect and m = mass of the particle.
l
can also be written as, Kinetic energy of the neutron is
hc
Also, = W + eV0 KE = f - f 0 3
2l KE = kT
hc é where, f = incident energy andù 2
Þ = 2W + 2eV0 …(ii) êë f 0 = work function úû
l \ de-Broglie wavelength of the neutron,
h h
Subtracting Eq. (i) from Eq. (ii), we get Given, KE 1 = 1 - 05
. = 05
. eV l= =
p 2m (KE)
0 = W - eV0 Þ W = eV0 KE 2 = 2.5 - 05. = 2 eV
From Eq. (i), KE 1 05 . 1 h h
So, = = = =
hc KE 2 2 4 3 3mkT
= eV0 + 3eV0 = 4eV0 2m ´ kT
l 2
MODULE 3
www.jeebooks.in
The NEET Edge ~ Physics 411

302. Since, it is given that electron has Initial momentum of the radiation is According to Einstein’s photoelectric
mass m. h E emission, we have
pi = =
de-Broglie’s wavelength for an electron l c KE max = E - W = 2.48 - 2.28 = 0.2 eV
will be given as The reflected momentum is For de-Broglie wavelength of the emitted
h h E electron,
le = …(i) pr = - = -
p l c
12.27 A° 12.27
where, h = Planck’s constant and So, the change in momentum of light is le min = =
2E KE max (eV ) 0.2
p = linear momentum of electron. Dplight = pr - pi = -
c = 27.436 Å = 27.436 ´ 10-10 m
As, kinetic energy of electron
Thus, the momentum transferred to the
p2 Thus, minimum wavelength of the emitted
E = Þ p = 2mE …(ii) surface is
2m 2E electron is
Dplight =
From Eqs. (i) and (ii), we get c l min = 2.7436 ´ 10-9 m » 2.8 ´ 10-9 m
le =
h
…(iii) h i.e. l ³ l min
306. For de-Broglie wavelength, l =
2mE p
Energy of a photon can be given as h h 26. Atoms
For first case, l1 = =
hc p 2mK 309. A particle of mass M1 and charge Z 1
E = hn Þ E =
lp For second case, l 2 =
h possess initial velocity u, when it is at a
hc 2m 16K large distance from the nucleus of an
Þ lp = …(iv)
E h l atom having atomic number Z 2 . At the
= = 1
4 2mK 4 distance of closest approach, the kinetic
Hence, l p = de-Broglie’s wavelength of
l 2 = 25% of l1 energy of particle is completely
photon.
converted to potential energy.
Now, dividing Eq. (iii) by Eq. (iv), we get So, 75% change in the wavelength
Mathematically,
le h E l 1 E takes place.
= × Þ e = . 1 1 Z 1Z 2
307. Energy of photon is given by M1u 2 =
lp 2mE hc lp c 2m 2 4 pe 0 r 0
hc
E = ...(i) So, the energy of the particle is directly
303. Cut-off wavelength occurs when l
proportional to Z 1Z 2 .
incoming electron looses its complete where, h is Planck’s constant, c is the
energy in collision. This energy appears velocity of light and l its wavelength. 310. When an a-particle of mass m moving
in the form of X-rays. with velocity v bombards on a heavy
de-Broglie wavelength is given by nucleus of charge Ze, then there will be
Given, mass of electrons = m and h
l= ...(ii) no loss of energy as in this case, where,
de-Broglie wavelength = l. p initial kinetic energy of a-particle =
p2 p being momentum of photon. potential energy of a-particle at closest
So, kinetic energy of electron =
2m From Eqs. (i) and (ii), we have approach.
2
æh ö hc E 1 2 Ze 2 1
ç ÷ E = = pc or p = Þ mv 2 = Þ r0 µ
èlø h2
= = h/p c 2 4 pe 0r 0 m
2m 2ml 2
Given, E = 1 MeV This is the required distance of closest
Now, maximum energy of photon can be
= 1 ´ 106 ´ 16
. ´ 10–19 J approach to a-particle from the nucleus.
hc h2
given by, E = = 19
1 eV = 1.6 ´ 10 J
l0 2ml 2 311. Kinetic energy of an electron in a Bohr
c = 3 ´ 108 m /s orbit of a hydrogen atom is given as
2
hc ´ 2 l × m 2mcl 2 Hence, after putting numerical values, Rhc
Þ l0 = = KE n = 2 …(i)
h2 h we obtain n
1 ´ 106 ´ 16
. ´ 10–19
304. Wavelength of electron is given by p = kg-m/s Total energy of an electron in a Bohr
h h 3 ´ 108 orbit of a hydrogen atom is given as
le = = [Q pe = 2mE ]
pe 2mE » 5 ´ 10–22 kg-m/s -Rhc
TE n = …(ii)
h2 308. As, energy of photon, n2
Þ l2e = hc Dividing Eq. (i) by Eq. (ii), we get
2mE E = hn Þ E =
and for photon, l æ Rhc ö
ç 2 ÷
hc hc 6.626 ´ 10-34 ´ 3 ´ 108 KE n
=
èn ø
Þ KE n : TE n = 1 : - 1
lp = or E = Þ E =
500 ´ 10-9 - æç 2 ö÷
E lp TE n Rhc
0.0397 ´ 10-34 ´ 108 èn ø
h2 h2 Þ E =
\ l2e = = l p Þ l2e µ l p
hc 2mhc 10-9 312. Energy of electron in the 3rd orbit of He+
2m ×
lp = 0.0397 ´ 10-21 J is
0.0397 ´ 10-21 Z2
305. The radiation energy is given by = E 3 = - 13.6 ´ eV
. ´ 10-19
16 n2
hc
E = = - 13.6 ´
4
eV
l = 0.0248 ´ 102 eV = 2.48 eV 32
MODULE 3
www.jeebooks.in
412 NEET Test Drive

4 Rhc hc 24 1
= - 13.6 ´ . ´ 10-19 J
´ 16 316. Excess energy of electron appears as and Rhc - = Þ R =
9 photon. From Rydberg’s formula, 25 l 25 l
From Bohr’s model, æ 1 1ö h h 24 Rh
= R ç 2 - 2 ÷ = R æç 2 - 2 ö÷ =
1 1 1 5R As, p = and v = =
1 l è f è ø l ml 25 m
E 3 = - KE 3 = - mev 2 n ni ø 2 3 36
2
= R æç 2 - 2 ö÷ =
1 1 1 1 7R 320. Number of spectral lines,
Þ . ´ 10-31
´ 91 ´ v2
2 l¢ è3 4 ø 144 n (n - 1) n (n - 1)
4 N = Þ =6
= - 13.6 ´ ´ 16 . ´ 10-19 1 1 5R
/ = ¸
7R 2 2
9 l l¢ 36 144 or 2
n - n - 12 = 0
136 ´ 16 ´ 4 ´ 2 ´ 10-11
Þv2 = l¢ 5R 144 20 20 or (n - 4)(n + 3) = 0
9 ´ 91 Þ = ´ = Þ l¢ = l
l 36 7R 7 7 or n=4
. ´ 106 m/s
or v = 146
317. In hydrogen atom, wavelength of Now, as the first line of the series has
313. Ionisation energy corresponding to the maximum wavelength, therefore
ionisation potential, E 1 = – 13 .6 eV characteristic spectrum
é 1 electron jumps from the fourth orbit to
Photon energy incident, D E = 12.1 eV 1 1ù
= RZ 2 ê 2 - 2 ú the third orbit.
So, the energy of electron in excited l ë n1 n2 û
state E 2 is given by For Lyman series n1 = 1, n2 = 2 27. Nuclei
E 2 - E 1 = DE 1 é 1 1 ù
= RZ 2 ê 2 - ú …(i) 321. Radius of the nucleus is given by
Þ E 2 = DE + E 1 l1 ë (1) (2)2 û
R = R 0 A1/ 3 Þ R µ A1/ 3
Þ E 2 = – 13 .6 + 12 .1 For Balmer series n1 = 2, n2 = 3 1/ 3 1/ 3
æA ö
= æç
Þ E 2 = – 1.5 eV R Al 27 ö 3
1 é 1 1 ù = ç Al ÷ ÷ =
13 .6 –13 .6 = RZ 2 ê 2 - ú …(ii) R Te è ATe ø è 125 ø 5
i.e. E 2 = – eV Þ –15 . = l2 ë (2) (3) 2 û
n2 n2 R Te =
5
R Al
–13 .6 Dividing Eq. (ii) by Eq. (i), we get 3
2
\ n = »9 Þ n=3
RZ 2 é - ù
1 1 5
–15
.
êë 4 9 úû 322. According to question, radius of 94 Be
l1 36
i.e. energy of electron in excited state = = nucleus be r and radius of germanium
l2 3
RZ 2 é1 - ù
1
corresponds to third orbit. êë ú (Ge) nucleus will be 2 r.
4û 4
The possible spectral lines is given by Radius of a nucleus is given by
l1 5 4 5
n (n - 1) 3 (3 - 1) = ´ = R = R 0 A1/ 3
Þ Þ 3 l2 36 3 27
2 2 where, R = radius of atom having mass
Ze 2 318. Energy provided to the ground state number A.
314. Kinetic energy of electron, KE =
8pe0r electron R µ A1/ 3 1/ 3
hc 6.6 ´ 10-34 ´ 3 ´ 108 R1 æ A1 ö
Potential energy of electron, = = So, =ç ÷
1 Ze 2 l 975 ´ 10-10 R 2 è A2 ø
U =- 6.6 ´ 3 1 /3
4 pe 0 r = ´ 10-16 r æ 9ö
975 Þ =ç ÷ (QA1 = 9)
\ Total energy, 2 r è A2 ø
Ze 2 Ze 2 = 0.0203 ´ 10-16 = 2.03 ´ 10-18 J
E = KE + U = - 3
8 pe0r 4 pe0r 20.3 ´ 10-19 20.3 æ 1ö = 9
= eV = eV or ç ÷
1.6 ´ 10-19 1.6 è2ø A2
Ze 2
or E =- or E = - KE A2 = 9 ´ (2) 3 = 9 ´ 8 = 72
8pe0r » 12.75 eV Hence,

or KE = - E = - (- 3 .4) = 3.4 eV It means the electron jumps to n = 4 Thus, in germanium (Ge) nucleus
from n = 1. number of nucleons is 72.
315. Wavelength of spectral lines are given by hn
When electron will fall back, number of 323. Momentum of a photon, p =
1 æ 1 1ö
= Z 2R ç 2 - 2÷ spectral lines emitted c
l è 1
n n 2 ø n (n - 1) 4 (4 - 1) p2
= = =6 Hence, recoil energy, E =
For last line of Balmer series, 2 2 2
2M
n1 = 2 and n2 = ¥ æ hn ö
319. According to third postulate of Bohr’s ç ÷
èc ø h 2 n2
= Z 2R æç 2 - 2 ö÷ = \ E = or E =
1 1 1 R model, when an atom makes a
⇒ [QZ = 1]
lB è2 ¥ ø 4 transition from higher energy state to 2M 2Mc 2
Similarly, for last line of Lyman series, lower energy state, the difference of 324. The binding energy for 1H1 is around zero
energy is carried away by a photon, and also not given in the question so we
n1 = 1 and n 2 = ¥
such that hn = E n i - E nf can ignore it
= Z R æç 2 - 2 ö÷ = R
1 1 1
⇒ 2
hc é 1 1ù
lL è1 ¥ ø or = Rhc ê 2 - 2 ú Q = 2 (E bn of He) - (E bn of Li)
1 l ë nf ni û = 2 (4 ´ 7.06) - (7 ´ 5.60)
R
lB 1 l 1 l Energy difference between fifth and first = (8 ´ 7.06) - (7 ´ 5.60)
\ = 4 = ⇒ Λ = ⇒ Β =4 hc
1 R 4 lΒ 4 lΛ orbit is E 5 - E 1 = = (56.48 - 39.2) MeV
lL l \ Q = 17.28 MeV ~ - 17.3 MeV

MODULE 3
www.jeebooks.in
The NEET Edge ~ Physics 413

T1/ 2 T T
325. After n half-life, the number of nuclei left 1 1/ 2 1 1/ 2 1 pairs increase. This leads to increase in
Þ 1 ¾® ¾® ¾®
undecayed is given as 2 4 8 the current in diode and overall
n
1 resistance of the diode changes.
N = N 0 æç ö÷ where, n =
1 t So, time taken to become unstable
è2ø t1/ 2 8 This in turn changes both the forward
part biasing and the reverse biasing.
Here, initially number of nuclei, N 0 = 600 = 3 ´ T1/ 2 = 3 ´ 1.4 ´ 109 = 4.2 ´ 109 yr
Thus, the overall V-I characteristics of
After disintegration, number of nuclei, Alternative
p-n junction diode gets affected.
N ¢ = 450 X Y
® 334. We know that, a diode only conducts in
\ Number of nuclei left undecayed, Active Stable
forward biased condition. In the given
N = N 0 - N ¢ = 600 - 450 = 150 As, we know that circuit, the diode D1 will be in reverse
Half-life, t1/ 2 = 10 min n 3
bias, so it will block the current and
= æç ö÷ = = æç ö÷ Þ n = 3
N 1 1 1 1
t /t 1/ 2
diode D2 will be in forward bias, so it will
= æç ö÷ N0 è2 ø 1 + 7 8 è2 ø
N 1
As, pass the current
N0 è2 ø t
As, T1/ 2 = V 10
Substituting the given values, we get n i = = = 2.5 A
t / 20 t /10
R1 + R 3 2+ 2
150 æ 1 ö 1 æ 1ö \ t = T1/ 2 ´ n
=ç ÷ Þ =ç ÷ 335. As it is forward biased so it takes positive
600 è 2 ø 4 è2ø = 1.4 ´ 109 ´ 3 = 4.2 ´ 109 yr
value. Hence, option (d) is correct.
2 t /10 329. Number of nuclei remained after time t
or æç ö÷ = æç ö÷
1 1 t
Þ = 2 Þ t = 20 min can be written as 336. V-I characteristics of a solar cell is
è2ø è2ø 10
N = N 0e - lt shown. Where, A represents open circuit
326. Let initial number of nuclei in A and B voltage
where, N 0 is initial number of nuclei of
is N 0 . (i.e.I = 0,V = emf) and B shows short
both the substances.
circuit current (i.e. I = I , V = 0).
Number of nuclei of A after time t is N1 = N 0e - 5 lt …(i)
N A = N 0e -8lt ...(i) 337. Let the energy of radiation falling on the
and N 2 = N 0e - lt …(ii) p-n photodiode be E = hn
Similarly, number of nuclei of B after time Dividing Eq. (i) by Eq. (ii), we obtain
t is The minimum energy required = 2 eV
N1 1
= e ( - 5 l + l ) t = e - 4 lt = 4 lt \ 2 eV = hn
NB = N 0e - lt ...(ii) N2 e
NA 1 2 eV 2 ´ 1.6 ´ 10-19
It is given that = [QNB > N A ] But, we have given \ n= =
NB e 2
h 6.6 ´ 10-34
N1 æ 1 ö 1
Now, from Eqs. (i) and (ii) =ç ÷ = 2 » 5 ´ 1014 Hz
N 2 èe ø e
e -8 lt 1 338. On applying forward bias to a p -n
= 1 1
e - lt e Hence, = junction diode, it increases number of
e 2 e 4 lt
Rearranging, e -1 = e -7lt Þ 7 lt = 1 donor on the n-side and decreases
Comparing the powers, we get potential barrier. It also decreases
1
Þ Time, t = 2 1 electric field of depletion layer.
7l 2 = 4 lt or t = =
4l 2 l
327. As, N = N 0e - lt and N = N 0 (½) n , where n 339. Given,VBE = 0 V,VCE = 0 V
is half-life. 28. Semiconductor Electronics andVi = 20 V
Given, N1 = 0.6 N 0 (Q40% decay) 330. Carbon, silicon and germanium are VCC=20 V
N 2 = 0.15 N 0 (Q85% decay) semiconductors.
RC=4 kΩ
Putting these in the formula, (E g) C = 5.2 eV =4×103Ω
2
N2 0.15 N 0 1 æ 1 ö (E g) Si = 121
. eV IC
= = =ç ÷ C
N1 0.6 N 0 4 è2ø (E g) Ge = 075
. eV IB
So, two half-life periods has passed. Thus,(E g) C > (E g) Si and (E g ) C > (E g ) Ge Vi VB
B
RB=500 kΩ
Thus, time taken 331. The four bonding electrons in the case = 500×103Ω E
= 2 ´ t1/ 2 = 2 ´ 30 = 60 min of C lie in the second orbit, whereas in
328. Ratio of X :Y is given = 1:7 case of Si they lies in the third orbit.
mx 1 So, loosely bounded valence electrons
= Þ 7m x = m y are present in Si as compared to C.
my 7
332. When a small amount of antimony is Applying Kirchhoff’s law to the
Let the initial total mass be m.
added to germanium crystal, the crystal
Þ mx + my = m base-emitter loop, we get
my 8 my becomes n-type semiconductor,
Þ + my = m Þ =m Vi = IB RB + VBE
because antimony is a pentavalent
7 7 Substituting the values, we get
substrate.
7
Þ my = m 333. Due to increase in temperature 20 = IB ´ (500 ´ 103 ) + 0
8 20
1 because of heating, thermal collision Þ IB = = 0.04 ´ 10-3
Only part remains between the electron and holes 500 ´ 103
8
increases. Thus, net electron-hole = 40 ´ 10-6 = 40 mA …(i)

MODULE 3
www.jeebooks.in
414 NEET Test Drive

Similarly, VCC = ICRC + VCE Putting all the values in given equation, 200
Þ 50 = b ´ Þ b = 25
Substituting the given values, we get we get 100
20 = IC ´ (4 ´ 103 ) + 0 R 2 kW Also, power gain = b 2 ´ impedance gain
A V = b out = 100 ´ Þ A V = 200
20 R in 1 kW 200
Þ IC = = 5 ´ 10-3 = 5 mA…(ii) = 25 2 ´ = 1250
4 ´ 103 (V ) 100
Now, A V = out AC = 200
I (V in ) AC 347. For a transistor, IE = IB + IC
Current gain is given as, b = C
IB 4 where, IE = emitter current,
Þ (V in ) AC = = 20 mV
Substituting the value of IB and IC from 200 IB = base current
Eqs. (i) and (ii), we get 343. Input signal of a CE amplifer, and IC = collector current
5 ´ 10-3 p DI
Þ b= = 0125
. ´ 103 = 125 Vin = 2 cos æç15t + ö÷ Current gain, b = C
D IB
40 ´ 10-6 è 3ø
Voltage gain, Av = 150 Here, DIC = 10 ´ 10-3 - 5 ´ 10-3
340. Collector current,
As CE amplifier gives phase difference = 5 ´ 10-3 A
V 3
iC = = = 10-3 A DIB = 200 ´ 10-6 - 100 ´ 10-6
R 3 ´ 103 of p between input and output signals.
So,
V
Av = 0 ÞV0 = AvVin = 100 ´ 10-6 A
Now base current,
Vin 5
i 10-3 \ b= ´ 1000 = 50
iB = C = = 10-5 A p 100
b 100 V0 = 150 ´ 2 cos æç15t + + p ö÷
è 3 ø 348. AC current gain b is defined as the ratio
As, voltage Vin = iBRB
æ 4p ö of the collector current to the base
\ Vin = 10-5 ´ 2 ´ 103 V = 300 cos ç15t + ÷ current at constant collector voltage,
è 3 ø
= 2 ´ 10-2 V æ Di ö
So, voltage gain, RL æb ö i.e, b = ç c÷
344. As, Av = b or G = ç ÷ RL è Dib øV
V 3 Ri è Ri ø c
AV = out = = 150 Given, Dic = 10 mA – 5 mA = 5 mA
Vin 2 ´ 10-2 Þ G = g mRL Þ G µ g m
Dib = 150 mA – 100 mA = 50 mA
Power gain, AP = AV ´ b é D Ic D IC bù
êQg m = DV = DI R = R ú \ b=
5 mA
= 100
= 150 ´ 100 = 15000 ë B B i i û
50 ´ 10–3 mA
341. Given, resistance across load, G2 gm2 0.02
\ = Þ G2 = ´G NOTE In common emitter amplifier, the
RL = 800 W G1 g m1 0.03
2 output voltage signal is 180° out of
Voltage drop across load,VL = 0.8 V So, voltage gain, G 2 = G
3 phase with the input voltage signal.
Input resistance of circuit, Ri = 192 W
Collector current is given by, DI 349. In a series LC-circuit, frequency of LC-
345. Current amplification factor, b = C
V 0.8 8 D IB oscillations is given by
IC = L = = = 1 mA
RL 800 8000 Collector current, 1 1
f = or f µ
Output current 2V 2p LC LC
QCurrent amplification = D IC = = 1 ´ 10-3 A
Input current 2 ´ 103 W Considering two cases of L and C,
IC 1 mA Base current,
= = 0.96 Þ IB = f1 L2C2
V VB =
IB 0.96 D IB = B = = VB ´ 10-3 f2 L1C1
RB 1 ´ 103
Q Voltage gain,
0.8 ´ 0.96 Given, L1 = L, C1 = C, L2 = 2 L,
V V Given, b = 100
AV = L = L = -3 =4
Vin IBRi 10 ´ 192 10-3 C2 = 4 C, f1 = f
Now, 100 =
Þ AV = 4 VB ´ 10-3 f 2 L ´ 4C f
\ = = 8 Þ f2 =
and power gain, 1 f2 LC 2 2
2
VB = V = 10 mV
IC2RL æI ö R 800 100
AP = = ç C ÷ . L = (0.96) 2 ´ 350. X = AB = A × B (i.e. AND gate)
IB2Ri è IB ø Ri 192 346. AC power gain
Change in output power If the output of NAND gate is connected
AP = 3.84 = to the two inputs of another NAND gate
Change in input power as in given figure, then we get back to
342. Voltage amplification is
DVc ´ Dic æ DVc ö æ Dic ö an AND gate.
R out (Vout ) AC = =ç ÷´ç ÷
AV = b = DVi ´ Dib è DVi ø è Dib ø 351. Gate 1 is OR gate,Y ¢ = A + B
R in (V in ) AC
= AV ´ b AC A
1
Given, collector resistance B 2 Y
where, AV is voltage gain and b AC is AC C
= Rout = 2 kW
current gain. Gate 2 is AND gate,Y = Y ¢ × C
Current amplification factor, b = 100
æR ö \ A = 1, B = 0, C = 1 will giveY = 1
Base resistance, Rin = 1 kW Also,AV = b AC ´ resistance gain ç o ÷
è Ri ø 352. The Boolean expression which satisfies
Output signal voltage = 4 V
Voltage gain = b ´ impedance gain the output of this logic gate is C = A × B ,
which is for AND gate.
MODULE 3
www.jeebooks.in

THE NEET EDGE


Chapterwise Collection of Most Difficult Chemistry Questions asked in last 15 Years’ NEET/AIPMT

CHEMISTRY
1. Some Basic Principles of Chemistry 6. 20.0 g of a magnesium carbonate sample
decomposes on heating to give carbon dioxide and
1. An element, X has the following isotopic
8.0 g magnesium oxide. What will be the percentage
composition :
purity of magnesium carbonate in the sample?
200
X : 90%, 199
X : 8.0%, 202
X : 2.0% (Atomic weight of Mg = 24) [CBSE-AIPMT 2015]

The weighted average atomic mass of the naturally (a) 75 (b) 96


(c) 60 (d) 84
occurring element X is closest to [CBSE-AIPMT 2007]
(a) 201 u (b) 202 u (c) 199 u (d) 200 u 7. A mixture of 2.3 g formic acid and 4.5 g oxalic acid
is treated with conc. H 2SO4. The evolved gaseous
2. If Avogadro number NA , is changed from mixture is passed through KOH pellets. Weight
−1 −1
6.022 × 10 mol
23
to 6.022 × 10 20
mol this would (in g) of the remaining product at STP will be
change [CBSE-AIPMT 2015] (a) 2.8 (b) 3.0 [NEET 2018]
(a) the definition of mass in units of grams (c) 1.4 (d) 4.4
(b) the mass of one mole of carbon
(c) the ratio of chemical species to each other in a balanced 8. Number of moles of MnO–4 required to oxidise one
equation mole of ferrous oxalate completely in acidic
(d) the ratio of elements to each other in a compound medium will be [CBSE-AIPMT 2008]
(a) 0.6 mol (b) 0.4 mol
3. In which case is the number of molecules of water (c) 7.5 mol (d) 0.2 mol
maximum? [NEET 2018]
(a) 0.00224 L of water vapours at 1 atm and 273 K 9. The number of moles of KMnO4 reduced by one mole
(b) 0.18 g of water of KI in alkaline medium is [CBSE-AIPMT 2005]
(c) 18 mL of water (a) one fifth (b) five (c) one (d) two
(d) 10−3 moles of water
10. What is the mass of precipitate formed when 50 mL
4. The number of atoms in 0.1 mole of a triatomic gas of 16.9% solution of AgNO3 is mixed with 50 mL of
−1
is ( N A = 6.023 × 10 mol )23
[CBSE-AIPMT 2010] 5.8% NaCl solution? (Ag = 107.8,N = 14, O = 16,
(a) 6.026 × 1022 (b) 1.806 × 1023 Na = 23, and Cl = 35.5) [CBSE-AIPMT 2015]
(c) 3.600 × 1023 (d) 1. 800 × 1022 (a) 28 g (b) 3.5 g
(c) 7 g (d) 14 g
5. How many grams of concentrated nitric acid
solution should be used to prepare 250 mL of 11. 10 g of hydrogen and 64 g of oxygen were filled in a
2.0 M HNO3? The concentrated acid is 70% HNO3 . steel vessel and exploded. Amount of water
[NEET 2013] produced in this reaction will be [CBSE-AIPMT 2009]
(a) 45.0 g conc. HNO 3 (b) 90.0 g conc. HNO 3 (a) 2 mol (b) 3 mol
(c) 70.0 g conc. HNO 3 (d) 54.0 g conc. HNO 3 (c) 4 mol (d) 1 mol

MODULE 3
www.jeebooks.in
416 NEET Test Drive

12. How many moles of lead (II) chloride will be 20. Which of the following is not permissible arrangement
formed from a reaction between 6.5 g of PbO and of electrons in an atom? [CBSE-AIPMT 2009]
3.2 g of HCl? [CBSE-AIPMT 2008] (a) n = 4, l = 0, m = 0, s = − 1/2
(a) 0.044 (b) 0.333 (c) 0.011 (d) 0.029 (b) n = 5, l = 3, m = 0, s = + 1/2
(c) n = 3, l = 2, m = − 3, s = − 1/2
13. What volume of oxygen gas (O2 ) measured at 0°C
(d) n = 3, l = 2, m = − 2, s = − 1/2
and 1 atm, is needed to burn completely 1 L of
propane gas (C3H 8 ) measured under the same 21. What is the maximum numbers of electrons that
conditions? [CBSE-AIPMT 2008] can be associated with the following set of quantum
(a) 7 L (b) 6 L (c) 5 L (d) 10 L numbers? n = 3 , l = 1 and m = − 1 [NEET 2013]
(a) 10 (b) 6 (c) 4 (d) 2
14. When 22.4 L of H 2 ( g) is mixed with 11.2 L of Cl 2 ( g),
each at STP, the moles of HCl ( g) formed is equal 22. What is the maximum number of orbitals that can
to [CBSE-AIPMT 2005] be identified with the following quantum numbers?
(a) 1 mole of HCl (g ) (b) 2 moles of HCl (g ) n = 3, l = 1 and ml = 0 [CBSE-AIPMT 2014]
(c) 0.5 mole of HCl (g ) (d) 1.5 moles of HCl (g ) (a) 1 (b) 2 (c) 3 (d) 4

15. 1.0 g of magnesium is burnt with 0.56 g of oxygen 23. How many electrons can fit in the orbital for which
in a closed vessel. Which reactant is left in excess n = 3 and l = 1 ? [NEET 2016, Phase II]
and how much? (Atomic weight of Mg = 24, O = 16) (a) 2 (b) 6 (c) 10 (d) 14
[CBSE-AIPMT 2014]
24. If n = 6, the correct sequence for filling of electrons
(a) Mg, 0.16 g (b) O 2 , 0.16 g (c) Mg, 0.44 g (d) O 2 , 0.28 g
will be [CBSE-AIPMT 2011]
(a) ns → (n − 1)d → (n − 2 )f → np
2. Structure of Atoms (b) ns → (n − 2 ) f → np → (n − 1)d
(c) ns → np → (n − 1)d → (n − 2 ) f
16. The frequency of radiation emitted when the (d) ns → (n − 2 ) f → (n − 1)d → np
electron falls from n = 4 to n = 1 in a hydrogen atom
will be (Given ionisation energy of 25. Which is the correct order of increasing energy of
H = 2.18 ×10–18 J atm –1 and h = 6.625 × 10–34 Js) the listed orbitals in the atom of titanium?
[CBSE-AIPMT 2004] [CBSE-AIPMT 2015]
(a) 1.54 × 1015 s –1 (b) 1.03 × 1015 s –1 (a) 3s, 4s, 3p, 3d (b) 4s, 3s, 3p, 3d
(c) 3.08 × 1015 s –1 (d) 2.00 × 1015 s –1 (c) 3s, 3p, 3d, 4s (d) 3s, 3p, 4s, 3d

17. Which one is the wrong statement? [NEET 2017] 26. Consider the following sets of quantum numbers.
h
(a) de-Broglie’s wavelength is given by λ = , where n l m s
mv I 3 0 0 +1/2
m = mass of the particle, v = group velocity of the particle II 2 2 1 +1/2
(b) The uncertainty principle is ∆E × ∆t ≥ h / 4π
III 4 3 −2 −1/2
(c) Half-filled and fully filled orbitals have greater stability due
IV 1 0 −1 −1/2
to greater exchange energy, greater symmetry and more
balanced arrangement V 3 2 3 +1/2
(d) The energy of 2s-orbital is less than the energy of Which of the following sets of quantum number is
2 p-orbital in case of hydrogen like atoms not possible? [CBSE-AIPMT 2007]
18. The value of Planck’s constant is 6.63 × 10−34 Js. The (a) II, III and IV (b) I, II, III and IV
speed of light is 3 × 1017 nm s −1. Which value is (c) II, IV and V (d) I and III
closest to the wavelength in nanometer of a 27. Which one is a wrong statement? [NEET 2018]
quantum of light with frequency of 6 × 1015 s−1? (a) The electronic configuration of N-atom is
(a) 10 nm (b) 25 nm [NEET 2013]
2p1x 2p1y 2p1z
(c) 50 nm (d) 75 nm 1s2 2s2

19. The measurement of the electron position is


associated with an uncertainty in momentum, which (b) An orbital is designated by three quantum numbers while
is equal to 1 × 10−18 g cm s −1. The uncertainty in an electron in an atom is designated by four quantum
electron velocity is (Mass of an electron is 9 × 10−28 g) numbers
[CBSE-AIPMT 2008] (c) Total orbital angular momentum of electron in ‘s’-orbital is
(a) 1 × 109 cm s −1 (b) 1 × 106 cm s −1 equal to zero
(c) 1 × 105 cm s –1 (d) 1 × 1011 cm s −1 (d) The value of m for d 2 is zero
z

MODULE 3
www.jeebooks.in
The NEET Edge ~ Chemistry 417

3. Classification of Elements and 4. Chemical Bonding and


Periodicity in Properties Molecular Structure
28. The element Z = 114 has been discovered recently. 36. The hybridisations of atomic orbitals of nitrogen in
It will belong to which of the following family/group NO+2 , NO −3 and NH +4 respectively are
[NEET 2016, Phase II]
and electronic configuration? [NEET 2017]
14 10 2 5 (a) sp, sp3 and sp2 (b) sp2 , sp3 and sp
(a) Halogen family, [Rn] 5f , 6d , 7 s , 7 p
(c) sp, sp2 and sp3 (d) sp2 , sp and sp3
(b) Carbon family, [Rn] 5f14 , 6d 10 , 7 s 2 , 7 p2
(c) Oxygen family, [Rn] 5f14 , 6d 10 , 7 s 2 , 7 p4 37. In which one of the following species the central
14 10 2 6 atom has the type of hybridisation which is not the
(d) Nitrogen family, [Rn] 5f , 6d , 7 s , 7 p same as that present in the other three?
[CBSE-AIPMT 2010]
29. Which of the following orders of ionic radii is
correctly represented? [CBSE-AIPMT 2014] (a) SF4 (b) I−3 (c) SbCl 2−
5 (d) PCl 5
− + + − 2−
(a) H > H >H (b) Na > F > O 38. The correct order of increasing bond angles in the
(c) F − > O 2 − > Na + (d) Al 3 + > Mg 2 + > N3 − following species is [CBSE-AIPMT 2010]
(a) Cl 2O < ClO 2 < ClO −2 (b) ClO 2 < Cl 2O < ClO −2
30. Identify the correct order of the size of the following.
(c) Cl 2O < ClO −2 < ClO 2 (d) ClO −2 < Cl 2O < ClO 2
(a) Ca 2+ < K+ < Ar < S2– < Cl – [CBSE-AIPMT 2007]
(b) Ca 2+ < K+ < Ar < Cl – < S2– 39. Which of the following species contains three bond
pairs and one lone pair around the central atom?
(c) Ar < Ca 2+ < K+ < Cl – < S2–
[NEET 2013]
(d) Ca 2+ < Ar < K+ < Cl – < S2–
(a) H2O (b) BF3 (c) NH−2 (d) PCl 3
31. Identify the wrong statement in the following.
[CBSE-AIPMT 2012]
40. The electronegativity difference between N and F
is greater than that between N and H yet the
(a) Amongst isoelectronic species, smaller the positive
dipole moment of NH 3 (1.5 D) is larger than that of
charge on the cation, smaller is the ionic radius
NF3 (0.2 D). This is because [CBSE-AIPMT 2006]
(b) Amongst isoelectronic species, greater the negative
(a) in NH3 as well as in NF3 , the atomic dipole and bond
charge on the anion, larger is the ionic radius
dipole are in the same direction
(c) Atomic radius of the elements increases as one moves (b) in NH3 , the atomic dipole and bond dipole are in the same
down the first group of the periodic table direction whereas in NF3 these are in opposite directions
(d) Atomic radius of the elements decreases as one moves (c) in NH3 as well as NF3 , the atomic dipole and bond dipole
across from left to right in the 2nd period of the periodic table are in opposite directions
32. The correct order of decreasing second ionisation (d) in NH3 the atomic dipole and bond dipole are in the
enthalpy of Ti(22), V(23), Cr(24) and Mn(25) is opposite directions whereas in NF3 these are in the same
[CBSE-AIPMT 2008] directions
(a) Cr > Mn > V > Ti (b) V > Mn > Cr > Ti 41. In BrF3 molecule, the lone pairs occupy equatorial
(c) Mn > Cr > Ti > V (d) Ti > V > Cr > Mn positions to minimise [CBSE-AIPMT 2004]
(a) lone pair-bond pair repulsion
33. Which of the following represents the correct order of
increasing electron gain enthalpy with negative sign (b) bond pair-bond pair repulsion
for the elements O, S, F and Cl? [CBSE-AIPMT 2010] (c) lone pair-lone pair repulsion and lone pair-bond pair
repulsion
(a) Cl < F < O < S (b) O < S < F < Cl
(d) lone pair-lone pair repulsion
(c) F < S < O < Cl (d) S < O < Cl < F
42. Which of the following is paramagnetic? [NEET 2013]
34. In which of the following options the order of (a) CO (b) O −2 (c) CN− (d) NO +
arrangement does not agree with the variation of
property indicated against it? [NEET 2016, Phase I] 43. Which one of the following species does not exist
(a) B < C < N < O (increasing first ionisation enthalpy) under normal conditions? [CBSE-AIPMT 2010]
(b) I < Br < Cl < F (increasing electron gain enthalpy) (a) Be +2 (b) Be 2 (c) B 2 (d) Li 2
(c) Li < Na < K < Rb (increasing metallic radius) 44. According to molecular orbital theory which of the
(d) Al 3+ < Mg 2+ < Na + <F − (increasing ionic size) following lists rank the nitrogen species in terms of
35. Which of the following oxides is not expected to increasing bond order? [CBSE-AIPMT 2009]
react with sodium hydroxide? [CBSE-AIPMT 2009] (a) N−2 < N2 < N 22 − (b) N 22 − < N2− < N2
(a) B 2O 3 (b) CaO (c) SiO 2 (d) BaO (c) N2 < N 22 − < N2− (d) N2− < N 22 − < N2

MODULE 3
www.jeebooks.in
418 NEET Test Drive

45. Four diatomic species are listed below in different


sequences. Which of these presents the correct 6. Thermodynamics
order of their increasing bond order?
[CBSE-AIPMT 2008] 53. A gas is allowed to expand in a well insulated
container against a constant external pressure of
(a) O –2 <NO <C 2– +
2 <He 2 (b) NO <C 2– – +
2 <O 2 <He 2 2.5 atm from an initial volume of 2.50 L to a final
(c) C 2 <He 2 <NO <O –2
2– + + – 2–
(d) He 2 <O 2 <NO <C 2 volume of 4.50 L. The change in internal energy
∆U of the gas in joules will be [NEET 2017]
(a) 1136.25 J (b) − 500 J
5. States of Matter (c) − 505 J (d) + 505 J

46. A 20 L container at 400 K contains CO2( g) at 54. The work done during the expansion of a gas from
pressure 0.4 atm and an excess of SrO (neglect the a volume of 4 dm 3 to 6 dm3 against a constant
volume of solid SrO). The volume of the container external pressure of 3 atm, is [CBSE-AIPMT 2004]
is now decreased by moving the movable piston (a) − 6 J (b) – 608 J
fitted in the container. The maximum volume of (c) + 304 J (d) – 304 J
the container, when pressure of CO2 attains its
55. The heat of combustion of carbon to CO2 is
maximum value, will be [NEET 2017]
−393.5 kJ / mol. The heat released upon the
(Given that : formation of 35.2 g of CO2 from carbon and oxygen
SrCO3 ( s ) q SrO( s ) + CO2( g) ; K p = 1.6 atm) gas is [CBSE-AIPMT 2015]
(a) 5 L (b) 10 L (c) 4 L (d) 2 L (a) −315 kJ (b) +315 kJ (c) −630 kJ (d) −315
. kJ
47. Equal masses of H 2 , O 2 and methane have been 56. Standard enthalpy of vaporisation ∆ vapH ° for water
taken in a container of volume V at temperature at 100°C is 40.66 kJ mol −1. The internal energy of
27°C in identical conditions. The ratio of the vaporisation of water at 100°C (in kJ mol −1 ) is
volumes of gases H 2 : O 2 : CH 4 would be
(assume water vapour to behave like an ideal gas).
[CBSE-AIPMT 2014]
[CBSE-AIPMT 2012]
(a) 8 : 16 : 1 (b) 16 : 8 : 1
(a) + 37.56 (b) − 43.76
(c) 16 : 1 : 2 (d) 8 : 1 : 2
(c) + 43.76 (d) + 40.66
48. A gaseous mixture was prepared by taking equal
57. The bond dissociation energies of X 2 , Y 2 and XY
moles of CO and N2 . If the total pressure of the
mixture was found 1 atm, the partial pressure of the are in the ratio of 1 : 0.5 : 1 . ∆H for the formation
nitrogen (N2 ) in the mixture is [CBSE-AIPMT 2011] of XY is −200 kJ mol −1. The bond dissociation
(a) 0.8 atm (b) 0.9 atm (c) 1 atm (d) 0.5 atm
energy of X 2 will be [NEET 2018]
(a) 800 kJ mol −1 (b) 100 kJ mol −1
49. Equal moles of hydrogen and oxygen gases are (c) 200 kJ mol −1 (d) 400 kJ mol −1
placed in container with a pin-hole through which
both can escape. What fraction of the oxygen 58. For a sample of perfect gas when its pressure is
escapes in the time required for one-half of the changed isothermally from pi to pf , the entropy
hydrogen to escape? [NEET 2016, Phase I] change is given by [NEET 2016, Phase II]

(a) 1/ 4 (b) 3/ 8 (c) 1/2 (d) 1/ 8 p p


(a) ∆S = nR ln  f  (b) ∆S = nR ln  i 
 pi   pf 
50. 50 mL of each gas A and of gas B takes 150 and
p p
200 s respectively for effusing through a pin-hole (c) ∆S = nRT ln  f  (d) ∆S = RT ln  i 
under the similar conditions. If molecular mass of  pi   pf 
gas B is 36, the molecular mass of gas A will be
(a) 96 (b) 128 [CBSE-AIPMT 2012]
59. For a given reaction, ∆H = 35.5 kJ mol−1 and
∆S = 83.6 JK −1 mol−1. The reaction is spontaneous
(c) 32 (d) 64
at (Assume that ∆H and ∆S do not vary with
51. By what factor does the average velocity of a temperature) [NEET 2017]
gaseous molecule increase when the temperature (a) T < 425 K (b) T > 425 K
(in Kelvin) is doubled? [CBSE-AIPMT 2011] (c) all temperatures (d) T > 298 K
(a) 2.8 (b) 4.0 (c) 1.4 (d) 2.0
60. The correct thermodynamic conditions for the
52. Maximum deviation from ideal gas is expected spontaneous reaction at all temperatures is
from [NEET 2013] [NEET 2016, Phase I]
(a) H2(g) (b) N2 (g ) (a) ∆H > 0 and ∆S < 0 (b) ∆H < 0 and ∆S > 0
(c) CH4 (g ) (d) NH3 (g ) (c) ∆H < 0 and ∆S < 0 (d) ∆H < 0 and ∆S = 0

MODULE 3
www.jeebooks.in
The NEET Edge ~ Chemistry 419

61. Consider the following liquid-vapour equilibrium : 67. Which of the following fluoro-compounds is most
Liquid Vapour ! likely to behave as a Lewis base? [NEET 2016, Phase II]
Which of the following relations is correct? (a) BF3 (b) PF3 (c) CF4 (d) SiF4
[NEET 2016, Phase I]
d ln p − ∆Hv d ln p − ∆Hv 68. The dissociation constants for acetic acid and HCN
= =
(a)
dT RT
(b)
dT 2
T 2 at 25°C are 1.5 × 10−5 and 4.5 × 10−10 , respectively.
d ln p − ∆Hv d ln G − ∆Hv The equilibrium constant for the equilibrium,
(c) = (d) =
dT RT 2 dT 2 RT 2 CN – + CH3COOH HCN + CH3COO– would be
r
[CBSE-AIPMT 2009]
7. Equilibrium (a) 3.0 × 105 (b) 3.0 × 10−5 (c) 3.0 × 10−4 (d) 3.0 × 104

62. If the value of an equilibrium constant for a 69. The percentage of pyridine (C5H5N ) that forms
particular reaction is 1.6 × 1012, then at equilibrium pyridinium ion (C5H5N +H ) in a 0.10 M aqueous
the system will contain [CBSE-AIPMT 2015] pyridine solution (K b for C5H5N = 1.7 × 10−9 ) is
(a) all reactants [NEET 2016, Phase II]
(b) mostly reactants (a) 0.0060% (b) 0.013% (c) 0.77% (d) 1.6%
(c) mostly products
(d) similar amounts of reactants and products 70. Which of the following salts will give highest pH in
water? [CBSE-AIPMT 2014]
63. The equilibrium constants of the following are (a) KCl (b) NaCl (c) Na 2CO 3 (d) CuSO4
N 2 + 3H 2 q 2NH3 ; K 1 71. Following solutions were prepared by mixing
N 2 + O2 q 2NO; K 2 different volumes of NaOH and HCl of different
concentrations :
1
H 2 + O2 → H 2O; K 3 I. 60 mL
M
HCl + 40 mL
M
NaOH
2 10 10
The equilibrium constant (K ) of the reaction M M
K II. 55 mL HCl + 45 mL NaOH
5 10 10
2NH3 + O2 q 2NO + 3H 2O, will be
2 M M
[NEET 2017, CBSE-AIPMT 2007] III. 75 mL HCl + 25 mL NaOH
(a) K1K 33 / K 2 (b) K 2 K 33 / K1 5 5
(c) K 2 K 3 / K1 (d) K 23 K 3 / K1 M M
IV. 100 mL HCl + 100 mL NaOH
10 10
64. If the concentration of OH − ions in the reaction,
Fe(OH)3 ( s) Fe3 + ( aq ) + 3OH − ( aq ) is decreased by
r pH of which one of them will be equal to 1? [NEET 2018]
(a) IV (b) I (c) II (d) III
1/4 times, then equilibrium concentration of Fe3 +
will increase by [CBSE-AIPMT 2008] 72. The ionisation constant of ammonium hydroxide is
(a) 8 times (b) 16 times (c) 64 times (d) 4 times 1.77 × 10−5 at 298 K. Hydrolysis constant of
ammonium chloride is [CBSE-AIPMT 2009]
65. The dissociation equilibrium of a gas AB2 can be
(a) 5.65 × 10−10 (b) 6.50 × 10−12
represented as
2 AB2( g) 3 2 AB( g) + B ( g) (c) 5.65 × 10−13 (d) 5.65 × 10−12
2
73. The solubility of BaSO4 in water is
The degree of dissociation is x and is small
2.42 × 10−3 g L−1 at 298 K. The value of its solubility
compared to 1. The expression relating the degree
of dissociation ( x ) with equilibrium constant K p product ( K sp ) will be
and total pressure p is [CBSE-AIPMT 2008] (Given, molar mass of BaSO4 = 233 g mol−1 )
[NEET 2018]
(a) (2 K p / p) (b) (2 K p / p)1/ 3
(a) 1.08 × 10−14 mol 2L−2 (b) 1.08 × 10−12 mol 2L−2
(c) (2 K p / p)1/ 2 (d) (K p / p) (c) 1.08 × 10−10 mol 2L−2 (d) 1.08 × 10−8 mol 2L−2

66. The value of ∆H for the reaction, 74. Using the Gibbs energy change, ∆G° = + 63.3 kJ
X 2( g) + 4Y 2( g) 2 XY 4 ( g) is less than zero.
r for the following reaction,
Formation of XY 4( g) will be favoured at Ag2 CO3 ( s) r2Ag+ ( aq) + CO32− (aq) the K sp of
(a) low pressure and low temperature [CBSE-AIPMT 2011]
Ag2CO3 ( s) in water at 25°C is (R = 8.314 JK −1mol−1 ).
(b) high temperature and low pressure [CBSE-AIPMT 2014]
(c) high pressure and low temperature −26 −12
(a) 3.2 × 10 (b) 8.0 × 10 (c) 2.9 × 10−3 (d) 7.9 × 10−2
(d) high temperature and high pressure

MODULE 3
www.jeebooks.in
420 NEET Test Drive

83. The alkali metals form salt like hydrides by the


8. Redox Reactions direct synthesis at elevated temperature. The
75. For the redox reaction, thermal stability of these hydrides decreases in
which of the following orders?
MnO−4 + C2O24− + H + → Mn2+ + CO2 + H 2O
[CBSE-AIPMT 2008]

the correct coefficients of the reactants for the (a) CsH > RbH > KH > NaH > LiH
(b) KH > NaH > LiH > CsH > RbH
balanced equation are [NEET 2018]
(c) NaH > LiH > KH > RbH > CsH
MnO −4 C 2O 2−
4 H+ MnO −4 C 2O 2−
4 H+
(d) LiH > NaH > KH > RbH > CsH
(a) 2 16 5 (b) 2 5 16
(c) 16 5 2 (d) 5 16 2 84. ‘‘Metals are usually not found as nitrates in their
ores’’. Out of the following two (I and II) reasons
76. Which of the following does not give oxygen on which is/are true for the above observation?
heating? [NEET 2013] I. Metal nitrates are highly unstable.
(a) Zn(ClO 3 )2 (b) K 2Cr2O 7 (c) (NH4 )2 Cr2O 7 (d) KClO 3 II. Metal nitrates are highly soluble in water.
[CBSE-AIPMT 2015]
77. Which one of the following compounds is a
peroxide? [CBSE-AIPMT 2018]
(a) I and II are true (b) I and II are false
(c) I is false but II is true (d) I is true but II is false
(a) KO 2 (b) BaO 2 (c) NO 2 (d) MnO 2
85. Among CaH 2 , BeH 2 , BaH 2 , the order of ionic
78. When Cl2 gas reacts with hot and concentrated
sodium hydroxide solution, the oxidation number character is [NEET 2018]
of chlorine changes from [CBSE-AIPMT 2012] (a) BeH2 < BaH2 < CaH2 (b) CaH2 < BeH2 < BaH2
(a) 0 to +1and 0 to −3 (b) 0 to −1and 0 to + 3 (c) BeH2 < CaH2 < BaH2 (d) BaH2 < BeH2 < CaH2
(c) 0 to +1and 0 to − 5 (d) 0 to −1and 0 to + 5 86. Equimolar solutions of the following were prepared
79. KMnO4 can be prepared from K 2MnO4 as per the in water separately. Which one of the solutions will
reaction, record the highest pH? [CBSE-AIPMT 2008]
(a) SrCl 2 (b) BaCl 2
3MnO24− + 2H 2O 2MnO4− + MnO2 + 4OH −
= (c) MgCl 2 (d) CaCl 2
The reaction can go to completion by removing OH −
ions by adding [NEET 2013] 87. Solubility of the alkaline earth's metal sulphates in
(a) KOH (b) HCl (c) CO 2 (d) SO 2 water decreases in the sequence [CBSE-AIPMT 2015]
(a) Mg > Ca > Sr > Ba (b) Ca > Sr > Ba > Mg
(c) Sr > Ca > Mg > Ba (d) Ba > Mg > Sr > Ca
9. Hydrogen
88. In which of the following the hydration energy is
80. Which of the following statements about hydrogen higher than the lattice energy? [CBSE-AIPMT 2007]
is incorrect? [NEET 2016, Phase I]
(a) BaSO 4 (b) MgSO 4
(a) Hydrogen never acts as cation in ionic salts (c) RaSO 4 (d) SrSO 4
(b) Hydronium ion, H 3 O + exists freely in solution
(c) Dihydrogen does not act as a reducing agent 89. The product obtained as a result of a reaction of
(d) Hydrogen has three isotopes of which tritium is the most nitrogen with CaC 2 is [NEET 2016, Phase I]
common (a) CaCN (b) CaCN 3
(c) Ca 2 CN (d) Ca(CN) 2
81. I. H2O2 + O3 → H2O + 2O2
II. H2O2 + Ag 2O → 2Ag + H2O + O2 90. A solid compound X on heating gives CO2 gas and
a residue. The residue mixed with water forms Y .
Role of hydrogen peroxide in the above reaction is On passing an excess of CO2 through Y in water, a
respectively [CBSE-AIPMT 2018]
clear solution Z is obtained. On boiling Z,
(a) reducing in I and oxidising in II compound X is reformed. The compound X is
(b) oxidising in I and reducing in II (a) Ca(HCO3 )2 (b) CaCO3 [CBSE-AIPMT 2004]
(c) oxidising in I and II (c) Na 2CO3 (d) K 2CO3
(d) reducing in I and II

10. s-block Elements 11. Some p-block Elements


91. The correct order of atomic radii in group 13
82. The sequence of ionic mobility in aqueous solution
elements is [NEET 2018]
is [CBSE-AIPMT 2008]
+ + + + + + + + (a) B < Ga < Al < Tl < In (b) B < Al < Ga < In < Tl
(a) K > Na > Rb > Cs (b) Cs > Rb > K > Na
(c) B < Al < In < Ga < Tl (d) B < Ga < Al < In < Tl
(c) Rb+ > K+ > Cs+ > Na + (d) Na + > K+ > Rb+ > Cs+

MODULE 3
www.jeebooks.in
The NEET Edge ~ Chemistry 421

92. The tendency of BF3 , BCl 3 and BBr3 behave as Lewis 100. Which of the following biphenyls is optically active?
acid decreases in the sequence [CBSE-AIPMT 2009] [NEET 2016, Phase I]
(a) BCl 3 > BF3 > BBr3 (b) BBr3 > BCl 3 > BF3 Br Br I
(c) BBr3 > BF3 > BCl 3 (d) BF3 > BCl 3 > BBr3
93. Al 2 O3 can be converted into anhy. AlCl3 by heating (a) (b)
(a) Al 2O3 with HCl gas [CBSE-AIPMT 2006]
I I I
(b) Al 2O 3 with NaCl in solid state
(c) a mixture of Al 2O 3 and carbon in dry Cl 2 gas CH3 O2N
(d) Al 2O3 with Cl 2 gas
(c) (d)
94. AlF3 is soluble in HF only in presence of KF. It is
due to the formation of [NEET 2016, Phase II]
CH3 I
(a) K 3 [AIF3H3 ] (b) K 3 [AIF6 ] (c) AIH 3 (d) K[AIF3H]
95. Boric acid is an acid because its molecule 101. The correct statement regarding a carbonyl
(a) contains replaceable H+ ion [NEET 2016, Phase II]
compound with a hydrogen atom on its
(b) gives up a proton
alpha-carbon, is [NEET 2016, Phase I]

(c) accepts OH − from water releasing proton (a) a carbonyl compound with a hydrogen atom on its
(d) combines with proton from water molecule alpha-carbon rapidly equilibrates with its corresponding
enol and this process is known as aldehyde-ketone
96. It is because of inability of ns2 electrons of the equilibration
valence shell to participate in bonding that (b) a carbonyl compound with a hydrogen atom on its
(a) Sn2+ is reducing while Pb 4+ is oxidising [NEET 2017] alpha-carbon rapidly equilibrates with its
(b) Sn2+ is oxidising while Pb 4+ is reducing corresponding enol and this process is known as
(c) Sn2+ and Pb 2+ are both oxidising and reducing carbonylation
(d) Sn4+ is reducing while Pb 4+ is oxidising (c) a carbonyl compound with a hydrogen atom on its
97. Name the type of the structure of silicate in which alpha-carbon rapidly equilibrates with its corresponding
enol and this process is known as keto-enol
one oxygen atom of [SiO4 ]4– is shared?
tautomerism
[CBSE-AIPMT 2011]
(d) a carbonyl compound with a hydrogen atom on its
(a) Sheet silicate (b) Pyrosilicate
alpha-carbon never equilibrates with its corresponding
(c) Three dimensional silicate (d) Linear chain silicate
enol
98. The straight chain polymer is formed by 102. The order of stability of the following tautomeric
[CBSE-AIPMT 2009]
compound is [NEET 2013]
(a) hydrolysis of (CH3 )3 SiCl followed by condensation
polymerisation OH O
(b) hydrolysis of CH3SiCl 3 followed by condensation  
polymerisation CH 2 == C  CH 2  C CH3 º
(c) hydrolysis of (CH3 )4 Si by addition polymerisation I
(d) hydrolysis of (CH3 )2 SiCl 2 followed by condensation O O
polymerisation  
CH3  C CH 2  C CH3 º
II
12. Organic Chemistry : Some Basic OH O
Principles and Techniques  
CH3 C == CH  C CH3
99. The IUPAC name of the compound [NEET 2017] III
O (a) I > II > III (b) III > II > I
O
(c) II > I> III (d) II > III > I
H C
is ......... . 103. How many stereoisomers does this molecule have?
CH3CH == CHCH 2CHBrCH3 [CBSE-AIPMT 2008]
(a) 3-keto-2-methylhex-4-enal (b) 5-formylhex-2-en-3-one (a) 4 (b) 6
(c) 5-methyl-4-oxohex-2-en-5-al (d) 3-keto-2-methylhex-5-enal (c) 8 (d) 2

MODULE 3
www.jeebooks.in
422 NEET Test Drive

104. Consider the following compounds


13. Hydrocarbons
CH3 Ph
109. The correct statement regarding the comparison
I. CH3 C CH II. Ph C Ph of staggered and eclipsed conformations of ethane,
is [NEET 2016, Phase I]
CH3 (a) the eclipsed conformation of ethane is more stable than
staggered conformation, because eclipsed
III. conformation has no torsional strain
CH3
(b) the eclipsed conformation of ethane is more stable than
hyperconjugation occurs in [CBSE-AIPMT 2015] staggered conformation even though the eclipsed
conformation has torsional strain
(a) I only (b) II only
(c) the staggered conformation of ethane is more stable
(c) III only (d) I and III
than eclipsed conformation, because staggered
105. Which of the following is the most correct electron conformation has no torsional strain
displacement for a nucleophilic reaction to take place? (d) the staggered conformation of ethane is less stable
[CBSE-AIPMT 2015] than eclipsed conformation, because staggered
H H2 H H2 conformation has torsional strain
(a) H3C →– C == C  C  Cl (b) H3C →– C == C  C  Cl
H H 110. Hydrocarbon (A) reacts with bromine by
H H2 H H2 substitution to form an alkyl bromide which by
(c) H3C →– C == C  C  Cl (d) H3C →– C == C  C  Cl Wurtz reaction is converted to gaseous
H H
hydrocarbon containing less than four carbon
106. Which one is most reactive towards nucleophilic atoms.A is [NEET 2018]
addition reaction? [CBSE-AIPMT 2014] (a) CH3  CH3 (b) CH2 == CH2 (c) CH ≡≡ CH (d) CH4
O
111. Given,
CHO C CH3 H3C CH3 H3C CH2 H2C CH2
(a) (c)

CHO CHO CH3 CH3 CH2


I II III

(c) (d) The enthalpy of hydrogenation of these


compounds will be in the order as [CBSE-AIPMT 2015]
(a) I > II > III (b) III > II > I (c) II > III > I (d) II > I > III
CH3 NO2
112. Which of the following compounds shall not
produce propene by reaction with HBr followed by
107. Among the following compound one that is most elimination or direct only elimination reaction?
reactive towards electrophilic nitration is [NEET 2016, Phase II]
[CBSE-AIPMT 2012] H2
(a) benzoic acid (b) nitrobenzene (a) H2 C CH2 (b) H3C  C  CH2OH
(c) toluene (d) benzene CH2
H2
108. Which one is most reactive towards electrophilic (c) H2C == C == O (d) H3C  C  CH2Br
reagent ? [CBSE-AIPMT 2010]
CH3 113. Reaction of HBr with propene in the presence of
CH3
peroxide gives [CBSE-AIPMT 2004]
OH CH2OH (a) iso-propyl bromide (b) 3-bromopropane
(a) (c) (c) allyl bromide (d) n-propyl bromide
114. 2,3-dimethyl-2-butene can be prepared by heating
which of the following compounds with a strong
CH3 CH3 acid? [NEET 2016, Phase I]
NHCOCH3 OCH3 (a) (CH3 )2 CH—CH — CH == CH3

(c) (d) CH3
(b) (CH3 )3 C —CH == CH2
(c) (CH3 )2 C ==CH — CH 2 —CH2
(d) (CH3 )2 CH — CH2 — CH == CH2
MODULE 3
www.jeebooks.in
The NEET Edge ~ Chemistry 423

115. Which one is the correct order of acidity? 122. Which one of the following statements is not true?
[NEET 2017] [CBSE-AIPMT 2011]
(a) CH2 == CH2 > CH3  CH == CH2 > CH3  C ≡≡ CH (a) Clean water would have a BOD value of less than 5 ppm
> CH ≡≡ CH (b) Concentration of DO below 6 ppm is good for the growth
(b) CH ≡≡ CH > CH3 C ≡≡ CH >CH2 == CH2 > CH3 CH3 of fish
(c) CH ≡≡ CH > CH2 == CH2 > CH3  C ≡≡ CH > CH3 CH3 (c) pH of drinking water should be between 5.5-9.5
(d) CH3  CH3 > CH2 == CH2 > CH3  C ≡≡ CH > CH ≡≡ CH (d) Oxides of sulphur, nitrogen and carbon are the most wide
(i) NaNH2 /liq.NH3 spread air pollutant
116. In the reaction, H C ≡≡ CH →
(ii) CH3 CH2Br 123. Which oxide of nitrogen is not a common pollutant
(i) NaNH2 /liq.NH3 introduced into the atmosphere both due to natural
X → Y , X and Y are and human activity? [NEET 2018]
(ii ) CH3 CH2Br [NEET 2016, Phase I]
(a) NO (b) NO 2 (c) N2O 5 (d) N2O
(a) X = 2 -butyne; Y = 3 -hexyne
(b) X = 2 -butyne; Y = 2 -hexyne
(c) X = 1-butyne; Y = 2 -hexyne 15. Solid State
(d) X = 1-butyne; Y = 3 -hexyne
117. Predict the product C obtained in the following 124. In calcium fluoride, having the fluorite structure,
reaction of butyne-1. [CBSE-AIPMT 2007] the coordination numbers for calcium ion (Ca 2+ ) and
fluoride ion (F − ) are [NEET 2016, Phase II]
HI
CH3CH 2 C ≡≡ CH + HCl → B → C (a) 4 and 2 (b) 6 and 6
I (c) 8 and 4 (d) 4 and 8

(a) CH3 —CH—CH2CH2I (b) CH3 — CH2 — CH2 —C — H 125. Which one of the following statements is an
  incorrect? [CBSE-AIPMT 2008]
Cl Cl (a) The fraction of the total volume occupied by the atoms in
I I a primitive cell is 0.48
  (b) Molecular solids are generally volatile
(c) CH3 — CH2 —CH — CH2Cl (d) CH3CH2  C  CH3
(c) The number of carbon atoms in an unit cell of diamond is 4

(d) The number of Bravais lattices in which a crystal can be
Cl
categorised is 14
118. Which of the following can be used as the halide 126. Percentage of free space in body centred cubic (bcc)
component for Friedel-Crafts reaction?
unit cell is [CBSE-AIPMT 2008]
[NEET 2016, Phase II]
(a) 30% (b) 32% (c) 34% (d) 28%
(a) Chlorobenzene (b) Bromobenzene
(c) Chloroethene (d) Isopropyl chloride 127. If ‘ a ’ stands for the edge length of the cubic systems
119. The reaction of toluene with Cl2 in the presence of : simple cubic, body centred cubic and face centred
cubic, then the ratio of radii of the spheres in these
FeCl3 gives ‘X’ and reaction in presence of light
systems will be respectively, [CBSE-AIPMT 2008]
gives ‘Y ’. Thus, ‘X’ and ‘Y’ are [CBSE-AIPMT 2010]
1 3 1 1 1
(a) X = benzal chloride, Y = o-chlorotoluene (a) a: a: a (b) a: 3a: a
(b) X = m-chlorotoluene, Y = p -chlorotoluene 2 4 2 2 2 2
(c) X = o and p-chlorotoluene, Y = trichloromethyl benzene (c)
1
a:
3
a:
2
a (d) 1a : 3 a : 2 a
(d) X = benzyl chloride, Y = m-chlorotoluene 2 2 2

128. Iron exhibits bcc structure at room temperature.


14. Environmental Chemistry Above 900°C , it transforms to fcc structure. The
ratio of density of iron at room temperature to that
120. Which of the following is a sink for CO ? [NEET 2017] at 900°C (assuming molar mass and atomic radii of
(a) Haemoglobin iron remains constant with temperature) is
(b) Microorganisms present in the soil 3 3 4 3 3 1
(a) (b) (c) (d)
(c) Oceans 4 2 3 2 2 2
(d) Plants 129. Lithium has a bcc structure. Its density is 530 kg
121. Which one of the following is not a common m −3 and its atomic mass is 6.94 g mol − 1. Calculate
component of photochemical smog?[CBSE-AIPMT 2014] the edge length of a unit cell of lithium metal.
(a) Ozone (b) Acrolein (N A = 6 . 02 × 1023 mol − 1) [NEET 2016, Phase I]
(c) Peroxyacetyl nitrate (d) Chlorofluorocarbons (a) 352 pm (b) 527 pm (c) 264 pm (d) 154 pm

MODULE 3
www.jeebooks.in
424 NEET Test Drive

130. The ionic radii of A+ and B- ions are 0.98 ´ 10-10 m (c) Not enough information is given to make a prediction
and 1.81 ´ 10-10 m. The coordination number of (d) The vapour will contain a higher percentage of benzene
each ion in AB is [NEET 2016, Phase I]
138. At 100°C the vapour pressure of a solution of 6.5 g
(a) 4 (b) 8 (c) 2 (d) 6 of a solute in 100 g water is 732 mm. If K b = 0.52,
131. Which is the incorrect statement? [NEET 2017] the boiling point of this solution will be
(a) FeO 0.98 has non-stoichiometric metal deficiency defect [NEET 2016, Phase I]
(b).Density decreases in case of crystals with Schottky’s defect (a) 100°C (b) 102°C (c) 103°C (d) 101°C
(c) NaCl(s) is insulator, silicon is semiconductor, silver is -1
139. The boiling point of 0.2 mol kg solution of X in
conductor, quartz is piezoelectric crystal water is greater than equimolal solution of Y in
(d) Frenkel defect is favoured in those ionic compounds in water. Which one of the following statements is
which sizes of cation and anions are almost equal true in this case? [CBSE-AIPMT 2015]
132. If NaCl is doped with 10-4 mol % of SrCl2 , the (a) X is undergoing dissociation in water
concentration of cation vacancies will be (b) Molecular mass of X is greater than the molecular mass of Y
( NA = 6.023 ´ 1023 mol-1 ) [CBSE-AIPMT 2007] (c) Molecular mass of X is less than the molecular mass of Y
(a) 6.023 ´ 1015 mol -1 (b) 6.023 ´ 1016 mol -1 (d) Y is undergoing dissociation in water while X undergoes
17
(c) 6.023 ´ 10 mol -1
(d) 6.023 ´ 1014 mol -1 no change
140. Which one of the following electrolytes has the same
16. Solutions value of van't Hoff factor ( i ) as that of Al2 (SO 4 )3 (if
all are 100% ionised)? [CBSE-AIPMT 2015]
133. A solution has 1 : 4 mole ratio of pentane to hexane. (a) K 2SO 4 (b) K 3 [Fe(CN)6 ]
The vapour pressure of the pure hydrocarbons at (c) AI(NO 3 )3 (d) K 4 [Fe(CN)6 ]
20°C are 440 mm of Hg for pentane and 120 mm of
141. The freezing point depression constant for water is
Hg for hexane. The mole fraction of pentane in the
– 1.86°C m -1. If 5.00 g Na 2 SO4 is dissolved in
vapour phase would be [CBSE-AIPMT 2005]
45.0 g H2O, the freezing point is changed by
(a) 0.549 (b) 0.200 (c) 0.786 (d) 0.478
– 3.82°C. Calculate the van’t Hoff factor for
134. If molality of the dilute solution is doubled, the Na 2 SO4 . [CBSE-AIPMT 2011]
value of molal depression constant ( K f ) will be (a) 2.63 (b) 3.11 (c) 0.381 (d) 2.05
(a) doubled (b) halved [NEET 2017]
142. A solution of sucrose (molar mass = 342 g mol -1 ) has
(c) tripled (d) unchanged
been prepared by dissolving 68.5 g of sucrose in
135. 25.3 g of sodium carbonate, Na 2CO3 is dissolved in 1000 g of water. The freezing point of the solution
enough water to make 250 mL of solution. If obtained will be (kf for water = 1.86 K kg mol -1 )
sodium carbonate dissociates completely, molar [CBSE-AIPMT 2010]
concentration of sodium ion, Na + and carbonate (a) - 0.372° C (b) - 0.520° C
ion, CO32- are respectively (molar mass of (c) + 0.372° C (d) - 0.570° C
Na 2CO3 = 106 g mol-1 ) [CBSE-AIPMT 2010] 143. A 0.0020 m aqueous solution of an ionic compound
(a) 0.955 M and 1.910 M (b) 1.910 M and 0.955 M Co(NH3 ) 5(NO2 )Cl freezes at – 0.00732°C. Number
(c) 1.90 M and 1.910 M (d) 0.477 M and 0.477 M of moles of ions which 1 mole of ionic compound
produces on being dissolved in water will be
136. An aqueous solution is 1.00 molal in KI. Which (kf = - 1.86° C / m ) [CBSE-AIPMT 2009]
change will cause the vapour pressure of the
solution to increase? [CBSE-AIPMT 2010]
(a) 2 (b) 3 (c) 4 (d) 1
3
(a) Addition of NaCl (b) Addition of Na 2SO 4 144. A solution containing 10 g per dm of urea
(c) Addition of 1.00 molal KI (d) Addition of water (molecular mass = 60 g mol –1) is isotonic with a 5%
solution of a non-volatile solute. The molecular
137. Which of the following statements about the mass of this non-volatile solute is [NEET 2006]
composition of the vapour over an ideal 1:1 molar
(a) 250 g mol –1 (b) 300 g mol –1
mixture of benzene and toluene is correct? Assume (c) 350 g mol –1 (d) 200 g mol –1
that the temperature is constant at 25°C.
(Given, vapour pressure data at 25°C, benzene 145. The vapour pressure of two liquids P and Q are 80
= 12 . 8 kPa, toluene = 3 . 85 kPa) [NEET 2016, Phase I] and 60 torr, respectively. The total vapour pressure
of solution obtained by mixing 3 moles of P and 2
(a) The vapour will contain a higher percentage of toluene
moles of Q would be [CBSE-AIPMT 2005]
(b) The vapour will contain equal amounts of benzene and
toluene (a) 140 torr (b) 20 torr (c) 68 torr (d) 72 torr

MODULE 3
www.jeebooks.in
The NEET Edge ~ Chemistry 425

153. In the electrochemical cell Zn| ZnSO4 (0.01 M)||


17. Electrochemistry CuSO4(1.0M)| Cu, the emf of this Daniel cell is E1.
146. The molar conductivity of a 0.5 mol/dm solution 3 When the concentration ZnSO4 is changed to 1.0 M
of AgNO3 with electrolytic conductivity of and that of CuSO4 changed to 0.01 M, the emf
5.76 ´ 10-3 S cm -1 at 298 K is changes to E2. From the followings, which one is
[CBSE-AIPMT 2016, Phase II] the relationship between E1 and E2 ? (Given,
RT
(a) 2.88 S cm 2 /mol (b) 11.52 S cm 2 /mol = 0.059) [NEET 2017, 2003]
(c) 0.086 S cm 2 /mol (d) 28.8 S cm 2 /mol
F
(a) E1 = E2 (b) E1 < E2
147. At 25° C molar conductance of 0.1 molar aqueous (c) E1 > E2 (d) E2 = 0 ¹ E1
solution of ammonium hydroxide is 9.54 W-1 cm 2
mol -1 and at infinite dilution its molar conductance
154. The pressure of H 2 required to make the potential
is 238 W-1 cm 2 mol-1. The degree of ionisation of of H 2 electrode zero in pure water at 298 K is
ammonium hydroxide at the same concentration [CBSE-AIPMT 2016, Phase I]
and temperature is [NEET 2013] (a) 10-12 atm (b) 10-10 atm (c) 10-4 atm (d) 10-14 atm
(a) 2.080 % (b) 20.800 % (c) 4.008 % (d) 40.800 % 155. For the reduction of silver ions with copper metal,
148. The weight of silver (at. wt. = 108) displaced by a the standard cell potential was found to be + 0.46 V
quantity of electricity which displaces 5600 mL of at 25°C. The value of standard Gibbs energy, DG°
O2 at STP will be [CBSE-AIPMT 2014] will be ( F = 96500C mol-1 ) [CBSE-AIPMT 2010]
(a) 5.4 g (b) 10.8 g (c) 54.0 g (d) 108.0 g (a) - 89.0 kJ (b) - 89.0 J
(c) - 44.5 kJ (d) - 98.0 kJ
149. Al2O3 is reduced by electrolysis at low potentials
and high currents. If 4.0 ´ 10 4 A of current is 156. Standard free energies of formation (in kJ/mol) at
passed through molten Al2O3 for 6 h, what mass of 298 K are –237.2, –394.4 and –8.2 for H 2 O ( l ), d
aluminium is produced? (Assume 100% current CO2( g) and pentane (g), respectively. The value of
efficiency, atomic mass of Al = 27 g mol-1) E°cell for the pentane-oxygen fuel cell is
[CBSE-AIPMT 2009]
[CBSE-AIPMT 2008]
(a) 9.0 ´ 103 g (b) 8.1 ´ 104 g (a) 1.968 V (b) 2.0968 V (c) 1.0968 V (d) 0.0968 V
(c) 2.4 ´ 105 g (d) 1.3 ´ 104 g
157. The standard EMF of a galvanic cell involving cell
150. A button cell used in watches functions as following. reaction with n = 2 is found to be 0.295 V at 25° C.
Zn( s) + Ag2O ( s) + H 2O ( l ) º 2 Ag ( s) The equilibrium constant of the reaction would be
+ Zn 2+
( aq ) + 2OH - ( aq ) (Given, F = 96500 C mol–1 , R = 8.314 JK –1 mol–1 )
[CBSE-AIPMT 2004]
If half-cell potentials are :
Zn2+ ( aq ) + 2e- ¾® Zn ( s ); E° = - 0.76 V (a) 2.0 ´ 1011 (b) 4.0 ´ 1012 (c) 10
. ´ 102 . ´ 1010
(d) 10
Ag2O ( s) + H 2O ( l ) + 2e- ¾® 2Ag ( s) + 2OH - ( aq);
E° = 0.34 V 18. Chemical Kinetics
The cell potential will be [NEET 2013]
(a) 1.10 V (b) 0.42 V (c) 0.84 V (d) 1.34 V 158. For the reaction, N 2 + 3H 2 ¾® 2NH3 , if
d [NH3 ] - d [H 2 ]
151. E°Fe 2+ / Fe = - 0.441 V and E°Fe3 + / Fe 2+ = 0.771V the = 2 ´ 10-4 mol L-1s-1, the value of
dt dt
standard emf of the reaction would be [CBSE-AIPMT 2009]
Fe + 2Fe3 + ¾® 3Fe2 + will be [CBSE-AIPMT 2006] (a) 3 ´ 10-4 mol L-1 s -1 (b) 4 ´ 10-4 mol L-1 s -1
(a) 0.111 V (b) 0.330 V (c) 1.653 V (d) 1.212 V (c) 6 ´ 10-4 mol L-1 s -1 (d) 1 ´ 10-4 mol L-1 s -1
152. Consider the change in oxidation state of bromine 159. Consider the reaction,
corresponding to different emf values as shown in N 2( g) + 3H 2( g) ¾® 2NH3 ( g)
the diagram below.
The equality relationship between
1.82 V 1.5 V
BrO-4 ¾¾® BrO3- ¾¾® HBrO d [NH3 ]
and –
d [H 2 ]
is [CBSE-AIPMT 2006]
1.595 V 1.0652 V
dt dt
¾¾® Br2 ¾¾¾® Br - d [NH3 ] 1 d [H2 ] d [NH3 ] 2 d [H2 ]
(a) =– (b) + =–
Then the species undergoing disproportionation is dt 3 dt dt 3 dt
(a) Br2 (b) BrO -4 [NEET 2018] (c) +
d [NH3 ]
=–
3 d [H2 ]
(d)
d [NH3 ]
=–
d [H2 ]
(c) BrO -3 (d) HBrO dt 2 dt dt dt

MODULE 3
www.jeebooks.in
426 NEET Test Drive

160. For the reaction, A + B ¾® products, it is 167. The activation energy of a reaction can be
observed that determined from the slope of which of the following
I. On doubling the initial concentration of A only, graphs? [CBSE-AIPMT 2015]

the rate of reaction is also doubled and In K I T I


(a) In K vs T (b) vs T (c) In K vs (d) vs
T T In K T
II. On doubling the initial concentrations of both A
and B, there is a change by a factor of 8 in the 168. The rate constants k1 and k2 for two different
rate of the reaction. reactions are 1016 × e-2000/ T and 1015 × e-1000/ T ,
The rate of this reaction is, given by respectively. The temperature at which k1 = k2 is
[CBSE-AIPMT 2009] [CBSE-AIPMT 2008]
(a) rate = k[ A]2 [B] (b) rate = k[ A][B]2 (a) 1000 K (b)
2000
K (c) 2000 K (d)
1000
K
(c) rate = k[ A]2 [B]2 (d) rate = k[ A][B] 2.303 2.303

161. Mechanism of a hypothetical reaction, 169. What is the activation energy for a reaction if its
rate doubles when the temperature is raised from
X 2 + Y 2 ¾® 2XY is given below : [NEET 2017]
20° C to 35° C ? (R = 8.314 J mol -1 K -1) [ NEET 2013]
I. X 2 q X + X (fast)
(a) 342 kJ mol -1 (b) 269 kJ mol -1
II. X + Y 2 ¾® XY + Y (slow)
(c) 34.7 kJ mol -1 (d) 15.1 kJ mol -1
III. X + Y ¾® XY (fast)
The overall order of the reaction will be
(a) 1 (b) 2 (c) 0 (d) 1.5 19. Surface Chemistry
162. The rate of reaction between two reactants A and B 170. Which of the following statements is correct for the
decreases by a factor of 4, if the concentration of spontaneous adsorption of a gas? [CBSE-AIPMT 2014]
reactant B is doubled. The order of this reaction (a) DS is negative and therefore, DH should be highly positive
with respect to reactant B is [CBSE-AIPMT 2005] (b) DS is negative and therefore, D H should be highly negative
(a) -1 (b) -2 (c) 1 (d) 2 (c) DS is positive and therefore, D H should be negative
(d) DS is positive and therefore, D H should also be highly
163. A first order reaction has a specific reaction rate of positive
10-2 s-1. How much time will it take for 20 g of the
reactant to reduce to 5 g? [NEET 2017] 171. If x is amount of adsorbate and m is amount of
(a) 238.6 s (b) 138.6 s (c) 346.5 s (d) 693.0 s
adsorbent, which of the following relations is not
related to adsorption process? [CBSE-AIPMT 2011]
164. For a first order reaction A ¾¾® B, the reaction
(b) p = f(T ) at constant æç ö÷
x x
rate at reactant concentration of 0.01 M is found (a) = f(T ) at constant p
m è mø
to be 2.0 ´ 10–5 mol L–1s–1. The half-life period of
x x
the reaction is [CBSE-AIPMT 2005] (c) = p ´ T (d) = f( p) at constant T
m m
(a) 220 s (b) 30 s (c) 300 s (d) 347 s
172. For adsorption of a gas on a solid, the plot of
165. The correct difference between first-and x
log vs log p is linear with slope equal to (n being
second-order reactions is that m
(a) a first-order reaction can be catalysed; a second-order a whole number) [CBSE-AIPMT 2006, 1994]
reaction cannot be catalysed 1
(a) k (b) log k (c) n (d)
(b) the half-life of a first-order reaction does not depend on n
[ A]0 the half-life of a second-order reaction does depend
173. The Langmuir adsorption isotherm is deduced by
on [ A]0
using the assumption that [CBSE-AIPMT 2007]
(c) the rate of a first-order reaction does not depend on
(a) the adsorption takes place in multilayers
reactant concentrations; the rate of a second-order
reaction does depend on reactant concentrations (b) the adsorption sites are equivalent in their ability to adsorb
the particles
(d) the rate of a first-order reaction does depend on reactant
concentrations; the rate of a second-order reaction does (c) the heat of adsorption varies with coverage
not depend on reactant concentrations (d) the adsorbed molecules interact with each other

166. For an endothermic reaction, energy of activation 174. Which one of the following statements is incorrect
is Ea and enthalpy of reaction is DH (both of these about enzyme catalysis? [CBSE-AIPMT 2012]
in kJ/mol). Minimum value of Ea will be (a) Enzymes are mostly proteinous in nature
[CBSE-AIPMT 2010] (b) Enzyme action is specific
(a) less than DH (b) equal to DH (c) Enzymes are denaturated by UV-rays and at high
temperature
(c) more than DH (d) equal to zero
(d) Enzymes are least reactive at optimum temperature
MODULE 3
www.jeebooks.in
The NEET Edge ~ Chemistry 427

175. Which property of colloidal solution is independent 182. Considering Ellingham diagram, which of the
of charge on the colloidal particles? following metals can be used to reduce alumina?
[CBSE-AIPMT 2015, 2014] [NEET 2018]
(a) Coagulation (b) Electrophoresis (a) Mg (b) Zn (c) Fe (d) Cu
(c) Electroosmosis (d) Tyndall effect
183. Aluminium is extracted from alumina (Al2O3 ) by
176. On which of the following properties does the electrolysis of a molten mixture of [CBSE-AIPMT 2012]
coagulating power of an ion depend? [NEET 2018]
(a) Al 2O 3 + HF + NaAlF4 (b) Al 2O 3 + CaF2 + NaAlF4
(a) Both magnitude and sign of the charge on the ion
(c) Al 2O 3 + Na 3 AlF6 + CaF2 (d) Al 2O 3 + KF + Na 3 AlF6
(b) Size of the ion alone
(c) The magnitude of the charge on the ion alone 184. In the extraction of copper from its sulphide ore,
(d) The sign of charge on the ion alone the metal finally obtained by the reduction of
cuprous oxide with [CBSE-AIPMT 2015]
177. The coagulation values in millimoles per litre of
the electrolytes used for the coagulation of As 2S3 (a) iron (II) sulphide (b) carbon monoxide
are given below : (c) copper (I) sulphide (d) sulphur dioxide
I. (NaCl) = 52 II. ( BaCl2) = 0.69 185. Match items of Column I with the items of Column II
III. ( MgSO4 ) = 0.22 and assign the correct code. [NEET 2016, Phase I]

The correct order of their coagulating power is Column I Column II


[NEET 2016, Phase II]
A. Cyanide process 1. Ultrapure Ge
(a) I > II > III (b) II > I > III (c) III > II > I (d) III > I > II
B. Froth floatation process 2. Dressing of ZnS
C. Electrolytic reduction 3. Extraction of Al
20. General Principles and Process D. Zone refining 4. Extraction of Au
of Isolation of Elements 5. Purification of Ni

178. Sulphide ores of metals are usually concentrated by Codes


froth floatation process. Which one of the following A B C D
sulphide ores offers an exception and is concentrated (a) 2 3 1 5
by chemical leaching? [CBSE-AIPMT 2007] (b) 1 2 3 4
(a) Argentite (b) Galena (c) 3 4 5 1
(c) Copper pyrite (d) Sphalerite (d) 4 2 3 1
179. Which of the following statements, about the
advantage of roasting of sulphide ore before
reduction is not true? [CBSE-AIPMT 2007]
21. p-block Elements
(a) Carbon and hydrogen are suitable reducing agents for 186. The correct order of N-compounds in its decreasing
metal sulphides order of oxidation states is [NEET 2018]
(b) The D f G° of the sulphide is greater than those for CS2 and
(a) HNO 3 , NH4Cl, NO, N2 (b) HNO 3 , NO, NH4Cl, N2
H2S
(c) The D f G° is negative for roasting of sulphide ore to oxide (c) HNO 3 , NO, N2 , NH4Cl (d) NH4Cl, N2 , NO, HNO 3
(d) Roasting of the sulphide to the oxide is thermodynamically 187. Which is the correct statement for the given acids?
feasible [NEET 2016, Phase I]

180. Roasting of sulphides gives the gas X as a (a) Phosphinic acid is a monoprotic acid while phosphonic
by-product. This is a colourless gas with choking acid is a diprotic acid
smell of burnt sulphur and causes great damage to (b) Phosphinic acid is a diprotic acid while phosphonic acid is
respiratory organs as a result of acid rain. Its a monoprotic acid
aqueous solution is acidic acts as a reducing agent (c) Both are triprotic acids
and its acid has never been insolated. The gas X is (d) Both are diprotic acids
[NEET 2013] 188. Strong reducing behaviour of H 3 PO 2 is due to
(a) H2S (b) SO 2 (c) CO 2 (d) SO 3 [CBSE-AIPMT 2015]
181. Extraction of gold and silver involves leaching with (a) presence of one ¾ OH group and two P ¾H bonds
CN - ion. Silver is later recovered by [NEET 2017] (b) high electron gain enthalpy of phosphorus
(a) liquation (b) distillation (c) high oxidation state of phosphorus
(c) zone refining (d) displacement with Zn (d) presence of two —OH groups and one P ¾ H bond

MODULE 3
www.jeebooks.in
428 NEET Test Drive

189. Which of the following statements is not valid for (a) The electronegativity of fluorine is much higher than for
oxoacids of phosphorus? [CBSE-AIPMT 2012] other elements in the group
(a) Orthophosphoric acid is used in the manufacture of triple (b) There is strong hydrogen bonding between HF molecules
superphosphate (c) The bond energy of HF molecules is greater than in other
(b Hypophosphorus acid is a diprotic acid hydrogen halides
(c) All oxoacids contain tetrahedral four coordinated (d) The effect of nuclear shielding is much reduced in fluorine
phosphorus which polarises the HF molecule
(d) All oxoacids contain at least one P ==O unit and one 196. When Cl 2 gas reacts with hot and concentrated
P ¾ OH group
sodium hydroxide solution, the oxidation number
190. The formation of the oxide ion O 2- ( g), from oxygen of chlorine changes from [CBSE-AIPMT 2012]
atom requires first an exothermic and then an (a) 0 to +1and 0 to -5
endothermic step as shown below : (b) 0 to -1and 0 to +5
O( g)+ e- —® O-( g); D f H ° = -141 kJ mol-1 (c) 0 to -1and 0 to +3
O-( g) + e - —® O2-( g) ; D f H ° = +780 kJ mol-1 (d) 0 to +1and 0 to -3

Thus, process of formation of O2 - in gas phase is 197. Match the interhalogen compounds of Column I
unfavourable even though O2 - is isoelectronic with with the geometry in Column II and assign the
neon. It is due to the fact that [CBSE-AIPMT 2015]
correct code. [NEET 2017]

(a) electron repulsion outweighs the stability gained by Column I Column II


achieving noble gas configuration
A. XX ¢ 1. T- shape
(b) O - ion has comparatively smaller size than oxygen atom
B. XX ¢3 2. Pentagonal bipyramidal
(c) oxygen is more electronegative
(d) addition of electron in oxygen result in large size of the ion C. XX ¢5 3. Linear
D. XX ¢7 4. Square pyramidal
191. In which pair of ions both the species contain S ¾ S
bond? [NEET 2017] 5. Tetrahedral

(a) S2O 27 - , S2O 23 - (b) S4O 26 - , S2O 23 - Codes


(c) S2O 27 - , S2O 28 - (d) S4O 26 - , S2O 27 - A B C D
(a) 3 4 1 2
192. Hot concentrated sulphuric acid is a moderately (b) 3 1 4 2
strong oxidising agent. Which of the following
(c) 5 4 3 2
reaction does not show oxidising behaviour?
(d) 4 3 2 1
[NEET 2016, Phase II]
(a) Cu + 2H2SO 4 ¾® CuSO 4 + SO 2 + 2H2O 198. Among the following, which one is a wrong
(b) 3S + 2H2SO 4 ¾® 3SO 2 + 2H2O statement? [NEET 2016, Phase II]
(c) C + 2H2SO 4 ¾® CO 2 + 2SO 2 + 2H2O (a) PH5 and BiCl 5 do not exist
(d) CaF2 + H2SO 4 ¾® CaSO 4 + 2HF (b) pp-dp bonds are present in SO 2
193. Which of the following statements is not true for (c) SeF4 and CH4 have same shape
halogens? [NEET 2018] (d) I+3 has bent geometry
(a) All but fluorine show positive oxidation states 199. Match the compounds given in Column I with the
(b) All are oxidising agents hybridisation and shape given in Column II and
(c) All form monobasic oxyacids mark the correct option. [NEET 2016, Phase I]
(d) Chlorine has the highest electron-gain enthalpy
Column I Column II
194. Which one of the following orders is not in A. XeF 6 1. Distorted octahedral
accordance with the property stated against it?
[CBSE-AIPMT 2006] B. XeO 3 2. Square planar
(a) F2 > Cl 2 > Br2 > I2 (oxidising power) C. XeOF 4 3. Pyramidal
(b) Hl > HBr > HCl > HF (acidic property in water) D. XeF 4 4. Square pyramidal
(c) F2 > Cl 2 > Br2 > I 2 (electronegativity)
(d) F2 > Cl 2 > Br2 > I 2 (bond dissociation energy) Codes
A B C D
195. The variation of the boiling point of the hydrogen (a) 1 2 4 3
halides is in the order HF > HI > HBr > HCl. What (b) 4 3 1 2
explains the higher boiling point of hydrogen (c) 4 1 2 3
fluoride? [CBSE-AIPMT 2015]
(d) 1 3 4 2
MODULE 3
www.jeebooks.in
The NEET Edge ~ Chemistry 429

207. In acidic medium, H 2 O 2 changes Cr2O72- to CrO5


22. d and f-block Elements which has two (—O—O—) bonds. Oxidation state
200. Four successive members of the first row transition of Cr in CrO 5 is [CBSE-AIPMT 2014]
elements are listed below with their atomic numbers. (a) +5 (b) +3
Which one of them is expected to have the highest (c) +6 (d) -10
third ionisation enthalpy? [CBSE-AIPMT 2005]
208. Which one of the following ions exhibits
(a) Vanadium (Z = 23) (b) Chromium (Z = 24) d-d transition and paramagnetism as well?
(c) Iron (Z = 26) (d) Manganese (Z = 25) [NEET 2018]
201. Match the metal ions given in Column I with the (a) MnO -4 (b) Cr2O 2-
7 (c) CrO 2-
4 (d) MnO 2-
4
spin magnetic moments of the ions given in
Column II and assign the correct code. [NEET 2018] 209. The reaction of aqueous KMnO4 with H 2 O 2 in acidic
conditions gives [CBSE-AIPMT 2014]
Column I Column II (a) Mn4 + and O 2 (b) Mn2 + and O 2
A. Co 3+ 1. 8 BM (c) Mn2 + and O 3 (d) Mn4 + and MnO 2
B. Cr 3+ 2. 35 BM
3+
210. The electronic configurations of Eu (at. no. = 63),
C. Fe 3. 3 BM
Gd (at. no. = 64) and Tb (at. no. = 65) are
D. Ni 2+ 4. 24 BM [NEET 2016, Phase I]
5. 15 BM (a) [Xe ] 4f 6 5d 1 6s 2 , [Xe ] 4f 7 5d 1 6s 2 and [Xe ] 4f 9 6s 2
Codes (b) [Xe ] 4f 6 5d 1 6s 2 , [Xe ] 4f 7 5d 1 6s 2 and [Xe ] 4f 8 5d 1 6s 2
A B C D (c) [Xe ] 4f 7 6s 2 , [Xe ] 4f 7 5d 1 6s 2 and [Xe ] 4f 9 6s 2
(a) 4 1 2 3 (d) [Xe ] 4f 7 6s 2 , [Xe ] 4f 8 6s 2 and [Xe ] 4f 8 5d 1 6s 2
(b) 1 2 3 4
(c) 4 5 2 1 211. Because of lanthanoid contraction, which of the
following pairs of elements have nearly same
(d) 3 5 1 2
atomic radii? (Numbers in the parenthesis are
202. Magnetic moment 2.84 BM is given by atomic numbers). [CBSE-AIPMT 2015]
(At. no. Ni = 28, Ti = 22, Cr = 24, Co = 27) (a) Ti (22) and Zr (40)
CBSE-AIPMT 2015, 2014]
(b) Zr (40) and Nb (41)
(a) Ni 2+ (b) Ti 3+ (c) Cr 3+ (d) Co 2+ (c) Zr (40) and Hf (72)
203. The d-electron configurations of Cr , Mn2+ Fe2+ 2+
(d) Zr (40) and Ta (73)
and Co2+ are d 4 , d5 , d 6 and d7 respectively. Which 212. Identify the incorrect statement among the
one of the following will exhibit minimum
following. [CBSE-AIPMT 2007]
paramagnetic behaviour? [CBSE-AIPMT 2011]
(a) There is a decrease in the radii of the atoms or ions as one
(At. no. Cr = 24, Mn = 25 , Fe = 26, Co = 27) proceeds from La or Lu
(a) [Fe(H2O)6 ]2+ (b) [Co(H2O)6 ]2+ (b) Lanthanide contraction is the accumulation of successive
2+
(c) [Cr(H2O)6 ] (d) [Mn(H2O)6 ]2+ shrinkages
(c) As a result of lanthanide contraction, the properties of 4 d
204. Out of TiF62- , CoF63 - , Cu2Cl2 and NiCl2-
4 (at. no. Z of series of the transition elements have no similarities with
Ti = 22 , Co = 27, Cu = 29, Ni = 28), the colourless the 5 d series of elements
species are [CBSE-AIPMT 2009]
(d) Shielding power of 4f electrons is quite weak
(a) TiF62 - and CoF63 - (b) Cu2Cl 2 and NiCl 2-
4 213. Which of the following ions will exhibit colour in
(c) TiF62 - and Cu2Cl 2 (d) CoF63 - and NiCl 24 - aqueous solutions? [CBSE-AIPMT 2010]
205. HgCl2 and I 2 both when dissolved in water (a) La 3+ (Z = 57 ) (b) Ti 3 + (Z = 22 )
containing I - ions the pair of species formed is (c) Lu3 + (Z = 71) (d) Sc 3 + (Z = 21)
[NEET 2017]
214. The reason for greater range of oxidation states in
(a) HgI2 , I-3 (b) HgI2 , I- (c) HgI24 - , I3- (d) Hg 2I2 , I-
actinoids is attributed to [NEET 2017]
206. Which one of the following statements is correct (a) the radioactive nature of actinoids
when SO 2 is passed through acidified K 2Cr 2O7 (b) actinoid contraction
solution? [NEET 2016, Phase I] (c) 5 f, 6d and 7s levels having comparable energies
(a) The solution is decolourised (b) SO 2 is reduced (d) 4f and 5d levels being close in energies
(c) Green Cr2 (SO 4 )3 is formed (d) The solution turns blue

MODULE 3
www.jeebooks.in
430 NEET Test Drive

215. More number of oxidation states are exhibited by 225. Which of the following coordination compounds
the actinides than by the lanthanides. The main would exhibit optical isomerism? [CBSE-AIPMT 2004]
reason for this is [CBSE-AIPMT 2006, 2005] (a) Pentaamminenitrocobalt (III) iodide
(a) more energy difference between 5f and 6d-orbitals than (b) Diamminedichloroplatinum (II)
that between 4f and 5d-orbitals
(c) trans-dicyanobis (ethylenediamine) chromium (III) chloride
(b) lesser energy difference between 5f and 6d-orbitals than
that between 4f and 5d-orbitals (d) tris-(ethylenediamine) cobalt (III) bromide
(c) greater metallic character of the lanthanides than that of 226. Which one of the following is an outer orbital
the corresponding actinides complex and exhibits paramagnetic behaviour?
(d) more active nature of the actinides [CBSE-AIPMT 2012]
(a) [Ni(NH3 )6 ]2+ (b) [Zn(NH3 )6 ]2+
23. Coordination Compounds (c) [Cr(NH3 )6 ]3+ (d) [Co(NH3 )6 ]3+
216. The correct order of the stoichiometrics of AgCl 227. Which of the following complexes exhibits the
formed when AgNO3 in excess is treated with the highest paramagnetic behaviour?
complexes CoCl3 × 6NH3 , CoCl3 × 5NH3 , CoCl3 × 4NH3 (where, gly = glycine, en = ethylenediamine and
respectively is [NEET 2017] bpy = bipyridyl moities)
(a) 1 AgCl, 3 AgCl, 2 AgCl (b) 3 AgCl, 1 AgCl, 2 AgCl (At. no. of Ti = 22, V = 23, Fe = 26, Co = 27)
(c) 3 AgCl, 2 AgCl, 1 AgCl (d) 2 AgCl, 3 AgCl, 1 AgCl [CBSE-AIPMT 2008]

217. The complex [Co(NH3 ) 6 ][Cr(CN)6 ] and (a) [V(gly)2 (OH)2 (NH3 )2 ]+ (b) [Fe(en)(py)(NH3 )2 ]2+
[Cr(NH3 )6 ][Co(CN)6 ] are the examples of which (c) [Co(ox)2 (OH)2 ]- (d) [Ti(NH3 )6 ]3+
type of isomerism? [CBSE-AIPMT 2011] 228. Which of the following complex ions is expected to
(a) Ionisation isomerism (b) Coordination isomerism absorb visible light?
(c) Geometrical isomerism (d) Linkage isomerism (At. no. of Zn = 30, Sc = 21, Ti = 22 , Cr = 24)
218. The complex, [Pt(Py)(NH3 )BrCl] will have how [CBSE-AIPMT 2009]

many geometrical isomers? [CBSE-AIPMT 2011] (a) [Sc(H2O)3 (NH3 )3 ]3+ (b) [Ti(en)2 (NH3 )2 ]4+
(a) 4 (b) 0 (c) 2 (d) 3 (c) [Cr(NH3 )6 ]3+ (d) [Zn(NH3 )6 ]2+
219. Number of possible isomers for the complex 229. [Cr(H 2O)6 ]Cl3 (at. no. of Cr = 24) has a magnetic
[Co(en)2 Cl2 ]Cl will be (en = ethylenediamine) moment of 3.83 BM, the correct distribution of 3d
[CBSE-AIPMT 2015] electrons in the chromium of the complex is
(a) 2 (b) 1 (c) 3 (d) 4 [CBSE-AIPMT 2006]

220. The geometry and magnetic behaviour of the (a) 3 d 1xy , 3 d 1yz , 3 d 1 2 (b) 3 d , 3 d 1 2 , 3 d 1xz
z ( x – y2)
2 z
complex [Ni(CO)4 ] are [NEET 2018] (c) 3 d xy , 3 d , 3 d 1yz (d) 3 d 1xy , 3 d 1yz , 3 d 1zx
( x2 - y 2 )
(a) square planar geometry and paramagnetic
(b) tetrahedral geometry and diamagnetic 230. Among the following complexes, the one which
(c) square planar geometry and diamagnetic shows zero crystal field stabilisation energy
(d) tetrahedral geometry and paramagnetic (CFSE) is [CBSE-AIPMT 2014]
221. The hybridisation involved in complex [Ni(CN)4 ]2 - (a) [Mn(H2O)6 ]3+ (b) [Fe(H2O)6 ]3+
is (atomic number of Ni = 28) [CBSE-AIPMT 2015] (c) [Co(H2O)6 ]2+ (d) [Co(H2O)6 ]3+
(a) dsp2 (b) sp3 (c) d 2 sp2 (d) d 2sp3
231. Crystal field stabilisation energy for high spin d 4
222. Which of the following does not show optical octahedral complex is [CBSE-AIPMT 2010]
isomerism? (en = ethylenediamine)[CBSE-AIPMT 2009] (a) -1.8Do (b) - 1.6Do + p
+ 0
(a) [Co(en)2 Cl 2 ] (b) [Co(NH3 )3 Cl 3 ]
+
(c) -1.2 Do (d) - 0.6Do
(c) [Co(en)Cl 2 (NH3 )2 ] (d) [Co(en)3 ]3+
232. Correct increasing order for the wavelengths of
223. Which of the following will give a pair of enantiomers? absorption in the visible region for the complexes
(en = NH 2CH 2CH 2NH 2 ) [CBSE-AIPMT 2007] of CO 3+ is [NEET 2017]
(a) [Cr(NH3 )6 ][Co(CN)6 ] (b) [Co(en)2 Cl 2 ]Cl (a) [Co(en)3 ]3+ , [Co(NH3 )6 ]3+ , [Co(H2O)6 ]3+
(c) [Pt(NH3 )4 ][PtCl 6 ] (d) [Co(NH3 )4 Cl 2 ]NO 2
(b) [Co(H2O)6 ]3+ , [Co(en)3 ]3+ , [Co(NH3 )6 ]3+
224. Which of the following pairs of d-orbitals will have (c) [Co(H2O)6 ]3+ , [Co(NH3 )6 ]3+ , [Co(en)3 ]3+
electron density along the axis? [NEET 2016, Phase II]
(d) [Co(NH3 )6 ]3+ , [Co(en)3 ]3+ , [Co(H2O)6 ]3+
(a) d 2 , d xz (b) d xz , d yz (c) d 2 , d (d) d xy , d
z z x2 - y 2 x2 - y 2

MODULE 3
www.jeebooks.in
The NEET Edge ~ Chemistry 431

233. Iron carbonyl, Fe(CO)5 is [NEET 2018] 239. What products are formed when the following
(a) trinuclear (b) mononuclear compound is treated with Br 2 in the presence of
(c) tetranuclear (d) dinuclear FeBr3 ? [CBSE-AIPMT 2014]
CH3
234. Which of the following has longest C¾ O bond
length? (Free C¾ O bond length in CO is 1.128 Å.)
[NEET 2016, Phase I]
(a) [Co(CO)4 ]- (b) [Fe(CO)4 ]2 - CH3
(c) [Mn(CO)6 ]+ (d) Ni(CO)4 CH3 CH3
Br
24. Haloalkanes and Haloarenes (a) and

CH3 CH3
235. Which of the following reaction(s) can be used for
Br
the preparation of alkyl halides? [CBSE-AIPMT 2015]
CH3 CH3
Anhy. ZnCl 2
I. CH3CH 2OH + HCl ¾¾¾¾¾® Br Br
II. CH3CH 2OH + HCl ¾® (b) and
III. (CH3 )3 COH + HCl ¾® CH3 CH3
Anhy. ZnCl 2
IV. (CH3 )2CHOH + HCl ¾¾¾¾¾® CH3 CH3
(a) I, III and IV (b) I and II Br
(c) Only IV (d) III and IV and
(c)
236. The reaction of C 6 H 5 CH == CHCH 3 with HBr CH3 CH3
produces [CBSE-AIPMT 2015]
Br
(a) C 6H5C HCH2CH3 (b) C 6H5CH2 C HCH3 CH3 CH3
½ ½
Br Br
(d) and
(c) C 6H5CH2CH2CH2Br (d) CH CHCH3
CH3 Br CH3
Br
240. Trichloroacetaldehyde, CCl 3 CHO reacts with
Br chlorobenzene in the presence of sulphuric acid
and produces. [CBSE-AIPMT 2009]
237. Identity Z in the sequence of reactions, Cl
[CBSE-AIPMT 2014]
HBr/H2O 2 C2H5 ONa (a) Cl C Cl
CH3CH 2CH == CH 2 ¾¾¾¾¾® Y ¾¾¾¾¾® Z
CH2Cl
(a) CH3 ¾ (CH2 )3 ¾ O ¾ CH2CH3
Cl
(b) (CH3 )2 CH2 ¾ O ¾ CH2CH3
(c) CH3 (CH2 )4 ¾ O ¾ CH3
(d) CH3CH2 ¾ CH(CH3 ) ¾O ¾ CH2CH3
238. In a S N2 substitution reaction of the type
DMF (b) Cl C Cl
R — Br + Cl– ¾¾® R — Cl + Br - .
Which one of the following has the highest relative H
rate? [CBSE-AIPMT 2008] OH
(a) CH3 — CH2 — CH2Br (b) CH3 — CH —CH2Br
½ (c) Cl C Cl
CH3 CH3
½ Cl
(c) CH3 —C — CH2Br (d) CH3CH2Br
½ (d) Cl CH Cl
CH3
CCl3

MODULE 3
www.jeebooks.in
432 NEET Test Drive

The major products A and C are respectively


25. Alcohols, Phenols and Ethers CH3 CH3
½ ½
241. In the following reaction, (a) CH3 ¾C == CH ¾CH3 and CH3 —C — CH2 — CH3
CH3 ½
½ Br
H 2 O/H +
H3C ¾ C ¾ CH == CH 2 ¾¾¾® CH3 CH3
½ ½ ½
CH3 (b) CH3 ¾C == CH ¾CH3 and CH3 —CH —CH — CH3
½
A + B Br
Major product Minor product [CBSE-AIPMT 2012] CH3 CH3
CH3 ½ ½
CH3 (c) CH2 ¾C == CH2 ¾CH3 and CH3 —C —CH2 — CH3
  ½
(a) H3C—C—CH—CH3 (b) CH2—C—CH2—CH3 Br
    CH3 CH3
OH CH3 OH CH3 ½ ½
(d) CH2 == C — CH2 — CH3 and CH2 — CH — CH2 — CH3
CH3 CH3 ½
  Br
(c) H3C—C—CH—CH3 (d) H3C—C—CH2—CH2 245. Which of the following will not form a yellow
    precipitate on heating with an alkaline solution of
CH3OH CH3 OH
iodine? [CBSE-AIPMT 2004]
242. Which of the following is not the product of (a) CH3CH(OH)CH3 (b) CH3CH2CH(OH)CH3
dehydration of ? (c) CH3OH (d) CH3CH2OH

OH [CBSE-AIPMT 2015]
246. Compound A, C8H10O, is found to react with NaOI
(produced by reacting Y with NaOH) and yields a
yellow precipitate with characteristic smell.
(a) (b) A and Y are respectively [NEET 2018]

(a) CH CH3 and I2


(c) (d)
OH
(b) CH2 CH2 OH and I2
243. Consider the following reaction,
3PBr KOH Alc.
Ethanol ¾¾® X ¾¾¾® Y (c) H3C CH2 OH and I2
(i) H SO , room temperature
¾¾¾¾¾¾¾¾¾¾®
2 4
Z CH3
(ii) H2O, heat

The product Z is [CBSE-AIPMT 2009] (d) CH3 OH and I2


(a) CH2 ==CH2 (b) CH3CH2OCH2CH3
(c) CH3CH2OSO 3H (d) CH3CH2OH 247. Which one is the most acidic compound? [NEET 2017]
244. In the following reactions, [CBSE-AIPMT 2011]
OH
OH OH OH
CH3 O2N NO2
½ H+/ Heat (c) (d)
I. CH3 —CH — CH — CH3 ¾¾¾¾® (a) (b)
½
OH NO2 NO2
CH3
A + B
Major Minor 248. Given are cyclohexanol (I), acetic acid (II),
product product
2, 4, 6-trinitrophenol (III) and phenol (IV). In
II. A ¾¾¾¾¾¾¾¾¾®
HBr, dark
C + D these, the order of decreasing acidic character will
in absence of peroxide æ Major ö
ç ÷
æ Minor ö
ç ÷
be [CBSE-AIPMT 2010]
è product ø è product ø
(a) III > II > IV > I (b) II > III > I > IV
(c) II > III > IV > I (d) III > IV > II > I

MODULE 3
www.jeebooks.in
The NEET Edge ~ Chemistry 433

249. Consider the following reaction, 253. The reaction,


Zn- dust CH3Cl Alk. KMnO4 CH3
Phenol ¾¾® X ¾¾¾¾® Y ¾¾¾® Z ½
Anhy. AlCl3
CH3 ¾ C ¾ ONa + CH3CH 2Cl ¾¾¾®
- NaCl
The product Z is [CBSE-AIPMT 2009] ½
CH3
(a) toluene (b) benzaldehyde CH3
(c) benzoic acid (d) benzene ½
250. In the reaction, [NEET 2018] CH3 ¾ C ¾ O — CH 2 — CH3
OH O–Na+ ½
CH3
CHO is called [CBSE-AIPMT 2015]
+ CHCl3+NaOH
(a) Williamson synthesis
(b) Williamson continuous etherification process
The electrophile involved is (c) Etard reaction
– + (d) Gatterman-Koch reaction
(a) dichloromethyl anion (CHCl 2 ) (b) formyl cation (CHO)
+
(c) dichloromethyl cation (CHCl 2 ) (d) dichlorocarbene ( ·· CCl 2 )
251. Identify the major products P , Q and R in the
26. Aldehyde, Ketone and
following sequence of reactions : Carboxylic Acid
Anhy. (i) O2 254. Predict the correct intermediate and product in the
+ CH3CH2CH2Cl P (ii) H3O+lD
Q+R
AlCl3 following reaction. [NEET 2017]
H 2O, H 2SO4
H3C ¾ C ºº CH ¾ ¾ ¾ ¾¾®
R HgSO4
P Q
CH(CH3)2 OH Intermediate ¾® Product
(a) CH3CH(OH)CH3 (A ) ( B)

(a) A =H3C ¾ C == CH2 ; B =H3C ¾ C ¾CH3


CH2CH2CH3 CHO COOH ½ ½½
SO 4 O
(b) A = H3C ¾ C == CH2 ; B =H3C ¾ C == CH2
(b) ½ ½
OH SO 4
CH2CH2CH3 CHO (c) A = H3C ¾ C ¾CH3 ; B =H3C ¾ C ºº CH
½½
O
(c) CH3CH2 OH (d) A = H3C ¾ C == CH2 ; B =H3C ¾ C ¾CH3
½ ½½
OH OH O

CH(CH3)2 255. A single compound of the structure is


(d) CH3 CH3
CH3 CO CH3
OHC C C
C H C O
252. Among the following ethers which one will produce H2 H2
methyl alcohol on treatment with hot concentrated
Obtainable from ozonolysis of which of the
HI? [CBSE-AIPMT 2013]
following cyclic compounds? [CBSE-AIPMT 2015]
(a) CH3 ¾ CH2 ¾ CH2 ¾ CH2 ¾O ¾ CH3
H3C H3C
(b) CH3 ¾ CH2 ¾ C H ¾ O ¾ CH3
(a) (b)
½ CH3
CH3 H3C
CH3 H3C
½ CH3
(c) CH3 ¾C ¾ O ¾CH3
½ (c) (d)
CH3
CH3
(d) CH3 ¾CH ¾CH2 ¾O ¾CH3 CH3
½
CH3
MODULE 3
www.jeebooks.in
434 NEET Test Drive

256. Reaction by which benzaldehyde cannot be 260. Acetophenone when reacted with a base, C2H5ONa,
prepared? [NEET 2013] yields a stable compound which has the structure.
CH3 —C CH—C—
(a) + CrO 2Cl 2 and CS2 followed by H3O + (a) [CBSE-AIPMT 2008]
CH3 O
COCl
(b) + H2 in presence of Pd-BaSO 4
—CH—CH2—C—
(c) + CO + HCl in presence of anhy. AlCl 3 (b)
CH3 O

COOH CH3 CH3


(d) + Zn /Hg and conc. HCl
—C—–C—–
(c)
257. Consider the reactions, OH OH
Cu [Ag(NH3)2]+
X 573 K A –OH, D
Silver mirror observed —CH—CH—
(C2H6O) (d)
–OH, D OH OH
Y
O
261. Predict the products in the given reaction,
NH2 NH C NH2
CHO
Z 
Identify A, X , Y and Z [NEET 2017] 50% KOH
(a) A-methoxymethane, X-ethanoic acid, Y-acetate ion,
Z-hydrazine
Cl [CBSE-AIPMT 2012]
(b) A-methoxymethane, X-ethanol, Y-ethanoic acid,
Z-semicarbazide CH2OH CH2COO–
(c) A-ethanol, X-acetaldelyde, Y-but-2-enal, Z-semicarbazone
(a)
(d) A-ethanol, X-acetaldehyde, Y-butanone, Z-hydrazone
+
258. The major organic product formed from the  
following reaction [CBSE-AIPMT 2005] Cl Cl

O CH2OH OH
(i) CH3NH2
... is
(ii) LiAlH4 (b)
(iii) H2O +
CH3  
(a) (b) OH OH
NHCH3
OH
CH2OH COO–
ONHCH3 NHCH3
(c)
(c) (d) +
OH OH
 
259. Of the following which is the product formed when Cl Cl
cyclohexanone undergoes aldol condensation CH2OH COO–
followed by heating? [NEET 2017]
(d)
O +
 
(a) (b) OH OH
262. A carbonyl compound reacts with hydrogen cyanide
OH OH to form cyanohydrin which on hydrolysis forms a
racemic mixture of a-hydroxy acid. The carbonyl
(c) (d) compound is [CBSE AIPMT 2006]
(a) acetaldehyde (b) acetone
O O O (c) diethyl ketone (d) formaldehyde

MODULE 3
www.jeebooks.in
The NEET Edge ~ Chemistry 435

263. The correct order of strengths of the carboxylic 267. Method by which aniline cannot be prepared is
acids [CBSE-AIPMT 2015]
COOH COOH COOH (a) hydrolysis phenyl isocyanide with acidic solution
(b) degradation of benzamide with bromine in alkaline solution
O
is
(c) reduction of nitrobenzene with H2 /Pd in ethanol
O (d) potassium salt of phthalimide treated with chlorobenzene
I II III
followed by the hydrolysis with aqueous NaOH solution
[NEET 2016, Phase II]
(a) I > II > III (b) II > III > I 268. Acetamide is treated with the following reagents
(c) III > II > I (d) II > I > III separately. Which one of these would yield methyl
264. In a set of reactions, ethyl benzene yielded a amine? [CBSE-AIPMT 2010]

product D. [CBSE-AIPMT 2010] (a) NaOH / Br2 (b) Sodalime


CH2CH3 (c) Hot conc. H2SO 4 (d) PCl 5
KMnO4 Br2 C2H5OH
B C D 269. The correct statement regarding the basicity of
KOH FeCl3 H+
arylamines is [NEET 2016, Phase I]
D would be (a) arylamines are generally more basic than alkylamines
Br because the nitrogen lone-pair electrons are not
delocalised by interaction with the aromatic ring
CH2 CH—COOC2H5 p-electron system
(a) (b) (b) arylamines are generally more basic than alkylamines
Br
Br because of aryl group
(c) arylamines are generally more basic than alkylamines,
CH2COOC2H5 because the nitrogen atom in arylamines is sp-hybridised
COOH COOC2H5 (d) arylamines are generally less basic than alkylamines
because the nitrogen lone-pair electrons are delocalised
by interaction with the aromatic ring p-electron system.
(c) (d)
270. Identify A and predict the type of reactions
OC2H5 Br
OCH3
265. In a set of reactions, acetic acid yielded a product D.
SOCl2 Benzene HCN HOH NaNH2
CH3COOH A Anhy. AlCl B C D A
3
Br [NEET 2017]
The structure of D would be [CBSE-AIPMT 2005]
OCH3
OH COOH
| |
C—COOH CH2—C—CH3 (a) and substitution reaction
(a) | (b)
|
CH3 NH2
OH
CN
OH OCH3
| |
CH2—C—CH3 C—CH3
(c) (d) NH2
| |
(b) and elimination addition reaction
CN OH

OCH3
27. Organic Compounds
Br
Containing Nitrogen (c) and cine substitution reaction
266. Which of the following reactions is appropriate for
converting acetamide to methanamine? [NEET 2017] OCH3
(a) Carbylamine reaction
(b) Hofmann Bromamide reaction
(d) and cine substitution reaction
(c) Stephen’s reaction
(d) Gabriel’s phthalimide synthesis

MODULE 3
www.jeebooks.in
436 NEET Test Drive

271. Nitration of aniline in strong acidic medium also The structure of C would be [CBSE-AIPMT 2008]
gives m-nitroaniline because [NEET 2018]
(a) in absence of substituents nitro group always goes to —N N—CH2 —N—
(a)
m-position
(b) in electrophilic substitution reactions amino group is meta CH3
directive CH3 CH3
(c) in spite of substituents nitro group always goes to only
m-position
—N N—
(d) in acidic (strong) medium aniline is present as anilinium ion (b)
272. In a set of reactions, m-bromobenzoic acid gave a
product D. Identify the product D. [CBSE-AIPMT 2011]
COOH CH3
—NH—NH— —N
(c) CH3
SOCl2 NH3 NaOH
B C Br2
D
CH3
Br —N==N — —N
(d) CH3
COOH NH2
(a) (b)
275. Aniline in a set of reactions yielded a product
NH2 H2
NH2 Br NaNO2 CuCN HNO2
A B C D
HCl Ni
CONH2 SO2NH2
[CBSE-AIPMT 2005]
(c) (d)
The structure of the product D would be
(a) C 6H5CH2NH2
Br Br (b) C 6H5NHCH2CH3
273. In the following reaction, the product (A) is (c) C 6H5NHOH
+ (d) C 6H5CH2OH
N NCl– NH2
276. Aniline when diazotised in cold and then treated
H+
A with dimethyl aniline, gives a coloured product. Its
+
Yellow dye structure would be [CBSE-AIPMT 2004]

[CBSE-AIPMT 2014] (a) CH3NH— N==N— NHCH3

(a) N N—NH
(b) CH3— N==N— NH2
H2N

(b) N N (c) (CH3)2N— N==N—


NH2
(d) (CH3)2N— N==N— —CH3
(c) N N

277. Which one of the following nitro-compounds does


(d) N N NH2 not react with nitrous acid? [NEET 2016, Phase II]
H2
H2 H3C C
274. In a reaction of aniline a coloured products C was (a) H3C C (b) CH NO2
obtained. C NO2 H3C
H2
CH3 CH3
NH2 —N H3C
NaNO2 CH3 (c) H3C C NO2 (d) H3C C
HCl
B C H NO
Cold H3C 2
O
A

MODULE 3
www.jeebooks.in
The NEET Edge ~ Chemistry 437

278. A given nitrogen-containing aromatic compound A 282. Which one of the following statements is not true
reacts with Sn/HCl, followed by HNO 2 to give an regarding (+)-lactose? [CBSE-AIPMT 2011]
unstable compound B. B, on treatment with (a) (+)-lactose is a b -glycoside formed by the union of a
phenol, forms a beautiful coloured compound C molecule of D-(+)-glucose and a molecule of D-(+)-
with the molecular formula C12 H10 N 2 O. The galactose
structure of compound A is [NEET 2016, Phase II] (b) (+)-lactose is a reducing sugar and does not exhibit
mutarotation
NH2 NO2
(c) (+)-lactose, C12H22O11 contains 8 ¾ OH groups
(a) (b)
(d) On hydrolysis (+) lactose gives equal amount of
D-(+)-glucose and D-(+)-galactose
CN CONH2
283. Which of the following compounds can form a
(c) (d)
Zwitter ion?
(a) Benzoic acid (b) Acetanilide
(c) Aniline (d) Glycine

28. Biomolecules 284. During the process of digestion, the proteins


present in food materials are hydrolysed to amino
279. Which one given below is a non-reducing sugar? acids. The two enzymes involved in the process
[NEET 2016, Phase I] Enzyme ( A )
(a) Lactose (b) Glucose (c) Sucrose (d) Maltose
Proteins ¾¾¾¾¾® Polypeptides
Enzyme ( B )
280. The correct corresponding order of names of four ¾¾¾¾¾® Amino acids,
aldoses with configuration given below : are respectively [CBSE-AIPMT 2006]
CHO CHO CHO CHO (a) amylase and maltase
H OH HO H HO H H OH (b) diastase and lipase
H OH H OH HO H HO H
(c) pepsin and trypsin
CH2OH CH2OH CH2OH CH2OH
(d) invertase and zymase
respectively, is [NEET 2016, Phase II] 285. Which one of the following structures represents
(a) L-erythrose, L-threose, L-erythrose, D-threose the peptide chain? [CBSE AIPMT 2004]
(b) D-threose, D-erythrose, L-threose, L-erythrose
H O
(c) L-erythrose, L-threose, D-erythrose, D-threose | ½ ||
(d) D-erythrose, D-threose, L-erythrose, L-threose (a) — N — C — N —C— NH —C — NH —
|| | ½
281. D-(+)-glucose reacts with hydroxyl amine and O H
yields an oxime. The structure of the oxime would H H
| ½ ½ ½ | ½ ½ ½
be [CBSE-AIPMT 2014] (b) — N —C —C—C—C— N — C—C—C—
|| ½ ½ ½ ½ ½ ½
CH NOH CH NOH
O
HO C H H H H H
H C OH | ½ | ½ | ½ | ½
(a) HO C H (b) H C OH (c) — N —C— C — N —C— C — N —C— C — N —C—
½ || ½ || ½ || ½
O O O
HO C H HO C H H O H
½ | ½ ½ || ½ ½ | ½ ½
H C OH H C OH (d) —C— N —C—C— C —N—C—C— N — C —C—C—
½ ½ ½ | ½ ½ || ½ ½
CH2OH CH2OH H O

CH NOH CH NOH 286. Which of the following statements is not correct?


[NEET 2017]
HO C H H C OH
(a) Insulin maintains sugar level in the blood of a human body
(c) HO C H (d) HO C H (b) Ovalbumin is a simple food reserve in egg white
H C OH H C OH (c) Blood proteins thrombin and fibrinogen are involved in
blood clotting
H C OH H C OH
(d) Denaturation makes the proteins more active
CH2OH CH2OH

MODULE 3
www.jeebooks.in
438 NEET Test Drive

287. The correct statement regarding RNA and DNA, CH3


(a) H2C ==C (b) (CH3 )2 C == C(CH3 )2
respectively is [NEET 2016, Phase I] CH3
(a) the sugar component in RNA is ribose and the sugar (c) CH3CH == CH × CH3 (d) CH3CH == CH2
component in DNA is 2¢-deoxyribose
(b) the sugar component in RNA is arabinose and the sugar 292. Which one of the following structures represents
component in DNA is ribose nylon-6,6 polymer? [NEET 2016, Phase II]
(c) the sugar component in RNA is 2¢-deoxyribose and the
sugar component in DNA is arabinose H2 H H2 H
(d) the sugar component in RNA is arabinose and the sugar C C C C
(a)
component in DNA is 2¢-deoxyribose
NH2 CH3
6,6

29. Polymers H2 H H2 H
C C C C
288. Regarding cross-linked or network polymers, which (b)
of the following statements is incorrect? [NEET 2018] NH2 NH3
6,6
(a) Examples are bakelite and melamine
(b) They are formed from bi- and tri-functional monomers H2 H H2 H
H2 H H2 H C C C C
(c) They contain covalent bonds between various linear C C C C
polymer chains (c)
6
(d) They contain strong covalent bonds in their polymer CH2 COOH
NH2 Cl
6
chains
O
289. Which one of the following is an example of a H2 H
thermosetting polymer? [CBSE-AIPMT 2014] C N (CH2)6 NH
(d) C C C
H2 2
(a) ¾( CH2 — C == CH ¾ CH2¾
)n
n
½ O
Cl
293. Which of the following organic compounds
(b) ¾ ( CH2 ¾ CH ¾ )n polymerises to form the polyester dacron?
½ [CBSE-AIPMT 2014]
H Cl H O O (a) Propylene and para —HO —(C 6H4 ) — OH
½ ½ ½½ ½½ (b) Benzoic acid and ethanol
(c) ¾( N ¾ (CH2 )6 ¾ N ¾ C ¾ (CH2 )4 ¾ C ¾)n
(c) Terephthalic acid and ethylene glycol
OH OH
(d) Benzoic acid and para —HO — (C 6H4 ) — OH
CH2 CH2
294. Which of the following structures represents
(d) neoprene polymer? [CBSE-AIPMT 2010]
n CN
|
290. Structures of some common polymers are given. ( CH2 ¾C == CH ¾CH2 —
(a) — )n ( CH2 ¾CH—
(b) — )n
Which one is not correctly presented? |
[CBSE-AIPMT 2009]
Cl

(a) Teflon —
( CF2 — CF2 —)n Cl
|
æ —CH2 — C === CH — CH2 — CH2 — ö ( CH2 ¾CH—
(c) — )n (d) —( CH ¾CH2 —
)n
ç ÷
(b) Neoprene ç ½ ÷ |
ç Cl ÷ C 6H5
è øn

(c) Terylene —OC


( COOCH2—CH2—O—)n
30. Chemistry in Everyday Life
(d) Nylon-6,6 —
[ NH(CH2 )6 NHCO(CH2 )4 — CO—]n 295. Which one of the following is employed as a
291. The monomer of the polymer tranquilizer drug ? [CBSE-AIPMT 2010]
CH3 (a) Promethazine
½ + CH3 (b) Valium
CH 2 ¾ C ¾ CH 2 ¾ C is (c) Naproxen
½ CH3
CH3 (d) Mifepristone
[CBSE-AIPMT 2005]

MODULE 3
www.jeebooks.in
The NEET Edge ~ Chemistry 439

296. Which one of the following is employed as (a) A 0.2% solution of phenol is an antiseptic while 1% solution
antihistamine? [CBSE-AIPMT 2011] acts as a disinfectant
(a) Diphenyl hydramine (b) Norethindrone (b) Chlorine and iodine are used as strong disinfectants
(c) Omeprazole (d) Chloramphenicol (c) Dilute solutions of boric acid and hydrogen, peroxide are
297. Which of the following is an analgesic? strong antiseptics
[NEET 2016, Phase I] (d) Disinfectants harm the living tissues
(a) Penicillin (b) Streptomycin (c) Chloromycetin (d) Novalgin 299. Artificial sweetener which is stable under cold
298. Antiseptics and disinfectants either kill or prevent conditions only is [CBSE-AIPMT 2014]
growth of microorganisms. Identify which of the (a) saccharine (b) sucralose
following is not true. [NEET 2013] (c) aspartame (d) alitame

Answer Sheet
1. (d) 2. (b) 3. (c) 4. (b) 5. (a) 6. (d) 7. (a) 8. (b) 9. (c) 10. (c)
11. (c) 12. (d) 13. (c) 14. (a) 15. (a) 16. (c) 17. (d) 18. (c) 19. (a) 20. (c)
21. (d) 22. (a) 23. (a) 24. (d) 25. (d) 26. (c) 27. (a) 28. (b) 29. (*) 30. (b)
31. (a) 32. (a) 33. (b) 34. (b) 35. (b) 36. (c) 37. (c) 38. (d) 39. (d) 40. (b)
41. (d) 42. (b) 43. (b) 44. (b) 45. (d) 46. (a) 47. (c) 48. (d) 49. (d) 50. (*)
51. (c) 52. (d) 53. (c) 54. (b) 55. (a) 56. (a) 57. (a) 58. (b) 59. (b) 60. (b, d)
61. (c) 62. (c) 63. (b) 64. (c) 65. (b) 66. (c) 67. (b) 68. (d) 69. (b) 70. (c)
71. (d) 72. (a) 73. (c) 74. (b) 75. (b) 76. (c) 77. (b) 78. (d) 79. (c) 80. (c, d)
81. (b) 82. (b) 83. (d) 84. (c) 85. (c) 86. (b) 87. (a) 88. (b) 89. (d) 90. (b)
91. (d) 92. (b) 93. (c) 94. (b) 95. (c) 96. (a) 97. (b) 98. (d) 99. (a) 100. (a)

101. (c) 102. (b) 103. (a) 104. (c) 105. (d) 106. (d) 107. (c) 108. (a) 109 (c) 110. (d)
111. (a) 112. (c) 113. (d) 114. (b) 115. (b) 116. (d) 117. (d) 118. (d) 119 (c) 120. (b)
121. (d) 122. (b) 123. (d) 124. (c) 125. (a) 126. (b) 127. (a) 128. (a) 129. (a) 130. (d)
131. (a, d) 132. (c) 133. (d) 134. (d) 135. (a) 136. (d) 137. (d) 138. (d) 139. (a) 140. (d)
141. (a) 142. (a) 143. (a) 144. (b) 145. (d) 146. (b) 147. (c) 148. (d) 149. (b) 150. (a)
151. (d) 152. (d) 153. (c) 154. (d) 155. (a) 156. (c) 157. (d) 158. (a) 159. (b) 160. (b)
161. (d) 162. (b) 163. (b) 164. (d) 165. (b) 166. (c) 167. (c) 168. (d) 169. (c) 170. (b)
171. (c) 172. (d) 173. (b) 174. (d) 175. (d) 176. (a) 177. (c) 178. (d) 179. (a) 180. (b)
181. (d) 182. (a) 183. (c) 184. (c) 185. (d) 186. (c) 187. (a) 188. (a) 189. (b) 190. (a)
191. (b) 192. (d) 193. (a) 194. (d) 195. (b, c) 196. (b) 197. (b) 198. (c) 199. (d) 200. (d)

201. (c) 202. (a) 203. (b) 204. (c) 205. (c) 206. (c) 207. (c) 208. (d) 209. (b) 210. (c)
211. (c) 212. (c) 213. (b) 214. (c) 215. (b) 216. (c) 217. (b) 218. (d) 219. (c) 220. (b)
221. (a) 222. (b) 223. (b) 224. (c) 225. (d) 226. (a, b) 227. (c) 228. (c) 229. (d) 230. (b)
231. (d) 232. (a) 233. (b) 234. (b) 235. (a) 236. (a) 237. (a) 238. (d) 239. (c) 240. (d)
241. (a) 242. (b) 243. (d) 244. (a) 245. (c) 246. (a) 247. (d) 248. (a) 249. (c) 250. (d)
251. (d) 252. (c) 253. (a) 254. (d) 255. (a) 256. (d) 257. (c) 258. (b) 259. (c) 260. (a)
261. (c) 262. (a) 263. (b) 264. (d) 265. (a) 266. (b) 267. (d) 268. (a) 269. (d) 270. (a)
271. (d) 272. (b) 273. (d) 274. (d) 275. (d) 276. (c) 277. (c) 278. (b) 279. (c) 280. (d)
281. (d) 282. (b) 283. (d) 284. (c) 285. (c) 286. (d) 287. (a) 288. (d) 289. (d) 290. (b)
291. (a) 292. (d) 293. (c) 294. (a) 295. (b) 296. (a) 297. (d) 298. (c) 299. (c)

MODULE 3
www.jeebooks.in

SOLUTIONS
1. Some Basic Principles =
Mass of substance in g (mH2 O ) From Eq. (i)
of Chemistry Molar mass in g mol −1 (MH2 O ) 1 mole of MgCO3 gives
mH2 O = 18 g = 1mol MgO
1. Average atomic mass
−1
A X + A2 X 2 + A3 X 3 [QDensity of water (dH2 O ) = 1 g L ] ∴0.238 mol MgCO3 will give
=A= 1 1
X1 + X 2 + X 3 18 = 0.238 mol MgO
∴ n H2 O = =1
18
where, A1A2 A3 = atomic mass = 0.238 × 40 g
Number of molecules of water
X1X 2 X 3 = percentage = 9.52 g MgO
= 1 × NA
200 × 90 + 199 × 8.0 + 202 × 2.0 Now, practical yield of MgO = 8 g
= (d) 10−3 moles of water
100 8
Number of molecules of water ∴% purity = × 100
= 199.96 ≈ 200 u 9.52
= 10−3 × N A
2. If Avogadro number N A , is changed = 84%
∴ Among the given options.
from 6.022 × 1023 mol − 1 to 6.022 × 1020 Option (c) contains the maximum Alternate Method
mol − 1, this would change the mass of number of water molecules. MgCO3 → MgO + CO2
one mole of carbon. 4. Number of atoms = number of moles 84 g 40 g
84
Q1 mole of carbon has mass = 12 g × NA × atomicity ∴ 8 g MgO will be form from g
5
or 6.022 × 1023 atoms of carbon have = 0.1 × 6.023 × 10 23
×3
of MgCO3 .
mass = 12 g = 1.806 × 10 atoms
23
84 100
∴ % purity = ×
∴6.022 × 1020 atoms of carbon have 5. Given, molarity of solution = 2 5 20
mass Volume of solution = 250 mL = 84%
12
= × 6.022 × 1020 =
250 1
= L 7. Firstly, write the reaction of formic
6.022 × 1023 1000 4 acid and oxalic acid with conc.
= 0.012 g Molar mass of HNO3 H2 SO4 , respectively. Then, find the
3. Number of molecules = Mole × = 1 + 14 + 3 × 16 gaseous products formed and identify
Avogadro’s number (N A ) the remaining gaseous product after
= 63 g mol −1 passing through KOH. Finally,
The number of molecules of water in
QMolarity calculate the total number of moles of
each of the given options is
weight of HNO3 gaseous product.
calculated as : =
Conc.H2SO4
molecular mass of HNO3 HCOOH CO(g)+H2O(l)
(a) 0.00224 L of water vapours at × volume of solution (L) Formic acid Dehydrating
1 atm and 273 K. At STP 2.3 1 mol agent

[1 atm and 273 K], ∴Weight of HNO3 Initial moles


46
= 20 0 0
1 1
Number of moles [with reference = molarity × molecular mass Final moles 0 20 20
to volume] × volume (L) Similarly,
Volume of gas in litres 1
= = 2 × 63 × = 31.5 g
4 COOH Conc.H SO
22.4 2 4
CO(g) + CO2(g)+H2O(l)
0.00224
= It is the weight of 100% HNO3 , but the COOH
Oxalic acid
22.4 given acid is 70% HNO3 . 4.5
Initial 1 mol
= 20 0 0 0
= 0.0001 100 moles 90
∴ Its weight = 31.5 × g 1 1 1
Number of molecules of water 70 Final 0 20 20 20
moles
= 0.0001 × NA = 45 g
(b) 0.18 g of water 6. In the given problem we have provided Now, H2O ()l gets absorbed by conc.
m practical yield of MgO. For calculation H2 SO4 in both reactions. Gaseous
nH2 O = H2 O
MH2 O of percentage yield of MgO, we need mixture CO and CO2 when passed
theoretical yield of MgO. For this we through KOH, only CO2 gets
0.18
= = 0.01 shall use mole concept. absorbed. Thus, CO is the remaining
18
MgCO3 (s) → MgO(s) + CO2 (g) …(i) gas.
Number of molecules of water Mass of substance Total number of moles of CO formed
= 0.01 × N A in gram
Moles of MgCO3 = in above both equations
(c) 18 mL of water Molecular mass
1 1 1
Number of moles (nH2 O) 20 = + =
= = 0.238 mol 20 20 10
84
MODULE 3
www.jeebooks.in
The NEET Edge ~ Chemistry 441

Given mass 1 As per equation,


Q Moles = Q mole of O2 gives
Molar mass 2 1 mole of Cl 2 = 2 moles of HCl
∴ Weight of CO formed = 1 mole of H2O ∴0.5 mole of Cl 2
1 ∴2 moles of O2 will give = 2 × 0.5 mole of HCl
= × 28 = 2.8 g
10 = 1× 2 × 2 = 1.0 mole of HCl
Thus, weight of the remaining product = 4 moles of water Hence, 1.0 mole of HCl (g) is produced
at STP will be 2.8 g. by 0.5 mole of Cl 2 [or 11.2 L].
12. PbO + 2HCl → PbCl 2 + H2O
8. In acidic medium MnO−4 oxidises 207 + 16 2 × 36.5 (1 mol) 15. The balanced chemical equation is
ferrous oxalate as follows : = 223 = 73
1
(1 mol) (2 mol)
Mg + O2 → MgO
2 MnO–4 + 5C2O2– 4 + 16H →
+
2+ 6.5 2
2Mn + 10CO2 + 8H2O Mole of PbO = = 0.029 24 g 16 g 40 g
223
Q5 moles of ferrous oxalate ions are From the above equation, it is clear that,
3.2
oxidised by 2 moles of MnO−4 . Mole of HCl = = 0.087
24 g of Mg reacts with 16 g of O2 .
36.5
∴1 mole of ferrous oxalate ion is 16
oxidised by Since, 1 mole of PbO reacts with Thus, 1.0 g of Mg reacts with g of
2 moles of HCl, thus in this reaction 24
2
= moles of MnO−4 PbO is the limiting reagent. O2 or 0.67 g of O2 .
5
Hence, 1 mole of PbO forms But only 0.56 g of O2 is available
= 0.4 mole of MnO–4 which is less than 0.67 g. Thus, O2 is
= 1 mole of PbCl 2
9. In alkaline medium, KMnO4 is the limiting reagent.
reduced to K 2MnO4 0.029 mole of PbO will form
Further, 16 g of O2 reacts with 24 g of Mg.
KI + H2O → KOH + HI = 0.029 mole of PbCl 2
∴ 0.56 g of O2 will react with Mg
2KMnO4 + 2KOH → 2K 2MnO4 13. C3H8 + 5O2 → 3CO2 + 4H2O
24
1 × 22.4 L 5 × 22.4 L = × 0.56 = 0. 84 g
+ H2O + [O] 16
For the combustion of 22.4 L propane,
Hence, one mole of KMnO4 is
oxygen required ∴ Amount of Mg left unreacted
reduced by one mole of KI. = (1.0 − 0.84) g Mg = 0.16 g Mg
= 5 × 22.4 L
10. For the calculation of mass of AgCl
precipitated, we find mass of AgNO3 For the combustion of 1 L of propane,
and NaCl in equal volume with the oxygen required 2. Structure of Atoms
help of mole concept. 5 × 22.4 16. Ionisation energy of H
= L= 5L
16.9% solution of AgNO3 means 22.4 = 2.18 × 10− 18 J atom–1
16.9 g AgNO3 is present in 100 mL 14. The given problem is related to the − 2 .18 × 10−18
solution. En = J atom–1
concept of stoichiometry of chemical n2
∴8.45 g AgNO3 will be present in equations. Thus, we have to convert Z = 1for H-atom
50 mL solution. the given volumes into their moles and
∴ E1 ( Energy of Ist orbit of H-atom)
then, identify the limiting reagent
Similarly, 5.8 g NaCl is present in 100
[possessing minimum number of = − 2.18 × 10−18 J atom–1
mL solution
moles and gets completely used up in ∆E = E 4 − E1
∴2.9 g NaCl is present in 50 mL the reaction]. The limiting reagent − 2 .18 × 10−18 − 2 .18 × 10−18
solution = −
gives the moles of product formed in 42 12
AgNO3 + NaCl → AgCl + NaNO3
× 2 − 2

the reaction. −18 1 1
= − 2 .18 × 10
8.45 2.9 H2(g ) + Cl 2 (g ) → 2HCl (g )  4 1 
Initial mole 0 0
169.8 58.5 15
Initial vol. 22.4 L 11.2 L 2 mol ∆E = − 2 .18 × 10−18 × −
= 0.049 = 0.049 16
Q22.4 L volume at STP is occupied by
After reaction 0 0 0.049 0.049 Cl 2 = 1 mol = + 2 .0437 × 10−18 J atom–1
∆E
∴ Mass of AgCl precipitated ∴11.2 L volume will be occupied by ∴ ν=
h
= 0.049 × 143.5 1 × 11.2
Cl 2 = mol = 0.5 mol 2 .0437 × 10−18 J atom–1
=7g 22.4 =
6 .625 × 10−34 J s
1 22 . 4 L volume at STP is occupied by
11. H2 + O2 → H2O = 3 .084 × 1015 s− 1 atom–1
2 H2 = 1mol
1 Thus, H2 (g) + Cl 2 (g) → 2HCl (g) 17. (a) According to de-Broglie’s equation,
General reaction 1 mol mol 1 mol
2 h
1 mol 0.5 mol Wavelength (λ) =
10 64 mv
Initial reaction mol mol ? Since, Cl 2 possesses minimum
2 32
number of moles, thus it is the limiting where, h = Planck’s constant.
5 mol 2 mol ? Thus, statement (a) is correct.
reagent.
MODULE 3
www.jeebooks.in
442 NEET Test Drive

(b) According to Heisenberg 21. According to Pauli’s exclusion 1


(iv) 1 0 0 − (because m = −1
uncertainty principle, the principle, the orbital of the electron 2
uncertainties of position (∆x) and having n = 3,l = 1 and m = − 1is 3p z is not possible for l = 0)
momentum (p = m∆v) are related as (as n/m) and an orbital can have a 1
h h maximum number of two electrons (v) 3 2 2 + (because m = 3 is
∆x . ∆p ≥ or, ∆x.m∆v ≥ with opposite spins. 2
4π 4π
h ∴ 3pz orbital contains only two not possible for l = 0)
∆x.m. ∆a . ∆t ≥
4π electrons or only 2 electrons are 27. According to Hund’s rule “the pairing
 ∆v = ∆a, a = acceleration associated with n = 3 , l = 1, m = − 1. of electrons in the orbitals of a
 ∆t 
h 22. The value of n = 3 and l = 1suggests particular subshell does not takes
or, ∆x ⋅ F ⋅ ∆t ≥ [QF = m ⋅ ∆a] that it is a 3p -orbital while the value of place until all the orbitals of a subshell

h ml = 0 [magnetic quantum number] are singly occupied. Moreover, the
or, ∆E ⋅ ∆t ≥
4π shows that the given 3p-orbital is 3pz singly orbitals must have the electrons
[Q∆E = F ⋅ ∆x, E = energy] in nature. with parallel spin.
l =1 i.e. 1s2 2s2 2p 1x 2p1y 2p1z
Thus, statement (b) is correct. 3pz
ml = 0
(c) According to Hund’s rule the half n=3
and fully filled orbitals have or
Hence, the maximum number of 1s2 2s2 2p 1x 2p1y 2p1z
greater stability due to greater
orbitals identified by the given
exchange energy, greater
quantum number is only 1, i.e. 3pz .
symmetry and more balanced
arrangement. Thus, statement (c) 23. According to Hund’s rule of maximum ∴Option (a) is the incorrect.
is correct. multiplicity, an orbital can
(d) For a single electronic species like
accommodate a maximum number of 3. Classification of Elements
2 electrons of exactly opposite spin.
H, energy depends on value of n
Hence, option (a) is correct. and Periodicity in Properties
and does not depend on l. Hence,
energy of 2s-orbital and 2p -orbital Caution Remember, maximum 28. The element with atomic number,
is equal in case of hydrogen like number of electrons in an orbital do Z = 114 is flerovium (Fl). It is a super
species. Therefore, statement (d) not depend upon the quantum heavy artificial chemical element. In
is incorrect. numbers as given in the question. the periodic table of the elements, it is
18. Given, Planck’s constant, 24. According to Aufbau principle, the a transactinide element in the p-block.
h = 6.63 × 10−34 Js electrons in an atom are filled in the It is a member of the 7th period and is
increasing order of their energies. The the heaviest known member of the
Speed of light, c = 3 × 1017 nm s−1
order of orbital energies is determined carbon family.
Frequency of quantum light
by (n + l ) rule. If (n + l ) value is same Electronic configuration for Z = 114 is
ν = 6 × 1015 s−1 for two orbitals, then the orbital with
Wavelength, λ = ? lower value of n is filled first. [Rn] 5f 14 , 6d10 , 7s 2 , 7 p 2
c
We know that, ν = For, n = 6 29. (No option is correct.)
λ
c
6s = 6 + 0 = 6 (a) H− > H+ > H
or λ= 6p = 6 + 1 = 7 It is known that radius of a cation
ν 5d = 5 + 2 = 7 is always smaller than that of a
3 × 1017 4f = 4 + 3 = 7
= neutral atom due to decrease in
6 × 1015 So, increasing order of energy will be the number of shells. Whereas, the
= 05. × 102 nm 6s → 4f → 5d → 6p. radius of anion is always greater
= 50 nm
25. Ti (Z = 22) electronic configuration than a cation due to decrease in
19. Given, ∆p = 1 × 10−18 g cm s–1 ⇒ 1s 2 , 2s 2 , 2 p 6 , 3s 2 , 3 p 6 , 4s 2 . 3d 2 effective nuclear charge. Hence,
(uncertainty in momentum) the correct order is
According to Aufbau rule
Mass = 9 × 10−28 g H− > H > H+
3s < 3 p < 4s < 3d
∆ p = m∆ v (b) Na > F > O2 −
+ −
26. The value of l varies from 0 to (n − 1)
1 × 10−18 = 9 × 10−28 × ∆v The given species are
and the value of m varies from − l to
(uncertainty in velocity) isoelectronic as they contain same
+ l through zero.
∆v = 1 × 109 cm s−1 1 number of electrons. For
The value of ‘s’ ± which signifies the
20. If n = 3, 2 isoelectronic species,
1
l = 0 to (3 − 1) = 0, 1, 2 spin of electron. The correct sets of Ionic radii ∝
m = − l to + l = − 2, − 1, 0, + 1, + 2 quantum number are following. atomic number
1 n l m s Ion: Na + , F − , O2−
s=± 1
2 (ii) 2 1 1 − (because l = 2 is Atomic number : 11 9 8
Therefore, option (c) is not a 2 Hence, the correct order of ionic
permissible set of quantum numbers. not possible for n = 2) radii is O2− > F − > Na + .
MODULE 3
www.jeebooks.in
The NEET Edge ~ Chemistry 443

(c) Similarly, the correct option is Between Cl and F, Cl has higher 1


H = [V + X − C + A]
O2− > F − > Na + . electron gain enthalpy then the F, 2
(d) Ions : Al 3+ Mg2+ N3− since the incoming electron where,V = number of valence
Atomic number : 13 12 7 experiences a greater force of electrons of central atom
Hence, the correct order is, repulsion because of small size of X = number of monovalent atoms
N3 − > Mg2 + > Al 3 + F-atom. Similar, it is true in case of C = charge on cation
O and S, i.e. the electron gain
30. A cation has always the lesser ionic A = charge on anion
enthalpy of S is higher as compared
5 = sp d , PCl 5 = sp d
SbCl 2− 3 2 3
size than a metal atom due to loss of
to O due to its small size. Thus, −
electrons and an anion has always the
the correct order of electron gain SF 4 = sp d , I 3 = sp d
3 3

greater size than metal atom due to 38. According to VSEPR theory repulsive
enthalpy of given elements is
gain of electrons. The given species interaction of electron pairs decreases
O < S < F < Cl.
are isoelectronic species as they in the order
contain same number of electrons. 34. For option (a), first ionisation energy is
the energy required to remove an lp − lp > lp − bp > bp − bp
For isoelectronic species, ionic radii
1 electron from outermost shell. As the number of lone pairs of
∝ . Hence, correct order is electrons increases, bond angle
atomic number
2+ + 2− − B < C < O < N. decreases due to repulsion between
Ion : Ca K Ar S Cl
So, option (a) is incorrect. lp − lp. Moreover, as the
Atomic no. 20 19 18 16 17 electronegativity of central atom
So, the correct order of size is For option (b), electron gain enthalpy
is the energy required to gain an decreases, bond angle decreases.
Ca 2+ < K+ < Ar < Cl – < S2– electron in the outermost shell. Hence, the order of bond angle is
31. Amongst isoelectronic species, ionic Hence, the correct order is –
radius increases with increase in Cl < O
I < Br < F < Cl.
negative charge or decrease in For option (c), as we move down the O O Cl Cl
(two lone pairs) (two lone pairs)
positive charge. group in alkali metal, metallic radius
Atomic radius of the elements increases Li < Na < K < Rb.
Cl
decreases across a period from left to For option (d), in case of isoelectronic O O
(one lone pair)
right due to increase in effective species, as positive charge
nuclear charge. On moving down a decreases or negative charge (Cl is less electronegative as
group, since, number of shells increases the ionic size of the species compared to O).
increases, so atomic radius increases. increases and vice-versa 39. (a) H2O ⇒ O
32. The amount of energy required to Al 3 + < Mg2 + < Na + < F − . H H
(2 bp + 2 lp)
remove an electron from unipositive 35. Generally, acids react with bases
ion is referred as second ionisation and bases (alkalies) react with acids. [bp = bond pair and lp = lone pair]
potential. Sodium hydroxide, NaOH, being a F
In Ti, V, Cr and Mn, generally second strong alkali never react with a basic 
(b) BF3 ⇒ B
ionisation energy increases with oxide (compound). Among the given F F
increase in atomic number but options, B 2O3 and BeO are amphoteric (3 bp + 0 lp)

second ionisation potential of Cr is oxides, SiO2 is an acidic oxide and –
(c) NH2 ⇒ N
greater than that of Mn due to the CaO is a basic oxide. Therefore, NaOH H H
presence of exactly half-filled does not react with CaO.
(2bp + 2lp)
d-subshell in Cr. Thus, the order of Cl
second ionisation enthalpy is 4. Chemical Bonding and (d) PCl3 ⇒ P
Cl Cl
Cr > Mn > V > Ti Molecular Structure (3 bp + 1 lp)
33. Electron gain enthalpy generally, Thus, in PCl 3 , the central P-atom is
increases in a period from left to right 36. Ion Structure Hybridisation
surrounded by three bond pairs and
and decreases in a group on moving NO +2 O==N==O sp one lone pair.
downwards. However, members of 3rd
period have some what higher electron NO −3 O sp 2 40.
N→O
gain enthalpy as compared to the O
N N
corresponding members of second NH+4 H sp 3
period, because of their small size. N
+ F F H H
O and S belong to VI A (16) group H H F H
and Cl and F belong to VII A (17) H µ = 0.24 D µ = 1.47 D
group. Thus, the electron gain Thus, option (c) is correct. F is more electronegative than N,
enthalpy of Cl and F is higher as therefore direction of bond is from N
compared to O and S. 37. Molecules having same hybridisation
to F whereas N is more
Cl and F > O and S have same number of hybrid orbitals,
electronegative than H, the direction

MODULE 3
www.jeebooks.in
444 NEET Test Drive

of the bond is from H to N. Thus, (d) Li 2 (3 + 3 = 6)


resultant moment of N—H bonds 5. States of Matter
= σ1s 2 , σ* 1s 2 , σ 2s 2
adds up to the bond moment of lone 46. For the reaction,
4−2
pair, that of 3N—F bonds partly BO = =1
2 SrCO3 (s) q SrO (s) + CO2 (g),
cancel the resultant moment of lone
pair. Hence, the net dipole moment of Thus, Be2 does not exist under normal K p = 16
. atm = pCO 2
NF3 is less than that of NH3 . conditions. = maximum pressure of CO2
41. In BrF3 molecule, Br is sp 3d 44. According to the molecular orbital Given, p1 = 0.4 atm,V1 = 20 L,T1 = 400 K
hybridised, but its geometry is theory (MOT), p2 = 16
. atm,V2 = ?,T2 = 400 K
T-shaped due to distortion of N2 (7 + 7 = 14) = σ 1s 2 , σ* 1s 2 , σ 2 s 2 , At constant temperature, p1V1 = p2V2
geometry from trigonal bipyramidal to
0.4 × 20 = 16
. × V2
T-shaped by the involvement of lone σ* 2 s , 2
π 2 px2 0.4 × 20
pair-lone pair repulsion. V2 = =5L
≈ π 2 py2 ,
σ 2 pz2 .
16
F 10 − 4
Bond order = =3 47. According to Avogadro’s hypothesis,
2
volume of a gas (V ) ∝ number of
Br— N−2 (7 + 7 + 1 = 15)
moles (n ).
F = σ1s 2 , σ* 1s 2 , σ 2 s 2 , σ* 2 s 2 , Therefore, the ratio of the volumes of
F gases can be determined in terms of
σ 2 pz2 , π 2 p x2 ≈ π 2 p y2 , π* 2 p1x their moles.
42. Paramagnetic species contains
unpaired electrons in their molecular 10 − 5 ∴The ratio of volumes of H2 : O2 :
BO = = 2.5
orbital electronic configuration. 2 methane (CH4 ) is given by
Molecular orbital configuration of the 2 (7 + 7 + 2 = 16)
N2− VH2 : VO 2 : VCH4 = nH2 : nO 2 : nCH4
given species is as : = σ1s 2 , σ* 1s 2 , σ 2 s 2 , σ* 2 s 2 , ⇒VH2 : VO 2 : VCH4
CO (6 + 8 = 14 ) = mH2 mO 2 mCH4
* 2 s 2, π 2 p 2 σ 2 pz2 , π 2 p x2
≈ π 2 p y2 , π* 2 p1x
≈ π* 2 p1y := : :
* 1s 2 , σ 2 s 2 , σ
σ1s 2 , σ x
10 − 6 MH2 M O 2 M CH4
≈ π 2 py2 , σ 2 pz2 BO = =2
2 Qn = mass 
(All the electrons are paired so, it is  molar mass 
diamagnetic). Hence, the increasing order of bond
order is Given, m H2 = mO 2 = mCH4 = m
O−2 (8 + 8 + 1 = 17) =
N22 − < N2− < N2 . m m m
Thus, VH2 : VO 2 : VCH4 = : :
* 2s 2 , σ 2 p 2 , π 2 p 2
* 1s 2 , σ 2 s 2 , σ
σ1s 2 , σ z x
45. The molecular orbital configuration of 2 32 16
≈ π2 py2 , *π 2 p 2 ≈ *π 2 p1
x y O−2 (8 + 8 + 1 = 17) = σ1s 2 , σ* 1s 2 , = 16 : 1 : 2
(It contains one unpaired electron so, σ 2 s 2 , σ* 2 s 2 , σ 2 pz2 , π 2 px2 48. Equal moles of CO and N2
it is paramagnetic.) ≈ π 2 py2 , π* 2 p x2 ≈ π* 2 p1y nCO = nN2
CN− (6 + 7 + 1 = 14) = same as CO Bond order (BO)
then, according to ideal gas equation,
pressure of both gases CO and N2
NO+ (7 + 8 − 1 = 14) = same as CO Nb − Na 10 − 7
= = = 1.5 becomes equal
Thus, among the given species only 2 2 ∴ pCO = p N2
O−2 is paramagnetic.
NO (7 + 8 = 15) = σ1s 2 , σ* 1s 2 , σ 2s 2 , Given, pCO + p N2 = total pressure of
43. Molecules with zero bond order, do mixture
σ* 2 s 2 , σ 2 pz2 ,
not exist. or 2 p N2 = 1 atm or p N2 = 0.5 atm
π 2 px ≈ π 2 p y , π 2 p x ≈ π* 2 p y0
2 2 * 1
According to molecular orbital theory,
10 − 5 49. Given, number of moles of hydrogen
(a) Be+2 (4 + 4 − 1 = 7) BO = = 2.5 (n H 2) and that of oxygen (n O2 ) are
2
= σ1s 2 , σ* 1s 2 , σ 2 s 2 , σ* 2 s 1 C2 (6 + 6 + 2 = 14)
2− equal.
4− 3 = σ1s 2 , σ* 1s 2 , σ 2 s 2 , σ* 2 s 2 , ∴ We have, the relation between ratio
Bond order (BO) = = 0.5
2 π 2 px2 ≈ π 2 p y2 , σ 2 p z2 of number of moles escaped and ratio
(b) Be2 (4 + 4 = 8) 10 − 4 of molecular mass.
BO = =3 nO 2 MH2
= σ1s 2 , σ* 1s 2 , σ 2 s 2 , σ* 2 s 2 2 =
4−4 He+2 (2 + 2 − 1 = 3) = σ1s 2 , σ* 1s 1 nH2 MO 2
BO = =0 2 −1
2 BO = where, M = molecular mass of the
(c) B 2 (5 + 5 = 10) 2 molecule.
1
= σ1s 2 , σ* 1s 2 , σ 2 s 2 , σ* 2 s 2 , π 2 p1x = = 0.5 n 2 n 1
2 ⇒ O2 = ⇒ O2 =
nH2 32 nH2 16
≈ π2 p1y Hence, order of increasing bond
6− 4 order is n 1 .
05 1
Bond order (BO) = =1 ⇒ O 2 = ⇒nO 2 = =
2 He+2 < O–2 < NO < C2– 2 05
. 4 4 8
MODULE 3
www.jeebooks.in
The NEET Edge ~ Chemistry 445

50. Given, VA = VB = 50 mL ∴ ∆U = W = − 505 J 58. Entropy change is given as,


TA = 150s,TB = 200 s Hence, change in internal energy, ∆U T p
∆S = nCp ln f + nR ln i K(i)
MB = 36, M A = ? of the gas is − 505 J. Ti pf
From Graham’s law of effusion 54. Work done (W ) = − pext (V2 − V1) For isothermal process,Ti = Tf
rB M A VBTA = − 3 × (6 − 4) = − 6 L atm T T
∴ nCp ln f = nCp ln i = 0 [ln1 = 0]
= =
rA MB TBVA = − 6 × 101.32 J Ti Ti
(Q 1 L atm = 101.32 J) p
M A VA × 150 From Eq. (i) ∆S = nR ln i
⇒ = = − 607.92 ≈ –608 J pf
36 200 × VA
55. Given, C(s) + O2 (g) → CO2 (g) ; 59. According to Gibbs-Helmholtz equation,
or
M A 15 3
= = ∆f H = − 393.5 kJ mol − 1 Gibbs energy (∆G) = ∆H − T∆S
36 20 4 QHeat released on formation of where, ∆H = enthalpy change
MA 9 44 g or 1 mol
= ∆S = entropy change
36 16 CO2 = −395.5kJ mol
9 × 36 9 × 9 T = temperature
MA = = QHeat released on formation of
For a reaction to be spontaneous
16 4 35.2 g of CO2
∆G < 0.
=
81
= 20.2 − 393.5 kJ mol − 1
4 = × 35.2 g ∴Gibbs -Helmholtz equation becomes,
44g
According to question, no option is ∆ G = ∆ H − T∆ S < 0
= −314.8 ≈ − 315 kJ mol −1 or, ∆ H < T∆ S
correct in this condition,
Note IfTA = 200 s andTB =150 s then
100 ° C
l →
56. H2O() H2O(g) ∆H 35.5 kJ mol −1
or, T > =
M A = 64. ∆ vapH ° = ∆ vapE ° + ∆ngRT ∆S 83.6 JK −1mol −1
35.5 × 1000
8RT ∆ vapH ° = enthalpy of vaporisation = = 425 K
51. Average velocity = 83.6
πM = 40.66 kJ mol − 1
T > 425 K
∴ Vav ∝ T For the above reaction,
(Vav ) 2 2T ∆ng = np − nr = 1 − 0 = 1 60. We have the Gibbs-Helmholtz reaction
or = = 1.4 for spontaneity as, ∆G = ∆H − T∆S
(Vav ) 1 T R = 8.314 JK −1 mol −1
T = 100° C For reaction to be spontaneous, ∆G
52. The extent to which a real gas = 273 + 100 = 373K must be negative.
deviates from ideal behaviour can be For, ∆H should be negative and ∆S
understood by a quantity ‘Z ’ called ∴ 40.66 kJ mol −1
should be positive.
the compressibility factor. Easily = ∆ vapE ° + 1 × 8.314 × 10−3 × 373 ∴ ∆H < 0 and ∆S > 0
liquefiable gases like NH3 , SO2 etc., ∆ vapE ° = 40.66 kJ mol −1 − 3 .1 kJ mol −1 And ∆S = 0 shows ∆G a negative
exhibit maximum deviation from ideal = + 37.56 kJ mol −1 quantity.
gas as for them Z < < < 1. 57. Relation between heat of reaction 61. The given phase equilibria is
CH4 also exhibits deviation but it is (∆r H ) and bond energies (BE) of Liquid e Vapour
less as compared to NH3 . reactants and products is given by
This equilibrium states that, when
∆r H = ΣBEreactants − ΣBEproducts liquid is heated, it converts into
6. Thermodynamics The reaction of formation for XY is vapour but on cooling, it further
1 1
53. According to first law of X 2 (g) + Y2 (g) → XY (g); converts into liquid, which is derived
2 2 by Clausius Clapeyron and the
thermodynamics, ∆H = − 200kJ mol −1 relationship is written as,
∆U = q + W Given, the bond dissociation energies d ln p ∆Hv
where, ∆U = internal energy =−
of X 2 , Y2 and XY are in the ratio dT RT 2
q = heat absorbed or evolved, 1 : 0.5 : 1. Let the bond dissociation where, ∆Hv = heat of vaporisation
W = work done. energies of X 2 , Y2 and XY are ‘a’ kJ
Also, work done against constant mol −1, 0.5a kJ mol −1 and ‘a’ kJ mol −1, 7. Equilibrium
external pressure (irreversible respectively.
process). ∴ ∆r H = ΣBEreactants − ∆ BEproducts 62. For a reaction, A eB
Reactant Product
W = − pext ∆V
=  × a + × 05 . a  − [1 × a]
1 1 [B]eq
K=
Work done in irreversible process,  2 2  [A]eq
W = − pext ∆V = − pext (V2 − V1) a a
−200 = + − a [B]eq
1. 6 × 1012 =
= − 2.5 atm (4.5 L − 2.5 L) 2 4 [A]eq
2a + a − 4a −a
= − 5 L atm = − 5 × 1013. J −200 = = [B]eq >> [A]eq ×1.6 × 1012
4 4
= − 506.5 ≈ − 505 J and [B]eq > [A]eq
a = 800 kJ mol −1
Since, the system is well insulated, So, mostly the product will be present
∴The bond dissociation energy of
q=0
X 2 = a kJ mol −1 = 800 kJ mol −1 in the equilibrium mixture.

MODULE 3
www.jeebooks.in
446 NEET Test Drive

63. Given, N2 + 3H2 q 2NH3 , K1 …(i) 4x 3p 3 (2 + x)2 or, percentage of dissociation


= ×
N2 + O2 q 2NO, K 2 …(ii) (2 + x) 3
p 4 (1 − x)
2 2 = (α × 100)%
1
H2 + O2 → H2O, K 3 ...(iii) 3 = (1.3 × 10−4 ) × 100
x p
2 = = 0. 013%
To calculate, (2 + x)(1 − x)2
K 70. The highest pH refers to the basic
5
2NH3 + O2 q 2NO + 3H2O, =
x 3p
[Q x <<< 1and 2] solution containing OH− ions.
2 2 Therefore, the basic salt releasing
K=? …(iv) 1/ 3 more OH− ions on hydrolysis will give
 2 K p  so, (1 − x) ≈ 1
On reversing the equation (i) and x=  highest pH in water.
 p  (2 + x) ≈ 2
multiplying the equation (iii) by 3, we Only the salt of strong base and weak
get 66. X 2 (g) + 4Y2 2 XY4 (g)
s acid would release more OH− ion on
1
2NH3 q N2 + 3H2 , …(v) where, ∆H < 0 and ∆n < 0 hydrolysis. Among the given salts,
K1 [∆n = nP − nR ] Na 2 CO3 corresponds to the basic salt
3
3H2 + O2 → 3H2O, K 33 …(vi) ∴ The forward reaction is favoured at as it is formed by the neutralisation of
2
high pressure and low temperature. NaOH [strong base] and H2 CO3
Now, add equation. (ii), (v) and (vi), [weak acid].
(According to Le-Chatelier’s principle)
we get the resultant equation, (iv).
K 67. The molecule with lone pair at centre CO23 − + H2O sHCO3− + OH−
5
2NH3 + O2 q 2NO + 3H2O atom, will behave as Lewis base. M M
2 71. 75 mL HCl + 25 mL NaOH
In the given molecules, only PF 3 has 5 5
K 2K 33
∴ K= lone pair at P as shown below: Milliequivalent of HCl
K1 F F F M 1
= 75 mL of HCl = × 75 = 15
3+ −
64. Fe(OH)3 (s) s Fe (aq) + 3OH (aq) C Si B P 5 5
F F F F F F F F Milliequivalent of NaOH
[Fe3+ ][OH– ]3
K= …(i) F F F M
[Fe(OH)3 ] Thus, PF 3 acts as a Lewis base = 25 mL of NaOH
5
Let the concentration of Fe3+ is (electron-pair donor due to presence 1
of lone pair on P-atom). = × 25 = 5
increased by x times and the 5
concentration of OH− decreases by 68. Given, CH3 COOH s CH COO 3

∴Milliequivalent of HCl left unused
1
times. + H+ , = 15 − 5 = 10
4 1 Ka = 1.5 × 10−5 …(i) Volume of solution = 100 mL
[x Fe3 + ][ × OH− ]3 ∴Molarity of [H+ ] in the resulting mixture
4 HCN s H+ + CN– ,
K= …(ii)
[Fe(OH)3 ] Ka1 = 4.5 × 10−10 …(ii) =
10
=
1
On dividing Eq. (ii) by (i) we get For CN− + CH3 COOH s HCN 100 10
+ CH3 COO– 1
1
× x = 1 ⇒ x = 64 times ∴ pH = log + = log(10) = 1
64 K=? [H ]
65. 2 AB 2 (g) 3 2 AB (g) + B (g) 2
On subtracting Eq. (ii) from Eq. (i), we 72. Given, Ka (NH4OH) = 1.77 × 10−5
get NH4OH NH+4 + OH–
3
Initial moles 1 0 0 CH3 COOH + CN– s HCN + CH3 COO –
[NH+4 ][OH– ]
At equil. 2 (1 − x) 2x x Ka 1.5 × 10−5 Ka = = 1.77 × 10−5 …(i)
K= = [NH4OH]
Ka1 4.5 × 10−10
where, x = degree of dissociation. Hydrolysis of NH4 Cl takes place as,
Total moles at equilibrium 105
= = 3.33 × 104 ≈ 3 × 104 NH4 Cl + H2O → NH4OH + HCl
= 2 − 2 x + 2 x + x = (2 + x) 3 or NH+4 + H2O → NH4OH + H+
2 (1 − x)p
So, pAB 2 = , 69. The percentage of pyridine can be Hydrolysis constant,
(2 + x) equal to the percentage of [NH4OH][H+ ]
2x p dissociation of pyridinium ion and Kh = …(ii)
pAB = [NH+4 ]
(2 + x) pyridine solution as shown below:
[NH4OH][H+ ][OH– ]
pB 2 =
xp – or Kh = …(iii)
(2 + x) + H2O
r
+OH [NH+4 ][OH– ]
N N
(p )2 (pB 2 ) From Eqs. (i), (ii) and (iii)
K p = AB
(pAB 2 )2 H Kw
As pyridinium is a weak base, so Kh = [Q [H+ ][OH− ] = Kw ]
2 Ka
 2x p   x   degree of dissociation is given as 10−14
    p = = 5.65 × 10−10
 2 + x   2 + x   Kb 1.7 × 10 −9 1.77 × 10−5
= 2
α= =
  2 (1 − x)   C 0.10 73. For a general reaction,
   p  Ax B y r xA y + + yB x

  (2 + x)   = 1.7 × 10−8 = 1. 3 × 10−4

MODULE 3
www.jeebooks.in
The NEET Edge ~ Chemistry 447
+6 +8
Solubility product 79. HCl and SO2 are reducing agents
C2O24 − → 2CO2 ; 2e − loss
(K sp) = [A y + ]x [B x − ]y which can reduce MnO2− 4 , CO2 which
ForBaSO4 (binary solute giving two ions) 2MnO−4 + 5C2O−4 + H+ → 2Mn2+ is neither oxidising nor reducing will
BaSO4 (s) r Ba 2+ (aq) + SO24 − (aq) + 10CO2 + H2O provide only acidic medium. It can
shift reaction in forward direction and
∴ K sp = [Ba 2+ ] [SO24 − ] Step IV Balance oxygen by adding
H2O on deficient site. reaction can go to completion.
= (S) (S) = S 2 …(i)
2MnO−4 +5C2O24 − +H+ →
[where, S = solubility] 9. Hydrogen
2Mn2+ +10CO2 + 8H2O
Given, S = 2.42 × 10−3 gL−1
Step V Balance hydrogen 80. ●
For ionic salts, hydrogen never
Molar mass of BaSO4 = 233 g mol −1 behaves as cation, but behaves as
∴Solubility of BaSO4 2MnO24 − + 5C2O24 − + 16H →+
2+ anion (H− ).
2.42 × 10−3 2Mn + 10CO2 + 8H2O
(S) = mol L−1

H 3 O + exists freely in solution.
233 ∴The coefficients of the reactants, ●
Dihydrogen acts as a reducing
= 1.04 × 10−5 mol L−1 MnO−4 , C2O24 − and H+ are 2, 5 and 16, agent.
On substituting the value of S in Eq. (i), respectively. ●
Hydrogen has three isotopes.
we get 76. (i) Zn(ClO ) → ZnCl + O∆ (i) Protium (11H) (ii) Deuterium (21 H)
3 2 2 2
K sp = (1.04 × 10−5 mol L−1)2 (iii) Tritium (13 H)

= 1.08 × 10−10 mol 2 L−2 (ii) 2K 2 Cr2O7 → 2K 2 Cr2O4 + Cr2O3 Protium is the most common
74. ∆G° is related to K sp by the equation, 3 isotopes of hydrogen with an
+ O3 abundance of 99.98%.
∆G ° = − 2.303RT log K sp 2
Given, ∆G° = + 63.3 kJ ∆ 81. In the reaction,
(iii) (NH4 )2 Cr2O7 → N2 + Cr2O3 Oxidation
= 63.3 × 103 J
+ 4H2O +1 −1 0, −2 +1, −2
Thus, substitute ∆G° = 63.3 × 103 J, 0
3
∆ H2O2 + O3 → H2 O + 2O2
R = 8.314 JK −1 mol −1 and T = 298 K (iv) KClO3 → KCl + O2
[25 + 273 K] from the above equation 2 Reduction
we get, 77. In peroxides, the oxidation state of O Since, H2O2 oxidises O3 to O2 and
63.3 ×103 = −2.303 ×8.314 is −1and gives H2O2 with dilute acids behaves as an oxidising agent.
× 298 log K sp and have peroxide linkage. In KO2 ,
Further, in the reaction,
∴ log K sp = −11.09 + 1 + (x × 2) = 0 +1 −1 +1 −2 0 +1 −2 0
⇒ K sp = antilog (− 11.09) −1 H2O2 + Ag2O → 2 Ag + H2 O + O2
x= Reduction
2
K sp = 8.0 × 10−12 Oxidation
(thus, it is a superoxide, not a
Here, H2O2 reduces Ag2O into metallic
peroxide) In BaO2 , + 2 + (x × 2) = 0
silver (Ag) (as oxidation number is
8. Redox Reactions x = −1
reducing from +1to 0).
Thus, it is a peroxide and gives H2O2
75. The given redox reaction is
when reacts with dilute acids and has
MnO−4 + C2O24 − + H+ → Mn2+ peroxide linkage as
10. s-block Elements
+ CO2 + H2O Ba 2 + [O − O]2 − 82. The smaller the size of the ion, the
The reaction can be balanced by ↑ greater is the degree of hydration,
considering the following steps: Peroxide linkage thus degree of hydration is highest for
Step I Balance the atoms except Li + and lowest for Cs+ . Thus, Li + holds
In MnO2 and NO2 , Mn and N exhibit more water molecules in its hydration
H and O. variable oxidation states, thus, the sphere and becomes largest in size
MnO4− + C2O24 − + H+ → Mn2+ oxidation state of O in these is − 2. among alkali metals and Cs+ ion hold
+ 2CO2 + H2O Hence, these are not peroxides. Thus, least number of water molecules.
Step II Write the oxidation number of it is clear that among the given Hence, ionic mobility is highest for
each atom. molecules only BaO2 is a peroxide. Cs+ (due to its smallest size in
Oxidation (−2e − ) 78. When chlorine gas reacts with hot and aqueous solution) and lowest for Na + .
+7 +6 conc. NaOH solution, it Thus, the order of ionic mobility in
+8
MnO−4 + C2O24 − + H+ → Mn2++ 2CO2 + H2O disproportionates into (Cl − ) chloride aqueous solution is
and (ClO−3 ) chlorate ions. Cs+ > Rb+ > K+ > Na+
Reduction (+5e − ) Oxidation
83. As the size of the alkali metal cation
Step III Cross multiply by change in 0
−1
+5 increases, thermal stability of their
oxidation number 3 Cl 2 + 6NaOH → 5 NaCl + NaCl O3 + 3H2 O
hydrides decreases.
+7
Reduction
MnO−4 → Mn2+ ; 5e − gain In this process, oxidation number of
Hence, the correct order of thermal
stability of alkali metal hydrides is
chlorine changes from 0 to −1and 0 to +5.
LiH > NaH > KH > RbH > CsH
MODULE 3
www.jeebooks.in
448 NEET Test Drive

84. Metals are usually not found as The magnitude of the lattice energy 92. As the size of halogen atom
nitrates in their ores, because metal remains almost constant as the size of increases, the acidic strength of
nitrates are highly soluble in water. the sulphate ion is so big that small boron halides increases.
For example, KNO3 (salt peter) would increase in the size of the cation from Thus, BF3 is the weakest Lewis acid.
be classified as completely soluble. Be to Ba does not make any This is because of the pπ-pπ back
Thus, KNO3 could be expected to difference. However, the hydration bonding between the fully filled
dissociate completely in aqueous energy decreases from Be2+ to Ba 2+ unutilised 2p -orbitals of F and vacant
solution to give K + and NO−3 ions. appreciably as the size of the cation 2p-orbitals of boron which makes BF3
+ −
KNO3 eK (aq) + NO (aq)3
increases down the group. The
significantly high solubility of MgSO4
less electron deficient. Such back
The nitrate anion has three equivalent donation is not possible in case of
is due to high enthalpy of solvation of
oxygen surrounding a central nitrogen BCl 3 or BBr3 due to larger energy
the smaller Mg2+ ions.
atom. This tends to spread the single difference between their orbitals.
negative charge and make it easier 88. Hydration energy of sulphate Thus, these are more electron
for water (using hydrogen bonds) to decreases from top to bottom in II deficient. Since, on moving down the
separate the ions in solution. group. Mg 2+ is smaller than other group the energy difference increases,
O O– O– given ions of II group, so Mg 2+ is the Lewis acid character also
readily hydrated. MgSO 4 has higher increases. Thus, the tendency to
N N N hydration energy than lattice energy.
behave as Lewis acid follows the order

O O– –
O O O O– 89. When calcium carbide (CaC 2 ) reacts
BBr3 > BCl 3 > BF3 .
with nitrogen (N2 ) under high
85. The covalent character in an ionic bond 93. Al 2O3 may be converted into anhyd.
temperature, it forms calcium
can be decided by Fajan’s rule. AlCl 3 by heating a mixture of Al 2O3
cyanamide which is also called
According to this rule, compounds nitrolim. and carbon in dry chlorine.
with small cation, large anion, more Al 2O3 + 3C + 3Cl 2 → Al 2 Cl 6
High
charge on cation or anion shows more CaC2 (s) + N2 (g) → Hot and dry Anhy. AlCl 3
temperature
covalent character. As the above Anhy. AlCl 3 exists in the form of dimer
conditions opposes, it shows ionic Ca (CN)2 (s) + C (s)
Calcium cyanamide as Al 2 Cl 6 .
character.
94. Al 3+ shows maximum coordination
Since, the size of cation decreases in 90. Compound X is CaCO3 .

number 6, thus it will form AlF63− .
the order Ba 2+ > Ca 2+ > Be2+ .
CaCO3 → CaO+ CO2 ↑ AlF3 forms K 3 [AlF6 ] when dissolved in
Therefore, the correct order of ionic X Residue
HF in the presence of KF as shown
character will be CaO + H2O → Ca(OH)2 below.
Residue Y
BeH2 < CaH2 < BaH2 .
Ca(OH)2 + CO2 +H2O → Ca(HCO3)2 HF
AlF3 + 3KF → K 3 [AlF6 ]
BeH2 has some covalent character. It Y Excess Z
is because of the effect of ∆
Ca(HCO3 ) 2 → CaCO3 + H2O+ CO2 ↑ 95. Boric acid can be considered as an
polarisation. According to Fajan’s rule, X
Z acid because its molecule accepts
smaller the size of cation and more
OH− from water, releasing proton.
the charge on the cation, greater is its
polarising power. Thus, BeH2 has 11. Some p-block Elements H3BO 3 + H2O q B(OH)–4 + H+
some covalent character also. 91. The atomic radii as well as ionic radii Acid Base Conjugate Conjugate
base acid
86. All salts are soluble in water and give increases on moving down the group
strong acid and weak base. 13 elements because of the 96. The inability of ns 2 electrons of the
SrCl 2 + 2H2O → Sr(OH)2 + 2HCl successive addition of one extra shell valence shell to participate in bonding
of electrons. is called as inert pair effect. Due to
BaCl 2 + 2H2O → Ba(OH)2 + 2HCl
However, there is an anomaly at this effect, the lower oxidation state
MgCl 2 + 2H2O → Mg(OH)2 + 2HCl becomes more stable on descending
gallium in case of atomic radii. Atomic
CaCl 2 + 2H2O → Ca(OH)2 + 2HCl the group. Thus, Sn2+ is a reducing
radii of Ga is lesser as compared to
The basic nature of alkaline earth Al. Gallium (Ga) with electronic agent while Pb4+ act as an oxidising
metals generally increases from Be to configuration, [Ar]18 3d10 4s 2 4 p1 has agent.
Ra. Thus, the order of basic nature of an extra d-electrons which do not 97. In pyrosilicate, only one oxygen atom
these hydroxides is screen the nucleus effectively. is shared.
Mg(OH)2 < Ca(OH)2 < Sr(OH)2 Consequently, electrons of Ga are – –
< Ba(OH)2 more attracted by nucleus.
Hence, pH is highest for BaCl 2 . (As Thus, the increasing order of atomic O=O
– –
pH increases with basic nature) radii of the group 13 elements is B (85 = Si
87. Solubility of the sulphates. The pm) < Ga (135 pm) < Al (143 pm) < In
sulphates become less soluble as you (167 pm) < Tl (170 pm). – –
go down the group. i.e. Pyrosilicate
Mg > Ca > Sr > Ba
MODULE 3
www.jeebooks.in
The NEET Edge ~ Chemistry 449

CH3 CH3 101. In keto-enol tautomerism, a carbonyl compound with a


 Hydrolysis  hydrogen atom on its alpha-carbon rapidly equilibrates with
98. Cl — Si — Cl → HO — Si — OH its corresponding enol.
–HCl
  O OH
CH3 CH3
 
Dimethyl dichlorosilane R — C— CH2R ′ r R — C == CHR ′
CH3 Ketone Enol
 (containing α-hydrogen)
nHO — Si — OH →
 –H 2 O 102. The enols of β-dicarbonyl compounds are more stable
(Condensation
CH3 polymerisation) because of conjugation and intramolecular H-bonding. Thus,
 CH3  CH3 the order of stability is
    OH O O O
—O — Si — O——
 Si — O 
    
  H3 C  C == CH  C  CH3 > CH3  C  CH2  C  CH3
 CH3  CH
 n − 1 3 (Stabilised by conjugation II
Straight chain polymer and H-bonding)
(silicon) OH O
III  
Straight chain silanes are silicon oils. These are more stable > CH2 == C  CH2  C  CH3
at high temperature than mineral oils and have less tendency I
to thicken at low temperature. Less stable as (== ) bond
is not in conjugation with carbonyl group
H
12. Organic Chemistry : Some Basic *
103. CH3—C — C—CH2—C—CH 3
Principles and Techniques
H H Br
99. O
O C* = asymmetric carbon
H C Number of optical isomers = 2n
1 2
3 4
where, n = number of asymmetric carbon atoms = 21 = 2
5 Number of geometrical isomers = 2n
6
where, n = number of double bonds = 21 = 2
—CHO group gets higher priority over C == O and Hence, total number of stereoisomers = Total optical isomers
C == C group in numbering of principal carbon chain. + Total geometrical isomers = 2 + 2 = 4
IUPAC name = 3-keto-2-methylhex-4-enal. 104. Hyperconjugation occurs through the H-atoms present on the
100. The biphenyl compounds having proper substitution at carbon atom next to the double bond, i.e. α-hydrogen atoms.
ortho-position of benzene rings resulting steric hindrance. There is no α-H in the structures I and II.
This steric hindrance makes the biphenyl system non-planar So, hyperconjugation occurs in structure III only. i.e.
and hence optically active compounds.
Br Br CH3

Proper substitution H α-hydrogen


H
Restricted rotation H C
I I about single bond C C
H CH2
I H H CH3
C C
H C
Improper substitution H α-hydrogen
H
105. Allylic and benzylic halides show high reactivity towards SN1
H I
reaction. Further, due to greater stabilisation of allyl and
CH3 H
benzyl carbocations intermediates by resonance, primary
allylic and primary benzylic halides show higher reactivity in
Improper substitution SN1 reactions than other simple primary halides.
H H
CH3 H   +
H3 C  C == CH  CH2  Cl ←→ H3 C  C == CH  CH2
O2N 1°-allyl cation
H
No bulkier group present

at ortho position arising H3 C  C  CH ==CH2 —
improper substitution +
I 2 °-allyl cation
Hence, it undergoes nucleophilic reaction readily.

MODULE 3
www.jeebooks.in
450 NEET Test Drive

106. Reactivity of carbonyl compounds toward nucleophilic 110. The given reaction takes place as follows :
addition reactions depends on the presence of substituted Br / hν Na/ dry ether
CH4 →
2
CH3Br → CH3  CH3
group. (A ) Step I Wurtz reaction (A )
(Step II)
Electron withdrawing (−I , − M ) groups increase reactivity
towards nucleophilic addition reactions. Thus, correct order is Step I Alkyl halide is formed by free radical halogenation of
O alkane in the presence of UV-light.
Step II The formed alkyl halide reacts with sodium in
CHO CHO CHO C CH3 presence of dry ether to form alkane containing double
number of carbon atoms present in alkyl halide.
This reaction is known as Wurtz reaction.
> > >
From the above mechanism, it is concluded that option (d) is
correct as in all other cases the hydrocarbon formed in step 2
will contain more than four carbon atoms.
NO2 (–I, –M) CH3 (+I )
111. The enthalpy of hydrogenation of given compounds is
107. Presence of electron releasing groups like R, OH etc.,
inversely proportional to stability of alkene.
increases the electron density at o/p-position and thus, +
+ +
+
H2C CH2 H2C – – CH2
makes the benzene ring more reactive (at o/p-positions)
towards an electrophile. On the other hand, electron
withdrawing groups like COOH, NO2 etc., if present, –
reduces electron density and thus, reduces the activity of +
+

benzene nucleus towards an electrophile. Thus, the order of CH2 CH2


the given compounds toward electrophilic nitration is III Less stable

Electron Electron releasing +


+
H3C CH2 H3C CH2
withdrawing group (EWG) group (ERG) –

NO2 COOH CH3


H
CH3 CH3
II
< < < H3C CH3
H2C – CH3
Nitrobenzene Benzoic acid Benzene Toluene

Thus, toluene is most reactive towards electrophilic nitration.


108. Electron withdrawing substituent deactivates the benzene CH3 CH3
nucleus towards electrophilic substitution while electron Stable
releasing substituent activates the ring towards electrophilic H3C CH3
substitution.
= Already aromatic compound,
Among the given, —OH has the higher electron donating most stable.
tendency and thus, activates the ring more towards
electrophilic substitution. CH3
Hence, I
CH3 Hence, correct order is I>II>III.
OH 112. The said reactions can be visualised as :
Br
H2 C CH2
Show + M-effect, due to this benzene HBr
CH3 CH2 CH2
ring becomes activate. C Electro-
H2 philic
addition
It is more reactive towards electrophilic reagent.
Elimination
H3C CH CH2
HBr
13. Hydrocarbons CH3  CH2  CH2  OH →
Electrophilic addition

109. Due to the absence of torsional strain staggered Elimination


conformation of ethane is more stable than eclipsed CH 3  CH 2  CH 2  Br → H 3C  CH == CH 2
conformation of it. Br O
H  
HBr
HH CH2 == C== O → H2C ==C OH q H3C  C Br
H H Nucleophilic
addition

H H H Direct elimination
H H CH3  CH2  CH2  Br → CH3 CH ==CH2
H H
Staggered conformation Eclipsed conformation Thus, option (c) is correct.

MODULE 3
www.jeebooks.in
The NEET Edge ~ Chemistry 451

113. Reaction of HBr with propene in the presence of peroxide 116. Since, NaNH2 /liq.NH3 behaves as a base, so it abstracts
gives n-propyl bromide. This addition reaction is an example proton from acetylene to form acetylide anion followed by
of anti-Markownikoff’s addition reaction. alkylation to give compound (X), i.e. 1-butyne. (X) further
(i.e. it is completed in form of free radical addition) reacts with NaNH2 /liq.NH3 followed by alkylation with ethyl
Peroxide
CH3 — CH == CH2 + HBr → bromide yields 3-hexyne (Y).
CH3 — CH2 — CH2Br (1) NaNH2/Liq.NH3
n −propyl bromide H — C ≡≡ C—H H — C ≡≡ Cs

Mechanism of this reaction is represented as follows : (2) CH3CH2—Br


alkylation
Step I Formation of free radical of peroxide by means of
HBr + H — C ≡≡ C — CH2CH3
decomposition. X

∆ 1-butyne
C6H5 — C— O O — C —C6H5  → 2C6H5 — COO
  Benzoate free radical (1) NaNH2/Liq.NH3
O O s
Benzoyl peroxide C ≡≡ C — CH2CH3
alkylation (2) CH3CH2—Br
Step II Benzoate free radical forms bromine free radical with
HBr. H3CH2C — C ≡≡ C — CH2CH3 + HBr
• • ( Y)
C6H5 COO + H Br → C6H5 COOH + Br 3-hexyne
Step III Bromine free radical attacks on C==C of propene to 117. Followed by Markownikoff’s rule.
form intermediate free radical. CH3 — CH2 — C ≡≡ CH + HCl →
• •
CH3 — CH == CH2 +Br → CH3 — CH— CH2 + I
| 
HI CH — CH —C
Br CH3 CH2 —C == CH2 → 3 2 — CH3
1° free radical (less stable)  
• Cl Cl
CH3 — C H — CH2Br 2-chloro-2-iodobutane
2 ° free radical
(more stable) 118. In chlorobenzene, bromobenzene and chloroethene, lone

Hence, CH3 — C H — CH2Br is the major product of this step. pair of halogen is delocalised with π-bonds so it attains
double bond character. Thus, these are not suitable as a
Step IV More stable free radical accept hydrogen free radical
halide component for Friedel-Crafts reaction.
from benzoic acid and give final product of reaction. i.e.
n-propyl bromide. Halides, i.e. chloro and bromobenzene along with
• chloroethene have carbon halogen bond as :
CH3 — CH— CH2Br + C6H5 COOH →
• C X
CH3 — CH2 — CH2Br + C6H5 COO
n -propyl bromide
But this is not a case with isopropyl chloride and used as
Step V Benzoate free radicals are changed into benzoyl component of Friedel-Crafts reaction.
peroxide for the termination of free radical chain. CH3
• •
C6H5 COO + C6H5 COO → (C6H5 COO)2 C
Anhy. AlCl3 H CH3
CH3 CH3 + CH3 CH Cl

H
+ r CH3
114. CH3 C CH CH2 CH3 C CH CH3
119. In the presence of halogen carrier, electrophilic substitution
CH3 CH3
occurs while in the presence of sunlight, substitution, occurs
CH3 CH3 CH3 CH3
at the side chain.
1-2-methyl ∆
CH3 C CH CH3 –H CH3 C CH CH3
shift r CH3 CH3 CH3
115. Greater the s-character of C-atom in hydrocarbons, greater the Cl
electronegativity of that carbon and thus greater the acidic Cl2
nature of the H attached to electronegative carbon. FeCl3
+
CH ≡≡ CH CH2 == CH2 CH3  CH3
o-chlorotoluene
Hybridisation : sp sp 2 sp 3 Cl
s-character : 50% 33% 25% p-chlorotoluene
(X)
Electronegativity : ←
Max.
FeCl 3 + Cl 2 → FeCl −4 + Cl +
Acidic character ←
Max.
Electrophile attacking species
of terminal H
(Q—CH3 is an o/p-directing group.)
Thus, CH ≡≡ CH > CH3 C ≡≡ CH > CH2 == CH2 > CH3  CH3

MODULE 3
www.jeebooks.in
452 NEET Test Drive

In presence of hν, reaction is free From the above figure, you can clearly where,
radical substitution reaction. see that coordination number of F − is Z = number of atoms per unit cell
CH3 CH2Cl 4 while that of Ca 2+ is 8. M = molar mass
125. Volume of atoms in a unit cell a 3 = volume of unit cell
4 [a = edge length]
Cl2 Cl2 (V ) = πr 3
3 NA = Avogadro’s number
hν hν
a = 6.022 × 1023
For primitive cell, r =
2 3a
CHCl2 CCl3 4 a
3 For bcc, Z = 2, radius (r ) =
V = π  4
3  2 4r
Cl2 πa 3 a=

= 3
6 a
Trichloromethyl Volume of the unit cell (V ) = a 3 For fcc, Z = 4, r =
2 2
benzene
Thus, total volume occupied by the ⇒ a = 2 2r
(Y)
atoms
Volume of the atoms in unit cell According to question,
14. Environmental Chemistry =
Volume of unit cell  ZM 
 
120. Microorganisms present in the soil πa 3 1 π d room temp.  N Aa 3  bcc
act as biggest source and sink. A = × 3 = = 0.52 =
6 a 6 d900 ° C  ZM 
sink is a natural or artificial reservoir  
 N a3 
that accumulates and stores some 126. In bcc unit cell, the number of atoms  A  fcc
chemical compound for an =2
On substituting the given values, we
indefinite period. Thus, (b) is correct Thus, volume of atoms in unit cell get
4
option. (V ) = 2 × πr 3 d room temp.
3
121. Among the given chlorofluorocarbons d900° C
3
are the compounds that are For bcc structure (r ) = a 2 ×M 4 ×M
responsible for the ozone depletion 4 =
3
4r 
3
N A × (2 2r )3
which degrade ozone into moleculer 4  3 
(V ) = 2 × π  a =
3
π a3 N A ×  
oxygen. It is not a component of 3  4  8  3
photochemical smog while other are Volume of unit cell (V ) = a 3 [QGiven, M and r of iron remains
component of smog. constant with temperature]
Percentage of volume occupied by
2 × 3 3 16 2 r 3
122. The fish growth is inhibited, if the unit cell = ×
dissolved concentration of oxygen in Volume of the atoms in unit cell 64 r 3 4
=
water is below 6 ppm. Volume of unit cell dbcc 3 3
=
123.N2O (nitrous oxide) occurs naturally in 3 d fcc 4 2
π a3
3
the environment. = 8 3 × 100 = π × 100 = 68% 129. Given, Li has a bcc structure.
In an automobile engine, when fuel is a 8
Density (ρ) = 530 kg m −3
burnt dinitrogen and dioxygen Hence, the free space in bcc unit cell
Atomic mass (M ) = 6.94 g mol −1
combine to yield NO and NO2 . = 100 − 68 = 32%
Avogadro’s number (N A )
127. If a = edge length of cubic systems = 6.02 × 1023 mol −1
15. Solid State For simple cubic structure, We know that, number of atoms per
a
124. In CaF2 (fluorite structure), Ca 2+ ions radius = unit cell in bcc (Z ) = 2.
2 ∴ We have the formula for density,
are arranged in ccp arrangement
For body centred cubic structure, ZM
(Ca 2+ ions are present at all corners ρ=
3
and at the centre of each face of the radius = a N Aa 3
cube) while F − ions occupy all the 4
where, a = edge length of a unit cell.
tetrahedral sites. For face centred cubic structure,
a ZM
radius = or a = 3
2 2 ρN A
Hence, the ratio of radii
2 × 6.94 g mol −1
Ca2+
1
= a:
3
a:
1
a = 3
2 4 2 2 . g cm−3 × 6.02 × 1023 mol −1
053
F–
128. Density of unit cell = 3 4.35 × 10−23 cm−3
Z ×M = 3.52 × 10−8 cm
d=
NA × a3 a = 352 pm

MODULE 3
www.jeebooks.in
The NEET Edge ~ Chemistry 453

130. Given, ionic radius of cation (A + ) ∴Concentration of cation vacancies Since, the molarity of Na 2 CO3 is
= 0.98 × 10 m−10 = 10−6 mol/mol of NaCl 0.955 M, the molarity of CO2−3 is also
Ionic radius of anion (B − ) = 10−6 × 6.023 × 1023 mol −1 0.955 M and that of Na + is 2 × 0.955
= 6.023 × 1017 mol −1 = 1.910 M
= 1.81 × 10−10 m
∴ Coordination number of each ion 136. Vapour pressure depends upon the
in AB = ? 16. Solutions surface area of the solution. Larger
Now, we have the surface area, higher is the vapour
Radius of cation 133. Total vapour pressure of mixture pressure.
Radius ratio = = Vapour pressure of pentane in Addition of solute decreases the
Radius of anion
mixture + Vapour pressure of vapour pressure as some sites of the
0.98 × 10−10 m
= = 0.541 hexane in mixture surface are occupied by solute
. × 10−10 m
181 Since, the ratio of pentane to hexane particles, result in decreased surface
If radius ratio range is in between = 1: 4 area. However, addition of solvent,
0.441 – 0.732, ion would have 1 i.e. dilution, increases the surface
∴Mole fraction of pentane =
octahedral structure with coordination 5 area of the liquid surface, thus results
number ‘six’. 4 in increased vapour pressure.
Mole fraction of hexane =
131. (a) FeO0.98 has non-stoichiometric 5 Hence, addition of water to the
metal excess defect. It occurs = (mole fraction of pentane aqueous solution of
due to missing of a negative ion × vapour pressure of pentane) (1 molal) KI, results in increased
from its lattice site, thus leaving a + (mole fraction of hexane × vapour vapour pressure.
hole which is occupied by an pressure of hexane) 137. Since, component having higher
=  × 440 + × 120

electron. Non-stoichiometric 1 4 vapour pressure will have higher
ferrous oxide is FeO0.93 − 0.96 and 5 5  percentage in vapour phase.
it is due to metal deficiency Benzene has vapour pressure
= (88 + 96) = 184 mm
defect. Thus, statement (a) is 12.8 kPa which is greater than
Q Vapour pressure of pentane in
incorrect. toluene 3.85 kPa.
mixture
(b) In an ionic crystal of A + B − type, if = Vapour pressure of mixture Therefore, the vapour will contain a
equal number of cations and higher percentage of benzene.
× mole fraction of pentane in vapour
anions are missing from their 138. From Raoult’s law of partial pressure,
phase
lattice sites, the defect is called pA° − pS nB
88 = 184 × mole fraction of pentane =
Schottky defect. Due to such
in vapour phase pS nA
defect, density of solid
∴Mole fraction of pentane in vapour 760 − 732 WB × M A
decreases. Thus, statement (b) ⇒ =
is correct. phase 732 MB × WA
88
(c) NaCl-insulator; Silicon (Si) - = = 0.478 28 . × 18
65
184 ⇒ =
semiconductor, Silver (Ag) - 732 MB × 100
conductor; Quartz - piezoelectric 134. For a dilute solution, the depression
in freezing point (∆Tf ) is directly ⇒ MB = 30.6
crystal.
proportional to molality (m) of the . × 1000
65
Thus, statement (c) is correct. ∴∆Tb = 052
. × = 1.10
solution. 30.6 × 100
(d) In an ionic crystal when an ion is ∆Tf ∝ m or ∆Tf = Kf m ∴ Boiling point = 100 + 1.10
missing from its lattice site and Where, Kf is called molal depression = 101.1°C ≈ 101° C
occupies interstitial site, the constant or freezing point depression
defect is called Frenkel defect. 139. Molality of solution X = molality of
constant or cryoscopic constant. The
This type of defect is seen in solution Y = 0. 2 mol/kg
value of Kf depends only on nature of
those crystals where the the solvent and independent of We know that, elevation in the boiling
difference in the size of cations composition of solute particles, i.e. point (∆Tb ) of a solution is
and anions is very large and their does not depend on the proportional to the molal
coordination number is low. concentration of solution. concentration of the solution. i.e.
Thus, statement (d) is incorrect. ∆Tb ∝ m
135. Molarity
132. Doping of NaCl with 10−4 mol% of Number of moles of solute or ∆Tb = Kbm
SrCl 2 means, 100 moles of NaCl are = × 1000
Volume of solution (in mL) where, m is the molality of the solution
doped with 10−4 moles of SrCl 2 . and Kb is molal boiling point constant
25.3 × 1000
∴1 mole of NaCl is doped with = = 0.9547 ≈ 0.955 M or ebullioscopic constant.
106 × 250
10−4
SrCl 2 = = 10−6 mol Na 2 CO3 in aqueous solution remains ∴By elevation in boiling point relation
100 ∆Tb = iKbm
dissociated as :
As each Sr 2+ ion introduces one ∆Tb ∝ i
Na 2 CO3 a 2 Na + + CO23 − or
cation vacancy.
x 2x x where, i is van’t Hoff factor.

MODULE 3
www.jeebooks.in
454 NEET Test Drive

Since, ∆Tb of solution X is greater Hence, 2 moles of ions are produced. We know that, degree of ionisation,
than ∆Tb of solutionY. [Co(NH3 )5 NO2 ]Cl → λ
α = ∞c × 100
(Observed colligative property is 1 mol [Co(NH3 )5 NO2 ]+ + Cl – λc
greater than normal colligative 14444244443 9.54
property). 2 ions = × 100 = 4 . 008%
238
∴ i of solution X > i of solutionY. 144. 10 g per dm 3 of urea is isotonic with
5% solution of a non-volatile solute. 148. Since, 22400 mL volume is occupied
∴ Solution X is undergoing by 1 mole of O2 at STP.
dissociation in water. Hence, between these solutions
osmosis is not possible, so their Thus, 5600 mL O2 means
140. Al 2 (SO4 )3 a 2 Al 3+ + 3 SO24 − molar concentrations are equal to 5600 1
= mol O2 = mol O2
i =5
Value of van’t Hoff’s factor () each other. 22400 4
2−
(a) K SO a 2K + SO ; (i = 3)
2 4
+
4
Thus, molar concentration of urea 1
∴ Weight of O2 = × 32 = 8 g
(b) K [Fe(CN) ] a 3 K + solution 4
3 6
+ [Fe(CN) ] ; (i = 4) 3− 10 g / dm3 According to problem,
6 =
3+ − Molecular weight of urea Equivalents of Ag = Equivalents of O2
(c) Al(NO ) a Al + 3 NO ;
3 3 3
(i = 4) 10 1 Weight of Ag
= M= M =
+ 60 6 Equivalent weight of Ag
(d) K [Fe(CN) ] a 4 K
4 6
Molar concentration of 5% WO2
+ [Fe(CN) 6 ]3− ; (i = 5) =
non-volatile solute Equivalent weight of O2
Therefore, K 4 [Fe(CN)6 ] has same 50 g / dm3 50
value of i that of Al 2 (SO4 )3 , i.e. i = 5. = = M w Ag w
Molecular weight of non m = O2
141. According to depression in freezing - volatile solute M Ag MO 2
point, Both solutions are isotonic to each w Ag 8
∴ ×1= × 4
∆Tf = i × kf ⋅ m 1 50
other, therefore, = 108 32
where, kf = cryoscopic constant 6 m [Q 2H2O → O2 + 4H+ +4e − ]
∆Tf × Wsolvent or m = 50 × 6 = 300 g mol −1 ⇒ w Ag = 108 g
or i=
kf × nsolute × 1000 3 3
145. χP = = 149. Al 2O3 ionises as :
3.82 × 45 3+ 2 5
= = 2.63 Al 2O3 Al 3+ + AlO3–
2 2 º
1.86 × 
5  χQ = =
3
 × 1000 3+ 2 5 At cathode
Cathode
Anode
 142 

Total vapour pressure Al + 3 e → Al
3+
142. Depression in freezing point, 3F 27 g
∆Tf = kf × m 3 2
= × 80 + × 60° = 48 + 24 = 72 torr Q Mass of aluminium deposited by
2 5
where, m = molality 3 F of electricity = 27
W × 1000 ∴ Mass of aluminium deposited by
= B
MB ⋅WA 17. Electrochemistry 4.0 × 104 × 6 × 3600 C of electricity
68.5 × 1000 68.5 146. The relation between molar 27 × 4.0 × 104 × 6 × 3600
= = = g
342 × 1000 342 conductivity (λm) and electrolytic 3F
∆Tf = 1.86 ×
68.5
= 0.372 °C conductivity (κ) is given as : 27 × 4.0 × 104 × 6 × 3600
κ × 1000 = g
342 λm = 3 × 96500
Tf = Tf 0 − ∆Tf = 0 − 0.372 ° C M
= 8.1 × 104 g
= − 0.372 ° C where, M is molarity of solution.
Given, concentration of solution, 150. Anode is always the site of oxidation
143. Given, molality, (m) = 0.0020 m thus, anode half-cell is
∆Tf = 0° C −0.00732 ° C M = 0.5 mol/dm3
Zn2 + (aq) + 2e − → Zn (s);
= −0.00732 ° C Electrolytic conductivity,
κ = 5 .76 × 10−3 S cm–1 E ° = − 0.76 V
kf = − 1.86° C / m Cathode half-cell is
Temperature, T = 298 K
∆Tf = i ⋅ kf × m Ag2O(s ) + H2O(l ) + 2e − →
∆Tf ∴ Molar conductivity,
i= κ × 1000 5 .76 × 10−3 × 1000 2Ag(s ) + 2OH− (aq ); E ° = 0.34 V
kf × m λm = = E ° cell = E ° cathode − E ° anode
0.00732 M 0.5
= = 0.34 − (− 0.76)
1.86 × 0.0020 = 11.52 S cm 2 /mol = + 1.10 V
= 1.96 ≈ 2 147. Given, molar conductance at 0.1 M 151. Given that E ° 2+ = − 0.441 V
concentration, Fe /Fe
Since, the compound is ionic, so
λc = 9.54 Ω −1 cm2 mol −1 So, Fe → Fe2+ + 2e − ;
number of moles produced is equal
to van’t Hoff factor, i. Molar conductance at infinite dilution, E ° = + 0.441 V …(i)
°
λ∞c = 238 Ω −1 cm2 mol −1 and E Fe 3+
/Fe 2+ = 0.771 V

MODULE 3
www.jeebooks.in
The NEET Edge ~ Chemistry 455

So, 2 Fe3 + + 2e − → 2 Fe2 + ; °


E cell °
= EBr −
°
+ EBr2 /HBrO
°
∆G ° = − nFE cell
2 /Br
E ° = 0.771 V …(ii) = 1.0652 − 1.595 n=2
Cell reaction = −0.5298 V ∆G° = − 2 × 96500 × 0.46
(i) Fe → Fe2 + + 2e − , E ° = 0.441 V (Non-spontaneous) = − 88780 J
(ii) 2Fe3 + + 2e − → 2Fe2 + , = − 88.7 kJ ≈ − 89.0 kJ
Among the given options, only
E ° = + 0.771 V HBrO undergoes disproportionation. 156. ∆G of H2O ()
l = − 237.2 kJ / mol
Fe + 2Fe3+ → 3Fe2+ , ∆G of CO2 (g) = − 394.4 kJ / mol
153. Calculate the value of Ecell , i.e. E1
° = 1.212 V
E cell and E 2 by substituting the ∆G of pentane (g) = − 8.2 kJ / mol
Alternative On the basis of cell reaction respective given values in the In pentane-oxygen fuel cell following
following half-cell reactions are written. Nernst equation, reaction takes place.
At anode 0.059 [Zn2 + ] C5H12 + 10H2O() l → 5CO2
E cell = E ° − log
Fe → Fe2 + + 2e − + 32 H+ + 32 e −
(oxidation) n [Cu2 + ] −
At cathode 8O2 + 32H + 32 e → 16 H2O(l )
+
Compare the calculated values of E1
2Fe3 + + 2e − → 2Fe2 + (reduction) C5H12 + 8O2 → 5CO2 + 6H2O(l ),
and E 2 and find the correct relation.
E°= ?
° = E cathode
So, E cell ° °
− E anode For the electrochemical cells, ∆Greaction = Σ∆Gproduct − Σ ∆Greactant
= (+ 0.771) − (− 0.441) Zn| ZnSO4 (0.01M)|| CuSO4 (1M)| Cu
= 5 × ∆G(CO 2 ) + 6∆G(H2 O)
= + 1.212 V. Cell reaction :
Zn + Cu2+ → Zn2+ + Cu; n = 2 − [∆G(C5H12 ) +8 × ∆GO 2 ]
152 The reaction in which same species is
oxidised as well as reduced is called 0.059 Zn2+ = 5 × (−394.4) + 6 × (−237.2)
E1 = E ° − log 2+
disproportionation reaction. Firstly, 2 Cu − (−8.2 + 0)
° = − 1972 − 1423.2 + 8.2
calculate the value of E cell of each 0.059 0.01
= E° − log
species undergoing disproportionation 2 1 = − 3387 kJ / mol
°
reaction. The reaction whose E cell value 0.059 1 = −3387 × 103 J / mol
E1 = E ° − log
is positive will be feasible 2 100 °
∆G = − nFE cell
(spontaneous). = (E ° + 0.059) °
−3387 × 103 = − 32 × 96500 × E cell
(i) Given, BrO−3 → HBrO ; For cell,
°
= 1.5 V ° =
−3387 × 103
EBrO −
/HBrO Zn| ZnSO4 (1M)|| CuSO4 (0.01M)| Cu E cell = 1.0968 V
3
– 32 × 96500
BrO−3 → BrO−4 ; E2 = E ° −
0.059
log
1
2 0.01 157. By Nernst equation,
°
EBrO − − = −1.82 V 2 .303 RT
3 /BrO 4 0.059 E cell = E ° cell − log10 K
+5 E 2 = E °− log100 = (E °− 0.059)
+1 +7 2 nF
∴ 2BrO−3 → HBrO + BrO−4 At equilibrium, E cell = 0
° ° °
⇒ E1 > E 2
E cell = Ered + E ox Given that, R = 8.314 JK – 1 mol – 1
154. From the question, we have an
°
= EBrO −
°
+ EBrO − equation T = 25 ° C + 273 = 298 K
/HBrO /BrO −
3 3 4
2H+ + 2e − → H2 (g) F = 96500 C and n = 2
= 1.5 − 1.82 = −0.32 V
2 .303 × 8.314 × 298
[Non-spontaneous] According to Nernst equation, ∴E ° cell = log10 K
+1 pH 2 × 96500
0 0.0591
(ii) HBrO → Br2 ; E = E° − log + 22 0.0591
° 2 [H ] = log10 K
EHBrO /Br2 = 1.595 V pH2
2
0.0591
+1 +5 = 0− log Given that E ° cell = 0.295 V
HBrO → BrO−3 ; 2 (10−7 )2 0.0591
∴ 0.295 = log10 K
°
EHBrO = −1.5 V [Q (H+ ) = 10−7 ] 2
/BrO − 3
∴ For potential of H2 electrode to be 0.295 × 2
+1 0 +5 log10 K = = 10
2HBrO → Br2 + BrO−3 zero, pH2 should be equal to [H+ ]2 , 0.0591
° ° ° i.e. 10−14 atm. antilog log10 K = antilog 10
E cell = EHBrO /Br2 + EHBrO /BrO −
3
10−14 and K = 1 × 1010
= 1.595 − 1.5 ∴ log =0
(10−7 )2
= 0.095 V [Spontaneous] 18. Chemical Kinetics
0 −1 155. We know that, standard Gibbs
− °
(iii) Br2 → Br ; EBr − = 1.0652 V energy, 158. For the reaction,
2 /Br
°
∆G ° = − nFE cell N2 + 3H2 → 2NH3
0 +1
°
Br2 → HBrO ; EBr2 /HBrO
= −1.595 V For the cell reaction, Rate = −
d [N2 ]
=−
1 d [H2 ]
0 −1 +1 2 Ag+ + Cu → Cu2 + + 2 Ag; dt 3 dt
2Br2 → Br − + HBrO ° = + 0.46 V 1 d [NH3 ]
E cell =+
2 dt

MODULE 3
www.jeebooks.in
456 NEET Test Drive

1 d [H2 ] 1 d [NH3 ] Rate (r ) = K1 (K eq)1/ 2 [X 2 ]1/ 2 [Y2 ] d[A]


or − =+ Rate = −
= k[A]
3 dt 2 dt dt
= K[X 2 ] [Y2 ]
1/ 2


d [H2 ] 3
= × 2 × 10−4 mol L−1s −1 1 3 [where, k = constant]
dt 2 ∴Order of reaction = + 1 = = 15
. 0.693
2 2 Half-life (t1/ 2 ) =
= 3 × 10−4 mol L−1s −1 k
162. A + B → Product ∴Rate of first order reaction depends
159. For the reaction,
Rate, r ∝ [A]x [B]y …(i) upon reactant concentrations and
N2 (g) + 3H2 (g) → 2NH3 (g)
The rate decreases by a factor 4 if half-life does not depend upon initial
The rate of reaction w.r.t.
d [N2 ] the concentration of reactant B is concentration of reactant, [A]0 .
N2 = − doubled. For second order reactions, the rate
dt r
[Rate of disappearance] ∝ [A]x [2 B]y …(ii) of reaction is proportional to the
4 second power of the concentration of
The rate of reaction with respect to From Eqs. (i) and (ii) the reactant.
1 d [H2 ]
H2 = − y
4 =   and y = − 2 For, 2A → B
1
3 dt
 2 Rate = k[A]2
[Rate of disappearance] 1
The rate of reaction with respect to Hence, order of reaction with respect Half-life (t1/ 2 ) =
to B is –2. k[A]0
1 d [NH3 ]
NH3 = + ∴Rate of second order reaction
2 dt 163. For a first order reaction,
2.303 a depends upon reactant concentration
[Rate of appearance] Rate constant (k) = log
t a−x and half-life also does depend on [A]0 .
Hence, at a fixed time
d [N2 ] 1 d [H2 ] 1 d [NH3 ] where, a = initial concentration 166. In endothermic reactions, energy of
− =− =+
dt 3 dt 2 dt a − x = concentration after time ‘t’ reactants is less than that of the
d [NH3 ] 2 d [H2 ] products.
or + =− t = time in sec.
3 dt Potential energy diagram for
dt Given, a = 20 g, a − x = 5 g,
d[NH3 ] 2d [N2 ] endothermic reactions is
or + =− k = 10−2
dt dt 2.303 20
∴ t = log
160. For the reaction, 10−2 5 Ea′

Potential energy
A + B → Products = 138.6 s
On doubling the initial concentration Alternatively, Ea Products
of A only, the rate of the reaction is Half-life for the first order reaction, ∆H
also doubled, therefore t1/ 2 0.693 0.693
= = −2 = 69.3 s
Rate ∝ [A]1 …(i) 2 k 10 Reactant
Let initial rate law is Two half-lives are required for the Progress of reaction
Rate = k[A][B]y …(ii) reduction of 20 g of reactant into 5 g.
t 1/ 2 t 1/ 2 where,
If concentration of A and B both are 20 g  → 10 g  → 5 g Ea = activation energy of forward
doubled, the rate gets changed by a
∴ Total time reaction
factor of 8.
= 2 t1/ 2 = 2 × 69.3 = 138.6 s E ′a = activation energy of backward
8 × rate = k[2 A][2 B]y …(iii)
164. For first order reaction, reaction
[Q Rate ∝ [A]1]
∆H = enthalpy of the reaction.
Dividing Eq. (iii) by Eq. (ii), we get A → B
Rate = k × [A] From the above diagram,
8 = 2 × 2y
4 = 2y Rate = 2.0 × 10–5 mol L–1s –1 Ea = Ea′ + ∆H
(2)2 = (2)y [A] = 0.01 M Thus, Ea > ∆ H
∴ y=2 So, 2 .0 × 10−5 = k × 0.01 167. By Arrhenius equation
Hence, rate law is, rate = k [A][B]2 2.0 × 10–5 –1 K = Ae − E a /RT
k= s
161. We know that, slowest step is the rate
0.01 where, Ea = energy of activation
determining step. = 2.0 × 10−3 s −1 Applying log on both the side,
For first order reaction, 2.303 A 2.303 0.04
∴ Rate (r ) = K1[X ] [Y2 ] …(i) k= log t = log
0.693 0.693
Now, from equation (i), i.e. t1/ 2 = = t A0 105 0.03
k 2 .0 × 10−3 2.303
X 2 → 2 X [fast] = × 0.124
[X ] 2 = 346.5 ≈ 347 s 105
K eq =
[X 2 ] 165. For first order reactions, the rate of k = 0.028 s−1
reaction is proportional to the first We know that,
[X ] = {K eq[X 2 ]}1/ 2 …(ii) 0.693 0.693
power of the concentration of the t1/ 2 = =
Now, substitute the value of [X ] from reactant . k 0.028 s−1
equation (ii) in equation (i), we get
For, A → B = 24.14 s ≈ 24.1 s
MODULE 3
www.jeebooks.in
The NEET Edge ~ Chemistry 457

ln k = ln A −
Ea
…(i) 0.3010 = −
Ea  −15  173. The main points of Langmuir’s theory
RT 2 . 303 × 8.314  293 × 308  of adsorption are as :
Ea
or log k = − + log A …(ii) 0.3010 × 2.303 × 8.314 × 293 (i) Adsorption takes place on the
2.303RT × 308 surface of the solid only till the
∴Ea =
Compare the above equation w.r.t. 15 whole of the surface is completely
straight line equation of y = mx + c. = 34673.48 J mol −1 covered with a unimolecular layer
1
Thus, if a plot of lnk vs is a straight = 34.7 kJ mol −1 of the adsorbed gas, i.e. the
T adsorption sites are equivalent in
line, the validity of the equation is their ability to absorb the
confirmed. 19. Surface Chemistry particles.
E
Slope of the line = − a 170. ∆S [change in entropy] and ∆H (ii) Adsorption consist of two
R opposing processes
[change in enthalpy] are related by
Thus, measuring the slope of the line, (a) condensation and (b)
the equation
the value of Ea can be calculated. evaporation.
∆G = ∆ H − T ∆S
(iii) The rate of condensation depend
[Here, ∆G = change in Gibbs free
upon the uncovered surface of
energy]
Ea the adsorbent available for
ln k

Slope =– For adsorption of a gas, ∆S is


R condensation.
negative because randomness
decreases. Thus, in order to make 174. Most of the enzymes have proteinous
∆G negative [for spontaneous nature. They are highly specific and
reaction], ∆H must be highly negative get denaturated by high temperature
1/T because reaction is exothermic. or UV-rays. At optimum temperature,
168. Given, k1 = 10 ⋅ e −2000 /T and
16
Hence, for the adsorption of a gas, if which is generally in between
k2 = 1015 ⋅ e −1000 /T ∆S is negative, therefore, ∆H should 15°-25°C, enzyme activity is
be highly negative. maximum.
On taking log of both the equations
x 175. Coagulation is generally brought
we get 171. = p × T is the incorrect relation.
2000 m about by the addition of electrolytes.
log k1 = 16 − When an electrolyte is added to a
2.303T The correct relation is amount of
x p colloidal solution, the particles of the
1000 adsorption ∝ .
and log k2 = 15 − m T sol take up the ions which are
2.303T
oppositely charged. As a result, their
172. If we plot a graph between log  
x
At k1 = k2 charge gets neutralised.
m
2000 1000 Electrophoresis The movement of
16 − = 15 − and log p, a straight line will be
2.303T 2.303T colloidal particles under an applied
obtained. The slope of the line is
1000 1 electric potential is called
⇒ T = K equal to and the intercept is equal electrophoresis.
2.303 n
to log k. Electroosmosis may be defined as a
169. Given, initial temperature,
phenomenon in which the molecules
T1 = 20 + 273 = 293 K of the dispersion medium are allowed
Final temperature, 1 to move under the influence of an
T2 = 35 + 273 = 308 K Slope = electric field whereas colloidal
x n
log
R = 8.314 J mol −1 K −1 m particles are not allowed to move.
Since, rate becomes double on Tyndall effect is the scattering of light
raising temperature, Intercept log k by sol particles, which cannot be
r affected by charge on them.
∴ r2 = 2r1 or 2 = 2
r1 log p
176. The process of settling of colloidal
x
As rate constant, k ∝ r where, = amount of adsorption particles due to the neutralisation of
k2 m their charge by any means is called
∴ =2
k1 According to Freundlich adsorption coagulation.
isotherm Coagulation power of an ion depends
From Arrhenius equation, we know x
that = Kp1/n both on magnitude and sign of the
m charge (positive or negative) on the
k Ea T1 − T2 
log 2 = −   Taking log of both sides, ion. This fact can be explained by
k1 2.303 R  T1T2  x 1
log = log k + log p Hardy-Schulze rule.
m n According to this rule “greater the
Ea  293 − 308 
log 2 = − y = zx + c
2.303 × 8.314  293 × 308 
from valency of the coagulating
1 ion/flocculating ion (oppositely
z = (slope)
n

MODULE 3
www.jeebooks.in
458 NEET Test Drive

charged ion) added, the greater is its 182. Ellingham diagrams help us in 185. A → 4, B → 2, C → 3, D → 1
power to cause coagulation. predicting the feasibility of thermal Cyanide process It is a metallurgical
To coagulate a positively charged reduction of an ore. The criterion of technique for extracting Au (gold)
sol, the order of coagulating power of feasibility is that at a given from low grade ore by converting the
negative ion is temperature, Gibbs energy of the Au to a water-soluble coordination
I− < SO24 − < PO34 − < [Fe(CN)6 ]4 − . reaction must be negative. complex.
Similarly, to coagulate a negatively According to Ellingham diagram, the Froth floatation process This process
charged sol, the order of coagulating temperature at which two lines is used for dressing of sulphide ore,
power of positive ions is intersect shows that the metal will i.e. ZnS.
Ag+ < Pb2+ < Fe3+ < Si 4+ . reduce the oxide of other metals Electrolytic reduction This process is
177. Lower the coagulating power, higher which lie above it in Ellingham used for extraction of Al which is
is the coagulation value in millimoles diagram. carried out in a steel tank lined inside
per litre, i.e. coagulating power is In other words, the metal oxide with graphite. Here, graphite serves
inversely proportional to coagulation having more negative value of ∆Gf° as cathode. The electrolyte consists
value. Thus, correct order of their can reduce the oxide having less of alumina dissolved in fused cryolite
coagulating power is negative ∆Gf° . As, Mg has more − ∆G ° (Na 3 AlF6 ) and fluorspar (CaF2 ).
MgSO4 > BaCl 2 > NaCl or III > II > I value than alumina, so it will be in Zone refining This process is used
lower part of Ellingham diagram. for ultra pure Ge element. An ingot of
Hence, Mg will be used to reduce Ge is first purified by zone refining.
20. General Principles and alumina. Then a small amount of antimony is
placed in the molten zone which is
Process of Isolation of –100
0
passed through the pure Ge with the
2Cu 2O
Elements –200
4Cu+O 2
proper choice of rate of heating and
2FeO
DG°/kJ mol–1of O2

–300 2Fe+O 2 C+O2 CO2 other variables.


178. Galena (PbS), copper pyrites –400
(CuFeS2 ) and argentite (Ag2 S) are 2CO 2 2C+O
–500 2 2CO
+O 2 2ZnO
2CO
concentrated by froth floatation –600 O2 21. p-block Elements
2Zn+
process but sphalerite (ZnS) is –700
A
concentrated by chemical leaching. –800 I O3
2/3A 2 186. Let the oxidation state of nitrogen in
–900 I+O 2 each of the given N-compounds be x.
4/3A
179. Carbon and hydrogen are not 2MgO
(i) HNO3 : + 1 + x + 3 (−2 ) = 0
–1000 O2
suitable reducing agents for metal 2Mg+
–1100
sulphides. –1200 x = +5
180. SO2 gas is obtained when any 0°C 400°C 800°C 1200°C 1600°C 2000°C ∴Oxidation state of N in HNO3 is
sulphide ore is roasted. 273K 673K 1073K 1473K 1873K 2273K +5.

2 M 2 S + 3 O2  → 2 M 2O + 2 SO2 Temperature
(ii) NO : x + 1(−2 ) = 0
This gas exhibits all the Gibbs energy ∆G° vs T plots x=+2
characteristics that are given in the (schematic) for formation of some
∴Oxidation state of N in NO is
question. oxides (Ellingham diagram).
+2.
181. Extraction of gold and silver involves 183. Alumina, Al 2O3 is a bad conductor of
(iii) NH4 Cl : x + 4 (+1) + 1(−1) = 0
leaching with CN− ion. Silver is later electricity and has very high melting
recovered by displacement of Zn. point, so before subjecting to x = −3

In the metallurgy of silver or gold, the electrolysis, it is mixed with fluorspar ∴Oxidation state of N in NH4 Cl is
respective metal is leached with a (CaF2 ) and cryolite (Na 3 AlF6 ), which −3.
dilute solution of NaCN or KCN in the lower its melting point and make it (iv) N2 : x = 0[QN2 is present in
presence of air to obtain the metal in more conducting. Mainly CaF2 and elemental state]
Na 3 AlF6 are mixed with Al 2O3 for
solution as complex. From the ∴Oxidation state of N in N2 is 0.
complex, metal is obtained later by converting Al 2O3 in molten state.
Thus, the correct decreasing
replacement. 184. In the extraction of copper from its
order of oxidation states of given
In general, sulphide ore, when ore is subjected
N- compounds will be
4M (s ) + 8CN− (aq ) + 2H2O(aq ) + O2 (g) to roasting, some of it is oxidised to +5 +2 0 −3
Cu2O which reacts with the remaining HNO3 > NO >N2 > NH4 Cl
→ 4[M (CN)2 ]− (aq ) + 4OH− (aq )
Cu2 S (sulphide ore) to give copper
2[M (CN)2 ]− (aq ) + Zn(s ) → metal.
187. Phosphinic acid
O
[Zn(CN)4 ] (aq ) + 2M (s ) ;
2−
2Cu2S + 3O 2 → 2Cu2O + 2SO 2 ↑
M = Ag or Au 2Cu2O + Cu2S → 6Cu + SO 2 ↑ P
This method is known as Mac-Arthur In this process Cu2 S behaves as H OH
forest cyanide process. reducing agent. H

MODULE 3
www.jeebooks.in
The NEET Edge ~ Chemistry 459

Phosphonic acid 194. Incorrect order of bond dissociation energy F2 > Cl 2 > Br2 > I2
O
due to following order of size I > Br > Cl > F.
P 195. Since, there is a strong hydrogen bonding between HF
HO H
molecules. Hence, boiling point is highest for HF.
HO
HF > HI > HBr > HI
Due to the presence of one replaceable proton in phosphinic
acid, it is monoprotic acid. And due to presence of two 196. When chlorine gas reacts with hot and concentrated NaOH
replaceable proton in phosphonic acid, it is diprotic acid. solution, it disproportionates into chloride (Cl − ) and chlorate
(ClO−3 ) ions.
188. The oxy acid of phosphorus which contain P—H bond act as Oxidation
a reducing agent or reductant.
O 0 –1 +5
3Cl2 + 6NaOH 5NaCl + NaClO3 + 3H2O
P Hot and
H OH concentrated Reduction
H
In H3PO2 one —OH group and two P—H bonds are present. In this process, oxidation number of chlorine changes from 0
to −1and 0 to +5.
189. Hypophosphorus acid, H3PO2 , has the following structure.
O Note In disproportionation reactions, the same element
undergoes oxidation as well as reduction.
H P OH
197. Two different halogens may react to form interhalogen
H
compounds as :
As it contains only one replaceable H-atom (that is attached A. XX ′ (ClF, BrF, BrCl, IF, ICl) → Linear
with O, not with P directly) so, it is a monoprotic acid. B. XX ′3 (ClF3 , BrF3 , IF3 , ICl 3 ) → Bent T-shaped
All other given statements are true. C. XX ′5 (ClF5 , BrCl 5 , IF5 ) → Square-pyramidal
190. Since, electron repulsion predominate over the stability D. XX ′7 (IF7 ) → Pentagonal bipyramidal
gained by achieving noble gas configuration. Hence,
198.PH5 does not exist due to very less electronegativity
formation of O2− in gas phase is unfavourable.
difference between P and H. Hydrogen is slightly more
191. S4O2− 2−
6 and S2O3 have S—S bond electronegative than phosphorus, thus could not hold
192. An oxidising agent is a species, which oxidises the other significantly the sharing electrons.
species and itself gets reduced. On the other hand, BiCl 5 does not exist due to inert pair effect.
0 +2 On moving down the group, +5 oxidation state becomes less
(i) Cu+ 2H2SO 4 → CuSO 4 + SO 2 +2H2O stable while +3 oxidation state becomes more stable.
0 +4
(ii) 3S+ 2H2SO 4 → 3 SO 2 +2H2O In SO2 , pπ-dπ and pπ-pπ both types of bonds are present.
0 +4
(iii) C + H2SO 4 → CO 2 + 2SO 2 +2H2O H

+2 –1 +2 –1
(iv) CaF 2 + H2SO 4 → CaSO 4 +2HF C
F—Se—F

H H
In reaction (iv), oxidation number of elements remains H
F F Tetrahedral
unchanged. Thus, in this reaction, H2 SO4 does not act as an See-saw shape
oxidising agent.
Thus, SeF4 and CH4 do not have same shape.
193. (a) Fluorine is the most electronegative element and cannot r
exhibit any positive oxidation state. Other halogens have
d-orbitals and therefore, can expand their octets and
I
show +1, +3, +5 and +7 oxidation states. Thus, option (a)
is incorrect. I I
Geometry-Bent
Fluorine can form an oxoacid, HOF in which oxidation
state of F is +1. But HOF is highly unstable compound. Thus, option (c) is incorrect statement.
(b) All halogens are strong oxidising agents as they have 199. A → 1, B → 3, C → 4, D → 2
strong tendency to accept an electron. Thus, option (b)
The structure of the xenon compounds are represented below:
is correct.
F O
(c) All halogens form monobasic oxyacids. Thus, option (c) F F F F
is also correct. F F
Xe Xe Xe Xe
(d) Electron gain enthalpy of halogens become less F F
O O F F
negative down the group. However, the negative F O F F
electron gain enthalpy of fluorine is less than chlorine Distorted Pyramidal (XeO3) Square pyramidal Square planar
due to small size of fluorine atom. octahedral (XeF6) (XeOF4) (XeF4)
Thus, option (d) is also correct.
MODULE 3
www.jeebooks.in
460 NEET Test Drive

∴ 2.84 = n (n + 2) BM But HgI2 is soluble in excess of I− .


22. d and f-block Elements (2 . 84)2 = n (n + 2) Hg2+ + 2I− → HgI2 ↓ + 2Cl −
200.In 23V = 1s 2 , 2s 2 2 p 6 , 3s 2 3 p 6 3d 3 , 4s 2 8 = n 2 + 2n Red ppt.

Third electron which is removed to n 2 + 2n − 8 = 0 HgI2 + 2I q −


[HgI4 ]2 −
give third ionisation potential, belongs n 2 + 4n − 2n − 8 = 0 206. When SO 2 is passed through
to 3d 3 -subshell. n (n + 4) − 2 (n + 4) = 0 acidified K 2 Cr2O7 solution, green
n=2
24 Cr = 1s 2 , 2s 2 2 p 6 , 3s 2 3 p 6 3d 5 , 4s 1 chromium sulphate is formed. In this
Ni 2+ = [Ar] 3d 8 4s 0 reaction, oxidation state of Cr
Third electron which is removed to
(two unpaired electrons) changes from +6 to +3.
give third ionisation potential, belongs
Ti 3+ = [Ar] 3d1 4s 0
to 3d 5 -subshell. K 2 Cr2O7 + H2 SO4 + 3SO2 →
(one unpaired electrons)
26 Fe = 1s , 2s 2 p , 3s 3 p 3d , 4s
2 2 6 2 6 6 2 OS of Cr=+6
Cr 3+ = [Ar] 3d 3 K 2 SO4 + Cr2 (SO4 )3 + H2O
Third electron which is removed to
(three unpaired electrons) OS of Cr= +3
give third ionisation potential, belongs (Green)
Co2+ = [Ar], 3d 7 , 4s 0
to 3d 6 -subshell. The appearance of green colour is
(three unpaired electrons)
25 Mn = 1s , 2s 2 p , 3s 3 p 3d , 4s
2 2 6 2 6 5 2
due to the reduction of chromium
So, only Ni 2+ has 2 unpaired
Third electron which is removed to metal.
electrons.
give third ionisation potential, belongs 207. When H2O2 is added to an acidified
203.
to 3d 5 -subshell. Cr2+ : d4 solution of a dichromate Cr2O2−
7 ,a
In all elements shell and subshells are (4 unpaired deep blue coloured complex,
same. Required amount of energy electrons) chromic peroxide CrO5 [or
(enthalpy) is based upon the stability Mn2+ : d5 CrO(O2 )2 ] is formed.
of d - subshell. (5 unpaired Cr2O27 − + 2 H+ + 4H2O →
The 3d 5 -subshell has highest stability electrons) 2 CrO(O2 ) 2
Fe2+ : d6 14 4244 3
in all because it is half-filled subshell. Chromic peroxide
So, Mn shows highest third ionisation (4 unpaired
electrons) [blue coloured complex] + 5H2O
potential.
Co2+ : d7
201. Spin magnetic moment can be This deep blue coloured complex has
(3 unpaired
calculated as : the following structure.
electrons)
µ = n (n + 2 ) BM O
∴ [Co(H2O)6 ]2+ has minimum
where, µ = magnetic moment O 115° O
number of unpaired electrons and
BM = Bohr Magneton (unit of µ) Cr
thus, shows minimum paramagnetic O O
n = number of unpaired behaviour.
Oxidation state of Cr in CrO5 is +6 due
electrons in d-orbital. Higher the unpaired e – .
3+
to the presence of two peroxide
The electronic configuration of Co is Higher the magnetic moment
linkages which can be calculated as
[Ar] 3d 6 . µ = n (n + 2)
x + (−1) × 4 + 1 × (−2)
Here, n = 4 n = number of unpaired e – [For Cr] [For 0 − 0] [For 0]

µ = 4 (4 + 2 ) = 24 BM 204. In TiF62− , Ti is present as Ti 4+ x − 6= 0 x = +6


The electronic configuration of Cr 3+ is Ti 4+ = [Ar] 3d 0 4s 0 208. In d-d transition, an electron in a
[Ar] 3d 3 . Hence, TiF62− is colourless due to the d-orbital of the metal is excited by a
Here, n = 3 absence of unpaired electrons. photon to another d-orbital of higher
µ = 3 (3 + 2 ) = 15 BM
In Cu2 Cl 2 , Cu is present as Cu+ . energy.
The electronic configuration of Fe3+ is Paramagnetism The complex
[Ar] 3d 5 . 3d10 4s
compound which contains unpaired
Here, n = 5 electrons shows paramagnetism
µ = 5 (5 + 2 ) = 35 BM while which contains paired electrons
Due to absence of unpaired
The electronic configuration of Ni 2+ is shows diamagnetism.
electrons, Cu2 Cl 2 is colourless.
[Ar] 3d 8 . The complex which contains
205.HgCl 2 and I2 both when dissolved in
Here, n = 2 unpaired electrons exhibit d-d
water containing I− ions, the pair of
µ = 2 (2 + 2 ) = 8 BM species formed is HgI2− − transition and paramagnetism.
4 and I3 .
So, the correct option is (c). In aqueous solution, I2 reacts with (i) In MnO4− The electronic
I− and maintains the following configuration of Mn7+ is [Ar] 3d 0 .
202. Magnetic moment, µ = n (n + 2) BM
where, equilibrium. Number of unpaired electrons = 0
n = number of unpaired electrons, I2 + I− q I−3 Therefore, it will be diamagnetic
µ = 2 . 84 (given) 2+
Hg gives ppt. of HgI2 on reaction and will not show d-d transition.
with I− .
MODULE 3
www.jeebooks.in
The NEET Edge ~ Chemistry 461

(ii) In Cr2 O27 − The electronic 212. The regular decrease in the radii of When AgNO3 in excess is treated
configuration of Cr 6+ is [Ar] 3d 0 . lanthanide ions from La 3+ to Lu3+ is with these complexes then following
Number of unpaired electrons = 0 known as lanthanide contraction. reactions takes place :
So, it will be diamagnetic and will It is due to the greater effect of [Co(NH3 )6 ]3 + 3Cl − + AgNO3 → 3AgCl
not show d-d transition. the increased nuclear charge than (Excess)

(iii) In CrO24 − The electronic that of screening effect (shielding + [Co(NH3 )6 ]3+
configuration of Cr 6+ is [Ar] 3d 0 . effect).
[Co(NH3 ) 5 Cl]2 + 2Cl − + AgNO3 →
Number of unpaired electrons = 0 As a result of lanthanide contraction, (Excess)

Therefore, it is also diamagnetic the atomic radii of element of 4d and 2AgCl + [Co(NH3 )5 Cl]2+
and will not show d-d transition. 5d come closer, so the properties of
4d and 5d-transition element shows [Co(NH3 )4 Cl 2 ]+ Cl − + AgNO3 → AgCl
(iv) In MnO24 −The electronic the similarities. (Excess)
configuration of Mn6+ is[Ar] 3d1. + [Co(NH3 )4 Cl 2 ]+
213. Colour is obtained as a consequence
Number of unpaired electrons = 1 of d-d (or f-f) transition, and for d-d 217. The complexes [Co(NH3 )6 ][Cr(CN)6 ]
Since, it contains one unpaired (or f-f ) transition, presence of and [Cr(NH3 )6 ][Co(CN)6 ] are the
electron so it will exhibit both d-d unpaired electrons is the necessary
examples of coordination isomerisms.
transition and paramagnetism. condition.
209. The reaction of aqueous KMnO4 with Electronic configuration of This isomerism occurs only in those
complexes in which both cation and
H2O2 in acidic medium is La 3+ (Z = 57 ) = [Xe] 4f 0 5 d 0 6s 0
anion are complex. It occurs due to
3H2 SO4 + 2KMnO4 + 5H2O2 → (no unpaired electron)
exchange of ligands between cation
5O 2 + 2MnSO4 + 8H2O + K 2 SO4 Ti 3+ (Z = 22) = [Ar] 3 d1 4s 0 and anion.
In the above reaction, KMnO4 (one unpaired electron)
oxidises H2O2 to O2 and itself, i.e. 218. The complex is square planar and is
Lu3+ (Z = 71) = [Xe]4f 14 5 d 0 6s 0 of the type [M (abcd)]. It has three
[MnO−4 ] gets reduced to Mn2+ ion as
(no unpaired electron) geometrical isomers.
MnSO4 . Hence, aqueous solution of
KMnO4 with H2O2 yields Mn2+ and O2 Sc3+ (Z = 21) = [Ar] 3 d 0 4s 0
Py NH3 Py Br
in acidic conditions. (no unpaired electron)
Hence, due to the presence of Pt Pt
210. Electronic configuration of 63 Eu unpaired electron in Ti 3+ , it exhibit
= [Xe]54 4f 7 6s 2
colour in aqueous solution. Cl Br ; Cl NH3 ;
Electronic configuration of 64 Gd
214. The reason for greater range of
= [Xe]54 4f 7 5d1 6s 2 Py NH3
oxidation states in actinoid is
Electronic configuration of 65 Tb attributed to the 5f , 6d and 7s levels
= [Xe]54 4f 9 6s 2 Pt
having comparable energies.
211. Because of the lanthanoid The 5f-orbitals extend into space Br Cl
contraction Zr (atomic radii 160 pm) beyond the 7s and 6p-orbitals and
and Hf (atomic radii 158 pm) have 219.[Co(en)2 Cl 2 ]Cl
participate in bonding. This is in
nearly same atomic radii. Possible isomers are
direct contrast to the lanthanides
Lanthanoids include the elements where the 4f-orbitals are buried deep Cl Cl
Cl
from lanthanum La (Z = 57) to inside the atom, totally shielded by
lutetium Lu(Z = 71. ) Zirconium Zr (40) outer orbitals and thus unable to take en Co en Co en
Zr (40) belong to the second transition part in bonding.
series (4d) and Hf (72) belongs to 215. More number of oxidation states are en
third transition series (5d). Lanthanoid cis Cl
exhibited by the actinides than by the Optically active trans
contraction is associated with the corresponding lanthanides due to stereoisomers =2 Optically inactive
intervention of the 4f orbitals which lesser energy difference between 5f
stereoisomers =1
are filled before the 5d-series of and 6d-orbitals than that between 4f Hence, total number of stereoisomers
elements. The filling of 4f-orbitals and 5d-orbitals. = 2 + 1= 3
before 5d-orbitals results in regular
220. The complexes having
decrease in atomic radii which
23. Coordination Compounds sp 3 -hybridisation are tetrahedral
compensates the expected increase
while having dsp 2 -hybridisation are
in atomic size with increasing atomic 216. According to Werner’s theory, square planar. The magnetic
number. As a result of this lanthanoid
CoCl 3 ⋅ 6NH3 → [Co(NH3 )6 ]3 + 3Cl − behaviour of complexes can be
contraction, the elements of second
and third transition series have CoCl 3 ⋅ 5NH3 → [Co(NH3 )5 Cl]2 + 2Cl − paramagnetic and diamagnetic
almost similar atomic radii. CoCl 3 ⋅ 4NH3 → [Co(NH3 )4 Cl 2 ]+ Cl − based on the presence and absence
of unpaired electrons, respectively.

MODULE 3
www.jeebooks.in
462 NEET Test Drive

Electronic configuration of Ni (Z = 28) is [Ar]18 3d 8 4s 2 . Due to Dichlorobis (ethylenediamine) cobalt (III)


presence of CO (neutral ligand), oxidation state of Ni in Mirror
[Ni(CO)4 ] is 0. en
en
Since, CO is a strong field ligand, it pair up the unpaired Cl Cl
electrons of Ni. Co
Co
3d
Cl Cl
Ni(CO)4 ×× ×× ×× ×× en
en
Mirror images
CO CO CO CO Enantiomorphs
sp3-hybridised 224.
(Tetrahedral geometry)
x y x
There is no unpaired electron, hence, Ni(CO)4 is
diamagnetic with tetrahedral geometry.
CO y z z
dxy dyz dzx
Ni
CO
OC Thus, dxy ,dyz and dzx orbitals have maximum electron
CO density between the axis.
221.[Ni(CN)4 ]2 − Z y
Let oxidation state of Ni in [Ni(CN)4 ]2 − is x.
∴ x−4=−2 y
x
or x=2 x
Now, Ni 2+ = [Ar] 3d 8 4s 0
dz2 d x2 – y2
3d 3s 4p
dz 2 and dx 2 − y 2 orbitals have maximum electron density
QCN– is a strong field ligand. Hence, all unpaired electrons
along the axis.
are paired up.
3d 225. tris-(ethylenediamine) cobalt (III) bromide [Co(en)3 ]Br3
×× ×× ×× × exhibits optical isomerism :

dsp2 en
3+
en 3+
2−
∴Hybridisation of [Ni(CN)4 ] 2
is dsp .
222. Optical isomerism is exhibited only by those complexes in en Co Co en
which plane of symmetry are absent. Octahedral complexes
of the types [M (aa)3 ], [M (aa)x 2 , y 2 ] and [M (aa)2 x 2 ] have en en
absence of plane of symmetry, thus exhibit optical
d-form Mirror l-form
isomerism. Here, (aa) represents bidentate ligand, x or y
represents monodentate ligand and M represents central 226. Outer orbital complex utilises (n − 1)d-orbitals for bonding and
exhibit paramagnetic behaviour, only if there present
metal ion.
unpaired electrons.
Hence, [Co(NH3 )3 Cl 3 ]0 due to presence of symmetry
(a) In [Ni(NH3 )6 ]2+ :
elements does not exhibit optical isomerism.
or Ni 2+ = [Ar] 3d 8 4 s 0
Octahedral complexes of [M (AA)2 B 2 ] type, e.g. 3d 4s
[Co(en)2 Cl 2 ]+ , [M (AA)B 2C2 ] type, e.g. [Co(en)Cl 2
(NH3 )2 ] and [M (AA)3 ] type, e.g. [Co(en)3 ]3+ show optical
3d
isomerism, whereas complexes of [MA3B3 ] type, e.g.
[Co(NH3 )3 Cl 3 ]0 do not show optical isomerism. [Ni(NH3)6]2+ =
223. Enantiomorphs or Enantiomers A pair of molecules related Two
to each other as an object and its mirror images are known unpaired
electrons
as enantiomorphs or enantiomers. These are not
superimposable on its mirror image.
The example is [Co(en)2 Cl 2 ]+ .
sp 3d 2

MODULE 3
www.jeebooks.in
The NEET Edge ~ Chemistry 463

So, this is an outer orbital complex as it involve x = +5


4d- orbitals for bonding, but having paramagnetic Co5+ = [Ar], 3d 4 [4 unpaired electrons]
character. ox and OH are weak field ligands, thus pairing of electron
(b) In [Zn(NH3 )6 ]2+ : Zn2+ = [Ar] 3d10 units does not occur.
3d The electronic configuration of Ti(22) = [Ar] 4s 2 , 3d 2
Oxidation state of Ti in [Ti(NH3 )6 ]3+ is 3.
3d 4d Ti 3+ = [ Ar] 3d1 (one unpaired electron)

[Zn(NH3)6]2+ = Hence, complex [Co(ox)2 (OH)2 ]− has maximum number of


unpaired electrons, thus show maximum paramagnetism.
sp 3d 2 228. In [Cr(NH3 )6 ]3+ , Cr is present as Cr 3+ .
Thus, it is also an outer orbital complex as it involve Cr 3+ = [Ar] 3d 3 , 4s 0
4d- orbitals for bonding but it is diamagnetic as all the
electrons are paired. 3d 4s
3+
(c) In [Cr(NH3 )6 ] :
Cr 3+ = [Ar] 3d 3 [Cr(NH3 )6 ]3+ = [Ar] 3d 3
3d 3d
[Cr(NH3)6]3+ =

Three unpaired d 2sp3 hybridisation d 2sp3 hybridised


electrons
Since, this complex has three unpaired electrons, excitation
Because of the involvement of (n − 1)d, i.e. 3d-orbital in
of electrons is possible and thus, it is expected that this
hybridisation, it is an inner orbital complex. Its nature is
complex will absorb visible light.
paramagnetic because of the presence of three
unpaired electrons. 229. Magnetic moment (µ ) = n (n + 2) BM
(d) In [Co(NH3 )6 ] 3+
: or 3 .83 = n (n + 2)
Co3+ = [Ar] 3d 6 or 3 .83 × 3 .83 = n 2 + 2n
3d 14 .6689 = n 2 + 2n
n~−3
Hence, number of unpaired electrons in d-subshell of
3d chromium (Cr= 24) = 3.
[Cr(NH3)6]3+= CO
OC
d2sp3 inner orbital complex
Fe CO
Because of the involvement of (n − 1)d orbital in
OC
hybridisation, it is an inner orbital complex. As all the
electrons are paired, it is a diamagnetic complex. CO

227. The electronic configuration of So, the configuration of chromium ion is


V(23) = [Ar] 4s 2 , 3d 3 Cr 3+ = 1s 2 , 2 s 2 2 p 6 , 3s 2 3 p 6 3d 3
Let in [V(gly)2 (OH)2 (NH3 )2 ]+ oxidation state of V is x. In [Cr(H2O)6 ]Cl 2 , oxidation state of Cr is +3.
x + (−1) × 2 + (−1)2 + (0 × 2) = + 1 Hence, in 3d 3 the distribution of electrons
x = +5
3d1xy , 3d1yz , 3d1zx , 3d 0 2 , 3d 02
x − y2 z
V 5+ = [Ar] 4s 0 , 3d 0 (no unpaired electrons)
230. The CFSE for octahedral complex is given by
The electronic configuration of Fe(26) = [Ar] 4s 2 , 3d 6 CFSE = [−0.4t 2 ge − + 0.6 ege − ]
Let the oxidation state of Fe in [Fe(en)(ppy)(NH3 )2 ]2+ is x. For Mn , [3d ] → t 23 ge1g
3+ 4

[x + (0) + (0) + (0) × 2] = + 2 ∴ CFSE = [(−0.4 × 3) + (0.6 × 1)] = − 0.6


x=+2
For Fe , [3d ] → t 23 geg2
3+ 5
Fe2+ = [Ar], 3d 6 (Q 4 unpaired electron)
CFSE = [− (0.4 × 3) + (0.6 × 2)] = 0
but, bpy, en and NH3 all are strong field ligands, so pairing
occurs and thus, Fe2+ contains no unpaired electron. 231. In case of high spin complex, ∆o is small. than the pairing
The electronic configuration of energy. That means, the energy required to pair up the
fourth electron with the electrons of lower energy d-orbitals
Co(27) = [Ar] 4s 2 , 3d 7
would be higher than that required to place the electrons in
Let the oxidation state of Co in [Co(ox)2 (OH)2 ]− is x. the higher d-orbital. Thus, pairing does not occur.
x + (−2) × 2 + (−1) × 2 = − 1 For high spin d 4 octahedral complex,

MODULE 3
www.jeebooks.in
464 NEET Test Drive

eg The correct order of bond length of the given complexes is


[Mn(CO)6 ]+ < [Ni (CO)4 ] < [Co(CO)4 ]− < [Fe(CO)4 ]2 −
0.6∆
0.4∆
Degenerate
t2g
24. Haloalkanes and Haloarenes
d-orbitals
∴Crystal field stabilisation energy 235. In (I) and (IV) due to the presence of Lucas reagent
(HCl + anhy. ZnCl 2 ) alcohols give alkyl halides while in (III)
= (− 3 × 0.4 + 1 × 0.6) ∆o
alkyl halide is formed due to SN1 reaction.
= ( − 1.2 + 0.6) ∆o = − 0.6 ∆o
+
232. Wavelength (λ) of absorption is inversely proportional to 236. C6H5 CH == CHCH3 + H+ →
Slow
C6H5  CH  CH2  CH3
Stable carbocation
CFSE (∆o value) of ligands attached with the central metal
ion. +
C6H5  CH  CH2  CH3 + Br − →
Fast
1
i.e. λ∝
∆o C6H5  CH  CH2  CH3

According to spectrochemical series. Br
I − < Br − < S2 − < SCN− < Cl − < F − < OH−< C2O24 − < O2 − Addition product

< H2O < NSS− < NH3 < en < NO−2 < CN− Electrophilic addition reaction takes place via more stable
carbocation.
Weak field Increasing order of ∆o Strong
ligands field HBr / H2 O2
ligands 237. CH3 CH2 CH == CH2 →
Anti - Markownikoff' s rule
The CFSE of ligands attached with Co3+ ion is in the order
en > NH3 > H2O (From spectrochemical series) CH3 CH2 CH2 CH2Br →
1 Bromo butane (1° product)
QWavelength of absorbed light (λ) ∝ . _ +
∆o C H ONa
CH3 CH2 CH2 CH2Br →
2 5

∴For ligand the order of wavelength of absorption in the SN 2 reaction


visible region will be : en < NH3 < H2O (Williamson’s synthesis)
or, [Co(en)3 ]3+ < [Co(NH3 )6 ]3+ < [Co(H2O)6 ]3+ CH3 CH2 CH2 CH2OC2H5
Ethoxy-butane
233. Compounds of transition metal with carbonyls (carbon
monoxide) are known as metal carbonyls. These are 238. A protic solvents like DMF increases the reactivity of
classified into mononuclear, dinuclear, trinuclear and so on nucleophile and favours SN 2 reaction.
based on the number of central metal atoms/ions present in a
The relative reactivity of alkyl halides towards
complex. SN 2 reactions is as follows :
Complexes following EAN rule have EAN of central metal/ion
CH3 X > Primary > Secondary > Tertiary
equal to nearest inert gas configuration and hence, are
stable. However, if the primary alkyl halide or the nucleophile/base is
Effective atomic number (EAN) of the metal in a complex is sterically hindered the nucleophile will have difficulty to
given by getting the back side of the α-carbon as a result of this, the
EAN = Atomic number (Z) − Oxidation number (O.N) elimination product will be predominant. Here, CH3 CH2Br is
the least hindered, hence it has the highest relative rate
+ 2 (Coordination number)
towards SN 2 reaction.
= 26 − 0 + 2 (5) = 36
Thus, Fe(CO)5 is a stable complex/ion. Since, there is only 239. CH3 is a o/p-directing group, thus electrophilic substitution
one central metal atom present in iron carbonyl, Fe(CO)5 , reaction of toluene.
thus it is mononuclear. The structure of Fe(CO)5 is shown CH3 CH3 CH3
below:
Br
3d10 4s Br2/FeBr3
+
Cu+ = [Ar] Electrophilic
CH3 substitution CH3 CH3
The examples of dinuclear, trinuclear complexes are Br
CH3 CH3
Co2 (CO)18 and Fe3 (CO)12 respectively.
234. As negative charge on metal carbonyl complex increases, Br2/FeBr3
Br Not possible
back π-bonding increases and hence bond length of C—O due to steric
Electrophilic
bond increases while bond length of metal-carbon bond CH3 substitution hinderance
CH3
decreases.
Hence, [Fe(CO)4 ]2− has longest C—O bond length among the
given complexes. 240.

MODULE 3
www.jeebooks.in
The NEET Edge ~ Chemistry 465

PBr
243. C2H5OH →
3
C2H5Br →
Alc. KOH
CH2 == CH2
H H —Cl Ethanol
–KBr
–H2 O
Conc. H2SO4 β - elimination
Cl3C C O+
H SO H O/ ∆
Trichloroacetaldehyde –H2O →
2 4
CH3 — CH2OSO3H →
2
CH3 CH2OH
H —Cl –H2 SO4
Ethanol
CH3
Chlorobenzene
 H + /∆
244. (i) CH3 — CH — CH— CH3 →
H —Cl 
Protonation
and dehydration
OH
Cl3C C CH3 CH3
—Cl  ⊕ 1, 2-hydride

CH3 — CH — C — CH3 → CH3 —C — CH2 — CH3
2 ° carbocation shift +
More stable (3 ° carbonium ion)
Dichlorodiphenyl CH3
trichloroethane (DDT)
CH3
− H+  
→ CH3 — C == CH — CH3 + CH3 —CH — CH == CH2
25. Alcohols, Phenols and Ethers Rearrangement Major (A) Minor (B)

241. CH3 CH3 A part is major because more substituted alkenes are
| | + more stable.
H+
H3C—C—CH== CH2 H3C—C—CH—CH3 CH3
| |  HBr (dark)
CH3 CH3 (ii) CH3 — C == CH — CH3 →
2° carbocation In the absence
CH3 Major (A ) of peroxide
(i) H2O CH3 CH3
H3C— C—CH—CH3
H+ | | (ii) –H+  
CH3 — C— CH2 — CH3 + CH3 —C — CH— CH3
CH3OH   
Minor product H Br
Br
‘B’ Major (C ) Minor (D )
CH3 CH3
| | 245. An organic compound form yellow precipitate of iodoform
CH3 — C — CH — CH3 → CH3 — C — CH — CH3 with I 2 in presence of alkali, if it has CH3 CO — group directly
+ H
| | |
CH3 O+ CH3 
or it has CH3 — C — group.
2 ° carbocation
H H (less stable) 
OH
CH3 CH3 NaOH
  (a) CH3 CH(OH)CH3 + I2 –—→ CH3 COCH3
1, 2-Me shift H2O
H3C—C—C—CH3 + 2HI+ 3NaI + CH3 COO−Na+ + 3H2O
+
 CH3 COCH3 + 3I2 + 4NaOH → CHI3 ↓
H Yellow ppt.
3° carbocation
(more stable)
+ 3NaI + CH3 COO−Na+ + 3H2O
CH3 CH3 CH3 H (b) CH2 — CH2 CH(OH) CH3 + I2 →
  –H+  
H3C—C—CH—CH3 H3C—C—C—CH3 CH3 — CH2 — C— CH3 + 2HI

   O
OH + O CH3 Ethyl methyl ketone
2,3-dimethyl butan-2-ol H H It gives iodoform test.
(major product)
‘A’ CH3 CH2 — C— CH3 + 3I2 + 4NaOH →
242. When intermediate carbocation is stable, no rearrangement 
takes place in carbocation. O
CHI3 ↓ + 3NaI + CH3 CH2 COONa + 3H2O
Yellow ppt.

OH + (c) CH3OH + I2 → HCHO + 2HI
–H2O
It does not have methyl ketonic group, so it does
not give yellow ppt. with I2 in presence of alkali.
+ (d) CH3 CH2OH+ I2 → CH3 — C— H + 2HI

O

MODULE 3
www.jeebooks.in
466 NEET Test Drive

Hence, the correct order of acidic strength is


CH3 — C— H+ 3I2 + 4NaOH → CHI3 ↓ 2, 4, 6-trinitrophenol > acetic acid > phenol > cyclohexanol
 Yellow ppt.
O III > II > IV > I
− +
+ HCOONa + 3NaI + 2H2O 249. OH CH3
Friedel-Craft's
Due to the presence of —COCH3 group, it gives reaction
Haloform test. Zn-dust CH3Cl
–ZnO anhy. AlCl3
246. Iodoform reaction with sodium hypoiodite is used for the reduction
detection of CH3 CO group. Also compounds containing Phenol of Phenol Benzene Toluene
CH3 CH(OH) group shows positive iodoform test as it X Y
produces CH3 CO group on oxidation. COOH
Since, among the compounds, CH3 CH(OH) group is given
only in the substrate of option (a) hence, it is correct. The
Alk. KMnO4
reaction of compound A with NaOI is given as follows :
2NaOH + I2 → NaOI + NaI + H2O
Benzoic acid
NaOI Z
CH CH3 C CH3
250. s
OH O OH O Na+
(A) Acetophenone CHO
I2/NaOH + CHCl3+NaOH

CONa++ CHI3
Iodoform
O (yellow ppt.) The above given reaction is known as Riemer-Tiemann
Sodium benzoate reaction. In this reaction, electrophile involved is
247. This problem is based on the acidic character of phenol. dichlorocarbene (•• CCl 2 ) which is formed in the Ist step of
Electron -withdrawing group at o and p-position w.r.t. —OH mechanism. It is given as follows :
group of phenol, increase the acidic strength. Mechanism
Picric acid (2, 4, 6-trinitrophenol) is extremely more acidic Step I Generation of electrophile
than given compounds because its pKa value is close to zero CHCl 3 + OH− r –
CCl 3 + H2O
also due to the presence of three strong electron withdrawing • CCl + Cl −

CCl 3 → • 2
group (NO2 group) at ortho and para-positions, picric is (Electrophile)
more acidic compound. Dichlorocarbene

248. Higher the tendency to give a proton, higher is the acidic Step II Reaction of etectrophile with phenoxide
character and tendency to lose a proton depends upon the O– O O–
stability of intermediate, i.e. carbanion formed.
H CHCl2
2, 4, 6-trinitrophenol after the loss of a proton gives –
2,4,6-trinitrophenoxide ion which is stabilised by resonance, + CCl2 CCl2
–I-effect and –M-effect, thus is most acidic among the given
compounds. o-dichloromethyl
phenoxide
Phenol after losing a proton form phenoxide ion which is also
stabilised by resonance, − M and – I effects but is less Step III Hydrolysis
stabilised as compared to 2, 4, 6-trinitrophenoxide ions. O– O– OH
Thus, it is less acidic as compared to 2, 4, 6-trinitrophenol. CHCl2 CH
(CH3 COOH) after losing a proton gives acetate 2 OH– OH
 O 
 CH3 C  ion which is stabilised by only resonance.
 
 O− 
O– OH
Carboxylate ion
However, it is more resonance stabilised as compared to a CHO CHO
phenoxide ion, thus more acidic as compared to phenol. –H2O H+

2, 4, 6-trinitrophenol, however, is more acidic than acetic


acid due to the presence of three electron withdrawing Salicylaldehyde
—NO2 groups. Cyclohexanol gives an anion that is least
stable among the given, thus, it is least acidic.

MODULE 3
www.jeebooks.in
The NEET Edge ~ Chemistry 467

251. The given reaction takes place as follows : 252. The ether which gives more stable carbocation, forms CH3OH
H3C CH3 as one of the product with hot conc. HI. The order of stability
CH
of carbocation is 3 ° > 2 ° > 1°.

Anhy.
CH3
AlCl3 (i) O2 
+ CH3CH2CH2Cl
(ii) H3O+/D
Thus, CH3  C  OCH3 gives CH3OH as one of the product.

Cumene CH3
(P)
OH The reaction proceeds as :
O CH3 CH3

⋅⋅  +
⋅⋅
+ CH3—C CH3
Acetone
H3 C  C  O  CH3 + H+ → H3 C  C  O  CH3
(R)   
Phenol
(Q)
CH3 CH3 H
3d
Mechanism
Step I Formation of carbocation.
+ −
CH3 CH2 CH2  Cl + AlCl 3 →CH3 CH2 CH2 + AlCl 4 CH3 CH3
+ 1, 2 H − shift +  
CH3 CH2 CH2 → CH3 CH  CH3 +I È
→ H3 C C + CH3OH → CH3  C  I + CH3OH
+
Carbocation Carbocation
(1° ) (2 ° )   Methanol
CH3 CH3
Step II Electrophilic substitution reaction. 3 °carbocation
H CH3 253. The reaction of alkyl halides with sodium alkoxide or sodium
+ phenoxide to form ethers is called Williamson synthesis.
+ CH3 CH CH3 + CH
Here, in this reaction alkyl halide should be primary and
CH3 alkoxide, should be bulkier as shown below :
r
–H – − +
AlCl4 CH3 R  X + R ′  O Na →R O R ′+ NaX
Alkyl Sodium alkoxide Ether
CH halide
CH3 CH3
(Cumene)  − +
(P) CH3  C O Na + CH3 CH2 Cl →
Step III Formation of peroxide.  −NaCl

CH3 CH3 CH3 CH3


CH

C O O H CH3  CO  CH2  CH3
O2
CH3 D CH3 
CH3
Step IV Hydrolysis of oxidised product formed in step III. 26. Aldehyde, Ketone and Carboxylic Acid
CH3 CH3
+
OH
CC
C O O
CC
H C O O H H2O,H2SO4
H+ 254. CH3—C CH CH3—C CH2
HgSO4
CH3 CH3 H Intermediate
(Enal) (A)
Migration of phenyl to
oxygen,–H2 O Tautomerisation
CH3 CH3
H2 O + O
+ CC CC
C O
CC
H O C O
CC
CH3—C—CH3
CH3 H CH3
(Acetone) (B)
CH3
+ H2 O OH
C O
C C O
–H3 O+

Therefore, A = CH3  C == CH2
H CH3 H
CH3 O
C== O + OH
CH3 
B = CH3  C  CH3
Acetone Phenol
(R) (Q ) 255.

MODULE 3
www.jeebooks.in
468 NEET Test Drive

CH3 [Ag(NH3 ) 2 ]
1 → Silver
O3 OH / ∆ mirror observed
CH3 Zn,H Tollen's reagent
2 2
CH3 CH3

2
NH
O
OHC1 C C2 O

C
CH2 CH2 O OH/∆
CH3 — CH == CH—C—H

NH
H3C H Aldol But-2-en-1-al
condensation (Y)

2
NH
O3
2
Zn,H2 O
H3C 1 O CH3 CH3 — CH == N—NH—C—NH2
Semicarbazone
H C CH2 CH CH CHO (Z)
1 2
258.
H3C CH3 CH3 O NCH3
CH3NH2
CH3
1 Ketone Schiff base
O3
Zn,H2
OHC C CH2 CH2 CHO NHCH3
2 1 2 [H]
(i) LiAlH4
or
CH3 (ii) H2O NHCH3
2° amine
CH3 O O
2 259. Aldehydes and ketones containing α-H atoms undergo aldol
O3 condensation in presence of dilute alkali as catalyst and
1 H3C C CH2 CH2 CH2 C CH
Zn,H2
1 2 gives α, β-unsaturated compound with the elimination of H2O
CH3
molecule.
H H

256. CH3 CHO OH –
2 O O O
CrO2Cl2,CS2
+
H3O –
OH O
Benzaldehyde H2O

This reaction is known as Etard reaction. – OH
H
O O O
C Cl CHO ∆ –H2O
Pd/BaSO4, S
(b) + HCl
(Boiling xylene)
Benzoyl
chloride

This reaction is called Rosenmund reaction. O


(α, β- unsaturated compound)
Anhy. AlCl3
CHO 260. Aldehydes and ketones with α-hydrogen atom, when reacted
(c) + CO + HCl + HCl with a base yields aldol which on heating loses water
molecule to give α, β-unsaturated aldehydes or ketones. This
The above reaction is known as Gattermann-Koch reaction is called aldol condensation reaction.

aldehyde synthesis. C2H5ONa iC H O 2 5 + Na+
Base
COOH COCl
Conc. HCl C O
(d) + Zn/Hg + H2O

+ C2H5O C2H5OH +
CH3 Base
Thus, from the reactants given in option (d) O

benzaldehyde is not obtained. CH2—C
(Abstract the acidic hydrogen)
257. Aldehydes gives silver mirror test so, ‘X’ may be alcohol
which is oxidised by Cu gives aldehydes.
Therefore, X is acetaldehyde (CH3 CHO). (Attacking species)
(Nu)

C2H5OH Cu/573
→
K
3CH CHO
(A ) oxidation Acetaldehyde
(X )

MODULE 3
www.jeebooks.in
The NEET Edge ~ Chemistry 469

CH3 O CH3 CH3 OH



C==O + HCN C
—C O + CH2—C— CH3 CH3 CN
Acetone
H2O CH3 OH
C
CH3 O CH3 COOH
—C—CH2—C— It is not optically active, racemic mixture is not formed.
– C2H5 C2H5 OH
O C==O + HCN C
CH3 O C2H5 C2H5 CN
Diethyl ketone
H
+ —C—–CH—C— H2O C2H5 OH
C
OH H C2H5 COOH
CH3 It is not optically active.
O
H H OH
—C CH—C— C==O + HCN C
α β
–H2O H H CN
Formaldehyde
H2O H OH
261. When benzaldehyde is treated with 50% alkali, it C
undergoes oxidation to give an acid salt as well as H COOH
reduction to give an alcohol. This reaction is called It is not optically active.
Cannizaro’s reaction.
263. Order of strengths of the given carboxylic acids can be
CHO CH2OH
–+ determined by the concept of I-effect.
 3
1 COOK The oxygen atom present in the ring shows I-effect. As the
50% KOH 2
+ distance between oxygen and ––COOH group increases,
2
Cl 1 Cl –I-effect of oxygen decreases.
3
3-hydroxy methyl  Thus, corresponding carboxylic acid will show less acidic
chlorobenzene Cl nature.
Potassium-3-chloro
benzoate The correct order of strengths of the carboxylic acids is
or COOH COOH COOH
COO–
> >

II III I
 More-effect Less I-effect No I-effect
Cl (most acidic) (moderately (least acidic)
acidic)
262. OH H2O OH
C==O + HCN C C 264. Alkaline KMnO4 converts complete carbon chain (that is directly
CN COOH attached to benzene nucleus) to —COOH group. Br2 in the
(It is α-hydroxy acid) presence of halogen carrier causes bromination by electrophilic
substitution reaction and ethyl alcohol in acidic medium results
In this reaction, complete hydrolysis of cyanide gives acid
in esterification.
and partial hydrolysis gives amide.
Racemic mixture, is obtained when C-atom is asymmetric. CH2CH3 COOH COOH
CH3 CH3 OH
C==O + HCN C KMnO4 Br2/FeCl3
H H CN KOH
Acetaldehyde
Br
CH3 CH3 (m-directing) C
H2O | | B
H—C—OH + HO—C—H COOC2H5
| |
COOH COOH
d-form l-form C2H5 OH
H+
Racemic mixture
Br
D

MODULE 3
www.jeebooks.in
470 NEET Test Drive

265. SOCl2 Benzene 268. The reagent which can convert —CONH2 group into —NH2
CH3COOH CH3COCl group is NaOH/ Br2 .
Anhy. AlCl3
A
OH Among the given reagents only NaOH/Br 2 converts
—CONH2 group to —NH2 group, thus it is used for
COCH3 C—CH3 converting acetamide to methyl amine. This reaction is called
HCN Hofmann Bromamide reaction, in which primary amides on
CN treatment with Br2 / NaOH form primary amines.
B C CH3 CONH2 + NaOH+ Br2 → CH3NH2 + NaBr + Na 2 CO3
OH OH Acetamide Methyl amine
+ H2O
269.
C—CH3 C—COOH NH2
2HOH
or
COOH CH3
D R — NH2
Aryl amine Alkyl amine
(less basic) (more basic)
27. Organic Compound Containing Nitrogen Due to delocalisation of lone pair of electrons of N-atom to
266. The conversion of amide with no substituent on nitrogen to an the benzene ring, it losses its basicity and becomes less
amine containing one carbon less by the action of alkaline basic than alkyl amine.
bromamide or bromine in presence of NaOH is known as On the other hand, alkyl amine has free lone pair of electron
Hofmann Bromamide reaction. It involves the migration of as well as + I-effect of alkyl group increases electron density
alkyl or aryl group with its electron pair to electron deficient N on N-atom enhancing its basic nature.
from adjacent carbon. The reaction involves the 270. OCH3 OCH3 OCH3
intermediates of isocyanate.
NH2
O NaNH2
 +

CH3  C  NH2 + Br2 + NaOH →
Acetamide
Br NH2
Direct Cine
CH3NH2 + NaBr + Na 2 CO3 + H2O substitution substitution
O Methanamine O If nucleophile occupies same position of the leaving group,
  product is called direct substitution product.
Step I CH3  C NH2 + Br2 → CH3  C N Br
 If nucleophile occupies adjacent position of the leaving
H group, product is called cine substitution product.
O O Intermediate formed in this reaction is benzyne.
  OCH3 OCH3
Step II CH3  C N Br + OH− → CH3  C N Br + H2O
 s H –
+ NH2
H
O O Br
Benzyne
 s  •• −
Step III CH3  C N  Br → CH3  C N•• + Br OCH3 OCH3

O – –
 •• + NH2 + H—NH2
Step IV CH3  C N•• Intermolecular
 → CH3NCO NH2
alkyl migration Attack of nucleophile
− ∆
at the original position
Step V CH3NCO + 2OH → CH3NH2 + CO23 − (from where Br– leaves)
OCH3
267. Due to resonance in chlorobenzene C—Cl bond acquires
double bond character hence, C—Cl bond is inert towards
nucleophile (phthalimide ion). Therefore, aniline cannot be NH2
prepared. Direct substitution
product

MODULE 3
www.jeebooks.in
The NEET Edge ~ Chemistry 471
+ –
OCH3 OCH3 OCH3 274. NH2 N NCl
NaNO2
NH2 NH2 HCl

+ NH2 273-278 K

A B
Attack of nucleophile H—NH2 Substitution Aniline Benzene diazonium
at the adjacent product chloride
CH3
carbon H— —N
CH3 CH3
271. In strongly acidic medium, aniline is protonated to form the —N N— —N
anilinium ion. Cold CH3
+ C
NH2 NH3 p-(N,N-dimethyl) amine azobenzene
(azo dye)
H+ 275. NH2 + –
N2Cl
NaNO2 CuCN
HCl (Sandmeyer's
Aniline Anilinium ion (Diazotisation) reaction)
A
Since, anilinium ion so formed is meta directing, thus Benzene diazonium
besides ortho and para- derivatives, significant amount of chloride
meta derivative is also formed. CN CH2NH2
H2 /Ni
NH2 NH2 NH2 NH2 (Reduction)
NO2
HNO3, B C
H2SO4 Cyanobenzene Benzyl amine
+ +
288K
NO2 CH2OH
Aniline o-nitroaniline HNO2
m-nitroaniline (2%)
NO2
(47%)
p-nitroaniline
(51%) D
Benzyl alcohol
272. COOH COCl 276. Aniline on diazotisation in cold (at 0° to 5° C) gives benzene
diazonium chloride.
SOCl2 NH3
0-5°C
NH2 + NaNH2 + 2HCl Diazotisation
Br Br
B
CONH2 NH2 + –
N N—Cl + NaCl + 2H2O
NaOH
Benzene diazonium chloride
+Br2
Br Br
C D This benzene diazonium chloride on coupling with dimethyl
The conversion of ‘C’ to ‘D’ is an example of Hofmann aniline gives a coloured product, i.e. p-(N,N-dimethyl) amino
Bromamide degradation reaction. azobenzene (azo dye).

273. The complete reaction is + – CH2


N N—Cl +H N
+ – H + CH2
N NCl + H NH2
– HCl
Diazonium Aniline
CH2
chloride N N N
CH2
N N NH2 p-(N, N-dimethyl) amino
azobenzene
A (azo-dye)
(Yellow dye)

(p-azo benzene compound) 277. 1° and 2° nitro compounds react with HNO2 while 3°-nitro
compound does not.
The above reaction is a coupling reaction of aniline with
diazonium salt to give azo benzene compound. This The reactions of given compounds with HNO2 are as follow:
coupling reaction takes place at the para-position to —NH2 HON == O
CH3 CH2 CH2NO2 → CH3  CH2  C NO2
group of benzene. This reaction act as electrophilic 1° -nitro 
substitution reaction of aniline. compound N OH

MODULE 3
www.jeebooks.in
472 NEET Test Drive

N OH CHO CHO
H2
H3C C HO N O H OH
CH3 CH C NO2 HO H
CH NO2
H3C CH3 HO H HO H
1°-nitro compound
CH2OH CH2OH
CH3 L-erythrose L-threose
HO N O
CH3 C CH NO2 CH3 CH C NO2 Thus, the correct option is (d).
O CH3 N OH 281. D-(+)-glucose contains aldehydic group which reacts with
1°-nitro compound hydroxyl amine (NH2OH) to yield an oxime. The complete
reaction is
H3C
HO N O CHO CH NOH
H3 C C NO2 No reaction
H3C H C OH H C OH
3°-nitro compound
HO C H HO C H
278. The complete road map of the reaction can be seen as: + NH2OH
H C OH – H2O H C OH
+ –
NO2 NH2 N2Cl
H C OH H C OH
Sn/HCl HNO2
CH2OH CH2OH
Reduction D-(+)-glucose Glucoxime
(A) Aniline (B)
Nitro benzene Benzene diazonium
chloride unstable 282. CH2OH H OH
Glycosidic linkage
Ph––OH O
N N OH HO OH
H O OH H
Red colour dye
OH H H
H H H H
O
28. Biomolecules H OH CH2OH
279. Sucrose is non-reducing sugar because reducing part of (+)-Lactose
O
 Lactose is a reducing sugar and all reducing sugars show
glucose (— C— H) and fructose ( C == O) are involved in mutarotation.
glycosidic linkage. 283. Ion containing positive as well as negative charge is called
CH2OH Zwitter ion.
O Among the given options, only glycine (H2N  CH2  COOH)
H H
H is an amino acid which contains both acidic (acquiring
OH H negative charge) and basic group (acquiring positive
HO O Glycosidic charge).
linkage Glycine can form a Zwitter ion. It is because glycine behave
H HO
like salts rather than simple amines or carboxylic acids. In
O aqueous solution, the carboxyl group can lose a proton and
HOH2C
amino group can accept a proton giving rise to a dipolar ion
OH CH2OH
H
H known as Zwitter ion.
H
H
OH 
Sucrose H C  COO−

While, lactose, glucose and maltose are reducing sugars. NH3
280. CHO CHO +
Zwitter ion
H OH HO H
Zwitter ion is a cation in acidic medium and migrates to
H OH H OH cathode on passing electric current. It is an anion in
basic medium and migrates to anode on passing electric
CH2OH CH2OH
D-erythrose D-threose current.

MODULE 3
www.jeebooks.in
The NEET Edge ~ Chemistry 473

284. In the process of digestion the proteins present in [ CH2  C == CH  CH2  ]n and PVC  [ CH2  CH  ]n
food material are hydrolysed to amino acid. In this  
process two enzymes pepsin and trypsin are involved as Cl Cl
follows: are thermoplastic polymers while nylon-6,6
Pepsin Trypsin
O O
Proteins → Polypeptide →
(Enzyme A ) (Enzyme B )
Amino acids  
[ NH  (CH2 )6  NH  C  (CH2 )4  C ]n is a polyamide
285. The peptide linkage (—NH — CO—) is formed by the which is commonly known as fibre.
condensation of amino acids molecules
290. Neoprene is a polymer of chloroprene (2-chloro -1,3-butadiene)
and also called homopolymer (addition polymer).
HNH CH — COH + H ⋅NH C H — COH →
| || | || nCH2 C—CH CH2
R O R O
—HN CH — C NH CH — C— Cl
Chloroprene Polymerisation
| || | ||
R O R O
Hence, following structure represents the peptide chain
—CH2—C CH—CH2—
H H H O
|  |  |  || Cl n
Neoprene
—N— C— C— N— C— C— N— C— C—
(synthetic rubber)
 ||  || 
O O 291. The monomer of polymer
286. Deprotonation of protein occur when it is subjected to CH3
physical change like change in temperature or chemical  + CH3 CH3
change like change in pH, the hydrogen bonds are —CH2 — C — CH2 — C is CH2 == C
disturbed. As a result, globules unfolds and helix get  CH3 CH3
uncoiled and protein losses its biological activity. CH3
Hence, the denaturation of protein makes the protein because 2-methylpropene shows cationic polymerisation.
inactive.
292. Nylon-6,6 polymer is formed as :
287. In DNA, two helically twisted strands connected together by
steps. Each strand consists of alternating molecules of HOOC—(CH2)4—COOH + H2N—(CH2)6—NH2
deoxyribose at 2 ′-position and phosphate groups. Adipic acid Hexamethylene
diamine
On the other hand, in RNA, the pentose sugar has an
identical structure with deoxyribose sugar except that there Polymerisation
is an —OH group instead of —H on carbon atom 2 ′.
Hence, it is only called ribose.
O O
C—(CH2)4—C—NH— (CH2)6—NH— n
29. Polymers Nylon-6,6
288. Cross-linked or network polymers are formed from
bi-functional and tri-functional monomers and contain strong Thus, option (d) is correct.
covalent bonds between various linear polymer chains. 293. Dacron commonly known as terylene, is obtained by
These are hard, rigid and brittle due to cross-links, e.g. heating a mixture of terephthalic acid and ethylene glycol at
bakelite, melamine etc. Thus, option (d) is incorrect. 420-460 K. In the presence of zinc acetate and antimony
289. trioxide as a catalyst.
OH OH

CH2 CH2 nHOOC COOH + nHO—CH2—CH2—OH
–H2O
Terephthalic acid Ethylene glycol

Novolac, a condensation polymer of phenol and —O—CH


[ 2—CH2—O—C C—] n
formaldehyde is a thermosetting polymer. Neoprene rubber Dacron O O

MODULE 3
www.jeebooks.in
474 NEET Test Drive

294. Neoprene (synthetic rubber) is a polymer of chloroprene, i.e. 296. Diphenylhydramine (benadryl) is used as an antihistamine.
2-chloro-1, 3-butadiene. 297. Novalgin (Dipyrone) is a non-narcotic analgesic used as pain
Polymerisation reliever.
nH2C==C—C==CH2

Penicillin is an antibiotic used for curing rheumatic
Cl H fever.
2-chloro-1,3-butadiene ●
Streptomycin is an antibiotic drug.
(chloroprene) ●
Chloromycetin is an antibiotic drug.
CH2—C==C—CH2 298. Antiseptics and disinfectants both either kill or prevent the
growth of microorganisms. The main point of difference
Cl H n between these two is that the former (antiseptics) are used
Neoprene for living beings whereas disinfectants are not safe for living
(synthetic rubber) tissues. These are actually used for inanimate objects like
floors, tiles, etc.
A substance like phenol in its lower concentration (0.2%)
30. Chemistry in Everyday Life behaves as antiseptic, whereas in higher concentration (1%)
295. Tranquilizers are the chemicals that reduce anxiety and as disinfectant. Chlorine and iodine are strong disinfectants
mental diseases.Tranquilizer is the strain reliever also used whereas dilute solutions of boric acid and hydrogen peroxide
for mild and essential component of sleeping pills. Thus, they are mild antiseptics.
are sometimes called psychotherapeutic drugs. Equanil, 299. Aspartame is the only artificial sweetener which is stable at
valium, serotonin and barbiturates (hypnotic) are some lower temperature and decomposes at higher temperature. It
commonly used tranquilizers. is also called ‘nutra sweet’. It’s relative sweetness value is
180 times sweetes than cane sugar.

MODULE 3
www.jeebooks.in

THE NEET EDGE


Chapterwise Collection of Most Difficult Biology Questions asked in last 15 Years’ NEET/AIPMT

BIOLOGY
1. The Living World 5. Carbohydrates the most abundant biomolecules on
1. The living organisms can be unexceptionally earth, are produced by [CBSE-AIPMT 2005]
distinguished from the non-living things on the basis (a) all bacteria, fungi and algae
of their ability for [CBSE-AIPMT 2007] (b) fungi, algae and green plant cells
(a) responsiveness to touch (c) some bacteria, algae and green plant cells
(b) interaction with the environment and progressive evolution (d) viruses, fungi and bacteria
(c) reproduction 6. The label of a herbarium sheet does not carry
(d) growth and movement information on [NEET 2016, Phase II]
2. How many organisms in the list given below are (a) date of collection (b) name of collector
autotrophs? (c) local names (d) height of the plant
Lactobacillus, Nostoc, Chara, Nitrosomonas, 7. Which one of the following is not a correct
Nitrobacter, Streptomyces, Saccharomyces, statement? [NEET 2013]
Trypanosoma, Porphyra, Wolffia. [CBSE-AIPMT 2012]
(a) Herbarium houses dried, pressed and preserved plant
(a) Four (b) Five (c) Six (d) Three
specimens
3. Nomenclature is governed by certain universal (b) Botanical gardens have collection of living plants for
rules. Which one of the following is contrary to the reference
rules of nomenclature? [NEET 2016, Phase I]
(c) A museum has collection of photographs of plants and
(a) The first word in a biological name represents the genus animals
name and the second is a specific epithet
(d) Key is a taxonomic aid for identification of specimens
(b) The names are written in Latin and are Italicised
(c) When written by hand, the names are to be underlined
(d) Biological names can be written in any language 2. Kingdom—Monera and Viruses
4. Study the four statements (I-IV) given below and 8. Viroids differ from viruses in having [NEET 2017]
select the two correct ones out of them.
[NEET 2016, Phase II] (a) DNA molecules with protein coat
I. Definition of biological species was given by (b) DNA molecules without protein coat
Ernst Mayr. (c) RNA molecules with protein coat
(d) RNA molecules without protein coat
II. Photoperiod does not affect reproduction in
plants. 9. Select the wrong statement. [CBSE-AIPMT 2015]
III. Binomial nomenclature system was given by (a) The viroids were discovered by DJ Ivanowski
RH Whittaker. (b) WM Stanley showed that viruses could be crystallised
IV. In unicellular organisms, reproduction is (c) The term ‘Contagium vivum fluidum’ was coined by MW
synonymous with growth. Beijerinck
The two correct statements are (d) Mosaic disease in tobacco and AIDS in human being are
(a) II and III (b) III and IV (c) I and IV (d) I and II caused by viruses

MODULE 3
www.jeebooks.in
476 NEET Test Drive

10. Which of the following statements is wrong for 18. When a freshwater protozoan possessing a
viroids? [NEET 2016, Phase I] contractile vacuole is placed in a glass containing
(a) They are smaller than viruses marine water, the vacuole will [CBSE-AIPMT 2004]
(b) They cause infections (a) increase in number (b) disappear
(c) Their RNA is of high molecular weight (c) increase in size (d) decrease in size
(d) They lack a protein coat
11. Which of the following statements is not true for 4. Kingdom—Fungi
retroviruses? [CBSE-AIPMT 2004] 19. Cellulose is the major component of cell walls of
(a) DNA is not present at any stage in the life cycle of [CBSE-AIPMT 2008]
retroviruses (a) Pythium (b) Xanthomonas
(b) Retroviruses carry gene for RNA dependent DNA (c) Pseudomonas (d) Saccharomyces
polymerase
(c) The genetic material in mature retroviruses is RNA 20. Which one of the following has haplontic life cycle?
(d) Retroviruses are causative agents for certain kinds of [CBSE-AIPMT 2009]
cancer in man (a) Funaria (b) Polytrichum
(c) Ustilago (d) Wheat
12. Which statement is wrong for viruses?
(a) All are parasites [CBSE-AIPMT 2012] 21. Choose the wrong statement. [CBSE-AIPMT 2015]
(b) All of them have helical symmetry (a) Penicillium is multicellular and produces antibiotics
(c) They have ability to synthesise nucleic acids and proteins (b) Neurospora is used in the study of biochemical genetics
(d) Antibiotics have no effect on them (c) Morels and truffles are poisonous mushrooms
(d) Yeast is unicellular and useful in fermentation
13. Barophilic prokaryotes [CBSE-AIPMT 2005]
(a) grow slowly in highly alkaline frozen lakes at high altitudes 22. Which pair of the following belongs to
(b) occur in water containing high concentrations of barium Basidiomycetes? [CBSE-AIPMT 2007]
hydroxide (a) Bird’s nest fungi and puff balls
(c) grow and multiply in very deep marine sediments (b) Puff balls and Claviceps
(d) readily grown and divide in sea water enriched in any
(c) Peziza and stink horns
soluble salt of barium
(d) Morchella and mushrooms
14. Which one of the following statements about
Mycoplasma is wrong? [CBSE-AIPMT 2007]
23. Trichoderma harzianum has proved a useful
microorganism for [CBSE-AIPMT 2008]
(a) They are also called PPLO
(a) bioremediation of contaminated soils
(b) They are pleomorphic
(b) reclamation of wastelands
(c) They are sensitive to penicillin
(c) gene transfer in higher plants
(d) They cause disease in plants
(d) biological control of soil-borne plant pathogens

3. Kingdom—Protista 24. Ergot of rye is caused by a species of [CBSE-AIPMT 2007]


(a) Phytophthora (b) Uncinula (c) Ustilago (d) Claviceps
15. Select the wrong statement. [NEET 2016, Phase II]
(a) The walls of diatoms are easily destructible 25. Which of the following environmental conditions
(b) ‘Diatomaceous earth’ is formed by the cell walls of are essential for optimum growth of Mucor on a
diatoms piece of bread? [CBSE-AIPMT 2006]
(c) Diatoms are chief producers in the oceans I. Temperature of about 25°C
(d) Diatoms are microscopic and float passively in water II. Temperature of about 5°C
III. Relative humidity of about 5%
16. Auxospores and hormocysts are formed IV. Relative humidity of about 95%
respectively by [CBSE-AIPMT 2005]
V. A shady place
(a) several diatoms and a few cyanobacteria
VI. A brightly illuminated place
(b) several cyanobacteria and several diatoms
(c) some diatoms and several cyanobacteria Choose the answer from the following options.
(a) I, IV and V (b) II, IV and V (c) II, III and VI (d) II, III and V
(d) some cyanobacteria and many diatoms
17. In which group of organisms the cell walls form 5. Plant Kingdom
two thin overlapping shells which fit together?
[CBSE-AIPMT 2015] 26. Zygotic meiosis is the characteristic of [NEET 2017]
(a) Chrysophytes (b) Euglenoids (a) Marchantia (b) Fucus
(c) Dinoflagellates (d) Slime moulds (c) Funaria (d) Chlamydomonas

MODULE 3
www.jeebooks.in
The NEET Edge ~ Biology 477

27. Life cycle of Ectocarpus and Fucus respectively are 36. Read the following statements and answer the
[NEET 2017] question which follows them
(a) haplontic, diplontic (b) diplontic, haplodiplontic I. In liverworts, mosses and ferns gametophytes
(c) haplodiplontic, diplontic (d) haplodiplontic, haplontic are free-living.
28. Which one of the following statements is wrong ? II. Gymnosperms and some ferns are
[NEET 2016, Phase II] heterosporous.
(a) Algae increase the level of dissolved oxygen in the III. Sexual reproduction in Fucus, Volvox and
immediate environment Albugo are oogamous.
(b) Algin is obtained from red algae and carrageenan from brown IV. The sporophyte in liverworts is more
algae elaborate than that in mosses.
(c) Agar-agar is obtained from Gelidium and Gracilaria
(d) Laminaria and Sargassum are used as food
How many of the above statements are correct?
(a) One (b) Two [NEET 2013]
29. Which one is a wrong statement? [CBSE-AIPMT 2015] (c) Three (d) Four
(a) Archegonia are found in bryophytes, pteridophytes and
gymnosperms 37. Select one of the following pairs of important
(b) Mucor has biflagellate zoospores features distinguishing Gnetum from Cycas and
(c) Haploid endosperm is typical feature of gymnosperms Pinus and showing affinities with angiosperms.
(d) Brown algae have chlorophyll-a and c, and fucoxanthin [CBSE-AIPMT 2008]
(a) Absence of resin duct and leaf venation
30. Select the wrong statement. [NEET 2013]
(b) Presence of vessel elements and absence of archegonia
(a) Isogametes are similar in structure, function and behaviour
(c) Perianth and two integuments
(b) Anisogametes differ either in structure, function and
(d) Embryo development and apical meristem
behaviour
(c) In Oomycetes, female gamete is smaller and motile, while 38. Flagellated male gametes are present in all the
male gamete is larger and non-motile three of which one of the following sets?
(d) Chlamydomonas exhibits both isogamy and anisogamy (a) Anthoceros, Funaria and Spirogyra [CBSE-AIPMT 2007]
and Fucus shows oogamy
(b) Zygnema, Saprolegnia and Hydrilla
31. Male and female gametophytes are independent (c) Fucus, Marselia and Calotropis
and free-living in [CBSE-AIPMT 2010] (d) Riccia, Dryopteris and Cycas
(a) mustard (b) castor
(c) Pinus (d) Sphagnum 39. Male gametophyte with least number of cells is
present in [CBSE-AIPMT 2014]
32. Which one of the following is a vascular (a) Pteris (b) Funaria (c) Lilium (d) Pinus
cryptogam? [CBSE-AIPMT 2009]
(a) Equisetum (b) Ginkgo 40. Replum is present in the ovary of flower of
[CBSE-AIPMT 2008]
(c) Marchantia (d) Cedrus
(a) lemon (b) mustard
33. Ectophloic siphonostele is found in [CBSE-AIPMT 2005] (c) sunflower (d) pea
(a) Adiantum and Cucurbitaceae
41. Match items in Column I with those in Column II.
(b) Osmunda and Equisetum
(c) Marsilea and Botrychium Column I Column II
(d) Dicksonia and maiden hair fern A. Peritrichous flagellation 1. Ginkgo
34. Select the correct statement. [NEET 2016, Phase I] B. Living fossil 2. Macrocystis
(a) Salvinia, Ginkgo and Pinus all are gymnosperms C. Rhizophore 3. Escherichia coli
(b) Sequoia is one of the tallest trees D. Smallest flowering plant 4. Selaginella
(c) The leaves of gymnosperms are not well-adapted to
E. Largest perennial alga 5. Wolffia
extremes of climate
(d) Gymnosperms are both homosporous and heterosporous Select the correct answer from the following.
35. The gametophyte is not an independent, free-living Codes [CBSE-AIPMT 2005]
generation in [CBSE-AIPMT 2011] A B C D E
(a) Adiantum (a) 3 1 4 5 2
(b) Marchantia (b) 2 1 3 4 5
(c) Pinus (c) 5 3 2 5 1
(d) Polytrichum (d) 1 2 5 3 2

MODULE 3
www.jeebooks.in
478 NEET Test Drive

6. Animal Kingdom 48. In which one of the following, the genus name, its
two characters and its class/phylum are correctly
42. Which one of the following statements about all the matched? [CBSE-AIPMT 2011]
four Spongilla, leech, dolphin and penguin is
Genus Two characters Class/Phylum
correct? [CBSE-AIPMT 2010]
(a) Salamandra (i) A tympanum
(a) Penguin is homeothermic, while the remaining three are
poikilothermic represents ear Amphibia
(b) Leech is a freshwater form, while all others are marine (ii) Fertilisation is external
(c) Spongilla has special collared cells called choanocytes, (b) Pteropus (i) Skin possesses hair Mammalia
not found in the remaining three (ii) Oviparous
(d) All are bilaterally symmetrical
(c) Aurelia (i) Cnidoblast Coelenterata
43. Ascaris is characterised by the [CBSE-AIPMT 2008] (ii) Organ level of
(a) absence of true coelom but presence of metamerism organisation
(b) presence of neither true coelom nor metamerism (d) Ascaris (i) Body segmented Annelida
(c) presence of true coelom but absence of metamerism (ii) Males and females
(d) presence of true coelom and metamerism distinct
(metamerisation)
49. Which one of the following groups of three animals
44. Which of the following is correctly stated as it each is correctly matched with their one
happens in common cockroach? [CBSE-AIPMT 2011] characteristic morphological feature?
(a) Oxygen is transported by haemoglobin in blood [CBSE-AIPMT 2008]
(b) Nitrogenous excretory product is urea Animals Morphological Feature
(c) The food is grinded by mandibles and gizzard (a) Liver fluke, sea anemone, — Bilateral symmetry
(d) Malpighian tubules are excretory organs projecting out sea cucumber
(b) Centipede, prawn, sea — Jointed appendages
from the colon
urchin
45. A jawless fish, which lays eggs in freshwater and (c) Scorpion, spider, cockroach — Ventral solid central
whose ammocoete larvae after metamorphosis nervous system
(d) Cockroach, locust, Taenia — Metameric segmentation
return to the ocean is [CBSE-AIPMT 2015]
(a) Eptatretus (b) Myxine 50. Which of the following pairs are correctly matched?
(c) Neomyxine (d) Petromyzon [CBSE-AIPMT 2007]

46. In which one of the following, the genus name, its Animals Morphological features
two characters and its phylum are not correctly I. Crocodile — Four-chambered heart
matched, whereas the remaining three are correct? II. Sea urchin— Parapodia
[CBSE-AIPMT 2012] III. Obelia — Metagenesis
Genus name Two characters Phylum IV. Lemur — Thecodont
(a) Pila (i) Body segmented (a) I, III and IV (b) II, III and IV
Mollusca
(ii) Mouth with radula (c) I and IV (d) I and II
(b) Asterias (i) Spiny-skinned 51. In Arthropoda, head and thorax are often fused to
Echinodermata
(ii) Water vascular system form cephalothorax, but in which one of the
(c) Sycon (i) Pore bearing following classes, is the body divided into head,
Porifera thorax and abdomen? [CBSE-AIPMT 2004]
(ii) Canal system
(d) Periplaneta (i) Jointed appendages Arthropoda (a) Insecta
(b) Myriapoda
47. Which one of the following groups of animals is (c) Crustacea
correctly matched with its one characteristic feature (d) Arachnida and Crustacea
without even a single exception? [CBSE-AIPMT 2011]
52. What will you look for to identify the sex of the
(a) Chordata — Possess a mouth provided with an following? [CBSE-AIPMT 2011]
upper and a lower jaw
(a) Male frog A copulatory pad on the first digit of
(b) Chondrichthyes — Possess cartilaginous
the hindlimb
endoskeleton
(b) Female cockroach Anal cerci
(c) Mammalia — Give birth to young ones (c) Male shark Claspers borne on pelvic fins
(d) Reptilia — Possess 3-chambered heart with (d) Female Ascaris Sharply-curved posterior end
one incompletely divided ventricle

MODULE 3
www.jeebooks.in
The NEET Edge ~ Biology 479

53. In which one of the following sets of animals do all 61. Pineapple (ananas) fruit develops from
the four give birth to young ones? [CBSE-AIPMT 2006] (a) a multipistillate syncarpous flower [CBSE-AIPMT 2006]
(a) Lion, bat, whale, ostrich (b) a cluster of compactly borne flowers on a common axis
(b) Platypus, penguin, bat, hippopotamus (c) a multilocular monocarpellary flower
(c) Shrew, bat, cat, kiwi (d) a unilocular polycarpellary flower
(d) Kangaroo, hedgehog, dolphin, loris
62. An example of a seed with endosperm, perisperm
54. From the following statements select the wrong and caruncle is [CBSE-AIPMT 2009]
one. [CBSE-AIPMT 2005] (a) cotton (b) coffee (c) lily (d) castor
(a) Millipedes have two pairs of appendages in each segment
63. Which one of the following statements is correct?
of the body
[CBSE-AIPMT 2011]
(b) Prawn has two pairs of antennae
(a) Seeds of orchids have oil rich endosperm
(c) Animals belonging to phylum–Porifera are exclusively
marine (b) Placentation in primrose is basal
(d) Nematocysts are characteristic of the phylum–Cnidaria (c) Flower of tulip is a modified shoot
(d) In tomato, fruit is a capsule

7. Morphology of Flowering Plants 64. Perisperm differs from endosperm in [NEET 2013]
(a) being a haploid tissue
55. In China rose, the flowers are [NEET 2013]
(b) having no reserve food
(a) actinomorphic, hypogynous with twisted aestivation (c) being a diploid tissue
(b) actinomorphic, epigynous with valvate aestivation (d) its formation by fusion of secondary nucleus with several
(c) zygomorphic, hypogynous with imbricate aestivation sperms
(d) zygomorphic, epigynous with twisted aestivation
56. How many plants among Indigofera, Sesbania, 8. Anatomy of Flowering Plants
Salvia, Allium, Aloe, mustard, groundnut, radish, 65. Palisade parenchyma is absent in leaves of
gram and turnip have stamens with different [CBSE-AIPMT 2009]
lengths in their flowers? [NEET 2016, Phase II]
(a) Sorghum (b) mustard
(a) Three (b) Four (c) soybean (d) gram
(c) Five (d) Six
66. Tracheids differ from other tracheary elements in
57. Among China rose, mustard, brinjal, potato, guava, [CBSE-AIPMT 2014]
cucumber, onion and tulip, how many plants have (a) having Casparian strips (b) being imperforate
superior ovary? [CBSE-AIPMT 2015]
(c) lacking nucleus (d) being lignified
(a) Five (b) Six
(c) Three (d) Four 67. Water containing cavities in vascular bundles are
found in [CBSE-AIPMT 2012]
58. How many plants in the list given below have
(a) sunflower (b) maize
composite fruits that develop from an
(c) Cycas (d) Pinus
inflorescence?
Walnut, Poppy, Radish, Fig, Pineapple, Apple, 68. Passage cells are thin-walled cells found in
[CBSE-AIPMT 2007]
Tomato, Mulberry [CBSE-AIPMT 2012]
(a) endodermis of roots facilitating rapid transport of water
(a) Four (b) Five
from cortex to pericycle
(c) Two (d) Three
(b) phloem elements that serve as entry points for substances
59. Among bitter gourd, mustard, brinjal, pumpkin, for transport to other plant parts
China rose, lupin, cucumber, sunnhemp, gram, (c) testa of seeds to enable emergence of growing embryonic
guava, bean, chilli, plum, Petunia, tomato, rose, axis during seed germination
Withania, potato, onion, Aloe and tulip, how many (d) central region of style through which the pollen tube grows
plants have hypogynous flower? [NEET 2013] towards the ovary
(a) Six (b) Ten
(c) Fifteen (d) Eighteen 69. The balloon-shaped structures called tyloses
[NEET 2016, Phase II]
60. The correct floral formula of chilli is (a) originate in the lumen of vessels
[CBSE-AIPMT 2011]
_ _ (b) characterise the sapwood
(a) Å O
+ K( 5 ) C( 5 ) A 5G( 2 ) (b) Å O
+ K( 5 ) C( 5 ) A( 5 )G 2 (c) are extensions of xylem parenchyma cells into vessels
_ _ (d) are linked to the ascent of sap through xylem vessels
(c) Å O
+K 5 C 5 A( 5 )G 2 (d) Å O
+K ( 5) C 5 A 5G( 2 )

MODULE 3
www.jeebooks.in
480 NEET Test Drive

9. Structural Organisation 10. Cell : The Unit of Life


in Animals 77. A major breakthrough in the studies of cells came
with the development of electron microscope. This
70. Choose the correctly matched pair. [CBSE-AIPMT 2014] is because [CBSE-AIPMT 2006]
(a) Inner lining of salivary ducts – Ciliated epithelium
(a) the resolving power of the electron microscope is
(b) Moist surface of buccal cavity – Glandular epithelium 200-350 nm as compared to 0.1-0.2 for the light microscope
(c) Tubular parts of nephrons – Cuboidal epithelium (b) electron beam can pass through thick materials, whereas
(d) Inner surface of bronchioles – Squamous epithelium light microscopy required thin sections
(c) the electron microscope is more powerful than the light
71. The function of the gap junction is to
microscope as it uses a beam of electrons which has
[CBSE-AIPMT 2015]
wavelength much longer than that of photons
(a) performing cementing to keep neighbouring cells together
(d) the resolution power of the electron microscope is much
(b) facilitate communication between adjoining cells by
higher than that of the light microscope
connecting the cytoplasm for rapid transfer of ions, small
molecules and some large molecules 78. A student wishes to study the cell structure under
(c) separate two cells from each other a light microscope having 10X eyepiece and 45X
(d) stop substance from leaking across a tissue objective. He should illuminate the object by which
72. Choose the correctly matched pair. [CBSE-AIPMT 2014] one of the following colours of light so as to get the
(a) Tendon – Specialised connective tissue best possible resolution? [CBSE-AIPMT 2005]
(b) Adipose tissue – Dense connective tissue (a) Blue (b) Green
(c) Areolar tissue – Loose connective tissue (c) Yellow (d) Red
(d) Cartilage – Loose connective tissue 79. Which one of the following does not differ in E. coli
and Chlamydomonas ? [CBSE-AIPMT 2012]
73. Which type of white blood cells are concerned with
the release of histamine and the natural (a) Ribosomes (b) Chromosomal organisation
anticoagulant heparin? [CBSE-AIPMT 2008] (c) Cell wall (d) Cell membrane
(a) Neutrophils (b) Basophils (c) Eosinophils (d) Monocytes
80. What is true about ribosomes? [CBSE-AIPMT 2012]
74. Which type of tissue correctly matches with its (a) The prokaryotic ribosomes are 80S, where S stands for
location? [NEET 2016, Phase I] sedimentation coefficient
(b) These are composed of ribonucleic acid and proteins
Tissue Location
(c) These are found only in eukaryotic cells
(a) Areolar tissue Tendons (d) These are self-splicing introns of some RNAs
(b) Transitional epithelium Tip of nose 81. Keeping in view the ‘fluid mosaic model’ for the
(c) Cuboidal epithelium Lining of stomach structure of cell membrane, which one of the
(d) Smooth muscle Wall of intestine following statements is correct with respect to the
movement of lipids and proteins from one lipid
75. Which one of the following correctly describes the monolayer to the other (described as flip-flop
location of some body parts in the earthworm
Pheretima? [CBSE-AIPMT 2009]
movement)? [CBSE-AIPMT 2008]
(a) Both lipids and proteins can flip-flop
(a) Two pairs of accessory glands in 16th -18th segments
(b) While lipids can rarely flip-flop, proteins cannot
(b) Four pairs of spermathecae in 4th -7th segments
(c) While proteins can flip-flop, lipids cannot
(c) One pair of ovaries attached at intersegmental septum of
(d) Neither lipids, nor proteins can flip-flop
14th and 15th segments
(d) Two pairs of testis in 10th and 11th segments 82. Cellular organelles with membranes are
[CBSE-AIPMT 2015]
76. Select the correct route for the passage of sperms
in male frogs. [NEET 2017] (a) nuclei, ribosomes and mitochondria
(b) chromosomes, ribosomes and endoplasmic reticulum
(a) Testes ® Bidder’s canal ® Kidney ® Vasa efferentia ®
Urinogenital duct ® Cloaca (c) endoplasmic reticulum, ribosomes and nuclei
(b) Testes ® Vasa efferentia ® Kidney ® Seminal vesicle ® (d) lysosomes, Golgi apparatus and mitochondria
Urinogenital duct ® Cloaca 83. Which of the following statements regarding
(c) Testes ® Vasa efferentia ® Bidder’s canal ® Ureter ®
mitochondrial membrane is not correct?
Cloaca
[CBSE-AIPMT 2006]
(d) Testes ® Vasa efferentia ® Kidney ® Bidder’s canal ®
(a) The enzymes of the electron transfer chain are embedded
Urinogenital duct ® Cloaca
in the outer membrane

MODULE 3
www.jeebooks.in
The NEET Edge ~ Biology 481

(b) The inner membrane is highly convoluted forming a series 92. Which one of the following structural formula of
of infoldings two organic compounds is correctly identified along
(c) The outer membrane resembles a sieve with its related function? [CBSE-AIPMT 2011]
(d) The outer membrane is permeable to all kinds of O
molecules
O CH2—O—C—R
84. The two subunits of ribosome remain united at a
critical ion level of [CBSE-AIPMT 2008] R2—C—O—CH O
(a) copper (b) manganese
(c) magnesium (d) calcium CH2—O—P—O—CH2—CH2

85. Which one of the following also acts as a catalyst in OH N


a bacterial cell? [CBSE-AIPMT 2011] CH3 CH3
CH3
(a) sn RNA (b) hnRNA
(A)
(c) 23S rRNA (d) 5S rRNA
NH2
86. Flagella of prokaryotic and eukaryotic cells N
differ in [CBSE-AIPMT 2004] N
(a) type of movement and placement in cell
N NH
(b) location in cell and mode of functioning
(c) microtubular organisation and type of movement (B)
(d) microtubular organisation and function
(a) A — Triglyceride — Major source of energy
87. Which of the following is true for nucleolus? (b) B — Uracil — A component of DNA
(a) It takes part in spindle formation [CBSE-AIPMT 2006]
(c) A — Lecithin — A component of cell membrane
(b) It is a membrane-bound structure (d) B — Adenine — A nucleotide that makes up
(c) Larger nucleoli are present in dividing cells nucleic acids
(d) It is a site for active ribosomal RNA synthesis
93. A phosphoglyceride is always made up of [NEET 2013]
(a) only a saturated fatty acid esterified to a glycerol molecule
11. Biomolecules to which a phosphate group is also attached
(b) only an unsaturated fatty acid esterified to a glycerol
88. About 98 per cent of the mass of every living molecule to which a phosphate group is also attached
organism is composed of just six elements (c) a saturated or unsaturated fatty acid esterified to a glycerol
including carbon, hydrogen, nitrogen, oxygen molecule to which a phosphate group is also attached
and [CBSE-AIPMT 2007] (d) a saturated or unsaturated fatty acid esterified to a
(a) phosphorus and sulphur (b) sulphur and magnesium phosphate group, which is also attached to a glycerol
(c) magnesium and sodium (c) calcium and phosphorus molecule
89. Which one of the following statements is wrong? 94. Given below is the diagrammatic representation of
[NEET 2016, Phase I] one of the categories of small molecular weight
(a) Cellulose is a polysaccharide organic compounds in the living tissues. Identify
(b) Uracil is a pyrimidine the category shown and the one blank component
(c) Glycine is a sulphur containing amino acid X in it.
(d) Sucrose is a disaccharide O X
HOCH2
90. The chitinous exoskeleton of arthropods is formed
by the polymerisation of [CBSE-AIPMT 2015]
(a) keratin sulphate and chondroitin sulphate
(b) D-glucosamine
OH OH
(c) N-acetyl glucosamine
(d) lipoglycans [CBSE-AIPMT 2012]
Category Component
91. Macromolecule chitin is [NEET 2013]
(a) Cholesterol — Guanine
(a) nitrogen containing polysaccharide (b) Amino acid — NH2
(b) phosphorus containing polysaccharide (c) Nucleotide — Adenine
(c) sulphur containing polysaccharide (d) Nucleoside — Uracil
(d) simple polysaccharide

MODULE 3
www.jeebooks.in
482 NEET Test Drive

95. Enzymes, vitamins and hormones can be classified 100. Given below is a schematic breakup of the
into a single category of biological chemicals, phases/stages of cell cycle
because all of these [CBSE-AIPMT 2005] A
(a) help in regulating metabolism
(b) are exclusively synthesised in the body of a living organism B C
as at present
(c) are conjugated proteins Mitosis
(d) enhance oxidative metabolism
Interphase
96. Which of the following statements regarding
enzyme inhibition is correct? [CBSE-AIPMT 2005] D
(a) Competitive inhibition is seen when a substrate competes
with an enzyme for binding to an inhibitor protein
(b) Competitive inhibition is seen when the substrate and the Which one of the following is the correct
inhibitor compete for the active site on the enzyme indication of the stage/phase in the cell
(c) Non-competitive inhibition of an enzyme can be overcome cycle? [CBSE-AIPMT 2009]
by adding large amount of substrate (a) B–Metaphase
(d) Non-competitive inhibitors often bind to the enzyme (b) C–Karyokinesis
irreversibly (c) D–Synthetic phase
(d) A–Cytokinesis
97. Which of the following describes the given graph
correctly? [NEET 2016, Phase II] 101. Which one of the following precedes reformation of
the nuclear envelope during M-phase of the cell
B cycle? [CBSE-AIPMT 2004]
(a) Decondensation from chromosomes and reassembly of
Potential energy

A the nuclear lamina


Substrate (b) Transcription from chromosomes and reassembly of the
nuclear lamina
(c) Formation of the contractile ring and formation of the
phragmoplast
Product
(d) Formation of the contractile ring and transcription from
Reaction
chromosomes
(a) Endothermic reaction with energy A in the presence of
enzyme and B in the absence of enzyme 102. A stage in cell division is shown in the figure.
(b) Exothermic reaction with energy A in the presence of Select the answer which gives correct
enzyme and B in the absence of enzyme identification of the stage with its characteristic
(c) Endothermic reaction with energy A in the absence of mentioned. [NEET 2013]
enzyme and B in the presence of enzyme
(d) Exothermic reaction with energy A in the absence of
enzyme and B in the presence of enzyme
98. Select the option which is not correct with respect
to enzyme action. [CBSE-AIPMT 2014]
(a) Substrate binds with enzyme as its active site
(b) Addition of a lot of succinate does not reverse the
inhibition of succinic dehydrogenase by malonate
(c) A non-competitive inhibitor binds the enzyme at a site
distinct from that which binds the substrate
(d) Malonate is a competitive inhibitor of succinic (a) Telophase — Nuclear envelope reforms, Golgi
dehydrogenase complex reforms
(b) Late anaphase — Chromosomes move away from
equatorial plate, Golgi complex not
12. Cell Cycle and Cell Division present
(c) Cytokinesis — Cell plate formed, mitochondria
99. During which phase(s) of cell cycle, amount of DNA distributed between two daughter
in a cell remains at 4C level if the initial amount is cells
denoted as 2C? [CBSE-AIPMT 2014] (d) Telophase — Endoplasmic reticulum and
(a) G 0 and G1 (b) G1 and S (c) Only G 2 (d) G 2 and M nucleolus not reformed yet

MODULE 3
www.jeebooks.in
The NEET Edge ~ Biology 483

103. Match the stages of meiosis in Column I to their 108. A column of water within xylem vessels of tall trees
characteristic features in Column II and select the does not break under its weight because of
correct option using the codes given below. (a) dissolved sugars in water [CBSE-AIPMT 2015]
[NEET 2016, Phase II] (b) tensile strength of water
(c) lignification of xylem vessels
Column I Column II
(d) positive root pressure
A. Pachytene 1. Pairing of homologous chromosomes
B. Metaphase-I 2. Terminalisation of chiasmata 109. A few drops of sap were collected by cutting across a
plant stem by a suitable method. The sap was
C. Diakinesis 3. Crossing-over takes place
tested chemically.
D. Zygotene 4. Chromosomes align at equatorial plate
Which one of the following test results indicates
Codes that it is phloem sap? [NEET 2016, Phase II]
A B C D A B C D (a) Acidic (b) Alkaline
(a) 3 4 2 1 (b) 1 4 2 3 (c) Low refractive index (d) The absence of sugar
(c) 2 4 3 1 (d) 4 3 2 1
110. The translocation of organic solutes in sieve tube
104. When cell has stalled DNA replication fork, members is supported by [CBSE-AIPMT 2006]
which checkpoint should be predominantly (a) P-proteins
activated? [NEET 2016, Phase II] (b) mass flow involving a carrier and ATP
(a) G1 /S (b) G 2 /M (c) cytoplasmic streaming
(c) M (d) Both G 2 /M and M (d) root pressure and transpiration pull

13. Transport in Plants 14. Mineral Nutrition


105. Two cells A and B are contiguous. Cell A has 111. In which of the following, all three are
osmotic pressure 10 atm, turgor pressure 7 atm macronutrients? [NEET 2016, Phase I]
and diffusion pressure deficit 3 atm. Cell B has (a) Iron, copper, molybdenum
osmotic pressure 8 atm, turgor pressure 3 atm and (b) Molybdenum, magnesium, manganese
diffusion pressure deficit 5 atm. The result will be (c) Nitrogen, nickel, phosphorus
[CBSE-AIPMT 2007] (d) Boron, zinc, manganese
(a) movement of water from cell B to A
(b) no movement of water 112. The deficiencies of micronutrients not only affect
(c) equilibrium between the two
growth of plants but also vital functions such as
photosynthetic and mitochondrial electron flow.
(d) movement of water from cell A to B
Among the list given below, which group of three
106. Which of the following facilitates opening of elements shall affect most, both photosynthetic
stomatal aperture? [NEET 2017] and mitochondrial electron transport?
(a) Contraction of outer wall of guard cells [CBSE-AIPMT 2005]
(b) Decrease in turgidity of guard cells (a) Co, Ni, Mo (b) Ca, K, Na
(c) Radial orientation of cellulose microfibrils in the cell wall of (c) Mn, Co, Ca (d) Cu, Mn, Fe
guard cells 113. Which one of the following elements in plants is
(d) Longitudinal orientation of cellulose microfibrils in the cell not remobilised? [CBSE-AIPMT 2011]
wall of guard cells
(a) Calcium (b) Potassium
107. Water vapour comes out from the plant leaf (c) Sulphur (d) Phosphorus
through the stomatal opening. Through the same
114. Which one of the following is correctly matched?
stomatal opening carbon dioxide diffuses into the
plant during photosynthesis. Reason out the above (a) Passive transport of nutrients – ATP [CBSE-AIPMT 2012]

statements using the following options. (b) Apoplast – Plasmodesmata


[NEET 2016, Phase I] (c) Potassium – Readily immobilisation
(a) Both processes can happen together because the (d) Bakane of rice seedlings – F Skoog
diffusion coefficient of water and CO 2 is different 115. Which one of the following helps in the absorption
(b) The above processes happen only during night-time of phosphorus from soil by plants?[CBSE-AIPMT 2011]
(c) One process occurs during daytime and the other at night (a) Rhizobium (b) Frankia
(d) Both processes cannot happen simultaneously (c) Anabaena (d) Glomus

MODULE 3
www.jeebooks.in
484 NEET Test Drive

116. Nitrogen-fixation in root nodules of Alnus is 125. As compared to a C 3 -plant, how many additional
brought about by [CBSE-AIPMT 2009, 08] molecules of ATP are needed for the net production
(a) Bradyrhizobium (b) Clostridium of one molecule of hexose sugar by C 4-plants?
(c) Frankia (d) Azorhizobium [CBSE-AIPMT 2005]
(a) 2 (b) 6 (c) 12 (d) zero
117. Which of the following is a flowering plant with
nodules containing filamentous nitrogen-fixing 126. Photosynthesis in C 4-plants is relatively less
microorganisms? [CBSE-AIPMT 2007] limited by atmospheric CO2 levels because
(a) Casuarina equisetifolia (b) Crotalaria juncea [CBSE-AIPMT 2005]
(c) Cycas revoluta (d) Cicer arietinum (a) effective pumping of CO 2 into bundle sheath cells
(b) RuBisCO in C 4 -plants has higher affinity for CO 2
(c) four carbon acids are the primary initial CO 2 -fixation
15. Photosynthesis in Higher Plants products
118. Plants adapted to low light intensity have (d) the primary fixation of CO 2 is mediated via PEP
[CBSE-AIPMT 2004] carboxylase
(a) larger photosynthetic unit size than the sun plants 127. With reference to factors affecting the rate of
(b) higher rate of CO 2 fixation than the sun plants
photosynthesis, which of the following statements
(c) more extended root system
is not correct? [NEET 2017]
(d) leaves modified to spines
(a) Light saturation for CO 2 -fixation occurs at 10% of full
119. Emerson’s enhancement effect and red drop have sunlight
been instrumental in the discovery of (b) Increasing atmospheric CO 2 concentration upto 0.05% can
[NEET 2016, Phase I] enhance CO 2 -fixation rate
(a) two photosystems operating simultaneously (c) C 3 -plants respond to higher temperature with enhanced
(b) photophosphorylation and cyclic electron transport photosynthesis, while C 4 -plants have much lower
(c) oxidative phosphorylation temperature optimum
(d) Tomato is a greenhouse crop, which can be grown in CO 2
(d) photophosphorylation and non-cyclic electron transport
enriched atmosphere for higher yield
120. In photosystem-I, the first electron acceptor is
[CBSE-AIPMT 2006]
(a) cytochrome (b) plastocyanin
16. Respiration in Plants
(c) an iron-sulphur protein (d) ferredoxin 128. In which one of the following processes CO 2 is not
121. Anoxygenic photosynthesis is the characteristic of released? [CBSE-AIPMT 2014]
[CBSE-AIPMT 2014] (a) Aerobic respiration in plants
(a) Rhodospirillum (b) Spirogyra (b) Aerobic respiration in animals
(c) Chlamydomonas (d) Ulva (c) Alcoholic fermentation
(d) Lactate fermentation
122. PGA as the first CO2 -fixation product was discovered
in the photosynthesis of [CBSE-AIPMT 2010] 129. How many ATP molecules could maximally be
(a) bryophyte (b) gymnosperm generated from one molecule of glucose, if the
(c) angiosperm (d) alga complete oxidation of one mole of glucose to CO 2 and
H 2 O yields 686 kcal and the useful chemical energy
123. The oxygen evolved during photosynthesis comes available in the high energy phosphate bond of one
from water molecules. Which one of the following mole of ATP is 12 kcal? [CBSE-AIPMT 2006]
pairs of elements involved in this reaction? (a) 30 (b) 57 (c) 1 (d) 2
(a) Manganese and chlorine [NEET 2016, Phase I]
130. The chemiosmotic coupling hypothesis of oxidative
(b) Manganese and potassium phosphorylation proposes that Adenosine
(c) Magnesium and molybdenum Triphosphate (ATP) is formed because
(d) Magnesium and chlorine [CBSE-AIPMT 2008]

124. During photorespiration, the oxygen consuming (a) high energy bonds are formed in mitochondrial proteins
(b) ADP is pumped out of the matrix into the intermembrane
reactions occur in the [CBSE-AIPMT 2006]
space
(a) stroma of chloroplasts and peroxisomes (c) a proton gradient forms across the inner membrane
(b) grana of chloroplasts and peroxisomes (d) there is a change in the permeability of the inner
(c) stroma of chloroplasts mitochondrial membrane toward Adenosine Diphosphate
(d) stroma of chloroplasts and mitochondria (ADP)

MODULE 3
www.jeebooks.in
The NEET Edge ~ Biology 485

131. What is the role of NAD+ in cellular respiration? (b) It is the basis for quantitative determination of small
(a) It is a nucleotide source of ATP synthesis [NEET 2018] amounts of growth-promoting substances
(c) It supports the hypothesis that IAA is auxin
(b) It functions as an electron carrier
(d) It demonstrated polar movement of auxins
(c) It functions as an enzyme
(d) It is the final electron acceptor for anaerobic respiration 137. During seed germination, its stored food is
mobilised by [NEET 2013]
132. All enzymes of TCA cycle are located in the
(a) ethylene (b) cytokinin (c) ABA (d) gibberellin
mitochondrial matrix except one which is located
in inner mitochondrial membranes in eukaryotes 138. Which one of the following acids is a derivative of
and in cytosol in prokaryotes. This enzyme is carotenoids? [CBSE-AIPMT 2009]
(a) lactate dehydrogenase [CBSE-AIPMT 2007] (a) Indole-butyric acid (b) Indole-3-acetic acid
(b) isocitrate dehydrogenase (c) Gibberellic acid (d) Abscisic acid
(c) malate dehydrogenase
139. Importance of day length in flowering of plants
(d) succinate dehydrogenase
was first shown in [CBSE-AIPMT 2008]
133. Which statement is wrong for Krebs’ cycle? (a) Lemna (b) tobacco (c) cotton (d) Petunia
[NEET 2017]
140. Phytochrome is a [NEET 2016, Phase II]
(a) There are three points in the cycle where NAD+ is reduced
to NADH + H+ (a) flavoprotein (b) glycoprotein
(b) There is one point in the cycle where FAD + is reduced to (c) lipoprotein (d) chromoprotein
FADH2 141. Anthesis is a phenomenon which refers to
(c) During conversion of succinyl Co-A to succinic acid, a
(a) reception of pollen by stigma [CBSE-AIPMT 2004]
molecule of GTP is synthesised
(b) formation of pollen
(d) The cycle starts with condensation of acetyl group (acetyl
Co-A) with pyruvic acid to yield citric acid (c) development of anther
(d) opening of flower bud
134. The three boxes in this diagram represent the
three major biosynthetic pathways in aerobic 142. One set of a plant was grown at 12 hr day and 12
respiration. Arrows represent net reactants or hr night period cycles and it flowered while in the
products [NEET 2013] other set, night phase was interrupted by flash of
9 10 light and it did not produce flower. Under which
1 5
one of the following categories will you place this
Glucose Pathway A 2 Pathway B 6 Pathway C 11
7 plant? [CBSE-AIPMT 2004]
4 8 12 (a) Long-day (b) Darkness neutral
3 (c) Day neutral (d) Short-day
Arrows numbered 4, 8 and 12 can all be 143. An enzyme that can stimulate germination of
(a) NADH (b) ATP barley seeds is [CBSE-AIPMT 2006]
(c) H2O (d) FAD+ or FADH2 (a) lipase (b) protease (c) invertase (d) a-amylase
135. Which one of these statements is incorrect?
[NEET 2018] 18. Digestion and Absorption
(a) Glycolysis operates as long as it is supplied with NAD that
can pick up hydrogen atoms 144. Which cells of ‘Crypts of Lieberkuhn’ secrete
(b) Glycolysis occurs in cytosol antibacterial lysozyme ? [NEET 2017]
(c) Enzymes of TCA cycle are present in mitochondrial matrix (a) Argentaffin cells (b) Paneth cells
(d) Oxidative phosphorylation takes place in outer (c) Zymogen cells (d) Kupffer cells
mitochondrial membrane 145. Which of the following gastric cells indirectly help
in erythropoiesis? [NEET 2018]
17. Plant Growth and Development (a) Goblet cells (b) Mucous cells
(c) Chief cells (d) Parietal cells
136. Dr. F Went noted that if coleoptile tips were
removed and placed on agar for one hour, the agar 146. If for some reason our goblet cells are non-
would produce a bending when placed on one side functional, this will adversely affect the
of freshly cut coleoptile stumps. Of what (a) production of somatostatin [CBSE-AIPMT 2010]
significance is this experiment? [CBSE-AIPMT 2014] (b) secretion of sebum from the sebaceous glands
(a) It made possible the isolation and exact identification of (c) maturation of sperms
auxin (d) smooth movement of food down the intestine

MODULE 3
www.jeebooks.in
486 NEET Test Drive

147. The initial step in the digestion of milk in humans D. Occurs in those countries where the staple diet is
is carried out by [CBSE-AIPMT 2014, 11] polished rice.
(a) lipase (b) trypsin (c) rennin (d) pepsin E. The symptoms are pain from neuritis, paralysis,
148. Which one of the following is the correct matching muscle wasting, progressive oedema, mental
of the site of action on the given substrate, the deterioration and finally heart failure.
enzyme acting upon it and the end product? (a) A, B and D (b) B, C and E
[CBSE-AIPMT 2008] (c) A, C and E (d) B, D and E
(a) Duodenum : Triglycerides trypsin monoglycerides 153. Which one of the following is a fat- soluble
(b) Small intestine : Starch a-amylase disaccharide (maltose) vitamin and its related deficiency disease?
(c) Small intestine : Proteins pepsin amino acids (a) Ascorbic acid — Scurvy [CBSE-AIPMT 2007]
(d) Stomach : Fats, Lipase micelles (b) Retinol — Xerophthalmia
149. Select the correct match of the digested products in (c) Cobalamine — Beri-beri
humans given in Column I with their absorption (d) Calciferol — Pellagra
site and mechanism in Column II. [NEET 2013]

Column I Column II 19. Breathing and Exchange of Gases


(a) Glycine and glucose Small intestine and active 154. The figure shows a diagrammatic view of human
absorption respiratory system with labels A, B, C and D. Select
+ the option, which gives correct identification and
(b) Fructose and Na Small intestine and passive
main function and/or characteristic. [NEET 2013]
absorption
(c) Glycerol and fatty acids Duodenum and move as
chilomicrons A
(d) Cholesterol and maltose Large intestine and active
absorption Bronchus

Cut end of rib B


150. Which one of the following statements is true
regarding digestion and absorption of food in Lung
humans? [CBSE-AIPMT 2009]
(a) Oxyntic cells in our stomach secrete the proenzyme D C
pepsinogen Heart
(b) Fructose and amino acids are absorbed through intestinal
mucosa with the help of carrier ions like Na + (a) A–Trachea–Long tube supported by complete
(c) Chylomicrons are small lipoprotein particles that are cartilaginous rings for conducting inspired air
transported from intestine into blood capillaries (b) B–Pleural membrane–Surrounds ribs on both sides to
(d) About 60% of starch is hydrolysed by salivary amylase in provide cushion against rubbing
our mouth (c) C–Alveoli–Thin-walled vascular bag-like structures for the
exchange of gases
151. The richest sources of vitamin–B12 are (d) D–Lower end of lungs–Diaphragm pulls it down during
(a) goat’s liver and Spirulina [CBSE-AIPMT 2004] inspiration
(b) chocolate and green gram 155. Lungs do not collapse between breaths and some
(c) rice and hen’s egg air always remains in the lungs which can never
(d) carrot and chicken’s breast be expelled because [NEET 2016, Phase II]
(a) there is a negative pressure in the lungs
152. Which group of three of the following five
(b) there is a negative intrapleural pressure pulling at the lung
statements (A–E) contains all three correct
walls
statements regarding beri-beri ? [CBSE-AIPMT 2005]
(c) there is a positive intrapleural pressure.
A. A crippling disease prevalent among the native (d) pressure in the lungs is higher than the atmospheric
population of sub-Sahara Africa. pressure
B. A deficiency disease caused by lack of thiamine
156. Reduction in the pH of blood will [NEET 2016, Phase I]
(vitamin-B1).
(a) reduce the blood supply to the brain
C. A nutritional disorder in infants and young
(b) decrease the affinity of haemoglobin with oxygen
children when the diet is persistently deficient in
(c) release bicarbonate ions by the liver
essential protein.
(d) reduce the rate of heartbeat

MODULE 3
www.jeebooks.in
The NEET Edge ~ Biology 487

157. Listed below are four respiratory capacities (1 - 4) Codes


and four jumbled respiratory volumes of a normal A B C A B C
human adult (a) 1 3 2 (b) 1 2 3
Respiratory Capacities Respiratory Volumes (c) 3 2 1 (d) 2 3 1
1. Residual volume 2500 mL 163. You are required to draw blood from a patient and
2. Vital capacity 3500 mL to keep it in a test tube for analysis of blood
3. Inspiratory reserve volume 1200 mL corpuscles and plasma. You are also provided with
4. Inspiratory capacity 4500 mL the following four types of test tubes. Which of them
Which one of the following is the correct matching of will you not use for the purpose? [CBSE-AIPMT 2004]
two capacities and volumes? [CBSE-AIPMT 2010] (a) Test tube containing calcium bicarbonate
(a) (2) 2500 mL, (3) 4500 mL (b) (3) 1200 mL, (4) 2500 mL (b) Chilled test tube
(c) (4) 3500 mL, (1) 1200 mL (d) (1) 4500 mL, (2) 3500 mL (c) Test tube containing heparin
(d) Test tube containing sodium oxalate
158. What is the vital capacity of our lungs?
[CBSE-AIPMT 2008] 164. Blood pressure in the pulmonary artery is
(a) Inspiratory reserve volume plus tidal volume (a) more than that in the carotid [NEET 2016, Phase I]
(b) Total lung capacity minus expiratory reserve volume (b) more than that in the pulmonary vein
(c) Inspiratory reserve volume plus expiratory reserve volume (c) less than that in the vena cava
(d) Total lung capacity minus residual volume (d) same as that in the aorta
159. Blood analysis of a patient reveals an unusually 165. Figure shows schematic plan of blood circulation in
high quantity of carboxyhaemoglobin content. human with labels A-D. Identify the correct label
Which of the following conclusions is most likely to with given function. [NEET 2013]
be correct? [CBSE-AIPMT 2004]
(a) The patient has been inhaling polluted air containing D
unusually high content of carbon disulphide A
(b) The patient has been inhaling polluted air containing
unusually high content of chloroform
(c) The patient has been inhaling polluted air containing
unusually high content of carbon dioxide C B
(d) The patient has been inhaling polluted air containing
unusually high content of carbon monoxide

20. Body Fluids and Circulation (a) A–Pulmonary vein–takes impure blood from body parts,
160. Which one of the following animals has two pO 2 = 60 mm Hg
separate circulatory pathways? [CBSE-AIPMT 2015] (b) B–Pulmonary artery–takes blood from heart to lungs,
(a) Frog (b) Lizard pO 2 = 90 mm Hg
(c) Whale (d) Shark (c) C–Vena cava–takes blood from body parts to right auricle,
pCO 2 = 45 mm Hg
161. The most popularly known blood grouping is the (d) D–Dorsal aorta–takes blood from heart to body parts,
ABO grouping. It is named ABO and not ABC pO 2 = 95 mm Hg
because ‘O’ in it refers to having [CBSE-AIPMT 2009]
(a) other antigens besides A and B on RBCs 166. If due to some injury the chordae tendinae of the
(b) overdominance of this type on the genes for A and B types tricuspid valve of the human heart is partially
(c) one antibody only—either anti-A or anti-B on the RBCs non-functional, what will be the immediate effect?
[CBSE-AIPMT 2010]
(d) no antigens A and B on RBCs
(a) The flow of blood into the aorta will be slowed down
162. Match the items given in Column I with those in (b) The ‘pacemaker’ will stop working
Column II and select the correct option given below. (c) The blood will tend to flow back into the left atrium
[NEET 2018] (d) The flow of blood into the pulmonary artery will be reduced
Column I Column II 167. If you suspect major deficiency of antibodies in a
A. Fibrinogen 1. Osmotic balance person, to which of the following would you look for
confirmatory evidence? [CBSE-AIPMT 2007]
B. Globulin 2. Blood clotting
(a) Serum albumins (b) Serum globulins
C. Albumin 3. Defence mechanism (c) Fibrinogen in the plasma (d) Haemocytes

MODULE 3
www.jeebooks.in
488 NEET Test Drive

21. Excretory Products and 171. Which one of the following correctly explains the
function of a specific part of a human nephron?
Their Elimination [CBSE-AIPMT 2011]

168. Which one of the following pairs of items correctly (a) Henle’s loop — Most reabsorption of the major
substances from the glomerular
belongs to the category of organs mentioned
filtrate
against it? [CBSE-AIPMT 2008]
(b) Distal convoluted — Reabsorption of ions into the
(a) Thorn of Bougainvillea and tendrils — Analogous organs tubule surrounding blood capillaries
of Cucurbita
(c) Afferent arteriole — Carries the blood away from the
(b) Nictitating membrane and blind — Vestigial organs glomerulus towards renal vein
spot in human eye
(d) Podocytes — Create minute spaces (slit
(c) Nephridia of earthworm and — Excretory organs pores) for the filtration of blood
Malpighian tubules of cockroach
into the Bowman’s capsule
(d) Wings of honeybee and wings of — Homologous organs
crow 172. Which one of the following statements in regard to
the excretion by the human kidneys is correct?
169. Consider the following four statements about [CBSE-AIPMT 2010]
certain desert animals such as kangaroo rat.
(a) Descending limb of loop of Henle is impermeable to water
I. They have dark colour and high rate of (b) Distal convoluted tubule is incapable of reabsorbing HCO 3
reproduction and excrete solid urine. (c) Nearly 99% of the glomerular filtrate is reabsorbed by the
II. They do not drink water, breathe at a slow rate to renal tubules
conserve water and have their body covered with (d) Ascending limb of loop of Henle is impermeable to
electrolytes
thick hairs.
III. They feed on dry seeds and do not require 173. If Henle’s loop were absent from mammalian
drinking water. nephron, which of the following is to be expected?
IV. They excrete very concentrated urine and do not (a) The urine will be more concentrated [CBSE-AIPMT 2003]
use water to regulate body temperature. (b) The urine will be more dilute
(c) There will be no urine formation
Which two of the above statements for such (d) There will be hardly any change in the quality and quantity
animals are true? [CBSE-AIPMT 2008]
of urine formed
(a) III and IV (b) II and II
(c) III and I (d) I and II 174. Which of the following statements is correct?
[NEET 2017]
170. Figure shows human urinary system with (a) The ascending limb of loop of Henle is impermeable to
structures labelled A-D. Select option, which water
correctly identifies them and gives their (b) The descending limb of loop of Henle is impermeable to
characteristics and/or functions. [NEET 2013] water
A (c) The ascending limb of loop of Henle is permeable to water
(d) The descending limb of loop of Henle is permeable to
B electrolytes
C Kidney
175. Which one of the following statements is correct
D with respect to kidney function regulation?
[CBSE-AIPMT 2011]
(a) Exposure to cold temperature stimulates ADH release
(b) An increase in glomerular blood flow stimulates formation
of angiotensin-II
Urinary bladder (c) During summer when body loses lot of water by
evaporation, the release of ADH is suppressed
(a) A-Adrenal gland—located at the anterior part of kidney. (d) When someone drinks lot of water ADH release is stopped
Secrete catecholamines, which stimulate glycogen
breakdown 176. A patient suffering from cholera is given saline
(b) B–Pelvis—broad funnel-shaped space inner to hilum, drip because [CBSE-AIPMT 2000, 1996]
directly connected to loops of Henle (a) Cl - ions are important component of blood plasma
(c) C–Medulla—inner zone of kidney and contains complete (b) Na + ions help to retain water in the body
nephrons (c) Na + ions are important in transport of substances across
membrane
(d) D–Cortex—outer part of kidney and do not contain any
(d) Cl - ions help in the formation of HCl in stomach for
part of nephrons
digestion

MODULE 3
www.jeebooks.in
The NEET Edge ~ Biology 489

(b) high concentration of Ca 2+ and Na +


22. Locomotion and Movement (c) decreased level of oestrogen
177. Which one of the following is the correct pairing of (d) accumulation of uric acid leading to inflammation of joints
a body part and the kind of muscle tissue that 183. Select the correct statement regarding the specific
moves it? [CBSE-AIPMT 2009]
disorder of muscular or skeletal system.
(a) Heart wall — Involuntary unstriated muscle [CBSE-AIPMT 2012]
(b) Biceps of upper arm — Smooth muscle fibres (a) Muscular dystrophy–Age related shortening of muscles
(c) Abdominal wall — Smooth muscle (b) Osteoporosis–Decrease in bone mass and higher chances
(d) Iris — Involuntary smooth muscle of fractures with advancing age
(c) Myasthenia gravis–Autoimmune disorder which inhibits
178. Number of cervical vertebrae in camel is
sliding of myosin filaments
(a) more than that of rabbit [CBSE-AIPMT 1990]
(d) Gout–Inflammation of joints due to extra deposition of
(b) less than that of rabbit calcium
(c) same as that of whale
(d) more than that of horse 184. Select the correct statement with respect to
locomotion in humans. [NEET 2013]
179. Out of ‘X’ pairs of ribs in humans only ‘Y ’ pairs are (a) A decreased level of progesterone causes osteoporosis in
true ribs. Select the option that correctly old people
represents values of X and Y and provides their (b) Accumulation of uric acid crystals in joints causes their
explanation. [NEET 2017] inflammation
(a) X = 12, Y = 7 True ribs are attached dorsally to (c) The vertebral column has 10 thoracic vertebrae
vertebral column and ventrally to the (d) The joint between adjacent vertebrae is a fibrous joint
sternum
(b) X = 12, Y = 5 True ribs are attached dorsally to 23. Neural Control and Coordination
vertebral column and sternum on the 185. Which one of the following statements is correct?
two ends [CBSE-AIPMT 2006]
(c) X = 24, Y = 7 True ribs are dorsally attached to (a) Neurons regulate endocrine activity, but not vice-versa
vertebral column, but are free on ventral (b) Endocrine glands regulate neural activity and nervous
side system regulates endocrine glands
(d) X = 24, Y = 12 True ribs are dorsally attached to (c) Neither hormones control neural activity nor the neurons
vertebral column, but are free on ventral control endocrine activity
(d) Endocrine glands regulate neural activity, but not
side
vice-versa
180. The characteristics and an example of a synovial
joint in humans is [NEET 2013] 186. How do parasympathetic neural signals affect the
working of the heart? [CBSE-AIPMT 2014]
Characteristics Examples (a) Reduce both heart rate and cardiac output
(a) Fluid cartilage between two bones, limited Knee joints (b) Heart rate is increased without affecting the cardiac output
movements
(c) Both heart rate and cardiac output increase
(b) Fluid-filled between two joints, provides Skull bones
(d) Heart rate decreases but cardiac output increases
cushion
(c) Fluid-filled synovial cavity between two Joint between 187. When a neuron is in resting state, i.e. not
bones atlas and axis conducting any impulse, the axonal membrane is
(d) Lymph-filled between two bones, Gliding joint [CBSE-AIPMT 2011]
limited movement between carpals (a) equally permeable to both Na + and K + ions
(b) impermeable to both Na + and K + ions
181. Which of the following pairs is correctly matched? (c) comparatively more permeable to K + ions and nearly
[CBSE-AIPMT 2005] impermeable to Na + ions
(a) Hinge joint — Between vertebrae (d) comparatively more permeable to Na + ions and nearly
(b) Gliding joint — Between zygapophyses of impermeable to K + ions
the successive vertebrae
188. During the propagation of a nerve impulse, the
(c) Cartilaginous joint — Skull bones action potential results from the movement of
(d) Fibrous joint — Between phalanges [CBSE-AIPMT 2008]
182. Osteoporosis, an age-related disease of skeletal (a) K + ions from extracellular fluid to intracellular fluid
system, may occur due to [NEET 2016, Phase II] (b) Na + ions from intracellular fluid to extracellular fluid
(a) immune disorder affecting neuromuscular junction leading (c) K + ions from intracellular fluid to extracellular fluid
to fatigue (d) Na + ions from extracellular fluid to intracellular fluid

MODULE 3
www.jeebooks.in
490 NEET Test Drive

189. During the transmission of nerve impulse through 195. Parts A, B, C and D of the human eyes are shown
a nerve fibre, the potential on the inner side of the in the diagram. Select the option, which gives
plasma membrane has which type of electric correct identification along with its functions/
charge? [CBSE-AIPMT 2007] characteristics. [NEET 2013]
(a) First negative, then positive and again back to negative
(b) First positive, then negative and continue to be negative
C
(c) First negative, then positive and continue to be positive
(d) First positive, then negative and again back to positive Lens B
Iris
190. In the resting state of the neural membrane,
diffusion due to concentration gradients, if allowed,
would drive [CBSE-AIPMT 2004] D
A
+ + +
(a) K into the cell (b) K and Na out of the cell
(c) Na + into the cell (d) Na + out of the cell (a) A–Retina–Contains photoreceptors–Rods and cones
(b) B–Blind spot–Has only a few rods and cones
191. An action potential in the nerve fibre is produced (c) C–Aqueous chamber–Reflects the light, which does not
when positive and negative charges on the outside pass through the lens
and the inside of the axon membrane are reversed, (d) D–Choroiditis–anterior part forms ciliary body
because [CBSE-AIPMT 2000]
(a) more potassium ions enter the axon as compared to 196. Which one of the following is the correct difference
sodium ions leaving it between rod cells and cone cells of our retina?
(b) more sodium ions enter the axon as compared to [CBSE-AIPMT 2008]
potassium ions leaving it Rod cells Cone cells
(c) all potassium ions leave the axon (a) Visual acuity High Low
(d) all sodium ions enter the axon
(b) Visual pigment Iodopsin Rhodopsin
192. A diagram showing axon terminal and synapse is contained
given. Identify correctly at least two of A-D. (c) Overall function Vision in poor Colour vision and detailed
[NEET 2013] light vision in bright light
B (d) Distribution More Evenly distributed all over
C
concentrated in retina
centre of retina

197. In a man, abducens nerve is injured. Which one of


A the following functions will be affected?
(a) Movement of the eyeball [CBSE-AIPMT 2005]
(b) Swallowing
D
(c) Movement of the tongue
(a) A–Receptor, C–Synaptic vesicles
(d) Movement of the neck
(b) B–Synaptic connection, D– K +
(c) A–Neurotransmitter, B–Synaptic cleft 198. Cornea transplantation is outstandingly successful
(d) C–Neurotransmitter, D– Ca 2+ because [CBSE-AIPMT 1996]
(a) cornea is easy to preserve
193. Destruction of the anterior horn cells of the spinal
(b) cornea is not linked up with blood vascular and immune
cord would result in loss of [CBSE-AIPMT 2015]
systems
(a) sensory impulses
(c) the technique involved is very simple
(b) voluntary motor impulses
(d) cornea is easily available
(c) commissural impulses
(d) integrating impulses 199. Choose the correct statement. [NEET 2016, Phase II]
(a) Nociceptors respond to changes in pressure
194. In mammalian eye, the ‘fovea’ is the centre of the
(b) Meissner’s corpuscles are thermoreceptors
visual field, where [CBSE-AIPMT 2015]
(c) Photoreceptors in the human eye are depolarised during
(a) high density of cones occurs, but has no rods
darkness and become hyperpolarised in response to the
(b) the optic nerve leaves the eye
light stimulus
(c) only rods are present
(d) Receptors do not produce graded potentials
(d) more rods than cones are found

MODULE 3
www.jeebooks.in
The NEET Edge ~ Biology 491

24. Chemical Coordination and 206. Which of the following endocrine glands stores its
secretion in the extracellular space before
Integration discharging into the blood? [CBSE-AIPMT 1995]
(a) Pancreas (b) Adrenal
200. GnRH, a hypothalamic hormone, needed in
(c) Testis (d) Thyroid
reproduction, acts on [NEET 2017]
(a) anterior pituitary gland and stimulates secretion of LH and 207. A person is having problems with calcium and
oxytocin phosphorus metabolism in his body. Which one of
(b) anterior pituitary gland and stimulates secretion of LH and the following glands may not be functioning
FSH properly? [CBSE-AIPMT 2007]
(c) posterior pituitary gland and stimulates secretion of (a) Parathyroid (b) Parotid
oxytocin and FSH (c) Pancreas (d) Thyroid
(d) posterior pituitary gland and stimulates secretion of LH
and relaxin 208. A person entering an empty room suddenly finds a
snake right in front on opening the door. Which
201. Hypersecretion of growth hormone in adults one of the following is likely to happen in his
does not cause further increase in height neurohormonal control system? [CBSE-AIPMT 2012]
because [NEET 2017]
(a) Sympathetic nervous system is activated releasing
(a) growth hormone becomes inactive in adults epinephrine and norepinephrine from adrenal medulla
(b) epiphyseal plates close after adolescence (b) Neurotransmitters diffuse rapidly across the cleft and
(c) bones loose their sensitivity to growth hormone in adults transmit a nerve impulse
(d) muscle fibres do not grow in size after birth (c) Hypothalamus activates the parasympathetic division of
202. Identify the hormone with its correct matching of brain
source and function. [CBSE-AIPMT 2014] (d) Sympathetic nervous system is activated releasing
(a) Oxytocin – Posterior pituitary, growth and maintenance of epinephrine and norepinephrine from adrenal cortex
mammary glands 209. Feeling the tremors of an earthquake, a scared
(b) Melatonin – Pineal gland, regulates the normal rhythm of resident of seventh floor of a multistoryed building
sleep-wake cycle starts climbing down the stairs rapidly. Which
(c) Progesterone – Corpus luteum, stimulation of growth and hormone initiated this action? [CBSE-AIPMT 2007]
activities of female secondary sex organs
(a) Thyroxine (b) Adrenaline
(d) Atrial natriuretic factor – Ventricular wall increases the blood
(c) Glucagon (d) Gastrin
pressure
203. The amino acid, tryptophan is the precursor for the 210. Which one of the following pairs of hormones are
synthesis of [NEET 2016, Phase I]
the examples of those that can easily pass through
the cell membrane of the target cell and bind to a
(a) thyroxine and tri-iodothyronine
receptor inside it (mostly in the nucleus)?
(b) oestrogen and progesterone
[CBSE-AIPMT 2012]
(c) cortisol and cortisone
(a) Insulin and glucagon (b) Thyroxine and insulin
(d) melatonin and serotonin
(c) Somatostatin and oxytocin (d) Cortisol and testosterone
204. A pregnant female delivers a baby, who suffers
from stunted growth, mental retardation, low
211. According to the accepted concept of hormone
action, if receptor molecules are removed from
intelligence quotient and abnormal skin. This is
target organs, then the target organ will
the result of [NEET 2013]
[CBSE-AIPMT 1995]
(a) deficiency of iodine in diet
(a) not respond to the hormone
(b) low secretion of growth hormone (b) continue to respond to hormone without any difference
(c) cancer of the thyroid gland (c) continue to respond to the hormone but in the opposite
(d) oversecretion of pars distalis way
(d) continue to respond to the hormone but will require higher
205. A health disorder that results from the deficiency
concentration
of thyroxine in adults and characterised by
I. a low metabolic rate [CBSE-AIPMT 2009] 212. Which of the following radioactive isotopes is used
II. increase in body weight in the detection of thyroid cancer?
[CBSE-AIPMT 2002, 1995]
III. tendency to retain water in tissues is
(a) hypothyroidism (b) simple goitre (a) Iodine-131 (b) Carbon-14
(c) myxoedema (d) cretinism (c) Uranium-238 (d) Phosphorus-32

MODULE 3
www.jeebooks.in
492 NEET Test Drive

213. Select the answer which correctly matches the Codes


endocrine gland with the hormone it secretes and A B C D A B C D
its function/deficiency symptom. [NEET 2013] (a) 4 3 2 1 (b) 2 1 4 3
(c) 1 2 4 3 (d) 3 1 4 2
Endocrine Hormone Function/Deficiency
gland symptoms 217. Why is vivipary an undesirable character for
(a) Anterior pituitary Oxytocin Stimulates uterus annual crop plants? [CBSE-AIPMT 2005]
contraction during (a) It reduces the vigour of the plant
childbirth (b) It adversely affects the fertility of the plant
(b) Posterior Growth Oversecretion (c) The seeds exhibit long dormancy
pituitary Hormone (GH) stimulates abnormal
(d) The seeds cannot be stored under normal conditions for
growth
the next season
(c) Thyroid gland Thyroxine Lack of iodine in diet
results in goitre 218. Which of the following pairs is not correctly
(d) Corpus luteum Testosterone Stimulates matched? [CBSE-AIPMT 2015]
spermatogenesis Mode of reproduction Example
(a) Offset Water hyacinth
214. Match the source gland with its respective
hormone as well as the function. [CBSE-AIPMT 2011] (b) Rhizome Banana
(c) Binary fission Sargassum
Source gland Hormone Function
(d) Conidia Penicillium
(a) Posterior pituitary Vasopressin Stimulates reabsorption
of water in the distal
tubules in the nephron
219. Which one of the following statements is not
correct? [NEET 2016, Phase II]
(b) Corpus luteum Oestrogen Supports pregnancy
(a) Offsprings produced by the asexual reproduction are
(c) Thyroid Thyroxine Regulates blood calcium
level called clone
(b) Microscopic, motile asexual reproductive structures are
(d) Anterior pituitary Oxytocin Contraction of uterus
called zoospores
muscles during childbirth
(c) In potato, banana and ginger, the plantlets arise from the
215. Given ahead is an incomplete table about certain internodes present in the modified stem
hormones, their source glands and one major effect (d) Water hyacinth, growing in the standing water,
of each on the body in humans. Identify the correct drains oxygen from water that leads to the death of fishes
option for the three blanks A, B and C
220. In which one pair both the plants can be
. [CBSE-AIPMT 2011]
vegetatively propagated by leaf pieces?
Gland Secretion Effect on body (a) Agave and Kalanchoe [CBSE-AIPMT 2005]
A Oestrogen Maintenance of secondary sexual (b) Bryophyllum and Kalanchoe
characters (c) Asparagus and Bryophyllum
Alpha cells of islets B Raises blood sugar level
(d) Chrysanthemum and Agave
of Langerhans
Anterior pituitary C Oversecretion leads to gigantism 221. Select the wrong statement. [NEET 2013]
(a) Isogametes are similar in structure, function and behaviour
A B C
(a) (b) Anisogametes differ either in structure, function and
Placenta Insulin Vasopressin
(b) behaviour
Ovary Insulin Calcitonin
(c) (c) In Oomycetes, female gamete is smaller and motile, while
Placenta Glucagon Calcitonin
(d) male gamete is larger and non-motile
Ovary Glucagon Growth hormone
(d) Chlamydomonas exhibits both isogamy and anisogamy
and Fucus shows oogamy
25. Reproduction in Organisms 222. In oogamy, fertilisation involves [CBSE-AIPMT 2004]
216. Match Column I with Column II and select the (a) a small non-motile female gamete and a large motile male
correct option using the codes given below. gamete
[NEET 2016, Phase II] (b) a large non-motile female gamete and a small motile male
Column I Column I gamete
A. Pistils fused together 1. Gametogenesis (c) a large non-motile female gamete and a small non-motile
B. Formation of gametes 2. Pistillate male gamete
C. Hyphae of higher ascomycetes 3. Syncarpous (d) a large motile female gamete and a small non-motile male
D. Unisexual female flower 4. Dikaryotic gamete

MODULE 3
www.jeebooks.in
The NEET Edge ~ Biology 493

26. Sexual Reproduction in 231. Double fertilisation leading to initiation of


endosperm in angiosperms requires [CBSE-AIPMT 2000]
Flowering Plants (a) fusion of one polar nucleus and the second male gamete
only
223. Which one of the following statements is not true? (b) fusion of two polar nuclei and the second male gamete
[NEET 2016, Phase I]
(c) fusion of four or more polar nuclei and the second male
(a) Exine of pollen grains is made up of sporopollenin gamete only
(b) Pollen grains of many species cause severe allergies (d) all of the above kinds of fusion in different angiosperms
(c) Stored pollen in liquid nitrogen can be used in the crop
breeding programmes 232. The scutellum observed in a grain of wheat or
(d) Tapetum helps in the dehiscence of anther maize is comparable to which part of the seed in
other monocotyledons? [CBSE-AIPMT 2010, 06]
224. Which one of the following statements is correct? (a) Cotyledon (b) Endosperm
(a) Hard outer layer of pollen is called intine [NEET 2013] (c) Aleurone layer (d) Plumule
(b) Sporogenous tissue is haploid
(c) Endothecium produces the microspores 233. Generative cell was destroyed by laser but a normal
(d) Tapetum nourishes the developing pollen pollen tube was still formed because
(a) vegetative cell is not damaged [CBSE-AIPMT 1989]
225. Which of the following statements is not correct? (b) contents of killed generative cell stimulate pollen growth
[NEET 2016, Phase I] (c) laser beam stimulates growth of pollen tube
(a) Insects that consume pollen or nectar without bringing (d) the region of emergence of pollen tube is not harmed
about pollination are called pollen nectar robbers
(b) Pollen germination and pollen tube growth are regulated 234. What would be the number of chromosomes in the
by chemical components of pollen interacting with those of cells of the aleuron layer in a plant species with
the pistil 8 chromosomes in its synergids? [CBSE-AIPMT 2006]
(c) Some reptiles have also been reported as pollinators in (a) 24 (b) 32 (c) 8 (d) 16
some plant species
235. Number of meiotic divisions required to produce
(d) Pollen grains of many species can germinate on the
200/400 seeds of pea would be [CBSE-AIPMT 1993]
stigma of a flower, but only one pollen tube of the same
species grows into the style (a) 200/400 (b) 400/800 (c) 300/600 (d) 250/500

226. How many pollen grains will be formed after 236. Ovule is straight with funiculus, embryo sac,
meiotic division in 10 microspore mother cells? chalaza and micropyle lying on one straight line.
[CBSE-AIPMT 1996] It is [CBSE-AIPMT 1993]
(a) 10 (b) 20 (c) 40 (d) 80 (a) orthotropous (b) anatropous
(c) campylotropous (d) amphitropous
227. In an angiosperm, how many microspore mother
cells are required to produce 100 pollen grains?
[CBSE-AIPMT 1995] 27. Human Reproduction
(a) 25 (b) 50 (c) 75 (d) 100
237. Which of the following depicts the correct pathway
228. Which type of association is found in between of transport of sperms? [NEET 2016, Phase II]
entomophilous flower and pollinating agent? (a) Rete testis ® Efferent ductules ® Epididymis ® Vas
[CBSE-AIPMT 2002] deferens
(a) Mutualism (b) Commensalism (b) Rete testis ® Epididymis ® Efferent ductules ® Vas
(c) Cooperation (d) Coevolution deferens
229. Flowers showing ornithophily show few (c) Rete testis ® Vas deferens ® Efferent ductules ®
Epididymis
characteristics like [CBSE-AIPMT 1999]
(d) Efferent ductules ® Rete testis ® Vas deferens ®
(a) blue flower with nectaries at base of corolla
Epididymis
(b) red sweet scented flower with nectaries
(c) bright red flower into thick inflorescence 238. Which one of the following statements is false in
(d) white flowers with fragrance respect of viability of mammalian sperm?
(a) Sperm is viable for only up to 24 hrs [CBSE-AIPMT 2012]
230. In majority of angiosperms, [NEET 2016, Phase II]
(b) Survival of sperm depends on the pH of the medium and is
(a) egg has a filiform apparatus
more active in alkaline medium
(b) there are numerous antipodal cells
(c) reduction division occurs in the megaspore mother cells (c) Viability of sperm is determined by its motility
(d) a small central cell is present in the embryo sac (d) Sperms must be concentrated in a thick suspension

MODULE 3
www.jeebooks.in
494 NEET Test Drive

239. Changes in GnRH pulse frequency in females is 246. Extrusion of second polar body from egg nucleus
controlled by circulating levels of [NEET 2016, Phase I] occurs [CBSE-AIPMT 1993]
(a) oestrogen and inhibin (b) progesterone only (a) after entry of sperm but before completion of fertilisation
(c) progesterone and inhibin (d) oestrogen and progesterone (b) after completion of fertilisation
(c) before entry of sperm
240. Select the incorrect statement. [NEET 2016, Phase I]
(d) without any relation of sperm entry
(a) LH and FSH trigger ovulation in ovary
(b) LH and FSH decrease gradually during the follicular phase 247. Which one of the following statements about
(c) LH triggers secretion of androgens from the Leydig cells morula in humans is correct? [CBSE-AIPMT 2010]
(d) FSH stimulates the Sertoli cells which help in (a) It has almost equal quantity of cytoplasm as an uncleaved
spermiogenesis zygote but much more DNA
241. What is the correct sequence of sperm formation? (b) It has far less cytoplasm as well as less DNA than in an
[NEET 2013] uncleaved zygote
(a) Spermatid, Spermatocyte, Spermatogonia, Spermatozoa (c) It has more or less equal quantity of cytoplasm and DNA
(b) Spermatogonia, Spermatocyte, Spermatozoa, Spermatid (d) It has more cytoplasm and more DNA than in an
(c) Spermatogonia, Spermatozoa, Spermatocyte, Spermatid uncleaved zygote
(d) Spermatogonia, Spermatocyte, Spermatid, Spermatozoa
248. Termination of gastrulation is indicated by
242. Which one of the following is the correct matching (a) obliteration of blastocoel
of the events occurring during menstrual cycle? (b) obliteration of archenteron
[CBSE-AIPMT 2009]
(c) closure of blastopore
(a) Ovulation – LH and FSH attain peak level and (d) closure of neural tube
sharp fall in the secretion of
progesterone 249. Ectopic pregnancies are referred to as
(b) Proliferative – Rapid regeneration of myometrium (a) pregnancies with genetic abnormality [CBSE-AIPMT 2015]
phase and maturation of Graafian follicle (b) implantation of embryo at site other than uterus
(c) Development of – Secretory phase and increased (c) implantation of defective embryo in the uterus
corpus luteum secretion of progesterone (d) pregnancies terminated due to the hormonal imbalance
(d) Menstruation – Breakdown of myometrium and 250. Select the correct option describing gonadotropin
ovum not fertilised activity in a normal pregnant female.
243. Which one of the following statements is incorrect [CBSE-AIPMT 2014, 12]
about menstruation? [CBSE-AIPMT 2008] (a) High level of FSH and LH stimulates the thickening of
(a) During normal menstruation, about 40 mL blood is lost endometrium
(b) The menstrual fluid can easily clot (b) High level of FSH and LH facilitates implantation of the
(c) At menopause in the female, there is especially abrupt embryo
increase in gonadotropic hormones (c) High level of hCG stimulates the synthesis of oestrogen
(d) The beginning of the cycle of menstruation is called and progesterone
menarche (d) High level of hCG stimulates the thickening of
endometrium
244. Fertilisation in humans is practically feasible only
if [NEET 2016, Phase I] 251. Which one of the following is not the function of
(a) the ovum and sperms are transported simultaneously to placenta? It [NEET 2013]
ampullary - isthmic junction of the Fallopian tube (a) facilitates supply of oxygen and nutrients to embryo
(b) the ovum and sperms are transported simultaneously to (b) secretes oestrogen
ampullary-isthmic junction of the cervix (c) facilitates removal of carbon dioxide and waste material
(c) the sperms are transported into cervix within 48 hrs of from embryo
release of ovum in uterus
(d) secretes oxytocin during parturition
(d) the sperms are transported into vagina just after the
release of ovum in Fallopian tube 252. The first movements of the foetus and appearance
245. If mammalian ovum fails to get fertilised, which of hair on its head are usually observed during
one of the following is unlikely? [CBSE-AIPMT 2005]
which month of pregnancy? [CBSE-AIPMT 2010]
(a) Fourth month
(a) Corpus luteum will disintegrate
(b) Fifth month
(b) Oestrogen secretion further decreases
(c) Sixth month
(c) Primary follicle starts developing
(d) Third month
(d) Progesterone secretion rapidly declines

MODULE 3
www.jeebooks.in
The NEET Edge ~ Biology 495

253. Match Column I with Column II and select the II. Generally chances of conception are nil until
correct option using the codes given below. mother breastfeeds the infant up to two years.
[NEET 2016, Phase II] III. Intrauterine devices like copper-T are effective
Column I Column II contraceptives.
IV. Contraception pills may be taken up to one week
A. Mons pubis 1. Embryo formation
after coitus to prevent contraception.
B. Antrum 2. Sperm
C. Trophectoderm 3. Female external genitalia Which two of the above statements are correct?
(a) I and III (b) I and II
D. Nebenkern 4. Graafian follicle (c) II and III (d) III and IV

Codes 258. Given below are four methods (A-D) and their
A B C D A B C D modes of action (1-4) in achieving contraception.
(a) 3 4 2 1 (b) 3 4 1 2 Select their correct matching from the four options
(c) 3 1 4 2 (d) 1 4 3 2 that follows. [CBSE-AIPMT 2008]

Method Mode of Action


28. Reproductive Health A. The pill 1. Prevents sperms reaching cervix
B. Condom 2. Prevents implantation
254. Match the following column I (sexually transmitted C. Vasectomy 3. Prevents ovulation
diseases) with their column II (causative agent) D. Copper-T 4. Semen contains no sperms
and select the correct option. [NEET 2017]
Codes
Column I Column II A B C D A B C D
A. Gonorrhoea 1. HIV (a) 3 4 1 2 (b) 2 3 1 4
B. Syphilis 2. Neisseria (c) 3 1 4 2 (d) 4 1 2 3
C. Genital warts 3. Treponema 259. In context of amniocentesis, which of the following
D. AIDS 4. Human papilloma virus statements is incorrect? [NEET 2016, Phase I]
Codes (a) It is used for prenatal sex-determination
A B C D A B C D (b) It can be used for the detection of Down syndrome
(a) 2 3 4 1 (b) 3 4 1 2 (c) It can be used for the detection of cleft palate
(c) 4 2 3 1 (d) 4 3 2 1 (d) It is usually done when a woman is between 14-16 weeks
pregnant
255. In India, human population is heavily weighed
towards the younger age groups as a result of 260. In case of a couple, where the male is having a
[CBSE-AIPMT 1995] very low sperm count, which technique will be
(a) short lifespan of many individuals and low birth rate suitable for fertilisation? [NEET 2017]
(b) long lifespan of many individuals and low birth rate (a) Intrauterine transfer
(c) short lifespan of many individuals and high birth rate (b) Gamete intracytoplasmic Fallopian transfer
(d) long lifespan of many individuals and high birth rate (c) Artificial insemination
(d) Intracytoplasmic sperm Injection
256. Which of the following approaches does not give the
defined action of contraceptive? [NEET 2016, Phase I] 261. In vitro fertilisation is a technique that involves
transfer of which one of the following into the
(a) Intrauterine devices – Increase phagocytosis of Fallopian tube? [CBSE-AIPMT 2010]
sperms, suppress sperm
motility and fertilising capacity (a) Embryo only, up to 8 cell stage
of sperms (b) Either zygote or early embryo up to 8 cell stage
(b) Hormonal – Prevent/ retard entry of sperms, (c) Embryo of 32 cell stage
contraceptives prevent ovulation and fertilisation (d) Only zygote
(c) Vasectomy – Prevents spermatogenesis
262. A childless couple can be assisted to have a child
(d) Barrier methods – Prevent fertilisation
through a technique called GIFT. The full form of
257. Consider the statements given below regarding this technique is [CBSE-AIPMT 2015]
contraception and answer as directed thereafter. (a) Gamete Inseminated Fallopian Transfer
[CBSE-AIPMT 2008] (b) Gamete Intra Fallopian Transfer
I. Medical Termination of Pregnancy (MTP) during (c) Gamete Internal Fertilisation and Transfer
first trimester is generally safe. (d) Germ Cell Internal Fallopian Transfer

MODULE 3
www.jeebooks.in
496 NEET Test Drive

263. Test tube baby means a baby born when 269. Match the terms in Column I with their description
[CBSE-AIPMT 2003] in Column II and choose the correct option.
(a) the ovum is fertilised externally and thereafter implanted in [NEET 2016, Phase I]
the uterus
(b) it develops from a non-fertilised egg Column I Column II
(c) it is developed in a test tube A. Dominance 1. Many genes govern a single
(d) it is developed through tissue culture method character
B. Codominance 2. In a heterozygous organism, only
264. Certain characteristic demographic features of one allele expresses itself
developing countries are [CBSE-AIPMT 2004] C. Pleiotropy 3. In a heterozygous organism, both
(a) high fertility, low or rapidly falling mortality rate, rapid alleles express themselves fully
population growth and a very young age distribution D. Polygenic inheritance 4. A single gene influences many
(b) high fertility, high density, rapidly rising mortality rate and a characters
very young age distribution
Codes
(c) high infant mortality, low fertility, uneven population growth
A B C D A B C D
and a very young age distribution
(a) 2 3 4 1 (b) 4 1 2 3
(d) high mortality, high density, uneven population growth and (c) 4 3 1 2 (d) 2 1 4 3
a very old age distribution
270. A tall true breeding garden pea plant is crossed
with a dwarf true breeding garden pea plant.
29. Principles of Inheritance When the F1 plants were selfed, the resulting
and Variation genotypes were in the ratio of [NEET 2016, Phase I]
(a) 1 : 2 : 1 :: Tall heterozygous : Tall homozygous : Dwarf
265. Which one of the following cannot be explained on
(b) 3 : 1 :: Tall : Dwarf
the basis of Mendel’s law of dominance?
[CBSE-AIPMT 2010]
(c) 3 : 1 :: Dwarf : Tall
(a) The discrete unit controlling a particular character is called (d) 1 : 2 : 1 :: Tall homozygous : Tall heterozygous : Dwarf
a factor 271. The genotypes of a husband and wife are IA IB and
(b) Out of one pair of factors one is dominant and the other IA i. Among the blood types of their children, how
recessive many different genotypes and phenotypes are
(c) Alleles do not show any blending and both the characters possible? [NEET 2017]
recover as such in F2 -generation
(a) 3 genotypes ; 3 phenotypes (b) 3 genotypes ; 4 phenotypes
(d) Factors occur in pairs
(c) 4 genotypes ; 3 phenotypes (d) 4 genotypes ; 4 phenotypes
266. In a test cross involving F1 dihybrid flies, more
272. ABO blood groups in humans are controlled by the
parental type offsprings were produced than the gene I. It has three alleles - IA , IB and i. Since
recombinant type offspring. This indicates
there are three different alleles, six different
[NEET 2016, Phase I]
genotypes are possible. How many phenotypes can
(a) chromosomes failed to separate during meiosis
occur? [CBSE-AIPMT 2010]
(b) the two genes are linked and present on the same
(a) Three (b) One (c) Four (d) Two
chromosome
(c) both of the characters are controlled by more than one 273. Which one of the following conditions correctly
gene describes the manner of determining the sex in the
(d) the two genes are located on two different chromosomes given example? [CBSE-AIPMT 2011]
267. Two genes R and Y are located very close on the (a) XO type of sex chromosomes determines male sex in
chromosomal linkage map of maize plant. When grasshopper
RRYY and rryy genotypes are hybridised, then F2 (b) XO condition in humans as found in Turner syndrome,
segregation will show [CBSE-AIPMT 2007] determines female sex
(c) Homozygous sex chromosomes (XX) produce male in
(a) higher number of the recombinant types
Drosophila
(b) segregation in the expected 9 : 3 : 3 : 1 ratio
(d) Homozygous sex chromosomes (ZZ) determine female
(c) segregation in 3 : 1 ratio sex in birds
(d) higher number of the parental types
274. A fruitfly heterozygous for sex-linked genes is
268. In a dihybrid cross AABB × aabb, F2 progeny of mated with normal female fruitfly. Male specific
AABB, AABb, AaBB and AaBb occurs in the ratio chromosome will enter egg cell in the proportion
of [CBSE-AIPMT 1994] [CBSE-AIPMT 1997]
(a) 1 : 1 : 1 : 1 (b) 9 : 3 : 3 : 1 (c) 1 : 2 : 2 : 1 (d) 1 : 2 : 2 : 4 (a) 1 : 1 (b) 2 : 1 (c) 3 : 1 (d) 7 : 1

MODULE 3
www.jeebooks.in
The NEET Edge ~ Biology 497

275. Which of the following statements is not true of 283. Down’s syndrome is caused by an extra copy of
two genes that show 50% recombination frequency? chromosome number 21. What percentage of
(a) The genes may be on different chromosomes [NEET 2013] offspring produced by an affected mother and a
(b) The genes are tightly linked normal father would be affected by this disorder?
(c) The genes show independent assortment (a) 50% (b) 25% [CBSE-AIPMT 2003]
(d) If the genes are present on the same chromosome, they (c) 100% (d) 75%
undergo more than one crossovers in every meiosis
284. A woman with albinic father marries an albinic man.
276. The linkage map of X-chromosome of fruitfly has The proportion of her progeny is [CBSE-AIPMT 1994]
66 units, with yellow body gene (y) at one end and (a) 2 normal : 1 albinic
bobbed hair gene (b) at the other end. The (b) all normal
recombination frequency between these two genes (c) all albinic
(y and b) should be [CBSE-AIPMT 2003] (d) 1 normal : 1 albinic
(a) £ 50% (b) 100%
(c) 66% (d) > 50% 285. In the following human pedigree, the filled symbols
represent the affected individuals. Identify the
277. There are three genes a, b, c. Percentage of type of given pedigree. [CBSE-AIPMT 2015]
crossing over between a and b is 20%, b and c is I.
28% and a and c is 8%. What is the sequence of
genes on chromosome? [CBSE-AIPMT 2002] II.
(a) b, a, c (b) a, b, c
(c) a, c, b (d) None of these III.
278. Which of the following is suitable for experiment
on linkage? [CBSE-AIPMT 1993] IV.
(a) aaBB × aaBB (b) AABB × aabb
(a) Autosomal dominant (b) X-linked recessive
(c) AaBb × AaBb (d) AAbb × AaBB
(c) Autosomal recessive (d) X-linked dominant
279. Mr. Kapoor has Bb autosomal gene pair and
d allele sex-linked. What shall be the proportion of 286. A colourblind man marries a woman with normal
Bd in sperms? [CBSE-AIPMT 1993]
sight who has no history of colour blindness in her
family. What is the probability of their grandson
(a) 0 (b) 1/2
being colourblind? [CBSE-AIPMT 2015]
(c) 1/4 (d) 1/8
(a) 0.5 (b) 1
280. A male human is heterozygous for autosomal genes (c) Nil (d) 0.25
A and B and is also hemizygous for haemophilic
gene h. What proportion of his sperms will be abh? 287. A man whose father was colourblind marries a
[CBSE-AIPMT 2004]
woman, who had a colourblind mother and normal
1 1 1 1 father. What percentage of male children of this
(a) (b) (c) (d) couple will be colourblind? [CBSE-AIPMT 2014]
8 32 16 4
(a) 25% (b) 0%
281. A man with a certain disease marries a normal (c) 50% (d) 75%
woman. They have eight children (3 daughters and
5 sons). All the daughters suffer from their father’s 288. Study the pedigree chart given below.
disease but none of the sons is affected. Which of
the following modes of inheritance do you suggest
for this disease? [CBSE-AIPMT 1996, 2002]
(a) Sex-linked recessive (b) Sex-linked dominant
(c) Autosomal dominant (d) Sex-limited recessive
282. Pick out the correct statements. [NEET 2016, Phase I]
I. Haemophilia is a sex-linked recessive disease. What does it show? [CBSE-AIPMT 2009]
II. Down's syndrome is due to aneuploidy. (a) Inheritance of a sex-linked inborn error of metabolism like
III. Phenylketonuria is an autosomal recessive gene phenylketonuria
disorder. (b) Inheritance of a condition like phenylketonuria as an
IV. Sickle-cell anaemia is an X-linked recessive gene autosomal recessive trait
disorder. (c) The pedigree chart is wrong as this is not possible
(a) II and IV are correct (b) I, III and IV are correct (d) Inheritance of a recessive sex-linked disease like
(c) I, II and III are correct (d) I and IV are correct haemophilia

MODULE 3
www.jeebooks.in
498 NEET Test Drive

289. If both parents are carriers for thalassemia, which 297. Select the two statements out of the four (I-IV)
is an autosomal recessive disorder, what are the given below about lac operon.
chances of pregnancy resulting in an affected I. Glucose or galactose may bind with the repressor
child? [NEET 2013] and inactivate it.
(a) No chance (b) 50% (c) 25% (d) 100% II. In the absence of lactose, the repressor binds with
290. Blue eye colour is recessive to brown eye colour. the operator region.
A brown eyed man whose mother was blue III. The z-gene codes for permease.
eyed marries a blue eyed woman. The children IV. This was elucidated by Francis Jacob and
shall be [CBSE-AIPMT 1991] Jacques Monod.
(a) both blue eyed and brown eyed 1 : 1
The correct statements are [CBSE-AIPMT 2010]
(b) all brown eyed
(a) I and III (b) I and IV
(c) all blue eyed
(d) blue eyed and brown eyed 3 : 1 (c) II and IV (d) I and II

291. Both husband and wife have normal vision though 298. Which of the following is not required for any of
their fathers were colourblind. The probability of the techniques of DNA fingerprinting available at
their daughter becoming colourblind is present? [NEET 2016, Phase I]
(a) 0% (b) 25% [CBSE-AIPMT 1990] (a) Zinc finger analysis (b) Restriction enzymes
(c) 50% (d) 75% (c) DNA-DNA hybridisation (d) Polymerase chain reaction
299. Which one of the following is wrongly matched?
30. Molecular Basis of Inheritance [CBSE-AIPMT 2014]
(a) Transcription – Writing information from DNA to t RNA
292. A molecule that can act as a genetic material (b) Translation–Using information in mRNA to make protein
must fulfil the traits given below, except (c) Repressor protein – Binds to operator to stop enzyme
[NEET 2016, Phase II] synthesis
(a) it should be able to express itself in the form of ‘Mendelian (d) Operon – Structural genes, operator and promoter
characters’
(b) it should be able to generate its replica
(c) it should be unstable structurally and chemically 31. Evolution
(d) it should provide the scope for slow changes that are 300. Following are the two statements regarding the
required for evolution origin of life. [NEET 2016, Phase I]
293. During transcription, holoenzyme RNA polymerase I. The earliest organisms that appeared on the
binds to a DNA sequence and the DNA assumes a Earth were non-green and presumably
saddle-like structure at that point. What is that anaerobes.
sequence called? [CBSE-AIPMT 2005] II. The first autotrophic organisms were the
(a) CAAT box (b) GGTT box (c) AAAT box (d) TATA box chemoautotrophs that never released oxygen.
294. During transcription, the nucleotide sequence of Of the above statements, which one of the
the DNA strand that is being coded is ATACG, following options is correct?
then the nucleotide sequence in the mRNA would (a) II is correct but I is false (b) Both I and II are correct
be [CBSE-AIPMT 2004] (c) Both I and II are false (d) I is correct but II is false
(a) TATGC (b) TCTGG (c) UAUGC (d) UATGG
301. Which of the following is the correct sequence of
295. If there are 999 bases in an RNA that codes for a events in the origin of life? [NEET 2016, Phase II]
protein with 333 amino acids and the base at I. Formation of protobionts.
position 901 is deleted such that the length of the
II. Synthesis of organic monomers.
RNA becomes 998 bases, how many codons will be
altered ? [NEET 2017]
III. Synthesis of organic polymers.
(a) 1 (b) 11 (c) 33 (d) 333
IV. Formation of DNA-based genetic systems.
(a) I–II–III–IV (b) I–III–II–IV (c) II–III–I–IV (d) II–III–IV–I
296. What would happen if in a gene encoding a
polypeptide of 50 amino acids, 25th codon (UAU) is 302. The process by which organisms with different
mutated to UAA? [CBSE-AIPMT 2003] evolutionary history evolve similar phenotypic
(a) A polypeptide of 49 amino acids will be formed
adaptations in response to a common
environmental challenge, is called [NEET 2013]
(b) A polypeptide of 25 amino acids will be formed
(a) natural selection (b) convergent evolution
(c) A polypeptide of 24 amino acids will be formed
(d) Two polypeptides of 24 and 25 amino acids will be formed (c) non-random evolution (d) adaptive radiation

MODULE 3
www.jeebooks.in
The NEET Edge ~ Biology 499

303. Which one of the following options gives one (a) they can be studied from the samples of fossil remains
correct example each of convergent evolution and (b) they are small and therefore, easy to study
divergent evolution? [CBSE-AIPMT 2012] (c) they are uniparental in origin and do not take part in
recombination
Convergent evolution Divergent evolution (d) their structure is known in greater detail
(a) Eyes of Octopus and Bones of forelimbs of
mammals vertebrates
309. Which one of the following sequences was proposed
by Darwin and Wallace for organic evolution?
(b) Thorns of Bougainvillea and Wings of butterflies and [CBSE-AIPMT 2003]
tendrils of Cucurbita birds
(a) Variations, natural selection, overproduction, constancy of
(c) Bones of forelimbs of Wings of butterflies and population size
vertebrates birds (b) Overproduction, variations, constancy of population size,
(d) Thorns of Bougainvillea and Eyes of Octopus and natural selection
tendrils of Cucurbita mammals (c) Variations, constancy of population size, overproduction,
natural selection
304. Age of fossils in the past was generally determined (d) Overproduction, constancy of population size, variations,
by radio-carbon method and other methods natural selection
involving radioactive elements found in the rocks.
More precise methods, which were used recently 310. Darwin’s theory of pangenesis shows similarity
and led to the revision of the evolutionary periods with theory of inheritance of acquired characters
for different groups of organisms, include then what will be correct according to it?
[CBSE-AIPMT 2001]
[CBSE-AIPMT 2004]
(a) Useful organs become strong and developed while
(a) study of carbohydrates/proteins in fossils
useless organs become extinct. These organs help in
(b) study of the conditions of fossilisation
struggle for survival
(c) Electron Spin Resonance (ESR) and fossil DNA (b) Size of organs increases with ageing
(d) study of carbohydrates/proteins in rocks (c) Development of organs is due to will power
305. The age of the fossil of Dryopithecus on the (d) There should be some physical basis of inheritance
geological time scale is [CBSE-AIPMT 1998] 311. Using imprints from a plate with complete medium
(a) 5 ´ 106 yr back (b) 25 ´ 106 yr back and carrying bacterial colonies, you can select
(c) 50 ´ 106 yr back (d) 75 ´ 106 yr back streptomycin resistant mutants and prove that
such mutations do not originate as adaptation.
306. The eyes of Octopus and eyes of cat show different These imprints need to be used [CBSE-AIPMT 2005]
patterns of structure, yet they perform similar (a) on plates with and without streptomycin
function. This is an example of [NEET 2013] (b) on plates with minimal medium
(a) homologous organs that have evolved due to convergent (c) only on plates with streptomycin
evolution (d) only on plates without streptomycin
(b) homologous organs that have evolved due to divergent
evolution 312. In the case of peppered moth (Biston betularia),
(c) analogous organs that have evolved due to convergent the black-coloured form became dominant over the
evolution light-coloured form in England during industrial
(d) analogous organs that have evolved due to divergent revolution. This is an example of [CBSE-AIPMT 2009]
evolution (a) natural selection whereby the darker forms were selected
(b) appearance of the darker coloured individuals due to very
307. In general, in the developmental history of a poor sunlight
mammalian heart, it is observed that it passes (c) protective mimicry
through a two-chambered fish-like heart, (d) inheritance of darker colour character acquired due to the
three-chambered frog-like heart and finally to darker environment
four-chambered stage. To which hypothesis can
this above cited statement be approximated? 313. Industrial melanism as observed in peppered moth
[CBSE-AIPMT 1998] proves that [CBSE-AIPMT 2007]

(a) Hardy-Weinberg law (b) Lamarck’s principle (a) the true black melanic forms arise by a recurring random
mutation
(c) Biogenetic law (d) Mendelian principles
(b) the melanic form of the moth has no selective advantage
308. In recent years, DNA sequences over lighter form in industrial area
(nucleotide sequence) of mtDNA and (c) the lighter form moth has no selective advantage either in
Y-chromosomes were considered for the study of polluted industrial area or non-polluted area
human evolution, because [CBSE-AIPMT 2003] (d) melanism is a pollution generated feature

MODULE 3
www.jeebooks.in
500 NEET Test Drive

314. Which one of the following statements is correct? IV. The acceptance or rejection of a kidney
[CBSE-AIPMT 2007] transplant depends on specific interferons.
(a) Stem cells are specialised cells
The two correct statements are [CBSE-AIPMT 2010]
(b) There is no evidence of the existence of gills during
(a) II and III (b) III and IV (c) I and III (d) I and II
embryogenesis of mammals
(c) All plant and animal cells are totipotent 319. Match the disease in Column I with the appropriate
(d) Ontogeny repeats phylogeny items(pathogen/prevention/treatment) in Column II.
[CBSE-AIPMT 2008]

32. Human Health and Diseases Column I Column II


A. Amoebiasis 1. Treponema pallidum
315. Common cold differs from pneumonia in that
[CBSE-AIPMT 2012] B. Diphtheria 2. Use only sterilised food and water
(a) pneumonia is a communicable disease, whereas the C. Cholera 3. DPT vaccine
common cold is a nutritional deficiency disease D. Syphilis 4. Use oral rehydration therapy
(b) pneumonia can be prevented by a live attenuated bacterial Codes
vaccine, whereas the common cold has no effective
A B C D A B C D
vaccine
(a) 1 2 3 4 (b) 2 4 1 3
(c) pneumonia is caused by a virus, while the common cold is
(c) 2 1 3 4 (d) 2 3 4 1
caused by the bacterium Haemophilus influenzae
(d) pneumonia pathogen infects alveoli whereas the common 320. Which of the following statements is not true for
cold affects nose and respiratory passage but not the lungs cancer cells in relation to mutations?
[NEET 2016, Phase I]
316. In which one of the following options the two (a) Mutations destroy telomerase inhibitor
examples are correctly matched with their
(b) Mutations inactivate the cell control
particular type of immunity? [CBSE-AIPMT 2012]
(c) Mutations inhibit the production of telomerase
Examples Type of immunity (d) Mutations in proto-oncogenes accelerate the cell cycle

(a) Polymorphonuclear leukocytes Cellular barriers 321. Which one of the following statements is correct?
and monocytes [CBSE-AIPMT 2009]
(b) Anti-tetanus and anti-snake bite Active immunity (a) Patients, who had undergone surgery are given
injections cannabinoids to relieve pain
(c) Saliva in mouth and tears in eyes Physical barriers (b) Benign tumours show the property of metastasis
(c) Heroin accelerates body functions
(d) Mucous coating of epithelium Physiological barriers
(d) Malignant tumours may exhibit metastasis
lining the urinogenital tract and
the HCl in stomach 322. Carcinoma refers to [CBSE-AIPMT 2003]
(a) malignant tumours of the colon
317. Which one of the following statements is correct
(b) benign tumours of the connective tissue
with respect to immunity? [CBSE-AIPMT 2012]
(c) malignant tumours of the connective tissue
(a) Preformed antibodies need to be injected to treat the bite
(d) malignant tumours of the skin or mucous membrane
by a viper snake
(b) The antibodies against smallpox pathogen are produced 323. Which of the following symptoms indicate radiation
by T-lymphocytes sickness? [CBSE-AIPMT 1997]
(c) Antibodies are protein molecules, each of which has four (a) Red and ulcerated skin
light chains (b) Nausea and anaemia
(d) Rejection of a kidney graft is the function of B-lymphocytes (c) Nausea and loss of hair
(d) Ulcerated skin, nausea and loss of hair
318. Consider the following four statements (I-IV)
regarding kidney transplant and select the two 324. Which of the following is correct regarding AIDS
correct ones out of these. causative agent HIV? [NEET 2016, Phase II]
I. Even if a kidney transplant is proper, the (a) HIV is enveloped virus containing one molecule of
single-stranded RNA and one molecule of reverse
recipient may need to take immunosuppressants
transcriptase
for a long time. (b) HIV is enveloped virus that contains two identical
II. The cell-mediated immune response is molecules of single-stranded RNA and two molecules of
responsible for the graft rejection. reverse transcriptase
(c) HIV is unenveloped retrovirus
III. The B-lymphocytes are responsible for the
(d) HIV does not escape but attacks the acquired immune
rejection of graft. response

MODULE 3
www.jeebooks.in
The NEET Edge ~ Biology 501

325. At which stage of HIV infection does one usually (a) Statements III and IV (b) Statements I, III and IV
show symptoms of AIDS? [CBSE-AIPMT 2014, 11] (c) Statements II, III and IV (d) Statements I and II
(a) Within 15 days of sexual contact with an infected person
331. Consider the following statements (I-IV) about
(b) When the infected retrovirus enters host cells
organic farming. [CBSE-AIPMT 2011]
(c) When HIV damages large number of helper T-lymphocytes
(d) When the viral DNA is produced by reverse transcriptase I. Utilises genetically modified crops like Bt cotton.
II. Uses only naturally produced inputs like
326. Retroviruses are implicated as a cause for cancer compost.
in humans because they [CBSE-AIPMT 1996]
III. Does not use pesticides and urea.
(a) carry gene for reverse transcriptase
IV. Produces vegetables rich in vitamins and
(b) may carry cellular proto-oncogenes in their genome
minerals.
(c) may carry v-oncogenes in their genome
(d) carry single-stranded RNA as their genetic material Which of the above statements are correct?
(a) II, III and IV (b) III and IV (c) II and III (d) I and II
327. Which one of the following is the correct statement
regarding the particular psychotropic drug 332. Consider the following four measures (I-IV) that
specified? [CBSE-AIPMT 2008] could be taken to successfully grow chickpea in an
(a) Hashish causes alter thought perceptions and area where bacterial blight disease is common.
hallucinations I. Spray with Bordeaux mixture.
(b) Opium stimulates nervous system and causes II. Control of the insect vector of the disease
hallucinations pathogen.
(c) Morphine leads to delusions and disturbed emotions
III. Use of only disease-free seeds.
(d) Barbiturates cause relaxation and temporary euphoria
IV. Use of varieties resistant to the disease.
328. Which one of the following is not correctly
Which two of the above measures can control the
matched? [CBSE-AIPMT 2004]
disease? [CBSE-AIPMT 2008]
(a) Glossina palpalis — Sleeping sickness
(a) III and IV (b) I and IV
(b) Culex pipiens — Filariasis
(c) II and III (d) I and II
(c) Aedes aegypti — Yellow fever
(d) Anopheles culicifacies — Leishmaniasis 333. Farmers in a particular region were concerned that
premature yellowing of leaves of a pulse crop
329. A person showing unpredictable moods, outbursts of might cause decrease in the yield. Which treatment
emotion, quarrelsome behaviour and conflicts with could be most beneficial to obtain maximum seed
others, is suffering from [CBSE-AIPMT 2006]
yield? [CBSE-AIPMT 2006]
(a) schizophrenia
(a) Frequent irrigation of the crop
(b) Borderline Personality Disorder (BPD)
(b) Treatment of the plants with cytokinins along with a small
(c) mood disorders dose of nitrogenous fertiliser
(d) addictive disorders (c) Removal of all yellow leaves and spraying the remaining
green leaves with 2,4,5-trichlorophenoxy acetic acid
33. Strategies for Enhancement in (d) Application of iron and magnesium to promote synthesis
of chlorophyll
Food Production 334. Of the world’s top five crops (in terms of annual
330. Consider the following four statements (I-IV) and production) [CBSE-AIPMT 1997]
select the option which includes all the correct ones (a) three belong to Poaceae (Gramineae), one to
only. Leguminosae, one to Solanaceae
I. Single cell Spirulina can produce large quantities (b) four belong to Poaceae, one to Leguminosae
of food rich in protein, minerals, vitamins, etc. (c) four belong to Poaceae, one to Solanaceae
II. Body weight-wise the microorganism (d) all five belong to Poaceae
Methylophilus methylotrophus may be able to
335. In crop improvement programme, haploids are
produce several times more proteins than the
important because they [CBSE-AIPMT 1989]
cows per day.
(a) require one half of nutrients
III. Common button mushrooms are a very rich
(b) are helpful in study of meiosis
source of vitamin-C.
(c) grow better under adverse conditions
IV. A rice variety has been developed which is very
(d) form perfect homozygous
rich in calcium. [CBSE-AIPMT 2012]

MODULE 3
www.jeebooks.in
502 NEET Test Drive

336. In cloning of cattle, a fertilised egg is taken out of (c) Biogas, commonly called gobar gas, is pure methane
the mother’s womb and [CBSE-AIPMT 2007] (d) Activated sludge-sediment in settlement tanks of sewage
(a) in the eight cell stage, cells are separated and cultured treatment plant is a right source of aerobic bacteria
until small embryos are formed which are implanted into
342. Match Column I with Column II and select the
the womb of other cows
correct option using the codes given below.
(b) in the eight cell stage, the individual cells are separated
[NEET 2016, Phase II]
under electrical field for further development in culture
media Column I Column II
(c) from this, up to eight identical twins can be produced A. Citric acid 1. Trichoderma
(d) the egg is divided into 4 pairs of cells which are implanted
B. Cyclosporin 2. Clostridium
into the womb of other cows
C. Statins 3. Aspergillus
337. One of the most important reason why wild plants D. Butyric acid 4. Monascus
should thrive is that these are good sources of
(a) unsaturated edible oils [CBSE-AIPMT 2000] Codes
(b) highly nutritive animals feed A B C D A B C D
(c) genes for resistance to diseases and pests (a) 3 1 2 4 (b) 3 1 4 2
(c) 1 4 2 3 (d) 3 4 1 2
(d) rare and highly sought after fruits of medical importance
338. Which one of the following pairs is mismatched? 343. Biological control component is central to advanced
agricultural production. Which of the following is
(a) Pila globosa — Pearl [CBSE-AIPMT 2007]
used as a third generation pesticide?
(b) Apis indica — Honey (a) Pathogens [CBSE-AIPMT 1998]
(c) Kenia lacca — Lac (b) Pheromones
(d) Bombyx mori — Silk (c) Insect repellents
339. The long-term prospects for a truly human (d) Insect hormone analogues
civilisation depend in a large measure on 344. One of the major difficulties in the biological
[CBSE-AIPMT 1996]
control of insect pests is the [CBSE-AIPMT 1995]
(a) the ability of humanity to moderate its fecundity
(a) practical difficulty of introducing the predator to specific
(b) increasing the food production areas
(c) colonisation of under populated areas (b) method is less effective as compared with the use of
(d) control of human diseases insecticides
(c) predator does not always survive when transferred to a
34. Microbes in Human Welfare new environment
(d) the predator develops a preference to other diets and may
340. Which of the following is wrongly matched in the itself become a pest
given table? [NEET 2016, Phase I]
345. Which of the following plants are used as green
Microbe Product Application manure in crop fields and in sandy soils?
[CBSE-AIPMT 2003]
(a) Monascus Statins Lowering of blood
(a) Saccharum munja and Lantana camara
purpureus cholesterol
(b) Dichanthium annulatum and Azolla nilotica
(b) Streptococcus Streptokinase Removal of clot from (c) Crotalaria juncea and Alhagi comelorum
blood vessel (d) Calotropis procera and Phyllanthus niruri
(c) Clostridium Lipase Removal of oil stains 346. Which one of the following statements is correct?
butylicum [CBSE-AIPMT 2007]
(d) Trichoderma Cyclosporin-A Immunosuppressive (a) Extensive use of chemical fertilisers may lead to
polysporum drug eutrophication of nearby water bodies
(b) Both Azotobacter and Rhizobium fix atmospheric nitrogen
341. Select the correct statement from the following. in root nodules of plants
[CBSE-AIPMT 2010] (c) Cyanobacteria such as Anabaena and Nostoc are
(a) Biogas is produced by the activity of aerobic bacteria on important mobilisers of phosphates and potassium for
animal waste plant nutrition in soil
(b) Methanobacterium is an aerobic bacterium found in rumen (d) At present it is not possible to grow maize without
of cattle chemical fertilisers

MODULE 3
www.jeebooks.in
The NEET Edge ~ Biology 503

35. Biotechnology : 354. Plasmids are suitable vectors for gene cloning
because [CBSE-AIPMT 2000]
Principles and Processes (a) these are small circular DNA molecules which can
347. Genetic engineering is possible, because integrate with host chromosomal DNA
[CBSE-AIPMT 1998] (b) these are small circular DNA molecules with their own
(a) the phenomenon of transduction in bacteria is replication origin site
well-understood (c) these can shuttle between prokaryotic and eukaryotic
(b) we can see DNA by electron microscope cells
(c) We can cut DNA at specific sites by endonucleases like (d) these often carry antibiotic resistance genes
DNase-I
(d) restriction endonucleases purified from bacteria can be used 355. The given figure is the diagrammatic
in vitro representation of the E. coli vector pBR322.
Which one of the given options correctly
348. A mutant strain of T4-bacteriophage R-II, fails to lyse identifies its certain component(s)?
the E. coli but when two strains R-IIx and R-IIy are [CBSE-AIPMT 2012]
mixed then they lyse the E. coli. What may be the
Eco RI Cla I Hind III
possible reason? [CBSE-AIPMT 2002]
(a) Bacteriophage transforms in wild Pvu I
Bam HI
(b) It is not mutated Pst I
ampR tetR
(c) Both strains have similar cistrons
(d) Both strains have different cistrons pBR322 Sal I
ori
349. What is the criterion for DNA fragment movement rop
on agarose gel during gel electrophoresis? [NEET 2017]
(a) The larger the fragment size, the farther it moves Pvu II
(b) The smaller the fragment size, the farther it moves (a) ori–original restriction enzyme
(c) Positively charged fragments move to farther end (b) rop–reduced osmotic pressure
(d) Negatively charged fragments do not move (c) Hind III, Eco RI–selectable markers
350. The DNA fragments separated on an agarose gel can (d) ampR, tetR–antibiotic resistance genes
be visualised after staining with [NEET 2017] 356. Which one is a true statement regarding DNA
(a) bromophenol blue (b) acetocarmine polymerase used in PCR? [CBSE-AIPMT 2012]
(c) aniline blue (d) ethidium bromide (a) It is used to ligate introduced DNA in recipient cells
351. Which vector can clone only a small fragment of (b) It serves as a selectable marker
DNA? [CBSE-AIPMT 2014] (c) It is isolated from a virus
(a) Bacterial artificial chromosome (d) It remains active at high temperature
(b) Yeast artificial chromosome
357. The genetic defect-Adenosine Deaminase (ADA)
(c) Plasmid deficinecy may be cured permanently by
(d) Cosmid [CBSE-AIPMT 2009]
352. The colonies of recombinant bacteria appear white in (a) periodic infusion of genetically engineered lymphocytes
contrast to blue colonies of non-recombinant bacteria having functional ADA cDNA
because of [NEET 2013] (b) administering adenosine deaminase activators
(a) non-recombinant bacteria containing b-galactosidase (c) introducing bone marrow cells producing ADA into cells
(b) insertional inactivation of a-galactosidase in non-recombinant at early embryonic stages
bacteria (d) enzyme replacement therapy
(c) insertional inactivation of a-galactosidase in recombinant
bacteria 358. Stirred-tank bioreactors have been designed for
(d) inactivation of glycosidase enzyme in recombinant bacteria [CBSE-AIPMT 2010]
(a) addition of preservatives to the product
353. PCR and restriction fragment length polymorphism (b) purification of the product
are the methods for [CBSE-AIPMT 2012] (c) ensuring anaerobic conditions in the culture vessel
(a) study of enzymes (b) genetic transformation (d) availability of oxygen throughout the process
(c) DNA sequencing (d) genetic fingerprinting

MODULE 3
www.jeebooks.in
504 NEET Test Drive

36. Biotechnology and Its Applications 366. The figure given below is a diagrammatic
representation of response of organisms to abiotic
359. Production of a human protein in bacteria by factors. What do (A), (B) and (C) represent
genetic engineering is possible because respectively? [CBSE-AIPMT 2010]
[CBSE-AIPMT 2005]
(a) bacterial cell can carry out the RNA splicing reactions (B)
(b) the human chromosome can replicate in bacterial cell
(c) the mechanism of gene regulation is identical in humans

Internal level
(A)
and bacteria (C)
(d) the genetic code is universal
360. An improved variety of transgenic basmati rice
[CBSE-AIPMT 2010] External level
(a) does not require chemical fertilisers and growth hormones
A B C
(b) gives high yield and is rich in vitamin-A
(a) Conformer Regulator Partial regulator
(c) is completely resistant to all insect pests and diseases of
paddy (b) Regulator Partial regulator Conformer
(d) gives high yield but has no characteristic aroma (c) Partial regulator Regulator Conformer
(d) Regulator Conformer Partial regulator
361. Genetic engineering has been successfully used for
producing [CBSE-AIPMT 2010] 367. Which one of the following processes during
(a) transgenic mice for testing safety of polio vaccine before decomposition is correctly described? [NEET 2013]
use in humans (a) Fragmentation–Carried out by organisms such as
(b) transgenic models for studying new treatments for certain earthworm
cardiac diseases (b) Humification–Leads to the accumulation of a dark coloured
(c) transgenic cow-Rosie which produces high fat milk for substance humus, which undergoes microbial action at a
making ghee very fast rate
(d) animals like bulls for farm work as they have super power (c) Catabolism–Last step in the decomposition under fully
anaerobic condition
362. The introduction of tDNA into plants involves (d) Leaching–Water soluble inorganic nutrients rise to the top
[CBSE-AIPMT 2015] layers of soil
(a) infection of the plant by Agrobacterium tumefaciens
(b) altering the pH of soil, heat-shocking the plants 368. The table below gives the populations
(c) exposing the plants to cold for a brief period (in thousands) of ten species (A- J) in four areas
(d) allowing the plant roots to stand in water
(I-IV) consisting of the number of habitats given
within brackets against each. Study the table and
363. Commonly used vectors for human genome answer the question which follows.
sequencing are [CBSE-AIPMT 2014]
(a) T-DNA (b) BAC and YAC Area and Species and their populations
(in thousands) in the areas
(c) Expression vectors (d) T/A cloning vectors number of
habitats A B C D E F G H I J
37. Organisms and Population I (11) 2.3 1.2 0.52 6.0 — 3.1 1.1 9.0 — 10.3

364. Which one of the following pairs is mismatched? II (11) 10.2 — 0.62 — 1.5 3.0 — 8.2 1.1 11.2
[CBSE-AIPMT 2005] III (13) 11.3 0.9 0.48 2.4 1.4 4.2 0.8 8.4 2.2 4.1
(a) Savanna — Acacia trees IV (12) 3.2 10.2 11.1 4.8 0.4 3.3 0.8 7.3 11.3 2.1
(b) Prairie — Epiphytes
(c) Tundra — Permafrost Which area out of I to IV shows maximum species
(d) Coniferous forest — Evergreen trees diversity? [CBSE-AIPMT 2008]
(a) II (b) III (c) IV (d) I
365. What is a keystone species? [CBSE-AIPMT 2004]
(a) A species which makes up only a small proportion of the
369. Consider the following four conditions (I-IV) and
total biomass of a community, yet has a huge impact on select a correct pair of them as adaptation to
the community’s organisation and survival environment in desert lizards. [CBSE-AIPMT 2011]
(b) A common species that has plenty of biomass, yet has a Conditions
fairly low impact on the community’s organisation
I. Burrowing in soil to escape high temperature.
(c) A rare species that has minimal impact on the biomass
II. Losing heat rapidly from the body during high
and on other species in the community
temperature.
(d) A dominant species that constitutes a large proportion of
III. Bask in sun when temperature is low.
the biomass and which affects many other species

MODULE 3
www.jeebooks.in
The NEET Edge ~ Biology 505

IV. Insulating body due to thick fatty dermis. 376. What type of human population is represented by
(a) III and IV (b) I and III the following age pyramid? [CBSE-AIPMT 2011]
(c) II and IV (d) I and II
370. Which of the following pairs is correctly matched?
Post-reproductive
[CBSE-AIPMT 2005]
(a) Uricotelism–Aquatic habitat
(b) Parasitism – Intraspecific relationship
(c) Excessive perspiration–Xeric adaptation Reproductive
(d) Stream-lined body–Aquatic adaptation
371. Gause’s principle of competitive exclusion states
Pre-reproductive
that [NEET 2016, Phase I]
(a) competition for the same resources excludes species (a) Vanishing population (b) Stable population
having different food preferences (c) Declining population (d) Expanding population
(b) no two species can occupy the same niche indefinitely for
the same limiting resources 377. A country with a high rate of population growth
(c) larger organisms exclude smaller ones through took measures to reduce it. The figure below shows
competition age sex pyramids of populations A and B twenty
(d) more abundant species will exclude the less abundant years apart. Select the correct interpretation about
species through competition them. [CBSE-AIPMT 2009]
Age
372. Competition for light, nutrients and space is most
70+
severe between [CBSE-AIPMT 1988] Males Females
(a) closely related organisms growing in different niches 60-69
(b) closely related organisms growing in the same area/niche 50-59

Age (in years)


(c) distantly related organisms growing in the same habitat ‘A’ 40-49
(d) distantly related organisms growing in different niches 30-39
373. Asymptote in a logistic growth curve is obtained, 20-29
when [NEET 2017] 10-19
(a) The value of ‘r’ approaches zero 0-9
(b) K = N
15 12 9 6 3 0 3 6 9 12 15
(c) K > N
Age
(d) K < N
70+
374. When does the growth rate of a population Males Females 60-69
following the logistic model equal zero? The 50-59

Age (in years)


logistic model is given as dN/dt = rN(l-N/K)
[NEET 2016, Phase I] ‘B’ 40-49
(a) when N nears the carrying capacity of the habitat 30-39
(b) when N/K equals zero 20-29
(c) when death rate is greater than birth rate 10-19
(d) when N/K is exactly one
0-9
375. The population of an insect species shows an 15 12 9 6 3 0 3 6 9 12 15
explosive increase in numbers during rainy season
Interpretations
followed by its disappearance at the end of the
season. What does this show? [CBSE-AIPMT 2007]
(a) ‘A’ is more recent and shows slight reduction in the growth
(a) S-shaped or sigmoid growth of this insect
rate
(b) The food plants mature and die at the end of the rainy (b) ‘B’ is earlier pyramid and shows stabilised growth rate
season (c) ‘B’ is more recent showing that population is very young
(c) Its population growth curve is of J-type (d) ‘A’ is the earlier pyramid and no change has occurred in
(d) The population of its predators increases enormously the growth rate

MODULE 3
www.jeebooks.in
506 NEET Test Drive

38. Ecosystem 382. In which of the following both pairs have correct
combination? [CBSE-AIPMT 2015]
378. Consider the following statements concerning food (a) Gaseous nutrient cycle – Carbon and nitrogen
chains. [CBSE-AIPMT 2008] Sedimentary nutrient cycle – Sulphur and phosphorus
I. Removal of 80% tigers from an area resulted in (b) Gaseous nutrient cycle – Carbon and sulphur
greatly increased growth of vegetation. Sedimentary nutrient cycle – Nitrogen and phosphorus
II. Removal of most of the carnivores resulted in an (c) Gaseous nutrient cycle – Nitrogen and sulphur
increased population of deers. Sedimentary nutrient cycle – Carbon and phosphorus
III. The length of food chains is generally limited to (d) Gaseous nutrient cycle – Sulphur and phosphorus
3-4 trophic levels due to energy loss. Sedimentary nutrient cycle – Carbon and nitrogen
IV. The length of food chains may vary from 2 to 8
trophic levels. 383. Given below is a simplified model of phosphorus
cycling in a terrestrial ecosystem with four blanks
Which of the two above statements are correct? (A-D). Identify the blanks. [CBSE-AIPMT 2014]
(a) I and II (b) II and III (c) III aqnd IV (d) I and IV
Consumers C
379. Identify the likely orgnaisms A, B, C and D in the
food web shown below. [CBSE-AIPMT 2012] D
A
Hawks
Lion Uptake
Snake Soil solution
Runoff
Foxes B
Garden
Owls lizard (C) A B C D
(D) Sparrow (a) Rock minerals Detritus Litter fall Producers
(b) Litter fall Producers Rock minerals Detritus
(A) Mice (B) Grasshopper (c) Detritus Rock minerals Producers Litter fall
(d) Producers Litter fall Rock minerals Detritus

384. During ecological succession, [CBSE-AIPMT 2015]


Vegetation/Seeds
(a) the gradual and predictable change in species
composition occur in a given area
A B C D
(b) the establishment of a new biotic community is very fast in
(a) Deer Rabbit Frog Rat its primary phase
(b) Dog Squirrel Bat Deer (c) the numbers and types of animals remain constant
(c) Rat Dog Tortoise Crow (d) the changes lead to a community that is in near equilibrium
with the environment and is called pioneer community
(d) Squirrel Cat Rat Pigeon
385. The correct sequence of plants in a hydrosere is
380. Given below is an imaginary pyramid of numbers. [CBSE-AIPMT 2009]
What could be one of the possibilities about certain (a) Oak ® Lantana ® Scirpus ® Pistia ® Hydrilla ® Volvox
organisms at some of the different levels? (b) Volvox ® Hydrilla ® Pistia ® Scirpus ® Lantana ® Oak
[CBSE-AIPMT 2012]
(c) Pistia ® Volvox ® Scirpus ® Hydrilla ® Oak ® Lantana
TC 10 (d) Oak ® Lantana ® Volvox ® Hydrilla ® Pistia ® Scirpus

SC 50 386. If the forest cover is reduced to half, what is most


likely to happen on a long basis? [CBSE-AIPMT 1996]
(a) Tribals living in these areas will starve to death
PC 500
(b) Cattle in these and adjoining areas will die due to lack of
fodder
PP 1 (c) Large areas will become deserts
(d) Crop breeding programmes will suffer due to a reduced
(a) Level PC is insects and level SC is small insectivorous birds availability of variety of germplasm
(b) Level PP is phytoplanktons in sea and whale on top level TC
(c) Level one PP is peepal trees and the level SC is sheep 387. Match the following and select the correct option.
[CBSE-AIPMT 2014]
(d) Level PC is rats and level SC is cats
Column I Column II
381. In Grass–Deer–Tiger food chain, grass biomass is
A. Earthworm 1. Pioneer species
one tonne. The tiger biomass shall be
[CBSE-AIPMT 1994] B. Succession 2. Detritivore
(a) 100 kg (b) 10 kg (c) 200 kg (d) 1 kg C. Ecosystem service 3. Natality
D. Population growth 4. Pollination

MODULE 3
www.jeebooks.in
The NEET Edge ~ Biology 507

Codes
A B C D A B C D
40. Environmental Issues
(a) 1 2 3 4 (b) 4 1 3 2 395. A scrubber in the exhaust of a chemical industrial
(c) 3 2 4 1 (d) 2 1 4 3 plant removes [CBSE-AIPMT 2014]
(a) gases like sulphur dioxide
39. Biodiversity and Conservation (b) particulate matter of the size 5 micrometer or above
388. Which one of the following pairs of organisms are (c) gases like ozone and methane
exotic species introduced in India?[CBSE-AIPMT 2007] (d) particulate matter of the size 2.5 micrometer or less
(a) Ficus religiosa, Lantana camara
(b) Lantana camara, water hyacinth 396. Steps taken by the Government of India to control
(c) Water hyacinth, Prosopis cineraria air pollution include [CBSE-AIPMT 2009]
(d) Nile perch, Ficus religiosa (a) compulsory mixing of 20% ethyl alcohol with petrol and
20% biodiesel with diesel
389. Which one of the following expanded forms of the (b) compulsory PUC (Pollution Under Control) certification of
following acronyms is correct? [CBSE-AIPMT 2011] petrol driven vehicles, which tests for carbon monoxide
(a) UNEP — United Nations Environmental Policy and hydrocarbons
(b) EPA — Environmental Pollution Agency (c) permission to use only pure diesel with a maximum of
(c) IUCN — International Union for Conservation of Nature 500 ppm sulphur as fuel for vehicles
and Natural Resources (d) use of non-polluting Compressed Natural Gas (CNG) only
(d) IPCC — International Panel for Climate Change as fuel by all buses and trucks
390. Select the correct statement about biodiversity. 397. Acid rains are produced by [CBSE-AIPMT 1988]
[CBSE-AIPMT 2012] (a) excess NO 2 and SO 2 from burning fossil fuels
(a) The desert areas of Rajasthan and Gujarat have a very (b) excess production of NH3 by industry and coal gas
high level of desert animal species as well as numerous (c) excess release of carbon monoxide by incomplete
rare animals
combustion
(b) Large scale planting of Bt cotton has no adverse effect on
(d) excess formation of CO 2 by combustion and animal
biodiversity
respiration
(c) Western Ghats have a very high degree of species
richness and endemism 398. A lake near a village suffered heavy mortality of
(d) Conservation of biodiversity is just a fad pursued by the fishes within a few days. Consider the following
developed countries reasons for this
391. Identify the odd combination of the habitat and the I. Lots of urea and phosphate fertiliser were used in
particular animal concerned. [CBSE-AIPMT 2007] the crops in the vicinity.
(a) Dachigam National Park — Snow leopard II. The area was sprayed with DDT by an aircraft.
(b) Sunderbans — Bengal tiger III. The lake water turned green and stinky.
(c) Periyar — Elephant IV. Phytoplankton populations in the lake declined
(d) Rann of Kutch — Wild ass initially thereby greatly reducing photosynthesis.
392. A number of natural reserves have been created to Which two of the above were the main causes of
conserve specific wildlife species. Identify the fish mortality in the lake? [CBSE-AIPMT 2008]
correct combination from the following. (a) II and III (b) III and IV
(a) Gir forest — Tiger [CBSE-AIPMT 1996] (c) I and III (d) I and II
(b) Kaziranga — Elephants
(c) Rann of Kutch — Wild ass 399. In which one of the following, the BOD
(d) Manas Wildlife Sanctuary — Musk deer
(Biochemical Oxygen Demand) of Sewage (S),
Distillery Effluent (DE), Paper Mill Effluent (PE)
393. Which of the following is the correct matching pair and Sugar Mill Effluent (SE) have been arranged
of a sanctuary and its main protected wild animal? in ascending order? [CBSE-AIPMT 2007]
[CBSE-AIPMT 1995] (a) SE < S < PE < DE
(a) Gir — Lion (b) Sariska — Tiger (b) SE < PE < S < DE
(c) Sunderban — Rhino (d) Kaziranga — Musk deer (c) PE < S < SE < DE
394. Which of the following is correctly matched? (d) S < DE < PE < SE
[NEET 2016, Phase II] 400. Lead concentration in blood is considered alarming
(a) Aerenchyma — Opuntia if it is [CBSE-AIPMT 2004]
(b) Age pyramid — Biome (a) 20 mg/100 mL (b) 30 mg/100 mL
(c) Parthenium hysterophorus — Threat to biodiversity (c) 4-6 mg/100 mL (d) 10 mg/100 mL
(d) Stratification — Population

MODULE 3
www.jeebooks.in

Answer Sheet
1. (b) 2. (c) 3. (d) 4. (c) 5. (c) 6. (d) 7. (c) 8. (d) 9. (a) 10. (c)
11. (a) 12. (b) 13. (c) 14. (c) 15. (a) 16. (a) 17. (a) 18. (b) 19. (a) 20. (c)
21. (c) 22. (a) 23. (d) 24. (d) 25. (a) 26. (d) 27. (c) 28. (b) 29. (b) 30. (c)
31. (d) 32. (a) 33. (b) 34. (b) 35. (c) 36. (c) 37. (b) 38. (d) 39. (c) 40. (b)
41. (a) 42. (c) 43. (b) 44. (c) 45. (d) 46. (a) 47. (b) 48. (a) 49. (c) 50. (a)
51. (a) 52. (c) 53. (d) 54. (b) 55. (a) 56. (b) 57. (b) 58. (d) 59. (c) 60. (a)
61. (b) 62. (d) 63. (c) 64. (b) 65. (a) 66. (b) 67. (b) 68. (a) 69. (c) 70. (c)
71. (b) 72. (c) 73. (b) 74. (d) 75. (d) 76. (d) 77. (d) 78. (a) 79. (d) 80. (b)
81. (b) 82. (d) 83. (a) 84. (c) 85. (c) 86. (c) 87. (d) 88. (a) 89. (c) 90. (c)
91. (a) 92. (c) 93. (c) 94. (d) 95. (a) 96. (b) 97. (b) 98. (b) 99. (d) 100. (c)

101. (a) 102. (a) 103. (a) 104. (a) 105. (d) 106. (c) 107. (a) 108. (b) 109. (b) 110. (b)
111. (c) 112. (d) 113. (a) 114. (c) 115. (d) 116. (c) 117. (a) 118. (a) 119. (a) 120. (c)
121. (a) 122. (d) 123. (a) 124. (a) 125. (c) 126. (d) 127. (c) 128. (d) 129. (b) 130. (c)
131. (b) 132. (d) 133. (d) 134. (b) 135. (d) 136. (b) 137. (d) 138. (d) 139. (b) 140. (d)
141. (d) 142. (d) 143. (d) 144. (b) 145. (d) 146. (d) 147. (d) 148. (b) 149. (a) 150. (c)
151. (a) 152. (d) 153. (b) 154. (c) 155. (b) 156. (b) 157. (c) 158. (d) 159. (d) 160. (c)
161. (d) 162. (d) 163. (a) 164. (b) 165. (c) 166. (d) 167. (b) 168. (c) 169. (a) 170. (a)
171. (d) 172. (c) 173. (b) 174. (a) 175. (d) 176. (b) 177. (d) 178. (c) 179. (a) 180. (c)
181. (b) 182. (c) 183. (b) 184. (b) 185. (a) 186. (a) 187. (c) 188. (a) 189. (a) 190. (c)
191. (b) 192. (a) 193. (b) 194. (a) 195. (a) 196. (c) 197. (a) 198. (b) 199. (c) 200. (b)

201. (b) 202. (b) 203. (d) 204. (a) 205. (c) 206. (d) 207. (a) 208. (a) 209. (b) 210. (d)
211. (a) 212. (a) 213. (c) 214. (a) 215. (d) 216. (d) 217. (d) 218. (c) 219. (c) 220. (b)
221. (c) 222. (b) 223. (d) 224. (d) 225. (d) 226. (c) 227. (a) 228. (a) 229. (b) 230. (c)
231. (b) 232. (a) 233. (a) 234. (a) 235. (d) 236. (a) 237. (a) 238. (a) 239. (d) 240. (b)
241. (d) 242. (c) 243. (b) 244. (a) 245. (b) 246. (a) 247. (a) 248. (a) 249. (b) 250. (c)
251. (d) 252. (b) 253. (b) 254. (a) 255. (c) 256. (c) 257. (a) 258. (c) 259. (c) 260. (c)
261. (b) 262. (b) 263. (a) 264. (a) 265. (c) 266. (b) 267. (d) 268. (d) 269. (a) 270. (d)
271. (c) 272. (c) 273. (a) 274. (a) 275. (b) 276. (b) 277. (c) 278. (b) 279. (c) 280. (a)
281. (b) 282. (c) 283. (a) 284. (a) 285. (c) 286. (d) 287. (a) 288. (d) 289. (c) 290. (a)
291. (a) 292. (c) 293. (d) 294. (c) 295. (c) 296. (c) 297. (b) 298. (a) 299. (a) 300. (b)

301. (c) 302. (b) 303. (a) 304. (c) 305. (b) 306. (c) 307. (c) 308. (c) 309. (c) 310. (d)
311. (c) 312. (a) 313. (a) 314. (d) 315. (d) 316. (a) 317. (a) 318. (d) 319. (d) 320. (c)
321. (d) 322. (d) 323. (d) 324. (b) 325. (c) 326. (b) 327. (a) 328. (d) 329. (a) 330. (d)
331. (c) 332. (b) 333. (d) 334. (c) 335. (d) 336. (a) 337. (c) 338. (a) 339. (d) 340. (c)
341. (d) 342. (b) 343. (b) 344. (d) 345. (c) 346. (a) 347. (d) 348. (d) 349. (b) 350. (d)
351. (c) 352. (c) 353. (d) 354. (b) 355. (d) 356. (d) 357. (a) 358. (d) 359. (d) 360. (b)
361. (a) 362. (a) 363. (b) 364. (b) 365. (a) 366. (d) 367. (a) 368. (c) 369. (b) 370. (d)
371. (b) 372. (b) 373. (b) 374. (d) 375. (c) 376. (c) 377. (a) 378. (b) 379. (a) 380. (a)
381. (b) 382. (a) 383. (c) 384. (a) 385. (b) 386. (c) 387. (d) 388. (c) 389. (c) 390. (c)
391. (a) 392. (c) 393. (b) 394. (c) 395. (a) 396. (b) 397. (a) 398. (d) 399. (d) 400. (b)

MODULE 3
www.jeebooks.in

EXPLANATIONS
1. The Living World Herbarium is a storehouse of collected to penicillin. Mycoplasma are inhibited
1. All living organisms interact with their plant specimens that are dried, pressed by metabolic inhibitors like
environment and show progressive and preserved on sheets. Botanical chloramphenicol and tetracycline.
evolution. They can sense and respond gardens have collection of living plants
to environmental uses. On the other for reference. Key is a taxonomic aid for 3. Kingdom—Protista
hand, reproduction, growth and the identification of plants and animals
15. Statement (a) is wrong. Diatoms are
movement cannot be all inclusive based on the similarities and
single-celled plant-like protists that
defining properties of living organisms. dissimilarities.
produce intricately structured cell walls
2. Autotrophs are the organisms which are made of nano(–) silica particles (SiO 2 )
capable of synthesising their food
2. Kingdom—Monera and which are indestructible. Rest of the
themselves from organic and inorganic Viruses options are correct.
substances. The autotrophs from the list 8. Viroids differ from viruses in having 16. Auxospores and hormocysts are formed
are Nostoc, Chara, Nitrosomonas, RNA molecules without protein coat. by several diatoms and a few
Nitrobacter, Porphyra and Wolffia. Viruses on the other hand possess DNA cyanobacteria, respectively.
3. Biological names originate from Latin or RNA with a protein coat as their Bacillariophyceae members (diatoms)
language and are printed in italics. genetic material. Viruses can infect a are microscopic, eukaryotic, unicellular
Nomenclature is the science of wide range of organisms including or colonial coccoid algae. These algae
providing distinct and proper name to plants, animals or bacteria, while viroids sexually reproduce by the formation of
the organisms. infect only plants. auxospores in most cases. Bozi (1914)
9. All statements are correct except the and Fermi (1930) reported that short
4. Statements I and IV are correct. The
statement (a), which can be corrected as sections of living cells at the tips of the
correct forms of II and III are as follows
trichomes of Wertiella lanosa become
II. Photoperiod does affect the Viroids were discovered by TO Diener invested by a thick, lamellated,
reproduction in plants. in 1971 as a new infectious agent that pigmented sheath. Such mullticellular
was smaller than virus. spore-like structures, function as
III. Binomial nomenclature was given
by Carolus Linnaeus. 10. Statement (c) is wrong about viroids. perennating bodies. They are specially
Viroids are infectious, non-protein modified hormogones and are called
5. Carbohydrates, the most abundant
coding, highly structured with small hormospores or hormocysts.
biomolecules on earth are produced by
circular RNAs, which have the ability to 17. Chrysophytes are placed under the
bacteria, algae and green plants. Some
replicate autonomously. These contain kingdom–Protista. This group includes
photosynthetic bacteria such as
RNA of low molecular weight and diatoms and golden algae (desmids).
‘Rhodopseudomonas’ can prepare
induce diseases in higher plants only. Most of them are photosynthetic. In
carbohydrates. But during this type of
food synthesis, O 2 is not evolved 11. The statement (a) is not true for diatoms, the cell walls form two thin
because in this case hydrogen donor is retroviruses. Retroviruses are so named overlapping cell wall plates, which fit
other than H2O. Algae (green and because they contain enzyme reverse together as in a soap box.
blue-green) and all green plant cells transcriptase or RNA dependent DNA 18. Freshwater protozoans live in hypotonic
prepare their food (carbohydrate) polymerase. The genetic material of environment so, for regulation of excess
through photosynthesis. Here, these viruses is RNA. It differs from of water which comes in the protoplasm
hydrogen ions are donated by water other RNA viruses in working pattern of through the process of endosmosis,
molecules by the process of photolysis first synthesis of DNA via reverse contractile vacuoles have developed.
of water, i.e. O 2 is released during this transcription, e.g. Rous Sarcoma Virus, When these protozoans are placed in
type of food synthesis. HIV, etc. marine water, i.e. hypertonic water, the
6. Herbarium does not have information on 12. All the viruses do not possess helical contractile vacuoles disappear because
height of the plant. It is a collection of symmetry. Two types of symmetry are the process of endosmosis does not
plants that usually have been dried, recognised in viruses, which occur and thus, water does not come in
pressed, preserved plant on sheets and correspond to the two primary shapes, the protoplasm.
are arranged in accordance with any rod and spherical. Rod-shaped viruses
accepted system of classification for have helical symmetry and spherical 4. Kingdom—Fungi
future reference and study. viruses have icosahedral symmetry.
19. Cellulose is a major component of cell
7. Statement (c) is incorrect. It is corrected 13. Barophilic prokaryotes grow and multiply wall of Pythium fungi. It belongs to
as, museums have collection of in very deep marine sediments. These Oomycetes (e.g. Pythium) and
preserved plant and animal specimens are generally found in deep sea floors Hyphochytridiomycetes. Fungal cell
for study and reference. Specimens are where pressure is more than 38 MPa. wall contains 80–90% carbohydrates,
preserved in the containers in 14. Penicillin is an antibiotic which acts on the remainder being proteins and lipids.
preservative solutions. cell wall and Mycoplasma lacks cell The typical feature of fungal cell wall is
wall. Thus, Mycoplasma is not sensitive the presence of chitin.

MODULE 3
www.jeebooks.in
510 NEET Test Drive

20. Ustilago has haplontic life cycle. In their 27. Ectocarpus and Fucus respectively Oogamy is the fusion of non-motile egg
sexual phase, only zygospore is diploid show haplodiplontic and diplontic with motile sperm. The gametes differ
structure, which divides by meiotic cell life cycle, respectively. both morphologically as well as
divisions to produce haploid zoospores. All physiologically. It occurs in
Sporophyte
others are haploid, such a sexual cycle is (2n) Chlamydomonas, Fucus, Chara,
termed as haploid or haplontic. Volvox, etc.
21. All statements are correct except 31. In Sphagnum, male and female
statement (c), which can be corrected as Germination gametophytes are independent and
2n phase Sporangium
Morels and truffles are edible and are free-living. In bryophytes, the most
(2n)
members of Ascomycetes in conspicuous phase in life cycle is the
Zygote gametophyte. It is independent and
kingdom–Fungi. They are also known as (2n)
sac fungi. Sporic concerned with reproduction.
meiosis
22. Bird’s nest fungi (Nidulariales) and puff ball Syngamy 32. Pteridophytes are also called vascular
fungi (Lycoperdales) belong to cryptogams as these have a
Basidiomycetes. The common example of n phase Meiospores well-developed vascular system but are
class—Basidiomycetes are smut, rusts, the Gametes (n) non-flowering plants, e.g. Equisetum.
mushrooms, the toad stools, the puff balls (n) (n) 33. In the ectophloic siphonostele, the
and the pore fungi. xylem surrounds pith and this xylem is
Sex organs Germination surrounded by phloem, pericycle and
23. Trichoderma harzianum are found in damp
(n) endodermis, respectively, e.g.
soils. They have an inhibitory effect on the
growth of the mycelium of Pythium. They Gametophyte Osmunda and Equisetum.
serve to suppress fungi causing damping off (n)
Phloem
disease of the seedlings and thereby Haplodiplontic life cycle of Ectocarpus
influence favourably the growth of crops. In Ectocarpus, sporic meiosis Xylem
24. The fungus Claviceps purpurea is occurs and haploid biflagellate
responsible for ergot disease of rye which meiozoospores are formed. They
lowers the yield of rye plant. It belongs to germinate to produce gametophytic Pith
Ascomycota division. thalli. The gametophytes liberate
25. Mucor shows the best growth on a piece gametes, which fuse to form diploid
of bread at a temperature of about 25°C, zygote, which gives rise to a diploid
plant. Ectophloic siphonostele
relative humidity of about 95% in a moist
and shady place. Mucor is a saprophytic 28. Only option (b) is wrong and all 34. The statement (b) is correct.
fungus belonging to the order–Mucorales other options are correct. Algin Sequoia is one of the tallest tree
and family–Mucoraceae and grows on extracted from brown algae, e.g. species, known as red wood tree. It is a
decaying dung and on some food stuffs Laminaria, etc., is a hydrocolloid gymnospermic plant.
having higher moisture contents. used in shaving creams, jellies,
Salvinia is an angiosperm, but Ginkgo
flameproof plastic, etc.
and Pinus are gymnosperms.
5. Plant Kingdom Carrageenan is extracted from red
Gymnosperms are well-adapted to
algae like Chondrus and used as
26. Zygotic meiosis is represented in the extremes of climate and are
emulsifier and clearing agent.
haplontic life cycle of many algae including heterosporous.
Chlamydomonas. In such a life cycle, all 29. The statement (b) is wrong. Mucor
They produce two types of spores, i.e.
cells are haploid except zygote. This is (fungus) belongs to the
microspores and megaspores.
because meiosis occurs in the zygote itself class—Zygomycetes. The members
resulting into four haploid cells that give of Zygomycetes bear non-motile, 35. In gymnosperms (e.g. Pinus, Cycas,
rise to haploid plants. non-flagellated gametes. etc.), the male and female
gametophytes do not have an
Zygote 30. Statement (c) is wrong as,
independent free-living existence. They
Sy (2n) Meiosis Oomycetes include water moulds,
ng remain within the sporangia retained on
am white rusts and downy mildews. In
y Spores the sporophytes.
(n) these, female gamete is large and
A 36. Statements I, II and III are correct.
non-motile, whereas, male
Gametogenesis B gamete is small and motile. Statement IV is incorrect and can be
Haplontic Isogametes are found in algae like corrected as the sporophyte in mosses
Ulothrix, Chlamydomonas, is more elaborate than in liverworts. The
Spirogyra, etc., which are similar sporophyte is dependent upon the
in structure, function and gametophyte for nutrition.
behaviour. Anisogametes are 37. The presence of vessels in the xylem is
Gametophyte
(n) found in Chlamydomonas in which an angiospermic character found in
one gamete is larger and Gnetum which distinguishes it from
Life cycle pattern : Haplontic
non-motile and the other one is Cycas and Pinus.
motile and smaller.

MODULE 3
www.jeebooks.in
The NEET Edge ~ Biology 511

Gnetum resembles angiosperms in 41. Column I Column II long, greenish-brown, cylindrical body
many other aspects also like with smooth scaleless, slimy skin,
Peritrichous Escherichia coli
(i) The leaves in Gnetum have jawless mouth, etc. They lay eggs in
flagella (flagella (a bacterium)
reticulate venation that is an freshwater, but their ammocoete larvae
all over the body) after metamorphosis return to the ocean.
angiospermic character.
Living fossil Ginkgo biloba 46. Option (a) is not correctly matched,
(ii) In Gnetum, female gametophyte is
(maiden hair because molluscs are bilaterally
only partly cellular before
tree) symmetrical, triploblastic, coelomate,
fertilisation and becomes
completely cellular only after Rhizophore Selaginella soft-bodied animals. Their soft body is
fertilisation. Some of the free nuclei (a form of aerial (a pteridophyte) covered by a calcareous shell and is
act as eggs as there are no adventitious unsegmented with a distinct head,
archegonia. roots) muscular foot and visceral hump,
The short apices in Gnetum and Smallest Wolffia e.g. Pila (apple snail), Sepia (cuttle
angiosperms show a distinct tunica and fish), Pinctada (pearl oyster), etc. Other
flowering plant
corpus configuration. options are correctly matched.
Largest perennial Macrocystis
The cortex of stem of Pinus is
47. The option (b) is correctly matched.
algae
transversed by large resin ducts or The members of class–Chondrichthyes
canals. Each duct or canal is lined by a are marine animals with streamlined
layer of thin-walled parenchymatous
6. Animal Kingdom body and have cartilaginous
glandular secretory cells constituting 42. The statement (c) is correct. endoskeleton without even single
epithelium. The epithelial cells secrete Spongilla have choanocyte cells. These exception.
resin into canal. Resin is the chief source are absent in leech, dolphin and Chordata These possess a notochord,
of turpentine. penguin. These distinctive cells line the a hollow dorsal nerve cord, pharyngeal
38. Flagellated male gametes are present interior body walls of sponges. These slits, an endostyle and a post anal tail
in Riccia, Dryopteris and Cycas. These cells have a central flagellum that is for atleast some period of their life
need a film of water to reach surrounded by a collar of microvilli. cycle.
archegonia and fertilise the egg cell. Choanocytes are versatile cells. Mammalia These are a clade of
39. Lilium (angiosperm) possesses the 43. Ascaris is characterised by the endothermic amniotes distinguished
male gametophyte with least number of presence of neither true coelom nor from reptiles and birds by the
cells. metamerism. It is endoparasite of man. possession of hair, three middle ear
It inhabits the small intestine more bones, mammary glands and a
The number of cells in male
frequently of children than of adults. neocortex.
gametophyte shows the pattern of
The body is elongate, cylindrical and Reptiles These are an evolutionary
reduction from bryophytes to
gradually tapering at both ends. There clade of animals, comprising today’s
angiosperms. In angiosperms, it is
is present a fluid-filled cavity between turtles, crocodilians, snakes, lizards and
reduced to about 2-3 celled and called
body wall and visceral organs. This tuatara, their extinct relatives and some
as pollen grains.
cavity is not true coelom as it is not of the extinct ancestors of mammals.
The number of cells in male lined by coelomic epithelium, has no
gametophyte decreases in the following relation with reproductive and excretory 48. The option (a) is correctly matched.
order organs and develops from blastocoel. Salamandra (salamander) is a member
Funaria > Pteris > Pinus > Lilium This body cavity is referred as of class–Amphibia. It has a tympanum
40. Replum is present in the ovary of pseudocoel. representing the ear and fertilisation is
flowers of mustard. The gynoecium in 44. The statement (c) is correct with external.
family—Cruciferae (Brassicaceae) is respect to common cockroach. Pteropus are mammals, so they are
bicarpellary, syncarpous, ovary In cockroach, mandibles are a pair of viviparous. Aurelia has cellular-tissue
superior, unilocular with parietal hard, strong, large, dark-coloured, grade of body organisation. Ascaris has
placentation. At maturity, ovary triangular structures which move in unsegmented body and belongs to
becomes bilocular due to the horizontal motion and crush food phylum–Aschelminthes.
formation of false septum (replum), between them. 49. Characteristic Animal
e.g. mustard. Gizzard or proventriculus has an outer Morphological feature
In family–Asteraceae (Compositae), the layer of thick circular muscles and thick
Bilateral symmetry — Liver fluke, Taenia
gynoecium is bicarpellary syncarpous, inner cuticle forming six highly chitinous
ovary inferior, unilocular, one ovule in Jointed appendages — Prawn, cockroach,
plate called teeth. The gizzard acts as
each locule, basal placentation and the grinding chamber and helps in scorpion
stigma branched, e.g. sunflower. grinding the food particles. Other Ventral solid central — Scorpion, spider,
statements do not hold true with respect nervous system cockroach
In family–Fabaceae (Leguminosae), the
gynoecium is monocarpellary, to cockroach. Metameric — Annelids
unilocular with superior ovary and 45. Petromyzon (the lamprey) belongs to segmentation
marginal placentation, e.g. pea. the section Agnatha of the Radial symmetry — Sea anemone
subphylum—Vertebrata. They have

MODULE 3
www.jeebooks.in
512 NEET Test Drive

50. Exception in class–Reptilia, crocodile 57. A superior ovary is an ovary attached to 63. The correct statement is (c), because
has four-chambered heart. In Obelia, the receptacle above the attachment of flower is highly condensed and
the alternation of generations is called other floral parts. In this case, the flower modified shoot meant for sexual
metagenesis in which an asexual is said to be hypogynous, e.g. China reproduction (Dr. Goethe; 1790). During
polypoid generation appears to rose, mustard, brinjal, potato, onion and the course of evolution, the nodes of the
alternate regularly with a sexual tulip. axis of shoot came in contact so that
medusoid generation. Thecodont Guava and cucumber have inferior internodes got reduced and leaves get
dentition is found in mammals. ovaries or epigynous flowers. modified and specialised to form four
51. An arthropod body consists of head, essential whorls of flower.
So, out of 8 given names, 6 have
thorax and abdomen, in some cases, superior ovaries. The other statements can be corrected
head and thorax may be fused to form as
58. Fig, pineapple and mulberry are
cephalothorax. Class—Insecta have Seeds of orchids lack endosperm.
composite fruits.
body divided into head, thorax and Placentation in primrose is free central
abdomen. Botanical placentation. In tomato, fruit is a berry.
Plant Fruit Inflorescence
52. A clasper is a male anatomical structure name 64. The main difference between perisperm
found in some groups of animals and Fig Ficus Syconus Hypanthodium and endosperm is that perisperm is
used in mating. Male cartilaginous carica present in seeds as persistent nucellus
fish-like shark have claspers formed Pineapple Annanas Sorosis Spike and endosperm is present in
from the posterior portion of their pelvic sativus developing embryo as its reserved food
fin which serves as intermittent organs Mulberry Sorosis Catkin which is completely consumed by it
Morus
used to channel semen into the during development.
sp.
female’s cloaca during mating.
53. Kangaroo, hedgehog, dolphin and loris 59. All the given plants except bitter gourd,
8. Anatomy of Flowering
are all mammals. These give birth to pumpkin, cucumber, guava, plum and Plants
young ones. rose are hypogynous flowers, i.e. 15. 65. Palisade parenchyma is absent in
54. Statement (b) is wrong regarding Hypogynous flowers have gynoecium leaves of Sorghum (family–Poaceae).
prawn. It does not have two pairs of present above all other floral parts and It is a monocot plant. The mesophyll of
antennae instead it has one pair of with superior ovary. monocot leaf is not differentiated into
antennae and one pair of antennules. palisade and spongy parenchyma, all
60. Floral formula of chilli (Capsicum
_ being thin-walled, chlorophyllous and
7. Morphology of Flowering annum) is Å O
+ K ( 5 )C ( 5 ) A 5G ( 2 ) . irregularly compactly arranged with
Plants It belongs to family–Solanaceae. In this fewer intercellular spaces.
55. Actinomorphic (star-shaped) can be family, flower is actinomorphic, Palisade cells occur in dicotyledonous
divided into 3 or more identical complete and bisexual, calyx has five plants and also in few of the net-veined
sections, which are related to each sepals which are gamosepalous monocots. Mostly the members of
other by rotation about the centre of the showing valvate aestivation, corolla has families, the Araceae and
five petals which are gamopetalous Dioscoreaceae.
flower, e.g. China rose. Zygomorphic
flowers can be divided by only a single showing valvate aestivation, 66. Tracheids differ from other tracheary
plane into two mirror image halves, e.g. androecium has five free stamens elements in being imperforate.
orchids. (polyandrous) but epipetalous, Tracheids and vessels both are called
gynoecium is bicarpellary, syncarpous, tracheary elements because their main
Valvate aestivation The sepals or petals
bilocular with superior ovary having function is conduction of sap. Tracheids
in a whorl just touch one another at the
axile placentation. are the specific cells which have pits to
margin, e.g. Calotropis.
61. The fruit of Ananas comosus (pineapple support upwards and lateral conduction
Twisted aestivation One margin of the of water sap.
or ananas) is sorosis, (a type of multiple
appendage overlaps that of the next Tracheids are comparatively short and
fruits), developing from spike, spadix or
one, e.g. China rose. In imbricate single cell, while vessels have more
catkin. In this type, the flowers
aestivation, the margins of sepals or than one cell and up to 10 cm long.
associate by their succulent tepals, the
petals overlap but not necessarily in
axis bearing them grows and becomes 67. The water containing cavities are found
specific direction, e.g. Cassia.
fleshy or woody. Thus, the whole in the vascular bundles of maize. In this,
The flowers containing superior ovary inflorescence turns into a compact fruit. vascular bundles are conjoint, collateral
are called hypogynous flowers. The 62. The seeds of castor (Ricinus communis, and closed. In this, vessels and xylem
other floral whorls like androecium, family–Euphorbiaceae) are parenchyma cells dissolve forming a
petals and sepals arise successively endospermic dicot seeds. They water containing schizolysigenous
below ovary. possess endosperm which acts as the cavity called protoxylem cavity or
56. Only Salvia, mustard, radish and turnip food storage tissue of seed. They also lacuna or lysigenous cavity. Protoxylem
have stamens of different length in their possess perisperm and caruncle. cavity and protophloem may be absent
flowers. Perisperm is persistent nucellus also in the smaller vascular bundles in maize.
Hence, correct answer is (b). Mustard, found in Nymphaea. The caruncle is 68. Some of the endodermal cells
turnip, radish belong to Brassicaceae. fleshy outgrowth near hilum of the present opposite to the xylem patches
seeds. are thin-walled and are called passage

MODULE 3
www.jeebooks.in
The NEET Edge ~ Biology 513

cells or transfusion cells. In roots, Adipose tissue is also loose connective


endodermis is the innermost layer of tissue, while the tendon is a dense 10. Cell : The Unit of Life
cortex. Passage cells help in transfer of connective tissue, which connects the 77. The resolution power of the electron
water and dissolved salts from cortex muscles with the bone. microscope is much higher than that of
directly into the xylem and ultimately to Cartilage is a skeletal connective tissue. the light microscope.
the pericycle.
It has chondrocytes that produce a As an average, the resolving power of
69. Tyloses are the extensions of xylem large amount of extracellular matrix a light microscope is 0.25–0.3 mm while
parenchyma cells into the vessel composed of collagen fibre. that of electron microscope is 2-10Å
elements. These are the structures though theoretically, it is 0.25Å. The
73. The basophils are probably like mast
found in the woody tissues of dicot magnification range of light microscope
cells of connective tissue. They release
stems. is 2000-4000 while that of electron
heparin (a natural anticoagulant),
histamine and serotonin. Their nucleus microscope is 100000-300000.
is usually three-lobed and their granules 78. Resolving power or resolution is the
take basic stain strongly. ability of the lens to distinguish fine
Parenchyma
Monocytes are largest of all types of details and structure. Specifically, it
leucocytes. Their nucleus is refers to the ability of the lenses to
bean-shaped. They are motile and distinguish between two points which
phagocytic in nature and engulf are a specified distance apart.
bacteria and cellular debris. Generally, Resolving power depends on two
they change into macrophages after factors
entering tissue spaces.
(a) Wavelength of light used for
Eosinophils have two-lobed nucleus. illumination.
They are non-phagocytic and help in
dissolving blood clot. Their number (b) Power of objective lenses.
increases in people with allergic Wavelength of light
Resolving power =
Structure of tyloses conditions such as asthma or hay fever. 2 ´ NA
in woody tissue Neutrophils are most numerous of all where, NA is numerical aperture.
(a) In longitudinal section (b) In cross-section leucocytes. They eat harmful germs and
Since, the limit of resolving power of a
are therefore, phagocytic in nature.
microscope is fixed by the structure of
Their nucleus is many lobed and stains
9. Structural Organisation in weakly with both acid and basic stains.
light, the shortest wavelength of visible
Animals light will give the maximum resolution.
74. Option (d) is correctly matched with its Among yellow, green, red and blue light
70. Cuboidal epithelium is present in the location. colour, blue (500 nm) have shortest
tubular parts of nephron (PCT and The smooth muscle is present in the wavelength so, it will give best
DCT). It consists of short, cube-shaped wall of intestine. resolution.
cells with round nuclei located in the Columnar epithelium is present in the
centre of the cell. These cells often 79. Cell membrane of prokaryotes is
lining of stomach. Tendon is dense
forms microvilli to increase the structurally similar to that of eukaryotes.
connective tissue and connects muscle
absorptive surface area of the cell. So, it does not differ in E. coli and
to bone. Tip of nose consists of elastic
Other correctly matched options are Chlamydomonas. Ribosomes, cell wall
cartilage.
and chromosomal organisation differ in
Inner lining of salivary ducts is lined by 75. Option (d) correctly describes the E. coli and Chlamydomonas.
compound epithelium. location of some body parts in the
Moist surface of buccal cavity is lined earthworm. In earthworm, two pairs of 80. The option (b) is true about ribosomes.
by compound epithelium. testis are found in 10th and 11th They are large, non-membranous, RNA
segments, accessory glands in 17th protein complexes which are necessary
Inner surface of bronchioles is lined by for protein synthesis. In prokaryotes,
and 19th segments, four pairs of
ciliated epithelium.
spermatheca from 6th-9th segments 70 S type of ribosomes are found while
71. The function of gap junction is to and one pair of ovaries in 13th 80 S type of ribosomes are found in
facilitate communication between segment. eukaryotes.
adjoining cells. These are specialised 76. In male frogs, germinal epithelium of 81. Lipid molecules very rarely migrate from
intercellular connection between a seminiferous tubules produce sperms, one lipid monolayer to other monolayer
multitude of animal cell types. They which are transferred to kidney via vasa of lipid bimolecular layer.
directly connect the cytoplasm of two efferentia from the kidney, these enter
cells, which allow various molecules, Such a type of movement is called
into Bidder’s canal, from where the
ions and electrical impulses to directly flip-flop or transbilayer movement and
sperms are carried to the transverse
pass through a regulated gate between occurs once a month for any individual
collecting tubules, longitudinal
cells. lipid molecule. But protein can never
collecting tubule and then to
perform flip-flop movement.
72. The option (c) is correctly matched pair. urinogenital duct. The later carries the
Areolar tissue is the body’s loose sperms to seminal vesical where, they 82. Membrane bound organelles include
connective tissue and provides are stored temporarily. From here, lysosomes, endoplasmic reticulum,
flexibility and cushioning. sperms are carried to cloaca and then Golgi apparatus, mitochondria,
these shed into water. chloroplasts, vacuoles and nucleus.

MODULE 3
www.jeebooks.in
514 NEET Test Drive

Non-membrane bound organelles 94.


include ribosomes, centrioles and 11. Biomolecules O
Uracil
microtubules. 88. About 98% of the mass of every living HOCH2
83. Statement (a) is incorrect regarding organism including bacterium and
mitochondria as enzymes of ETC are human beings is composed of just six
present in inner mitochondrial elements, i.e. Carbon (C), Hydrogen
membrane. (H), Nitrogen (N), Oxygen (O),
Outer membrane contains enzymes Phosphorus (P) and Sulphur (S). OH OH
Nucleoside Uridine
involved in mitochondrial lipid synthesis 89. Statement (c) is wrong as, glycine is the
and those enzymes that convert lipid simplest amino acid in which functional Nucleoside is made up of ribose sugar
substrates into other forms that are group ‘R’ is replaced by hydrogen atom and nitrogenous base only. Uracil forms
subsequently metabolised in the matrix. (H). Cysteine and methionine are nucleoside with only ribose sugar. So,
The outer membrane resembles a sieve sulphur containing amino acids. the option with category nucleoside
that is permeable to all molecules of 90. The chitinous exoskeleton of arthropods component uracil is correct.
10,000 daltons mole weight or less, is formed by the polymerisation of 95. Enzymes, vitamins and hormones are
including small proteins. N-acetyl glucosamine, which is classified into a single category of
The inner membrane is impermeable a derivative of glucose. It is also a biological chemical because all these
and highly convoluted, forming a series characteristic component of the cell help in regulation of metabolism.
of infoldings, known as cristae in the wall of fungi, the radulae of molluscs Enzyme is a proteinaceous catalyst
matrix space. and the beaks and internal shells of produced by a cell and responsible for
84. The two subunits of ribosomes remain cephalopods, which include squid and the high rate and specificity of one or
united at a critical ion level of octopuses. more intercellular or intracellular
magnesium. It is constituent of 91. Macromolecule chitin is a complex biochemical reactions.
chlorophyll, middle lamella and polysaccharide containing amino Vitamin is an organic substance which
connected with phosphate transfer in sugars and chemically modified sugars is generally synthesised by plants
respiration. It is concerned with (e.g. glucosamine, N-acetyl (exception vitamin-D). The absence of a
binding of ribosomes, DNA and RNA vitamin from the diet for sufficient time
galactosamine, etc).
synthesis. gives symptoms of a resulting
Polysaccharides are long carbohydrate
85. 23S rRNA in bacteria is the enzyme molecules of monosaccharide units deficiency disease.
ribozyme for the formation of peptide Hormones are chemical messengers
joined together by glycosidic bonds.
bond. 23S r RNA is found in large which on secretion bring about a
They have a general formula Cx (H2O )y .
subunit (70S) of ribosome of bacteria. specific and adaptive physiological
Chitin is the main component of the cell
86. Flagella of prokaryotic and eukaryotic wall of fungi, the exoskeleton of response.
species differ in microtubular arthropods, insects and radulae of 96. Statement (b) is correct regarding the
organisation and type of movement. In molluscs, etc. enzyme inhibition. A competitive
eukaryotes, the arrangement is (9 + 2) inhibitor competes with substrate
and specialised while in prokaryotes, the
92. Lecithin is a phospholipid composed of
choline and inositol. It is found in all molecule for occupying the active site
arrangement is (9 + 0) and is simple. of an enzyme. These inhibitors have
living cells as a major component of cell
87. The option (d) is true for nucleolus. It is membrane. structural resemblance with substrate
a site for active ribosomal RNA (r RNA) molecules due to which they easily
synthesis. It is a naked, round or slightly 93. Phosphoglyceride is always made up of
bind with active site of an enzyme and
irregular structure in nucleus. It lacks a two fatty acids, phosphoric acid and a
form an enzyme-inhibitor complex.
membrane and its contents are in direct trifunctional alcohol glycerol. The fatty
acids are attached to the glycerol at the E + I ¾® EI
contact with the nucleoplasm. (Enzyme) (Inhibitor) (Complex)
Microtubules take part in the spindle 1 and 2 position on glycerol through
ester bonds. A fat is formed of two In non-competitive inhibition, there is no
formation. Mitochondria, vacuoles and competition between the molecule for
plastids, etc., are membrane-bound kinds of smaller molecules, i.e. glycerol
and fatty acids. In making a fat-free the active site of enzyme. The effect of
structures. The dividing cells possess a inhibitor cannot be reversed by
large number of mitochondria. fatty acid, molecules join to glycerol by
an ester linkage. A fatty acid has a long increasing the concentration of
Perinucleolar substrate, thus the inhibition is
chromatin carbon skeleton, usually 16 or 18
Intranucleolar carbon atoms in length. If there are no irreversible.
chromatin double bonds between carbon atoms 97. The graph shows the exothermic
Matrix composing the chain, then as many reaction A in the presence of enzyme
(Pars amorpha)
hydrogen atoms as possible are as it lowers down the activation energy
Granular portion substantially.
(Ribosomal bonded to the carbon skeleton.This is
precursor) called saturated fatty acid. Unsaturated The B graph shows this reaction in the
Fibrillar portion fatty acid has one or more double absence of enzyme when activation
(RNA fibrils) bonds. energy is quite high. Thus, option (b) is
correct.
Structure of nucleolus

MODULE 3
www.jeebooks.in
The NEET Edge ~ Biology 515

98. Option (b) is not correct with respect to i.e. nuclear envelope reforms, Golgi to these forces only, transpiration driven
enzyme action because addition of a lot complex reforms, etc. In late anaphase, ascent of xylem sap occurs in tall trees
of succinate reverses the inhibition of centromeres split and chromatids of even 100 m height.
succinic dehydrogenase by malonate. separate and chromatids move to 109. The phloem sap is alkaline in nature.
Inhibition of succinic dehydrogenase by opposite poles. Cytokinesis is the It continuously pumps protons from its
malonate is an example of competitive process in which cell itself is divided companion cells to the other
inhibition. into two daughter cells. neighbouring cells during transport of
Competitive inhibition occurs when 103. Various phases of meiosis and their sugars.
enzyme and inhibitor both have more or characteristic features are 110. According to mass flow hypothesis, the
less similar structure and are present in Pachytene — Crossing over takes transport of organic solutes takes place
higher concentration. place from source to sink. This transport also
Thus, both enzyme and inhibitor Metaphase-I — Chromosomes align at depends on metabolic energy.
compete for the active site of enzyme equatorial plate According to cytoplasmic streaming
resulting in the decrease of the Diakinesis — Terminalisation of hypothesis (which was given by de
enzymatic activity. chiasmata Vries, 1885), the transport of organic
Zygotene — Pairing of homologous solutes takes place by the combination
12. Cell Cycle and Cell Division chromosomes of diffusion and cytoplasmic streaming.
104. Stalled forks activate checkpoint Cytoplasmic streaming carries organic
99. During the S or synthetic phase, the
signaling and pause replication. Since, solutes from one end to the other end of
DNA content doubles, i.e. from 2C to
G1/S checkpoint checks DNA damage, sieve tube.
4C for all diploid cells.
cells size prior to S-phase (i.e. DNA P-proteins has a role as defence
The G 2 -phase follows the S-phase and
replication phase), this checkpoint against phloem feeding insects and
is called second growth phase or would be activated by stalled DNA sealing of damaged sieve tubes.
pre-mitotic gap phase. In G 2 -phase, the replication fork.
synthesis of DNA stops therefore, the Root pressure is a positive hydrostatic
DNA level remains 4C if initial was 2C. pressure developed in xylem vessels
13. Transport in Plants due to the metabolic activities of roots.
However, the formation of RNA and
105. The difference between the diffusion Transpiration pull is a force developed
protein continues as they are required
pressure of a solution and that of its due to transpiration.
for the multiplication of cell organelles,
spindle formation and cell growth. This pure solvent at a particular temperature
amount becomes half (i.e.) 2C only and pressure is called diffusion 14. Mineral Nutrition
during anaphase (in mitosis) when pressure deficit.
111. None of the option is correct with
chromosomes separate. The water moves from lower DPD to respect to question. The option (c)
higher DPD, i.e. from cell A to cell B. seems to be more appropriate.
100. In the given figure, D is representing the
This is because when a cell is placed in None of the options consists of all three
S-phase (synthetic phase) of cell cycle.
pure water, the water enters into the cell macronutrients, but option (c) have
Cell cycle completes in two steps, i.e.
as a result of the Diffusion Pressure
interphase and M-phase. Interphase is nitrogen and phosphorus which are
Deficit (DPD) of cell sap.
completed in three successive stages macronutrients, but nickel is a
G 1-phase (post-mitotic phase), S-phase DPD = OP - TP micronutrients. Mo, Zn and B are
(synthetic phase) and G 2 -phase 106. The radial orientation of cellulose micronutrients.
(pre-mitotic or post-synthetic phase). microfibrils facilitates the opening of
stomatal aperture. These are tiny pore 112. Micronutrients are minerals obtained
101. At telophase stage of M-phase of cell
complexes. Each stoma is surrounded from the soil and present in plant
cycle, nuclear membrane vesicles
by two specialised green epidermal tissues at concentrations usually less
associate with the surface of individual
cells called guard cells. Cellulose than 3 mmol g –1 dry matter. Cu (copper),
chromosomes and fuse to reform the
microfibrils are oriented radially rather Mn (manganese) and Fe (iron) are
nuclear membranes, which partially
than longitudinally, making it easier for those micronutrients which affect both
enclose clusters of chromosomes
the stoma to open, when turgor photosynthesis and mitochondrial
before coalescing to reform the
pressure inside the cell increases. electron transport. It is because they
complete nuclear envelope. During this
are the main constituents of various
process, the nuclear pores reassemble 107. Diffusion of water vapour and CO 2 are electron transport carrier proteins.
and the dephosphorylated reassociate independent processes. Their diffusion
to form the nuclear lamina. One of the depends on the difference in their 113. Elements like calcium are a part of
lamina proteins (lamina-B) remains with partial pressure in the atmosphere as structural component of the cell (mainly
the nuclear membrane fragments well as inside the cells of leaves. cell wall) and hence, are not released.
throughout mitosis and may help The deficiency symptoms tend to
108. A column of water within xylem vessels appear first in the young tissues
nuclear reassembly. After the nucleus
of tall trees does not break under its whenever the elements are not
reforms, the chromosomes decondense
weight because of high tensile strength demobilised.
and RNA synthesis resumes, causing
of water, i.e. an ability to resist a pulling
the nucleolus to reappear. 114. None of the options is correct. Option
force. This high tensile property
102. Telophase is the reverse of prophase. depends on cohesion, adhesion and (c) is correctly matched. If statement be
The chromosomes that have reached surface tension property of water. Due read as ‘potassium readily mobilisation’
their respective poles decondense, instead of ‘potassium readily

MODULE 3
www.jeebooks.in
516 NEET Test Drive

immobilisation’. Active transport of 120. In photosystem-I, the primary electron C3 -plants for the synthesis of one
nutrients requires ATP. Symplast acceptor is probably a Fe-S protein. molecule of hexose sugar.
includes translocation through The reduced primary acceptor transfers 126. The photosynthesis in C4 -plants is
plasmodesmata. Bakane disease of rice the electrons to secondary electron
relatively less limited by atmospheric
was found by Hori (1918) to be caused acceptor (most probably P430 ). The
CO 2 levels because, the primary
due to Gibberella fujikuroi. sequence of electron transfer is as
fixation of CO 2 is mediated via PEP
115. Glomus helps in the absorption of follows
carboxylase enzyme. The fixation of
phosphorus from soil by plants. It is a The reduced P430 passes its electrons to CO 2 in C4 -plants takes place in two
genus of Arbuscular Mycorrhizal (AM) ferredoxin (Fd) present at outer surface places and by two different organic
fungi, which forms symbiotic of thylakoid membrane. compounds. Phosphoenol Pyruvate
relationship with plant roots. It is a 121. Anoxygenic photosynthesis (in which (PEP) is found in mesophyll cells which
longest genus of AM fungi. O 2 is not released) is seen in primarily fixes atmospheric CO 2 into
116. Nitrogen-fixation in root nodules of Rhodospirillum which is a purple oxaloacetic acid (4C).
Alnus is brought about by Frankia. non-sulphur bacterium. It helps an RuBisCO is present in bundle sheath
Nitrogen is the most critical element. organism to trap light energy and stores it cells where final fixation of CO 2 in
Certain non-leguminous plants also as chemical energy. hexose sugars takes place. CO 2 is
form nodules to fix nitrogen. The best primarily fixed by PEP carboxylase
Other than this, anoxygenic
known example in temperate region is because this enzyme has greater
photosynthesis commonly occurs in
alder (Alnus sp). The bacterium affinity to CO 2 than RuBisCO.
purple non-sulphur bacteria, green
involved in nodule formation is Frankia
sulphur/non-sulphur bacteria and 127. With reference to factors affecting the
(Actinomycetes).
heliobacteria, etc. Spirogyra, rate of photosynthesis, statement (c) is
Clostridium is anaerobic, saprotrophic, Chlamydomonas and Ulva are algae not correct. In C4 -plants, the initial
free-living nitrogen-fixing bacterium. which perform oxygenic photosynthesis. fixation of CO 2 occurs in mesophyll
Bradyrhizobium sp are symbiont in 122. The PGA as the first product of cells. The primary acceptor of CO 2 is
plants of Paraspania and soybean. photosynthesis was discovered in alga Phosphoenol Pyruvate (PEP). It
The Azorhizobium forms both stem and named Chlorella by Melvin Calvin. He combines with CO 2 in the presence of
2

root nodules in Sesbania (aquatic made use of radioactive 14 C in enzyme PEP carboxylase to form the
plant). photosynthesis studies which had led to first stable product, i.e. Oxalo Acetic
the discovery that the first CO 2 fixation Acid (OAA), whereas C3 -plants lack
117. The Casuarina tree has nitrogen-fixing PEP carboxylase enzyme. They possess
product was a 3-carbon organic acid.
root nodules. It is a flowering plant. RuBisCO enzyme. This enzyme can
The first product identified was
Cicer arietinum (gram) is a leguminous work as both carboxylase (fixation of
3-Phosphoglyceric Acid (PGA).
plant. Cycas revoluta is a gymnosperm. CO 2 ) and oxygenase (fixation of O 2 ).
Crotolaria juncea is leguminous fibre 123. In the presence of manganese, calcium
RuBisCO has a much greater affinity
yielding plant that harbors a filamentous and chloride ions, photolysis of water
for CO 2 than for O 2 and the binding is
actinomycete nitrogen-fixing organism during photosynthesis evolves nascent
competitive. At higher temperature, its
called Frankia. oxygen as a byproduct. The hydrogen
affinity for CO 2 decreases and it works
atoms of water molecules are used up
as oxygenase.
15. Photosynthesis in Higher for reducing carbon of CO 2 to produce
Therefore, at higher temperature,
glucose molecule (C6H12O 6 ).
Plants photosynthesis decreases in C3 -plants,
124. The first reaction of photorespiration while in C4 -plants it increases.
118. Plants adapted to low light intensity occurs in the stroma of chloroplast. In
have larger photosynthetic size than
this reaction, the RuBP (Ribulose 1-5
sun plants. These plants have lower
bisphosphate) consumes one oxygen
16. Respiration in Plants
photosynthetic rates and hence, lower 128. Lactate fermentation does not release
molecule in the presence of enzyme
growth rates. any molecule of CO 2 .
RuBisCO.
These plants have larger leaves to use
In peroxisome, the glycolate transferred Glucose 2 lactate
sunlight more efficiently. The leaves are
from chloroplast takes up O 2 and forms
thinner to catch more sunlight. P+ + ADP + 2P NAD
the glyoxylate whereas, the H2O 2
Glycolysis
119. The enhancement effect has led to the released as byproduct. ATP + P P P 2NADH+
discovery of two photosystems, i.e. PS-I +2H+
125. In C4 -plants, every CO 2 molecule has 2 pyruvate
and PS-II operating in photosynthesis.
Emerson performed photosynthetic to be fixed twice, so these plants
needed more energy for the synthesis Lactic acid fermentation is process by
experiment on Chlorella. He provided which glucose, fructose and sucrose
monochromatic light of more than of hexose sugar molecules than
C3 -plants in which CO 2 has to be fixed are converted into energy and the
680 nm and observed decrease in the metabolite lactate. It is an anaerobic
rate of photosynthesis known as red only once. 18 ATP molecules are
required by C3 -for the synthesis of one fermentation reaction that occurs in
drop.Later, he provided synchronised some bacteria and animal cells and
light of 680 nm and 700 nm and molecule of hexose sugar while 30 ATP
molecules are needed by the C4 -plants allows glycolysis to continue by
observed increase in the rate of ensuring that NADH is returned to its
photosynthesis. for the same. Thus, C4 -plants have a
need of 12 ATP molecules extra than oxidised state (NAD+ ) .

MODULE 3
www.jeebooks.in
The NEET Edge ~ Biology 517

129. One mole of ATP liberates 12 kcal of Arrow 9 and 10—O 2 and ADP (either long-short day plants (e.g. Bryophyllum)
energy, so 686 kcal will be liberated by one can be 9 or 10) intermediate plants (e.g. wild kidney
686 / 12 = 57.1 ATP molecules. Arrow 11 and 12 — H 2 O and ATP bean) and day-neutral plants
(either one (e.g. cotton).
130. The chemiosmotic coupling hypothesis
of oxidative phosphorylation proposes can be 11 or 12) 140. Phytochrome is a chromoprotein which
that ATP is formed because a proton 135. Statement given in the option (d) is exists in two forms, Pr (Phytochrome red)
gradient forms across the inner incorrect. Oxidative phosphorylation is and Pfr (Phytochrome far- red). These
membrane. The production of ATP with the process of ATP formation due to the are inter-convertible forms. When
the help of energy liberated during transfer of electrons from NADH or plants get red light this protein gets
oxidation of reduced coenzymes and FADH2 to oxygen molecule (O 2 ) by a converted into Pfr from Pr and
terminal oxidation is called oxidative series of electron carriers. This process vice-versa. It controls the
phosphorylation. Peter Mitchell (1961) occurs in the inner mitochondrial photoperiodism in all flowering plants.
gave a hypothesis known as membrane because of its less 141. Anthesis is the opening of floral buds.
chemiosmotic hypothesis for ATP permeability, the presence of ETC Reception of pollen by stigma is called
synthesis. According to this, when proteins and ATP synthase. pollination. Formation of pollen is called
electrons flow from dual proton-electron The rest three statements are correct. microsporogenesis.
carrier to a non-hydrogen carrier, the
142. The condition shows that the plant
H+ are released and expelled into
the intermembrane space and thus,
17. Plant Growth and requires photoperiod shorter than the
create a proton gradient with higher Development critical day length. This plant needs
uninterrupted dark period for flowering.
concentration of H+ in the inter- 136. This experiment is significant for laying Therefore, it is a short-day plant and
membranous space than the matrix. the basis of quantitative determination these do not flower if the dark period is
Due to the proton motive force, the of growth promoting substances interrupted with flashes of light.
protons flow back and energy liberated (auxin). Dr. F Went isolated auxin from
during this backflow of protons Avena coleoptile tip. His experiment 143. An enzyme that can stimulate
activates ATPase present in F1 head to demonstrated the polar movement of germination of barlay seeds is
synthesise ATP. auxins, i.e. it showed that the plants a-amylase. Barley seeds are rich in
grow towards light in response to a carbohydrate (starch). The starch is
131. NAD+ functions as an electron carrier in hydrolysed by a-amylase to
cellular respiration. NAD is an oxidising signal generated in the tip of coleoptile
by a plant hormone auxin. monosaccharide units at the time of the
agent which accepts electrons and then germination of seeds.
transfers them to the Electron Transport 137. During seed germination, its stored
System (ETS). As a result, 3ATP food is mobilised by gibberellin Lipase enzyme causes hydrolysis of
molecules are formed, by oxidation of hormone. It induces aleurone cells to fats. Protease enzyme is responsible for
one NAD+ molecule. secrete enzyme that breaks stored food breakdown of proteins. Invertase
in seed. enzyme is responsible for hydrolysis of
132. Succinate dehydrogenase enzyme is sucrose.
present on inner membrane of Cytokinins promote nutrient mobilisation
mitochondria and catalyses the which helps in the delay of leaf
oxidation of succinate to fumarate in senescence. ABA plays an important
18. Digestion and Absorption
eukoryotes. This reaction occurs in role in seed development, maturation 144. The Paneth cells of Crypts of
cytosol in prokaryotic cells. and dormancy. Ethylene induces fruit Lieberkuhn secrete antibacterial
133. Statement given in option (d) is wrong ripening and breaks seed dormancy. lysozyme. These are present in
which can be corrected as 138. Abscisic acid is a derivative of between the bases of villi of small
carotenoid. It is a terpenoid. intestine.
Krebs’ cycle starts with the condensation
of acetyl group with oxaloacetic acid and Indole butyric acid and indole-3-acetic
water to yield citric acid. acid are auxins which are weak organic Villi
acids. Lacteal
During conversion of succinic acid to
fumaric acid, FAD + is reduced to Gibberellic acid (gibberellin) is a
FADH2 . terpene. Capillaries
134. Pathway A is glycolysis, pathway B is 139. The importance of day length in
the Krebs’ cycle and pathway C is flowering of plants was first shown in Artery
oxidative phosphorylation tobacco. It is called photoperiodism Crypts of Lieberkuhn
Vein
Arrow 1 — ADP or NAD + discovered by Garner and Allard (1920,
1922). They observed that maryland A section of small intestinal mucosa showing
Arrow 2 — Pyruvate villi and the Crypts of Lieberkuhn
mammoth variety of tobacco could be
Arrow 3 — NADH made to flower only by reducing the
Kupffer cells are phagocytic cells of liver.
Arrow 4 — ATP light hours with artificial darkening.
Arrow 5 — ADP, NAD + or FAD Zymogen cells produce inactive
On the basis of photoperiodic response
precursors of enzyme.
Arrow 6 and 7— FADH 2 and NADH to flowering, plants have been divided
into short day plants (tobacco), long Argentaffin cells produce hormones.
(either one can be 6
or 7) day plants (e.g. wheat, hanbane), 145. Parietal cells indirectly help in
Arrow 8 — ATP or CO 2 short-long day plants (e.g. Campanula), erythropoiesis. These cells also called

MODULE 3
www.jeebooks.in
518 NEET Test Drive

oxyntic cells secrete hydrochloric acid 150. Statement in option (c) is true regarding because there is a negative intrapleural
and castle intrinsic factor. HCl converts digestion and absorption of food in pressure pulling at the lung walls.
iron (in diet) from ferric to ferrous form humans. Chylomicrons are lipoprotein Alveoli are basic functional unit of
which can be easily absorbed and used particles synthesised by intestinal lungs. The outer alveolar wall surface
during erythropoiesis (formation of epithelial cells and consisting mainly of has cells which secrete DPPC also
RBCs). triglycerides. Chylomicrons are the called as lipid surfactant.
Castle intrinsic factor helps in form, in which dietary fat is transported The surfactant expands the alveoli due
absorbing vitamin-B12 and its in the circulatory system. to which the negative pressure inside
deficiency causes pernicious anaemia. Oxyntic cells secrete HCl and CIF the alveoli increases. This prevents the
146. If goblet cells become non-functional, (Castle’s Intrinsic Factor). Sodium alveoli from collapsing. Since, in this
this will adversely affect smooth pumps of cell membrane play a question, both options (a) and (b) are
movement of food down the intestine supportive role in absorption of correct and option (b) provides more
due to the absence of mucin. These fructose. Absorption of amino acids appropriate explanation, therefore, it
cells are something like a wine glass also involves active transport coupled must be chosen.
that are present in the columnar with active sodium transport. Fructose 156. Reduction in the pH of blood, i.e.
epithelium of the mammalian intestine is absorbed by facilitated diffusion. increase in acidity favours the
and secrete mucin, a mucoprotein that 151. The richest sources of vitamin B12 are dissociation of oxyhaemoglobin thereby
forms mucus when in solution. goat’s liver and Spirulina. Vitamin-B12 giving up more O 2 . When this
147. The initial step in digestion of milk in (cyanocobalamin) is the only vitamin phenomenon occurs due to increase in
humans is carried out by pepsin. In which is not found in vegetables. It is CO 2 concentration, then it is called
calves, it is rennin. It is also present in present in animal protein such as meat, Bohr effect.
small amounts in human infants but not liver, fish and Spirulina (single cell 157. Option (c) is correct matching of two
adults. Pepsin acts on water soluble protein). It promotes DNA synthesis, capacities and its volumes.
‘caseinogen (milk protein) to form maturation of RBCs and myelin
Inspiratory Capacity (IC) is the maximum
soluble ‘casein’. This combines with formation.
amount of air that can be inspired after a
calcium salts to form insoluble calcium 152. The deficiency of vitamin-B1 or thiamine normal expiration, IC = TV + IRV . It is
paracaseinate, which gets readily causes the disease beri-beri. This 3500 mL in adult male and 2400 mL in
digested enzymatically. disease occurs in those countries adult female.
148. Option (b) is the correct matching of the where the staple diet is polished rice.
Residual Volume (RV) is the amount of
site of action on the given substrate, the The symptoms of this disease are pain air remaining in the lungs after a forced
enzyme acting on it and the end from neuritis, paralysis, muscle wasting, exhalation. Its average value is 1200 mL
product. In small intestine, food meets progressive oedema, mental and 1100 mL in adult male and female,
with the pancreatic juice containing deterioration and finally heart failure. respectively.
a-amylase, which converts starch 153. Xerophthalmia is caused due to the 158. Vital capacity of our lungs is total lung
into maltose, isomaltose and a-dextrins deficiency of vitamin-A (retinol). capacity minus residual volume. It is
in small intestine.
Retinol and calciferol are fat soluble about 4800 mL.
Pancreatic a -amylase vitamins but pellagra is not the
Starch ¾¾¾¾¾¾¾® Maltose Total lung capacity is the sum of vital
deficiency disease of calciferol. capacity and residual volume, i.e. vital
(disaccharide)
Pellagra is caused due to the deficiency capacity of our lungs is total lung
The pancreatic juice also contains of Vitamin-B 3 . capacity minus residual volume.
proenzymes trypsinogen,
Tidal volume is the amount of air which
chymotrypsinogen and
procarboxypeptidase. The trypsinogen
19. Breathing and Exchange of normally passes into and out of the lungs
is converted to active trypsin in intestine Gases during each cycle of quite breathing. It is
about 800 mL in adult person.
by enterokinase of intestinal juice. The 154. Option (c) gives correct identification
trypsin converts proteins into large Inspiratory reserve volume is the extra
and functions of lungs.
peptides and the large peptides are volume of air that can be inhaled into
C – Alveoli are thin-walled vascular lungs during deepest possible
converted to dipeptide and amino acids bag-like structures for the exchange inspiration.
by carboxypeptidase. of gases.
159. The presence of high quantity of
149. Amino acids, monosaccharides like A – Trachea or windpipe is an air
+ carboxyhaemoglobin in the blood
glucose, electrolytes like Na are conducting tube through which
transport of gases takes place. It has reveals that person is exposed to
absorbed into the blood by active
incomplete rings of hyaline cartilage. inhalation of polluted air having high
transport. Fructose and some amino
B – Pleural membrane is double-layered CO. Carbon monoxide forms a stable
acids are absorbed with the help of the
compound with haemoglobin called
carrier ions like Na + by facilitated which reduces friction on the lung
surface. carboxyhaemoglobin as affinity of Hb
transport.
D – Diaphragm is involved in the for CO is 210 times greater than its
Fatty acid and glycerol cannot be affinity for O 2 . In this form, haemoglobin
inspiration and expiration process of
absorbed into the blood. They are first does not carry oxygen resulting in
breathing.
incorporated into small droplets called death due to hypoxia.
micelles, which move into the intestinal 155. Lungs do not collapse between breaths
and some air always remains in the Hb + CO a HbCO
mucosa.
lungs which can be never expelled (Haemoglobin) (Carboxyhaemoglobin )

MODULE 3
www.jeebooks.in
The NEET Edge ~ Biology 519

165. The correct labelling of parts with their 169. Statements III and IV about kangaroo
20. Body Fluids and respective functions is as follows rat are true. Kangaroo rat is a desert
Circulation rodent. Its body is covered by hairs. Its
Pulmonary vein – Takes oxygenated blood urine is more than 20 times
160. Whale has two separate circulatory from lung and carries it to
pathways. The circulatory system in left auricle concentrated as its plasma. This
which two distinct and separate concentrated waste enables it to live in
Dorsal aorta – Takes blood from heart to
circulatory pathways for blood flow are dry or desert environment where little
body parts, pO 2 = 95 mm
involved, is called double circulatory water is available for him to drink.
Hg
system (also, double-loop circulatory Most of its water is metabolically
Vena cava – Takes blood from body
system). It occurs in mammals and produced from the oxidation of
parts to right auricle
birds. Whale is a mammal, so it shows carbohydrates, fats and proteins in the
pCO 2 = 45 mm Hg
above characteristic. seeds that it eats. The animal remains in
Pulmonary – Takes blood from heart to cool burrow during daytime and the
161. Landsteiner divided human population artery lungs, pO 2 = 90 mm Hg respiratory moisture condensed in nasal
into four groups based on the
passages.
presence of antigens found in their 166. If chordae tendinae of the tricuspid
RBCs. Each group represented a valve became partially non-functional 170. A–Adrenal gland is correctly mentioned
blood group. Thus, there are four types due to the injury, then the flow of blood with its function. It is located at the
of blood groups A, B, AB and O. Blood into the pulmonary artery will be anterior part of kidney and secretes
group ‘O’ does not contain any antigen reduced because chordae tendinae catecholamines, which stimulate
on RBCs, hence can be given to any arise from papillary muscles and insert glycogen breakdown.
person, that’s why this blood group is upon the flaps of tricuspid and bicuspid
valves. The valves in the heart allow the 171. Option (d) correctly explains function of
called universal donor. human nephron. Podocytes or visceral
flow of blood only in one direction, i.e.
162. Fibrinogen is a soluble plasma protein from the atria to the ventricles and from epithelial cells are the cells in
that is stimulated by thrombin and gets the ventricles to the pulmonary artery or Bowman’s capsule in the kidneys that
converted into insoluble form fibrin at aorta. wrap around the capillaries of
the site of injury. The latter helps in the 167. Deficiency of antibodies can be glomerulus. They create minute pores
formation of blood clot to seal the confirmed by serum globulins as (slit pores) for the filtration of blood into
wound and stop bleeding and further antibodies are also called the Bowman’s capsule.
loss of blood from the injured part of immunoglobulins and constitute the 172. Statement in option (c) regarding
body. gamma globulin part of blood proteins. kidney is correct. The plasma fluid that
Globulins are simple proteins that form These are secreted by activated B-cells filters out from glomerular capillaries
a large fraction of blood serum proteins or plasma cells. into Bowman’s capsule of nephron is
involved in defence mechanism. There called glomerular filtrate. A comparison
are four main types of globulins that are 21. Excretory Products and of the volume of the filtrate formed per
manufactured in liver, namely alpha-1, day (180 L /day) with that of the urine
alpha-2, beta and gamma. Their Elimination released (1.5L), suggests that nearly
Albumin is a plasma protein that is 168. In annelids like Nereis, earthworm, 99% of the filtrate has to be reabsorbed
manufactured by the liver. It helps in leech, etc., the tubular coiled structures by the renal tubules in a process called
maintaining osmotic pressure which called nephridia are excretory organs. reabsorption.
prevents the fluid-leakage out into the In phylum–Arthropoda, insects 173. The main function of the Henle’s loop is
tissues from the bloodstream. centipedes, millipedes and arachnids to absorb water from the tubular lumen,
163. Test tube containing calcium possess Malpighian tubules as their thus making the urine concentrated. If
bicarbonate will not be used for the principal excretory organ. Analogous loop of Henle is absent, then the urine
purpose. Clotting of collected blood organs have almost similar appearance becomes more dilute.
can be prevented by coating the test and perform the same function, but 174. Statement given in option (a) is correct.
tubes with silicon or adding chelating develop in totally different groups and The ascending limb of loop of Henle is
agents. Citrate, oxalate, heparin and are totally different in their basic impermeable to water and permeable to
EDTA are anticoagulants. structure and developmental origin, K+, Cl - and Na+ and partially
e.g. wings of butterfly, birds, bats. permeable to urea. Due to this, sodium,
164. Blood pressure in different blood
vessels The homologous organs have potassium, calcium, magnesium and
Artery > Arteriole > Capillary > Venule common origin, perform different type chloride are reabsorbed here, making
> Vein (vena cava) of functions and have different the filtrate hypotonic.
So, blood pressure in the pulmonary appearance, e.g. thorns of 175. When someone drinks lot of water
artery is more than that in the Bougainvillea and tendrils of which is not required by body, the
pulmonary vein. Cucurbita. osmoregulation of the blood will
Vestigial organs are useless remnants, decrease. The decrease in
The pulmonary arteries have thicker
which might have been large and osmoregulation will inhibit the release of
smooth muscle and connective tissue
functional in the ancestors, e.g. ADH.
than the pulmonary veins to
accommodate the higher pressure and nictitating membrane, vermiform 176. A patient suffering from cholera is given
high rate of blood flow. appendix, etc. saline drip because Na + ions are
important in retaining water in the body.

MODULE 3
www.jeebooks.in
520 NEET Test Drive

Severe diarrhoea, vomiting, watery Oestrogen deficiency causes both the channels are present on the neural
stools are the chief symptoms of early and late forms of osteoporosis in membrane.
cholera. All these lead to dehydration. post-menopausal women.
188. During the nerve impulse when a
The toxin secreted by Vibrio cholerae Osteoporosis is thinning or weakening stimulus of adequate strength is applied
causes a continuous activation of of bones, which makes them fragile and to a polarised membrane, the
adenylate cyclase of intestinal epithelial more likely to break. Women have low permeability of the membrane to Na + is
cells. oestrogen level when they are increased at the point of stimulation. As
The resultant high concentration of transitioning through menopause. a result, the sodium ion channels permit
cAMP triggers continual secretion of 183. Osteoporosis is an age related disease, the influx of Na + by diffusion into the
Cl - , HCO -3 and water into the lumen of in which bones loose minerals and fibres intracellular fluid from extracellular fluid.
the intestine. Administration of saline from the matrix causing decreased bone 189. During the transmission of nerve
not only supports the sodium-potassium mass and higher chances of fractures
pump through which water in cell is impulse through a nerve fibre, the
with advancing age. Major causative
restored, but glucose is also symported potential on the inner side of the
factors of osteoporosis are imbalance of
along with sodium. plasma membrane has first become
hormones like calcitonin of thyroid,
parathormone of parathyroids, sex negatively charged, then positive and
22. Locomotion and hormones and deficiencies of calcium again negatively charged by
Movement and vitamins. The disease may be repolarisation.
classified as primary type 1, primary type 190. In the resting nerve fibre, the cytoplasm
177. Pair given in option (d) is correct about
2 or secondary. inside the axon has a high
body part and its muscle. Smooth
muscles are plain, non-striated, The form of osteoporosis most common concentration of K + and a low
involuntary or unstriped muscles due to in women after menopause is referred concentration of Na + in contrast to the
the absence of striations. These occur to as primary type 1 or fluid outside the axon. Thus, if diffusion
in the walls of hollow internal organs, in post-menopausal osteoporosis. occurs then through concentration
capsules of lymph glands, spleen, etc., Secondary osteoporosis may arise at gradient Na + enters the fibre.
in iris and ciliary body of eyes, skin any age and affect men and women
dermis, penis and other accessory equally. 191. When a nerve fibre is stimulated, its
genitalia, etc. membrane becomes more permeable
184. Inflammation of joints due to the to sodium ions because of opening of
178. Cervical vertebrae are seven in number, accumulation of uric acid crystals is ion gated channels for sodium ions.
constant in most of the mammals. gout. Fibrous joints are formed by the Hence, more sodium ions enter the
Whale, camel, giraffe, etc., have same flat skull bones, which fuse end-to-end axon than potassium ions leaving it.
number of cervical vertebrae. However, with the help of dense fibrous As a result, the positive and negative
long neck in camel and giraffe is due to connective tissues in the form of sutures charges on the outside and inside of
the more length of vertebrae. to form cranium. The vertebral column the membrane are reversed. The
179. In the rib cage, the true ribs are those is formed by 26 serially arranged units membrane with reversed polarity is
which are attached to the sternum in the called vertebrae. The less secretion of called depolarised.
front and vertebral column at back. progesterone causes abortion as it
basically supports pregnancy. 192. A – Receptor, B – Synaptic cleft,
These are 7 in numbers. Although, there C – Synaptic vesicles,
are total 12 ribs in the rib cage. The 8th, D – Neurotransmitter
9th and 10th ribs are called false ribs. 23. Neural Control and
193. Destruction of the anterior horn cells of
The 11th and 12th ribs are attached to Coordination the spinal cord would result in loss of
the vertebral column and keep floating
185. The statement given in option (a) is voluntary motor impulses because the
in the thoracic cavity, so are called
correct. The autonomous nervous anterior horn cells which is the front
floating ribs.
system regulates the secretion of column of grey matter in the spinal cord
180. Joint between atlas and axis is pivot glands, whereas the glands do not contains motor neurons that affect the
joint, which is an example of synovial regulate the nervous system. axial muscle activity.
joint characterised by the presence of a
186. The parasympathetic neural signals 194. In mammalian eye, the fovea is the
fluid-filled synovial cavity between the
affect the working of the heart by centre of the visual field where high
articulating surface of the two bones.
reducing heart rate and cardiac input density of cones, but no rods occur.
Knee joint (hinge joint) is a synovial joint
through the post-ganglionic fibres. At the posterior pole of the eye lateral to
characterised by the presence of
These fibres are very short and are the blind spot, there is a yellowish
fluid-filled synovial cavity between the
cholinergic in nature. pigmented spot called macula lutea
articulating surfaces of the two bones.
187. When a neuron is in resting state, i.e. with a central pit called the fovea. It is a
181. Gliding joint is present between thinned-out portion of the retina where
zygapophyses of the successive not conducting any impulse, the axonal
membrane is comparatively more only the cones are densely packed. It is
vertebrae. This joint permits sliding the point where the visual acuity
movements of two bones over each permeable to K + ions and nearly
(resolution) is the highest.
other. impermeable to Na + ions. Neurons are
excitable cells because their 195. A–Retina—Contains photoreceptors—
182. Osteoporosis is an age related disease rods and cones. The daylight vision is
of skeletal system, which may occur membranes are in a polarised state.
the function of cones and twilight vision
due to decreased level of oestrogen. Different types of selectively permeable
is related to rods.

MODULE 3
www.jeebooks.in
The NEET Edge ~ Biology 521

B–Blind spot—Photoreceptor cells are cycle in human female. FSH is also Simple goitre is caused by lower intake
not present in this part. involved in spermatogenesis in human of iodine through diet.
C–Aqueous chamber contains a thin males. Cretinism occurs due to the
watery fluid called aqueous humor. 201. Chronic hypersecretion of Growth hyposecretion of thyroxine during the
D–Sclera is the external layer of eye Hormone (GH) leads to gigantism or growth years or birth.
having dense connective tissue. acromegaly depending on the age of 206. Thyroid gland secretes thyroxine.
the individual. Thyroxine hormones are stored in the
196. The rods contain the rhodopsin (visual
purple) pigment and enable the animals If its hypersecretion occurs before the lumen of the follicle, the extracellular
to see in darkness. Therefore, present ossification of epiphyseal plates, it space.
in large number in nocturnal animals. causes exaggerated and prolonged 207. The parathormone secreted by
The cones contain the iodopsin (visual growth in long bones. It results in parathyroid gland regulates the calcium
violet) pigment and chiefly concerned gigantism. and phosphate balance between the
with distinction in colour and light vision In adults, hypersecretion of GH leads to blood and other tissues.
during daytime. accromegaly. No increase in height 208. Epinephrine and norepinephrine are
occurs because of the ossified secreted by adrenal medulla (under the
197. Abducens (abducent) nerve is a cranial
epiphyseal plate. control of sympathetic nervous system)
nerve which originates from the ventral
surface of medulla oblongata. It Epiphyseal plate is responsible for the in response to stress of any kind or
innervates the lateral rectus muscle of growth of bone. It closes after during emergency situations. These are
eyeball. It is a motor nerve and controls adolescence. also called emergency hormones. Thus,
the movements of the eyeball. Hence, if 202. The option (b) is correct matching of the they would be released when the
abducens nerve is injured in a man, source of endocrine hormone and its person enter an empty room and
movement of eyeball will be affected. function. In animals, melatonin allows suddenly finds a snake.
198. Cornea transplantation is outstandingly the regulation of circadian rhythms. 209. Adrenaline hormone is responsible for
successfull because it is non-vascular, Oxytocin is a neurohypophyseal this action, as adrenaline hormone is
i.e. no blood supply and is not linked hormone which stimulates the muscle known as 3F hormone, i.e. fright, flight
with immune system. contraction (smooth muscle) in the wall and fight.
of uterus during childbirth.
199. The photosensitive compounds 210. Cortisol and testosterone are lipid
(rhodopsin) in the human eye is Progesterone is a female hormone soluble hormones, which can directly
composed of opsin (a protein) and produced by the corpus luteum after pass through the cell membrane of the
retinal (an aldehyde of vitamin-A, i.e. ovulation in the Graafian follicles of target cell and bind with intracellular
retinol). ovary. This hormone maintains the wall receptors.
of uterus throughout the pregnancy.
It is present in the rod cells 211. Hormones are released in normal blood
(photoreceptors). Light induces ANF stimulates the secretion of Na and
circulation, but each hormone
dissociation of retinol, from opsin thus H2O by the kidneys and helps in
stimulates only a specific target organ
changing the structure of opsin. This regulating blood pressure.
to initiate a specific response. It is
creates potential differences in the 203. Melatonin and serotonin are derivatives because of the presence of specific
photoreceptors and they become of tryptophan amino acid, while receptor protein only in the specific
hyperpolarised. thyroxine and tri-iodothyronine are target cell. If these receptor molecules
However, during darkness rhodopsin is iodinated tyrosine amino acid are removed from target organs, the
resynthesised from opsin and retinine to derivatives. Tyrosine is an essential organ will not respond to hormone.
restore the dark vision and amino acid.
212. Radioactive iodine, i.e. Iodine-131 is
photoreceptors are depolarised. 204. Hypothyroidism during pregnancy
administered to patients suffering from
The correct form of other statements are causes defective development and
thyroid cancer for its detection. Iodine is
maturation of the growing foetus leading
(a) Nociceptors are sensory nerve cells integral part of thyroxine hormone.
to stunted growth. Low secretion of GH
that respond to potentially damaging results in stunted growth resulting in 213. Lack of iodine in diet results in goitre.
chemical or mechanical stimuli and pituitary dwarfism. Oxytocin is produced by
send them to brain and spinal cord. neurohypophysis, which stimulates
205. Myxoedema (Gull’s disease) occurs
(b) Meissner’s receptors are tactile uterus contraction during childbirth.
due to the deficiency of thyroxine in
receptors receiving the stimuli of Anterior pituitary secretes Growth
adults. It causes low BMR (by 30-40%),
pressure. Hormones (GH) whose oversecretion
low body temperature, tendency to
(d) Receptors always produce graded stimulates abnormal growth.
retain water in tissues, reduced heart
potentials. Testosterone is secreted by Leydig cells
rate, pulse rate, blood pressure and
of testes in males.
cardiac output, low sugar and iodine
24. Chemical Coordination level in blood, muscular weakness and 214. Posterior pituitary stores and releases
oedema (accumulation of interstitial two hormones called oxytocin and
and Integration vasopressin. Vasopressin acts mainly at
fluid that causes the facial tissues to
200. GnRH is a hypothalamic hormone. swell and look fluffy). the kidney and stimulates reabsorption
It stimulates the anterior lobe of pituitary of water and electrolytes by the distal
Decreased secretion of thyroxine
gland to secrete LH and FSH. These two tubules in the nephron and thereby
hormone from thyroid gland causes
hormones play important role in menstrual reducing the loss of water through urine
hypothyroidism.

MODULE 3
www.jeebooks.in
522 NEET Test Drive

(diuresis). Hence, it is also called as 219. Statement (c) is incorrect as in potato, 225. The statement in option (d) is incorrect.
Anti-Diuretic Hormone (ADH). banana and ginger new plantlets Pollen grains of different species are
Oestrogen is secreted by the ovarian always arise from the nodes of the incompatible, so they fail to germinate.
follicles influenced by FSH. Oxytocin is modified stem. Internodes are the area Only the pollen of the same species
produced by neurohypophysis, which between the two nodes. germinate and can form pollen tube,
stimulates uterus contraction during 220. Marginal notches in Kalanchoe and which grows and finally dispatches
childbirth. Blood calcium level is Bryophyllum possess adventitious buds male gamete to embryo sac. Rest, all
regulated by parathyroid gland. in their leaves for vegetative the statements are correct.
215. Ovary produces two steroid hormones, propagation. Once these are detached 226. 40 pollen grains will be formed after
i.e. oestrogen and progesterone. from leaves, give rise to new individual meiotic division in 10 microspore
Oestrogens produce wide ranging plant. mother cells. One microspore mother
actions such as stimulation of growth 221. The statement in option (c) is wrong. cell forms four pollen grains after
and activities of female secondary sex meiotic division. So, 10 microspore
Oomycetes include water moulds,
organs, development of growing mother cells will form 40 pollen grains.
white rusts and downy mildews. In
ovarian follicles, appearance of female
these, female gamete is larger and 227. In angiosperms, each Microspore
secondary sex characters (e.g. high
non-motile, whereas, male gamete is Mother Cell (MMC) undergoes meiosis
pitch of voice, etc.) mammary gland
smaller and motile. Isogametes are to produce four microspores which
development. Oestrogens also regulate
found in algae like Ulothrix, develop into pollen grains. Thus, to
female sexual behaviour.
Chlamydomonas, Spirogyra, etc., produce 100 pollen grains,
Alpha cells of islets of Langerhans of which are similar in structure, 25 microspore mother cells are
the endocrine pancreas secrete a function and behaviour. required.
hormone called glucagon. It is a Anisogametes are found in 228. Mutualism type of association is found
peptide hormone and plays an Chlamydomonas in which one in between entomophilous flower and
important role in maintaining the normal gamete is larger and non-motile and pollinating agent. The plant uses its
blood glucose levels. It acts mainly on the other one is motile and smaller. pollinator to ensure cross-pollination,
the liver cells (hepatocytes) and Oogamy is the fusion of non-motile while pollinator uses the plant as food.
stimulates glycogenolysis resulting in egg with motile sperm. The gametes,
an increased blood sugar differ both morphologically as well as 229. Flowers showing ornithophily have sweet
(hyperglycemia). physiologically. It occurs in scented, red coloured flowers with
Chlamydomonas, Fucus, Chara, nectaries. Ornithophily is an allogamy
The pars distalis region of pituitary,
Volvox, etc. performed by birds, such as long
commonly called anterior pituitary,
beaked small birds (sun birds, humming
secretes Growth Hormone (GH). 222. The option (b) is correct. birds), crow, parrot, bulbul, etc.
Over secretion of GH stimulates
In oogamous type of sexual Ornithophilous flowers are large and
abnormal growth of the body leading to
reproduction, the female gamete showy cup-shaped with abundant nectar
gigantism and low secretion of GH
( ovum/egg) is big, passive while male or edible part, e.g. Bombax, Agave, etc.
results in stunted growth resulting in
gametes (spermatozoids) are smaller, 230. In most of the angiosperms, the
dwarfism.
active and motile. It is found in the reduction division occurs in megaspore
members of fungi grouped in class mother cell to produce 4 cells. Out of
25. Reproduction in Oomycetes. these, 3 degenerate and one remains
Organisms which forms functional megaspore. This
216. Pistils fused together — Syncarpous 26. Sexual Reproduction in divides mitotically and forms embryo
Formation of gametes Flowering Plants sac which contains following structures.
— Gametogenesis 223. The statement in option (d) is incorrect. Linear tetrad
Hyphae of higher ascomycete of megaspores
Tapetum is the inner layer of Parietal cell
—Dikaryotic
microsporangium (anther), which Megaspore
Unisexual female flower—Pistillate provides nourishment to developing mother cell
217. Vivipary is an undesirable character for pollen grain after meiotic cell division.
annual crop plants because germinated Rest, all the statements are correct. Degenerating
megaspores
seeds cannot be stored under normal 224. The statement in option (d) is true. Functional
conditions for the next season. It is a Tapetum is the innermost layer of megaspore
condition, in which seeds germinate on microsporangium. It nourishes the (i) One egg cell with 2 synergids
plant itself. pollen grains. The inner wall of pollen forming an egg apparatus.
218. The pair in option (c) is not correctly grain is called intine. Endothecium is (ii) There are 3 antipodal cells.
matched. The plant body Sargassum is the wall around the microsporangium,
which provides protection and helps in (iii) There are two central cells which are
a diploid sporophyte. It does not
dehiscence of anther to release the seen as secondary nucleus (2n).
multiply asexually by means of binary
fission. Instead, it reproduces by pollen. Sporogenous tissue is diploid. Egg does not have filiform apparatus. It
vegetative means, i.e. fragmentation The cells of this tissue undergo is the synergids, which have special
which is the only known method of meiotic division to form microspore cellular thickenings at the micropylar tip
vegetative reproduction in the tetrads. which is called filiform apparatus. This
free-floating species of Sargassum. guides pollen tube into the synergids.

MODULE 3
www.jeebooks.in
The NEET Edge ~ Biology 523

231. In angiosperms, the endosperm is a (small tubular structures between rete transported simultaneously at
special tissue, which is formed as a testis and epididymis). ampullary-isthmic junction of Fallopian
result of triple fusion. In triple fusion, the The sperms reach to epididymis tube.
second male gamete fuses with through vasa efferentia where they are 245. If mammalian ovum fails to get
secondary nucleus (diploid nucleus temporarily stored for maturation and fertilised, the oestrogen secretion does
formed by fusion of two polar nuclei) to then transferred to seminal vesicle not decrease further, while corpus
form triploid primary endosperm through vas deferens. luteum will disintegrate. Primary follicle
nucleus. starts developing and progesterone
Thus, the correct route is
232. In wheat or maize (family–Poaceae), the secretion rapidly declines.
Rete testis ® Efferent ductules ®
scutellum is thought to be a modified 246. The sperm entry stimulates the
Epididymis ® vas deferens.
cotyledon or seed leaf. It contains secondary oocyte to undergo meiotic-II
reserve food material for germinating 238. Statement given in the option (a) is false division, which produces the ovum and
seeds. with respect to viability of mammalian second polar body.
sperm. Viability of a sperm means the
233. Pollen grain or microspore divides 247. Cleavage divisions are mitotic division,
capability of a sperm, to fertilise an egg.
mitotically forming a larger tube or in which the single-celled zygote is
Sperms are viable for 24 h to 48 h,
vegetative cell and a small generative converted into a multicellular morula.
whereas the ovum is viable for only 24 h.
cell. Since, vegetative cell gives rise to But during cleavage divisions, there is
pollen tube and generative cell divides 239. High levels of oestrogen and no growth of resultant daughter cells
to form two male gametes, thus if a progesterone give negative feedback to /blastomeres. So, the DNA content will
generative cell is destroyed by laser, a hypothalamus for the release of GnRH. increase, but there is no increase or
normal pollen tube will still form through Thus, inhibiting the gonadotropin insignificant increase in amount of
the vegetative cell which is there release. protoplasm.
undestroyed. 240. The statement in option (b) is incorrect. 248. Termination of gastrulation is indicated
234. Synergids are two, short-lived, haploid In follicular phase of menstrual cycle, by obliteration of blastocoel. The three
cells lying close to the egg in mature LH and FSH increase gradually and embryonic membranes are formed
embryo sac of flowering plant ovule. stimulate follicular development as well during this stage. These are ectoderm,
While, the endosperm is a triploid tissue as secretion of oestrogens by the mesoderm and endoderm. These layers
formed after triple fusion. Thus, if the growing follicles. give rise to different organs of
synergids have 8 chromosomes, the 241. Spermatogonia ® Spermatocyte ® developing foetus.
aleurone layer (part of endosperm) will
Spermatid ® Spermatozoa 249. Ectopic pregnancy develops when an
have just triple of that of chromosomes embryo implants somewhere else, other
in the synergids, i.e. 24 chromosomes. Spermatogonium is present on the inside
wall of seminiferous tubule, which than the uterus, such as in one of
235. 200 seeds of pea would be produced undergoes mitotic division and increases Fallopian tube. It is also known as
from 200 pollen grains and 200 embryo eccysis or tubal pregnancy.
their number. Spermatocytes are some of
sacs. 200 pollen grains will be formed 250. hCG (human Chorionic Gonadotropin)
the spermatogonia, which periodically
by 50 microspore mother cells, while secretion occurs about 48-72 hours after
undergo meiosis. The secondary
200 embryo sacs will be formed by 200 implantation. Its level increases and
spermatocytes undergo the second
megaspore mother cells. Similarly, 400 excess of hCG leaks into urine, which is
meiotic division to produce four, equal
seeds of pea would be produced from the indication of pregnancy.
haploid spermatids. The spermatids are
400 pollen grains and 400 embryo
transformed into spermatozoa (sperm). This hormone like LH stimulates the
sacs. 400 pollen grains will be formed
by 100 Microspore Mother Cells 242. In secretory phase during ovulation, the corpus luteum to secrete high levels of
(MMCs) and 400 embryo sacs fromed follicle breaks and collapse under the progesterone and some oestrogen to
by 400 Megaspore Mother Cells continuous influence of Luteinising maintain pregnancy. These steroids are
(MMCs). Thus, number of meiotic Hormone (LH). It begins to enlarge and required to maintain the development of
divisions required to produce 200/400 forms a yellowish structure, called the placenta, initiate the development of
seeds would be 250/500. corpus luteum. The corpus luteum plays mammary glands and inhibit ovulation.
an important role in the preparation of the 251. Pituitary secretes oxytocin during
236. Orthotropous or atropous is the erect endometrium for the implantation of the
ovule in which the body of ovule lies parturition. The functions of placenta
fertilised egg by secreting oestrogens are supply of oxygen and nutrients to
straight and upright over the funicle. and progesterone.
Hilum, chalaza and micropyle occur on embryo. It also secretes oestrogen,
one straight line, e.g. family
243. The statement in option (b) is incorrect. facilitates removal of carbon dioxide
Rest, all the statements are correct. and waste materials from embryo.
Polygonaceae and Piperaceae.
During normal menstruation, 252. The first movement of the foetus and
27. Human Reproduction approximately 40 mL of blood and an appearance of hair on its head are
additional 35 mL of serous fluid are lost. usually observed during fifth month of
237. The pathway given in option (a) dipicts The menstrual fluid is normally pregnancy.
correct pathway of transport of non-clotting because a fibrinolysin is
sperm. The sperms are produced in the During development of foetus in human
released along with necrotic
seminiferous tubules. The rete testis is by week 20, hair begins to grow
endometrial material.
connected to these tubules at one end including eyebrows and eyelashes,
244. Fertilisation in humans, is practically fingerprints develop. Finger nails and
and transfers sperms to vasa efferentia
feasible only if the sperm and ovum are toe nails grow. Firm hand grip. Between

MODULE 3
www.jeebooks.in
524 NEET Test Drive

16 and 20 weeks, the first movements 259. Cleft palate is a developmental


of the foetus are observed. abnormality which may occur in the 29. Principles of Inheritance
253. The correct matches are developing foetus and so it can be and Variation
Mona pubis–Female external genitalia detected by sonography, not by 265. Out of the given statements, statement
amniocentesis. (c) is incorrect because the law of
Antrum–Graafian follicle
Trophectoderm–Embryo development Amniocentesis is being misused for dominance does not occur universally.
foetal sex-determination test so, it is After Mendel several cases were
Nebenkern–Sperm
banned in India. recorded by scientists, where a clear
deviation from law of dominance was
28. Reproductive Health 260. Assisted Reproductive Technologies
seen.
(ARTs) is a general term referred to the
254. Gonorrhoea Neisseria method used to achieve pregnancy by Such a deviation may be seen in the
Syphilis Treponema artificial means or partial artificial form of incomplete dominance or
means and is primarily used in infertility blending inheritance and codominance.
Genital warts Human papilloma virus
treatment. Artificial insemination is a 266. When two genes in a dihybrid cross
AIDS HIV type of ARTs. are situated on the same chromosome,
261. In vitro fertilisation (IVF) or test tube the proportion of parental gene
255. In India, human population is heavily baby technique involves fertilising one combinations are much higher than the
weighed towards the younger age or more eggs outside the body and non-parental or recombinant type as
group as a result of high birth rate and then transferring the fertilised eggs linked genes are inherited together in
short lifespan of individuals. known as pre-embryos back into the offspring.
256. The option (c) does not define action of uterus. 267. Higher number of the parental types
contraceptive because in vasectomy, a Zygote Intra Fallopian Transfer (ZIFT) is formed when RRYY and rryy genotypes
small part of the vas deferens is an example of IVF. In this, the zygote or are hybridised, giving the condition
removed or tied up through a small early embryos up to 8 blastomeres are that R and Y genes are closely linked.
incision on the scrotum in males. transferred into the Fallopian tube. If the Law of independent assortment is not
Vasectomy blocks the gamete embryo has more than 8 blastomeres applicable when the genes of different
then it is transferred into uterus this is character occupy the same homologous
transport and does not affect
known as IUT. chromosomes, i.e. are linked gene.
spermatogenesis.
262. The full form of GIFT is Gamete Intra 268. In a dihybrid cross, the genotypic ratio
257. Option (a) with statements I and III are can be represented as follows—
Fallopian Transfer. This method is used
correct. Intrauterine devices like assuming a cross between AABB and
in females who cannot produce ova, but
copper-T are effective contraceptives aabb
can provide suitable environment for
for birth control. It suppresses sperm fertilisation and further development of AABB × aabb — Parents
motility and the fertilising capacity of the embryo in the oviducts. In such cases, AB ¯ ab — Gametes
sperm. Medical termination of ovum from the donor female is
AaBb — F1-generation
pregnancy or induced abortion is surgically removed and is then
voluntary or intentional termination of introduced into the Fallopian tube of Selfing ¯ — F2 -generation
pregnancy before full term of foetus. It such females. Such women then accept 1BB — 1 AABB
is comparatively safe up to 12 weeks sperms from her husband during 1 AA 2 Bb — 2 AABb
(the first trimester) of pregnancy. copulation.
1 bb — 1 Aabb
258. The contraceptive pills are hormones 263. The test tube baby means a baby born
1 BB — 2AaBB
when the ovum is fertilised externally
either in combination or progesterone
and thereafter implanted in the uterus. 2 Aa 2Bb — 4AaBb
only that primarily prevent release of
Patrick Stepote and Robert Edward first 1 bb — 2Aabb
egg. It is convenient and highly
time developed test tube baby 1 BB — 1 aaBB
effective, significant non-contraceptive technique in 1978. In these cases,
health benefits such as protection 1 aa 2Bb — 2aaBb
where normal fertilisation is not
against ovarian and endometrial possible, ovum from the female and the 1bb — aabb
cancers. sperm from the male are mated in vitro. So, the ratio of AABB, AABb, AaBB and
Condom is thin rubber sheath for penis The zygote, later on is implanted in the AaBb will be 1 : 2 : 2 : 4 in
that collects semen. It is easy to use, uterus where further development into F2 -generation.
baby take place. 269. Dominance—Expression of only one
effective and inexpensive.
264. The characteristic demographic allele in a heterozygous organism
Vasectomy is the cutting and tying off
features of developing countries is high Codominance—Side by side full
the ductus deferens so, that sperm
fertility, low or rapidly falling mortality expression of both alleles. F1 resembles
connot enter the ejaculate. rate, rapid population growth and a very both parents
Copper-T is small plastic device placed young age distribution. In India, it is all
due to the reproductive and child health Pleiotropy—Single gene can exhibit
in the uterus that prevents fertilisation or
care programmes operating throughout multiple phenotypic expression, e.g.
implantation. phenylketonuria
the country.

MODULE 3
www.jeebooks.in
The NEET Edge ~ Biology 525

Polygenic inheritance—Many genes female produces only one type of content of O 2 as their haemoglobin is
govern a single character, e.g. human gametes, i.e. female specific. As male malformed. The person suffering from
skin colour. produces two types of gametes in equal this disease show symptoms of anaemia.
270. Parents – TT ´ tt proportion. There is an equal opportunity 283. In the given question, 50% of offsprings
to getting a male or female offspring. would be affected by this disorder
(Tall) Tt (Dwarf)
275. Out of the given statements (b) is not true because Down’s syndrome is the result
F1-generation (Heterozygous tall)
because the tightly linked genes on of trisomy, in which chromosome pair
chromosomes show 100% parental types number 21st contains an extra copy of
K-Pollen ® On selfing
and 0% recombinants. Two genes chromosome (2A + 1). Affected mother
T t
that undergo independent assortment will produce 50% normal egg cells and
Egg TT Tt
indicated by a recombinant frequency of rest 50% eggs are of abnormal type.
T (Tall) (Tall)
F2-generation 50% are either on non-homologous 284. Daughter of an albinic father will be
t Tt tt chromosomes or located far apart in a carrier of this disease, when such
(Tall) (Dwarf) single chromosome. woman gets married to an albinic man,
Phenotypic ratio = 3 : 1 [Tall : Dwarf] As the distance between two genes 50% of her progeny will be normal and
Genotypic ratio Þ 1 : 2 : 1 increases, crossover frequency 50% will be albinic.
increases. 285. The given pedigree shows the
(Homozygous tall : Heterozygous tall : Dwarf)
276. The actual distance between two genes autosomal recessive disorder. In this
271. A cross between two individuals, one is said to be equivalent to the disorder, the individual inherits two
with AB blood group and other with A percentage of crossing over between mutated genes, one from each parent.
blood group will produce four these two genes. Since, the two genes This disorder is usually passed on by
genotypes and three phenotypes. lie at the ends of the chromosome, two carriers. Health is rarely affected,
Parents Male Female there are 100% chances of their but individuals have one mutated gene
segregation during crossing over. (recessive gene) and one normal gene
Phenotype AB A
277. According to the given question, the (dominant gene) for the condition. The
Genotype I AIB IA i sequence of genes on chromosome are carrier individuals have a 25% chance
b, a, c. of having an unaffected child with
Gametes I A, IB I A, i
8% normal genes, 50% chance of having
20%
´ an unaffected child who also is a carrier
IA IB and a 25% chance of having an
IA I I AA AB
I I b a c affected child with recessive genes.
(A) (AB) 286. When a colourblind man (X C Y) marries
28%
i IA i IB i
to a woman with normal sight (XX) who
(A) (B) 278. AABB ´ aabb is suitable for experiment has no family history of colour
Offsprings genotypes : on linkage. Linkage may be defined as blindness, all of their sons will be
A A AB A B the tendency of two genes of the same normal pure and all of their daughters
4(I I , I I , I i, I i)
chromosome to remain together in the will be carriers as shown below
Phenotypes : 3 (A, B, AB) process of inheritance.
272. The ABO blood group system has at Normal woman Colourblind man
279. Mr. Kapoor will have the genotype Bb,
least 6 genotypes. On the basis of the d, so 1/4th of the sperms will have Bd. XX XCY
presence or absence of antigens and +
280. The genotype of human male in
antibodies, four blood groups
(phenotypes) have been question must be Aa Bb X hY. XC Y
differentiated—A, B, AB and O blood Hence, 2 ´ 2 ´ 2 = 8 types of gametes +
groups. In ABO blood group system, would be formed, AB X h, ABY, aB X h.
inheritance of grouping is controlled by aBY, Ab X h, AbY, ab X h, abY X XXC XY
a single autosomal gene on
chromosome 9 with three major alleles Hence, 1/8 proportion of his sperms
A, B and O (IA , IB and IO). would be abh.
X XXC XY
273. Grasshopper is an example of XO type 281. Daughters have two X-chromosomes
of sex-determination in which the males one of them is from father and other
All carrier All normal
have only one X-chromosome besides comes from mother, in this case, all the
daughters son
the autosomes, whereas females have a daughters are suffering from the fathers
pair of X-chromosomes. disease hence, X-chromosome of father So in the next generation, the children
must be carrying a dominant trait, i.e. of all of their son will be normal in all
274. Genes which are present on sex inheritance pattern is sex-linked conditions (except the case in which
chromosomes are called sex-linked dominant. the wife involved is not carrier
genes. Male Drosophila contains XY
282. Statement given in option (c) is correct. neither colourblind). For carrier
sex chromosome, while female contains
Sickle-cell anaemia is an autosomal daughters.
XX-chromosomes.
recessive gene disorder in which (i) If they many to a normal man 50%
During gamete formation, male sickle-celled RBCs are formed instead of their grandsons will become
produces 50% male specific gametes of normal ones. They carry very less colourblind as
and 50% female specific gametes while

MODULE 3
www.jeebooks.in
526 NEET Test Drive

Carrier woman Normal man can be easily observed from the cross The criteria that a molecule must fulfil to
c XY given below) act as a genetic material are as
XX
following
+
X Xc × XY
(i) It should be able to replicate.
X Y (ii) It should be chemically and
+ X X, Xc X, XY, Xc Y structurally stable.
Normal Carrier Normal Colourblind 25% (iii) It should provide the scope for slow
X XX XY daughter daughter son son each changes, i.e. mutations which are
required for evolution.
288. The statement in option (d) is correct. In
the given pedigree chart, squares are (iv) It should be able to express itself in
XC XcX XcY
representing males and circles females. the form of ‘Mendelian characters’.
Colourblind In F1-generation, 1-male and 1-female 293. TATA box is present in eukaryotic
son are diseased and in next generation promoter region. It has a resemblance
only male is diseased. This shows the with Pribnow box of prokaryotes. TATA
(ii) If carrier daughter marries to a inheritance of a recessive sex-linked box was identified by Dr. Hogness and
colourblind man 50% of their disease like haemophilia. so, it is also called as Hogness box. It is
grandson will be colourblind along
289. In the given question, since both a 7 bp long region located 20 bp
with 50% of the grand daughter
parents carry a haemoglobinopathy trait upstream to the start point.
while rest 50% of the grand
daughters will be carriers as of thalassemia the risk is 25% for each During the process of transcription, the
pregnancy for an affected child. RNA polymerase (a holoenzyme which
Carrier woman Colourblind man
R r has a core unit and a sigma factor for
XXc XcY proper initiation of transcription) binds
R RR Rr
+ to TATA box due to which DNA
r Rr rr assumes a saddle-like structure at this
Xc Y RR–Unaffected (25%) place.
+ Rr–Carrier (50%) 294. If DNA has ATACG nucleotide
rr–Affected (25%) sequence then the mRNA would
X XXc XY contain UAUGC sequence. The
Carrier Normal So, the chances of pregnancy resulting
in an affected child is 25%. formation of mRNA from DNA is termed
as transcription. This process takes
Xc XcXc XcY 290. A brown eyed man, whose mother was
place in the nucleus (eukaryotes) or in
Colourblind Colourblind blue eyed must have the genotype Bb
the cytoplasm (prokaryotes).
where B represents brown eye colour
Daughters Son and b represents blue eye colour. When The base sequence of mRNA is
a man of such genotype marries a blue complementary copy of the template
So, in both the above cases, the result
eyed woman, the children shall be DNA strand.
shows 50% of grand sons will be
colourblind which in terms of overall Man Woman 295. 33 codons will be altered if the 901st
progenies (son + daughters) comes as (Brown eye) (Blue eye) base is deleted and RNA has only 998
25% thus confirming the probability as Bb bb Parents bases instead of 999 bases.
0.25. B b b b Gametes Total bases present in RNA = 999
287. As colour blindness is a sex-linked Bases left after deletion of 901st base in
recessive genetic disorder, for it is RNA = 999 - 901= 98
present at X-chromosome. Thus, Number of codon present in 98 = 33
according the gene to the situation Bb Bb bb bb Genotype of
progenies (Approximately as three codons code
given in the question, a man whose (Brown eyed (Blue eyed for one amino acid).
father was colourblind (will be, i.e. XY children) children)
normal) marries a woman whose mother 296. UAA is the ‘stop’ codon hence,
291. In this case, when the fathers of both polypeptide chain will not grow after
was colourblind and father was normal husband and wife were colourblind and
( i.e. this woman will be a carrier) 24th amino acid. In the absence of new
they have normal vision, husband have initiating codon, rest of codons will not
according to the cross given below normal vision, while wife is carrier of this be able to translate.
Xc Xc × (XY) disease. Daughter of such parents will
be carrier but there is no chance of her 297. Statements II and IV are true about lac
to be colourblind. operon. In prokaryotes, a hypothesis
was given in 1961 to explain the
Xc X, Xc X, Xc Y, Xc Y
protein synthesis regulation. This
50% carrier
30. Molecular Basis of hypothesis was given by F Jacob and
50% colourblind
daughter son Inheritance J Monod and for this they were
Thus, when marriage will happen 292. The statement given in option (c) is an awarded Nobel Prize in 1965, the
between a normal man and a carrier exception to the genetic molecule’s hypothesis was known by the name of
woman, in that case percentage of a characteristics. It should be stable Operon Model.
male child to be colourblind is 25% (this structurally and chemically.

MODULE 3
www.jeebooks.in
The NEET Edge ~ Biology 527

The operator gene is the segment of 303. The correct example of convergent and in origin and do not take part in
DNA, which exercise a control divergent evolution is given in option recombinations. Wilson and Sarich
overtranscriptions. In the absence of (a). Convergent evolution involves the choose mitochondrial DNA (mtDNA) for
lactose, the repressor binds with the independent development of similar the study of maternal line inheritance.
operator gene. structures in organisms that are not While, Y-chromosomes were
298. Zinc finger analysis is not required for directly related. It is represented by considered for the study of human
any of the techniques of DNA analogous organs, e.g. eyes of Octopus evolution particularly male domain.
fingerprinting available. It is a small and mammals, wings of insects and 309. The sequence of organic evolution
protein structural motif that is birds. In divergent evolution, same proposed by Darwin and Wallace is
characterised by the coordination of basic organ becomes adapted by variations, constancy of population size,
one or more Zn ions in order to stabilise specialisation to perform different overproduction and natural selection.
functions. Though living organisms tend to
the folds.
It is represented by homologous multiply geometrically, the number of
299. Statement (a) is wrongly matched. It is
organs, e.g. bones of forelimbs of individuals of a species tend to remain
because transcription is a process of
vertebrates (like seal’s flipper, bat’s constant over a long period of time. Out
mRNA synthesis from a DNA template.
wing, cat’s paw, horse’s front leg and of heterogenous population (due to the
It involves three main events,
human hand), thorns of Bougainvillea variation), best adapted individuals are
i.e. initiation (binding of RNA
and tendrils of Cucurbita. selected by nature.
polymerase to as DNA), elongation
(development of a short stretch of DNA) 304. Electron Spin Resonance (ESR) 310. According to both the views, Darwin’s
and termination (recognition of the measures number of charges theory and pangenesis something is
transcription termination sequence and occupying deep traps in crystal band passed from parent to offspring which
the release of RNA polymerase). gap. The basic principle of ESR is same causes development of specific
as those for luminescence, i.e. characters, i.e. all that has been
electrons become trapped and stored acquired by the organism during its
31. Evolution as a result of ionising radiations, e.g. lifetime is preserved by generation and
300. The statement given in option (b) is dating of tooth enamel. transmitted to offsprings in the form of
correct. The earliest organisms that pangenesis or gemmules.
305. The age of the fossil of Dryopithecus on
appeared on Earth were anaerobic
the geological time scale is 25 ´ 106 311. Plates having streptomycin allow to
chemoautotrophs. propagate only those bacteria which
years back, i.e. about 20-25 million years
Chemoautotrophs were the first ago. Dryopithecus had the combined are resistant to the antibiotic. While,
autotrophic organisms. They were characters of great apes, old world those plates in which streptomycin is
unable to perform photolysis of water monkeys and man. The main structural absent, both resistant and non-resistant
and never released oxygen, instead bacteria can grow normally.
characteristics of Dryopithecus are
they used H2 S and produced sulphur as broad jaws, large canines, semi-erect 312. The given case is an example of natural
a byproduct, e.g. sulphur bacteria. walking, 5 cusped molars and the selection. As a result of struggle for
absence of brow ridges. existence only those organisms could
301. Option (c) is correct regarding the
survive, which have favourable
sequential manner of events of origin of 306. The eyes of Octopus and eyes of cat
variations to adapt to the environmental
life is as follows are examples of analogous organs,
conditions. With so many variations in
Synthesis of organic monomers though they are different in structure but
population of species, the struggle for
similar in function. The analogous
¯ existence results in survival of the fittest.
organs are not anatomically similar
Synthesis of organic polymers In this case, the darker peppered moths
structures though they perform similar
were naturally selected.
¯ functions. Hence, analogous structures
are a result of convergent evolution, i.e. The survival of the fittest is the result of
Formation of protobionts selection and proliferation of only those
different structures evolving for the
¯ same function and hence, having organisms, which were most suitably
Formation of DNA based genetic similarity. adapted to the environment and most
systems successful in mating, i.e. natural
Homologous organs develop along
selection.
302. The process by which organisms with different directions due to the
different evolutionary history evolve adaptations to various needs. This is 313. Industrial melanism as observed in
similar phenotypic adaptations in divergent evolution and the structures peppered moth proves that the true
response to a common environmental are homologous. black melanic forms arise by a
challenge is called convergent recurring random mutation.
307. Biogenetic law was propounded by
evolution. It occurs in unrelated group Ernst Haeckel in 1866. According to Industrial melanism is a term used to
of organisms. The process of evolution this, during its early development, an describe the evolutionary process in
of different species in a given animal passes through ancestral adult which darker individuals come to
geographical area starting from a point stages. predominate over lighter individuals
and literally radiating to other areas of since the industrial revolution as a result
308. In the recent years, DNA sequences of of natural selection. In 1848, a black
geography is called adaptive radiation.
mtDNA and Y-chromosomes were
Natural selection is the basis of form of the moth was recorded in
considered for the study of human
evolution. Manchester and by 1895, 98% of the
evolution because they are uniparental
peppered moth population in

MODULE 3
www.jeebooks.in
528 NEET Test Drive

Manchester was black. This black and bind antigens and may differentiate process is called as metastasis. These
‘melanic’ forms arose by a recurring to memory cells or plasma cells tumours are generally located in
random mutation, but its phenotypic (produce antibody). T-cells cause epithelial tissue and glands, e.g. breast
appearance had a strong selective transplant rejection. cancer, skin cancer, stomach cancer,
advantage in industrial areas. 318. Both statements I and II are correct. lung cancer, pancreas cancer, etc.
314. The statement given in option (d) is The correct form of statement III is 323. The symptom given in option (d)
correct. Recapitulation theory or B-lymphocytes are not responsible for indicates radiation sickness. Even lower
biogenetic law states that ontogeny the rejection of graft. IV is the doses of radiations cause serious
(development of embryo) is acceptance or rejection of a kidney damages like skin burns, nausea, loss of
recapitulation of phylogeny (ancestral transplant does not depend on specific hairs and nails, change in blood cell
sequence) of embryonic development interferons. Interferons are antiviral count, prolonged exposure causes
of organisms. proteins. formation of tumours, cancer. High dose
319. Amoebiasis is caused by protozoan (lethal radiation exposer) may cause
32. Human Health called Entamoeba histolytica. instant death.
and Diseases Prevention of infection is entirely a 324. Statement (b) is correct. The correct
matter of hygiene, both personal as well form of other statements are
315. Common cold differs from pneumonia in
as municipal. Their prevention include (a) HIV is a virus containing ssRNA
that pneumonic pathogen infects alveoli
use of properly cooked food and and reverse transcriptase enzyme
whereas the common cold affects nose,
sterilised water. enveloped inside protein coat.
respiratory passage but not lungs.
Pneumonia is caused by the bacteria Diphtheria is caused by bacterium (c) HIV is enveloped retrovirus.
Diplococcus pneumoniae which infects named Corynebacterium diphtheriae. (d) HIV escapes the immune cells and
the alveoli of lungs. It generally spreads The symptoms are fever, sore throat, attacks helper T-cells of immune
through the sputum of patient. Fever, severe damage to heart, nerve cell and system.
cold and difficulty in breathing are adrenal glands. The DPT vaccine is
used for diphtheria, pertussis and 325. HIV infection does usually show
some common symptoms of symptoms of AIDS when HIV damages
pneumonia. It can be treated by the tetanus.
large number of helper T-lymphocytes.
antibiotics. Cholera is caused by bacterium named T-lymphocyte receptors can recognise
Common cold is caused by a variety of Vibrio cholerae, a Gram negative only antigen that bound to cell membrane
viruses, most commonly by rhinovirus bacterium. It spreads by faecal proteins. These lymphocytes mediate
(RNA virus). It spreads through droplet contamination. The dehydration and CMI (Cell-Mediated Immunity).
infection. It affects the upper respiratory loss of mineral salts can cause death. It B-lymphocytes are the major effector
tract but not the lungs. Nasal and is treated by the use of oral rehydration molecules of humoral immunity.
bronchial irritation, sneezing and therapy. Erythrocytes are red blood cells.
coughing are some common symptoms Syphilis is caused by Treponema Thrombocytes or platelets secrete factors
of cold. pallidum, a spirochaete and spread by that are involved in vascular repair.
316. Option (a) correctly match the example sexual contact and is a STD. 326. Retroviruses are implicated as a cause
of immunity and lymphocyte. 320. Cancerous cells have high telomerase of cancer in humans because they may
Phagocytosis is an important feature of activity. The maintenance of telomere carry cellular proto-oncogenes in their
cellular innate immunity, performed by stability is required for the long term genome, when these proto-oncogenes
cells called phagocytes that engulf or proliferation of tumours. This makes get converted into oncogenes due to
eat pathogens or foreign particles. telomerase a target not only for cancer some physical, chemical or biological
Common examples of these diagnosis but also for the development agents, they cause cancer.
phagocytes are monocytes, of novel anti-cancer therapeutic agents, 327. The statement given in option (a) is
macrophages, neutrophil granulocytes e.g. telomerase inhibitors are used in correct regarding psychotropic drug.
(often referred to as polymorphonuclear cancer treatment.
Charas is the dried resinous extract
leukocytes or PMN or PML, because of 321. Statement given in option (d) is correct. from the flowering tops and leaves of
the varying shapes of nucleus), tissue Malignant tumour first grows slowly. No Cannabis sativa. In some countries, it is
dendritic cells, mast cells, etc. symptoms are noticed. This stage is called hashish. It is a hallucinogen,
Anti-tetanus and anti-snake bite called the latent stage. The tumour later which alters a person’s thoughts,
injections are examples of passive grows quickly. The cancer cells go feelings and perceptions.
immunity. beyond adjacent tissue and enter the
328. Pair given in option (d) is not correctly
317. Statement (a) is correct with respect to blood and lymph. Once this happens,
matched. Leishmaniasis or kala-azar is
immunity. In artificially acquired passive they migrate to many other sites in the
caused by a protozoan, Leishmania
immunity, preformed antibody in an body where the cancer cells continue to
donovani. It is spread by sandfly. It is
immune serum of some other animal is divide. It is called as metastasis. Only
also known as dum-dum fever. Its
introduced into the body, as the malignant tumours are properly
control includes eradication of vector
antivenum used to treat snake bites. In designated as cancer.
and use of antibiotics.
this case, the body does not produce 322. Carcinoma refers to malignant tumours
any antibodies. Antibody is a protein 329. Schizophrenia is a group of severe
of skin and mucous membrane. It is a
molecule having two light chains and mental disorders that have common
malignant or metastatic tumour. It can
two heavy chains. B-cells recognise extend to the neighbouring cells, this symptoms as hallucinations, delusions,

MODULE 3
www.jeebooks.in
The NEET Edge ~ Biology 529

blunted emotions, disordered thinking. corn–Zea mays (Poaceae), rice–Oryza 343. Insect hormones, i.e. pheromones are
It can be caused by excessive sativa (Poaceae), potato–Solanum third generation pesticides.
dopamine production, alternation of tuberosum (Solanaceae) and Pheromones are the chemical
neuropeptide and decreased frontal barley–Hordeum vulgare (Poaceae). substances which when released into
lobe activities. Recovery is possible 335. Haploids are important in crop an animal’s surroundings, influence the
with regular use of chloropromazine improvement programme because they behaviour or development of other
along with psychosocial therapy. produce a pureline and form perfect individuals of the same species.
homozygous. Inorganic substances, oils, plant
33. Strategies for Enhancement extracts used as insecticides are called
336. The statement in option (a) is correct. first generation pesticides and synthetic
in Food Production During cloning of a cattle, a fertilised organic compounds as second
330. Out of the given statements, (I) and (II) egg is taken out of the mother’s womb generation pesticides.
are correct as single cell Spirulina can and in the eight cell stage, cells are
344. The major difficulties in the biological
produce large quantities of food rich in separated and cultured until small
control of insects–pest is that the
protein, minerals, vitamins, etc. And embryos are formed which are
predator develops a preference to other
body weight-wise the microorganism implanted into the womb of other cows.
diets and may itself become a pest.
Methylophilus methylotrophus may be 337. Wild plants have to survive without Biological control mainly refers to the
able to produce several times more getting any protection and for this, they introduction of living organisms which
proteins than cows per day. evolve various strategies/characters destroy other harmful organisms.
which are exploited by plant breeders
331. Out of the given statements, (II) and (III) 345. Crotalaria juncea (sunnhemp) and
like disease resistance.
are correct as organic farming is the Alhagi camelorum are among the plants
form of agriculture that relies on 338. Out of the given options, option (a) is which are used as green manures in
techniques such as crop rotation, green mismatched because pearl is obtained India. These green manures help the
manure, compost and biological pest from pearl oyster (Pinctada vulgaris) soil to become enriched through
control to maintain soil productivity and while, honey from Apis indica, lac from increase in water holding capacities
control pest on a farm. Organic farming Kenia lacca and silk from Bombyx mori. and fertility.
excludes or strictly limits the use of 339. The long term prospects for a truly 346. Out of the given statements, option (a)
manufactured chemical fertilisers, is correct as eutrophication is caused
human civilisation depend on a large
pesticides (which include herbicides, by runoff water from fertilised fields,
measure on control of human disease.
insecticides and fungicides), plant sub-urban lawns, feed lots and
growth regulators such as hormones, detergent rich sewage. It is the
food additives and genetically modified 34. Microbes in Human phenomenon of nutrient enrichment of a
organisms. Welfare water body which enhances the growth
332. Bacterial blight of chickpea is caused 340. Option (c) is wrongly matched as, of water weeds and depletes dissolved
by bacterium Xanthomonas campestris. lipase is obtained from Candida oxygen.
The stems and the leaves of infected albicans. Butyric acid is produced by
plant give blighted or burnt up fermentive activity of the bacteria called 35. Biotechnology :
appearance. Clostridium butylicum. It does not Principles and Processes
Control measures include roguing, produce lipase.
347. Genetic engineering is possible
3-year crop rotation, disease-free seeds, 341. Out of the given statements, option (d) because restriction endonuclease
spray of copper fungicides (Bordeaux is correct because activated sludge is a purified from bacteria can be used in
mixture) and antibiotics besides sowing process dealing with the treatment of vitro. It is the manipulation of genetic
disease resistant varieties. sewage and industrial waste waters. material of an organism using enzyme
333. If a pulse crop possesses premature Atmospheric air or pure oxygen is restriction endonuclease. Nathans and
yellowing of leaves and decreases in introduced to a mixture of primary Smith (1970) isolated the first restriction
yield, an application of magnesium and treated or screened sewage (or endonuclease. Jackson, Symons and
iron to promote synthesis of chlorophyll industrial waste water) combined with Paul Berg (1972) successfully
may become most beneficial to organisms to develop a biological flock, generated recombinant DNA molecules
overcome the problem and to obtain which reduces the organic content of in vitro.
maximum seed yield. the sewage. Sediment in settlement
348. The possible reason is that both strains
tanks of sewage treatment, plant is a
Magnesium is an important part of ring have different cistron because the
rich source of aerobic bacteria because
structure of chlorophyll molecule and its enzymes required for lysing E. coli could
small amounts are used as inoculum in
deficiency causes chlorosis and not be synthesised by the mutant strain.
secondary treatment or biological
premature leaf abscission. Two different strains had cistrons for
treatment stage of sewage treatment.
In iron deficiency also, the leaves synthesising different enzymes which
342. Option (b) is correct.. The correct acted together.
become chlorotic because iron is
matches are as follows
required for the synthesis of some of the 349. Gel electrophoresis is used for the
chlorophyll protein complexes in the Citric acid — Aspergillus separation of molecules of similar
chloroplast. Cyclosporin— Trichoderma electric charge on the basis of their
334. Top five crops of today are Statins — Monascus size. Hence, smaller the DNA fragment
wheat–Triticum aestivum (Poaceae), size the farther it moves.
Butyric acid— Clostridium

MODULE 3
www.jeebooks.in
530 NEET Test Drive

Agarose gel matrix functions as sieve. signal situated at their replication origin Golden rice has been bred to be
Smaller DNA fragments easily move which determines how many copies are especially disease-resistant, resulting in
and larger fragments take time to move to be made and this number can be better crop yields.
in gel matrix. artificially increased for cloning a given 361. Genetic engineering has been
350. The DNA fragments separated on an gene. successfully used for producing
agarose gel can be visualised after 355. ampR and tetR are the antibiotic transgenic mice which are being
staining with ethidium bromide. It is resistance genes. Ori represents the developed for use in testing the safety
intercalating agent and a fluorescent site of origin of replication, rop of vaccines before they are used on
agent. The stained DNA fragments are represents the proteins that take part in humans. Transgenic mice are being
seen as bright orange coloured bands the replication of plasmid. Hind III, Eco used for testing toxicity of drugs.
under UV-light. RI are the recognition sites of restriction Transgenic animals are made to carry
Intercalation is the insertion of endonucleases. genes, which make them more sensitive
molecules between the planar bases of to toxic substances than non-transgenic
356. Polymerase Chain Reaction (PCR) is
DNA. This process is used as a method animals.
used to amplify a DNA segment or to
for analysing DNA. Intercalation occurs, synthesise in vitro the multiple copies of They are then exposed to the toxic
when ligands of an appropriate size gene (or DNA) of interest, using two sets substances and the effects studied.
and chemical nature fit themselves in of primers and the enzyme DNA Toxicity testing in such animals will
between base pairs of DNA. These polymerase. This enzyme is isolated allow us to obtain results in less time.
ligands are mostly polycyclic, aromatic from a bacterium Thermus aquaticus 362. Agrobacterium tumefaciens, a
and planar and therefore often make and it remains active during the high pathogen of several dicot plants is able
good nucleic acid stains. Intensively temperature but high temperature to deliver a piece of DNA known as
studied DNA intercalator include induced denaturation of double-stranded t DNA to transform normal plant cells
ethidium bromide, proflavine, etc. DNA. into a tumour and direct these tumour
351. Plasmid is a small fragment of DNA 357. Severe Combined Immuno Deficiency cells to produce the chemicals required
(about 10 Kbp size) that is physically (SCID) caused by Adenosine by the pathogen.
separate from and can replicate freely Deaminase Deficiency (ADA) is the first 363. Commonly used vector for human
of chromosomal DNA within a cell. It genetic disorder to be treated with gene genome sequencing are BAC (Bacterial
can clone small DNA fragments. therapy. T-cell directed gene transfer Artificial Chromosome) and YAC. BAC is
Cosmid –45 Kbp was useful in the treatment of ADA-SCID, a DNA construct, based on a functional
BAC –300-350 Kbp whereas the retroviral-mediated gene fertility plasmid (F plasmid) used for
YAC –1 Mbp (1,000 Kbp-2,500 Kbp) transfer to haematopoietic stem cells transforming and cloning in bacteria (E.
was insufficient for achievement of coli) and YAC are genetically engineered
352. The colonies of recombinant bacteria
clinical benefits. chromosomes derived from the DNA of
appear white in contrast to blue
colonies of non-recombinant bacteria 358. The most common type of aerobic the yeast (Saccharomyces cerevisiae),
because of insertional inactivation of bioreactor in use today is the stirred which is then ligated into a bacterial
alpha galactosidase in recombinant tank bioreactor, which may feature a plasma.
bacteria. Alpha galactosidase is a specific internal configuration designed
glycoside hydrolase enzyme that to provide a specific circulation pattern. 37. Organisms and Population
hydrolyses the terminal alpha galactosyl The stirred tank bioreactor have been 364. The pair given in option (b) is
moieties from glycolipids and designed for availability of oxygen mismatched. Prairies contain tall
glycoprotein. It is encoded by the GLA throughout the processes. grasses and shrubs, not the epiphytes.
gene. b-galactosidase is an
Savanna — Acacia trees
exoglycosidase, which hydrolyses the 36. Biotechnology and Its
b-glycosidic bond formed between a Tundra — Permafrost
Applications
galactose and its organic moiety. Coniferous forest — Evergreen trees
359. Production of human protein in bacteria
353. PCR and RFLP are methods used for by genetic engineering is possible 365. The statement in option (a) is correct.
genetic fingerprinting. As Restriction because the genetic code is universal Keystone species is a species which
Fragment Length Polymorphism (RFLP) as a codon codes for the same amino has much greater influence on
is the basis of genetic (or DNA) acid in all the living organisms. community characteristics, relative to
fingerprinting and is useful in identifying their low abundance or biomass
individuals from their semen, blood or 360. An improved variety of transgenic removal of these cause serious
tissues or any other DNA sample and basmati rice gives high yields and is disruption in functioning of community,
resolution of parenthood disputes. rich in vitamin-A. It is named golden e.g. in tropical forests, figs are keystone
rice (transgenic basmati rice). It is a species.
Polymerase Chain Reaction (PCR) is
variety of Oryza sativa produced
also useful in genetic fingerprinting as it 366. In the given diagrammatic
through genetic engineering to
can amplify the DNA sample even if representation of response of
biosynthesise beta-carotene, a
available in a very small amount. organisms to abiotic factors
precursor of provitamin-A in the edible
354. Plasmids are suitable vectors for gene parts of rice. The research that led to (i) Regulator Some organisms are
cloning because these are small golden rice was conducted with the able to maintain homeostasis by
circular DNA molecules with their own goal of helping children who suffer from physiological (sometimes
replication origin site. These carry a vitamin-A deficiency in poor countries. behavioural also) means which

MODULE 3
www.jeebooks.in
The NEET Edge ~ Biology 531

ensures constant body 371. Gause’s principle of competitive 377. Interpretation (a) is correct for the given
temperature, constant osmotic exclusion states that no two species figures. As ‘A’ is more recent and shows
concentration, etc. They are known can occupy the same niche indefinitely slight reduction in the growth rate.
as regulators. for the same limiting resources.
(ii) Conformer Most animals and 372. Niche word was used for the first time 38. Ecosystem
plants cannot maintain a constant by Joseph Grinnel (1917). Niche means 378. Statements II and III are correct. A
internal environment. Their body functional role of an organism in an
simple food chain consists of producers,
temperature changes with the ecosystem. Competition becomes most
herbivores and carnivores. The length of
ambient temperature. These severe between the closely related
food chain is generally limited to 3-4
animals and plants are simply organisms which share same niche.
trophic levels due to the energy loss.
called conformer. 373. When K = N in a logistics growth curve, In grazing food chain, the producers
(iii) Partial regulator During the course it is asymptote. (i.e. plants) are eaten by herbivores
of evolution, the costs and benefits It means a population growing in a (i.e. rabbit, deer, cow, etc.) and the
of maintaining a constant internal habitat with limited resources show herbivores are eaten by carnivores.
environment are taken into initially a lag phase, followed by phase Therefore, the removal of most of the
consideration. Some species have of acceleration and deceleration and carnivores resulted in an increased
evolved the ability to regulate but finally an asymptote, i.e. when the population of deers.
only over a limited range of population density (N ) reaches the Rest, statements I and IV are incorrect.
environmental conditions, beyond carrying capacity (K ). There will be a decrease of vegetation
which they simply conform. They are when 80% tigers are removed from an
Population density (N)

partial regulators. K area.


367. Fragmentation is one of the steps 379. In the given food web, option (a) is
during decomposition, in which detritus dN correct as producers utilise the
is converted into small fragments. = rN K-N
dt K radiant energy of sun which is
Humification leads to dark coloured transformed to chemical form during
amorphous substance called humus photosynthesis. Thus, green plants
Time (t)
that is highly resistant to microbial occupy the first trophic level. The
action and undergoes decomposition at Population growth curve is logistic, herbivores constitute the secondary
an extremely slow rate. when responses are limiting the growth, trophic level and the carnivores the
Catabolism is the set of metabolic here, K is carrying capacity and N is third trophic level. Deer is herbivores,
pathways that breaks down complex population density. rabbit and rat are also herbivores but
molecules into smaller units to release frog eats the grasshoppers. Also, deer
374. In logistic growth model population,
energy. growth equation is described as is been eaten by lion.
Leaching refers to the loss of water K -Nö 380. The given figure shows spindle-shaped
= rN æç
dN
soluble plant nutrients from the soil due ÷ pyramid of number in single tree
dt è K ø
to the rain and irrigation. ecosystem. Here, a single large sized
where, N = Population density at time t
368. Habitat is the place where an organism tree provides food to a large number of
r = Intrinsic rate of natural increase
live. Population has been defined as a herbivores which support a few
K = Carrying capacity
collective group of organisms of the N K -N carnivores and the later are eaten by
same species occupying a particular when, = 1then =0 small number of top carnivores. So,
K K
place at a particular time. In the given dN here PP is used for Primary Producer,
table, the area IV has maximum Therefore, =0
dt i.e. single tree, PC is Primary
species diversity, as there are 10 Consumers, i.e. large number of
375. Its population growth curve is J-shaped
species (A-J) reside in 12 habitats, insects, SC is Secondary Consumers,
in which density increases rapidly in
while in area III the 10 species reside in i.e. small insectivorous birds and TC is
exponential fashion and then stops
13 habitats. So, exhibit less diversity
abruptly as environmental resistance or Top Consumers which may be eagles
than area IV.
another limiting factor becomes or falcon, etc.
369. The pair given in option (b) correctly effective more or less suddenly.
matched. The adaptation of lizard to its 381. According to 10% law of Lindemann, if
376. An age pyramid is a graphic 1 tonne (1000 kg) biomass is present in
environment. Desert lizards bask in the
representation of proportion of various grass, only 10% of it means 100 kg will
sun and absorb heat when their body
age groups of a population with go into deer and in tiger the biomass
temperature drops below the comfort
pre-reproductive at the base, will be only 10 kg, i.e. 10% of deer’s
zone, but move into shade when the
reproductive in the middle and biomass.
ambient temperature starts increasing.
post-reproductive at the top. For human 382. The biogeochemical cycles are of two
Some species are capable of
population, the age pyramids show age types
burrowing into the soil to hide and
distribution of males and females in a
escape from the above ground heat. (i) Gaseous cycles, in which the
combined diagram. The shape of the
370. The correct pair is in option (d), i.e. age pyramids reflects the growth status reservoir for the nutrient elements is in
stream-lined body is an aquatic of the population. In a declining the atmosphere (air) or hydrosphere
adaptation which helps these animals in populations, the shape of pyramid is (water). The four most abundant
swimming. urn-shaped. elements in the living systems, i.e.

MODULE 3
www.jeebooks.in
532 NEET Test Drive

hydrogen, carbon, oxygen and e.g. biodiversity maintenance, The other options are wrong because
nitrogen have predominantly gaseous crop pollination, etc., and natality is the (a) Aerenchyma is found in aquatic
cycles. term used for population growth or birth plants (Vallisneria, Hydrilla), but
(ii) Sedimentary cycles, the reservoir for rate in population ecology. Opuntia is a xerophytic plant.
the nutrient elements is in the (b) Biome is total sum of all ecosystem
sediments of the earth. Elements, 39. Biodiversity and present in the planet earth whereas
such as phosphorus, sulphur, Conservation age pyramid is the graphical
potassium representation of age of
388. Water hyacinth, Prosopis cineraria are organisms of one population at a
and calcium have sedimentary exotic species introduced in India. A specific time.
cycles. species of organism that is not native to
(d) Stratification is related with different
383. A - Detritus, B - Rock mineral, a locality where it is flourishing and
layers of vegetation in an ecosystem
have been moved there from its natural (like forest/grass land) and population
C - Producer, D - Litter fall range by humans or other agents is is a term referred to a group of same
Phosphates called exotic species.
Consumers Producers kind of organisms which can freely
inorganic
By action of 389. The expanded form of IUCN given in interbreed.
organism
option (c) is correct. IUCN or IUCNNR
Detritus Litter fall Uptake (International Union for Conservation of 40. Environmental Issues
Phosphates dissolved
inorganic Nature and Natural Resources) is now 395. Scrubber is an electrostatic precipitator
are released
in soil
Soil solution particles known as WCU (World Conservation in which the dirty air is cleaned by
Union). Its headquarter is at Gland, capturing the gas like SO 2 and other
Switzerland. It studies the threat to oxides in water/lime spray (CaCO 3 ).
Rock minerals
biodiversity in all parts of the world by The calcium in lime stone combines
Phosphorus cycle gathering information about the chemically with the sulphur to produce
geographical distribution, population calcium sulphate (CaSO 4 ), which is
384. During ecological succession, the size and population changes of various separately collected.
gradual and predictable change in taxa. It prepares a Red List or Red Data
species composition occurs in a given 396. Government of India have taken many
Book categorising different organisms
area. During this process, some steps to control air pollution. Out of
belonging to different categories.
species colonise an area and their which one includes compulsory PUC
populations become more numerous, 390. Western Ghats occur along the Western (Pollution Under Control) certification of
whereas populations of other species coast of India in Maharashtra, petrol driven vehicles, which test for
decline and even disappear. Karnataka, Tamil Nadu and Kerala. carbon monoxide and hydrocarbons
There is high degree of endemism as emissions of the vehicles.
385. The various stages in a hydrosere are well as richness of species of flowering
well-studied in ponds, pools or lakes. 397. SO 2 and NO 2 are the gases
plants, amphibians, reptiles, some
The various stages of hydrosere are responsible for acid rains. SO 2 mainly
mammals and butterflies. comes from burning of coal, fossil fuel
(i) Phytoplankton stage, e.g. some
391. Odd combination of animal and its part in the form of smoke from industries.
blue-green algae, green algae
is given in option (a). Dachigam 398. Reasons given in option (d) are
(Volvox), diatoms and bacteria, etc. National Park is situated near Dal Lake responsible for fish mortality rate in
(ii) Rooted submerged stage, e.g. in Jammu and Kashmir. It is known lake. It is because lots of urea and
Hydrilla, Vallisneria, etc. for conservation of the most phosphate fertiliser were used in the
(iii) Floating stage, e.g. Nelumbo, endangered Hangul or Kashmir stag in crops in the vicinity and the area was
Nymphaea, etc. Some free-floating paramount. sprayed with DDT by an aircraft.
species are Pistia, Azolla, Lemna, etc. 392. The correct combination is given in Inorganic phosphorus and nitrogen are
(iv) Red-swamp stage, e.g. species of option (c). Rann of Kutch is situated in responsible for the growth of algae.
Gujarat and provides protection mainly In polluted water, these increase due to
Scirpus, Typha, etc.
to wild ass, whereas musk deers are which algae increase greatly at the
(v) Sedge-meadow stage, e.g. species mainly protected in Kedarnath surface of water forming water bloom.
of Cyperaceae and Gramineae. sanctuary. Gir is famous for Asiatic Due to the death of these algae their
(vi) Woodland stage, e.g. Lantana, lions. Kaziranga is famous for organic matter gets decomposed due
Salix, Populus, etc. one-horned rhinoceros. to which oxygen gets depleted and
(vii) Forest stage, e.g. Tropical rainforests, 393. Sariska is a wildlife sanctuary and is aquatic animals die.
mixed forests of Almus, Acer, Quercus situated in Alwar, Rajasthan. Tiger is 399. The ascending order of BOD is Sewage
(oak), tropical deciduous forests. main protected animal in Sariska as it (S) < Distillery Effluent (DE) < Paper Mill
was selected as a Tiger Reserve in Effluent (PE) < Sugar Mill Effluent (SE).
386. If the forest cover is reduced to half than
Project Tiger (1973). Gir is a National
it will lead to desertification (formation of 400. The concentration of lead in blood
Park, associated with lions.
desert) of that area in long term. averages about 25 mg / 100 mL.
394. Parthenium hysterophorus (carrot Increase to 70 mg / 100 mL is generally
387. The species that invade a base area in
grass) is a threat to biodiversity. It is an associated with clinical symptoms.
succession is called pioneer species
alien species introduced inadvertantly Hence, a level of 30 mg / 100 mL is
and earthworm is a detritivore. Ecosystem
for some economic use, turned invasive considered alarming. The chief sources
services are the products of ecosystem
causing decline or extinction of the of Pb to water are the effluents of lead
process,
indigenous species. and lead processing industries.

MODULE 3
www.jeebooks.in
NEET Solved Paper 2019 1

NEET (National)
SOLVED PAPER 2019 (Held on : 5 May 2019)

Duration : 3 Hours Max. Mark : 720

Important Instructions
The test is of 3 hours duration and Test Booklet contains 180 questions.
Each question carries 4 marks. For each correct response, the candidate will get 4 marks.
For each incorrect response, one mark will be deducted from the total scores.
The maximum marks are 720.

PHYSICS
1. For a p-type semiconductor, which of the following 5. A small hole of area of cross-section 2 mm 2 is present
statements is true ? near the bottom of a fully filled open tank of height 2 m.
(a) Holes are the majority carriers and trivalent atoms are the Taking g = 10 m/s 2, the rate of flow of water through
dopants. the open hole would be nearly
(b) Holes are the majority carriers and pentavalent atoms are the (a) 8.9 ´ 10-6 m 3 /s (b) 2. 23 ´ 10-6 m 3 /s
dopants. (c) 6.4 ´ 10-6 m 3 /s (d) 12.6 ´ 10-6 m 3 /s
(c) Electrons are the majority carriers and pentavalent atoms are
the dopants. 6. A force F = 20 + 10 y acts on a particle in y-direction,
(d) Electrons are the majority carriers and trivalent atoms are the
dopants. where F is in newton and y in meter.
Work done by this force to move the particle from y = 0
2. A body weighs 200 N on the surface of the earth. How to y = 1 m is
much will it weigh half way down to the centre of the (a) 5 J (b) 25 J (c) 20 J (d) 30 J
earth ?
(a) 200 N (b) 250 N (c) 100 N (d) 150 N 7. Two point charges A and B, having charges +Q and -Q
respectively, are placed at certain distance apart and
3. In which of the following processes, heat is neither force acting between them is F.
absorbed nor released by a system? If 25% charge of A is transferred to B, then force
(a) Adiabatic (b) Isobaric (c) Isochoric (d) Isothermal between the charges becomes
9F 16F 4F
4. Six similar bulbs are connected as (a) (b) (c) (d) F
A B 16 9 3
shown in the adjoining figure with a
DC source of emf E and zero internal 8. When an object is shot from the bottom of a long
resistance. smooth inclined plane kept at an angle 60º with
The ratio of power consumption by horizontal, it can travel a distance x1 along the plane.
the bulbs when (i) all are glowing and But when the inclination is decreased to 30º and the
(ii) in the situation when two from same object is shot with the same velocity, it can travel
section A and one from section B are x 2 distance. Then x1 : x 2 will be
glowing, will be (a) 2:1 (b) 1: 3
(a) 9 : 4 (b) 1 : 2 E (c) 1:2 3 (d) 1: 2
(c) 2 : 1 (d) 4 : 9
www.jeebooks.in
2 NEET Test Drive

9. Pick the wrong answer in the context with rainbow. 17. +6 V


(a) The order of colours is reversed in the secondary rainbow R
0
(b) An observer can see a rainbow when his front is towards the LED
sun A 1
(Y)
(c) Rainbow is a combined effect of dispersion refraction and R
reflection of sunlight 0
(d) When the light rays undergo two internal reflections in a water
drop, a secondary rainbow is formed B 1

10. Average velocity of a particle executing SHM in one The correct Boolean operation represented by the
complete vibration is circuit diagram drawn is
Aw2 Aw (a) OR (b) NAND (c) NOR (d) AND
(a)Aw (b) (c) zero (d)
2 2
18. A soap bubble, having radius of 1 mm, is blown from
11. The total energy of an electron in an atom in an orbit is a detergent solution having a surface tension of
-3.4 eV. 2.5 ´ 10 -2 N/m. The pressure inside the bubble equals at
Its kinetic and potential energies are, respectively: a point Z 0below the free surface of water in a container.
(a) -3.4 eV, -6.8 eV (b) 3.4 eV, -6.8 eV Taking g = 10 m/s 2, density of water = 10 3kg/m 3, the
(c) 3.4 eV, 3.4 eV (d) -3.4 eV, -3.4 eV value of Z 0 is
12. At a point A on the earth’s surface the angle of dip, (a) 10 cm (b) 1 cm (c) 0.5 cm (d) 100 cm
d = +25º.
19. The displacement of a particle executing simple
At a point B on the earth’s surface the angle of dip,
harmonic motion is given by y = A0 + A sin wt + Bcos wt
d = -25º.
We can interpret that Then the amplitude of its oscillation is given by
(a) A is located in the southern hemisphere and B is located in (a) A 2 + B2 (b) A02 + ( A + B)2
the northern hemisphere.
(c) A + B (d) A0 + A 2 + B2
(b) A is located in the northern hemisphere and B is located in
the southern hemisphere.
20. The work done to raise a mass m from the surface of
(c) A and B are both located in the southern hemisphere.
the earth to a height h, which is equal to the radius of
(d) A and B are both located in the northern hemisphere.
the earth, is
13. In which of the following devices, the eddy current 1 3
(a) 2mgR (b) mgR (c) mgR (d) mgR
effect is not used? 2 2
(a) Magnetic braking in train (b) Electromagnet
21. The speed of a swimmer in still water is 20 m/s. The
(c) Electric heater (d) Induction furnace
speed of river water is 10 m/s and is flowing due east. If
14. Two particles A and B are moving in uniform circular he is standing on the south bank and wishes to cross
motion in concentric circles of radii rA and rB with speed the river along the shortest path the angle at which he
v A and vB respectively. Their time period of rotation is should make his strokes w.r.t. north is given by
the same. The ratio of angular speed of A to that of B (a) 0º (b) 60° west (c) 45º west (d) 30º west
will be
22. Two parallel infinite line charges with linear charge
(a) v A : v B (b) rB : rA (c) 1 : 1 (d) rA: rB
densities +l C/m and -l C/m are placed at a distance of
15. Two similar thin equi-convex lenses, of focal length f 2R in free space. What is the electric field mid-way
each, are kept co-axially in contact with each other between the two line charges?
such that the focal length of the combination is F1 . 2l l l
(a) N/C (b) N/C (c) N/C (d) Zero
When the space between the two lenses is filled with pe0 R pe0 R 2pe0 R
glycerine (which has the same refractive index (m = 15 .)
as that of glass) then the equivalent focal length is F2. 23. Which of the following acts as a circuit protection
The ratio F1 : F2 will be device?
(a) Inductor (b) Switch (c) Fuse (d) Conductor
(a) 1 : 2 (b) 2 : 3 (c) 3 : 4 (d) 2 : 1
24. A parallel plate capacitor of capacitance 20mF is being
16. An electron is accelerated through a potential charged by a voltage source whose potential is changing
difference of 10,000 V. at the rate of 3 V/s. The conduction current through the
Its de-Broglie wavelength is, (nearly) : connecting wires and the displacement current through
(me = 9 ´ 10 -31 kg) the plates of the capacitor, would be, respectively.
(a) 12.2 ´ 10-12 m (b) 12.2 ´ 10-14 m (a) 60 mA, 60 mA (b) 60 mA, zero
(c) 12.2 nm (d) 12.2 ´ 10-13 m (c) zero, zero (d) zero, 60 mA
www.jeebooks.in
NEET Solved Paper 2019 3

25. When a block of mass M is suspended by a long wire of 33. A disc of radius 2 m and mass 100 kg rolls on a
length L, the length of the wire becomes (L + l). horizontal floor. Its centre of mass has speed of 20
The elastic potential energy stored in the extended wire cm/s. How much work is needed to stop it?
is (a) 30 kJ (b) 2 J
1 1 (c) 1 J (d) 3 J
(a) MgL (b) Mgl (c) MgL (d) Mgl
2 2
34. In an experiment, the percentage of error occurred in
26. A hollow metal sphere of radius R is uniformly the measurement of physical quantities A , B, C and D
charged. are 1%, 2%, 3% and 4% respectively. Then the maximum
The electric field due to the sphere at a distance r from percentage of error in the measurement X , where,
the centre A 2B1 / 2
X = 1 / 3 3 will be
(a) zero as r increases for r < R, decreases as r increases for C D
r>R
(d) æç ö÷ %
3
(b) zero as r increases for r < R, increases as r increases for (a) 16% (b) -10% (c) 10%
è 13 ø
r>R
(c) decreases as r increases for r < R and for r > R 35. Body A of mass 4m moving with speed u collides with
(d) increases as r increases for r < R and for r > R another body B of mass 2m, at rest. The collision is
27. A mass m is attached to a thin wire and whirled in a head on and elastic in nature. After the collision the
vertical circle. fraction of energy lost by the colliding body A is
8 4 5 1
The wire is most likely to break when: (a) (b) (c) (d)
9 9 9 9
(a) the wire is horizontal
(b) the mass is at the lowest point 36. The radius of circle, the period of y
(c) inclined at an angle of 60° from vertical P(t=0)
revolution, initial position and
(d) the mass is at the highest point
sense of revolution are indicated in T=4 s
28. Which colour of the light has the longest wavelength? the alongside figure. x
(a) Blue (b) Green (c) Violet (d) Red y-projection of the radius vector of 3m
rotating particle P is
29. A particle moving with velocity v is acted by three pt
(a) y(t ) = 4sinæç ö÷, where y in m
forces shown by the vector triangle PQR. è2 ø
3pt ö
The velocity of the particle will (b) y(t ) = 3cosæç ÷, where y in m
P è 2 ø
pt
(c) y(t ) = 3cosæç ö÷, where y in m
è2 ø
(d) y(t ) = -3cos 2pt , where y in m

37. In total internal reflection when the angle of incidence


R Q
is equal to the critical angle for the pair of media in
(a) decrease contact, what will be angle of refraction?
(b) remain constant
(a) 0º (b) Equal to angle of incidence
(c) change according to the smallest force QR
(c) 90º (d) 180º
(d) increase

30. Ionised hydrogen atoms and a -particles with same 38. A solid cylinder of mass 2 kg and radius 4 cm is rotating
about its axis at the rate of 3 rpm. The torque required
momenta enters perpendicular to a constant magnetic
to stop after 2p revolutions is
field, B.
The ratio of their radii of their paths rH : ra will be (a) 2 ´ 10-3 N-m (b) 12 ´ 10-4 N-m
(c) 2 ´ 106 N-m (d) 2 ´ 10-6 N-m
(a) 1 : 2 (b) 4 : 1 (c) 1 : 4 (d) 2 : 1
39. A block of mass 10 kg is in contact against the inner wall
31. The unit of thermal conductivity is :
-1 -1 -1 -1 -1 -1
of a hollow cylindrical drum of radius 1 m. The
(a) J m K (b) W m K (c) W m K (d) J m K
coefficient of friction between the block and the inner
32. A coil of 800 turns effective area 0.05 m 2 is kept wall of the cylinder is 0.1. The minimum angular
perpendicular to a magnetic field 5 ´ 10 -5 T. When the velocity needed for the cylinder to keep the block
stationary when the cylinder is vertical and rotating
plane of the coil is rotated by 90 º around any of its
about its axis, will be (g = 10 m/s 2 )
co-planar axis in 0.1 s, the emf induced in the coil will
be 10
(a) rad/s (b) 10 rad/s (c) 10p rad/s (d) 10 rad/s
-3 2p
(a) 0.2 V (b) 2 ´ 10 V (c) 0.02 V (d) 2 V
www.jeebooks.in
4 NEET Test Drive

40. A cylindrical conductor of radius R is carrying a 42. Increase in temperature of a gas filled in a container
constant current. The plot of the magnitude of the would lead to
magnetic field B with the distance d from the centre of (a) increase in its kinetic energy
the conductor, is correctly represented by the figure (b) decrease in its pressure
B
(c) decrease in intermolecular distance
B
(d) increase in its mass
(a) (b) 43. A copper rod of 88 cm and an aluminium rod of
unknown length have their increase in length
R d R d independent of increase in temperature. The length of
B B aluminium rod is
(a) 113.9 cm (b) 88 cm
(c) (d)
(c) 68 cm (d) 6.8 cm

44. a-particle consists of


R d R d
(a) 2 electrons, 2 protons and 2 neutrons
(b) 2 electrons and 4 protons only
41. In the circuits shown below, the readings of voltmeters (c) 2 protons only
and the ammeters will be (d) 2 protons and 2 neutrons only
10 Ω i1 10 Ω i2
45. In a double slit experiment, when light of wavelength
400 nm was used, the angular width of the first
10 Ω
V1 V2 A2
minima formed on a screen placed 1 m away, was
A1
found to be 0.2º. What will be the angular width of the
first minima, if the entire experimental apparatus is
10 V 10 V immersed in water? (m water = 4 / 3)
(a) V1 = V2 and i1 > i 2 (b) V1 = V2 and i i = i 2 (a) 0.15º (b) 0.051°
(c) V2 > V1 and i1 > i 2 (d) V2 > V1 and i1 = i 2 (c) 0.1º (d) 0.266º

CHEMISTRY
46. For the chemical reaction,N 2(g) + 3H 2(g) - 2NH (g) (b) 100 mL of 0.1 M HCl + 200 mL of 0.1 M NH4 OH
3
(c) 100 mL of 0.1 M HCl + 100 mL of 0.1 M NaOH
The correct option is
(d) 50 mL of 0.1 M NaOH + 25 mL of 0.1 M CH3COOH
d [N2 ] d [NH3 ] d [N2 ] 1 d [NH3 ]
(a) - =2 (b) - =
dt dt dt 2 dt 51. The number of moles of hydrogen molecules required
d [H2 ] d [NH3 ] 1 d [H2 ] 1 d [NH3 ] to produce 20 moles of ammonia through Haber’s
(c) 3 =2 (d) - =-
dt dt 3 dt 2 dt process is
(a) 20 (b) 30 (c) 40 (d) 10
47. The non-essential amino acid among the following is
(a) leucine (b) alanine (c) lysine (d) valine 52. Which of the following diatomic molecular species has
48. For an ideal solution, the correct option is only p-bonds according to molecular orbital theory?
(a) N2 (b) C 2 (c) Be2 (d) O 2
(a) D mix V ¹ 0 at constant T and p
(b) Dmix H = 0 at constant T and p 53. The biodegradable polymer is
(c) DmixG = 0 at constant T and p (a) nylon-2-nylon-6 (b) nylon-6
(d) DmixS = 0 at constant T and p (c) buna-S (d) nylon-6,6
49. The manganate and permanganate ions are tetrahedral,
54. Which of the following reactions are
due to
disproportionation reaction?
(a) there is no p-bonding
(i) 2Cu+ ¾® Cu2+ + Cu0
(b) the p-bonding involves overlap of p -orbitals of oxygen with
(ii) 3MnO24 - + 4H+ ¾® 2MnO4- + MnO2 + 2H2 O
p -orbitals of manganese D
(c) the p-bonding involves overlap of d-orbitals of oxygen with (iii) 2KMnO4 ¾®K2 MnO4 + MnO2 + O2
d-orbitals of manganese
(iv) 2MnO4- + 3Mn2+ + 2H2O ¾®5MnO2 + 4HÅ
(d) the p-bonding involves overlap of p-orbitals of oxygen with
d-orbitals of manganese Select the correct option from the following.
50. Which will make basic buffer? (a) (i), (ii) and (iii) (b) (i), (iii) and (iv)
(a) 100 mL of 0.1 M CH3COOH + 100 mL of 0.1 M NaOH (c) (i) and (iv) only (d) (i) and (ii) only
www.jeebooks.in
NEET Solved Paper 2019 5

55. The compound that is most difficult to protonate is 62. The number of sigma (s) and pi (p) bonds in
O O pent-2-en-4-yne is
(a) H3C H (b) H3C CH3
(a) 8s-bonds and 5p-bonds (b) 11s-bonds and 2p-bonds
O O (c) 13s-bonds and no p-bonds (d) 10s-bonds and 3p-bonds
(c) Ph H (d) H H
63. For the second period elements the correct increasing
56. Which of the following species is not stable? order of first ionisation enthalpy is
(a) [GeCl 6 ]2- (b) [Sn(OH)6 ]2- (c) [SiCl 6 ]2- (d) [SiF6 ]2- (a) Li < B < Be < C < O < N < F < Ne
(b) Li < B < Be < C < N < O < F < Ne
57. An alkene ‘A’ on reaction with O 3 and Zn - H2O gives (c) Li < Be < B < C < O < N < F < Ne
propanone and ethanal in equimolar ratio. Addition of (d) Li < Be < B < C < N < O < F < Ne
HCl to alkene ‘A’ gives ‘B’ as the major product.
The structure of product ‘B’ is 64. Among the following, the narrow spectrum antibiotic is
CH2Cl CH3 (a) ampicillin (b) amoxycillin
| | (c) chloramphenicol (d) penicillin G
(a) H3C ¾CH2 ¾C H ¾CH3 (b) H3C ¾ CH2 ¾ C ¾ CH3
| 65. The method used to remove temporary hardness of
Cl water is
CH3 CH3
| | (a) Clark’s method (b) ion-exchange method
(c) H3C ¾ C H ¾ C H (d) Cl ¾ CH2 ¾CH2 ¾ C H (c) synthetic resins method (d) Calgon’s method
| | |
Cl CH3 CH3 66. Which of the following series of transitions in the
spectrum of hydrogen atom fall in visible region?
58. Match the following : (a) Balmer series (b) Paschen series
(c) Brackett series (d) Lyman series
A. Pure nitrogen (i) Chlorine
B. Haber process (ii) Sulphuric acid 67. Match the xenon compounds in Column I with its
C. Contact process (iii) Ammonia structure in Column II and assign the correct code:
D. Deacon’s process (iv) Sodium azide or barium azide
Column I Column II
Which of the following is the correct option? A. XeF4 (i) Pyramidal
A B C D A B C D B. XeF6 (ii) Square planar
(a) (ii) (iv) (i) (iii) (b) (iii (iv) (ii) (i)
(c) (iv) (iii) (ii) (i) (d) (i) (ii) (iii) (iv) C. XeOF4 (iii) Distorted octahedral
D. XeO 3 (iv) Square pyramidal
59. The most suitable reagent for the following conversion,
is Codes
A B C D A B C D
H3 C CH3
(a) (ii) (iii) (iv) (i) (b) (ii) (iii) (i) (iv)
H3C—C≡≡C—CH3
H H (c) (iii) (iv) (i) (ii) (d) (i) (ii) (iii) (iv)
cis-2-butene

(a) H2 , Pd/C, quinoline (b) Zn/HCl


68. The structure of intermediate A in the following
reaction, is
(c) Hg 2+ /H+ , H2O (d) Na/liquid NH3
CH3
CH OH
60. The correct order of the basic strength of methyl CH3 O
substituted amines in aqueous solution is O2 H+
A + H3C CH3
(a) (CH3 )3 N > CH3NH2 > (CH3 )2NH H2O

(b) (CH3 )3 N > (CH3 )2NH > CH3NH2 CH3 CH3


(c) CH3NH2 > (CH3 )2NH > (CH3 )3N
(a) H3C—C—O—O—H (b) O—O—CH
(d) (CH3 )2 NH > CH3NH2 > (CH3 )3N
CH3
61. A gas at 350 K and 15 bar has molar volume 20 percent
smaller than that for an ideal gas under the same
conditions. The correct option about the gas and its CH3
compressibility factor (Z) is CH2—O—O—H
CH
(a) Z > 1and repulsive forces are dominant
HC O
(b) Z < 1and attractive forces are dominant CH3
CH3
(c) Z < 1and repulsive forces are dominant
(d) Z > 1and attractive forces are dominant (c) (d)
www.jeebooks.in
6 NEET Test Drive

69. The correct structure of tribromooctaoxide is (a) -23.16 kJ mol -1 (b) 46.32 kJ mol -1
(c) 23.16 kJ mol -1 (d) -46.32 kJ mol -1
O O O O O O–
(a) O==Br—Br—Br—O –

(b) O—Br—Br—Br==O
– 78. Conjugate base for Bronsted acids H2O and HF are
O O– –
O O– (a) H3O+ and F - , respectively (b) OH- and F - , respectively
O– O–
(c) H3O+ and H2F+ , respectively (d) OH- and H2F+ , respectively
O O– O O O O 79. Which one is malachite from the following?
(c) O==Br—Br—Br—O – (d) O==Br—Br—Br==O (a) Cu(OH)2 (b) Fe 3O 4
O O– O O (c) CuCO 3 × Cu(OH)2 (d) CuFeS2
O– O
80. pH of a saturated solution of Ca(OH)2 is 9.
70. Which is the correct thermal stability order for H2 E
The solubility product (K sp) of Ca(OH)2 is
(E = O, S, Se, Te and Po)?
. ´ 10-10
(a) 025 (b) 0125
. ´ 10-15
(a) H2O < H2S < H2Se < H2 Te < H2Po
(c) 0.5 ´ 10-10 (d) 0.5 ´ 10-15
(b) H2Po < H2 Te < H2Se < H2S < H2O
(c) H2Se < H2 Te < H2Po < H2O < H2S 81. Which of the following is incorrect statement?
(d) H2S < H2O < H2Se < H2 Te < H2Po (a) SiCl 4 is easily hydrolysed
(b) GeX 4 ( X =, F, Cl, Br, I) is more stable than GeX 2
71. In which case change in entropy is negative? (c) SnF4 is ionic in nature
(a) Expansion of a gas at constant temperature (d) PbF4 is covalent in nature
(b) Sublimation of solid to gas
(c) 2H(g )¾® H2 (g ) 82. Which mixture of the solutions will lead to the
(d) Evaporation of water formation of negatively charged colloidal [AgI]I- sol ?
(a) 50 mL of 1 M AgNO 3 + 50 mL of 2 M KI
72. A compound is formed by cation C and anion A. The (b) 50 mL of 2 M AgNO 3 + 50 mL of 1.5 M KI
anions form hexagonal close packed (hcp) lattice and (c) 50 mL of 1 M AgNO 3 + 50 mL of 0.1 M KI
the cations occupy 75% of octahedral voids. (d) 50 mL of 1 M AgNO 3 + 50 mL of 1.5 M KI
The formula of the compound is
(a) C 3 A2 (b) C 3 A4 (c) C 4 A3 (d) C 2 A3 83. The mixture that forms maximum boiling azeotrope is
(a) ethanol + water
73. Among the following, the one that is not a green house (b) acetone + carbon disulphide
gas is (c) heptane + octane
(a) methane (b) ozone (d) water + nitric acid
(c) sulphur dioxide (d) nitrous oxide
° = 0 .59 V at 298 K,
84. For a cell involving one electron, E cell
74. Identify the incorrect statement related to PCl5 from the equilibrium constant for the cell reaction is
the following: é 2.303 RT ù
(a) Two axial P¾ Cl bonds make an angle of 180° with each other êëGiven that, = 0.059 V at T = 298 K ú
F û
(b) Axial P¾ Cl bonds are longer than equatorial
P¾ Cl bonds . ´ 105
(a) 10 . ´ 1010
(b) 10
(c) PCl 5 molecule is non-reactive . ´ 1030
(c) 10 . ´ 10 2
(d) 10
(d) Three equatorial P¾Cl bonds make an angle of 120° with
each other
85. Among the following, the reaction that proceeds
through an electrophilic substitution, is
75. Which of the following is an amphoteric hydroxide? AlCl3
(a) Ca(OH)2 (b) Mg(OH)2 (c) Be(OH)2 (d) Sr(OH)2 (a) + Cl2 —Cl + HCl

76. 4 d , 5p , 5 f and 6 p-orbitals are arranged in the order of Cl Cl


decreasing energy. The correct option is UV light
(a) 6 p > 5f > 5 p > 4 d (b) 5 p > 5f > 4 d > 5 p (b) + Cl2 Cl Cl
(c) 5f > 6 p > 4 d > 5 p (d) 5f > 6 p > 5 p > 4 d
Cl Cl
77. For the cell reaction,
Heat
(c) —CH2Cl + H2O
2Fe3+ (aq) + 2I- (aq) ¾® 2Fe2+ (aq) + I2 (aq) —CH2OH + HCl

È
E cell = 0 .24 V at 298 K. The standard Gibbs energy + Cu2Cl2
(D rG ) of the cell reaction is
È (d) —N2Cl– —Cl + N2

[Given that Faraday constant F = 96500 C mol -1 ]


www.jeebooks.in
NEET Solved Paper 2019 7

86. The major product of the following reaction is: (a) t 26g e g0 (b) e 3 t 23
COOH (c) e 4 t 22 (d) t 24g e g2
Strong heating
+ NH3
COOH 88. If the rate constant for a first order reaction is k, the
O t required for the completion of 99% of the
time ()
COOH
reaction is given by
(a) (b) (a) t = 6.909/k (b) t = 4.606/k
NH
(c) t = 2.303/k (d) t = 0.693/k
NH2
O 89. Under isothermal condition, a gas at 300 K expands
from 0.1 L to 0.25 L against a constant external
NH2 COOH
pressure of 2 bar. The work done by the gas is
(c) (d)
(Given that 1 L bar = 100 J)
NH2 CONH2 (a) 5 kJ (b) 25 J (c) 30 J (d) -30 J

87. What is the correct electronic configuration of the 90. Enzymes that utilise ATP in phosphate transfer require
central atom in K4 [Fe(CN)6 ] based on crystal field an alkaline earth metal (M) as the cofactor. M is
theory? (a) Mg (b) Ca (c) Sr (d) Be

BIOLOGY
91. Cells in G 0 phase Select the correct option.
A B C D A B C D
(a) enter the cell cycle (a) (iii) (i) (ii) (iv) (b) (iii) (i) (iv) (ii)
(b) suspend the cell cycle (c) (iii) (iv) (i) (ii) (d) (i) (iii) (ii) (iv)
(c) terminate the cell cycle
(d) exit the cell cycle 96. In some plants, the female gamete develops into
embryo without fertilisation.
92. Phloem in gymnosperms lacks
This phenomenon is known as
(a) sieve tubes only
(a) parthenocarpy (b) syngamy
(b) companion cells only
(c) parthenogenesis (d) autogamy
(c) both sieve tubes and companion cells
(d) albuminous cells and sieve cells 97. Under which of the following conditions will there be
no change in the reading frame of following mRNA?
93. Which of the following factors is responsible for the 5¢AACAGCGGUGCUAUU3’
formation of concentrated urine?
(a) Deletion of G from 5th position
(a) Maintaining hyperosmolarity towards inner medullary
(b) Insertion of A and G at 4th and 5th positions, respectively
interstitium in the kidneys
(c) Deletion of GGU from 7th, 8th and 9th positions
(b) Secretion of erythropoietin by juxtaglomerular complex
(d) Insertion of G at 5th position
(c) Hydrostatic pressure during glomerular filtration
(d) Low levels of antidiuretic hormone 98. Which of the following muscular disorders is
inherited?
94. Match the hominids with their correct brain size (a) Muscular dystrophy (b) Myasthenia gravis
A. Homo habilis (i) 900 cc (c) Botulism (d) Tetany
B. Homo neanderthalensis (ii) 1350 cc
99. Select the correct option.
C. Homo erectus (iii) 650-800 cc
(a) 11th and 12th pairs of ribs are connected to the sternum with
D. Homo sapiens (iv) 1400 cc
the help of hyaline cartilage
Select the correct option. (b) Each rib is a flat thin bone and all the ribs are connected
A B C D A B C D dorsally to the thoracic vertebrae and ventrally to the sternum
(a) (iii) (ii) (i) (iv) (b) (iii) (iv) (i) (ii) (c) There are seven pairs of vertebrosternal, three pairs of
(c) (iv) (iii) (i) (ii) (d) (iii) (i) (iv) (ii) vertebrochondral and two pairs of vertebral ribs
(d) 8th, 9th and 10th pairs of ribs articulate directly with the sternum
95. Match the following genes of the Lac operon with their 100. Grass leaves curl inwards during very dry weather.
respective products Select the most appropriate reason from the following
A. i gene (i) b-galactosidase (a) Flaccidity of bulliform cells
B. z gene (ii) Permease (b) Shrinkage of air spaces in spongy mesophyll
C. a gene (iii) Repressor (c) Tyloses in vessels
D. y gene (iv) Transacetylase (d) Closure of stomata
www.jeebooks.in
8 NEET Test Drive

101. In Antirrhinum (Snapdragon), a red flower was crossed 109. Which of the following statements regarding
with a white flower and in mitochondria is incorrect?
F1 -generation, pink flowers were obtained. (a) Enzymes of electron transport are embedded in outer
When pink flowers were selfed, the membrane
F2-generation showed white, red and pink flowers. (b) Inner membrane is convoluted with infoldings
Choose the incorrect statement from the following (c) Mitochondrial matrix contains single circular DNA molecule
(a) Pink colour in F1 is due to incomplete dominance and ribosomes
1 2 1 (d) Outer membrane is permeable to monomers of
(b) Ratio of F 2 is (Red) : (Pink) : (White) carbohydrates, fats and proteins
4 4 4
(c) Law of segregation does not apply in this experiment 110. What triggers activation of protoxin to active Bt toxin
(d) This experiment does not follow the principle of dominance of Bacillus thuringiensis in bollworm?
102. Xylem translocates (a) Moist surface of midgut
(a) water and mineral salts only (b) Alkaline pH of gut
(b) water, mineral salts and some organic nitrogen only (c) Acidic pH of stomach
(d) Body temperature
(c) water, mineral salts, some organic nitrogen and hormones
(d) water only 111. It takes very long time for pineapple plants to produce
103. Placentation in which ovules develop on the inner wall flowers. Which combination of hormones can be
of the ovary or in peripheral part, is applied to artificially induce flowering in pineapple
plants throughout the year to increase yield?
(a) axile (b) parietal (c) free central (d) basal
(a) Gibberellin and Cytokinin
104. Which of the following glucose transporters is (b) Gibberellin and Abscisic acid
insulin-dependent? (c) Cytokinin and Abscisic acid
(a) GLUT II (b) GLUT III (d) Auxin and Ethylene
(c) GLUT IV (d) GLUT I
112. In a species, the weight of newborn ranges from 2 to
105. Select the incorrect statement. 5 kg. 97% of the newborn with an average weight
(a) Inbreeding is essential to evolve purelines in any animal between 3 to 3.3 kg survive whereas 99% of the infants
(b) Inbreeding selects harmful recessive genes that reduce born with weights from 2 to 2.5 kg or 4.5 to 5 kg die.
fertility and productivity Which type of selection process is taking place?
(c) Inbreeding helps in accumulation of superior genes and (a) Stabilising selection
elimination of undesirable genes (b) Disruptive selection
(d) Inbreeding increases homozygosity (c) Cyclical selection
(d) Directional selection
106. Select the incorrect statement.
(a) In male grasshoppers, 50% of sperms have no 113. Which of the following can be used as a biocontrol
sex-chromosome agent in the treatment of plant disease?
(b) In domesticated fowls, sex of progeny depends on the type (a) Chlorella (b) Anabaena
of sperm rather than egg (c) Lactobacillus (d) Trichoderma
(c) Human males have one of their sex chromosomes much
shorter than the other 114. Concanavalin A is
(d) Male fruitfly is heterogametic (a) an essential oil (b) a lectin
(c) a pigment (d) an alkaloid
107. Colostrum, the yellowish fluid, secreted by mother
during the initial days of lactation is very essential to 115. Match the following hormones with the respective
impart immunity to the new born infants because it disease.
contains
A. Insulin (i) Addison's disease
(a) monocytes (b) macrophages
(c) immunoglobulin A (d) natural killer cells B. Thyroxin (ii) Diabetes insipidus
C. Corticoids (iii) Acromegaly
108. A gene locus has two alleles A, a. If the frequency of
dominant allele A is 0.4, then what will be the D. Growth hormone (iv) Goitre
frequency of homozygous dominant, heterozygous and (v) Diabetes mellitus
homozygous recessive individuals in the population?
(a) 0.16(AA); 0.24(Aa); 0.36(aa) Select the correct option.
(b) 0.16(AA); 0.48(Aa); 0.36(aa) A B C D A B C D
(c) 0.16(AA); 0.36(Aa); 0.48(aa) (a) (ii) (iv) (iii) (i) (b) (v) (iv) (i) (iii)
(d) 0.36(AA); 0.48(Aa); 0.16(aa) (c) (ii) (iv) (i) (iii) (d) (v) (i) (ii) (iii)
www.jeebooks.in
NEET Solved Paper 2019 9

116. What is the fate of the male gametes discharged in the 124. Which of the statements given below is not true about
synergid? formation of annual rings in trees?
(a) All fuse with the egg (a) Differential activity of cambium causes light and dark bands
(b) One fuses with the egg, other(s) fuse(s) with synergid of tissue early and late wood, respectively
nucleus (b) Activity of cambium depends upon variation in climate
(c) One fuses with the egg and other fuses with central cell (c) Annual rings are not prominent in trees of temperate region
nuclei (d) Annual ring is a combination of springwood and
(d) One fuses with the egg other(s) degenerate (s) in the autumnwood produced in a year
synergid 125. Purines found both in DNA and RNA are
117. Match the following structures with their respective (a) adenine and guanine (b) guanine and cytosine
location in organs. (c) cytosine and thymine (d) adenine and thymine
126. Variations caused by mutation, as proposed by Hugo de
Column I Column II
Vries are
A. Crypts of Lieberkuhn (i) Pancreas (a) random and directionless (b) small and directional
B. Glisson's capsule (ii) Duodenum (c) small and directionless (d) random and directional
C. Islets of Langerhans (iii) Small intestine 127. Which of the following statements is not correct?
D. Brunner's glands (iv) Liver (a) The hydrolytic enzymes of lysosomes are active under acidic pH
(b) Lysosomes are membrane bound structures
Select the correct option from the following (c) Lysosomes are formed by the process of packaging in the
A B C D A B C D endoplasmic reticulum
(a) (ii) (iv) (i) (iii) (b) (iii) (iv) (i) (ii) (d) Lysosomes have numerous hydrolytic enzymes
(c) (iii) (ii) (i) (iv) (d) (iii) (i) (ii) (iv)
128. The shorter and longer arms of a submetacentric
118. Which one of the following is not a method of in situ chromosome are referred to as
conservation of biodiversity? (a) p-arm and q-arm, respectively
(a) Wildlife sanctuary (b) Botanical garden (b) q-arm and p-arm, respectively
(c) Sacred grove (d) Biosphere reserve (c) m-arm and n-arm, respectively
(d) s-arm and l-arm, respectively
119. Expressed Sequence Tags (ESTs) refers to
129. The ciliated epithelial cells are required to move particles
(a) polypeptide expression (b) DNA polymorphism
or mucus in a specific direction. In humans, these cells
(c) novel DNA sequences (d) genes expressed as RNA
are mainly present in
120. Which of the following ecological pyramids is generally (a) Fallopian tubes and pancreatic duct
inverted? (b) eustachian tube and salivary duct
(a) Pyramid of energy (c) bronchioles and Fallopian tubes
(b) Pyramid of biomass in a forest (d) bile duct and bronchioles
(c) Pyramid of biomass in a sea 130. Which of the following statements is incorrect?
(d) Pyramid of numbers in grassland
(a) Claviceps is a source of many alkaloids and LSD
121. Which of the following pairs of gases is mainly (b) Conidia are produced exogenously and ascospores
responsible for greenhouse effect? endogenously
(c) Yeasts have filamentous bodies with long thread-like hyphae
(a) Oxygen and Nitrogen
(d) Morels and truffles are edible delicacies
(b) Nitrogen and Sulphur dioxide
(c) Carbon dioxide and Methane 131. Match Column I with Column II.
(d) Ozone and Ammonia
Column I Column II
122. Which of the following immune responses is A. Saprophyte (i) Symbiotic association of fungi with
responsible for rejection of kidney graft? plant roots
(a) Humoral immune response B. Parasite (ii) Decomposition of dead organic
(b) Inflammatory immune response materials
(c) Cell-mediated immune response C. Lichens (iii) Living on living plants or animals
(d) Auto-immune response
D. Mycorrhiza (iv) Symbiotic association of algae and
123. Pinus seed cannot germinate and establish without fungi
fungal association. This is because Choose the correct answer from the option given
(a) it has obligate association with mycorrhizae below:
(b) it has very hard seed coat A B C D A B C D
(c) its seeds contain inhibitors that prevent germination (a) (iii) (ii) (i) (iv) (b) (ii) (i) (iii) (iv)
(d) its embryo is immature (c) (ii) (iii) (iv) (i) (d) (i) (ii) (iii) (iv)
www.jeebooks.in
10 NEET Test Drive

132. Which of the following is a commercial blood (c) A unit of distance between genes on chromosomes,
representing 50% cross over
cholesterol lowering agent?
(d) A unit of distance between two expressed genes
(a) Statin (b) Streptokinase representing 10% cross over
(c) Lipases (d) Cyclosporin A
141. Extrusion of second polar body from egg nucleus
133. Thiobacillus is a group of bacteria helpful in carrying out occurs
(a) chemoautotrophic-fixation (b) nitrification (a) after fertilisation
(c) denitrification (d) nitrogen-fixation (b) before the entry of sperm into ovum
134. Polyblend, a fine powder of recycled modified plastic, (c) simultaneously with first cleavage
has proved to be a good material for (d) after the entry of sperm but before fertilisation
(a) use as a fertiliser (b) construction of roads 142. What is the site of perception of photoperiod necessary
(c) making tubes and pipes (d) making plastic sacks for induction of flowering in plants?
135. Select the correct sequence of organs in the alimentary (a) Pulvinus (b) Shoot apex
canal of cockroach starting from mouth (c) Leaves (d) Lateral buds
(a) Pharynx ® Oesophagus ® Gizzard ® Crop ® Ileum
® Colon ® Rectum
143. Match the following organisms with their respective
(b) Pharynx ® Oesophagus ® Gizzard ® Ileum ® Crop characteristics.
® Colon ® Rectum Column I Column II
(c) Pharynx ® Oesophagus ® Ileum ® Crop ® Gizzard
® Colon ® Rectum A. Pila (i) Flame cells
(d) Pharynx ® Oesophagus ® Crop ® Gizzard ® Ileum B. Bombyx (ii) Comb plates
® Colon ® Rectum C. Pleurobrachia (iii) Radula
136. Identify the correct pair representing the causative D. Taenia (iv) Malpighian tubules
agent of typhoid fever and the confirmatory test for
Select the correct option from the following :
typhoid.
A B C D A B C D
(a) Streptococcus pneumoniae / Widal test (a) (iii) (iv) (ii) (i) (b) (ii) (iv) (iii) (i)
(b) Salmonella typhi / Anthrone test (c) (iii) (ii) (iv) (i) (d) (iii) (ii) (i) (iv)
(c) Salmonella typhi / Widal test
(d) Plasmodium vivax / UTI test 144. The correct sequence of phases of cell cycle is
137. What would be the heart rate of a person if the cardiac (a) G1 ® G 2 ® S ® M (b) S ® G1 ® G 2 ® M
output is 5 L, blood volume in the ventricles at the end (c) G1 ® S ® G 2 ® M (d) M ® G1 ® G 2 ® S
of diastole is 100 mL and at the end of ventricular 145. Which of the following protocols did aim for reducing
systole is 50 mL?
emission of chlorofluorocarbons into the atmosphere?
(a) 75 beats per minute (b) 100 beats per minute
(a) Kyoto Protocol (b) Gothenburg Protocol
(c) 125 beats per minute (d) 50 beats per minute
(c) Geneva Protocol (d) Montreal Protocol
138. Which of the following statements is incorrect?
146. Match the following organisms with the products they
(a) Viruses are obligate parasites
produce
(b) Infective constituent in viruses is the protein coat
(c) Prions consist of abnormally folded proteins A. Lactobacillus (i) Cheese
(d) Viroids lack a protein coat B. Saccharomyces cerevisiae (ii) Curd
139. Consider the following statement C. Aspergillus niger (iii) Citric acid
(A) Coenzyme or metal ion that is tightly bound to D. Acetobacter aceti (iv) Bread
enzyme protein is called prosthetic group. (v) Acetic acid
(B) A complete catalytic active enzyme with its bound
prosthetic group is called apoenzyme. Select the correct option.
Select the correct option. A B C D A B C D
(a) (A) is true but (B) is false (b) Both (A) and (B) are false (a) (ii) (iv) (iii) (v) (b) (iii) (iv) (v) (i)
(c) (A) is false but (B) is true (d) Both (A) and (B) are true (c) (ii) (i) (iii) (v) (d) (ii) (iv) (v) (iii)

140. What map unit (Centimorgan) is adopted in the 147. Which of the following contraceptive methods do
construction of genetic maps? involve a role of hormone?
(a) A unit of distance between two expressed genes (a) Barrier method, Lactational amenorrhea, Pills
representing 100% cross over (b) Cu-T, Pills, Emergency contraceptives
(b) A unit of distance between genes on chromosomes, (c) Pills, Emergency contraceptives, Barrier methods
representing 1% cross over (d) Lactational amenorrhea, Pills, Emergency contraceptives
www.jeebooks.in
NEET Solved Paper 2019 11

148. Which of the following sexually transmitted diseases is (a) Annelida, Arthropoda and Mollusca
not completely curable? (b) Arthropoda, Mollusca and Chordata
(c) Annelida, Mollusca and Chordata
(a) Genital warts (b) Genital herpes
(d) Annelida, Arthropoda and Chordata
(c) Chlamydiasis (d) Gonorrhoea
157. Select the correct group of biocontrol agents.
149. The frequency of recombination between gene pairs on
(a) Trichoderma, Baculovirus, Bacillus thuringiensis
the same chromosome as a measure of the distance
(b) Oscillatoria, Rhizobium, Trichoderma
between genes was explained by (c) Nostoc, Azospirillum, Nucleopolyhedrovirus
(a) Gregor J Mendel (b) Alfred Sturtevant (d) Bacillus thuringiensis, Tobacco mosaic virus, Aphids
(c) Sutton-Boveri (d) TH Morgan
158. Identify the cells whose secretion protects the lining of
150. From evolutionary point of view, retention of the
gastrointestinal tract from various enzymes.
female gametophyte with developing young embryo on
the parent sporophyte for some time, is first observed in (a) Goblet cells (b) Oxyntic cells
(c) Duodenal cells (d) Chief cells
(a) mosses (b) pteridophytes
(c) gymnosperms (d) liverworts 159. Which one of the following equipments is essentially
151. What is the direction of movement of sugars in phloem? required for growing microbes on a large scale, for
industrial production of enzymes?
(a) Upward (b) Downward
(c) Bidirectional (d) Non-multidirectional (a) Sludge digester (b) Industrial oven
(c) Bioreactor (d) BOD incubator
152. The Earth Summit held in Rio de Janeiro in 1992 was
called 160. Which of the following is true for Golden rice?
(a) It is pest resistant, with a gene from Bacillus thuringiensis
(a) for conservation of biodiversity and sustainable utilisation of
its benefits (b) It is drought tolerant, developed using Agrobacterium vector
(b) to assess threat posed to native species by invasive weed (c) It has yellow grains, because of a gene introduced from a
species primitive variety of rice
(c) for immediate steps to discontinue the use of CFCs that were (d) It is vitamin-A enriched, with a gene from daffodil
damaging the ozone layer
161. Which of the following statements is correct?
(d) to reduce CO 2 emissions and global warming
(a) Cornea consists of dense connective tissue of elastin and
153. Use of an artificial kidney during hemodialysis may can repair itself
result in : (b) Cornea is convex, transparent layer which is highly
vascularised
A. Nitrogenous waste build-up in the body
(c) Cornea consists of dense matrix of collagen and is the most
B. Non-elimination of excess potassium ions
sensitive portion of the eye
C. Reduced absorption of calcium ions from gastrointestinal
(d) Cornea is an external, transparent and protective
tract
proteinaceous covering of the eyeball
D. Reduced RBC production
Which of the following options is the most appropriate? 162. Respiratory Quotient (RQ) value of tripalmitin is
(a) (B) and (C) are correct (b) (C) and (D) are correct (a) 0.7 (b) 0.07 (c) 0.09 (d) 0.9
(c) (A) and (D) are correct (d) (A) and (B) are correct
163. Which of these following methods is the most suitable
154. Conversion of glucose to glucose-6-phosphate, the first for disposal of nuclear waste?
irreversible reaction of glycolysis, is catalysed by
(a) Bury the waste under Antarctic ice-cover
(a) hexokinase (b) enolase (b) Dump the waste within rocks under deep ocean
(c) phosphofructokinase (d) aldolase (c) Bury the waste within rocks deep below the Earth’s surface
155. Which one of the following statements regarding (d) Shoot the waste into space
post-fertilisation development in flowering plants is
incorrect? 164. Match the following columns.
(a) Zygote develops into embryo Column I Column II
(b) Central cell develops into endosperm
(c) Ovules develop into embryo sac A. P - wave (i) Depolarisation of ventricles
(d) Ovary develops into fruit B. QRS complex (ii) Repolarisation of ventricles
156. Consider the following features. C. T-wave (iii) Coronary ischemia
A. Organ system level of organisation D. Reduction in the Size of (iv) Depolarisation of atria
B. Bilateral symmetry T-wave
C. True coelomates with segmentation of body
Select the correct option of animal groups which possess (v) Repolarisation of atria
all the above characteristics.
www.jeebooks.in
12 NEET Test Drive

Select the correct option. (b) Seminiferous tubules ® Vasa efferentia ® Epididymis ®
A B C D A B C D Inguinal canal ® Urethra
(a) (iv) (i) (ii) (v) (b) (ii) (i) (v) (iii) (c) Testis ® Epididymis ® Vasa efferentia ® Vas deferens ®
(c) (ii) (iii) (v) (iv) (d) (iv) (i) (ii) (iii) Ejaculatory duct ®Inguinal canal ®Urethra ®Urethral meatus
(d) Testis ® Epididymis ® Vasa efferentia ® Rete testis ®
165. Tidal Volume and Expiratory Reserve Volume of an Inguinal canal ® Urethra
athlete is 500 mL and 1000 mL, respectively. What will
be his Expiratory Capacity if the Residual Volume is 173. The concept of ‘Omnis cellula-e-cellula’ regarding cell
1200 mL? division was first proposed by
(a) 1700 mL (b) 2200 mL (c) 2700 mL (d) 1500 mL (a) Theodor Schwann
(b) Schleiden
166. Which of the following is the most important cause for (c) Aristotle
animals and plants being driven to extinction? (d) Rudolf Virchow
(a) Drought and floods
(b) Economic exploitation
174. Drug called ‘Heroin’ is synthesised by
(c) Alien species invasion (a) acetylation of morphine
(d) Habitat loss and fragmentation (b) glycosylation of morphine
(c) nitration of morphine
167. Select the correctly written scientific name of Mango (d) methylation of morphine
which was first described by Carolus Linnaeus.
(a) Mangifera indica Linn. (b) Mangifera indica 175. Persistent nucellus in the seed is known as
(c) Mangifera Indica (d) Mangifera indica Car. Linn. (a) perisperm (b) hilum
(c) tegmen (d) chalaza
168. Following statements describe the characteristics of the
enzyme Restriction Endonuclease. 176. Due to increasing air-borne allergens and pollutants,
Identify the incorrect statement. many people in urban areas are suffering from
(a) The enzyme binds DNA at specific sites and cuts only one of respiratory disorder causing wheezing due to
the two strands (a) inflammation of bronchi and bronchioles
(b) The enzyme cuts the sugar-phosphate backbone at specific (b) proliferation of fibrous tissues and damage of the alveolar
sites on each strand walls
(c) The enzyme recognises a specific palindromic nucleotide (c) reduction in the secretion of surfactants by pneumocytes
sequence in the DNA (d) benign growth on mucous lining of nasal cavity
(d) The enzyme cuts DNA molecule at identified position within
the DNA 177. DNA precipitation out of a mixture of biomolecules can
be achieved by treatment with
169. What is the genetic disorder in which an individual has
(a) chilled ethanol
an overall masculine development gynaecomastia and
(b) methanol at room temperature
is sterile ?
(c) chilled chloroform
(a) Klinefelter’s syndrome (b) Edward syndrome (d) isopropanol
(c) Down’s syndrome (d) Turner’s syndrome
178. Select the hormone- releasing Intra-Uterine Devices.
170. Which of the following pairs of organelles does not
(a) Multiload 375, Progestasert
contain DNA?
(b) Progestasert, LNG-20
(a) Chloroplast and Vacuoles (c) Lippes Loop, Multiload 375
(b) Lysosomes and Vacuoles (d) Vaults, LNG-20
(c) Nuclear envelope and Mitochondria
(d) Mitochondria and Lysosomes 179. Which of the following features of genetic code does
171. How does steroid hormone influence the cellular allow bacteria to produce human insulin by
activities? recombinant DNA technology?
(a) Binding to DNA and forming a gene-hormone complex (a) Genetic code is redundant
(b) Activating cyclic AMP located on the cell membrane (b) Genetic code is nearly universal
(c) Using aquaporin channels as second messenger (c) Genetic code is specific
(d) Genetic code is not ambiguous
(d) Changing the permeability of the cell membrane
172. Select the correct sequence for transport of sperm cells 180. Which part of the brain is responsible for
in male reproductive system. thermoregulation?
(a) Seminiferous tubules ® Rete testis ® Vasa efferentia ® (a) Hypothalamus (b) Corpus callosum
Epididymis ® Vas deferens ® Ejaculatory duct ® Urethra ® (c) Medulla oblongata (d) Cerebrum
Urethral meatus
www.jeebooks.in
NEET Solved Paper 2019 13

Answers
1. (a) 2. (c) 3. (a) 4. (a) 5. (d) 6. (b) 7. (a) 8. (b) 9. (b) 10. (c)
11. (b) 12. (b) 13. (c) 14. (c) 15. (a) 16. (a) 17. (b) 18. (b) 19. (a) 20. (b)
21. (d) 22. (b) 23. (c) 24. (a) 25. (b) 26. (a) 27. (b) 28. (d) 29. (b) 30. (d)
31. (c) 32. (c) 33. (d) 34. (a) 35. (a) 36. (c) 37. (c) 38. (d) 39. (b) 40. (b)
41. (b) 42. (a) 43. (c) 44. (d) 45. (a) 46. (b) 47. (b) 48. (b) 49. (d) 50. (b)
51. (b) 52. (b) 53. (a) 54. (d) 55. (c) 56. (c) 57. (b) 58. (c) 59. (a) 60. (d)
61. (b) 62. (d) 63. (a) 64. (d) 65. (a) 66. (a) 67. (a) 68. (a) 69. (d) 70. (b)
71. (c) 72. (b) 73. (c) 74. (c) 75. (c) 76. (d) 77. (d) 78. (b) 79. (c) 80. (d)
81. (d) 82. (a) 83. (d) 84. (b) 85. (a) 86. (a) 87. (a) 88. (b) 89. (d) 90. (a)
91. (d) 92. (c) 93. (a) 94. (b) 95. (b) 96. (c) 97. (c) 98. (a) 99. (c) 100. (a)
101. (c) 102. (c) 103. (b) 104. (c) 105. (b) 106. (b) 107. (c) 108. (b) 109. (a) 110. (b)
111. (d) 112. (a) 113. (d) 114. (b) 115. (b) 116. (c) 117. (b) 118. (b) 119. (d) 120. (c)
121. (c) 122. (c) 123. (a) 124. (c) 125. (a) 126. (a) 127. (c) 128. (a) 129. (c) 130. (c)
131. (c) 132. (a) 133. (c) 134. (b) 135. (d) 136. (c) 137. (b) 138. (b) 139. (a) 140. (b)
141. (d) 142. (c) 143. (a) 144. (c) 145. (d) 146. (a) 147. (d) 148. (b) 149. (b) 150. (b)
151. (c) 152. (a) 153. (b) 154. (a) 155. (c) 156. (d) 157. (a) 158. (a) 159. (c) 160. (d)
161. (c) 162. (a) 163. (c) 164. (d) 165. (d) 166. (d) 167. (a) 168. (a) 169. (a) 170. (b)
171. (a) 172. (a) 173. (d) 174. (a) 175. (a) 176. (a) 177. (a) 178. (b) 179. (b) 180. (a)

Answer with Explanations


Physics
mg¢ = 200 ´ æç1 -
1. (a) p-type semiconductors are obtained when a trivalent R / 2ö æ 1ö 1
÷ = 200 ç1 - ÷ = 200 ´ = 100
è R ø è 2ø 2
impurity (e.g. boron, aluminium, gallium or indium) is added
to a intrinsic semiconductor. (e.g. germanium or silicon). \ The body will weigh 100 N half way down to the centre of
In other words, the dopants in p-type semiconductor is the earth.
trivalent atom.
3. (a) In an adiabatic process, the system is completely insulated
Thus, this addition creates deficiencies of valence electron from the surroundings. Thus, heat is neither absorbed nor
which are most commonly known as holes. These are the released by the system to the surroundings. So, DQ = 0.
majority charge carriers in this type of semiconductor. However,
Sudden processes are adiabatic like bursting of cycle tyre, etc.
in n-type semiconductors, the dopants are pentavalent
impurities. Also, the majority charge carriers are electrons. If the pressure of gas is kept constant, then the process is
called isobaric, i.e. Dp = 0.
2. (c) Given, weight of the body, w = 200 N If the temperature of the system remains constant, then it is
As we know, w = mg, where m is the mass of the body and g called isothermal process, i.e. DT = 0.
(» 10 m/s 2 ) is acceleration due to gravity of the body at the If the volume of gas is constant in a system, then it is called
surface of the earth. isochoric process, i.e. DV = 0.
Since, mass m remains constant irrespective of the position of
the body on the earth. However, g is not constant and its
4. (a) Case I When all bulbs are glowing, then the circuit can
be realised as shown in the figure below.
value at a depth d below the earth’s surface is given as
g¢ = gæç1 - ö÷
d A B
… (i)
è Rø R R
where, R is the radius of the earth.
R R
Multiplying m on the both sides of Eq. (i), we get
R R
mg¢ = mgæç1 - ö÷
d
è Rø

Thus, the weight of the body at half way down æç i.e. d = ö÷ to


R
è 2ø E
the centre of the earth is \ The equivalent resistance of this circuit is
Req = R A + RB
www.jeebooks.in
14 NEET Test Drive

As, section A has three parallel resistance, so equivalent As the velocity of liquid flow is given as v = 2gh
resistance,
∴R = av = a 2gh
R
RA = Substituting the given values, we get
3
Similarly, for section B, equivalent resistance, R = 2 × 10−6 × 2 × 10 × 2

RB =
R = 2 × 10−6 × 6.32 = 12.64 × 10−6 m 3/s ~
− 12.6 × 10−6 m 3/s
3
R R 2R 6. (b) Work done by a force F, which is variable in nature in
∴ Req = + = … (i) moving a particle from y1 to y2 is given by
3 3 3
y2
Thus, power consumed in this circuit,
V 2 E 2 3E 2
W= ∫ F ⋅ dy … (i)
P1 = = = [using Eq. (i)] …(ii) y1
R Req 2R Here, force, F = 20 + 10 y, y1 = 0 and y2 = 1 m
Case II When two from section A and one from section B Substituting the given values in Eq. (i), we get
glow, the circuit can be realised as shown in the figure below. 1 1
 10 y 2 
⇒ W = ∫ (20 + 10 y)dy =  20 y + 
A B
0  2 0
R
R = 20 (1 − 0) + 5(1 − 0)2 = 25 J
R
∴Work done will be 25 J.
7. (a) The force between two point charges A and B having
charge + Q and − Q respectively is given by
+Q –Q
E A B
∴Equivalent resistance of section A, r
R
RA = K Q AQB KQ(− Q) KQ2
2 F= 2
= 2
=− 2 …(i)
r r r
and of section B, RB = R 1
Thus, equivalent resistance of the entire circuit becomes where, K = constant =
4 πε0
R 3R
Req = R A + RB = + R = … (iii) and r = distance between two charges A and B.
2 2
When 25% charge of A is transferred to B, then new amount
∴Power consumed by this circuit,
of charge on A and B respectively become
V 2 E 2 2E 2 75 3
P2 = = = [using Eq. (iii)]… (iv) Q′ A = (Q A) = Q
R Req 3R 100 4
−3
Q′B =  Q + QB  =  Q − Q =
So, ratio of power of two cases is obtained from Eqs. (ii) and 25 1
Q
(iv), we get  100 A  4  4
P1 3E 2 3R 9 So, the force between the two charges now becomes
= × = or 9 : 4
2R 2E 2 4
K    − Q
P2 3Q 3
K Q′ A Q′B  4   4  −9KQ2 9
5. (d) The rate of liquid flow moving with velocity v through an F′ = = = = F
r2 r2 16r 2 16
area a is given by
[from Eq. (i)]
Rate (R) = Area (a) × Velocity (v)
Thus, the new force between the charges is 9/16 time the
Given, area of hole,
initial force between the charges.
a = 2 mm 2 = 2 × 10−6 m 2
8. (b) The motion of object shot in two cases can be depicted
height of tank, h = 2m. as below
The given situation can also be depicted as shown in the
figure below.

u u
2m 60° 30°

60° g cos 60° 30° g cos 30°


g sin 60° g sin 30°
g g
Case I Case II
a=2mm2
Using third equation of motion,
v2 = u2 − 2gh … (i)
www.jeebooks.in
NEET Solved Paper 2019 15

As the object stops finally, so v = 0 This means the values δ is positive in northern hemisphere
For inclined motion, and is negative in southern hemisphere.
g = gsinθ and h = x ∴For point A, δ = + 25°,
Substituting these values in Eq. (i), we get So, A lies in the northern hemisphere.
u2 Similarly, for B, δ = − 25°, so
⇒ u2 = 2gsinθ x ⇒ x =
2gsinθ B lies in the southern hemisphere.
u2 13. (c) Electric heaters are not based on the eddy current effect.
For case (I), x1 =
2gsin 60° Rather their working is based on Joule’s heating effect of
u2 current. According to this effect, the passage of an electric
For case (II), x 2 = current through a resistor produces heat.
2gsin 30°
However, when a changing magnetic flux is applied to a bulk
x1 u2 2gsin 30° 1 2 1 piece of conducting material, then circulating current is
⇒ = × = × = or 1 : 3
x 2 2gsin 60° u2 2 3 3 induced in the body of this conductor, which is usually
known as eddy currents. This current shows both heating
9. (b) The necessary conditions for the rainbow to take place is and magnetic effects.
(i) Sun should be shining in part of the sky while it is Thus, it is the basic principle behind the working of magnetic
raining in opposite part of the sky. braking in train, electromagnet and induction furnace.
(ii) The observer must stand with his back towards the sun.
∴Statement in option (b) is wrong. 14. (c) The angular speed of a particle in a uniform circular
motion is given by
However, rest statements regarding the rainbow are correct. angle of circle
ω=
10. (c) The average velocity of a particle executing simple Time

harmonic motion (SHM) is ω= , where T is the time period of rotation
T
Total displacement x f − x i 2π
vav = = For particle A, ω A =
Time interval T TA
where, x f and x i are the initial and final position of the 2π
For particle B,ωB =
particle executing SHM. TB
As, in vibrational motion, the particle executes SHM about its ω A 2π TB T 1
mean position. So, after one complete vibration of the ∴ = × = B = or 1 : 1 [Q TA = TB (given)]
ωB TA 2π TA 1
particle, it will reaches its initial position, i.e.
Displacement, x f − x i = 0 15. (a) Key Idea When the space between two convex lenses is
0 filled with a liquid of refractive index same as that of glass of
∴ vav = lens, then it behaves like a diverging lens, i.e. it forms a
T
bi-concave lens.
Hence, the average velocity is zero.
11. (b) According to Bohr’s model, the kinetic energy of electron Case I When two equi-convex lens of focal length f1 and f2
in term of Rydberg constant R is given by respectively, are kept co-axially in contact, then the equivalent
Rhc focal length of combination is
KE = 2 … (i)
n Glass Glass
where, h = Planck’s constant, c = speed of light
and n = principal quantum number.
Similarly, potential energy is given by,
2Rhc Air
PE = − 2 … (ii)
n 1 1 1 1 1 2
= + = + = [QHere, f1 = f2]
Rhc F1 f1 f2 f f f
∴ Total energy, E = PE + KE = − 2 [from Eqs. (i) and (ii)]
n f
⇒ F1 = … (i)
⇒ KE = − E and PE = 2E 2
Given, E = − 34
. eV Case II When glycerine of same
Glass
∴ KE = − (− 34
. ) = 34
. e V and PE = 2 (− 34 . ) = − 6.8eV refractive index at that of the glass Glass

12. (b) The angle of dip (δ) is the angle is filled in the space between two
between the horizontal component BV lens, then the combination will now
of earth’s magnetic field and the comprises of three lenses; first
total magnetic field of the earth. –ve bi-convex, second bi-concave and Glycerine (µ=1.5)
Its value is different at different third is bi-convex. So, the focal
places. It is zero at equator, as the δ length of the combination now is given as
BH
dip neddle becomes parallel to δ 1 1 1 1 1
= + + = ⇒ F2 = f … (ii)
horizontal component. It varies F2 f (− f) f f
from − 90° in South pole to + 90° +ve
From Eqs. (i) and (ii), we get
in the North pole.
F1 : F2 = 1 : 2
www.jeebooks.in
16 NEET Test Drive

16. (a) Given, potential difference, V = 10000 V 19. (a) The displacement of given particle is
If electron is accelerated through a potential of V volt, then y = A0 + Asinωt + B cosωt … (i)
the wavelength associated with it is given by
The general equation of SHM can be given as
h
λ= … (i) x = a sinωt + b cosωt … (ii)
2eVme
So, from Eqs. (i) and (ii), we can say that A0 be the value of
where, h = Planck’s constant = 6.63 × 10−34 J-s,
mean position, at which y = 0.
e = electronic charge = 1.6 × 10−19C
∴Amplitude, R = A2 + B2 + 2 AB cosθ
and me = mass of electron = 9 × 10−31 kg
Substituting these values in Eq. (i), we get As two function sine and cosine have phase shift to 90°.
12.27 12.27 12.27 × 10−10 ∴ R= A 2 + B2 [Q cos 90° = 0]
λ= Å= × 10−10 = = 12.27 × 10−12 m
V 10000 100 20. (b) Key Idea Amount of work done in moving the given body
17. (b) The LED will glow when the current flows through it, i.e. from one point to another against the gravitational force is
when the voltage across it is high. The truth table can be equal to the change in potential energy of the body.
formed from this
As we know, the potential energy of body of mass m of the
+6V
surface of earth is
GMm
R U1 = − … (i)
0 R
where, G = gravitational constant,
A 1
Y M = mass of earth and R = radius of earth.
When the mass is raised to a height h from the surface of the
R earth, then the potential energy of the body becomes

0
h=R
B 1

A B Y
0 0 1 R

0 1 1 M
1 0 1
1 1 0
The output Y is same as that come from NAND gate.
GMm
U2 = −
18. (b) The excess pressure inside a soap bubble of radius r is (R + h)
given by Here, h = R (given)
4T ⇒ U2 = −
GMm
p= … (ii)
r 2R
where, T = surface tension. Thus, the change in potential energy, ∆U = U 2 − U1
If p0 be the outside pressure from the water, then total Substituting the values from Eqs. (i) and (ii), we get
pressure inside the bubble becomes GMm GMm GMm gR2m Q g = GM 
∆U = − + = =
p1 = p0 +
4T
… (i) 2R R 2R 2R  R2 
r mgR
=
The pressure at the depth Z0 below the water surface is 2
p2 = p0 + Z0ρg … (ii) Thus, the work done in raising the mass to a height R is
mgR
As it is given that the pressure inside the bubble is same as equals to .
the pressure at depth Z0 , then equating 2
Eqs. (i) and (ii), we get 21. (d) Given, speed of river, vR = 10 m/s
4T 4T
p0 + = p0 + Z0ρg ⇒ Z0 = … (iii) speed of swimmer in still water and vSN = 20 m/s.
r rρg
vR
Here, T = 2.5 × 10−2 N/m, ρ = 103 kg/m 3, River flow
−2 −3
g = 10 ms and r = 1 mm = 1 × 10 m N
Substituting these values in Eq. (iii), we get vSN vS
W E
4 × 2.5 × 10−2
Z0 = = 10 × 10−3 m = 1 cm θ
1 × 10−3 × 103 × 10 S
www.jeebooks.in
NEET Solved Paper 2019 17

For the shortest path to cross the river, he should swim at an ε0 A


As the capacitance, C =
angle (90° + θ) with the stream flow. From the figure above, d
v SN = v R + v S dV
So, angle θ is given by ∴ Id = C
dt
v 10 1
sinθ = R = = ⇒ θ = 30° Substituting the given values, we get
v SN 20 2
I d = 20 × 10−6 × 3 = 60 × 10−6 A = 60 µ A
As the river is flowing in East direction, so he should swim
towards West. As displacement current is in between the plates of capacitor
and conduction current is in the connecting wires which are
22. (b) Consider two infinite line charges with linear charge equal to each other. So,
densities + λ C/m and − λ C/m respectively, which are lying I c = I d = 60 µA
in y-direction as shown in the figure below.
+λ –λ 25. (b) In stretching a wire, the work
+ EB – done against internal restoring force
L L
+ EA – is stored as elastic potential energy in
+ – wire and given by
+ –
R 1
+ – U = W = × Force (F) × Elongation () l
+ – 2 l
+ 2R – 1 1
A B = Fl = × Mg × l M
2 2
Then, the electric field due to line A at the mid way between
1
the two line charges, i.e. at R is = Mgl Mg
λ 2
|E A| = N/C … (i)
2πε0 R 26. (a) As the hollow sphere is uniformly charged, so the net
which lies along +ve x-axis (outward), i.e. from A to B. charge will appear on the surface of the sphere.
Similarly, the electric field due to line B at the mid-way (i) The electric field at a point outside the hollow sphere is
between the two line charges, i.e. at R is Q
φ = ∫ E ⋅ dS = enclosed [from Gauss’ law]
λ ε0
|EB| = N/C … (ii) S
2πε0 R
E
Due to negative charge on B, EB also lies along
+ve x-axis (inward), i.e. from A to B.
dS
So, the resultant electric field at R is given as ++ +
+
|ER| = |E A| + |EB| + +
Substituting the values from Eqs. (i) and (ii), we get + +
λ λ λ R +
|ER|= + = N/C + O
r
2πε0 R 2πε0 R πε0 R + +
Which also lies along the + ve x-axis, i.e. from A to B. + Q
+ + +
23. (c) Among given devices fuse is used in electric circuit as a
protective device. Q Q 1
⇒ E (4 πr 2) = ⇒ E= ⇒ E∝
It helps in preventing excessive amount of current to flow in ε0 4 πε0 r 2 r2
the circuit or from short circuiting.
It has low melting point and low resistivity, so when excess
(ii) The electric field at the surface (r = R),
amount of current flow in the circuit, it melts and break the E=
Q
circuit. 4 πε0 R
24. (a) Key Idea The displacement current is precisely equals to (iii) The electric field inside hollow sphere is
the conduction current, when the two are present in different Q
parts of the circuit. E= =0 [Q Qinside = 0]
4 πε0 r
dV
Given, C = 20µF = 20 × 10−6 F and = 3V/s Thus, the electric field is zero inside the hollow sphere
dt maximum at the surface and decreases as r increases for r > R.
The displacement current in a circuit is given by This can be shown graphically as
dφ E
I d = ε0 [from Maxwell’s equation)
dt
Emax
d
= ε0 (EA) [Q φ = EA]
dt
E ∝ 12
= ε0 A  
d V E=0 r
[Q V = Ed]
dt  d 
ε A dV r
= 0 O R
d dt
www.jeebooks.in
18 NEET Test Drive

27. (b) Let the mass m which is attached to a thin wire and is As, momentum, p = mvp
whirled in a vertical circle is shown in the figure below. p
∴ r= [from Eq. (i)]
C qB
According to the question, moment of both particle is same.
1
⇒ r∝
q
D B
T For ionised hydrogen atom, q H = e
m r q 2e 2
l and for α-particle, q α = 2e ⇒ H = α = = or 2 : 1
P rα qH e 1
θ mg cos θ
A 31. (c) The rate of heat flow through a conductor of length L and
mg
area of cross-section A is given by
The tension in the string at any point P be T. dQ ∆T
= KA J/s or watt
According to Newton’s law of motion, In equilibrium, net force dt L
towards the centre = centripetal force where, K = coefficient of thermal conductivity and
mv2 L dQ
⇒ T − mg cosθ = ∆T = change in temperature ⇒ K =
l A ∆T dt
Here, l = length of wire and v = linear velocity of the particle metre
whirling in a circle. ∴Unit of K = × watt = Wm −1 K −1
(metre)2 × kelvin
mv2
⇒ T = mg cosθ +
l 32. (c) Given, area of coil, A = 0.05 m 2
mv2 magnetic field, B = 5 × 10−5 T and
At A, θ = 0° ⇒ TA = mg +
l
number of turns, N = 800 .
mv2
At B, θ = 90° ⇒ TB = The magnetic flux linked with the coil is
l
mv2 φ = N(B ⋅ A) = N BA cosθ … (i)
At C, θ = 180° ⇒ TC = − mg +
l where, θ is the angle between B and A.
mv2 The emf induced when coil is rotated from θ1 = 0° to θ2 = 90°
At D, θ = 90° ⇒ TD = TB =
l is
So, from the above analysis, it can be concluded that the ∆φ ∆
tension is maximum at A i.e. the lowest point of circle, So e=− =− (NBA cosθ) [using Eq. (i)]
chance of breaking is maximum. ∆t ∆t
NBA
28. (d) Different colours of white light have different =− (cosθ2 − cosθ1) … (ii)
∆t
wavelengths. The descending order of the wavelength of the
Here, ∆t = 01. s
component of white light is
Thus, substituting the given values in Eq. (ii), we get
λ Red > λ Green > λ Blue > λ Violet
800 × 5 × 10−5 × 0.05 × [cos 90° − cos 0° ]
e=−
29. (b) As the three forces are represented by three sides of a 01
.
triangle taken in order, then they will be in equilibrium.
= 2000 × 10−5 = 0.02 V
P
33. (d) Given, radius R = 2 m, mass, m = 100 kg and
vCM (velocity centre of mass) = v = 20 cm/s = 20 × 10−2 m/s.
Then, according to work energy theorem, the work done in
stopping the disc is equal to the change in its kinetic energy,
i.e. W = KE f − KE i
R Q As, the disc stops at the end, so final velocity is zero. Thus,
⇒ Fnet = FPQ + FQR + FRP = 0 KE f = 0
dv Since, the disc is rolling so, its initial kinetic energy would
Fnet = m × a = m =0 have both rotational and translational kinetic energy
dt
dv component.
⇒ = 0 or v = constant 1 1
dt KE i = KE R + KE T = Iω2 + mv2
So, the velocity of particle remain constant. 2 2
1 1 2 2 1 2 Q for disc, I = 1 mR2 
30. (d) The centripetal force required for circular motion is =  mR  ω + mv
2 2  2  2 
provided by magnetic force
mvp2 1 1 1 v2 1
mvp = mR2ω2 + mv2 = mR2 2 + mv2 [Q v = rω]
⇒ = Bqvp ⇒ r = … (i) 4 2 4 R 2
r qB
3 2
where, vp = perpendicular velocity of particle and = mv
q = charge on particle. 4
www.jeebooks.in
NEET Solved Paper 2019 19
3 2
∴ W = |KE i| = mv The velocity after collision is given by
4
 m − m2  2m2u2
Substituting the given values, we get v1 =  1  u1 +
3 3  m1 + m2  m1 + m2
W = × 100 × (20 × 10−2)2 = × 400 × 100 × 10−4 = 3 J
4 4  4m − 2m  2(2m) × 0
= u + [Q u2 = u′ = 0]
2 1/ 2  4m + 2m  (4m + 2m)
34. (a) Given, X = A1 /B3 3 2m 1
C D = u= u
6m 3
The percentage error in X is given by
∆X ∆A  1  ∆B  ∴Net decreases in kinetic energy of A
× 100 = 2  × 100 +   × 100 ∆KE = KE A − KE ′A = 2mu2 − 2mv12 = 2m(u2 − v12)
X  A  2 B 
1 ∆C  Substituting the value of v1 , we get
+   ∆D  × 100 …(i)
 × 100 + 3    u2  16 mu2
3 C   D  ∆KE = 2m  u2 −  =
∆A ∆B  9 9
Here, × 100 = 1%, × 100 = 2%,
A B ∴The fractional decreases in kinetic energy is
∆C ∆D ∆KE 16 mu2 1 8
× 100 = 3%, × 100 = 4% = × =
C D KE A 9 2mu2 9
Substituting these values in Eq. (i), we get
∆X 1 1 36. (c) Let O be the centre of circle, then at t = 0, the
× 100 = 2(1%) + (2%) + (3%) + 3(4%)
X 2 3 displacement y is maximum and have value 3 m.
y
= 2% + 1% + 1% + 12% = 16%
Thus, maximum % error in X is 16%. t=0

35. (a) Key Idea In head-on elastic collision, momentum and t=t
kinetic energy before and after the collision is conserved. ωt
The given situation of collision can be drawn as O x
3m
4m 2m 4m 2m

u u'= 0 v1 v2 T=4s
A B A B As, the general equation of displacement of a particle will be
144442444443 144442444443 in the form
Before collision After collision y = A cosωt
Applying conservation of linear momentum, Here, A = 3 m
2π 2π π
Initial momentum of system = Final momentum of system Then, ω= = = [given, T = 4 s]
T 4 2
⇒ (4m)u + (2m)u′ = (4m)v1 + (2m)v2 π
4mu = 4mv1 + 2mv2 ∴ y = 3cos  t  (in metre)
2 
or 2 u = 2v1 + v2 … (i)
The kinetic energy of A before collision is 37. (c) The total internal reflection Normal
1 is the phenomenon of reflection
KE A = (4m)u2 = 2mu2 back of light in the denser
2
medium when it travel from
Kinetic energy of B before collision, KE B = 0 denser to rarer medium, when
ic
Denser
The kinetic energy of A after collision is the angle of incidence is greater
1 than the critical angle. While the r=90°
KE′A = (4m)v12 = 2 mv12 Rarer
2 critical angle for a pair of given
media in contact is the angle of
Kinetic energy of B after collision,
incidence in denser medium for
1
KE′B = (2m)v22 = mv22 which the angle of refraction in
2 rarer medium is 90°.
As, Initial kinetic energy of the system = Final kinetic energy
of the system 38. (d) Key Idea According to work-energy theorem, the
change in kinetic energy of a particle is the amount
⇒KE ′A + KE ′B = KE ′A + KE ′B of work done on the particle to move, i.e.
2mu2 + 0 = 2mv12 + mv22 W = − ∆KE = KE f − KE i

2mu2 = 2mv12 + mv22 or 2u2 = 2v12 + v22 … (ii) Given, mass of cylinder, m = 2 kg
Solving Eqs. (i) and (ii), we get radius of cylinder, r = 4 cm = 4 × 10 m
−2
1 4
v1 = u and v2 = u 2π π
3 3 rotational velocity, ω = 3 rpm = 3 × = rad/s and θ = 2π
60 10
or the final velocity of A can be directly calculated by using
the formula. revolution = 2π × 2π = 4 π 2 rad.
www.jeebooks.in
20 NEET Test Drive

The work done in rotating an object by an angle θ from rest is (i) The magnetic field at point outside cylinder, i.e. d > R.
given by W = τθ From Ampere’s circuital law, ∫ B ⋅ dl = µ 0 I
As the cylinder is brought to rest, so the work done will be
µ 0I
negative. ⇒ B∫ dl = µ 0 I ⇒ B(2πd) = µ 0 I ⇒ B=
2π d
According to work-energy theorem,
where, µ 0 = permeability of free space.
Work done = Change in rotational kinetic energy
(ii) The magnitude field at surface, i.e. d = R
1 1 1 I(−ω2i )
− τθ = Iω2f − Iωi2 = I(ω2f − ωi2) ⇒ τ = µ I
2 2 2 2θ B= 0
2πR
[Qω f = 0]
(iii) The magnetic field at inside point. The current for a
1  1 2  ωi2  I = MR (for cylinder)
1 2 point inside the cylinder is given by I′ = current per unit
=  mr 
22  θ  2  cross-sectional area of cylinder × cross-section of loop
1 ω2 I Id 2
= mr 2 [Qωi = ω] = 2
πd 2 = 2
4 θ πR R
2
π µ 0 I ′ µ 0 Id 2 µ I
× 2 × (4 × 10−2)2 ×   ×
1 1 ∴ B= = = 0 d
=
4  10  4π2 2πd 2πR2d 2πR2
1 π2 1 So, the variation of magnetic field can be plotted as
= × 2 × 16 × 10−4 × ×
4 100 4 π 2 B
2
= × 10−4 = 2 × 10−6 N-m
100 B∝d
B∝ 1
39. (b) Given, mass of cylinder m = 10 kg, d
radius of cylinder, r = 1 m ω
coefficient of friction, µ = 01.. O d=R
d
The given situation can be as shown
in the figure given alongside. fl 41. (b) For an ideal voltmeter, the resistance is infinite and for an
From the above figure, it can be mrω2 N ideal ammeter, the resistance is zero.
concluded that the block will be 10 V i1
stationary when the limiting friction mg
( fl) is equal to or greater than the
downward force or weight of block, V1 A1
i.e.
fl ≥ mg …(i)
Also, the magnitude of limiting friction between two bodies
Circuit 1
is directly proportional to the normal reaction (N) between 10 V
them, i.e. So, the current in circuit 1 is
fl ∝ N or fl = µ N …(ii) R × i = V or 10 i1 = 10
From Eqs. (i) and (ii), we get 10
⇒ i1 = =1A
µN ≥ mg or µ( mrω2 ) ≥ mg [Q N = mrω2] 10
g ∴ V1 = i1 × R = 1 × 10 = 10 V
⇒ ω≥
rµ Similarly, for circuit 2, the addition of 10 Ω to voltmeter does
not affect the current and hence
Thus, the minimum angular velocity is
10 Ω i2
g 10
ωmin = = = 10 rad/s
rµ 1 × 01.
10 Ω
V2
40. (b) The cylinder can be considered to be made from A2
concentric circles of radius R.
R
Circuit 2
10 V
10
d 10 i 2 = 10 ⇒ i 2 = =1A
10
V2 = i 2R = 1 × 10 = 10 V
∴ V1 = V2 and i1 = i 2
www.jeebooks.in
NEET Solved Paper 2019 21

− d[N 2]
42. (a) As the temperature of gas in the container is increased, the Rate of reaction =
kinetic energy also increases. This is because the average kinetic dt
energy of a gas is given by 1 d[H 2] 1 d[ NH3 ]
f
=− =+
KE = nRT … (i) 3 dt 2 dt
2 Hence, option (b) is correct.
where, f = degree of freedom,
n = number of moles of gas molecules, 47. (b) The amino acids that can be synthesised in our body and
hence are not essentially required in our diet are called
R = universal gas constant, and T = absolute temperature of
non-essential amino acids. e.g. glycine, alanine, serine,
the gas.
proline, cysteine, glutamine, tyrosine, aspartic acid, glutamic
From Eq. (i), acid, asparagine. Rest given options are essential amino acids,
KE ∝ T i.e. those can’t be synthesised in our body hence essentially
option (b) is incorrect as increase in temperature will lead to required in our diet. Their other examples are isoleucine,
increase in pressure as p ∝ T. Other options (c) and (d) are phenylalanine, methionine, tryptophan, threonine, arginine
also incorrect as molecular distance increases while mass and histidine.
remains the same for increase in the temperature. Hence, option (b) is correct.
43. (c) Due to change in temperature, the thermal strain 48. (b) Ideal solutions are those which obey Raoult’s law over all
produced in a rod of length L is given by concentration ranges at a given temperature, e.g.
∆L benzene-toluene, n-hexane-n-heptane, etc.
= α ∆T ⇒ ∆L = L α ∆T
L
For an ideal solution,
where l = original length of rod and α = coefficient of liner
expansion of solid rod ∆Vmix = 0, ∆H mix = 0,
As the change in length (∆l) of the given two rods of copper ∆Gmix < 0, ∆Smix > 0.
and aluminium are independent of temperature change, i.e. Hence, option (b) is correct.
∆T is same for both copper and aluminium.
LCuα Cu = LAlα Al … (i) 49. (d) The structures of manganate and permanganate ions are
as follows:
Here, α Cu = 1.7 × 10−5 K −1 O– O
−5 −1
α Al = 22
. × 10 K and LCu = 88 cm
Mn Mn
Substituting the given values in Eq. (i), we get O O– O O–
L α 1.7 × 10−5 × 88 ~ O O
LAl = Cu Cu = − 68 cm
. × 10−5
Manganate Permanganate
α Al 22 2–
(MnO4 ) (MnO4– )

44. (d) α-particles are doubly ionised helium nucleus (He 2+) The manganate and permanganate ions are tetrahedral and
contain π-bonds of dπ - pπ-type, oxygen does not have any
which are emitted in any radioactive process. So, they have
d-orbital.
two protons, 2 neutrons in its nucleus and no electron.
Hence, the π-bonding takes place by overlap of p-orbital of
45. (a) The angular width in YDSE is given by oxygen and d-orbitals of manganese.
β
θ= 50. (b) Key Idea A buffer solution having pH more than 7 is known
D as basic buffer. It is obtained by mixing weak base and its salt
where, β is the separation between two fringes. D is the with strong acid in a fixed proportion.
distance between the planed the slits and screen. If YDSE
apparatus is immersed in a liquid of refractive index µ, then Let us consider all the options,
the wavelength of light and hence the angular width (a) 100 mL of 01. M CH 3COOH + 100 mL of 0.1M NaOH
decreases µ times. CH 3COOH + NaOH → CH 3COONa + H 2O
β θ Initial 100 mL × 100 mL × 0 mmol
i.e. θ′ = =
µD µ conc. 0.1 M 0.1 M
0.2 = 10 mmol = 10 mmol
Here, µ (for water) = 4 / 3 and θ = 0.2° ⇒θ′ = = 015
. ° Final conc. 0 0 10 mmol
4/ 3
It is not basic buffer because hydrolysis of salt takes
Chemistry place and final solution contains salt of weak acid with
strong base only.
46. (b) Rate of reaction,
Hence, option (a) is incorrect.
Decrease in the concentration of reactants
(b) 100 mL of 01 . M HCl + 200 mL of 0.1 M NH 4 OH
or increase in the concentration of products
= HCl + NH 4 OH → NH 4 Cl + H 2O
Time taken
Initial conc. 100 mL × 200 mL × 0 mmol
For the given reaction, 0.1 M HCl 0.1 M
N 2(g) + 3H 2(g) - 2NH 3(g) = 10 mmol = 20 mmol
Final conc. 0 10 mmol 10 mmol
www.jeebooks.in
22 NEET Test Drive

It is basic buffer because final solution contains weak 54. (d) The reaction in which the same species is simultaneously
base and its salt with strong acid. Hence, option (b) is oxidised and reduced are called disporportionation reactions.
correct.
Let us, consider the given reaction one by one:
(c) 100 mL of 01 . M HCl + 100 mL of 0.1 M NaOH
HCl + NaOH → NaCl + H 2O (i) 2Cu + → Cu 2 + + Cu 0
Initial conc. 100 mL 100 mL × 0 mmol The above reaction is a disproportionation reaction as
× 01
. M 0.1 M Cu(+1) is oxidised to Cu(+2) and reduced to Cu(0).
= 10 mmol = 10 mmol + 6 + 7 + 4
Final conc. 0 0 10 mmol (ii) 3Mn O42 − + 4H + → 2Mn O4− + Mn O2 + 2H 2OThe above
It is a neutral solution. Hence, option (c) is incorrect. reaction is a disproportionation reaction as Mn(+ 6) is
+ 4
(d) 50 mL of 01 . M NaOH + 25mL of 0.1 M CH 3COOH
oxidised to MnO−4 (Mn + 7) and reduced to MnO2 (Mn).
CH 3COOH + NaOH → CH 3COONa + H 2O
+ 7 + 6 + 4
Initial 25 mL 50 mL ∆
0 mmol (iii) 2KMn O4  → K 2 Mn O 4 + Mn O 2 + O2
conc. × 0.1 M × 0.1 M
= 2 .5 mmol = 5 mmol The above reaction is not a disproportionation reaction as
Final conc. 0 2.5 mol 2.5 mmol Mn (+7) is only reduced to K 2MnO4 (Mn + 6 ) and
It is basic solution. Hence, option (d) is incorrect. MnO2 (Mn + 4).
+ 7 + 4
51. (b) According to Haber’s process, (iv) 2Mn O−4 + 3Mn 2 + + 2H 2O → 5Mn O2 + 4H +
N 2(g) + 3H 2(g) - 2NH 3(g) The above reaction is not a disproportionation reaction as Mn
Now, according to above equations (+7) is only reduced to MnO2 (+ 4).
2 moles of ammonia (NH 3) require = 3 moles of H 2 Hence, option (d) is correct.
∴1 mole of NH 3 require = 3 / 2 moles of H 2 55. (c) In CH 3OH and CH 3OCH 3,  CH 3 group shows + I-effect
3 due to which oxygen acquires partial negative charge in both
or, 20 moles of NH 3 require = × 20 moles of H 2
2 the compounds.
= 30 moles of H 2. δ– δδ–
O O
Note Involvement of any limiting reagent is not mentioned in H3C H CH3 CH3
(+I-effect) (+I-effect) (+I-effect)
question.

52. (b) The molecular orbital configuration of C 2 is On the other hand in Ph  OH, Ph shows −I effect due to
2 * 2 2 * 2
which oxygen acquire partial positive charge.
C2(Z = 12) = σ1s , σ 1s , σ2s , σ 2s , π 2px2 = π 2p 2y . δ+
O
Double bond in C2 consists of both π-bonds because of the
Ph H
presence of last (valence) four electrons in two π-molecular
orbitals. Along with, the lone pair of oxygen also show conjugation
The configuration of N 2, Be 2 and O2 are as follows: with benzene ring which further increases the positive charge
N 2(Z = 14) − σ1s 2, σ *1s 2, σ2s 2, σ * 2s 2, π 2px2 = π 2p 2y , σ2pz2 (1 σ on oxygen as:
δ+
and 2π-bonds) O—H O—H
Be 2(Z = 8) − σ1s 2, σ *1s 2, σ2s 2, σ * 2s 2 (σ bonds only) δ– δ–
2 * 2 2 * 2 *
O2(Z = 16) − σ1s , σ 1s , σ2 s , σ 2 s , σ 2pz2, π 2px2 = π 2p 2y ,
π * 2p1x = π * 2p1z (σ, π and 1π * bond) δ–

Hence, option (b) is correct. Thus, further decreasing the process of protonation.
O Therefore, incoming proton will not be able to attack easily on
partial positive ‘O’ atom. Hence, phenol is most difficult to

53. (a) Nylon-2-nylon-6 ( HN  CH 2CONH(CH 2 )5 C  )n is a protonate. Thus, option (c) is correct.
biodegradable polymer. It is an alternating polyamide 56. (c)[SiCl 6]2 − is not stable and does not exist because
copolymer of glycine (H 2 N  CH 2  COOH) and amino caproic
(i) six large chloride ions cannot be accomdated around
acid (H 2 N(CH 2)5 COOH).
Si 4 + due to limitation of its size.
The remaining polymers, i.e. nylon-6,6,
(ii) interaction between lone pair of chloride ion and Si 4 + is
( NH(CH 2)6 NHCO(CH 2) 4 CO) n , not very strong.
nylon-6- ( CO(CH 2)5NH ) n and buna-S
C6H 5 On the other hand, due to presence of d-orbital in Si, Ge and
Sn they form species like [SiF6 ]2 − , [GeCl 6]2 − and [Sn(OH)6 ]2 − .

( CH 2 CH == CH CH 2  CH  ) n are non-biodegradable Hence, option (c) is correct.
polymers.
Hence, option (a) is correct.
www.jeebooks.in
NEET Solved Paper 2019 23

57. (b) The given road map problem is 60. (d) Basic strength of methyl substituted amines in aqueous
H3C solution depends upon the ease of formation of cation by
O3
Alkene (A) C == O + O == CH—CH3 accepting a proton from the acid.
Zn/H2O
H3C Ethanal
H H
Addition Propanone
HCl
reaction + +
R—N +H R—N—H
B
(Major product) H H
The products of first reaction, i.e. ozonolysis can give an idea The basic strength can be decided by both inductive effect
and solvation effect of alkyl group. In aqueous phase, the
about the probable alkene as —O
C— +
substituted NH 4 cations get stabilised not only by electron
group of these arose from the π-bonds of alkene. Thus, most releasing effect of alkyl group (+ I) but also by solvation
probable alkene will be with water molecules. Greater the size of ion, lesser will be
solvation and less stabilised is the ion. The order is as follows:
H3C H H3C H
C == O O ==C C == C Inductive effect (CH 3)3N > (CH 3) 2(NH) > (CH 3)NH 2
H3C CH3 H3C CH3
Alkene
H
+ H 3C H----OH2 H 3C +
+
The reaction taking place will be H3C—N—H----OH2 > N > N—H----OH2
H3C O3 H3 C H 3C H----OH2 H 3C
C==
=CHCH3 C==O+O==CHCH3 H CH3
Zn/H2O
H3C 2-methyl but-2-ene H3 C Ethanal (2°) (3°)
OH2
(A) Propanone
(1°)
Addition
HCl
reaction Strong +I-effect and hydrogen bonding favours higher basic
Cl strength of 2º amine. Thus, the correct order of basic strength
H3C H3 C in aqueous media will be (CH 3) 2 NH > CH 3NH 2 > (CH 3) 3 N
C—CH2—CH3 + CH—CH—CH3
H3C H3 C 61. (b) Compressibility factor (Z) is the factor which decides the
Cl 2-chloro-3-methylbutane extent of deviation of real gases from ideal gases.
2-chloro-2-methylbutane (minor product)
Vreal
(B)
(major product) Z=
Videal
58. (c) The correct match is as follows : Real gases deviate from ideal gas behaviour because for ideal
(A) Pure nitrogen Pure nitrogen can be obtained by gas it is considered that there is no force of attraction
thermal decomposition of sodium or barium azide. between gas molecules.

Ba(N 3 )2  → Ba + 3N 2 Also, for ideal gas, volume of gas molecules is negligible as
compared to volume of gas container.

2NaN 3  → 2Na + 3N 2 Given, Vreal < Videal
(B) Haber’s process On large scale, ammonia is
∴ Z <1
manufactured by Haber’s process.
If Z < 1, the gas is more compressible than expected from
N 2(g) + 3H 2(g) - 2NH 3(g)
ideal behaviour. As a result, attractive forces are present
(C) Contact process Sulphuric acid is manufactured by the between molecules and are dominant.
contact process.
62. (d) Single bond = 1σ, Double bond = 1σ + 1 π, Triple bond
(D) Deacon’s process Chlorine is prepared by Deacon’s = 1σ + 2π.
process.
The structure of pent-2-en-4-yne is
4HCl + O2 CuCl
  2 → 2Cl + 2H O.
2 2
H H
Hence, the correct match is 1σ |1σ 1σ | 1σ 1σ 1σ
(A) → (iv), (B) →(iii), (C) →(ii), (D) →(i) H — C — C == C — C ≡≡C—H
1σ 2π
1σ | 1σ | 1π

Time Saver If someone know correct match of B, i.e. (iii) H H
then according to the options (c) is the only correct answer as ∴The number of sigma (σ) bonds are 10 and pi (π) bonds
this match is given in this option only.
are 3.
59. (a) Hydrogenation of alkynes in the presence of Pd/C, 63. (a) The increasing nuclear charge outweighs the shielding
quinoline proceeds through syn addition of hydrogen and the effect across a period. As a result, the outermost electrons are
alkene thus formed has cis-configuration. held more firmly, hence ionisation enthalpy (IE) increases
H3C CH3 across a period as:
H2, Pd/C

H3C—C=
=C—CH3 C=
=C Li < B < Be < C < O < N < F < Ne
Quinoline
But-2-yne H H Exceptions (i) IE of boron (B) is less than IE of beryllium
cis-2-butene
(syn addition) (Be). It is because presence of fully-filled 2s-orbitals in Be.
www.jeebooks.in
24 NEET Test Drive

(ii) IE of oxygen (O) is less than that of nitrogen (N). It is O


because of the presence of half-filled p-orbitals in N which F F
(C) XeOF4 Xe (Square pyramidal)
provides extra stability.
64. (d) Key Idea Antibiotics which are effective mainly against F F
either gram positive or gram negative bacteria are called
narrow spectrum antibiotics.
(D) XeO3 Xe (Pyramidal)
Penicillin G has a narrow spectrum. Ampicillin and O O
amoxycillin are synthetic modifications of penicillin. O
These have broad spectrum. Also, chloramphenicol is a broad Hence, the correct match is
spectrum antibiotic.
(A) → (ii), (B) → (iii), (C) → (iv), (D) → (i)
Hence, option (d) is correct.
68. (a) The given reaction is of cumene process for phenol
65. (a) Temporary hardness in water is due to presence of production and intermediate (A) is cumene hydroperoxide.
magnesium and calcium hydrogen carbonates. In the process, cumene (isopropylbenzene) is oxidised in the
Temporary hardness in water can be removed by Clark’s presence of air to cumene hydroperoxide. Which is then
method. In this method calculated amount of lime is added converted to phenol and acetone by treating with dilute acid.
to hard water. It precipitates out calcium carbonate and Acetone, a by-product of this reaction is also obtained in
magnesium hydroxide which can be filtered off. large quantities by this method. The reaction takes place as
Ca(HCO3)2 + Ca(OH)2 → 2CaCO3 ↓ + 2H 2O follows:

Mg(HCO3 )2 + 2Ca(OH)2 → 2CaCO3 ↓ + Mg(OH)2 ↓ + 2H 2O CH3


CH3
Besides this, temporary hardness can also be removed by CH H3C—C—OOH OH
boiling. All the other given methods are used to remove CH3
permanent hardness of water. O2 H+
H2O + CH3COCH3
66. (a) Balmer series of transitions in the spectrum of hydrogen Cumene Cumene
Acetone
Phenol
atom fall in visible region. Lyman series fall in ultraviolet hydroperoxide
while paschen, brackett and pfund fall in infrared region. (A)
n=∞
n=6
n=5 69. (d) The correct structure of tribromooctaoxide is
n=4 O
Paschen series (infrared) O O
n=3
Balmer series (visible) O=
=Br—Br—Br=
=O
O O O
n=2
Red
Green
Blue
Violet

In this compound, Br exhibits variable oxidation state. The


oxidation state of Br from left to right are + 6, + 4 and + 6,
respectively.

70. (b) For group 16 elements, the hydrides with high molar
Energy

Lyman series (ultraviolet)


mass (e.g. H 2 Po) are less thermally stable than hydride with
lower molar mass (e.g. H 2O). This is due to the increase in
size of central atom which results to the weakening of M H
bond due to increased bond length.
Thus, the correct order of thermal stability for
H 2E(E = O, S, Se, Te and Po) is as follows:
n=1 H 2Po < H 2Te < H 2Se < H 2S < H 2O
Transitions of the electron in the hydrogen atom
71. (c) The explanation of given statements are :
67. (a) The given xenon compounds with its structures are as (a) Entropy is positive in case of expansion of a gas at
follows: constant temperature. It is because during expansion of
gas volume increases and hence randomness increases.
F F (b) Entropy is positive in case of sublimation of solid to gas
(A) XeF4 Xe (Square planar) as gas is much disordered than a solid.
F F (c) Entropy is negative in case of 2H(g) → H 2(g) as the
number of moles of gaseous reactants are more than
F that of gaseous products.
F F (d) Entropy is positive in case of evaporation of water as gas
(B) XeF6 Xe (Distorted octahedral) is much disordered than a liquid.
F F Hence, option (c) is correct.
F
www.jeebooks.in
NEET Solved Paper 2019 25

72. (b) Anions ( A) form hexagonal close packed (hcp) lattice, so 78. (b) An acid on losing a proton produces a species which has
+
Number of anions (A) = 6 the tendency to accept H .
Number of octahedral voids = Number of atoms in the close It is called conjugate base of that acid.

packed structure = 6. H2O - OH + H + , HF - F − + H+
Acid Conjugate Acid Conjugate
Cations (C) occupy 75% of octahedral voids, so number of base base
75
cations (C) = 6 × = 6 × 3/ 4 = 9 / 2 Water (H 2 O) is amphoteric in nature and thus act both as an
100
∴The formula of compound = C9/ 2 A6 = C9 A12 = C3 A4 acid and base. e.g.
Thus, option (b) is correct. HF + H 2 O - F− + H 3 O+
Acid Base Conjugate Conjugate
base acid
73. (c) Sulphur dioxide (SO2) is not a green house gas. Carbon
dioxide (CO2), methane (CH 4), water vapour, nitrous oxide 79. (c) Malachite is an ore of copper and its composition is
(N 2O), CFCs and ozone (O3) are the green house gases. CuCO3 ⋅ Cu(OH)2 .
These gases are responsible for global warming. Azurite (Cu(OH)2) and copper pyrites (CuFeS2) are also the
ores of copper (Cu).
74. (c) In gaseous and liquid phases, PCl 5 has a trigonal Mgnetite (Fe 3 O4) is an ore of iron (Fe).
bipyramidal structure with sp 3d-hybridisation.
Hence, option (c) is correct.
Axial
Cl
Cl bond 80. (d) For the reaction,

90° 202
Ca(OH)2 (s) - Ca2 + (aq) + 2OH(aq) [where, S = solubility]
120°

Cl——P pm
S 2S

Equatorial 240 Cl K sp = [Ca2 + ] [OH − ]2 = S(2S) 2 … (i)


pm
bond Cl
Given, pH = 9
Due to presence of longer and weaker axial bonds PCl 5 is a We know that, pH + pOH = 14
reactive molecule.
∴ pOH = 14 − 9 = 5
Hence, statement in option (c) is incorrect while the

remaining options contain correct statements. pOH = − log[OH]
75. (c) Be(OH)2 is amphoteric in nature as it reacts with acid and −
5 = − log[OH] or, [OH − ] = 10− 5
alkali both as :
Be(OH)2 + 2HCl → BeCl 2 + 2H 2O From above equation,

Be(OH)2 + 2NaOH → Na2[Be(OH)4 ] [OH] = 2S = 10− 5
This amphoteric nature of Be is due to small size of Be. The 10−5
∴ S=
other hydroxides of alkaline earth metals are basic in nature. 2
76. (d) The order of energy of orbitals can be calculated from On substituting the value of ‘S’ in eqn. (i), we get
3
(n + l) rule. The lower the value of (n + l) for an orbital, lower  10−5 
is its energy. If two orbitals have same (n + l) value, the K sp = 4S 3 = 4   = 0.5 × 10− 15
 2 
orbital with lower value of n has the lower energy.
(i) 6p = 6 + 1 = 7 (ii) 5 f = 5 + 3 = 8 81. (d) All the tetrahalides of group 14 elements are covalent in
(iii) 4d = 4 + 2 = 6 (iv) 5p = 5 + 1 = 6 nature and sp 3-hybridised with tetrahedral geometry.
∴The order of decreasing energy will be Exceptions are SnF4 and PbF4 which are ionic in nature. Thus,
5 f > 6p > 5p > 4d. statement (d) is incorrect while the remaining statements are
correct.
º
77. (d) The relation between E cell and ∆ r Gº is as follows:
∆ r Gº = − nFE cell
º 82. (a) Key Idea The colloidal particles acquire positive or
negative charge by preferential adsorption of positive ions on
For the cell reaction, negative ions.
2Fe 3 + (aq) + 2 I − (aq) → 2Fe 2 + (aq) + I 2(aq)
n= 2 When silver nitrate (AgNO3) solution is added to potassium
−1 iodide (KI) solution, the precipitated silver iodide (AgI)
Given, F = 96500 C mol , E cell
º
= 0.24 V
adsorbs iodide ions (I −) from the dispersion medium and
Now, we know that
negatively charged colloidal solution results. Among the
∆ r Gº = − n FE cell
º
given options, in option (a) and option (b) millimole of KI is
On substituting the given values in above equation we get higher than AgNO3 . Hence, both the mixture will lead to the
∆ r Gº = − 2 mol × 96500 C mol − 1 × 0.24 J mol − 1 formation of negatively charged colloidal[AgI]I − sol.
= − 46320 J mol − 1 = − 46.32 KJ mol − 1 Note The most appropriate option is (a) because millimole of KI
Hence, option (d) is correct. (50 mL × 2 M = 100 mmol ) is maximum in this option.
www.jeebooks.in
26 NEET Test Drive

83. (d) Key Idea The binary liquid mixtures having the same The splitting of the d-orbitals in two sets orbitals in an
composition in liquid and vapour phase and boil at constant octahedral complex, K 4 [Fe(CN)6 ] may be represented as :
temperature are called azeotropic mixtures or azeotropes. dx2–y2 dz2

eg
The solutions that show large negative deviation from
Raoult’s law forms maximum boiling azeotrope. e.g. nitric
dxy dxz dyz
acid and water.
Average energy of the t2g
The remaining option containing different mixtures forms d-orbitals in spherical
minimum boiling azeotrope. crystal field Splitting of d-orbitals
d-orbitals in octahedral
84. (b) We known that, free ion crystal field

2.303 RT
E cell = E cell
º
− log Q Hence, the electronic configuration of the central atom in
nF
K 4 [Fe(CN) 6 ] is t 26g e g0 .
At equilibirum, Q = K eq and E cell = 0
2.303RT 88. (b) Key Idea For first order reaction,
∴ º
E cell = log K eq … (i)
nF 2 .303 a
t= log
Given, º
E cell = 0.59 V K a− x
2 .303RT where, a = initial concentration, a − x = final concentration.
= 0.059 V,
F
Let the initial concentration (a) = 100
T = 298K, n = 1
After time t, final concentration
On substituting the given values in eqn. (i), we get
(a − x) = 100 − 99 = 1
0.059
0.59 = log K eq 2 .303 a
1 We know that, t = log
0.59 K a− x
= 10 ⇒ K eq = 10 = 1 × 10
10 10
∴ log K eq =
0.059 On substituting the given values in above eqn. we get
Thus, option (b) is correct. 2.303 100 2 .303 2.303 4.606
t= log = log102 = × 2log10 =
K 1 K K K
85. (a) Halogenation of benzene is an example of electrophilic Thus, option (b) is correct.
substitution reaction.
89. (d) Key Idea For an isothermal irreversible expansion,
AlCl3
+ Cl2 —Cl + HCl W irrev = − pext (V 2 − V 1)

Chlorobenzene
Given, V1 = 01
. L, V2 = 0.25 L, pext = 2 bar
Reaction given in option (b) is an example of addition
We know that,
reaction, reaction in option (c) is an example of nucleophilic
substitution and option (d) is an example of substitution Wirrev = − pext (V2 − V1)
reaction. On substituting the given values in the above equation,
we get
86. (a) Carboxylic acids react with ammonia to give ammonium
salt which on further heating at high temperature give Wirrev = − 2 bar (0.25 − 01
.) L
amides. Further, on strong heating, ammonia is removed = − 2 × 015
. L bar = − 0.3 L bar
from phthalamide and phthalimide is formed.
= − 0.3 × 100 J [Q1 L bar = 100 J]
The reaction takes place as follows:
– + = − 30 J
COOH COONH4
+ NH3
– +
90. (a) All enzymes that utilise ATP in phosphate transfer
COOH COONH4 require magnesium as the cofactor. The cofactor are required
Phthalic Ammonium by the enzyme for their efficient activity.
acid phthalate

O ∆ –2H2O Biology
C CONH2 91. (d) G0 phase is the stage in which the cells exit the cell cycle.
Strong heating
NH It is the resting or quiescent phase in which the cells do not
– NH3
C CONH2 divide. It is the permanent state for some cells, e.g., neurons.
Phthalamide
O 92. (c) Phloem in gymnosperms lacks both sieve tube and
Phthalimide companion cells.
In gymnosperms, the phloem has albuminous cells and sieve
87. (a) In K 4[Fe(CN)6 ], cells only. On the other hand, the phloem of angiosperms is
Fe 2 + = [Ar]3d 6 4s 0 . composed of four elements namely, sieve tube, companion

CN is a strong field ligand and as it approaches the metal cells, phloem parenchyma and phloem fibers.
ion, the electrons must pair up.
www.jeebooks.in
NEET Solved Paper 2019 27

93. (a) Maintaining hyperosmolarity towards inner medullary Rest statements are incorrect. Correct information about the
interstitium in the kidneys is the factor responsible for the statements is as follows
formation of concentrate urine because it provides Vertebrosternal ribs are first seven pairs of true ribs which
concentration gradient necessary for water reabsorption in are attached dorsally to thoracic vertebrae and ventrally to
renal tubules. the sternum.
Erythropoietin secretion by juxtaglomerular complex is Vertebrochondral ribs (8th, 9th, 10th pairs) are false ribs
related to red blood cell production and hydrostatic pressure which are not attached to sternum directly. They are attached
during glomerular filtration to amount of filtrate formed by to the seventh rib with the help of hyaline cartilage.
kidneys. High levels of antidiuretic hormone and not low Vertebral ribs are the last two pairs of floating ribs which are
levels produce urine that is more concentrated. attached to vertebrae dorsally and are not attached ventrally.

94. (b) (A)–(iii) (B)–(iv), (C)–(i) (D)–(ii) 100. (a) Flaccidity of bulliform cells is the most appropriate
reason for the curling of grass leaves during dry weather.
The brain capacity of Homo habilis or the tool maker was
650-800 cc. Homo neanderthalensis, who existed in the late Bulliform cells are present between the epidermal cells of the
Pleistocene period had the brain size of about 1400 cc. Homo leaf and they help to minimise the water loss due to
erectus were the first to walk upright and stand erect. Their transpiration during water stress period.
brain size was about 900 cc. Homo sapiens or the living 101. (c) The statement that ‘law of segregation does not apply in
modern man has the brain size of about 1350 cc, which is this experiment’ is incorrect because the law of segregation
lesser than their immediate ancestors, Cro-Magnon man. applies universally. The reappearence of parental (red and
white) flowers in the F 2-generation also confirms, that law of
95. (b) (A)–(iii), (B)–(i), (C)–(iv), (D)–(ii) segregation applies in this experiment. Rest statements are
In a Lac operon, i gene is a regulator gene which produces a correct.
repressor that binds to operator gene and stops its
functioning. z, y and a are the three structural genes in the lac 102. (c) Xylem in plants helps in the translocation of water,
operon of E. coli. z gene produces β-galactosidase for mineral salts, some organic nitrogen and hormones from the
hydrolysing galactoside. y gene produces permease for soil to the aerial parts of the plant. This process is known as
allowing the entry of lactose from outside. the ascent of sap and it involves four major forces namely
root pressure, capillarity, transpirational pull and cohesion
a gene produces transacetylase which helps to transfer an and adhesion of water molecule and cell wall.
acetyl group from acetyl Co-A to beta- galactoside.
103. (b) In parietal placentation, the ovules develop on the inner
96. (c) Parthenogenesis is the process by which the female wall of the ovary or in peripheral part. In this type, the ovary
gamete develops into embryo without fertilisation. It is of is compound or syncarpous type which is either unilocular or
two type-haploid and diploid. In former, embryo develops falsely two or more locular, e.g. mustard, Argemone, Fumaris,
from haploid egg and in latter case, a diploid egg develops etc. Axial placentation is when ovules are placed along the
into embryo. central axis of the ovary.
Parthenocarpy is the development of seedless fruits from an Basal placentation is when ovules are placed at the base of
unfertilised egg of the flower. the ovary.
Syngamy is the fusion of male and female gamete during Free-central placentation is when placenta develops at the
sexual reproduction in plants. centre of ovary as prolongation of floral axis. On this axis,
Autogamy is a type of self-pollination in which a flower is the ovules are attached.
pollinated by its own pollen.
104. (c) GLUT IV is an insulin dependent glucose transporter. It
97. (c) The reading frame of given mRNA will not change even helps in the transport of glucose into muscles and adipose
after the deletion of GGU from 7th, 8th and 9th positions. cells under anabolic conditions. It permits the facilitated
However, the amino acid glycine will not be formed at third diffusion of circulating glucose down its concentration
position in this case which is being coded by GGU. In rest of gradient into muscle and fat cells. On the other hand, GLUT
the cases, insertion or deletion of one or two nucleotide bases I, II and III are insulin independent glucose transporters.
would result in the complete alteration in the reading frame
of mRNA.
105. (b) Statement that inbreeding selects harmful recessive
genes that reduce fertility and productivity is incorrect. The
98. (a) Muscular dystrophy is an inherited muscular disorder in correct information regarding the statement is as follows
which the skeletal muscles degenerate progressively. It is Inbreeding does not select harmful recessive genes. It
caused due to the absence of dystrophin protein which helps exposes harmful recessive genes that are eliminated by
to keep muscle cells intact. selection and thus reduces fertility and productivity. This is
called inbreeding depression.
Myasthenia gravis is an autoimmune neuromuscular disease
that causes paralysis of skeletal muscles. Statements in other options are correct.
Botulism is a type of food poisoning caused by bacterium 106. (b) The statement ‘‘in domesticated fowls, sex depends on
Clostridium botulinum. the type of sperm rather than egg’’ is incorrect. The correct
Tetany is an involuntary muscle contraction caused due to form of statement is, in domesticated fowls, sex of progeny
the low level of calcium in body. depends on the type of egg rather than type of sperm. In
fowls, the females are heterogametic and produce two type of
99. (c) The statement that there are seven pairs of eggs containing either (A+Z) or (A+W) chromosomes. The
vertebrosternal, three pairs of vertebrochondral and two pairs males are homogametic and produce only one type of sperm
of vertebral ribs is correct. containing (A+Z) chromosomes. Rest statements are correct.
www.jeebooks.in
28 NEET Test Drive

107. (c) Colostrum, the yellowish fluid, secreted by mother during 114. (b) Concanavalin A is a lectin or a carbohydrate binding
the initial days of lactation is very essential to impart protein. It is a T-cell mitogen that can activate the immune
immunity to the newborn infants because it contains system, recruit lymphocytes and elicit cytokine production. It
immunoglobulin A. The type of immunity provided by can also induce programmed cell death via
colostrum is natural passive immunity. As IgA is secreted in mitochondria-mediated apoptosis.
mother’s milk, it is also called secretory immunoglobulin.
Monocytes are a type of white blood cell having simple oval 115. (b) (A)–(v), (B)–(iv), (C)–(i), (D)–(iii)
nucleus. Macrophages are cells of the immune system. These The deficiency of insulin causes diabetes mellitus which is
cells can engulf bacteria, fungi, viruses and parasites. characterised by high blood sugar levels. Excess production of
thyroid hormone, thyroxine causes goitre, deficiency of
Natural killer cells are lymphocytes and are a component of
corticoids causes Addison’s disease or adrenal insufficiency.
innate immune system.
Hypersecretion of growth hormones causes acromegaly in
108. (b) The frequency of homozygous dominant, heterozygous adults. It is characterised by enlarged hands and feet of
and homozygous recessive individuals would be 0.16 (AA); affected person.
0.48 (Aa); 0.36 (aa). The frequencies are calculated as
follows:
116. (c) Out of the male gametes discharged in the synergid, one
fuses with the egg and other fuses with central cell nuclei.
Frequency of dominant allele (p) = 0.4 (given) The fusion between male gamete and egg is called syngamy
Frequency of recessive allele (q) = 1 − 0.4 = 0.6 or true fertilisation which forms zygote (2)n. The fusion
Frequency of homozygous dominant individuals (AA) between male gamete and central cell nuclei is called triple
= p 2 = (0.4)2 = 0.16 fusion and it results in the formation of a triploid primary
endosperm nucleus (3)n.
Frequency of heterozygous individual (Aa)= 2pq
= 2 (0.4) (0.6) = 0.48 117. (b) (A)–(iii), (B)–(iv), (C)–(i), (D)–(ii)
Frequency of homozygous recessive individual (aa) Crypts of Lieberkuhn are simple, tubular intestinal glands
= q 2 = (0.6)2 = 0.36 which occur throughout the small intestine between the villi.
They secrete digestive enzymes and mucus. Glission’s capsule
109. (a) The statement ‘‘enzymes of electron transport are is the inner thin layer of connective tissue in liver. Islets of
embedded in outer membrane’’ is incorrect. The correct form Langerhans constitute the endocrine part of pancreas which
of statement is secrete hormones.
Enzymes of electron transport are embedded in the inner Brunner’s glands are located in the submucosa of duodenum
membrane of mitochondria. An electron transport chain is a and they open into the crypts of Lieberkuhn.
series of coenzymes and cytochromes that take part in the
118. (b) Botonical garden is not a method of in situ conservation
passage of electrons from a chemical to its ultimate acceptor.
of biodiversity. It is a type of ex situ or off site conservation in
Rest statements are correct.
which rare plants are conserved in places outside their
110. (b) Alkaline pH of gut triggers activation of protoxin to active natural habitat. Rest all are methods of in situ conservation of
Bt toxin of Bacillus thuringiensis in bollworm. The inactive biodiversity.
protoxins contain toxic insecticidal protein crystals.
119. (d) Expressed Sequence Tags (EST) refers to the genes
When the alkaline pH of insect gut solubilises the crystals, expressed as RNA. These are the DNA sequences that are
the activated toxin binds to the epithelial cells of the midgut expressed as mRNA for protein synthesis.
and creates pores. It causes the cell to swell and burst,
eventually causing the death of insect. 120. (c) Pyramid of biomass in sea is generally inverted because
the biomass of a trophic level depends upon reproductive
111. (d) Auxin and ethylene can be applied to artificially induce potential and longevity of its members. In a sea, the biomass
flowering in pineapple plants throughout the year to increase of phytoplanktons is usually lesser than that of zooplanktons
yield. Auxin induces flowering in pineapple and ethylene
while the biomass of carnivores is greater than small
helps to synchronise flower and fruit growth in this plant.
carnivores and zooplanktons. On the other hand, pyramid of
Though in other cases, ethylene causes fading of flowers.
energy is always upright. Pyramid of biomass in terrestrial
112. (a) The given data represents stabilising selection. It ecosystems (forests, grasslands) is also upright.
eliminates individuals from both ends of a phenotypic
distribution and hence maintains the same distribution 121. (c) Greenhouse effect is mainly contributed by carbon
average. In the given situation, most of the newborn of dioxide (60%) and methane (20%) along with nitrous oxide,
average weight 3-3.3 kg survive. Babies having more or less nitrogen dioxide and chlorofluorocarbons. Greenhouse effect
weight had low survival rate. Disruptive selection favours results in the rise in temperature of earth because greenhouse
both extremes of continuous variation. Directional selection gases has the ability to trap the heat of solar radiations.
favours one extreme of continuous variation. Cyclical
selection is regarded as a source of polymorphism. 122. (c) Cell-mediated immune response is responsible for the
rejection of kidney graft. Cell-mediated immune response is
113. (d) Trichoderma can be used as a biocontrol agent in the conferred by sensitised T-lymphocytes and here, antibodies
treatment of plant disease. It is a filamentous soil fungus are not produced. T-cells confer a long term memory and
having mycoparasitic activity. On the other hand, Anabaena they are able to discriminate between self and non-self. These
helps in nitrogen-fixation, Lactobacillus helps in the cells sometimes consider graft as non-self and attack the
production of organic acid, e.g. lactic acid and Chlorella is a same which causes its rejection.
single cell protein which acts as food supplement.
www.jeebooks.in
NEET Solved Paper 2019 29

123. (a) Pinus has an obligate association with mycorrhizae due humans. Lichens represent symbiotic association between
to which the Pinus seeds are unable to germinate and algae and fungi. Mycorrhiza is symbiotic association of fungi
establish in the absence of fungal partner. and plant roots.
Fungus or mycorrhizae help the Pinus roots to absorb water 132. (a) Statins are commercial blood cholesterol lowering agent
and minerals by increasing their surface area. In turn, the as they competitively inhibit the enzymes involved in
fungus derives food from the plant. cholesterol synthesis. They are obtained from a yeast,
Monascus purpureus.
124. (c) The statement “annual rings are not prominent in trees of
temperate region’’ is incorrect. Correct information about the Streptokinase is a thrombolytic agent which is used to treat
statement is as follows: pulmonary embolism and myocardial infarction.
Annual rings are formed due to the seasonal activity of Lipases help to digest fat molecules while cyclosporin A is an
cambium. In the plants of temperate region, cambium is immunosuppressant.
highly active in spring and less active in autumn season.
Hence, prominent rings are formed in these plants having
133. (c) Thiobacillus bacteria help to carry out denitrification
during nitrogen cycle. This bacteria brings about the
light and dark bands of tissue. Rest statements are correct
reduction of nitrate to free nitrogen (N 2 ). Nitrosomonas and
about the formation of annual rings in trees.
Nitrobacter are chemoautotrophs that cause nitrification.
125. (a) Adenine and guanine are the purines which are found
both in DNA and RNA. Cytosine and thymine are the
134. (b) Polyblend has proved to be a good material for the
constructions of roads. It is a fine powder of recycled plastic
pyrimidines which are found in DNA. In case of RNA,
and it is mixed with bitumen to lay roads. The first polyblend
thymine is replaced by uracil.
road was laid in Bangaluru by the effort of Ahmed Khan.
126. (a) Hugo de Vries proposed that the variations caused by
mutation are random and directionless. These are the
135. (d) The correct sequence of organs in the alimentary canal of
cockroach starting from mouth is
sudden, heritable changes in the genetic material and these
variations constitute the raw material for evolution. He also Pharynx → Oesophagus → Crop → Gizzard → Ileum → Colon
proposed that mutations play a key role in speciation and → Rectum.
used the term saltation for single step large mutations. Both crop and gizzard are the structures of foregut. The
former serves as the food reservoir while the latter helps to
127. (c) The statement “lysosomes are formed by the process of masticate the food due to the presence of six chitinous teeth
packaging in the endoplasmic reticulum” is incorrect. The in it.
correct form of the statement is ‘lysosomes are actually
formed by the budding off from the trans-face of Golgi bodies. 136. (c) Typhoid fever is caused by the bacterium Salmonella typhi
These membrane bound structures contain hydrolytic and widal test is the confirmatory test for typhoid, which is
enzymes whose precursors are synthesised by rough based on antigen antibody reaction.
endoplasmic reticulum. Rest statements are correct. Typhoid fever or enteric fever has the incubation period of 1
to 2 weeks and it is usually transmitted through
128. (a) The shorter and longer arms of submetacentric contaminated food and water.
chromosome are designated as p and q arm, respectively.
Here, ‘p’ signifies petite or short. In a submetacentric 137. (b) As per the given data, the heart rate of the person would
chromosome, centromere is located near the centre due to be 100 beats per minute. It can be calculated as follows
which the two arms appear unequal in length. Given, Cardiac output = 5L (5000 mL)
129. (c) In humans, ciliated epithelial cells are present in the Blood volume in ventricles at the end of diastole = 100 mL
bronchioles and Fallopian tube. In bronchioles, these cells Blood volume at the end of ventricular systole = 50 mL
help in the movement of mucus and in Fallopian tube, these So, Stroke volume = 100 − 50 = 50 mL
are required to move the egg towards uterus. Cardiac output = Stroke volume × Heart rate, i.e.
On the other hand, salivary and pancreatic ducts are lined by 5000 mL = 50 mL × Heart rate
simple cuboidal epithelium. Bile duct is lined by simple Therefore, Heart rate = 100 beats/min.
columnar epithelium and Eustachian tube is lined by
columnar epithelium having ciliated cells. 138. (b) The statement “infective constituent in viruses is protein
coat” is incorrect. The correct information about the
130. (c) The statement ‘‘ yeasts have filamentous bodies with long statement is as follows. Viruses infect their host organisms
thread-like hyphae” is incorrect. Correct information about through their genetic material, i.e either DNA or RNA and
the statement is as follows. not protein. They take over the biosynthetic machinery of the
Yeast is a unicellular sac fungus which lacks filamentous host cell and produce chemicals required for their own
structures or hyphae. However, they may form short multiplication. Rest statements are correct.
temporary filamentous structure called pseudomycelium.
Rest statements are correct. 139. (a) Statement A is true but B is false. Correct information
about statement B is as follows.
131. (c) (A)–(ii), (B)–(iii) , (C)–(iv) (D)–(i) A complete catalytic, active enzyme with its bound prosthetic
Saprophytes are decomposers which help in the group is called holoenzyme. An apoenzyme is an inactive
decomposition of dead organic material, e.g. Agaricus. enzyme which gets activated by the binding of an organic or
Parasites are the entitites which live on other living plants or inorganic cofactor.
animals and derive nutrition from them, e.g. tapeworm in
www.jeebooks.in
30 NEET Test Drive

140. (b) In the construction of genetic maps, map unit or contraceptives also contain progesterone and estradiol
centimorgan is a unit or distance between genes on preparation. Other contraceptive methods include
chromosomes, representing 1% crossover. Intrauterine Devices (IUDs) which release copper and
i. e. 1 map unit = 1% crossover destroys the sperms. On the other hand, barrier method
represents a physical method of contraception. Also, copper-T
Hence, the genetic distance between genes is based on
average number of cross over frequency between them. (Cu-T) acts by releasing copper and not any hormone.

141. (d) Extrusion of second polar body from egg nucleus occurs 148. (b) Genital herpes is a sexually transmitted disease which is
after the entry of sperm but before fertilisation. The entry of not completely curable. It is caused by type-II herpes simplex
virus. Treatment is available for herpes symptoms but the
sperm into female egg causes the breakdown of Metaphase
virus remains in the body and becomes active again. Genital
Promoting Factor (MPF) and turns on Anaphase Promoting
warts are small bump on the genitals caused by HPV (Human
Factor (APF). Hence, the secondary oocyte completes its Papilloma Virus) Chlamydiasis is caused by Chlamydia
meiotic division after fertilisation and is said to be activated. trachomatis and may not show symptoms.
142. (c) For the induction of flowering in plants, photoperiod Gonorrhoea is caused by Neisseria gonorrhoeae and is referred
stimulus is percieved by the leaves of plants. As a result, as “the clap”.
floral hormones are produced in the leaves which are then
translocated to the apical part and subsequently cause the 149. (b) Alfred Sturtevant was the first to explain the concept of
chromosomal mapping. It is drawn on the basis of
initiation of floral primordial growth.
recombination frequency between gene pairs on the same
143. (a) (A)–(iii), (B)–(iv), (C)–(ii), (D)–(i) chromosome. This frequency is directly proportional to the
Pila or apple snail contains a file-like rasping organ called distance between these two genes.
radula for feeding. It can be used to determine the exact location of a gene on
Bombyx or silkworm is an arthropod in which excretion the chromosome.
occurs through Malpighian tubules. The body of ctenophore 150. (b) From the evolutionary point of view pteridophytes were
Pleurobranchia bears eight rows of ciliated comb plates, which the first to show the retention of female gametophyte with
help in locomotion. young embryo on the parent sporophyte for some time. It
In Taenia, excretion occurs through specialised cells called also represents the origin of seed habit during the course of
flame cells which contain a protonephridia. evolution of seed plants.

144. (c) The correct sequence of phases of cell cycle is 151. (c) Sugars show bidirectional movements in phloem unlike
G1 → S → G2 → M unidirectional movement of water in xylem. In phloem,
sugars move in both directions depending upon the
Here G1 and G2 represent first and second growth phase, source-sink relationship.
respectively. S-phase represents synthesis phase during
Initially leaves act as source of ‘food from where it moves to
which DNA replicates. M-phase is mitotic phase during
the sink (parts requiring food). Later, the food in sink is
which cell begins to divide.
mobilised towards the growing buds of the plant.
145. (d) Montreal Protocol aimed to reduce the emission of 152. (a) The Earth Summit held in Rio de Janeiro in 1992 was
chlorofluorocarbons into atmosphere because it has the
called for the conservation of biodiversity and sustainable
deleterious effects on stratospheric ozone. This protocol was
utilisation of its benefits. In this summit, more than 130
signed in Montreal, Canada in 1987. Kyoto Protocol aimed to
nations signed a convention on Biodiversity and Climate
reduce the emission of CO 2 , NO 2 and methane.
Change. Canada was the key player in the development of
It was signed by 160 countries in a convention held in Kyoto,
this convention.
Japan in 1997. Geneva Protocol is a treaty to prohibit the use
of chemical or biological weapons in international armed 153. (b) Statements (C) and (D) are correct.
conflicts. Use of an artificial kidney during haemodialysis may result in
Gothenberg Protocol is a multipollutant protocol which reduced RBC production and reduced absorption of calcium
focuses to reduce eutrophications, acidification, emission ions from gastrointestinal tract. The former would occur due
standards for SO 2, etc. to the low level of erythropoietin hormone secreted by
juxtaglomerular cells.
146. (a) (A)–(ii), (B)–(iv), (C)–(iii), (D)–(v) The later would be caused due to the elimination of calcium
Lactobacillus bacteria helps in the production of curd. The ions along with phosphate ions during dialysis.
yeast, Saccharomyces cerevisiae helps in bread making. The Statements (A) and (B) are incorrect because dialysis helps
fungus, Aspergillus Niger is used for citric acid production. The in the removal of nitrogenous waste and potassium ions from
bacteria Acetobacter aceti is used in the production of acetic the body.
acid.
154. (a) Conversion of glucose to glucose-6-phosphate during
147. (d) Lactational amenorrhea, pills and emergency glycolysis is catalysed by the enzyme hexokinase. During this
contraceptives provide contraception due to the role of step, glucose is phosphorylated to glucose-6-phosphate by
hormones. ATP. It is the first step of activation phase of glycolysis.
In lactational amenorrhoea, high prolactin level during active
lactation period decreases the gonadotropin level in the blood. 155. (c) The statement that “ovules develop into embryo sac’’ is
incorrect. Correct information about the statement is as
Pills usually contain progesterone or progesterogen- oestrogen
follows
combinations which prevent ovulation. Emergency
During post-fertilisation event, ovule develops into seed.
www.jeebooks.in
NEET Solved Paper 2019 31

On the other hand, embryo sac is a multicellular structure 162. (a) The RQ value of tripalmitin is 0.7. It can be calculated as
which is derived from the megaspore. Rest statements follows
regarding post-fertilisation development in flowering plants Amount of CO 2 released
are correct. Respiratory Quotient (RQ) =
Amount of O 2 Consumed
156. (d) All the three animal groups namely Annelida, 2(C51 H 98 O6) + 145O 2 → 102 CO2 + 98 H 2 O
Arthropoda and Chordata possess organ system level of Tripalmitin
organisation, bilateral symmetry and true coelom with
102 CO2
segmented body. Molluscans are also bilaterally symmetrical RQ = = 0.7
and show organ system grade of organisation but they do not 1450 O2
possess segmented body. It is to note that RQ of common fats is usually less than 1
under aerobic conditions.
157. (a) The correct group of biocontrol agents is Trichoderma,
Baculovirus and Bacillus thuringiensis. Baculovirus are pathogens 163. (c) Nuclear waste is usually disposed by burying it within
that attack insects and other arthropods. Most of Baculoviruses rocks deep below the earth’s surface. Nuclear waste disposal
used as biocontrol agent belong to the genus is extremely hazardous. Before burrying the waste, it is
Nucleopolyhedrovirus. sealed in large containers so as to reduce its radiation effects.
Trichoderma is extensively used against pathogenic fungi 164. (d) (A)–(iv), (B)–(i), (C)–(ii), (D)–(iii)
which causes soil borne diseases.
In an Electrocardiograph (ECG), P-wave represents the
Bacillus thuringiensis secretes toxin crystals which kill the depolarisation of atria which is caused by the activation of SA
insect larvae. On the other hand, Rhizobium, Nostoc, node. QRS complex represents depolarisation of ventricles
Azospirillum and Oscillatoria are used as biofertilisers. Tobacco which is caused by the impulse of contraction from AV node.
mosaic virus is a pathogen and aphids are pests that harm
T-wave represents repolarisation of ventricles and reduction
crop plants.
in its size signifies coronary ischemic, i.e. when the heart
158. (a) Secretions of goblet cells protect the lining of muscles receive insufficient oxygen as in arteriosclerotic
gastrointestinal tract from various enzymes. These cells heart disease.
secrete mucus which along with bicarbonate ions helps in the
165. (d) The Expiratory Capacity of athlete will be 1500 mL.
lubrication and protection of the mucosal epithelium from
It can be calculated as
the excoriation by the highly concentrated HCl. On the other
hand, oxyntic or parietal cells secrete hydrochloric acid. Chief Given, Tidal Volume (TV) = 500 mL
cells or peptic cells secrete proenzymes-pepsinogen and Expiratory Reserve Volume (ERV) = 1000 mL
prorenin. Expiratory Capacity = TV+ERV = 500 + 1000 = 1500 mL

159. (c) Bioreactors are required for growing microbes on large 166. (d) Habitat loss and fragmentation is the most important
scale for the industrial production of enzymes. These large cause for animals and plants being driven to extinction. Due
vessels provide biologically active environment. On the other to habitat fragmentation and loss, a 14% of the earth’ land
hand, sludge digesters are used to decompose organic solid surface, rainforest has shrunk to only 6% in last few years.
waste under aerobic conditions. BOD incubators are used to Fragmentation and loss of large habitats due to various
maintain the temperature for tissue culture growth, bacterial human activities badly affects mammals and birds requiring
cultures, etc. large territories. Certain animals with migratory habits are
also affected by habitat loss and fragmentation. Thus, their
160. (d) Statement that Golden rice is vitamin-A enriched, with a populations are driven towards decline and extinction.
gene from daffodil is true. Golden rice is genetically On the other hand, droughts and floods, economic
engineered variety of rice to biosynthesise β-carotene which exploitation and alien species invasion affect only a small
is a precursor of vitamin-A. It contains psy gene (phytoene part of population at a time.
synthase) which is derived from daffodil.
Other statements are not true for golden rice. The correct 167. (a) The correct form of writing the scientific name of mango
information about the statements is as follows The grains of as described by Carolus Linnaeus is Mangifera indica Linn.
golden rice appear yellow due to high level of β-carotene in it. As per binomial nomenclature rules, the name of an
Golden rice is neither drought tolerant nor pest resistant. organism contains a generic and specific name. The former
begins with capital letter while the later begins with small
161. (c) The statement that cornea consists of dense matrix of letter. The name of taxonomist is written in Roman script
collagen and is the most sensitive portion of the eye is and it is written in abbreviated form.
correct.
Rest statements are incorrect. The correct information about
168. (a) The statement about restriction enzymes that the enzyme
binds DNA at specific sites and cuts only one of the two
the statements is as follows
strands is incorrect.
The outer layer of the wall of eyeball, sclera, consists of a
These enzymes cut both the strands of DNA helix at specific
dense connective tissue containing mainly collagen and some
sites in their sugar phosphate backbone. The sequences being
elastic fibre. Cornea is convex, transparent layer which is
recognised by restriction enzymes are called palindromic
non-vascularised. The cornea is the clear part of eye’s
sequences which have same reading frame in both 5’→ 3’ and
protective covering.
3’→ 5’ directions. Rest statements are correct.
www.jeebooks.in
32 NEET Test Drive

169. (a) In Klinefelter’s syndrome, individual has overall masculine 173. (d) Rudolf Virchow proposed the concept of “omnis
development, gynaecomastia and is sterile. This condition is cellula-e-cellula’’, i.e. all cells are derived from the
represented as 44 + X (47) due to the presence of an extra pre-existing cells.
X-chromosome in males. Schleiden and Theodor Schwann jointly put forward the
Edward syndrome is 18 trisomy and it causes severe cell theory in 1839.
developmental delay.
Down’s syndrome is 21-trisomy and it is identified as Mongolism
174. (a) Drug ‘Heroin’ is synthesised by the acetylation of
morphine.
due to the short stature of affected individual.
Chemically heroin is diacetymorphine and commonly it
Turner’s syndrome is characterised by a missing X-chromosome in
is called smack. It is an opium derivative which is used
females. It causes sterility in females.
as medicine. Excessive use of it causes addiction.
170. (b) Lysosomes and vacuoles do not contain DNA. Lysosomes are
single membrane bound small vesicles which contain hydrolytic
175. (a) Persistent nucellus in the seed is known as
perisperm.
enzymes. Vacuoles are a large membranous sac found in the
cytoplasm. These contain substances that are not essentially useful The albuminous seeds usually retain a part of
for the cell like water, sap, excretory products and other materials. endosperm as it is not completely used up during
Chloroplast and mitochondria are semi-autonomous organelles embryo development.
because they contain their own DNA and are believed to be But in some seeds, remnants of nucellus are also
prokaryotic symbionts. persistent, e.g. black pepper and beet.

171. (a) Steroid hormones are able to bind to DNA and form a 176. (a) Wheezing occurs due to the inflammation of
gene-hormone complex. These hormones can easily cross the lipid bronchi and bronchioles. It is one of the most
bilayer of the cell and do not require secondary messengers for the significant feature of asthma in which people face
same. Steroid hormones bind to intracellular receptors in the difficulty in breathing. It is usually caused due to
nucleus to form hormone receptor complex which in turn interact increasing air borne allergens and pollutants. The
with the genome. allergens stimulate the release of histamine from the
cAMP pathway, secondary messengers and change in cell membrane mast cells which in turn contracts the smooth muscles
permeability is required in case of proteinaceous hormones which of bronchioles.
cannot pass through lipid bilayer on their own.
177. (a) Chilled ethanol is used to precipitate DNA out of a
172. (a) The correct sequence of sperm transport in male reproductive mixture of biomolecules. Low temperature protects the
system is seminiferous tubules→Rete testis→Vasa DNA by slowing down the activity of enzymes that
efferentia→Epididymis→Vas deferens→Ejaculatory duct →Urethra could break it apart and ethanol helps in the quick
→Urethral meatus. precipitation of DNA.
The pathway of sperm transport is shown in the diagram below 178. (b) Progestasert and LNG-20 are hormone releasing
Intrauterine Devices (IUDs). These devices release small
quantities of hormone which suppresses endometrial
changes, cause an ovulation and insufficient luteal
activity.

Ejaculatory 179. (b) Bacteria is able to produce human insulin because


duct (6) genetic code is nearly universal in all organisms. For
example, the codon AGG specifies amino acid Arginine
Urethra (7) Vas deferens (5)
in bacteria, animals and plants.
But there are also some exceptions to it, e.g. in
Epididymis (4) mitochondria, stop codon UGA specifies amino acid
Urethral tryptophan.
meatus (8)
180. (a) Hypothalamus is the thermoregulatory centre in the
brain and it maintains the constant body temperature of
37°C. The hypothalamus contains a number of centres,
which control body temperature.
Vasa efferentia (3) Corpus callosum is the thick band of nerve fibres that
divide the cerebrum into left and right hemispheres.
Rete testis (2) Medulla oblongata is the component of hindbrain. It
receives and integrates signals from spinal cord and
sends them to cerebellum.
Seminiferous
Cerebrum is the large part of the brain and consists of
tubules
(1) two hemispheres.

Enlarged view of testis


www.jeebooks.in
NEET Solved Paper 2019 33

NEET (Odisha)
SOLVED PAPER 2019 (Held on : 20 May 2019)

Duration : 3 Hours Max. Mark : 720

Important Instructions
The test is of 3 hours duration and Test Booklet contains 180 questions.
Each question carries 4 marks. For each correct response, the candidate will get 4 marks.
For each incorrect response, one mark will be deducted from the total scores.
The maximum marks are 720.

PHYSICS
1. Two metal spheres, one of radius R and the other of 5. An object flying in air with velocity (20 $i + 25$j − 12k)
$
radius 2R respectively have the same surface charge suddenly breaks in two pieces whose masses are in the
density σ. They are brought in contact and separated. ratio 1 : 5. The smaller mass flies off with a velocity
What will be the new surface charge densities on (100 $i + 35$j + 8 k$ ).
them?
5 5 5 5 The velocity of the larger piece will be
(a) σ1 = σ, σ 2 = σ (b) σ1 = σ, σ 2 = σ
6 2 2 6 (a) 4$i + 23$j − 8k$ (b) − 100$i − 35$j − 8k $
5 5
(c) σ1 = σ, σ 2 = σ
5 5
(d) σ1 = σ, σ 2 = σ (c) 20$i + 15$j − 80k$ (d) − 20 i − 15 j − 80k
$ $ $
2 3 3 6
6. An object kept in a large room having air temperature
2. The distance covered by a particle undergoing SHM in of 25°C takes 12 minutes to cool from 80°C to 70°C.
one time period is (amplitude = A) The time taken to cool for the same object from 70°C to
(a) zero (b) A 60°C would be nearly
(c) 2A (d) 4A (a) 10 min (b) 12 min (c) 20 min (d) 15 min
3. A mass falls from a height ‘h’ and its time of fall ‘t’ is 7. Two small spherical metal balls, having equal masses,
recorded in terms of time period T of a simple are made from materials of densities ρ1 and ρ2 (ρ1 = 8 ρ2)
pendulum. On the surface of earth it is found that t = 2 and have radii of 1 mm and 2 mm, respectively. They
T. The entire set up is taken on the surface of another are made to fall vertically (from rest) in viscous
planet whose mass is half of earth and radius the same. medium whose coefficient of viscosity equals η and
Same experiment is repeated and corresponding times whose density is 0.1 ρ2 . The ratio of their terminal
noted as t′ and T ′. velocities would be
(a) t ′ = 2 T ′ (b) t ′ > 2 T ′ 79 19 39 79
(a) (b) (c) (d)
(c) t ′ < 2T ′ (d) t ′ = 2 T ′ 72 36 72 36
8. A particle starting from rest, moves in a circle of radius
4. A tuning fork with frequency 800 Hz produces
resonance in a resonance column tube with upper end ‘r’. It attains a velocity of v0 m/s in the nth round. Its
open and lower end closed by water surface. Successive angular acceleration will be
resonance are observed at length 9.75 cm, 31.25 cm and v0 v 02
(a) rad/s 2 (b) rad/s 2
52.75 cm. The speed of sound in air is n 2 πnr 2
(a) 500 m/s (b) 156 m/s v 02 v2
(c) rad/s 2 (d) 0 rad/s 2
(c) 344 m/s (d) 172 m/s 4πnr 2 4πnr
www.jeebooks.in
34 NEET Test Drive

9. A person standing on the floor of an elevator drops a 13. An LED is constructed from a p-n junction diode using
coin. The coin reaches the floor in time t1 if the elevator GaAsP. The energy gap is 1.9 eV. The wavelength of the
is at rest and in time t2 if the elevator is moving light emitted will be equal to
(a) 10.4 × 10−26 m (b) 654 nm
uniformly. The which of the following option is
correct? (c) 654 Å (d) 654 × 10−11 m
(a) t 1 < t 2 or t 1 > t 2 depending upon whether the lift is going up
14. The circuit diagram shown here corresponds to the
or down
logic gate,
(b) t 1 < t 2
+6V
(c) t 1 > t 2
R
(d) t 1 = t 2 A 0
1
10. A truck is stationary and has a bob suspended by a B 0
light string, in a frame attached to the truck. The truck, 1
suddenly moves to the right with an acceleration of a. LED (Y)
The pendulum will tilt R
(a) to the left and the angle of inclination of the pendulum with
the vertical is sin−1  
g
 a (a) NOR (b) AND (c) OR (d) NAND
(b) to the left and angle of inclination of the pendulum with the 
Cp 
 a 15. The value of γ  =  , for hydrogen, helium and
vertical is tan−1    C V
g
another ideal diatomic gas X (whose molecules are not
(c) to the left and angle of inclination of the pendulum with the
 a rigid but have an additional vibrational mode), are
vertical is sin1−   respectively equal to
g
7 5 9 5 7 9 5 7 7 7 5 7
(d) to the left and angle of inclination of the pendulum with the (a) , , (b) , , (c) , (d) , ,
5 3 7 3 5 7 3 5 5 5 3 5
vertical is tan−1  
g
 a 16. An equi-convex lens has power P it is cut into two
symmetrical halves by a plane containing the principal
11. In a u-tube as shown in a figure, water and oil are in
axis. The power of one part will be
the left side and right side of the tube respectively. The
P
heights from the bottom for water and oil columns are (a) 0 (b)
15 cm and 20 cm respectively. The density of the oil is 2
P
[take ρwater = 1000 kg/m 3 ] (c) (d) P
4

17. In a Young’s double slit experiment, if there is no


initial phase-difference between the light from the two
slits, a point on the screen corresponding to the fifth
20 cm minimum has path difference.
15 cm λ λ λ λ
(a) 5 (b) 10 (c) 9 (d) 11
2 2 2 2

18. A double convex lens has focal length 25 cm. The


radius of curvature of one of the surfaces is doubled of
Water Oil
3 3
the other. Find the radii, if the refractive index of the
(a) 1200 kg/m (b) 750 kg/m material of the lens is 1.5.
(c) 1000 kg/m 3 (d) 1333 kg/m 3
(a) 100 cm, 50 cm (b) 25 cm, 50 cm
12. A deep rectangular pond of surface area A, containing (c) 18.75 cm, 37.5 cm (d) 50 cm, 100 cm
water (density = ρ, specific heat capacity = s), is located
in a region where the outside air temperature is a 19. Two bullets are fired horizontally and simultaneously
steady value at the − 26 °C. The thickness of the frozen towards each other from roof tops of two buildings 100
ice layer in this pond, at a certain instant is x. m apart and of same height of 200 m with the same
velocity of 25 m/s. When and where will the two
Taking the thermal conductivity of ice as K, and its
specific latent heat of fusion as L, the rate of increase of bullets collides. (g = 10 m/s 2 )
the thickness of ice layer, at this instant would be given (a) After 2s at a height 180 m
by (b) After 2s at a height of 20 m
(a) 26K/ρ r(L−4s) (b) 26K/(ρx2 − L) (c) After 4s at a height of 120 m
(d) They will not collide
(c) 26K/(ρxL) (d) 26K/ρr(L + 4s)
www.jeebooks.in
NEET Solved Paper 2019 35

20. The stress-strain curves are drawn for two different 26. A straight conductor carrying current i splits into two
materials X and Y. It is observed that the ultimate parts as shown in the figure. The radius of the circular
strength point and the fracture point are close to each loop is R. The total magnetic field at the centre P at the
other for material X but are far apart for material Y. We loop is
can say that materials X and Y are likely to be
(respectively)
i1
(a) ductile and brittle (b) brittle and ductile R
(c) brittle and plastic (d) plastic and ductile
i P
21. A body of mass m is kept on a rough horizontal surface 90º
(coefficient of friction = µ). Horizontal force is applied on i2
the body, but it does not move. The resultant of normal
i
reaction and the frictional force acting on the object is
given F, where F is (a) Zero (b) 3 µ 0 i / 32 R, outward
(a) |F| = mg + µ mg (b) |F| = µmg µ i
(c) 3 µ 0 i /32R, inward (d) 0 , inward
(c) |F| ≤ mg 1 + µ 2 (d) |F| = mg 2R

22. A particle of mass 5 m at rest suddenly breaks on its 27. The variation of EMF with time for four types of
own into three fragments. Two fragments of mass m generators are shown in the figures. Which amongst
each move along mutually perpendicular direction with them can be called AC?
each speed v. The energy released during the process is E E
3 5
(a) mv 2 (b) mv 2
5 3
3 4 (a) t (b) t
(c) mv 2 (d) mv 2
2 3

23. An object of mass 500 g, initially at rest acted upon by


E E
a variable force whose X component varies with X in the
manner shown. The velocities of the object a point
X = 8 m and X = 12 m, would be the respective values (c) t (d) t
of (nearly)
F (N)
(a) (a) and (d) (b) (a), (b), (c) and (d)
20
(c) (a) and (b) (d) only (a)

28. The radius of the first permitted Bohr orbit for the
10
electron, in a hydrogen atom equals 0.51 Å and its
x (m ) ground state energy equals − 13.6 eV. If the electron in
4 5 8 10 12 the hydrogen atom is replaced by muon (µ −1) [Charge
–10 same as electron and mass 207 me ], the first Bohr
radius and ground state energy will be
–20 (a) 0.53 × 10−13 m, − 3.6 eV
–25
(b) 25.6 × 10−13 m, − 2.8 eV
(a) 18 m/s and 24.4 m/s (b) 23 m/s and 24.4 m/s (c) 2.56 × 10−13 m, − 2.8 keV
(c) 23 m/s and 20.6 m/s (d) 18 m/s and 20.6 m/s (d) 2.56 × 10−13 m, − 13.6 eV
24. A solid cylinder of mass 2 kg and radius 50 cm rolls up 29. The reading of an ideal voltmeter in the circuit shown
an inclined plane of angle inclination 30°. The centre of is
mass of cylinder has speed of 4 m/s. The distance 20 Ω 30 Ω
travelled by the cylinder on the inclined surface will be
: (Take g = 10 m/s 2 )
V
(a) 2.2 m (b) 1.6 m (c) 1.2 m (d) 2.4 m

25. Two toroids 1 and 2 have total number of turns 200 30 Ω 20 Ω


and 100 respectively with average radii 40 cm and 20
cm respectively. If they carry same current i, the ratio
of the magnetic fields along the two loops is 2V
(a) 0.6 V (b) 0 V (c) 0.5 V (d) 0.4 V
(a) 1 : 1 (b) 4 : 1 (c) 2 : 1 (d) 1 : 2
www.jeebooks.in
36 NEET Test Drive

30. The meter bridge shown in the balance position with 37. A sphere encloses an electric dipole with charge
P l1
= . If we now interchange the positions of ± 3 × 10 −6 C. What is the total electric flux across the
Q l2 sphere?
galvanometer and cell, will the bridge work? If yes, (a) − 3 × 10−6 N-m 2 /C (b) zero
that will be balanced condition? (c) 3 × 106 N-m 2 /C (d) 6 × 10−6 N-m 2 /C

38. Two identical capacitors C1 and C 2 of equal capacitance


P Q are connected as shown in the circuit. Terminals a and
b of the key k are connected to charge capacitor C1
G using battery of emf V volt. Now, disconnecting a and
b the terminals b and c are connected. Due to this,
l1 l2 what will be the percentage loss of energy?
P l2 − l1 k
(a) yes, = (b) no, no null point a c
Q l2 + l1
P l P l1 b
(c) yes, = 2 (d) yes, =
Q l1 Q l2
V C1 C2
31. The relations amongst the three elements earth’s
magnetic field, namely horizontal component H,
vertical component V and dip δ are, (BE = total
magnetic field) (a) 75% (b) 0% (c) 50% (d) 25%
(a) V = BE tan δ, H = BE (b) V = BE sinδ, H = BE cos δ
39. The main scale of a vernier calliper has n divisions/cm.
(c) V = BE cos δ, H = BE sin δ (d) V = BE , H = Be E tan δ n divisions of the vernier scale coincide with (n −1)
32. The rate of radioactive disintegration at an instant for a divisions of main scale. The least count of the vernier
radioactive sample of half life2.2 × 10 9 s is 10 10 s −1 . The callipers is
1 1
number of radioactive atoms in that sample at that (a) cm (b) cm
(n + 1) (n − 1) n
instant is
1 1
. × 1020
(a) 317 . × 1017
(b) 317 (c) 2 cm (d) cm
n n (n + 1)
. × 10
(c) 317 18
. × 1019
(d) 317
40. A person travelling in a straight line moves with a
33. The time period of a geo-stationary satellite is 24 h, at a constant velocity v1 for certain distance ‘x’ and with a
height 6 RE (RE is the radius of earth) from surface of constant velocity v2 for next equal distance. The
earth. The time period of another satellite whose average velocity v is given by the relation
height is 2.5 RE from surface will be 1 1 1 2 1 1
24 12 (a) = + (b) = +
(a) 6 2 h (b) 12 2 h (c) h (d) h v v1 v 2 v v1 v 2
2.5 2.5
v v + v2
(c) = 1 (d) v = v1v 2
34. A circuit when connected to an AC source of 12 V gives 2 2
a current of 0.2 A. The same circuit when connected to
a DC source of 12 V, gives a current of 0.4 A. The circuit 41. Assuming that the gravitational potential energy of an
is object at infinity is zero, the change in potential energy
(a) series LR (b) series RC
(final – initial) of an object of mass m, when taken to a
(c) series LC (d) series LCR height h from the surface of earth (of radius R) is given
by,
35. A cycle wheel of radius 0.5 m is rotated with constant GMm GMmh
(a) − (b)
angular velocity of 10 rad/s in a region of magnetic R+ h R (R + h)
field of 0.1 T which is perpendicular to the plane of the GMm
(c) mgh (d)
wheel. The EMF generated between its centre and the R+ h
rim is
(a) 0.25 V (b) 0.125 V 42. 1g of water, of volume 1 cm 3 at 100°C is converted into
(c) 0.5 V (d) zero steam at same temperature under normal atmospheric
pressure= (~
− 1 × 10 5 Pa). The volume of steam formed
36. For a transparent medium relative permeability and
permittivity µ r and ∈r are 1.0 and 1.44 respectively. The equals 1671 cm 3 . If the specific latent heat of
velocity of light in this medium would be vaporisation of water is 2256 J/g, the change in internal
(a) 2.5 × 108 m/s (b) 3 × 108 m/s energy is
(a) 2423 J (b) 2089 J (c) 167 J (d) 2256 J
(c) 2.08 × 10 m/s
8
(d) 4.32 × 108 m/s
www.jeebooks.in
NEET Solved Paper 2019 37

43. Angular width of the central maxima in the 44. The work function of a photosensitive material is 4.0
Fraunhoffer diffraction for λ = 6000 Å is θ 0 . When the eV. This longest wavelength of light that can cause
same slit is illuminated by another monochromatic photon emission from the substance is (approximately)
light, the angular width decreases by 30%. The (a) 3100 nm (b) 966 nm (c) 31 nm (d) 310 nm
wavelength of this light is
45. A proton and an α-particle are accelerated from rest to
(a) 1800 Å (b) 4200 Å
the same energy. The de-Broglie wavelengths λ p and
(c) 6000 Å (d) 420 Å λ α are in the ratio
(a) 2 : 1 (b) 1 : 1 (c) 2 :1 (d) 4 : 1

CHEMISTRY
46. The reaction that does not give benzoic acid as the 50. The artificial sweetner stable at cooking temperature
major product is and does not provide calories is
CH2OH (a) saccharin (b) aspartame
(a) K2Cr2O7 (c) sucralose (d) alitame

51. The liquified gas that is used in dry cleaning along with
COCH3 a suitable detergent is
(i) NaOCl (a) water gas (b) petroleum gas
(b)
(ii) H3O+ (c) NO 2 (d) CO 2

CH2OH 52. The hydrolysis reaction that takes place at the slowest
(c)
PCC rate, among the following is
(Pyridinium
aq.NaOH
chlorochromate) (a) Cl ONa
CH2OH
KMnO4/H+ CH3 CH3
(d) aq . NaOH
(b) H3C CH2 Cl → H3C CH2 OH
aq . NaOH
47. The amine that reacts will Hinsberg’s reagent to give (c) H2C == CH CH2Cl →
an alkali insoluble product is H2C == CH CH2OH
(a) CH3  CH NH  CH CH3 aq.NaOH
 CH2CH3 (d) CH2Cl CH2OH
CH3  CH3
(b) CH3 CH2 N CH2C H3
53. When vapours of a secondary alcohol is passed over
NH2
heated copper at 573 K, the product formed is

(c) CH3 C C H2CH2CH3 (a) a carboxylic acid (b) an aldehyde
 (c) a ketone (d) an alkene
CH3
CH3 54. The major products C and D formed in the following
 reactions respectively are
(d) CH3 C  CH  NH2
Excess HI
  H 3C CH 2 CH 2 OC(CH 3)3 → C + D
CH3 CH3 ∆

(a) H3C CH2 CH2 I and I C(CH3 )3


48. Which structure(s) of proteins remains(s) intact (b) H3C CH2 CH2 OH and I C(CH3 )3
during denaturation process? (c) H3C CH2 CH2 I and HO C(CH3 )3
(a) Both secondary and tertiary structures (d) H3C CH2 CH2 OH and HO C(CH3 )3
(b) Primary structure only
(c) Secondary structure only 55. Match the oxide given in column A with its property
(d) Tertiary structure only
given in column B.
Column-A Column-B
49. The polymer that is used as a substitute for wool in
making commercial fibres is 1. Na 2O i. Neutral
(a) melamine 2. Al 2O 3 ii. Basic
(b) nylon-6, 6 3. N2O iii. Acidic
(c) polyacrylonitrile
4. Cl 2O 7 iv. Amphoteric
(d) buna-N
www.jeebooks.in
38 NEET Test Drive

Which of the following options has all correct pairs? (c) Ores are minerals that may contain a metal
1 2 3 4 1 2 3 4 (d) Gangue is an ore contaminated with undesired materials
(a) (ii), (i), (iv), (iii) (b)(iii), (ii), (i), (iv)
63. A compound ‘X ’ upon reaction with H 2O produced a
(c) (i), (ii), (ii), (iii) (d) (ii), (iv), (i), (iii)
colourless gas ‘Y’ with rotten fish smell. Gas ‘Y ’ is
56. Match the catalyst with the process absorbed in a solution of CuSO 4 to give Cu 3P2 as one of
the products. Predict the compound ‘X ’
Catalyst Process
(a) Ca 3P2 (b) NH4Cl
1. V2O 5 i. The oxidation of ethyne to (c) As 2O 3 (d) Ca 3 (PO 4 )2
ethanal
2. TiCl 4 + Al(CH3 )3 ii. Polymerisation of alkynes 64. Which of the following oxoacids of phosphorus has
strongest reducing property?
3. PdCl 2 iii. Oxidation of SO 2 in the (a) H4P2O 7 (b) H3PO 3
manufacture of H2SO 4 (c) H3PO 2 (d) H3PO 4
4. Nickel iv. Polymerisation of ethylene 65. Identify the correct formula of oleum from the
complexes following
Which of the following is the correct option? (a) H2S2O 7 (b) H2SO 3
(c) H2SO 4 (d) H2S2O 8
1 2 3 4 1 2 3 4
(a) (iii), (iv), (i), (ii) (b) (i), (ii), (iii), (iv) 66. When neutral or faintly alkaline KMnO 4 is treated with
(c) (i), (iii), (ii), (iv) (d)(iii), (i), (iv), (ii)
potassium iodide, iodide ion is converted into ‘X ’. ‘X ’ is
57. The most stable carbocation, among the following is (a) I2 (b) IO −4

(c) IO −3 (d) IO −
(a) (CH3 )3 C CH CH3

(b) CH3 CH2 CH CH2 CH3 67. The Crystal Field Stabilisation Energy (CFSE) for
⊕ [CoCl 6 ]4− is 18000 cm −1 . The CFSE for [CoCl 4 ]2− will be
(c) CH3 CH CH2 CH2 CH3
⊕ (a) 6000 cm−1 (b) 16000 cm−1
(d) CH3 CH2 CH2 (c) 18000 cm−1 (d) 8000 cm−1

58. The alkane that gives only one mono-chloro product on 68. Following limiting molar conductivities are given as
chlorination with Cl 2 in presence of diffused sunlight is
λ 0m (H 2SO 4 ) = x S cm 2 mol −1
(a) 2, 2-dimethylbutane (b) neopentane
λ 0m(K 2SO 4 ) = y S cm 2mol −1
(c) n-pentane (d) isopentane
59. In the following reaction, λ 0m(CH 2COOK) = z S cm 2 mol −1
Red hot iron tube
H 3C C ≡≡ CH →
873 K
A, λ 0m (inS cm 2mol −1) for CH 3COOH will be
the number of sigma(σ) bonds present in the product A (a) x − y + 2 z (b) x + y − z
is ( x − y)
(c) x − y + z (d) + z
(a) 21 (b) 9 (c) 24 (d) 18 2

60. Aluminium chloride in acidified aqueous solution 69. A first order reaction has a rate constant of2.303 × 10 −3
forms a complex ‘A’, in which hybridisation state of Al s −1 . The time required for 40 g of this reactant to reduce
is ‘B’. What are ‘A’ and ‘B’, respectively? to 10 g will be
(a) [Al(H2O)6 ]3+, sp3d 2 (b) [Al(H2O)4 ]3+, sp3
3+ 2 [Given that log 10 2 = 0 .3010]
(c) [Al(H2O)4 ] , dsp (d) [Al(H2O)6 ]3+, d 2 sp3
(a) 230.3 s (b) 301 s
61. Which of the following compounds is used in cosmetic (c) 2000 s (d) 602 s
surgery? 70. For a reaction, activation energy E a = 0 and the rate
(a) Silica (b) Silicates
constant at 200K is 1.6 × 10 6 s −1 . The rate constant at
(c) Silicones (d) Zeolites
400 K will be
62. Identify the incorrect statement. [Given that gas constant R = 8 .314 JK −1 mol −1 ]
(a) The scientific and technological process used for isolation
(a) 3.2 × 104 s −1 (b) 1.6 × 106 s −1
of the metal from its ore is known as metallurgy −1
(b) Minerals are naturally occurring chemical substances in the (c) 1.6 × 10 s3
(d) 3.2 × 106 s −1
earth’s crust
www.jeebooks.in
NEET Solved Paper 2019 39

71. The correct option representing a Freundlich 83. In water saturated air the mole fraction of water
adsorption isotherm is vapour is 0.02. If the total pressure of the saturated air
(a)
x
= kp0. 3
x
(b) = kp2. 5 is 1.2 atm, the partial pressure of dry air is
m m (a) 1.18 atm (b) 1.76 atm
x x
(c) = kp−0. 5 (d) = kp−1 (c) 1.176 atm (d) 0.98 atm
m m
84. The standard electrode potential (E −) values of
72. Which of the following is paramagnetic?
(Al 3+ / Al, Ag + / Ag, K + / K and Cr3+ / Cr are − 166
. V,
(a) N2 (b) H2 (c) Li 2 (d) O 2
0.80V, 2.93 V and − 0 .74 V, respectively. The correct
73. Which of the following is the correct order of dipole decreasing order of reducing power of the metal is
moment? (a) Ag > Cr > Al > K
(a) NH3 <BF3 <NF3 <H2O (b) BF3 <NF3 <NH3 <H2O (b) K > Al > Cr > Ag
(c) BF3 <NH3 <NF3 <H2O (d) H2O <NF3 <NH3 <BF3 (c) K > Al > Ag > Cr
(d) Al > K > Ag > Cr
74. Crude sodium chloride obtained by crystallisation of
brine solution does not contain 85. The density of 2 M aqueous solution of NaOH is 1.28
g/cm 3 . The molality of the solution is [Given that
(a) MgSO 4 (b) Na 2SO 4 (c) MgCl 2 (d) CaSO 4
molecular mass of NaOH = 40 g mol −1 ]
75. Which of the alkali metal chloride (MCl) forms its (a) 1.20 m (b) 1.56 m
dihyrate salt (MCl⋅2H 2O) easily? (c) 1.67 m (d) 1.32 m
(a) LiCl (b) CsCl (c) RbCl (d) KCl
86. Orbital having 3 angular nodes and 3 total nodes is
76. The pH of 0.01 M NaOH (aq) solution will be (a) 5 p (b) 3 d (c) 4 f (d) 6 d
(a) 7.01 (b) 2 (c) 12 (d) 9
87. In hydrogen atom, the de-Broglie wavelength of an
77. Which of the following cannot act both as Bronsted electron in the second Bohr orbit is
acid and as Bronsted base? [Given that, Bohr radius, a 0 = 52.9 pm]
(a) HCO −3 (b) NH3 (c) HCl (d) HSO −4 (a) 211.6 pm (b) 211.6 π pm
−11 (c) 52.9 π pm
78. The molar solubility of CaF2 (K sp = 5.3 × 10 ) in 0.1 M (d) 105.8 pm

solution of NaF will be 88. The volume occupied by 1.8 g of water vapour at 374°C
(a) 5.3 × 1011 mol L −1 (b) 5.3 × 10−8 mol L −1 and 1 bar pressure will be
(c) 5.3 × 10−9 mol L −1 (d) 5.3 × 10−10 mol L −1 [Use R = 0 .083 bar L K −1 mol −1 ]
(a) 96.66 L (b) 55.87 L (c) 3.10 L (d) 5.37 L
79. The oxidation state of Cr in CrO 5 is
(a) − 6 (b) + 12 (c) + 6 (d) + 4 89. An ideal gas expands isothermally from10 −3 m 3 to10 −2
m 3 at 300 K against a constant pressure of 10 5 Nm −2 . The
80. The number of hydrogen bonded water molecule(s) work done on the gas is
associated with CuSO 4 ⋅ 5H 2O is (a) + 270 kJ (b) − 900 J (c) + 900 kJ (d) − 900 kJ
(a) 3 (b) 1 (c) 2 (d) 5
90. Reversible expansion of an ideal gas under isothermal
81. Formula of nickel oxide with metal deficiency defect in
2+ 3+ and adiabatic conditions are as shown in the figure.
its crystal is Ni 0.98O. The crystal contains Ni and Ni
A ( pA, VA, TA)
ions. The fraction of nickel existing as Ni2+ ions in the
crystal is B ( pB, VB, TB)
(a) 0.96 (b) 0.04 p
(c) 0.50 (d) 0.31
C ( pC, VC, TC)
82. Which of the following statements is correct regarding
a solution of two compounds A and B exhibiting V
positive deviation from ideal behaviour? AB → Isothermal expansion
(a) Intermolecular attractive forces between A—A and B—B are AC → Adiabatic expansion
stronger than those between A—B. Which of the following option is not correct?
(b) ∆mix H = 0 at constant T and p
(a) ∆Sisothermal > ∆Sadiabatic
(c) ∆mix V = 0 at constant T and p
(d) Intermolecular attractive forces between A—A and B—B (b) TA = TB
are equal to those between A—B. (c) Wisothermal > Wadiabatic
(d) Tc > TA
www.jeebooks.in
40 NEET Test Drive

BIOLOGY
91. Which of the following cell organelles is present in the 99. Select the correct sequence of events
highest number in secretory cells? (a) Gametogenesis → Gamete transfer → Syngamy → Zygote
(a) Mitochondria (b) Golgi complex → Cell division (Cleavage) → Cell differentiation →
(c) Endoplasmic reticulum (d) Lysosomes Organogenesis
(b) Gametogenesis → Gamete transfer → Syngamy → Zygote
92. Non-membranous nucleoplasmic structures in nucleus → Cell divison (Cleavage) → Organogenesis→ Cell
are the site for acitive synthesis of differentiation
(a) protein synthesis (b) mRNA (c) Gametogenesis → Syngamy → Gamete transfer → Zygote
(c) rRNA (d) tRNA → Cell division (Cleavage) → Cell differentiation →
Organogenesis
93. Which of the following nucleic acids is present in an (d) Gametogenesis → Gamete transfer → Syngamy → Zygote
organism having 70S ribosomes only? → Cell differentiation → Cell division (Cleavage) →
(a) Single-stranded DNA with protein coat Organogenesis
(b) Double-stranded circular naked DNA
100. Which of the following hormones is responsible for
(c) Double-stranded DNA enclosed in nuclear membrane
both the milk ejection reflex and the foetal ejection
(d) Double-stranded circular DNA with histone proteins
reflex?
94. After meiosis-I, the resultant daughter cells have (a) Oestrogen (b) Prolactin
(a) same amount of DNA as in the parent cell in (c) Oxytocin (d) Relaxin
S-phase
(b) twice the amount of DNA in comparison to haploid gamete 101. No new follicles develop in the luteal phase of the
(c) same amount of DNA in comparison to haploid gamete menstrual cycle because
(d) four times the amount of DNA in comparison to haploid (a) follicles do not remain in the ovary after ovulation
gamete (b) FSH levels are high in the luteal phase
(c) LH levels are high in the luteal phase
95. Which of the following organic compounds is the main (d) both FSH and LH levels are low in the luteal phase
constituent of lecithin?
(a) Arachidonic acid (b) Phospholipid 102. In Australia, marsupials and placental mammals have
(c) Cholesterol (d) Phosphoprotein evolved to share many similar characteristics. This type
of evolution may be referred to as
96. The main difference between active and passive
(a) adaptive radiation (b) divergent evolution
transport across cell membrane is
(c) cyclical evolution (d) convergent evolution
(a) passive transport is non-selective whereas active transport
is selective 103. Match the items of Column I with Column II.
(b) passive transport requires a concentration gradient across
a biological membrane whereas active transport requires Column I Column II
energy to move solutes 1. XX-XO method of i. Turner’s
(c) passive transport is confined to anionic carrier proteins sex-determination syndrome
whereas active transport is confined to cationic channel
proteins 2. XX-XY method of ii. Female
(d) active transport occurs more rapidly than passive transport sex-determination heterogamety
3. Karyotype-45 iii. Grasshopper
97. Match the items given in Column I with those in
Column II and choose the correct option. 4. ZW-ZZ method of iv. Female
sex- determination homogamety
Column I Column II
1. Rennin i. Vitamin-B12 Select the correct option from the following
2. Enterokinase ii. Facilitated transport 1 2 3 4 1 2 3 4
(a) (ii) (iv) (i) (iii) (b) (i) (iv) (ii) (iii)
3. Oxyntic cells iii. Milk proteins (c) (iii) (iv) (i) (ii) (d) (iv) (ii) (i) (iii)
4. Fructose iv. Trypsinogen
104. What will be the sequence of mRNA produced by the
1 2 3 4 1 2 3 4 following stretch of DNA?
(a) (iii) (iv) (ii) (i) (b) (iv) (iii) (i) (ii)
3'ATGCATGCATGCATG5' TEMPLATE STRAND
(c) (iv) (iii) (ii) (i) (d) (iii) (iv) (i) (ii)
5'TACGTACGTACGTAC3' CODING STRAND
98. Kwashiorkor disease is due to (a) 3' - AUGCAUGCAUGCAUG 5'
(a) simultaneous deficiency of proteins and fats (b) 5' - UACGUACGUACGUAC 3'
(b) simultaneous deficiency of protein and calories
(c) 3' - UACGUACGUACGUAC 5'
(c) deficiency of carbohydrates
(d) 5' - AUGCAUGCAUGCAUG 3'
(d) protein deficiency not accompanied by calorie deficiency
www.jeebooks.in
NEET Solved Paper 2019 41

105. Select the incorrect statement regarding inbreeding. 112. Match the following RNA polymerases with their
(a) Inbreeding helps in the elimination of deleterious alleles transcribed products
from the population
1. RNA polymerase I i. tRNA
(b) Inbreeding is necessary to evolve a pureline in any animal
(c) Continued inbreeding reduces fertility and leads to 2. RNA polymerase II ii. rRNA
inbreeding depression 3. RNA polymerase III iii. hnRNA
(d) Inbreeding depression cannot be overcome by outcrossing
Select the correct option from the following
106. A biocontrol agent to be a part of an integrated pest 1 2 3 1 2 3
management should be (a) (i) (iii) (ii) (b) (i) (ii) (iii)
(a) species-specific and symbiotic (c) (ii) (iii) (i) (d) (iii) (ii) (i)
(b) free-living and broad spectrum 113. In a marriage between male with blood group A and
(c) narrow spectrum and symbiotic female with blood group B, the progeny had either
(d) species-specific and inactive on non-target organisms blood group AB or B. What could be the possible
107. Match the following enzymes with their functions genotype of parents?
(a) IAi (Male) : IBIB (Female)
Column I Column II
(b) IAIA (Male) : IBIB (Female)
1. Restriction i. Joins the DNA (c) IAIA (Male) : IBi (Female)
endonuclease fragments (d) IAi (Male) : IBi (Female)
2. Restriction ii. Extends primers on
exonuclease genomic DNA template 114. A population of a species invades a new area. Which of
the following conditions will lead to adaptive
3. DNA ligase iii. Cuts DNA at specific
position
radiation?
(a) Area with large number of habitats having very low food
4. Taq iv. Removes nucleotides
supply
polymerase from the ends of DNA
(b) Area with a single type of vacant habitat
Select the correct option from the following (c) Area with many types of vacant habitats
1 2 3 4 1 2 3 4 (d) Area with many habitats occupied by a large number of
(a) (iii) (i) (iv) (ii) (b) (iii) (iv) (i) (ii) species
(c) (iv) (iii) (i) (ii) (d) (ii) (iv) (i) (iii)
115. Identify A, B and C in the diagrammatic representation
108. The two antibiotic resistant genes on vector pBR322 are of the mechanism of hormone action.
A
for
(a) Ampicillin and Tetracycline B
(b) Ampicillin and Chloramphenicol
(c) Chloramphenicol and Tetracycline Response 1
(d) Tetracycline and Kanamycin
C
109. Exploitation of bioresources of a nation by
multinational companies without authorisation from Physiological responses
the concerned country is referred to as
(a) bioweapon (b) biopiracy Select the correct option from the following
(c) bioethics (d) biowar (a) A – Steroid hormone; B–Hormone-receptor complex,
C – Protein
110. Carnivorous animals-lions and leopards, occupy the (b) A–Protein hormone, B–Receptor; C–Cyclic AMP
same niche but lions predate mostly larger animals and (c) A – Steroid hormone; B-Receptor, C-Second messenger
leopards take smaller ones. This mechanism of (d) A – Protein hormone; B – Cyclic AMP, C – Homone-receptor
competition is referred to as complex
(a) character displacement
(b) altruism 116. Humans have acquired immune system that produces
(c) resource partitioning antibodies to neutralise pathogens. Still innate
(d) competitive exclusion immune system is present at the time of birth because
it
111. Decline in the population of Indian native fishes due to (a) is very specific and uses different macrophages
introduction of Clarias gariepinus in river Yamuna can (b) produces memory cells for mounting fast secondary
be categoriesd as response
(a) co-extinction (c) has natural killer cells which can phagocytose and destroy
(b) habitat fragmentation microbes
(c) overexploitation (d) provides passive immunity
(d) allien species invasion
www.jeebooks.in
42 NEET Test Drive

117. Which of the following statements is not correct? Select the correct option from the following
(a) An action potential in an axon does not move backward 1 2 3 4 1 2 3 4
because the segment behind is in a refractory phase (a) (iv) (i) (iii) (ii) (b) (iii) (iv) (i) (ii)
(b) Depolarisation of hair cells of cochlea results in the (c) (i) (iii) (iv) (ii) (d) (iii) (iv) (ii) (i)
opening of the mechanically gated potassium-ion channels 125. Which of the following animals are true coelomates
(c) Rods are very sensitive and contribute to daylight vision with bilateral symmetry?
(d) In the knee-jerk reflex, stimulus is the stretching of muscle
(a) Adult echinoderms (b) Aschelminthes
and response is its contraction
(c) Platyhelminthes (d) Annelids
118. Match the following joints with the bones involved 126. The contrasting characteristics generally in a pair used
Column I Column II for identification of animals in taxonomic key are
1. Gliding joint i. Between carpal and referred to as
metacarpal of thumb (a) lead (b) couplet (c) doublet (d) alternate
2. Hinge joint ii. Between atlas and axis
127. Mach the following cell structures with their
3. Pivot joint iii. Between the carpals characteristic features.
4. Saddle joint iv. Between humerus and
ulna Column I Column II

Select the correct option from the following 1. Tight junctions i. Cement neighbouring
cells together to form
1 2 3 4 1 2 3 4
sheet
(a) (iii) (iv), (ii) (i) (a) (iv) (i) (ii) (iii)
(b) (iv) (ii) (iii) (i) (d) (i) (iii) (ii) (iv) 2. Adhering ii. Transmit information
junctions through chemical to
119. Which of the following diseases is an autoimmune another cells
disorder?
(a) Myasthenia gravis (b) Arthritis 3. Gap junctions iii. Establish a barrier to
(c) Osteoporosis (d) Gout prevent leakage of fluid
across epithelial cells
120. Artificial light, extended work-time and reduced
sleep-time disrupt the activity of 4. Synaptic iv. Cytoplasmic channels
junctions to facilitate
(a) thymus gland (b) pineal gland communication
(c) adrenal gland (d) posterior pituitary gland between adjacent cells
121. Which of the following conditions will stimulate
parathyroid gland to release parathyroid hormone? Select the correct option from the following
1 2 3 4 1 2 3 4
(a) Fall in active vitamin-D levels
(a) (ii) (iv) (i) (iii) (b) (iv) (ii) (i) (iii)
(b) Fall in blood Ca +2 levels
(c) (iii) (i) (iv) (ii) (d) (iv) (iii) (i) (ii)
(c) Fall in bone Ca +2 levels
(d) Rise in blood Ca +2 levels 128. Which of the following statements is incorrect?
122. Which of the following is a correct statement? (a) Cockroaches exhibit mosaic vision with less sensitivity and
(a) IUDs once inserted need not be replaced more resolution
(b) IUDs are generally inserted by the user herself (b) A mushroom-shaped gland is present in the 6-7th
abdominal segments of male cockroach
(c) IUDs increase phagocytosis of sperms in the uterus
(d) IUDs suppress gametogenesis (c) A pair of spermatheca is present in the 6th segment of
female cockroach
123. Which of the following sexually transmitted diseases (d) Female cockroach possesses sixteen ovarioles in the
do not specifically affect reproductive organs? ovaries
(a) Genital warts and Hepatitis-B
(b) Syphilis and Genital herpes 129. Select the correct statement.
(c) AIDS and Hepatitis-B (a) Expiration occurs due to external intercostal muscles
(d) Chlamydiasis and AIDS (b) Intrapulmonary pressure is lower than the atmospheric
pressure during inspiration
124. Match the following genera with their respective (c) Inspiration occurs when atmospheric pressure is less than
phylum intrapulmonary pressure
Column I Column II
(d) Expiration is initiated due to contraction of diaphragm

1. Ophiura (i) Mollusca 130. The maximum volume of air a person can breathe in
2. Physalia (ii) Platyhelminthes after a forced expiration is known as
3. Pinctada (iii) Echinodermata (a) expiratory capacity (b) vital capacity
(c) inspiratory capacity (d) total lung capacity
4. Planaria (iv) Coelenterata
www.jeebooks.in
NEET Solved Paper 2019 43

131. All the components of the nodal tissue are (a) Basilar membrane and otoliths
autoexcitable. Why does the SA node act as the normal (b) Hair cells and organ of Corti
pacemaker? (c) Tectorial membrane and macula
(d) Crista ampullaris and macula
(a) SA node has the lowest rate of depolarisation
(b) SA node is the only component to generate the threshold 136. Which of the following is against the rules of ICBN?
potential (a) Handwritten scientific names should be underlined
(c) Only SA node can convey the action potential to the other (b) Every species should have a generic name and a specific
components epithet
(d) SA node has the highest rate of depolarisation (c) Scientific names are in Latin and should be italicised
132. A specialised nodal tissue embedded in the lower (d) Generic and specific names should be written starting with
corner of the right atrium, close to atrio-ventricular small letters
septum, delays the spreading of impulses to heart apex 137. Mad cow disease in cattle is caused by an organism
for about 0.1 sec. The delay allows which has
(a) blood to enter aorta (a) inert crystalline structure
(b) the ventricles to empty completely (b) abnormally folded protein
(c) blood to enter pulmonary arteries (c) free RNA without protein coat
(d) the atria to empty completely (d) free DNA without protein coat
133. Match the following parts of a nephron with their 138. Which of the following statements is correct?
function.
(a) Lichens do not grow in polluted areas
Column I Column II (b) Algal component of lichens is called mycobiont
1. Descending limb i. Reabsorption of salts (c) Fungal component of lichens is called phycobiont
of Henle’s loop only (d) Lichens are not good pollution indicators
2. Proximal ii. Reabsorption of water 139. Match the organisms in Column I with habitats in
convoluted only Column II
tubule
Column I Column II
3. Ascending limb iii. Conditional
of Henle’s loop reabsorption of sodium 1. Halophiles i. Hot springs
ion and water
2. Thermoacidophiles ii. Aquatic environment
4. Distal convoluted iv. Reabsorption of ion,
tubule water and organic 3. Methanogens iii. Guts of ruminants
nutrients 4. Cyanobacteria iv. Salty area

Select the correct option from the following Select the correct option from the following
1 2 3 4 1 2 3 4 1 2 3 4
(a) (i) (iii) (ii) (iv)
(a) (iv) (i) (iii) (ii) (b) (i) (ii) (iii) (iv)
(b) (ii) (iv) (i) (iii)
(c) (i) (iv) (ii) (iii) (c) (iii) (iv) (ii) (i) (d) (ii) (iv) (iii) (i)
(d) (iv) (i) (iii) (ii)
140. In dicot root, the vascular cambium originates from
134. Match the items in Column-I with those in Column II (a) tissue located below the phloem bundles and a portion of
pericycle tissue above protoxylem
Column I Column II (b) cortical region
1. Podocytes i. Crystallised oxalates (c) parenchyma between endodermis and pericycle
(d) intrafascicular and interfascicular tissue in a ring
2. Protonephridia ii. Annelids
3. Nephridia iii. Amphioxus 141. Which of the following shows whorled phyllotaxy?
4. Renal calculi iv. Filtration slits (a) Mustard (b) China rose
(c) Alstonia (d) Calotropis
Select the correct option from the following
142. Regeneration of damaged growing grass following
1 2 3 4 1 2 3 4
grazing is largely due to
(a) (iii) (iv) (ii) (i) (b) (iii) (ii) (iv) (i)
(a) lateral meristem (b) apical meristem
(c) (iv) (iii) (ii) (i) (d) (iv) (ii) (iii) (i) (c) intercalary meristem (d) secondary meristem
135. Which of the following receptors are specifically 143. Bicarpellary ovary with obliquely placed septum is seen
responsible for maintenance of the balance of body and in
posture? (a) Brassica (b) Aloe (c) Solanum (d) Sesbania
www.jeebooks.in
44 NEET Test Drive

144. Which is the most common type of embryo sac in 3. Vacuoles iii. Formation of
angiosperms? glycoproteins and
glycolipids
(a) Tetrasporic with one mitotic stage of divisions
(b) Monosporic with three sequential mitotic divisions 4. Ribosomes iv. Digesting biomolecules
(c) Monosporic with two sequential mitotic divisions
(d) Bisporic with two sequential mitotic divisions Select the correct option from the following
1 2 3 4 1 2 3 4
145. From the following, identify the correct combination of (a) (iii) (iv) (ii) (i) (b) (iv) (iii) (i) (ii)
salient features of Genetic code. (c) (iii) (ii) (iv) (i) (d) (i) (ii) (iv) (iii)
(a) Universal, Non-ambiguous, Overlapping 153. Prosthetic groups differ from coenzymes in that
(b) Degenerate, Overlapping, Commaless
(a) they require metal ions for their activity
(c) Universal, Ambiguous, Degenerate
(d) Degenerate, Non-overlapping, Non-ambiguous (b) they (prosthetic groups) are tightly bound to apoenzymes
(c) their association with apoenzymes is transient
146. Which scientist experimentally proved that DNA is the (d) they can serve as cofactors in a number of enzyme
sole genetic material in bacteriophage? catalysed reactions
(a) Beadle and Tatum (b) Meselson and Stahl 154. Crossing over takes place between which chromatids and
(c) Hershey and Chase (d) Jacob and Monod in which stage of the cell cycle?
147. In the process of transcription in eukaryotes, the RNA (a) Non-sister chromatids of non-homologous chromosomes at
zygotene stage of prophase I
polymerase I transcribes
(b) Non-sister chromatids of homologous chromosomes at
(a) mRNA with additional processing, capping and tailing
pachytene stage of prophase I
(b) tRNA, 5 srRNA and snRNAs
(c) Non-sister chromatids of homologous chromosomes at
(c) rRNAs-28 S, 18 S and 5.8 S
zygotene stage of prophase I
(d) precursor of mRNA, hnRNA
(d) Non-sister chromatids of non-homologous chromosomes
148. In which genetic condition, each cell in the affected at pachytene stage of prophase I
person, has three sex chromosomes XXY? 155. “Ramachandran plot” is used to confirm the structure
(a) Thalassemia (b) Klinefelter’s syndrome of
(c) Phenylketonuria (d) Turner’s syndrome (a) RNA (b) proteins
(c) triacylglycerides (d) DNA
149. What initiation and termination factors are involved in
transcription in eukaryotes? 156. Which of the following is not a feature of active
transport of solutes in plants?
(a) σ and ρ, respectively (b) α and β, respectively
(a) Occurs against concentration gradient
(c) β and γ, respectively (d) α and σ, respectively (b) Non-selective
(c) Occurs through membranes
150. Which of the following statements is correct about the
(d) Requires ATP
origin and evolution of men?
(a) Agriculture came around 50,000 years back 157. Which of the following bacteria reduces nitrate in soil
(b) The Dryopithecus and Ramapithecus primates existing 15 into nitrogen?
million years ago, walked like men (a) Nitrobacter (b) Nitrococcus
(c) Homo habilis probably ate meat (c) Thiobacillus (d) Nitrosomonas
(d) Neanderthal men lived in Asia between 1,00,000 and 158. What will be the direction of flow of water when a
40,000 years back plant cell is placed in a hypotonic solution?
(a) Water will flow in both directions
151. The production of gametes by the parents, the
(b) Water will flow out of the cell
formation of zygotes, the F1 and F2 plants, can be
(c) Water will flow into the cell
understood using (d) No flow of water in any direction
(a) pie diagram (b) a pyramid diagram
(c) Punnett square (d) Venn diagram 159. Where is respiratory Electron Transport System (ETS)
located in plants?
152. Match the Column I with Column II. (a) Mitochondrial matrix
(b) Outer mitochondrial membrane
Column I Column II (c) Inner mitochondrial membrane
1. Golgi i. Synthesis of protein (d) Intermembrane space
apparatus 160. In Hatch and Slack pathway, the primary CO 2 acceptor
2. Lysosomes ii. Trap waste and is
excretory products (a) oxaloacetic acid (b) phosphoglyceric acid
(c) phosphoenol pyruvate (d) RuBisCO
www.jeebooks.in
NEET Solved Paper 2019 45

161. Removal of shoot tips is very useful technique to boost 169. Which of the following statements about ozone is
the production of tea leaves. This is because correct?
(a) gibberellins prevent bolting and are inactivated (a) Tropospheric ozone protects us from
(b) auxins prevent leaf drop at early stages UV- radiations
(c) effect of auxins is removed and growth of lateral buds is (b) Stratospheric ozone is ‘bad’
enhanced (c) Tropospheric ozone is ‘good’
(d) gibberellins delay senescence of leaves (d) Stratospheric ozone protects us from
UV- radiations
162. One scientist cultured Cladophora in a suspension of
Azotobacter and illuminated the culture by splitting light 170. Exploration of molecular, genetic and species level
through a prism. He observed that bacteria diversity for novel products of economic importance is
accumulated mainly in the region of known as
(a) violet and green light (b) indigo and green light
(a) biopiracy (b) bioenergetics
(c) orange and yellow light (d) blue and red light
(c) bioremediation (d) bioprospecting
163. In order to increase the yield of sugarcane crop, which
of the following plant growth regulators should be 171. Which of the following is an innovative remedy for
sprayed? plastic waste?
(a) Ethylene (b) Auxins (a) Burning in the absence of oxygen
(c) Gibberellins (d) Cytokinins (b) Burrying 500 m deep below soil surface
(c) Polyblend
164. What type of pollination takes place in Vallisneria? (d) Electrostatic precipitator
(a) Pollination occurs in submerged condition by water
(b) Flowers emerge above surface of water and pollination 172. Between which among the following, the relationship
occurs by insects is not an example of commensalism?
(c) Flowers emerge above water surface and pollen is carried (a) Orchid and the tree on which it grows
by wind (b) Cattle egret and grazing cattle
(d) Male flowers are carried by water currents to female flowers (c) Sea anemone and clown fish
at the surface of water (d) Female wasp and fig species
165. In which of the following, both autogamy and 173. If an agricultural field is liberally irrigated for a
geitonogamy are prevented? prolonged period of time, it is likely to face problem of
(a) Wheat (b) Papaya (c) Castor (d) Maize (a) metal toxicity (b) alkalinity
166. Match the placental types (Column I) with their (c) acidity (d) salinity
examples (Column II) 174. Which of the following statements about methanogens
Column I Column II is not correct?
1. Basal i. Mustard (a) They can be used to produce biogas
2. Axile ii. China rose (b) They are found in the rumen of cattle and their excreta
(c) They grow aerobically and breakdown cellulose rich food
3. Parietal iii. Dianthus
(d) They produce methane gas
4. Free-central iv. Sunflower
175. In mung bean, resistance to yellow mosaic, virus and
Select the correct option from the following powdery mildew were brought about by
1 2 3 4 1 2 3 4
(a) mutation breeding
(a) (ii) (iii) (iv) (i) (b) (i) (ii) (iii) (iv)
(b) biofortification
(c) (iv) (ii) (i) (iii) (d) (iii) (iv) (i) (ii)
(c) tissue culture
167. A selectable marker is used to (d) hybridisation and selection
(a) help in eliminating the non-transformants, so that the
transformants can be regenerated 176. Coca alkaloid or cocaine is obtained from
(b) identify the gene for a desired trait in an alien organism (a) Papaver somniferum
(c) select a suitable vector for transformation in a specific crop (b) Atropa belladonna
(d) mark a gene on a chromosome for isolation using (c) Erythroxylum coca
restriction enzyme (d) Datura

168. Western Ghats have a large number of plant and 177. Among the following pairs of microbes, which pair has
animal species that are not found anywhere else. both the microbes that can be used as biofertilisers?
Which of the following terms will you use to notify (a) Aspergillus and Rhizopus
such species? (b) Rhizobium and Rhizopus
(a) Endemic (b) Vulnerable (c) Cyanobacteria and Rhizobium
(c) Threatened (d) Keystone (d) Aspergillus and Cyanobacteria
www.jeebooks.in
46 NEET Test Drive

178. Given below are four statements pertaining to Select the correct option from the following
separation of DNA fragments using gel electrophoresis. (a) 1, 3 and 4 (b) 1, 2 and 3
(c) 2, 3 and 4 (d) 1, 2 and 4
Identify the incorrect statements.
1. DNA is negatively charged molecule and so it is 179. An enzyme catalysing the removal of nucleotides from
loaded on gel towards the anode terminal. ends of DNA is
(a) DNA ligase
2. DNA fragments travel along the surface of the gel
(b) endonuclease
whose concentration does not affect movement of
(c) exonuclease
DNA. (d) protease
3. Smaller the size of DNA fragment larger is the
distance it travels through it. 180. In RNAi, the genes are silenced using
4. Pure DNA can be visualised directly by exposing (a) dsRNA (b) ssDNA
UV-radiation. (c) ssRNA (d) dsDNA

Answers
1. (d) 2. (d) 3. (d) 4. (c) 5. (a) 6. (d) 7. (d) 8. (c) 9. (d) 10. (b)
11. (b) 12. (c) 13. (b) 14. (a) 15. (a) 16. (d) 17. (c) 18. (c) 19. (a) 20. (b)
21. (c) 22. (d) 23. (c) 24. (d) 25. (a) 26. (a) 27. (b) 28. (c) 29. (d) 30. (d)
31. (b) 32. (d) 33. (a) 34. (a) 35. (b) 36. (a) 37. (b) 38. (c) 39. (c) 40. (b)
41. (b) 42. (b) 43. (b) 44. (d) 45. (a) 46. (c) 47. (a) 48. (b) 49. (c) 50. (c)
51. (d) 52. (a) 53. (c) 54. (a) 55. (d) 56. (a) 57. (c) 58. (b) 59. (a) 60. (a)
61. (c) 62. (d) 63. (a) 64. (c) 65. (a) 66. (c) 67. (d) 68. (d) 69. (d) 70. (b)
71. (a) 72. (d) 73. (b) 74. (a) 75. (a) 76. (c) 77. (c) 78. (c) 79. (c) 80. (b)
81. (a) 82. (a) 83. (c) 84. (b) 85. (c) 86. (c) 87. (b) 88. (d) 89. (b) 90. (d)
91. (b) 92. (c) 93. (b) 94. (b) 95. (b) 96. (b) 97. (d) 98. (d) 99. (a) 100. (c)
101. (d) 102. (d) 103. (c) 104. (b) 105. (d) 106. (d) 107. (b) 108. (a) 109. (b) 110. (c)
111. (d) 112. (c) 113. (a) 114. (c) 115. (b) 116. (c) 117. (c) 118. (a) 119. (a) 120. (b)
121. (b) 122. (c) 123. (c) 124. (b) 125. (d) 126. (b) 127. (c) 128. (a) 129. (b) 130. (b)
131. (d) 132. (d) 133. (b) 134. (c) 135. (d) 136. (d) 137. (b) 138. (a) 139. (a) 140. (a)
141. (c) 142. (c) 143. (c) 144. (b) 145. (d) 146. (c) 147. (c) 148. (b) 149. (a) 150. (d)
151. (c) 152. (a) 153. (b) 154. (b) 155. (b) 156. (b) 157. (c) 158. (c) 159. (c) 160. (c)
161. (c) 162. (d) 163. (c) 164. (d) 165. (b) 166. (c) 167. (a) 168. (a) 169. (d) 170. (d)
171. (c) 172. (d) 173. (d) 174. (c) 175. (a) 176. (c) 177. (c) 178. (d) 179. (c) 180. (a)
www.jeebooks.in
NEET Solved Paper 2019 47

Answer with Explanations


Physics
1. (d) The surface charge density of a closed surface area having 3. (d) The distance covered by the mass falling from height ‘h’
charge Q is given by during its time of fall ‘t’ is given by
Charge Q 1
σ= = or Q = σA s = h = ut + gt 2
Area A 2
Thus, the charges on sphere P and Q having same charge density 1 2 2h
As, u = 0 ⇒h = gt ⇒t = …(i)
as shown in the figure below is given by 2 g
σ
The time period of simple pendulum is
σ
l
T = 2π …(ii)
g
R 2R where, l is the length of the pendulum. From Eq. (i) and (ii),
since ‘h’ and ‘l’ are constant so, we can conclude that,
P 1 1 t
Q t∝ and T ∝ ∴ =1
g g T
QP = σ × 4 πR2 = 4 πσR2 …(i)
Thus, the ratio of time of fall and time period of pendulum is
and QQ = σ × 4 π(2R)2 = 16 πσR2 …(ii) independent of value of gravity (g) or any other parameter like
when they are brought in contact with each other, the total mass and radius of the planet. Thus, the relation between t′ and
charge will be T′ on another planet irrespective of its mass or radius will
Q t = QP + QQ = 4 πσR2 + 16 πσR2 [From Eq. (i) and (ii)] remains same as it was on earth i.e.
t ′ = 2T ′
= 20 πσR2 …(iii)
4. (c) For vibrating tunning fork over a resonance tube, the first
In connection of two charged conducting bodies, the potential
resonance is obtained at the length
will become same on both, i.e.
QP QQ QP QQ QP 1 A
= ⇒ = ⇒ =
4 π ∈0 R 4 π ∈0 2R R 2R QQ 2
l1 λ/4
So, the charges on the sphere P and Q after separation will be
distributed as N
1 2
⇒ QP′ = Qt and QQ′ = Qt
3 3
After separation, the new surface charge densities on P and Q
will be λ
l1 = …(i)
Q′ 1 Qt 1 20 πσR2 5 4
σP = P = = = σ
Area 3 Area 3 4 πR2 3 and for second resonance,
QQ′ 2 Qt 2 20 πσR2 2 5 5 A
and σ Q = = = × = × σ= σ
Area 3 Area 3 4 π(2R)2 3 4 6
λ/4
2. (d) In a simple harmonic motion (SHM) the particle N
oscillates about its mean position on a straight line.
The particle moves from its mean position (O) to an extreme l2
position (P) and then return to its mean position covering same
distance of A. λ/2
Then by the conservative force, it is moved in opposite direction
to a point Q by distance A and then back to mean position
covering a distance of A. This comprises of one time period as
shown below
λ λ 3λ
O l2 = + = …(ii)
P 4 2 4
Q From Eq. (i) and (ii), we get
3λ λ λ
l 2 − l1 = − = ⇒ λ = 2(l 2 − l1)
A A 4 4 2
 v
extreme mean extreme ⇒ v = 2 f (l 2 − l1) …(iii) Q λ = 
position position position  f
In one time period Here, f = 800 Hz, l1 = 9.75 cm, l 2 = 31.25 cm
Hence, in one time period it covers a distance of Substituting the given values in Eq. (iii), we get
x = OP + PO + OQ + QO = A + A + A + A = 4 A ⇒ v = 2 × 800(31.25 − 9.75) = 34400 cm/s = 344 m/s
www.jeebooks.in
48 NEET Test Drive

5. (a) Let m be the mass of an object flying with velocity v in air. 2(ρ2 − σ)r22 g
vt2 = …(ii)
When it split into two pieces of masses in ratio 1 : 5, the mass 9η
5m From Eq. (i) and (ii), we get
of smaller piece is m/6 and of bigger piece is .
6 vt1 2(8ρ2 − σ)r12 g 9η
This situation can be interpreted diagrammatically as below = ×
vt2 2(ρ2 − σ)r22 g 9η
v1 2
 8ρ − 01. ρ2   r1 
m/6 = 2   …(iii) [Qσ = 0.1ρ2]
 ρ2 − 01
. ρ2   r2 
m v Here, r1 = 1 mm and r2 = 2 mm
Substituting these values in Eq. (iii), we get
5m/6
vt1  7.9ρ2   1  2 79
v2 ⇒ =   =
As the object breaks in two pieces, so the momentum of the vt
2
 0.9ρ2   2 36
system will remains conserved i.e. the total momentum (before
8. (c) From third equation of motion for circular motion
breaking) = total momentum (after breaking)
m 5m v 5v 2 ω2 − ω20 = 2αθ …(i)
mv = v1 + v2 ⇒ v = 1 +
6 6 6 6 where, ω = final angular velocity of particle
Here, v = 20$i + 25$j − 12k
$ ω0 = initial angular velocity
α = angular acceleration and
v1 = 100$i + 35$j + 8k $
θ = angular displacement
$ = (100 i + 35$j + 8k) + 5v 2
$ $ v
⇒ 20$i + 25$j − 12k Here, ω = 0 rad/s (where, r radius of the circle)
6 6 r
⇒ (120$i + 150$j − 72k$ ) = (100$i + 35$j + 8k$ ) + 5v
2
ω0 = 0 (initially particle is at rest)
1 θ = 2πn (for n rounds)
⇒ v 2 = (20 i + 115 j − 80k)
$ $ $
5 Substituting these values in Eq. (i), we get
2
= 4$i + 23$j − 16k $  v0  − 0 = 2α(2πn)
 
 r
6. (d) Key Idea From Newton’s law of cooling, the time taken (t ) v02
by a body to cool from T1 to T2 when placed in a medium of ⇒ α= rad/s 2
temperature T0 can be calculated from relation 4 πnr 2
T 1 − T 2 1  T1 + T2
− T0 
= 
9. (d) Let h be the height through which the coin is dropped.
t K 2  Then, according to the equation of motion, it is given as
1 2h
h = ut + gt 2 ⇒ t = [Qu = 0]
When the object cool from 80°C to 70°C in 12 minutes, then 2 g
from Newton’s law of cooling, 1
80 − 70 1  80 + 70 ⇒ t∝
=  − 25 [QT0 = 25° C] g
12 K 2 
As the elevator is moving uniformly i.e. its velocity is constant,
5 1
= 50 …(i) so the acceleration is zero.
6 K ∴Relative acceleration of the lift when it is either moving
Similarly, when object cool from 70°C to 60°C we get upward or downward is given as, g′ = g ± a = g ± 0 = g
70 − 60 1  70 + 60
=  − 25 Hence, the time for the coin to reach the floor will remains
t K 2  same i.e. t1 = t2.
10 1
= 40 …(ii) 10. (b) As the truck move to the right, so the bob will move to
t K the left due to inertia of rest with acceleration a.
Divide Eq. (i) and (ii), we get Thus, the given situation can be drawn as
5 t 50 5
× = ⇒ t = × 12 = 15 minutes
6 10 40 4 ma θ
a ⇒
7. (d) The terminal velocity achieved by ball in a viscous fluid is θ
2(ρ − σ)r 2 g mg
vt =
9η (a) (b)
where, ρ = density of metal of ball, From the above diagram (b) as the string moves by an angle ofθ
σ = density of viscous medium, with the vertical then the tangent angle is
r = radius of ball and ma
tanθ =
η = coefficient of viscosity of medium mg
Terminal velocity of first ball,  a
2(ρ − σ)r12 g 2(8ρ2 − σ)r12 g ⇒ θ = tan−1  
vt1 = 1 = …(i) [Qρ 1 = 8ρ2]  g
9η 9η
Similarly, for second ball
www.jeebooks.in
NEET Solved Paper 2019 49

11. (b) According to Pascal’s law ‘‘Pressure applied to an +6V


enclosed fluid is transmitted undiminished to every point of R
the fluid and the walls of the containing vessel.’’ A 0
In the given situation as shown in the figure below 1
B 0
1
LED (Y)

R
20 cm
15 cm Thus, the truth table for the circuit diagram can be formed as
circuit diagram in given solution.
A B Y
Water Oil
0 0 1
Pressure due to water column of height 15 cm = Pressure due to
oil column of height 20 cm 0 1 0
⇒ hwρw g = h0ρ0 g 1 0 0
15
15ρw = 20ρ0 ⇒ ρ0 = ρω
20 1 1 0
15
ρ0 = × 1000 (Qgiven, ρw = 1000 kg m −3)
20 The output (Y) is equivalent to that of NOR gate.
= 750 kgm −3 15. (a) The poisson’s ratio,
CP
12. (c) Key Idea If area of cross-section of a surface is not uniform γ= …(i)
or if the steady state condition is not reached, the heat flow CV
equation can be applied to a thin layer of material where, Cp = molar heat capacity constant pressure
perpendicular to direction of heat flow. and CV = molar heat capacity at constant volume
The rate of heat flow by conduction for growth of ice is given Also, Cp = CV + R (from Mayer’s relation)
by, f
CV = R (where, f =degree of freedom)
dθ KA(θ 0 − θ 1)] 2
=
Cp =  + 1 R
dt x f

where, dθ = ρAdxL, θ 0 = 0 and θ 1 = − θ 2 
So, Eq. (i) becomes,
Given, θ0 = 0° C, θ1 = − 26° C 2
⇒ γ =1 +
The rate of increase of thickness can be calculated from Eq. f
dθ KA(θ0 − θ1)
= For hydrogen gas, which is diatomic, the degree of freedom is 5
dt x (3 translational, 2 rotational).
ρAdxL KA(θ0 − θ1) 2 7
⇒ = ∴ γ =1 + =
dt x 5 5
dx KA(θ0 − θ1) K [0 − (−26)] 26K For helium gas, which is monoatomic, the degree of freedom is
⇒ = = =
dt ρAxL ρxL ρxL 3 (3 translational only).
2 5
13. (b) The energy of light of wavelength λ is given by ∴ γ =1 + =
3 3
hc hc
E = hν = ⇒ λ= …(i) The diatomic gas X also have vibrational motion, so degree of
λ E freedom is 7 (3 translational, 2 rotational and 2 vibrational).
Here, h = Planck’s constant = 6.63 × 10−34 J-s 2 9
∴ γ =1 + =
c = speed of light = 3 × 108 m/s 7 7
E = energy gap = 1.9 eV = 1.9 × 1.6 × 10−19 J 16. (d) When the lens is cut along its principal axis, the focal
Substituting the given values in Eq. (i), we get length of the two halves will remain same because the radius
6.63 × 10−34 × 3 × 108 of curvature of both the surfaces are still same. So, the power
⇒ λ= also remains same as
1.9 × 1.6 × 10−19
1
= 6.54 × 10−7 m ≈ 654 nm P=
f
Thus, the wavelength of light emitted from LED will be 654 f
nm.

14. (a) From the circuit diagram given below, it can be seen that
the current will flow to ground if any of the switch is closed.
Also, the LED will only glow when current flows through it.
f f
www.jeebooks.in
50 NEET Test Drive

17. (c) In a YDSE, the path difference for nth minima is given by 20. (b) The stress-strain curve for a material is shown
λ
∆y = (2n − 1)
2 Stress Elastic D Ultimate
For 5th minima, n = 5 B limit strength point
λ 9λ A C
∴ ∆y = [2(5) − 1] = E Fracture point
2 2 Plastic behaviour
Elastic behaviour
18. (c) Using the lens maker’s formula
1  µ 2 − µ1   1 1 
=  −  O
f  µ 1   R1 R2  Strain

Here, f = 25cm This curve specifies the behaviour of material.


µ 2 = 1.5, µ 1 = 1 (for air) For the material, if distance between strength point and
let R1 = R and R2 = − 2R fracture point is small, so it is brittle and will break easily on
1.5 − 1   1  1   the application of some extra stress after point D.
= 
1
⇒  − −  For material Y, if the distance between strength point and
25  1   R  2R  
fracture point is large, so it is a ductile material and can
1 1 1
= + withstand for some extra stress beyond point D.
25 × 0.5 R 2R
3 21. (c) The situation can be drawn as
⇒ R = × 25 × 0.5
2 F N
= 18.75 cm
∴ R1 = 18.75 cm
R2 = 2 × 18.75 f FH
= 37.5 cm

19. (a) Given, distance between the two buildings


d = 100 m mg
height of each tower, h = 200 m The frictional force, f = µN = µ mg [QN = mg]
speed of each bullet, v = 25 ms−1 From Free body diagram (FBD), the resultant force is
The situation can be shown as below |F| = N 2 + f 2 = (mg)2 + (µmg) 2 = mg 1 + µ 2
This is the minimum force required to move the object. But as
x 25 m/s the body is not moving
∴ |F|≤ mg 1 − µ 2
200 m 200 m
22. (d) The particle of mass 5m breaks in three fragments of
mass m, m and 3m respectively. Two fragments of mass m
each, move in perpendicular direction with velocity v and the
100 m left fragment will move in a direction with velocity v′ such
where, x be the vertical distance travelled from the top of the that the total momentum of the system must remain
building and t be the time at which they collide. conserved.
As two bullets are fired toward each other, v
So, their relative velocity will be
v
vrel = 25 − (−25) = 50 ms−1 5m m m
d 100
Then, time t = = = 2s v=0
vrel 50 3m
v
The distance or height at which they collide is calculated from By law of conservation of momentum,
equation of motion,
5m × 0 = mv$i + mv$j + 3mv ′
1
x = ut + at 2 v v
2 ⇒ v ′ = − $i − $j
3 3
The bullet is initially a rest i.e. u = 0 and as it is moving under 2 2
the effect of gravity a = − g, so ∴ |v′| =  − v +  − v = v 2
   
1  3  3 3
x = − gt 2
2 ∴ Energy released
1 2
x = − × 10(2)2 = − 20 m 1 2 1 2 1  v 2
2 E= mv + mv + × 3m 
2 2 2  3 
The negative sign shows that the bullets will collide 20 m below
the top of tower i.e. at a height of (200 − 20) = 180 m from the mv2 4 2
= mv2 + = mv
ground after 2 s. 3 3
www.jeebooks.in
NEET Solved Paper 2019 51

23. (c) The area under the force displacement curve give the From the above diagram
amount of work done. h
sinθ =
x
⇒x =
h
=
1.2
= 1.2 × 2 = 2 . 4 m Qsin 30° = 1 
A B
20 sinθ sin 30°  2

10
C E K L 25. (a) The magnetic field within the turns of toroid is
µ 0 NI
O D F J M B=
0
5 8 10 12
x(m) 2πr
4
–10 where, N = number of turns, I = current in loops and
r = radius of each turn
G Here, N1 = 200, N 2 = 100 , r1 = 40 cm, r2 = 20 cm
–20 I
and current I is same, then
–25
H B1 µ 0 N1 I 2πr2
= ×
From work-energy theorem, B2 2πr1 µ 0 N 2I
W = ∆KE …(i) Substituting the given values in the above relation, we get
∴At x = 8 m, B1  N1   r2   200   20  1
=    =    = 2 × =1
W = Area ABDO + Area CEFD = 20 × 5 + 10 × 3= 130 J B2  N 2   r1   100   40  2
Using Eq. (i) ∴ B1 : B2 = 1 : 1
1 1 500 2
⇒ 130 = mv2 = × v
2 2 1000 26. (a) The magnetic field at the centre of an arc subtended at an
⇒ v = 2 130 = 22.8 ms ≈ 23 ms−1
−1 angle θ is given by
µ 0i θ
At x = 12m B= ×
2R 2π
W = Area ABDO + Area CEFD + Area FGHIJ
+ Area KLMJ
i1
 1 
W = 20 × 5 + 10 × 3 + (−20 × 2) +  × − 5 × 2 + 10 × 2 270°
2  R
P
[QArea FGHIJ = Area FGIJ + Area GHI] i A 90°
= 100 + 30 − 40 − 5 + 20 = 105 J R
i2
Using Eq. (i)
B
1 1
∴ 105 = × × v2 ⇒ v = 2 105 ~ − 20.6 ms−1 i
2 2
Then, the magnetic field due to larger arc AB is
24. (d) When a body rolls i.e. have rotational motion, the total µ i 270
kinetic energy of the system will be B1 = 0 1 × …(i)
2R 2π
1  k2 
KE = mv2 1 + 2  which acts in inward direction according to right hand thumb
2  R  rule.
where, m = mass of body, v = velocity and And magnetic field due to smaller arc AB is
k = radius of gyration µ i 90
B2 = 0 2 × …(ii)
2R 2π
which acts in outward direction.
The resultant magnetic field
x h BR = B1 + B2
–1 µ i × 270 µ 0 i 2 × 90
ms
=− 01 + [From Eq. (i) and (ii)] …(iii)
v=
4 4 πR 4 πR
30°
which acts in inward direction as B1 > B2.
Given, m = 2kg, θ = 30°, v = 4 ms−1 Two arcs can also be seen as the two resistances in parallel
combination.
Let h be the height of the inclined plane, then from law of
So, the potential across them will be same i.e.
conservation of energy,
V1 = V2
KE = PE
i1 R1 = i 2R2 …(iv)
1 2 k2 
mv 1 + 2  = mgh where, R1 and R2 = Resistance of respective segments
2  R  The wire is uniform so
Substituting the given values in the above equation, we get R1 L1 R × 270
= = [Qlength of arc = radius × angle]
k2 1 R × 90
× 2 × 161 +  = 2 × 10 × h [QFor cylinder 2 = ]
1 1 R2 L2
2  2 R 2 From Eq. (iv), we get
3 90 1
⇒ 8 × = 10 h ⇒ h = 1.2 m ⇒
i1 R2
= = = or 3i1 = i 2 …(v)
2 i 2 R1 270 3
www.jeebooks.in
52 NEET Test Drive

From Eq. (iii) and (v), we get e4


where, k = = constant.
µ µ 8ε20 h2
BR = 0 (−270i1 + 90i 2) = 0 [−270i1 + 90(3i1)]
4 πR 4 πR For muon in first orbit
µ E1 ′ = kmµ ′
= 0 (−270i1 + 270i1) = 0
4 πR = k × 186me [from Eq. (i)]
= 186kme
27. (b) The emf generated due to a rotating conductor in a
= 186E1 [from Eq. (iv)]
generator is given by
= 186(−136
. ) eV (given)
dφB = − 2529.6 eV
E=−
dt = − 2.5keV
where, φB = magnetic flux linked with conductor ∴ The values are closest to that of options (c).
From the above equation we can conclude that the emf
generated is a varying function of time with opposite polarity. 29. (d) The given circuit diagram can be drawn as shown below
So, all the graph are correct for the given variations. I1 20Ω 30Ω
B
A E
28. (c) Key Idea Hydrogen atom can be considered to be the
system of two charges, positive charged nucleus and negative
V
charged electron. A system of this kind is equivalent to a single
particle of mass m′ that revolves around the position of the 30Ω 20Ω
D F
heavier particle. Then, the reduced mass of electron is I2 C
mM I
m′ =
m+ M
where, m = mass of electron and
M = mass of nucleus
Its values is less than m. 2V
The equivalent resistance of circuit is given by
Given, radius of first orbit for electron r1 = 0.51 Å, 1 1 1
= +
ground state energy of electron, E1 = − 136 . eV, Req RAE RDF
mass of electron = me 1 1
= +
mass of muon, mµ = 207me and (20 + 30) (30 + 20)
mass of nucleus, M = 1836 me 1 1 2
When electron in hydrogen atom is replaced by muon, the = + =
50 50 50
reduced mass of muon is
mµ M ⇒ Req = 25 Ω
mµ ′ = …(i) The current in circuit, I = =
V 2
A
mµ + M R 25
Substituting the given values in Eq. (i), we get As the resistance of two branches is same i.e. 50 Ω.
207me × 1836me So, the current I1 = I 2
mµ′ = ≈ 186 me …(ii)
207me + 1836me ⇒ I = I1 + I 2
The radius of first orbit in hydrogen atom for electron is given 2 1
= 2I1 ⇒ I 2 = I1 = A
by, 25 25
h2ε0 ∴The voltage across AB
r1 = …(iii) 1
πme e 2 V1 = I1 R1 = × 20
25
The radius of first orbit for muon is
and voltage across CD
h2ε0
r1 ′ = [Qcharge of µ = charge of e − ] V2 = I 2R2 =
1
× 30
πmµ′ e 2 25
h2ε0 30 20 10
= [from Eq. (ii)] ∴Voltmeter reading = V2 − V1 = − = = 0.4 V
π × 186me e 2 25 25 25
 h2ε0  1 r 30. (d) For balanced position in a meterbridge
= 2
 = 1 [from Eq. (iii)]
 πme e  186 186
0.51
= Å [Q r1 = 0.51Å]
186 P Q
= 2.74 × 10−13 m [Q1 Å = 10−10 m]
The total energy of electron is given by G
− mZ 2e 4  1 
En =   ⇒ En ∝ m l1 l2
8ε20 h2  n2 
P l1
For electron in first orbit of hydrogen atom, =
E1 = kme …(iv) Q l2
www.jeebooks.in
NEET Solved Paper 2019 53

Now, if position of G and cell is interchanged, When it is connected to DC source,


Voltage, V = 12V
G
Current, I = 0.4 A
V 12
⇒ Resistance, R = = = 30 Ω
P Q I 0.4
As in case of DC supply, the capacitor act as an open circuit and
no current flows through the circuit. So the given circuit will
not have capacitor in series combination. Therefore the circuit
l1 l2
should be a series LR circuit.

The balance condition still remains the same if the jockey 35. (b) When a conducting disc or wheel of radius r rotates with
points as the same point as given in the initial condition, for constant angular velocity of ω about its axis in a uniform
which there is no deflection in the galvanometer or no current magnetic field perpendicular to its plane and parallel to its
will be drawn from the cell. Thus, the bridge will work as usual axis of rotation, then,
and balance condition is same, P / Q = l1 / l 2

31. (b) Let BE be the net magnetic field at H


δ ω=10 rad s–1
same point. H and V be the horizontal
and vertical components of BE . Let δ
be the angle of dip, which is the angle
between direction of earth’s magnetic V P
r=0.5m
field BE and horizontal line in the
magnetic meridian.
Thus, from figure, we can see that
H = BE cosδ and V = BE sinδ BE
Axis
32. (d) Given, half life T1 / 2 = 2.2 × 109 s 1
Induced emf is given by, e = Bωr 2 …(i)
10 −1
Rate of disintegration, R = 10 s 2
If N be the number of nuclei present, then the rate of Here, B = 01
. T, ω = 10 rad/s, r = 0.5 m
disintegration is Substituting these values in Eq. (i), we get
1 1
dN
= λN (λ = decay constant) ∴ e = × 01 . × 10 × (0.5)2 = V = 0125. V
dt 2 8
R
⇒ R = λN or N = …(i) 36. (a) The speed of any electromagnetic wave (EM) like light in
λ
any medium of permeability, µ and permittivity, ε is given by
Also, the half life is given by, 1
0.693 0.693 v=
T1 / 2 = ⇒ λ= …(ii) µε
λ T1 / 2
Also, µ = µ 0µ r and ε = ε0 εr
From Eq. (i) and (ii), we get
where, µ 0 and ε0 are permeability and permittivity in free space
R 1010 × 2.2 × 109 and µ r and εr are relative permeability and relative permittivity
N= × T1 / 2 = . × 1019
= 317
0.693 0.693 of medium respectively.
Here, µ r = 1.0
33. (a) From Kepler’s third law, the time period of revolution of
satellite around earth is εr = 1.44
1  1 
T 2 ∝ r 3 or T ∝ r 3/ 2 …(i) ∴ v= =
c
Q c = 
where, r is the radius of satellite’s orbit. µ r εr µ 0 ε0 µ r εr  µ 0 ε0 
Here, r1 = 6RE + RE , T1 = 24 h 3 × 108
= = 2.5 × 108 ms −1
r2 = 2.5RE + RE , T2 = ? 1 × 1.44
where RE = radius of earth
So, from Eq. (i), we get 37. (b) When a sphere encloses a charged dipole,
3/ 2 3/ 2 3/ 2
T1  r1  24  6RE + RE 
=  
7
=  ⇒ = 
T2  r2  T2  2.5RE + RE   35
. 
24 24 12
⇒ T2 = 3/ 2 = = = 6 2h +q –q
(2) 2 2 2

34. (a) When the circuit is connected to AC source,


Voltage, V = 12V
Current, I = 0.2 A
V 12
⇒ Impedance Z = = = 60 Ω
I 0.2 Here, q = ± 3 × 10−6 C
www.jeebooks.in
54 NEET Test Drive

Thus, according to Gauss’s law, the net electric flux across the total distance D
The average velocity, Vav = =
closed surface is equal to the net charge enclosed by it divided total time T
by ε0 , i.e. 2x 2
q +3 × 10−6 − 3 × 10−6 v= = [Qvav = v]
φE = in = =0 x x 1 1
+ +
ε0 ε0 v1 v2 v1 v2
Hence, electric flux across the sphere is zero. 1 1 2
⇒ + =
v1 v2 v
38. (c) When C1 is connected to voltage source, it is charged to a
potential V and this will be stored as a potential energy in the 41. (b) The gravitational potential energy of an object placed at
capacitor given by earth’s surface is
1
U = CV 2 U1 = −
GMm
…(i)
2 R
a k c
where, G = gravitational constant,
b M = Mass of earth, m = mass of object and
V C2 R = radius of the earth
C1 The negative sign in the above relation indicates that it is the
work done in bringing the object from infinity to a distance R.
The gravitational potential energy of object at a height h above
When key is disconnected from battery and b and c are the surface of earth is
connected, the charge will be transformed from the capacitor GMm
C1 to capacitor C2, then U2 = − …(ii)
(R + h)
a k c
So, the change in potential energy is
b GMm  GMm 
∆U = U 2 − U1 = − − −  [From Eq. (i) and (ii)]
V C2 R+ h  R 
C1  1 1  h  GMmh
= − GMm  −  = − GMm − =
 (R + h) R   R(R + h)  R(R + h)
The loss of energy due to redistribution of charge is given by
C1 C2 42. (b) Given, mass of water, m = 1 g
∆U = (V1 − V2)2
2(C1 + C2) Volume of 1 g of water = 1 cm 3 = 10−6 m 3
C×C 1 Volume of 1 g of steam = 1671 cm3 = 1671 × 10−6 m3
= (V − 0)2 = CV 2 [QC1 = C2]
2(C + C) 4 Pressure, p = 1 × 105 Pa
1
CV 2 Latent heat of vaporization of water, L = 2256 J/g
∆U 4
∴Percentage loss = × 100 = × 100 = 50% Change in volume, ∆V = (1671 − 1) × 10−6 m3
U 1 2
CV
2 = 1670 × 10−6 m3 …(i)
39. (c) As it is given that n divisions of vernier scale coincide Heat supplied, ∆Q = mL = 1 × 2256 = 2256 J …(ii)
with (n − 1) divisions of main scale i.e. As the steam expands, so the work done in expansion is
n(VSD) = (n − 1) MSD ∆W = p∆V = 1 × 105 × 1670 × 10−6 [from Eq. (i)]
(n − 1) = 167 J …(iii)
⇒ 1VSD = MSD …(i)
n According to first law of thermodynamics,
The least count is the difference between one main scale ∆Q = ∆U + ∆W ⇒ ∆U = ∆Q − ∆W
division (MSD) and one vernier scale division (VSD). = 2256 − 167 = 2089 J [from Eq. (ii) and (iii)]
∴Least Count (LC) = 1 MSD − 1VSD
(n − 1) 43. (b) The angular width of central maxima is given by
= 1 MSD − MSD [From Eq. (i)] 2λ
n 2θ = …(i)
(n − 1)  a
= 1 −
1
 MSD = MSD where, λ = wavelength of light used
 n  n
1 1 1 1 a = width of the slit
Here,1 MSD = cm ⇒ LC = × cm = 2 cm For λ1 = 6000Å, 2θ = θ0 (given)
n n n n
For another light of wavelength λ 2 (says), the angular width
40. (b) For distance x, the person moves with constant velocity v1 decreases by 30% so,
100 − 30 
2θ = 
and for another x distance, he moves with constant velocity 70
θ = θ = 0.7θ0
of v2, then  100  0 100 0
Total distance travelled, D = x + x = 2x As slit width is constant, so using Eq. (i) for these values, we
Total time-taken, T = t1 + t2 get
x x  Distance  θ0 λ
= + = 1
Q t = Velocity  0.7θ0 λ 2
v1 v2  
⇒ λ 2 = λ1 × 0.7 = 6000 × 0.7 = 4200 Å
www.jeebooks.in
NEET Solved Paper 2019 55

44. (d) The work function of material is given by 47. (a) Secondary amines on reaction with Hinsberg’s reagent
φ = hν (benzene sulphonyl chloride) forms N,N-dialkyl benzene
sulphonamide. The product formed is not acidic as no
…(i) Q ν = 
hc c
φ=
λ  λ  H-atom is attached to N thus it is insoluble in alkali.
where, h = Planck’s constant = 6.63 × 10−34 J-s The reaction of 2º amine given in option (a) takes place as
8 −1 follows:
c = speed of length = 3 × 10 ms and
λ = wavelength of light
Here, φ = 4 eV = 4 × 1.6 × 10−19 J —S—Cl+CH3—CH—NH—CH—CH3
Substituting the given values in Eq. (i), we get CH3 CH3
6.63 × 10−34 × 3 × 108
⇒ 4 × 1.6 × 10−19 = 2º amine
λ
6.63 × 10−34 × 3 × 108 CH3
or λ = = 3108
. × 10−7 m ~
− 310 nm
4 × 1.6 × 10−19 —S———N—CH—CH3 + HCl
45. (a) The de-Broglie wavelength associated with a charged H3C– CH
particle is given by
h CH3
λ=
p On the other hand, 1º amines react with Hinsberg’s reagent to
where, h = planck’s constant and yield N-alkyl benzene sulphonamide which is soluble in alkali
p = momentum = 2mKE (here, KE is the kinetic energy of the and 3º amines do not react with C6H 5SO 2Cl. The remaining
charged particle) options contain 1º and 3º amines.
h CH 2CH 3 NH 2
⇒ λ=
2mKE  
CH 3CH 2 N  CH 2 CH 3 , CH 3  C CH 2 CH 2CH 3,
For proton and α-particle, the wavelengths are respectively 
3º amine
given as, CH 3
h h 1º amine
λp = and λ α =
2mpKE p 2mαKE α CH 3
λp 2mαKE α 
∴ = …(i) CH 3 C  CH  NH 2
λα 2mPKE P  
Here, KE α = KE P and mα = 4mp CH 3 CH 3
1º amine
Substituting these above mentioned relations in Eq. (i), we get
Thus, option (a) is correct.
λP 4mP 2
⇒ = = or λ P : λ α = 2 : 1
λα mp 1 48. (b) The loss in biological activity of proteins due to unfolding
of globules and uncoiling of helix is called denaturation of
proteins. During the denaturation, primary structure remains
Chemistry intact but secondary and tertiary structures are destroyed.
46. (c) Primary aromatic alcohols on reaction with pyridinium e.g. The coagulation of egg white on boiling, curding of milk
chlorochromate (PCC) which is a mild oxidising agent forms caused by bacteria due to formation of lactic acid.
aromatic aldehydes. CN
CH2OH CHO 
PCC
49. (c) Polyacrylonitrile or orlon or acrilan, ( CH 2 CH )n is
(Pyridinium an addition homopolymer of monomer CH 2 == CH CN (vinyl
chlorochromate)
Benzaldehyde cyanide). It is used in making synthetic fibres and synthetic
In the remaining options benzoic acid is formed as follows: wool. Thus, it is a substitute for wool in making commercial
fibres.
CH2OH COOH
K2Cr2O7 50. (c) Sucralose is a trichloro derivative of sucrose and is about
650 times sweeter than cane sugar. It is a zero calorie sugar
and stable at cooking temperature.
Benzoic acid H
COCH3 COOH OH
(i) NaOCl O
+ CHCl3 Cl H OH H
(ii) H3O+ OH
Benzoic acid HO
OH CH2Cl
CH2OH COOH H O
H HO
KMnO4/H+

ClH2C O H
Benzoic acid Sucralose
Thus, option (c) is correct.
www.jeebooks.in
56 NEET Test Drive

51. (d) Solvents used to dry clean clothes are usually chlorinated 58. (b) Key Idea Alkanes which contain all equivalent hydrogen
atoms forms only one monochloro derivative on
compounds which are carcinogenic. Suitable detergents
halogenation.
which work in liquid carbon dioxide have been discovered to
replace the chlorinated compounds. Thus, CO 2 is the liquified CH 3
gas that is used in dry cleaning along with a suitable 
detergent. Neo-pentane, H 3C  C  CH 3 contains all

52. (a) —Cl is a benzylic halide. Thus, there is a partial CH 3
equivalent hydrogen atoms. So, it will give only one
double bond character between sp 3-hybridised C atom next to monochloro derivative on halogenation.
an aromatic ring and Cl. It is most difficult to break this bond
and hence it undergoes hydrolysis reaction with slowest rate. 59. (a) Key Idea Single bond = 1σ , Double bond = 1σ , 1π; Triple
bond = 1σ and 2 π.
53. (c) When vapours of alcohols are passed over heated copper Alkynes form aromatic compounds when their vapours are
at 573 K, primary and secondary alcohols undergo
passed over red hot copper or iron tubes.
dehydrogenation to give aldehydes and ketones, respectively.
CH3
While tertiary alcohols undergo dehydration to give alkenes.
H3C H Cu/573 K
H3C
C =O+H2↑
C= 3CH3—C≡≡CH
Red hot iron tube
H3C OH H3C 873 K
H 3C CH3
Propan-2-ol Propanone
Mesitylene
(2°) (Ketone)
(A)
Thus, option (c) is correct. The number of sigma (σ) bonds present in the product ( A) is 21.
H
54. (a) Ethers are readily cleaved by heating in presence of σ|
halogen acids to form alcohol and an alkyl halide. In case of σ
H—C—H
σ
unsymmetrical ethers, halogen goes preferentially with σ
smaller alkyl group or more stable carbocation. H σ σ σ σ H
If excess of acid is used then only alkyl halide is formed π
because alcohol formed reacts further with halogen acid to H σ π σ H
σ| σ π σ |σ
form corresponding alkyl halide.
σ C σ σ C
HI
CH3CH2CH2OC(CH3)3 CH3CH2CH2OH + (CH3)3 I σ σ| σ H
∆ H σ|
‘D’ H H
∆ Excess H
HI

CH3CH2CH2I 60. (a) Aluminium chloride in acidified aqueous solution forms


‘C’ an octahedral complex. Aqueous solution is mostly water and
when the compound is dissolved in acidified aqueous
55. (d) Key Idea Metal oxides are basic, non-metal oxides are solution, the water fills its vacancies and an octahedral
acidic while semi-metal oxides are amphoteric in nature. complex, ‘A’ which is [Al(H 2O) 6]3+ is formed.
Thus, the basic character of oxides decreases across the
period and increases down the group. Al(13) =1s ,2 2s 2 , 2p 6 , 3s 2 , 3p1

The correct match of oxide with its property are as follows: Al 3+ =1s 2 , 2s 2 , 2p 6.
Na 2O- Basic 2s 2p

Al 2O 3- Amphoteric Al3+=
N 2O- Neutral [Al(H2O)6]3+=
Cl 2O 7- Acidic
2s 2p 3s 3p 3d
Thus, option (d) is correct.
56. (a)(1) V2 O 5 –(iii) Oxidation of SO 2 in the manufacture of
H 2SO 4 by contact process. H2O H2O H2O H2O H2O H2O
(2) TiCl 4 + Al(CH 3)3– (Ziegler-Natta catalyst)– (iv)
sp3d2
Polymerisation of ethylene.
Thus, the hybridisation state of Al in [Al(H 2O)6]3+ is sp 3d 2 (B).
(3) PdCl 2–(i) Oxidation of ethyne to ethanal.
(4) Nickel complexes–(ii) Polymerisation of alkynes. 61. (c) Silicones are synthetic organo-silicon polymers contaning
Thus, the correct match is repeated R2SiO units. It is used in surgical and cosmetic
(1) → (iii), (2) → (iv), (3) → (i), (4) → (ii) implants. They are also used as low temperature lubricants,
+ in making waterproof clothes and paper, etc.
57. (c) CH 3 CH CH 2 CH 2 CH 3 is the most stable carbocation
62. (d) The earthly impurities like sand, clay, mica, etc.,
among the given carbocations. It is because the number of
associated with ores are called gangue or matrix. In other
α-H atom attached to carbocation is maximum in
+ words, contaminated undesired materials present in an ore is
CH 3 CH CH 2 CH 2 CH 3. Thus, it has maximum number of called gangue. Thus, statement (d) is incorrect while other
hyperconjugating structure hence, it is most stable. options contain correct statements.
www.jeebooks.in
NEET Solved Paper 2019 57

63. (a) The given road map problem is Given λºm( H = 2λ°H + + λºSO 2 − = x S cm2mol −1 …(i)
2SO4 ) 4
‘X’ + H2O ‘Y’ (colourless) λºm( K = 2λºK + + λºSO 2 − = y S cm2mol −1 …(ii)
Rotten fish smell 2SO4 ) 4
−1
CuSO4 λºm(CH 3COOK) = λºCH COO − + λºK + = z S cm mol
2
…(iii)
3

Cu3P2 + Product To find : λºm( CH 3 COOH) = λºCH COO− + λºH+


3

As compound ‘X’ produces a colorless gas ‘Y’ with rotten fish The above equation can be obtained by
smell, it can be Ca 3P2 (calcium phosphide). The reaction takes Eq. (i) Eq. (ii)
Eq. (iii) + −
palce as follows : 2 2
Ca 3P2 + H 2O → Ca(OH) 2 + PH 3 λº λº
∴ λºm( CH 3 COOH) = λºm ( CH COOK) + m( H 2SO4 ) − m( K 2SO4 )
Colorless gas with rotten fish smell. ‘Y’ 3 2 2
CuSO 4 + PH 3 → Cu 3P2 + H 2SO 4  x y −1
=  z + −  S cm mol 2
'Y '
 2 2
64. (c) P H bonds found in phosphorus acids have reducing x−y
properties. Thus, reducing property is directly proportional to =  + z  S cm 2 mol −1
 2 
number of P H bonds. The structures of given oxoacids of
phosphorus are as follows: 69. (d) For first order reaction,
O O O 2.303 a
t= log …(i)
k a− x
P P P
HO O OH H OH Given : k = 2. 303 × 10−3 s−1 , a = 40 g, a − x = 10 g
HO OH OH
(H4P2O7) (H3PO3) On substituting the given values in Eq. (i), we get
2. 303 40
O O t= log
2. 303 × 10−3 10
P P
H OH HO OH = 103 log 22
H HO = 2 × 103 × log 2
(H3PO2) (H3PO4)
= 2 × 103 × 0.3010 = 602 s
H 3PO 2 contains 2P H bonds which is maximum among given Alternative method
options. Thus, H 3PO 2 has strongest reducing property.
For first order reaction,
65. (a) The correct formula of oleum is H 2 S2O 7 which is also 0.693
t1 / 2 =
known as pyrosulphuric acid. k
0.693
O O t1 / 2(t50%) = = 301s
2.303 × 10−3
HO—S—O—S—OH Also, t75% = 2t50%
O O
∴ t75% = 2 × 301 = 602 s

66. (c) When neutral or faintly alkaline KMnO 4 is treated with 70. (b) Key Idea Rate constants at two different temperatures is
given by Arrhenius equation as follows:
potassium iodide, KMnO 4 is converted into MnO 2 while k2 Ea  1 1 
iodide (I −) ion is converted inot IO −3 (iodate). The reaction log = − , T2 > T1
k 1 2.303R  T1 T2 
takes place as follows:
Given, E a = 0, T1 = 200 K, k1 = 1.6 × 106 s−1

KMnO 4 + I – + OH → MnO 2 + IO –3 + H 2O
T2 = 400 K, R = 8.314 JK −1 mol −1
' X'
4− 2−
67. (d) [CoCl 6] is an octahedral while [CoCl 4 ] is a tetrahedral According to Arrhenius equation,
complex.
k Ea  1 1
∆ octahedral =18000 cm −1 log 2 = −
k1 2.303R  T1 T2 
We know that, On substituting the given values in above equation, we get
4 4
∆tetrahedral =∆ octahedral = × 18000 cm −1 = 8000 cm −1 k2 0  1 − 1 
9 9 log =
1.6 × 106 2.303 × 8.314  200 400 
Hence, option (d) is correct.
log 
k2 
68. (d) Key Idea According to Kohlrausch’s law,  =0
 1.6 × 106 
λºm for A x By = xλº( A ) y + + yλº( B ) x − k2
= 100 = 1
where, λºm = limiting molar conductivity of electrolyte and 1.6 × 106
λº( A ) y + and λº( B ) x − are the limiting molar conductivities of k2 = 1.6 × 106 s−1 at 400K
cation ( Ay + ) and anion ( B x − ), respectively. Thus, option (b) is correct.
www.jeebooks.in
58 NEET Test Drive

71. (a) According to Freundlich adsorption isotherm, 76. (c) NaOH is a strong base, thus
x
= kp1 / n [OH − ] = 0.01M = 10−2 M
m pOH = − log [OH − ] = − log(10−2) = 2
x
where, = amount of the gas adsorbed per unit We know that, pH + pOH = 14
m
mass of adsorbent ∴ pH = 14 − 2 = 12
p = pressure k and n = constants. Thus, option (c) is correct.
The value of n lies in between 0 to 1.
x Key Idea Bronsted acid is a substance which has a
Thus, = kp 0. 3 and option (a) is correct. 77. (c) tendency to donate proton. Bronsted base is a substance
m which has a tendency to accept proton.
72. (d) Key Idea If all the electron in a molecule are paired, the HCl can act as Bronsted acid becuase it can only donate proton.
molecule is diamagnetic and if there are unpaired electrons
in a molecule, it is paramagnetic. HCl + H 2O H 3O + + Cl −
-
Acid Base
Molecular orbital configuration of given molecules are as The remaining options contains substances which act both as
follows: Bronsted acid and Bronsted base.
(1) N 2(Z = 14) − σ1s 2 , σ *1s 2 , σ 2s 2 , σ * 2s 2 , π 2px2 = π 2p 2y , σ 2pz2 HCO –3 + HCO –3 H 2CO 3 + CO 2–
- 3

It is a diamagnetic molecule due to absence of unpaired NH 3 + NH 3 - NH +4 + NH −2


electron.
HSO –4 + HSO –4 - H 2SO 4 + SO 2–
(2) H 2 (Z = 2) − 1σ 2 4

Thus, option (c) is correct.


It is a diamagnetic molecule.
(3) Li 2 (Z = 6) − σ1s 2 , σ *1s 2 , σ 2s 2 78. (c) Let the solubility of CaF2 in 0.1 M NaF is ‘S’ mol L −1
Ca 2+ (aq) + 2F − (aq)
It is a diamagnetic molecule.
(4) O 2(Z = 16) − σ1s 2 , σ *1s 2 , σ 2s 2 , σ * 2s 2,
CaF2(s) - S 2S
σ 2pz2 , π 2px2 = π 2p 2y , π * 2p1x = π * 2p1y
NaF(aq) - Na+ + F − (aq)
It is a paramagnetic molecule due to presence of unpaired . M 01
01 . M
electrons in each orbital of degenerate levels.
[F − ] = 2S + 01
.
Thus, option (d) is correct.
K sp of CaF2 = [Ca 2+ ] [F − ]2 = [S] [2S+ 0.1]2
73. (b) BF3 has zero dipole moment as it is symmetrical in nature.
. × 10−11 = [S] [2S + 01
= 53 . ]2
H 2O has maximum dipole moment as it possess two lone pair
of electrons. Between NH 3 and NF3, NH 3 has greater dipole ⇒ . × 10−11 = [S] [01
53 . ]2 [Q 2S << 01
. ]
moment though in NH 3 and NF3, both N possesses one lone . × 10−11
53
pair of electrons. [S] =
. )2
(01
This is beacuse in case of NH 3, the net N H bond dipole is in
= 5.3 × 10−9 mol L−1
the same direction as the direction of dipole of lone pair. But in
case of NF3, the direction of net dipole moment of three N F 79. (c) The structure of CrO 5 is
bonds is opposite to that of the dipole moment of the lone pair.
O
Thus, the correct of dipole moment is
O O
Cr
F O O
O > N > N >F B Oxidation state of Cr is +6 due to the presence of two peroxide
H H H H F F F
H F linkages, which can be calculated as.
(H2O) (NH3) (NF3) (BF3) In CrO(O 2)2, let the oxidation state of Cr be x
x + (−1) 4 + (−2) = 0 ⇒ x − 6 = 0 ⇒ x = +6
74. (a) Crude sodium chloride obtained by crystallisation of
brine solution contains CaSO 4 , Na 2SO 4 , CaCl 2 , MgCl 2 as 80. (b) Cu 2+ ion has 17 electrons in its outermost orbital. 4H 2O
impuirities. CaCl 2 and MgCl 2 are deliquescent, i.e. absorb molecules donate 4 pairs of electrons for coordinate covalent
moisture from atmosphere and give wet appearance to the sharing. The one H 2O molecule is attached to the sulphate ion
salt. Crude salt is dissolved in water and insoluble impurities by hydrogen bonding, the oxygen being oriented to the
are removed by filtration. Hence, MgSO 4 is not present in hydrated cupirc ion.
crude sodium chloride.
H2O OH2 H----O O
75. (a) The size of the ion and its charge determines extent of Cu O S
hydration. Greater the charge and smaller the size of ion, H2O OH2 H----O O
greater is the extent of hydration and hence greater is the
hydration enthalpy. Thus, Li + having maximum hydration Thus, the number of hydrogen bonded water molecule
tendency forms dihydrate salt (LiCl.2H 2O) easily. associated with CuSO 4 . 5H 2O is 1.
www.jeebooks.in
NEET Solved Paper 2019 59

81. (a) Let, in the given crystal Ni 0. 98O Alternative method


M
Ni 2+
= x and Ni 3+
= 0.98 − x Molality (m) = × 1000
1000d − Mωt
Total charge on M 2+
and M 3+ = (+2) x + (+3) (0.98 − x)
where, M = molarity, d = density of solution
= 2x + 2.94 − 3x = 2.94 − x
Mωt = molar mass of solute.
As metal oxide is neutral. Therefore,
On substing the given values, we get
total charge on cations = total charge on anions. 2
2.94 − x = 2 [Q Charge of oxygen atom = −2] Molality = × 1000
1000 × 1.28 − 40
x = 2.94 − 2 = 0.94
2 2000
So, the fraction of Ni 2+ ions in the crystal =
0.94
= 0.96 = × 1000 = = 1.612 m
0.98 1280 − 40 1240

82. (a) Solution exhibits positive deviation from ideal behaviour 86. (c) Angular node (l) = 3
if the intermolecular interactions, i.e., A  B interactions are Total node = radial node + angular node
more stronger than in pure components, i.e. between A  B or 3 = (n − l − 1) + l
B  B. Thus. option (a) is correct. The remaining options are
3 = n −1 ⇒ n = 4
valid only for ideal solutions.
∴Orbital having 3 angular nodes and 3 total nodes is = nl = 4 f [Q
83. (c) Partial pressure of dry air = total pressure × mole fraction l = 3for f- orbital]
of dry air
87. (b) According to Bohr,
⇒ pdry air = ptotal × λ dry air nh
Given, λsaturated air = 0.02 mvr =

λ dry air = 1 − 0.02 = 0.98
…(i) Q λ =
nh h 
ptotal = 1.2atm 2πr = = nλ
mv  mv 
∴ pdry air = 1.2 atm × 0.98 = 1176
. atm
where, r = radius, λ = wavelength
84. (b) More negative the value of standard reduction potential, n = number of orbit
higher is the reduction power. a n2
1 Also, r= 0 …(ii)
i.e. Reducing power ∝ Z
standard reduction potential where, a 0 = Bohr radius = 52.9 pm
Thus, the correct decreasing order of reducing power of the Z = atomic number
metal is On substituting the value of ‘r ’ from Eq. (ii) to Eq. (i), we get
K > Al >
2πn2a 0 2πna 0
(E Kº + / K = −2.93V) º
(E Al = − 1.66V) nλ = ⇒ λ=
3+
/ Al Z Z
Cr >Ag λ = 2π × 2 × 52.9 [Q n = 2, Z = 1]
º
(E Cr 3+ = − 0.74V) º
(E Ag + / Ag = 0.80V)
= 211.6 π pm
/ Cr

85. (c) Given, molarity = 2M [i.e 2 mole NaOH in 1 L solution] 88. (d) According to ideal gas equation,
Density (d) = 1.28 cm −3 pV = nRT
…(i) Q n =
−1 nRT w RT w 
Molecular weight of NaOH = 40 gmol or V= =
p M. wt p  M. wt 
We know that,
mass of solution Given, w = 1.8 g, T = 374º C
Density =
volume of solution = (374 + 273) K = 647 K
∴ Mass of solution = 1.28 g cm −3 × 1000 mL p = 1 bar, R = 0.083 bar LK −1 mol −1
= 1280 g On substituting the given values in Eq. (i), we get
Moreover, molarity 1.8g 0.083 bar LK –1 mol –1 × 647 K
number of moles of solute V= × = 537
. L
= × 1000 18 g mol -1
1 bar
volume of solution (in mL)
∴ 2=
mass of solute
× 1000 = 80 g
89. (b) For an isothermal irreversible expansion,
40 × 1000 Work done (W) = − pext (V2 − V1)
Now, mass of solution = mass of solvent + mass of solute where, V1 = initial volume
1280 g = mass of solvent +80 V2 = final volume
∴Mass of solvent = 1280 − 80 = 1200 g = 1.2 kg Given, pext = 105 Nm −2, V1 = 10−3 m3, V2 = 10−2 m3
number of moles of solute
Now, molality = On substituting the given values in Eq, (i),
mass of solvent (in kg) We get,
2 20 5 W = −105Nm−2(10−2 m3 − 10−3 m3)
= = = = 1.67 m
1.2 12 3
= −105Nm−2 × 10−3 (10 − 1)m3 = −900 Nm = −900 J
www.jeebooks.in
60 NEET Test Drive

90. (d) From first law of thermodynamics, 98. (d) Kwashiorkor disease is due to protein deficiency not
∆U = q + W accompanied by calorie deficiency in the children of age 1-5
In adiabatic expansion, q = 0 years. It’s symptoms are weak muscle, thin limbs, retarded
growth of the body and brain, swelling of legs due to
∴ ∆U = W retention of water (oedema), reddish hair, pot belly, etc.
During expansion of a gas w is negative i.e w < 0 or ∆U < 0.
We know that, ∆U = nCV ∆T 99. (a) The correct sequence of events are
∴ nCV ∆T < 0 or ∆T < 0 Gametogenesis (i.e. production of gametes) → Gamete transfer
(i.e. movement of gamete at the site of fertilisation) →
∴ TC − TA < 0 or TC < TA
Syngamy (i.e. the fusion of gametes) → Zygote (i.e. a
Thus, option (d) is incorrect while the remaining options are eukaryotic cell formed by fertilisation of gametes) → Cell
correct. division (cleavage) → Cell differentiation (i.e the process
where a cell changes from one cell type to another) →
Biology Organogenesis (i.e. the process of formation of three germ
91. (b) Golgi complex (Golgi apparatus) is a cell organelle layers).
present in highest number in secretory cells. These are the 100. (c) Oxytocin hormone is responsible for both, the milk
site of modification, packaging and secretions of secretory ejection reflex and foetal ejection reflex. It is a peptide
proteins and glycoproteins outside the cell. hormone normally produced in the hypothalamus and
released by the posterior pituitary gland.
92. (c) Non-membranous nucleoplasmic structure in the nucleus
of the cell are the site for active synthesis of rRNA. These 101. (d) No new follicles develop in the luteal phase of menstrual
structures are called nucleolus. Larger and more numerous cycle. It is because during this phase, Luteinising Hormone
nucleoli are present in the cell actively carrying out protein (LH) and Follicle Stimulating Hormone (FSH) levels
synthesis. decrease. Instead, the already ruptured follicle closes after
releasing the egg and forms a corpus luteum during luteal
93. (b) Double-stranded circular naked DNA type of nucleic acid phase, which produces progesterone.
is present in an organism having 70S ribosomes.
These are present in prokaryotic organisms or cells. All
102. (d) In Australia, marsupials and placental mammals have
evolved to share many similar characteristics. This type of
prokaryotic cells have a single double-stranded (double helix),
evolution is referred to as convergent evolution.
circular DNA molecule for their genetic material. This DNA is
attached to the inner cell membrane where the DNA Convergent evolution is the independent evolution of similar
replicating machinery is located. The DNA is ‘‘naked’’, it does features in species of different lineages. For example, a number
not have proteins associated with it as eukaryotic DNA does. of marsupials, each different from the other evolved from an
ancestral stock, but all within the Australian island continent.
94. (b) After meiosis-I, the resultant daughter cells have twice Also, marsupials in Australia resemble placental mammals in
the amount of DNA in comparison to haploid gamete. the rest of the world, they evolved in isolation after Australia
Meiosis-I causes segregation of homologous pairs of separated from other continents.
chromosomes. However, each chromosome is
double-stranded, having two sister chromatids due to DNA 103. (c) The correct matches are
replication before meiosis began. 1. XX-XO method of sex-determination is seen in (iii)
Grasshopper, where males have only one X-chromosome
95. (b) Phospholipids are main constituents of lecithin. These besides autosomes and females have a pair of
molecules are composed of choline and inositole. It is found X-chromosome.
in all living cells as a major component of cell membrane. 2. XX-XY method of sex-determination-(iv) Female
96. (b) The main difference between active and passive transport homogamety as seen in human beings.
across the cell is that passive transport requires concentration 3. Karyotype-45 (i) Turner’s syndrome with 45+XO.
gradient across biological membrane whereas during active 4. ZW-ZZ method of sex-determination-(ii) Female
transport, the movement of molecules is from high heterogamety as seen in birds.
concentration to low concentration that means they move
against the concentration gradient by using ATP. 104. (b) The mRNA will be complementary to the DNA strand, but
in RNA, uracil will be present in place of thymine. If the
97. (d) The correct matches are template strand is 3′ - A T G C A T G C A T G C A T G - 5 then
1. Rennin is a proteolytic enzyme that causes the base sequence of mRNA for the given DNA strand will be
coagulation of milk. 5′ - U A C G U A C G U A C G U A C - 3’.
2. Enterokinase converts trypsinogen into its 105. (d) The incorrect statement regarding inbreeding is the
active form trypsin. option (d). It is because continuous inbreeding among cattle
3. Oxyntic cells (also called parietal cells) during digestion causes inbreeding depression. It decreases the fertility and
release stomach acid to allow release of vitamin- B12 from even productivity of an animal. It can be overcome by
food. applying outbreeding in which mating is done between
4. Fructose is absorbed by facilitated transport into the different breeds or individuals of the same breed but having
blood capillaries. no common ancestors. Outbreeding includes outcrossing,
cross-breeding and interspecific hybridisation.
www.jeebooks.in
NEET Solved Paper 2019 61

106. (d) A biocontrol agent to be a part of an Integrated Pest 114. (c) Option (c) is correct.
Management (IPM) programme should be species-specific The adaptive radiation occurs when a population of a species
and inactive or have no negative impacts on non-target invades a new area. It is because it provides organisms of a
organisms like plants, mammals, birds, fish and even on population a new habitat with plenty of niche spaces. Darwin’s
other non-target insects. It should kill only targeted finches represent one of the best examples of this phenomenon.
insects/pests (organisms).
115. (b) In the given mechanism of hormone action, the labels are
107. (b) The correct matches are
A–Protein hormone, B–Receptor, C–Cyclic AMP
1. Restriction (iii) Cuts DNA at specific site Hormones which interact with membrane bound receptors do
endonuclease not enter the target cell. These bind with membrane proteins
(receptors) and generate second messengers which in turn
2. Restriction (iv) Removes nucleotides from the
regulate cellular metabolism, e.g. FSH on ovarian cell
exonuclease ends of DNA
membrane.
3. DNA ligase (i) Joins the DNA fragments
116. (c) Innate immunity is non-specific type of defence that is
4. Taq polymerase (ii) Extends primars on genomic present at the time of birth because it has natural killer cells
DNA template. which can phagocytose and destroy microbes (cellular
barriers). Other forms of innate immunity are physical
108. (a) The two antibiotic resistance gene on E.coli cloning barriers, physiological and cytokine barriers.
vector pBR322 are for ampicillin and tetracycline. Cloning 117. (c) Option (c) is not correct because rods and cones are
vectors are DNA molecules that carry a foreign DNA photoreceptor cells in our eyes. The rod cells contain a purple
segment and replicate inside host cell. Plasmid in E.coli is a pigment rhodopsin that is useful in night vision or scotopic
cloning vector. vision. Daylight (photopic) vision and colour vision are the
functions of cones.
109. (b) Exploitation of bioresources of a nation by multinational
companies without authorisation from concerned country is 118. (a) The correct matches are
referred to as biopiracy. For example, a patent granted in 1. Gliding joint–(iii) Between the carpals
USA covers the entire basmati rice germplasm of our
2. Hinge joint – (iv) Between humerus and ulna
country.
3. Pivot joint – (ii) Between atlas and axis
110. (c) Carnivorous animals, lions and leopard, occupy the same 4. Saddle joint – (i) Between carpal and metacarpal of thumb
niche but lion predates mostly larger animals and leopard
takes smaller ones. This is called resource partitioning. It is 119. (a) Myasthenia gravis is a chronic autoimmune
a mechanism in which there is the division of limited neuromuscular disorder that causes weakness in the skeletal
resources by species to help avoid competition in an muscles. This is responsible for breathing and moving parts of
ecological niche. In any environment, organisms compete the body including the arms and legs.
for limited resources, so organisms and different species
have to find ways to coexist with one another. That is why
120. (b) Artificial light, extended work time and reduced sleep time
disrupt the activity of pineal gland. It is a small pea-shaped
lions predate mostly larger animals and leopards take
gland in the brain. It produces melatonin, which helps
smaller ones.
maintain circadian rhythm and regulate reproductive
111. (d) Decline in the population of Indian native fishes due to hormones.
introduction of Clarias gariepinus in river Yamuna can be
categorised as alien species invasions. It is posing a threat to 121. (b) The fall in blood Ca +2 levels is the condition which will
the indigenous catfishes in our rivers and causing a loss of stimulate parathyroid gland to release parathyroid hormone.
biodiversity. This hormone exerts its effects on bones and kidneys. When
calcium levels are low, parathyroid hormone is released by the
112. (c) The correct matches are parathyroid glands into the blood and causes the bones to
1. RNA polymerse I transcribes (ii) rRNAs (28S,18S and release calcium and increase levels in the bloodstream.
5.8S) 122. (c) Option (c) is correct as Intrauterine Devices (IUDs)
2. RNA polymerase II (iii) hn RNA increase phagocytosis of sperms within the uterus and the Cu
ions released suppress sperm motility and fertilising capacity
3. RNA polymerase III (i) tRNA of sperms.
Other statements can be corrected as
113. (a) The possible genotype of parents having progeny with IUDs can be removed as these are a reversible contraception
either blood group AB or B is I A i (male): I BI B (female) method.
IA i IB IB IUDs are inserted by doctors or expert nurses in the uterus
´
through vagina.
IUDs do not affect gametogenesis.
I AI B I AI B I Bi I Bi 123. (c) AIDS and Hepatitis-B are sexually transmitted diseases
Blood Blood Blood Blood which do not specifically affect reproductive organs. AIDS
group group group group affects the overall immune system of the individual and
AB AB B B Hepatitis-B affects the liver. These are called STD because
these spread through unsafe sex or unprotected sex.
www.jeebooks.in
62 NEET Test Drive

124. (b) The correct matches are 132. (d) Atrio-Ventricular Node (AVN) present in the lower
corner of the right atrium, delays the spreading of impulses
1. Ophiura (Brittle star) (iii) Echinodermata to heart ventricles for about 0.1 second. This pause allows the
2. Physalia (Portuguese man of war) (iv) Coelenterata atria to empty completely into the ventricles before the
ventricles pump out the blood.
3. Pinctada (ii) Mollusca
(Pearl oyester) 133. (b) The correct matches are
4. Planaria (Flatworm) (ii) Platyhelminthes Part of nephron Function
1. Descending limb of Henle’s (ii) Reabsorption of water
125. (d) Annelids are true coelomates with bilateral symmetry. loop only
These exhibit organ-system level of body organisation with 2. Proximal convoluted tubule (iv) Reabsorption of ion,
true coelom. They are triploblastic, metamerically segmented water and organic
and coelomate animals, e.g. earthworm. nutrients
126. (b) Couplet is the contrasting characteristic generally in a 3. Ascending limb of Henle’s (i) Reabsorption of salts
pair used for identification of animals in taxonomic key. It loop
represents the choice made between two opposite options, 4. Distal convoluted tubule (iii) Conditional
each half of a couplet is called lead. Separate taxonomic keys reabsorption of
are required for each taxonomic category like family, genus sodium ion and water
species.
134. (c) The correct matches are
127. (c) The correct matches are
1. Podocytes (iv) Filtration slit
1. Tight junctions (iii) Establish a barrier to
2. Protonephridia (iii) Amphioxus
prevent leakage of fluid
across epithelial cells. 3. Nephridia (ii) Annelids
2. Adhering junctions (i) Cement neighbouring 4. Renal calculi (i) Crystallised oxalates
cells together to form Podocytes are cells in Bowman’s capsule in kidneys. They
sheet. have filtration slits through which the blood is filtered.
Protonephridia help in osmoregulation. Nephridia in annelids
3. Gap junctions (iv) Cytoplasmic channels to help in osmoregulation and excretion. Renal calculi are
facilitate communication kidney stones which mainly consist of crystallised oxalates.
between adjacent cells
135. (d) The inner ear contains crista ampullaris and macula as
4. Synaptic junctions (ii) Transmit information the specific receptors of the vestibular apparatus responsible
through chemical to for maintenance of balance of the body and posture.
another cells.
136. (d) Statement (d) is against the rules of ICBN because the
universal rule of nomenclature is that the first word denoting
128. (a) Statement (a) is incorrect because cockroach vision is the genus starts with a capital letter while the specific epithet
very sensitive but provides less resolution. Such vision is starts with a small letter. It can be illustrated with the
called mosaic vision. It has compound eye and each eye example of Mangifera indica.
contains about 2000 ommatidia.
137. (b) Mad cow disease in cattle is caused by prions which are
129. (b) Statement (b) is correct as intrapulmonary pressure is abnormally folded proteins. It is also known as Bovine
lower than the atmospheric pressure during inspiration. Spongiform Encephalopathy (BSE). It is a progressive
Other statements can be corrected as neurological disorder of cattle.
(a) Inspiration occurs due to external intercoastal
138. (a) (a) Statement (a) is correct. Lichens do not grow in
muscles. polluted area. Rest statements are incorrect.
(c) Inspiration occurs when atmospheric pressure The correct forms of the statements are as follows
is more than intrapulmonary pressure. (b) Algal component of lichens is called phycobiont.
(d) Inspiration is initiated due to contraction of (c) Fungal component of lichens is called mycobiont.
diaphragm. (d) Lichens are good pollution indicators.
130. (b) Vital Capacity (VC) is the maximum volume of air a 139. (a) The correct match of organisms with their habitats are
person can breathe in after a forced expiration. This includes ● Halophiles live in salty areas
ERV, TV and IRV or the maximum volume of air a person can ● Thermoacidophiles live in hot springs
breathe out after a forced inspiration. ● Methanogens live in guts of ruminants
131. (d) The nodal musculature has the ability to generate action ● Cyanobacteria live in aquatic environment
potentials without any external stimuli, i.e. it is
140. (a) In dicot root, the vascular cambium originates from
autoexcitable. However, the number of action potentials that tissues located below the phloem bundles and a portion of
could be generated in a minute vary at different parts of the pericycle tissue above protoxylem.
nodal system. The SAN (Sino-Atrial Node) can generate the
maximum number of action potentials, i.e 70-75 min, i.e. the Vascular cambium is the meristematic layer that is responsible
highest rate of depolarisation and is responsible for initiating for cutting off vascular tissues (xylem and phloem). In young
and maintaining the rhythmic contractile activity of the stem, it is present in patches as a single layer between the
heart. Therefore, it is called pacemaker. xylem and phloem.
www.jeebooks.in
NEET Solved Paper 2019 63

141. (c) Alstonia shows whorled phyllotaxy. In whorled phyllotaxy, ● About 15 mya , primates called Dryopithecus and
more than two leaves arise at a node and form a whorl. Mustard Ramapithecus were existing. They were hairy and walked
and China rose show alternate phyllotaxy. Calotropis shows like gorillas and chimpanzees.
opposite phyllotaxy. ● Homo habilis probably did not eat meat while Homo erectus
probably ate meat.
142. (c) Regeneration of damaged growing grass following grazing is
largely due to intercalary meristem. It is the meristem which 151. (c) The production of gametes by the parents, the
occurs between mature tissues. It is found in grasses and formation of zygotes, the F1 and F2 plants can be
regenerates parts damaged by the grazing herbivores. understood from a diagram called Punnett square. It was
developed by a British geneticist, Reginald C. Punnett. It is
143. (c) Solanum (family Solanaceae) has bicarpellary ovary with a graphical representation to calculate the probability of
obliquely placed septum. Brassica (family Brassicaceae) has all possible genotypes of offspring in a genetic cross.
bicarpellary ovary with false septum. In Sesbania (subfamily
Fabaceae), ovary is monocarpellary. Aloe (family Liliaceae) 152. (a) The correct matches are
shows tricarpellary ovary. Golgi apparatus – Formation of glycoproteins and
glycolipids
144. (b) The most common type of female gametophyte (embryo sac) Lysosomes – Digesting biomolecules
in angiosperms is the monosporic embryo sac in which the Vacuoles – Trap waste and excretory products
embryo sac develops from a single functional megaspore (n) Ribosomes – Synthesis of protein
while the other three megasproes degenerate. The functional
megaspore undergoes three sequential mitotic divisions and 153. (b) Prosthetic groups are organic compounds and are
gives rise to 8-nucleate and 7-celled mature embryo sac. distinguished from other cofactors in that they are tightly
bound to the apoenzyme. For example, in peroxidase and
145. (d) The correct combination of salient features of Genetic code catalase, which catalyse the breakdown of hydrogen
is degenerate, Non-overlapping, Non-ambiguous. These are peroxide to water and oxygen, haem is the prosthetic
explained as one codon codes for only one amino acid, hence group and it is a part of the active site of the enzyme.
genetic code is unambiguous and specific. Some amino acids are
coded by more than one codon, hence the code is degenerate. 154. (b) Crossing over takes place between non-sister
The codon is read in mRNA in a contiguous fashion. There are no chromatids of homologous chromosomes at pachytene
punctuations and overlapping. stage of prophase-I. This stage of prophase-I in meiosis is
characterised by the appearance of recombination nodules,
146. (c) Alfred Hershey and Martha Chase (1952) experimentally the site at which crossing over occurs between non-sister
proved that DNA is the sole genetic material in bacteriophage. chromatids of the homologous chromosomes.
On the other hand, Beadle and Tatum (1940s) experimentally
showed one gene-one enzyme hypothesis using Neurospora. 155. (b) ‘Ramachandran plot’ is used to confirm the structure
Meselson and Stahl first showed that DNA replicates of proteins. Ramachandran plot is a plot of the
semiconservatively through experiments on E.coli. Jacob and angles-phi (ϕ) and psi (ψ) of amino acids found in a
peptide chain. This plot was developed by GN
Monod were first to explain lac operon.
Ramachandran, an Indian Scientist in 1963.
147. (c) In the process of transcription (i.e. copying of genetic 156. (b) Option (b) is not a feature of active transport of
information from one strand of the DNA into RNA) in solutes in plants. Active transport of solutes in plants is
eukaryotes, the RNA polymerase I transcribes rRNA - 28S, 18S carried out by membrane proteins. Like enzymes, the
and 5.8S. On the other hand, tRNA, 5srRNA and snRNAs are carrier proteins are very specific (i.e. selective) in what
transcribed by RNA polymerase III. RNA polymerase II they carry across the membranes. Active transport uses
transcribes precursor of mRNA, hnRNA. energy (ATP) to pump molecules against a concentration
148. (b) Klinefelter’s syndrome is a genetic condition in which each gradient.
cell in the affected person has three sex chromosomes XXY. It is 157. (c) Thiobacillus reduces nitrate in soil into nitrogen. The
caused due to the presence of an additional copy of process is called denitrification.
X-chromosome resulting into a karyotype of 47, XXY. Such
individuals are sterile. On the other hand, Nitrosomonas and Nitrococcus oxidise
ammonia into nitrite. The bacterium, Nitrobacter oxidises
149. (a) This question is not correct because out of the given nitrite to nitrate. These processes together are known as
initiation and termination factors, none is involved in nitrification.
transcription in eukaryotes. Only option (a) gives initiation and
termination factors which are involved in transcription. These 158. (c) The behaviour of the plant cells with regard to water
factors (σ and ρ) initiate and terminate transcription in movement depends on the surrounding solution. When a
prokaryotes (not in eukaryotes). Initiation and termination plant cell is placed in hypotonic solution then the water
factors involved in transcription in eukaryotes are General will flow into the cell and the cell will swell.
Transcription Factors (TF IIA - TF II H) and Transcription 159. (c) Respiratory Electron Transport System (ETS) in plants
Termination Factor-1 (TTF-1), respectively. is located in inner mitochondrial membrane. It serves as
150. (d) Statement (d) is correct about the origin and evolution of the site of oxidative phosphorylation through the action of
men. ATP synthase.
Neanderthal men lived in Asia between 1,00,000 and 40,000 years 160. (c) In Hatch and Slack pathway, the primary CO2 acceptor
back. Other statements are corrected as follows is phosphoenol pyruvate. This occurs in C4-plants.
●Agriculture came around 10,000 years back Phosphoenol pyruvate, a 3-carbon compound, accepts CO2
and forms oxaloacetic acid which is a 4-carbon compound.
www.jeebooks.in
64 NEET Test Drive

161. (c) Removal of shoot tips is a very useful technique to boost 171. (c) Polyblend is an innovative remedy for plastic waste.
the production of tea leaves. This is because effect of auxin is Polyblend is a fine powder of recycled modified plastic which
removed and growth of lateral bud is enhanced. This when mixed with bitumen, can be used to lay roads.
phenomenon in most higher plants in which growing apical 172. (d) Among the given examples, relationship between wasp
buds (shoot tips) inhibit growth of lateral buds due to effects and fig species does not show commensalism. In this
of auxin is called apical dominance. relationship, one species derives the benefit and other neither
harmed nor benefitted.
162. (d) Engelmann used a prism to split light into its spectral
components and then illuminated a green alga, Cladophora Wasp and fig tree show mutualism. Here fig flower is
placed in a suspension of aerobic bacteria (Azotobacter). The pollinated by wasp and wasp lays its egg into fruit and leaves
bacteria were used to detect the sites of oxygen evolution. them there for development.
He observed that bacteria mainly accumulated in the region of Other options show examples of commensalism.
blue and red light of the split spectrum, thus giving the first 173. (d) Salinity of soil is a problem faced by farmers if they
action spectrum of photosynthesis. liberally irrigate agricultural field for a prolonged period of
163. (c) In order to increase the yield of sugarcane crop, time.
gibberellins should be sprayed.Sugarcane stores Irrigation salinity is the accumulation of salts in the topsoil
carbohydrates as sugar in their stems. Spraying sugarcane under irrigation. It is caused by over irrigation of agricultural
crop with gibberellins increases the length of the stem, thus land, inefficient water use, poor drainage and the irrigation of
increasing the yield by as much as 20 tonnes per acre. unsuitable and leaky soils.

164. (d) Vallisneria is a water pollinated plant. In Vallisneria, the 174. (c) Statement (c) is incorrect. Correct information about the
female flowers reach the surface of water by the long stalk statement is as follows
and the male flowers or pollen grains are released on to the Certain bacteria, which grow anaerobically on cellulosic
surface of water. They are carried passively by water currents material, produce large amount of methane along with CO 2
to female flowers at surface of water. and H 2. These bacteria are collectively called methanogens
and one such example is Methanobacterium.
165. (b) Autogamy and geitonogamy both are prevented in Rest statements are correct.
papaya plant. In papaya, male and female flowers are present
on different plants that is each plant is either male or female 175. (a) In mung bean, resistance to yellow mosaic virus and
(dioecy). powdery mildew were induced by mutation breeding.
Mutation breeding is the process of exposing seeds to
166. (c) The correct matches are chemicals or radiation in order to generate mutants with
The placental types Examples desirable traits to be bred with other cultivars.

Basal Sunflower 176. (c) Coca alkaloid or cocaine is obtained from coca plant
Erythroxylum coca, native to
Axile China rose South America. It interferes with the transport of the
Parietal Mustard neurotransmitter dopamine.
Free-central Dianthus 177. (c) Biofertilisers are organisms that enrich the nutrient
quality of the soil.
167. (a) To facilitate cloning into a vector, the vector requires a For example, Cyanobacteria and Rhizobium. Aspergillus and
selectable marker, which helps in identifying and eliminating Rhizopus are not used as biofertilisers.
non-transformants and selectively permitting the growth of
178. (d) Statements (1), (2) and (4) are incorrect because DNA
the transformants. fragments are negatively charged molecules they can be
168. (a) Endemic species are those species which are confined to a separated by forcing them to move towards the anode under
particular region and not found anywhere else. For example, an electric field through a medium/matrix. The concentration
species of Western Ghats. Vulnerable and threatened species of gel does affect the resolution of DNA separation.
are the species which face high risk of extinction. Keystone The separated DNA fragments can be visualised only after
species are the species whose presence or absence significantly staining the DNA with a compound known as ethidium
affects other species in an ecosystem. bromide followed by exposure to UV radiation. Only statement
3 is correct. The DNA fragments spearate (resolve) according to
169. (d) Statement (d) is correct. Stratospheric ozone protects us their size through sieving effect provided by the agarose gel.
from UV radiations of the sun. Correct information about Hence, the smaller the fragment size, the farther it moves.
incorrect statements is as follows.
Good ozone is found in the upper part of the atmosphere, i.e. 179. (c) An enzyme catalysing the removal of nucleotides from
stratosphere. Bad ozone is formed in the lower atmosphere ends of DNA is exonuclease. Endonuclease made cuts at
(troposphere) that harms plants and animals. specific positions within the DNA. DNA ligase joins DNA
fragments. Proteases are protein-degrading enzymes.
170. (d) Exploration of molecular, genetic and species level
diversity for novel products of economic importance is known 180. (a) In RNAi, the genes are silenced using dsRNA. RNA
as bioprospecting. Biopiracy refers to illegal use of interference (RNAi) takes place in all eukaryotic organisms
bioresources. Bioremediation is use of organisms to clean up as a method of cellular defence. This method involves
silencing of a specific mRNA due to a complementary dsRNA
contamination during waste treatment. Bioenergetics deals
molecule that binds to and prevents translation of the mRNA
with energy metabolism in living organisms.
(silencing).

You might also like